You are on page 1of 788

ED

U
JO
U
R
N
AL
.IN
EBD_7179
• Corporate Office : 45, 2nd Floor, Maharishi Dayanand Marg, Corner Market,
Malviya Nagar, New Delhi-110017
Tel. : 011-26692293 / 26692294

By
Sanjeev Kumar Jha

.IN
Typeset by Disha DTP Team

downloaded from : AL
N
R

www.youtube.com/c/rishimudgal
U
JO
U
ED

DISHA PUBLICATION
All Rights Reserved

© Copyright Publisher
No part of this publication may be reproduced in any form without prior permission of the publisher. The author and
the publisher do not take any legal responsibility for any errors or misrepresentations that might have crept in. We
have tried and made our best efforts to provide accurate up-to-date information in this book.

For further information about the books from DISHA PUBLICATION,


Log on to www.dishapublication.com or www.aiets.co.in or email to info@aiets.co.in

(ii)
Contents
1. Physical World 1-7

2. Units and Measurements 8-34

3. Motion in a Straight Line 35-66

4. Motion in a Plane 67-98

Laws of Motion

.IN
5. 99-138

6. Work, Energy and Power


AL 139-172
N
7. System of Particles and Rotational Motion 173-210
R
U

8. Gravitation 211-240
JO

9. Mechanical Properties of Solids


U

241-259
ED

10. Mechanical Properties of Fluids 260-279

11. Thermal Properties of Matter 280-302

12. Thermodynamics 303-325

13. Kinetic Theory 326-345

14. Oscillations 346-372

15. Waves 373-400

(iii)
EBD_7179
16. Electric Charges and Fields 401-429

17. Electrostatic Potential and Capacitance 430-463

18. Current Electricity 464-511

19. Moving Charges and Magnetism 512-542

20. Magnetism and Matter 543-566

21. Electromagnetic Induction 567-587

.IN
22. Alternating Current 588-609

23. Electromagnetic Waves


AL 610-623
N
24. Ray Optics and Optical Instruments 624-660
R
U

25. Wave Optics 661-684


JO

26. Dual Nature of Radiation and Matter 685-710


U
ED

27. Atoms 711-727

28. Nuclei 728-748

29. Semiconductor Electronics :


Materials, Devices and Simple Circuits 749-784

(iv)
1 Physical World

WHAT IS SCIENCE ? Keep in Memory


Science refers to a system of acquiring knowledge. This system
uses observation and experimentation to describe and explain 1. Information received through the senses is called observation.
natural phenomena. The term science also refers to the organized 2. An idea that may explain a number of observations is called

.IN
body of knowledge that people have gained using that system. hypothesis.
Less formally, the word science often describes any systematic
3. A hypothesis that has been tested many times is called
field of study or the knowledge gained from it.
scientific theory.
The purpose of science is to produce useful models of reality.
AL
Most scientific investigations use some form of the scientific 4. A scientific theory that has been tested and has always
method. proved true is called scientific law.
N
The scientific method is a logical and rational order of steps by
SCOPE AND EXCITEMENT OF PHYSICS
R

which scientists come to conclusions about the world around


them. The Scientific method helps to organize thoughts and The scope of physics is very vast. It covers a tremendous range
U

procedures so that scientists can be confident in the answers of magnitude of physical quantities like length, mass, time, energy,
etc. Basically, there are two domains of interest : macroscopic and
JO

they find. Scientists use observations, hypotheses, and deductions


to make these conclusions. microscopic. The macroscopic domain includes phenomena at
Science as defined above, is sometimes called pure science to the laboratory, terrestrial and astronomical scales. The microscopic
U

differentiate it from applied science, which is the application of domain includes atomic, molecular and nuclear phenomena.
research to human needs. Fields of science are commonly divided Classical physics deals mainly with macroscopic phenomena
ED

into two major categories.


consisting of the study of heat, light, electricity, magnetism, optics,
1. Natural science : The science in which we study about the
acoustics, and mechanics. Since the turn of the 20th century, the
natural world. Natural science includes physics, chemistry,
biology, etc. study of quantum mechanics and relativity has become more
2. Social science : It is the systematic study of human behavior important. Today, physics is often divided into branches such as
and society. nuclear physics, particle physics, quantum physics, theoretical
WHAT IS PHYSICS ? physics, and solid-state physics. The study of the planets, stars,
and their interactions is known as astrophysics, the physics of
The word physics originates from a Greek word which means
nature. Physics is the branch of science that deals with the study the Earth is called geophysics, and the study of the physical laws
of basic laws of nature and their manifestation of various natural relating to living organisms is called biophysics.
phenomena. Physics is exciting in many ways. Application and exploitation of
There are two main thrusts in physics : physical laws to make useful devices is the most interesting and
1. Unification : In physics, attempt is made to explain various exciting part and requires great ingenuity and persistence of effort.
physical phenomena in terms of just few concepts and laws. PHYSICS, TECHNOLOGY AND SOCIETY
Attempts are being made to unify fundamental forces of Physics generates new technologies and these technologies have
nature in the persuit of unification. made our lives comfortable and materially prosperous. We hear
2. Reductionism. Another attempt made in physics is to explain terms like science, engineering , technology all in same context
a macroscopic system in terms of its microsocopic
though they are not exactly same.
constituents. This persuit is called reductionism.
EBD_7179
2 PHYSICS

• Science uses a process--the scientific method--to generate


Technology knowledge
Scientists The products and
Investigate the processes created Engineering:
natural world by engineers • Design under constraint
• Seeks solutions for societal problems and needs
• Aims to produce the best solution from given resources and
Society constraints
Values
• Engineering uses a process–the engineering design
Needs
Environment process–to produce solutions and technologies
Economy
Technology:
Scientific • The body of knowledge, processes and artifacts that result
knowledge Engineers
from engineering
Create the
What scientists • Almost everything made by humans to solve a need is a
designed world
have learned about
technology
the natural world
• Examples of technology include pencils, shoes, cell phones,

.IN
and processes to treat water
Science: In the real world, these disciplines are closely connected.
• A body of knowledge Scientists often use technologies created by engineers to
AL
• Seeks to describe and understand the natural world and its conduct their research. In turn, engineers often use
physical properties knowledge developed by scientists to inform the design of
N
• Scientific knowledge can be used to make predictions the technologies they create.
R

Link between technology and physics


U

Technology Scientific principle(s)


JO

Aeroplane Bernoulli’s principle in fluid dynamics


Bose-Einstein condensate Trapping and cooling of atoms by laser beams and magnetic fields
U

Computers Digital logic


ED

Electric generator Faraday’s laws of electromagnetic induction


Electron microscope Wave nature of electrons
Fusion test reactor (Tokamak) Magnetic confinement of plasma
Giant Metrewave Radio Telescope (GMRT) Detection of cosmic radio waves
Hydroelectric power Conversion of gravitational potential energy into electrical energy
Lasers Light amplification by stimulated emission of radiation
Non-reflecting coatings Thin film optical interference
Nuclear reactor Controlled nuclear fission
Optical fibres Total internal reflection of light
Particle accelerators Motion of charged particles in electromagnetic fields
Photocell Photoelectric effect
Production of ultra high magnetic fields Superconductivity
Radio and television Generation, propagation and detection of electromagnetic waves
Rocket propulsion Newton’s laws of motion
Sonar Reflection of ultrasonic waves
Steam engine Laws of thermodynamics
Physical World 3

Some physicists from different countries of the world and their major contributions
Name of physicists Major contribution/discovery Country of origin
Abdus Salam Unification of weak and electromagnetic interactions Pakistan
Albert Einstein Explanation of photoelectric effect; Theory of relativity Germany
Archimedes Principle of buoyancy; Principle of the lever Greece
C.H. Townes Maser; Laser U.S.A.
Christiaan Huygens Wave theory of light Holland
C.V. Raman Inelastic scattering of light by molecules India
Edwin Hubble Expanding universe U.S.A.
Enrico Fermi Controlled nuclear fission Italy
Ernest Orlando Lawrence Cyclotron U.S.A.
Ernest Rutherford Nuclear model of atom New Zealand
Galileo Galilei Law of inertia Italy
Heinrich Rudolf Hertz Generation of electromagnetic waves Germany
Hideki Yukawa Theory of nuclear forces Japan
Homi Jehangir Bhabha Cascade process of cosmic radiation India

.IN
Issac Newton Universal law of gravitation; Laws of motion; Reflecting telescope U.K.
James Clark Maxwell Electromagnetic theory; Light - an electromagnetic wave
AL U.K.
James Chadwick Neutron U.K.
J.C. Bose Ultra short radio waves India
N
J.J. Thomson Electron U.K.
John Bardeen Transistors; Theory of super conductivity U.S.A.
R

Lev Davidovich Landau Theory of condensed matter; Liquid helium Russia


U

Louis Victor de Broglie Wave nature of matter France


Marie Sklodowska Curie Discovery of radium and polonium; Studies on natural radioactivity Poland
JO

Michael Faraday Laws of electromagnetic induction U.K.


M.N. Saha Thermal ionisation India
U

Niels Bohr Quantum model of hydrogen atom Denmark


Paul Dirac Relativistic theory of electron; Quantum statistics U.K.
ED

R.A. Millikan Measurement of electronic charge U.S.A.


S. Chandrashekhar Chandrashekhar limit, structure and evolution of stars India
S.N. Bose Quantum statistics India
Victor Francis Hess Cosmic radiation Austria
Werner Heisenberg Quantum mechanics; Uncertainty principle Germany
W.K. Roentgen X-rays Germany
Wolfgang Pauli Exclusion principle Austria

Example 1. Does imagination play any role in physics? frictional force, muscular force, forces exerted by springs and
Solution : Yes, imagination has clayed an important role in the strings etc. These forces actually arise from four fundamental forces
development of physics. Huygen’s principle, Bohr’s theory, of nature.
Maxwell equations, Heisenberg’s uncertainty principle, etc. were Following are the four fundamental forces in nature.
the imaginations of the scientists which successfully explained 1. Gravitational force
the various natural phenomena.
2. Weak nuclear force
Example 2. How is science different from technology?
3. Electromagnetic force
Solution : Science is the study of natural laws while technology is
the practical application of these laws to the daily life problems. 4. Strong nuclear force
Among these forces gravitational force is the weakest and strong
FUNDAMENTAL FORCES IN NATURE
nuclear force is the strongest force in nature.
We come across several forces in our day-to-day lives eg.,
EBD_7179
4 PHYSICS

Fundamental forces of nature


Name Relative Range Exchange Major role Important properties
strength particles
Gravitational force (Force of Universal attractive, weakest,
Large-scale
attraction between any two boies 10 –39
Infinite Gravitons long range, central, conservative
structure
by virtue of their masses) force.
Weak nuclear force (Changes Very short, Govern process involving
quark types as in Beta-decay of sub-nuclear Weak Nuclear neutrino and antineutrino,
nucleus 10 –13 size (on bosons reactions operates only through a range of
10 –16m) nuclear size.
Electromagnetic force (Force Either attractive or repulsive, long
Chemistry
between particles with 10–2 Infinite Photons range, central, conservative force.
and Biology
charge/magnetism)
Strong nuclear force (Strong Basically an attractive becomes
attractive force which binds Very Short, Holding repulsive (when distance between
together the protons and neutrons 1 nuclear size Gluons nuclei nucleons < 0.5 fermi) strongest,
in nucleus) (on 10 –15m) together short range, non-central, non-

.IN
conservative force.
Physicists are trying to derive a unified theory that would describe all the forces in nature as a single fundamental law. So far, they have
AL
succeeded in producing a unified description of the weak and electromagnetic forces, but a deeper understanding of the strong and
gravitational forces has not yet been achieved. Theories that postulate the unification of the strong, weak, and electromagnetic forces
are called Grand Unified Theories (often known by the acronym GUTs). Theories that add gravity to the mix and try to unify all four
N
fundamental forces into a single force are called Superunified Theories. The theory that describes the unified electromagnetic and
R

weak interactions is called the Standard Electroweak Theory, or sometimes just the Standard Model.
Progress in unification of different forces/domains in nature
U
JO

Name of the physicist Year Achievement in unification

Unified celestial and terrestrial mechanics; showed that the same laws of
Issac Newton 1687
U

motion and the law of gravitation apply to both the domains.


ED

Hans Christian Oersted 1820 Showed that electric and magnetic phenomena are inseparable aspects of
a unified domain : electromagnetism.
Michael Faraday 1830
Unified electricity, magnetism and optics; showed that light is an
James Clark Maxwell 1873
electromagnetic wave.
Sheldon Glashow, Abdus Salam, Showed that the 'weak' nuclear force and the electromagnetic force could
1979
Steven Weinberg be viewed as different aspects of a single electro-weak force.
Verified experimentally the predictions of the theory of electro-weak
Carlo Rubia, Simon Vander Meer 1984
force.

Example 3 . What is electromagnetic force? scientific community. The production of a summary description of
Solution : It is the force due to interaction between two moving nature in the form of such laws is a fundamental aim of science.
Physical laws are distinguished from scientific theories by their
charges. It is caused by exchange of photons (g) between two
simplicity. Scientific theories are generally more complex than laws.
charged particles. They have many component parts, and are more likely to be
NATURE OF PHYSICAL LAWS changed as the body of available experimental data and analysis
develops. This is because a physical law is a summary observation
A physical law, scientific law, or a law of nature is a scientific
of strictly empirical matters, whereas a theory is a model that
generalization based on empirical observations of physical
accounts for the observation, explains it, relates it to other
behavior. Empirical laws are typically conclusions based on
observations, and makes testable predictions based upon it. Simply
repeated scientific experiments and simple observations over many
stated, while a law notes that something happens, a theory explains
years, and which have become accepted universally within the
why and how something happens.
Physical World 5

.IN
AL
N
R
U
JO
U
ED
EBD_7179
6 PHYSICS

1. The man who has won Nobel Prize twice in physics is 11. Madam Marie Curie won Nobel Prize twice which were in the
(a) Einstein (b) Bardeen field of
(c) Heisenberg (d) Faraday (a) Physics and chemistry(b) Chemistry only
2. Prof. Albert Einstein got nobel prize in physics for his work (c) Physics only (d) Biology only
on 12. The man who is known as the Father of Experimental Physics
(a) special theory of relativity is
(b) general theory of relativity (a) Newton (b) Albert Einstein
(c) Galileo (d) Rutherford
(c) photoelectric effect
13. The person who has been awarded the title of the Father of
(d) theory of specific heats
Physics of 20th century is
3. Which of the following is wrongly matched ?
(a) Madame Curie (b) Sir C.V. Raman
(a) Barometer-Pressure
(c) Neils Bohar (d) Albert Einstein
(b) Lactometer-Milk 14. Which of the following is true regarding the physical science?
(c) Coulomb’s law-charges

.IN
(a) They deal with non-living things
(d) Humidity-Calorimeter (b) The study of matter are conducted at atomic or ionic
4. C.V. Raman got Nobel Prize for his experiment on AL levels
(a) dispersion of light (b) reflection of light (c) Both (a) and (b)
(c) deflection of light (d) scattering of light (d) None of these
5. A scientific way of doing things involve 15. The branch of science which deals with nature and natural
N
(a) identifying the problem phenomena is called
R

(b) collecting data (a) Sociology (b) Biology


(c) hypothesising a possible theory (c) Civics (d) Physics
U

(d) All of the above 16. Who gave general theory of relativity?
(a) Einstein (b) Marconi
JO

6. The scientific principle involves in production of ultra high


magnetic fields is (c) Ampere (d) Newton
(a) super conductivity (b) digital logic 17. Who discovered X-rays?
U

(c) photoelectric effect (d) laws of thermodynamics (a) Chadwick (b) Roentgen
7. Which of the following has infinite range? (c) Thomson (d) Madam Curie
ED

(a) Gravitational force (b) Electromagnetic force 18. Which of the following is the weakest force?
(c) Strong nuclear force (d) Both (a) and (b) (a) Nuclear force (b) Gravitational force
(c) Electromagnetic force (d) None of these
8. Which of the following is the correct decreasing order of the
strengths of four fundamental forces of nature ? 19. The field of work of S. Chandrashekar is
(a) theory of black hole (b) Cosmic rays
(a) Electromagnetic force > weak nuclear force >
gravitational force > strong nuclear force (c) theory of relativity (d) X-rays
20. Two Indian born physicists who have been awarded Nobel
(b) Str ong nuclear force > weak nuclear force >
Prize in Physics are
electromagnetic force > gravitational force
(a) H. J. Bhabha and APJ Kalam
(c) Gravitational force > electromagnetic force > strong
(b) C.V. Raman and S. Chandrasekhar
nuclear force > weak nuclear force
(c) J.C. Bose and M.N. Saha
(d) Strong nuclear force > electromagnetic force > weak
(d) S. N. Bose and H. J. Bhabha
nuclear force > gravitational force
21. Science is exploring, ...x... and ...y... from what we see
9. The exchange particles for the electromagnetic force are
around us. Here, x and y refer to
(a) gravitons (b) gluons
(a) qualitative, modify (b) experiment, predict
(c) photons (d) mesons
(c) verification, predict (d) reasoning, quantitative
10. Louis de-Broglie is credited for his work on 22. Macroscopic domain includes
(a) theory of relativity (a) phenomena at the laboratory
(b) electromagnetic theory (b) terrestrial scales
(c) matter waves (c) astronomical scales
(d) law of distribution of velocities (d) All of the above
Physical World 7

23. In Rutherford, alpha particle scattering experiment as shown DIRECTIONS for Qs. (24-25) : Each question contains
in given figure, A and B refer to STATEMENT-1 and STATEMENT-2. Choose the correct answer
(ONLY ONE option is correct ) from the following
(a) Statement -1 is false, Statement-2 is true
Lead Flash of (b) Statement -1 is true, Statement-2 is true; Statement -2 is a
block light correct explanation for Statement-1
Microscope (c) Statement -1 is true, Statement-2 is true; Statement -2 is not
Fluorescent a correct explanation for Statement-1
a q
screen (d) Statement -1 is true, Statement-2 is false
A Scattering 24. Statement-1 : The concept of energy is central to Physics
angle and expression for energy can be written for every
B physical system.
Statement-2 : Law of conservation of energy is not valid
for all forces and for any kind of transformation between
(a) polonium sample and aluminium foil different forms of energy.
(b) polonium sample and gold foil 25. Statement-1 : Electromagnetic force is much stronger than
the gravitational farce.
(c) uranium sample and gold foil Statement-2 : Electromagnetic force dominates all
(d) uranium sample and aluminium foil phenomena at atomic and molecular scales.

.IN
AL
Hints & Solutions
N

EXERCISE - 1 24. (d) The concept of energy is central to Physics and the
R

expressions for energy can be written for every physical


1. (b) 2. (c) 3. (d) 4. (d) 5. (d)
system. When all forms of energy e.g., Heat, mechanical
U

6. (a) 7. (d) 8. (d) 9. (c) 10. (c) energy, electrical energy etc., are counted, it turns out
11. (a) 12. (c) 13. (d) 14. (c) 15. (d)
JO

that energy is conserved. The general law of


16. (a) 17. (b) 18. (b) 19. (a) 20. (b) conservation of energy is true for all forces and for any
21. (b) Science is exploring, experimenting and predicting kind of transformation between different forms of
U

from what we see around us. energy.


22. (d) The macroscopic domain includes phenomena at the 25. (b) It is mainly the electromagnetic force that governs the
ED

laboratory, terrestrial and astronomical scales. structure of atoms and molecules, the dynamics of
23. (b) The alpha particle scattering experiment of Rutherford chemical reactions and the mechanical, thermal and
gave the nuclear model of the atom as shown in figure other properties of materials.
Lead Flash of
block light

a Fluorescent
A q
screen
Scattering
B angle
Polonium Gold foil
sample
EBD_7179
8 PHYSICS

Units and
2 Measurements
PHYSICAL QUANTITY Fundamental (or Base) and Derived Units
All the quantities in terms of which laws of physics are described Fundamental units are those, which are independent of unit of
and which can be measured directly or indirectly are called other physical quantity and cannot be further resolved into any
physical quantities. For example mass, length, time, speed, force other units or the units of fundamental physical quantities are

.IN
etc. called fundamental or base units. e.g., kilogram, metre, second
Types of Physical Quantity etc,
1. Fundamental quantities : The physical quantities which do All units other than fundamental are derived units (which are
AL
not depend upon other physical quantities are called dependent on fundamental units) e.g., unit of speed (ms–1) which
fundamental or base physical quantities. e.g. mass, length, depends on unit of length (metre) and unit of time (second), unit
time temperature electric current, luminous intensity and of momentum (Kgms–1) depends on unit of mass, length and time
N
amount of substance. etc.
R

2. Derived quantities :The physical quantities which depend SYSTEM OF UNITS


on fundamental quantities are called derived quantities A system of units is a complete set of fundamental and derived
U

e.g. speed, acceleration, force, etc. units for all physical quantities.
JO

UNIT Different types of system of units


The process of measurement is a comparison process. F.P.S. (Foot - Pound - Second) system. (British engineering system
Unit is the standard quantity used for comparision. of units.): In this system the unit of length is foot, mass is pound
U

The chosen standard for measurement of a physical quantity, and time is second.
which has the same nature as that of the quantity is called the C.G.S. (Centimetre - Gram - Second) system. (Gaussian system
ED

unit of that quantity. of units): In this system the unit of length is centimetre, mass is
Choice of a unit (Characteristics of a unit): gram and time is second.
(1) It should be suitable in size (suitable to use) M.K.S (Metre - Kilogram - Second) system. This system is related
(2) It should be accurately defined (so that everybody to mechanics only. In this system the unit of length is metre, mass
understands the unit in same way) is kilogram and time is second.
(3) It should be easily reproducible. S.I. (International system) units: (Introduced in 1971) Different
(4) It should not change with time. countries use different set of units. To avoid complexity, by
(5) It should not change with change in physical conditions international agreement, seven physical quantities have been
i.e., temperature, pressure, moisture etc. chosen as fundamental or base physical quantities and two as
(6) It should be universally acceptable. supplementary. These quantities are
Every measured quantity (its magnitude) comprises of a number
and a unit. Ex: In the measurement of time, say S.No Base physical quantity Fundamental unit Symbol
15 s 1 Mass kilogram kg
Number (n) Unit (u) 2 Length metre m
If Q is the magnitude of the quantity (which does not depend on 3 Time second s
the selection of unit) then
4 Temperature kelvin K
1
Q = n u = n1 u1 = n2 u2 Þ n µ 5 Electric current ampere A
u
6 Luminous intensity candela cd
Where u1 and u2 are the units and n1 and n2 are the numerical
values in two different system of units. 7 Amount of substance mole mol
Units and Measurements 9

S.No Supplementary physical Supplementary unit Symbol Prefix Power of 10 Symbol


quantity exa 18 E
1 Plane angle radian rad
peta 15 P
tera 12 T
2 Solid angle steradian sr
giga 9 G
Merits of S.I. Units : mega 6 M
(1) SI is a coherent system of units: This means that all derived kilo 3 k
units are obtained by multiplication and division without hecto 2 h
introducing any numerical factor.
deca 1 da
(2) SI is a rational system of units: This is because it assigns
deci –1 d
only one unit to a particular physical quantity.
centi –2 c
(3) SI is an absolute system of units: There is no gravitational
unit in this system. milli –3 m
micro –6 m
(4) SI system is applicable to all branches of science.
nano –9 n
Conventions of writing of Units and their Symbols pico –12 p
· Unit is never written with capital initial letter.
femto –15 f
· For a unit named after scientist the symbol is a capital

.IN
letter otherwise not. atto –18 a
· The unit or symbol is never written in plural form. Some Important Practical Units :
· Punctuations marks are not written after the symbol. (1) For large distance (macro-cosm)
AL
Definitions of Fundamental Units (a) Astronomical unit: It is the average distance of the
(i) Metre : One metre is equal to 1650763.73 wavelength in centre of the sun from the centre of the earth.
1 A.U. = 1.496 × 1011m
N
vacuum of the radiation corresponding to transition between
the levels 2p10 and 5d5 of the krypton – 86 atom (b) Light year: It is the distance travelled by the light in
R

Or vacuum in one year. 1 ly = 9.46 × 1015m


The distance travelled by light in vacuum in (c) Parsec: One parsec is the distance at which an arc
U

1A.U. long subtends an angle of one second.


1 1 parsec = 3.1 × 1016m
JO

second is called 1 metre.


299,792, 458 (2) For small distance (micro-cosm)
(ii) Kilogram : The mass of cylinder (of height and diameter 1 micron = 10–6m 1nanometre = 10–9m
1angstorm = 10 m –10 1fermi = 10–15m
U

39 cm) made of Platinum-iridium alloy kept at International


Bureau of weights and measures in paris is defined as 1kg. (3) For small area 1 barn = 10–28m2
ED

(iii) Second : It is the duration of 9,192,631,770 periods of (4) For heavy mass 1 ton = 1000kg
radiation corresponding to the transition between the two 1quintal = 100kg 1slug = 14.57kg
hyperfine levels of the ground state of Caesium (133) atom. 1 C.S.L (chandrasekhar limit) = 1.4 times the mass of the sun
(5) For small mass 1 amu = 1.67 x 10–27kg
(iv) Ampere : It is the current which when flows through two
1 pound = 453.6g = 0.4536 kg
infinitely long straight conductors of negligible cross-
(6) For small time 1 shake = 10–8s
section placed at a distance of one metre in air or vacuum
(7) For large time
produces a force of 2 × 10–7 N/m between them.
Lunar month: It is the time taken by the earth to complete
(v) Candela : It is the luminous intensity in a perpendicular one rotation about its axis with respect to sun.
direction, of a surface of 1/600,000 square metre of a black 1L.M. = 27.3 days.
body at the temperature of freezing platinum under a pressure Solar day: It is the time taken by the earth to complete one
of 1.013 × 105 N/m2. rotation about its axis with respect to sun.
(vi) Kelvin : It is the 1/273.16 part of thermodynamic temperature Sedrial day: It is the time taken by earth to complete one
of triple point of water. rotation on its axis with respect to distant star.
(vii) Mole : It is the amount of substance which contains as many (8) For measuring pressure
elementary entities as there are in 0.012 kg of Carbon-12. 1 bar = 1atm pressure = 105N/m2 = 760mmHg
S.I. Prefixes : 1torr = 1 mmHg
The magnitudes of physical quantities vary over a wide range. 1 poiseuille = 10 Poise.
For example, the atomic radius, is equal to 10 –10m, radius of earth DIMENSIONS
is 6.4×106 m and the mass of electron is 9.1×10–31 kg. The The powers to which the fundamental units of mass, length and
internationally recommended standard prefixes for certain powers time must be raised to represent the physical quantity are called
of 10 are given in the table: the dimensions of that physical quantity.
EBD_7179
10 PHYSICS

For example : Force = mass × acceleration called dimensional formula. It shows how and which of the
fundamental quantities represent the dimensions.
v–u [LT -1]
= mass × = [M] T–2]
= [MLT For example, the dimensional formula of work is [ML2T–2]
t [T]
Hence the dimensions of force are 1 in mass 1 in length and (– 2) Dimensional Equation :
in time. When we equate the dimensional formula with the physical
Dimensional Formula : quantity, we get the dimensional equation.
Unit of a physical quantity expressed in terms of M, L and T is For example Work = [ML2T–2]
Classification of Physical Quantities (On the basis of dimensions) :

Dimensional Dimensional For e.g. c (velocity of light in vaccum)


physical quantity constant R(universal gas constant),
s(stefan's constant), h (Planck's constant),
k (Boltzmann constant), G (universal gravitational constant) etc.

Physical Dimensional For e.g. distance, displacement, force, mass, time etc.
Quantity variable

Dimensionless Dimensionless For e.g. 0, 1, 2, ....., e, p , sin q , cos q , tan q , etc.

.IN
physical quantity constant

Dimensionless For e.g. plane angle, solid angle, strain,


variable
AL
refractive index, dielectric constant, relative density,
specific gravity, poisson's ratio etc.

Dimensional Formula of Some Important Physical Quantities :


N

S.No. Physical quantity Relation with other quantities Dimensional formula


R
U

Length
1. Velocity (v) [M0LT–1]
Time
JO

Velocity
2. Acceleration (a) [M0LT–2]
Time
U

3. Momentum (p) Mass × velocity [MLT–1]


ED

4. Force (F) Mass × acceleration [MLT–2]


5. Work Force × displacement [ML2T–2]

6. Power (P) Work [ML2T–3]


Time

Fr 2
7. Universal gravitational constant G= [M–1L3T–2 ]
m1m 2

8. Torque t = r ´F [ML2T–2]

F
9. Surface tension S= [MT–2]
l

W
10. Gravitational potential VG = [M0L2T–2]
m

F
11. Coefficient of viscosity h= [ML–1T–1]
dv
A
dx
12. Impulse Force×time(F×t) [MLT–1]
Units and Measurements 11

Change in length æ DL ö
13. Strain ç
Original length è L ø
÷ [M0L0T0]

Pressure æ P ö
14. Pressure gradient ç ÷ [ML–2T–2]
Distance è l ø

Arc æsö
15. Plane angle ç ÷ [M0L0T0]
Radius of circle è r ø

Angle æ q ö
16. Angular velocity ç ÷ [M0L0T–1]
Time è t ø

Moment of inertia of body æ I ö


17. Radius of gyration ç ÷÷ [M0L1T0]
Total mass of the body çè å mi ø
18. Moment of force, moment of couple Force × distance (F × s) [ML2T–2]
19. Angular frequency 2p×frequency (2pn) [M0L0T–1]

.IN
Force
20. Pressure [ML–1T–2]
Area AL
Output work or energy æ W ö
21. Efficiency ç ÷ [M0L0T0]
Input work or energy è Q ø
N
22. Angular impulse forque×time (t × t) [ML2T–1]
R

Energy æ E ö
23. Planck’s constant ç ÷ [ML2T–1]
Frequency è n ø
U

Heat energy æ Q ö
JO

24. Heat capacity, Entropy ç ÷


Temperature è T ø
[ML2T–2K–1]

Heat energy æ Q ö
U

25. Specific heat capacity ç ÷ [M0L2T –2K–1 ]


Mass ´ temperature è m ´ DT ø
ED

Heat energy ´ thickness æ Q DT ö


26. Thermal conductivity, K ç = -KA ÷ [MLT–3K–1]
Area ´ temperature ´ time è t Dx ø

Length
27. Thermal Resistance, R [M–1L–2T3K]
Thermal conductivity × area

Volume ´ (change in pressure) æ DP ö


28. Bulk modulus (B) or (compressibility)–1 ç -V ÷ [ML–1T–2]
Change in volume è DV ø

(Energy/area) æ Q = sAtT 4 ö
29. Stefan’s constant (s) ç ÷ [ML0T–3K–4]
Time ´ (temperature) 4 çè E = Q / A.t = sT 4 ÷ø

Pressure ´ volume æ PV ö
30. Universal gas constant R ç ÷
Mole ´ temperature è nT ø [ML2 T -2 K -1mol-1 ]

Work æ W ö
31. Voltage, electric potential (V) or ç ÷ [ ML2T -3A -1 ]
Charge è q ø
electromotive force (e)
EBD_7179
12 PHYSICS

Charge æqö
32. Capacitance (C) ç ÷ [ M -1L-2T 4A 2 ]
Potential difference è V ø

r Electric force æ F ö
33. ( )
Electric field E Charge çè q ÷ø
[ MLT-3A -1 ]

r Force
34. Magnetic field(B), magnetic induction, [F = IlBsin q] [ML0T–2 A–1 ]
Current ´ length
magnetic flux density
35. Magnetic flux (fm) f = BAcosq [ML2T–2 A–1 ]
36. Inductance [ ML2T -2 A -2 ]
Magnetic flux æ f m ö
Current çè I ÷ø
coefficient of self inductance (L) or
coefficient of mutual inductance (M)
37. Magnetic field strength or
Magnetic moment æ M ö
magnetic moment density (I) ç ÷ [M0L–1 T0A]

.IN
Volume èVø

(Charge)2 æ q2 ö
ç ÷ é M -1L-3T 4A 2 ù
38. Permittivity constant in free space eo
AL
4π ´ electrostatic force(distance)2 çè 4p´ F ´ r 2 ÷
ø ë û

39. Faraday constant (F), charge Avagadro constant × elementry charge [M0 L0 TA mol-1 ]
N

[ML0 T 0 ]
R

40. Mass defect, (Dm) (Sum of masses of nucleons


– mass of nucleus)(MP+MN –Mnucleus)
U

1
JO

41. Resonant frequency (fr) Tr [M 0 L0 T -1 ]

æ1ö é M 0 L-1T0 ù
(Focal length)-1 ç ÷
U

42. Power of lens ë û


èf ø
ED

Speed of light in vacuum


43. Refractive index Speed of light in medium [M 0 L0 T 0 ]

2p é M 0 L-1T0 ù
44. Wave number
Wavelength ë û

45. Binding energy of nucleus Mass defect × (speed of light in vacuum)2 é ML2 T -2 ù
ë û
1
46. Conductance (c) é M -1L-2 A 2T 3 ù
Resistance ë û

æ pö (Pressure) ´ (radius) 4 é M 0 L3T –1 ù


47. Fluid flow rate çè ÷ø ë û
8 (Viscosity coefficient) ´ (length)

48. Inductive reactance (Angular frequency × inductance) é ML2T -3A -2 ù


ë û

49. Capacitive reactance (Angular frequency × capacitance)–1 é ML2T -3A -2 ù


ë û
Torque é M 0 L2 T0 A ù
50. Magnetic dipole moment or Current ´ area ë û
Magnetic field
Units and Measurements 13

Short cuts / Time saving techniques Keep in Memory


1. To find dimensions of a typical physical quantity which is
The dimensional formula of
involved in a number of formulae, try to use that formula
• all trigonometric ratio is [M0L0T0]
which is easiest for you. For example if you want to find the
• x in ex is [M0L0T0]
dimensional formula of magnetic induction then you can use
• ex is [M0L0T0]
the following formulae
• x in log x is [M0L0T0]
m 0 Idl sin q • log x is [M0L0T0]
dB = , B = m 0 nI, F = qvB, t = MB sin q
4p r 2 Example 1.
Out of these the easiest is probably the third one. Find out the unit and dimensions of permittivity of free
space.
2. If you have to find the dimensional formula of a combination Solution :
of physical quantities, then instead of finding the According to Coulomb’s law
dimensional formula of each, try to correlate the combination
é A 2T 2 ù
of physical quantities with a standard formula. For example, [q 2 ] ë û = [M -1L-3T 4 A 2 ]
if you have to find the dimension of CV2, then try to use e0 = =
-2
4p[F][r 2 ] [MLT ][L2 ]
1 2
formula E = CV where E is energy of a capacitor..
2 coulomb2 (coulomb)2 coulomb
Its unit = = =
1 newton ´ metre2 joule ´ metre volt ´ metre

.IN
3. = c = velocity of light in vacuum
m0e 0 CV–1m–1
• Dimensions of the following are same Example 2.
AL Find out the unit and dimensions of coefficient of self or
q2 V2 mutual inductance of.
Work = PV = nRT = qV = CV 2 = = ´ t = LI 2
c R Solution :
N
[ML2T–2] æ dI ö
• Dimensions of the following are same e = Læç dI ö÷ or M ç ÷ , where e is induced electromotive
R

è dt ø è dt ø
Force = Impulse / time
=qvB= qE force (e.m.f.)
U

= Thrust æ dt ö W æ t ö
\ L = eç ÷ = ç ÷
JO

= weight = energy gradient [MLT–2]


è dI ø q è I ø
L
• The dimension of RC = is same as that of time [ ML2 T -2 ][T ]
R = [ ML2 T - 2 A - 2 ]
U

or [L] =
• Dimensions of the following are same [ AT ][A ]
ED

Its unit is volt × sec/amp or ohm × sec or henry.


T 1 Example 3.
velocity = m = = f ´l T–1]
[M°LT
m0 e 0 Find out the unit and dimensions of magnetic field intensity .
Solution :
• Dimensions of the following are same
B 1 Idl sin q
R k MB 1 As B = mH, hence H = = ;
Frequency = = = = [M°L°T–1] m 4p r 2
L m I LC
[A][L]
• Dimensions of the following are same \H = = [M° L-1T° A]
(E) Modulus of elasticity = Y (Young's modulus) [L2 ]
= B (Bulk modulus) Its unit is ampere /metre in SI system. In c.g.s. system, the
= h (Modulus of rigidity) unit is oersted.
= Stress Example 4.
Find out the unit and dimensions of magnetic permeability
1 of free space or medium.
= Pressure =
Compressibility Solution :
[ML–1T–2] According to Biot-Savart’s law
• Dimensions of the following are same m 0 Idl sin q
Acceleration, retardation, centripetal acceleration, centrifugal B= and F = BI l sinq
4p r 2
acceleration, gravitational intensity/strength. [M°LT–2]
• Dimensions of the following are same F m Idl sin q
Water equivalent, thermal capacity, entropy, Boltzmann's
or =B= 0 ;
Il sin q 4p r 2
constant. [ML2T–2K–1]
EBD_7179
14 PHYSICS

(2) Checking the accuracy of various formulae


Fr 2 Example : Check the correctness of the following
\ m or m0 = (dimensionally)
I2 l 2 equation dimensionally
dv
[MLT -2 ][L2 ] F = ηA sinθ where F = force, h = coefficient of viscosity,,
Hence [m] or [m 0 ] = = [MLT -2 A - 2 ] dx
[A 2 ][L2 ]
dv
Its units are A = area, = velocity gradient w.r.t distance, q = angle of
dx
contact
N - m2 newton joule / metre volt ´ coulomb Sol. L.H.S = force = [MLT–2]
= = =
amp 2 - m 2 amp 2 amp 2 amp ´ amp ´ metre dv LT -1 é
R.H.S = h A (sin q) = M1L-1T -1L2 ´ = MLT -2 ù
dx L ë û
ohm ´ sec henry tesla ´ metre
= = = The equation is dimensionally correct.
metre metre amp.
Example 5. CAUTION : Please note that the above equation is not correct
The dimensions of physical quantity X in the equation numerically. The conclusion is that an equation, if correct
X dimensionally, may or may not be numerically correct. Also
Force = is given by remember that if an equation is dimensionally incorrect, we can
Density
conclude with surety that the equation is incorrect.
(a) [ML4T–2] (b) [M2L–2T–1]

.IN
2 –2
(c) [M L T ]–2 (d) [ML–2T–1] (3) Derivation of formula
Solution : Example : The air bubble formed by explosion inside water
AL
X performed oscillation with time period T which is directly
(c) Q Force =
denisty proportional to Pa db Ec where P is pressure, d is density and E
\ X = Force × density is the energy due to explosion. Find the values of a, b and c.
N
Hence X has dimensions Sol. Let us assume that the required expression for time period is
T = K Pa db Ec
R

[M]
[MLT–2] = [M2L–2T–2] where K is a dimensionless constant.
[L3 ]
U

Writing dimensions on both sides,


Example 6.
[M 0 L0 T1 ] = [ML-1T -2 ]a [ML-3 ]b [ML2 T -2 ]c
JO

If force, acceleration and time are taken as fundamental


quantities, then the dimensions of length will be- = [M]a + b + c [L]- a -3b + 2 c [T]-2a - 2c = [T1 ]
(a) [FT2] (b) [F–1A2T–1]
U

Equating the powers,


2
(c) [FA T] (d) [AT2] a+ b+ c=0 ....(1)
ED

Solution : -a - 3b + 2c = 0 ....(2)
(d) L = Fx A y Tz – 2a – 2c = 1 ....(3)
Solving these equations, we get,
M0 L1T 0 = [MLT -2 ]x [LT -2 ]y T z
5 1 1
x x + y -2x - 2y + z a= - ,b= ,c= .
= M L T 6 2 3
x = 0, x + y = 1, – 2x – 2y + z = 0 Limitations of Dimensional Analysis :
x = 0, y = 1, z = 2 (1) No information about the dimensionless constant is obtained
Hence, L = AT 2 during dimensional analysis
(2) Formula cannot be found if a physical quantity is dependent
DIMENSIONAL ANALYSIS AND ITS APPLICATIONS
on more than three physical quantities.
Principle of Homogeneity : (3) Formula containing trigonometrical /exponential function
Only those physical quantities can be added /subtracted/equated cannot be found.
/compared which have the same dimensions. (4) If an equation is dimensionally correct it may or may not be
Uses of Dimensions : absolutely correct.
(1) Conversion of one system of unit into another
Example : Convert a pressure of 106 dyne/cm2 in S.I units. Example 7.
Sol. We know that 1N = 105 dyne Þ 1 dyne = 10–5 N Find the dimensions of a and b in the Van der waal's
Also 1m = 100 cm Þ 1cm = 10–2 m æ a ö
Now, the pressure 10 dyne/cm2 in SI unit is
6 equation ç P + 2 ÷ (V - b) = RT
è V ø
dyne 10 -5 N
106 = 10 6 ´ = 105 N / m 2 where P is pressure and V is volume of gas.
cm ´ cm –2 -2
10 m ´ 10 m
Units and Measurements 15

Solution : Example 11.


A calorie is a unit of heat or energy and it equals about 4.2
Dimensionally P = a [By principle of homogeneity] J where 1J = 1 kg m2 s–2. Suppose we employ a system of
V2
units in which the unit of mass equals a kg, the unit of
a length equals b m, the unit of time is g s. Show that a calorie
Þ ML-1T -2 = Þ a = [ML5 T -2 ]
V2 has a magnitude 4.2 a–1 b –2 g2 in terms of the new units.
Also dimensionally V=b [By principle of homogeneity] Solution :
\ b = [L3] 1 cal = 4.2 kg m2 s–2.
Example 8. SI system New system
n1 = 4.2 n2 = ?
3mgl (1 - cos q) M1 = 1 kg M2 = a kg
The formula v2 =
M (1 + sin 2 q) is obtained as the L1 = 1m L2 = b metre
solution of a problem. Use dimensions to find whether T1 = 1s T2 = g second
this is a reasonable solution (v is a velocity, m and M are Dimensional formula of energy is [ML2T–2]
masses, l is a length and g is gravitational acceleration). Comparing with [MaLbTc], we find that
Solution : a = 1, b = 2, c = – 2
Dimensions of L.H.S. = [LT–1]2 = [L2T–2] a b c
éM ù éL ù éT ù

.IN
-2
[M] [LT ] [L] Now, n 2 = n1 ê 1 ú ê 1 ú ê 1 ú
Dimensions of R.H.S. = = [L2 T -2 ] ë M 2 û ë L 2 û ë T2 û
[M]
1 2 -2
Hence formula is reasonable. é 1 kg ù é 1m ù é 1s ù
AL
= 4.2 ê ú ê ú ê ú
Example 9. ë a kg û ë bm û ë gs û
é n - n1 ù = 4.2 a–1 b –2 g2
N
In the formula; N = - D ê 2 ú,
ë x 2 - x1 û SIGNIFICANT FIGURES
R

D = diffusion coefficient, n1 and n2 is number of molecules The number of digits, which are known reliably in our
in unit volume along x1 and x2 which represents distances measurement, and one digit that is uncertain are termed as
U

where N is number of molecules passing through per unit significant figures.


JO

area per unit time calculate the dimensions of D. Rules to determine the numbers of significant figures:
Solution : 1. All non-zero digits are significant. 235.75 has five significant
By homogeneity theory of dimension figures.
U

2. All zeroes between two non-zero digits are significant.


Dimension of (N) 2016.008 has seven significant figures.
ED

3. All zeroes occurring between the decimal point and the non-
dimension of (n 2 - n1 )
= Dimension of D × zero digits are not significant. provided there is only a zero
dimension of (x 2 - x1 ) to left of the decimal point. 0.00652 has three significant
figures.
1 L-3 4. All zeroes written to the right of a non-zero digit in a number
= Dimension of D ×
L2T L written without a decimal point are not significant. This rule
L L2 does not work if zero is a result of measurement. 54000 has
Þ Dimensions of 'D' = -3 2 = = [L2T–1] two significant figures whereas 54000m has five significant
L ´L T T
Example 10. figures.
5. All zeroes occurring to the right of a non-zero digit in a
1
Let us consider an equation mv 2 = mgh where m is the number written with a decimal point are significant. 32.2000
2 has six significant figures.
mass of the body, v its velocity, g is the acceleration due to 6. When a number is written in the exponential form, the
gravity and h is the height. Check whether this equation is exponential term does not contribute towards the significant
dimensionally correct. figures. 2.465 × 105 has four significant figures.
Solution : Keep in Memory
The dimensions of LHS are
[M] [L T–1 ]2 = [M] [ L2 T–2] = [M L2 T–2] 1. The significant figures depend upon the least count of the
instrument.
The dimensions of RHS are
2. The number of significant figure does not depend on the
[M][L T–2] [L] = [M][L2 T–2] = [M L2 T–2]
units chosen.
The dimensions of LHS and RHS are the same and hence
the equation is dimensionally correct.
EBD_7179
16 PHYSICS

ROUNDING OFF The following diagrams illustrate the meaning of terms accuracy
1. If digit to be dropped is less than 5 then preceding digit and precision :
should be left unchanged.
2. If digit to be dropped is more than 5 then one should raise
preceding digit by one.
3. If the digit to be dropped is 5 followed by a digit other than
zero then the preceding digit is increased by one.
4. If the digit to be dropped is 5 then the preceding digit is not
changed if it is even.
5. If digit to be dropped is 5 then the preceding digit is increased
by one if it is odd.
Arithmetical Operations with Significant Figures and
Rounding off :
(1) For addition or subtraction, write the numbers one below
In the above figure : The centre of the target represents the

.IN
the other with all the decimal points in one line. Now locate accepted value. The closer to the centre, the more accurate the
the first column from the left that has a doubtful digit. All experiment. The extent of the scatter of the data is a measure of
digits right to this column are dropped from all the numbers the precision.
AL
and rounding is done to this column. Addition subtraction A- Precise and accurate, B- Accurate but imprecise, C- Precise but
not accurate, D- Not accurate nor precise
is then done. When successive measurements of the same quantity are repeated
N
Example : Find the sum of 23.623 and 8.7 to correct there are different values obtained. In experimental physics it is
significant figures. vital to be able to measure and quantify this uncertainty. (The
R

words "error" and "uncertainty" are often used interchangeably


Sol. Step-1 :- 23.623 + 8.7 Step-2 :- 23.6 + 8.7=32.3 by physicists - this is not ideal - but get used to it!)
U

(2) In multiplication and division of two or more quantities, the Error in measurements is the difference of actual or true value
JO

and measured value.


number of significant digits in the answer is equal to the
Error = True value – Measured value
number of significant digits in the quantity, which has
Keep in Memory
minimum number of significant digits.
U

The insignificant digits are dropped from the result if they 1. Accuracy depends on the least count of the instrument used
ED

for measurement.
appear after the decimal point. They are replaced by zeroes
2. In the addition and subtraction operation, the result contains
if they appear to the left of the decimal point. The least
the minimum number of decimal places of the figures being
significant digit is rounded off. used
Example : 107.88 (5. S. F.) 3. In the multiplication and division operation, the result
contains the minimum number of significant figures.
× 0.610 (3 S. F.)
4. Least count (L.C.) of vernier callipers = one MSD – one VSD
= 65.8068 @ 65.8
where MSD = mains scale division
ACCURACY, PRECISION OF INSTRUMENTS AND ERRORS VSD = vernier scale division
IN MEASUREMENTS : 5. Least count of screw gauge (or spherometer)
Accuracy and Precision are two terms that have very different pitch
=
meanings in experimental physics. We need to be able to no of divisions on circular scale
distinguish between an accurate measurement and a precise where pitch is the ratio of number of divisions moved on
measurement. An accurate measurement is one in which the linear scale and number of rotations given to circular scale.
results of the experiment are in agreement with the ‘accepted’ 6. Pure number or unmeasured value do not have significant
value. This only applies to experiments where this is the goal like numbers
7. Change in the position of decimal does not change the
measuring the speed of light.
number of significant figures.
A precise measurement is one that we can make to a large number Similarly the change in the units of measured value does not
of decimal places. change the significant figures.
Units and Measurements 17

Example 12. Methods of Expressing Error:


Each side of a cube is measured to be 7.203 m. What are the Absolute error: It is the difference between the mean value and
total surface area and the volume of the cube to appropriate the measured value of the physical quantity.
significant figures? |D X1| = |X mean–X1|
..................................
Solution : ..................................
The number of significant figures in the measured length |DX n| = |Xmean–Xn|
7.203 m is 4. The calculated area and the volume should Mean absolute error:
therefore be rounded off to 4 significant figures. | DX1 | + | DX 2 | +.........+ | DX n |
Surface area of the cube = 6(7.203)2 m2 DXmean or D X =
n
= 311.299254 m2 = 311.3 m2 Relative error: It is the ratio of the mean absolute error and the
Volume of the cube = (7.203)3 m3 = 373.714754 m3 value of the quantity being measured.
= 373.7 m3 DX mean
Example 13. Relative error (da) =
X mean
5.74 g of a substance occupies volume 1.2 cm3. Express its Percentage error: It is the relative error expressed in percent
density by keeping the significant figures in view.
Solution : DX
Percentage error = ´ 100%
There are 3 significant figures in the measured mass whereas X
there are only 2 significant figures in the measured volume. To find the maximum error in compound quantities we proceed as :
Hence the density should be expressed to only 2 significant (i) Sum and difference : We have to find the sum or difference

.IN
figures. of two values given as (a ± Da) and (b ± Db), we do it as
follows
mass 5.74 X ± DX = (a ± Da) + (b ± Db) = (a + b) ± (Da + Db)
Density = = gcm -3 = 4.8 g cm–3 .
volume 1.2
AL
Þ X = a + b and DX = Da + Db in case of sum
And X = (a – b) and DX = Da + Db in case of difference.
Example 14.
(ii) Product and quotient : We add the fractional or percentage
The mass of a box measured by a grocer’s balance is 2.300
N
errors in case of finding product or quotient.
kg. Two gold pieces of masses 20.15 g and 20.17 g are added
DP æ Da Db ö
R

to the box. What is (a) the total mass of the box, (b) the If P = ab then = ±ç + ÷
difference in the masses of the pieces to correct significant P è a bø
U

figures ? a DQ æ Da Db ö
Solution : If Q = then = ±ç + ÷
b Q è a bø
JO

(a) Total mass = 2.3403 kg = 2.3 kg (upto 2 S. F.)


DX æ Da ö
(b) Difference = 20.17 g – 20.15 g (upto 4 S. F.) (iii) Power of a quantity : If x = an then = nç ÷
X è a ø
COMMON ERRORS IN MEASUREMENTS
U

Example :
It is not possible to measure the 100% correct value of any physical Dy
x2 Dx
ED

quantity, even after measuring it so many times. There always For Q = , If ´ 100 = 3% and ´ 100 = 4%
y x y
exists some uncertainty, which is usually referred to as experimental
error. DQ
Experimental errors : then × 100 = (2 × 3 + 4)% = 10%
Q
(i) Random error : It is the error that has an equal chance of Similarly :
being positive or negative.
mp nq DA é Dm Dn Dc ù
It occurs irregularly and at random in magnitude and If A = then = ± êp +q +r ú
direction. It can be caused cr A ë m n c û
(a) by the lack of perfection of observer Keep in Memory
(b) if the measuring instrument is not perfectly sensitive.
1. More the accuracy, smaller is the error.
(ii) Systematic error : It tends to occur in one direction either
positive or negative. It occurs due to 2. Absolute error |DX| is always positive.
(a) measuring instrument having a zero error. 3. |DX| has the same dimensions as that of X.
(b) an instrument being incorrectly calibrated (such as 4. If the least count of measuring instrument is not given and
slow- running-stop clock) the measured value is given the least error in the measurement
(c) the observer persistently carrying out a mistimed action can be found by taking the last digit to be 1 and rest digit to
(e.g., in starting and stopping a clock) be zero. For e.g. if the measured value of mass m = 2.03 kg
For measuring a particular physical quantity, we take a number of then Dm = ±0.01 kg.
readings. Let the readings be X1, X2............,Xn. Then the mean 5. If a number of physical quantities are involved in an
value is found as follows expression then the one with higher power contributes more
X + X 2 + ..... + X n in errors and therefore should be measured more accurately.
X mean (or true value) = X = 1 6. Relative error is a dimensionless quantity.
n
EBD_7179
18 PHYSICS

7. We are always interested in calculating the maximum possible Example 17.


error. A body travels uniformly a distance of (13.8 ± 0.2) m in a
Example 15. time (4.0 ± 0.3) s. Calculate its velocity with error limits.
What is percentage error in velocity?
In an experiment, the refractive index of water was observed
Solution :
as 1.29, 1.33, 1.34, 1.35, 1.32, 1.36, 1.30 and 1.33. Calculate the
Here, s = (13.8 ± 0.2) m, t = (4.0 ± 0.3) s
mean value, mean absolute error and percentage error in the
measurement. s 13.8
velocity, v= = = 3.45 ms–1 = 3.5 ms–1
Solution : t 4.0
Mean value of refractive index, (rounding off to two significant figures)
1.29 + 1.33 + 1.34 + 1.35 + 1.32 + 1.36 + 1.30 + 1.33
m=
8 Dv æ Ds Dt ö
=±ç + ÷ =±ç
æ 0.2 0.3 ö (0.8 + 4.14)
+ =±
= 1.33 v è s t ø è 13.8 4.0 ÷ø 13.8 ´ 4.0
Absolute error in measurement,
Dm1 = 1.33 - 1.29 = +0.04 , Dm 2 = 1.33 - 1.33 = 0.00 , Dv 4.94
Þ =± = ±0.0895
v 13.8 ´ 4.0
Dm3 = 1.33 - 1.34 = -0.01 , Dm 4 = 1.33 - 1.35 = -0.02 ,
D v = ± 0.0895 × v = ± 0.0895 × 3.45 = ± 0.3087 = ± 0.31

.IN
Dm5 = 1.33 - 1.32 = +0.01 , Dm 6 = 1.33 - 1.36 = -0.03 , (Rounding off to two significant fig.)
Hence, v = (3.5 ± 0.31) ms–1
Dm 7 = 1.33 - 1.30 = +0.03 , Dm8 = 1.33 - 1.33 = 0.00
% age error in velocity
So, mean absolute error,
AL
Dv
| 0.04 | + | 0.01| + | 0.02 | + | 0.01| + | 0.03 | + | 0.03 | + | 0 | = ´ 100 = ± 0.0895 × 100 = ± 8.95 % = ± 9%
( Dm ) = v
N
8
Example 18.
= 0.0175 » 0.02
R

Two resistances are expressed as R1 = (4 ± 0.5) W and


Dm 0.02 R2 = (12 ± 0.5) W. What is the net resistance when they are
U

Relative error = ± =+ = ±0.015 = ±0.02 connected (i) in series and (ii) in parallel, with percentage
m 1.33
error ?
JO

0.02 (a) 16W ± 23%, 3W ± 6.25% (b) 3W ± 2.3%, 3W ± 6.25%


Percentage error = ± ´ 100% = ±1.5% (c) 3W ± 23%, 16W ± 6.25% (d) 16W ± 6.25%, 3W ± 23%
1.33
U

Solution :
Example 16.
ED

The length and breadth of a rectangle are (5.7 ± 0.1) cm and R1 R 2 RR


(d) R S = R1 + R 2 = 16 W ; R P = = 1 2 =3W
(3.4 ± 0.2) cm. Calculate area of the rectangle with error limits. R1 + R 2 RS
Solution :
Here, l = (5.7 ± 0.1) cm, b = (3.4 ± 0.2) cm DR S = DR1 + DR 2 = 1 W
Area A = l × b
DR S 1
= 5.7 × 3.4 = 19.38 cm² Þ R ´ 100 = 16 ´ 100% = 6.25%
= 19.0 cm² (Rounding off to two significant figures) S

DA æ Dl Db ö æ 0.1 0.2 ö Þ R S = 16W ± 6.25%


= ± çè + ÷ø = ± çè + ÷
A l b 5.7 3.4 ø
R1 R 2
Similarly, R P =
æ 0.34 + 1.14 ö RS
= ± çè ÷
5.7 ´ 3.4 ø
DR P DR1 DR 2 DR S
= + +
DA 1.48 1.48 RP R1 R2 RS
=± Þ DA = ± ´A
A 19.38 19.38
DR P 0.5 0.5 1
Þ = + + = 0.23
1.48 RP 4 12 16
=± ´ 19.38 = ±1.48
19.38
DA = ± 1.5 (Rounding off to two significant figures) DR P
Þ ´ 100 = 23% Þ R P = 3W ± 23%
\ Area = (19.0 ± 1.5) sq.cm. RP
Units and Measurements 19

.IN
AL
N
R
U
JO
U
ED
EBD_7179
20 PHYSICS

1. Temperature can be expressed as derived quantity in terms 13. Light year is


of (a) light emitted by the sun in one year.
(a) length and mass (b) time taken by light to travel from sun to earth.
(b) mass and time (c) the distance travelled by light in free space in one year.
(c) length, mass and time (d) time taken by earth to go once around the sun.
(d) None of these 14. The SI unit of pressure is
2. What is the unit of “a” in Vander Waal’s gas equation? (a) atmosphere (b) bar
(a) Atm litre–2 mol2 (b) Atm litre2 per mol (c) pascal (d) mm of Hg
(c) Atm litre mol–1 2 (d) Atm litre2 mol–2 15. Electron volt is a unit of
3. Random error can be eliminated by (a) potential difference (b) charge
(a) careful observation (c) energy (d) capacity
(b) eliminating the cause 16. Dimensions of impulse are
(c) measuring the quantity with more than one instrument (a) [ML T -1 ] (b) [MLT2 ]

.IN
(d) taking large number of observations and then their
mean. (c) [ MT -2 ] (d) [ML-1 T -3 ]
4. If e is the charge, V the potential difference, T the temperature,
17. The S.I. unit of pole strength is
eV
AL (a) Am2 (b) Am
then the units of are the same as that of
T (c) A m–1 (d) Am–2
18. Which is dimensionless?
N
(a) Planck’s constant (b) Stefan’s constant
(c) Boltzmann constant (d) gravitational constant (a) Force/acceleration (b) Velocity/acceleration
R

5. If f = x2, then the relative error in f is (c) Volume/area (d) Energy/work


2 Dx 19. Potential is measured in
(Dx ) 2
U

(a) (b) (a) joule/coulomb (b) watt/coulomb


x x
JO

(c) newton-second (d) None of these


Dx 20. Maxwell is the unit of
(c) (d) (Dx)2
x (a) magnetic susceptibility
U

6. Two quantities A and B have different dimensions which (b) intensity of Magnetisation
mathematical operation given below is physically (c) magnetic Flux
ED

meaningful? (d) magnetic Permeability


(a) A/B (b) A + B 21. The mass of the liquid flowing per second per unit area of
(c) A – B (d) A = B cross-section of the tube is proportional to (pressure
7. Which of the following systems of units is not based on difference across the ends)n and (average velocity of the
units of mass, length and time alone liquid)m. Which of the following relations between m and n
(a) SI (b) MKS is correct?
(b) CGS (d) FPS (a) m = n (b) m = – n
8. Unit of latent heat is 2
(c) m = n (d) m = – n2
(a) J Kg–1 (b) J mol–1 22. Which of the following is a derived physical quantity?
(c) N Kg –1 (d) N mol–1 (a) Mass (b) Velocity
9. Dyne-sec is the unit of (c) Length (d) Time
(a) momentum (b) force 23. N kg–1 is the unit of
(c) work (d) angular momentum (a) velocity (b) force
10. Illuminance of a surface is measured in (c) acceleration (d) None of these
(a) Lumen (b) candela 24. Which physical quantities have same dimensions?
(c) lux (d) lux m–2 (a) Moment of couple and work
11. SI unit of electric polarisation is (b) Force and power
(a) Cm–2 (b) coulomb (c) Latent heat and specific heat
(c) ampere (d) volt (d) Work and power
12. The SI unit of coefficient of mutual inductance of a coil is 25. The expression [ML–1 T–2] does not represent
(a) henry (b) volt (a) pressure (b) power
(c) farad (c) weber (c) stress (d) Young’s modulus
Units and Measurements 21

1. What are the units of magnetic permeability? 12. The potential energy of a particle varies with distance x from
(a) Wb A–1 m–1 (b) Wb–1 Am
–1 A x
(c) Wb A m (d) Wb A–1 m a fixed origin as V = where A and B are constants.
x+B
2. The dimensions of pressure gradient are
(a) [ML–2 T–2] (b) [ML–2 T–1] The dimensions of AB are
(c) [ML–1 T–1] (d) [ML–1 T–2] (a) [M1 L5/ 2 T -2 ] (b) [M1 L2 T -2 ]
3. The dimensions of Rydberg’s constant are
(a) [M0 L–1 T] (b) [MLT–1] (c) [M 3/ 2 L5/ 2 T -2 ] (d) [M1 L7 / 2 T -2 ]
(c) [M0 L–1 T0] (d) [ML0 T2] 13. Distance travelled by a particle at any instant ‘t’ can be
4. The dimensions of universal gas constant are represented as S = A (t + B) + Ct 2. The dimensions of B are
(a) [L2 M1 T–2 K–1] (b) [L1 M2 T–2 K–1] (a) [M 0 L1 T -1 ] (b) [M 0 L0 T1]
1 1
(c) [L M T K ] –2 –1 (d) [L2 M2 T–2 K–1]
[M 0 L-1 T -2 ] (d) [M 0 L2 T -2 ]

.IN
5. The dimensions of magnetic moment are (c)
(a) [L2 A1] (b) [L2 A–1] 14. The deBroglie wavelength associated with a particle of mass
2
(c) [L / A ] 3 (d) [LA2]
AL m and energy E is h / 2 m E . The dimensional formula of
6. The dimensions of Wien’s constant are
(a) [ML0 T K] (b) [M0 LT0 K] Planck’s constant h is
(c) [M0 L0 T K]
N
(d) [MLTK] (a) [M 2 L2 T -2 ] (b) [ M L2 T -1 ]
7. The unit and dimensions of impedance in terms of charge Q
R

are (c) [M LT -2 ] (d) [M L2 T -2 ]


(a) mho, [ML2 T–2 Q–2] (b) ohm, [ML2 T–1 Q–2]
U

15. The velocity of a body which falls under gravity varies as ga


(c) ohm, [ML2 T–2 Q–1] (d) ohm, [MLT–1 Q–1] hb, where g is acc. due to gravity and h is the height. The
JO

8. If L denotes the inductance of an inductor through which a values of a and b are


current i is flowing, the dimensions of L i2 are (a) a = 1, b = 1/2 (b) a = b = 1
(a) [ML2 T–2] (c) a = 1/2, b = 1 (d) a = 1/2; b = 1/2
U

(b) [MLT–2] b
16. The velocity v of a particle at time t is given by v = a t +
ED

(c) [M2 L2 T–2] t+c


(d) Not expressible in M, L, T The dimensions of a, b c are respectively
9. The dimensional formula of wave number is (a) [LT–2], [L], [T] (b) [L2], [T] and [LT2]
(a) [M0 L0 T–1] (b) [M0 L–1 T0] (c) [LT2], [LT] and [L] (d) [L], [LT] and [T2]
–1
(c) [M L T ] –1 0 (d) [M0 L0 T0] 17. The dimensions of Hubble’s constant are
10. The period of a body under S.H.M. is represented by: T = Pa
Db Sc where P is pressure, D is density and S is surface (a) [T -1 ] (b) [M 0 L0 T -2 ]
tension, then values of a, b and c are
(c) [MLT 4 ] (d) [MT - 1 ]
3 1
(a) - , , 1 (b) -1, - 2, 3 18. Error in the measurement of radius of a sphere is 1%. Then
2 2
error in the measurement of volume is
1 3 1
(c) ,- ,- (d) 1, 2 1/3 (a) 1% (b) 5%
2 2 2 (c) 3% (d) 8%
11. The time of oscillation T of a small drop of liquid depends 19. Subtract 0.2 J from 7.26 J and express the result with correct
on radius r, density r and surface tension S. The relation number of significant figures.
between them is given by
(a) 7.1 J (b) 7.06 J
S r r3 (c) 7.0 J (d) 7 J
(a) Tµ (b) Tµ 20. Multiply 107.88 by 0.610 and express the result with correct
r r3 S
number of significant figures.
S2 r 3 r r3 (a) 65.8068 (b) 65.807
(c) Tµ (d) Tµ
r S (c) 65.81 (d) 65.8
EBD_7179
22 PHYSICS

21. When 97.52 is divided by 2.54, the correct result is (a) M–1 L–3 T4 A2 (b) ML3 T–4 A–2
(a) 38.3937 (b) 38.394 (c) M0 L0 T0 A0 (d) M–1 L–3 T2 A
(c) 38.39 (d) 38.4 33. The dimensional formula for entropy is
22. Relative density of a metal may be found with the help of (a) [MLT–2 K1] (b) [ML2 T–2]
spring balance. In air the spring balance reads (5.00 ± 0.05) 2
(c) [ML T K ]–2 –1 (d) [ML2 T–2 K]
N and in water it reads (4.00 ± 0.05) N. Relative density
34. Dimensions of specific heat are
would be
(a) [ML2 T–2 K] (b) [ML2 T–2 K–1]
(a) (5.00 ± 0.05)N (b) (5.00 ± 11%)
(c) [ML2 T2 K–1] (d) [L2 T–2 K–1]
(c) (5.00 ± 0.10) (d) (5.00 ± 6%)
35. L, C, R represent physical quantities inductance, capacitance
23. Area of a square is (100 ± 2) m2. Its side is
and resistance respectively. The combinations which have
(a) (10 ± 1) m (b) (10 ± 0.1) m
the dimensions of frequency are
(c) (10 ± 2 )m (d) 10 ± 2 % (a) 1/RC (b) R/L
24. Let Q denote the charge on the plate of a capacitor of (c) 1 / LC (d) C/L
Q2 36. The physical quantity which has the dimensional formula
capacitance C. The dimensional formula for is
C [M1T–3] is
(a) [L2 M 2 T ] (b) [LMT 2 ] (a) surface tension (b) solar constant

.IN
(c) density (d) compressibility
(c) [L2 MT -2 ] (d) [L2 M 2 T 2 ] 37. Which of the following is the most accurate?
25. If L and R denote inductance and resistance then dimension (b) 20 × 101 m
AL (a) 200.0 m
of L/R is 2
(c) 2 × 10 m (d) data is inadequate
(a) [M 0 L0 T 0 ] (b) [M 0 L0 T ] 38. The velocity of water waves (v) may depend on their
N
wavelength l, the density of water r and the acceleration
[M 2 L0 T 2 ] (d) [MLT 2 ] due to gravity, g. The method of dimensions gives the
R

(c)
relation between these quantities is
26. The dimensional formula of current density is
U

(a) v (b) v2 µ g l
(a) [M 0 L-2 T -1Q] (b) [M 0 L2 T1Q -1 ]
JO

(c) v2 µ g l 2 (d) v2 µ g -1 l 2
(c) [ MLT -1Q] -2 -1 2
(d) [ML T Q ]
39. The time dependence of a physical quantity p is given by
U

27. The least count of a stop watch is 0.2 second. The time of 20
oscillations of a pendulum is measured to be 25 second. p = p0 exp. (– a t2), where a is a constant and t is the time.
ED

The percentage error in the measurement of time will be The constant a


(a) 8% (b) 1.8% (a) is dimensionless (b) has dimensions T–2
(c) has dimensions T 2 (d) has dimensions of p.
(c) 0.8% (d) 0.1%
28. The dimensional formula for relative density is
æ a ö
(a) [M L–3] (b) [M0 L–3] 40. In the eqn. ç P + 2 ÷ (V - b) = constant, the unit of a is
0 0 –1 è V ø
(c) [M L T ] (d) [M0 L0 T0]
29. The solid angle sustended by the total surface area of a (a) dyne × cm5 (b) dyne × cm4
sphere, at the centre is (c) dyne/cm3 (d) dyne × cm2
(a) 4p (b) 2p 41. Dimensions of ‘ohm’ are same as (where h is Planck’s
(c) p (d) 3p constant and e is charge)
30. If C and L denote the capacitance and inductance, the
h h2 h h2
dimensions of LC are (a) (b) (c) (d)
2
(a) [M0 L0 T–1] (b) [M0 L–1 T0] e e e e2
–1
(c) [M L T ] –1 0 (d) [M0 L0 T2] 42. The Richardson equation is given by I = AT2e–B/kT. The
31. The dimensions of solar constant is dimensional formula for AB2 is same as that for
(a) [M0 L0 T0] (b) [MLT–2] (a) I T2 (b) kT
2 –2 2 (d) I k2/T
(c) [ML T ] (d) MT –3 (c) I k
43. The unit of current in C.G..S. system is
1 e2 (a) 10 A (b) 1/10 A
32. The dimensions of are
Îo hc (c) 1/100 A (d) 1/1000 A
Units and Measurements 23

44. Which of the following do not have the same dimensional (a) 10% (b) 13%
formula as the velocity? (c) 30% (d) 20%
Given that m0 = permeability of free space, e0 = permittivity 52. Using mass (M), length(L), time (T) and electric current (A)
of free space, n = frequency, l = wavelength, P = pressure, r as fundamental quantities the dimensions of permittivity
= density, w = angular frequency, k = wave number, will be
(a) [MLT–1 A–1 ] (b) [MLT–2 A–2 ]
(a) 1 m 0 eo (b) n l –1 –3
(c) [M L T A ] +4 2 (d) [M2L–2 T –2 A2 ]
53. The percentage errors in the measurement of mass and speed
(c) P/r (d) wk are 2% and 3% respectively. The error, in kinetic energy
45. A cube has numerically equal volume and surface area. The obtained by measuring mass and speed, will be
volume of such a cube is (a) 12 % (b) 10 %
(a) 1000 unit (b) 200 unit (c) 8 % (d) 2 %
(c) 216 unit (d) 300 unit 54. The density of a cube is measured by measuring its mass
and length of its sides. If the maximum error in the
46. A spherical body of mass m and radius r is allowed to fall in
measurement of mass and length are 4% and 3% respectively,
a medium of viscosity h. The time in which the velocity of
the maximum error in the measurement of density will be
the body increases from zero to 0.63 times the terminal
(a) 7% (b) 9%
velocity (v) is called time constant t. Dimensionally t can be
(c) 12% (d) 13%
represented by

.IN
gRT
55. The speed of sound in a gas is given by v =
æ 6pmrh ö M
mr 2 ç ÷ R = universal gas constant,
(a) (b)
6ph ç g2 ÷
è ø
AL T = temperature
M = molar mass of gas
m The dimensional formula of g is
N
(c) 6 ph r v
(d) None of these
(a) [M 0 L0 T 0 ] (b) [M 0 LT -1 ]
R

47. A quantity is represented by X = Ma Lb Tc. The % error in


[ MLT -2 ] (d) [M 0 L0 T -1 ]
U

measurement of M, L and T are a%, b% and g% (c)


respectively. The % error in X would be 56. Specific gravity has ............ dimensions in mass, ............
JO

(a) (a a + b b + g c ) % (b) (a a - b b + g c) % dimensions in length and ............ dimensions in time.


(a) 0, 0, 0 (b) 0, 1, 0
(c) (a a - b b - g c) ´100% (d) None of these (c) 1, 0, 0 (d) 1, 1, 3
U

48. In a Vernier calliper, N divisions of vernier scale coincide 57. If I is the moment of inertia and w the angular velocity,,
ED

with (N–1) divisions of main scale (in which one division what is the dimensional formula of rotational kinetic energy
represents 1 mm). the least count of the instrument in cm. 1 2
Iw ?
should be 2
(a) N (b) N – 1
(a) [ ML2T -1 ] (b) [ M 2 L-1T -2 ]
1 1
(c)
10 N
(d) (c) [ ML2 T - 2 ] (d) [ M 2 L-1T -2 ]
N -1
58. Given that r = m2 sin pt , where t represents time. If the unit
49. What is the fractional error in g calculated from of m is N, then the unit of r is
T = 2 p l / g ? Given fraction errors in T and l are ± x and (a) N (b) N2
± y respectively.. (c) Ns (d) N 2 s
(a) x + y (b) x – y 59. The dimensional formula of farad is
(c) 2x + y (d) 2x – y
50. Conversion of 1 MW power in a New system of units having (a) [M -1L-2 TQ] (b) [M -1L-2T 2Q 2 ]
basic units of mass, length and time as 10 kg, 1 dm and 1
(c) [M -1L-2 TQ 2 ] (d) [ M -1L-2 T 2 Q ]
minute respectively is
(a) 2.16 × 1010 unit (b) 2 × 104 unit 60. If time T, acceleration A and force F are regarded as base
units, then the dimensional formula of work is
(c) 2.16 × 1012 unit (d) 1.26 × 1012 unit
51. A resistor of 10 k W having tolerance 10% is connected in (a) [FA] (b) [FAT]
series with another resistor of 20 k W having tolerance 20%.
The tolerance of the combination will be
(c) [FAT 2 ] (d) [FA 2 T]
EBD_7179
24 PHYSICS

61. Turpentine oil is flowing through a capillary tube of length


æ Da Db ö æ Da Db ö
l and radius r. The pressure difference between the two (c) ç + ÷ ´ 100% (d) ç - ÷ ´100%
ends of the tube is p. The viscosity of oil is given by : èa-b a -bø èa-b a -bø

p(r 2 - x 2 ) 69. The heat generated in a circuit is given by Q = I 2 Rt, where


h= . Here v is velocity of oil at a distance x from I is current, R is resistance and t is time. If the percentage
4vl
the axis of the tube. From this relation, the dimensional errors in measuring I, R and t are 2%, 1% and 1% respectively,
formula of h is then the maximum error in measuring heat will be
(a) 2% (b) 3%
(a) [ML-1T -1 ] (b) [ MLT -1 ] (c) 4% (d) 6%
(c) [ ML2 T -2 ] (d) [M 0 L0 T 0 ] 70. The pressure on a square plate is measured by measuring
the force on the plate and length of the sides of the plate by
62. The dimensional formula of velocity gradient is
F
(a) [M 0 L0 T -1 ] (b) [ MLT1 ] using the formula P = . If the maximum errors in the
l2
(c) [ML0T -1 ] (d) [ M 0 LT -2 ] measurement of force and length are 4% and 2%
respectively, then the maximum error in the measurement of
63. The thrust developed by a rocket-motor is given by
pressure is
F = mv + A ( P1 - P2 ) where m is the mass of the gas ejected
(a) 1% (b) 2%

.IN
per unit time, v is velocity of the gas, A is area of cross-
(c) 8% (d) 10%
section of the nozzle, P1 and P2 are the pressures of the
71. In a simple pendulum experiment for the determination of
exhaust gas and surrounding atmosphere. The formula is AL acceleration due to gravity, time period is measured with an
dimensionally accuracy of 0.2% while length was measured with an
(a) correct accuracy of 0.5%. The percentage accuracy in the value of
(b) wrong g so obtained is
N
(c) sometimes wrong, sometimes correct (a) 0.25% (b) 0.7%
R

(d) Data is not adequate (c) 0.9% (d) 1.0%


64. If E, m, J and G represent energy, mass, angular momentum 72. The dimensions of a rectangular block measured with
U

and gravitational constant respectively, then the callipers having least count of 0.01 cm are 5 mm × 10 mm × 5
JO

mm. The maximum percentage error in the measurement of


dimensional formula of EJ 2 / m5G 2 is
the volume of the block is
(a) angle (b) length
(a) 5% (b) 10%
(c) mass (d) time
U

(c) 15% (d) 20%


65. In a vernier callipers, ten smallest divisions of the vernier
73. Consider the following pairs of quantities
ED

scale are equal to nine smallest division on the main scale. If


1. Young's modulus; pressure
the smallest division on the main scale is half millimeter,
then the vernier constant is 2. Torque; energy
(a) 0.5 mm (b) 0.1 mm 3. Linear momentum; work
(c) 0.05 mm (d) 0.005 mm 4. Solar day; light year.
66. A vernier calliper has 20 divisions on the vernier scale, which In which cases are the dimensions, within a pair, same?
coincide with 19 on the main scale. The least count of the (a) 1 and 3 (b) 1 and 4
instrument is 0.1 mm. The main scale divisions are of (c) 1 and 2 (d) 2 and 4
(a) 0.5 mm (b) 1mm 74. Which one of the following has the same dimension as that
of time, if R is resistance, L inductance and C is capacitance?
1 (a) RC (b) LC
(c) 2 mm (d) mm
4
(c) L (d) All of the above
67. The pitch of the screw gauge is 0.5 mm. Its circular scale R
contains 50 divisions. The least count of the screw gauge is 75. The equation of a wave is given by
(a) 0.001 mm (b) 0.01 mm
(c) 0.02 mm (d) 0.025 mm æx ö
y = a sin w ç - k ÷ , where w is angular velocity and v is
68. If x = a – b, then the maximum percentage error in the è v ø
measurement of x will be linear velocity. The dimensions of K will be
(a) [T2] (b) [T–1]
æ Da Db ö æ Da Db ö
(a) ç + ÷ ´100% (b) ç - ÷ ´ 100% (c) [T] (d) [L T]
è a b ø è a b ø
Units and Measurements 25

76. In the equation X = 3YZ2, X and Z are dimensions of 84. The density of a material in CGS system of units is 4g/cm3.
capacitance and magnetic induction respectively. In MKSQ In a system of units in which unit of length is 10 cm and unit
system, the dimensional formula for Y is of mass is 100 g, the value of density of material will be
(a) [M–3 L–2 T–2 Q–4] (b) [M L–2] (a) 0.4 (b) 40
–3 –2
(c) [M L Q T ] 4 8 (d) [M–3 L–2 Q4 T4] (c) 400 (d) 0.04
77. A force is given by F = at + bt2 , where t is time, the 85. In an experiment four quantities a, b, c and d are measured
dimensions of a and b are with percentage error 1%, 2%, 3% and 4% respectively.
Quantity P is calculated as follows
(a) [M L T–4] and [M L T–1]
(b) [M L T–1] and [M L T0] a 3b 2
P=
(c) [M L T–3] and [M L T–4] cd
(d) [M L T–3] and [M L T0] % error in P is
n (a) 10% (b) 7%
78. The frequency of vibration of a string is given by f =
2L (c) 4% (d) 14%
T 86. What is the fractional error in g calculated from
, where T is tension in the string, L is the length, n is
m
number of harmonics. The dimensional formula for m is T = 2 p l / g ? Given fractional errors in T and l are ± x
(a) [M0 L T] (b) [M1 L–1 T–1] and ± y respectively..

.IN
(c) [M1 L–1 T0] (d) [M0 L T –1] (a) x + y (b) x – y
79. The dimensions of voltage in terms of mass (M), length (L) (c) 2x + y (d) 2x – y
and time (T) and ampere (A) are rl
AL
87. The resistance R of a wire is given by the relation R = .
(a) [ML2T–2 A–2 ] (b) [ML2T 3A–1 ] p r2
Percentage error in the measurement of r, l and r is 1%, 2%
(c) [ML2T –3A1 ] (d) [ML2T–3 A–1 ] and 3% respectively. Then the percentage error in the
N
80. Suppose the kinetic energy of a body oscillating with measurement of R is :
R

amplitude A and at a distance x is given by (a) 6 (b) 9


(c) 8 (d) 10
U

Bx
K= 88. What are the dimensions of permeability ?
x + A2
2
(a) [M1L1T1 A–2 ] (b) [M1L1 T–2 A–2 ]
JO

The dimensions of B are the same as that of 2 2


(c) [M L T A ]1 0 (d) [M1L2T2 A–2 ]
(a) work/time (b) work × distance 89. The physical quantity having the dimensions
U

(c) work/distance (d) work × time [M–1L–3T3A2] is


81. The dimensions of magnetic field in M, L, T and C (coulomb) (a) resistance (b) resistivity
ED

are given as (c) electrical conductivity (d) electromotive force


(a) [MLT–1 C–1] (b) [MT2 C–2] 90. The time of reverberation of a room A is one second. What
(c) [MT–1 C–1] (d) [MT–2 C–1] will be the time (in seconds) of reverberation of a room,
having all the dimensions double of those of room A?
82. Two full turns of the circular scale of a screw gauge cover a
(a) 2 (b) 4
distance of 1mm on its main scale. The total number of
divisions on the circular scale is 50. Further, it is found that 1
(c) (d) 1
the screw gauge has a zero error of – 0.03 mm. While 2
measuring the diameter of a thin wire, a student notes the 91. Which of the following is the unit of molar gas constant?
main scale reading of 3 mm and the number of circulr scale (a) JK–1 mol–1 (b) Joule
divisions in line with the main scale as 35. The diameter of
(c) JK–1 (d) J mol–1
the wire is
92. Density of liquid is 16.8 g cm–3. Its value in the International
(a) 3.32 mm (b) 3.73 mm System of Units is
(c) 3.67 mm (d) 3.38 mm
83. If the dimensions of a physical quantity are given by
(a) 16.8 kgm -3 (b) 168 kgm -3
Ma Lb Tc, then the physical quantity will be
(c) 1680 kgm -3 (d) 16800 kgm -3
(a) velocity if a = 1, b = 0, c = – 1
93. The dimensional formula of couple is
(b) acceleration if a = 1, b = 1, c = – 2
(c) force if a = 0, b = – 1, c = – 2 (a) [ML2 T -2 ] (b) [MLT 2 ]
(d) pressure if a = 1, b = – 1, c = – 2
(c) [ML-1T -3 ] (d) [ML-2 T -2 ]
EBD_7179
26 PHYSICS

94. The refractive index of water measured by the relation DIRECTIONS for Qs. (97 to 100) : Each question contains
real depth STATEMENT-1 and STATEMENT-2. Choose the correct answer
m= is found to have values of 1.34, 1.38,
apparent depth (ONLY ONE option is correct ) from the following
1.32 and 1.36; the mean value of refractive index with (a) Statement -1 is false, Statement-2 is true
percentage error is
(b) Statement -1 is true, Statement-2 is true; Statement -2 is a
(a) 1.35 ± 1.48 % (b) 1.35 ± 0 %
(c) 1.36 ± 6 % (d) 1.36 ± 0 % correct explanation for Statement-1
95. Diameter of a steel ball is measured using a Vernier callipers (c) Statement -1 is true, Statement-2 is true; Statement -2 is not
which has divisions of 0.1 cm on its main scale (MS) and 10 a correct explanation for Statement-1
divisions of its vernier scale (VS) match 9 divisions on the (d) Statement -1 is true, Statement-2 is false
main scale. Three such measurements for a ball are given
below: 97. Statement 1 : The number of significant figures depends on
the least count of measuring instrument.
S.No. MS(cm) VS divisions
Statement 2 : Significant figures define the accuracy of
1. 0.5 8 measuring instrument.
2. 0.5 4
98. Statement 1 : Absolute error may be negative or positive.
3. 0.5 6
Statement 2 : Absolute error is the difference between the
If the zero error is – 0.03 cm, then mean corrected diameter is real value and the measured value of a physical quantity.

.IN
(a) 0.52 cm (b) 0.59 cm
99. Statement 1 : In the measurement of physical quantities
(c) 0.56 cm (d) 0.53 cm
L direct and indirect methods are used.
The period of oscillation of a simple pendulum is T = 2p
96. .
AL
Statement 2 : The accuracy and precision of measuring
g
Measured value of L is 20.0 cm known to 1 mm accuracy and instruments along with errors in measurements should be
time for 100 oscillations of the pendulum is found to be 90 s taken into account, while expressing the result.
N
using a wrist watch of 1s resolution. The accuracy in the
100. Statement 1 : Energy cannot be divided by volume.
R

determination of g is
(a) 1% (b) 5% Statement 2 : Because dimensions for energy and volume
U

(c) 2% (d) 3% are different.


JO
U
ED

Exemplar Questions 6. Which of the following pairs of physical quantities does


1. The number of significant figures in 0.06900 is not have same dimensional forrmula?
(a) 5 (b) 4 (a) Work and torque
(c) 2 (d) 3 (b) Angular momentum and Planck's constant
2. The sum of the numbers 436.32, 227.2 and 0.301 in (c) Tension and surface tension
appropriate significant figures is
(d) Impulse and linear momentum
(a) 663.821 (b) 664
7. Measure of two quantities along with the precision of
(c) 663.8 (d) 663.82
respective measuring instrument is
3. The mass and volume of a body are 4.237 g and 2.5 cm 3,
respectively. The density of the material of the body in A = 2.5 ms–1 ± 0.5 ms–1,
correct significant figures is B = 0.10 s ± 0.01 s. The value of AB will be
(a) 1.6048 g cm–3 (b) 1.69 g cm–3 (a) (0.25 ± 0.08) m (b) (0.25 ± 0.5) m
(c) 1.7 g cm –3 (d) 1.695 g cm–3 (c) (0.25 ± 0.05) m (d) (0.25 ± 0.135) m
4. The numbers 2.745 and 2.735 on rounding off to 3 8. You measure two quantities as A = 1.0 m ± 0.2 m, B = 2.0 m ±
significant figures will give
0.2 m. We should report correct value for AB as
(a) 2.75 and 2.74 (b) 2.74 and 2.73
(c) 2.75 and 2.73 (d) 2.74 and 2.74 (a) 1.4 m ± 0.4 m (b) 1.41 m ± 0.15 m
5. The length and breadth of a rectangular sheet are 16.2 ± 0.1 (c) 1.4 m ± 0.3 m (d) 1.4 m ± 0.2 m
cm and 10.1 ± 0.1 cm, respectively. The area of the sheet in 9. Which of the following measurement is most precise?
appropriate significant figures and error is
(a) 5.00 mm (b) 5.00 cm
(a) 164 ± 3 cm2 (b) 163.62 ± 2.6 cm2
(c) 5.00 m (d) 5.00 km
(c) 163.6 ± 2.6 cm2 (d) 163.62 ± 3 cm2
Units and Measurements 27

10. The mean length of an object is 5 cm. Which is the following 15. If force (F), velocity (V) and time (T) are taken as fundamental
measurement is most accurate? units, then the dimensions of mass are : [2014]
(a) 4.9 cm (b) 4.805 cm (a) [F V T– 1] (b) [F V T– 2]
(c) 5.25 cm (d) 5.4 cm (c) [F V– 1 T– 1] (d) [F V– 1 T]
11. Young's modulus of steel is 1.9 × 1011 N/m2. When expressed 16. If energy (E), velocity (V) and time (T) are chosen as the
in CGS units of dyne/cm2, it will be equal to (1N = 105 dyne, fundamental quantities, the dimensional formula of surface
1 m2 = 104 cm2) tension will be : [2015]
(a) 1.9 × 1010 (b) 1.9 × 1011 (a) [EV–1T–2] (b) [EV–2T–2]
(c) 1.9 × 1012 (d) 1.9 × 1013 (c) [E–2V–1T–3] (d) [EV–2T–1]
12. If momentum (p), area (A) and time (T) are taken to be 17. If dimensions of critical velocity ucof a liquid flowing
fundamental quantities, then energy has the dimensional
through a tube are expressed as [ hx ry r x ] , where h, r and
formula
(a) [pA–1T1] (b) [p2AT] r are the coefficient of viscosity of liquid, density of liquid
and radius of the tube respectively, then the values of x, y
(c) [pA–1/2T] (d) [ pA1/2T]
and z are given by : [2015 RS]
NEET/AIPMT (2013-2017) Questions (a) –1, –1, 1 (b) –1, –1, –1
13. In an experiment four quantities a, b, c and d are measured (c) 1, 1, 1 (d) 1, –1, –1
with percentage error 1%, 2%, 3% and 4% respectively. 18. A physical quantity of the dimensions of length that can be

.IN
a 3b 2 e2
Quantity P is calculated as follows P = % error in P is: formed out of c, G and is [c is velocity of light, G is
cd AL 4pe0
(a) 10% (b) 7% [2013] universal constant of gravitation and e is charge] [2017]
(c) 4% (d) 14% 1/ 2 1/ 2
é e2 ù 1 é e2 ù
14. The pair of quantities having same dimensions is 2
c êG ú ê ú
N
(a) (b)
(a) Young’s modulus and energy [NEET Kar. 2013] ëê 4pe 0 ûú c 2 ëê G4pe0 ûú
R

(b) impulse and surface tension


1/ 2
(c) angular momentum and work 1 e2 1 é e2 ù
U

(d) work and torque (c) G (d) ê G ú


c 4pe0 c 2 êë 4pe 0 úû
JO
U
ED
EBD_7179
28 PHYSICS

Hints & Solutions


EXERCISE - 1 21. (b) Let M = pnvm
1. (d) Temperature is one of the basic physical quantities. ML-2 T -1 = ( ML-1 T -2 ) n (LT -1 ) m
2. (b) The vander Waal’s gas equation is
= M n L- n + m T -2n - m
a
(P + )(V - b) = RT for one mole. \ n = 1; - n + m = -2
V2
\ m = -2 + n = -2 + 1 = -1 \ m = -n
æ ma ö 22. (b)
& çç P + ÷÷(V - mb) = mRT for m mole
è V2 ø 23. (c) N kg–1 = force/mass = acceleration
Dimensionally in first bracket on L.H.S 24. (a) Moment of couple = force × distance = [M1 L2 T -2 ]
é ma ù [P ]V 2
[ P] = ê ú Þ [a ] = work = force × distance = [M1 L2 T -2 ] .
ë V2 û m
energy ML2 T -2
é atm. litre2 ù 25. (b) Power = = = [ ML2 T -3 ] .

.IN
Dimension of [a ] = ê ú time T
ëê mole ûú
EXERCISE - 2
3. (d)
AL
eV W PV 1. (a) From Biot Savart’s law
4. (c) = = =R
T T T
N
m 0 i dl sin q
R B=
and = Boltzmann constant. 4p r2
R

N
5. (a) 6. (a) 4 p Br2 Wb m-2m2
U

7. (a) SI is based on seven fundamental units. m0 = = = Wb A-1 m-1


idl sin q Am
JO

Q J
8. (a) L= = = J kg -1 Pressure difference
m kg 2. (a) Pressure gradient = .
distance
U

9. (a) As force = change in momentum/time.


\ force × time = change in momentum ML-1T -2
= é ML- 2T -2 ù
ED

10. (c) Illuminance is intensity of illumination measured in lux. [Pressure gradient] = ë û


L
Qi C
11. (a) P= = 2 = Cm - 2 1 æ 1 1 ö÷
A m 3. (c) From = Rç - ,
l ç n2 n2 ÷
12. (a) è 1 2 ø

1
13. (c) 1 light year = speed of light in vacuum × no. of seconds dimensions of R = = L-1 = [M 0 L-1 T 0 ]
in one year = (3 × 108) × (365 × 24 × 60 × 60) = 9.467 × L
1015 m.
14. (c) 1 pascal = 1 N / m2. PV W ML2 T -2
4. (a) R= = =
15. (c) Electron volt is a unit of energy & mT m T mol K
1eV = 1.6×10–19 joule where m is number of mole of the gas
16. (a) Impulse = force × time = MLT -2 1 -1
´ T = [M LT ] . = [M1L2T–2K–1mol–1]
5. (a) M = current × area = AL2 = [L2 A1]
2
M Am
17. (b) Pole strength, m = = = A m. 6. (b) b = l m T = LK = [M0 L1 T 0 K1 ]
2l m
18. (d) Both energy and work have same unit. V W ML2 T -2
\ energy/work is a pure number. 7. (b) Impedance = = =
I QI Q QT -1
19. (a) Potential is work done per unit charge.
20. (c) Maxwell is the unit of magnetic flux in C.G.S system. = [ML2 T -1 Q -2 ] .
1 Wb(S.I unit) = 108 maxwell
Units and Measurements 29

1 2 2 -2 Weight of body in air


8. (a) Energy stored in an inductor = L i = [ML T ] 22. (d) Relative density =
Loss of weight in water
2
5.00 5.00
= =
5.00 – 4.00 1.00
9. (b) Wave number n = 1 = 1 = [M 0 L-1 T 0 ] .
l L Dr æ 0.05 0.05 ö
´ 100 = ç + ´ 100
10. (a) T = Pa Db Sc r è 5.00 1.00 ÷ø
= (0.01 + 0.05) × 100
M 0 L0 T1 = (ML-1 T -2 )a (ML-3 ) b (MT -2 ) c
= 0.06 × 100 = 6%
= M a + b + c L- a -3b T -2a - 2c \ Relative density = 5.00 ± 6%
Applying principle of homogeneity 23. (a) Area = (Length)2
a + b + c = 0; – a – 3b = 0; – 2a – 2c = 1 or length = (Area)1 2
on solving, we get a = – 3/2, b = 1/2, c = 1
= (100 ± 2)1 2
3 -3 3
rr ML L
11. (b) = =T 1
S MT -2 = (100)1 2 ± ´2
2
12. (d) B = x = [L]; A x = Vx; A = V x = (10 ± 1)m

.IN
= M L2 T -2 L1 / 2 = M L5 / 2 T -2 Q2
24. (c) We know that is energy of capacitor so it represent
2C
AB = (ML5 / 2 T -2 ) (L) = [M1 L7 / 2 T -2 ]
AL
13. (b) In S = A (t + B) + Ct2 ; B is added to time t. Therefore, the dimension of energy = [ML2 T -2 ] .
dimensions of B are those of time. Volt ´ sec/amp.
N
25. (b) L/R = = sec. = [M 0 L0 T ]
2 -2 2 -1 Volt/amp.
14. (b) h = l 2m E = L M ( ML T ) = [M L T ]
R

Current Q
15. (d) v = ga h b ; [M 0 LT -1 ] = (LT -2 ) a Lb = La + b T -2a 26. (a) Current density = =
U

area area ´ t
\ a + b = 1; - 2a = -1 a = 1/ 2
JO

0.2
27. (c) ´ 100 = 0.8
\ b = 1/ 2 25
16. (a) As c is added to t, \ c = [T] 28. (d) 29. (a)
U

1
v LT -1 30. (d) From n =
a= = = [LT -2 ] ,
ED

t T
2p LC
1 1
b = v (t + c) = LT -1 ´ T = [L] LC = = = T 2 = [M 0 L0 T 2 ]
2 -1 2
(2p n) (T )
velocity [ LT -1 ] 31. (d) Solar constant = energy/sec/area
17. (a) Hubble’s constant, H = =
dis tan ce [ L]
M L2 T -2
= = [MT-3 ]
= [T -1 ] = 70 ´ 10 -3 N / m. TL 2

4 1 e2
18. (c) V = p r 3; 32. (c) From F =
3 4 p eo r2
DV æ Dr ö
´ 100 = 3ç ÷ ´ 100 = 3 ´ 1 % = 3 % e2
V è r ø \ = 4 p F r 2 (dimensionally)
eo
19. (a) Subtraction is correct upto one place of decimal,
corresponding to the least number of decimal places. e2 4pFr 2 (MLT-2 )L2
= = = [M 0 L0 T 0 A 0 ] ,
7.26 – 0.2 = 7.06 = 7.1 J. eo hc hc ML2T-1[LT-1]
20. (d) Number of significant figures in multiplication is three,
corresponding to the minimum number e2
107.88 × 0.610 = 65.8068 = 65.8 is called fine structure constant & has value
e o hc
97 .52
21. (d) = 38 . 393 = 38 .4 (with least number of 1
2 .54 .
significant figures, 3). 137
EBD_7179
30 PHYSICS

45. (c) Volume (L3) = surface area (6L2)


Q ML2 T -2
33. (c) Entropy = = = [ML2 T -2 K -1 ] \ L = 6, volume = 63 = 216
T K 46. (d) None of the expressions has the dimensions of time.
2 -2
Q ML T X = Ma Lb T c ;
34. (d) s = = = [L2 T -2 K -1 ] 47. (a)
mq MK
DX æ a DM b DL c DT ö
35. (c) 1 1 ´ 100 = ç + + ÷ ´ 100
= X è M L T ø
LC (ML2 T -2 A -2 ) ´ (M -1L-2 T 4 A 2 )
= (a a + b b + c g )%
1 48. (c) N VD = (N – 1) MD
= = T -1
2
T æ N -1 ö
36. (b) Solar constant = energy/area/time 1 VD = ç ÷MD
è N ø
M L2 T -2 L.C. = Least count = 1MD – 1VD
= = [M1 T -3 ] .
2
L T
æ N -1 ö
37. (a) L.C. = ç1 - ÷ MD
è N ø
38. (b) v = k l a rb g c

.IN
1 0.1 1
0 -1 a -3 b -2 c
= M.D. = cm = cm
[M LT ] = L (ML ) (LT ) AL N N 10 N

= M b La - 3b + c T -2c value of 1 part on main scale


=
\ b = 0; a - 3b + c = 1 number of parts on vernier scale
N
1 where V.D. = vernier division, M.D. Main scale division.
-2c = -1 Þ c = 1 / 2 \ a=
R

2 l l
49. (c) From T = 2p ; g = 4p 2
v µ l1/ 2 r0 g1/ 2 or v 2 µ l g g T2
U

39. (b) In p = p0 exp. (– a t2) is dimensionless


JO

D g Dl 2 DT
1 1 -2 = + = ( y + 2x )
\ a= = = [T ] g l T
t2 T2
50. (c) We have, n 1u1 = n2u2
U

a
40. (b) As =P æu ö
ED

V2 n2 = n1 ç 1 ÷
è u2 ø
dyne
\ a = PV 2 = 2
(cm 3 ) 2 = dyne ´ cm 4
cm 2 -3
æ M öæL ö æT ö
= 106 ´ ç 1 ÷ ç 1 ÷ ç 1 ÷
h ML2 T -1 è M 2 ø è L 2 ø è T2 ø
41. (c) = = ML2 T -3A -2 = Resistance (ohm)
e2 (AT) 2 -3
æ 1 kg ö æ 1 m ö 2 æ 1 s ö
42. (c) I = AT2 e–B/kT = 106 ´ ç ç ÷
Dimensions of A = I /T2; Dimensions of B = kT è 10 kg ÷ø è 1 ´ 10-1 m ø çè 60 s ÷ø
(Q power of exponential is dimensionless)
æ 1ö
= 106 ´ ç ÷ (10) ( 60)
2 3
I è 10 ø
AB2 = (kT)2 = I k 2
T2
= 107 ´ ( 60) = 2.16 × 1012 units.
3
43. (a) The C.G.S unit of current is called biot (Bi) i.e.,
1C (1/10)e.m.u of charge 1 51. (c) Effective resistance
1A = = = Bi
1sec sec 10 R S = (10kW ± 10%) + (20kW ± 20%)
or 1Bi = 10A
\ Tolerance of the combination = (30kW ± 30%)
1 1
44. (d) w k = ´ = [L-1 T -1 ] 1 q 1q 2 q .q
T L 52. (c) Force, F = Þ e 0 = 1 22
The dimensions of the quantities in a, b, c are of velocity 4pe 0 r 2
4pFr
[LT–1]
Units and Measurements 31

So dimension of e0
0.1 æ 19 ö 1 1
2 66. (c) = ç1 - ÷ MSD Þ = ´ 1MSD
=
[AT]
= [M -1L-3T 4 A 2 ] 10 è 20ø 100 20
-2 2
[MLT ][L ] 1
Þ 1 MSD = ´ 10 = 2
5
æ Dm ö
53. (c) Percentage error in mass ç ´ 100 ÷ = 2% and
è m ø 0.5
67. (b) Least count = = 0.01mm
50
percentage error in speed æç Dv ´ 100 ö÷ = 3% . 68. (c) Maximum absolute error is Da + Db . Therefore the
è v ø
absolute error
1 percentage error = actual value ´ 100
E = mv2
2
DQ 2DI DR Dt
DE Dm DV 69. (d) ´ 100 = ´ 100 + ´100 + ´100
\ ´ 100 = ´ 100 + 2 ´ 100 Q I R t
E m V
= 2% + 2 × 3% = 8%. = 2 ´ 2% +1% +1% = 6%.

Mass DP DF Dl
70. (c) ´ 100 = ´ 100 + 2 ´ 100 = 4% + 2 × 2%
54. (d) Density = P F l

.IN
Volume
= 8%
M Dr DM DL
r= , = +3 l l
L 3 r M L 71.
AL(c) T = 2p , gµ 2
g T
% error in density = % error in Mass + 3 (% error
in length) Dg
N
= 4 + 3(3) = 13% \ ´ 100 = 0.5% + 2 × 0.2% = 0.9%
g
R

Cp
55. (a) Ratio of specific heat, g = 72. (a) % error =
0.01
´ 100 +
0.01
´100 +
0.01
´ 100
U

Cv
0.5 1.0 0.5
56. (a) Specific gravity is the ratio of density of substance =2+1+2=4+1=5
JO

and density of water at 4°C. The ratio of like quantities 73. (c)
is dimensionless.
57. (c) Dimensionally K.E = Work 74. (b) [ LC] = ML2T -2 A -2 .M -1L-2 T4 A 2 = T
U

58. (b) Trigonometric ratio are a number and hence


wx
ED

demensionless 75. (c) The quantity ( – wk) has dimension of angle and
v
éQù é Q2 ù
59. (b) [C] = ê ú = ê
-1 -2 2 2
ú = [M L T Q ] hence wk is dimensionless being angle.
ëVû ëê W ûú
[X] M -1L-2 T 4 A 2
76. (d) [Y] = = = M -3 L-2 Q 4 T 4
60. (c) [A ] = [LT -2 ] or [L ] = [AT 2 ] [Z2 ] M 2 T -4 A -2

[Work] = [Force ×Distance] = [FL] = [FAT 2 ] æ Qö


çèQ A = ÷ø
61. (a) h is the coefficient of viscosity.. T
62. (a) Velocity gradient is velocity per unit distance. 77. (c) [at] = [F] amd [bt2] = [F]
63. (a) Use principle of homogeneity. Þ [a] = MLT–3 and [b] = MLT–4

[ML2 T -2 ][ML2 T -1 ]2 n 2T MLT -2


64. (a) = [M 0 L0 T 0 ] = angle. 78. (c) Clearly, m = ; [m] =
[M 5 ][M -1L3T - 2 ]2 4f 2 L2 T -2 .L2

9 é W ù ML2T -2
65. (c) 10 VD = 9MD, 1VD = MD 79. (d) [V] = ê ú = = ML2 A -1T -3
10 ëQû AT
Vernier constant = 1 MD – 1 VD
Bx Bx B
æ 9ö 1 1 1 80. (b) From K = 2 2
= 2 =
= ç1- ÷ MD = MD = ´ = 0.05 mm x +A x x
è 10ø 10 10 2
\ B = K × x = K.E. × distance = work × distance
EBD_7179
32 PHYSICS

81. (c) We know that F = q v B


m
89. (c) Resistivity, r =
F MLT -2 ne 2t
\ B= = = MT -1C-1
qv C ´ LT -1
r = [ML3T –3 A –2 ]
0.5 So, electrical conductivity
82. (d) Least count of screw gauge = mm = 0.01mm
50 1
\ Reading = [Main scale reading s=
r
+ circular scale reading × L.C] – (zero error)
= [3 + 35 × 0.01] – (–0.03) = 3.38 mm s = [M –1L–3 T 3 A 2 ]
90. (a) Reverberation time,
MLT –2
83. (d) Pressure = = [ ML T–1 –2 ] æ dö æ dö
L2 t = çè V ÷ø +ç ÷
forward
è V ø backward
Þ a = 1, b = – 1, c = – 2.
when dimensions double then, d¢ = 2d
g
84. (b) In CGS system, d = 4 d ¢ d ¢ 2d 2d æ d dö
cm3 \ t¢ = + = + = 2ç + ÷
V V V V è V Vø
The unit of mass is 100g and unit of length is 10 cm, so

.IN
æ d d ö
æ 100g ö æ 4 ö Þ t¢ = 2t çèQ + = t ÷ø
4ç V V
è 100 ÷ø çè ÷
100 ø (100g) PV J
density = = = J K -1 mol -1 .
æ 10 ö
3 3 3 91.
AL(a) R= =
æ 1 ö (10cm) nT mol K
çè cm÷ø çè ÷ø
10 10 92. (d) 16.8 gcm–3 = 16800 Kgm–3.
N
4 100g 93. (a) Dimensionally couple = Torque = Work
= ´ (10)3 · = 40 unit 94. (a) The mean value of refractive index,
R

100 (10cm) 3
1.34 + 1.38 + 1.32 + 1.36
U

a 3b2 DP Da Db m= = 1.35
85. (d) P = , × 100% = 3 × 100% + 2 × 4
cd P a b and
JO

| (1.35 - 1.34) | + | (1.35 - 1.38) | + | (1.35 - 1.32) | + | (1. 35 - 1.36) |


Dc Dd Dm =
4
100% + × 100% + × 100%.
c d = 0.02
U

= 3 × 1% + 2 × 2% + 3% + 4% = 14% Dm 0.02
Thus m ´ 100 = ´ 100 = 1.48
ED

1.35
l l
86. (c) From T = 2p ; g = 4p 2 0.1
g T2 95. (b) Least count = = 0.01 cm
10
D g Dl 2 DT d1 = 0.5 + 8 × 0.01 + 0.03 = 0.61 cm
= + = ( y + 2x ) d2 = 0.5 + 4 × 0.01 + 0.03 = 0.57 cm
g l T
d3 = 0.5 + 6 × 0.01 + 0.03 = 0.59 cm
rl DR 0.61 + 0.57 + 0.59
87. (b) Given = R = , then ´ 100 Mean diameter =
2 R
pr 3
Dr Dl Dr = 0.59 cm
= r ´ 100 + ´ 100 + 2 ´ 100
l r
2 l
= 1% + 2% + 2 × 3% = 9% 96. (d) As, g = 4 p
T2
88. (b) The magnetic field at a point near a long straight
conductor is given by Dg Dl DT
So, ´ 100 = ´ 100 + 2 ´ 100
g l T
m0 2I 4 prB
B = . Þ m0 =
4p r 2I 0.1 1
= ´ 100 + 2 ´ ´ 100 = 2.72 ; 3%
20 90
[r ][ B ] L.MT -2 A -1 æ Fö 97. (c) 98. (b)
\ [m0] = = çQ B = qv ÷
[I ] I è ø 99. (b) 100. (a)
= MLT–2A–2
Units and Measurements 33

EXERCISE - 3 b. Angular momentum (L)


= mvr = [M][LT–1][L]
Exemplar Questions
= [ML2T–1]
1. (b) In 0.06900, the two zeroes before six are not siginificant
E
and two zeroes on right side of 9 are significant figures. Planck's constant h =
Hence, number of significant figures are four (6900). n
2. (b) In addition the result will be in least number of places [ML2 T -2 ] æ 1ö
after decimal and minimum number of significant figure. = -1
= [ML2 T -1 ] çèQ E = hn and n = ÷ø
[T ] T
The sum of the given numbers can be calculated as
663.821. The number with least decimal places is 227.2 So, dimensions of h and L are equal.
is correct to only one decimal place but in addition of c. Tension = force = [MLT–2]
numbers, the final result should be rounded off to one
decimal place i.e., 664. force [MLT -2 ]
Surface tension = =
3. (c) As we know that in multiplication or division, the final length [L]
result should retain as many significant figures as are = [ML0T–2]
there in the original number with the least significant d. Impulse = force × time = [MLT–2][T]
figures. = [MLT–1]
The significant figure in given numbers 4.237 g and 2.5 Momentum = mass × velocity
cm3 are four and two respectivey so, density should

.IN
= [M][LT–1] = [MLT–1]
be reported to two significant figures.
LHS and RHS has same dimensions.
4.237 g Hence only (c) options of Physical quantities does not
Density = = 1.6948 = 1.7 g.cm -3
2.5 cm 3
AL have same dimensions.
7. (a) By applying the Rule of significant figure in
\ As rounding off the number upto 2 significant
multiplication and addition.
figures, we get density = 1.7.
N
As given that,
4. (d) Rounding off 2.745 upto 3 significant figures here IVth
A = 2.5 ms–1 ± 0.5 ms–1, B = 0.10 s ± 0.01 s
R

digit is 5 and its preceding is even, so no change in 4.


Thus answer would be 2.74. Rounding off 2.735 upto 3 x = AB = (2.5)(0.10) = 0.25 m (consider only significant
U

significant figures, here IV digit is 5 and its preceding figure value)


digit is 3 (odd). So 3 is increased by 1 answer become Dx DA DB
JO

would be 2.74. = +
x A B
5. (a) If Dx is error in a physical quantity, then relative error is
Dx Dx 0.5 0.01 0.05 + 0.025 0.075
U

calculated as . = + = =
x x 2.5 0.10 0.25 0.25
ED

Given that, Dx = 0.075 = 0.08 m


Length l = (16.2 ± 0.1) cm (rounding off to two significant figures.)
Breadth b = (10.1 ± 0.1) cm AB = (0.25 ± 0.08) m
Q Dl = 0.1 cm, Db = 0.1 cm 8. (d) As given that,
Area (A) = l × b = 16.2 × 10.1 = 163.62 cm2 A = 1.0 m ± 0.2 m, B = 2.0 m ± 0.2 m
In significant figure rounding off to three significant
So, X = AB = (1.0)(2.0) = 1.414 m
digits, area A = 164 cm2
Rounding off upto two significant digit
DA Dl Db 0.1 0.1
= + = + X = 1.4 m = (r) (Let)
A l b 16.2 10.1
1.01 + 1.62 2.63 Dx 1 é DA DB ù 1 é 0.2 0.2 ù
= = = + = +
16.2 ´10.1 163.62 x 2 êë A B úû 2 êë 1.0 2.0 úû
2.63 2.63
So, DA = A ´ = 164 ´ 0.6
163.62 163.62 =
2 ´ 2.0
= 2.636 cm2
Now rounding off up to one significant figure 0.6 x 0.6 ´ 1.4
DA = 3 cm2. Þ Dx = = = 0.212
2 ´ 2.0 2 ´ 2.0
So, Area A = A ± DA = (164 ± 3) cm2.
Rounding off upto one significant digit
6. (c) a. Work = force × distance
Dx = 0.2 m = Dr (Let)
= [MLT–2][L] = [ML2T–2]
So, correct value of
Torque = F × d = [ML2T–2]
LHS and RHS has same dimensions. AB = r + Dr = (1.4 ± 0.2) m
EBD_7179
34 PHYSICS

9. (a) For the most precise measurement, the unit must be 14. (d) Work = Force × displacement
least and number of digits including zeroes after Torque = Force × force arm
decimal must be zero. = mass × acceleration × length
Now, take first option, = [M] × [LT–2] × [L] = [M L2T–2]
As here 5.00 mm has the smallest unit and the error in
15. (d) Force = mass × acceleration
5.00 mm is least (commonly taken as 0.01 mm if not
specified), hence, 5.00 mm is most precise. Þ [Mass]
10. (a) Now, checking the errors with each options one by é force ù

one, ë acceleration úû
|Dl1| = |5 – 4.9| = 0.1 cm
|Dl2| = |5 – 4.805| = 0.195 cm é force ù –1
= ê ú = [F V T]
|Dl3| = |5.25 – 5| = 0.25 cm ë velocity / time û
|Dl4| = |5.4 – 5| = 0.4 cm 16. (b) Let surface tension
Error Dl1 is least or minimum.
s = Ea Vb Tc
So, 4.9 cm is most precise.
b
11. (c) It is given that Young's modulus (Y) is, MLT –2 æ Lö
= (ML2 T –2 )a ç ÷ (T)C
Y = 1.9 × 1011 N/m2 L è Tø
1N = 105 dyne Equating the dimension of LHS and RHS
So, Y = 1.9 × 1011 × 105 dyne/m2

.IN
ML0T–2 = MaL2a + b T–2a – b + c
Convert meter to centimeter
Þ a = 1, 2a + b = 0, –2a – b + c = –2
Q 1m = 100 cm
Þ a = 1, b = – 2, c = – 2
Y = 1.9 × 1011 × 105 dyne/(100)2 cm2
AL
= 1.9 × 1016 – 4 dyne/cm2 Hence, the dimensions of surface tension are
[E V–2 T–2]
Y = 1.9 × 1012 dyne/cm2
N
12. (d) Given that fundamental quantities are momentum (p), 17. (d) Applying dimensional method :
area (A) and time (T). vc = hxryrz
R

Let us consider the dimensional formula for [M0LT–1] = [ML–1T–1]x [ML–3T0]y [M0LT0]z
U

E µ [ p a AbT c ] Equating powers both sides


x + y = 0; –x = –1 \ x = 1
JO

E = kp a AbT c ]
1 + y = 0 \ y = –1
where k is dimensionless constant of proportionality.
–x – 3y + z = 1
Dimensions of energy [E] = [ML2T–2] and Dimension
–1 – 3(–1) + z = 1
U

of momentum p = mv = [MLT–1]
Dimension of Area [A] = [L2] –1 + 3 + z = 1
ED

Dimension of Time [T] = [T] \ z = –1


Dimension of energy [E] = [K] [p]a[A]b[T]c 18. (d) Let dimensions of length is related as,
Putting all the dimensions, value z
é e2 ù
ML2T–2 = [MLT–1]a [L2]b [T]c x
L = [c] [G] ê
y
ú
= MaL2b + aT–a + c êë 4pe 0 úû
By principle of homogeneity of dimensions,
1 e2
a = 1, 2b +a = 2 Þ 2b + 1 = 2 Þ b = = ML3T–2
2 4pe0
- a + c = -2 L = [LT–1]x [M–1L3T–2]y[ML3T–2]z
c = –2 + a = –2 + 1 = –1 [L] = [Lx + 3y + 3z M –y + z T–x – 2y – 2z]
So, Dimensional formula (of energy) Comparing both sides
E = [pA1/2T–1] –y + z = 0 Þ y = z ...(i)
E = [ pA1/2T -1] x + 3y + 3z = 1 ...(ii)
–x – 4z = 0 (Q y = z) ...(iii)
NEET/AIPMT (2013-2017) Questions From (i), (ii) & (iii)
a 3 b 2 DP Da Db 1
13. (d) P = , × 100% = 3 × 100% + 2 × z = y = , x = –2
cd P a b 2
1/2
Dc Dd é e2 ù
100% + × 100% + × 100%. -2
c d Hence, L = c êG × ú
= 3 × 1% + 2 × 2% + 3% + 4% = 14% ëê 4pe 0 ûú
Motion in a
3
BASIC DEFINITIONS
Straight Line
3. Displacement of an object is independent of the path
Mechanics : Branch of physics, which deals with the study of followed by the object but distance depends upon path.
objects in rest and in motion. 4. Displacement and distance both have same unit as that of
Statics : Study of objects at rest or in equilibrium. length i.e. metre.
Kinematics : Study of motion of objects without considering the

.IN
cause of motion. Distance
5. ³1
Dynamics : Study of motion of objects considering the cause of |Displacement|
motion.
Rest : An object is said to be at rest if it does not change its 6. For a moving body distance always increases with time
AL
position with time, with respect to its surrounding (a reference 7. For a body undergoing one dimensional motion, in the same
point which is generally taken as origin in numerical problems) direction distance = | displacement |. For all other motion
Motion : An object is said to be in motion if it changes its position distance > | displacement |.
N
with time, with respect to its surroundings.
SPEED
R

Rest and motion are relative terms.


Point mass/Point object : An object is said to be a point mass if It is the distance travelled per unit time by an object. It is a scalar
U

during its motion it covers distance much greater than its own quantity. It cannot be negative.
size. Uniform speed : An object is said to be moving with a uniform
JO

One dimensional motion : An object travels in a straight line. It is speed, if it covers equal distances in equal intervals of time,
also called rectilinear or linear motion. The position change of howsoever small the time intervals may be.
the object with time in one dimension can be described by only Non-uniform speed : If an object covers unequal distances in
U

one co-ordinate. equal interval of time or equal distances in unequal interval of


Ex. A stone falling freely under gravity. time.
ED

Two dimensional motion or motion in a plane : For an object Instantaneous speed : The speed of an object at a particular instant
travelling in a plane two coordinates say X and Y are required to of time is called the instantaneous speed.
describe its motion.
Ex. An insect crawling over the floor. Ds ds
Instantaneous speed, Vinst = lim = .
Three dimensional motion : An object travels in space.To describe Dt dt
Dt ® 0
motion of objects in three dimension require all three coordinates Average speed : It is ratio of the total distance travelled by the
x, y and z. object to the total time taken.
Ex. A kite flying in the sky.
Dx
DISTANCE AND DISPLACEMENT Average speed Vav =
Dt
Distance or Path length : The length of the actual path travelled Dimensions : [M0LT-1]; Unit: In SI systems.
by an object during motion in a given interval of time is called
the distance travelled by that object or path length. It is a scalar VELOCITY
quantity. It is the displacement of an object per unit time. It is a vector
Displacement : It is the shortest distance between the initial and quantity. It can be positive negative or zero.
final position of an object and is directed from the initial position Uniform velocity : An object is said to be moving with a uniform
to the final position. It is a vector quantity. velocity, if it covers equal displacements in equal intervals of
time, howsoever small the time intervals may be.
Keep in Memory Non-uniform velocity : If an object covers unequal displacements
1. Displacement may be positive, negative or zero but distance in equal interval of time or equal displacements in unequal interval
is always positive. of time.
Instantaneous velocity : The velocity of an object at a particular
2. Displacement is not affected by the shift of the coordinate
axes. instant of time is called the instantaneous velocity.
EBD_7179
36 PHYSICS

r r r Variable or non-uniform acceleration : If the velocity of body


Dr dr
Instantaneous velocity, Vinst = lim = . changes in different amounts during same time interval, then the
Dt ® 0 Dt dt
acceleration of the body is known as variable acceleration.
Average Velocity : It is ratio of the total displacement to the total
Acceleration is variable if either its direction or magnitude or
time taken.
r both changes with respect to time. A good example of variable
Dr acceleration is the acceleration in uniform circular motion.
Average velocity, vav =
Dt
EQUATIONS FOR UNIFORMLY ACCELERATED MOTION
Dimensions : [M0LT–1] ; Unit: In SI system, m/s
When the motion is uniformly accelerated i.e., when acceleration
Keep in Memory is constant in magnitude and direction :
(i) v = u + at
1. | Average velocity | can be zero but average speed cannot be
zero for a moving object. 1 2
(ii) s = ut + at
2
|Average velocity|
2. £1 (iii) v2 – u2 = 2as
Average speed
where u = initial velocity; v = final velocity;
3. | Instantaneous velocity | = Instantaneous speed. a = uniform acceleration and
4. A particle may have constant speed but variable velocity. It
s = distance travelled in time t,
happens when particle travels in curvilinear path.

.IN
5. If the body covers first half distance with speed v1 and æ u + vö
(iv) s =ç t
next half with speed v2 then
AL è 2 ÷ø
2v1v2 a
Average speed v = (v) Distance travelled in nth second sn = u + (2n - 1) ;
v1 + v2 2
6. If a body covers first one-third distance at a speed v1, next sn = distance covered in n th second
N
one-third at speed v2 and last one-third at speed v3, then Above equations in vector form
R

3v1 v2 v3 1
Average speed v = v = u + a t, s = u t + a t 2 ,
v1 v 2 + v 2 v3 + v1 v3
U

2
7. If a body travels with uniform speed v1 for time t1 and with r2 r2 rr rr rr rr
v - u = 2a.s (or v.v + u.u = 2a.s )
JO

uniform speed v2 for time t2, then


r 1 r r
v t +v t s = ( u + v) t
Average speed v = 1 1 2 2 2
U

t1 + t2 When displacement (s) is given as a function of time t [s = f(t)]


then
ED

ACCELERATION
The rate of change of velocity with respect to time is called s µ t0 Body at rest v= 0 a= 0
acceleration. It is a vector quantity.
r s µ t1 Uniform velocity;
Let velocity changes by Dv during some interval of time Dt .
r acceleration zero v µ t0 a= 0
Average acceleration a av is given by
r
r Dv s µ t2 Uniform acceleration v µ t1 a µ t0
aav =
Dt
r s µ t 3 or more Non uniform
Instantaneous acceleration a is given by
r r acceleration v µ t 2 or more a µ t1 or more
r Dv dv
a = lim = We use calculus method (integration and differentiation)
Dt ®0 Dt dt
for displacement, velocity, acceleration as a function of time.
Its SI unit is meter/sec2 (ms–2).
ds dv
A body moving with uniform velocity has zero acceleration. It We know that v =
dt ò
Þ s = v dt ; a =
dt ò
Þ v = a dt ;
means that neither its speed nor its direction of motion is changing
with time. when a = f(s)
dv
Uniform acceleration : If the velocity of the body changes in
equal amount during same time interval, then the acceleration of
a=v
ds ò
Þ a ds = v dv , ò
the body is said to be uniform. Acceleration is uniform when where s = displacement, v = instantaneous velocity,
neither its direction nor magnitude changes with respect to time. a = instantaneous acceleration
Motion in a Straight Line 37

VERTICAL MOTION UNDER GRAVITY


1 2 g
(i) For a body thrown downward with initial velocity u from a h= gt ; h th = (2n - 1)
2 n 2
height h, the equations of motion are
(iii) When a body is thrown upwards with initial velocity u, the
v = u +gt ; v 2 = u 2 + 2gh equations of motion are
v = u – gt
1 2 g
h = ut + gt ; h th = u + (2n - 1)
2 n 2 1 2
h = ut – gt
(ii) If initial velocity is zero, then the equations are 2
v2 = u2 – 2gh.
v = gt ; v = 2gh

To summarise :
Motion of a body

Velocity constant Accelerated motion


Displacement

.IN
Velocity =
Time
Uniformly accelerated Non-uniformly
motion accelerated motion
AL
v = u + at ds dv dv
v= a= =v
1 dt dt ds
s = ut + at 2
N
2 Differentiation Differentiation
R

2 2 Displacement Velocity Acceleration


v - u = 2as
a Integration Integration
U

s nth = u + (2n - 1)
2
ò ds = ò v dt ò dv = ò a dt
JO

1
s = (u + v) ´ t
2
U

N OTE: Calculus method as shown in non-uniformly accelerated Keep in Memory


ED

motion may also be used for uniformly accelerated motion. 1. The direction of average acceleration vector is the direction
r r
Uniformly Accelerated Motion : A Discussion r vf - vi
While using equations of motion we can have two approaches. of the change in velocity vector. a =
t
Approach 1 : Take a = +ve when velocity increases and r r r r r
a has a direction of v f - v i = v f + (– v i )
a = –ve when velocity decreases. r r
Take rest of physical quantities such as u, v, t and s as positive. i.e., the resultant of vf and – v i
Approach 2 : (Vector method)
2. There is no definite relationship between velocity vector
Assume one direction to be positive and other negative. Assign
and acceleration vector.
sign to all the vectors (u, v, a, s), +ve sign is given to a vector
which is directed to the positive direction and vice-versa 3. For a body starting from rest and moving with uniform
– + acceleration, the ratio of distances covered in t 1 sec.,
+

t2 sec, t3 sec, etc. are in the ratio t 12 : t22 : t32 etc.


+

4. A body moving with a velocity v is stopped by application


– of brakes after covering a distance s. If the same body moves

with a velocity nv, it stops after covering a distance n 2s by
Normally the direction taken is as drawn above. But it is important the application of same retardation.
to note that you can take any direction of your choice to be positive
Example 1.
and the opposite direction to be negative.
The displacement of particle is zero at t = 0 and
displacement is ‘x’ at t = t. It starts moving in the positive
N OTE: The second method (or approach) is useful only when
x-direction with a velocity which varies as v = k x where
there is reversal of motion during the activity concerned. k is constant. Show that the velocity varies with time.
EBD_7179
38 PHYSICS

Solution : r r
r v - vi 5jˆ - 5iˆ r 1 1
v = k x or
dx
= k x or
dx
= k dt
a= f = Þ a = – $i + $j
Dt 10 2 2
dt x
Given that when t = 0, x = 0 and when t = t, x = x, r FG 1 IJ + FG 1 IJ
2 2 1
m/s2.
x
dx
t |a |=
H 2K H 2 K =
2
Hence ò x
ò
= k dt ;
Keep in Memory
0 0
x
é - 1 +1 ù 1. An object moving under the influence of earth's gravity in
x t êx 2 ú which air resistance and small changes in g are neglected is
ú = k [ t ]0
t
\ ò x -½ dx = k ò dt or ê called a freely falling body.
1
ê - + 1ú
0 0 2. In the absence of air resistance, the velocity of projection
ëê 2 ûú 0
is equal to the velocity with which the body strikes the
kt ground.
or 2 x = k t or x=
2 3. Distance travelled by a freely falling body in 1st second is
2 always half of the numerical value of g or 4.9 m, irrespective
ék tù k t
Now, v = k ´ ê ú = of height h.
ë2û 2 4. For a freely falling body with initial velocity zero
Thus the velocity varies with time. (i) Velocity µ time (v = gt)

.IN
Example 2. (ii) Velocity µ Distance fallen (v2 = 2gs)
A particle covers each 1/3 of the total distance with speed
vl, v2 and v3 respectively. Find the average speed of the æ 1 ö
(iii) Distance fallen a (time)2 ç s = gt 2 ÷ , where g is the
particle.
AL è 2 ø
Solution : acceleration due to gravity.
Average speed 5. If maximum height attained by a body projected vertically
N
Total distance travelled s upwards is equal to the magnitude of velocity of projection,
v= = then velocity of projection is 2g ms–1 and time of flight is 4
Total time taken s s s
R

+ + sec.
3v1 3v 2 3v 3
6. If maximum height attained by a body projected upward is
U

3v1 v 2 v 3 equal to magnitude of acceleration due to gravity i.e., 'g',


Þ v=
JO

v1v 2 + v 2 v 3 + v 3 v1 the time of ascent is 2 sec. and velocity of projection is


Example 3.
A cheetah can accelerate from 0 to 96 km/h in 2 sec., whereas g 2.
U

a cat requires 6 sec. Compute the average accelerations for 7. Ratio of maximum heights reached by different bodies
the cheetah and cat. projected with velocities u1, u2, u3 etc. are in the ratio of
ED

Solution : u12 : u 22 : u 33 etc. and ratio of times of ascent are in ratio of


r r
r | vf - vi | 96km / h - 0 u1 : u2 : u3 etc.
For cheetah | a av| = =
Dt 2sec 8. During free fall velocity increases by equal amount every
1000 m decend and distance covered during 1st, 2nd, 3rd seconds
96 ´ of fall, are 4.9m, 14.7m, 24.5m.
3600 sec
= = 15 m/s2
2 sec u=0 t=0
10 4.9 m
96 ´ 9.8 m/s t=1
r
For cat | a av| = 36 = 5 m/s2. 14.7 m
6 19.6 m/s t=2
Example 4.
A particle is moving in east direction with speed 5 m/s.
After 10 sec it starts moving in north direction with same 24.5 m
speed. Find average acceleration.
Solution : N 29.4 m/s t=3
r r
| v f| = | v i| = 5 m/s
9. If a body is projected horizontally from top of a tower, the
Acceleration ¹ 0
W E time taken by it to reach the ground does not depend on
(due to change in direction the velocity of projection, but depends on the height of
of velocity Av. acceleration,
2h
r
D v = vr f – vr i = vr f + (– vr i) S
tower and is equal to t =
g
.
Motion in a Straight Line 39

10 If velocity v of a body changes its direction by q without


Correct. Average velocity = V Dr r -r i
change in magnitude then the change in velocity will be av = = f
Dt tf – ti
q
2v sin .
2 (where r i is position vector at time ti and r f is position
11. From the top of a tower a body is projected upward with a
vector at time tf).
certain speed, 2nd body is thrown downward with same
Whereas instantaneous velocity
speed and 3rd is let to fall freely from same point then
ur ur
uur D r dr
t3 = t1t 2 V inst. = lim =
Dt ®0 Dt dt
where t 1 = time taken by the body projected upward,
t 2 = time taken by the body thrown downward and It is important to note that average velocity is equal to
t3 = time taken by the body falling freely. instantaneous velocity only when the case is of uniform
12. If a body falls freely from a height h on a sandy surface and velocity.
it buries into sand upto a depth of x, then the retardation Incorrect. Taking acceleration as negative (– a) even when
acceleration is an unknown.
æ h + xö
produced by sand is given by a = g ç . Correct. Take acceleration as (a) when it is unknown even if
è x ÷ø
we know that the motion is a case of deceleration or
13. In case of air resistance, the time of ascent is less than time retardation. On solving, we will find the value of (a) to be
of descent of a body projected vertically upward i.e. ta < td. negative .

.IN
14. When atmosphere is effective, then buoyancy force always Incorrect. Magnitude of instantaneous velocity is different
acts in upward direction whether body is moving in upward from instantaneous speed .
or downward direction and it depends on volume of the Correct. Magnitude of instantaneous velocity is equal to
body. The viscous drag force acts against the motion.
AL
the instantaneous speed in any case.
15. If bodies have same volume but different densities, the Example 5.
buoyant force remains the same.
N
The numerical ratio of average velocity to average speed is
CAUTION : Please note that dropping body gets the velocity (a) always less than one (b) always equal to one
R

of the object but if the object is in acceleration, the body dropped (c) always more than one (d) equal to or less than one
will not acquire the acceleration of the object. Solution : (d)
U

It is equal to or less than one as average velocity depends


COMMON DEFAULT
JO

upon displacement whereas average speed depends upon


Incorrect. In the question, if it is given that a body is path length.
dropped, taking its initial velocity zero. Example 6.
U

Correct. The initial velocity is zero if the object dropping The distance travelled by a body is directly proportional
the body is also at rest (zero velocity). But if the object to the time taken. Its speed
ED

dropping the body is having a velocity, then the body being (a) increases (b) decreases
dropped will also have initial velocity which will be same as (c) becomes zero (d) remains constant
that of the object. Solution : (d)
For example : s
When s µ t, so = v = constant.
(a) When an aeroplane flying horizontally drops a bomb. t
(b) An ascending helicopter dropping a food packet. Example 7.
(c) A stone dropped from a moving train etc. A particle is projected vertically upwards. Prove that it will be
Incorrect. Applying equations of motion in case of non- at 3/4 of its greatest height at time which are in the ratio 1 : 3.
uniform acceleration of the body. Solution :
If u is the initial velocity of a particle while going vertically
Correct. The equations of motion are for uniformly
accelerated motion of the body. u2
Please note that when the case is of non-uniform acceleration upwards, then the maximum height attained is h = .
2g
we use calculus (differentiation and integration). 3
dv vdv ds If t is the time when particle reaches at a height h, then
a= = ; v= . 4
dt ds dt using the relation
In fact calculus method is a universal method which can be 1 2 3 1
s = ut + at ; we have h = ut + (-g ) t 2
used both in case of uniform as well as non-uniform 2 4 2
acceleration.
Incorrect. Taking average velocity same as that of 3 æç u 2 ö÷ 1 2 2 2u 3 u2
or, 4 ç 2 g ÷ = ut - gt or t - t + =0
instantaneous velocity. è ø 2 g 4 g2
EBD_7179
40 PHYSICS

Solving it for t, we have Total distance = 100 + 100 = 200 m.


200
2u 4u 2 3 u2 \ 20 = or u - v = 10
± - 4 ´1´ u-v
2
g g 4 g2 u u
t= = ± 200
2 g 2g and 10 = or u + v = 20
u+v
t = u/g By solving, we get, u = 15 m/sec and v = 5 m/sec.

Example 10.
A body starts from rest and moves with a uniform
t1 t2 = 3u/2g acceleration. The ratio of the distance covered in the nth
h sec to the distance covered in n sec is
3h/4 2 1 1 1
(a) – (b) –
n n2 n 2 n

2 1 2 1
u u u (c) 2
– 2 (d) +
Taking negative sign, t 1 = - = ; n n n n2
g 2g 2g Solution : (a)

.IN
The distance covered in nth second
u u
Taking positive sign, t 2 = + = 3u / 2g.
g 2g a a
sn = u + (2 n - 1) or s n = 0 + (2 n - 1) ...(1)
AL 2 2
t 1 æ u / 2g ö 1 Further distance covered in n second
=ç ÷=
t 2 çè 3u / 2g ÷ø 3 .
N
1 2 1
s = ut+ a t = 0 + a n2 ...(2)
Example 8. 2 2
R

A police party in a jeep is chasing a dacoit on a straight


road. The jeep is moving with a maximum uniform speed v. a
(2 n - 1)
U

sn 2 2 1
The dacoit rides on a motorcycle of his waiting friend when \ = = -
2
s n n2
JO

the jeep is at a distance d from him and the motorcycle (an / 2)


starts with constant acceleration a. Show that the dacoit Example 11.
will be caught if v ³ 2ad . The water drop falls at regular intervals from a tap 5 m
U

Solution : above the ground. The third drop is leaving the tap at
Suppose the dacoit is caught at a time t sec after the instant the first drop touches the ground. How far above
ED

motorcycle starts. The distance travelled by the motorcycle the ground is the second drop at that instant?
during this interval is (a) 1.25 m (b) 2.50 m
(c) 3.75 m (d) 4.00 m
1 2 Solution : (c)
s= at ...(1)
2 See fig. Let t be the time interval between any two drops.
During the interval, the jeep travels a distance,
s + d = vt ...(2) Tap
3rd drop
x
1 2 v ± v 2 - 2ad 2nd drop
By (1) and (2), at - vt + d = 0 or t =
2 a 5m
The dacoit will be caught if t is real and positive. This will be h
Ist drop
possible if v 2 - 2ad > 0 or v > 2ad .
ground
Example 9.
Two trains, each of length 100 m, are running on parallel
1
tracks. One overtakes the other in 20 second and one For third drop 5 = g (2 t ) 2 or g t 2 = 5 / 2
crosses the other in 10 second. Calculate the velocities of 2
each train. 1 2 1 5 5
Solution : For second drop x = g t or x = ´ = = 1.25 m
2 2 2 4
Let u and v be the velocities of the trains.
Therefore h = 5 – x = 5 – 1.25 = 3.75 m
The relative velocity of overtaking is u – v while the relative
Hence option (c)
velocity of crossing is u + v.
Motion in a Straight Line 41

Example 12. Solution :


The height of a tower is h metre. A body is thrown from the 1
top of tower vertically upward with some speed, it takes From formula, Sn th = u + a(2n –1)
2
t1, second to reach the ground. Another body thrown from
a
the top of tower with same speed downwards and takes t2 S7 th = u + (2 ´ 7 - 1) but S7 th = 20m
2
seconds to reach the ground. If third body, released from
same place takes ‘t’ second to reach the ground, then a 13a
\ 20 = u + ´ 13 Þ 20 = u + ... (1)
2 2
t1 +t2 t1
(a) t = (b) t = t 17a
2 2 also s 9 th = 24m \ 24 = u + ... (2)
2
2 1 1 13a
(c) t = t + t (d) t = (t1 t2 ) From eqn. (1) u = 20 - ... (3)
1 2 2
Solution : (d) Substitute this value of in eqn. (2)
Let u be the initial velocity of the body. Then at time t1 13a 17a
24 = 20 - +
1 2 2
h = - u t1 + g t12 ... (1)
2 17a 13a
24 - 20 = -
1 2 2

.IN
and at time t2, h = u t 2 + g t 22 ...(2)
2 4a 4
4= Þ 4 = 2a Þ a = = 2 m / s2
From eqs. (1) and (2), we get AL 2 2
1 1 13a
- u t 1 + g t 12 = u t 2 + g t 2 2 Substituting this value of a in eqn. (3) u = 20 -
2 2 2
1 13 ´ 2
N
or g ( t 12 - t 2 2 ) = u ( t 1 + t 2 ) Þ u = 20 - Þ u = 20 - 13
2 2
R

1 \ u = 7 m/s
or g ( t1 + t 2 ) ( t 1 - t 2 ) = u ( t1 + t 2 )
U

2 a
Now, s15th = u + (2 ´ 15 - 1)
1 2
JO

or u = g ( t1 - t 2 ) ...(3)
2 2
Substituting the value of u in eqn. (2) we get = 7 + (29) = 7 + 29 = 36m
2
Example 15.
U

1 1 1
h= g ( t1 - t 2 ) t 2 + g t 2 2 or h = g t1 t 2 ...(4) A train starts from rest and for the first kilometer moves
2 2 2
ED

with constant acceleration, for the next 3 kilometers it has


1 2 constant velocity and for another 2 kilometers it moves
For third body, h = gt ...(5)
2 with constant retardation to come to rest after 10 min.
Find the maximum velocity and the three time intervals in
From eqs. (4) and (5) we get, t = t1t 2
the three types of motion.
Example 13. Solution :
A bullet moving with a speed 10 m/s hits the wooden plank Let the three time intervals be t1 min, t2 min, and t3 min.
and is stopped when it penetrates the plank 20 cm deep. respectively.
Calculate retardation of the bullet. Let the maximum velocity attained be v m/min.
Solution : A t1 B v–t2 C t3 D
2
v0 = 10 m/s, v = 0 and s = 20 cm. = = 0.02m 1000m 3000m 2000m
100
Using v2 – v02 = 2ax æ 0 + vö
1000 = ç t
For A to B è 2 ÷ø 1 Þ 2000 = vt1 ........... (1)
-100
0 - (10)2 = 2a (0.02) Þ =a For B to C 3000 = vt 2 ........... (2)
2 ´ 0.02
or a = – 2500 m/s2 [Using in both equations disp. = mean vel. × time]
Retardation = 2500 m/s2 æ v + 0ö
and for C to D 2000 = çè ÷ t 3 Þ 4000 = vt 3 ...... (3)
Example 14. 2 ø
A body covers a distance of 20m in the 7th second and 24m Adding eqs. (1), (2) and (3), we get
in the 9th second. How much distance shall it cover in 9000 = v (t1 + t 2 + t 3 )
15th sec?
EBD_7179
42 PHYSICS

9000 900 ´ 10-3 10 + 102 - 4 ´ 5(-40) 10 + 100 + 800


\ v= = 900 m / min . = km = 54 km / hr. or t= =
10 1/ 60 hr. 2´5 10
Now, from eqs. (1), (2) and (3) we get 10 + 30
= = 4 sec.
2000 20 2 10
t1 = = = 2 min. , (ii) It will pass from where it was projected after
900 9 9
2 ´ 10
3000 10 1 t= = 2 sec.
t2 = = = 3 min. g
900 9 3 (iii) Velocity with which stone strikes the ground
4000 40 4 V = 10 + g × 2 = 30 m/s
and t 3 = = = 4 min.
VARIOUS GRAPHS RELATED TO MOTION
900 9 9
Example 16. (a) Displacement-time graph - In this graph time is plotted on
A falling stone takes 0.2 seconds to fall past a window x-axis and displacement on y-axis.
which is 1m high. From how far above the top of the window (i) For a stationary body (v = 0) the time-displacement
was the stone dropped ? graph is a straight line parallel to time axis.
Solution :
1 1
h = gt 2 ; h +1 = g(t + 0.2)2
2 2

.IN
Displ
1 2 1 1 1
gt + 1 = gt 2 + g(0.2) 2 + g ´ 2 ´ 0.2t Velocity = 0
2 2 2 2
Body is at rest
u=0
AL
h
N
1m Time
R

(ii) When the velocity of a body is constant then time-


U

displacement graph will be an oblique straight line.


1 4 2
1= + 0.2gt ; = 2t Þ t = æ dx ö
JO

5 5 5 Greater the slope ç = tan q ÷ of the straight line,


è dt ø
1 4 4
h= g = m higher will be the velocity.
U

2 25 5
Example 17.
ED

From the top of a multi-storeyed building 40m tall, a boy


projects a stone vertically upwards with an initial velocity
of 10 ms–1 such that it eventually falls to the ground. (i)
After how long will the stone strike the ground ? (ii) After
how long will it pass through the point from where it was
projected ? (iii) What will be its velocity when it strikes the
ground ? Take g = 10 ms–2.
Solution :
r r 1r
Using, S = ut + at 2 (iii) If the velocity of a body is not constant then the time-
2 displacement curve is a zig-zag curve.
1 2
(i) -40 = 10t - gt or -40 = 10t - 5t 2
2
[Q taking g = 10m/s2]
or 5t 2 - 10t - 40 = 0

u=10m/s
u
40m
(iv) For an accelerated motion the slope of time-
displacement curve increases with time while for
v
decelerated motion it decreases with time.
h
Motion in a Straight Line 43

(iv) When the particle moves with non-uniform acceleration


Displ. Displ. and its initial velocity is zero.
a>0
a<0

t t
(v) When the particle returns towards the point of
reference then the time-displacement line makes an
angle q > 90° with the time axis.
(v) When the acceleration decreases and increases.

Displ
q > 90º

g
in
V

as
re
ec
ad
.IN
t g
asin
(b) Velocity-time graph - In this curve time is plotted along re
x-axis and velocity is plotted along y-axis.
AL a inc
(i) When the velocity of the particle is constant or t
acceleration is zero. (vi) The total area enclosed by the time - velocity curve
N
represents the distance travelled by a body.
R

Velocity Vel. const, a = 0


U
JO

t
U

(ii) When the particle is moving with a constant


acceleration and its initial velocity is zero.
ED

N OT E: While finding displacement through v – t graph,


keeping sign under consideration.

(c) Acceleration-time graph - In this curve the time is plotted


along X-axis and acceleration is plotted along Y-axis.
(i) When the acceleration of the particle is zero.

(iii) When the particle is moving with constant retardation.


a Acc = 0

t
V Retardation (ii) When acceleration is constant

a
a = const

t t
EBD_7179
44 PHYSICS

Total distance covered in 10 s


(iii) When acceleration is increasing and is positive.
1 1 1
= ´ 6 ´ 20 + ´ 2 ´ 20 + ´ 2 ´ 10 = 90 m
2 2 2
a a increasing Total displacement in 10s
1 1 1
= ´ 6 ´ 20 - ´ 2 ´ 20 + ´ 2 ´ 10 = 50 m
2 2 2
Example 19.
t Two trains, which are moving along different tracks in
(iv) When acceleration is decreasing and is negative opposite directions, are put on the same track due to a
mistake. Their drivers, on noticing the mistake, start slowing
a a de down the trains when the trains are 300 m apart. Graphs
cr e given below show their velocities as function of time as the
as
trains slow down, The separation, between the trains when
in
g

both have stopped, is


V(m/s) V(m/s)

t 40
(v) When initial acceleration is zero and rate of change of 20 8
acceleration is non-uniform t (s) t (s)
10
Train I

.IN
–20 Train II
a
(a) 120 m (b) 280 m
(c) 60 m (d) 20 m.
AL
Solution : (d)
N
t
R

(vi) The change in velocity of the particle = area enclosed


by the time-acceleration curve.
U

Initial distance between trains is 300m. Displacement of 1st


JO

train is calculated by area under v – t.


a 1
Curve of train I = displacement = ´ 10 ´ 40 = 200m
2
U

1
Displacement of train II = ´ 8 ´ (-20) = -80m
ED

2
\ Distance between the two trains is 20m.
t Example 20.
Example 18. Figure given below shows the variation of velocity of a
The velocity-time graph of a body moving in a straight line particle with time.
is shown in fig. Find the displacement and distance Y
travelled by the body in 10 seconds.
u (m/s)

8
20 A
6
velocity

D 4
(m/s)

10
2
0 t (s)
2 4 6 8 10 X
1 2 3 4 5 6 7
–10
Find the following : time(sec)
–20
(i) Displacement during the time intervals
C (a) 0 to 2 sec, (b) 2 to 4 sec. and (c) 4 to 7 sec.
Solution : (ii) Accelerations at –
The area enclosed by velocity-time graph with time axis (a) t = 1 sec, (b) t = 3 sec. and (c) t = 7 sec.
measures the distance travelled in a given time. Displacement (iii) Average acceleration –
covered from 0 to 6 seconds is positive; from 6 to 8 seconds (a) between t = 0 to t = 4 sec.
is negative and from 8 to 10 seconds is positive; whereas (b) between t = 0 to t = 7 sec.
distance covered is always positive. (iv) Average velocity during the motion.
Motion in a Straight Line 45
Solution : RELATIVE VELOCITY (In one dimension)
(i) (a) Displacement between t = 0 sec. to t = 2 sec. The velocity of A relative to B is the velocity with which A appears
1 to be moving w.r.t.an observer who is moving with the velocity of B
Þ ´ 2 sec ´ 8 m / s = 8m Relative velocity of A w.r.t. B
2 r r r
(b) Between t = 2 sec. to t = 4 sec. v AB = vA - vB
Þ 2 sec ´ 8 m / s = 16m Similarly, relative velocity of B w.r.t. A
(c) Between t = 4 sec. to t = 7 sec. r r r
vBA = vB - vA
1 Case 1: Bodies moving in same direction :
Þ ´ 3 sec ´ 8 m / s = 12m
2
(ii) Acceleration = slope of v – t curve vA vB
(a) At t = 1 sec, r r r
v AB = vA - vB Þ vAB = vA – vB
8 m / sec
slope = m / sec = 4 m / s 2
2 sec Case 2: Bodies moving in opposite direction :
(b) At t = 3 sec, slope = 0 – +
vA vB
(c) At t = 7 sec,
8 2 r r r
slope = - = -2 m / s
2 v AB = v A - ( -vB ) Þ vAB = vA + vB
3 3 Example 22.
Total change in velocity Two trains, one travelling at 54kmph and the other at
(iii) Average acceleration = 72kmph, are headed towards each other on a level track.

.IN
Total change in time
When they are two kilometers apart, both drivers
(a) Between t = 0 to t = 4 sec.,
simultaneously apply their brakes. If their brakes produces
8m/s equal retardation in both the trains at a rate of 0.15 m/s2,
Average acceleration = = 2m / s 2
4
ALdetermine whether there is a collision or not.
(b) Between t = 0 to t = 7 sec., Solution :
0 Speed of first train = 54 kmph = 15m/s.
Average acceleration = = 0
N
7 Speed of second train = 72kmph = 20 m/s
As both the trains are headed towards each other, relative
R

Total displacement
(iv) Average velocity = velocity of one train with respect to other is given as
Total time vr=15 + 20 = 35 m/s
U

8 + 16 + 12 36 1 Both trains are retarded by acceleration of 0.15 m/s2. Relative


= = = 5 m /s retardation ar = 0.15 + 0.15 = 0.3 m/s2.
JO

7 7 7 Now, we assume one train is at rest and other is coming at


Example 21.
The velocity-time graph of a particle moving along a 35m/s retarded by 0.3 m/s2 at a distance of two kilometer.
straight line is shown below. The maximum distance travelled by the moving train while
U

The acceleration and deceleration are same and it is equal retarding is


ED

to 4 m/s². If the average velocity during the motion is 15 m/ v 2r


s and total time of motion is 20 second then find (35) 2
smax = = = 2041.66m
2a r 2 ´ 0.3
It is more than 2km, which shows that it will hit the second
20 – 2t train.
v
Example 23.
Two cars started simultaneously towards each other from
t towns A and B which are 480 km apart. It took first car
t 20–t 20 travelling from A to B 8 hours to cover the distance and
(a) the value of t (b) the maximum velocity of the particle second car travelling from B to A 12 hours. Determine
during the journey. (c) the distance travelled with uniform when the car meet after starting and at what distance from
velocity. town A. Assuming that both the cars travelled with constant
Solution : v = 0 + at speed.
1
Total displacement = (20 - 2t + 20) ´ 4t = 2t (40 - 2t) 480
2 Solution : Velocity of car from A = = 60 km / hour
Total displacement 8
Average velocity =
Total time 480
2t (40 - 2t) velocity of car from B = = 40 km / hour
15 = 12
20 Let the two cars meet at t hour
Solving quadratic equation, 150 = 40t – 2t2 Þ t = 5 sec.
(another solution not acceptable think why!) total distance 480
\ t= = = 4.8 hour
Maximum velocity = 4t = 4 × 5 = 20 m/s relative velocity of meeting 60 + 40
Distance travelled with uniform velocity The distance s = vA × t = 60 × 4.8 = 288 km.
= (20 – 2t) V = (20 – 2 × 5) × 20 = 200 m
46

ED
U
JO
U
R
N
AL
.IN
PHYSICS

EBD_7179
Motion in a Straight Line 47

1. The study of motion, without consideration of its cause is (a) 1/2 (b) 1 / 3
studied in
(a) statistics (b) kinematics (c) 3 (d) 1/3
(c) mechanics (d) modern physics 12. A person travels along a straight road for the first half time
2. The ratio of the numerical values of the average velocity with a velocity v1 and the second half time with a velocity
and average speed of a body is always: v2. Then the mean velocity v is given by
(a) unity (b) unity or less
v1 + v 2 2 1 1
(c) unity or more (d) less than unity (a) v= (b) = +
3. A particle has moved from one position to another position 2 v v1 v 2
(a) its distance is zero
v2
(b) its displacement is zero (c) v= v1 v 2 (d) v=
(c) neither distance nor displacement is zero v1
(d) average velocity is zero 13. A particle covers half of the circle of radius r. Then the
4. The displacement of a body is zero. The distance covered displacement and distance of the particle are respectively

.IN
(a) is zero (a) 2pr, 0 (b) 2r, pr
(b) is not zero pr
(c) may or may not be zero (c) , 2r (d) pr, r
2
(d) depends upon the acceleration
AL
14. The distance through which a body falls in the nth second
5. Which of the following changes when a particle is moving is h. The distance through which it falls in the next second is
with uniform velocity? g
(a) h (b) h +
N
(a) Speed (b) Velocity
2
(c) Acceleration (d) Position vector
(c) h – g (d) h + g
R

6. The slope of the velocity time graph for retarded motion is 15. It is given that t = px2 + qx, where x is displacement and t is
(a) positive (b) negative
U

time. The acceleration of particle at origin is


(c) zero (d) can be +ve, –ve or zero
7. The area of the acceleration-displacement curve of a body 2p 2q 2p 2q
JO

(a) - - 3
(b) (c)
3
(d)
gives q 3 p q p3
(a) impulse 16. Figure shows the v-t graph for two particles P and Q. Which
(b) change in momentum per unit mass
U

of the following statements regarding their relative motion


(c) change in KE per unit mass is true ?
ED

(d) total change in energy Their relative velocity is


8. If the displacement of a particle varies with time as V
x = t + 7 , the P
(a) velocity of the particle is inversely proportional to t Q
(b) velocity of the particle is proportional to t
(c) velocity of the particle is proportional to t
(d) the particle moves with a constant acceleration
9. The initial velocity of a particle is u (at t = 0) and the
O T
acceleration a is given by f t.
Which of the following relation is valid? (a) is zero
(a) v = u + f t2 (b) v = u + f t2/2 (b) is non-zero but constant
(c) v = u + f t (d) v = u (c) continuously decreases
10. The displacement x of a particle moving along a straight line (d) continuously increases
at time t is given by 17. A stone is dropped into a well in which the level of water is
x = a 0 + a1 t + a 2 t2 h below the top of the well. If v is velocity of sound, the time
What is the acceleration of the particle T after which the splash is heard is given by
(a) a 1 (b) a 2
æ 2h ö h
(c) 2 a2 (d) 3 a2 (a) T = 2h/v (b) T= ç ÷ +
11. The displacement-time graphs of two particles A and B are è g ø v
straight lines making angles of respectively 30º and 60º with
the time axis. If the velocity of A is vA and that of B is vB, the æ 2h ö h æ h ö 2h
(c) T= ç ÷+ (d) T = çç ÷÷ +
value of vA/vB is è v ø g è 2g ø v
EBD_7179
48 PHYSICS

18. A point traversed half of the distance with a velocity v0. a


(c) the particle cannot go beyond x =
The half of remaining part of the distance was covered with b
velocity v1 & second half of remaining part by v2 velocity. (d) the particle will not come back to its starting point at
The mean velocity of the point, averaged over the whole t®¥
time of motion is
22. The displacement of a particle is given by x = t + 1 . Which
v 0 + v1 + v 2 2 v 0 + v1 + v 2
(a) (b) of the following statements about its velocity is true ?
3 3 (a) It is zero
v 0 + .2 v1 + 2 v 2 2 v 0 (v1 + v 2 ) (b) It is constant but not zero
(c) (d)
3 (2v 0 + v1 + v 2 ) (c) It increases with time
19. The acceleration of a particle is increasing linearly with time (d) It decreases with time
t as bt. The particle starts from the origin with an initial 23. Two bodies of masses m1 and m2 fall from heights h1 and h2
velocity v0. The distance travelled by the particle in time t respectively. The ratio of their velocities, when they hit the
will be ground is
1 2 1 3 h1 h1
(a) v 0 t + bt (b) v 0 t + bt (a) (b)
3 3 h2 h2
1 3 1 2
(c) v 0 t + bt (d) v 0 t + bt
6 2 m1h1 h12
20. The deceleration experienced by a moving motorboat after (c) (d)
its engine is cut off, is given by dv/dt = – kv3 where k is
m1h 2 h 22

.IN
constant. If v0 is the magnitude of the velocity at cut-off, 24. Two cars A and B are travelling in the same direction with
the magnitude of the velocity at a time t after the cut-off is velocities vA and vB (vA>vB). When the car A is at a distance
v0 d behind the car B the driver of the car A applies brakes
(a) (c) v 0 e - kt
AL
producing a uniform retardation a. There will be no collision
(2 v 0 2 kt + 1) when
(c) v 0 / 2 (d) v 0 (v A - v B )2 v 2A - v 2B
N
(a) d< (b) d<
21. The displacement x of a particle varies with time according 2a 2a
R

a
to the relation x = (1 - e - bt ). Then select the false (v A - v B ) 2 v 2 - v 2B
b (c) d> (d) d > A
U

alternatives. 2a 2a
1 25. A body is thrown upwards and reaches its maximum height.
JO

(a) At t = , the displacement of the particle is nearly


b At that position
2æa ö (a) its acceleration is minimum
ç ÷ (b) its velocity is zero and its acceleration is also zero
U

3èbø
(c) its velocity is zero but its acceleration is maximum
(b) the velocity and acceleration of the particle at t = 0 are
(d) its velocity is zero and its acceleration is the acceleration
ED

a and –ab respectively


due to gravity.

1. The position x of a particle varies with time (t) as The initial velocity and acceleration are respectively
x = A t2 – B t3. The acceleration at time t of the particle will be (a) b, – 4 d (b) – b, 2 c
equal to zero. What is the value of t? (c) b, 2 c (d) 2 c, – 4 d
2A A 4. A passenger travels along the straight road for half the
(a) (b)
3B B distance with velocity v1 and the remaining half distance
A with velocity v2. Then average velocity is given by
(c) (d) zero
3B (a) v1 v 2 (b) v2 2/ v1 2
2. The acceleration of a particle, starting from rest, varies with (c) (v1 + v2 )/2 (d) 2v1v2 / (v1 + v2)
time according to the relation 5. A point moves with uniform acceleration and v1, v2 and v3
2
a = -s w sin w t denote the average velocities in t1, t2 and t3 sec. Which of
The displacement of this particle at a time t will be the following relation is correct?
(a) s sin w t (b) s w cos w t (a) ( v 1 - v 2 ) : ( v 2 - v 3 ) = ( t 1 - t 2 ) : ( t 2 + t 3 )
1 (b) ( v 1 - v 2 ) : ( v 2 - v 3 ) = ( t 1 + t 2 ) : ( t 2 + t 3 )
(c) s w sin w t (d) - (s w 2 sin wt ) t 2
2 (c) ( v1 - v 2 ) : ( v 2 - v 3 ) = ( t1 - t 2 ) : ( t1 - t 3 )
3. The displacement of a particle is given by
y = a + b t + c t2 – d t4 (d) ( v1 - v 2 ) : ( v 2 - v 3 ) = ( t 1 - t 2 ) : ( t 2 - t 3 )
Motion in a Straight Line 49

6. A bus starts moving with acceleration 2 m/s2. A cyclist 96 m (b) from rest and moves with uniform acceleration
behind the bus starts simultaneously towards the bus at (c) with an initial velocity and moves with uniform
20 m/s. After what time will he be able to overtake the bus? acceleration
(a) 4 sec (b) 8 sec (d) with an initial velocity and moves with uniform velocity
(c) 12 sec (d) 16 sec 16. A stone thrown upward with a speed u from the top of the
7. When the speed of a car is v, the minimum distance over tower reaches the ground with a velocity 3u. The height of
which it can be stopped is s. If the speed becomes n v, what the tower is
will be the minimum distance over which it can be stopped (a) 3u2/g (b) 4u2/g
2
(c) 6u /g (d) 9u2/g
during same retardation
(a) s/n (b) n s 17. A smooth inclined plane is inclined at an angle q with
(c) s/n 2 (d) n2 s horizontal. A body starts from rest and slides down the
8. The two ends of a train moving with constant acceleration inclined surface.
pass a certain point with velocities u and v. The velocity with
which the middle point of the train passes the same point is
(a) (u + v)/2 (b) (u2 + v2)/2
h
(c) (u 2 + vu2 ) / 2 (d) u 2 + v2
9. A particle accelerates from rest at a constant rate for some q
time and attains a velocity of 8 m/sec. Afterwards it Then the time taken by it to reach the bottom is

.IN
decelerates with the constant rate and comes to rest. If the
total time taken is 4 sec, the distance travelled is æ 2h ö æ 2l ö
(a) çç ÷÷ (b) çç ÷÷
(a) 32 m (b) 16 m AL è g ø è g ø
(c) 4 m (d) None of the above
10. The velocity of a particle at an instant is 10 m/s. After 5 sec, 1 2h ( 2h )
the velocity of the particle is 20 m/s. Find the velocity at 3 (c) (d) sin q
sin q g g
N
seconds before from the instant when velocity of a particle
18. A ball is dropped downwards, after 1 sec another ball is
is 10m/s.
R

dropped downwards from the same point. What is the


(a) 8 m/s (b) 4 m/s
distance between them after 3 sec?
U

(c) 6 m/s (d) 7 m/s


(a) 25 m (b) 20 m
11. A particle experiences constant acceleration for 20 seconds (c) 50 m (d) 9.8 m
JO

after starting from rest. If it travels a distance s1 in the first 19. Two trains are each 50 m long moving parallel towards each
10 seconds and distance s2 in the next 10 seconds, then other at speeds 10 m/s and 15 m/s respectively. After what
(a) s2 = s1 (b) s2 = 2 s1 time will they pass each other?
U

(c) s2 = 3 s1 (d) s2 = 4 s1
12. A train of 150 m length is going towards north direction at a 2
ED

(a) 5 sec (b) 4 sec


speed of 10 ms–1. A parrot flies at a speed of 5 ms–1 towards 3
south direction parallel to the railway track. The time taken (c) 2 sec (d) 6 sec
by the parrot to cross the train is equal to 20. A ball is projected vertically upwards with kinetic energy E.
(a) 12 s (b) 8 s The kinetic energy of the ball at the highest point of its
(c) 15 s (d) 10 s flight will be
13. A particle is moving along a straight line path according to (a) E (b) E / 2
the relation (c) E/2 (d) zero
s2 = at2 + 2bt + c 21. A particle is moving in a straight line with initial velocity
s represents the distance travelled in t seconds and a, b, c and uniform acceleration a. If the sum of the distance travelled
are constants. Then the acceleration of the particle varies in tth and (t + 1)th seconds is 100 cm, then its velocity after
as t seconds, in cm/s, is
(a) s – 3 (b) s 3/2 (a) 80 (b) 50
(c) s –2/3 (d) s 2 (c) 20 (d) 30
14. A stone thrown vertically upwards with a speed of 5 m/sec 22. Similar balls are thrown vertically each with a velocity
attains a height H1. Another stone thrown upwards from 20 ms–1, one on the surface of earth and the other on the
the same point with a speed of 10 m/sec attains a height H2. surface of moon. What will be ratio of the maximum heights
The correct relation between H1 and H2 is attained by them? (Acceleration on moon = 1.7 ms–2 approx)
(a) H2 = 4H1 (b) H2 = 3H1 1
(c) H1 =2H2 (d) H1 = H2 (a) 6 (b)
6
15. A body covers 26, 28, 30, 32 meters in 10th, 11th, 12th and
13th seconds respectively. The body starts 1
(c) (d) None of these
(a) from rest and moves with uniform velocity 5
EBD_7179
50 PHYSICS

23. The relative velocity VAB or VBA of two bodies A & B may 32. A particle travels half the distance with a velocity of 6 m s -1 .
be
(1) greater than velocity of body A The remaining half distance is covered with a velocity of 4
(2) greater than velocity of body B m s -1 for half the time and with a velocity of 8 m s -1 for the
(3) less than the velocity of body A rest of the half time. What is the velocity of the particle
(4) less than the velocity of body B averaged over the whole time of motion ?
(a) (1) and (2) only (b) (3) and (4) only
(a) 9 ms–1 (b) 6 ms–1
(c) (1), (2) and (3) only (d) (1), (2), (3) and (4) –1
24. A stone is thrown vertically upwards. When the particle is (c) 5.35 ms (d) 5 ms–1
at a height half of its maximum height, its speed is 10m/sec, 33. A ball released from a height falls 5 m in one second. In 4
then maximum height attained by particle is (g = 10m/sec2) seconds it falls through
(a) 8 m (b) 10 m (a) 20 m (b) 1.25 m
(c) 15 m (d) 20 m (c) 40 m (d) 80 m
25. From a 10m high building a stone ‘A’ is dropped & 34. A food packet is released from a helicopter rising steadily at
simultaneously another stone ‘B’ is thrown horizontally with the speed of 2 m/sec. After 2 seconds the velocity of the
an initial speed of 5 m/sec–1. Which one of the following packet is
statements is true?
(g = 10 m/sec2)
(a) It is not possible to calculate which one of two stones
(a) 22 m/sec (b) 20 m/sec
will reach ground first

.IN
(b) Both stones ‘A’ & ‘B’ will reach the ground (c) 18 m/sec (d) none of these
simultaneously. 35. The displacement x of a particle along a straight line at time
(c) ‘A’ stones reach the ground earlier than ‘B’ AL a1t a
(d) ‘B’ stones reach the ground earlier than ‘A’ t is given by : x = a0 + + 2 t2. The acceleration of the
2 3
26. An automobile travelling with a speed of 60 km/h, can apply
brake to stop within a distance of 20m. If the car is going particle is
N
twice as fast i.e., 120 km/h, the stopping distance will be a2 2a 2 a1 a
(a) 60 m (b) 40 m (a) (b) (c) (d) a0 + 2
R

3 3 2 3
(c) 20 m (d) 80 m
36. A rubber ball is dropped from a height of 5 metre on a plane
U

27. The motion of a particle is described by the equation u = at.


The distance travelled by particle in first 4 sec is where the acceleration due to gravity is same as that onto
JO

(a) 4a (b) 12a the surface of the earth. On bouncing, it rises to a height of
(c) 6a (d) 8a 1.8 m. On bouncing, the ball loses its velocity by a factor of
28. If you were to throw a ball vertically upward with an initial 2 16
3
(b) 9
U

velocity of 50 m/s, approximately how long would it take for (a) (c) (d)
5 25 5 25
the ball to return to your hand? Assume air resistance is
ED

37. –1
A boy moving with a velocity of 20 km h along a straight
negligible.
(a) 2.5 s (b) 5.0 s line joining two stationary objects. According to him both
(c) 7.5 s (d) 10 s objects
29. A body travels 2 m in the first two second and 2.20 m in the (a) move in the same direction with the same speed of
next 4 second with uniform deceleration. The velocity of the 20 km h–1
body at the end of 9 second is (b) move in different direction with the same speed of
(a) – 10 m s -1 (b) – 0.20 m s -1 20 km h–1
(c) move towards him
(c) – 0.40 m s -1 (d) – 0.80 m s -1
(d) remain stationary
30. From a 200 m high tower, one ball is thrown upwards with
38. A man leaves his house for a cycle ride. He comes back to
speed of 10 m s -1 and another is thrown vertically his house after half-an-hour after covering a distance of
downwards at the same speeds simultaneously. The time one km. What is his average velocity for the ride ?
difference of their reaching the ground will be nearest to (a) zero (b) 2 km h–1
(a) 12 s (b) 6 s
1
(c) 2 s (d) 1 s (c) 10 km s–1 (d) km s -1
31. A rocket is fired upward from the earth’s surface such that it 2
39. A car travels from A to B at a speed of 20 km h–1 and returns
creates an acceleration of 19.6 m s - 2 . If after 5 s, its engine
is switched off, the maximum height of the rocket from earth’s at a speed of 30 km h –1. The average speed of the car for the
surface would be whole journey is
(a) 980 m (b) 735 m (a) 5 km h–1 (b) 24 km h–1
(c) 25 km h –1 (d) 50 km h–1
(c) 490 m (d) 245 m
Motion in a Straight Line 51

40. A body dropped from a height ‘h’ with an initial speed zero, (b) increases
strikes the ground with a velocity 3 km/hour. Another body (c) increases then decreases
of same mass dropped from the same height ‘h’ with an (d) decreases then increases
initial speed u' = 4 km/hour. Find the final velocity of second 48. The displacement x of a particle varies with time t as
mass, with which it strikes the ground x = ae-at + bebt, where a, b, a and b are positive constants.
(a) 3 km/hr (b) 4 km/hr The velocity of the particle will
(c) 5 km/hr (d) 6 km/hr (a) be independent of a and b
41. An electron starting from rest has a velocity that increases (b) drop to zero when a = b
-2
linearly with time i.e. v = kt where k = 2 m s . The distance (c) go on decreasing with time
(d) go on increasing with time
covered in the first 3 second is
49. Which one of the following equations represents the motion
(a) 9 m (b) 16 m
of a body with finite constant acceleration ? In these
(c) 27 m (d) 36 m
equations, y denotes the displacement of the body at time t
42. A body released from the top of a tower falls through half and a, b and c are constants of motion.
the height of the tower in 2 s. In what time shall the body fall
through the height of the tower ? (a) y = at y = at + bt 2
(b)
(a) 4 s (b) 3.26 s a
(c) y = at + bt 2 + ct 3 (d) y = + bt
(c) 3.48 s (d) 2.828 s t
43. The displacement x of a particle at the instant when its 50. The dependence of velocity of a body with time is given by

.IN
velocity is v is given by v = 3x + 16 . Its acceleration and the equation v = 20 + 0.1t 2 . The body is in
initial velocity are
AL(a) uniform retardation
(a) 1.5 units, 4 units (b) 3 units, 4 units (b) uniform acceleration
(b) 16 units, 1.6 units (d) 16 units, 3 units (c) non-uniform acceleration
44. Let A, B, C, D be points on a vertical line such that (d) zero acceleration.
N
AB = BC = CD. If a body is released from position A, the 51. Two stones are thrown from the top of a tower, one straight
down with an initial speed u and the second straight up
R

times of descent through AB, BC and CD are in the ratio.


with the same speed u. When the two stones hit the ground,
U

(a) 1 : 3 - 2 : 3 + 2 (b) 1 : 2 - 1 : 3 - 2 they will have speeds in the ratio


(a) 2 : 3 (b) 2 : 1
JO

(c) 1 : 2 -1 : 3 (d) 1 : 2 : 3 - 1
(c) 1 : 2 (d) 1 : 1
45. A body moves in a straight line along Y-axis. Its distance y 52. A graph of acceleration versus time of a particle starting
(in metre) from the origin is given by y = 8t – 3t2. The average from rest at t = 0 is as shown in Fig. The speed of the particle
U

speed in the time interval from t = 0 second to t = 1 second at t = 14 second is


is
ED

(a) – 4 ms–1 (b) zero 4


(c) 5 ms–1 (d) 6 ms–1
46. The acceleration due to gravity on planet A is nine times the 2
12
a (in ms-2 )

acceleration due to gravity on planet B. A man jumps to a 0


2 4 6 8 10 14 t (in sec .)
height 2m on the surface of A. What is height of jump by -2
same person on planet B?
(a) 2/3 m (b) 2/9 m -4
(c) 18 m (d) 6 m (a) 2 ms–1 (b) 34 ms–1
47. In the given figure the distance PQ is constant. SQ is a (c) 20 ms–1 (d) 42 ms–1
vertical line passing through point R. A particle is kept at R
53. In the displacement d versus time t graph given below,
and the plane PR is such that angle q can be varied such the value of average velocity in the time interval 0 to 20 s is
that R lies on line SQ. The time taken by particle to come (in m/s)
down varies, as the q increases

S 50
40
d/m

30
R 20
10
q 0
0 10 20 30 t / s
P Q
(a) 1.5 (b) 4
(a) decreases continuously (c) 1 (d) 2
EBD_7179
52 PHYSICS

54. Figure shows the position of a particle moving along the 61. For the velocity time graph shown in the figure below the
X-axis as a function of time. distance covered by the body in the last two seconds of its
x (m) motion is what fraction of the total distance travelled by it in
all the seven seconds?
20
10
10 8 B C
­ 6
–1
2 4 6 t(s) ms 4
Which of the following is correct? velocity 2
A D
(a) The particle has come to the rest 6 times 0 1 2 3 4 5 6 7 8
(b) The maximum speed is at t = 6 s.
(c) The velocity remains positive for t = 0 to t = 6 s. 1 1 2 1
(a) (b) (c) (d)
(d) The average velocity for the total period shown is 2 4 3 3
negative. 62. Which of the following graph cannot possibly represent
55. A steel ball is bouncing up and down on a steel plate with a one dimensional motion of a particle?
period of oscillation of 1 second. If g = 10 ms–2, then it x x
bounces up to a height of
(a) 5 m (b) 10 m

.IN
(c) 2.5 m (d) 1.25 m (a) t (b)
56. A body starts from rest and travels a distance x with uniform t
acceleration, then it travels a distance 2x with uniform speed, AL
finally it travels a distance 3x with uniform retardation and
comes to rest. If the complete motion of the particle is along speed
a straight line, then the ratio of its average velocity to
N
maximum velocity is
(a) 2/5 (b) 3/5
R

(c) (d) All of the above


(c) 4/5 (d) 6/7 t
57. When two bodies move uniformly towards each other, the
U

distance decreases by 6 ms–1. If both bodies move in the


JO

same directions with the same speeds (as above), the 63. The distance time graph of a particle at time t makes angles
distance between them increases by 4 ms–1. Then the speeds 45° with the time axis. After one second, it makes angle 60°
of the two bodies are with the time axis. What is the acceleration of the particle?
(a) 3 ms–1 and 3 ms–1 (b) 4 ms–1 and 2 ms–1
U

–1 –1 (a) 3 - 1 (b) 3 + 1 (c) 3 (d) 1


(c) 5 ms and 1 ms (d) 7 ms–1 and 3 ms–1
ED

58. A ball is thrown vertically upward with a velocity ‘u’ from 64. Two particles A and B are connected by a rigid rod AB. The
the balloon descending with velocity v. The ball will pass rod slides along perpendicular rails as shown here. The
by the balloon after time velocity of A to the left is 10 m/s. What is the velocity of B
u+v when angle a = 60°?
(a) u - v (b)
2g 2g B
2(u - v) 2( u + v )
(c) (d)
g g
59. Two bodies begin a free fall from the same height at a time
interval of N s. If vertical separation between the two bodies a A
is 1 after n second from the start of the first body, then n is (a) 9.8 m/s (b) 10 m/s (c) 5.8 m/s (d) 17.3 m/s
equal to
65. A balloon starts rising from the ground with an acceleration
(a) (b)
1 of 1.25 ms–2. After 8 s, a stone is released from the balloon.
nN
gN The stone will (Taking g = 10 m s–2)
(a) begin to move down after being released
1 N 1 N
(c) + (d) - (b) reach the ground in 4 s
gN 2 gN 4
(c) cover a distance of 40 m in reaching the ground
60. A particle starting from rest falls from a certain height.
(d) will have a displacement of 50 m.
Assuming that the acceleration due to gravity remain the
same throughout the motion, its displacements in three 66. A point initially at rest moves along x-axis. Its acceleration
successive half second intervals are S1, S2 and S3 then varies with time as a = (6t + 5)m / s 2 . If it starts from origin,
(a) S1 : S2 : S3 = 1 : 5 : 9 (b) S1 : S2 : S3 = 1 : 3 : 5 the distance covered in 2 s is
(c) S1 : S2 : S3 = 9 : 2 : 3 (d) S1 : S2 : S3 = 1 : 1 : 1 (a) 20 m (b) 18 m (c) 16 m (d) 25 m
Motion in a Straight Line 53

67. The relation between time t and distance x is t = ax 2 + b x 76. A stone falls freely under gravity. It covers distances h 1, h2
where a and b are constants. The retardation is and h 3 in the first 5 seconds, the next 5 seconds and the next
5 seconds respectively. The relation between h 1, h2 and h3
(a) 2av3 (b) 2b v3 (c) 2ab v 3 (d) 2b 2 v3 is
68. A stone is just released from the window of a train moving h h
along a horizontal straight track. The stone will hit the ground (a) h1 = 2 = 3 (b) h2 = 3h 1 and h3 = 3h2
3 5
following a (c) h1 = h2 = h3 (d) h1 = 2h 2 = 3h 3
(a) straight line path (b) circular path 77. A ball is released from the top of a tower of height h meters.
(c) parabolic path (d) hyperbolic path It takes T seconds to reach the ground. What is the position
69. A parachutist after bailing out falls 50 m without friction. T
When parachute opens, it decelerates at 2 m/s2 . He reaches of the ball at second
3
the ground with a speed of 3 m/s. At what height, did he bail
out ? 8h
(a) meters from the ground
(a) 182 m (b) 91 m 9
(c) 111 m (d) 293 m 7h
(b) meters from the ground
70. A car accelerates from rest at a constant rate a for some time 9
after which it decelerates at a constant rate b to come to h
rest. If the total time elapsed is t, the maximum velocity (c) meters from the ground
9
acquired by the car is given by
17 h
(d) meters from the ground

.IN
18
æ a2 + b2 ö æ a2 - b2 ö 78. The motion of particle is described by the equation x = a +
ç ÷t ç ÷
(a) ç ab ÷ (b) ç a b ÷ t bt2, where a = 15 cm and b = 3 cm/sec2. Its instant velocity at
è ø è ø
ALtime 3 sec will be
(a) 36 cm/sec (b) 9 cm/sec
æ a +b ö æ ab ö
çç ÷÷ t çç ÷÷ t (c) 4.5 cm/sec (d) 18 cm/sec
(c) (d)
è ab ø è a +b ø
N
79. Which one of the following equation represents the motion
71. A car moving with a speed of 40 km/hour can be stopped by of a body moving with constant finite acceleration? In these
R

applying brakes after at least 2m. If the same car is moving equation, y denotes the displacement in time t and p, q and
with a speed of 80km/hour, what is the minimum stopping r are constant:
U

distance. (a) y = (p + qt )(t + pt)


(a) 8 m (b) 6 m (b) y = p + t/r
JO

(c) 4 m (d) 2 m (c) y = (p + t) (q + t ) (r + t)


72. A man throws balls with same speed vertically upwards one (p + qt)
(d) y =
after the other at an interval of 2 sec. What should be the rt
U

speed of throw so that more than two balls are in air at any 80. A ball is thrown up with velocity 19.6 m/s. The maximum
height attained by the ball is
ED

time
(a) only with speed 19.6 m/s (a) 29.2 m (b) 9.8 m
(b) more than 19.6 m/s (c) 19.6 m (d) 15.8 m
(c) at least 9.8 m/s 81. A train A which is 120 m long is running with velocity 20 m/
(d) any speed less then 19.6 m/s. s while train B which is 130 m long is running in opposite
73. A ball is dropped from a high rise platform at t = 0 starting direction with velocity 30 m/s. What is the time taken by
from rest. After 6 seconds another ball is thrown downwards train B to cross the train A ?
from the same platform with a speed v. The two balls meet at (a) 5 sec (b) 25 sec
t = 18s. What is the value of v? (c) 10 sec (d) 100 sec
(take g = 10 m/s2) 82. A car travelling at a speed of 30 km h–1 is brought to a halt in
(a) 75 m/s (b) 55 m/s 8 m by applying brakes. If the same car is travelling at 60 km
(c) 40 m/s (d) 60 m/s h–1, it can be brought to a halt with the same braking power
74. A particle moves a distance x in time t according to equation in
x = (t + 5)–1. The acceleration of particle is proportional to (a) 32 m (b) 24 m
(c) 16 m (d) 8 cm
(a) (velocity) 3/2 (b) (distance)2 83. Velocity time curve for a body projected vertically upwards
(c) (distance)–2 (d) (velocity)2/3 is
r r (a) parabola (b) ellipse
75. A particle has initial velocity (2i + 3 j ) and acceleration
r r (c) hyperbola (d) straight line
(0.3i + 0.2 j ) . The magnitude of velocity after 10 seconds 84. A train is moving towards east and a car is along north, both
will be with same speed. The observed direction of car to the
(a) 9 2 units (b) 5 2 units passenger in the train is
(a) east-north direction (b) west-north direction
(c) 5 units (d) 9 units
(c) south-east direction (d) None of the above
EBD_7179
54 PHYSICS

85. A metro train starts from rest and in 5 s achieves 108 km/h. 94. The displacement time graph of a moving particle is shown
After that it moves with constant velocity and comes to rest below
after travelling 45 m with uniform retardation. If total distance S
travelled is 395 m, find total time of travelling.

DISPLACEMENT
(a) 12.2 s (b) 15.3 s
D
(c) 9 s (d) 17.2 s
86. A particle moves along a straight line OX. At a time t (in C E F
second) the distance x (in metre) of the particle from O is
given by x = 40 + 12t – t3. How long would the particle travel Time
The instantaneous velocity of the particle is negative at the
before coming to rest?
point
(a) 24 m (b) 40 m
(a) D (b) F
(c) 56 m (d) 16 m
(c) C (d) E
87. The displacement of particle is given by
95. A particle moves along a straight line such that its
a t a t2 displacement at any time t is given by
x = a0 + 1 - 2 . What is its acceleration?
2 3 s = (t3 – 6t2 + 3t + 4) metres
2a2 2a The velocity when the acceleration is zero is
(a) (b) - 2 (a) 3 ms–1 (b) – 12 ms –1
3 3
(c) a 2 (d) zero (c) 42 ms–2 (d) – 9 ms–1
96. A body starts from rest, what is the ratio of the distance
88. Figure here gives the speed-time graph for a body. The

.IN
travelled by the body during the 4th and 3rd seconds ?
displacement travelled between t = 1.0 second and t = 7.0
second is nearest to 7 5
(a) (b)
5 7
4
AL 7 3
8 (c) (d)
v (in ms -1 )

0 6 3 7
2 4 t
97. Which of the following curve does not represent motion in
N
( in sec .)

-4 one dimension?
R

v v
(a) 1.5 m (b) 2 m
U

(c) 3 m (d) 4 m
89. A boat takes 2 hours to travel 8 km and back in still water (a) (b)
JO

lake. With water velocity of 4 km h –1, the time taken for t t


going upstream of 8 km and coming back is v
v
(a) 160 minutes (b) 80 minutes
U

(c) 100 minutes (d) 120 minutes


90. A lift in which a man is standing, is moving upwards with a (c) (d)
ED

speed of 10 ms–1. The man drops a coin from a height of 4.9 t t


metre and if g = 9.8 ms–2, then the coin reaches the floor of
the lift after a time DIRECTIONS for Qs. (98 to 100) : Each question contains
STATEMENT-1 and STATEMENT-2. Choose the correct answer
(a) 2 s (b) 1 s (ONLY ONE option is correct ) from the following.
1 1 (a) Statement -1 is false, Statement-2 is true
(c) (d)
2 2 (b) Statement -1 is true, Statement-2 is true; Statement -2 is a
correct explanation for Statement-1
91. If a ball is thrown vertically upwards with a velocity of (c) Statement -1 is true, Statement-2 is true; Statement -2 is not
40m/s, then velocity of the ball after two seconds is :
a correct explanation for Statement-1
(g = 10 m/sec2) (d) Statement -1 is true, Statement-2 is false
(a) 15 m/s (b) 20 m/s 98. Statement 1 : Velocity-time graph for an object in
(c) 25 m/s (d) 28 m/s
uniform motion along a straight path is a straight
92. If a car at rest accelerates uniformly to a speed of 144 km/h line parallel to the time axis.
in 20 sec., it covers a distance of Statement 2 : In uniform motion of an object velocity
(a) 20 cm (b) 400 m
increases as the square of time elapsed.
(c) 1440 cm (d) 2980 cm 99. Statement 1 : A positive acceleration can be associated with
93. The water drops fall at regular intervals from a tap 5 m above a ‘slowing down’ of the body.
the ground. The third drop is leaving the tap at an instant
Statement 2 : The origin and the positive direction of an
when the first drop touches the ground. How far above the axis are a matter of choice.
ground is the second drop at that instant ? 100. Statement 1 :In a free fall, weight of a body becomes
(Take g = 10 m/s2)
effectively zero.
(a) 1.25 m (b) 2.50 m Statement 2 : Acceleration due to gravity acting on a body
(c) 3.75 m (d) 5.00 m having free fall is zero.
Motion in a Straight Line 55

Exemplar Questions t1t2


t1 + t2
(a) (b)
1. Among the four graph shown in the figure there is only one 2 t2 - t1
graph for which average velocity over the time interval (O,
T) can vanish for a suitably chosen T. Which one is it? t1t2
(c) t2 + t1
(d) t1 – t2
x x
NEET/AIPMT (2013-2017) Questions
(a) t (b) 7. A stone falls freely under gravity. It covers distances h 1, h2
and h3 in the first 5 seconds, the next 5 seconds and the
t next 5 seconds respectively. The relation between h 1, h2
and h3 is [2013]
x x
h2 h
(a) h1 = = 3 (b) h2 = 3h 1 and h3 = 3h2
3 5

.IN
(c) (d)
(c) h1 = h2 = h3 (d) h1 = 2h 2 = 3h 3
t t 8. The displacement ‘x’ (in meter) of a particle of mass ‘m’ (in
AL
2. A lift is coming from 8th floor and is just about to reach 4th kg) moving in one dimension under the action of a force, is
floor. Taking ground floor as origin and positive direction related to time ‘t’ (in sec) by t = x + 3 . The displacement
upwards for all quantities, which one of the following is of the particle when its velocity is zero, will be
N
correct? [NEET Kar. 2013]
R

(a) x < 0, v < 0, a > 0 (b) x > 0, v < 0, a < 0 (a) 2 m (b) 4 m
(c) x > 0, v < 0, a > 0 (d) x > 0, v > 0, a < 0
U

(c) zero (d) 6 m


3. In one dimensional motion, instantaneous speed v satisfies 9. A particle of unit mass undergoes one-dimensional motion
JO

0 £ v < v0 . such that its velocity varies according to v(x) = bx–2n


(a) The displacement in time T must always take non- where b and n are constants and x is the position of the
particle. The acceleration of the particle as d function of x, is
U

negative values
given by: [2015]
(b) The displacement x in time T satisfies – v0T < x < v0T
ED

(a) –2nb x 2 –4n–1 (b) –2b x2 –2n+1

(c) The acceleration is always a non-negative number (c) –2nb2 e–4n+1 (d) –2nb2x–2n–1
(d) The motion has no turning points 10. If the velocity of a particle is v = At + Bt 2, where A and B are
4. A vehicle travels half the distance l with speed v1 and the constants, then the distance travelled by it between 1s and
other half with speed v2, then its average speed is 2s is : [2016]
3
v1 + v2 2v1 + v2 (a) A + 4B (b) 3A + 7B
(a) (b) 2
2 v1 + v2
3 7 A B
(c) A+ B (d) +
2v1v2 L(v1 + v2 ) 2 3 2 3
(c) (d)
v1 + v2 v1v2 11. Preeti reached the metro station and found that the escalator
was not working. She walked up the stationary escalator in
5. The displacement of a particle is given by x = (t – 2)2 where
time t1. On other days, if she remains stationary on the
x is in metre and t in second. The distance covered by the moving escalator, then the escalator takes her up in time t2.
particle in first 4 seconds is The time taken by her to walk up on the moving escalator
(a) 4 m (b) 8 m will be: [2017]
(c) 12 m (d) 16 m t1t 2 t1t 2
6. At a metro station, a girl walks up a stationary escalator in (a) (b)
t 2 - t1 t 2 + t1
time t1. If she remains stationary on the escalator, then the
escalator take her up in time t2. The time taken by her to t1 + t 2
(c) t1 – t2 (d)
walk up on the moving escalator will be 2
EBD_7179
56 PHYSICS

Hints & Solutions


EXERCISE - 1 g
Also, h = (2n - 1) ......(ii)
1. (b) 2. (b) 3. (c) 4. (c) 2
5. (d) 6. (b) 7. (c) From (i) and (ii)
y= h + g
8. (b) x = ( t + 7) or x = ( t + 7) 2
15. (a) Differentiate two times and put x = 0.
dx 16. (d) The difference in velocities is increasing with time as
= 2 ( t + 7 ), \ velocity µ time both of them have more constant but different
dt
acceleration.
dv 17. (b) Time taken by the stone to reach the water level
9. (b) a = f t, a = = f t at t = 0, velocity = u
dt 2h
t1 =
v t g
t2 t2
ò dv = ò f t dt , v – u = f 2
Þ v = u+ f
2 Time taken by sound to come to the mouth of the well,
u 0

.IN
h
NOTE : Do not use v = u + at directly because the t2 =
v
acceleration is not constant. AL
dv 2h h
dx \ a= = 2a2 \ Total time t1 + t 2 = +
10. (c) v= = a1 + 2 a 2 t g v
dt dt
18. (d) Let the total distance be d. Then for first half distance,
N
11. (d) v A = tan 30º and v B = tan 60º
d
R

v tan 30º 1/ 3 1 time = , next distance. = v1t and last half distance
\ A = = = 2v 0
U

vB tan 60º 3 3 = v2t


12. (a) Let for the first half time t, the person travels a distance
JO

d t= d
\ v1t + v 2 t =;
s1. Hence v1 =
s1 2 2(v1 + v2 )
or s1 = v1 t
t Now average speed
U

s2
For second half time, v 2 = or s2 = v 2 t d
ED

t t=
d d d
Total displacement s1 + s 2 + +
Now, v = = 2 v 0 2( v1 + v 2 ) 2( v1 + v 2 )
Total time 2t
2v0 (v1 + v2 )
v t + v 2 t v1 + v2 =
= 1 = (v1 + v 2 ) + 2v0
2t 2
dv
13. (b) When a particle cover half of circle of radius r, then 19. (c) a = bt or = bt . Integrating, we get
displacement is AB = 2r dt
& distance = half of circumference of circle = pr bt 2
v= + c , where c is a constant of integration.
2
B At t = 0, v = v0. Thus v0 = c.
r
pr ds bt 2 æ bt 2 ö
Now, v = = + v o \ ds = çç + v o ÷dt
÷
r dt 2 è 2 ø
3
A bt
Integrating we get, s = + vot
6
1 1 dv
14. (d) y= g (n + 1) 2 - gn 2 20. (a)
dv
= - kv 3 or 3 = - k dt
2 2 dt v
g g 1
= [(n + 1) 2 - n 2 ] = (2n + 1) ......(i) Integrating we get, - = - kt + c ...(1)
2 2 2v 2
Motion in a Straight Line 57

1 EXERCISE -2
At t = 0, v = v0 \ - =c
2v2o
1. (c) Given that x = A t2 – B t3
Putting in (1)
dx
1 1 1 1 \ velocity = = 2 A t - 3B t 2
- = - kt - or - = - kt dt
2v2 2v02 2v20 2v2
d æ dx ö
and acceleration = ç ÷ = 2 A - 6 Bt
dt è dt ø
é 1 ù
[ ]
1 2
or ê + kt ú = or 1 + 2v 02 kt = v 0 For acceleration to be zero 2A – 6Bt = 0.
êë 2 v 02 úû 2 v 2 v2 2A A
\ t= =
6 B 3B
v 20 v0
or v 2 = or v = d 2x
1 + 2 v 20 kt 1 + 2v 02 kt 2. (a) a= = -s w2 sin wt
2
dt
1 w
a - b´ a a 1 On integrating, dx = s w 2 cos t = s w cos w t
21. (d) x= (1 - e b )= (1 - e -1 ) = (1 - ) dt w
b b b e
Again on integrating, we get

.IN
a (e - 1) a ( 2 .718 - 1) a (1 .718 ) a 2
= = = = 0 .637 ~
- a/b sin w t
b e b 2 .718 b 2 .718 b 3 x = sw = s sin w t
AL w
dx
velocity v = = ae - bt , v0 = a 3. (c) v=
dy
= b + 2 c t - 4 d t3
dt dt
N
dv v0 = b + 2c(0) - 4d (0)3 = b
accleration a = = - abe - bt & a 0 = - ab
dt
(Q for initial velocity, t = 0)
R

a dv
At t = 0, x = (1 - 1) = 0 and = 2 c - 12 d t 2
U

Now a =
b dt
JO

1 a a 1 2 \ a 0 = 2c - 12d (0) 2 = 2c , (at t = 0)


At t = , x = (1 - e -1 ) = (1 - ) = a / b
b b b e 3
U

a x x
At t = ¥, x = +
b 1 2 v1 v2
ED

4. (d) 2 2 = =
a x x æ v 2 + v1 ö v1 + v2
It cannot go beyond this, so point x > is not reached +
b 2 v1 2 v 2 çè 2 v1 v 2 ÷ø
by the particle.
a 5. (b) Let u be the initial velocity
At t = 0, x = 0, at t = ¥, x = , therefore the particle
b \ v1¢ = u + a t1, v2¢ = u + a (t1 + t 2 )
does not come back to its starting point at t = ¥ .
and v3¢ = u + a (t1 + t 2 + t 3 )
22. (c) x = t 2 + 2t + 1
dx u + v1¢ u + (u + a t1 ) 1
Now v1 = = = u + a t1
Hence v = = 2t + 2 . It increases with time. 2 2 2
dt
v ¢ + v2¢ 1
23. (b) When a body falls through a height h, it acquires a v2 = 1 = u + a t1 + a t 2
2 2
velocity 2gh . v2¢ + v3¢ 1
v3 = = u + a t1 + a t 2 + a t 3
24. (c) Initial relative velocity vA – vB, is reduced to 0 in 2 2
distance d¢ (<d) with retardation a. 1
So, v1 - v 2 = - a (t1 + t 2 )
\ 0 2 - ( v A - v B ) 2 = -2 ad ¢ 2
1
(v - v B ) 2 (v - v B ) 2 and v 2 - v3 = - a (t 2 + t 3 )
d¢ = A \ d> A 2
2a 2a
\ (v1 - v 2 ) :(v 2 - v3 ) = (t1 + t 2 ) : (t 2 + t 3 )
25. (d)
EBD_7179
58 PHYSICS

6. (b) Let after a time t, the cyclist overtake the bus. Then 12. (d) So by figure the velocity of parrot
1 w.r. t. train is = 5–(–10) = 15m/sec
96 + ´ 2 ´ t 2 = 20 ´ t or t2 – 20 t + 96 = 0 so time taken to cross the train is
2
length of train 150
20 ± 400 - 4 ´ 96 = = = 10 sec
\t = relative velocity 15
2 ´1
20 ± 4 North
= = 8 sec . and 12 sec .
2
7. (d) v2 = u2 + 2 a s or v2 – u2 = 2 a s train
Maximum retardation, a = v2/2 s 10m/sec
West East
When the initial velocity is n v, then the distance over parrot
which it can be stopped is given by 5m/sec
u 2 (n v) 2
sn = 0 = = n2 s South
2 a 2 (v 2 / 2s)
8. (c) Let the length of train is s, then by third equation of ds
13. (a) s2 = at2 + 2bt + c \ 2 s = 2 at + 2 b
motion, v 2 = u 2 + 2a ´ s ....(1) dt
Where v is final velocity after travelling a distance s
ds at + b
with an acceleration a & u is initial velocity as per or = , again differentiating

.IN
question dt s
æ at + b ö
Let velocity of middle point of train at same point is v', as - (at + b ) ç ÷
d 2 s a . s - ( at + b ) ds è s ø
then = . =
dt 2 s2 dt s2
( v¢) 2 = u 2 + 2a ´ (s / 2) ....(2)
AL 2 2 2
d s as - ( at + b )
\ 2 =
dt s3
v2 + u2
N
By equations (1) and (2), we get v¢ =
2 d 2s
\a = ¥ s -3 .
R

9. (b) 8 = a t1 and 0 = 8 – a (4 – t1) dt 2


8 æ 8ö
U

or t 1 = \ 8 = a ç4 - ÷ 14. (a) From third equation of motion v 2 = u 2 + 2ah


a è aø
JO

8 = 4 a – 8 or a = 4 and t1 = 8/4 = 2 sec In first case initial velocity u1 = 5 m/sec


final velocity v1 = 0, a = – g
1
Now, s1 = 0 ´ 2 + ´ 4 ( 2) 2 or s1 = 8 m 25
U

2 and max. height obtained is H1, then, H1 =


1 2g
s2 = 8 ´ 2 - ´ 4 ´ ( 2) 2 or s 2 = 8 m
ED

2 In second case u2 = 10 m/sec, v2 = 0, a = –g


\ s1 + s 2 = 16 m
100
10. (b) u = 10 m/s, t = 5 sec, v = 20 m/s, a = ? and max. height is H2 then, H 2 = .
2g
20 - 10
a= = 2 ms -2 It implies that H2 = 4H1
5
15. (c) The distance covered in n th second is
From the formula v1 = u1 + a t, we have
1
10 = u1 + 2 × 3 or u1 = 4 m/sec. S n = u + ( 2 n - 1)a
11. (c) Let a be the constant acceleration of the particle. Then 2
where u is initial velocity & a is acceleration
1 1
s = u t + a t 2 or s1 = 0 + ´ a ´ (10 ) 2 = 50 a 19 a
2 2 then 26 = u + ....(1)
2
é 1 2ù
and s 2 = ê0 + a(20) ú - 50a = 150a 21a
ë 2 û 28 = u + ....(2)
2
\ s 2 = 3s1
Alternatively : 23 a
30 = u + ....(3)
Let a be constant acceleration and 2
1 1 25 a
s = ut + at 2 , then s1 = 0 + ´ a ´ 100 = 50a 32 = u + ....(4)
2 2 2
Velocity after 10 sec. is v = 0 + 10a From eqs. (1) and (2) we get u = 7m/sec, a=2m/sec2
1 \ The body starts with initial velocity u =7m/sec
So, s 2 = 10a ´ 10 + a ´ 100 = 150a Þ s 2 = 3s1
2 and moves with uniform acceleration a = 2m/sec2
Motion in a Straight Line 59

16. (b) The stone rises up till its vertical velocity is zero and 400
again reached the top of the tower with a speed u So s1 = = 20 m
2 ´ 10
(downward). The speed of the stone at the base is 3u.
For second case (at moon) u2 =20m/sec, v2=0,
u
g
– a2 = = 1 . 7 m / sec 2 , s 2 = ?
6
400 400 s1 1
+ s2 = = so =
2 ´ 1 .7 2 ´ 10 / 6 s2 6
v, g, h 23. (d) All options are correct :
(i) When two bodies A & B move in opposite
4u 2 directions then relative velocity between A & B
Hence (3u)2 = (-u)2 + 2gh or h = either VAB or VBA both are greater than VA & VB.
g
(ii) When two bodies A & B move in parallel direction
1 2
17. (c) So by second equation of motion, we get S = ut + at then VAB = VA - VB Þ VAB < VA
2
here S = l, u = 0, a = g sinq VBA = VB - VA Þ VBA < VB
2l 2h 1 2h æ 24. (b) From third equation of motion

t= = = Q sin q = ÷ v2 = u2 – 2gh (Q a = -g)
a g sin 2 q sin q g çè lø

.IN
Given, v = 10 m/sec at h/2. But v = 0, when particle
attained maximum height h.
q Therefore (10)2 = u2 – 2gh/2
sin
AL or 100 = 2gh –2gh/2 (Q 0 = u2 – 2gh)
g
g cosq

h Þ h = 10 m
g 25. (b) Since in both case the height of building and down
N
q ward acceleration ‘g’ is same. So both stones reach
R

1 2 1 2
18. (a) S = ut + ½at2 here a = g simultaneously i.e., S = gt Þ 10 = 10 ´ t
2 2
U

For first body u1 =0 Þ S1=½g × 9


For second body u2=0 Þ S2= ½g × 4 or t = 2 sec, for both stone.
JO

So difference between them after 3 sec. = S1 – S2 5 50


26. (d) Speed v1 = 60 ´ m/s = m/s
= ½ g×5 18 3
If g = 10m/sec2 then S1–S2 = 25 m. 5 100
U

19. (b) Relative speed of each train with respect to each other d 1 = 20m, v'1 = 120 ´ = m/s
18 3
be, n = 10 + 15 = 25 m/s
ED

Let dceleration be a
Here distance covered by each train = sum of their
lengths = 50 + 50 = 100 m \ 0 = v12 - 2ad1 ....(1)

100 or v12 = 2ad1


\ Required time = = 4 sec .
25
(2v1 )2 = 2ad 2 ...(2)
20. (d) At highest point of the trajectory velocity becomes
zero and all kinetic energy changes to potential energy (2) divided by (1) gives,
so at highest point, K.E. = 0 d2
21. (b) The distance travel in n th second is 4= Þ d 2 = 4 ´ 20 = 80 m
d1
Sn = u + ½ (2n–1)a ....(1)
so distance travel in tth & (t+1)th second are 27. (d) Equation of motion is u = at
St = u +½ (2t–1)a ....(2)
St+1= u+½ (2t+1)a ....(3) we know that u = ds Þ ds = at or ds = atdt
dt dt
As per question,
St+St+1 = 100 = 2(u + at) ....(4) integrating it we get, ò0s ds = a ò04 tdt
Now from first equation of motion the velocity, of a
particle after time t, if it moves with an accleration a is s = [ t 2 ]40 = 8a
2
v=u+at ....(5) 28. (d) The only force acting on the ball is the force of gravity.
where u is initial velocity The ball will ascend until gravity reduces its velocity
So from eq(4) and (5), we get v = 50cm./sec. to zero and then it will descend. Find the time it takes
22. (b) Since v2 = u2 + 2as for the ball to reach its maximum height and then double
For first case u1=20m/sec, v1= 0, a1= g = 10, s1= ? the time to cover the round trip.
EBD_7179
60 PHYSICS

Using vat maximum height = v0 + at = v0 – gt, we get:


9.9 - 5.94
0 m/s = 50 m/s – (9.8 m/s2) t Fractional loss = = 0.4
Therefore, 9.9
t = (50 m/s)/(9.8 m/s2) ~ (50 m/s)/ (10 m/s2) ~ 5s r r r
37. (a) Use v AB = v A - v B .
This is the time it takes the ball to reach its maximum
height. The total round trip time is 2t ~ 10s. 38. (a) Since displacement is zero.
29. (b) 2 ´ 20 ´ 30
A B C 39. (b) Average velocity = = 24 km h -1 .
A to B 20 + 30
1 40. (c) From third equation of motion, v2 = u2 + 2as
2 = u ´ 2 + ´ a ´ 2 ´ 2 Þ 1 = u + a, where v & u are final & initial velocity, a is acceleration,
2
A to C s is distance.
For first case v1= 3km/hour, u1= 0, a1=g & s1=?
1
4.20 = u ´ 6 + a ´ 6 ´ 6 Þ 0.7 = u + 3a , 9 ´ 100
2 s1 = metre
36 ´ 36 ´ 20
2a = -0.3 or a = -0.15 m s -2 , For second case v2=?, u2=4km/hour ,
a2= g = 10m/sec
u = 1 - a = (1 + 0.15)m s -1 = 1.15m s -1
9 ´ 100
Velocity at t = 9 sec. & s1 = s 2 =
36 ´ 36 ´ 20

.IN
v = 1.15 - 0.15 ´ 9 = 1.15 - 1.35 = -0.2 m s -1
16 ´ 1000 ´ 1000 2 ´ 10 ´ 9 ´ 100
30. (c) The ball thrown upward will lose velocity in 1s. It return so v 22 = +
back to thrown point in another 1 s with the same 3600 ´ 3600 20 ´ 36 ´ 36
AL
velocity as second. Thus the difference will be 2 s. or v2 = 5 km/hour
31. (b) Velocity when the engine is switched off ds 1 1
41. (a) = kt Þ s = kt 2 = ´ 2 ´ 3 ´ 3 = 9 m.
N
-1
v = 19.6 ´ 5 = 98 m s dt 2 2
2
R

42. (d) For constant acceleration and zero initial velocity


h max = h1 + h 2 where h1 = 1 at 2 & h 2 = v
2 2a h µ t2
U

1 98 ´ 98
h max = ´ 19.6 ´ 5 ´ 5 +
2 ´ 9.8 h1 t12 h2
JO

2
= Þ t2 = t1 = 2 ´ t1 = 2 ´ 2s
= 245 + 490 = 735 m h 2 t 22 h1
32. (b) Average velocity for the second half of the distance is
U

v1 + v 2 4 + 8 43. (a) v = 3x + 16 Þ v 2 = 3x + 16
= = = 6 m s -1
2 2 Þ v 2 - 16 = 3x
ED

Given that first half distance is covered with a velocity


Comparing with v 2 - u 2 = 2aS, we get, u = 4 units, 2a
of 6 m s -1 . Therefore, the average velocity for the
= 3 or a = 1.5 units
whole time of motion is 6 m s -1 1 1
44. (b) S = AB = g t 12 Þ 2S = AC = g (t1 + t 2 )2
33. (d) Since S = ut + ½ gt2 2 2
where u is initial velocity & a is acceleration. 1 2
In this case u = 0 & a = g and 3S = AD = g ( t 1 + t 2 + t 3 ) A
2
so distance travelled in 4 sec is,
S = ½ × 10 × 16 = 80m 2S S
t1 =
34. (c) The food packet has an initial velocity of 2 m/sec in g B
upward direction, therefore
4S 4S 2S 2S S
v = – u + gt or v = –2 + 10 × 2 = 18 m /sec. t1 + t 2 = , t2 = - 3S
35. (b) Differentiated twice. g g g C
36. (c) Downward motion
6S S
v2 - 02 = 2 ´ 9.8 ´ 5 t1 + t 2 + t 3 =
g
D
Þ v = 98 = 9.9
Also for upward motion 6S 4S
t3 = -
g g
02 - u 2 = 2 ´ ( -9.8) ´1.8

Þ u = 3528 = 5.94 t1 : t 2 : t 3 : : 1 : ( 2 - 1 ) : ( 3 - 2 )
Motion in a Straight Line 61

(8 ´ 1 - 3 ´ 1 ´ 1) - 0 x + 2 x + 3x
45. (c) v= = 5ms -1 56. (b) v av =
1 t1 + t 2 + t 3
46. (c) Since the initial velocity of jump is same on both planets
So 0 = u2– 2gAhA 2x 2x 6x
t1 = , t2 = , t3 =
0 = u2–2gBhB vmax vmax vmax

or g A ´ h A = h B Þ h B = 9 ´ 2 = 18m 6x v max
gB 1 v av =
10x
47. (d) S
vav 3
R =
q o sq vmax 5
in c
gs mg
m 57. (c) Let v A and vB are the velocities of two bodies.
mg
q In first case, v A + vB = 6m/s ........(1)
P Q In second case, v A – vB = 4m/s .........(2)
Let distance (PR) is covered by the particle in time ‘t’.
From (1) & (2) we get, v A = 5 m/s and vB =1 m/s.
1 1
Þ PR = 0 + g sin q . t 2 = gt 2 sin q r

.IN
2 2 58. (d) v BB = Relative velocity of ball w.r.t. balloon = u + v
PQ
Further PR = (Given PQ = constant) u+v
0 = -(u + v) + gt of t =
cos q
AL g
1 2 1
Þ PQ = gt sin q cos q = gt 2 sin 2q 2( u + v)
Total time =
2 4
N
g
PQ 1 1 2 1
y1 = gn , y 2 = g ( n - N ) 2
R

Þ t=2 59. (c)


\ tµ 2 2
g sin 2q sin 2q
U

1
So as q increases, sin 2q first increases and then \ y1 - y 2 = g[n 2 - (n - N) 2 ]
2
JO

decreases. Hence ‘t’ first decreases and then increases.


48. (d) Given x = ae–at + bebt Þ 1=
g
(2n - N)N [Q y1 - y 2 = 1]
dx aa 2
bt
U

Velocity, v = = –aae–at + bbebt = - at + bbe 1 N


dt e Þ n= +
ED

i.e., go on increasing with time. gN 2


49. (b) y µ t 2 ; v- µ t'; a µ t° 1 æ 1 ö2 1
60. (b) S1 = g ç ÷ , S1 + S2 = g(1)2
50. (c) On differentiating, acceleration = 0.2t Þ a = f ( t ) 2 è2ø 2
2
51. (d) Use v2 – u2 = 2aS. In both the cases, (u positive or 1 æ3ö
negative) u2 is positive. g ç ÷ = S1 + S2 + S3
2 è2ø
52. (b) Area under a-t graph is change in velocity. By solving we get
1 1 1 S1 : S2 : S3 = 1 : 3 : 5
Area = (4 ´ 4) + 6 ´ 4 + ´ 2 ´ 4 - ´ 2 ´ 2 61. (b) Distance in last two second
2 2 2
1
= × 10 × 2 = 10 m.
= 36 - 2 = 34 ms-1 2
1
As initial velocity is zero therefore, the velocity at 14 Total distance = × 10 × (6 + 2) = 40 m.
2
second is 34 m s -1 . 62. (d) In (a), at the same time particle has two positions which
53. (c) At t = 20s, d = 20 m is not possible. In (b), particle has two velocities at the
same time. In (c), speed is negative which is not
54. (a) At six points in the graph the tangents have zero slope
possible.
i.e. velocity is zero. 63. (a) Velocity at time t is tan 45° = 1. Velocity at time (t = 1) is
1 tan 60° = 3 . Acceleration is change in velocity in
55. (d) Time fall is second.
2 one second = 3 - 1 .
2 64. (c) Here, the particle B moves upwards. Let the upward
1 æ 1ö 10 v
h = g ç ÷ = = 1.25 m velocity of B be v then = tan 60°
2 è 2ø 8 10
EBD_7179
62 PHYSICS

65. (b) v = 1.25 × 8 ms–1 = 10 ms–1


v v
1 Now t1 + t2 = t or + =t
s = ´ 1.25 ´ 8 ´ 8m = 40m a b
2
1 2
Now , 40 = -10t + ´ 10 ´ t t æ ab ö
2 \v= =ç ÷ t and
or 5t 2 - 10t - 40 = 0 æ 1 1 ö è a+bø
ç + ÷
èa bø
or t 2 - 2t - 8 = 0 or t = 4 s.
66. (b) Given acceleration a = 6t + 5 v2 v2 v2 æ 1 1 ö
s = s1 + s2 = + = ç + ÷
dv 2 a 2b 2 è a b ø
\a= = 6t + 5 , dv = (6t + 5)dt
dt v t

Integrating it, we have ò


dv = ò
(6t + 5)dt 71. (a) From third equation of motion : v 2 = u 2 + 2as
0 0
v = 3t 2 + 5t + C, where C is constant of integration. 40 ´ 10
for first case u = m / sec ,
When t = 0 , v = 0 so C = 0 36
v =0, a = ?, s = 2 m
\ v = ds = 3t 2 + 5t or ds = (3t 2 + 5t)dt
dt 2
æ 40 ´ 10 ö 1

.IN
Integrating it within the conditions of motion, i.e., as t so, a = ç ÷ m / sec 2
changes from 0 to 2 s, s changes from 0 to s, we have è 36 ø 4
s 2
80 ´ 10
AL
ò ò
ds = (3t 2 + 5t)dt for second case u =
36
m / sec , v = 0,
0 0
2 2 2
N
3 5 2 æ 80 ´10 ö 1 æ 40 ´ 10 ö
\s=t + t = 8 + 10 = 18m So s 2 = ç ÷ / 2´ ´ç ÷ = 8meter
2 0 è 36 ø 4 è 36 ø
R

67. (a) t = ax 2 + b x 72. (b) Height attained by balls in 2 sec is


U

Differentiating w.r.t. time on both sides, we get


1
= ´ 9. 8 ´ 4 = 19 . 6 m
JO

dx dx
1 = 2a .x + b 2
dt dt the same distance will be covered in 2 second (for
dx 1 dv -2av descent)
\v= = = = -2av3
U

;
dt b + 2ax dt (b + 2ax) 2 Time interval of throwing balls, remaining same. So, for
Negative sign shows retardation. two balls remaining in air, the time of ascent or descent
ED

68. (c) The horizontal velocity of the stone will be the same as must be greater than 2 seconds. Hence speed of balls
that of the train. In this way, the horizontal motion will must be greater than 19.6 m/sec.
be uniform motion. The vertical motion will be controlled 73. (a) Clearly distance moved by 1st ball in 18s = distance
by the force of gravity, i. e., vertical motion is accelerated
moved by 2nd ball in 12s.
motion. Thus the resultant motion will be along a
parabolic trajectory. Now, distance moved in 18 s by 1st ball
69. (d) Initial velocity of parachute 1
after bailing out, 50 m = × 10 × 182 = 90 × 18 = 1620 m
2
u= 2gh v Distance moved in 12 s by 2nd ball
a = - 2 m / s2 1 2
u = 2 ´ 9.8 ´ 50 = 14 5 = ut + gt \ 1620 = 12 v + 5 × 144
The velocity at ground, 2
v = 3m/s 3m / s Þ v = 135 – 60 = 75 ms –1

v -u2 2
3 - 9802 1 dx = -1
S= = » 243 m 74. (a) x= \ v=
2´ 2 4 t +5 dt (t + 5) 2
Initially he has fallen 50 m.
d2x 2
\ Total height from where he bailed out \ a= = = 2x3
= 243 + 50 = 293 m dt 2 (t + 5)3
70. (d) As per question, 1 3
1 1
Let max. velocity is v Now µ v2 \ µ v2 µ a
then v = a t1 & v – b t2 = 0, where t = t1 + t2 (t + 5) (t + 5)3
Motion in a Straight Line 63
r r r Total distance to be covered by train B
75. (b) v = u + at
= LA + LB= (120 + 130) m
v = (2iˆ + 3 ˆj) + (0.3iˆ + 0.2 ˆj) ´ 10 = 5iˆ + 5 ˆj = 250 m
r r Hence, time required by train B to cross train A
| v | = 52 + 52 ; | v | = 5 2
250
1 2 t = sec = 5sec
76. (a) Q h = gt 50
2
v 2 - u 2 0 - (25 / 3) 2
1 82. (a) Retardation, a = =
\ h1 = g(5)2 = 125 2s 2 ´8
2
2
æ 25 ö 1
1
g(10)2 = 500 a = -ç ÷ ´
h1 + h2 =
2 è 3ø 16
Þ h2 = 375 50 –1
1 For u = 60 km h–1 = ms
3
h1 + h2 + h3 = g(15)2 = 1125
2
Þ h3 = 625 0 - (50 / 3) 2
s= = 32 m
h2 = 3h1 , h3 = 5h 1 2 ´ [-(25 / 3) 2 ´1/16]

.IN
h2 h3 83. (d) Velocity time curve will be a straight line as shown:
or h1 = =
3 5
AL v
1 2
77. (a) h = gT
2
now for t = T/3 second vertical distance moved is given by
N
2 o
1 æ Tö 2 t
h¢ = g ç ÷ Þ h¢ = 1 ´ gT = h
R

2 è 3ø 2 9 9 At the highest point v = 0.


U

84. (b) Let O be the origin, then


h 8h
\ position of ball from ground = h - = N
JO

9 9 N
Q
78. (d) x = a + bt2 = 15 + 3t2
dx Car W E
U

v= = 3 × 2t = 6t
dt
Þ v t = 3s = 6 × 3 = 18cm/s
ED

S
O E
79. (a) Motion with constant acceleration is represented by a Train P
quadratic equation of t passenger in the train at P observes the car at Q along
Y = (p + qt) (r + pt) = pr + qrt + p2t + pqt2 the direction PQ; i.e. west north direction.
80. (c) Let the maximum height attained by the ball be h. 85. (d) Given : u = 0, t = 5 sec, v = 108 km/hr = 30m/s
At maximum height , velocity of ball, v = 0 By eqn of motion
Given, initial velocity, u = 19.6 m/s v = u + at
Using the equation of motion, v 30
or a= = = 6 m / s 2 [Q u = 0]
v2 = u2 + 2gh t 5
We get 0 = (19.6)2 + 2 (– 9.8) × h 1
S 1 = at 2
(19.6)2 2
Þ h = 1
2 ´ 9.8 = ´ 6 ´ 52 = 75 m
= 19.6 m 2
Distance travelled in first 5 sec is 75m.
81. (a) Here, length of train A, LA = 120 m
Distance travelled with uniform speed of 30 m/s is S2
length of train B, LB = 130 m
395 = S1 + S2 + S3
velocity of train A, vA = 20 m/s
395 = 75 + S2 + 45
velocity of train B, vB = 30 m/s
\ S2 = 395 – 120 = 275 m
Train B is running in opposite direction to train B,
\ velocity of train B relative to train A, Time take n to travel 275 m =
275
= 9.2 sec
vBA = vB + vA 30
= (30 + 20) m/s For retarding motion, we have
= 50 m/s 02 – 302 = 2 (– a) × 45
EBD_7179
64 PHYSICS

We get, a = 10 m/s2 93. (c) Height of tap = 5m and (g) = 10 m/sec2.


1 2 1 2
Now by, S = ut + at For the first drop, 5 = ut + gt
2 2
1 1
(–10)t 2 = (0 ´ t ) + ´ 10t 2 = 5t2 or t2 = 1 or t = 1 sec.
45 = 30t + 2
2
45 = 30t – 5t2 It means that the third drop leaves after one second of
Pn solving we get, t = 3 sec the first drop. Or, each drop leaves after every 0.5 sec.
Total time taken = 5 + 9.2 + 3 = 17.2 sec. Distance covered by the second drop in 0.5 sec
86. (c) When particle comes to rest, 1 1
= ut + gt 2 = (0 ´ 0.5) + ´ 10 × (0.5)2
dx d 2 2
V=0= = (40 + 12t – t3) = 1.25 m.
dt dt
Þ 12 – 3t2 = 0 Therefore, distance of the second drop above the
ground = 5 – 1.25 = 3.75 m.
12 94. (d) At E, the slope of the curve is negative.
Þ t2 = = 4 \ t = 2 sec
3 ds
Therefore distance travelled by particle before coming 95. (d) Velocity, v = = 3t 2 – 12t + 3
dt
to rest,
x = 40 + 12t – t3 = 40 + 12 × 2 – (2)3 = 56m dv
Acceleration, a = = 6t – 12; For a = 0, we have,

.IN
87. (b) We get acceleration by double differentiation of dt
displacement. 0 = 6 t – 12 or t = 2s. Hence, at t = 2 s the velocity will be
dx d æ a t a ö
= ç a0 + 1 - 2 t2 ÷ v = 3 ´ 22 –12 ´ 2 + 3 = -9 ms –1
V=
dt dt è 2 3 ø
AL
a
a 2 D4 0 + 2 (2 ´ 4 - 1) 7
= 1 - a2t 96. (a) = =
N
2 3 D3 a 5
0 + (2 ´ 3 - 1)
2
R

æa 2 ö
d ç 1 - a 2t÷
dv è 2 3 ø -2 97. (b) In one dimensional motion, the body can have at a time
a= = = a2
U

dt dt 3 one velocity but not two values of velocities.


98. (d) In uniform motion the object moves with uniform velocity,
JO

1 1 1
88. (b) (2 + 4) ´ 2 + ´ 1 ´ 4 - ´ 3 ´ 4 = 2 m the magnitude of its velocity at different instane i.e., at
2 2 2 t = 0, t =1, sec, t = 2sec ..... will always be constant. Thus
8+8 velocity-time graph for an object in uniform motion along
U

89. (a) Velocity of boat = = 8 km h -1 a straight path is a straight line parallel to time axis.
2
99. (b) 100. (d)
ED

Velocity of water = 4 km h -1
8 8 8
EXERCISE -3
t= + = h = 160 minute
8-4 8+ 4 3 Exemplar Questions
90. (b) Using relative terms
1. (b) If we draw a line parallel to time axis from the point (A)
u rel. = 0 m / s on graph at t = 0 sec. This line can intersect graph at B.
a = 9.8 m s -2 , S = 4.9 m, t = ? In graph (b) for one value of displacement there are
two different points of time. so, for one time, the average
1 velocity is positive and for other time is equivalent
4.9 = 0 ´ t + ´ 9.8 ´ t 2
2 negative.
As there are opposite velocities in the inteval 0 to T
Þ 4.9 t 2 = 4.9 Þ t = 1 s hence average velocity can vanish in (b). This can be
91. (b) From first equation of motion v = u + a t seen in the figure given below.
here u = 40, a = g = – 10, t = 2
x
so v = 40 – 10 × 2 = 20 m/sec
92. (b) v = [144 × 1000/(60 × 60)] m/sec.
v = u + at A B
or (144 × 1000)/(60 × 60) = 0 + a × 20
144 ´1000
\a= = 2 m / sec 2 t
60 ´ 60 ´ 20 O T
1 2 1 2 Here, OA = BT (same displacement) for two different
Now s = u t + at = 0 + ´ 2 ´ ( 20 ) = 400 m points of time.
2 2
Motion in a Straight Line 65

2. (a) As the lift is moving downward directions so 5. (b) As given that, x = (t – 2)2
displacement is negative (zero). We have to see whether
dx d
the motion is accelerating or retarding. Now, velocity v = = (t - 2) 2
Due to downward motion displacement is negative the dt dt
lift reaches 4th floor is about to stop hence, motion is = 2 (t – 2) m/s
retarding (–a) downward in nature hence, x < 0; a > 0.
dv d
Acceleration, a = = [2(t - 2)]
dt dt
8th floor
= 2 [1 – 0] = 2 m/s2 = 2 ms–2
x<0 at t = 0; v0 = 2 (0 – 2) = –4 m/s
6th floor
t = 2 s; v2 = 2 (2 – 2) = 0 m/s
4th floor t = 4 s; v4 = 2 (4 – 2) = 4 m/s
x<0 O
Ground floor V
As displacement is in negative direction,
4 m/s B
x < 0 velocity will also be negative i.e., v < 0 but net
acceleration is +ve a > 0, that can be shown in the O A D
t
graph. 2 4 (Time)
–4 m/s C

.IN
3. (b) In one dimensional motion, for the maximum and
minimum displacement we must have the magnitude
and direction of maximum velocity.
v-t graph is shown in diagram.
As maximum velocity in positive direction is v0, hence
AL Distance travelled
maximum velocity in opposite direction is also –v0.
= area between time axis of the graph
Maximum displacement in one direction = v0T
N
= area OAC + are ABD
Maximum displacement in opposite directions = –v0T.
1 1
R

Hence, -v0T < x < v0T = OA ´ OC + AD ´ BD


2 2
4. (c) Time taken to travel first half distance,
U

l/2 l 4´ 2 1
= + ´ 2´ 4 = 8 m
JO

t1 = = (Q L1 = l / 2) 2 2
v1 2v1
If displacement occurs
Time taken to travel second half distance,
U

1 1
l = ´ OA ´ OC + ´ AD ´ BD
t2 = (Q L2 = l / 2) 2 2
ED

2v2
So, total time taken to travel full distance 1 1
= ´ 2(-4) + ´ 2 ´ 4 = 0
= t1 + t2 2 2
6. (c) Let us consider, displacement is L, then
l l l é1 1 ù
= + = ê + ú velocity of girl with respect to ground,
2v1 2v2 2 ë v1 v2 û
L
vg =
l é v2 + v1 ù t1
Total time = 2 ê v v ú
ë 1 2 û Velocity of escalator with respect to ground,
So, average speed, L
ve =
Total distance t2
vav. = =
Total time Net velocity of the girl on moving escalator with respect
to ground
l 2v1v2
Þ vav. = =
l é1 1 ù v1 + v2 L L
= v g + ve = +
ê + ú t1 t2
2 ë v1 v2 û

2v1v2 ét + t ù
\ vav. = Þ vge = L ê 1 2 ú
v1 + v2 ë t1t2 û
EBD_7179
66 PHYSICS

Now, if t is total time taken by girl on moving escalator dv


in covering distance L, then So, = – 2 nb x– 2n – 1
dx
distance Acceleration of the particle as function of x,
t=
speed
a=v
dv
dx
{
= bx–2n b (–2 n) x
–2n –1
}
L tt
= = 12 = – 2nb2x–4n–1
æ t + t ö t1 + t2
Lç 1 2 ÷ 10. (c) Given : Velocity
è t1t 2 ø
dx
NEET/AIPMT (2013-2017) Questions V = At + Bt2 Þ = At + Bt2
dt
1 2 By integrating we get distance travelled
7. (a) Q h = gt
2 x 2

ò ò ( At + Bt ) dt
1 2
Þ dx =
\ h1 = g(5)2 = 125
2 0 1
1 Distance travelled by the particle between 1s and 2s
h1 + h2 = g(10)2 = 500
2
A ( 2 2 ) B ( 3 3 ) 3A 7B
Þ h2 = 375 x= 2 -1 + 2 -1 = +

.IN
2 3 2 3
1
h1 + h2 + h3 = g(15)2 = 1125
2 d
11. (b) Velocity of preeti w.r.t. elevator v1=
Þ h3 = 625
AL t1
h2 = 3h1 , h3 = 5h 1 d
h2 h3 Velocity of elevator w.r.t. ground v 2 = then
t2
N
or h1 = =
3 5 velocity of preeti w.r.t. ground
R

x +3 v = v1 + v2
8. (c) Q t =
U

d d d
Þ x = t – 3 Þ x = (t – 3)2 = +
t t1 t 2
JO

dx
v= = 2(t – 3) = 0
dt 1 1 1
= +
Þ t=3 t t1 t 2
U

\ x = (3 – 3)2
t1t 2
ED

Þ x = 0. \ t= (time taken by preeti to walk up on


9. (a) According to question, (t1 + t 2 )
V (x) = bx–2n the moving escalator)
Motion in a
4 Plane
SCALARS AND VECTORS (v) Unit vector : Vector which has unit magnitude. It represents
Scalars : The physical quantities which have only magnitude r
direction only. For example take a vector B . Unit vector in
but no direction, are called scalar quantities.
For example - distance, speed, work, temperature, mass, etc. r B
· Scalars are added, subtracted, multiplied and divided by the direction of B is , which is denoted as B̂ . B̂ , is
|B |

.IN
ordinary laws of algebra. read as “B cap” or "B caret".
Vectors: For any quantity to be a vector, (vi) Orthogonal unit vector : A set of unit vectors, having the
(i) it must have magnitude.
directions of the positive x, y and z axes of three dimensional
(ii) it must have direction.
AL
(iii) it must satisfy parallelogram law of vector addition. rectangular coordinate system are denoted by ˆi, ˆj and kˆ .
For example – displacement, velocity, force, etc. They are called orthogonal unit vectors because angle
N
Electric current has magnitude as well as direction but between any of the two unit vectors is 90º.
R

still it is not treated as a vector quantity because it is added by Z


ordinary law of algebra.
U


Types of Vectors
JO

(i) Like vectors : Vectors having same direction are called like
vectors. The magnitude may or may not be equal. ˆj
o Y
U

A î
B
r r
ED

A and B are like vectors. These are also called parallel X


vectors or collinear vectors.
The coordinate system which has shown in fig. is called
(ii) Equal vectors : Vectors having same magnitude and same
direction are called equal vectors. right handed coordinate system. Such a system derives its
name from the fact that right threaded screw rotated through
A 90º from OX to OY will advance in positive Z direction as
B shown in the figure.
r r r r
Here A and B are equal vectors A = B (vii) Null vector (zero vector) : A vector of zero magnitude is
Thus, equal vector is a special case of like vector. called a zero or null vector. Its direction is not defined. It is
(iii) Unlike vectors : Vectors having exactly opposite directions denoted by 0.
are called unlike vectors. The magnitude may or may not be Properties of Null or Zero Vector :
equal. ur
(a) The sum of a finite vector A and the zero vector is
A
B equal to the finite vector
r r ur ur
A and B are unlike vectors. i.e., A + 0 = A
(iv) Negative vectors : Vectors having exactly opposite direction (b) The multiplication of a zero vector by a finite number n
and equal magnitudes are called negative vectors. is equal to the zero vector
i.e., 0 n = 0
A ur
(c) The multiplication of a finite A by a zero is equal to
B zero vector
r r r r ur
Here A and B are negative vectors, A = -B i.e., A 0 = 0
Thus negative vectors is a special case of unlike vectors.
EBD_7179
68 PHYSICS

(viii) Axial vector : Vector associated with rotation about an axis ur ur


Direction of R : Let the resultant R makes an angle b with the
i.e., produce rotation effect is called axial vector. Examples ur
are angular velocity, angular momentum, torque etc. direction of A . Then from right angle triangle QNO,
(ix) Coplanar vectors : Vectors in the same plane are called QN QN B sin q
coplanar vectors. tan b = = =
ON OP + PN A + B cos q
(x) Position vectors and displacement vectors : The vector ur
drawn from the origin of the co-ordinate axes to the position (i) | R | is maximum, if cosq = 1, q = 0° (parallel vector)
ur ur
of a particle is called position vector of the particle. If A (x1, A B
y1, z1) and B (x2, y2, z2) be the positions of the particle at ¾¾® P ¾¾®
two different times of its motion w.r.t. the origin O, then
Rmax = A 2 + B2 + 2AB = A + B
position vector of A and B are ur
Y (ii) | R | is minimum, if cosq = –1, q = 180° (opposite vector)
uur uuur
B¾ ¾¾
¬¾ A®
Pa

A D
th

Rmin = A 2 + B2 - 2AB = A - B
of

isp
ve lace
pa

rA (iii) If the vectors A and B are orthogonal,


rti

c mto
r ent
cl
e

B i.e., q = 90 o , R = A 2 + B2
rB Parallelogram Law of Vector Addition
X

.IN
O It states that if two vectors are represented in magnitude and
uur uuur
rA = OA = x1 ˆi + y1 ˆj + z1 kˆ direction by the two adjacent sides of a parallelogram then their
uur uuur resultant is represented in magnitude and direction by the
rB = OB = x 2 ˆi + y2 ˆj + z 2 kˆ .
AL
diagonal of the parallelogram.
ur ur
The displacement vector is Let the two vectors A and B , inclined at angle q are represented
uuur uuur
AB = OB - OA .
N
by sides OP and OS of parallelogram OPQS, then resultant vector
ur uuur
= (x 2 - x 2 )iˆ + (y 2 - y1 )ˆj + (z 2 - z1 )kˆ R is represented by diagonal OQ of the parallelogram.
R

Laws of Vector Algebra S Q


r r r r
U

1. A + B = B + A (Commutative law of addition)


r r r r r r
JO

2. A + (B + C) = (A + B) + C (Associative law of addition) R


r r B
3. mA = Am q
r r b
U

4. m(nA) = (mn)A O
P
r r r A
ED

5. (m + n)A = mA + nA B sin q
r r r r R = A2 + B 2 + 2 AB cos q ; tan b =
6. m(A + B) = mA + mB A + B cos q
ur ur ur ur
ADDITION OF VECTORS If q < 90° , (acute angle) R = A + B , R is called main
(major) diagonal of parallelogram
Triangle Law of Vector Addition ur ur ur ur
It states that if two vectors acting on a particle at the same time If q > 90° , (obtuse angle) R = A + B , R is called minor
diagonal.
are represented in magnitude and direction by the two sides of a
triangle taken in one order, their resultant vector is represented Polygon Law of Vector Addition
in magnitude and direction by the third side of the triangle taken If a number of non zero vectors are represented by the (n–1)
in opposite order. sides of an n-sided polygon then the resultant is given by the
closing side or the nth side of the polygon taken in opposite
Q
order. D D C
C
E
R B B
E
q
b R B
O A P N O A
A
ur ur ur ur ur ur
ur So, R = A + B + C + D + E
Magnitude of R is given by R = A 2 + B 2 + 2 AB cos q uuur uuur uuur uuur uuur uuur
ur ur or, OA + AB + BC + CD + DE = OE
where q is the angle between A and B .
Motion in a Plane 69

15. Magnitude of a vector is independent of co-ordinate axes


system.
1. Resultant of two unequal vectors cannot be zero. 16. Component of a vector perpendicular to itself is zero.
2. Resultant of three co-planar vectors may or may not be 17. (a) Resultant of two vectors is maximum when angle
zero. between the vectors is zero i.e., q = 0°
3. Minimum no. of coplanar vectors for zero resultant is 2 (for Rmax = A + B
(b) Resultant of two vectors is minimum when
equal magnitude) and 3 (for unequal magnitude).
q = 180°
4. Resultant of three non coplanar vectors cannot be zero. Rmin = A – B
Minimum number of non coplanar vectors whose sum can
be zero is four. (c) The magnitude of resultant of A and B can vary
between (A + B) and (A – B)
5. Polygon law should be used only for diagram purpose for
calculation of resultant vector (For addition of more than 2 SUBTRACTION OF VECTORS
vectors) we use components of vector. We convert vector subtraction into vector addition.

Keep in Memory B
r r r r q
1. If A = B , then A - B = 0 is a null vector..

.IN
2. Null vector or zero vector is defined as a vector whose A
–B 180° – q
magnitude is zero and direction indeterminate. Null vector
differs from ordinary zero in the sense that ordinary zero is
AL
not associated with direction. A - B = A + (-B )
r
A If the angle between A and B is q then the angle between
 = r is called a unit vector. It is unitless and
N
3.
|A|
A
R

dimensionless vector. Its magnitude is 1. It represents


direction only. a
U

r r r r
180

4. If A = B , then | A |=| B | and  = B̂ , where Aˆ and B ˆ are –B


°–q
JO

unit vectors of A and B respectively.


5. A vector can be divided or multiplied by a scalar. R
6. Vectors of the same kind can only be added or subtracted.
A and - B is (180° – q).
U

It is not possible to add or subtract the vectors of different


kind. This rule is also valid for scalars.
| A - B |= A 2 + B2 - 2AB cos q
ED

7. Vectors of same as well as different kinds can be multiplied.


8. A vector can have any number of components. But it can B sin(180 o - q) B sin q
have only three rectangular components in space and two tan a = =
A + B cos (180o - q) A - Bcos q
rectangular components in a plane. Rectangular
components are mutually perpendicular. RESOLUTION OF A VECTOR
9. The minimum number of unequal non-coplanar whose
vector sum is zero is 4. Rectangular Components of a Vector in Plane
ur Y
10. When A = A x ˆi + A y ˆj + A z kˆ
r r
| A |= A 2x + A 2y + A z2 , where | A | is modulus or
A y ˆj A y ˆj
r A
magnitude of vector A .
11. ˆi + ˆj makes 45° with both X and Y-axes. It makes angle 90°
q
with Z-axis.
A x î X
12. î + ˆj + k̂ makes angle 54.74° with each of the X, Y and
Z-axes. The vector A may be written as
uur
13. A -B ¹ B -A A = Axiˆ + Ay ˆj
14. If | A + B | = | A - B | then angle between A and B is
where A x î is the component of vector A in X-direction and
p
.
2 A y ĵ is the component of vector A in the Y-direction.
EBD_7179
70 PHYSICS

Also Ax = A cos q and Ay = A sin q Solution :


\ A = ( A cos q) î + ( A sin q) ĵ Here OP = 6 cos 60° î + 6 sin 60 ° ĵ = 3 î + 5.2 ĵ `
Þ A cos q and A sin q are the magnitudes of the components of PQ = 4 î and QM = 3 ˆj
A in X and Y-direction respectively.. \ OM = OP + PQ + QM = 7 î + 8.2 ˆj
uur Example 3.
Also | A |= Ax2 + Ay2 The resultant of two forces F1 and F2 is P. If F2 is reversed,
Rectangular components of a vector in 3D : Three rectangular the resultant is Q. Show that P 2 + Q 2 = 2( F12 + F22 ) .
components along X, Y and Z direction are given by
A x ˆi , A y ˆj, A z k.
ˆ Therefore, F1
r
vector A may be written as
ur F2 P F2
q
Q
A = Ax ˆi + A y ˆj + A z kˆ and A = A 2x + A 2y + A 2z
q
If a , b and g are the angles subtended by the rectangular a

components of vector then F1


A Ay Az Solution :
cos a = x , cos b = and cos g = r r
A A A Suppose q be the angle between the forces F1 and F2 ,

.IN
2 2
Also, cos a + cos b + cos 2 g =1 then P 2 = F12 + F22 + 2F1F2 cos q ......(i)
CAUTION : Do not resolve the vector at its head. The vector
is always resolved at its tail. F1
AL
B cos q
q B sin q Q
P
F2 F2
B
N
q
B
R

B sin q q
F1
B cos q
U

Wrong Correct When the force F2 is reversed, then the magnitude of their
JO

Example 1. resultant is
X and Y component of vector A are 4 and 6 m respectively. Q 2 = F12 + F22 + 2 F1 F2 cos(180° - q)
uur uur
The X and Y component of A + B are 10 m and 9 m
U

= F12 + F22 - 2F1F2 cos q ......(ii)


respectively. Calculate the length of vector B and its angle
with respect to X-axis Adding equations (i) and (ii),
ED

Solution : P 2 + Q 2 = 2F12 + 2F22 = 2(F12 + F22 )


A = 4 î + 6 ˆj and A + B = 10 î + 9 ˆj Example 4.
r
Find the components of vector A = 2iˆ + 3jˆ along the
\ B = ( A + B ) – A = (10î + 9 ĵ) – ( 4 î + 6 ˆj) = 6 î + 3 ĵ
directions of ˆi + ˆj and ˆi – ˆj .
\ length of B is | B |= 6 2 + 3 2 = 3 5 m Solution :
r
By
3 1 æ1ö Here A = 2î + 3ˆj
Also tan q = = Þ q = tan -1 ç ÷
= r
Bx
6 2 è2ø In order to find the component of A along the direction of
r r
where q is the angle which (A + B) is making with î + ĵ , let us find out the unit vector along î + ĵ . If â is the
X-axis.
Example 2. unit vector along î + ĵ , then
Find the resultant of vectors given in figure
î + ˆj î + ˆj
Y M â = =
| î + ˆj | 2
3 cm r
Hence, the magnitude of the component vector of A along
P 4 cm
Q î + ĵ
6cm
r î + ĵ 1 5
60° = A.â = (2î + 3ˆj). = (2 + 3) =
X 2 2 2
O
Motion in a Plane 71

Therefore, component vector of A along ˆi + ˆj Solution :


F2
r 5 æ ˆi + ˆjö 5 ˆ ˆ r r
= (A.a)a
ˆ ˆ= ç ÷ = (i + j) F2 = 250ˆj , F1 = 250iˆ
2è 2ø 2 F1
r r
Similarly, if b̂ is the unit vector along the direction of î - ĵ , F2 - F1 = 250ˆj - 250iˆ ,
r r
then magnitude of the component vector of A along î - ĵ | F2 - F1 | = 2502 + 2502 = 250 2 N (N-W direction)

r (î - ĵ) (î - ĵ) (2 - 3) 1 N


= ( A.b̂) = (2î + 3 ĵ). = (2 î + 3 ĵ). = =-
| î - ĵ | 2 2 2

\ Component vector of A along ˆi - ˆj 135°


W E

r æ ˆ ˆö
ˆ ˆ = - 1 i - j = - 1 (iˆ - ˆj)
= (A.b)b ç ÷ 2
2è 2ø S
Example 5.
250
If 0.3î + 0.4 ĵ + ck̂ is a unit vector, then find the value of c. tan q = = -1 Þ q = 135 °

.IN
-250
Solution : Example 9.
r r r
Unit vector is a vector of unit magnitude. r r r
If a,b and c are unit vectors such that a +b + c = 0 , then
\ Þ 0.09 + 0.16 + c 2 = 1
AL
0.32 + 0.4 2 + c 2 = 1 r r
find the angle between a and b .
Þ c 2 = 1 - 0.25 = 0.75 Þ c = 0.75 Solution :
N
r r r r
Example 6. Given : ar + b + cr = 0 Þ c = -(a + b)
R

What is the vector joining the points (3, 1, 14) and r r r


Also, | a | = | b | = | c | = 1
U

(–2, –1, –6) ?


Solution : r r
Let angle between a and b = q
JO

If P and Q be the points represented by the coordinates


(3, 1, 14) and (–2, –1, –6) respectively then, \ 1 = 12 + 12 + 2 ´ 1 ´ 1 ´ cos q
PQ = p.v. of Q – p.v. of P
U

\ cos q = -1/ 2 Þ q = 120° = 2 p / 3


= (-2î - ĵ - 6k̂ ) - (3î + ĵ + 14 k̂ ) = -5î - 2 ĵ - 20k̂ PRODUCT OF TWO VECTORS
ED

uuur uuur Scalar or Dot Product


and QP = - PQ = 5iˆ + 2jˆ + 20kˆ
The scalar or dot product of two vectors A and B is a scalar,
Example 7. r r
Find the angle between two vectors of magnitude 12 and which is equal to the product of the magnitudes of A and B and
18 units, if their resultant is 24 units. cosine of the smaller angle between them.
r r
Solution : i.e., A . B = A B cosq
r B
Magnitude of first vector (A) = 12; Magnitude of second
r r
r r r
e.g. W = F × s;P = F × vr
vector (B) = 18 and resultant of the given vectors (R) = 24
q
X
\ 24 = 2 2
A + B + 2AB cos q B cosq
A
Properties of Scalar or Dot Product:
2 2
24 = (12) + (18) + 2 ´ 12 ´ 18 cos q
1. iˆ . iˆ = ˆj . ˆj = kˆ . kˆ =1
or (24)2 = 144 + 324 + 432 cos q or 432 cos q = 108
108 iˆ . ˆj = ˆj . kˆ = kˆ . iˆ = 0
or cos q = = 0.25 or q = 75°52¢ r r
432 2. A . B = A (B cosq) = B (A cosq)
Example 8.
r r The dot product of two vectors can be interpreted as the
Two forces F1 = 250N due east and F2 = 250 N due north
th product of the magnitude of one vector and the magnitude
have their common initial point. Find the magnitude and of the component of the other vector along the direction of
r r the first vector.
direction of F2 - F1
EBD_7179
72 PHYSICS
r r r r
3. A . B = B . A Dot product of two vectors is commutative. S R
r r k
4. A . A = A2
B
r r r r r r r
5. A.( B + C ) = A . B + A .C Dot product is distributive. q
r r P Q
6. A .B = (A x ˆi + A y ˆj + A z k).(B
ˆ ˆ ˆ ˆ
x i + B y j + B z k) A
= (Ax Bx + Ay By + Az Bz) r r
| A ´ B |= A( B sin q)
Vector or Cross Product = area of parallelogram PQRS
The vector product of two vectors is defined as a vector having
= 2 (area of DPQR)
magnitude equal to the product of two vectors and sine of the
angle between them. Its direction is perpendicular to the plane 8. A unit vector which is perpendicular to A as well as B is
containing the two vectors (direction of the vector is given by uur uur r r
A´B A´ B
right hand screw rule or right hand thumb rule. uur uur =
ur ur ur | A ´ B | AB sin q
C = A ´ B = (AB sin q) n̂
ur ur
The direction of (A ´ B) perpendicular to the plane containing Keep in Memory

.IN
ur ur ur ur
vectors A and B in the sense of advance of a right handed A x B x + A y B y + A z Bz A.B
ur ur 1. cos q = = ur ur
screw rotated from A to B is through the smaller angle between A x2 + A 2y + A z2 B2x + B2y + Bz2 | A || B |
AL
them.
r r
A | A ´ B | A B sin q
tan q = r r =
N
2.
q A.B A B cos q
ur ur ur
( A ´ B) = C
R

B r r r r
| A ´ B |2 + | A.B |2 = A 2 B2
U

3.
r r r r r r r r r r r r r r r
JO

e.g., v = w´ r ; t = r ´ F; L = r ´ p
4. | A + B | ´ | A – B |= 2 | (B) ´ (A ) |
Properties of Vector or Cross Product r r r r r r r r r
U

i 5. If A + B + C = 0 , then A ´ B = B ´ C = C ´ A
1. iˆ ´ iˆ = ˆj ´ ˆj = kˆ ´ kˆ = 0 k
r r r r
ED

6. | A.B |2 - | A ´ B |2 = A 2 B 2 cos 2q
2. iˆ ´ ˆj = kˆ; ˆj ´ kˆ = iˆ; kˆ ´ iˆ = ˆj
r r p
3. A´ A = 0 7. If | A ´ B |= A . B then angle between A and B is .
j 4
r r r r r r r r
4. A ´ B ¹ B ´ A (not commutative) [Q A ´ B = - B ´ A]
8. If A || B then A ´ B = 0
r r r r r r r
5. A ´ ( B + C ) = ( A ´ B) + ( A ´ C ) (follows distributive law)
9. Division by a vector is not defined. Because, it is not
r r
6. A ´ B = ( Ax iˆ + Ay ˆj + Az kˆ ) ´ ( B x iˆ + B y ˆj + B z kˆ ) possible to divide by a direction.

ˆi ˆj 10. The sum and product of vectors is independent of



co-ordinate axes system.
= Ax Ay Az
Bx By Bz CONDITION OF ZERO RESULTANT VECTOR AND LAMI'S
THEOREM
= (Ay Bz – Az By) î + (Az Bx – Ax Bz) ĵ If the three vectors acting on a point object at the same time are
represented in magnitude and direction by the three sides of a
+ (Ax By – Ay Bx) k̂ triangle taken in order, then their resultant is zero and the three
7. The cross product of two vectors represents the area of the vectors are said to be in equilibrium.
parallelogram formed by them. r r r
i.e. F1 + F2 + F3 = 0
Motion in a Plane 73
ur uur
R ˆ s = 2cos 45º iˆ - 2 sin 45º ˆj
s1 = 4j, 2
uur
a s3 = -2 cos 45º ˆi - 2 sin 45º ˆj
F3 F2 r ur uur uur
s = s1 + s 2 + s3 = 4jˆ + 2 ˆi - 2 ˆj - 2 ˆi - 2 ˆj
r
b g \ Displacement s = (4 - 2 2)jˆ = (1.17)jˆ = (1.17)
P Q
F1 (northward)
And total distance covered = 4 + 2 + 1 = 7m
Lami's Theorem
It states that if three forces acting at a point are in equilibrium, Example 12.
r r
then each force is proportional to the sine of the angle between Prove that vectors A= iˆ + 2jˆ +3kˆ and B = 2iˆ - ˆj are
the other two forces.
r r r F2 perpendicular to each other.
F1 F F Solution :
= 2 = 3 g
sin a sin b sin g r r
a Here, A = î + 2 ĵ + 3k̂ and B = 2î - ĵ
r r r b
F1 F2 F3 F1 r r
or, = = Two vectors are perpendicular to each other if, A.B = 0
PQ QR PR
F3 r r
Example 10. Now A.B = (î + 2 ĵ + 3k̂ ) . (2î - ĵ) = 0

.IN
Calculate the area of a parallelogram formed from the
r r = 1 × 2 + 2 × (–1) + 3 × (0) = 2 – 2 + 0 = 0
vectors A = î + 2 ĵ + 3k̂ and B = 2î - 3 j + k̂ , as adjacent r r r r
Sin ce A.B = 0 , therefore vectors A and B are
sides.
AL
perpendicular to each other.
Solution :
r r
The area of a parallelogram is given by | A ´ B |
N
Example 13.
Here, r
Find the angle between the vectors A = iˆ + ˆj - 2kˆ and
R

ˆi ˆj kˆ r
r r B = -iˆ + 2 ˆj - kˆ .
U

A ´ B = 1 +2 3
Solution :
2 -3 1
JO

r r
= î [(2 × 1) – (–3 × 3)] + ĵ [(3 × 2) – (1 × 1)] Here A = î + ĵ - 2k̂ , B = - î + 2 ĵ - k̂
r r
+ k̂ [(1 × –3) – (2 × 2)] r r A.B
U

We know that A.B = AB cos q or cos q =


AB
= 11 î + 5 ĵ – 7 k̂
ED

ur ur Now A= 12 + 12 + ( -2) 2 = 6,
\ | A ´ B |= (11)2 + (5)2 + (-7) 2 @ 14

B = ( -1)2 + 22 + ( -1)2 = 6
Example 11.
r r
A particle suffers three displacements 4m in the northward, A.B = (iˆ + ˆj - 2k).(
ˆ -ˆi + 2ˆj - k)
ˆ = -1 + 2 + 3 = 3
2 m in the south-east and 1 m in the south-west directions.
What is the displacement of the particle and the distance 3 3 1
cos q = = = or q = 60°
covered by it? 6´ 6 6 2
Solution : Example 14.
Taking a frame of reference with the x-axis in the eastward A particle is displaced from a point (3, – 4, 5) to another point
and the y-axis in the northward direction
(–2, 6, – 4) under a force 2î + 3 ĵ - k̂ . Find the work done by
North (N) the force.
Y Solution :
r
F = 2î + 3 ĵ - k̂
West (W) X East (E) The displacement of the particle is
r
s = position vector of point (–2, 6, – 4) – position vector of
point (3, – 4, 5)
South (S) r
s = (-2î + 6 ĵ - 4k̂ ) - (3î - 4 ĵ + 5k̂ ) = -5î + 10 ĵ - 9k̂
EBD_7179
74 PHYSICS

\ work done by the force is Dr ˆ Dx


rr Average velocity v = =i + ˆj Dy = vxiˆ + vy ˆj
W = F.s = (2î + 3 ĵ - k̂ ).(-5î + 10 ĵ - 9k̂ ) Dt Dt Dt
W = (2) (–5) + (3) (10) + (–1) (–9) = 29 units. Y
Dr
Example 15. Instantaneous velocity vinst = lim
Dt ® 0 Dt v ^
r vy j
A force F = 6iˆ + xjˆ acting on a particle displaces it from the q
The magnitude of v = v x2 + v y 2 vx ^i
point A (3, 4) to the point B (1, 1). If the work done is 3 units,
X
then find value of x. O
vy v y
Solution : The direction of the velocity, tan q = \ q = tan -1
r r vx vx
d = -2iˆ - 3jˆ , F = 6iˆ + xjˆ
rr Acceleration
\ W = F.d ; 3 = – 12 – 3x Þ x = –5 The average acceleration in a x–y plane in time interval Dt is the
change in velocity divided by the time interval.
MOTION IN A PLANE OR MOTION IN TWO r
DIMENSIONS a = ax iˆ + a y ˆj
The motion in which the movement of a body is restricted to a
The magnitude of a = ax2 + a y2
plane is called motion in a plane.
Example : A ball is thrown with some initial velocity (u) and Dv Dvx ˆ Dv y ˆ

.IN
making angle q with harizontal. Average acceleration a = = i+ j
Dt Dt Dt
The general approach to solve problem on this topic is to resolve Instantaneous acceleration
the motion into two mutually perpendicular co-ordinates. One
AL
along X-axis and other along Y-axis. These two motions are Dv Dv x ˆ Dv y
ainst = lim = lim i + lim ˆj
independent of each other and can be treated as two separate D t ® 0 Dt D t ® 0 Dt D t ® 0 Dt
N
rectilinear motions. In two or three dimensions, the velocity and acceleration vectors
The velocity v and acceleration a can be resolved into its x and y may have any angle between 0°and 180° between them.
R

components. RELATIVE VELOCITY IN TWO DIMENSIONS


U

Y If two objects A and B moving with velocities VA and VB with


JO

respect to some common frame of reference, then :


vy v (i) Relative velocity of A w.r.t B
b r r r
u vx v AB = v A - vB
U

u sin q P(x,y)
(ii) Relative velocity of B w.r.t. A
r r r
ED

q vBA = vB - v A
X
O u cos q r r r r
Therefore, v AB = v BA and vAB = vBA
v = v x ˆi + v y ˆj
PROJECTILE MOTION
a = axi + ay j
Projectile is the name given to a body thrown with some initial
x-component of motion y- component of motion velocity in any arbitrary direction and then allowed to move
vx = ux + axt vy = uy + ayt under the influence of gravity alone.
1 2 1 Examples : A football kicked by the player, a stone thrown from
x = ux t+ at y = uyt + a t2 the top of building, a bomb released from a plane.
2 x 2 y
The path followed by the projectile is called a trajectory.
vx2 – ux2 = 2 axx vy2 – uy2 = 2 ayy The projectile moves under the action of two velocities:
(1) A uniform velocity in the horizontal direction, which does
æ u + vx ö æ u y + vy ö not change (if there is no air resistance)
x= ç x ÷t y = çç ÷÷ t
è 2 ø è 2 ø (2) A uniformly changing velocity in the vertical direction due
to gravity.
Velocity The horizontal and vertical motions are independent of each other.
The ratio of the displacement and the corresponding time interval Types of Projectile:
is called the average velocity. 1. Oblique projectile : In this, the body is given an initial
r velocity making an angle q with the horizontal and it moves
v = v x ˆi + v y ˆj
under the infuence of gravity along a parabolic path.
Motion in a Plane 75

2. Horizontal projectile : In this, the body is given an initial Case 2: If a particle is projected at an angle (q) in upward direction
velocity directed along the horizontal and then it moves from the top of tower of height h with velocity u, then
under the influence of gravity along a parabolic path. uy = u sin q
Motion along x-axis g
ay = – g
ux = u, ax = 0 o ux = u x ux = u cos q
y
P(x,y) ax = 0
1
uy = 0

x = uxt + axt2 x b vx = ux = u
2 1
h = + u sin q.t - gt 2 and x = u cosq.t
2
x = ut + 0 vy=gt v
x y
\ t= …… (1)
u +
Motion along y-axis u sin q u
uy = 0, ay = g A q

1 1 u cos q
y = uyt + ayt2 Þ 0 + gt2
2 2 h
1
y = gt2 …… (2)
2

.IN
x B
æ g ö 2
From equations (1) and (2) we get y = çè 2 ÷ø x
2u Case 3: If a body is projected at an angle (q) from the top of tower
AL
which is the equation of a parabola. in downward direction then
Velocity at any instant : uy = – u sin q, ux = u cosq, ax = 0
r
v = v ˆi + v ˆj
N
x y 1 2
ay = +g , - h = - u sin q.t - gt and x = u cosq.t
R

v= u 2 + g2 t 2 2
r
U

If b is the angle made by v with the horizontal, then


+ ucos q
JO

vygt q u
tanb = = –
vx u usin q
U

Time of flight and horizontal range: h


If h is the distance of the ground from the point of projection, T
ED

is the time taken to strike the ground and R is the horizontal range
of the projectile then

2h x
2h
T= and R = u
g g Equation of Trajectory
Let the point from which the projectile is thrown into space is
taken as the origin, horizontal direction in the plane of motion is
Case 1: If the projectile is projected from the top of the tower of taken as the X-axis, the vertical direction is taken as the Y-axis,
height 'h', in horizontal direction, then the height of tower h, Let the projectile be thrown with a velocity u making an angle q
range x and time of flight t are related as : with the X-axis.
1 Y
h = gt 2 and x = vt
2
vy v
b
u vx
u
u sin q P(x,y)

q
h X
1b v x O u cos q
v The components of the initial velocity in the X-direction and Y-
vy direction are u cos q and u sin q respectively. Then at any instant
x of time t,
EBD_7179
76 PHYSICS

Motion along x – axis Keep in Memory


ux = ucosq, ax = 0
1. The horizontal range of the projectile is same at two angles
1
x = ux t + a x t2 of projection for q and (90° – q).
2
2. The height attained by the projectile above the ground is
x = (u cosq) t ...(1)
Motion along y–axis the largest when the angle of projection with the horizontal
uy = u sin q, ay = –g is 90° (vertically upward projection). In such a case time of
flight is largest but the range is the smallest (zero).
1 2
y = uyt + at 3. If the velocity of projection is doubled. The maximum height
2 y
attained and the range become 4 times, but the time of flight
1 2 is doubled.
y = u sinq t + gt ...(2)
2 4. When the horizontal range of the projectile is maximum, (q =
From equations (1) and (2) we get 45°), then the maximum height attained is ¼th of the range.
g 5. For a projectile fired from the ground, the maximum height is
y = x tanq - 2 2
x2 attained after covering a horizontal distance equal to half of
2u cos q
which is the equation of a parabola. Hence the path followed by the range.
the projectile is parabolic. The velocity of the projectile is minimum but not zero at the
Velocity at any Point highest point, and is equal to u cosq i.e. at the highest point

.IN
Let vy be the vertical velocity of projectile at time t. (at P) of the trajectory, the projectile has net velocity in the
And vx be the horizontal component of velocity at time t. horizontal direction (vertical component is zero). Horizontal
AL
\ v y = u sin q - gt ¼¼ (1) component of velocity also remains same as the component
of g in horizontal direction is zero i.e., no acceleration in
vx = u cos q ¼¼ (2) horizontal direction.
N
v = vx2 + v y2 Example 16.
R

A boat takes 2 hours to travel 8 km and back in still water


= u 2 cos 2 q + u 2 sin 2 q - 2gt u sin q + g 2 t 2 lake. If the velocity of water is 4 km h–1, the time taken for
U

going upstream of 8 km and coming back is


(a) 2 hours
JO

v = u 2 + g 2 t 2 - 2gt u sin q
(b) 2 hours 40 minutes
and the instantaneous angle (b) with horizontal is given by (c) 1 hour and 20 minutes
v y u sin q - gt (d) can't be estimated with given information
U

gt
tan b = = = tan q - Solution : (b)
vx u cos q u cos q
ED

Total distance travelled by boat in 2 hours = 8 + 8 = 16 km.


Time of Flight :
Therefore speed of boat in still water, vb = 16/2 = 8 km h–1.
The time of flight of the projectile is given by Effective velocity when boat moves upstream = vb – vw
2u sin q = 8 – 4 = 4 km h –1.
T = 2t = ,
g Therefore time taken to travel 8 km distance
where 't' is the time of ascent or descent. = 8/4 = 2h.
Maximum Height : Effective velocity when boat moves along the stream
Maximum height attained by the projectile is given by = vb + vw = 8 + 4 = 12 km h –1.
The time taken to travel 8 km distance = 8/12 = 2/3h = 40 min.
u2 Total time taken = 2h + 40 min = 2hours 40 min.
H= sin 2 q .
2g Example 17.
In case of vertical motion, q = 90º so maximum height attained A boat man can row a boat with a speed of 10 km/h in still
water. If the river flows at 5 km/h, the direction in which
u2 the boat man should row to reach a point on the other
H=
2g bank directly opposite to the point from where he started
Horizontal Range : (width of the river is 2 km).
The horizontal range of the projectile is given by (a) is in a direction inclined at 120º to the direction of river
flow
u 2 sin 2q u2 (b) is in a direction inclined at 90º to the direction of
R= and R max = at q = 45º river flow.
g g
(c) is 60º in the north-west direction
(Q maximum value of sin2q = 1) (d) is should row directly along the river flow
Motion in a Plane 77

Solution : (a) 2 u 2 sin q cos q 2 u 2 sin 2 q


Refer to fig., the boat man should go along OC in order to \ =
cross the river straight (i.e. along OB). g 2g
Solving we get tan q = 2 .....(3)
C B
Hence sin q = (2 / 5 ) and cos q = (1 / 5 )
2u2 2 1 4u2
From eqn. (2) R = ´ ´ =
2km g 5 5 5g
nb
q
Example 21.
O nr A A body is projected downwards at an angle of 30º to the
horizontal with a velocity of 9.8 m/s from the top of a tower
CB n r 5 1 29.4 m high. How long will it take before striking the
sin q = = = = = sin 30º ; q = 30º; ground?
OC n b 10 2 Solution :
\ Boat man should go along in a direction inclined at The situation is shown in fig.
90º + 30º = 120º to the direction of river flow.
Example 18. 30°

A man swims at an angle q = 120º to the direction of water u

.IN
flow with a speed vmw = 5km/hr relative to water. If the +
speed of water vw = 3km/hr, find the speed of the man.
Solution :
r r r
AL
v mw = v m – v w
r r r
v m = v mw + v w
N
r r A B
Þ vm = | v mw + v w| = v 2mw + v 2w + 2 v mw . v w cos q
The time taken by the body is equal to the time taken by the
R

freely falling body from the height 29.4 m with initial velocity
Þ vm = 5 2 + 3 2 + 2 (5) (3) cos120 º u sin q. This is given by
U

Þ vm = 25 + 9 - 15 = 19 m/sec. 9.8
u sinq = = 4.9 m / s
JO

Example 19. 2
A gun throws a shell with a muzzle speed of 98 m/sec. When 1
the elevation is 45º, the range is found to be 900 m. How Applying the formula, s = u t + g t2, we have
2
U

much is the range decreased by air resistance?


Solution : 1
ED

29.4 = 4.9 t + (9.8) t2 or 4.9 t2 + 4.9 t – 29.4 = 0


Without air resistance, the expected range 2
u 2 sin 2 q (98) 2 ´ sin 90 (98) 2 (because s, u and g are all in downward direction)
R= = = = 980 m t2 + t – 6 = 0or t = 2 or –3
g 9.8 9.8 \ Time taken to reach ground = 2 second
Decrease in range = 980 m – 900 m = 80 m
Example 20. Example 22.
A particle is projected with velocity u at an angle q with Two boys stationed at A and B fire bullets simultaneously
the horizontal so that its horizontal range is twice the at a bird stationed at C. The bullets are fired from A and B
greatest height attained. The horizontal range is at angles of 53° and 37° with the vertical. Both the bullets
(a) u2/g (b) 2 u2/3 g fire the bird simultaneously. What is the value of vA if
2 vB = 60 units? (Given : tan 37° = 3/4)
(c) 4 u /5 g (d) None of these
Solution :
Solution : (c)
The vertical components must be equal.
Greatest height attained
u 2 sin 2 q
H= …… (1)
2g
Horizontal range,
u 2 sin 2 q 2 u 2 sin q cos q
R= = …… (2)
g g
[Qsin 2q = 2 sin q cosq]
Given that R = 2 H
EBD_7179
78 PHYSICS

\ vA cos 53° = vB cos 37° Condition for horizontal range R on the inclined plane to be
cos 37° maximum :
or vA = vB cos (90° - 37°) u2
Since R = [2sin(q - a ) cos q]
g cos 2 a
60
or vA = 60 cot 37° = tan 37° [QvB = 60 units] u2
= [sin(2q - a) - sin a]
g cos 2 a
60 ´ 4
= = 80 units {2 sin A cos B = sin(A+B)+sin(A–B)}
3
R is maximum when sin (2q –a) is maximum
PROJECTILE ON AN INCLINED PLANE
Let a body is thrown from a plane OA inclined at an angle a with p a
i.e., sin (2q – a) = 1 or q = +
the horizontal, with a constant velocity u in a direction making an 4 2
angle q with the horizontal.
The body returns back on the same plane OA. Hence the net u2
displacement of the particle in a direction normal to the plane OA Þ R max = [1 - sin a]
is zero. g cos 2 a

R max (on horizontal plane)


or Rmax(on inclined plane) =
x 1 + sin a

.IN
u2
A where Rmax (on horizontal plane) = .
2g
a)
AL
y sin(
g Condition for time of flight T to be maximum :
u ) g cos(a)
-a a
(q
N
2 u sin(q - a )
q T= so T is max when sin (q – a) is maximum
uy a g g cos a
R

O B p 2u
U

i.e., sin (q –a) = 1 or q = +a Þ T =


ux = u cos (q – a ) along the incline, + x-axis) 2 g cos a
JO

uy = u sin (q – a ) along the incline, + y-axis) It means that if q1 is the angle for projectile for which T is maximum
ax = g sin a along – x-axis, as retardation and q2 is the angle for which R is maximum, then q1 = 2q2.
ay = g cos a along – y-axis, as retardation Example 23.
U

The time of flight of the projectile is given by The slopes of wind screen of two cars are b1 = 30° and
1 2 b2 = 15° respectively. At what ratio v1/v2 of the velocities
ED

s = ut + at
2 of the cars will their drivers see the hailstorms bounced by
windscreen of their cars in the vertical direction? Assume
1
or 0 = u sin(q - a )T - g cos aT 2 hailstorms falling vertically.
2 Solution :
v
2 u sin(q - a ) From the fig tan a = and a = 90° - 2b1
T= v1
g cos(a )
where v is velocity of hail
If maximum height above the inclined plane is H, v
tan(90° - 2b1 ) = cot 2b1 =
v1
u 2 sin 2 (q - a )
H=
2g
v b1 b
1
The horizontal range R of the projectile is given by a
–v1
2 u 2 sin(q - a) cos q
OB = u cos q t = =R
g cos a b1
The range of the projectile at the inclined plane is given by v
Similarly, cot 2b 2 =
v2
OB 2u 2 sin(q - a)cos q
OA = = =R v1 cot 2b 2 cot 30
cos a g cos 2 a = = = 3.
v 2 cot 2b1 cot 60
Motion in a Plane 79

Example 24. Angular displacement : Change in angular position is called


A particle is projected up an inclined plane of inclination angular displacement (dq).
a to the horizontal. If the particle strikes the plane
horizontally then tan a = ... . Given angle of projection B DS
with the horizontal is b. A
dq
(a) 1/2 tanb (b) tanb q2
(c) 2 tanb (d) 3 tanb q1
Solution : (a) O
If the projectile hits the plane horizontally then

A
Angular velocity : Rate of change of angular displacement is called
angular velocity w

b dq
i.e., w =
a B dt
C
Relation between linear velocity (v) and angular velocity (w).
r r r

.IN
1
Tplane = Thorizontal plane
v = w´r
2 In magnitude, v = r w
2u sin(b - a ) u sin b
=
AL
Angular acceleration : Rate of change of angular velocity is called
or g cos a g angular accelaeration.

2 sinb cos a –2 cosb sina = cosa sinb d w d 2q


N
i.e., a = =
tan b dt dt 2
R

or sinb cosa = 2 cosb sina or tan a =


2 Relation between linear acceleration and angular accelaration.
r ur r
U

Keep in Memory a = a´r


JO

1. Equation of trajectory of an oblique projectile in terms of In magnitude, a = r a


range (R) is Centripetal acceleration : Acceleration acting on a body moving
in uniform circular motion is called centripetal acceleration. It
U

æ xö arises due to the change in the direction of the velocity vector.


y = x tan q - çè1 - ÷ø
ED

R Magnitude of certipetal acceleration is


v2
2. There are two unique times at which the projectile is at the ac = = rw2
same height h(< H) and the sum of these two times. r
2p æ 1 ö
1 2 Q w= = 2pu çè u = = frequency÷ø
Since, h = (u sin q)t - gt is a quadratic in time, so it has T T
2
\ ac = 4p2 u2 r
two unique roots t1 and t2 (say) such that sum of roots
This acceleration is always directed radially towards the centre of
2u sin q 2h the circle.
(t1 + t2) is g and product (t1t2) is g .
Centripetal force: The force required to keep a body moving in
uniform circular motion is called centripetal force.
mv2
4u sin q
2 2
8h Fc = = mr w2
The time lapse (t1– t2) is - . r
g2 g It is always directed radially inwards.
Centrifugal force : Centrifugal force is a fictitious force which
UNIFORM AND NON-UNIFORM CIRCULAR MOTION acts on a body in rotating (non-inertial frames) frame of reference.
Uniform Circular Motion mv2
An object moving in a circle with a constant speed is said to be Magnitude of the centrifugal force F = .
r
in uniform circular motion.Ex. Motion of the tip of the second This force is always directed radially outwards and is also called
hand of a clock. corolious force.
EBD_7179
80 PHYSICS

Non-uniform Circular Motion : Solution :


An object moving in a circle with variable speed is said to be in Centripetal force is provided by component T sin q, therefore
non-uniform circular motion.
mn 2
If the angular velocity varies with time, the object has two T sin q = ;
accelerations possessed by it, centripetal acceleration (a c) and r
Tangential accelaration (aT) and both perpendicular to each other. and, T cos q = mg;
so,
aT v2 r O
a tan q = = ;
rg l2 - r 2
b q l
é r ù
êQ tan q = ú
ac êë l 2 - r 2 úû
1/2
T cos q
é r 2g ù T
v = ê 2 2 1/2 ú q
ë (l - r ) û T sin q A
Net acceleration B r
1/2
é 0.09 ´ 10 ù
=ê 1/2 ú
a = ac 2 + aT 2 ë (0.25 - 0.09) û mg
= 1.5 m/s.

.IN
a = (r 2 w4 + r 2a 2 ) Example 26.
A cyclist is riding with a speed of 27 km/h. As he approaches
a circular turn on the road of radius 80 m, he applies
a = r w4 + a2
AL brakes and reduces his speed at the constant rate of 0.50
m/s every second. What is the magnitude and direction of
ac the net acceleration of the cyclist on the circular turn ?
and, tan b =
N
aT Solution :
R

5
Keep in Memory Speed, v = 27 km/h = 27 ´ ms -1 = 7.5ms -1
18
U

1. Angular displacement behaves like vector, when its v2


JO

magnitude is very very small. It follows laws of vector centripetal acceleration, a c =


addition. r
2. Angular velocity and angular acceleration are axial vectors. (7.5) 2 -2
or a c = ms = 0.7ms -2
U

3. Centripetal acceleration always directed towards the centre 80


ED

of the circular path and is always perpendicular to the


instantaneous velocity of the particle. v
4. Circular motion is uniform if aT = ra = 0, that is angular
velocity remains constant and radial acceleration
ac
P
v2
ac = = rw2 is constant. q
r a at
5. When aT or a is present, angular velocity varies with time
and net acceleration is
P is the point at which cyclist applies brakes. At this point,
a = a c2 + a T 2 tangential acceleration at, being negative, will act opposite
r
to v .
6. If aT = 0 or a = 0, no work is done in circular motion.
Total acceleration, a = a 2c + a 2t
Example 25.
A sphere of mass 0.2 kg is attached to an inextensible string or, a = (0.7) 2 + (0.5) 2 ms -2 = 0.86ms -2
of length 0.5 m whose upper end is fixed to the ceiling. The a c 0.7
tan q = = = 1.4
sphere is made to describe a horizontal circle of radius a t 0.5
0.3 m. What will be the speed of the sphere? \ q = 54°28¢
Motion in a Plane 81

.IN
AL
N
R
U
JO
U
ED
EBD_7179
82 PHYSICS

1. It is found that A + B = A . This necessarily implies, 10. Given that A + B = R and A2 + B2 = R2. The angle between
A and B is
(a) B = 0 (a) 0 (b) p/4
(b) A, B are antiparallel (c) p/2 (d) p
(c) A, B are perpendicular
11. Given that A + B = R and A = B = R. What should be the
(d) A.B £ 0 angle between A and B ?
2. Which one of the following statements is true? (a) 0 (b) p/3
(a) A scalar quantity is the one that is conserved in a (c) 2p/3 (d) p
process. 12. Let A = iA cos q + jA sin q be any vector. Another vector B,
(b) A scalar quantity is the one that can never take negative which is normal to A can be expressed as
values. (a) i B cos q – j B sin q (b) i B cos q + j B sin q
(c) A scalar quantity is the one that does not vary from (c) i B sin q – j B cos q (d) i B sin q + j B cos q
one point to another in space. 13. Three particles A, B and C are projected from the same point
(d) A scalar quantity has the same value for observers with same initial speeds making angles 30º, 45º and 60º

.IN
with different orientations of the axes. respectively with the horizontal. Which of the following
3. Two balls are projected at an angle q and (90º – q) to the statements is correct?
horizontal with the same speed. The ratio of their maximum (a) A, B and C have unequal ranges
vertical heights is
AL
(b) Ranges of A and C are equal and less than that of B
(a) 1 : 1 (b) tanq : 1
(c) Ranges of A and C are equal and greater than that of B
(c) 1 : tanq (d) tan2q : 1
N
(d) A, B and C have equal ranges
4. Which of the following is not correct ?
r r r 14. Two particles of equal masses are revolving in circular paths
R

(a) A ´ B = - B ´ A of radii r1 and r2 respectively with the same period. The


r r r
(b) A ´ B ¹ B ´ A ratio of their centripetal force is
U

r r r r r r r
(c) A ´ ( B + C ) = A ´ B + A ´ C (a) r1/r2 (b) r2 / r1
r r r r r r
JO

(d) A ´ ( B + C ) = ( A ´ B ) + C
(c) (r1/r2)2 (d) (r2/r1)2
5. The greatest height to which a man can through a ball is h.
15. A bomb is released from a horizontal flying aeroplane. The
What is the greatest horizontal distance to which he can
trajectory of bomb is
U

throw the ball?


h (a) a parabola (b) a straight line
ED

(a) 2h (b) (c) a circle (d) a hyperbola


4
h 16. A projectile can have the same range for two angles of
(c) (d) None of these projection. If h 1 and h2 are maximum heights when the
2
6. If A and B are two vectors, then the correct statement is range in the two cases is R, then the relation between R, h 1
(a) A + B = B + A (b) A – B = B – A and h2 is
(c) A × B = B × A (d) None of these
(a) R = 4 h1h 2 (b) R = 2 h1h 2
7. Three vectors A, B and C satisfy the relation
A · B = 0 and A · C = 0. The vector A is parallel to (c) R = h1h 2 (d) None of these
(a) B (b) C
(c) B · C (d) B × C 17. A bomb is dropped from an aeroplane moving horizontally
at constant speed. If air resistance is taken into
8. If n̂ is a unit vector in the direction of the vector A, then consideration, then the bomb
A (a) falls on earth exactly below the aeroplane
(a) n̂ = (b) n̂ = A A
A (b) falls on the earth exactly behind the aeroplane
A (c) falls on the earths ahead of the aeroplane
(c) n̂ = (d) nˆ = nˆ × A (d) flies with the aeroplane
A
18. Two vectors A and B lie in a plane, another vector C lies
9. A projectile thrown with a speed v at an angle q has a range
outside this plane, then the resultant of these three vectors
R on the surface of earth. For same v and q, its range on the
i.e., A + B + C
é g Earth ù
surface of moon will be ê g moon = (a) can be zero
ë 6 úû (b) cannot be zero
(a) R/6 (b) R
(c) lies in the plane containing A + B
(c) 6 R (d) 36 R
(d) lies in the plane containing A – B
Motion in a Plane 83

19. A cannon on a level plane is aimed at an angle q above the (a) vA = vB = vC (b) vA = vB > vC
horizontal and a shell is fired with a muzzle velocity n0 towards (c) vB > vC > vA (d) vA > vB = vC
a vertical cliff a distance D away. Then the height from the 22. An aeroplane flying at a constant speed releases a bomb.
bottom at which the shell strikes the side walls of the cliff is As the bomb moves away from the aeroplane, it will
(a) always be vertically below the aeroplane only if the
g D2 aeroplane was flying horizontally
(a) D sin q -
2 v 0 2 sin 2 q (b) always be vertically below the aeroplane only if the
aeroplane was flying at an angle of 45° to the horizontal
g D2 (c) always be vertically below the aeroplane
(b) D cos q - (d) gradually fall behind the aeroplane if the aeroplane was
2 v 0 2 cos 2 q
flying horizontally.
23. In uniform circular motion, the velocity vector and
g D2
(c) D tan q - acceleration vector are
2 v 0 2 cos2 q (a) perpendicular to each other
(b) same direction
g D2 (c) opposite direction
(d) D tan q -
2 v0 2 sin 2 q (d) not related to each other
24. The time of flight of a projectile on an upward inclined plane
20. A projectile thrown with velocity v making angle q with
depends upon
vertical gains maximum height H in the time for which the

.IN
(a) angle of inclination of the plane
projectile remains in air, the time period is
(b) angle of projection
(a) H cos q / g (b) 2 H cos q / g (c) the value of acceleration due to gravity
AL
(d) all of the above.
(c) 4H / g (d) 8H / g 25. At the highest point on the trajectory of a projectile, its
21. Three particles A, B and C are thrown from the top of a (a) potential energy is minimum
N
tower with the same speed. A is thrown up, B is thrown (b) kinetic energy is maximum
(c) total energy is maximum
R

down and C is horizontally. They hit the ground with speeds


vA, vB and vC respectively then, (d) kinetic energy is minimum.
U
JO
U

1. A projectile is projected with a kinetic energy E. Its range is R. (a) 20 (b) 23


ED

It will have the minimum kinetic energy, after covering a


(c) 5 33 (d) 26
horizontal distance equal to
(a) 0.25 R (b) 0.5 R 6. If range is double the maximum height of a projectile, then q is
(c) 0.75 R (d) R (a) tan–1 4 (b) tan–1 1/4
(c) tan 1 –1 (d) tan–1 2
2. The range of a projectile when launched at an angle of 15º
with the horizontal is 1.5 km. What is the range of the 7. A body is projected such that its KE at the top is 3/4th of its
projectile when launched at an angle of 45º to the horizontal initial KE. What is the angle of projectile with the horizontal?
(a) 1.5 km (b) 3.0 km (a) 30º (b) 60º (c) 45º (d) 120º
(c) 6.3 km (d) 0.75 km 8. Consider a vector F = 4 i – 3 j. Another vector that is
perpendicular to F is
3. A gun fires two bullets at 60º and 30º with horizontal. The
(a) 4 i + 3 j (b) 6 i
bullets strike at some horizontal distance. The ratio of
(c) 7 k (d) 3 i – 4 j
maximum height for the two bullets is in the ratio
9. From a point on the ground at a distance 2 meters from the
(a) 2 : 1 (b) 3 : 1
foot of a vertical wall, a ball is thrown at an angle of 45º
(c) 4 : 1 (d) 1 : 1 which just clears the top of the wall and afterward strikes
4. The angular speed of a fly-wheel making 120 revolutions/ the ground at a distance 4 m on the other side. The height of
minute is the wall is
(a) p rad/sec (b) 4p rad/sec
2 3
(c) 2p rad/sec (d) 4p2 rad/sec (a) m (b) m
3 4
r r
5. Consider two vectors F1 = 2iˆ + 5kˆ and F2 = 3jˆ + 4kˆ . The 1 4
(c) m (d) m
magnitude of the scalar product of these vectors is 3 3
EBD_7179
84 PHYSICS

10. The velocity of projection of a body is increased by 2%. 18. A body is projected horizontally from a point above the
Other factors remaining unchanged, what will be the ground and motion of the body is described by the equation
percentage change in the maximum height attained? x = 2t, y = 5t2 where x, and y are horizontal and vertical
(a) 1% (b) 2 % coordinates in metre after time t. The initial velocity of the
(c) 4 % (d) 8 % body will be

11. A ball is thrown from the ground with a velocity of 20 3 (a) 29 m / s horizontal (b) 5 m/s horizontal
m/s making an angle of 60º with the horizontal. The ball will (c) 2 m/s vertical (d) 2 m/s horizontal
be at a height of 40 m from the ground after a time t equal to uur
(g = 10 ms–2) 19. A vector P1 is along the positive x-axis. If its vector product

(a) 2 sec (b) 3 sec with another vector P2 is zero then P2 could be
(c) 2 sec (d) 3 sec
12. Forces of 4 N and 5 N are applied at origin along X-axis and (a) 4 ĵ (b) - 4î
Y-axis respectively. The resultant force will be
(c) ( ĵ + k̂ ) (d) - (î + ˆj)
æ5ö æ4ö
(a) 41N, tan -1 ç ÷ (b) 41N, tan -1 ç ÷ 20. A wheel rotates with constant acceleration of
è4ø è5ø 2.0 rod/s2, if the wheel starts from rest the number of

.IN
revolutions it makes in the first ten seconds will be
æ5ö æ4ö
(c) - 41N, tan - 1 ç ÷ (d) - 41N, tan - 1 ç ÷ approximately
è4ø è5ø
(a) 32 (b) 24
13. A particle moves in a circle of radius 25 cm at two revolutions
AL
(c) 16 (d) 8
per second. The acceleration of the particle in meter per
21. A projectile of mass m is fired with velocity u making angle
second2 is
N
q with the horizontal. Its angular momentum about the point
(a) p2 (b) 8 p2 of projection when it hits the ground is given by
R

(c) 4 p2 (d) 2 p2
14. The vector sum of the forces of 10 N and 6 N can be 2mu sin 2 q cos q 2mu 3 sin 2 q cos q
U

(a) 2 N (b) 8 N (a) (b)


g g
JO

(c) 18 N (d) 20 N
15. A bomb is dropped on an enemy post by an aeroplane flying mu sin 2 q cos q mu3 sin 2 q cos q
horizontally with a velocity of 60 km h–1 and at a height of (c) (d)
2g 2g
U

490 m. At the time of dropping the bomb, how far the


aeroplane should be from the enemy post so that the bomb 22. A bucket, full of water is revolved in a vertical circle of
ED

may directly hit the target ? radius 2 m. What should be the maximum time-period of
revolution so that the water doesn’t fall out of the bucket?
400 500
(a) m (b) m (a) 1 sec (b) 2 sec
3 3
(c) 3 sec (d) 4 sec
(c)
1700
m (d) 498 m. r r r r r r
3 23. If | a + b | = | a - b | then angle between a & b is
16. A projectile is thrown horizontally with a speed of (a) 45º (b) 30º
20 ms–1. If g is 10 ms–2, then the speed of the projectile (c) 90º (d) 180º
after 5 second will be nearly 24. If the sum of two unit vectors is a unit vector, then the
(a) 0.5 ms–1 (b) 5 ms–1 magnitude of their difference is
(c) 54 ms–1 (d) 500 ms–1
17. A ball is projected at such an angle that the horizontal range (a) 3 (b) 2
is three times the maximum height. The angle of projection
of the ball is 1
(c) 5 (d)
2
-1 æ 3 ö -1 æ 4 ö
(a) sin ç ÷ (b) sin ç ÷ 25. Out of the following sets of forces, the resultant of which
è4ø è3ø
cannot be zero ?
æ4ö æ4ö (a) 10, 10, 10 (b) 10, 10, 20
(c) cos -1 ç ÷ (d) tan -1 ç ÷
è3ø è3ø (c) 10, 20, 20 (d) 10, 20, 40
Motion in a Plane 85
r
26. The magnitude of the vector product of two vectors is 3 32. The angles which the vector A = 3iˆ + 6jˆ + 2kˆ makes with
times the scalar product. The angle between vectors is the co-ordinate axes are :
p 3 4 1
(a)
p
(b) (a) cos -1 , cos -1 , cos -1
4 6 7 7 7
-1 3 6 2
(b) cos , cos -1 , cos -1
p p 7 7 7
(c) (d)
3 2 4 5 3
(c) cos -1 , cos -1 , cos -1
27. If A = B + C and the magnitudes of A, B and C are 5, 4 and 3 7 7 7
units, the angle between A and C is (d) None of these
(a) cos–1 (3/5) (b) cos–1 (4/5) 33. The resultant of two forces 3P and 2P is R. If the first force
is doubled then the resultant is also doubled. The angle
p
(c) (d) sin–1 (3/4) between the two forces is
2 (a) 60º (b) 120º
28. A rectangular sheet of material has a width of 3 m and a (c) 70º (d) 180º
length of 4 m. Forces with magnitudes of 3 N and 4N. r r r
34. Let C = A + B
respectively, are applied parallel to two edges of the sheet,
r r
as shown in the figure below. (a) | C | is always greater than | A |
r r r r

.IN
4N 4m (b) It is possible to have | C | < | A | and | C | < | B |
r r r
F
AL (c) C is always equal to A + B
3m q r r r
(d) C is never equal to A + B
35. At what angle must the two forces (x + y) and (x – y) act so
N
3N
that the resultant may be (x 2 + y 2 ) ?
R

A third force F, is applied to the centre of the sheet, along a (a) cos -1[-(x 2 + y2 ) / 2(x 2 - y 2 )]
U

line in the plane of the sheet, at an angle q = tan 0.75 with


(b) cos -1[-2(x 2 - y 2 ) /(x 2 + y 2 )]
JO

respect to the horizontal direction. The sheet will be in


translational equilibrium when F has what value? (c) cos -1[-(x 2 + y 2 ) /(x 2 - y 2 )]
(a) F = 3 N (b) F = 4N
U

(c) F = 5 N (d) F = 7N (d) cos -1[-(x 2 - y 2 ) /(x 2 + y 2 )]


29. The linear velocity of a rotating body is given by :
ED

uur uur ur 36. A force of – Fk̂ acts on O, the origin of the coordinate
v = w´ r system. The torque about the point (1, –1) is
If w = î - 2ˆj + 2 k̂ and r = 4 ĵ - 3k̂ , then the magnitude of
Z
v is
(a) 29 units (b) 31 units
O
(c) (d) Y
37 units 41 units
30. Two forces are such that the sum of their magnitudes is 18 N
and their resultant is 12 N which is perpendicular to the X
smaller force. Then the magnitudes of the forces are
(a) 12 N, 6 N (b) 13 N, 5 N (a) F(î - ˆj) (b) - F(î + ĵ)
(c) 10 N, 8 N (d) 16N, 2N.
r r r r r r (c) F(î + ĵ) (d) - F(î - ˆj)
31. For any two vectors A and B , if A . B = | A ´ B | , the
r r r r
magnitude of C = A + B is 37. The component of vector a = 2iˆ + 3jˆ along the vector
i + j is
(a) A + B (b) A 2 + B 2 + 2 AB
5
(a) (b) 10 2
2 2 AB 2
(c) 2
A +B 2 (d) A +B +
2 (c) 5 2 (d) 5
EBD_7179
86 PHYSICS

38. A body is projected at an angle of 30º to the horizontal with (a) It would depend upon the weights of the bullets.
speed 30 m/s. What is the angle with the horizontal after 1.5 (b) The slower one.
seconds? Take g = 10 m/s2. (c) The faster one.
(a) 0º (b) 30º (d) Both will reach simultaneously.
(c) 60º (d) 90º
39. A projectile is moving at 60 m/s at its highest point, where it 47. A cricket ball is hit with a velocity 25 m s -1 , 60° above the
breaks into two equal parts due to an internal explosion. horizontal. How far above the ground, ball passes over a
One part moves vertically up at 50 m/s with respect to the fielder 50 m from the bat (consider the ball is struck very
ground. The other part will move at a speed of close to the ground)?
(a) 110 m/s (b) 120 m/s Take 3 = 1.7 and g = 10 ms–2
(c) 130 m/s (d) 10 61 m / s (a) 6.8 m (b) 7 m
40. A particle having a mass 0.5 kg is projected under gravity (c) 5 m (d) 10 m
with a speed of 98 m/sec at an angle of 60º. The magnitude 48. The equation of a projectile is
of the change in momentum (in N-sec) of the particle after 10 gx 2
seconds is y = 3x -
(a) 0.5 (b) 49 2
(c) 98 (d) 490 The angle of projection is given by
41. A large number of bullets are fired in all directions with the 1
tan q = tan q = 3

.IN
same speed v. What is the maximum area on the ground on (a) (b)
3
which these bullets will spread?
p
p v2 pv4 (c) (d) zero.
(a) (b)
AL 2
g 2
g
49. A body is thrown horizontally with a velocity 2gh from
N
2
v p v
2 4
the top of a tower of height h. It strikes the level ground
(c) p2 (d)
R

2 2 through the foot of the tower at a distance x from the tower.


g g
The value of x is
U

42. A force of (3iˆ + 4j)


ˆ N acts on a body and displaces it by
gh
(a) gh (b)
JO

(3iˆ + 4j)
ˆ m. The work done by the force is 2
(a) 5 J (b) 30 J 2gh
(c) 2h (d)
(c) 25 J (d) 10 J 3
U

43. A person aims a gun at a bird from a point at a horizontal 50. A plane flying horizontally at a height of 1500 m with a
ED

distance of 100 m. If the gun can impact a speed of 500 ms– velocity of 200 ms–1 passes directly overhead on antiaircraft
1 to the bullet. At what height above the bird must he aim
gun. Then the angle with the horizontal at which the
his gun in order to hit it? (g = 10 ms–2) gun should be fired from the shell with a muzzle velocity of
(a) 10.4 cm (b) 20.35 cm 400 ms–1 to hit the plane, is
(c) 50 cm (d) 100 cms (a) 90° (b) 60°
r ˆ ˆ ˆ r (c) 30° (d) 45°
44. If a = i - j + k and b = 2iˆ + ˆj + 3kˆ , then the unit vector along
51. A projectile A is thrown at an angle of 30° to the horizontal
® ® from point P. At the same time, another projectile B is thrown
a + b is with velocity v2 upwards from the point Q vertically below
3i + 4 k -3i + 4 k v
the highest point. For B to collide with A, 2 should be
(a) (b) v1
5 5
Highest
-3i - 4 k point
(c) (d) None of these
5
45. A body is thrown with a velocity of 9.8 ms–1 making an v1
angle of 30º with the horizontal. It will hit the ground after a v2
time A B
30°
(a) 3.0 s (b) 2.0 s
P Q
(c) 1.5 s (d) 1 s
(a) 1 (b) 2
46. Two bullets are fired horizontally, simultaneously and with
different velocities from the same place. Which bullet will (c) 1 (d) 4
hit the ground earlier ? 2
Motion in a Plane 87

52. The velocity of projection of oblique projectile is (a) -4(xˆ + y)


ˆ y
(6î + 8ˆj) m s -1 . The horizontal range of the projectile is P
(b) 4(xˆ + y)
ˆ
(a) 4.9 m (b) 9.6 m
(c) 19.6 m (d) 14 m (c) -(xˆ + y)
ˆ / 2 O x
Q
53. A gun is aimed at a horizontal target. It takes 1 s for the
2 (d) (xˆ - y)
ˆ /4
bullet to reach the target. The bullet hits the target x metre 59. A projectile is thrown in the upward direction making an
below the aim. Then, x is equal to angle of 60° with the horizontal direction with a velocity of
9.8 9 .8 147 ms–1. Then the time after which its inclination with the
(a) m (b) m
4 8 horizontal is 45°, is
(c) 9.8 m (d) 19.6 m. (a) 15 s (b) 10.98 s
54. The equation of trajectory of projectile is given by (c) 5.49 s (d) 2.745 s
x gx 2 60. A cyclist moving at a speed of 20 m/s takes a turn, if he
y= - , where x and y are in metre.
3 20 doubles his speed then chance of overturn
The maximum range of the projectile is (a) is doubled (b) is halved
8 4 (c) becomes four times (d) becomes 1/4 times
(a) m (b) m

.IN
3 3 61. A person swims in a river aiming to reach exactly on the
3 3 opposite point on the bank of a river. His speed of swimming
(c) m (d) m is 0.5 m/s at an angle of 120º with the direction of flow of
4 8 ALwater. The speed of water is
55. A bullet is fired with a speed of 1500 m/s in order to hit a
target 100 m away. If g = 10 m/s2. The gun should be aimed (a) 1.0 m/s (b) 0.5 m/s
(a) 15 cm above the target (c) 0.25 m/s (d) 0.43 m/s
N
(b) 10 cm above the target 62. The position vector of a particle is
(c) 2.2 cm above the target
R

r
(d) directly towards the target r = (a cos wt )î + (a sin wt ) ĵ. The velocity of the particle is
U

56. A projectile of mass m is thrown with a velocity v making an


angle 60° with the horizontal. Neglecting air resistance, the (a) directed towards the origin
JO

change in momentum from the departure A to its arrival at B, (b) directed away from the origin
along the vertical direction is (c) parallel to the position vector
v (d) perpendicular to the position vector
U

63. A ball whose kinetic energy is E is thrown at an angle of 45º


ED

with horizontal. Its kinetic energy at highest point of flight


will be
60°
A B (a) E (b) E/2

(a) 2 mv (b) 3mv E


(c) (d) O
mv 2
(c) mv (d)
3 64. Two projectiles are fired from the same point with the same
57. A person sitting in the rear end of the compartment throws speed at angles of projection 60º and 30º respectively.
a ball towards the front end. The ball follows a parabolic Which one of the following is true?
path. The train is moving with velocity of 20 m/s. A person (a) Their maximum height will be same
standing outside on the ground also observes the ball. How (b) Their range will be same
will the maximum heights (ym) attained and the ranges (R)
(c) Their landing velocity will be same
seen by the thrower and the outside observer compare with
(d) Their time of flight will be same
each other?
(a) Same ym different R (b) Same ym and R 65. A body of 3kg. moves in X-Y plane under the action of force
(c) Different ym same R (d) Different ym and R given by 6t î + 4 tĵ . Assuming that the body is at rest at time
58. A particle moves in a circle of radius 4 cm clockwise at
t = 0, the velocity of body at t = 3 sec is
constant speed 2 cm/s. If x̂ and ŷ are unit acceleration
vectors along X and Y-axis respectively (in cm/s2), the (a) 9î + 6ˆj (b) 18î + 6 ĵ
acceleration of the particle at the instant half way between P
and Q is given by (c) 18î + 12ˆj (d) 12î + 68 ĵ
EBD_7179
88 PHYSICS

66. From a 10 m high building a stone A is dropped, and Directions for Qs. (71 to 75) : Each question contains
simultaneously another identical stone B is thrown STATEMENT-1 and STATEMENT-2. Choose the correct answer
horizontally with an initial speed of 5 ms–1. Which one of (ONLY ONE option is correct ) from the following-
the following statements is true? (a) Statement -1 is false, Statement-2 is true
(a) It is not possible to calculate which one of the two
(b) Statement -1 is true, Statement-2 is true; Statement -2 is a
stones will reach the ground first
correct explanation for Statement-1
(b) Both the stones (A and B) will reach the ground
simultaneously (c) Statement -1 is true, Statement-2 is true; Statement -2 is not
(c) A stone reaches the ground earlier than B a correct explanation for Statement-1
(d) B stone reaches the ground earlier than A (d) Statement -1 is true, Statement-2 is false
67. The vector sum of two forces is perpendicular to their vector r r
differences. In that case, the forces 71. Statement -1 : tr = rr ´ F and rt ¹ F ´ rr
(a) cannot be predicted Statement -2 : Cross product of vectors is commutative.
(b) are equal to each other r
(c) are equal to each other in magnitude 72. Statement -1 : If dot product and cross product of A and
(d) are not equal to each other in magnitude r r r
B are zero, it implies that one of the vector A and B must
æ 20 ö be a null vector
68. A particle moves along a circle of radius ç ÷ m with
è p ø Statement -2 : Null vector is a vector with zero magnitude.
constant tangential acceleration. It the velocity of particle

.IN
73. Statement-1 Two stones are simultaneously projected from
is 80 m/sec at end of second revolution after motion has level ground from same point with same speeds but different
begun, the tangential acceleration is angles with horizontal. Both stones move in same vertical
(a) 40 p m/sec2 (b) 40 m/sec2
2
AL
plane. Then the two stones may collide in mid air.
(c) 640 p m/sec (d) 160 p m/sec2
r r r r r r Statement-2 : For two stones projected simultaneously from
69. If | A ´ B |= 3A.B then the value of | A + B | is same point with same speed at different angles with
N
horizontal, their trajectories may intersect at some point.
(a) (A 2 + B 2 3AB)½ (b) (A 2 + B 2 + AB)½
R

74. Statement-1 : K.E. of a moving body given by as2 where s is


½ the distance travelled in a circular path refers to a variable
æ 2 AB ö
U

(c) çç A + B2 + ÷÷ (d) A + B acceleration.


è 3ø
JO

Statement-2 : Acceleration varies with direction only in this


Ù Ù Ù case of circular motion.
70. If a vector 2 i + 3 j + 8 k is perpendicular to the vector
Ù Ù Ù
75. Statement-1 : Centripetal and centrifugal forces cancel each
U

4 j - 4 i + a k , then the value of a is other.


(a) 1/2 (b) –1/2 Statement-2 : This is because they are always equal and
ED

(c) 1 (d) –1 opposite.

Exemplar Questions 3. Figure shows the orientation of two vectors u and v in the
xy-plane.
1. The angle between A = iˆ + ˆj and B = iˆ - ˆj is Y
(a) 45° (b) 90°
(c) – 45° (d) 180° v
u
2. Which one of the following statements is true?
(a) A scalar quantity is the one that is conserved in a
process O X
(b) A scalar quantity is the one that can never take negative If u = aiˆ + bjˆ and v = piˆ + qjˆ
values Which of the following is correct?
(c) A scalar quantity is the one that does not vary from (a) a and p are positive while b and q are negative
one point to another in space (b) a, p and b are positive while q is negative
(d) A scalar quantity has the same value for observers (c) a, q and b are positive while p is negative
with different orientation of the axes (d) a, b, p and q are all positive
Motion in a Plane 89
r
4. The component of a vector r along X-axis will have maximum Average velocity vector (Vav ) from t = 0 to t = 5s is : [2014]
value if
1 7 ˆ ˆ
(a) r is along positive Y-axis (a) (13iˆ + 14j)
ˆ (b) (i + j)
(b) r is along positive X-axis 5 3
(c) r makes an angle of 45° with the X-axis 11 ˆ ˆ
(c) 2(iˆ + ˆj) (d) (i + j)
(d) r is along negative Y-axis 5
5. The horizontal range of a projectile fired at an angle of 15° is 12. A ship A is moving Westwards with a speed of 10 km h–1
50 m. If it is fired with the same speed at an angle of 45°, its and a ship B 100 km South of A, is moving Northwards with
range will be a speed of 10 km h –1. The time after which the distance
(a) 60 m (b) 71 m between them becomes shortest, is : [2015]
(c) 100 m (d) 141 m
(a) 5h (b) 5 2 h
6. Consider the quantities, pressure, power, energy, impulse,
gravitational potential, electrical charge, temperature, area. (c) 10 2 h (d) 0 h
Out of these, the only vector quantities are : r r wt wt
(a) impulse, pressure and area 13. If vectors A = cos wtiˆ + sinwtjˆ and B = cos ˆi + sin ˆj
(b) impulse and area 2 2
(c) area and gravitational potential are functions of time, then the value of t at which they are
(d) impulse and pressure orthogonal to each other is : [2015 RS]
7. In a two dimensional motion, instantaneous speed v0 is a p p
(a) t = (b) t =
positive constant. Then, which of the following are 2w w

.IN
necessarily true? p
(c) t = 0 (d) t =
(a) The average velocity is not zero at any time 4w
(b) Average acceleration must always vanish r
14. The position vector of a particle R as a function of time is
(c) Displacements in equal time intervals are equal
AL
given by:
(d) Equal path lengths are traversed in equal intervals r
8. In a two dimensional motion, instantaneous speed v0 is a R = 4sin(2pt)iˆ + 4cos(2pt) ˆj
N
positive constant. Then, which of the following are Where R is in meter, t in seconds and î and ĵ denote unit
necessarily true?
R

vectors along x-and y-directions, respectively.


(a) The acceleration of the particle is zero
Which one of the following statements is wrong for the
U

(b) The acceleration of the particle is bounded


motion of particle? [2015 RS]
(c) The acceleration of the particle is necessarily in the
JO

plane of motion v2
(a) Magnitude of acceleration vector is , where v is
(d) The particle must be undergoing a uniform circular R
motion the velocity of particle
U

NEET/AIPMT (2013-2017) Questions (b) Magnitude of the velocity of particle is 8 meter/second


(c) path of the particle is a circle of radius 4 meter.
ED

9. ( )
The velocity of a projectile at the initial point A is 2$i + 3$j (d) Acceleration vector is along - R
r
m/s. It’s velocity (in m/s) at point B is [2013] 15. A particle moves so that its position vector is given by
r
r = cos wtxˆ + sin wtyˆ . Where w is a constant. Which of the
following is true ? [2016]
r
(a) Velocity and acceleration both are perpendicular to r
r
(b) Velocity and acceleration both are parallel to r
r
(c) Velocity is perpendicular to r and acceleration is
directed towards the origin
(a) -2$i + 3 $j (b) 2$i - 3$j r
(d) Velocity is perpendicular to r and acceleration is
(c) 2$i + 3$j (d) -2$i - 3 $j directed away from the origin
r r r r r r r 16. If the magnitude of sum of two vectors is equal to the
10. Vectors A, B and C are such that A × B = 0 and A × C = 0.
r magnitude of difference of the two vectors, the angle
Then the vector parallel to A is [NEET Kar. 2013] between these vectors is : [2016]
r r r r
(a) B and C (b) A ´ B (a) 0° (b) 90°
r r r r (c) 45° (d) 180°
(c) B + C (d) B ´ C
11. A particle is moving such that its position coordinate (x, y) 17. The x and y coordinates of the particle at any time are x = 5t
are – 2t2 and y = 10t respectively, where x and y are in meters
(2m, 3m) at time t = 0 and t in seconds. The acceleration of the particle at t = 2s is
(6m, 7m) at time t = 2 s and (a) 5 m/s2 (b) –4 m/s2 [2017]
(c) –8 m/s 2 (d) 0
(13m, 14m) at time t = 5s.
EBD_7179
90 PHYSICS

Hints & Solutions


EXERCISE - 1 u 2 sin 2 q
1. (b) 2. (d) 16. (a) h1 =
2g
H1 u 2 sin 2 q / 2g
3. (d) = 2 2 = tan 2 q
H 2 u sin (90º -q) / 2g u 2 sin 2 (90 - q) u 2 sin 2q
h2 = , R =
4. (d) 5. (a) 2g g
6. (a) In vector addition, the commutative law is obeyed Range R is same for angle q and (90° – q)
r r r r
i.e., A + B = B + A u 2 sin 2 q u 2 sin 2 (90 - q)
\ h1h 2 = ´
Vector subtraction does not follow commutative law. 2g 2g
r r r r r
7. (d) Since A.C = 0 = A.B , it means that A is perpendicular
r r r r r u 4 (sin 2 q) ´ sin 2 (90 - q)
= [Q sin(90 - q) = cos q]
to both C & B , hence A is parallel to (B ´ C) or 4g 2
r r
(C ´ B ) . u 4 (sin 2 q) ´ cos 2 q

.IN
r = [Q sin 2q = 2sin q cos q]
8. (a) The unit vector of any vector A is defined as 4g 2
r AL
A u 4 (sin q cos q)2 u 4 (sin 2q)2
 = r = =
|A| 4g 2 16g2
9. (c) On earth, R = u2 sin 2q/g.
N
On moon, g' = g/6 (u 2 sin 2q)2 R2
= =
R

R' = u2 sin 2q/g' = 6u2 sin2q/g = 6R. 16g 2 16


U

R 2 - A 2 - B2 R2 - R2 or, R2 = 16 h1h2 or R = 4 h1h 2


10. (c) cos q = = =0
2AB 2A B 17. (b) If there is no resistance, bomb will drop at a place exactly
JO

below the flying aeroplane. But when we take into


\ q = p/2 account air resistance, bomb will face deceleration in
its velocity. So, it will fall on the earth exactly behind
R 2 = [A 2 + B 2 + 2 AB cos q ]
U

11. (c)
the aeroplane.
ED

R 2 = R 2 + R 2 + 2 R 2 cos q 18. (b)


19. (c) From the resultant path of the particle, when it is
- R 2 = 2 R 2 cos q or cos q = -1 / 2 or q = 2p / 3 projected at angle q with its velocity u is
12. (c) The dot product should be zero. A

u 2 sin 60º 3 2
13. (c) R 30º = = (u / g)
g 2
on

y
n
Ca

u 2 sin 90 1 gx 2
R 45º = = u2 / g y = x tan q -
g 2 u 2 cos 2 q
q
O D B
2 2 2
u sin120º u cos 30º u
R 60 º = = = ( 3 / 2) Where y denotes the instantaneous height of particle
g g g
when it travels an instantaneous horizontal distance x.
so R 30º = R 60º > R 45º or R A = R C > R B here x = D, u = vo
gD 2
14. (a) F1 = mr1w 2 ; F2 = mr2 w 2 so y = D tan q - 1
2 v 20 cos 2 q
2p
since period T is same, so w is same, because T = .
w v 2 sin 2 (90 - q)
20. (d) Max. height = H = .....(i)
F1 æ r1 ö 2g
Hence =ç ÷
F2 çè r2 ÷ø 2 v sin( 90 - q )
Time of flight, T = ...(ii)
15. (a) A parabola g
Motion in a Plane 91

EXERCISE - 2
v 1. (b) K.E. is minimum at the highest point. So, the horizontal
Vertical distance is half of the range R i.e., 0.5 R.
u 2 sin 2 q u 2 sin 30 u2 u2
q 2. (b) R1 = = or 1.5 = or =3
g g 2g g

u 2 sin 90 u 2
R2 = = = 3 km
Horizontal g g
3. (b) The bullets are fired at the same initial speed
v cos q 2H
From (i), = ,From (ii), H u 2 sin 2 60º 2g sin 2 60º
g g = ´ 2 =
H¢ 2g u sin 2 30 º sin 2 30º
2H 8H
T=2 = ( 3 / 2) 2 3
g g = =
(1/ 2) 2 1
21. (a) For A: It goes up with velocity u will it reaches its
maximum height (i.e. velocity becomes zero) and comes 4. (b)
r r
back to O and attains velocity u. 5. (a) F1 .F2 = ( 2 î + 5k̂ ).(3ˆj + 4k̂ ) = 20
Using v 2 = u 2 + 2as Þ v A = u 2 + 2gh

.IN
u 2 2 sin q cos q u 2 sin 2 q
u 6. (d) = 2´ or tan q = 2
AL g 2g

O u = vx 1 3 1
7. (a) m(u cos q)2 = ´ m u 2
u 2 4 2
N
h 2 3 3
or cos q = or cos q = = cos 30 º .
4 2
R

vB vA 2 2
u = vX vc = v x + v y 8. (c)
U

2
u2 u2 gx
vC 9. (d) R= or 6 = ; y = x tan q -
JO

For B, going down with velocity u g g 2 u cos 2 q


2

Þ v B = u 2 + 2gh 4 æ1ö 2 4
or h = 2 tan 45 º - ç ÷ = 2- = m
U

For C, horizontal velocity remains same, i.e. u. Vertical 2 cos 2 45 º è 6 ø 3 3


velocity = 0 + 2gh = 2gh
ED

(u sin q) 2 u 2 sin 2 q
10. (c) We know that, y m = H = =
The resultant v C = v 2x + v 2y = u 2 + 2gh . 2g 2g
Hence v A = v B = v C DH 2 Du Du
\ = . Given = 2%
22. (c) Since horizontal component of the velocity of the bomb H u u
will be the same as the velocity of the aeroplane, DH
therefore horizontal displacements remain the same at \ = 2 ´ 2 = 4%
H
any instant of time.
23. (a) In uniform circular motion speed is constant. So, no 1 2
11. (c) As, s = u sin qt - gt
tangential acceleration. 2
1
v2 so 40 = 20 3 ´ ( 3 / 2 ) t - ´ 10 ´ t 2
It has only radial acceleration a R = [directed towards 2
R
or 5t2 – 30t + 40 = 0 or t2 – 6t + 8 = 0
center]
and its velocity is always in tangential direction. So these or t = 2 or 4.
two are perpendicular to each other. The minimum time t = 2s.

2u sin (q - a) 12. (a) R = 4 2 + 5 2 = 41N


24. (d) T=
g cos a
25 (d) Velocity and kinetic energy is minimum at the highest
point.
5N R
1
K.E = m v 2 cos 2 q q
2
4N
EBD_7179
92 PHYSICS

5 æ 5ö 50
The angle q will be given by tan q = or q = tan-1 ç ÷ \ Number of revolution = = 16
4 è 4ø 3.14
r r
1 21. (b) L = R´p
13. (c) Here T = sec the required centripetal acceleration
2
for moving in a circle is u 2 sin 2q
Where R = range =
g
v2 (rw )2
aC = = = rw 2 = r ´ (2p / T) 2 r r
r r The angle between R and p = q
Also, p = m u.
so a c = 0.25 ´ (2p / 0.5) 2 = 16 p 2 ´ .25 = 4.0p 2
u 2 sin 2q
14. (b) R max = (10 + 6) = 16N , R min = (10 - 6) = 4N Hence, L = ´ mu sin(q)
g
Þ Values can be from 4N to 16N
15. (b) Time taken for vertical direction motion 2mu3 sin 2 q cos q
=
2h 2 ´ 490 g
t= = = 100 = 10 s
g 9.8 22. (c) Let its angular velocity be w at all points (uniform
The same time is for horizontal direction. motion). At the highest point weight of the body is
balanced by centrifugal force, so
æ 5ö 500

.IN
\ x = vt = ç 60 ´ ÷ ´ 10 = m g
è 18 ø 3 mw2 r = mg Þ w =
r
16. (c) Even after 5 second, the horizontal velocity v x will be AL 2p r 2 2p
20 m s -1 . The vertical velocity v y is given by T= = 2p = 2p =
w g 10 5
v y = 0 + 10 ´ 5 = 50 m s -1
N
2 ´ 3.14
= = 3 sec .
2 2 2
Now, v = v x + v y = 20 + 50 » 54 ms 2 -1 2. 1
R

r r r r r r r r
23. (c) | a + b | = | a - b | Þ | a + b |2 = | a - b |2
U

u 2 sin 2q u 2 sin 2 q r r r r
17. (d) Given =3
g 2g Þ | a |2 + | b |2 + 2a . b = | a |2 + | b |2 - 2a . b
JO

24. (a)
-1 æ 4 ö 25. (d) R2 = P2 + Q2 – PQ cos q
Þ q = tan ç ÷
è3ø (40)2 = (10)2 + (20)2 – 2 × 10 × 20 × cosq
U

18. (d) The horizontal velocity of the projectile remains 400 cos q = 500 – 1600 = – 1100
constant throughout the journey. 1100 11
ED

Since the body is projected horizontally, the initial cos q = - =- which is not possible.
400 4
velocity will be same as the horizontal velocity at any In this way, the set of forces given in option (d) can not
point. be represented both in magnitude and direction by the
dx sides of a triangle taken in the same order. Thus their
Since, x = 2t , =2 resultant can not be zero.
dt r r r r
\ Horizontal velocity = 2 m/s 26. (c) Here, | A ´ B |= 3 | A.B |
\ Initial velocity = 2 m/s
19. (b) Vector product of parallel vectors is zero. Þ ABsin q = 3ABcos q Þ tan q = 3
20. (c) For circular angular motion, the formula for angular p
displacement q and angular acceleration a is Þ q = 60° =
3
1 27. (a) See fig. Clearly A is the resultant of B and C. Further B
q = wt + a t 2 where w = initial velocity
2 is perpendicular to C
1
or q = 0 + 1 at 2 or q = ´ (2)(10)2
2 2 4
or q = 100 radian 5
2p radian are covered in 1 revolution B 4
A
1
\ 1 radian is covered in revolution q
2p
100 C=3
or 100 radian are covered in revolution
2p cos q = 3/5 or q = cos–1 (3/5)
Motion in a Plane 93

28. (c) A body is in translational equilibrium when the ur r ur


36. (c) Torque t = r ´ F = (iˆ - ˆj) ´ (-Fk)
ˆ
components of all external forces cancel.
For the sheet : F cos q = 4 N, F sinq = 3 N. The magnitude = F[- ˆi ´ kˆ + ˆj ´ k]
ˆ = F(ˆj + ˆi) = F(iˆ + ˆj)
of F is found by adding the squares of the components:
F2cos2 q+ F2 sin 2 q = F2 = 42 + 32 = 25 N2. Therefore F éSince kˆ ´ iˆ = ˆj and ˆj ´ kˆ = iˆ ù
ë û
= 5 N. The F vector points in the proper direction, r r r
r a.b
since tan q = 0.75 = 3/4. 37. (a) Component of a along b = r
|b|
ˆi ˆj kˆ
r 38. (a) u x = 30cos30º = 30 3 / 2, , uy = 30 sin 30°,
29. (a) v = 1 -2 2
v y = 30sin 30° - gt
0 4 -3
v y = 30 sin 30 º - 10 ´ 1. 5 = 0
r
v = î[6 - 8] + ˆj[0 + 3] + k̂[4 - 0] As vertical velocity = 0,
r uur Angle with horizontal a = 0º
v = -2î + 3ˆj + 4k̂ Þ | v |= 29 units . It is a state, when a particle reach to a highest point of
its path.
P sin q
30. (b) Use tan a = 39. (c) From conservation of linear momentum,
Q + P cos q
r mr mr
mv = v + v
P sin q 2 1 2 2

.IN
Þ tan 90° = =¥
Q + P cos q v1 = 50m/sec
\ Q + P cos q = 0 Þ P cos q = -Q. ode y
p l
AL Ex
R= P 2 + Q 2 + 2PQ cos q
120 m/s
N
R= P 2 + Q 2 - 2Q 2 or R = P 2 - Q 2 = 12 q
144 = (P + Q)(P - Q) or P - Q = 144/18 = 8.
R

\ P = 13 N and Q = 5 N.
r r r r 50 m/s
v2
U

31. (b) | A.B |=| A ´ B | Þ AB cos q = ABsin q


(here we take particle & earth as a system so in this
JO

1 case external force is zero & linear momentum is


Þ tanθ = 1 Þ cosθ =
2 conserved)
r
r r Where v is velocity of particle before explosion &
U

Now, | A + B |= A 2 + B2 + 2ABcos q r r
v1, v2 are velocity of its equal pieces.
ED

= A 2 + B2 + 2AB.
1
2 2 here vr = 60iˆ (in x direction),
2 = A + B + 2AB r
v1 = 50ˆj (in y direction)
3iˆ + + bjˆ + 2kˆæ3 6 2 ö r r
32. (b) Â = = ç ˆi + ˆj + kˆ ÷ . If a, b and g are so v 2 = 120iˆ - 50ˆj or | v2 |= 130m / sec &
9 + 36 + 4 è 7 7 7 ø
[From conservation of linear momentum]
angles made by A with coordinate axes, then -50
tan q =
3 6 2 120
cos a = , cos b = and cos g = .
7 7 7 40. (b) There is no change in horizontal velocity, hence no
33. (b) Solve two equations : R2 = 9P2 + 4P2 + 12P2 cosq and change in momentum in horizontal direction. The
4R2 = 36P2 + 4P2 + 24P2 cosq. vertical velocity at t = 10sec is
34. (b) v = 98 ´ sin 60 º - (9.8) ´ 10 = –13.13 m/sec
2 so change in momentum in vertical direction is
æ x 2 + y 2 ö = (x + y)2 + (x - y)2 + 2(x + y)(x - y) cos q
35. (a) ç ÷
è ø = (0.5 ´ 98 ´ 3 / 2) - [ -(0.5 ´ 13.13)]
Þ x 2 + y 2 = x 2 + y 2 + 2xy + x 2 + y 2 - 2xy + 2(x 2 - y 2 ) cos q = 42.434+6.56 = 48.997 » 49
41. (b) Maximum possible horizontal range = v2/g
or 2(x 2 - y2 ).cos q = -(x 2 + y 2 ) Maximum possible area of the circle
2
é -(x 2 + y2 ) ù æ v2 ö p v4 é v2 ù
-1
Þ q = cos ê ú = pç ÷ = ê Here r = ú
2 2
êë 2(x - y ) úû ç g ÷ g2 êë g úû
è ø
EBD_7179
94 PHYSICS
r
42. (c) Here , F = (3iˆ + 4ˆj)N 49. (c) 2gh
r rr
d = (3iˆ + 4j)m
ˆ \ W = F.d = (9 + 16)J = 25J
43. (b) Let the gun be fired with velocity u from point O on the
h
bird at B, making an angle q with the horizontal direction.
Therefore the height of the aims of the person is at height
BA (=h) above the bird.
A x
uy = 0, sy = –h, ay = –g, ty = ?
1
h s = ut + at 2
u 2

q 1 2h
O B \ -h = - gt 2 Þ t =
100 m 2 g

u 2 sin 2q x
Here, horizontal range = = 100 velocity =
g t
2h
5002 sin 2q \ x = 2gh ´ = 2h

.IN
or = 100 or sin 2q g
10
50. (b) Horizontal distance covered should be same for the
100 ´ 10 1 time of collision.
= = = sin14¢
(500) 2 250
AL
1
400 cos q = 200 or cos q = or q = 60°
7 p 2
or 2q = 14' or q = 7' = ´ radian 51. (c) This happen when vertical velocity of both are same.
N
60 180
arc v2 1
\ v2 = v1 sin30° or
R

As, angle = \ q = AB =
radius v1 2
OB
U

or AB = q × OB 52. (b) v = 6 î + 8ˆj


JO

7 p
= ´ ´ (100 ´ 100) cm = 20.35 cm
60 180
r r
44. (a) a + b = 3iˆ + 4kˆ
U

10 8
\ Required unit vector
ED

r r
a+b 3iˆ + 4kˆ 3iˆ + 4kˆ q
= r r = = .
|a+b| 32 + 4 2 5 6
uur
2 u sin q Comparing with v = vx ˆi + v yˆj , we get
45. (d) Time of flight =
g
v x = 6 m s -1 and v y = 8 m s -1
2 ´ 9.8 ´ sin 30º 1
= = 2 ´ = 1 sec . Also, v 2 = v x 2 + v y 2 = 36 + 64 = 100
9. 8 2
46. (d) The initial velocity in the vertically downward direction or v = 10 m s -1
is zero and same height has to be covered.
8 6
1 gx 2 sin q = and cos q =
47. (c) y = x tan q - 10 10
2 u 2 cos 2 q
v 2 sin 2q 2v 2 sin q cos q
R= =
10 ´ 50 ´ 50 =5 m g g
y = 50 tan 60° -
2 ´ 25 ´ 25 ´ cos 2 60° 8 6 1
48. (b) Comparing the given equation with R = 2 ´ 10 ´ 10 ´ ´ ´ = 9.6 m
10 10 10
gx 2 1 2
y = x tan q - , we get 53. (b) x= gt
2u 2 cos 2 q 2
1 1 1 9.8
tan q = 3 = ´ 9.8 ´ ´ = m
2 2 2 8
Motion in a Plane 95

54. (b) Comparing the given equation with the equation of Hence, change in momentum is given by :
trajectory of a projectile,
Dp = mvsin 60° - (-mv sin 60°) = 2mv sin 60°
gx 2
y = x tan q - 3
2u 2 cos 2 q = 2mv = 3mv
2
1 57. (a) The motion of the train will affect only the horizontal
we get, tan q = Þ q = 30°
3 component of the velocity of the ball. Since, vertical
20 component is same for both observers, the ym will be
and 2u 2 cos 2 q = 20 Þ u 2 = same, but R will be different.
2cos 2 q
v2
10 10 40 58. (c) a= = 1 cm/s. Centripetal acceleration is directed
= = = r
cos 2 30° æ 3ö
2 3
towards the centre. Its magnitude = 1. Unit vector at
çç ÷÷ the mid point on the path between P and Q is
è 2 ø
ˆ / 2.
-(xˆ + y)
u2 40 4
Now, R max = = = m
59. (c) v sin 45°
g 3 ´ 10 3
v
55. (c) The bullet performs a horizontal journey of 100 cm with

.IN
constant velocity of 1500 m/s. The bullet also performs 45°
a vertical journey of h with zero initial velocity and u sin 60° u
A v cos 45°
downward acceleration g. AL
Distance 60°
\ For horizontal journey, time (t) = O u cos 60° B
Velocity
Velocity of projectile u = 147 ms–1
N
100 1 angle of projection a = 60°
\t= = sec … (1)
R

1500 15 Let, the time taken by the projectile from O to A be t


The bullet performs vertical journey for this time. where direction b = 45°. As horizontal component of
U

1 2 velocity remains constant during the projectile motion.


For vertical journey, h = ut + gt
JO

2 Þ v cos 45° = u cos 60°


1 1 147 -1
2 Þ v× = 147 ´ Þ v = ms
1 æ1ö 2 2 2
h = 0 + ´10 ´ ç ÷
U

2 è 15 ø For Vertical motion, vy = uy – gt


Þ v sin 45° = 45sin 60° – 9.8 t
ED

10 10 ´ 100
or, h = m= cm 147 1 3
2 ´ 15 ´ 15 2 ´ 15 ´ 15 Þ ´ = 147 ´ - 9.8 t
2 2 2
20 147
or, h = cm = 2.2cm Þ 9.8 t = ( 3 - 1) Þ t = 5.49 s
9 2
60. (d) When a cyclist moves on a circular path, it experiences
æ 20 ö a centrifugal force which is equal to mv2 / r. It tries to
The gun should be aimed ç ÷ cm above the target.
è 9 ø overturn the cyclist in outward direction. If speed
increases twice, the value of centrifugal force too
56. (b)
increases to 4 times its earlier value. Therefore the
PA= mv sin 60°

v chance of overturning is 1/4 times.


61. (c) Here v = 0.5 m/sec. u = ?
u u 1
so sin q = Þ = or u = 0.25 ms–1
60° B v .5 2
A
60° B u C direction
of flow

PB= mvsin 60° v


river
30º
As the figure drawn above shows that at points A and 120º
B the vertical component of velocity is v sin 60° but
their directions are opposite. A
EBD_7179
96 PHYSICS
r r r
62. (d) r = (a cos wt ) î + (a sin wt )ˆj 69. (b) | A ´ B |= A B sin q
r r
r d(r ) d A.B = A B cos q
v= = {(a cos wt )î + (a sin wt )ˆj} r r r r
dt dt | A ´ B |= 3 A.B
= (- aw sin w t )î + (aw cos w t ) ĵ Þ AB sin q = Ö3 AB cos q or tan q = Ö3
= w[( -a sin wt ) î + (a cos wt )ˆj] \ q = 60º
r r r r
r .v = 0 \| A + B |= A2 + B2 + 2ABcos 60º
\ velocity is perpendicular to the displacement.
1 2E = A 2 + B 2 + AB
63. (b) Since E = mv 2 Þ v = 70. (b) For two vectors to be perpendicular to each other
2 m
Now at highest point of flight, the vertical component ® ®
A × B =0
of velocity is zero & only horizontal component is non
Ù Ù Ù Ù Ù Ù
1 2 ( 2 i + 3 j+ 8 k ) · ( 4 j - 4 i + a k ) = 0
zero. So K.E. at highest point is E ' = m(v cos 45º )
2 4 1
=E /2 –8 + 12 + 8a = 0 or a=- =-
8 2
64. (b) Given, u1 = u2 = u, q1 = 60º, q2 = 30º 71. (d) 72. (c) 73. (d) 74. (d) 75. (a)
In Ist case, we know that range

.IN
u 2 sin 2(60°) u 2 sin 120° u 2 sin(90° + 30°) EXERCISE - 3
R1 = = =
g g g AL Exemplar Questions
2 2
u (cos 30 ° ) 3u
= = 1. (b) Given, A = iˆ + ˆj
g 2g
B = iˆ - ˆj
N
In IInd case when q 2 = 30° , then As we know that
r r
R

2 2
u sin 60° u 3 A × B =| A || B | cos q
R2 = = Þ R1 = R2
g 2g
U

(iˆ + ˆj ) × (iˆ - ˆj ) = ( 12 + 12 )( 12 + 12 ) cos q


(we get same value of ranges).
JO

6t 4t (i + j )(i - j ) = 2 ´ 2 cos q
65. (a) F = 6tiˆ + 4tjˆ or a x = ,a y =
3 3 where q is the angle between A and B
so u x = òot a x dt = t 2 Þ (u x ) t=3 = 9m/sec 1- 0 + 0 -1
U

cos q = =0
2 2
2t 2 Þ ( u )
ED

and u y = òot a y dt = y t = 3 = 6 m / sec \ q = 90°


3
2. (d) A scalar quantity does not depend on direction so it
(because ux & uy = 0 at t = 0 sec) has the same value for observers with different
1 2 orientations of the axes.
66. (b) s = gt , s and g are same for both the balls, so time
2 3. (b) From the diagram, u = aiˆ + bjˆ
of fall ‘t’ will also be the same for both of them (s is
As u is in the first quadrant, so both components a
vertical height)
r r r and b will be positive.
67. (c) P = vector sum = A + B
For v = piˆ + qjˆ, as it is in positive x-direction and
r r r
Q = vector differences = A - B located downward so x-component p will be positive
r r and y-component q will be negative.
Since P and Q are perpendicular Hence, a, b and p are positive but q is negative.
r r 4. (b) Let r makes an angle q with positive x-axis component
\ P.Q=0
r r r r of r along x-axis.
Þ (A + B).(A - B) = 0 Þ A2 = B2 Þ A = B rx =| r | cos q
68. (b) Circumference of circle is 2pr = 40m
(rx )maximum =| r | (cos q) maximum
Total distance travelled in two revolution is 80m. Initial
velocity u =0, final veloctiy v = 80 m/sec =| r | cos 0° =| r |
so from (Q cos q is maximum of q = 0°)
v2 =u2+2as q = 0°.
Þ (80)2 = 02+2×80×a Hence the vector r has maximum value along positive
Þ a = 40m/sec2 x-axis.
Motion in a Plane 97

5. (c) Consider, projectile is fired at an angle q. NEET/AIPMT (2013-2017) Questions


According to question, 9. (b) At point B the direction of velocity component of the
q = 15° and R = 50 m projectile along Y - axis reverses.
®
Y Hence, VB = 2i$ - 3j$
10. (d) Vector triple product
u r r r r r r r r r
A ´ ( B ´ C ) = B( A × C ) - C ( A × B ) = 0
r r r r r r r
q Þ A || ( B ´ C ) [Q A × B = 0 and A × C = 0] 1.
X r r r
R (a) ( A + B )2 = (C ) 2
r r
u 2 sin 2q Þ A2 + B 2 + 2 A.B = C 2
Range, R =
g r r
Þ 32 + 42 + 2 A.B = 52
r r
u 2 sin(2 ´ 15°) Þ 2 A.B = 0
R = 50 m = r r
g or Þ A.B = 0
1 r r
50 ´ g = u 2 sin 30° = u 2 ´ \A^ B

.IN
2 uur uur
50 × g × 2 = u2 Here A2 + B2 = C2. Hence, A ^ B
r
u 2 = 50 ´ 9.8 ´ 2 = 100 ´ 9.8 = 980
AL r Dr (displacement)
11. (d) vav =
Dt (time taken)
u = 980 = 31.304 m/s = 14 5
N
(13 - 2)iˆ + (14 - 3)ˆj 11 ˆ ˆ
(Q g = 9.8 m/s 2 ) = = (i + j)
5-0 5
R

Now, q = 45°; ur
( )
V A = 10 –i$
U

12. (a)
u sin 2 ´ 45° u
2 2
R= = ur
V B = 10 ( $j)
JO

g g
ur
(14 5)2 14 ´14 ´ 5 V BA = 10 $j + 10 ˆi = 10 2 km / h
R= = = 100 m
U

g 9.8
Distance OB = 100 cos 45° = 50 2 km
6. (b) As we know that,
ED

N($j)
æ Dp ö
Impulse, I = F Dt = ç ÷ Dt = Dp
è Dt ø
where F is force, Dt is time duration and Dp is change
in momentum. As Dp is a vector quantity, hence impulse
is also a vector quantity. Sometimes area can also be w 10 km/h A 100 km
treated as vector. 45°
7. (d) As speed is a scalar quantity, hence it will be related
with path length (scalar quantity) only. 100 km O
V BA = 10 2 km / h
total distance travelled
Hence, Speed v0 = B
time taken
S
So, total distance travelled = Path length Time taken to reach the shortest distance between
= (speed) × time taken
OB 50 2
Hence, path length which is scalar and traversed in A and B = uuuur = = 5h
equal intervals. VBA 10 2
8. (c) As given that in two dimensional motion the 13. (b) Two vectors are
r
A = cos wti + sin wtj
instanteous speed v0 is positive constant and we know ˆ ˆ
that acceleration is rate of change of velocity or
r wt wt
instantaneous speed and hence it will also be in the B = cos ˆi + sin ˆj
plane of motion. 2 2
EBD_7179
98 PHYSICS
r r r r r r r r
For two vectors A and B to be orthogonal A.B = 0 16. (b) A+B = A-B
r r wt wt
A.B = 0 = cos wt.cos + sin wt.sin Squaring on both sides
2 2
r r2 r r2
æ wt ö æ wt ö A+B = A-B
= cos ç wt - ÷ = cos ç ÷
è 2 ø è 2 ø r r r r r r
Þ A · A + 2A · B + B · B
wt p p
So, = \ t= r r r r r r
2 2 w
= A · A – 2A · B + B · B
14. (b) Here,x = 4sin(2pt) ...(i)
y = 4cos(2pt) ...(ii) r r
Squaring and adding equation (i) and (ii) Þ 4A · B = 0 Þ 4AB cos q = 0
x2 + y2 = 42 Þ R = 4 Þ cos q = 0 Þ q = 90°
Motion of the particle is circular motion, acceleration
17. (b) Given:
ur V2
vector is along – R and its magnitude = x = 5t – 2t2 y = 10t
R
Velocity of particle, V = wR = (2p) (4) = 8p dx dy
15. (c) Given: Position vector vx = = 5 – 4t vy = = 10
dt dt

.IN
r = cos wt + sin wt ŷ
r x̂
r
\ Velocity, v = – wsin wt x̂ + wcos wt ŷ dv x dv y
ax = =–4 ay = =0
and acceleration,
AL dt dt
r r r
a = –w2 cos wt x̂ – w2sin wt ŷ = – w2 r a = a xi + a y j r
r a = -4i m / s2
r . r = 0 hence r ^ vr and
N
r v
r is directed towards the origin. Hence, acceleration of particle at (t = 2 s) = –4m/s2
R

a
U
JO
U
ED
5 Laws of Motion

ARISTOTLE’S FALLACY
According to Aristotelian law an external force is required to keep Third law : To every action there is an equal and opposite
a body in motion. However an external force is required to reaction. For example – walking , swimming , a horse pulling a
overcome the frictional forces in case of solids and viscous forces cart etc.
r r
in fluids which are always present in nature. FAB = – FBA

.IN
LINEAR MOMENTUM (p) Action and reaction act on different bodies and hence cannot
Linear momentum of a body is the quantity of motion contained balance each other. Action and reaction occur simultaneously.
r r
in the body. Momentum p = mv
AL
Forces always occur in pairs.
It is a vector quantity having the same direction as the direction EQUILIBRIUM OF A PARTICLE
of the velocity. Its SI unit is kg ms–1. A body is said to be in equilibrium when no net force acts on the
N
body.
NEWTON’S LAWS OF MOTION r
R

i.e., SF = 0
First law : A body continues to be in a state of rest or of uniform
motion, unless it is acted upon by some external force to change Then SFx = 0, SFy = 0 and SFz = 0
U

its state. Stable equilibrium : If a body is slightly displaced from equilbrium


JO

Newton’s first law gives the qualitative definition of force according position, it has the tendency to regain its original position, it is
to which force is that external cause which tends to change or said to be in stable equilibrium.
actually changes the state of rest or motion of a body.
æ d 2u ö
U

Newton’s first law of motion is the same as law of inertia given by


In this case, P.E. is minimum. ç 2 = +ve ÷
Galileo. ç dr ÷
è ø
ED

Inertia is the inherent property of all bodies because of which


So, the centre of gravity is lowest.
they cannot change their state of rest or of uniform motion unless
Unstable equilibrium : If a body, after being displaced from the
acted upon by an external force.
equilibrium position, moves in the direction of displacement, it is
Second law : The rate of change of momentum of a body is directly
said to be in unstable equilibrium.
proportional to the external force applied on it and the change
takes place in the direction of force applied. æ d 2u ö
In this case, P.E. is maximum. ç 2 = -ve ÷
r dpr mdvr r ç dr ÷
i.e., F = = = ma è ø
dt dt So, the centre of gravity is highest.
This is the equation of motion of constant mass system. For Neutral equilibrium : If a body, after being slightly displaced
variable mass system such as rocket propulsion from the equilibrium position has no tendency to come back or to
r d ( mvr ) move in the direction of displacement the equilibrium is known to
F= be neutral.
dt
æ d 2u ö
r m(dvr ) r dm In this case, P.E. is constant ç 2 = constant ÷
And, F = +v ç dr ÷
dt dt è ø
The SI unit of force is newton. (One newton force is that much The centre of gravity remains at constant height.
force which produces an acceleration of 1ms–2 in a body of mass COMMON FORCES IN MECHANICS
1 kg. 1. Weight : It is the force with which the earth attracts a body
The CGS unit of force is dyne. (1N = 105 dyne) and is called force of gravity, For a body of mass m, where
The gravitational unit of force is kg-wt (kg-f) or g-wt (g-f) acceleration due to gravity is g, the weight
1 kg-wt (kg-f) = 9.8 N, 1 g-wt (g-f) = 980dyne W = mg
EBD_7179
100 PHYSICS

2. Tension : The force exerted by the ends of a loaded/stretched


string (or chain) is called tension. The tension has a sense
of pull at its ends. Case 3
q N
Case 1 Case 2 mg sinq mg cos q
2T mg
T
2T
T Massless N = mg cos q
T T pulley q
T T
m1 m2 4. Spring force : If an object is connected by spring and spring
m1g m2g is stretched or compressed by a distance x, then restoring
force on the object F = – kx
Case 3 where k is a spring contact on force constant.
T T a 5. Frictional force : It is a force which opposes relative motion
T' T' T1
T between the surfaces in contact. f = mN
T m
T1 – T = ma This will be discussed in detail in later section.
T
If m = 0, T1 = T 6. Pseudo force : If a body of mass m is placed in a non-inertial
frame having aceleration ar , then it experiences a Pseudo

.IN
i.e tension is same
The tension in a string remains the same throughout the string if force acting in a direction opposite to the direction of ar .
(a) string is massless, AL r r
(b) pulley is massless or pulley is frictionless Fpseudo = – ma
Case 4 : String having mass Negative sign shows that the pseudo force is always directed
in a direction opposite to the direction of the acceleration of
N
the frame.
y
R
U
JO

Let the total mass of the string be M and length be L. Then mass a
Fpseudo
per unit length is
M m x
U

L
Let x be the distance of the string from the mass m. Then the mass z
ED

æM ö CONSTRAINT MOTION :
of the shaded portion of string is ç ´ x ÷ When the motion of one body is dependent on the other body, the
è L ø
relationship of displacements, velocities and accelerations of
If the string is at rest then the tension T has to balance the wt of
the two bodies are called constraint relationships.
shaded portion of string and weight of mass m.
Case 1 Pulley string system :
æ M ö
\T = çm + x÷ g
è L ø X

Þ as x increases, the tension increases. Thus tension is non- F


uniform in a string having mass.
3. Normal force : It measures how strongly one body presses x Block
the other body in contact. It acts normal to the surface of
contact. Step 1 : Find the distance of the two bodies from fixed points.
Step 2 : The length of the string remain constant. (We use of
mg
Case 1 N = mg this condition)
N Therefore X + (X – x) = constant Þ 2X – x = constant
Case 2 dX dx dX dx
Þ 2 – =0 Þ 2 =
a N – mg = ma dt dt dt dt
m mg Þ N = m(g + a)
é dX
Þ 2Vp = v B êQ = Vp = velocity of pulley
N ë dt
Laws of Motion 101

dx ù When the observer is in non-inertial reference frame a


= v B = velocity of block ú pseudo force is applied on the body under observation.
dt û
Again differentiating we get, 2ap = aB Free Body Diagram (FBD) :
é dVp dvB ù Free body diagram of a mass is a separate diagram of that mass.
êa p = dt and a B = dt ú All forces acting on the mass are sketched. A FBD is drawn to
ë û
visualise the direct forces acting on a body.
ap = acceleration of pulley, aB = acceleration of block Case 1 : Masses M1 and M2 are tied to a string, which goes over
2 2 a frictionless pulley
Case 2 Here h + x + y = constt. On differentiating w.r.t ‘t’
(a) If M2 > M1 and they move with acceleration a

h
T
1 q 2 a T
F M1
x a
[Negative sign with dy/dt shows that with increase in time, y

.IN
M1g M2
decreases]
1´ 2x dx dy
- = 0 Þ cos q (v1 – v2) = 0 M2g
AL
2 h 2 + x 2 dt dt FBD of M1, FBD of M2
é x ù T
N
T
êQ cos q = ú
ëê h 2 + x 2 ûú
R

M1 a M2 a
Case 3 Wedge block system : Thin lines represents the condition
U

of wedge block at t = 0 and dotted lines at t = t


M1g M2g
JO

c T - M1g = M1a M 2g - T = M 2a
ax where T is the tension in the string. It gives
U

ay ay M 2 - M1 2M 1 M 2
a= g and T = g
ED

q M1 + M 2 M1 + M 2
B ax Ax A (b) If the pulley begins to move with acceleration f,
Ax
q downwards
Ax = acceleration of wedge towards left uur M - M uur uur ur 2 M M uur uur
a = 2 1 ( g - f ) and 1 2
ax, ay = acceleration of block as shown T = (g - f )
M1 + M 2 M1 + M 2
ay Case 2 : Three masses M1, M2 and M3 are connected with strings
From D ABC , tan q =
ax + Ax as shown in the figure and lie on a frictionless surface. They are
Frame of Reference : pulled with a force F attached to M1.
Reference frames are co-ordinate systems in which an event is T2 T2 T1 T 1
M3 M2 M1 F
described.
There are two types of reference frames The forces on M2 and M3 are as follows
(a) Inertial frame of reference: These are frames of reference
in which Newton’s laws hold good. These frames are at rest M 2 + M3 M3
T1 = F and T2 = F;
with each other or which are moving with uniform speed M1 + M 2 + M 3 M1 + M 2 + M 3
with respect to each other.
All reference frames present on surface of Earth are F
Acceleration of the system is a =
supposed to be inertial frame of reference. M1 + M 2 + M 3
(b) Non – inertial frame of reference: Newton’s law do not
Case 3 : Two blocks of masses M1 and M2 are suspended
hold good in non-inertial reference frame.
All accelerated and rotatory reference frames are non – vertically from a rigid support with the help of strings as shown
inertial frame of reference. Earth is a non-intertial frame. in the figure. The mass M2 is pulled down with a force F.
EBD_7179
102 PHYSICS

T1 y
R=N T x
T1
M1 g M1
T2 M1g cos q
T2 M1g sin q M1g

M2g
M2 FBD of M2

é M M ù g
F T =ê 2 1 ú
ë M1 + M2 û (1+sin q) T a
The tension between the masses M1 and M2 will be
M2g
T2 = F + M2g
Tension between the support and the mass M1 will be
(ii) When the mass M 1 moves downwards with
T1 = F + (M1 + M2)g
Case 4 : Two masses M1 and M2 are attached to a string which acceleration a.
passes over a pulley attached to the edge of a horizontal table. Equation of motion for M1 and M2,
The mass M1 lies on the frictionless surface of the table. M1g sin q – T = M1a ...(1)
T – M2g = M2a ...(2)

.IN
T
M1 Solving eqns. (1) and (2) we get,
a é M sin q - M 2 ù é M 2 M1 ù g
T
AL a=ê 1 ú g; T = ê ú
ë M1 + M 2 û ë M1 + M 2 û (1 + sin q)
M2 (a) If (M2/M1 = sinq) then the system does not accelerate.
N
M2 g (b) Changing position of masses, does not affect the
R

tension. Also, the acceleration of the system remains


Let the tension in the string be T and the acceleration of the unchanged.
U

system be a. Then
T = M1a ...(1) (c) If M1 = M2 = M (say), then
JO

M2g – T = M2a ...(2) 2 2


Adding eqns. (1) and (2), we get æ q qö æ g ö æ q q ö æ Mg ö
a = ç cos - sin ÷ ç ÷ ; T = ç cos + sin ÷ ç ÷
è 2 2ø è 2 ø è 2 2ø è 2 ø
é M2 ù é M1 M 2 ù
U

a=ê ú g and T = ê úg Case 6 : Two masses M1 and M2 are attached to the ends of a
ë M1 + M 2 û ë M1 + M 2 û
string over a pulley attached to the top of a double inclined
ED

Case 5 : Two masses M1 and M2 are attached to the ends of a


string, which passes over a frictionless pulley at the top of the plane of angle of inclination a and b.
inclined plane of inclination q. Let the tension in the string be T. Let M2 move downwards with acceleration a and the tension in
the string be T then

M1 M1 M2

q M2
M1g sinq M1g cosq
M1g a b
M2g
q
FBD of M1 a
T
(i) When the mass M1 moves upwards with acceleration a.
M1
From the FBD of M1 and M2,
T – M1g sin q = M1a ...(1) a
na M1gcosa
M2g – T = M2a ...(2) gsi M1g
Solving eqns. (1) and (2) we get,
M 1

é M - M1 sin q ù Equation of motion for M1


a=ê 2 úg T – M1g sin a = M1a
ë M1 + M 2 û or T = M1g sin a + M1a ...(1)
FBD of mass M1
Laws of Motion 103

FBD of M2 By work-energy theorem loss in P.E. = gain in K.E.


T a
M 1
2
Þ mgh = mv 2 Þ v = 2gh
2
M
M2gcosb b 2 gs
inb Also, from the figure, h = l sin q. \ v = 2gh = 2gl sin q
M2g
Equation of motion for M2 (a) Acceleration down the plane is g sin q.
M2g sinb – T = M2a (b) Its velocity at the bottom of the inclined plane will be
or T = M2g sin b – M2a ...(2) 2 gh = 2 g l sin q
Using eqn. (1) and (2) we get,
(c) Time taken to reach the bottom will be
M1g sin a + M1a = M2g sin b – M2a
1/ 2 1/ 2
Solving we get, æ 2l ö æ 2h ö 1 1 2h
t =ç =ç = =
÷ ç g sin 2 q ÷÷
( M 2 sin b - M1 sin a ) g M 1M 2 g è g sin q ø è ø æ g ö
sin q ç ÷
1/ 2 sin q g
a= and T = [sin b + sin a]
M1 + M 2 M1 + M 2 è 2h ø
Case 7 : A person/monkey climbing a rope (d) If angles of inclination are q1 and q2 for two inclined planes
T ½
t1 æ sin q 2 ö
Keeping the length constant then =ç ÷
a t2 è sin q1 ø

.IN
Case 9 : Weight of a man in a lift :
a
(i) When lift is accelerated upward : In this case the man also
Mg r
moves in upward direction with an acceleration a .
AL
(a) A person of mass M climbs up a rope with acceleration a.
The tension in the rope will be M(g+a).
N
T – Mg = Ma Þ T = M(g + a)
(b) If the person climbs down along the rope with acceleration a a
R

a, the tension in the rope will be M(g–a).


mg
U

T N
JO

Then from Newton’ second law


N – mg = ma or N = m(g + a)
a a
or Wapp = m(g + a) = Wo (1 + a / g ) (as W = mg)
U

Mg Where Wapp is apparent weight of the man in the lift, Wo is


ED

Mg – T = Ma Þ T = M(g – a) the real weight, N is the reaction of lift on the man. It is clear
(c) When the person climbs up or down with uniform speed, that N = Wapp
tension in the string will be Mg. When the lift moves upward and if we measure the weight
Case 8 : A body starting from rest moves along a smooth inclined of the man by any means (such as spring balance) then we
plane of length l, height h and having angle of inclination q. observe more weight (i.e., Wapp) than the real weight (Wo)
Wapp >Wo
(ii) When lift is accelerated downward : In this case from
l Newton’s second law
h FBD of body
q
a
N=R
mg
N
q
mg cosq mg – N = ma
mg sinq
mg or N = m(g – a) = Wo(1– a/g)
(where N=R is normal reaction applied by plane on the body or W'app= Wo(1– a/g) {Q Wo = mg}
of mass m) If we measure the weight of man by spring balance, we
For downward motion, along the inclined plane, observe deficiency because Wapp< Wo.
mg sin q = ma Þ a = g sin q
EBD_7179
104 PHYSICS

(iii)When lift is at rest or moving with constant velocity : From From figure, dl = R d q ;
Newton’s second law N –mg = 0 or N = mg Mass of the element,
In this case spring balance gives the true weight of the man. m m
dm = dl ; or dm = . R d q
Case 10 : Three masses M1, M2 and M3 are placed on a smooth l l
surface in contact with each other as shown in the figure. Force responsible for acceleration, dF = (dm)g sinq ;
A force F pushes them as shown in the figure and the three
masses move with acceleration a, æm ö mgR
dF = ç R d q ÷(g sin q) = sin q d q
è l ø l
M3
M2 Net force on the chain can be obtained by integrating the
M1 above relation between 0 to a, we have
F2
F1 F a
F2 F1 mg R mg R a mg R
F= ò l
sin q dq =
l
(- cos q ) =
0 l
[1 - cos a]
a 0

M1 mg R é lù
Þ F – F1 = m1a ...(i) = ê1 - cos R ú ;
F1 F l ë û
M2
F2 F1 Þ F1 – F2 = m2a ...(ii) F gR æ lö
\ Acceleration, a = = ç 1 - cos ÷ .

.IN
M3 m l è Rø
F2 Þ F2 = M3 a ...(iii) Example 2.
A block slides down a smooth inclined plane to the ground
Adding eqns. (i), (ii) and (iii) we get, a =
F
AL
when released at the top, in time t second. Another block
M1 + M 2 + M 3 is dropped vertically from the same point, in the absence
of the inclined plane and reaches the ground in t/2 second.
N
M 3F (M 2 + M 3 )F
Þ F2 = and F1 = Then find the angle of inclination of the plane with the
M1 + M 2 + M 3 M1 + M 2 + M 3
R

vertical.
Solution :
U

Keep in Memory If q is the angle which the inclined plane makes with the
vertical direction, then the acceleration of the block sliding
JO

1. When a man jumps with load on his head, the apparent down the plane of length l will be g cosq.
weight of the load and the man is zero.
2. (i) If a person sitting in a train moving with uniform A
q
U

velocity throws a coin vertically up, then coin will fall


back in his hand.
ED

(ii) If the train is uniformly accelerated, the coin will fall l h


behind him.
(iii) If the train is retarded uniformly, then the coin will fall
in front of him.
C B
Example 1.
A chain of length l is placed on a smooth spherical surface 1 2
Using the formula, s = ut + at , we have s = l, u = 0, t = t
of radius R with one of its ends fixed at the top of the 2
sphere. What will be the acceleration a of each element of and a = g cos q.
the chain when its upper end is released? It is assumed
1 1
æ Rö so l = 0 ´ t + g cos q t 2 = (g cos q)t 2 ...(i)
that the length of chain l < ç π ÷ . 2 2
è 2ø Taking vertical downward motion of the block, we get
Solution :
Let m be the mass of the chain of length l. Consider an 1 1
h = 0 + g ( t / 2) 2 = gt 2 / 4 ...(ii)
element of length dl of the chain at an angle q with vertical, 2 2
Dividing eqn. (ii) by (i), we get
h 1
= [Q cos q = h / l]
dl l 4 cos q
q dq 1 1
1
or cos q = ; or cos 2 q = ; or cos q =
R 4 cos q 4 2
or q = 60º
Laws of Motion 105

Example 3. Example 6.
A large mass M and a small mass Two masses each equal to m are lying on X-axis at (–a, 0)
m hang at the two ends of a string and (+ a, 0) respectively as shown in fig. They are connected
that passes through a smooth by a light string. A force F is applied at the origin and along
tube as shown in fig. The mass m the Y-axis. As a result, the masses move towards each other.
moves around a circular path in q What is the acceleration of each mass? Assume the
a horizontal plane. The length of l
instantaneous position of the masses as (– x, 0) and (x, 0)
the string from mass m to the top r m respectively
of the tube is l, and q is the angle
the string makes with the F
vertical. What should be the
frequency (n) of rotation of mass
T
m so that mass
M remains stationary? M
(–a, 0) (a, 0)
Solution : –X X
m O m
Tension in the string T = Mg. Solution :
Centripetal force on the body = mrw2 =mr ( 2p n )2. This is
provided by the component of tension acting horizontally F
i.e. T sinq ( = Mg sinq).

.IN
A
1 Mg
\ mr ( 2pn)2 = Mg sinq = Mgr/l. or n=
2p ml AL T q T
Example 4. B C
A string of negligible mass going over a clamped pulley of O
(–x, 0) (x, 0)
mass m supports a block of mass M as shown in fig. The
N
force on the pulley by the clamp is given by
From figure F = 2 T cos q or T = F/(2 cos q)
The force responsible for motion of masses on X-axis is T
R

(a) 2 Mg
m sin q
U

(b) 2 mg F
\ m a = T sin q = ´ sin q
2 cos q
JO

(c) [ (M + m)2 + m2 ] g
F F OB F x
= tan q = ´ = ´
(d) [ (M + m)2 + M2 ] g 2 2 OA 2 (a - x 2 )
2
U

Solution : (c)
ED

F x
Force on the pulley by the clamp = resultant of so, a = ´
T = (M + m)g and mg acting along horizontal and vertical 2m (a - x 2 )
2

respectively
Example 7.
\ F = [(M + m)g]2 + (mg)2 = [ (M + m)2 + m2 ]g A block of mass M is pulled along horizontal frictionless
surface by a rope of mass m. Force P is applied at one end
Example 5.
The masses of 10 kg and 20 kg respectively are connected of rope. Find the force which the rope exerts on the block.
by a massless spring in fig. A force of 200 newton acts on the Solution :
20 kg mass. At the instant shown, the 10 kg mass has The situation is shown in fig
acceleration 12 m/sec2. What is the acceleration of 20 kg T O
mass? M P
m
20 kg Let a be the common acceleration of the system. Here
10 kg
200 newton T = M a for block
P – T = m a for rope
Solution :
P
Force on 10 kg mass = 10 × 12 = 120 N \ P – M a = m a or P = a (M + m) or a =
The mass of 10 kg will pull the mass of 20 kg in the backward ( M + m)
direction with a force of 120 N. MP
\ Net force on mass 20 kg = 200 – 120 = 80 N \T =
(M + m)
force 80 N
Its acceleration a = = = 4 m / s2
mass 20 kg
EBD_7179
106 PHYSICS

Example 8. r r r
where p1, p 2 ...............p n are individual linear momentum of first,
In the system shown below, friction and mass of the pulley second and nth particle respectively.
are negligible. Find the acceleration of m 2 if If this rigid body is isolated i.e., no external force is applied on it,
m1 = 300 g, m2 = 500 g and F = 1.50 N r
then Ptotal = constant (from Newton’s second law).
Further we know that internal forces (such as intermolecular forces
etc.) also act inside the body, but these can only change individual
linear momentum of the particles (i.e., p1, p2.........), but their total
r
momentum Ptotal remains constant.
Solution : Gun Firing a Bullet
When the pulley moves a distance d, m1 will move a distance If a gun of mass M fires a bullet of mass m with velocity v. Then
2d. Hence m1 will have twice as large an acceleration as m2 from law of conservation of momentum, as initially bullet & gun
has. are at rest position i.e., initial momentum is zero, so final momentum
For mass m1, T1 = m1 (2a) ...(1) (gun + bullet) must also be zero.
For mass m2, F – T2 = m2(a) ...(2) r
Since on firing, the bullet moves with velocity v b in forward
T2 direction, then from Newton’s third law, the gun moves in backward
Putting T1 = in eqn. (1) gives T2 = 4m1a r
2 direction v g . So,

.IN
Initial momentum = final momentum
r r
0 = mvb + MVg uuur
uuur - mvb
Momentum Momentum \ V =
g
AL of bullet of gun M
(–ve sign shows that the vel. of gun will have the opposite
Substituting value of T2 in equation (2), direction to that of bullet)
N
F = 4m1a + m2a = (4m1 + m2)a IMPULSE
According to Newton’s second law the rate of change of
R

F 1.50 momentum of a particle is equal to the total external force applied


Hence a = = = 0.88 m / s 2
4m1 + m 2 4(0.3) + 0.5
U

on it (particle) i.e.,
r
LAW OF CONSERVATION OF LINEAR MOMENTUM dP r
JO

= Fext ...(i)
A system is said to be isolated, when no external force acts on it. dt
r r
For such isolated system, the linear momentum ( P = mv ) is r r r r r tf r
or dP = Fext .dt or DP = Pf - Pi = ò Fext .dt ...(ii)
U

constant i.e., conserved. ti


The linear momentum is defined as r
ED

r r Where Pi is momentum of the particle at initial time ti and when


P = mv .....(1) r r
r we apply some external force Fext its final momentum is Pf at
where v is the velocity of the body, whose mass is m. The direction
r r
of P is same as the direction of the velocity of the body. It is a time tf . The quantity Fext × dt on R.H.S in equation (ii) is called the
vector quantity. From Newton’s second law, impulse.
r d r d r We can write equation (ii) as
Fext . = (mv) = P .....(2)
t r r
dt dt I = ò f Fext .dt = DP ...(iii)
i.e., time rate of change in momentum of the body is equal to total ti
external force applied on the body. r
So, the impulse of the force Fext is equal to the change in
r d r r
If Fext . = 0 Þ (P) = 0 or P = constant .....(3) momentum of the particle. It is known as impulse momentum
dt theorem.
This is called law of conservation of momentum.
Now let us consider a rigid body consisting of a large number of Fext.
particles moving with different velocities, then total linear
momentum of the rigid body is equal to the summation of individual
linear momentum of all particles
n r r r r r
impulse

i.e., å p i = p1 + p 2 + p 3 + ..........p n
Area=

i =1
r n r r r r r
or Ptotal = å pi = p1 + p2 + p3 + .......... + pn t
i =1 ti (a) tf
Laws of Motion 107

Force vary with time and impulse is area under force versus Example 10.
time curve A hammer of mass M strikes a nail of mass m with velocity
of u m/s and drives it ‘s’ meters in to fixed block of wood.
Fext. Find the average resistance of wood to the penetration of
nail.
Area=Fext.Dt Solution :
Applying the law of conservation of momentum,
Fext.
. æ M ö
Fav m u = (M + m) v0 Þ v 0 = ç ÷u
t èm+Mø
ti (b) tf There acceleration a can be obtained using the formula
r (v2 = u2 + 2as).
Force constant with time i.e., Fext. constant with time (shown Here we have 0 – v02 = 2as or a = v02 /2s
by horizontal line) and it would give same impulse to particle 2 2
æ M ö u
in time Dt = tf – ti as time varying force described. \ a =ç ÷
It is a vector quantity having a magnitude equal to the area under è m + M ø 2s
the force-time curve as shown in fig. (a). In this figure, it is assumed
æ M2 ö u2
that force varies with time and is non-zero in time interval Dt = tf– Resistance = (M + m) a = ç ÷
r ç m + M ÷ 2s
ti. Fig.(b) shows the time averaged force Fext. i.e., it is constant è ø

.IN
in time interval Dt, then equation (iii) can be written as Example 11.
A ball of mass 0.5 kg is thrown towards a wall so that it
r t r r
I = Fext. ò f dt = F (t - t ) I = Fext. Dt ...(iv)
AL strikes the wall normally with a speed of 10 ms–1. If the ball
it ext. f i bounces at right angles away from the wall with a speed of
The direction of impulsive vector I is same as the direction of 8ms–1, what impulse does the wall exert on the ball ?
change in momentum. Impulse I has same dimensions as that of Solution :
N
momentum i.e, [MLT–1] Approaching wall
R

Rocket propulsion (A case of system of variable mass ) : It is u = –10 ms–1


based on principle of conservation of linear momentum. 10
f
U

In rocket, the fuel burns and produces gases at high temperature.


These gases are ejected out of the rocket from nozzle at the 8 Leaving wall
JO

backside of rocket and the ejecting gas exerts a forward force on v = +8 ms–1
the rocket which accelerates it.
Taking the direction of the impulse J as positive and using
U

dM J = mv – mu
Let the gas ejects at a rate r = - and at constant velocity u
dt 1 1
ED

w.r.t. rocket then from the conservation of linear momentum we have J= ´8- (-10) = 9 N-s
2 2
dv ru ru Therefore the wall exerts an impulse of 9 N-s on the ball.
= = where M = M0 - rt and M0 is mass of rocket
dt M M 0 - rt
Example 12.
æ M0 ö Two particles, each of mass m, collide head on when their
with fuel and solving this equation, we get v = u log e çç ÷
÷ speeds are 2u and u. If they stick together on impact, find
è M 0 - rt ø
their combined speed in terms of u.
where v = velocity of rocket w.r.t. ground. Solution :
Example 9. m m
Two skaters A and B approach each other at right angles. Before impact u
Skater A has a mass 30 kg and velocity 1 m/s and skater 2u
B has a mass 20 kg and velocity 2 m/s. They meet and
2m
cling together. Find the final velocity of the couple. After impact v
Solution : Using conservation of linear momentum (in the direction of
Applying principle of conservation of linear momentum, the velocity 2u) we have
p = p12 + p 22 ; ( m1 + m 2 ) v = ( m1 v1 ) 2 + ( m 2 v 2 ) 2 1
(m) (2u) – mu = 2m × V Þ V = u
2
( 30 + 20 ) v = ( 30 ´1)2 + ( 20 ´ 2 )2 = 50 The combined mass will travel at speed u/2.
(Note that the momentum of the second particle before impact
50 is negative because its sense is opposite to that specified
v= =1 m / s
50 as positive.)
EBD_7179
108 PHYSICS

FRICTION Fig.(c) (fs )max is equal to mkN. When the book is in motion,
When a body is in motion on a rough surface, or when an object
we call the retarding frictional force as the force of kinetic
moves through water (i.e., viscous medium), then velocity of the
friction fk.
body decreases constantly even if no external force is applied on
the body. This is due to friction. Since fk< (fs )max , so it is clear that, we require more force to
So “an opposing force which comes into existence, when two start motion than to maintain it against friction.
surfaces are in contact with each other and try to move relative By experiment one can find that (fs )max and f k are
to one another, is called friction”. proportional to normal force N acting on the book (by rough
Frictional force acts along the common surface between the two surface) and depends on the roughness of the two surfaces
bodies in such a direction so as to oppose the relative movement in contact.
of the two bodies. Note :
(a) The force of static friction fs between book and rough (i) The force of static friction between any two surfaces
surface is opposite to the applied external force Fext. The r
r in contact is opposite to Fext. and given by f s £ ms N
force of static friction fs = Fext .
and (fs )max = ms N (when the body just moves in the
R=N
right direction).
fs Book Fext. where N = W = weight of book and ms is called
coefficient of static friction, fs is called force of static
friction and (fs )max is called limiting friction or

.IN
(a)
W maximum value of static friction.
r
(b) When Fext . exceeds the certain maximum value of static AL (ii) The force of kinetic friction is opposite to the direction
friction, the book starts accelerating and during motion of motion and is given by fk = mkN
Kinetic frictional force is present. where mk is coefficient of kinetic friction.
(iii) The value of mk and ms depends on the nature of
R=N Body just starts moving
N
surfaces and mk is always less then ms.
Book Friction on an inclined plane : Now we consider a book on an
R

fk Fext. inclined plane & it just moves or slips, then by definition


U

(f s) m
a x
(b) R=N
W
ok
JO

r Bo
(c) A graph Fext . versus | f | shown in figure. It is clear that
q q mg cos q
fs, ,max > fk s in mg=W
mg q
U

|f|
ED

( f s )max = m s R
(fs)max Now from figure, f s,max = mg sin q and R = mg cosq
Body is Body starts with
=msN at rest acceleration Þ ms= tanq or q = tan–1(ms)
where angle q is called the angle of friction or angle of repose
fk=mk N Some facts about friction :
(1) The force of kinetic friction is less than the force of static
O static kinetic friction and the force of rolling friction is less than force of
region region kinetic friction i.e.,
(c)
Fig.(a) shows a book on a horizontal rough surface. Now if fr < fk < fs or mrolling < mkinetic < mstatic
r hence it is easy to roll the drum in comparison to sliding it.
we apply external force Fext. , on the book, then the book
r (2) Frictional force does not oppose the motion in all cases,
will remain stationary if Fext. is not too large. If we increase infact in some cases the body moves due to it.
r
Fext. then frictional force f also increase up to (fs )max B
(called maximum force of static friction or limiting friction)
r A Fext
and (fs )max = msN. At any instant when Fext. is slightly
greater than (fs )max then the book moves and accelerates to In the figure, book B moves to the right due to friction
the right. between A and B. If book A is totally smooth (i.e., frictionless)
Fig.(b) when the book is in motion, the retarding frictional then book B does not move to the right. This is because of
no force applies on the book B in the right direction.
force become less than, (fs )max
Laws of Motion 109

Laws of limiting friction : Figure shows three processes A, B and C by which we can reach
(i) The force of friction is independent of area of surfaces from an initial position to final position. If force is conservative,
in contact and relative velocity between them (if it is then work done is same in all the three processes i.e., independent
not too high). of the path followed between initial and final position.
(ii) The force of friction depends on the nature of material
of surfaces in contact (i.e., force of adhesion). If force is non conservative then work done from i to f is different
m depends upon n ature of the surface. It is in all three paths A,B and C.
independent of the normal reaction. Hence it is clear that work done in conservative force depends
(iii) The force of friction is directly proportional to normal
only on initial & final position irrespective of the path followed
reaction i.e., F µ N or F = mn.
While solving a problem having friction involved, follow between initial & final position. In case of non-conservative
the given methodology forces the work done depends on the path followed between
initial and final position.
If Fapp < fl
We can say also that there is no change in kinetic energy of the
Body does not move and
Fapp = frictional force body in complete round trip in case of conservative force. While
in case of non conservative forces, when a body return to its
Check If Fapp = fl initial position after completing the round trip, the kinetic energy
(a) Fapp Body is on the verge of movement of the body may be more or less than the kinetic energy with
(b) Limiting if the body is initially at rest which it starts.

.IN
friction (fl) Body moves with constant velocity Example 13.
AL Pushing force making an angle q to the horizontal is
applied on a block of weight W placed on a horizontal
table. If the angle of friction is f, then determine the
magnitude of force required to move the body.
N
Rolling Friction : Solution :
R

The name rolling friction is a misnomer. Rolling friction has nothing


The various forces acting on the block are shown in fig.
to do with rolling. Rolling friction occurs during rolling as well as
U

sliding operation.
N
JO

f cosq
f q
U

Cause of rolling friction : When a body is kept on a surface of F


mg F sinq
ED

another body it causes a depression (an exaggerated view shown


in the figure). When the body moves, it has to overcome the
depression. This is the cause of rolling friction. Here,
Rolling friction will be zero only when both the bodies f
incontact are rigid. Rolling friction is very small as compared to m = tan f = ; or f = N tanf...(i)
N
sliding friction. Work done by rolling friction is zero
The condition for the block just to move is
CONSERVATIVE AND NON-CONSERVATIVE FORCES
Fcosq = f = N tanf ...(ii)
If work done on a particle is zero in complete round trip, the
and F sinq + W = N ...(iii)
force is said to be conservative. The gravitational force,
electrostatics force, elastic force etc., are conservative forces. From (ii) and (iii),
On the other hand if the work done on a body is not zero during F cosq = (W + F sinq ) tan f = W tanf + F sin q tanf ;
a complete round trip, the force is said to be non-conservative.
or F cos q – F sinq sinf/cosf = W sinf/cosf
The frictional force, viscous force etc. are non-conservative
forces. or F (cosq cosf – sinq sinf) = W sinf ;
Final or F cos (q + f) = W sinf or F = W sinf / cos (q + f)
position
A f Example 14.
An object of weight W is resting on an inclined plane at an
B
angle q to the horizontal. The coefficient of static friction
C is m. Find the horizontal force needed to just push the object
i
Initial up the plane.
position
EBD_7179
110 PHYSICS

Solution : (loss in kinetic energy of the particle) = (gain in potential energy)


The situation is shown in fig. In conservative force system (such as gravity force) the
mechanical energy (i.e., kinetic energy + potential energy) must
be constant.
osq
R Fc Total energy will be constant
q F Now from eqns.(2) and (3), we get
B
inq
Ws q F sinq vB B
f=m R
q W cosq
mg R
W D
TB
D TA
Let F be the horizontal force needed to just push the object R
up the plane. From figure R = W cos q + F sin q
Now f = mR = m [W cos q + F sin q] ...(1) A vA= vC
Further, F cos q = W sin q + f ...(2) A mg
F cos q = W sin q + m [W cos q + F sin q] m m
TA - TB = 2 mg + (VA2 - VB2 ) = 2 mg + (4gR )
F cos q – m F sin q = W sin q + m W cos q R R

W (sin q + m cos q) Þ TA - TB = 6mg ...(5)


\ F= or TA = TB + 6mg ...(6)

.IN
(cos q - m sin q)
So it is clear from eqn. (6) that tension in string at lowest point
CASES OF CIRCULAR MOTIONS of vertical circle is greater then the tension at highest point of
Motion in a Vertical Circle : vertical circle by 6mg.
AL
Let us consider a particle of mass m attached to a string of length Condition to complete a vertical circle :
R let the particle be rotated about its centre O.
If we reduce the velocity vA in equation (2), then TA will be reduce
At t = 0 the particle start with velocity u from the point A (lowest
N
point of vertical circle) and at time t its position is P. Then the and at some critical velocity vc, TB will be zero, then put TB = 0
tension at point P is given by and vB = vC in equation (3) and we obtain
R

B vC = vB = gR ...(7)
U

In this condition the necessary centripetal force at point B is


JO

provided by the weight of the particle [see again equation (3)]


then from equation (4), we get
O vP
q T v 2A - gR = 4 gR Þ v A = 5 gR ...(8)
U

R P
then the tension at the point A will be
q
ED

A u mg cos q m(5gR )
mg sin q mg TA = mg + = 6mg ...(9)
R
mv 2P mv2P Hence if we rotate a particle in a vertical circle and tension in
TP - mg cos q = or TP = mg cos q + ...(1) string at highest point is zero, then the tension at lowest point of
R R
So tension at point A (lowest point of vertical circle) is vertical circle is 6 times of the weight of the particle.
mv 2A
TA - mg = (Q q = 0º) ...(2)
R Some Facts of Vertical Motion :
and tension at point B (highest point of vertical circle) is (i) The body will complete the vertical circle if its velocity at
mv 2B
TB + mg = (Q q =180º) ...(3) lowest point is equal to or greater then 5gR
R
mv 2 (ii) The body will oscillate about the lowest point if its velocity
Where is centripetal force required for the particle to move
r at lowest point is less then 2 gR . This will happen when
in a vertical circle.
the velocity at the halfway mark, i.e.
Now from law of conservation of energy
1 1 é 1 ù
mv 2A - mv 2B = 2 mgR v D = 0 êQ mv 2A = mgR ú
2 2 ë 2 û
or, v 2A - v 2B = 4gR ...(4) (iii) The string become slack and fails to describe the circle
when its velocity at lowest point lies between
(change in kinetic energy of particle)
= (change in potential energy of particle) 2gR to 5gR
or
Laws of Motion 111

Example 15. Negotiating a Curve :


A mass m is revolving in a vertical circle at the end of a Case of cyclist
string of length 20 cm. By how much does the tension of the To safely negotiate a curve of radius r, a cyclist should bend at an
string at the lowest point exceed the tension at the topmost angle q with the vertical.
point? N Ncosq
Solution :
The tension T 1 at the topmost point is given by, q
m v12
T1 = -mg
20 Nsinq
Centrifugal force acting outward while weight acting
downward
m v 22
The tension T2 at the lowest point, T2 = + mg v2
20 Which is given by tan q = . Angle q is also called as angle of
rg
Centrifugal force and weight (both) acting downward
banking.
m v 2 2 - m v12 2 2
T2 - T1 = + 2mg ; v1 = v 2 - 2 g h or
20 mv 2
N sin q = and N cos q = mg

.IN
v 2 2 - v12 = 2 g (40 ) = 80 g r
Case of car on a levelled road
80 m g A vehicle can safely negotiate a curve of radius r on a rough level
\ T2 - T1 = + 2mg = 6 mg
20
AL
road when coefficient of sliding friction is related to the velocity
Example 16.
v2
A stone of mass 1 kg tied to a light inextensible string of as m s ³ .
N
length L = (10/3) m is whirling in a circular path of radius rg
L in a vertical plane. If the ratio of the maximum to the minimum Now consider a case when a vehicle is moving in a circle, the
R

tension in the string is 4 and g = 10 m/s2, then find the speed


mv 2
U

of the stone at the highest point of the circle. centrifugal force is whereas m is mass of vehicle, r = radius
r
Solution :
JO

of circle and v is its velocity.


P
mv 2
VP r
U

L fs
ED

O The frictional force is static since wheels are in rolling motion


T because point of contact with the surface is at rest
q
mg cos q mv 2
\ fs = f s £ f max = m s mg
VO q r
Q
mg mv 2 v2
m
£ m s mg or s ³
The tension T in the string is given by r rg
é
Case of banking of road (frictionless)
vQ2 ù é v 2ù
Tmax = m êg + ú and T = m ê- g + P ú A vehicle can safely negotiate a curve of radius r on a smooth
ê L ú min
ë û êë L úû (frictionless) road, when the angle q of banking of the road is

According to the given problem v2


given by tan q = .
rg
g + ( v Q 2 / L) vQ 2
= 4 or vP2
2 g+ = -4 g + 4
- g + ( v P / L) L L N
q
Vertical

2 2
vP + 4 g L v
or g+ = -4 g + 4 P
L L
L = (10/3) m and g = 10 m/s2 (given) q mg
Solving we get vP = 10 m/s. Horizontal
EBD_7179
112 PHYSICS

When the banked surface is smooth, the force acting will be gravity Now in the case of minimum velocity with which body could move
and normal force only. in a circular motion, the direction of friction will be opposite to
that one in maximum velocity case.
N
fs
mv2
r N

mg mv 2
q r

Balancing forces mg
N cos q = mg ...(1) q
mv 2
N sin q = ...(2) 2 æ m - tan q ö
r and vmin = rg ç ÷
è 1 + m tan q ø
v2
= tan q ...(3) Keep in Memory
rg
Case of banking of road (with friction) 1. Whenever a particle is moving on the circular path then
The maximum velocity with which a vehicle can safely negotiate there must be some external force which will provide the
necessary centripetal acceleration to the particle.

.IN
a curve of radius r on a rough inclined road is given by
For examples :
rg (m + tan q)
v2 = ; where m is the coefficient of friction of the (i) Motion of satellite around a planet : Here the centripetal
1 - m tan q force is provided by the gravitational force.
rough surface on which the vehicle is moving, and q is the angle
AL
of inclined road with the horizontal.
Suppose a vehicle is moving in a circle of radius r on a rough V
N
μ
inclined road whose coefficient of friction is and angle of
mv 2 Satellite
R

M GMm
banking is q. i.e. 2
= (m)
r r (M) Planet
U

N
N
JO

mv 2 mv 2 (ii) Motion of electron around the nucleus : Here the


r r
required centripetal force is provided by the
Coulombian force
U

fs mg fs mg
q 1 (ze)(e) mv2
ED

i.e. =
Let velocity of object (vehicle) be V. 4pe o r 2 r

mv2
If we apply pseudo force on body, centrifugal force is
r
when v is max. and friction force will be acting down the slope.
mv2 Nucleus
Balancing the force horizontally, = f s cos q + N sin q ...(1) Electron
r r
(Ze) (e)
Balancing the force vertically,
N cos q = f s sin q + mg ...(2)
when v = maximum, f = fmax = fs = mN ...(3)
From eqn. (2), (iii) Motion of a body in horizontal and vertical circle:
N cos q = mN sin q + mg Þ N (cos q - m sin q) = mg Here the centripetal force is provided by the tension.
Horizontal circle
mg
or N =
cos q - m sin q V
2 2
mv mmg cos q + mg sin q mv
From eqns.(1) and (3), = T= (m)
r cos q - m sin q r
T
mv 2 mg (m + tan q) 2 (m + tan q)
Þ = Þ vmax = rg
r 1 - m tan q 1 - m tan q
Laws of Motion 113

Vertical circle (d) The vertical depth h of P below A is independent of the


length of the string since from eqn. (1) and (4)
mv A 2
At point A, TA = ; h lmg
r VB T = mg Þ T = but T = mlw 2
B l h
V mlg g
mg
Therefore mlw 2 = Þh= 2
TB h w
mv B 2 A
which is independent of l.
At point B, TB + mg =
r T
Example 17.
mg
TC A particle of mass m is moving in a circular path of constant
V
mv C 2 C mg C radius r such that its centripetal acceleration ac is varying
And at point C, TC - mg = with time t as ac = k2rt2, where k is a constant. Determine
r
the power delivered to the particle by the forces acting on
CONICAL PENDULUM it.
Consider an inextensible string of length l which is fixed at Solution :
one end, A. At the other end is attached a particle P of mass Here tangential acceleration also exists which requires power.
m describing a circle with constant angular velocity w in a Given that centripetal acceleration
horizontal plane. ac = k2rt2 also, ac = v2/r ;

.IN
\ v2/r = k2rt2 or v2 = k2r2t2 or v = k r t ;
A
Tangential acceleration, a = dv = k r
AL dt
h Tsin Now, force F = ma = m k r ;
P O So, power, P = F v = m k r × k r t = m k2 r2 t.
N
r 2
P O
R

r Example 18.
The string of a pendulum is horizontal. The mass of the bob
U

is m. Now the string is released. What is the tension in the


mg Vertical section Horizontal Plane string in the lowest position?
JO

Solution :
O
As P rotates, the string AP traces out the surface of a cone.
Consequently the system is known as a conical pendulum.
U

Vertically, T cos q = mg ... (1)


ED

Horizontally, ... (2) T


Tsin q = mrw 2
In triangle AOP, r = l sin q ... (3) v

and h = l cos q ... (4)


mg
Several interesting facts can be deduced from these Let v be the velocity of the bob at the lowest position. In
equations : this position, The P.E. of bob is converted into K.E. Hence,
(a) It is impossible for the string to be horizontal.
1
mgl = m v 2 or v 2 = 2 g l ...(1)
mg 2
This is seen from eqn. (1) in which cos q = cannot be
T If T be the tension in the string, then
zero. Hence q cannot be 90°.
æ m v2 ö
(b) The tension is always greater than mg. T - mg = ç ÷
...(2)
ç l ÷
This also follows from eqn. (1) as cos q < 1 (q is acute but è ø
not zero). Hence, T > mg From eqns. (1) and (2).
(c) The tension can be calculated without knowing the T – m g = 2 m g or T = 3 m g
inclination of the string since, from eqn. (2) and (3)
T sin q = ml sin q w 2 Þ T = mlw 2
114

ED
U
JO
U
R
N
AL
.IN
PHYSICS

EBD_7179
Laws of Motion 115

1. A rectangular block is placed on a rough horizontal surface 8. A uniform rope of length L resting on a frictionless horizontal
in two different ways as shown, then surface is pulled at one end by a force F. What is the tension
in the rope at a distance l from the end where the force is
applied.
F
F (a) F (b) F (1 + l/L)
(c) F/2 (d) F (1 – l/L)
(a) (b) 9. A particle of mass m is moving with velocity v1, it is given
(a) friction will be more in case (a) an impulse such that the velocity becomes v2 . Then
(b) friction will be more in case (b) magnitude of impulse is equal to
r r r r
(c) friction will be equal in both the cases (a) m( v2 - v1 ) (b) m( v1 - v 2 )
(d) friction depends on the relations among its dimensions. r r r r
(c) m ´ (v 2 - v1 ) (d) 0.5m(v 2 - v1 )
2. Centripetal force :

.IN
(a) can change speed of the body. 10. A constant force F = m2g/2 is applied on the block of mass
(b) is always perpendicular to direction of motion m1 as shown in fig. The string and the pulley are light and
the surface of the table is smooth. The acceleration of m1 is
(c) is constant for uniform circular motion.
AL
(d) all of these m1
3. When a horse pulls a cart, the horse moves down to F
N
(a) horse on the cart.
(b) cart on the horse.
R

m2
(c) horse on the earth.
U

(d) earth on the horse. m2g


(a) towards right
JO

4. The force of action and reaction 2 (m1 + m 2 )


(a) must be of same nature
(b) must be of different nature m 2g
(b) towards left
U

2 (m1 - m 2 )
(c) may be of different nature
ED

(d) may not have equal magnitude m 2g


(c) towards right
5. A body is moving with uniform velocity, then 2 (m 2 - m1 )
(a) no force must be acting on the body.
(b) exactly two forces must be acting on the body m 2g
(d) towards left
2 (m 2 - m1 )
(c) body is not acted upon by a single force.
(d) the number of forces acting on the body must be even. 11. A mass is hanging on a spring balance which is kept in a lift.
6. The direction of impulse is The lift ascends. The spring balance will show in its readings
(a) same as that of the net force (a) an increase
(b) opposite to that of the net force (b) a decrease
(c) same as that of the final velocity (c) no change
(d) same as that of the initial velocity (d) a change depending on its velocity
7. A monkey is climbing up a rope, then the tension in the rope 12. A cart of mass M has a block of mass m attached to it as
(a) must be equal to the force applied by the monkey on shown in fig. The coefficient of friction between the block
the rope and the cart is m. What is the minimum acceleration of the
(b) must be less than the force applied by the monkey on cart so that the block m does not fall?
the rope. (a) mg
(c) must be greater than the force applied by the monkey (b) g/m M m
on the rope.
(c) m/g
(d) may be equal to, less than or greater the force applied
by the monkey on the rope. (d) M mg/m
EBD_7179
116 PHYSICS

13. A particle of mass m moving eastward with a speed v collides 20. A particle starts sliding down a frictionless inclined plane.
with another particle of the same mass moving northward If Sn is the distance traveled by it from time t = n – 1 sec to
with the same speed v. The two particles coalesce on
t = n sec, the ratio Sn/Sn+1 is
collision. The new particle of mass 2m will move in the north-
external direction with a velocity : 2n - 1 2n + 1
(a) (b)
(a) v/2 (b) 2v 2n + 1 2n
(c) v / 2 (d) None of these 2n 2n + 1
(c) (d)
14. A spring is compressed between two toy carts of mass m1 2n + 1 2n - 1
and m2. When the toy carts are released, the springs exert 21. A block is kept on a inclined plane of inclination q of length l .
equal and opposite average forces for the same time on The velocity of particle at the bottom of inclined is (the
each toy cart. If v1 and v2 are the velocities of the toy carts coefficient of friction is m )
and there is no friction between the toy carts and the ground,
then : (a) [2gl(m cos q - sin q)]1 / 2 (b) 2gl(sin q - m cos q)
(a) v1/v2 = m1/m2 (b) v1/v2 = m2/m1
(c) 2gl(sin q + m cos q) (d) 2gl(cos q + m sin q)
(c) v1/v2 = –m2/m1 (d) v1/v2 = –m1/m2
22. A bird is in a wire cage which is hanging from a spring
15. Two mass m and 2m are attached with each other by a rope
balance . In the first case, the bird sits in the cage and in the
passing over a frictionless and massless pulley. If the pulley
second case, the bird flies about inside the cage. The reading
is accelerated upwards with an acceleration ‘a’, what is the
in the spring balance is

.IN
value of T?
(a) more in the first case
g+a g -a (b) less in first case
(a) (b)
3 3 (c) unchanged
AL
4 m (g + a ) m (g - a ) (d) zero in second case.
(c) (d) 23. In an explosion, a body breaks up into two pieces of unequal
3 3
masses. In this
N
16. A rider on a horse back falls forward when the horse (a) both parts will have numerically equal momentum
suddenly stops. This is due to
R

(b) lighter part will have more momentum


(a) inertia of horse (c) heavier part will have more momentum
U

(b) inertia of rider (d) both parts will have equal kinetic energy
(c) large weight of the horse 24. A block of mass m on a rough horizontal surface is acted
JO

(d) losing of the balance upon by two forces as shown in figure. For equilibrium of
17. A ball of mass m is thrown vertically upwards. What is the block the coefficient of friction between block and surface is
U

rate at which the momentum of the ball changes? F2


q
(a) Zero (b) mg
ED

(c) Infinity (d) Data is not sufficient. F1 m


18. A small block is shot into each of the four tracks as shown
below. Each of the tracks rises to the same height. The
speed with which the block enters the track is the same in all F1 + F2 sin q F1 cos q + F2
(a) (b)
cases. At the highest point of the track, the normal reaction mg + F2 cos q mg - F2 sin q
is maximum in F1 + F2 cos q F1 sin q - F2
(c) (d)
mg + F2 sin q mg - F2 cos q
25. A plate of mass M is placed on a horizontal of frictionless
(a) (b) surface (see figure), and a body of mass m is placed on this
v v plate. The coefficient of dynamic friction between this body
and the plate is m . If a force 2 m mg is applied to the body
of mass m along the horizontal, the acceleration of the plate
will be
(c) (d) m
v v 2m mg
M
19. A weight W rests on a rough horizontal plane. If the angle
of friction be q , the least force that will move the body mm mm
(a) g (b) g
along the plane will be M (M + m)
(a) W cos q (b) W cot q 2mm
2mm g.
(c) W tan q (d) W sin q (c) g (d)
M (M + m)
Laws of Motion 117

1. An object of mass 10 kg moves at a constant speed of


10 ms–1. A constant force, that acts for 4 sec on the object, F (N)
gives it a speed of 2 ms–1 in opposite direction. The force
acting on the object is 1
(a) –3 N (b) –30 N 0 x
2 4 6 7 8
(c) 3 N (d) 30 N t (s)
2. A solid sphere of 2 kg is suspended from a horizontal beam
2
by two supporting wires as shown in fig. Tension in each
wire is approximately (g = 10 ms–2)
(a) 30 N 30º 30º (a) zero (b) 4 N-s
(c) 8 Ns (d) None of these
T T
(b) 20 N 9. Fig. shows a uniform rod of length 30 cm having a mass of

.IN
3.0 kg. The strings shown in the figure are pulled by constant
(c) 10 N forces of 20 N and 32 N. All the surfaces are smooth and the
strings and pulleys are light. The force exerted by 20 cm part
AL
(d) 5 N mg of the rod on the 10 cm part is
3. A toy gun consists of a spring and a rubber dart of mass 16 10
cm 20 cm
g. When compressed by 4 cm and released, it projects the
N
dart to a height of 2 m. If compressed by 6 cm, the height
R

achieved is 20 N 32 N
(a) 3 m (b) 4 m
U

(c) 4.5 m (d) 6 m (a) 20 N (b) 24 N


JO

4. A player stops a football weighting 0.5 kg which comes (c) 32 N (d) 52 N


flying towards him with a velocity of 10m/s. If the impact 10. A force of 10 N acts on a body of mass 20 kg for 10 seconds.
lasts for 1/50th sec. and the ball bounces back with a velocity Change in its momentum is
U

of 15 m/s, then the average force involved is (a) 5 kg m/s (b) 100 kg m/s
ED

(a) 250 N (b) 1250 N (c) 200 kg m/s (d) 1000 kg m/s
(c) 500 N (d) 625 N 11. Consider the system shown in fig. The pulley and the string
5. A car travelling at a speed of 30 km/h is brought to a halt in are light and all the surfaces are frictionless. The tension in
the string is (take g = 10 m/s2)
4 m by applying brakes. If the same car is travelling at 60 km/h,
it can be brought to halt with the same braking power in
1 kg
(a) 8 m (b) 16 m
(c) 24 m (d) 32 m 1 kg
6. A body of mass 4 kg moving on a horizontal surface with an (a) 0 N (b) 1 N
initial velocity of 6 ms–1 comes to rest after 3 seconds. If (c) 2 N (d) 5 N
one wants to keep the body moving on the same surface 12. The elevator shown in fig. is descending with an acceleration
with the velocity of 6 ms–1, the force required is of 2 m/s2. The mass of the block A = 0.5 kg. The force exerted
(a) Zero (b) 4 N by the block A on block B is
(c) 8 N (d) 16 N
(a) 2 N
7. A machine gun has a mass 5 kg. It fires 50 gram bullets at the
rate of 30 bullets per minute at a speed of 400 ms–1. What
(b) 4 N 2
force is required to keep the gun in position? 2 m/s
(a) 10 N (b) 5 N (c) 6 N A
(c) 15 N (d) 30 N
8. A force time graph for the motion of a body is shown in Fig. (d) 8 N
Change in linear momentum between 0 and 8s is B
EBD_7179
118 PHYSICS

13. Two blocks of masses 2 kg and 1 kg are placed on a smooth 20. A ball of mass 0.5 kg moving with a velocity of 2 m/sec
horizontal table in contact with each other. A horizontal force strikes a wall normally and bounces back with the same
of 3 newton is applied on the first so that the block moves speed. If the time of contact between the ball and the wall is
with a constant acceleration. The force between the blocks one millisecond, the average force exerted by the wall on
would be the ball is :
(a) 3 newton (b) 2 newton (a) 2000 newton (b) 1000 newton
(c) 5000 newton (d) 125 newton
(c) 1 newton (d) zero
21. The mass of the lift is 100 kg which is hanging on the string.
14. A 4000 kg lift is accelerating upwards. The tension in the The tension in the string, when the lift is moving with
-
supporting cable is 48000 N. If g = 10m s 2 then the constant velocity, is (g = 9.8 m/sec2)
acceleration of the lift is (a) 100 newton (b) 980 newton
-2 -2 (c) 1000 newton (d) None of these
(a) 1 m s (b) 2 m s 22. In the question , the tension in the strings, when the lift is
-2 accelerating up with an acceleration 1 m/sec2, is
(c) 4 m s -2 (d) 6 m s (a) 100 newton (b) 980 newton
15. A rocket has a mass of 100 kg. Ninety percent of this is fuel. It (c) 1080 newton (d) 880 newton
ejects fuel vapors at the rate of 1 kg/sec with a velocity of 500 23. A block of mass 5 kg resting on a horizontal surface is
m/sec relative to the rocket. It is supposed that the rocket is connected by a cord, passing over a light frictionless pulley
outside the gravitational field. The initial upthrust on the to a hanging block of mass 5 kg. The coefficient of kinetic

.IN
rocket when it just starts moving upwards is friction between the block and the surface is 0.5. Tension in
(a) zero (b) 500 newton the cord is : (g = 9.8 m/sec2)
A
(c) 1000 newton (d) 2000 newton
AL
16. A 0.1 kg block suspended from a massless string is moved 5 kg

first vertically up with an acceleration of 5 ms -2 and then


N

moved vertically down with an acceleration of 5 ms -2 . If 5 kg B


R

T1 and T2 are the respective tensions in the two cases,


(a) 49 N (b) Zero
U

then (c) 36.75 N (d) 2.45 N


24. A 40 kg slab rests on frictionless floor as shown in fig. A 10
JO

(a) T2 > T1
kg block rests on the top of the slab. The static coefficient
(b) T1 - T2 = 1 N, if g = 10 ms -2 of friction between the block and slab is 0.60 while the kinetic
friction is 0.40. The 10 kg block is acted upon by a horizontal
U

(c) T1 - T2 = 1kg f force of 100 N. If g = 9.8 m/s2, the resulting acceleration of


ED

the slab will be:


-2
(d) T1 - T2 = 9.8N, if g = 9.8 ms
17. The coefficient of friction between two surfaces is 0.2. The 100 N
angle of friction is

(a) sin -1 (0.2) (b) cos -1 (0.2)


40 kg
-1 No friction
(c) -1 (d) cot (5)
tan (0.1)
18. A man weighing 80 kg is standing on a trolley weighing 320 (a) 0.98 m/s2 (b) 1.47 m/s2
(c) 1.52 m/s 2 (d) 6.1 m/s2
kg. The trolley is resting on frictionless horizontal rails. If
the man starts walking on the trolley along the rails at a 25. Two blocks are connected over a massless pulley as shown in
speed of one metre per second, then after 4 seconds, his fig. The mass of block A is 10 kg and the coefficient of kinetic
displacement relative to the ground will be : friction is 0.2. Block A slides down the incline at constant speed.
(a) 5 metres (b) 4.8 metres The mass of block B in kg is:
(c) 3.2 metres (d) 3.0 metres
19. Starting from rest, a body slides down a 45º inclined plane in
twice the time it takes to slide down the same distance in the A
absence of friction. The coefficient of friction between the
body and the inclined plane is: 30º B

(a) 0.33 (b) 0.25 (a) 3.5 (b) 3.3


(c) 0.75 (d) 0.80 (c) 3.0 (d) 2.5
Laws of Motion 119

26. Two trolleys of mass m and 3m are connected by a spring. 33. A trailer of mass 1000 kg is towed by means of a rope
They were compressed and released at once, they move off attached to a car moving at a steady speed along a level
in opposite direction and come to rest after covering a road. The tension in the rope is 400 N. The car starts to
distance S1, S2 respectively. Assuming the coefficient of accelerate steadily. If the tension in the rope is now 1650 N,
friction to be uniform, ratio of distances S1 : S2 is : with what acceleration is the trailer moving ?
(a) 1.75 ms–2 (b) 0.75 ms–2
(a) 1 : 9 (b) 1 : 3 –2
(c) 2.5 ms (d) 1.25 ms–2
(c) 3 : 1 (d) 9 : 1 34. A rocket of mass 5000 kg is to be projected vertically upward.
27. A particle of mass 10 kg is moving in a straight line. If its The gases are exhausted vertically downwards with velocity
displacement, x with time t is given by x = (t3 – 2t – 10) m, 1000 ms–2 with respect to the rocket. What is the minimum
then the force acting on it at the end of 4 seconds is rate of burning the fuel so as to just lift the rocket upwards
(a) 24 N (b) 240 N against gravitational attraction ?
(c) 300 N (d) 1200 N (a) 49 kg s–1 (b) 147 kg s–1
(c) 98 kg s –1 (d) 196 kg s–1
28. When forces F1, F2, F3 are acting on a particle of mass m
35. Blocks A and B of masses 15 kg and 10 kg, respectively, are
such that F2 and F3 are mutually perpendicular, then the connected by a light cable passing over a frictionless pulley
particle remains stationary. If the force F1 is now removed as shown below. Approximately what is the acceleration
then the acceleration of the particle is experienced by the system?
(a) F1/m (b) F2F3/mF1 (a) 2.0 m/s2

.IN
(c) (F2 – F3)/m (d) F2/m
(b) 3.3 m/s2
29. One end of massless rope, which passes over a massless
and frictionless pulley P is tied to a hook C while the (c) 4.9 m/s2
AL B
other end is free. Maximum tension that the rope can (d) 9.8 m/s2 A
bear is 360 N. With what value of maximum safe
36. A 50 kg ice skater, initially at rest, throws a 0.15 kg snowball
acceleration (in ms –2 ) can a man of 60 kg moves
N
with a speed of 35 m/s. What is the approximate recoil speed
downwards on the rope? [Take g = 10 ms–2]
of the skater?
R

(a) 0.10 m/s (b) 0.20 m/s


U

P (c) 0.70 m/s (d) 1.4 m/s


37. Block A is moving with acceleration A along a frictionless
JO

C horizontal surface. When a second block, B is placed on top


of Block A the acceleration of the combined blocks drops to
1/5 the original value. What is the ratio of the mass of A to
U

(a) 16 (b) 6 the mass of B?


(a) 5 : 1 (b) 1 : 4
ED

(c) 4 (d) 8
(c) 3 : 1 (d) 2 : 1
r
30. A force F = 8î - 6 ĵ - 10k̂ newton produces an acceleration 38. A force F is used to raise a 4-kg mass M from the ground to
a height of 5 m.
of 1 ms–2 in a body. The mass of the body is

(a) 10 kg (b) 10 2 kg

(c) 10 3 kg (d) 200 kg 60°

31. A uniform chain of length 2 m is kept on a table such that a


length of 60 cm hangs freely from the edge of the table. The F
total mass of the chain is 4 kg. What is the work done in M
pulling the entire chain on the table ? What is the work done by the force F? (Note : sin 60° = 0.87;
(a) 12 J (b) 3.6 J cos 60° = 0.50. Ignore friction and the weights of the pulleys)
(c) 7.2 J (d) 1200 J (a) 50 J (b) 100 J
32. A body of mass 1 kg moving with a uniform velocity of (c) 174 J (d) 200 J
39. A 5000 kg rocket is set for vertical firing. The exhaust speed
1 ms -1 . If the value of g is 5 ms -2 , then the force acting on is 800 m/s. To give an initial upward acceleration of 20 m/s2,
the frictionless horizontal surface on which the body is the amount of gas ejected per second to supply the needed
moving is thrust will be (Take g = 10 m/s2)
(a) 5 N (b) 1 N (a) 127.5 kg/s (b) 137.5 kg/s
(c) 155.5 kg/s (d) 187.5 kg/s
(c) 0 N (d) 10N
EBD_7179
120 PHYSICS

40. A bullet is fired from a gun. The force on the bullet is given 47. A 1 kg block and a 0.5 kg block move together on a horizontal
by F = 600 – 2 × 105 t frictionless surface . Each block exerts a force of 6 N on the
Where, F is in newtons and t in seconds. The force on the other. The block move with a uniform acceleration of
bullet becomes zero as soon as it leaves the barrel. What is a
the average impulse imparted to the bullet? F 1 kg 0.5 kg
(a) 1.8 N-s (b) Zero
(c) 9 N-s (d) 0.9 N-s (a) 3 ms -2 (b) 6 ms -2
41. A rifle man, who together with his rifle has a mass of 100 kg, (c) 9 ms -2 (d) 12 ms -2
stands on a smooth surface and fires 10 shots horizontally. 48. A body of mass 32 kg is suspended by a spring balance
Each bullet has a mass 10 g and a muzzle velocity of 800 ms– from the roof of a vertically operating lift and going
1. The velocity which the rifle man attains after firing 10
downward from rest. At the instant the lift has covered 20 m
shots is and 50 m, the spring balance showed 30 kg and 36 kg
-1 respectively. Then the velocity of the lift is
(a) 8 ms -1 (b) 0.8 ms
(a) decreasing at 20 m, and increasing at 50 m
(c) 0.08 ms -1 (d) – 0.8 ms -1 (b) increasing at 20m and decreasing at 50 m
42. A block of mass 4 kg rests on an inclined plane. The (c) continuously decreasing at a steady rate throughout
inclination to the plane is gradually increased. It is found the journey
that when the inclination is 3 in 5, the block just begins to (d) constantly increasing at constant rate throughout the

.IN
slidedown the plane. The coefficient of friction between the journey.
block and the plane is 49. An object at rest in space suddenly explodes into three
parts of same mass. The momentum of the two parts are
(a) 0.4 (b) 0.6
(c) 0.8 (d) 0.75.
AL 2pî and pĵ . The momentum of the third part
43. The minimum velocity (in ms-1) with which a car driver must
(a) will have a magnitude p 3
traverse a flat curve of radius 150 m and coefficient of friction
N
0.6 to avoid skidding is (b) will have a magnitude p 5
R

(a) 60 (b) 30
(c) will have a magnitude p
(c) 15 (d) 25
U

(d) will have a magnitude 2p.


44. A body of mass 2 kg is placed on a horizontal surface 50. A triangular block of mass M with angles 30°, 60°, and 90°
JO

having kinetic friction 0.4 and static friction 0.5. If the force rests with its 30°–90° side on a horizontal table. A cubical
applied on the body is 2.5 N, then the frictional force acting block of mass m rests on the 60°–30° side. The acceleration
on the body will be [g = 10 ms–2] which M must have relative to the table to keep m stationary
U

(a) 8 N (b) 10 N relative to the triangular block assuming frictionless contact is


(c) 20 N (d) 2.5 N g
(a) g (b)
ED

45. A bag of sand of mass m is suspended by a rope. A bullet of 2


g g
mass m is fired at it with a velocity v and gets embedded (c) (d)
20 3 5
into it. The velocity of the bag finally is 51. A body of mass 1.0 kg is falling with an acceleration of 10 m/
sec2. Its apparent weight will be (g = 10 m/sec2)
v 20v (a) 1.0 kg wt (b) 2.0 kg wt
(a) ´ 21 (b)
20 21 (c) 0.5 kg wt (d) zero
52. In the figure a smooth pulley of negligible weight is
v v suspended by a spring balance. Weight of 1 kg f and
(c) (d)
20 21 5 kg f are attached to the opposite ends of a string passing
46. For the arrangement shown in the Figure the tension in the over the pulley and move with acceleration because of
gravity, During their motion, the spring balance reads a
string is [Given : tan -1 (0.8) = 39° ] weight of

(a) 6 kg f
m = 1 kg
(b) less then 6 kg f
m = 0.8
39° (c) more than 6 kg f

(a) 6 N (b) 6.4 N (d) may be more or less then 6 kg f 1 kg


(c) 0.4 N (d) zero. 5 kg
Laws of Motion 121

53. A particle moves so that its acceleration is always twice its 60. A horizontal force F is applied on back of mass m placed on
velocity. If its initial velocity is 0.1 ms–1, its velocity after it a rough inclined plane of inclination q . The normal reaction
has gone 0.1 m is N is
(a) 0.3 ms–1 (b) 0.7 ms–1
(c) 1.2 ms –1 (d) 3.6 ms–1
54. An object is resting at the bottom of two strings which are F
inclined at an angle of 120° with each other. Each string can
withstand a tension of 20N. The maximum weight of the
object that can be supported without breaking the string is
(a) 5 N (b) 10 N
(c) 20 N (d) 40 N
55. On a smooth plane surface (figure) two block A and B are (a) mg cos q (b) mg sin q
accelerated up by applying a force 15 N on A. If mass of B is mg cos q - Fcos q
(c) (d) mg cos q + F sin q
twice that of A, the force on B is
61. The coefficient of friction between the rubber tyres and the
(a) 30 N (b) 15 N
road way is 0.25. The maximum speed with which a car can
(c) 10 N (d) 5 N be driven round a curve of radius 20 m without skidding is
(g = 9.8 m/s2)

.IN
15 N A B (a) 5 m/s (b) 7 m/s
AL (c) 10 m/s (d) 14 m/s
62. A bucket tied at the end of a 1.6 m long string is whirled in a
56. A 10 kg stone is suspended with a rope of breaking strength vertical circle with constant speed. What should be the
30 kg-wt. The minimum time in which the stone can be raised minimum speed so that the water from the bucket does not
through a height 10 m starting from rest is (Take
N
spill when the bucket is at the highest position?
g = 10 N / kg) (a) 4 m/sec (b) 6.25 m/sec
R

(a) 0.5 s (b) 1.0 s (c) 16 m/sec (d) None of the above
U

63. A cane filled with water is revolved in a vertical circle of


(c) 2/3 s (d) 2 s
radius 4 meter and the water just does not fall down. The
JO

-1 time period of revolution will be


57. A ball of mass 0.4 kg thrown up in air with velocity 30 ms
reaches the highest point in 2.5 second . The air resistance (a) 1 sec (b) 10 sec
(c) 8 sec (d) 4 sec
U

encountered by the ball during upward motion is


(a) 0.88 N (b) 8800N 64. A circular road of radius r in which maximum velocity is v,
ED

(c) 300 dyne (d) 300 N. has angle of banking


58. A player caught a cricket ball of mass 150 g moving at a rate æ v2 ö æ rg ö
of 20 m/s. If the catching process is completed in 0.1s, the (a) tan -1ç ÷ (b) tan -1 çç ÷÷
ç rg ÷ è v2 ø
force of the blow exerted by the ball on the hand of the è ø
player is equal to
ævö æ rg ö
(a) 150 N (b) 3 N (c) tan -1 çç ÷÷ (d) tan -1 ç ÷
(c) 30 N (d) 300 N è rg ø èvø
59. In the system shown in figure, the pulley is smooth and 65. A small sphere is attached to a cord and rotates in a vertical
massless, the string has a total mass 5g, and the two circle about a point O. If the average speed of the sphere is
suspended blocks have masses 25 g and 15 g. The system increased, the cord is most likely to break at the orientation
is released from state l = 0 and is studied upto stage l' = 0 when the mass is at
During the process, the acceleration of block A will be
A
g m
(a) constant at l
9
C D
O
g l l'
(b) constant at
4 A B
(c) increasing by factor of 3 25 g
B (a) bottom point B (b) the point C
(d) increasing by factor of 2 15 g
(c) the point D (d) top point A
EBD_7179
122 PHYSICS

66. A person with his hand in his pocket is skating on ice at the 71. A ball of mass 400 gm is dropped from a height of 5 m. A boy
rate of 10m/s and describes a circle of radius 50 m. What is on the ground hits the ball vertically upwards with a bat
his inclination to vertical : (g = 10 m/sec2) with an average force of 100 newton so that it attains a
(a) tan–1(½) (b) tan–1 (1/5) vertical height of 20 m. The time for which the ball remains in
contact with the bat is (g = 10 m/s2)
(c) tan–1 (3/5) (d) tan –1(1/10)
(a) 0.12 s (b) 0.08 s
67. When the road is dry and the coefficient of the friction is m,
the maximum speed of a car in a circular path is 10 ms–1. If (c) 0.04 s (d) 12 s
72. Block A of weight 100 kg rests on a block B and is tied with
m
the road becomes wet and m' = , what is the maximum horizontal str ing to the wall at C. Block B is of
2
200 kg. The coefficient of friction between A and B is 0.25
speed permitted?
1
(a) 5 ms–1 (b) 10 ms–1 and that between B and surface is . The horizontal force F
3
(c) 10 2 ms -1 (d) 5 2 ms -1 necessary to move the block B should be (g = 10 m/s2)
68. Two pulley arrangements of figure given are identical. The
mass of the rope is negligible. In fig (a), the mass m is lifted A
by attaching a mass 2m to the other end of the rope. In fig C

.IN
(b), m is lifted up by pulling the other end of the rope with a
B F
constant downward force F = 2mg. The acceleration of m in
the two cases are respectively
(a) 1050 N (b) 1450 N
AL
(c) 1050 N (d) 1250 N
73. An open topped rail road car of mass M has an initial velocity
N
v0 along a straight horizontal frictionless track. It suddenly
starts raising at time t = 0. The rain drops fall vertically with
R

velocity u and add a mass m kg/sec of water. The velocity of


U

car after t second will be (assuming that it is not completely


filled with water)
JO

m 2m m F = 2 mg
u mv0
(a) (b) (a) v0 + m (b)
M M + mt
U

Mv0 + ut mut
ED

(a) 3g, g (b) g /3, g (c) (d) v0 +


M + ut M + ut
(c) g / 3 , 2g (d) g, g / 3
74. A ball mass m falls vertically to the ground from a height h1
69. The linear momentum p of a body moving in one dimension and rebounds to a height h 2. The change in momentum of
varies with time according to the equating P = a + bt2 where the ball of striking the ground is
a and b are positive constants. The net force acting on the
body is (a) m 2g(h1 + h 2 ) (b) n 2g(m1 + m 2 )
(a) proportional to t2
(c) mg(h1 - h 2 ) (d) m( 2gh1 - 2gh 2 )
(b) a constant
(c) proportional to t 75. In the given figure, the pulley is assumed massless and
frictionless. If the friction force on the object of mass m is f,
(d) inversely proportional to t
then its acceleration in terms of the force F will be equal to
70. Three blocks of masses m1, m2 and m3 are connected by
massless strings, as shown, on a frictionless table. They are
pulled with a force T3 = 40 N. If m1 = 10 kg, m2 = 6 kg and m3
= 4kg, the tension T2 will be F
m
T1 T2 T3
M1 M2 M3
æF ö
(a) (F - f ) / m (b) ç -f ÷/m
(a) 20 N (b) 40 N è2 ø
(c) 10 N (d) 32 N (c) F/m (d) None of these
Laws of Motion 123

76. A smooth block is released at rest on a 45° incline and then 82. A block of mass m is placed on a smooth wedge of
slides a distance ‘d’. The time taken to slide is ‘n’ times as inclination q. The whole system is accelerated horizontally
much to slide on rough incline than on a smooth incline. so that the block does not slip on the wedge. The force
The coefficient of friction is exerted by the wedge on the block (g is acceleration due to
gravity) will be
1 1 (a) mg/cos q (b) mg cos q
(a) m k = 1- (b) m k = 1-
n 2
n2 (c) mg sin q (d) mg
83. The coefficient of static friction, ms, between block A of
1 1 mass 2 kg and the table as shown in the figure is 0.2. What
(c) m s = 1- (d) m s = 1- would be the maximum mass value of block B so that the
n 2
n2 two blocks do not move? The string and the pulley are
assumed to be smooth and massless. (g = 10 m/s2)
77. The upper half of an inclined plane with inclination f is
perfectly smooth while the lower half is rough. A body 2 kg
A
starting from rest at the top will again come to rest at the
bottom if the coefficient of friction for the lower half is given
by B

(a) 2 cos f (b) 2 sin f (a) 0.4 kg (b) 2.0 kg

.IN
(c) 4.0 kg (d) 0.2 kg
(c) tan f (d) 2 tan f
84. A body under the action of a force
78. A particle of mass 0.3 kg subject to a force F = – kx with k = r
F = 6 ˆi – 8 ˆj+10 k,
ˆ acquires an acceleration of 1 m/s2. The
15 N/m . What will be its initial acceleration if it is released
AL
from a point 20 cm away from the origin ? mass of this body must be
(a) 15 m/s2 (b) 3 m/s2 (a) 10 kg (b) 20 kg
N

(c) 10 m/s2 (d) 5 m/s2 (c) 10 2 kg (d) 2 10 kg


R

79. A block is kept on a frictionless inclined surface with angle 85. A conveyor belt is moving at a constant speed of 2m/s. A box
is gently dropped on it. The coefficient of friction between
U

of inclination ‘ a ’ . The incline is given an acceleration ‘a’ to


keep the block stationary. Then a is equal to them is µ = 0.5. The distance that the box will move relative to
JO

belt before coming to rest on it taking g = 10 ms–2, is


(a) 1.2 m (b) 0.6 m
(c) zero (d) 0.4 m
U

86. A person of mass 60 kg is inside a lift of mass 940 kg and


ED

presses the button on control panel. The lift starts moving


a upwards with an acceleration 1.0 m/s2. If g = 10 ms–2, the
tension in the supporting cable is
(a) 8600 N (b) 9680 N
(a) g cosec a (b) g / tan a (c) 11000 N (d) 1200 N
(c) g tan a (d) g 87. The upper half of an inclined plane of inclination q is
perfectly smooth while lower half is rough. A block
80. Consider a car moving on a straight road with a speed of 100
starting from rest at the top of the plane will again come to
m/s . The distance at which car can be stopped is [µk = 0.5] rest at the bottom, if the coefficient of friction between the
(a) 1000 m (b) 800 m block and lower half of the plane is given by
(c) 400 m (d) 100 m 2
(a) m = (b) m = 2 tan q
81. A round uniform body of radius R, mass M and moment of tan q
inertia I rolls down (without slipping) an inclined plane 1
making an angle q with the horizontal. Then its acceleration (c) m = tan q (d) m =
tan q
is
88. A bridge is in the from of a semi-circle of radius 40m. The
(a) g sin q (b) g sin q greatest speed with which a motor cycle can cross the bridge
1, MR 2 / I 1 ∗ I / MR 2 without leaving the ground at the highest point is
(g = 10 m s–2) (frictional force is negligibly small)
g sin q g sin q
(c) (d) (a) 40 m s–1 (b) 20 m s–1
1 ∗ MR 2 / I 1, I / MR 2 (c) 30 m s–1 (d) 15 m s–1
EBD_7179
124 PHYSICS

89. An explosion breaks a rock into three parts in a horizontal 95. A heavy uniform chain lies on horizontal table top. If the
plane. Two of them go off at right angles to each other. The coefficient of friction between the chain and the table surface
first part of mass 1 kg moves with a speed of 12 ms–1 and the is 0.25, then the maximum fraction of the length of the chain
second part of mass 2 kg moves with speed 8 ms–1. If the that can hang over one edge of the table is
third part flies off with speed 4 ms–1 then its mass is
(a) 5 kg (b) 7 kg (a) 20% (b) 25%
(c) 17 kg (d) 3 kg (c) 35% (d) 15%
90. Two particles of masses m and M (M > m ) are connected by 96. A body of mass 5 kg explodes at rest into three fragments
a cord that passes over a massless, frictionless pulley. The with masses in the ratio 1 : 1 : 3. The fragments with equal
tension T in the string and the acceleration a of the particles masses fly in mutually perpendicular directions with speeds
is of 21 m/s. The velocity of heaviest fragment in m/s will be
2mM Mm
(a) T= g ;a = g (a) (b)
(M - m) (M + m) 7 2 5 2

2mM æ M-m ö (c) 3 2 (d) 2


(b) T= g; a = ç g
(M + m) è (M + m) ø÷ 97. Two bodies of masses m and 4m are moving with equal
kinetic energies. The ratio of their linear momenta will be
æ m-M ö æ Mm ö
(c) T=ç g;a = ç g (a) 1 : 4 (b) 4 : 1
÷ è (M + m) ø÷

.IN
è (M + m) ø
(c) 1 : 2 (d) 2 : 1
æ mM ö æ 2Mm ö
(d) T=ç g;a = ç g
è (M + m) ø÷ è (M + m) ø÷
Directions for Qs. (98 to 100) : Each question contains
AL
STATEMENT-1 and STATEMENT-2. Choose the correct answer
91. A bullet of mass m is fired from a gun of mass M. The
(ONLY ONE option is correct ) from the following-
recoiling gun compresses a spring of force constant k by a
N
distance d. Then the velocity of the bullet is (a) Statement -1 is false, Statement-2 is true
R

d (b) Statement -1 is true, Statement-2 is true; Statement -2 is a


(a) kd M / m (b) km correct explanation for Statement-1
M
U

d kM (c) Statement -1 is true, Statement-2 is true; Statement -2 is not


(c) (d)
JO

kM d a correct explanation for Statement-1


m m
92. A spring of force constant k is cut into two pieces whose (d) Statement -1 is true, Statement-2 is false
lengths are in the ratio 1 : 2. What is the force constant of 98. Statement -1 : The work done in bringing a body down from
U

the longer piece ? the top to the base along a frictionless incline plane is the
ED

k 3k same as the work done in bringing it down the vartical side.


(a) (b)
2 2 Statement -2 : The gravitational force on the body along
(c) 2 k (d) 3k the inclined plane is the same as that along the vertical side.
93. A motor cycle is going on an over bridge of 99. Statement -1 : On a rainy day, it is difficult to drive a car or
radius R. The driver maintains a constant speed. As the bus at high speed.
motor cycle is ascending on the overbridge, the normal force
Statement -2 : The value of coefficient of friction is lowered
on it
due to wetting of the surface.
(a) increases (b) decreases
(c) remains the same (d) fluctuates erratically 100. Statement -1 : The two bodies of masses M and m (M > m)
94. A body of mass M hits normally a rigid wall with velocity V are allowed to fall from the same height if the air resistance
and bounces back with the same velocity. The impulse for each be the same then both the bodies will reach the
experienced by the body is earth simultaneously.
(a) MV (b) 1.5 MV Statement -2 : For same air resistance, acceleration of both
(c) 2 MV (d) zero the bodies will be same.
Laws of Motion 125

Exemplar Questions 8. A body with mass 5 kg is acted upon by a force


r
1. A ball is travelling with uniform translatory motion. This F = (-3iˆ + 4 ˆj ) N . If its initial velocity at t = 0 is
means that v = (6iˆ - 12 ˆj ) ms -1 , the time at which it will just have a
(a) it is at rest velocity along the y-axis is
(b) the path can be a straight line or circular and the ball (a) never (b) 10 s
travels with uniform speed (c) 2 s (d) 15 s
(c) all parts of the ball have the same velocity (magnitude 9. A car of mass m starts from rest and acquires a velocity
r
and direction) and the velocity is constant along east, v = viˆ (v > 0) in two seconds. Assuming the
(d) the centre of the ball moves with constant velocity and car moves with uniform acceleration, the force exerted on
the car is
the ball spins about its centre uniformly
mv
2. A metre scale is moving with uniform velocity. This implies (a) eastward and is exerted by the car engine
2
(a) the force acting on the scale is zero, but a torque about
mv
(b) eastward and is due to the friction on the tyres

.IN
the centre of mass can act on the scale
2
(b) the force acting on the scale is zero and the torque
exerted by the road
acting about centre of mass of the scale is also zero
mv
(c) the total force acting on it need not be zero but the
AL
(c) more than eastward exerted due to the engine and
2
torque on it is zero
overcomes the friction of the road
(d) neither the force nor the torque need to be zero mv
N
3. A cricket ball of mass 150 g has an initial velocity (d) exerted by the engine
2
r r
R

u = (3iˆ + 4 ˆj )ms -1 and a final velocity v = -(3iˆ + 4 ˆj )ms -1 ,


NEET/AIPMT (2013-2017) Questions
after being hit. The change in momentum (final momentum-
U

initial momentum) is (in kgms–1) 10. Three blocks with masses m, 2 m and 3 m are connected by
strings as shown in the figure. After an upward force F is
JO

(a) zero -(0.45iˆ + 0.6 ˆj )


(b) applied on block m, the masses move upward at constant
(c) -(0.9 ˆj + 1.2 ˆj ) (d) -5(iˆ + ˆj )iˆ speed v. What is the net force on the block of mass 2m?
4. In the previous problem (3), the magnitude of the momentum (g is the acceleration due to gravity) [2013]
U

transferred during the hit is (a) 2 mg


ED

(a) zero (b) 0.75 kg-m s–1


(c) 1.5 kg-m s –1 (d) 1.4 kg-m s–1 (b) 3 mg
5. Conservation of momentum in a collision between particles
(c) 6 mg
can be understood from
(a) Conservation of energy (d) zero
(b) Newton's first law only
11. A car is moving in a circular horizontal track of radius 10 m
(c) Newton's second law only with a constant speed of 10 m/s. A bob is suspended from
(d) both Newton's second and third law the roof of the car by a light wire of length 1.0 m. The angle
6. A hockey player is moving northward and suddenly turns made by the wire with the vertical is [NEET Kar. 2013]
westward with the same speed to avoid an opponent. The p
force that acts on the player is (a) 0° (b)
3
(a) frictional force along westward p p
(b) muscle force along southward (c) (d)
6 4
(c) frictional force along sotuh-west 12. A balloon with mass ‘m’ is descending down with an
(d) muscle force along south-west acceleration ‘a’ (where a < g). How much mass should be
7. A body of mass 2 kg travels according to the law removed from it so that it starts moving up with an
acceleration ‘a’? [2014]
x(t ) = pt + qt 2 + rt 3 where, q = 4 ms–2, p = 3 ms–1 and r = 5 2ma 2ma
ms–3. The force acting on the body at t = 2s is (a) g + a (b) g - a
(a) 136 N (b) 134 N
ma ma
(c) 158 N (d) 68 N (c) (d)
g+a g-a
EBD_7179
126 PHYSICS

13. The force ‘F’ acting on a particle of mass ‘m’ is indicated by 17. Two stones of masses m and 2 m are whirled in horizontal
the force-time graph shown below. The change in momentum r
of the particle over the time interval from zero to 8 s is : circles, the heavier one in radius and the lighter one in
2
[2014] radius r. The tangential speed of lighter stone is n times that
of the value of heavier stone when they experience same
6 centripetal forces. The value of n is : [2015 RS]
3 (a) 3 (b) 4
(c) 1 (d) 2
0
F(N)

2 4 6 8 18. A plank with a box on it at one end is gradually raised about


–3 the other end. As the angle of inclination with the horizontal
reaches 30º the box starts to slip and slides 4.0 m down the
t(s) plank in 4.0s. The coefficients of static and kinetic friction
between the box and the plank will be, respectively :
(a) 24 Ns (b) 20 Ns
[2015 RS]
(c) 12 Ns (d) 6 Ns
14. A system consists of three masses m1, m2 and m3 connected
by a string passing over a pulley P. The mass m1 hangs
freely and m2 and m3 are on a rough horizontal table (the
coefficient of friction = m). The pulley is frictionless and of

.IN
mg
negligible mass. The downward acceleration of mass m1 is : q
(Assume m1 = m2 = m3 = m) [2014]
g(1 – gm) (a) 0.6 and 0.5 (b) 0.5 and 0.6
AL
(a) m2 m3 (c) 0.4 and 0.3 (d) 0.6 and 0.6
g P
19. What is the minimum velocity with which a body of mass m
2gm
N
(b) must enter a vertical loop of radius R so that it can complete
3 the loop ? [2016]
R

g(1 – 2m )
(c) (a) gR (b) 2gR
U

3 m1
g(1 – 2m )
JO

(d) (c) 3gR (d) 5gR


2
15. Three blocks A, B and C of masses 4 kg, 2 kg and 1 kg 20. One end of string of length l is connected to a particle of
mass 'm' and the other end is connected to a small peg on a
U

respectively, are in contact on a frictionless surface, as


shown. If a force of 14 N is applied on the 4 kg block then smooth horizontal table. If the particle moves in circle with
ED

the contact force between A and B is [2015] speed 'v' the net force on the particle (directed towards
centre) will be (T represents the tension in the string) : [2017]

A B mv2 mv2
C (a) T+ (b) T–
l l
(a) 6 N (b) 8 N
(c) Zero (d) T
(c) 18 N (d) 2 N
21. Two blocks A and B of masses 3 m and m respectively are
16. A block A of mass m1 rests on a horizontal table. A light connected by a massless and inextensible string. The whole
string connected to it passes over a frictionless pulley at system is suspended by a massless spring as shown in
the edge of table and from its other end another block B of figure. The magnitudes of acceleration of A and B
mass m2 is suspended. The coefficient of kinetic friction immediately after the string is cut, are respectively : [2017]
between the block and the table is µk. When the block A is
sliding on the table, the tension in the string is [2015] g
(a) ,g
(m 2 – mk m1 ) g m1m 2 (1 + m k )g 3
(a) (m1 + m 2 ) (b) (m1 + m 2 )
(b) g, g
m1m 2 (1 – m k )g (m 2 + m k m1 )g
(c) (d) g g
(m1 + m 2 ) (m1 + m 2 ) (c) ,
3 3 A 3m

g
(d) g, B m
3
Laws of Motion 127

Hints & Solutions


EXERCISE - 1 Let p = momentum after collision. Then,
1. (c) 2. (b) 3. (d) 4. (a) 5. (c) r r r r
p = p1 + p 2 or p = (mv) 2 + (mv) 2
6. (a) 7. (a)
v
8. (d) Let n be the mass per unit length of rope. Therefore, 2 mv ¢ = mv 2 or v¢ = m/sec
2
mass of rope = nL. 14. (c) Applying law of conservation of linear momentum
Acceleration in the rope due to force F will be m1 v v m
a = F/nL. m1v1 + m2v2 = 0, = - 2 or 1 = - 2
Mass of rope of length (L – l) will be n (L – l). m2 v1 v2 m1
Therefore, tension in the rope of length (L – l), is equal 15. (c) The equations of motion are
to pulling force on it 2 mg – T = 2ma
= n (L – l) a = n (L – l) × F/nL = F (1 – l/L) T– mg = ma Þ T = 4ma & a = g/3 so T = 4mg/3
r r If pulley is accelerated upwards with an accleration a,
9. (a) Impulse = change in momentum = m v 2 - m v1
then tension in string is
10. (a) Let a be the acceleration of mass m 2 in the downward
4m
direction. Then T= (g + a )

.IN
T – m2 (g/2) = m1 a ....(i) 3
and m2 g – T = m2 a ....(ii) 16. (b) Inertia is resistance to change.
17. (b) The time rate of change of momentum is force.
Adding eqs. (1) and (2), we get
AL
(m1 + m2) a = m2g – m2 (g/2) = m2 g/2 mv ' 2
18. (a) At the highest point of the track, N + mg =
m2 g r
\a=
N
2 (m 1 + m 2 )
11. (a) Let acceleration of lift = a and
R

mg
let reaction at spring balance = R
U

N
R where r is the radius of curvature at that point and v¢ is
JO

the speed of the block at that point.


mv ' 2
Now N = - mg
r
U

N will be maximum when r is minimum (v¢ is the same


ED

mg for all cases). Of the given tracks, (a) has the smallest
radius of curvature at the highest point.
Applying Newton’s law
19. (c) f = mW
R – mg = ma Þ R = m(g + a )
thus net weight increases, f = W tan q [\ m = tan q ]
So reading of spring balance increases. a
20. (a) Sn = (2n - 1)
12. (b) See fig. 2
mN a
S n +1 = ( 2n + 1)
2
ma N Sn 2n - 1
=
Sn +1 2n + 1
mg
21. (b) From the F.B.D.
If a = acceleration of the cart, then N = ma N = mg cos q
\ mN = mg or m ma = mg or a = g/m F = ma = mg sin q – mN
13. (c) p1 = mv northwards, p2 = mv eastwards
Þ a = g (sin q - m cos q)
N
v N mN
m
W E
v mg sin q m g cos q
xmg
m q
S
EBD_7179
128 PHYSICS

Now using, v 2 - u 2 = 2as 5. (b) As, (1/2)m v2 = Fs


or, v 2 = 2 ´ g (sin q - m cos q) l 1 1
So m (30 ) 2 = F ´ 4 and m (60 ) 2 = F ´ s
( l = length of incline) 2 2
or, v = 2gl (sin q - m cos q) \ s/4 = (60)2 / (30)2 = 4 or s = 4 × 4 = 16 m.
22. (a) Based on Newton’s third law of motion.
23. (a) If m1, m2 are masses and u1, u2 are velocity then by 6. (c) Acceleration, a = v - u = 0 - 6 = -2 ms -2
t 3
conservation of momentum m 1 u1 + m 2 u2 = 0 or
| m1u1 | = | m2 u 2 | Force = m×a = 4×2 = 8 N
24. (a) Here, on resolving force F2 and applying the concept change in momentum
of equilibrium 7. (a) Force required =
time taken
F2cos q (50 ´ 10-3 ´ 30) ´ 400 - (5 ´ 0)
F2sin q = = 10 N
60
F1 m 8. (a) Change in momentum = Force × time = Area which the
force-time curve encloses with time axis.
f N 9. (b) 10cm 20cm

.IN
mg 20N(F1) F F 32N(F2)
l1 l2
N = mg + F2 cos q , and f = µN AL L
\ f = m[mg + F2 cos q] … (i) It is clear F2 > F1, so rod moves in right direction with
an acceleration a, whereas a is given by
Also f = F1 + F2 sin q … (ii)
N
From (i) and (ii) (F2–F1)= mL×a................(i)
where m is mass of rod per unit length.
m[mg + F2 cos q] = F1 + F2 sin q
R

Now consider the motion of length l1 from first end,


F1 + F2 sin q
U

Þ m= then
mg + F2 cos q F– F1 = ml1a..................(ii)
JO

25. (a) The frictional force acting on M is µmg Dividing eq (ii) by (i), we get
mmg F - F1 l
\ Acceleration = = 1 or F = (F - F ) ´ l1 + F
U

M F2 - F1 L 2 1 1
L
EXERCISE - 2
ED

here l1 = 10 cm., L = 30 cm., F1 = 20 N, F2 = 32N


1. (b) Here u = 10 ms–1, v = –2 ms–1,
so F = 24 N
t = 4 s, a = ?
10. (b) Change in momentum = F × t
Using a = v - u = - 2 - 10 = - 3 m / s 2 = 10 × 10 = 100 Ns or 100 kg. m/s
t 4
\ Force, F = ma = 10×(–3) = –30 N 11. (d) See fig.
2. (b) 2 T cos 60º = mg T
or T = mg = 2×10 = 20 N. T
3. (c) If k is the spring factor, then P.E. of the spring 1 kg
T
compressed by distance x æç = kx 2 ö÷ will equal to gain
1
1 kg
è 2 ø
1 From figure, 1 g – T = 1 a ...(i)
2
in P.E. of the dart ( = mgh) i.e. kx = mgh and T = 1 a ....(ii)
2
1 From eqs. (i) and (ii), we get
\ k ( 4 ) 2 = 16 ´ g ´ 200 ....(i)
2 1g – 1a = 1a or 2a = g
1
and k (6) 2 = 16 ´ g ´ h ...(ii) \ a = (g/2) = (10/2) = 5 m/s2
2
On solving, (i) and (ii), we get h = 450 cm = 4.5 m. So, T = ma = 1 × 5 = 5 N
4. (d) Here m = 0.5 kg ; u = – 10 m/s ; 12. (b) R = mg – ma = 0.5 × 10 – 0.5 × 2 = 5 – 1 = 4 N
t = 1/50 s ; v = + 15 ms–1
Force = m (v– u)/t = 0.5 (10 + 15) × 50 = 625 N 13. (c) See fig. Let F be the force between the blocks and a their
common acceleration. Then for 2 kg block,
Laws of Motion 129

N
3N 2 kg
1 kg
F F A a
T
3–F=2a ...(1) 5 kg
for 1 kg block, F = 1 × a = a ....(2) mN
a
\ 3 – F = 2 F or 3 F = 3 or F = 1 newton T
14. (b) T = m (g + a )
5 kg 5 kg B
48000 = 4000(10 + a )
-2
Þ a = 2 ms
Dm for block B, 5g – T = 5a
15. (b) Initial thrust on the rocket = v rel
Dt Þ T = 36.75N, a = 2.45 m/sec2
= 500 × 1 = 500 N 24. (a) Force on the slab (m = 40 kg) = reaction of frictional
force on the upper block
Dm
where = rate of ejection of fuel.
Dt 100 N
10 kg
16. (b) T1 = m(g + a ) = 0.1(10 + 5) = 1.5N m k × 10 × g

.IN
T2 = m(g - a ) = 0.1(10 - 5) = 0.5N 40 kg

Þ T1 - T2 = (1.5 - 0.5) N = 1N \ 40a = mk × 10 × g or a = 0.98 m/sec2


AL
-1
25. (b) Considering the equilibrium of B
17. (d) Angle of friction = tan µ –mBg + T = mBa
18. (c) Displacement of the man on the trolley = 1 × 4 = 4m
N
Since the block A slides down with constant speed.
Now applying conservation of linear momentum a = 0.
R

Therefore T = mBg
1
80 × 1 + 400 v = 0 or v = - m/sec. Considering the equilibrium of A, we get
U

5 10a = 10g sin 30º – T – mN


The distance travelled by the trolley
JO

where N = 10g cos 30°


= – 0.2 × 4 = –0.8 m. a
N
(In opposite direction to the man.) T
Thus, the relative displacement of the man with the
U

A mN T a
ground = (4 – 0.8) = 3.2 m. 0º
si n3
ED

19. (c) In presence of friction a = (g sinq – mg cos q)


10g B
\ Time taken to slide down the plane 10g cos30º
mBg
2s 2s 10g
t1 = =
a g (sin q - m cos q )
10
\ 10 a = g - T - m ´ 10 g cos 30 º
2
2s
In absence of friction t 2 = but a = 0, T = mBg
g sin q
Given : t 1 = 2 t 2 0 .2 3
0 = 5g - m B g - × 10 ×g
2
2s 2s´ 4 Þ mB = 3.268 » 3.3 kg
\ t12 = 4t 22 or =
g (sin q - m cos q) g sin q
v1
sin q = 4 sinq – 4m cos q 26. (d) mv1 + 3mv2 = 0 or = -3
v2
3 3
m= tan q = = 0.75 (since q = 45°) 1
4 4 Now mv 12 = F. S1 = m. mg . S1
2
mv - (- mv ) 2mv 2 ´ 0.5 ´ 2
20. (a) F= = = = 2 × 103 N 1
(3m ) v 22 = F. S 2 = m. 3 mg . S 2
t t 10 -3 2
21. (b) T = m (g + a) = 100 (9.8 + 0) = 980 N
22. (c) T = m(g+a) = 100(9.8+1) = 1080N S1 v12 9
or = =
23. (c) For block A, T – mN = 5a and N = 5g S2 v 22 1
EBD_7179
130 PHYSICS

27. (b) m = 10 kg, x = (t3 – 2t – 10) m The total mass that must be set in motion is
2 15 kg + 10 kg = 25kg
dx d x
= n = 3t 2 - 2 = a = 6t Since Ftotal = m total a , a = Ftotal / mtotal
dt dt 2
= 49 N / 25 kg @ 2 m/s2
At the end of 4 seconds, a = 6 × 4 = 24 m/s2
36. (a) Momentum is always conserved. Since the skater and
F = ma = 10 × 24 = 240 N
snowball are initially at rest, the initial momentum is
28. (a) F3 zero. Therefore, the final momentum after the toss must
also be zero.
Pskater + Psnowball = 0
F2
or m skater v skater + m snowball v snowball = 0
m
F1 v skater = -m snowball vsnowball / m skater

The formula for force is given by F1 = ma = -(0.15kg)(35m / s) =


- 0.10m / s
F (50kg)
Acceleration of the particle a = 1 ,
m The negative sign indicates that the momenta of the
because F1 is equal to the vector sum of F2 & F3. skater and the snowball are in opposite directions.
29. (c) 37. (b) Apply Newton’s second law
FA = FAB, therefore :

.IN
P mA aA = (mA + mB)aAB and aAB = aA / 5
Therefore : mA aA = (mA + mB)aA/5 which reduces to
C T
AL 4 mA = mB or 1 : 4
38. (d) Work is the product of force and distance. The easiest
a way to calculate the work in this pulley problem is to
mg
multiply the net force or the weight mg by the distance
N
mg - T = ma it is raised: 4 kg x 10 m/s2 x 5 m = 200 J.
39. (d) Given : Mass of rocket (m) = 5000 Kg
R

60 ´ 10 - 360 Exhaust speed (v) = 800 m/s


=a
60 Acceleration of rocket (a) = 20 m/s2
U

a = 4 ms - 2 Gravitational acceleration (g) = 10 m/s2


JO

We know that upward force


82 + (-6)2 + (-10)2 F = m (g + a) = 5000 (10 +20)
30. (b) m= = 10 2kg
1 = 5000 × 30 = 150000 N.
We also know that amount of gas ejected
U

4
31. (b) Mass of over hanging chain m’ = ´ (0.6) kg
2 æ dm ö F 150000
ED

ç ÷= = = 187.5 kg / s
Let at the surface PE = 0 è dt ø v 800
C.M. of hanging part = 0.3 m below the table
4 40. (d) Given F = 600 – 2 ´ 105 t
U i = - m ¢gx = - ´ 0.6 ´ 10 ´ 0.30 The force is zero at time t, given by
2
DU = m 'gx = 3.6J = Work done in putting the entire 0 = 600 – 2 ´ 105 t
chain on the table 600
Þ t= = 3 ´ 10 – 3 seconds
32. (a) Weight of body = m g = 5 N 2 ´ 105
33. (d) Here, the force of friction is 400N.
t 3´10 –3
Fnet = (1650 - 400) = 1250N \ Impulse = ò Fdt = ò (600 – 2 ´ 105 t) dt
0 0
1250
\ a= = 1.25ms - 2 3´10 –3
1000 é 2 ´ 105 t 2 ù
= ê 600t – ú
dm mg 5000 ´ 9.8 = 49 kg s -1 êë 2 úû
34. (a) = = 0
dt vr 1000
35. (a) Two external forces, FA and FB, act on the system and = 600 ´ 3 ´ 10 –3 – 105 (3 ´10 –3 ) 2
move in opposite direction. Let’s arbitrarily assume that
the downward direction is positive and that FA provides = 1.8 – 0.9 = 0.9Ns
downward motion while FB provides upward motion. 41. (b) According to law of conservation of momentum,
FA = (+15 kg) (9.8 m/s2) = 147 N 10
and FB = (–10 kg)(9.8 m/s2) = – 98 N 100v = - ´ 10 ´ 800
1000
Ftotal = FA + FB = 147 N + (–98 N) = 49 N ie, v = 0.8 ms–1.
Laws of Motion 131

3 52. (b) Reading of spring balance


42. (d) sin q =
5 4m1m 2 4 ´ 5 ´ 1 10
2T = = = kgf
5 m1 + m 2 6 3
3
53. (a) a = 2v (given)
q
4 dv
3 3 Þ v = 2v
\ tan q = Þ m = tan q = = 0.75 ds
4 4
or dv = 2ds
43. (b) The condition to avoid skidding, v = mrg v

ò dv = 2[s]0
0.1
= 0.6 ´ 150 ´ 10 = 30 m/s. = 0.2
44. (d) Limiting friction = 0.5 ´ 2 ´10 = 10N 0.1
The applied force is less than force of friction, therefore v - 0.1 = 0.2
the force of friction is equal to the applied force. Þ v = 0.3ms -1
45. (d) Applying law of conservation of momentum 54. (c) If W is the maximum weight, then
Momentum of bullet = Momentum of sand-bullet W = 2T cos 60°
system or W = T = 20N
m æ mö 21 15 5
v = çm + ÷V = mV

.IN
55. (c) The acceleration of both the blocks = =
20 è 20 ø 20 3x x
46. (d) Here tan q = 0.8 5
\ Force on B = ´ 2x = 10 N
x
where q is angle of repose
AL
56. (b) The maximum acceleration that can be given is a
q = tan -1 (0.8) = 39°
\ 30g = 10g + 10a
N
The given angle of inclination is equal to the angle of
repose. So the 1 kg block has no tendency to move. Þ a = 2g = 20ms -2
R

\ mg sin q = force of friction 1 2


We know that s = ut + at
U

Þ T=0 2
47. (d) For 0.5 kg block, 6 = 0.5 a
JO

2s 2 ´ 10
48. (b) While moving down, when the lift is accelerating the \t= = =1s
weight will be less and when the lift is decelerating the a 20
weight will be more. 57. (a) Let the air resistance be F. Then
U

mg + F = ma Þ F = m[a - g]
49. (b) Total momentum = 2p î + pˆj
ED

Magnitude of total momentum 30


Here a = = 12ms - 2
2. 5
= ( 2 p) 2 + p 2 = 5p 2 = 5p
m( v - u ) 0.15(0 - 20)
This must be equal to the momentum of the third part. 58. (c) F= = = 30 N
t 0. 1
59. (c) Considering the two masses and the rope a system,
°
30 then
50. (c) cos
ma Initial net force = [25 - (15 + 5)] g = 5g
30° 60°
° Final net force = éë( 25 + 5 ) - 15 ùû g = 15 g
30 ma (pseudo force)
sin
mg Þ (acceleration)final = 3 (acceleration)initial
M a 60. (d)
30° 90°
q
os
Fc
ma cos 30° = mg sin 30° N q
F
g
\a= F sin q
3 q
sin mg cos q
51. (d) Apparent weight when mass is falling down is given mg mg
by W ' = m(g - a)
\ W ' = 1´ (10 - 10) = 0 From figure N = mg cos q + F sin q
EBD_7179
132 PHYSICS

61. (b) m mg = m v 2 / r or v = m g r

or v = ( 0 .25 ´ 9 . 8 ´ 20 ) = 7 m / s
62. (a) Since water does not fall down, therefore the velocity
of revolution should be just sufficient to provide a
T T a
centripetal acceleration at the top of vertical circle. So,
v = (g r ) = {10 ´ (1.6)} = (16) = 4 m/sec.

63. (d) The speed at the highest point must be v ³ rg mg 2mg


Now v = rw = r (2p / T ) (a)

T – mg = ma … (1)
æ ö
\ r ( 2 p / T ) > rg or T < 2pr < 2p çç r ÷÷ and 2mg – T = 2ma … (2)
rg ègø
Adding (1) and (2), we get
\ T = 2p æç 4 ö÷ = 4 sec mg = 3ma
è 9.8 ø
g
64. (a) From figure, \a=
3

.IN
mv2 For fig (b),
N sin q = ....... (i)
r AL
N cos q = mg ...... (ii)
Dividing, we get
N
v2 v2
tan q = or q = tan -1
rg a¢ T¢
R

rg T¢ a¢
65. (a) In the case of a body describing a vertical circle,
U

A
F = 2mg
JO

mg
(b)
O D
C
U

q T T '- mg = ma ' … (3)


ED

and 2mg - T ' = 0 … (4)


q Mg cos q
B Solving (3) and (4)
Mg sin q Mg a'=g
mn 2 mn 2 g
T - mg cos q = T = mg cos q + \ a= and a ' = g
l l 3
Tension is maximum when cos q = +1 and velocity is
maximum 69. (c) Linear momentum, P = a + bt 2
Both conditions are satisfied at q = 0º (i.e. at lowest dP
point B) = 2bt (on differentiation)
dt
66. (b) Since surface (ice) is frictionless, so the centripetal
force required for skating will be provided by inclination dP
\ Rate of change of momentum, µt
of boy with the vertical and that angle is given as dt

v2 dP
tan θ = By 2nd law of motion, µF
where v is speed of skating & r is radius dt
rg
\ Fµ t
of circle in which he moves. 70. (d) For equilibrium of all 3 masses,
67. (d) v max = mgr T3 = (m1 + m2 + m3)a or

68. (b) Let a and a' be the accelerations in both cases T3


a=
respectively. Then for fig (a), m1 + m 2 + m3
Laws of Motion 133

For equilibrium of m1 & m2 74. (d) Let v1 = velocity when height of free fall is h 1
v2 = velocity when height of free rise is h2
T2 = (m1 + m 2 ).a
\ v12 = u 2 + 2gh1 for free fall
(m1 + m 2 )T3
or, T2 = or
m1 + m 2 + m3 For free rise after impact on ground
Given m1 = 10 kg, m2 = 6 kg, m3 = 4 kg, T3 = 40 N 0 = v22 - 2gh 2 or v 22 = 2gh 2
(10 + 6).40 Initial momentum = mv1
\ T2 = = 32N Final momentum = mv2
10 + 6 + 4
\ Change in momentum = m(v1 – v2)
71. (a) Velocity of ball after dropping it from a height of
5m = m( 2gh 2 - gh 2 )
75. (b) T = tension is the string
\ Applied force F = 2T
10 m/sec 20 m/sec T = F/2 … (i)

(using v2 = u2 + 2gh) T
v2 = 0 + 2 × 10 × 5 Þ v = 10 m/s
T T F
Velocity gained by ball by force exerted by bat f m

.IN
0 = u2 – 2gh
u2 = 2 × 10 × 20 or u = 20 m/s For block of mass m, force of friction due to surface f.
For sliding the block
Change in momentum = m(u + v) AL T – f = force on the block = mass × acceleration
= 0.4 (20 + 10) = 12 kg m/s
T -f
DP DP or acceleration of block = . Put T from (i)
F= or Dt = m
N
Dt F
F
-f
R

Dt =
12
= 0.12 sec \ Acceleration = 2
100 m
U

72. (d) F1 = Force of friction between B and A g sin q - mg cos q


76. (b)
JO

= m1m1g d q
n
= 0.25 × 100 × g = 25 g newton g si d
F2 = Force of friction between (A + B) and surface 45° 45°
U

smooth rough
= m2 m 2 g = m2 (mass of A and B) g
ED

When surface is When surface is


1 300 smooth rough
= (100 + 200)g = g = 100g newton
3 3 1 1 2
d= (g sin q)t 12 , d = (g sin q - mg cos q) t 2
\ F = F1 + F2 2 2
= 25 g + 100 g = 25g = 125 × 10 N 2d 2d
t1 = , t2 =
\ F = 1250 N g sin q g sin q - mg cos q
73. (b) The rain drops falling vertically with velocity u do not
According to question, t 2 = nt1
affect the momentum along the horizontal track. A
vector has no component in a perpendicular direction 2d 2d
Rain drops add to the mass of the car n =
g sin q g sin q - mg cos q
Mass added in t sec = (mt) kg
Momentum is conserved along horizontal track. m , applicable here, is coefficient of kinetic friction as
Initial mass of car = M the block moves over the inclined plane.
Initial velocity of car = v0 1 æ 1 ö
Final velocity of (car + water) = v n= ççQ cos 45° = sin 45° = ÷÷
1- mk è 2ø
Mass of (car + water) after time t = (M + mt)
\ final momentum = initial momentum 1 1
n2 = or 1 - mk =
(M + mt)v = Mv0 1- mk n2

Mv 0 1
\ v= or m k = 1-
(M + mt) n2
EBD_7179
134 PHYSICS

77. (d) Acceleration of block while sliding down upper half = r


84. (c) F = 6 ˆi – 8 ˆj+10 k,
ˆ

( )
g sin f ;
| F |= 36 + 64 + 100 = 10 2 N Q F = Fx2 + Fy2 + Fz2
retardation of block while sliding down lower half = –
(g sin f - mg cos f) a = 1 ms–2
Q F = ma
For the block to come to rest at the bottom, acceleration
in I half = retardation in II half. 10 2
\ m= = 10 2 kg
g sin f = -(g sin f - mg cos f) 1
85. (d) Frictional force on the box f = mmg
Þ m = 2 tan f \ Acceleration in the box
Alternative method : According to work-energy a = mg = 5 ms–2
theorem, W = DK = 0 v2 = u2 + 2as
(Since initial and final speeds are zero) Þ 0 = 22 + 2 × (5) s
\ Work done by friction + Work done by gravity 2
=0 Þ s=– w.r.t. belt
5
Þ distance = 0.4 m
l
i.e., -(µ mg cos f) + mgl sin f = 0
2 86. (c) a=1
µ
or cos f = sin f or µ = 2 tan f
2

.IN
78. (c) Mass (m) = 0.3 kg Þ F = m.a = – 15 x m = 1000 kg
15 -150
a=– x= x = - 50 x
0.3 3
AL
a = – 50 × 0.2 = 10 m / s 2 Total mass = (60 + 940) kg = 1000 kg
N
Let T be the tension in the supporting cable, then
79. (c) From free body diagram,
T – 1000g = 1000 × 1
R

ma
g cos Þ T = 1000 × 11 = 11000 N
N 87. (b)
U

a a
a
JO

a
mg cosa
+ ma sina mg mg sin a S/2 h
oot S/2 sin q
For block to remain stationary, Sm
U

q
mg sin a = ma cos a Þ a = g tan a S/2
h
oug
ED

S/2 sin q
q R
80. (a) v 2 - u 2 = 2as or 02 - u 2 = 2(-mk g) s
1 For upper half of inclined plane
- 100 2 = 2 ´ - ´10 ´ s v2 = u2 + 2a S/2 = 2 (g sin q) S/2 = gS sin q
2
s = 1000 m For lower half of inclined plane
81. (b) This is a standard formula and should be memorized. 0 = u2 + 2 g (sin q – m cos q) S/2
Þ – gS sin q = gS ( sinq – m cos q)
g sin q Þ 2 sin q = m cos q
a=
I 2 sin q
1+ Þ m= = 2 tan q
MR 2 cos q
82. (a) N = m a sin q + mg cos q ......(1)
88. (b) v = gr = 10 ´ 40 = 20 m s -1
also m g sin q = m a cos q ......(2)
from (2) a = g tan q 89. (a)
m y
2 ac
sin q os
\ N = mg + mg cos q , q 2 kg m2
cos q N
ma 8 m/sec Presultant
mg q
or N = cos q
mg m 12 m/sec
cos q o sq g
ma
c mg sin m1
83. (a) mBg = ms mAg {Q mAg = ms mAg} q c x
/se
Þ mB = ms mA 4m 1 kg
or mB = 0.2 × 2 = 0.4 kg 3
m
Laws of Motion 135

94. (c) Impulse experienced by the body


Presultant = 122 + 162
= change in momentum = MV – (–MV) = 2MV.
= 144 + 256 = 20 95. (a) The force of friction on the chain lying on the table
m3v3 = 20 (momentum of third part) should be equal to the weight of the hanging chain.
20 Let
or, m3 = = 5 kg r = mass per unit length of the chain
4
90. (b) Mg – T = Ma µ = coefficient of friction
T – mg = ma l = length of the total chain
x = length of hanging chain
On solving, we get Now, µ(l – x) rg = xrg or µ(l – x) = x
or µl = (µ + 1)x or x = µl/(µ + 1)
(M - m)g
a= T T 0.25l 0.25l
M+m m \ x= = = 0.2l
M (0.25 + 1) 1.25
2Mmg mg
and T = mg x
M+m = 0.2 = 20%
l
91. (c) Let velocity of bullet be v. If velocity of gun is V, then
mv + MV = 0
96. (a) Masses of the pieces are 1, 1, 3 kg. Hence
mv (1 ´ 21)2 + (1 ´ 21)2 = (3 ´ V ) 2
Þ V= -

.IN
M That is, V = 7 2 m/s
As spring compresses by d, so
1 2 1 K1 p2 m 2
kd = MV 2 = 1 ´ 22 [Q p = mv Þ K = p ]
97.
AL
(c)
2 2 K 2 m1 p2 2m
2
1 2 1 æ mv ö p1 M1 1 1
kd = M ç
N
or ÷
2 2 è Mø Hence, p = M2
= =
4 2
2
R

d
Þ v= kM 98. (d) Work done in moving an object against gravitational
m force depends only on the initial and final position of
U

92. (b) Here, l2 = 2l1 the object, not upon the path taken. But gravitational
force on the body along the inclined plane is not same
JO

1
As for a spring, force constant k µ as that along the vertical and it varies with angle of
l
inclination.
1 1 1 99. (b) On a rainy day, the roads are wet. Wetting of roads
U

\ k1 µ l , k 2 µ l , k µ l + l lowers the coefficient of friction between the types


1 2 1 2
and the road. Therefore, grip on a road of car reduces
ED

k1 l1 + l2 k2 l1 + l2 and thus chances of skidding increases.


= and = 100. (a) The force acting on the body of mass M are its weight
k l1 k l2
Mg acting vertically downward and air resistance F
é l2 ù é l1 ù acting vertically upward.
or k1 = k ê1 + ú and k2 = k ê1 + ú
ë l1 û ë l2 û F
\ Acceration of the body , a = g -
\ k1 = k [1 + 2] = 3k [using (i)] M
Now M > m, therefore, the body with larger mass will
3
k2 = k êé1 + úù = k [using (i)]
1
have great acceleration and it will reach the ground
ë 2û 2 first.
mV 2 EXERCISE - 3
93. (a) R = mg cos q –
r Exemplar Questions
R 1. (c) In a uniform translatory motion if all the parts of the
V
car body moves with (same velocity in same straight line,
q mg cos q so the velocity is constant.
mg q v
A v
O Over bridge v
When q decreases, cosq increases i.e. R increases The situation is shown in (figure) where a body A is in
unfirom translatory motion.
EBD_7179
136 PHYSICS

2. (b) According to question we have to apply Newton's r r r


So, F12 = - F21 (Q Fext = 0)
second law of motion, in terms of force and change in
momentum. Total force on the system will be zero.
r r
= - 21 or dpr12 = -dpr 21
dp dp12 dp
We know that F = dt dt
dt
r r
As given that the meter scale is moving with uniform Þ (dp12 + dp21 = 0)
velocity, hence So prove the law of conservation of momentum and
Force (F) = m × 0 = 0 verifies the option (d).
No change in its velocity i.e., acceleration of it zero by 6. (c) Consider the adjacent diagram.
Newton's second law.
Hence, net or resultant force must act on body is zero. N
r r r so,
t = r ´ F,
As all part of the scale is moving with uniform velocity
P2
and total force is zero, hence, torque will also be zero. B A
3. (c) As given that,
P1
Mass of the ball = 150 g = 0.15 kg
( P2– P1) O
r

.IN
B
u = (3iˆ + 4 ˆj ) m/s W E
r
v = -(3iˆ + 4 ˆj ) m/s AL
(Dp) Change in momentum S
= Final momentum – Initial momentum The force on player is due to rate of change of
r r
N
= mv - mu momentum. The direction of force acting on player will
r r be the same as the direction of change in momentum.
R

= m(v - u ) = (0.15)[ -(3iˆ + 4 ˆj ) - (3iˆ + 4 ˆj )] Let OA = P1 i.e., Initial momentum of player northward
AB = P2 i.e., Final momentum of player towards west.
U

= (0.15)[-6iˆ - 8 ˆj ] Clearly, OB = OA + OB
JO

Change in momentum
= -[0.15 ´ 6iˆ + 0.15 ´ 8 ˆj] = P2 – P1
= AB – OA = AB + (–OA)
= -[0.9iˆ + 1.20 ˆj ]
U

= Clearly resultant AR will be along south-west.


Dp = -[0.9iˆ + 1.2 ˆj ] So, it will be also the direction of force on player.
ED

7. (a) As given that, mass = 2 kg


Hence verifies option (c). p = 3 m/s, q = 4 m/s2, r = 5 m/s3
4. (c) From previous solution As given equation is
Dp = -(0.9iˆ + 1.2 ˆj) = -0.9iˆ - 1.2 ˆj x(t ) = pt + qt 2 + rt 3
2
Magnitude of | Dp |= (-0.9) + (-1.2)
2 ds (t )
v= = p + 2qt + 3rt 2
dt
= 0.81 + 1.44
dv d 2 x (t )
= 2.25 = 1.5 kg-m s–1 a= = = 0 + 2q + 6rt
Verifies the option (c). dt dt 2
5. (d) By Newton's second law : é d 2 x (t ) ù
r ê 2 ú
= 2q + 12r
dp ë dt û ( t = 2)
Fext =
dt
r = 2q + 12r
As Fext in law of conservation of momentum is zero.
= 2 × 4 + 12 × 5
If Fext = 0, dp = 0 Þ p = constant = 8 + 60 = 68 m/s2
Hence, momentum of a system will remain conserve if r
Force acting on body ( F ) = ma
external force on the system is zero.
In case of collision between particle equal and opposite = 2 × 68 = 136 N.
forces will act on individual particle by Newton's third 8. (b) As given that mass = m = 5 kg
r
law. Acting force = F = (-3iˆ + 4 ˆj ) N
Laws of Motion 137
r Fnet on block of mass 2 m
Initial velocity at t = 0, u = (6iˆ - 12 ˆj ) m/s
r = T – T' – 2 mg = 0
F æ 3iˆ 4 ˆj ö
Retardation, aˆ = = ç - + ÷ m/s 2 ALTERNATE :
m è 5 5 ø
Q v = constant
As the final velocity along Y-component only. So its x-
component must be zero. so, a = 0, Hence, Fnet = ma = 0
11. (d) Given; speed = 10 m/s; radius r = 10 m
From v x = u x + ax t , for X-component only,,
Angle made by the wire with the vertical
-3iˆ
0 = 6iˆ + t v2 102
5 tan q = = =1
rg 10 ´ 10
3iˆ p
(t ) = 6iˆ
5 Þ q = 45° =
4
5´ 6 12. (a) Let upthrust of air be Fa then
t= = 10 s
3 For downward motion of balloon
t = 10 sec. Hence verifies the option (b). Fa = mg – ma
9. (b) As given that mass of the car = m mg – Fa = ma
As car starts from rest, u = 0
r For upward motion

.IN
Velocity acquired along east (v ) = viˆ Fa – (m – Dm)g = (m – Dm)a
Time (t) = 2 s.
We know that, v = u + at 2ma
Therefore Dm =
AL g+a
Þ viˆ = 0 + a ´ 2
13. (c) Change in momentum,
r v
Þ a = iˆ
N
2 Dp = ò Fdt
R

(Force, by engine is internal force)


= Area of F-t graph
r r mv ˆ
U

F = ma = i = ar of D – ar of + ar of
2
JO

1
mv = ´ 2 ´ 6 - 3 ´ 2 + 4 ´ 3 = 12 N-s
Hence, force acting on the car is towards east due 2
2
14. (c) Acceleration
U

mv ˆ
to force of friction is i which moves the car in Net force in the direction of motion
2 =
ED

eastward direction. Hence, force by engine is internal Total mass of system


force.
m1g - m(m 2 + m3 )g g
NEET/AIPMT (2013-2017) Questions = = (1 - 2m )
m1 + m 2 + m3 3
(Q m1 = m2 = m3 = m given)
F T T' F
m v
15. (a) Acceleration of system a = net
mg M total
mg m 2m 3m 14 14 2
2m = = = 2m/s
10. (d) T T' T" 4 + 2 +1 7
2mg
mg 2mg 3mg
3m 14N A B C
4kg 2kg 1kg
6 mg
From figure The contact force between A and B
F = 6 mg, = (mB + mC) × a = (2 + 1) × 2 = 6N
As speed is constant, acceleration a = 0 16. (b) For the motion of both the blocks
\ 6 mg = 6ma = 0, F = 6 mg m1a = T – mkm1g
\ T = 5 mg , T¢ = 3 mg m2g – T = m2a
T² = 0
EBD_7179
138 PHYSICS
a 20. (d) Net force on particle in uniform circular motion is cen-
T
mk m1g m1
æ mv2 ö
mk tripetal force çç l ÷÷ which is provided by tension in
è ø
string so the net force will be equal to tension i.e., T.
m 2g – m k m1g m2
a= a
m1 + m 2

æ m 2 g – m k m1g ö m2g
m2g – T = (m2) ç m + m ÷
è 1 2 ø 21. (a) m
solving we get tension in the string
m1m 2 g (1 + m k ) g
T=
m1 + m 2 mg
17. (d) According to question, two stones experience same
Before cutting the string
centripetal force
kx = T + 3 mg ...(i)
i.e. FC1 = FC2 T = mg ...(ii)
Þ

.IN
kx = 4mg
mv12 2mv 22 After cutting the string T = 0
or, = or, V12 = 4V22
r (r / 2)
4mg - 3mg
So, V1 = 2V2 i.e., n = 2
AL aA =
3m
18. (a) Coefficient of static friction,
1
N
4mg
ms = tan 30° = = 0.577 @ 0.6
3
R

1 2
S = ut + at
U

2
JO

1 1
4= a(4)2 Þ a = = 0.5
2 2
[Q s = 4m and t = 4s given]
U

a = gsinq – mk(g) cosq


ED

0.9 g
Þ mk = = 0.5 aA =
3 3
19. (d) To complete the loop a body must enter a vertical loop
mg
of radius R with the minimum velocity v = 5gR . and a B = =g
m
Work, Energy
6 and Power
WORK The area under each segment of the curve is approximately equal
Work Done by a Constant Force to the area of a rectangle. The height of the rectangle is a constant
ur value of force, and its width is a small displacement Dx. Thus, the
Work done (W) by a force F in displacing a body through a step involves an amount of work DWn = Fn Dxn. The total work
displacement x is given by done is approximately given by the sum of the areas of the
rr

.IN
rectangles.
W = F.x = Fx cos q
i.e., W » å Fn Dxn.
Fsinq AL
F

q
Body
N
x
F cos q
R

r
Where q is the angle between the applied force F and displacement
U

r
x.
As the size of the steps is reduced, the tops of the rectangle more
JO

The S.I. unit of work is joule, CGS unit is erg and its dimensions closely trace the actual curve shown in figure. If the limit Dx ® 0,
are [ML2T–2]. which is equivalent to letting the number of steps tend to infinity,
1 joule = 107 erg the discrete sum is replaced by a continuous integral.
U

(a) When q = 0° then W = Fx


W = lim å Fn Dxn = ò Fx dx
ED

(b) When q is between 0 and p/2 then Dxn ®0


W = Fx cos q = positive Thus, the work done by a force Fx from an initial point A to final
(c) When q = p/2 then W = Fx cos 90° = 0 (zero) xn
Work done by centripetal force is zero as in this case angle
q = 90°
point B is WA® B = ò Fx dx
xA
(d) \ When q is between p / 2 and p then The work done by a variable force in displacing a particle from x 1
W = Fx cos q = negative to x2
Work Done by a Variable Force x2

When the force is an arbitrary function of position, we need the


W= ò Fdx = area under force displacement graph
x1
techniques of calculus to evaluate the work done by it. The figure
shows Fx as function of the position x. We begin by replacing CAUTION : When we find work, we should be cautious about
the actual variation of the force by a series of small steps. the question, work done by which force? Let us take an example
to understand this point. Suppose you are moving a body up
without acceleration.
Fapplied
Work done by applied force
uur uur
Wapp = F app · x = Fapp x

Work done by gravitational force mg


x
uuur uur uur
W grav = Fg x = - mgx
EBD_7179
140 PHYSICS

ENERGY Kinetic Energy


It is the capacity of doing work. Its units and dimensions are The energy possessed by a body by virtue of its motion is called
same as that of work. kinetic energy.
Potential Energy The kinetic energy Ek is given by
Ek = ½ mv2 ...(i)
The energy possessed by a body by virtue of its position or Where m is mass of body, which is moving with velocity v. We
configuration is called potential energy. Potential energy is know that linear momentum (p) of a body which is moving with a
defined only for conservative forces. It does not exist for non velocity v is given by
conservative forces. p = mv ...(ii)
(a) Elastic potential energy (Potential energy of a spring) : So from eqs. (i) and (ii), we have
Let us consider a spring, its one end is attached to a rigid
p2
wall and other is fixed to a mass m. We apply an external Ek = ...(iii)
r 2m
force Fext . on mass m in the left direction, so that the spring This is the relation between momentum and kinetic energy.
is compressed by a distance x. The graph between E k and p is a straight line

Q p = 2m E k
x
m
Fext

.IN
Ek
l
If spring constant is k, then energy stored in spring is given
AL
by
P.E. of compressed spring = ½kx2
p
N
Now if the external force is removed, the mass m is free to
1
move then due to the stored energy in the spring, it starts The graph between E k and p is a rectangular hyperbola
R

oscillating
(b) Gravitational potential energy : When a body is raised to
U

some height, above the ground, it acquires some potential Ek


energy, due to its position. The potential energy due to
JO

height is called gravitational potential energy. Let us


consider a ball B, which is raised by a height h from the
ground.
U

1
ED

Fapp p
B
h 1
mg The graph between Ek and is a rectangular hyperbola
m

Ground p is constant
Ek
In doing so, we do work against gravity and this work is
stored in the ball B in the form of gravitational potential
energy and is given by
W = Fapp. h = mgh = gravitational potential energy ...(i) 1
Further if ball B has gravitational P.E. (potential energy) Uo m
at ground and at height h, Uh, then Keep in Memory
Uh–Uo =mgh ...(ii)
1. Work done by the conservative force in moving a body in
If we choose Uo= 0 at ground (called reference point) then a closed loop is zero.
absolute gravitational P.E of ball at height h is
Work done by the non-conservative force in moving a
Uh = mgh ...(iii) body in a closed loop is non-zero.
In general, if two bodies of masses m1 and m2 are separated 2. If the momenta of two bodies are equal then the kinetic
by a distance r, then the gravitational potential energy is energy of lighter body will be more.
m1m2 Q p1 = p 2 or 2 m1 E 1 = 2 m 2 E 2
U = -G
r
Work, Energy and Power 141

E1 m 2
Keep in Memory
\ =
E 2 m1 1. Work done against friction on horizontal surface
3. If the kinetic energies of two bodies are same then the = m mgx and work done against force of friction on inclined
momentum of heavier body will be more. plane = (mmg cosq) x where m = coefficient of friction.
2. If a body moving with velocity v comes to rest after
p1 m1 covering a distance ‘x’ on a rough surface having
Q E1 = E 2 \
p2
=
m2 coefficient of friction m, then (from work energy theorem),
ur ur
2m gx = v2. Here retardation is a = -mg
WORK-ENERGY THEOREM
Let a number of forces acting on a body of mass m have a resultant 3. Work done by a centripetal force is always zero.
r 4. Potential energy of a system decreases when a
force Fext. And by acting over a displacement x conservative force does work on it.
r r 5. If the speed of a vehicle is increased by n times, then its
(in the direction of Fext. ), Fext. does work on the body, and there
stopping distance becomes n2 times and if momentum is
by changing its velocity from u (initial velocity) to v (final velocity). increased by n times then its kinetic energy increases by
Kinetic energy of the body changes. n2 times.
So, work done by force on the body is equal to the change in
6. Stopping distance of the vehicle = Kinetic energy

.IN
kinetic energy of the body.
Stopping force
W = ½ mv 2 - ½mu 2 7. Two vehicles of masses M1 and M2 are moving with
velocities u1 and u2 respectively. When they are stopped
AL
This expression is called Work energy (W.E.) theorem. by the same force, their stopping distance are in the ratio
as follows :
LAW OF CONSERVATION OF MECHANICAL ENERGY Since the retarding force F is same in stopping both the
N
The sum of the potential energy and the kinetic energy is called vehicles. Let x1 and x2 are the stopping distances of vehicles
R

the total mechanical energy. of masses M1 & M2 respectively, then


The total mechanical energy of a system remains constant if F.x1 (work done in stopping the mass M1 )
U

only conservative forces are acting on a system of particles and F.x 2 (work done in stopping the mass M 2 )
JO

the work done by all other forces is zero.


½M1u12 E k1
i.e., DK + DU = 0 = = ....(i)
½M 2u 22 E k2
or Kf – Ki + Uf – Ui = 0
U

where u1 and u2 are initial velocity of mass M1 & M2


or Kf + Uf = Ki + Ui = constant
respectively & final velocity of both mass is zero.
ED

VARIOUS FORMS OF ENERGY : THE LAW OF


CONSERVATION OF ENERGY x 1 E k1 ....(ii)
Þ =
Energy is of many types – mechanical energy, sound energy, x 2 E k2
heat energy, light energy, chemical energy, atomic energy, nuclear Let us apply a retarding force F on M1 & M2, a1 & a2 are
energy etc. the decelerations of M1 & M2 respectively. Then from third
In many processes that occur in nature energy may be transformed
from one form to other. Mass can also be transformed into energy (
equation of motion v 2 = u 2 + 2ax : )
and vice-versa. This is according to Einstein’s mass-energy
u12
equivalence relation, E = mc2. 0 = u12 - 2a1x1 Þ a 1 = ....(iii a)
2x1
In dynamics, we are mainly concerned with purely mechanical
energy.
u 22
and 0 = u 22 - 2a 2 x 2 Þ a 2 = ....(iii b )
Law of Conservation of Energy : 2x 2
The study of the various forms of energy and of transformation If t1 & t2 are the stopping time of vehicles of masses
of one kind of energy into another has led to the statement of a M1 & M2 respectively, then from first equation of motion
very important principle, known as the law of conservation of
energy. (v = u+at) 0 = u1 - a 1t1 Þ t1 = u1 ....(iv a)
a1
"Energy cannot be created or destroyed, it may only be
transformed from one form into another. As such the total amount u2
and 0 = u 2 - a 2 t 2 Þ t 2 = ....(iv b)
of energy never changes". a2
EBD_7179
142 PHYSICS

Then by rearranging equation (i), (iii) & (iv), we get (a) If a spring is divided into n equal parts, the spring
constant of each part = nK.
t1 u /a æ x ö æu ö
= 1 1 = ç 1 ÷´ç 2 ÷ (b) If spring of spring constant K1, K2, K3 .......... are
t 2 u 2 / a 2 çè x 2 ÷ø çè u1 ÷
ø connected in series, then effective force constant
t (½ M1u12 ) u 2 æ M1u1 ö 1 1 1 1
Þ 1 = ´ =ç ÷÷ = + + + ......
t 2 (½ M 2 u 22 ) u1 çè M 2 u 2 ø K eff K1 K 2 K 3
(c) If spring of spring constant K1, K2, K3........... are
t1 M1
(a) If u1 = u 2 Þ = connected in parallel, then effective spring constant
t 2 M2 Keff = K1 + K2 + K3 +.............
t1 u 1
(b) If M1 = M 2 Þ = POWER
t2 u2 Power of the body is defined as the time rate of doing work by
(c) If M1u1 = M2u2 Þ t1 = t2 the body.
The average power Pav over the time interval Dt is defined by
x1 (M1u1 ) 2 M 2 x M
and = 2
´ Þ 1 = 2 DW
x 2 (M 2 u 2 ) M1 x 2 M1 Pav = ...(i)
Dt
(d) Consider two vehicles of masses M 1 & M 2
And the instantaneous power P is defined by

.IN
respectively.
If they are moving with same velocities, then the ratio DW dW
P = Lim = ...(ii)
of their stopping distances by the application of same AL dt ® 0 Dt dt
retarding force is given by Power is a scalar quantity
x1 M1 The S.I. unit of power is joule per second
= and let M2 > M1 then x1 < x2 1 joule/sec = 1watt
N
x 2 M2
The dimensions of power are [ML2T–3]
Þ lighter mass will cover less distance then the r
R

heavier mass dW d r r r dS rr
P= = ( F .S ) = F . = F .v
U

And the ratio of their retarding times are as follows : dt dt dt


t1 M1 x1 t 1 (force is constant over a small time interval)
JO

= i.e =
t2 M2 x2 t2 So instantaneous power (or instantaneous rate of working) of a
man depends not only on the force applied to body, but also on
8. If kinetic energy of a body is doubled, then its momentum
the instantaneous velocity of the body.
U

p2 Example 1.
becomes 2 times, E k = Þ pµ E k
ED

2m A metre stick of mass 600 mg, is pivoted at one end and


9. If two bodies of masses m1 and m2 have equal kinetic displaced through an angle of 60º. The increase in its
energies, then their velocities are inversely proportional potential energy is (g = 10 ms–2)
to the square root of the respective masses. i.e. (a) 1.5 J (b) 15 J
(c) 150 J (d) 0.15 J
1 1 v1 m2 Solution : (a)
m1v12 = m 2 v 2 2 then =
2 2 v2 m1 O
l/2

x
10. (a) The spring constant of a spring is inversely 60º
l/2 G´
1 M
proportional to the no. of turn s i.e. Kµ h
n G A´

or kn = const.
(b) Greater the no. of turns in a spring, greater will be the
work done i.e. W µ n A
(c) The greater is the elasticity of the spring, the greater
The C.G. of stick rises from G to G´.
is the spring constant.
\ Increase in P.E. = mgh
11. Spring constant : The spring constant of a spring is
mg l
1 = mg (l/2 – x) = (1 - cos 60º )
inversely proportional to length i.e., K µ or 2
l
0.6 x 10 x 1 é 1 ù
Kl = constant. = ê1 - 2 ú = 1.5 J
2 ë û
Work, Energy and Power 143

Example 2. Example 5.
A block of mass M slides along the sides of a flat bottomed A particle of mass m is moving along a circular path of
bowl. The sides of the bowl are frictionless and the base constant radius R. The centripetal acceleration varies as
has a coefficient of friction 0.2. If the block is released a = K2 Rt2, where K is a constant and t is the time elapsed.
from the top of the side which is 1.5 m high, where will the What is the power delivered to the particle by the force
block come to rest if, the length of the base is 15 m ? acting on it?
(a) 1 m from P (b) Mid point of flat part PQ v2
(c) 2 m from P (d) At Q Sol. For circular motion, ac = v2 / R here K2 Rt2 = or v2 =
R
Solution : (b) K2 R2 t2
P.E. of the block at top of side = 1.5 mg. 1 1
This is wasted away in doing work on the rough flat part, Now, KE = mv 2 = m. K 2 R 2 t 2
2 2
Work done in time t = W = DK
1.5 m
P Q (from work energy theorem)
15 m
m 2 2 2 m
1.5 \ DK = (K R t ) - 0 = K 2 R 2 t 2
\ 1.5 mg = m mg.x or x = = 7.5 m. 2 2
m dW m 2 2
i.e, the block comes to rest at mid-point of PQ. Power = = K .R .2t = mK 2 R 2 t
dt 2
Example 3. Example 6.

.IN
Fig. given below shows a smooth curved track terminating
An electron of mass 9.0 × 10–28 g is moving at a speed of
in a smooth horizontal part. A spring of spring constant
400 N/m is attached at one end to the wedge fixed rigidly AL 1000 m/sec. Calculate its kinetic energy if the electron
with the horizontal part. A 40 g mass is released from rest takes up this speed after moving a distance of 10 cm from
at a height of 4.9 m on the curved track. Find the maximum rest. Also calculate the force in kg weight acting on it.
compression of the spring. Solution :
N
1 1
K.E. = mn 2 = ´ 9 ´ 10-31 (103 ) 2 = 4.5 × 10–25 J ;
R

2 2
From n 2 – u2 =2as, n 2 = 2as Q u=0
U

4.9 m
n2 (103 ) 2
JO

\a= =
2s 2 ´ 10 - 1

4.5 ´ 10-24
U

F = ma = 9 × 10–31 (0.5 × 107) N = kg wt.


Solution : 9.8
ED

According to the law of conservation of energy, = 0.46 × 10–24 kg wt.


1 Example 7.
mg h = k x2 The bob of a simple pendulum of length 1 m is drawn aside
2
where x is maximum compression. until the string becomes horizontal. Find the velocity of
the bob, after it is released, at the equilibrium position.
æ 2m g hö
\ x= ç
è k ÷ø
ì 2 ´ (0.4 kg ) ´ (9.8 m / s 2 )0 ´ ( 4.9 m) üï
or x = ïí ý = 9.8 cm.
ïî ( 400 N / m) ïþ
Example 4.
The K.E. of a body decreases by 19%. What is the
percentage decrease in momentum?
Solution :
Solution :
As p = 2m K.E When the bob is raised from A to B the height through
which it is raised is the length of the pendulum.
p f - pi K.E f - K.E i
´ 100 = ´ 100 h = 1m
pi K.E i Taking A as the standard level.
æ 81 - 100 ö P.E. at B = mgh = m × 9.8 × 1 = (9.8) m joule, where m is the
=ç ÷ ´ 100
è 100 ø mass of the bob.
= -10% Since it is at rest at B it has no K.E.
EBD_7179
144 PHYSICS

When the bob reaches A after it is released from B, its Oblique Elastic Collision :
energy at A is kinetic one. P.E. at A is zero. When a body of mass m collides obliquely against a stationary
If v be the velocity at A, from the law of conservation of body of same mass then after the collision the angle between
energy these two bodies is always 90°.
K.E. at A = P.E. at B
Elastic Collision in One Dimension (Head on)
1
mv2 = mgh or v2 = 2gh Let two bodies of masses M1 and M2 moving with velocities u1
2 and u2 along the same straight line, collide with each other. Let
Þ v = 2gh = 2 ´ 9.8 ´ 1 = 196 = 4.427 m/s u1>u2. Suppose v1 and v2 respectively are the velocities after
the elastic collision, then:
The bob has a velocity 4.427 m/s at A.
According to law of conservation of momentum
COLLISION
M1u1 + M 2 u 2 = M1v1 + M 2 v 2 ...(1)
Collision between two bodies is said to take place if either of
two bodies come in physical contact with each other or even M1 M2 M1 v1 M2 v2
u1 u2
when path of one body is affected by the force exerted due to
the other. Before collision After collision
From law of conservation of energy
Head on collision

.IN
Elastic collision (e = 1) (Impact parameter b = 0) 1 1 1 1
• Total energy conserved M1u12 + M 2 u 2 2 = M1v12 + M 2 v 2 2 ...(2)
• Total momentum conserved 2 2 2 2
• K.E is conserved Oblique collision
u1 - u 2 = – (v1 - v2 ) ...(3)
(Impact parameter b ¹ 0)
AL
Collision Relative velocity of a Relative velocity of a
Inelastic collision body before collision body after collision
N
Inelastic collision (0 < e < 1) Solving eqs. (1) and (2) we get,
• Total energy conserved Head on Oblique
R

• Total momentum conserved collision collision (M1 – M 2 )u1 2M 2 u 2


• K.E is not conserved v1 = + ...(4)
U

Perfectly inelastic collision (M1 + M 2 ) (M1 + M 2 )


(e = 0)
JO

(M 2 – M1 )u 2 2M1u1
(1) Elastic collision : The collision in which both the v2 = + ...(5)
(M1 + M 2 ) (M1 + M 2 )
momentum and kinetic energy of the system remains
U

conserved is called elastic collision. From eqns. (4) and (5), it is clear that :
Forces involved in the interaction of elastic collision are (i) If M1 = M2 and u2 = 0 then v1 = 0 and v2 = u1. Under this
ED

conservative in nature. condition the first particle comes to rest and the second
(2) Inelastic collision : The collision in which only the particle moves with the velocity of first particle after
collision. In this state there occurs maximum transfer of
momentum of the system is conserved but kinetic energy
energy.
is not conserved is called inelastic collision.
(ii) If M1>> M2 and (u2=0) then, v1 = u1, v2 = 2u1 under this
Perfectly inelastic collision is one in which the two bodies
condition the velocity of first particle remains unchanged
stick together after the collision.
and velocity of second particle becomes double that of
Forces involved in the interaction of inelastic collision are first.
non-conservative in nature.
Coefficient of Restitution (or coefficient of resilience) : 2M1
(iii) If M1 << M2 and (u2 = 0) then v1 = –u1 and v2 = u1 » 0
It is the ratio of velocity of separation after collision to the M2
velocity of approach before collision. under this condition the second particle remains at rest
i.e., e = | v1 – v2 |/ | u1 – u2 | while the first particle moves with the same velocity in the
Here u1 and u2 are the velocities of two bodies before collision opposite direction.
and v1 and v2 are the velocities of two bodies after collision. (iv) If M1 = M2 = M but u2 ¹ 0 then v1 = u2 i.e., the particles
1. 0 < e < 1 (Inelastic collision) mutually exchange their velocities.
Collision between two ivory balls, steel balls or quartz ball (v) If second body is at rest i.e., u2 = 0, then fractional decrease
is nearly elastic collision. in kinetic energy of mass M1, is given by
2. For perfectly elastic collision, e = 1 E k1 - E 'k1 v12 4M1M 2
3. For a perfectly inelastic collision, e = 0 = 1- =
E k1 u12 (M1 + M 2 ) 2
Work, Energy and Power 145

Inelastic Collision : Oblique Collision :


Let two bodies A and B collide inelastically. Then from law of This is the case of collision in two dimensions. After the collision,
conservation of linear momentum the particles move at different angle.
M1u1 + M2u2 = M1v1+M2v2 ...(i) y axis v1
u1
u2 = 0 A
æ velocity of separation ö
Coefficient of restitution = - ç m1
è velocity of approach ÷ø
q x axis
m1 m2 f
(v - v 2 ) A
e =- 1 ...(ii) B
( u1 - u 2 ) v2
B m2
From eqns.(i) and (ii), we have,
Before collision After collision
æ M - eM 2 ö æ M (1 + e) ö We will apply the principle of conservation of momentum in the
v1 = ç 1 ÷ u1 + ç 2 u2 ...(iii)
è M +M ø
1 2 è M + M ÷ø 1 2
mutually perpendicular direction.
Along x-axis, m1u1 = m1v1 cosq + m2 v2 cosf
æ (1 + e)M 1 ö æ M - eM 1 ö Along y-axis, 0 = m1v1 sinq - m2 v2 sinf
v 2 = çç ÷ u1 + ç 2
÷
÷
ç M + M ÷u 2 ...(iv)
M
è 1 + M 2 ø è 1 2 ø Keep in Memory
Loss in kinetic energy (–DEk) = initial K.E. – final K.E

.IN
1. Suppose, a body is dropped from a height h 0 and it strikes
é1 1 ù é1 1 ù the ground with velocity v0. After the (inelastic) collision
Þ - D E k = ê M 1 u 12 + M 2 u 22 ú - ê M 1 v 12 + M 2 v 22 ú
ë2 2 û ë2 2 û let it rise to a height h1. If v1 be the velocity with which the
ALbody rebounds, then the coefficient of restitution.
Þ - DE k =
1 æ M1 M 2
ç
2 çè M1 + M 2
(
ö 2
)
÷÷ e - 1 (u1 - u 2 )2 ...(v)
ø
N
1
Negative sign indicates that the final kinetic energy is less than 1
R

v æ 2gh1 ö 2 æ h ö 2 h0 h1
initial kinetic energy. e= 1 = ç = çç ÷÷
v0 è 2gh 0 ÷ø è h0 ø
U

Perfectly Inelastic Collision v0 v1


JO

In this collision, the individual bodies A and B move with velocities


2. If after n collisions with the ground, the velocity is vn and
u1 and u2 but after collision move as a one single body with
the height to which it rises be hn, then
velocity v.
U

1
So from law of conservation of linear momentum, we have æh ö2
v
e = n = çç n
n
÷
ED

M1u1+M2u2=(M1+M2)V ...(i) vo è h o ÷
ø

æ M u + M 2u 2 ö 3. When a ball is dropped from a height h on the ground,


or V = çç 1 1 ÷÷ ...(ii) then after striking the ground n times , it rises to a height
è M1 + M 2 ø hn = e2n ho where e = coefficient of restitution.
4. If a body of mass m moving with velocity v, collides
M1 + M2 elastically with a rigid ball, then the change in the
M1 M2 V momentum of the body is 2 m v.
u1 u2
A B A B (i) If the collision is elastic then we can conserve the
energy as
Before collision After
(u1 > u2) 1 1 1
collision 2 2
m1u1 = m1v 1 + m 2 v 2
2
2 2 2
(ii) If two particles having same mass and moving at right
And loss in kinetic energy, –DE k = total initial K.E angles to each other collide elastically then after the
– total final K.E collision they also move at right angles to each other.
(iii) If a body A collides elastically with another body of
2
1 1 1 æ M u + M2u 2 ö same mass at rest obliquely, then after the collision
= M1u 12 + M 2 u 22 - ( M1 + M 2 )çç 1 1 ÷÷ the two bodies move at right angles to each other, i. e.
2 2 2 è M1 + M 2 ø p
(q + f) =
2
1 M 1M 2 5. In an elastic collision of two equal masses, their kinetic
or, - DE k = (u1 - u 2 ) 2 ...(iii)
2 (M1 + M 2 ) energies are exchanged.
EBD_7179
146 PHYSICS

6. When two bodies collide obliquely, their relative velocity Example 9.


resolved along their common normal after impact is in A bullet of mass m moving horizontally with velocity v hits
constant ratio to their relative velocity before impact a block of wood of mass M, resting on a smooth horizontal
(resolved along common normal), and is in the opposite
plane. Find the fraction of energy of the bullet dissipated
direction.
in the collision itself (assume collision to be inelastic).
m1 m1 v1
q1
Solution :
Applying the law of conservation of momentum, we have,
m v = (m + M) v1
u1
æ mv ö
u2 m2 v1 = ç ÷
è m+Mø
b
1 2 1 2
q2 Loss of K.E. = mv - (m + M)v1
v2
2 2
m2
After collision 2
Before collision 1 1 æ mv ö
= m v 2 - (m + M ) ç ÷
v1 sin q1 - v 2 sin q 2 2 2 èM+mø
= -e
u 1 sin a - u 2 sin b é m ù 1
1 2 é M ù
= m v 2 ê1 - ú = mv êm + Mú
2 ë m + Mû 2 ë û
Example 8.
Loss of K.E. = æ M ö

.IN
A body of mass m moving with velocity v collides head on Fraction of K.E. dissipated = ç ÷
with another body of mass 2m which is initially at rest. Initial K.E. èm+Mø
What will be the ratio of K.E. of colliding body before and
after collision?
AL
Example 10.
Solution : A smooth sphere of mass 0.5 kg moving with horizontal
mv + 2m × 0 = mv1 +2mv2 ; speed 3 m/s strikes at right angles a vertical wall and bounces
N
v - v1 off the wall with horizontal speed 2 m/s. Find the coefficient
v = v1 + 2v2 or v 2 = ; of restitution between the sphere and the wall and the
2
R

impulses exerted on the wall at impact.


1 1 1
mv 2 = mv12 + 2mv 22 Solution :
U

2 2 2
Just before impact 3m/s

//////////////////
2
æ v - v1 ö
JO

v 2 = v12 + 2v 22 = v12 + 2ç ÷
è 2 ø At impact J J
v 2 + v 2 - 2vv 1 2
2v12 + v12 2
2
v = v12 + 1 or 2v = + v - 2vv1
U

2 Just after impact 2m/s


or 3v12 - 2vv1 - v 2 = 0 ; v1 = - v e = separation speed : approach speed = 2 : 3
ED

3 Therefore the coefficient of restitution is 2/3.


1 Using impulse = change in momentum for the sphere we
mv 2
K.E. of colliding body before collision = 2 have : = 0.5 × 2 – 0.5 (–3) = 2.5
2 =9:1
K.E. of colliding body after collision 1 ævö The equal and opposite impulse acting on the wall is
mç ÷
2 è3ø therefore 2.5 N s.
Work, Energy and Power 147

.IN
AL
N
R
U
JO
U
ED
EBD_7179
148 PHYSICS

1. The magnitude of work done by a force : 10. A metallic wire of length L metre extends by l metre when
(a) depends on frame of reference stretched by suspending a weight Mg from it. The mechanical
(b) does not depend on frame of reference energy stored in the wire is
(c) cannot be calculated in non-inertial frames. (a) 2 Mg l (b) Mg l
(d) both (a) and (b) Mg l Mg l
(c) (b)
2. Work done by a conservative force is positive if 2 4
(a) P.E. of the body increases 11. A body of mass m accelerates uniformly from rest to v1 in
(b) P.E. of the body decreases time t1. As a function of t, the instantaneous power delivered
(c) K.E. of the body increases to the body is
(d) K.E. of the body decreases m n1 t m n12 t
3. A vehicle is moving with a uniform velocity on a smooth (a) (b)
t2 t1
horizontal road, then power delivered by its engine must be
(a) uniform (b) increasing m n12 t
m n1 t 2
(c) decreasing (d) zero

.IN
(c) (d)
4. Which of the following force(s) is/are non-conservative? t1 t12
(a) Frictional force (b) Spring force 12.
AL A block is acted upon by a force, which is inversely
(c) Elastic force (d) All of these proportional to the distance covered (x). The work done
5. A ball of mass m and a ball B of mass 2m are projected with will be proportional to
equal kinetic energies. Then at the highest point of their (a) x (b) x1/2
N
respective trajectories. (c) x 2 (d) None of these
(a) P.E. of A will be more than that of B 13. A small body is projected in a direction inclined at 45º to the
R

(b) P.E of B will be more than that of B horizontal with kinetic energy K. At the top of its flight, its
U

(c) P.E of A will be equal to that of B kinetic energy will be


(d) can’t be predicted. (a) Zero (b) K/2
JO

6. In case of elastic collision, at the time of impact.


(c) K/4 (d) K/ 2
(a) total K.E. of colliding bodies is conserved.
(b) total K.E. of colliding bodies increases 14. A motor cycle is moving along a straight horizontal road
U

(c) total K.E. of colliding bodies decreases with a speed v0. If the coefficient of friction between the
(d) total momentum of colliding bodies decreases. tyres and the road is m, the shortest distance in which the
ED

7. The engine of a vehicle delivers constant power. If the car can be stopped is
vehicle is moving up the inclined plane then, its velocity, v20 v2
(a) must remain constant (a) (b)
2m g m
(b) must increase
(c) must decrease 2
æ v0 ö v0
(d) may increase, decrease or remain same. (c) çè m g ÷ø (d) mg
8. A ball projected from ground at a certain angle collides a
smooth inclined plane at the highest point of its trajectory. 15. Consider the following two statement:
If the collision is perfectly inelastic then after the collision, I. Linear momentum of a system of particles is zero.
ball will II. Kinetic energy of a system of particles is zero.
(a) come to rest Then
(b) move along the incline (a) I implies II but II does not imply I.
(c) retrace its path. (b) I does not imply II but II implies I.
9. The vessels A and B of equal volume and weight are (c) I implies II and II implies I.
immersed in water to depth h. The vessel A has an opening (d) I does not imply II and II does not imply I.
at the bottom through which water can enter. If the work 16. Which of the following must be known in order to determine
done in immersing A and B are WA and WB respectively, the power output of an automobile?
then (a) Final velocity and height
(a) WA = WB (b) WA < WB (b) Mass and amount of work performed
(c) WA > WB (d) WA > WB (c) Force exerted and distance of motion
<
(d) Work performed and elapsed time of work
Work, Energy and Power 149

17. The work done in stretching a spring of force constant k 22. A ball of mass m moving with a constant velocity strikes
from length l1 and l 2 is against a ball of same mass at rest. If e = coefficient of
restitution, then what will be the ratio of velocity of two
1 balls after collision?
(a) k(l 22 - l12 ) (b) k (l 22 - l 12 )
2 1- e e -1
(a) (b)
k 1+ e e +1
(c) k (l 2 - l 1 ) (d) (l 2 + l 1 )
2 1+ e 2+e
18. If the force acting on a body is inversely proportional to its (c) (d)
1- e e -1
velocity, then the kinetic energy acquired by the body in 23. Which one of the following physical quantities is
time t is proportional to represented by the shaded area in the given graph?
(a) t 0 (b) t 1
(c) t 2 (d) t 4
19. The engine of a truck moving along a straight road delivers

Force
constant power. The distance travelled by the truck in time
t is proportional to
(a) t (b) t 2
(c) t (d) t 3/2 Distance

.IN
20. A bullet of mass ‘a’ and velocity ‘b’ is fired into a large (a) Torque (b) Impulse
block of wood of mass ‘c’. The bullet gets embedded into (c) Power (d) Work done
the block of wood. The final velocity of the system is 24. A particle of mass m1 moving with velocity v collides with
AL
b a +b a mass m2 at rest, then they get embedded. Just after
(a) ´c (b) ´a collision, velocity of the system
a+b c
N
(a) increases (b) decreases
a a +c (c) remains constant (d) becomes zero
(c) ´b (d) ´b
R

a +c a
21. A ball is dropped from a height h. If the coefficient of 25. A mass m1 moves with a great velocity. It strikes another
U

restitution be e, then to what height will it rise after jumping mass m2 at rest in a head on collision. It comes back along
JO

twice from the ground? its path with low speed, after collision. Then
(a) e h/2 (b) 2 e h (a) m1 > m2 (b) m1 < m2
(c) e h (d) e4 h (c) m1 = m2 (d) cannot say
U
ED

1. A particle describe a horizontal circle of radius 0.5 m with 4. A particle moving in the xy plane undergoes a displacement
r r
uniform speed. The centripetal force acting is 10 N. The of s = (2 î + 3 ĵ) while a constant force F = (5 î + 2 ĵ ) N
work done in describing a semicircle is
acts on the particle. The work done by the force F is
(a) zero (b) 5 J
(a) 17 joule (b) 18 joule
(c) 5 p J (d) 10 p J
2. A cord is used to lower vertically a block of mass M, (c) 16 joule (d) 15 joule
a distance d at a constant downward acceleration of g/4. 5. A simple pendulum 1 metre long has a bob of 10 kg. If the
The work done by the cord on the block is pendulum swings from a horizontal position, the K.E. of the
bob, at the instant it passes through the lowest position of
d d
(a) Mg (b) 3Mg its path is
4 4
(a) 89 joule (b) 95 joule
d
(c) -3Mg (d) Mg d (c) 98 joule (d) 85 joule
4 6. A particle moves under the effect of a force F = cx from
3. A boy pushes a toy box 2.0 m along the floor by means of a
x = 0 to x = x1, the work done in the process is
force of 10 N directed downward at an angle of 60º to the
horizontal. The work done by the boy is 1 2
(a) 6 J (b) 8 J (a) cx 12 (b) cx1
2
(c) 10 J (d) 12 J
(c) 2 cx12 (d) zero
EBD_7179
150 PHYSICS

7. A motor of 100 H.P. moves a load with a uniform speed of 72 18. A long string is stretched by 2 cm and the potential energy
km/hr. The forward thrust applied by the engine on the car is V. If the spring is stretched by 10 cm, its potential energy
is will be
(a) 1111 N (b) 3550 N (a) V / 25 (b) V/5
(c) 2222 N (d) 3730 N (c) 5 V (d) 25 V
8. Two bodies A and B having masses in the ratio of 3 : 1 19. When the kinetic energy of a body is increased to three
possess the same kinetic energy. The ratio of linear times, then the momentum increases
momentum of B to A is (a) 6 times (b) 1.732 times
(a) 1 : 3 (b) 3 : 1 (c) 2 times (d) 2 times
20. Two bodies of masses 2 m and m have their KE in the ratio
(c) 1: 3 (d) 3 :1
8 : 1. What is the ratio of their momenta ?
9. When a U238 nucleus, originally at rest, decays by emitting (a) 8 : 1 (b) 4 : 1
an a-particle, say with speed of v m/sec, the recoil speed of (c) 2 : 1 (d) 1 : 1
the residual nucleus is (in m/sec.) 21. A body of mass 5 kg initially at rest explodes into 3
(a) – 4 v/234 (b) – 4 v/238 fragments with mass ratio 3 : 1 : 1. Two of fragments each of
(c) 4 v/238 (d) – v/4 mass ‘m’ are found to move with a speed 60 m/s in mutually
10. Calculate the K.E and P.E. of the ball half way up, when a perpendicular directions. The velocity of third fragment is
ball of mass 0.1 kg is thrown vertically upwards with an (a) 60 2 (b) 20 3
initial speed of 20 ms–1.

.IN
(a) 10 J, 20 J (b) 10 J, 10 J (c) 10 2 (d) 20 2
(c) 15 J, 8 J (d) 8 J, 16 J 22. A machine, which is 75% efficient, uses 12 J of energy in
lifting up a 1 kg mass through a certain distance. The mass
11. A spring of force constant 800 N/m has an extension of 5
AL
cm. The work done in extending it from 5 cm to 15 cm is is then allowed to fall through that distance. The velocity
(a) 16 J (b) 8 J at the end of its fall is (in m/s)
N
(c) 32 J (d) 24 J. (a) 24 (b) 12
12. If the linear momentum is increased by 5%, the kinetic energy
R

(c) 18 (d) 9
will increase by
23. A body accelerates uniformly from rest to a velocity of 1
U

(a) 50% (b) 100%


ms–1 in 15 seconds. The kinetic energy of the body will be
(c) 125% (d) 10%
JO

13. A cord is used to lower vertically a block of mass M, through 2


a distance d at a constant downward acceleration of g/8. J when 't' is equal to [Take mass of body as 1 kg]
9
Then the work done by the cord on the block is (a) 4s (b) 8s
U

(a) Mg d/8 (b) 3 Mg d/8 (c) 10s (d) 12s


(c) Mg d (d) – 7 mg d/8 24. A machine gun fires a bullet of mass 40 g with a velocity
ED

r r r r
14. A force F = (5i + 3 j + 2k)N is applied over a particle which 1200 ms–1. The man holding it can exert a maximum force of
r r r 144 N on the gun. How many bullets can he fire per second
displaces it from its origin to the point r = (2 i - j)m. The
at the most?
work done on the particle in joule is
(a) 2 (b) 4
(a) +10 (b) +7
(c) 1 (d) 3
(c) –7 (d) +13 25. A crane is used to lift 1000 kg of coal from a mine 100 m
15. A spring of spring constant 5 × 103 N/m is stretched initially deep. The time taken by the crane is 1 hour. The efficiency
by 5 cm from the unstretched position. Then the work of the crane is 80%. If g = 10 ms–2, then the power of the
required to stretch it further by another 5 cm is crane is
(a) 18.75 J (b) 25.00 J (a) 104 W (b) 105 W
(c) 6.25 J (d) 12.50 J
16. Two solid rubber balls A and B having masses 200 & 104 105
(c) W (d) W
400 gm respectively are moving in opposite direction with 36 ´ 8 36 ´ 8
velocity of A equal to 0.3 m/sec. After collision the two 26. In figure, a carriage P is pulled up from A to B. The relevant
balls come to rest when the velocity of B is coefficient of friction is 0.40. The work done will be
(a) 0.15 m/sec (b) 1.5 m/sec (a) 10 kJ
(c) –0.15 m/sec (d) None of these B
17. A bomb of mass 9 kg explodes into the pieces of masses (b) 23 kJ 0 kg
P 5
3 kg and 6 kg. The velocity of mass 3 kg is 16 m/s. The (c) 25 kJ
kinetic energy of mass 6 kg in joule is m 30 m
50
(a) 96 (b) 384 (d) 28 kJ q
(c) 192 (d) 768 A C
Work, Energy and Power 151

27. A neutron with velocity v strikes a stationary deuterium 35. A body of mass m is suspended from a massless spring of
atom, its K.E. changes by a factor of natural length l. It stretches the spring through a vertical
distance y. The potential energy of the stretched spring is
15 1
(a) (b) 1
16 2 mg (l + y)
(a) mg(l + y) (b)
2
2
(c ) (d) None of these 1
1 (c) mgy (d) mgy
28. A body moves a distance of 10 m along a straight line under 2
the action of a force of 5 newtons. If the work done is 25 36. Figure here shows the frictional force versus displacement
joules, the angle which the force makes with the direction for a particle in motion. The loss of kinetic energy in
of motion of body is travelling over s = 0 to 20 m will be
(a) 0º (b) 30º
(c) 60º (d) 90º f(N)
29. A sphere of mass 8m collides elastically (in one dimension) 15
with a block of mass 2m. If the initial energy of sphere is E.
What is the final energy of sphere? 10
(a) 0.8 E (b) 0.36 E
(c) 0.08 E (d) 0.64 E 5

.IN
30. Johnny and his sister Jane race up a hill. Johnny weighs
twice as much as jane and takes twice as long as jane to 0 x(m)
0 5 10 20
reach the top . Compared to Jane AL
(a) Johnny did more work and delivered more power. (a) 250 J (b) 200 J
(b) Johnny did more work and delivered the same amount (c) 150 J (d) 10 J
of power. 37. Ten litre of water per second is lifted from a well through 10
N
(c) Johnny did more work and delivered less power m and delivered with a velocity of 10 ms –1 . If
(d) Johnny did less work and johnny delivered less power. g = 10 ms–2 , then the power of the motor is
R

31. A body of mass m moving with velocity v makes a head on (a) 1 kW (b) 1.5 kW
U

elastic collision with another body of mass 2m which in (c) 2 kW (d) 2.5 kW
initially at rest. The loss of kinetic energy of the colliding 38. A nucleus ruptures into two nuclear parts which have their
JO

body (mass m ) is velocity ratio equal to 2 : 1 . The ratio of their respective


1 nuclear sizes (nuclear radii )is
(a) of its initial kinetic energy
2
U

(a) 1 : 2 (b) 1 : 2
(b) 1 of its initial kinetic energy (c) 1 : 2 1/3 (d) 1 : 8
ED

9 39. The rest energy of an electron is 0.511 MeV. The electron is


8 accelerated from rest to a velocity 0.5 c. The change in its
(c) of its initial kinetic energy energy will be
9
(a) 0.026 MeV (b) 0.051 MeV
1 (c) 0.08 MeV (d) 0.105 MeV
(d) of its initial kinetic energy
4 40. A one-ton car moves with a constant velocity of
32. In the non-relativistic regime, if the momentum, is increase 15 ms–1 on a rough horizontal road. The total resistance to
by 100% , the percentage increase in kinetic energy is the motion of the car is 12% of the weight of the car. The
(a) 100 (b) 200 power required to keep the car moving with the same
(c) 300 (d) 400 constant velocity of 15ms–1 is
33. A body is dropped from a height of 20m and rebounds to a [Take g = 10 ms–2]
height 10m. The loss of energy is (a) 9 kW (b) 18 kW
(a) 10% (b) 45% (c) 24 kW (d) 36 kW
(c) 50% (d) 75% 41. Hail storms are observed to strike the surface of the frozen
34. A moving body with a mass m1 and velocity u strikes a lake at 300 with the vertical and rebound at 600 with the
stationary body of mass m2. The masses m1 and m2 should vertical. Assume contact to be smooth, the coefficient of
be in the ratio m1/m2 so as to decrease the velocity of the restitution is
first body to 2u/3 and giving a velocity of u to m2 assuming 1 1
a perfectly elastic impact. Then the ratio m1/m2 is (a) e= (b) e=
3 3
(a) 5 (b) 1 / 5
(c) 1 / 25 (d) 25. (c) e= 3 (d) e = 3
EBD_7179
152 PHYSICS

42. A body starts from rest and acquires a velocity V in time T. mV (M + m)


The work done on the body in time t will be proportional to (c) momentum =
M
V V2 2 m2 v2
(a) t (b) t kinetic energy =
T T (d)
2 (M + m)
V2 V2 2 51. A particle, initially at rest on a frictionless horizontal surface,
(c) t (d) t is acted upon by a horizontal force which is constant in
T2 T2
magnitude and direction. A graph is plotted of the work
43. A ball is allowed to fall from a height of 10 m. If there is 40% done on the particle W, against the speed of the particle v.
loss of energy due to impact, then after one impact ball will If there are no other horizontal forces acting on the particle,
go up to the graph would look like
(a) 10 m (b) 8 m
(c) 4 m (d) 6 m W W
44. The potential energy of a conservative system is given by
U = ay2 – by, where y represents the position of the particle
and a as well as b are constants. What is the force acting on (a) (b)
the system ? v v
(a) – ay (b) – by
(c) 2ay – b (d) b – 2ay W W

.IN
45. An automobile engine of mass M accelerates and a constant
power p is applied by the engine. The instantaneous speed
of the engine will be AL (c) (d)
(a) [Pt / M]1/ 2 (b) [2Pt / M]1/ 2 v v
(c) [Pt / 2M]1/ 2 (d) [Pt / 4M] 1/ 2 52. A particle of mass m moving eastward with a speed v
collides with another particle of the same mass moving
N
46. A bullet fired into a fixed target loses half of its velocity
northwards with the same speed. If two particles coalesce
after penetrating 3 cm. How much further it will penetrate
R

on collision, the new particle of mass 2 m will move in the


before coming to rest assuming that it faces constant
north-east direction with a velocity
resistance to motion ?
U

(a) 2.0 cm (b) 3.0 cm (a) v / 2 (b) v 2


JO

(c) 1.0 cm (d) 1.5 cm (c) v / 2 (d) None of these


47. A bomb of mass 16kg at rest explodes into two pieces of 53. A small block of mass m is kept on a rough inclined surface
masses 4 kg and 12 kg. The velolcity of the 12 kg mass is of inclination q fixed in an elevator. The elevator goes up
4ms–1. The kinetic energy of the other mass is
U

with a uniform velocity v and the block does not slide on


(a) 144 J (b) 288 J the wedge. The work done by the force of friction on the
ED

(c) 192 J (d) 96 J block in time t as seen by the observer on the inclined p
48. Given that a force F̂ acts on a body for time t, and displaces lane will be
the body by d̂ . In which of the following cases, the speed (a) zero (b) mgvt cos2q
(c) mgvt sin q 2 (d) mgvt sin 2q
of the body must not increase? r
(a) F > d (b) F < d 54. A nucleus moving with a velocity v emits an a-particle.
Let the velocities of the a-particle and the remaining nucleus
(c) F̂ = dˆ (d) F̂ ^ dˆ r r
49. A body is attached to the lower end of a vertical helical be v1 and v2 and their masses be m1 and m2.
r r r
spring and it is gradually lowered to its equilibrium position. (a) v , v1 and v 2 must be parallel to each other
This stretches the spring by a length x. If the same body r r r
(b) None of the two of v , v1 and v2 should be parallel to
attached to the same spring is allowed to fall suddenly,
what would be the maximum stretching in this case? each other
r r r
(a) x (b) 2 x (c) m1 v1 + m 2 v 2 must be parallel to (m1 + m 2 )v .
(c) 3 x (d) x/2 (d) None of these
50. A bag of mass M hangs by a long thread and a bullet (mass 55. A shell is fired from a cannon with a velocity V at an angle
m) comes horizontally with velocity V and gets caught in q with the horizontal direction. At the highest point in its
the bag. Then for the combined (bag + bullet) system path, it explodes into two pieces of equal masses. One of
mvM the pieces retraces its path to the cannon. The speed of the
(a) momentum = other piece immediately after the explosion is
M+m
(a) 3 V cos q (b) 2 V cos q
m V2 3
(b) kinetic energy = (c) V cos q (d) V cos q
2 2
Work, Energy and Power 153

56. The bob A of a simple pendulum is released when the string


makes an angle of 45º with the vertical. It hits another bob P
P
B of the same material and same mass kept at rest on the (a) (b)
table. If the collision is elastic
t t

45º P P
(c) (d)
A
t t
B 63. A glass marble dropped from a certain height above the
(a) both A and B rise to the same height horizontal surface reaches the surface in time t and then
(b) both A and B come to rest at B continues to bounce up and down. The time in which the
marble finally comes to rest is
(c) both A and B move with the velocity of A
(d) A comes to rest and B moves with the velocity of A (a) ent (b) e 2 t
57. Two masses ma and mb moving with velocities va and vb in
opposite direction collide elastically and after the collision é1+ e ù é1 - e ù
(c) tê (d) t ê ú
ma and mb move with velocities Vb and Va respectively. ë1 - e úû ë1 + e û

.IN
Then the ratio ma/mb is
64. A weight suspended from the free end of a vertically
Va - Vb ma + mb hanging spring produces an extension of 3 cm. The spring
(a) (b) is cut into two parts so that the length of the longer part is
Va + Vb ma
AL
2
1 of the original length, If the same weight is now
(c) 1 (d) 3
N
2
suspended from the longer part of the spring, the extension
58. A particle moves in a straight line with retardation
R

produced will be
proportional to its displacement. Its loss of kinetic energy
(a) 0.1 cm (b) 0.5 cm
U

for any displacement x is proportional to


(a) x (b) e x (c) 1 cm (d) 2 cm
A 10 m long iron chain of linear mass density 0.8 kg m–1 is
JO

(c) x 2 (d) loge x 65.


hanging freely from a rigid support. If g = 10 ms–2, then the
59. A particle of mass m1 moving with velocity v strikes with a power required to left the chain upto the point of support in
mass m2 at rest, then the condition for maximum transfer of
U

10 second
kinetic energy is (a) 10 W (b) 20W
ED

(a) m1 >> m2 (b) m2 >> m2 (c) 30 W (d) 40 W


(c) m1 = m2 (d) m1 = 2m2 66. A smooth sphere of mass M moving with velocity u directly
60. Two blocks of masses 10 kg and 4 kg are connected by a collides elastically with another sphere of mass m at rest.
spring of negligible mass and placed on a frictionless After collision, their final velocities are V and v respectively.
horizontal surface. An impulse gives a velocity of 14 m/s to The value of v is
the heavier block in the direction of the lighter block. The
velocity of the centre of mass is 2uM 2um
(a) (b)
(a) 30 m/s (b) 20 m/s m M
(c) 10 m/s (d) 5 m/s
61. A mass m is moving with velocity v collides inelastically 2u 2u
(c) (d)
with a bob of simple pendulum of mass m and gets embedded m M
1+ 1+
into it. The total height to which the masses will rise after M m
collision is 67. The kinetic energy of particle moving along a circle of radius
R depends upon the distance covered S and is given by K
v2 v2 = aS where a is a constant. Then the force acting on the
(a) (b)
8g 4g particle is

v2 2v 2 aS 2(aS) 2
(c) (d) (a) (b)
2g g R R
62. A motor drives a body along a straight line with a constant aS2 2aS
force. The power P developed by the motor must vary with (c) 2 (d)
R R
time t according to
EBD_7179
154 PHYSICS

68. A ramp is constructed in parabolic shape such that the 74. A body falls freely under gravity. Its velocity is v when it
height y of any point on its surface is given in terms of the has lost potential energy equal to U. What is the mass of
point's horizontal distance x from the bottom of the ramp be the body ?
y = x2/2L. A block of granite is to be set on the ramp; the (a) U2 /v2 (b) 2U2/v2
coefficient of static friction is 0.80. What is the maximum x (c) 2U/v 2 (d) U /v2
coordinate at which the block can be placed on the ramp 75. If v be the instantaneous velocity of the body dropped
and remain at rest, if L = 10 m? from the top of a tower, when it is located at height h, then
which of the following remains constant ?
v2
y (a) gh + v2 (b) gh +
2
x v2
(a) 8 m (b) 8.4 m (c) gh - (d) gh – v2
2
(c) 9 m (d) 9.4 m
69. The force F acting on a body moving in a circle of radius r 76. The coefficient of friction between the tyres and the road is
is always perpendicular to the instantaneous velocity v. m. A car is moving with momentum p. What will be the
The work done by the force on the body in half rotation is stopping distance due to friction alone ? The mass of the
(a) Fv (b) F·2pr car is m.
(c) Fr (d) 0 (a) p2/2mg (b) p2/2mmg

.IN
70. The negative of the distance rate of change of potential (c) p2/2m2mg (d) p2/2mg
energy is equal to 77. A particle moves in the X–Y plane under the influence of a
r
(a) force acting on the particle in the direction of force F such that its instantaneous momentum is
displacement
AL r ˆ
p = i2cos t + ˆj2sin t .
(b) acceleration of the particle, perpendicular to
displacement What is the angle between the force and instantaneous
N
(c) power momentum ?
(a) 0° (b) 45°
R

(d) impulse.
71. n small balls each of mass m impinge elastically each second (c) 90° (d) 180°
U

on a surface with velocity v. The force experienced by the 78. A particle of mass 10 kg moving eastwards with a speed 5
surface will be ms–1 collides with another particle of the same mass moving
JO

north-wards with the same speed 5 ms–1. The two particles


1
(a) mnv (b) 2 mnv coalesce on collision. The new particle of mass 20 kg will
2 move in the north-east direction with velocity
U

(c) mnv (d) 2 mnv (a) 10 ms–1 (b) 5 ms–1


72. A horse drinks water from a cubical container of side 1 m.
ED

The level of the stomach of horse is at 2 m from the ground. (c) (5 / 2)ms -1 (d) none of these
Assume that all the water drunk by the horse is at a level of 79. A metal ball of mass 2 kg moving with a velocity of
2m from the ground. Then minimum work done by the horse 36 km/h has a head on collision with a stationary ball of
in drinking the entire water of the container is mass 3 kg. If after the collision, the two balls move together,
(Take rwater = 1000 kg/m3 and g = 10 m/s2 ) – the loss in kinetic energy due to collision is
(a) 10 kJ (a) 140 J (b) 100 J
(c) 60 J (d) 40 J
(b) 15 kJ 80. A force acts on a 30 gm particle in such a way that the position
(c) 20 kJ of the particle as a function of time is given by x = 3t – 4t 2
+ t3, where x is in metres and t is in seconds. The work done
(d) zero during the first 4 seconds is
73. The ball rolls down without slipping (which is at rest at a) (a) 576mJ (b) 450mJ
along ab having friction. It rolls to a maximum height h c (c) 490mJ (d) 530mJ
where bc has no friction. Ka, Kb and Kc are kinetic energies 81. Arubber ball is dropped from a height of5m on a plane, where
at a, b and c. the acceleration due to gravity is not shown. On bouncing
Which of the following is correct ? it rises to 1.8 m. The ball loses its velocity on bouncing by
a a factor of
c
16 2
ha hc (a) (b)
b 25 5
(a) Ka = Kc, ha = hc (b) Kb > Kc, ha = hc 3 9
(c) (d)
(c) Kb > Kc, ha < hc (d) Kb > Kc, ha > hc 5 25
Work, Energy and Power 155

82. A 3 kg ball strikes a heavy rigid wall with a speed of 10 m/ 89. A mass m moving horizontally (along the x-axis) with
s at an angle of 60º. It gets reflected with the same speed and velocity v collides and sticks to mass of 3m moving vertically
angle as shown here. If the ball is in contact with the wall for upward (along the y-axis) with velocity 2v. The final velocity
0.20s, what is theaverageforceexertedon theball bythe wall? of the combination is
1 ˆ 3 ˆ 1 ˆ 2 ˆ
(a) 150N (a) vi + v j (b) vi + v j
4 2 3 3
(b) Zero 60º 2 ˆ 1 ˆ 3 ˆ 1 ˆ
(c) vi + v j (d) vi + v j
3 3 2 4
(c) 150 3N 60º 90. The potential energy of particle in a force field is
A B
(d) 300N U = 2 - , where A and B are positive constants and r is
r r
83. Abombofmass1kgis thrown verticallyupwards with a speed the distance of particle from the centre of the field. For
of 100 m/s. After 5 seconds it explodes into two fragments. stable equilibrium, the distance of the particle is
One fragment of mass 400 gm is found to go down with a (a) B / 2A (b) 2A / B
speed of 25 m/s. What will happen to the second fragment (c) A / B (d) B / A
just after the explosion? (g = 10 m/s2) 91. A uniform force of (3iˆ + ˆj ) newton acts on a particle of
(a) It will go upward with speed 40 m/s
mass 2 kg. The particle is displaced from position (2$i + k$ )
(b) It will go upward with speed 100 m/s
meter to position (4$i + 3 $j - k$ ) meter. The work done by

.IN
(c) It will go upward with speed 60 m/s
(d) It will also go downward with speed 40m/s the force on the particle is
84. In a simple pendulum of length l the bob is pulled aside AL (a) 6 J (b) 13 J
from its equilibrium position through an angle q and then (c) 15 J (d) 9 J
released. The bob passes through the equilibrium position 92. An explosion breaks a rock into three parts in a horizontal
with speed plane. Two of them go off at right angles to each other. The
N
first part of mass 1 kg moves with a speed of 12 ms–1 and
(a) 2gl(1 + cos q) (b) 2gl sin q the second part of mass 2 kg moves with speed 8 ms–1. If the
R

third part flies off with speed 4 ms–1 then its mass is
(c) 2gl (d) 2gl(1 - cos q) (a) 5 kg (b) 7 kg
U

85. A stationary particle explodes into two particles of masses (c) 17 kg (d) 3 kg
JO

m1 and m2 which move in opposite directions with velocities 93. If the kinetic energy of a body is increased by 300%, the
v1 and v2. The ratio of their kinetic energies E1/E2 is momentum of the body is increased by
(a) m1v2/m 2v1 (b) m2/m1 (a) 300% (b) 200%
U

(c) m1/m2 (d) 1 (c) 100% (d) 50%


86. A mass of 0.5 kg moving with a speed of 1.5 m/s on a 94. If the mass of the body is halved and velocity gets doubled
ED

horizontal smooth surface, collides with a nearly weightless then final kinetic energy would be .........of initial.
spring of force constant k = 50 N/m. The maximum (a) same (b) four times
compression of the spring would be (c) double (d) eight times
95. A train of weight 107 N is running on a level track with
uniform speed of 36 km h–1. The frictional force is 0.5 kg per
quintal. If g = 10 m/s2, then power of engine is
(a) 500 kW (b) 50 kW
(a) 0.5 m (b) 0.15 m (c) 5 kW (d) 0.5 kW
(c) 0.12 m (d) 1.5m DIRECTIONS (Qs. 96 to 100): Each question contains Statement-
87. A ball moving with velocity 2 m/s collides head on with 1 and Statement-2. Choose the correct answer (ONLY ONE option
another stationary ball of double the mass. If the coefficient is correct ) from the following.
of restitution is 0.5, then their velocities (in m/s) after (a) Statement -1 is false, Statement-2 is true
collision will be (b) Statement -1 is true, Statement-2 is true; Statement -2 is a
(a) 0, 1 (b) 1, 1 correct explanation for Statement-1
(c) 1, 0.5 (d) 0, 2 (c) Statement -1 is true, Statement-2 is true; Statement -2 is not
88. A body projected vertically from the earth reaches a height a correct explanation for Statement-1
equal to earth's radius before returning to the earth. The (d) Statement -1 is true, Statement-2 is false
power exerted by the gravitational force is greatest 96. Statement-1 : A quick collision between two bodies is
(a) at the highest position of the body more violent than slow collision, even when initial and
(b) at the instant just before the body hits the earth final velocities are identical.
(c) it remains constant all through Statement -2 : The rate of change of momentum
(d) at the instant just after the body is projected determines that the force is small or large.
EBD_7179
156 PHYSICS

97. Statement -1 : If collision occurs between two elastic 99. Statement -1 : An object of mass m is initially at rest. A
bodies their kinetic energy decreases during the time of constant force F acts on it. Then the velocity gained by
collision. the object during a fixed displacement is proportional to

Statement -2 : During collision intermolecular space 1/ m .


decreases and hence elastic potential energy increases. Statement -2 : For a given force and displacement velocity
is always inversely proportional to root of mass.
98. Statement -1 :A work done by friction is always negative. 100. Statement -1 : Mechanical energy is the sum of
Statement -2 : If frictional force acts on a body its K.E. macroscopic kinetic & potential energies.
may decrease. Statement - 2 : Mechanical energy is that part of total
energy which always remain conserved.

Exemplar Questions (c) Total mechanical energy


1. An electron and a proton are moving under the influence of (d) Total linear momentum
mutual forces. In calculating the change in the kinetic energy 6. During inelastic collision between two bodies, which of the

.IN
of the system during motion, one ignores the magnetic force following quantities always remain conserved?
of one on another. This is, because (a) Total kinetic energy
(a) the two magnetic forces are equal and opposite, so AL (b) Total mechanical energy
they produce no net effect (c) Total linear momentum
(b) the magnetic forces do not work on each particle (d) Speed of each body
(c) the magnetic forces do equal and opposite (but non- 7. Two inclined frictionless tracks, one gradual and the other
N
zero) work on each particle steep meet at A from where two stones are allowed to slide
(d) the magnetic forces are necessarily negligible down from rest, one on each track as shown in figure.
R

2. A proton is kept at rest. A positively charged particle is Which of the following statement is correct?
released from rest at a distance d in its field. Consider two
U

A
experiments; one in which the charged particle is also a
JO

proton and in another, a positron. In the same time t, the I II


work done on the two moving charged particles is h
(a) same as the same force law is involved in the two
experiments q1 q2
U

B
(b) less for the case of a positron, as the positron moves
(a) Both the stones reach the bottom at the same time but
ED

away more rapidly and the force on it weakens


not with the same speed
(c) more for the case of a positron, as the positron moves
(b) Both the stones reach the bottom with the same speed
away a larger distance
and stone I reaches the bottom earlier than stone II
(d) same as the work done by charged particle on the
(c) Both the stones reach the bottom with the same speed
stationary proton.
and stone II reaches the bottom earlier than stone I
3. A man squatting on the ground gets straight up and stand.
(d) Both the stones reach the bottom at different times
The force of reaction of ground on the man during the
and with different speeds
process is
8. The potential energy function for a particle executing linear
(a) constant and equal to mg in magnitude
(b) constant and greater than mg in magnitude 1 2
SHM is given by V ( x) = kx where k is the force
(c) variable but always greater than mg 2
(d) at first greater than mg and later becomes equal to mg constant of the oscillator (Fig.). For k = 0.5 N/m, the graph
4. A bicyclist comes to a skidding stop in 10 m. During this of V(x) versus x is shown in the figure. A particle of total
process, the force on the bicycle due to the road is 200N energy E turns back when it reaches x = ±xm. If V and K
and is directly opposed to the motion. The work done by indicate the PE and KE, respectively of the particle at x =
the cycle on the road is +xm, then which of the following is correct?
(a) + 2000 J (b) – 200 J V(x)
(c) zero (d) – 20,000 J
5. A body is falling freely under the action of gravity alone in
vaccum. Which of the following quantities remain constant
x
during the fall? –xm xm
(a) Kinetic energy
(b) Potential energy (a) V = O, K = E (b) V = E, K = O
(c) V < E, K = O (d) V = O, K < E
Work, Energy and Power 157

9. Two identical ball bearings in contact with each other and KE KE


resting on a frictionless table are hit head-on by another ball
bearing of the same mass moving initially with a speed v as
shown in figure. (a) (b)

1 2 3 t

KE KE

v (c) (d)
If the collision is elastic, which of the following (figure) is a
possible result after collision?
13. Which of the diagrams shown in figure represents variation
1 of total mechanical energy of a pendulum oscillating in air
(a) as function of time?
v=0 v/2 E E

1 2 3
(a) (b)
t
(b) t

.IN
v=0 v
E E
1 2 3
AL
(c) (c) (d)
v/3 t
N
t

1 2 3
R

14. A mass of 5 kg is moving along a circular path of radius 1 m.


(d) If the mass moves with 300 rev/min, its kinetic energy would
U

be
(a) 250 p2 (b) 100 p2
JO

v/1 v/2 v/3


10. A body of mass 0.5 kg travels in a straight line with velocity (c) 5 p2 (d) 0
v = a x3/2 where a = 5 m–1/2s–1. The work done by the net 15. A raindrop falling from a height h above ground, attains a
U

force during its displacement from x = 0 to x = 2 m is near terminal velocity when it has fallen through a height
(a) 15 J (b) 50 J (3/4)h. Which of the diagrams shown in figure correctly
ED

shows the change in kinetic and potential energy of the


(c) 10 J (d) 100 J
drop during its fall up to the ground?
11. A body is moving unidirectionally under the influence of a
source of constant power supplying energy. Which of the h PE
PE h
diagrams shown in figure correctly shown the displacement-
time curve for its motion?
(a) (b) h/4
KE
d d KE
t t
x
h
(a) (b) PE h KE
(c) (d) PE
t t
KE
O t O t
d d
16. In a shotput event an athlete throws the shotput of mass 10
kg with an initial speed of 1 m s–1 at 45° from a height 1.5 m
(c) (d) above ground. Assuming air resistance to be negligible
and acceleration due to gravity to be 10 m s–2, the kinetic
t t energy of the shotput when it just reaches the ground will
12. Which of the diagrams shown in figure most closely shows be
the variation in kinetic energy of the earth as it moves once (a) 2.5 J (b) 5.0 J
around the sun in its elliptical orbit? (c) 52.5 J (d) 155.0 J
EBD_7179
158 PHYSICS

17. Which of the diagrams in figure correctly shows the change 22. A particle with total energy E is moving in a potential energy
in kinetic energy of an iron sphere falling freely in a lake region U(x). Motion of the particle is restricted to the region
having sufficient depth to impart it a terminal velocity? when [NEET Kar. 2013]
(a) U(x) > E (b) U(x) < E
(c) U(x) = O (d) U(x) £ E
KE KE 23. One coolie takes 1 minute to raise a suitcase through a
(a) (b) height of 2 m but the second coolie takes 30 s to raise the
t O t same suitcase to the same height. The powers of two coolies
O Depth Depth
are in the ratio of [NEET Kar. 2013]
(a) 1 : 2 (b) 1 : 3
(c) 2 : 1 (d) 3 : 1
KE KE 24. A body of mass (4m) is lying in x-y plane at rest. It suddenly
(c) (d) explodes into three pieces. Two pieces, each of mass (m)
t t move perpendicular to each other with equal speeds (v).
O Depth O Depth
The total kinetic energy generated due to explosion is :

.IN
18. A cricket ball of mass 150 g moving with a speed of 126 km/ 3
h hits at the middle of the bat, held firmly at its position by (a) mv2 (b) mv 2 [2014]
2
the batsman. The ball moves straight back to the bowler AL
after hitting the bat. Assuming that collision between ball (c) 2 mv2 (d) 4 mv2
and bat is completely elastic and the two remain in contact 25. A particle of mass m is driven by a machine that delivers a
for 0.001s, the force that the batsman had to apply to hold constant power of k watts. If the particle starts from rest the
N
the bat firmly at its place would be force on the particle at time t is [2015]
R

(a) 10.5 N (b) 21 N (a) mk t –1/2 (b) 2mk t –1/2


U

(c) 1.05 × 104 N (d) 2.1 × 104 N


NEET/AIPMT (2013-2017) Questions 1
JO

mk –1/2
(c) mk t –1/2 (d) t
2 2
19. A uniform force of (3iˆ + ˆj) newton acts on a particle of
mass 2 kg. The particle is displaced from position (2$i + k$ ) 26. Two similar springs P and Q have spring constants K P and
U

meter to position (4$i + 3 $j - k$ ) meter. The work done by KQ, such that KP > KQ. They are stretched, first by the
ED

the force on the particle is [2013] same amount (case a,) then by the same force (case b). The
work done by the springs WP and WQ are related as, in
(a) 6 J (b) 13 J
case (a) and case (b), respectively [2015]
(c) 15 J (d) 9 J
(a) WP = WQ ; WP = WQ
20. An explosion breaks a rock into three parts in a horizontal
(b) WP > WQ ; WQ > WP
plane. Two of them go off at right angles to each other. The
first part of mass 1 kg moves with a speed of 12 ms–1 and (c) WP < WQ ; WQ < WP
the second part of mass 2 kg moves with speed 8 ms–1. If (d) WP = WQ ; WP > WQ
the third part flies off with speed 4 ms–1 then its mass is 27. A block of mass 10 kg, moving in x direction with a constant
(a) 5 kg (b) 7 kg [2013] speed of 10 ms–1, is subject to a retarding force F = 0.1 × J/m
(c) 17 kg (d) 3 kg during its travel from x = 20 m to 30 m. Its final KE will be:
21. A person holding a rifle (mass of person and rifle together (a) 450 J (b) 275 J [2015]
is 100 kg) stands on a smooth surface and fires 10 shots (c) 250 J (d) 475 J
horizontally, in 5 s. Each bullet has a mass of 10 g with a 28. The heart of man pumps 5 litres of blood through the arteries
muzzle velocity of 800 ms–1. The final velocity acquired by per minute at a pressure of 150 mm of mercury. If the density
the person and the average force exerted on the person are
of mercury be 13.6 ×103 kg/m3 and g = 10m/s2 then the
[NEET Kar. 2013]
power of heart in watt is : [2015 RS]
(a) –1.6 ms–1; 8 N (b) –0.08 ms–1; 16 N
(a) 2.35 (b) 3.0
(c) – 0.8 ms–1; 8 N (d) –1.6 ms–1; 16 N
(c) 1.50 (d) 1.70
Work, Energy and Power 159

29. A ball is thrown vertically downwards from a height of 20 m r r r r r r r r


(a) r1.v1 = r2 .v 2 (b) r1 ´ v1 = r2 ´ v2
with an initial velocity v0. It collides with the ground loses r r r r
50 percent of its energy in collision and rebounds to the r r r r r1 - r2 v 2 - v1
(c) r1 - r2 = v1 - v2 (d) r r = r r
same height. The initial velocity v0 is : [2015 RS] | r1 - r2 | | v 2 - v1 |
(Take g = 10 ms–2) 32. A body of mass 1 kg begins to move under the action of a
r
(a) 20 ms–1 (b) 28 ms–1 time dependent force F=(2tiˆ+3t 2 ˆj) N, where î and ĵ are
(c) 10 ms –1 (d) 14 ms–1 unit vectors alogn x and y axis. What power will be
30. On a frictionless surface a block of mass M moving at speed developed by the force at the time t? [2016]
v collides elastically with another block of same mass M 2 3
(a) (2t + 3t )W 2 4
(b) (2t + 4t )W
which is initially at rest. After collision the first block moves (c) (2t3 + 3t4) W (d) (2t3 + 3t5)W
v 33. A particle of mass 10 g moves along a circle of radius 6.4 cm
at an angle q to its initial direction and has a speed . The
3 with a constant tangential acceleration. What is the
second block's speed after the collision is : [2015 RS] magnitude of this acceleration if the kinetic energy of the
particle becomes equal to 8 × 10–4 J by the end of the
3 3 second revolution after the beginning of the motion ? [2016]
(a) v (b) v
4 2 (a) 0.1 m/s2 (b) 0.15 m/s2
(c) 0.18 m/s 2 (d) 0.2 m/s2
3 2 2

.IN
(c) v (d) v 34. Consider a drop of rain water having mass 1 g falling from a
2 3 height of 1 km. It hits the ground with a speed of 50 m/s.
r Take 'g' constant with a value 10 m/s2.
31. Two particles A and B, move with constant velocities v1
r r
AL The work done by the (i) gravitational force and the (ii)
and v 2 . At the initial moment their position vectors are r1 resistive force of air is [2017]
r
and r2 respectively. The condition for particles A and B for (a) (i) 1.25 J (ii) –8.25 J
N
their collision is: [2015 RS] (b) (i) 100 J (ii) 8.75 J
R

(c) (i) 10 J (ii) – 8.75 J


(d) (i) – 10 J (ii) –8.25 J
U
JO
U
ED
EBD_7179
160 PHYSICS

Hints & Solutions


EXERCISE - 1 s s
Become = Ek µ Þ E k µ
1. (a) 2. (b) 3. (d) 4. (a) 5. (c) v s/t
6. (c) 7. (a) 8. (b) Þ Ek µ t
9. (b) Because water enters into the vessel A, it becomes
heavier. Gravity helps it sink. External work required mv 2 ms 2
19. (d) = 3
for immersing A is obviously less than that for t t
immersing B. since both P and m are constants
10. (c) Weight Mg moves the centre of gravity of the spring
(0 + l ) s2
through a distance = l/2 \ = constant
2 t3
\ Mechanical energy stored = Work done = Mg l/2. m1 v1 + m 2 v 2 a ´ b + 0 a (b)
20. (c) v= = = .
11. (d) From v = u + at, v1 = 0 + at1 m1 + m 2 a+c a+c
v1 1/ 2
\ a= æh ö

.IN
t1 21. (d) As e n = çç n ÷÷
è h0 ø
F = ma = m v1 / t1
\ h n = e 2 n h 0 = e 2 ´ 2 h = e 4 h.
v1
AL
t 22. (a) As u2 = 0 and m1 = m2, therefore from
Velocity acquired in t sec = at = m1 u1 + m2 u2 = m1 v1 + m2 v2 we get u1 = v1 + v2
t1
N
v 2 - v1 v 2 - v1 1 - v1 / v 2
m v1 v1 t m v12 t Also, e = = = ,
Power = F ´ v = ´ = u1 v 2 + v1 1 + v1 / v 2
R

t1 t1 t12
v1 1 - e
=
U

12. (d) W = F × s which gives


v2 1 + e
1
JO

W µ (x) \ W µ x 0
x 23. (d) Work done = Fdx ò
13. (b) At the top of flight, horizontal component of
U

velocity = u cos 45º = u / 2 24. (b)


m1 m2 m1 + m 2
ED

1 æ u ö
2
1 æ m u 2 ö÷ 1
\ K.E. = m = ç = K.
2 çè 2 ÷ø 2 çè 2 ÷ø 2 u v u=0 u = v1
14. 2 2
(a) From v - u = 2 a s applying conservation of momentum
m1
v2 m1v + m 2 (0) = (m1 + m 2 ) v1 Þ v1 = ( m + m ) v
0 - v 20 = 2 (m g) s \ s= 0 1 2
2mg
as m1 < (m1 + m 2 ) so velocity decreases.
r
15. (b) If L = 0 Þ K.E may or may not be zero. (m1 - m 2 ) u1
r 25. (b) v1 = As v1 is negative and less than
If K.E = 0 Þ L = 0 . m1 + m 2
16. (d) Power is defined as the rate of doing work. For the u1, therefore, m1 < m2.
automobile, the power output is the amount of work
done (overcoming friction) divided by the length of EXERCISE - 2
time in which the work was done. 1. (a) W = F s cos 90º = zero
2. (c) As the cord is trying to hold the motion of the block,
1 2 1 2 1 work done by the cord is negative.
17. (b) W= kl 2 - kl1 = k (l 22 - l12 )
2 2 2
æ gö - 3M gd
W = – M (g – a) d = - M ç g - ÷ d =
1 è 4 ø 4
18. (b) F µ (given)
v 3. (c) W = F s cos q = 10 × 2 cos 60º = 10 J.
rr
Then W = E k = F.s 4. (c) W = F.s = (5 î + 2 ˆj).(2 î + 3 ĵ) = 10 + 6 = 16 J.
Work, Energy and Power 161

5. (c) From horizontal position to lowest position, height 1


through which the bob falls = l 15. (a) W1 = ´ 5 ´10 3 (0.05) 2
2
\ At lowest position, v = 2 l g 1
Þ W2 = ´ 5 ´10 3 (0.10) 2
K.E. at lowest point 2
1 1 1
= m v 2 = m (2 l g) = mlg = 10 × 1 × 9.8 = 98 J. \ DW = ´ 5 ´10 3 ´ 0.15 ´ 0.05 = 18.75J.
2 2 2
x1 x1 x 16. (a) m1 = 0.2 kg, m 2 = 0.4 kg, v1 = 0.3m / s, v 2 = ?
é1 ù 1
6. (b) W= ò ò
F dx = c x dx = ê c x 2 ú
ë2 û0
Applying law of conservation of momentum
0 0 0.2 ´ 0.3
m1v1 – m2v2 = = 0.15 m / s.
1 1 0.4
= c ( x12 - 0) = c x 12
2 2 17. (c) v = - m1 v1 = - 3 ´ 16 = -8 m / s
2
P 100 ´ 746 m2 6
7. (d) Forward thrust, F = = = 3730 N.
v 20 1 1
E2 = m 2 v 22 = ´ 6( - 8) 2 = 192 J.
1 2 1 2 2 2
8. (c) As mA vA = mB vB
2 2 1 1 2V V
18. (d) V= k ( x ) 2 = k (2) 2 or k = =

.IN
2 2 4 2
vA mB
= ;
vB mA 1 1 æVö
V¢ = k (10) 2 = ´ ç ÷ (10) 2 = 25V
AL 2 2 è2ø
PB mB vB mB mA mB 1 19. (b) If P = momentum, K = kinetic energy, then
= = = =
PA mA vA mA mB mA 3 P12 = 2 mK1, P22 = 2mK 2
N

2
R

9. (a) From m1 v1 + m 2 v2 = 0, æ P2 ö K 2 3K1 P 3


\ç ÷ = = =3\ 2 = = 1.732
è P1 ø K1 K1 P1 1
U

-m1v1 4
v2 = =-
m2 234 p1 2m1K1
JO

20. (b) =
10. (b) Total energy at the time of projection p2 2m 2 K 2
1 1 21. (d) Applying the principle of conservation of linear
= m v 2 = ´ 0.1(20) 2 = 20J
U

2 2 momentum, we get
Half way up, P.E. becomes half the P.E. at the top i.e.
ED

3m ´ v = (m ´ 60)2 + (m ´ 60)2 = m ´ 60 2
20
P.E. = = 10J \ K.E. = 20 – 10 = 10J.
2 v = 20 2 m / s

(c) Energy stored (E) = 75 ´ (12) = 9 J


11. (b) Workdone, W =
1
2
(
k x 22 - x12 ) 22.
100
1
=
1 é
k ( 0.15) - (0.05)2 ù
2 As E = mv 2
2 ë û 2

1 2E 2´9
= ´ 800 ´ 0.02 = 8J v= = = 18 m / sec
2 m 1
1- 0 1
p2 \
dE æ dp ö
= 2 ç ÷ = 2 ´ 5% = 10% 23. (c) The uniform acceleration is a = = ms -2
12. (d) As E = E è pø 15 15
2m
2
7 Let v be the velocity at kinetic energy J
13. (d) T = M (g - g / 8) = Mg 9
8
Work done by the cord = T × d cos 180º 1 2 2
therefore ´ 1 ´ v 2 = or v = ms -1
2 9 3
7
= M g d (-1) = -7 M g d / 8. Using v = u + at
8
rr 2 1
= 0 + ´ t Þ t = 10s
14. (b) W = F.x = (5î + 3ˆj + 2k̂).(2î - ĵ) = 10 - 3 = 7 joules 3 15
EBD_7179
162 PHYSICS

24. (d) Let n be the number of bullets that the man can fire in 29. (b) For elastic collision in one dimension
one second.
2m 2 u 2 (m1 - m 2 )u1
\ change in momentum per second = n ´ mv = F v1 = +
[ m= mass of bullet, v = velocity] (Q F is the force) m1 + m 2 (m1 + m 2 )

F 144 ´ 1000 As mass 2m, is at rest, So u 2 = 0


\n = = =3
mv 40 ´ 1200 (8m - 2m )u 3
Þ v1 = = u
mgh 8m + 2 m 5
25. (d) Power supplied =
t Final energy of sphere = (K.E.)f
mgh 100
Power used by crane = ´ 1 æ 3u ö 1
2
æ3ö
2
t 80 = (8m )ç ÷ = (8m) u 2 ´ ç ÷
2 è 5 ø 2 è5ø
1000 ´ 10 ´ 100 100 10 5
= ´ = W
3600 80 36 ´ 8 9
= E = 0.36 E
26. (b) Work done against gravity 25
Wg = 50 × 10 × 30 = 15 kJ 30. (b) The work is done against gravity so it is equal to the
Work done against friction change in potential energy. W = Ep = mgh
4 For a fixed height, work is proportional to weight lifted.
Wf = mmg cos q ´ s = 0.4 ´ 50 ´ 10 ´ ´ 50 = 8 kJ

.IN
Since Johnny weighs twice as much as Jane he works
5
twice as hard to get up the hill.
Total work done = Wg + Wf = 15 kJ + 8 kJ = 23 kJ AL Power is work done per unit time. For Johnny this is
W/Dt. Jane did half the work in half the time, (1/2 W)/
27. (d) Let mass of neutron = m
(1/2 Dt) = W/Dt which is the same power delivered by
then mass of deuterium = 2m Johnny.
[Q it has double nuclides thus has neutron].
N
Let initial velocity of neutron = v and final velocities 4´ 2 8
31. (c) Fraction of energy transferred = =
of neutron and deuterium are v1and v2 respectively. 2 9
R

(1 + 2)
v v=0
U

2
32. (c) p 2 Þ E1 = p1 Þ E = E ´ 4
2m m ® v1 2m ® v 2 E= 2 1
m E 2 p 22
JO

2m
neutron deuterium \ E 2 - E1 = 3E1
Applying conservation of momentum
33. (c) Since the new height gained is half , therefore there is
U

mv + 2m (0) = mv1 + 2mv 2 50% loss of energy.


ED

Þ v = v1 + 2v 2 ...........(i) 2u
34. (a) m1u = m1 + m2u (By condition of linear
applying conservation of energy 3
1 1 1 momentum)
mv2 = mv12 + 2m.v22
2 2 2 1
2
Þ m1u = m 2 v ...... (i)
Þv = v12 2
+ 2v 2 ......(ii) 3

from (i) and (ii), v 2 = ( v - 2v 2 ) 2 + 2v 2 2 | v1 - v 2 |


Also e =
| u 2 - u1 |
Þ v 2 = v 2 + 4v 2 2 - 4v 2 v + 2v 2 2
2u 5
Þ v- =u Þ v= u ...... (ii)
6v2 2 - 4v2 v = 0 3 3
2v v 1 5 m
Þ v2 = & v1 = - From (i) and (ii), m1u = m 2 u Þ 1 = 5
3 3 3 3 m2
now fractional change in kinetic energy
mg
1 1 2 35. (c) At Equilibrium , ky = mg Þ k =
Ki - K f mv 2 - mv12 v 2 - v y
= = 2 2
= 9 =8
Ki 1 2
mv2 v 9 1 æ mg ö 2 1
U= ç ÷ y = mgy
2 2 çè y ÷ø 2
W 25 1
28. (c) W = F s cos q, cos q = = = , q = 60º. 36. (a) Loss in K.E = Area under the curve
F s 5 ´ 10 2
Work, Energy and Power 163

Kinetic energy after the impact


1
mgh + mv 2
2 60
37. (b) In this case, P = = ´ mg ´ 10 = 6mg
t 100
If the ball rises to a height h, then mgh = 6 mg.
mé v2 ù Hence, h = 6 m.
Þ P= êgh + ú
t ëê 2 ûú dU
44. (d) F=- = b - 2ay
dy
10 é 10 ´ 10 ù
Þ P= 10 ´ 10 + W = 1500W
1 êë 2 úû dv
45. (b) Fv = P . or M v=P
dt
m1 v2
38. (c) By conservation of linear momentum, =
P
m 2 v1
That is ò vdv = ò M dt
4 3
rx pr Hence v = [2Pt / M]1/ 2
or 3 1 = 1 Þ r1 = 1
4 2 r2 21 / 3
r x pr23 46. (c)
3
3 cm x
m0 m0 u

.IN
2 2 2
39. (c) mc = c = c
v2 (0.5c)2 u v=0
1- 1-
c2 c2
AL 2

m0 m0 Case I :
= c2 = c 2 = 1.15m 0 c 2
1 - 0.25 0.75
N
2
æuö 2
Change in energy = 1.15 m0c2 – m0c2 ç ÷ - u = 2. a . 3
è2ø
R

= 0.15 × 9.1 × 10–31 × (3 × 108)2


= 12.285 × 10–15 J
3u 2 u2
U

12.285 ´ 10 -15 or – = 2. a. 3 Þ a = –
= MeV 4 8
JO

1.6 ´ 10 -13 Case II :


= 0.07678 MeV æ u2 ö
æuö
2
u2 ç- ÷ ×x
0 - ç ÷ = 2. a. x or – = 2.
U

12 4 ç 8 ÷
40. (b) F = ´ 1000 ´ 10 N = 1200 N è 2ø è ø
100
ED

P = Fv = 1200 N × 15 ms–1 = 18 kW. Þ x = 1 cm


41. (b) Components of velocity before and after collision Alternative method : Let K be the initial energy and F
parallel to the plane are equal, So be the resistive force. Then according to work-energy
v sin 60° = u sin 30°.......(1) theorem,
Components of velocity normal to the plane are related W = DK
to each other 2
1 1 ævö
v cos 60° = e u (cos 30°) ........(2) i.e., 3F = mv2 - m ç ÷
2 2 è2ø
cos 60°
Þ cot 60° = e cot 30° Þ e =
cot 30° 1 æ 1ö
3F = mv 2 ç 1 - ÷
1 2 è 4ø
3 1
Þ e= Þ e= . 3æ1 ö
3 3 3F = ç mv2 ÷ ...(1)
4è2 ø
42. (d) Work done on the body is gain in the kinetic energy.
Acceleration of the body is a = V/T. 2
1 æ vö 1
and Fx = m ç ÷ - m(0) 2
V 2 è 2ø 2
Velocity acquired in time t is v = at = t
T
1æ1 2ö
V2 t 2 i.e., ç mv ÷ = Fx ...(2)
K.E. acquired µ v2. That is work done µ 4è2 ø
2
T Comparing eqns. (1) and (2)
43. (d) Kinetic energy of ball when reaching the ground F = Fx
= mgh = mg × 10 or x = 1 cm
EBD_7179
164 PHYSICS

47. (b) Let the velocity and mass of 4 kg piece be v1 and m1 55. (a) Let M be the mass of shell. Applying law of
and that of 12 kg piece be v2 and m2. conservation of linear momentum
Applying conservation of linear momentum
æ MV ö M
m2v2 = m1v1 MV cos q = çè - cos q÷ + v
ø 2
2
12 ´ 4
Þ v1 = = 12 ms -1 M M
4 ie, MV cos q + Vcos q = v
2 2
1 1
\ K.E.1 = m1v12 = ´ 4 ´144 = 288 J or v = 3 Vcos q .
2 2 56. (d) As bob B is of same material and same mass as the
48. (d) Velocity will increase when force is along the direction bob A, therefore, on elastic collision, their velocities
of displacement i.e. F̂ = d̂ . are exchanged. Bob A comes to rest and B moves with
the velocity of A.
49. (b) When body is lowered gradually, its weight acts at
57. (c) As velocities are exchanged on perfectly elastic
C.G. of the spring. When same body is allowed to fall
collision, therefore masses of two objects must be
freely, the same weight acts at lower end of the spring.
equal.
In the latter case, original length (L) of spring is double.
As DL µ L, therefore, DL becomes twice in second ma
\ = 1 or m a = m b .
case i.e. 2x. mb
50. (d) If V is velocity of combination (bag + bullet), then

.IN
from principle of conservation of linear momentum dv
58. (c) a = - kx Þ = -kx
mv dt
(m + M) V = m v or V =
(m + M) dx dx
AL Also = v or dt =
dt dv
1 m 2 v2
K.E. = (m + M) V 2 = 2 v x
2(m + M) vdv
N
2
\ = -kx Þ ò v dv = -ò kxdx
51. (c) Workdone W = [ML2 T–2] dx
R

v1 0
It shows that W µ (LT -1 )2 i.e. W µ v 2 .
U

kx2 Þ 1 m v 2 - v 2 = 1 m æ - kx ö
( )
2
\ graph between W & v is a parabola.
( v22 - v12 ) =- 2
2 1 ç
2 è 2 ø
÷
JO

Alternatively : 2
According to work energy theorem
\DK a x2
1 1 1
W= mv 2 - mu 2 = mv 2 (Q u = 0) 1
U

2 2 2 59. (c) Ki = m1u12 ,


2
Þ W µ v2
ED

1 m - m2
52. (c) Applying principle of conservation of linear momentum K f = m1v12 , v1 = 1 u1
2 m1 + m 2
(2 m) V = (m v) 2 + (m v)2 = m v 2 Fractional loss
1 1
v 2 v m1u12 - m1 v12
\ V= = Ki - Kf
2 2 =2 2
Ki 1
53. (a) Since block does not slide on wedge so displacement m1u 12
2
is zero & hence work done by force is zero.
u = 1-
v12
= 1-
(m1 - m 2 )2 4m1m 2
=
u 12 (m1 + m 2 )2 (m1 + m 2 )2
f
Kf 4n
m 2 = m ; m1 = nm 1- =
K i (1 + n )2
q
in
gs mg Energy transfer is maximum when K f = 0
m q
4n
54. (c) If mass of nucleus is m, mass of a particle is m1 & = 1 Þ 4n = 1 + n 2 + 2n Þ n 2 + 1 - 2 n = 0
mass of remaining nucleus is m2, then from the law of (1 + n ) 2

conservation of momentum.
r r r (n - 1)2 = 0 n = 1 ie. m 2 = m , m1 = m
mv = m1v1 + m 2 v2 Transfer will be maximum when both masses are equal
initial momentum momentum of momentum of
of nucleus a particle remaining nucleus and one is at rest.
Work, Energy and Power 165

66. (c) By law of conservation of momentum,


10 ´ 14 + 4 ´ 0
60. (c) Vc = = 10 m / s ; since spring force is Mu = MV + mv ....(i)
10 + 4
internal force. | v1 - v 2 |
Also e = Þ Mu = Mv - MV ....(ii)
61. (a) For inelastic collision, linear momentum is conserved | u1 - u 2 |
v1 From (i) and (ii), 2Mu = (M + m)v
Þ mv1 = 2mv 2 Þ v 2 =
2 2uM 2u
Loss in K.E. = Gain in P.E. Þv = Þ v=
M+m m
1+
1 1 M
= mv12 - (2m) v 22 = 2 mgh
2 2
mv2
67. (d) Centripetal force =
mv12 mv12 mv 2
R
Þ 4 mgh = mv12 – = =
2 2 2
æ1 ö 2 2 K 2aS
= ç mv 2 ÷ = =
v2 è2 øR R R
Þ h=
8g 68. (a) As the block is at rest at P.
62. (d) P = F´ v Þ P = F a t \ Pµt mg sin q = mmg cos q

.IN
2h
dy = d æç x ö÷ = x
2
63. (c) t AB =
g m = tan q = slope =
dx dx çè 2L ÷ø L
A AL
2h1
t BC + t CB = 2 h C
g
D
N
2e 2 h 2h
=2 = 2e h1 y
g g N P
R

h2
U

2h q mg cosq
t BD + t DB = 2e 2 mg sinq mg
g B
JO

x
\ Total time taken by the body in coming to rest x
\
= 0.8
2h 2h 2h L
U

= + 2e + 2e 2 + .........
g g g Þ x = 0.8 × 10 = 8 m
ED

69. (d) Work done by centripetal force is zero.


2h 2h dU
= + 2e [1 + e + e 2 + .........] 70. (a) F=-
g g dx
71. (c) The change in momentum in the ball after the collision
2h 2h 1 = 2h é1 + e ù æ 1 + e ö with surface is m(0–v) = –mv
+ 2e ´ =t
g ëê1 - e ûú èç 1 - e ø÷
=
g g 1- e Since n balls impinge elastically each second on the
surface, then rate of change of momentum of ball per
mg second is
64. (d) Initially, 3k = mg or k =
3 mvn (consider magnitude only)
New force constant of longer part Now According to Newton’s second law
3 3 mg mg rate of change of momentum per second of ball = force
k' = k= ´ = experienced by surface.
2 2 3 2
Finally, k ' y = mg 72. (d)
mg mg
y= = ´ 2 = 2cm h=1.5m
k' mg
65. (d) m = 10 ´ 0.8kg = 8kg
height of iron chain = 5m
The mass of water is m = 1 × 103 kg
mgh 8 ´ 10 ´ 5 \ The increase in potential energy of water is
P= = W = 40W
t 10 = mgh = (1 × 103) (10) (1.5) = 15 kJ
EBD_7179
166 PHYSICS

73. (d) Loss in velocity, Dv = v1 – v 2 = 2gh1 – 2gh 2


2
74. (c) U = (1/ 2)Mv \ fractional loss in velocity
75. (b) P. E + K.E = constant, mass being constant
Dv 2gh1 – 2gh 2 h2
gh + v2/2 = constant =1 –
= =
2gh1 h1
v1
76. (c) K = p2 / 2m = mmgx

Hence, x = p 2 / 2m 2mg. 1.8 2


=1– = 1 – 0.36 = 1 – 0.6 = 0.4 =
5 5
r dpr 82. (c) Change in momentum along the wall
77. (c) F = ˆ sin t + ˆj2 cot t
= -i2
dt = mv cos60º – mv cos 60º = 0
rr r r Change in momentum perpendicular to the wall
Hence F.p = 0 , hence angle between F and p is 90° = mv sin60º – (– mv sin60º) = 2mv sin60º
r
78. (c) Here ˆimv + ˆjmv = 2mV Change in momentum
\ Applied force =
Time
r v
That is V = (iˆ + ˆj) 2 mv sin 60º 2 ´ 3 ´ 10 ´ 3
2 = =
0.20 2 ´ 0.20
v v
Hence V = ´ 2= . Hence v = 5 ms–1

.IN
2 2 = 50 ´ 3 3 = 150 3 newton
79. (c) Apply conservation of momentum, 83. (b) Speed of bomb after 5 second,
m1v1 = (m1 + m2)v v = u – gt = 100 –10×5 = 50m/s
AL Momentum of 400 g fragment
m1 v1
v = (m + m ) 400
1 2
= ´ (-25) (downward)
1000
N
Here v1 = 36 km/hr = 10 m/s,
600
R

m1 = 2 kg, m2 = 3 kg Momentum of 600g fragment = v


1000
10 ´ 2
U

v= = 4 m/s Momentum of bomb just before explosion


5 = 1 × 50 = 50
JO

1 From conservation of momentum


K.E. (initial) = ´ 2 ´ (10) 2 = 100 J Total momentum just before collision = Total
2
momentum just after collision
U

1 400 600
K.E. (Final) = ´ (3 + 2) ´ (4) 2 = 40 J Þ 50 = - ´ 25 + v
2
ED

1000 1000
Loss in K.E. = 100 – 40 = 60 J Þ v = 100 m/s (upward)
Alternatively use the formula 84. (d) If l is length of pendulum and q be angular amplitude
1 m1m2 then height.
-DE k = ( u1 - u 2 )2
2 ( m1 + m 2 ) A

q
dx
80. (a) x = 3t –4t2 + t3 = 3 - 8t + 3t 2 l
dt

d 2x C
Acceleration = = -8 + 6 t P
2 h
dt
Acceleration after 4 sec = –8 + 6 × 4 = 16 B

Displacement in 4 sec = 3 ×4 – 4 × 42 + 43 = 12 m h = AB – AC = l – l cos q = l(1 – cos q)


\ Work = Force × displacement At extreme position, potential energy is maximum and
= Mass × acc. × disp. = 3 × 10–3 × 16 × 12 = 576 mJ kinetic energy is zero; At mean (equilibrium) position
81. (b) According to principle of conservation of energy potential energy is zero and kinetic energy is
Potential energy = kinetic energy maximum, so from principle of conservation of energy.

1 (KE + PE ) at P = (KE + PE ) at B
Þ mgh = mv2 Þ v = 2gh
2 1
0 + mgh = mv 2 + 0
If h 1 and h2 are initial and final heights, then 2
Þ v1 = 2gh 1 , v 2 = 2gh 2 Þ v = 2gh = 2gl(1– cos q)
Work, Energy and Power 167

85. (b) From law of conservation of momentum, before for stable equilibrium
collision and after collision linear momentum (p) will
be same. d 2U
should be positive for the value of r.
or initial momentum = final momentum. dr 2
p2 d 2U 6A 2B
E= = -
2m here 2 4 is +ve value for
dr r r3
According to question,
2A
E1 p 2 2m 2 E m r= So
= 1 ´ Þ 1 = 2 [p = p ] B
E 2 2m1 p 22 E 2 m1 1 2
r
91. (d) Given : F = 3i$ + $j
1 1 2 2 2 2
86. (b) mv 2 = kx 2 Þ mv = kx or 0.5 × (1.5) = 50×x ur r
$ $ uur
2
\ x = 0.15 m
2 ( )$ $
r1 = 2i + k , r2 = 4i + 3j - k ( )
r uur ur r
87. (a) Clearly v1 = 2 ms –1, v2 = 0
m1 = m (say), m2 = 2m
$ $
( $ $
r = r2 - r1 = 4i + 3j - k – 2i + k ) ( )
v1' = ?, v'2 = ? r
or r = 2i$ + 3$j – 2k$
v1 '- v2 '

.IN
e= v -v ....(i) rr
2 1
So work done by the given force w = f .r
By conservation of momentum,
( )(
= 3i$ + $j . 2i$ + 3j$ - 2k$ = 6 + 3 = 9J)
2m = mv1' + 2mv2' ... (ii)
AL
v2 '- v1 ' 92. (a)
From (i), 0.5 = y
N
2
\ v2' = 1 + v1' 2 kg m2
R

From (ii), 2 = v1'+ 2 + 2 v1' 8 m/sec Presultant


Þ v1 = 0 and v2 = 1 ms–1
U

88. (b) Power exerted by a force is given by 12 m/sec


m1
JO

P = F.v c x
When the body is just above the earth’s surface, its /se 1 kg
4m
velocity is greatest. At this instant, gravitational force 3
U

is also maximum. Hence, the power exerted by the m


gravitational force is greatest at the instant just before
ED

the body hits the earth.


89. (a) As the two masses stick together after collision, hence Presultant = 122 + 162
it is inelastic collision. Therefore, only momentum is
conserved. = 144 + 256 = 20
m3v3 = 20 (momentum of third part)
2v
20
or, m3 = = 5 kg
4
m v x 93. (c) p = 2mK
3m
r p ' = 2m[K + 3K] = 2p
\ mviˆ + 3m(2v)ˆj = (4m)v
Dp ´ 100 2p - p
r v 6 = ´ 100 = 100%
v = ˆi + vjˆ p p
4 4
v 3 94. (c) Let m and v be the mass and the velocity of the body.
= ˆi + vjˆ Then initial K.E. ,
4 2
90. (b) For equilibrium 1 2
Ki = mv
dU -2 A B 2
=0 Þ + 2 =0
dr r3 r m
Now, mass =
2A 2
r=
B velocity = 2v
EBD_7179
168 PHYSICS

When the man does not squat and gets straight up in


1æmö
\ Final K.E. = ç ÷ (2v )
2 that case friction ( f ) » 0
2è 2 ø R (Reactional force) » mg
Hence, the reaction force (R) is larger when squatting
1
= (2mv 2 ) and become equal to mg when no squatting.
2 4. (c) According to the question, work done by the frictional
force on the cycle is :
æ 1 2ö
= 2 çè mv ÷ø = 200 × 10 = –2000 J
2 As the road is not moving, hence work done by the
= 2 Ki cycle on the road is zero.
95. (a) Power of engine = Force × velocity 5. (c) As the body is falling freely under gravity and no
= Fv external force act on body in vaccum so law of
conservation, the potential energy decreases and
107 kinetic energy increases because total mechanical
Here, mass of engine = kg = 106 kg
10 energy (PE + KE) of the body and earth system will be
F = frictional force remain constant.
= 0.5 kgf per quintal 6. (c) According to the question, consider the two bodies
= (0.5 × 10) N per quintal as system, the total external force on the system will
= 5 N per quintal be zero.
Hence, in an inelastic collision KE does not conserved

.IN
æ 5 ´ 106 ö but total linear momentum of the system remain
=ç ÷N
è 100 ø conserved.
= (5 × 104) N 7. (c) As the (inclined surface) are frictionless, hence,
AL mechanical energy will be conserved. As both the
and v = 36 km h–1
tracks having common height, h (and no external force
36 ´1000
kmh -1 = 10 ms–1 acts on system).
N
=
60 ´ 60 KE & PE of stone I at top = KE + PE at bottom of I.
\Power of engine= (5 × 104 × 10)W
R

From conservation of mechanical energy,


= 5 × 105 W 1
U

= 500 kW 0 + mv12 = mgh + 0


2
96. (b) 97. (b)
JO

98. (a) When frictional force is opposite to velocity, kinetic Þ v1 = 2 gh similarly v2 = 2 gh


energy will decrease. Hence, speed is same for both stones.
99. (c) 100. (d) For stone I, acceleration along inclined plane a1 = g
U

sin q1
EXERCISE - 3 Similarly, for stone II a2 = g sin q2
ED

Exemplar Questions sin q1 < sin q2 Thus, q2 > q1 hence a2 > a1.
a2 is greater than a1 and both length for track II is also
1. (b) When electron and proton are moving under influence
less hence, stone II reaches earlier than stone I.
of their mutual forces, then according to the flemings
8. (b) Total Mechanical energy is E = PE + KE at any instant.
left hand rule, the direction of force acting on a charge
When particle is at x = xm i.e., at extreme position,
particle is perpendicular to the direction of motion.
partical returns back and its velocity become zero for
In magnetic field, work-done = F. s. cosq
an instant. Hence, at x = xm; x = 0, K.E. = 0.
= F . s. cos 90° = 0. From Eq. (i),
So magnetic forces do not work on moving charge
1
particle. E = PE + 0 = PE = V(xm) = kxm2
2. (c) Forces between two protons is same as that of between 2
proton and a positron. but at mean position at origin V(xm) = 0.
As positron is much lighter than proton, it moves away 9. (b) If two bodies of equal masses collides elastically, their
through much larger distance compared to proton. velocities are interchanged.
When ball 1 collides with ball-2, then velocity of ball-
Work done = Force × Distance
1, v1 becomes zero and velocity of ball-2, v2 becomes
As forces are same in case of proton and positron but
v, i.e., similarly then its own all momentum is mV.
distance moved by positron is larger, hence, work done
on positron will be more than proton. So, v1 = 0 Þ v2 = v, P1 = 0, P2 = mV
3. (d) When the man squatting on the ground he is tilted Now ball 2 collides to ball 3 and its transfer it's
somewhat, hence he also has to apply frictional force momentum is mV to ball 3 and itself comes in rest.
besides his weight. So, v2 = 0 Þ v3 = v, P2 = 0, P3 = mV
R (reactional force) = friction force (f) + mg So, ball 1 and ball 2, become in rest and ball 3 move
i.e. R > mg with velocity v in forward direction.
Work, Energy and Power 169

10. (b) As we know that, So, option (b) and (c) represents wrongly the variation
in kinetic energy of earth.
x2
r uur x2
r uur
W.D. = ò F × dx = ò ma0 ×dx
x1 x1

As given that, A SUN B


m = 0.5 kg, a = 5 m–1/2 s–1,
work done (W) = ?
v = ax 3/ 2 When the earth is closest to the sun, speed of the
We also know that, earth is maximum, hence, KE is maximum. When the
Acceleration, earth is farthest from the sun speed is minimum hence,
KE is minimum.
dv dv d
a0 = = v× = ax3/ 2 (ax 3/ 2 ) So, K.E. of earth increases (B to A) and then decrease,
dt dx dx variation is correctly represented by option (d).
3 3 13. (c) When a pendulum oscillates in air, due to air resistance
= ax 3/ 2 ´ a ´ ´ x1/ 2 = a 2 x 2 the force of friction acts between bob of pendulum
2 2
and air, so it will lose energy continuously in
3 2 2 overcoming. Therefore, total mechanical energy (KE
Now, Force = ma0 = m a x
2 + PE) of the pendulum decreases continuously with
time and finally becomes zero.

.IN
From (i),
x= 2 Sum of KE and PE can never be negative. So, option
Work done = ò Fdx (a) and (d) are incorrect. Hence option (c) is verifies.
x= 0 AL
14. (a) As given that, mass (m) = 5 kg,
2é3 ù
= ò ê ma 2 x 2 ú dx n = 300 revolution
0 ë2 û Radius (R) = 1 m
N
2 t = 60 sec
3 2 æ x3 ö
= ma ´ çç ÷÷ æ 2pn ö
R

2 è 3 ø0 w=ç ÷ = (300 ´ 2 ´ p) rad / 60s


è t ø
U

1 2 600 ´ p
= ma ´ 8 = rad/s = 10 p rad/s
JO

2 60
linear speed (v) = wR = (10p × 1)
1
´ (0.5) ´ (25) ´ 8 = 50 J
= v = 10p m/s
2
U

11. (b) As given that power = constant 1 2


KE = mv
ED

As we know that power (P) 2


r uur 1
dW F × dx F dx = ´ 5 ´ (10p)2
P= = = 2
dt dt dt
As the body is moving unidirectionally. 1
= 100p 2 ´ 5 ´
Hence, F × dx = Fdx cos 0° = Fdx 2

Fdx = 250p 2J
P= = constant (Q P = constant by question)
dt So, verifies the option (a).
Now, by dimensional formula 15. (b) P.E. is maximum when drop start falling at
F ×v = 0 t = 0 as it fall is P.E. decrease gradually to zero. So, it
[F] [v] = constant rejects the graph (a), (c) and (d).
[MLT–2] [LT–1] = constant K.E. at t = 0 is zero as drop falls with zero velocity, its
[ML2T–3] = constant velocity increases (gradually), hence, first KE also
increases. After sometime speed (velocity) is constant
T3
L2 = this is called terminal velocity, so, KE also become
M
æ 3ö
(As mass of body constant) constant. It happens when it falls ç ÷ height or
è 4ø
L2 µ T 3 Þ L µ T3/ 2 Þ Displacement (d ) µ t 3/ 2
Verifies the graph (b). æ 4ö
remains at ç ÷ from ground, then PE decreases
12. (d) The speed of earth around the sun can never be zero è 4ø
or negative, so the kinetic energy of earth cannot be continuously as the drop is falling continuously.
zero and negative. The variation in PE and KE is best represented by (b).
EBD_7179
170 PHYSICS

16. (d) As given that, h = 1.5 m, v = 1 m/s, m = 10 kg, g = 10 ms–2 20. (a)
By the law of conservation of mechanical energy as y
no force acts on shotput after thrown. 2 kg m2
(PE)i + (KE)i = (PE)f + (KE)f
8 m/sec
1 Presultant
mghi + mvi2 = 0 + (KE) f
2 12 m/sec
1 2 m1
(KE)f = mghi + mvi c x
2 /se 1 kg
4m
Total energy when it reaches ground, so 3
m
1
(KE) f = 10 ´ 10 ´ 1.5 + ´ 10 ´ (1) 2
2 Presultant = 122 + 162
E = 150 + 5 = 155 J.
17. (b) First velocity of the iron sphere = 144 + 256 = 20
m3v3 = 20 (momentum of third part)
V = 2 gh after sometime its velocity becomes 20
constant, called terminal velocity. Hence, according or, m3 = = 5 kg
4
first KE increases and then becomes constant due to
21. (c) According to law of conservation of momentum
resistance of sphere and water which is represented
MV + mnv = 0

.IN
by (b).
18. (c) As given that, - mNv -0.01 kg ´ 10 ´ 800 m/s
ÞV = =
150 M 100
m = 150 g = kg = 0.15 kg Þ – 0.8 m/s
1000
AL
Dt = time of contact = 0.001 s According to work energy theorem,
Average work done = Change in average kinetic energy
126×1000
u = 126 km/h =
N
m/s 1
60 ´ 60 i.e., Fav ´ Sav = mVrms 2

5 2
R

= 126 ´ = 35 m/s F V t 1 V2
18 Þ av max = m rms
U

5 2 2 2
v = –126 km/h = -126 ´ = -35 m/s Þ Fav = 8 N
18
JO

So, final velocity is acc. to initial force applied by 22. (d) As the particle is moving in a potential energy region.
batsman. \ Kinetic energy ³ 0
So, change in momentum of the ball And, total energy E = K.E. + P.E.
U

Þ U(x) £ E
3
Dp = m ( v - u ) = ( -35 - 35) kg-m/s w
ED

20 23. (a) Q Power P =


t
3 21
= (-70) = - P1 t 2 30s 30s 1
20 2 Þ = = = =
As we know that, force P2 t1 1 minute 60s 2
Dp -21/ 2 (t1 = 1 minute; t2 = 30 second given)
F= = N = –1.05 × 10 4 N 24. (b) By conservation of linear momentum
Dt 0.001
Hence negative sign shown that direction of force will v
be opposite to initial velocity which taken positive 2mv1 = 2mv Þ v1 =
2
direction. Hence verify the option (c).
NEET/AIPMT (2013-2017) Questions
v
m
r
19. (d) Given : F = 3i$ + $j
ur r m
$ $ uur
( ) $ $
(
r1 = 2i + k , r2 = 4i + 3j - k ) v

r uur ur r 2m
r = r -
2 1r = ( ) (
$ + 3j$ - k – 2i$ + k$
4i ) v1
r As two masses of each of mass m move perpendicular
or r = 2i$ + 3$j – 2k$ to each other.
rr Total KE generated
So work done by the given force w = f .r
1 1 1
( )( )
= 3i$ + $j . 2i$ + 3j$ - 2k$ = 6 + 3 = 9J =
2
mv 2 + mv 2 + (2m)v12
2 2
Work, Energy and Power 171

29. (a) When ball collides with the ground it loses its 50% of
mv2 3 2 energy
= mv2 + = mv
2 2
1
dw
KEf 1 mVf2
25. (d) As we know power P = = 2 1
dt \ Þ = M
KEi 2 1
1
Þ w = Pt = mV 2 mVi2 2
2 2
Vf 1
2Pt or V =
So, v = i 2
m
dV 2P 1 2gh 1 M
Hence, acceleration a = = . =
dt m 2 t or, 2
Therefore, force on the particle at time ‘t’ V02 + 2gh

2Km 2 1 Km mK –1/2 or, 4gh = V02 + 2gh


.
= ma = = = t
m 2 t 2t 2 \ V0 = 20ms–1
26. (b) As we know work done in stretching spring 30. (d) Here, M1 = M2 and u2 = 0
1 2 V
w= kx u1 = V, V1 = ; V2 = ?
2 3

.IN
where k = spring constant
V1=V/3
x = extension
Case (a) If extension (x) is same, AL
u2=0 M1 q
1 u1=V
W = K x2 M1 M2
2 M2 f
N
So, WP > WQ(Q KP > KQ)
R

F2 V2=?
Case (b) If spring force (F) is same W =
2K
U

From figure, along x-axis,


So, WQ > WP M1u1 + M2u2 = M1V1 cosq + M2V2 cosf ...(i)
JO

27. (d) From, F = ma Along y-axis


F 0.1x dV 0 = M1V1 sinq – M2Vs sinf ...(ii)
a= = = 0.01x = V By law of conservation of kinetic energy
m 10 dx
U

1 1 1 1
v2
x30 M1u12 + M 2 u 22 = M1V12 + M 2 V22 ...(iii)
So, ò vdV = ò 2 2 2 2
ED

dx
v1 20 100 Putting M1 = M2 and u2 = 0 in equation (i), (ii) and
(iii) we get
V2 30
V2 x2 30 ´ 30 20 ´ 20 p
– = = –
2 200 200 200 q + f= = 90°
V1 20 2
= 4.5 – 2 = 2.5 2 2 2
and u1 = V1 + V2
1
2
( )
m V22 – V12 = 10 ´ 2.5 J = – 25J æVö
2
2 é Vù
V2 = ç ÷ + V2 êQ u1 = V and V1 = ú
è3ø ë 3û
Final K.E.
2
1 2 1 2 1 æVö
= mv2 = mv1 – 25 = ´ 10 ´ 10 ´ 10 – 25 or, V2 – ç ÷ = V22
2 2 2 è3ø
= 500 – 25 = 475 J
rr r V2
28. (d) Power F.V = PAV = rghAV V2 - = V22
9
é F ù
êëQ P = A and P = rgh úû
8 2 2 2
or V22 = V Þ V2 = V
9 3
= 13.6 × 103 × 10 × 150 × 10–3 × 0.5 × 10–3/60 ur
31. (d) For collision V B/A should be along
102 r
=
60
= 1.70 watt B ® A ( rA/B )
EBD_7179
172 PHYSICS
ur ur r r
V 2 - V1 r1 - r2 1 1 æ 10 ö 2
So, V - V = mv 2 = E Þ ç ÷ v = 8 × 10–4
2 1 r1 - r2 2 2 è 1000 ø
Þ v2 = 16 × 10–2
V1 V2
Þ v = 4 × 10–1 = 0.4 m/s
Now, using

A B v2 = u2 + 2ats(s = 4pR)
A B
æ 22 6.4 ö
r (0.4)2 = 02 + 2at ç 4 ´ ´ ÷
32. (d) Given force F = 2tiˆ + 3t 2 ˆj è 7 100 ø
According to Newton's second law of motion,
7 ´ 100
r Þ at = (0.4)2 × = 0.1 m/s2
dv 8 ´ 22 ´ 6.4
m = 2tiˆ + 3t 2 ˆj (m = 1 kg)
dt
34. (c) From work-energy theorem,
r
v t
r Wg + Wa = DK.E
ò dv = ò ( 2tiˆ + 3t ˆj) dt
2
Þ
1
0 0 or, mgh + Wa = mv 2 - 0

.IN
r 2
Þ v = t 2 ˆi + t3ˆj
r r 1
10 -3 ´ 10 ´ 103 + Wa = ´ 10 -3 ´ (50) 2
Power P = F·v = (2t iˆ + 3t 2 ˆj) · (t 2 ˆi + t 3 ˆj)
AL 2
= (2t3 + 3t5)W Þ Wa = –8.75 J
N
10 which is the work done due to air resistance
33. (a) Given: Mass of particle, M = 10g = kg
1000 Work done due to gravity = mgh
R

radius of circle R = 6.4 cm = 10–3 × 10 × 103 = 10 J


U

Kinetic energy E of particle = 8 × 10–4J


JO

acceleration at = ?
U
ED
System of Particles
7 and Rotational Motion
CENTRE OF MASS Rigid Bodies
Centre of mass for a system of particles is defined as that point If a body does not undergo any change in shape by the action of
where the entire mass of the system is imagined to be a force, it is said to be rigid.
concentrated. If such body undergoes some displacement, every particle in it
undergoes the same displacement. No real body can be perfectly

.IN
If r1 , r2 , r3 ......... be the position vectors of masses m1, m2, m3 rigid.
....... respectively from the origin O, Rotatory Motion
AL
y A body rotating about a fixed axis then every particle of the
body moves in a circle and the centres of all these circles lie on
axis of rotation. The motion of the body is said to possess
m4 rotational motion.
N
m2
r4 Keep in Memory
R

m3 r2
r3
1. The centre of mass of a system of two identical particles
U

m1
r1 lies in between them on the line joining the particles.
JO

x 2. If m1 = m2
O uur uur ur uur
r (m 1 r1 + m 2 r 2 ) (r + r )
r cm = = 1 2
(m 1 + m 2 ) 2
U

then the centre of mass of the system is so, for particles of equal masses the centre of mass is
ur uur uur
uuur (m1 r1 + m2 r2 + m3 r3 + .....) located at the mean position vector of the particles.
ED

1 n uur
rcm = = å mi ri 3. The position of centre of mass remains unchanged in pure
(m1 + m2 + m3 + ......) M i =1
rotatory motion. But it changes with time in translatory
where M is the total mass of the system of particles. The product
motion or rolling motion.
of mass of the particles and its position vector w.r.t. the reference
4. The position of centre of mass of a body is independent
point is called moment of mass
r of the choice of co-ordinate system.
i.e., moment of mass = m ´ r 5. If we take the centre of mass at the origin, then the sum of
MOTION OF CENTRE OF MASS the moments of the masses at of the system about the
The motion of the centre of mass is governed by the equation r
origin Sm i ri is zero.
r r r r
d 2 (rcm) 6. In pure rotatory motion, the axis of rotation passes through
MAcm = Fext where Acm =
dt 2 the centre of mass.
Momentum conservation of a system of particles : 7. If external force is zero then the velocity of the centre of
In the absence of external forces, the velocity of the centre of mass of a body remains constant.
mass remains constant. 8. The centre of mass and centre of gravity of a body
We have, MAcm = F ext coincide, if the value of g is same throughout the dimension
If Fext = 0 of the body.
d 9. In kinematics and dynamics, whole of the mass of a body
M ( vcm ) = 0
dt can be assumed to be concentrated at the centre of mass.
\ vcm = constant. 10. The location of the centre of mass depends on the shape
Hence, momentum (Mvcm = constant) of the centre of mass and nature of distribution of mass of the body.
system is conserved. (a) The position of centre of mass of continuous bodies
can be found using integration as
EBD_7179
174 PHYSICS

1 1 1
Relationship between angular velocity and linear velocity:
X cm =
M ò
x dm, Ycm =
M
y dm, Z cm =òM
z dm ò z
where, x, y and z are the co-ordinates of small mass
w v
dm and M is the total mass of the system.
(b) The C.M. of a uniform rod of length L is at its middle r q P x
point.
(c) Centre of mass of a uniform semicircular wire is at
æ 2R ö y
ç 0, ÷ , where R is the radius of the semicircular wire.
è p ø r r r
v = w× r
It does not depend on mass.
Þ v = rw sin q
Y
where q is the angle between w & r.
MOMENT OF INERTIA AND RADIUS OF GYRATION
R A rigid body having constituent particles of masses m1, m2,
X ....mn and r1, r2 ... r n be their respective distances from the axis of
O
rotation then moment of inertia is given by,
(d) For symmetrical bodies of uniform mass distribution, n

.IN
the C. M. lies at the geometrical centre I = m1 r12 + m2 r22 + ... + mn rn2 = å m i ri2
i =1
The moment of inertia of continuous mass distribution is given
ANGULAR VELOCITY AND ANGULAR ACCELERATION
AL
by
The angular velocity is defined as the angle covered by the
I = ò r 2 dm
radius vector per unit time. It is denoted by w.
N
where r is the perpendicular distance of the small mass dm from
Y the axis of rotation.
R

Its SI unit is kgm2. It is a tensor.


U

Radius of gyration :
P The radius of gyration of a body about its axis of rotation may
JO

r s be defined as the distance from the axis of rotation at which, if


q the entire mass of the body were concentrated, its moment of
X inertia about the given axis would be same as with its actual
U

O Q
distribution of mass.
Radius of gyration k is given by,
ED

I = MK2
1
Dq
Average angular velocity w = 1 én ù2
Dt ê S mi ri 2 ú
é ù2
I
The unit of angular velocity is rad/sec. or K = ê ú = ê i ú
ëM û êë S mi úû
where, M = Sm i
The instantaneous angular velocity w (similar to instantaneous
linear velocity) is defined as X
Dq d q
w = lim = r4
Dt ®o Dt dt m4
r3 m3
The angular acceleration is the rate of change of angular
velocity. It is denoted by a. k Sm
m2
The average angular acceleration aavg. of a rotating body is r2
w - w1 Dw m1
a avg . = 2 =
r1
t2 - t1 Dt
r
In analogy to linear acceleration a , the instantaneous angular
Y
acceleration is defined as
r12 + r22 + r32 + .....rn2
Dw d w Also, k=
a = lim = n
dt ® o Dt dt
Therefore, radius of gyration (k) equals the root mean square of
The unit of angular acceleration is rad/sec2.
the distances of particles from the axis of rotation.
System of Particles and Rotational Motion 175

GENERAL THEOREMS ON MOMENT OF INERTIA so moment of inertia of whole lamina about AB is


Theorem of perpendicular axis : I = Sm(x+r)2 = Smx2 + Smr2 +2 Smxr
According to this theorem “ the moment of inertia of a plane Where first term on R.H.S is S mx2 = Ic.m. moment of inertia of
lamina (a plane lamina is a 2-dimensional body. Its third lamina about PQ through its centre of mass, second term on
dimension is so small that it can be neglected.) about an axis, R.H.S. is Smr2 = r2Sm = Mr2, M is whole mass of lamina, third
perpendicular to the plane of lamina is equal to the sum of the term on R.H.S is (Smx) r = 0, because Smx is equal to moments of
moment of inertia of the lamina about two axes perpendicular all particles of lamina about an axis PQ, passing through its centre
to each other, in its own plane and intersecting each other at of mass. Hence
the point, where the perpendicular axes passes through it. I = Ic.m. + M.r2
If Ix and Iy be the moment of inertia of a plane lamina (or 2D rigid i.e., the moment of inertia of lamina about AB = its moment of
body) about the perpendicular axis OX and OY respectively, inertia about a parallel axis PQ passing through its centre of mass
which lie in plane of lamina and intersect each other at O, then + mass of lamina×(distance between two axes)2
moment of inertia (Iz) about an axis passing through (OZ) and Example 1.
perpendicular to its plane is given by Three rings each of mass P and radius Q are arranged as
shown in fig. What will be the moment of inertia of the
Z
arrangement about YY’?
Y

.IN
x
o y X
r p(x, y)
1 2
P P
Q Q
AL
Y Q
Ix + Iy = Iz
N
3 P
Let us consider a particle of mass m at point P distance r from
R

origin O, where r = x 2 + y 2
U

s o Ix + Iy = Smy2 + Smx2 = Smr2 Solution : Y’


JO

i.e., Iz = Ix + Iy Moment of inertia of each ring about its diameter


Theorem of parallel axes : 1
= PQ 2 .
(Derived by Steiner) This theorem is true for both plane laminar 2
U

body and thin 3D body. It states that “the moment of inertia of a So total moment of inertia of all three rings about Y Y' is
Itotal = I1 + I2+ I3
ED

body about any axis is equal to its moment of inertia about a


parallel axis through its centre of mass plus the product of the Using theorem of parallel axes (for rings 1 and 2), we get
mass of the body and the square of the distance between two æ1 2 2 ö æ12 2 2 ö 1 7
Itotal = ç PQ + PQ ÷ + ç PQ + PQ ÷ + PQ = PQ 2
axes. è2 ø è2 ø 2 2
Example 2.
P A
Four particles each of mass m are lying symmetrically on
the rim of a disc of mass M and radius R. Find MI of this
system about an axis passing through one of the particles
r and perpendicular to plane of disc.
Solution :
O According to the theorem of parallel axes, MI of disc about
c.m an axis passing through K and perpendicular to plane of
M
disc, is
1 2 2 3 2
= MR + MR = MR
Q B 2 2
Particle 2
Let AB be the axis in plane of paper about which, the moment of
inertia (I) of plane lamina is to be determined and PQ an axis R
Particle 1 K Particle 3
O R
parallel to AB, passing through centre of mass O of lamina is at a
distance ‘r’ from AB. Particle 4
Consider a mass element m of lamina at point P distant x from PQ. Total MI of the system =
Now the moment of inertia of the element about AB = m (x + r)2 3 MR 2
MR 2 + m(2 R ) 2 + m( 2 R ) 2 + m( 2 R ) 2 = 19
2 2
EBD_7179
176 PHYSICS

Example 3.
Three identical rods, each of length l, are joined to from a Solution :
rigid equilateral triangle. Find its radius of gyration about
an axis passing through a corner and perpendicular to C2 O C1
the plane of the triangle. m2 m1
m
Solution :
A

Area of whole plate = p (56/2)2 = 784 p sq. cm.


l
Area of cut portion = p (42/2)2 = 441 p sq. cm. ;
o Area of remaining portion = 784p – 441p = 343 p cm2;
60 As mass µ area.
B C
D
Mass of cut portion m1 441 p 9
\ = = =
ml 2 ml 2 Mass of remaining portion m 2 343 p 7
I = IAB + IAC + IBC = + + (I D + mh 2 )
3 3 Let C2 be centre of mass of remaining portion and C1 be
centre of mass of cut portion.

.IN
2 2 ml 2 é2 1 3ù O is centre of mass of the whole disc.;
= ml + + m(l sin 60) 2 = ml 2 ê + + ú OC1 = r1 = 28 - 21 = 7 cm.
3 12 ë 3 12 4 û
AL OC2 = r2 = ?;
3 2 l Equating moments of masses about O,
= ml = (3m)k 2 or k = m 9
2 2 we get m2 × r2 = m1 × r1 Þ r2 = 1 ´ r1 = ´ 7 = 9
m2 7
N
Example 4. \ Centre of mass of remaining portion is at 9 cm to the left
A uniform rod of mass m and length l makes a constant
R

of centre of disc.
angle q with the axis of rotation which passes through Example 6.
U

one end of the rod. Determine its moment of inertia about Show that the centre of mass of a rod of mass M and length
this axis. L lies midway between its ends, assuming the rod has a
JO

Solution : uniform mass per unit length.


æmö
Mass of element of uniform rod = ç ÷dx
èlø
U

Axis
dm
ED

O x
x sinq
x dx
dx
q
x Solution :
By symmetry, we see that yCM = zCM = 0 if the rod is placed
along the x axis. Furthermore, if we call the mass per unit
length l (the linear mass density), then l = M/L for a uniform
Moment of inertia of the element about the axis rod.
If we divide the rod into elements of length dx, then the
æm ö 2 mass of each element is dm = λ dx. Since an arbitrary element
= ç dx ÷( x sin q) .
èl ø of each element is at a distance x from the origin, equation
gives
l
m 2 2 ml 2 L L
lL2
I= sin θ. ò x dx = sin 2 θ 1 1
l 3 xCM = M ò x dm =
M ò0
x ldx =
2M
0 0
Example 5.
A circular plate of uniform thickness has a diameter of L2 æ M ö L
Because l = M/L, this reduces to xCM = ç ÷=
56 cm. A circular portion of diameter 42 cm is removed 2M è L ø 2
from one edge of the plate as shown. Find the position of One can also argue that by symmetry, xCM = L/2.
centre of mass of the remaining portion.
System of Particles and Rotational Motion 177

MOMENT OF INERTIA AND RADIUS OF GYRATION OF DIFFERENT OBJECTS

Shape of body Rotational axis Figure Moment of Radius of


inertia gyration

(1) Ring (a) Perpendicular to plane IC


M = mass passing through centre
R = radius of mass M
MR2 R
R cm

(b) Diameter in the plane


M IB
1 R
R cm MR2
2 2

.IN
I 'd I CM
(c) Tangent perpendicular
to plane
AL M
R cm 2MR2 2R
N
R

(d) Tangent in the plane M


U

R cm
3
JO

3 R
MR2 2
ID 2
U

(2) Disc (a) Perpendicular to plane Ic


passing through centre
ED

of mass
M
R
cm
1 R
MR 2
2 2

Y
(b) Diameter in the plane Z

Id
MR 2 R
4 2

(c) Tangent in the plane

M
R 5 5
Ic MR2
cm
4 2 R
Id
EBD_7179
178 PHYSICS

(d) Tangent perpendicular Id Ic


to plane
3 3 R
M MR2
2 2
R
cm

Ic

(3) Thin walled (a) Geometrical axis MR2 R


R
M

cylinder
L
cm

.IN
(b) Perpendicular to length Ic
M æ R 2 L2 ö R 2 L2
passing through centre of mass Mç + ÷ +
AL è 2 12 ø 2 12
R
cm
N
L
R

I
d Ic
U

(c) Perpendicular to length æ R 2 L2 ö


M
R 2 L2
Mç + ÷ +
JO

passing through one end R è 2 3ø 2 3


cm
U

L
ED

(4) Solid cylinder (a) Geometrical axis Ic

R MR 2 R
M 2 2
L
cm

I Ic
(b) Perpendicular to length M
passing through centre of mass æ R 2 L2 ö R 2 L2
R
Mç + ÷ +
è 4 12 ø 4 12
cm

Ic
Id
M
(c) Perpendicular to length
é R 2 L2 ù R 2 L2
passing through one end R
Mê + ú +
cm
êë 4 3 úû 4 3

L
System of Particles and Rotational Motion 179

(5) Annular disc (a) Perpendicular to plane Ic


M R12 + R 22
passing through centre [ R12 + R 22 ]
2 2
of mass M

R2 C

R1

M
(b) Diameter in the plane
C
Ic M[R12 + R 22 ] R12 + R 22
R2
R1 4 4

M
R2

.IN
R1
é R 2 + R 22 ù
L Mê 1 ú R12 + R 22
(6) Hollow cylinder (a) Geometrical axis 2
ëê ûú 2
AL
cm
N
R
U

R2 M
R1
JO

(b) Perpendicular to length L é L2 (R 2 + R 22 ) ù


Mê + 1 ú
L2 R12 + R 22
passing through centre I +
ëê 12 4 ûú 12 4
U

of mass cm
ED

Ic
2 2
(7) Solid sphere (a) Along the diameter M MR2 R
5 5

R
cm

Id Ic
7 7
(b) Along the tangent M MR2 R
5 5
R
cm
EBD_7179
180 PHYSICS

Ic

2 2
(8) Thin spherical shell (a) Along the diameter R MR2 R
3 3

Id Ic

.IN
5 5
(b) Along the tangent R MR2 R
AL 3 3
N
R
U

I
JO

Cavity
R
2 é R5 - r5 ù 2 (R 5 - r5 )
U

(9) Hollow sphere Along the diameter M ê 3 3ú


r 5 ëê R - r ûú 5 (R 3 - r3 )
ED

Hollow sphere

Ic
ML2 L
(10) Thin rod (a) Perpendicular to length M 12 2 3
passing through centre cm
of mass L

Id Ic
ML2 L
(b) Perpendicular to length M
3 3
passing through one end cm
L
System of Particles and Rotational Motion 181

I AB
A
(11) Rectangular plate (a) Perpendicular to length in
the plane passing through b O D Ma 2 a
centre of mass C 12 2 3
B
a

(b) Perpendicular to breadth in A


the plane passing through Mb2 b
b O D
centre of mass I 12 2 3
C CD

B
a
I CM
(c) Perpendicular to plane
passing through centre of mass A

.IN
M(a 2 + b2 ) a 2 + b2
O D 12 2 3
b C
AL
B
a
N

I1
R

(12) Square Plate (a) Perpendicular to plane passing M Ma 2 a


through centre of mass I1 =
U

a 6 6
JO

a
U

I3 M I2
(b) Diagonal passing through a
ED

a Ma 2
centre of mass I2 = I3 2 3
12
a

I1

(13) Cube (a) Perpendicular to plane passing M Ma 2 a


through centre of mass I1 =
6 6

I2

(b) Perpendicular to plane passing


M
through one end 2Ma 2 2
I2 = a
3 3

a
EBD_7179
182 PHYSICS

TORQUE, ANGULAR MOMENTUM AND ANGULAR The magnitude of angular momentum for a system is given by
IMPULSE n n
2
Torque : | L |= å (m i ri ) vi = å (m i ri )w [Q v = rw]
The moment of force is called torque. It is defined as the product i =1 i =1
of force and the perpendicular distance of the force from the n
axis of rotation. Þ | L |= å (Ii ´ w) = Iw ...(ii)
Y i=1
Where Ii is the moment of inertia of the ith particle of that system
F
q and I is total moment of inertia of the system
n
r q I = å I i = I1 + I 2 + I3 + ....................+ I n
i =1
O X
or I = m1r12 + m 2 r22 + ....................+ m n rn2 ...(iii)
r r r So if a system undergoes a redistribution of its mass, then its
i.e., t = r ´ F moment of inertia changes but since no external torque is applied
or, t = rF sin q on the system so total angular momentum is constant before and
r r after the distribution of mass, even if moment of inertia of the
where q is the angle between r and F .
system is changed.
Its S.I. unit is (N-m). The dimensions of torque [ML2T–2] are uur uur

.IN
the same as that of energy but it is not energy. L Initial = L Final ...(iv)
Note : If the line of action of a force passes through axis of or, I1w1 = I2w2 ...(v)
rotation then no torque will be formed.
Angular Momentum :
AL
where L initial denote the state previous to the redistribution of
mass and final denote the state after the redistribution of mass in
The angular momentum of a particle about an arbitrry point 'O' is
the moment of linear momentum taken about that point. that system.
N
Y A comparison of useful relations in rotational and translational
R

or linear motion :
p
U

Rotational motion about Linear motion


q a fixed axis
JO

dq dx
q Angular velocity w = Linear velocity v =
r dt dt
dw
U

dv
Angular acceleration a = Linear acceleration a =
O X dt dt
ED

Resultant torque t = Ia Resultant force F = ma


Equations of rotational Equations of linear motion
r r r motion
It is given as L = r ´ p
ì v = u + at
ìw = w0 + at ï
or, L = rp sin q ïï ï 1
a = constant then íq - q0 = w0 t + ½ at 2 a = constant then ís = ut + at 2
r r ï 2 ï 2
where q is the angle between r and F . 2
ïîw = w0 + 2a(q - q0 ) ï v 2 - u 2 = 2as
î
Angular Impulse : where (s = x – x0)
t2 q r r x r r
Work W = ò td q work W = ò Fdx
J = ò tdt = L2 - L1 (= Change in angular momentum) qo xo
t1 Kinetic energy Ek = ½ Iw2 Kinetic energy Ek=½ mv2
rr
CONSERVATION OF ANGULAR MOMENTUM Power P = tw Power P = Fv
rr
Angular momentum L = I w Linear momentum p = mv
r dL
From equation t ext = . dL dp
dt Torque t = Force F =
r dt dt
If t ext = 0 Þ L = constant ...(i)
This is called law of conservation of angular momentum. Work Energy Theorem in Rotational Motion :
r According to this theorem “the work done by external forces in
According to this “if resultant external torque text acting on
rotating a rigid body about a fixed axis is equal to the change
the system is zero then total angular momentum of the system is in rotational kinetic energy of the body.”
constant.
System of Particles and Rotational Motion 183

Since, we can express the torque as vcm = linear velocity of the centre of mass of the rigid body
dw dw dq K = radius of gyration (I = MK 2 = SM i ri2 )
t = Ia = I =I .
dt dq dt r = radius of moving rigid body
I = moment of inertia of the rigid body about centre of mass
dw
t = Iw but tdq = dW ½ Iw2 = rotational kinetic energy about the centre of mass.
dq Hence it is clear from the expression that total kinetic energy of
Þ tdq = dW = Iwdw rolling body is equal to the sum of rotational kinetic energy about
By integrating the above expression, we get total work done by centre of mass (C.M.) and translational kinetic energy of the centre
all external force on the body, which is written as of mass of body.
Body Rolling without Slipping on an Inclined Plane :
q2 w2 1 1
òq1 tdq = òw1 Iwdw = 2 Iw2 - 2 Iw1 When a body performed translatory as well as rotatory motion
2 2
W=
then we can say that the body is in rolling motion.
where the angular velocity of the body changes from w1 to w2 as Acceleration for body rolling down an inclined plane without
the angular displacement changes from q1 to q2 due to external slipping.
r Let M is the mass of the body, R is its radius and I is the moment
force Fext. on the body.. of inertia about the centre of mass and K is the radius of gyration.
Rotational Kinetic Energy : R
Let us consider a rigid body (collection of small particles) of high f

.IN
symmetry which is purely rotating about z-axis with an angular Mg
co s q Mg
velocity w. Each particle has some energy, determined by m i and q Mg sin
q
vi. The kinetic energy of mi particle is AL
(E k )i = ½ m i v i2 ... (i) q

Force equation, Mg sin q - f = Ma ...(1)


w
N
y Torque equation, fR = Ia ...(2)
vi
R

mi Also, I = MK 2 ...(3)
ri
U

2
MK a
Þ f= ...(4)
x
JO

R
O
Adding, eqn. (1) & (4)

Fig. The total kinetic energy of the body is ½Iw2. æ K 2a ö


U

Mg sin q = M ç + a ÷ a = a (Q Motion is pure rolling)


ç R ÷ R
Now we know that in rigid body every particle moves with same è ø
ED

angular velocity, the individual linear velocities depends on the


distance ri from the axis of rotation according to the expression æ K 2a ö g sin q
Mg sin q = M ç + a÷ Þ a =
(vi = riw). Hence the total kinetic energy of rotating rigid body is ç R2 ÷ K2
è ø 1+
equal to the sum of kinetic energies of individual particles.
R2
2 w2 Now assume in fig. that a body (which has high symmetry such
E k = S(E k )i = S½ m i v i2 = (Sm i ri )
2 as cylinder, sphere etc.) is rolling down an incline plane without
slipping. This is possible only if friction is present between object
Ek = ½ I w2 ... (ii) and incline plane, because it provides net torque to the body for
rotating about the centre of mass (since the line of action of the
where I = Sm i ri2 is moment of inertia of the rigid body.. other forces such as mg and R pass through the centre of mass of
Now consider a rigid body which is rolling without slipping. In rigid body, hence they do not produce torque in a body about
this case it possesses simultaneous translatory motion and the centre of mass).
rotatory motion and the total kinetic energy of the rigid body f=msR R
M
K.ETotal = rotation K.E. + translational K.E. of C.M. c. m
Ek = (E k ) rotational + (E k ) translatio nal ... (iii) mgsinq
Ek = ½ Iw 2 + ½ Mv cm
2
... (iv) h c
mg o x
po nt
in act
2 t w
E k = ½ Mv cm (1 + K 2 / r 2 ) = ½ Iw2 (1 + r 2 / K 2 ) ... (v)
q
where, vc
EBD_7179
184 PHYSICS

A round object rolling down an incline, mechanical energy is Rolling Motin Combination of translatory and rotatory
conserved if no slipping occurs. motion :
But mechanical energy of the body remains constant despite of (i) In pure rolling motion, the contact point O remains at rest.
friction because the contact point is at rest relative to the surface (ii) In pure rolling, the velocity of top most point is,
at any instant. V = Vcm + wR = 2Vcm
For pure rolling motion vc = rw
P
so E k = ½ I(v c / r ) 2 + ½ Mv 2c Vcm +wR

1 æ Ic ö C.M
Ek = çç + M ÷÷ vc2 ...(i)
2èr 2
ø Vcm
R
As body rolls down an incline, it loses potential energy Mgh (h
Vcm
is the height of incline). Since body starts from rest at top, hence O Contact
wR
its total kinetic energy at bottom given by eqn.(i) must be equal point
to Mgh at top i.e., (iii) In rolling without slipping Vcm = wR.
1/ 2
1 æ Ic ö æ 2gh ö 2wR = 2Vcm
çç + M ÷÷Vc2 = Mgh Þ v c = ç ÷
...(ii)
2 ç 2 ÷

.IN
2èr ø è 1 + I c / MR ø
since h =x sin q, where x is length of incline and Ic =MK2
(K is radius of gyration), then
AL Vcm
½
é 2gx sin q ù
vc = ê ....(iii)

N
2
ë1 + K / R û O Surface at rest
For rolling down an inclined plane without slipping, the
R

essential condition is : Keep in Memory


U

I cm
ms ³ tan q ; m is the coefficient of static friction 1. The axis of the rolling body is parallel to the plane on
JO

MR 2 + I cm s
which the body rolls in case of sphere, disc, ring.
where Icm is moment of inertia of the body about its C.M. 2. Let (Ek)r = rotational kinetic energy
U

(Ek)t = translation kinetic energy


DIFFERENT TYPES OF MOTION (a) For solid sphere, (Ek)r = 40% of (Ek)t
ED

Pure translational motion : In this case the velocities at all three


(b) For shell (Ek)r = 66% of (Ek)t
points : (i) top most point P, (ii) C.M. and (iii) contact point O are
same. (c) For disc, (Ek)r = 50% of (Ek)t
(d) For ring, (Ek)r = (Ek)t
P v
3. Translational kinetic energy is same for rolling bodies
having same M, R and w irrespective of their shape.
C.M (v) 4. Total energy is minimum for solid sphere and maximum for
(A)
ring having same mass and radius.
v 5. Rotational kinetic energy is maximum for ring and minimum
O
for solid sphere of same mass and radius.
Pure rotational motion : In this case the velocity of 6. (i) The acceleration down the inclined plane for different
(i) top most point P is Rw shapes of bodies of same mass and radius are as
(ii) C.M. is zero
follows :
(iii) contact point O is – Rw
Sphere > disc > shell > ring
Rw (ii) The velocity down the plane is related as follows:
Sphere > disc > shell > ring
(B) w Vcm = 0 (iii) The time taken to reach the bottom of the inclined
C.M
plane is related as follows :
O
Ring > shell > disc > sphere.
–Rw
System of Particles and Rotational Motion 185

This is because
Y
2
K
(i) For ring =1 log I
R2

K2 1
(ii) For disc = X
2 2
R
log K
K2 2 1 2
(iii) For sphere = 11. Rotational KE, (E k ) r = Iw \ Er µ w2
2 5
R 2
\ graph between (E k ) r and w will be as below
K2 2
(iv) For shell =
R 2 3 Y
7. The angular speed of all particles of a rotating/revolving
rigid body is same, although their linear velocities may be Er
different.
8. (i) Two particles moving with angular speed w1 and w2
on the same circular path and both in anti-clockwise X
w

.IN
direction then their relative angular speed will be wr =
w1 – w2. 12. Angular momentum L = Iw, hence L µ w.
AL\ Graph between L and w is a straight line.
w1
p1 Y
w2
r L
N
p2
R

(ii) In the above case one particle will complete one X


U

revolution more or less as compared to the other in w


time 13. The moment of inertia is not a vector quantity because
JO

2p 2p TT 1 1 1 clockwise or anti-clockwise, direction is not associated


T= = = 1 2 or = - with it. It is a tensor.
wr w1 - w 2 T1 - T2 T T2 T1 14. When a spherical/circular/cylindrical body is given a push,
U

9. Let two particles move on concentric circles having radius it only slips when the friction is absent, It may roll with
r1 and r 2 and their linear speeds v1, v2 both along anti- slipping if friction is less than a particular value and it may
ED

clockwise direction then their relative angular speed will roll without slipping if the friction is sufficient. (i.e.
ur ur I cm tan q
| v1 - v 2 | ms ³ )
be ωr = r r (MR 2 + I cm
| r1 -r 2 |
15. When a body rolls without slipping no work is done against
friction.
v2

r2 Example 7.
A thin circular ring of mass M and radius R rotating about
its axis with a constant angular speed w. Two blocks, each
r1 of mass, m are attached gently to opposite ends of a
v1 diameter of the ring. Find the angular speed of the ring.
10. As I µ K2 and I = MK2 ,
hence graph between I and K will be Solution :
a parabola. However graph between log I and log K will be As L = Iw = constant. Therefore,
straight line.
I1w1 MK 2 w Mw
Y I 2 w2 = I1w1 or w2 = = =
I2 ( M + 2m ) K 2 M + 2m
I Example 8
A solid sphere of mass 500 g and radius 10 cm rolls without
slipping with a velocity of 20 cm/s. Find the total K.E. of
X the sphere.
K
EBD_7179
186 PHYSICS

Solution : Example 12.


A cord is wound round the circumference of wheel of radius
1 2 1 1 2 2 2 1 2 r. The axis of the wheel is horizontal and moment of inertia
Total K.E. = Iω + mv 2 = ´ m r ω + mv
2 2 2 5 2 about it is I. A weight mg is attached to the end of the cord
2 2 and falls from rest. After falling through a distance h, what
As, Isphere = mr will be the angular velocity of the wheel?
5 Solution :
1 1 7 1 1 1 1
= mv 2 + mv 2 = mv 2 m g h = Ι ω 2 + m v 2 = Iw 2 + m r 2 w 2
5 2 10 2 2 2 2
2 1/ 2
7 500 æ 1 ö é 2m g h ù
= ´ ´ ç ÷ = 0.014 J . 2 2
10 1000 è 5 ø or 2 m g h = (I + m r )w \ w=ê ú
2
Example 9. ëê I + m r ûú
A body of radius R and mass M is rolling horizontally Example 13.
without slipping with speed v, it then rolls up a hill to a Let g be the acceleration due to gravity at earth’s surface
maximum height h. If h = 3v2/4 g, (a) what is the moment and K be the rotational kinetic energy of the earth.
of inertia of the body? (b) what might be the shape of the Suppose the earth’s radius decrease by 2%, keeping all
body? other quantities same, then
Solution : (a) g decreases by 2% and K decreases by 4%
(b) g decreases by 4% and k increases by 2%

.IN
1 1
(a) K total = K trans. + K rot. = M v2 + I ω2 (c) g increases by 4% and K decreases by 4%
2 2
(d) g decreases by 4% and K increases by 4%
1 1 æ v2 ö v2 é I ù Solution : (c)
= M v2 + I ç ÷= M+
AL
2 2 çè R 2 ÷ 2 êë R2 û
ú
GM
ø
We know that, g =
When it rolls up a hill to height h, the entire kinetic R2
N
energy is converted into potential energy M g h
dg æ 2dR ö
é v2 ù Differentiating, = -ç ÷ ....(1)
R

v2 é I ù g è R ø
Thus ê M+ ú = M g h = M g ê3 ú
2 ë R2 û êë 4g úû
U

1 2 1 é3 ù
Further, K = Iw = ê M R 2 ú w2
2 2 2 ë5
é I ù 3 MR û
JO

or ê M + 2 ú = M \ I =
ë R û 2 2 dK 3 æ 2dR ö
(b) The body may be a circular disc or a solid cylinder. or = M w2 ´ ç ÷ ....(2)
K 10 è R ø
U

Example 10. When radius decreases by 2%, then g increases by 4% and


ED

The angular velocity of a body changes from w1 to w2 K decreases by 4%.


without applying torque but by changing moment of Example 14.
inertia. What will be the ratio of initial radius of gyration A small solid ball rolls without slipping along the track
to the final radius of gyration? shown in fig. The radius of the circular part of track is R. If
Solution : the ball starts from rest at a height 8 R above the bottom,
Since I1w1 = I2w2 or M K12 w1 = M K 22 w 2 what are the horizontal and vertical forces acting on it at
P?
K1 æw ö Ball
Þ = çç 2 ÷÷
K2 è w1 ø
Example 11.
A rod of mass M and length l is suspended freely from its
end and it can oscillate in the vertical plane about the 8R
point of suspension. It is pulled to one side and then R
released. It passes through the equilibrium position with O P
angular speed w. What is the kinetic energy while passing R
through the mean position? Solution :
Solution : Suppose m is the mass of the ball of radius r.
1 Ml 2 On reaching P, the net height through which the ball
K.E, = Iw2 and I = (M l2/12) + M (l/2)2 = descends is
2 3
8 R – R = 7R, (from the fig.)
1 é M l2 ù 2 1 2 2 \ decrease in P.E. of ball = mg × 7 R
\ K= ê úw = M l w
2 ëê 3 ûú 6 This appears as total KE of ball at P.
System of Particles and Rotational Motion 187

Thus mg × 7R = KE of translation + KE of rotational


g sin q 2
a1 = = g sin q ; From v 2 = u2 + 2 a s
1 1 (1 + 1 / 2) 3
= mv 2 + Iω 2
2 2
æ2 ö
v12 = 0 + 2 ç gsin θ ÷ ´ l = 4 g sin q ´ l ...(i)
1 1 æ2 ö 7 è 3 ø 3
= mv 2 + ´ ç mr 2 ÷w 2 = mv 2
2 2 è5 ø 10 (ii) Acc. of the sliding cylinder, a2 =g sin q

2 From v 2 = u2 + 2 a s; v 22 = 0 + 2g sin θ ´ l ...(ii)


\ v = 10 g R
(where v = rw and r is radius of solid ball) v12 4g sin θ ´ l 2 v
\ = = ; 1 = 2/3 .
The horizontal force acting on the ball, v 22 3 ´ 2g sin θ ´ l 3 v 2
Fh = centripetal force towards O Example 16.
A disc of mass M and radius R is rolling with angular
mv 2 m(10g R )
= = = 10 mg speed w on a horizontal plane (fig.). Determine the
R R magnitude of angular momentum of the disc about the
Vertical force on the ball, Fv = weight of the ball = mg. origin O.
Y
Example 15.

.IN
A solid cylinder first rolls and then slides from rest down a
smooth inclined plane. Compare the velocities in the two AL
cases when the cylinder reaches the bottom of the incline.
w
Solution :
R
(i) We know that acc. of a body rolling down an inclined M
N
O X
g sin q Solution :
R

plane is a1 =
1+ K2 / R2 The angular momentum L is given by
U

æ1 2ö 3 2
R2 L = Ιcm ω + M v R = ç M R ÷w + M(w R ) R = M R w
JO

2 è 2 ø 2
For a solid cylinder K = ;
2
U
ED
188

ED
U
JO
U
R
N
AL
.IN
PHYSICS

EBD_7179
System of Particles and Rotational Motion 189

1. Centre of mass of the earth and the moon system lies 8. A ball tied to a string is swung in a vertical circle. Which of
(a) closer to the earth the following remains constant?
(b) closer to the moon (a) tension in the string
(c) at the mid-point of line joining the earth and the moon (b) speed of the ball
(d) cannot be predicted (c) centripetal force
2. Four particles of masses m1,m2,m3 and m4 are placed at the (d) earth's pull on the ball
vertices A,B,C and D as respectively of a square shown. 9. Angular momentum of a system of a particles changes,
The COM of the system will lie at diagonal AC if when
(a) m1 = m3 A m1 B m2 (a) force acts on a body
(b) m2 = m4 (b) torque acts on a body
(c) direction of velocity changes
(c) m1 = m2
m4 m3 (d) None of these
(d) m3 = m4 C

.IN
D 10. Angular momentum is
3. Two spheres A and B of masses m and 2m and radii 2R and (a) a polar vector (b) an axial vector
R respectively are placed in contact as shown. The COM AL (c) a scalar (d) None of these
of the system lies 11. If a running boy jumps on a rotating table, which of the
following is conserved?
A
(a) Linear momentum
N
B
(b) K.E
2R R
R

(c) Angular momentum


(d) None of these
U

12. A gymnast takes turns with her arms & legs stretched. When
JO

(a) inside A (b) inside B she pulls her arms & legs in
(c) at the point of contact (d) None of these (a) the angular velocity decreases
4. Moment of inertia does not depend upon (b) the moment of inertia decreases
U

(a) distribution of mass (c) the angular velocity stays constant


ED

(b) axis of rotation (d) the angular momentum increases


(c) point of application of force 13. Moment of inertia of a circular wire of mass M and radius R
(d) None of these about its diameter is
5. A disc is given a linear velocity on a rough horizontal surface (a) MR2/2 (b) MR2
then its angular momentum is (c) 2MR2 (d) MR2/4.
(a) conserved about COM only 14. A solid sphere is rotating in free space. If the radius of the
(b) conserved about the point of contact only sphere is increased keeping mass same which one of the
(c) conserved about all the points following will not be affected ?
(d) not conserved about any point. (a) Angular velocity
6. A body cannot roll without slipping on a (b) Angular momentum
(a) rough horizontal surface (c) Moment of inertia
(b) smooth horizontal surface (d) Rotational kinetic energy
(c) rough inclined surface 15. One solid sphere A and another hollow sphere B are of
(d) smooth inclined surface same mass and same outer radii. Their moment of inertia
7. A body is projected from ground with some angle to the about their diameters are respectively IA and IB Such that
horizontal. The angular momentum about the initial position
will (a) IA < IB (b) IA > IB
(a) decrease IA d A
(c) IA = IB (d) =
(b) increase IB d B
(c) remains same
where d A and d B are their densities.
(d) first increase then decrease
EBD_7179
190 PHYSICS

16. Angular momentum is


m P
(a) moment of momentum
(b) product of mass and angular velocity M
(c) product of M.I. and velocity Q

(d) moment of angular motion A


q
B
17. The angular momentum of a system of particle is conserved
(a) when no external force acts upon the system (a) (m/M) L cos q (b) m L/(M + m)
(b) when no external torque acts upon the system (c) (M + m)/(m L cos q) (d) (m L cos q) / (m + M)
(c) when no external impulse acts upon the system 22. A solid sphere and a hollow sphere of the same material
(d) when axis of rotation remains same and of same size can be distinguished without weighing
18. Analogue of mass in rotational motion is (a) by determining their moments of inertia about their
coaxial axes
(a) moment of inertia (b) angular momentum
(b) by rolling them simultaneously on an inclined plane
(c) gyration (d) None of these
(c) by rotating them about a common axis of rotation
19. Moment of inertia does not depend upon
(d) by applying equal torque on them
(a) angular velocity of body
23. A stick of length L and mass M lies on a frictionless
(b) shape and size
horizontal surface on which it is free to move in any way. A
(c) mass
ball of mass m moving with speed v collides elastically with
(d) position of axis of rotation
the stick as shown in fig.

.IN
20. A hollow sphere is held suspended. Sand is now poured
into it in stages. L
M
v
AL m
If after the collision ball comes to rest, then what should be
N
the mass of the ball?
(a) m = 2 M (b) m = M
R

SAND (c) m = M/2 (d) m = M/4


24. A flywheel rotates about an axis. Due to friction at the axis,
U

it experiences an angular retardation proportional to its


JO

angular velocity. If its angular velocity falls to half while it


The centre of gravity of the sphere with the sand makes n rotations, how many more rotations will it make
(a) rises continuously before coming to rest?
(b) remains unchanged in the process
U

(a) 2n (b) n
(c) First rises and then falls to the original position (c) n/2 (d) n/3
ED

(d) First falls and then rises to the original position 25. A raw egg and a hard boiled egg are made to spin on a table
21. A block Q of mass M is placed on a horizontal frictionless with the same angular momentum about the same axis. The
surface AB and a body P of mass m is released on its ratio of the time taken by the two to stop is
frictionless slope. As P slides by a length L on this slope of (a) = 1 (b) < 1
inclination q, the block Q would slide by a distance (c) > 1 (d) None of these

1. A solid cylinder of mass 20 kg rotates about its axis with 3. Two particles A and B, initially at rest, moves towards each
angular speed 100 rad/s. The radius of the cylinder is 0.25 other under a mutual force of attraction. At the instant when
m. The K.E. associated with the rotation of the cylinder is the speed of A is v and the speed of B is 2 v, the speed of
(a) 3025 J (b) 3225 J centre of mass is
(c) 3250 J (d) 3125 J (a) zero (b) v
2. What is the moment of inertia of a solid sphere of density r (c) 1.5 v (d) 3 v
and radius R about its diameter? 4. Point masses 1, 2, 3 and 4 kg are lying at the points (0, 0, 0),
105 5 105 2 (2, 0, 0), (0, 3, 0) and (–2, –2, 0) respectively. The moment of
(a) R r (b) R r inertia of this system about X-axis will be
176 176
(a) 43 kg –m2 (b) 34 kg–m2
176 5 176 2
(c) R r (d) R r (c) 27 kg – m 2 (d) 72 kg – m2
105 105
System of Particles and Rotational Motion 191

5. A body having moment of inertia about its axis of rotation 13. A particle of mass m is observed from an inertial frame of
equal to 3 kg-m2 is rotating with angular velocity equal to 3 reference and is found to move in a circle of radius r with a
rad/s. Kinetic energy of this rotating body is the same as uniform speed v. The centrifugal force on it is
that of a body of mass 27 kg moving with a speed of
(a) 1.0 m/s (b) 0.5 m/s mv2
(a) towards the centre
(c) 1.5 m/s (d) 2.0 m/s r
6. A particle moves in a circle of radius 0.25 m at two mv2
revolutions per second. The acceleration of the particle in (b) away from the centre
metre per second2 is r
(a) p2 (b) 8p2
mv2
(c) 4p 2 (d) 2p2 (c) along the tangent through the particle
r
7. The radius of gyration of a body about an axis at a distance (d) zero
6 cm from its centre of mass is 10 cm. Then, its radius of
14. A circular disc X of radius R is made from an iron plate of
gyration about a parallel axis through its centre of mass will
thickness t, and another disc Y of radius 4R is made from an
be
t
(a) 80 cm (b) 8 cm iron plate of thickness . Then the relation between the
4
(c) 0.8 cm (d) 80 m moment of inertia IX and IY is

.IN
8. A particle of mass m is moving in a plane along a circular
path of radius r. Its angular momentum about the axis of (a) Ι Y = 32 Ι X (b) Ι Y = 16 Ι X
rotation is L. The centripetal force acting on the particle is
(c)Ι Y = ΙX (d) Ι Y = 64 ΙX
(a) L2 /mr (b) L2 m/r
AL
15. A particles performing uniform circular motion. Its angular
(c) L2 /m r3 (d) L2 /m r2 frequency is doubled and its kinetic energy halved, then the
N
9. A small disc of radius 2 cm is cut from a disc of radius 6 cm. new angular momentum is
If the distance between their centres is 3.2 cm, what is the
R

shift in the centre of mass of the disc? (a) L (b) 2 L


(a) 0.4 cm (b) 2.4 cm 4
U

(c) 1.8 cm (d) 1.2 cm L


JO

10. A solid cylinder of mass m & radius R rolls down inclined (c) 4 L (d)
2
plane without slipping. The speed of its C.M. when it
16. A simple pendulum is vibrating with angular amplitude of
reaches the bottom is
U

90° as shown in figure.


(a) 2 gh
ED

q
(b) 4gh / 3
h
(c) 3 / 4 gh
For what value of q is the acceleration directed
4 gh (i) vertically upwards
(d)
(ii) horizontally
11. The ratio of moment of inertia of circular ring & circular disc
(iii) vertically downwards
having the same mass & radii about on axis passing the c.m
& perpendicular to plane is 1
(a) 0°, cos -1
1
, 90° (b) cos -1 , 0°, 90°
(a) 1 : 1 (b) 2 : 1 3
3
(c) 1 : 2 (d) 4 : 1
1
12. A rod of length L is pivoted at one end and is rotated with
(c) 90°, cos -1
1
, 0° (d) cos -1
, 90°, 0°
a uniform angular velocity in a horizontal plane. Let T1and 3 3
T 2 be the tensions at the points L/4 and
17. A mass is tied to a string and rotated in a vertical circle, the
3L/4 away from the pivoted end. Then
minimum velocity of the body at the top is
(a) T1 > T2
(b) T2 > T1 (a) gr (b) g/r
(c) T1 – T2
3/ 2
(d) the relation between T1 and T2 depends on whether ægö
(c) ç ÷ (d) gr
the rod rotates clockwise and anticlockwise. èrø
EBD_7179
192 PHYSICS

18. A couple is acting on a two particle systems. The resultant 25. A uniform bar of mass M and length L is horizontally
motion will be suspended from the ceiling by two vertical light cables as
(a) purely rotational motion shown. Cable A is connected 1/4th distance from the left
(b) purely linear motion end of the bar. Cable B is attached at the far right end of the
(c) both a and b bar. What is the tension in cable A?
(d) None of these
19. A mass m is moving with a constant velocity along a line Cable A Cable B
parallel to the x-axis, away from the origin. Its angular 1L
momentum with respect to the origin 4
L
(a) is zero (b) remains constant
(c) goes on increasing (d) goes on decreasing. (a) 1/4 Mg (b) 1/3 Mg
20. A smooth sphere A is moving on a frictionless horizontal (c) 2/3 Mg (d) 3/4 Mg
plane with angular speed w and centre of mass velocity v. It 26. ABC is a triangular plate of uniform thickness. The sides are
collides elastically and head on with an identical sphere B in the ratio shown in the figure. IAB, IBC and ICA are the
at rest. Neglect friction everywhere. After the collision, their moments of inertia of the plate about AB, BC and CA as axes
angular speeds are wA and wB, respectively. Then respectively. Which one of the following relations is correct?
(a) wA< wB (b) wA= wB (a) I > I
AB BC
A
(c) wA= w (d) wB= w
I BC > I AB

.IN
(b)
21. A particle moves in a circle of radius 4 cm clockwise at 5
3
constant speed 2 cm s–1 . If x̂ and ŷ are unit acceleration (c) I AB + I BC = I CA
vectors along X and Y respectively (in cm s–2 ), the 90°
AL (d) ICA is maximum B 3 C
acceleration of the particle at the instant half way between
P and Q is given by 27. In carbon monoxide molecule, th e carbon and
Y the oxygen atoms are separated by a distance
N
1.12 × 10–10 m. The distance of the centre of mass, from the
R

carbon atom is
P
(a) 0.64 × 10–10 m (b) 0.56 × 10–10 m
U

(c) 0.51 × 10 m –10 (d) 0.48 × 10–10 m


28. A weightless ladder 20 ft long rests against a frictionless
JO

O X
Q wall at an angle of 60º from the horizontal. A 150 pound man
is 4 ft from the top of the ladder. A horizontal force is needed
to keep it from slipping. Choose the correct magnitude from
U

(a) -4(x̂ + ŷ) (b) 4(x̂ + ŷ) the following.


ED

(a) 175 lb (b) 100 lb


(c) - ( x̂ + ŷ) / 2 (d) ( x̂ - ŷ) / 4
(c) 120 lb (d) 17.3 lb
22. A particle is confined to rotate in a circular path decreasing 29. The moment of inertia of a disc of mass M and radius R
linear speed, then which of the following is correct? about an axis, which is tangential to the circumference of the
r
(a) L (angular momentum) is conserved about the centre disc and parallel to its diameter, is
r 2
3
(b) only direction of angular momentum L is conserved (a) MR 2 (b) MR 2
(c) It spirals towards the centre 2 3
(d) its acceleration is towards the centre. 5 4
23. Two rings of radius R and nR made up of same material (c) MR 2 (d) MR 2
4 5
have the ratio of moment of inertia about an axis passing
through centre as 1 : 8. The value of n is 30. A tube one metre long is filled with liquid of mass 1 kg. The
tube is closed at both the ends and is revolved about one
(a) 2 (b) 2 2 end in a horizontal plane at 2 rev/s. The force experienced
by the lid at the other end is
1
(c) 4 (d) (a) 4p2N (b) 8p2N
2 (c) 16p N 2 (d) 9.8 N
24. A cylinder rolls down an inclined plane of inclination 30°, 31. If the linear density (mass per unit length) of a rod of length
the acceleration of cylinder is 3m is proportional to x, where x is the distance from one end
g of the rod, the distance of the centre of gravity of the rod
(a) (b) g
3 from this end is
g 2g (a) 2.5 m (b) 1 m
(c) (d) (c) 1.5 m (d) 2 m
2 3
System of Particles and Rotational Motion 193

32. A composite disc is to be made using equal masses of æ1 ö


aluminium and iron so that it has as high a moment of inertia (a) v cos ç q ÷
è2 ø P
as possible. This is possible when q
(a) the surfaces of the disc are made of iron with aluminium æ1 ö
(b) 2v cosç q ÷
inside è2 ø
(b) the whole of aluminium is kept in the core and the iron (c) v (1 + sin q)
at the outer rim of the disc (d) v (1 + cos q)
(c) the whole of the iron is kept in the core and the 38. Acertain bicycle can go up a gentle incline with constant
aluminium at the outer rim of the disc speed when the frictional force of ground pushing the rear
(d) the whole disc is made with thin alternate sheets of wheel is F2 = 4 N. With what force F1 must the chain pull on
iron and aluminium the sprocket wheel if R1=5 cm and R2 = 30 cm?
33. A ball rolls without slipping. The radius of gyration of the
ball about an axis passing through its centre of mass is K. If in F1
Cha
radius of the ball be R, then the fraction of total energy
associated with its rotational energy will be d
R2 Roa
R2 K2 + R2 R1
(a) (b) 4N
K2 + R2 R2
F2 =

.IN
K2 K2 Horizontal
(c) (d)
R2 K2 + R2 (a) 4 N (b) 24 N
AL
34. When a bucket containing water is rotated fast in a vertical
35
circle of radius R, the water in the bucket doesn't spill (c) 140 N (d) N
4
provided
N
39. Auniform rod of length l is free to rotate in a vertical plane
(a) the bucket is whirled with a maximum speed of 2gR
R

about a fixed horizontal axis through O. The rod begins


(b) the bucket is whirled around with a minimum speed of rotating from rest from its unstable equilibrium position.
U

When it has turned through an angle q, its angular velocity


[(1 / 2) gR ] w is given as O
JO

(c) the bucket is having a rpm of 900g /( p 2 R ) q


6g 6g q
U

(a) sin q (b) sin


(d) the bucket is having a rpm of 3600g /( p 2 R ) l l 2
ED

35. A coin placed on a gramophone record rotating at 6g q 6g


33 rpm flies off the record, if it is placed at a distance of (c) cos (d) cos q
l 2 l
more than 16 cm from the axis of rotation. If the record is
revolving at 66 rpm, the coin will fly off if it is placed at a 40. A sphere of mass 2000 g and radius 5 cm is rotating at the
distance not less than rate of 300 rpm .Then the torque required to stop it in 2p
(a) 1 cm (b) 2 cm revolutions, is
(c) 3 cm (d) 4 cm (a) 1.6 × 102 dyne cm (b) 1.6 × 103 dyne cm
36. Two fly wheels A and B are mounted side by side with (c) 2.5 × 104 dyne cm (d) 2.5 × 105 dyne cm
frictionless bearings on a common shaft. Their moments of 41. Five masses are placed in a plane as shown in figure. The
inertia about the shaft are 5.0 kg m2 and 20.0 kg m2 coordinates of the centre of mass are nearest to
respectively. Wheel A is made to rotate at 10 revolution per y
2 3 kg 4 kg
second. Wheel B, initially stationary, is now coupled to A
with the help of a clutch. The rotation speed of the wheels
will become 5 kg
1
(a) 2 5 rps (b) 0.5 rps
(c) 2 rps (d) None of these
37. A wheel is rolling straight on ground without slipping. If 0 1 kg 2 kg x
0 1 2
the axis of the wheel has speed v, the instantenous velocity
of a point P on the rim, defined by angle q, relative to the (a) 1.2, 1.4 (b) 1.3, 1.1
ground will be (c) 1.1, 1.3 (d) 1.0, 1.0
EBD_7179
194 PHYSICS

42. A solid sphere of mass 1 kg rolls on a table with linear 48. The wheel of a car is rotating at the rate of 1200 revolutions
speed 1 ms–1. Its total kinetic energy is per minute. On pressing the accelerator for 10 seconds it
(a) 1 J (b) 0.5 J starts rotating at 4500 revolutions per minute. The angular
(c) 0.7 J (d) 1.4 J acceleration of the wheel is
43. A boy and a man carry a uniform rod of length L, horizontally (a) 30 radian / second2 (b) 1880 degrees/ second2
in such a way that the boy gets 1/4th load. If the boy is at (c) 40 radian / second2 (d) 1980 degree/second2
one end of the rod, the distance of the man from the other 49. Four masses are fixed on a massless rod as shown in the
end is adjoining figure. The moment of inertia about the dotted
(a) L/3 (b) L/4 axis is about
(c) 2 L/3 (d) 3 L/4
44. A solid sphere of mass M and radius R is pulled horizontally 0.2 m 0.2 m
on a sufficiently rough surface as shown in the figure.

2kg 5kg 5kg 2kg


0.4 m 0.4 m

(a) 2 kg × m2 (b) 1 kg × m2
(c) 0.5 kg × m 2 (d) 0.3 kg × m2

.IN
Choose the correct alternative. 50. The moment of inertia of a body about a given axis is 1.2 kg
(a) The acceleration of the centre of mass is F/M m2. Initially, the body is at rest . In order to produce a
rotational kinetic energy of 1500 J, an angular acceleration
2 F
AL
(b) The acceleration of the centre of mass is of 25 rad s–2 must be applied about that axis for a duration
3M
of
(c) The friction force on the sphere acts forward
N
(a) 4 s (b) 2 s
(d) The magnitude of the friction force is F/3
(c) 8 s (d) 10 s
R

45. Three identical particles each of mass 1 kg are placed


51. Fig. shows a disc rolling on a horizontal plane with linear
touching one another with their centres on a straight line.
velocity v. Its linear velocity is v and angular velocity is w.
U

Their centres are marked A, B and C respectively. The


Which of the following gives the velocity of the particle P
distance of centre of mass of the system from A is
JO

on the rim of the disc


AB + AC + BC AB + AC
(a) (b) A
3 3 w
U

P
AB + BC AC + BC
r
ED

(c) (d)
3 3
46. M.I of a circular loop of radius R about the axis in figure is q
v
O M
y

Axis of rotation
R/2 B N
x
O (a) v (1 + cos q) (b) v (1 – cos q)
(c) v (1 + sin q) (d) v (1 – sin q)
52. A toy car rolls down the inclined plane as shown in the fig.
It loops at the bottom. What is the relation between H and
(a) MR2 (b) (3/4) MR2 h?
(c) MR /2 2 (d) 2MR2 H
(a) =2
47. If the earth is treated as a sphere of radius R and mass M, h
its angular momentum about the axis of its diurnal rotation H h
with period T is (b) =3
h H D
r
4 pMR 2
2pMR 2 H
(a) (b) (c) =4
5T 5T h
B
MR 2 T pMR 3 H
(c) (d) (d) =5
T T h
System of Particles and Rotational Motion 195

53. A sphere rolls down on an inclined plane of inclination q.


(I1 + I 2 )w I2w
What is the acceleration as the sphere reaches bottom? (a) (b)
I1 I1 + I 2
5 3
(a) g sin q (b) g sin q
7 5 I1w
(c) w (d)
2 2 I1 + I 2
(c) g sin q (d) g sin q
7 5 60. Three particles, each of mass m gram, are situated at the
54. In a bicycle, the radius of rear wheel is twice the radius of vertices of an equilateral triangle ABC of side/cm (as shown
front wheel. If r F and rF are the radii, vr and vr are the speed in the figure). The moment of inertia of the system about a
of top most points of wheel. Then line AX perpendicular to AB and in the plane of ABC, in
(a) v r = 2v F (b) v F = 2v r gram-cm2 units will be
(c) vF = v r (d) vF > v r 3 X
(a) ml 2
55. An annular ring with inner and outer radii R1 and R 2 is 2
m C
rolling without slipping with a uniform angular speed. The 3
ratio of the forces experienced by the two particles situated (b) ml 2
4 l l
F
on the inner and outer parts of the ring , 1 is (c) 2 ml2
F2

.IN
5 B
2 (d) ml 2 A
æ R1 ö R2 4 m l m
(a) çç ÷÷ (b)
è R2 ø R1
AL
61. The moment of inertia of a uniform circular disc of radius ‘R’
and mass ‘M’ about an axis passing from the edge of the disc
(c) R1 (d) 1 and normal to the disc is
N
R2
1
(a) MR2 (b) MR 2
56. A wheel having moment of inertia 2 kg-m2 about its vertical
R

2
axis, rotates at the rate of 60 rpm about this axis, The torque
U

3 7
which can stop the wheel’s rotation in one minute would (c) MR 2 (d) MR 2
2 2
be
JO

62. Two bodies have their moments of inertia I and 2I


p 2p
(a) Nm (b) Nm respectively about their axis of rotation. If their kinetic
18 15
energies of rotation are equal, their angular momenta will
U

p p be in the ratio
(c) Nm (d) Nm
12 15
ED

(a) 2 : 1 (b) 1 : 2
57. Consider a system of two particles having masses m1 and
m2 . If the particle of mass m1 is pushed towards the centre (c) 2:1 (d) 1 : 2
of mass particles through a distance d, by what distance 63. A drum of radius R and mass M, rolls down without
would the particle of mass m2 move so as to keep the mass slipping along an inclined plane of angle q. The frictional
centre of particles at the original position? force
m2 m1 (a) dissipates energy as heat.
(a) d (b) d
m1 m1 + m 2 (b) decreases the rotational motion.
m1 (c) decreases the rotational and translational motion.
(c) d (d) d
m2 (d) converts translational energy to rotational energy
58. The ratio of the radii of gyration of a circular disc about a 64. A tube of length L is filled completely with an incompressible
tangential axis in the plane of the disc and of a circular ring liquid of mass M and closed at both ends. The tube is then
of the same radius about a tangential axis in the plane of rotated in a horizontal plane about one of its ends with
the ring is uniform angular speed w. What is the force exerted by the
(a) 1 : Ö2 (b) 1 : 3 liquid at the other end?
(c) 2 : 1 (d) Ö5 : Ö6
MLw2
59. A round disc of moment of inertia I 2 about its axis (a) (b) MLw2
2
perpendicular to its plane and passing through its centre
is placed over another disc of moment of inertia I 1 rotating
MLw2 MLw2
with an angular velocity w about the same axis. The final (c) (d)
angular velocity of the combination of discs is 4 8
EBD_7179
196 PHYSICS

65. A circular disk of moment of inertia It is rotating in a 70. If the angular velocity of a body rotating about an axis is
horizontal plane, its symmetry axis, with a constant angular doubled and its moment of inertia halved, the rotational
speed wi . Another disk of moment of inertia Ib is dropped kinetic energy will change by a factor of :
coaxially onto the rotating disk. Initially the second disk (a) 4 (b) 2
has zero angular speed. Eventually both the disks rotate 1
(c) 1 (d)
with a constant angular speed w f . The energy lost by the 2
71. One quarter sector is cut from a uniform circular disc of
initially rotating disk to friction is
radius R. This sector has mass M. It is made to rotate about
2
1 Ib I t2 a line perpendicular to its plane and passing through the
(a) wi2 (b) wi2
2 ( It + I b ) ( It + I b ) centre of the original disc. Its moment of inertia about the
axis of rotation is
Ib - It 2 1 Ib It
(c) wi (d) wi2 1
mR 2
1
mR 2
( It + I b ) (
2 t I + I ) (a) (b)
b 2 4
66. The moment of inertia of a thin uniform rod of mass M and 1
length L about an axis passing through its midpoint and (c) mR 2 (d) 2mR 2
8
perpendicular to its length is I0. Its moment of inertia about

.IN
an axis passing through one of its ends and perpendicular Directions for Qs. (72 to 75) : Each question contains
to its length is STATEMENT-1 and STATEMENT-2. Choose the correct answer
(a) I0 + ML2/2 (b) I0 + ML2/4 (ONLY ONE option is correct ) from the following-
AL
(c) I0 + 2ML 2 (d) I0 + ML2 (a) Statement -1 is false, Statement-2 is true
67. The instantaneous angular position of a point on a rotating (b) Statement -1 is true, Statement-2 is true; Statement -2 is a
wheel is given by theequation q(t) = 2t3 – 6t2. The torque
N
correct explanation for Statement-1
on the wheel becomes zero at (c) Statement -1 is true, Statement-2 is true; Statement -2 is not
R

(a) t = 1s (b) t = 0.5 s a correct explanation for Statement-1


U

(c) t = 0.25 s (d) t = 2s (d) Statement -1 is true, Statement-2 is false


68. The moment of inertia of a uniform circular disc is 72. Statement 1 : When you lean behind over the hind legs of
JO

maximum about an axis perpendicular to the disc and the chair, the chair falls back after a certain angle.
passing through Statement 2 : Centre of mass lying outside the system
U

makes the system unstable.


73. Statement 1: A rigid disc rolls without slipping on a fixed
ED

C rough horizontal surface with uniform angular velocity. Then


D the acceleration of lowest point on the disc is zero.
B Statement 2 : For a rigid disc rolling without slipping on a
A
fixed rough horizontal surface, the velocity of the lowest
point on the disc is always zero.
74. Statement 1 : If no external force acts on a system of
particles, then the centre of mass will not move in any
(a) B (b) C direction.
(c) D (d) A Statement 2 : If net external force is zero, then the linear
69. Three masses are placed on the x-axis : 300 g at origin, 500 momentum of the system remains constant.
g at x = 40 cm and 400 g at x = 70 cm. The distance of the 75. Statement 1 : A wheel moving down a frictionless inclined
centre of mass from the origin is plane will slip and not roll on the plane.
(a) 40 cm (b) 45 cm Statement 2 : It is the frictional force which provides a
(c) 50 cm (d) 30 cm torque necessary for a body to roll on a surface.
System of Particles and Rotational Motion 197

Exemplar Questions 6. In problem-5, the CM of the plate is now in the following


quadrant of x-y plane.
1. For which of the following does the centre of mass lie outside
the body? (a) I (b) II
(a) A pencil (b) A shotput (c) III (d) IV
(c) A dice (d) A bangle 7. The density of a non-uniform rod of length 1 m is given by
2. Which of the following points is the likely position of the r(x) = a (1 + bx 2) where, a and b are constants and
centre of mass of the system shown in figure? 0 £ x £ 1. The centre of mass of the rod will be at
Hollow sphere 3(2 + b ) 4(2 + b)
Air (a) (b)
4(3 + b ) 3(3 + b)
R/2 A
B 3(3 + b ) 4(3 + b )
C (c) (d)
4(2 + b) 3(2 + b )

.IN
R/2
D
8. A merry-go-round, made of a ring-like platform of radius R
Sand
and mass M, is revolving with angular speed w. A person of
(a) A (b) B
AL mass M is standing on it. At one instant, the person jumps
(c) C (d) D off the round, radially away from the centre of the round (as
3. A particle of mass m is moving in yz-plane with a uniform seen from the round). The speed of the round of afterwards
N
velocity v with its trajectory running parallel to +ve y-axis is
and intersecting z-axis at z = a in figure. The change in its
R

(a) 2w (b) w
angular momentum about the origin as it bounces elastically
U

from a wall at y = constant is w


z (c) (d) 0
(a) mvaeˆx 2
JO

v
(b) 2mvaeˆx a NEET/AIPMT (2013-2017) Questions
U

(c) ymveˆ x 9. A small object of uniform density rolls up a curved surface


with an initial velocity ‘n’. It reaches upto a maximum height
ED

(d) 2 ymveˆ x y
3n 2
4. When a disc rotates with uniform angular velocity, which of of with respect to the initial position. The object is a
the following is not true? 4g
(a) The sense of rotation remains same (a) solid sphere (b) hollow sphere [2013]
(b) The orientation of the axis of rotation remains same (c) disc (d) ring
(c) The speed of rotation is non-zero and remains same 10. A rod PQ of mass M and length L is hinged at end P. The
(d) The angular acceleration is non-zero and remains same rod is kept horizontal by a massless string tied to point Q as
5. A uniform square plate has a small piece Q of an irregular shown in figure. When string is cut, the initial angular
shape removed and glued to the centre of the plate leaving acceleration of the rod is [2013]
a hole behind in figure. The moment of inertia about the z-
axis is then,
y y
hole
Q Q
x x
P P

(a) increased
(a) g /L (b) 2g/L
(b) decreased
(c) the same 2g 3g
(c) (d)
(d) changed in unpredicted manner 3L 2L
EBD_7179
198 PHYSICS

11. Two discs are rotating about their axes, normal to the discs string which passes through a smooth hole in the plane as
and passing through the centres of the discs. Disc D1 has shown.
2 kg mass and 0.2 m radius and initial angular velocity of 50
v0
rad s–1. Disc D2 has 4kg mass, 0.1 m radius and initial angular
velocity of 200 rad s–1. The two discs are brought in contact
face to face, with their axes of rotation coincident. The final m
angular velocity (in rad s–1) of the system is
(a) 40 (b) 60 [NEET Kar. 2013]
(c) 100 (d) 120
12. The ratio of radii of gyration of a circular ring and a circular The tension in the string is increased gradually and finally
disc, of the same mass and radius, about an axis passing R
m moves in a circle of radius 0 . The final value of the
through their centres and perpendicular to their planes are 2
kinetic energy is [2015]
(a) 2 :1 (b) 1 : 2 [NEET Kar. 2013]
(c) 3 : 2 (d) 2 : 1 1
(a) mv02 (b) 2mv02
13. The ratio of the accelerations for a solid sphere (mass ‘m’ 4
and radius ‘R’) rolling down an incline of angle ‘q’ without
slipping and slipping down the incline without rolling is : 1
(c) mv02 (d) mv02
(a) 5 : 7 (b) 2 : 3 [2014] 2

.IN
(c) 2 : 5 (d) 7 : 5 r
18. A force F= a ˆi + 3jˆ + 6kˆ is acting at a point
14. A solid cylinder of mass 50 kg and radius 0.5 m r
r = 2iˆ - 6jˆ - 12kˆ . The value of a for which angular
is free to rotate about the horizontal axis. A massless string
AL
is wound round the cylinder with one end attached to it momentum about origin is conserved is : [2015 RS]
and other hanging freely. Tension in the string required to (a) 2 (b) zero
N
produce an angular acceleration of 2 revolutions s– 2 is : (c) 1 (d) –1
(a) 25 N (b) 50 N [2014] 19. Point masses m1 and m2 are placed at the opposite ends of
R

(c) 78.5 N (d) 157 N a rigid rod of length L, and negligible mass. The rod is to be
set rotating about an axis perpendicular to it. The position
U

15. Three identical spherical shells, each of mass m and radius of point P on this rod through which the axis should pass
r are placed as shown in figure. Consider an axis XX' which so that the work required to set the rod rotating with angular
JO

is touching to two shells and passing through diameter of velocity w0 is minimum, is given by : [2015 RS]
third shell. Moment of inertia of the system consisting of
w0
U

these three spherical shells about XX' axis is [2015]


(a) 3mr2
ED

X
16 2 m1 P m2
(b) mr
5
(c) 4mr2 x (L–x)

11 2 X¢
(d) mr
5
16. A rod of weight W is supported by two parallel knife edges m1 m2
A and B and is in equilibrium in a horizontal position. The (a) x= L (b) x = m L
m2 1
knives are at a distance d from each other. The centre of
mass of the rod is at distance x from A. The normal reaction m2 L m1L
(c) x= (d) x = m + m
on A is [2015] m1 + m 2 1 2

Wd W(d – x) 20. An automobile moves on a road with a speed of 54 km h -1.


(a) (b) The radius of its wheels is 0.45 m and the moment of inertia
x x
of the wheel about its axis of rotation is 3 kg m2. If the
W(d – x) Wx vehicle is brought to rest in 15s, the magnitude of average
(c) (d) torque transmitted by its brakes to the wheel is : [2015 RS]
d d
(a) 8.58 kg m2 s-2 (b) 10.86 kg m2 s-2
17. A mass m moves in a circle on a smooth horizontal plane
with velocity v0 at a radius R0. The mass is attached to (c) 2.86 kg m2 s-2 (d) 6.66 kg m2 s-2
System of Particles and Rotational Motion 199

21. A disk and a sphere of same radius but different masses roll (B) Centre of mass of a body is the point at which the total
off on two inclined planes of the same altitude and length. gravitational torque on the body is zero
Which one of the two objects gets to the bottom of the (C) A couple on a body produce both translational and
plane first ? rotation motion in a body
(a) Disk [2016] (D) Mechanical advantage greater than one means that
(b) Sphere small effort can be used to lift a large load
(c) Both reach at the same time (a) (A) and (B) (b) (B) and (C)
(d) Depends on their masses (c) (C) and (D) (d) (B) and (D)
22. A uniform circular disc of radius 50 cm at rest is free to turn 25. Two discs of same moment of inertia rotating about their
about an axis which is perpendicular to its plane and passes regular axis passing through centre and perpendicular to
through its centre. It is subjected to a torque which produces the plane of disc with angular velocities w1 and w2 . They
a constant angular acceleration of 2.0 rad s–2 . Its net are brought into contact face to face coinciding the axis of
acceleration in ms–2 at the end of 2.0s is approximately : rotation. The expression for loss of energy during this
process is:- [2017]
(a) 8.0 (b) 7.0 [2016]
(c) 6.0 (d) 3.0 1
(a) I(w1 - w2 )2 (b) I(w1 - w2 )2
23. From a disc of radius R and mass M, a circular hole of diameter 4
R, whose rim passes through the centre is cut. What is the

.IN
1 1
moment of inertia of the remaining part of the disc about a (c) (w1 - w2 )2 (d) I( w1 + w2 ) 2
perpendicular axis, passing through the centre ? [2016] 8 2
(a) 15 MR2/32 (b) 13 MR2/32 26. A rope is wound around a hollow cylinder of mass 3 kg and
AL
(c) 11 MR2/32 (d) 9 MR2/32 radius 40 cm. What is the angular acceleration of the
cylinder if the rope is pulled with a force of 30 N ? [2017]
24. Which of the following statements are correct ? [2017]
N
(a) 0.25 rad/s2 (b) 25 rad/s2
(A) Centre of mass of a body always coincides with the 2
(c) 5 m/s (d) 25 m/s2
R

centre of gravity of the body


U
JO
U
ED
EBD_7179
200 PHYSICS

Hints & Solutions


EXERCISE - 1 20. (d) Initially centre of gravity is at the centre. When sand
is poured it will fall and again after a limit, centre of
1. (a) 2. (b) 3. (c) 4. (c) gravity will rise.
5. (b) 6. (d) 7. (b) 21. (d) Here, the centre of mass of the system remains
8. (d) The pull of earth changes only when the body moves unchanged. in the horizontal direction. When the mass
so that g changes. It is an exceptional case and should m moved forward by a distance L cos q, let the mass
not be considered unless otherwise mentioned. (m + M) moves by a distance x in the backward
9. (b) If we apply a torque on a body, then angular momentum direction. hence
of the body changes according to the relation (M + m) x – m L cos q = 0
r
r dL r \ x = (m L cos q)/(m + M)
τ= Þ if t = 0 then, L = constant 22. (b) Acceleration of solid sphere is more than that of
dt r r r r hollow sphere, it rolls faster, and reaches the bottom
10. (b) Angular momentum L is defined as L = r ´ m(v)
r of the inclined plane earlier.
so L is, an axial vector.. Hence, solid sphere and hollow sphere can be
11. (c) The boy does not exert a torque to rotating table by distinguished by rolling them simultaneously on an

.IN
jumping, so angular momentum is conserved i.e., inclined plane.
r 23. (d) Applying the law of conservation of momentum
dL r
= 0 Þ L = constant m v= M V ...(1)
dt
AL By conservation of angular momentum
12. (b) Since no external torque act on gymnast, so angular
æ M L2 ö
momentum (L=I ω ) is conserved. After pulling her arms m v (L / 2) = ç ÷ω
ç 12 ÷ ...(2)
& legs, the angular velocity increases but moment of
N
è ø
inertia of gymnast, decreases in, such a way that As the collision is elastic, we have
R

angular momentum remains constant. 1 1 1


13. (a) m v 2 = M V 2 + Ι ω2 ...(3)
U

2 2 2
14. (b) Angular momentum will remain the same since external
Substituting the values, we get m = M/4
torque is zero.
JO

24. (b) a is propotional to w


2 1 2 Let a = kw (Q k is a constant)
15. (a) I B = MR I A = MR
2 dω
= kω dθ dθ
U

\ IA < IB [also = ω Þ dt = ]
r dt dt ω
16. (a) Angular momentum = r ´ ( linear m omentum )
ED

ωdω
dL \ = k ω Þ dw = kdq
17. (b) We know that τ ext = dθ
dt
if angular momentum is conserved, it means change in ω/ 2
angular momentum = 0
dL
Now ò dω = kò dθ
or, dL = 0 , dt = 0 Þ τ ext = 0
ω
0 θ
ω ω
18.
Thus total external torque = 0.
(a) Analogue of mass in rotational motion is moment of
ò dω = k ò dθ Þ -
2
= kθ Þ - = kθ1
2
ω/2 0
inertia. It plays the same role as mass plays in (Q q1 = 2pn)
translational motion.
19. (a) Basic equation of moment of inertia is given \ θ = θ1 or 2pn1 = 2pn
n1 = n
25. (b) So raw egg is like a spherical shell & hard bioled egg is
mi like solid sphere. Let I1 , I2 be M. I. of raw egg and
boiled egg respectively.
n ri Given that angular momentum L, is same
by I = S m i ri2
i =1 \ I1w1= I2w2 Þ w2>w1 Q I1>I2
Now from first equation of angular motion (wf=wi+at)
here a is retarding decceleration for both cases &
where m i is the mass of i th particle at a distance of wf = 0 for both case.
t (raw egg) ω /α t
ri from axis of rotation. So 1 = 1 Þ 1 <1
Thus it does not depend on angular velocity. t 2 (hard egg) ω 2 / α t2
System of Particles and Rotational Motion 201

EXERCISE - 2 1 1
K.E. = m v 2 = ´ 27 ´ v 2 = 13.5
1 2 2 2
1. (d) K.E. of rotation = Iw
2 v =1 m/s
1 æ 1 2ö 2 6. (c) Centripetal acceleration
= ´ ç mr ÷ w
2 è2 ø a c = 4p2 n 2 r = 4 p 2 ´ 2 ´ 2 ´ 0.25 = 4p 2 ms -2
7. (b) From the theorem of parallel axes, the moment of inertia
1
= ´ 20 ´ (0.25)2 ´ 100 ´ 100 = 3125 J I = ICM + Ma2
4 where ICM is moment of inertia about centre of mass
2. (c) For solid sphere and a is the distance of axis from centre.
2 2æ4 ö mk2 = m(k1)2 + m(6)2
I= M R 2 = ç p R 3 r÷ R 2 (Q I = mk2 where k is radius of gyration)
5 5è3 ø or, k2 = (k1)2 + 36
8 22 5 176 5 or, 100 = (k1)2 + 36
=
´ R r= R r Þ (k1)2 = 64 cm
15 7 105
\ k1 = 8 cm
3. (a) Force FA on particle A is given by 8. (c) L = m v r or v = L /mr
mA v

.IN
2
FA = m A a A = ...(1) m v 2 m (L / m r) 2 = L
t Centripetal force =
r r m r3
mB ´ 2 v 9. (a) The situation can be shown as : Let radius of complete
Similarly FB = m B a B = ...(2)
AL
t disc is a and that of small disc is b. Also let centre of
mass now shifts to O2 at a distance x2 from original
mA v mB ´ 2 v
(Q FA = FB ) centre.
N
Now =
t t
R

So mA = 2 mB
For the centre of mass of the system a
U

O b
mA vA + m B vB O2 O1 x-axis
v=
JO

mA + mB x2 x1

2 mB v - mB ´ 2 v
or v= =0
U

2 mB + m B The position of new centre of mass is given by


ED

Negative sign is used because the particles are -s .pb 2 .x1


travelling in opposite directions. XCM =
s .pa 2 - s .pb 2
Alternatively, if we consider the two masses in a
system then no external force is acting on the system. Here, a = 6 cm, b = 2 cm, x1 = 3.2 cm
Mutual forces are internal forces. Since the centre of -s ´ p (2) 2 ´ 3.2
mass is initially at rest, it well remain at rest Hence, XCM =
s ´ p ´ (6)2 - s ´ p ´ (2)2
4. (a) Moment of inertia of the whole system about the axis
of rotation will be equal to the sum of the moments of 12.8p
inertia of all the particles. = = – 0.4 cm.
32p
Y 10. (b) By energy conservation
(K.E )i + (P.E )i = (K.E ) f + (P.E ) f
(0, 3, 0)
3 kg
(K.E )i = 0, (P.E )i = mgh, (P.E ) f = 0
X
(0, 0, 9) (2, 0, 0) (K.E)f = ½Iω2 + ½mvcm 2

(–2,–2, 0) 1 kg 2 kg Where I (moment of inertia) = ½mR2


4 kg (for solid cylinder)

\ I = I 1 + I2 + I3 + I4 æ v2 ö
so mgh = ½(½mR 2 ) ç cm2 ÷
+ ½mv 2cm
= 0 + 0 + 27 + 16 = 43 kg m2 è R ø
1 2 1
5. (a) Er = I w = ´ 3 ´ (3) 3 = 13.5 J Þ vcm = 4gh / 3
2 2
EBD_7179
202 PHYSICS

11. (b) Moment of inertia (M.I) of circular ring I1 = mr2 18. (a) A couple consists of two equal and opposite forces
moment of inertia (M.I) of circular disc I2 = ½mr2 whose lines of action are parallel and laterally
I 2 separated by same distance. Therefore, net force (or
Þ 1 = resultant) of a couple is null vector, hence no
I2 1
translatory motion will be produced and only
12. (a) Tension provides the necessary centripetal force for rotational motion will be produced.
the rest of rod. 19. (b) Angular momentum of mass m moving with a constant
13. (d) Centrifugal force is observed when the observer is in velocity about origin is
a frame undergoing circular motion. v
line parallel
1 to x-axis
14. (d) I = mR2 y
m
2
M µ t µ R2 O x
1 2 1 2 2 L = momentum × perpendicular distance of line of
For disc X, ΙX = (m)(R) = (πr t).(R)
2 2 action of momentum from origin
L = mv × y
1
for disc Y, ΙY = [π(4R)2 .t / 4][4R]2 In the given condition mvy is a constant. Therefore
2 angular momentum is constant.

.IN
ΙX 1 20. (c) Since the spheres are smooth, there will be no transfer
Þ = Þ IY = 64 IX of angular momentum from the sphere A to sphere B.
ΙY (4)3 AL The sphere A only transfers its linear velocity v to the
sphere B and will continue to rotate with the same
1
15. (a) Angular momentum µ angular speed w.
Angular frequency
N
-( x̂ + ŷ)
µ Kinetic energy 21. (c) n̂ = –a cos45o x
r K.E. 2 45o
R

Þ L=
ω a
U

L1 æ K.E1 ö ω
=ç ÷ ´ 2 = 4 Þ L2 = L
JO

L 2 è ω1 ø K.E 2 4 –a sin45o y
16. (a) When q = 0°, the net force is directed vertically
ur v2 4 -(xˆ + y)
ˆ
upwards.
U

a = nˆ = ´
17. (a) Let velocity at A = v A and velocity at B = v B r 4 2
ED

A 22. (b) Since v is changing (decreasing), L is not conserved


vA in magnitude. Since it is given that a particle is confined
r T to rotate in a circular path, it can not have spiral path.
Since the particle has two accelerations ac and at
therefore the net acceleration is not towards the centre.
vB v
B
Applying conservation of energy at A & B L at
ac
1 1
mv 2A + 2gmr = mv 2B [Q (P.E)A = mg (2r)]
2 2

v 2B = v 2A + 4gr..........(i ) The direction of L remains same even when the


Now as it is moving in circular path it has centripetal speed decreases.
force. 23. (a) The moment of inertia (I) of circular ring whose axis of

mv 2A rotation is passing through its center, I1 = m1R 2


At point A Þ T + mg =
r
Also, I 2 = m 2 (nR) 2
for minimum velocity T ³ 0 Since both rings have same density,
mv 2A m2 m1
or ³ mg Þ v 2A ³ gr Þ v A ³ gr Þ =
r 2p (nR) ´ A 2 2pR ´ A1
System of Particles and Rotational Motion 203

Where A is cross-section of ring, From definition of centre of mass.


A1 = A 2 (Given) \ m 2 = nm1 16 ´ 1.12 ´ 10 -10 + 12 ´ 0 = 16 ´ 1.12 ´ 10
-10
d=
16 + 12 28
I1 1 m1R 2 m1R 2
Given = = = = 0.64 × 10–10 m.
I2 8 m 2 (nR) 2 nm1 (nR) 2 28. (d) AB is the ladder, let F be the horizontal force and W is
the weigth of man. Let N1 and N2 be normal reactions
1 1 of ground and wall, respectively. Then for vertical
Þ = or n = 2
8 n3 equilibrium
24. (a) Remember that acceleration of a cylinder down a W = N1 .....(1)
smooth inclined plane is For horizontal equilibrium N2 = F .....(2)
Taking moments about A
N mN
N2(AB sin60°) – W(B Cos 60°) = 0......(3)
Using (2) and AB = 20 ft, BC = 4 ft, we get
mg sin q m g cos q
xmg N2 B
q

t
4f
2
g sin q mR C
a= where I = is the moment of
æ I ö

.IN
2
ç1 + ÷
è mR 2 ø N1 W

g sin 30°
Inertia for cylinder a =
AL 60°
æ mR 2 1 ö÷ O
ç1 + ´ A F
ç 2 mR 2 ÷ø
è æ 3 ö÷ æ 1ö
N
Fç 20 ´ – wç 4 ´ ÷ = 0
ç 2 ø ÷ è 2ø
1 è
R


= 2 =g 2w ´ 2 w 150
1 3 ÞF= = =
U

1+ pound (lb)
2 20 3 5 3 5 3
JO

25. (c) This is a torque problem. While the fulcrum can be = 10 3 = 10 ´ 1.73 = 17.3 pound
placed anywhere, placing it at the far right end of the 29. (c) Moment of inertia of disc about its diameter is
bar eliminated cable B from the calculation. There are 1
I d = MR 2
U

now only two forces acting on the bar ; the weight


4
that produces a counterclockwise rotation and the
ED

I
tension in cable A that produces a clockwise rotation.
Since the bar is in equilibrium, these two torques must R
Id
sum to zero.
3
L
Cable 4 MI of disc about a tangent passing through rim and in
the plane of disc is
fulcrum 1 5
1L I = I G + MR 2 = MR 2 + MR 2 = MR 2
4 4
W = mg 2 1 2 2
30. (b) F = mrw2 = 1 ´ ´ 4 p ´ 2 ´ 2 = 8p N
2
St = TA (3 / 4L) - Mg(1 / 2L) = 0
31. (d) Consider an element of length dx at a distance x from
Therefore
end A.
TA = ( MgL / 2) /(3L / 4) = (MgL / 2)(4 / 3L) = 2Mg / 3 Here : mass per unit length l of rod
26. (b) The intersection of medians is the centre of mass of (l µ x Þ l = kx)
the triangle. Since distances of centre of mass from
the sides are related as : xBC < xAB < xAC therefore IBC \ dm = ldx = kx dx
> IAB > IAC or IBC > IAB.
–10
27. (a) (12 amu) 1.12×10 (16 amu)
C c.m. O
–10
d 1.12×10 – d A x dx B
EBD_7179
204 PHYSICS

Position of centre of gravity of rod from end A


37. (b) v
L qv
x CG =
ò0 x dm q
L
ò0 dm
3
é x3 ù
3 ê ú(3)3
\
ò
x CG = 0
x(kx dx)
=
ëê 3 ûú0
= 3 = 2m
v R = v 2 + v 2 + 2 v 2 cos q = 2 v 2 (1 + cos q)
3 3
(3)2
ò0 kx dx éx ù
ê ú
2 q
= 2v cos
2 2
êë 2 úû0
38. (b) For no angular acceleration tnet = 0
32. (b) Density of iron > density of aluminium Þ F1 × 5 = F2 × 30 (given F2 = 4N) Þ F1 = 24 N
moment of inertia = ò r dm .
2
1 2
39. (c) Iw = Loss of gravitational potental energy
2

1 ml 2 2 mgl

.IN
r ´ w = (1 + cos q)
dm 2 3 2

3g æ 2 qö 6g q
Þ w2 =
AL ç 2 cos ÷ Þ w = cos
l è 2ø l 2
\ Since riron > raluminium 40. (d) Use t = Ia
N
so whole of aluminium is kept in the core and the iron
2
at the outer rim of the disc. and w 2 - w02 = 2aq Here I = MR 2
R

5
1 1
33. (d) Rotational energy = I(w) 2 = (mK 2 )w2 Given M = 2000g = 2kg
U

2 2 R = 5cm = 5 × 10–2m
JO

1 q = 2p ´ 2p radians
Linear kinetic energy = mw2 R 2
2 n = 300 rpm = 5 rps
\ Required fraction
æ 2 2ö
2 ö w - w0
U

æ2
1 \ t = Ia = = çè 5 MR ÷ø ç 2q ÷
(mK 2 )w2 K2 è ø
ED

= 2 = 2
1 1
(mK 2 )w 2 + mw 2 R 2
K + R2
(
4p 2 52 - 02 )
2
( )
2
2 2 = ´ 2 ´ 5 ´ 10 -2
5 2
34. (c) At the highest point 2 ´ 4p
= 0.025 N – m.
g = 2.5 × 105 dyne cm.
mg = mR4p 2 n2 Þ n =
4p 2 R 1´ 0 + 2 ´ 2 + 3 ´ 0 + 4 ´ 2 + 5 ´ 1
41. (c) X C.M. =
900 g 1+ 2 + 3 + 4 + 5
Revolution per minute = 60n =
p2R 4 + 8 + 5 17
= = = 1.1
15 15
35. (d) m mg = mr 4p 2 n 2
1´ 0 + 2 ´ 0 + 3 ´ 2 + 4 ´ 2 + 5 ´ 1
YC.M =
r1n12 r1n12 1+ 2 + 3 + 4 + 5
Þ r2 n 22 = r1 n12 Þ r2 = =
n 22 4n12 6+8+5
= = 1.3
15
r1
r2 = = 4 cm
4 1 1
42. (c) E= Mv 2 + Iw 2
36. (c) By conservation of angular momentum, 2 2
50 1 1 2 v2 7
5 ´ 10 = 5w + 20w Þ w = = 2 rps = Mv 2 + ´ MR 2 ´ = Mv 2 = 0.7 J
25 2 2 5 R 2 10
System of Particles and Rotational Motion 205

43. (a) So couple about the C.M must be zero for rotational 53. (a) Acceleration of a body rolling down an inclined plane
equilibrium, It means that g sin q
is given by , a =
W L 3W
´ = ´x Þ x =
L K2
1+ 2
4 2 4 6 R
L/2
boy C.M man In case of a solid sphere , we have
L/2 x
K2 [(I / M)] I (2 / 5)MR 2 2
= = = =
W 3W R2 R2 MR 2 MR 2 5
4 4 K2 2
and the distance of man from other end is Substituting = in equation (1) we get
2 5
R
L L L L 5
–x = - = a = g sin q
2 2 6 3 7
44. (b,c) 54. (c) The velocity of the top point of the wheel is twice that
45. (b) Position of C.M w.r. to A of centre of mass. And the speed of centre of mass is
same for both the wheels.

a2
A B C R2

.IN
R1 v 2 = wR 2
1´ 0 + 1 ´ AB + 1 ´ AC AB + AC a1
= = v1 = wR1
1+ 1+1 3 AL
46. (b) Use theorem of parallel axes.
2 2p 4pMR 2
47. (a) Angular momentum = Iw = MR 2 . =
v12 w2 R 12
N
5 T 5T
55. (c) a1 = = = w2 R 1
w2 - w1 R1 R1
R

48. (d) a=
t 2 - t1 v 22
a2 = = w2 R 2
U

R2
2 p ´1200 2 p ´ 4500
w1 = = 40p ; w2 = = 150p
JO

Taking particle masses equal


60 60
F1 ma 1 a 1 R 1
110p = = =
a= rad / sec2 F2 ma 2 a 2 R 2
U

10 Alternative method : The force experienced by any


Now, p radian = 180° particle is only along radial direction or the centripetal
ED

force.
180 Force experienced by the particle, F = mw2R
\ 1 rad = degree
p F R
\ 1 = 1
11p ´180 F2 R 2
\ a= degree /sec 2 = 1980 degree/sec2 56. (d) t × Dt = L0 {Q since Lf = 0}
p
Þ t × Dt = Iw
49. (b) I = 2 ´ 5 ´ (0.2)2 + 2 ´ 2 ´ (0.4) 2 = 1kg ´ m 2 or t × 60 = 2 × 2 × 60p/60
50. (b) E = 1500 = 1/2 × 1.2 w2 60 ö
(Q f = 60rpm \ w = 2pf = 2p ´ ÷
3000 60 ø
w2 = = 2500 p
1.2 t = N-m
15
w = 50rad / sec
57. (c) m1d = m2 d2 Þ d 2 = m1d
w 50 m2
t= = = 2 sec
a 25
y1 y¢1
51. (c) Velocity of P = (NP)w = (NM + MP)w
= r(r + sin q)w = v(1 + sin q) 58. (d) Circular disc (1)
52. (d) Velocity at the bottom and top of the circle is 5gr
and gr . Therefore (1/2)M(5gr) = MgH and
MR 2
(1/2) M (gr) = Mgh. Iy =
1 4
EBD_7179
206 PHYSICS

From parallel axis theorem


MR 2 5
\ I¢y = + MR 2 = MR 2 1 3
1 4 4 I T = I C.M . + MR 2 = MR 2 + MR 2 = MR 2
2 2
y2 y¢2 L
2
62. (d) K= Þ L2 = 2KI Þ L = 2 KI
2I
Circular ring (2)
L1 K1 I1 K I 1
= × = × =
L2 K2 I 2 K 2I 2

L1 : L2= 1 : 2
MR 2
Iy = 63. (d) Net work done by frictional force when drum rolls
2 2
down without slipping is zero.
MR 2 3 M
\ I¢y = + MR 2 = MR 2 R f
2 2 2

I¢y = MK 12 , I¢y 2 = MK 22
1
q
I¢y

.IN
K2 Wnet = 0, Wtrans. + Wrot. = 0
\ 1 = 1
Þ K1 : K 2 = 5 : 6 DKtrans. + DKrot. = 0
K 22 I¢y
2 AL DKtrans = –DKrot.
59. (d) Angular momentum will be conserved i.e., converts translation energy to rotational energy.
64. (a) Tube may be treated as a particle of mass M at distance
I1w L/2 from one end.
I 1w = I1w' + I2w' Þ w¢ = I + I
N
1 2 2 ML 2
Centripetal force = Mrw = w
R

60. (d) IAX = m(AB)2 + m(OC)2 = ml2 + m (l cos 60º)2 2


= ml2 + ml2/4 = 5/4 ml2 65. (d) By conservation of angular momentum, It wi =(It+Ib) w f
U

where w f is the final angular velocity of disks


JO

X
Cm æ It ö
O \ wf = ç ÷ wi
è It + Ib ø
U

60º
l l Loss in K.E., D K = Initial K.E. – Final K.E.
ED

1 1
= It wi2 – ( It + Ib ) w2f
60º 2 2
Am l Bm
1 1 It2
Þ DK = It wi2 - ( It + Ib ) w2
61. (c) M.I. of a uniform circular disc of radius ‘R’ and 2 i
2 2 ( It + Ib )
mass ‘M’ about an axis passing through C.M.
and normal to the disc is 1 2 It ( I + I - I ) 1 2 I t Ib
= w t b t = w
1 2
2 i It + Ib 2 i I t + Ib
I C.M. = MR 66. (b) By theorem of parallel axes,
2
I = I cm + Md2
I = I0 + M (L/2)2 = I0 + ML2/4
67. (a) When angular acceleration (a) is zero then torque on
the wheel becomes zero.
q(t) = 2t3 – 6t2
dq
Þ = 6t 2 - 12t
dt

d 2q
Þa = = 12t - 12 = 0
dt 2
\ t = 1 sec.
System of Particles and Rotational Motion 207

68. (a) According to parallel axis theorem of the moment of 2. (c) Centre of mass of a system lies towards the part of the
Inertia system, having bigger mass. In the above diagram,
lower part is heavier, hence CM of the system lies
I = Icm + md2
below the horizontal diameter at C point.
d is maximum for point B so Imax about B.
3. (b) The initial velocity is vi = veˆy and after reflection from
m1 x1 + m2 x2 + m3 x3
69. (a) X cm = the wall, the final velocity is v f = -veˆ y . The trajectory
m1 + m2 + m3
is at constant distance a on z axis and as particle moves
along y axis, its y component changes .
300 ´ (0) + 500(40) + 400 ´ 70 So position vector (moving along y-axis),
X cm =
300 + 500 + 400 r
r = yeˆy + aeˆz
500 ´ 40 + 400 ´ 70 Hence, the change in angular momentum is
X cm =
1200 r
r ´ m(v f - v j ) = 2mvaeˆx .
50 + 70 120 dw
X cm = = = 40 cm 4. (d) Angular acceleration a =
3 3 dt
where w is angular velocity of the disc and is uniform

.IN
1 2 or constant.
70. (b) K= Iw
2 dw
a= =0
dt
1 æ 1ö
AL
K ' = ç ÷ (2w) 2 = 2K Hence, angular acceleration is zero.
2 è 2ø
5. (b) According to the question, when the small piece Q
N
71. (a) For complete disc with mass '4M', M.I. about given removed it is stick at P through axis of rotation passes,
axis = (4M)(R2/2) = 2 MR2 but axis of rotation does not passes through Q and It
R

is glued to the centre of the plate, the mass comes


Hence, by symmetry, for the given quarter of the disc
U

closer to the z-axis, hence, moment of inertia decreases.


1 2 6. (c) Let us consider the diagram of problem 5, there is a
JO

M.I. = 2 MR2 /4 = MR line shown in the figure drawn along the diagonal.
2
First, centre of mass of the system was on the dotted
72. (d) line and was shifted towards Q from the centre (1st
U

73. (a) For a disc rolling without slipping on a horizontal rough quadrant).
ED

surface with uniform angular velocity, the acceleration y y


of lowest point of disc is directed vertically upwards
Q Q hole
and is not zero (Due to translation part of rolling,
x x
acceleration of lowest point is zero. Due to rotational x
part of rolling, the tangential acceleration of lowest
point is zero and centripetal acceleration is non-zero When the mass Q is decrease, it will be on the same
and upwards). Hence statement 1 is false. line but shifted towards other side of Q on the line
74. (a) 75. (b) joining QPor away from the centre so, new CM will
lies in IIIrd quadrant in x-y plane.
EXERCISE - 3
7. (a) As given that density :
Exemplar Questions
r( x) = a(1 + bx 2 )
1. (d) A bangle is in the form of a ring as shown in figure.
The centre of mass of the ring lies at its centre, which where a and b are constants and 0 £ x £ 1
is outside the ring or bangle. At b = 0, r( x) = a = constant
In that case, centre of mass will be at
x = 0.5 m (i.e., mid-point of the rod)
C Putting, b = 0 in options.
Centre
3 2 1
(a) ´ = = 0.5 m
4 3 2
EBD_7179
208 PHYSICS

4 2
(b) ´ ¹ 0.5 m
3 3
3 3 10. (d)
(c) ´ ¹ 0.5 m
4 2
4 3
(d) ´ ¹ 0.5 m
3 2
Weight of the rod will produce the torque
So, only (a) option gives 0.5.
8. (a) As there is external torque acting on the system, L mL2 é ML2 ù
t = mg =Ia= a êQ Irod = ú
angular momentum should be conserved. 2 3 êë 3 úû
Hence Iw = constant ...(i)
where, I is moment of inertia of the system and w is 3g
Hence, angular acceleration a =
angular velocity of the system. 2L
From Eq. (i), 11. (c) Given: m1 = 2 kg m2 = 4 kg
I1w1 = I 2 w2 r1 = 0.2 m r2 = 0.1 m
w1 = 50 rad s–1 w2 = 200 rad s–1
where w1 and w2 are angular velocities before and

.IN
after jumping. As, I1W1 = I2W2 = Constant

(Q I = mr 2 ) So, given that, 1 1


m r2w + m r2 w
I1W1 + I 2W2 2 1 1 1 2 2 2 2
m1 = 2M, m2 = M, w1 = w, w2 = ?
AL \ Wf = =
I1 + I 2 1 1
m1r12 + m2 r22
r1 = r2 = R m1r12 w1 = m2 r22w2 2 2
N

2MR 2 w = MR2 w2 By putting the value of m1, m2, r1, r2 and solving we
R

get = 100 rad s–1


as mass reduced to half, hence, moment of inertia also
U

reduced to half. I
12. (a) Q I = MK2 \ K =
M
JO

w2 = 2w
1
NEET/AIPMT (2013-2017) Questions Iring = MR2 and Idisc = MR 2
U

2
9. (c)
ED

K1 I1 MR 2
= = = 2 :1
3V 2 K2 I2 æ MR 2 ö
ç ÷
4g V è 2 ø

13. (a) For solid sphere rolling without slipping on inclined


plane, acceleration
From law of conservation of mechanical energy
g sin q
a1 =
1 2 1 3v2 K2
2
Iw + 0 + mv = mg ×
4g 1+
2 2 R2
1 2 3 1 For solid sphere slipping on inclined plane without
Þ Iw = mv2 – mv2 rolling, acceleration
2 4 2
a2 = g sin q
mv2 æ 3 - 1 ö
= ç ÷ a1
2 è2 ø Therefore required ratio =
a2
1 V2 mv2 1
or, I 2 = or, I = mR2 1 1 5
2 R 4 2 = = =
K 2 2 7
Hence, object is a disc. 1+ 1+
R2 5
System of Particles and Rotational Motion 209

14. (d) Here a = 2 revolutions/s2 = 4p rad/s2 (given) \ v1 = 2V0


1 1 1
I cylinder = MR 2 = (50)(0.5) 2 Therefore, new KE =
2
m (2V0)2 = 2mv0
2 2 2
25 18. (d) From Newton's second law for rotational motion,
= Kg-m2 r
4 r dL r r
As t = Ia so TR = Ia t = , if L = constant then t = 0
dt
r r r
æ 25 ö So, t = r ´ F = 0
(4 p )
Ia çè 4 ÷ø
ÞT=
R
=
(0.5)
N = 50 pN = 157 N ( 2iˆ - 6ˆj -12kˆ ) ´ (aˆi + 3jˆ + 6k)ˆ = 0
Solving we get a = –1
15. (c) Moment of inertia of shell 1 along diameter
19. (c) Work required to set the rod rotating with angular
2 2 velocity w0
Idiameter = MR
3 1 2
Moment of inertia of shell 2 = m. i of shell 3 K.E. = Iw
2
2 2 2 5 2 Work is minimum when I is minimum.
= Itangential = MR + MR = MR
3 3 I is minimum about the centre of mass
X So, (m1) (x) = (m2) (L – x)

.IN
or, m1x = m2L – m2x
1 m2L
\x= m + m
2 3
AL 1 2
20. (d) Given : Speed V = 54 kmh–1 = 15 ms–1
X¢ Moment of inertia, I = 3 kgm2
N
So, I of the system along x x1 Time t = 15s
= Idiameter + (Itangential) × 2
R

2 æ5 ö wi = V = 15 = 100 wf = 0
U

or, Itotal = MR 2 + ç MR 2 ÷ ´ 2 r 0.45 3


3 è3 ø wf = wi + at
JO

12 100 100
=MR 2 = 4MR 2 0= + (– a) (15) Þ a =
3 3 45
16. (c) By torque balancing about B
U

Average torque transmitted by brakes to the wheel


NA (d) = W (d – x)
100
ED

W(d – x) t = (I) (a) = 3 × = 6.66 kgm2s–2


NA = 45
d
K2
NA NB 21. (b) Time of descent µ
R2

A B K2
Order of value of
R2
d
x d–x K2 1
for disc; 2 = = 0.5
R 2
W
17. (b) Applying angular momentum conservation K2 2
for sphere; 2 = = 0.4
R 5
V0
m (sphere) < (disc)
\ Sphere reaches first
22. (a) Given: Radius of disc, R = 50 cm
angular acceleration a = 2.0 rads–2; time t = 2s
Particle at periphery (assume) will have both radial
æ R0 ö
mV0R0 = (m) (V1) ç ÷ (one) and tangential acceleration
è 2 ø at = Ra = 0.5 × 2 = 1 m/s2
EBD_7179
210 PHYSICS

From equation, 24. (d) Centre of mass may or may not coincide with centre
w = w0 + at of gravity. Net torque of gravitational pull is zero about
w = 0 + 2 × 2 = 4 rad/sec centre of mass.
ac = w2R = (4)2 × 0.5 = 16 × 0.5 = 8m/s2 t g = Sti = Sri ´ mig = 0
Net acceleration,
Load
Mechanical advantage , M. A.=
atotal = a 2t + a 2c = 2
1 +8 2 » 8 m/s2 Effort
23. (b) Moment of inertia of complete disc about point 'O'. If M.A. > 1 Þ Load > Effort
M 25. (a) Here, Iw1 + Iw2 = 2Iw
MR 2
ITotal disc =
2 w1 + w2
Þ w=
Mass of removed disc O R 2
M R 1 2 1 2
MRemoved = (Mass µ area) (K.E.)i = Iw1 + Iw2
4 2 2
Moment of inertia of removed disc about point 'O'. 2
IRemoved (about same perpendicular axis) 1 æ w + w2 ö
(K.E.)f = ´ 2Iw2 = I ç 1
= Icm + mx2 2 è 2 ÷ø

.IN
M (R / 2)
2 2
3MR 2 1
MæRö Loss in K.E. = (K.E) f - (K.E)i = I(w1 - w2 )2
= + ç ÷ = 4
4 2 4è2ø 32
Therefore the moment of inertia of the remaining part
AL
26. (b) Given, mass of cylinder m = 3kg
of the disc about a perpendicular axis passing through R = 40 cm = 0.4 m
the centre, F = 30 N ; a = ? 40
N
IRemaing disc = ITotal – IRemoved As we know, torque t = Ia cm
F × R = MR2a
R

MR 2 3 13
= - MR 2 = MR 2 F´ R
U

2 32 32 a=
MR 2
JO

F = 30 N
30 ´ (0.4)
a= or, a = 25rad / s 2
3 ´ (0.4) 2
U
ED
8 Gravitation

NEWTON'S UNIVERSAL LAW OF GRAVITATION


GM e G(4 / 3pRe3r) 4
Gravitational force is an attractive force between any two point g= = = prGRe ……(iv)
2 2 3
Re Re
masses M1 and M2 separated by any distance r.
where r is the density of earth.

.IN
æM M ö This is the relation between universal gravitational constant
It is given by F = G ç 1 2 2 ÷ 'G' and acceleration due to gravity 'g'.
è r ø
Change in the Value of Acceleration due to Gravity (g)
AL
where G is the universal gravitational constant. (i) Due to rotation or latitude of earth : Let us consider a
Its value, G = 6.67 × 10–11 Nm2 kg–2 particle P at rest on the surface of earth at a latitude f.
Dimensions of G are [M–1 L3 T–2]
g ( f ) = g - Re w 2 cos 2 f
N
(a) The gravitational force is the central force and follows
R

inverse square law. It acts along the line joining the particles. w
(b) Since the work done by the gravitational force is
U

independent of the path followed and hence it is a r


f
JO

conservative force.
(c) It is the weakest force in nature. It is 1038 times smaller R g¢
than nuclear force and 1036 times smaller than electric force.
U

Strongest force being nuclear force (for small range)


followed by electric force.
ED

(d) Gravitation is independent of the presence of other bodies


At poles f = 90º, so g(f) = g ...(v)
around it.
At equator f = 0º so g(f) = g –w2Re ...(vi)
ACCELERATION DUE TO GRAVITY (g) (ii) Due to shape of earth : The shape of the earth is not
The force of attraction exerted by earth on a body of mass m is perfectly spherical. It is flattened at poles. The polar radius
the force of gravity. is 21 km less than the equatorial radius. Hence acceleration
due to gravity at poles is greater than at the equator.
So the force of gravity from Newton’s gravitational law is gp = 9.83 ms–2 = value of g at poles
GM e m ge = 9.78 ms–2 = values of g at equator
F= ……(i) i.e. gp > ge
r2 (iii) At a depth 'd' below the earth surface : The acceleration
where Me = mass of earth due to gravity at the surface of earth.
r = distance of the body from the centre of earth. GM e éæ 4 ö ù
g= = êç ÷pR e rú G (from eqn. iv) ...(vii)
The force of gravity can be written as F = mg ……(ii) Re 2
ë è 3 ø û
where g is called the acceleration due to gravity.
From the expression (i) and (ii), we get d
P
GM e R-d
g= ……(iii)
r2 O R
If body is located at the surface of earth i.e., r = Re (radius of
earth) then Earth
Surface of earth
EBD_7179
212 PHYSICS

If a body is taken at a depth d below the earth surface, then


the body is attracted by inner core of mass M' of earth (Mi )1 a 2
=
radius (Re– d). Then acceleration due to gravity at point P (M i )2 a1
is
(ff applied force on (Mi)1 and (Mi)2 is same.)
GM ¢ G é (4 / 3) p(R e - d)3 r ù (b) Gravitational mass Mg defined by Newton's law of
g¢ = = ë û
gravitation
(R e - d) 2 (R e - d) 2
= G ((4 / 3)p(R e - d)r) ...(viii) Fg W Weight of body
Mg = g = g = Accelerati on due to gravity
From eqns. (vii) and (viii) we have
æ R -d ö æ d ö 8. If two spheres of same material, mass and radius are put in
g¢ = g ç e ÷ = g ç1 - ÷ ...(ix) contact, the gravitational attraction between them is directly
è Re ø è Re ø proportional to the fourth power of the radius.
The value of acceleration due to gravity at the centre of the
earth is zero.
(iv) At a height 'h' above the earth surface : The acceleration R
2 B A
due to gravity at the surface of earth from expression (iv) 1 æ4 ö 4 R
is defined as F= Gç ps ÷ R
4 è3 ø
GM e m ¬ 2R ® m
g= 2
Re

.IN
This system may be considered to be made up of two mass
At a height h above the earth surface, the acceleration due each separated by 2R distance.
to gravity is
9. If the earth stops rotating about its axis, the value of g at
GM e GM e
AL the equator will increase by about 0.35%, but that at the
g¢ = = ...(x)
(R e + h ) 2 r2 poles will remain unchanged.
If the earth starts rotating at the angular speed of about 17
N
-2
æR +hö times the present value, there will be weightlessness on
Þ Þ g ¢ = gç e ÷ [from eqs. (ix) & (x)]
ç R ÷
R

è e ø
equator, but g at the poles will remain unchanged. In such a
case, the duration of the day will be about 84 min.
If h << Re
U

10. If the radius of planet decreases by n%, keeping the mass


æ 2h ö unchanged, the acceleration due to gravity on its surface
JO

g ¢ » g ç1 -
è R ÷ø
...(xi)
e
Dg - 2 DR
increases by 2n%. i.e., =
Keep in Memory g R
U

11. If the mass of the planet increases by m% keeping the


1. The value of the acceleration due to gravity on the moon radius constant, the acceleration due to gravity on its
ED

is about one sixth of that on the earth and on the sun is


Dg DM
about 27 times that on the earth. surface increases by m% i.e., = where R = constant.
g M
æ gE ö 12. If the density of planet decreases by r % keeping the radius
çè gs = 27g E and g M = ÷ø
6 constant, the acceleration due to gravity decreases by r%.
2. The value of g is minimum on the mercury, among all 13. If the radius of the planet decreases by r% keeping the
planets. density constant, the acceleration due to gravity decreases
3. For h<<R, the rate of decrease of the acceleration due to by r%.
gravity with height is twice as compared to that with depth.
Example 1.
4. The value of g increases with the increase in latitude. Its
value at latitude q is given by : gq= g – Rw2 cos2q. What will be the acceleration due to gravity on the surface
of the moon with its radius as 1/4th the radius of the earth
5. Rotation of the earth about its own axis is responsible for
and its mass as 1/80th the mass of earth?
decrease in the value of g with latitude.
Solution :
6. The weight of the body varies along with the value of g (i.e.
W = mg) GM e GM m
g= and g' = ;
7. Inertial mass and gravitational mass- Re 2
R m2
(a) Inertial mass (Mi) defined by Newton's law of motion
2 2
External force applied on the body g' M m æ R e ö 1 æ4ö 1
Mi = F/a = \ = ç ÷÷ = ´ ç ÷ = ; \ g' = g / 5
Acceleration produced in it g M e çè R m ø 80 è 1 ø 5
Gravitation 213

Example 2.
A mass M splits into two parts m and (M–m), which are R m
then separated by a certain distance. What will be the x
Center of earth
ratio m/M which maximizes the gravitational force
between the two parts?
Solution :
æ 4 px 3 ö
Gm (M - m) Gç d ÷m
F= ; For maxima, ç 3 ÷
x2 \F = è ø
2
dF G m 1 x
= 2 (M - 2m) = 0 or = 3M
dm x M 2 where d = density of earth = ;
Example 3. 4p R 3
Identical point masses each equal to m are placed at GMm
\F= .x [M is total mass of earth]
x = 0, x = 1, x = 2, x = 4, ... The total gravitational force R3
on mass m at x = 0 due to all other masses is Example 6.
(a) infinite (b) (4/3) Gm2 If the radius of the earth were to shrink by two per cent,
(c) (4/3 GM2) (d) zero its mass remaining the same, the acceleration due to
Solution : (c) gravity on the earth’s surface would
o m m m (a) decrease by 2% (b) increase by 2%

.IN
(c) increase by 4% (d) decrease by 4%
x=0 x=1 x=2 x=4 Solution : (c)
2 2 2
Gm Gm Gm
F0 = + + + .... GM
12
2 2
42
AL As g = 2 ; So, if R decreases then g increases.
R
é1 1 1 ù Taking logarithm of both the sides;
F0 = Gm 2 ê + + + ...ú
2 2 2 log g = log G + log M – 2 log R
N
ë1 2 4 û
It is a G.P. with common ratio 1/4. dg 2 dR
R

Differentiating it, we get =0+0– ;


é 1 ù
F0 = Gm 2 ê 4Gm 2 g R
\ ú F0 =
U

ë1 - 1 / 4 û 3 dg æ -2 ö 4
Example 4. \ = -2ç ÷=
g è 100 ø 100
JO

Four identical point masses each equal to m are placed


dg 4
at the corners of a square of side a. The force on a point \ % increase in g = ´100 = ´ 100 = 4%
mass m’ placed at the point of intersection of the two g 100
U

diagonals is Example 7.
(a) (4Gmm¢)/a2 (b) (2Gmm¢)/a2 An apple of mass 0.25 kg falls from a tree. What is the
ED

(c) (Gmm¢)/a2 (d) zero acceleration of the apple towards the earth? Also calculate
Solution : (d) the acceleration of the earth towards the apple.
The forces acting on the mass m¢ placed at the point of Given : Mass of earth = 5.983 × 1024 kg,
intersection of diagonals are balanced by each other as Radius of earth = 6.378 × 106 metre,
shown in figure. Therefore net force is zero. G = 6.67 × 10–11 Nm2kg–2.
Solution :
m m Mass of apple, m = 0.25 kg,
Mass of earth, M = 5.983 × 1024 kg,
F
F m¢ Radius of earth, R = 6.378 × 106 metre
Gravitational constant, G = 6.67 × 10–11 Nm2kg–2
F Let F be the gravitational force of attraction between the
F apple and earth.
m m GmM
Then, F=
Example 5. R2
A tunnel is dug along the diameter of the earth. Determine Let a be the acceleration of apple towards the earth.
the force on a particle of mass m placed in the tunnel at a
distance x from the centre. F GmM GM
a= = = 2
Solution : m mR 2 R
Force on a mass m placed at a distance x from the centre is
equal to the force of gravity due to a mass of spherical 6.67 ´ 10 -11 ´ 5.983 ´ 1024
volume of earth of radius x. Þa= ms-2 = 9.810 ms-2
(6.378 ´ 10 )6 2
EBD_7179
214 PHYSICS

Let a¢ be the acceleration of the earth towards the apple. (i) |Eg|= 0 at points inside the spherical shell (i.e. r < R)

F Gm -GM
a¢ = = (ii) | E g |= at the surface of shell (i.e. r = R)
M R2 R2
-GM
6.67 ´ 10-11 ´ 0.25 -2 (iii) | E g |= for outside the spherical shell (r >R).
= ms r2
(6.378 ´ 106 ) 2
It is clear that as r increases, E g decreases.
= 4.099 ´ 10 -25 ms -2 .
Gravitational intensity for solid sphere :
Example 8.
The acceleration due to gravity at the moon’s surface is Eg Solid
1.67 ms–2. If the radius of the moon is 1.74 × 106 m, calculate sphere
the mass of the moon. Use the known value of G. R
Solution :
GM gR 2 |Eg|
g= or M = r
R2 G
This relation is true not only for earth but for any heavenly
body which is assumed to be spherical. GM .r
(i) | E g |= - for points inside the solid sphere (r<R)
R3
Here, g = 1.67ms - , R = 1.74 ´ 10 m and
2 6

.IN
GM
G = 6.67 ´10-11 Nm 2 kg -2 (ii) At the surface of solid sphere (r = R) | E g |= -
R2
\ Mass of moon,
AL
GM
1.67 ´ (1.74 ´ 106 ) 2 (iii) Outside the solid sphere | E g |= - (r > R)
M= kg = 7.58 ´ 1022 kg r2
6.67 ´ 10-11
N
GRAVITATIONAL POTENTIAL
GRAVITATIONAL FIELD Gravitational potential at any point is the work done by
R

(or Gravitational Field Intensity) gravitational force in carrying a body of unit mass from infinity
The region or space around a body in which its gravitational to that point in gravitational influence of source.
U

influence is experienced by other bodies is the gravitational W -GM


JO

field of that body. i.e., VA = =


mo r
The gravitational field strength Eg, produced by a mass M at
any point P is defined as the force exerted on the unit mass where r is the distance of point from source mass M.
The S.I. unit of V is joule/kg. Its dimensions are [M0L2T–2]. It is
U

placed at that point P. Then


a scalar quantity.
ED

F æ G Mm ö GM Gravitational potential at the earth surface is


Eg = = m= 2 ...(i)
m çè r 2 ÷ø r -GM e
where m = test mass VA = = - gRe
Re
r = distance between M and m
Gravitational potential due to a uniform ring at a point on its
(i) The direction of E g always points towards the mass axis
producing it.
(ii) The gravitational field can be represented by gravitational
lines of force. a
(iii) The S.I unit of E g is newton/kg. Its dimensions are
q
[M0LT–2]. GM O r P
Vg = -
Gravitational intensity (Eg) for spherical shell a2 + r 2

Radius of
R spherical shell Gravitational potential due to a uniform thin spherical shell
Vg
GM
|Eg| | Eg |= 2 r
r O
|Eg|=0
r
O
GM
-
R
Gravitation 215

Relation between gravitational potential energy and


-GM
(i) at a point outside the shell, Vg = (r > R) gravitational potential.
r
Gravitational potential energy = gravitational potential × mass
GM
(ii) at a point inside the shell, Vg = - (r < R) Keep in Memory
R
where R = radius of spherical shell. 1. The gravitational potential energy of a mass m at a point
Potential due to a uniform spheral shell is constant throughout above the surface of the earth at a height h is given by
the cavity of the shell. -GMm
Gravitational potential due to a uniform solid sphere Ug = . The –ve sign shows that if h increases,
R+h
GM the gravitational PE decreases and becomes zero at infinity.
(i) at an external point Vg = - (r ³ R)
r 2. (a) If we take reference level to be at the surface of earth
(not at infinity) i.e., we assume that the gravitational
P.E of a mass m is zero at the surface of earth, then the
solid gravitational potential energy at a height h above the
R state surface of earth is (mgh), where h << Re (radius of
earth)
(b) The gravitational P.E of mass m on the earth’s surface
is

.IN
-GMe m
Ug = = -mgR
V Re
If we assume that gravitational P.E of mass m is zero
AL
at infinity i.e., we take reference level at infinity then
P.E.of any mass m at a height h above the earth surface
N
GM -GM e m
(ii) at an internal point Vg = - (3R2 - r 2 ) (r < R) Uh =
3
2R (R e + h )
R

So work done against the gravitational force, when


U

the particle is taken from surface of earth to a height


h above the earth surface
JO

Work done = change in potential energy


O r -GM e m GM e m @ GM e m ´ h
Þ Uh–Ug = +
Re + h Re R e (R e + h )
U

GM e m æ h ö
» ç ÷ (if h<<R)
ED

R
R e çè R e ÷
ø
æ GMe ö
-3GM = mç ÷h
(iii) Potential at the centre of solid sphere is V = ç R 2 ÷
2R è e ø
GRAVITATIONAL POTENTIAL ENERGY So, work done = mgh is stored in the particle as
Let Vg be the gravitational potential at a point. If we place a mass particle potenital energy, when kept at height ‘h’.
m at that point, then we say that the gravitational potential energy Example 9.
possessed by the mass is Four particles each of mass 1 kg are at the four corners of
a square of side 1m. Find the work done to remove one of
GM é GM ù
êë\ Vg = - r úû
U g = Vg ´ m Þ U g = - ´m the particles to infinity.
r Solution :
We can also therefore say that the gravitational potential energy The gravitational potential energy associated with any pair
of particle of mass m1 and m2 separated by a distance r is
of a system containing two masses m1 and m2 placed such that
the centre to centre distance between them is r then -Gm1m 2
U=
Gm1m2 r
Ug = - G ´ 1 ´ 1 2 G ´ 1´ 1
r Initial P.E. of system, – U1 = 4. +
This formula is valid taking into consideration the fact that we 1 2
have taken the gravitational potential of few at infinity. Please é4 2 + 2ù
remember that unless otherwise stated it is understood that the or U1 = -G ê ú
gravitational potential is taken to be zero at infinity. êë 2 úû
EBD_7179
216 PHYSICS

1 Kg 1m 1 Kg Example 11.
Find the gravitational potential energy of a system of four
particles, each having mass m, placed at the vertices of a

2m
1m 1m square of side l. Also obtain the gravitational potential at
2m the centre of the square.
Solution :
1 Kg 1m 1 Kg The system has four pairs with distance l and two diagonal
When one mass is removed, then pairs with distance 2l.
é G ´ 1´1 G ´ 1´1 ù m m
P.E. = U2 = - ê2. + ú
ë 1 2 û
é 2 2 + 1ù
U 2 = -ê úG ; 2l l
ëê 2 ûú

é 4 2 + 2 - 2 2 -1ù é 2 2 + 1ù
\ Work done = G ê ú = Gê ú m m
êë 2 úû êë 2 úû
[Q Work done = change in P.E. = –U2–[–U1] = U1–U2] Gm2 Gm2 2Gm 2 æ 1 ö Gm 2
U = -4 -2 =- 2 + = - 5.41

.IN
\
Example 10. l 2l l çè ÷
2ø l
A solid sphere of uniform density and mass M has a radius The gravitational potential at the centre of the square is
of 4m. Its centre is at the origin of coordinate system. Two
(r = )
spheres of radii 1m are taken out so that their centres are
AL 2l / 2
at P(0, –2, 0) and Q(0, 2, 0) respectively. This leaves two
V = Algebraic sum of potential due to each particle
spherical cavities. What is the gravitational field at the
N
centre of each cavity? 4 2Gm
z Þ V=-
l
R

Example 12.
U

The radius of earth is R. Find the work done in raising a


JO

P Q body of mass m from the surface of earth to a height R/2.


y
O Solution :
If a body of mass m is placed at a distance x from the centre
U

of earth, the gravitational force of attraction F between the


R
ED

GM m
body and the earth is F =
Solution : x2
If the cavities are not made, then the intensity at the point
P (or Q), due to whole sphere Small amount of work done in raising a body through a
small distance dx is given by
GM GM GM r r r r
I = 3 .x = ´2 = where I = IP + IQ + IR
R 64 32 GM m
r r r r dW = F dx = dx
\ IR = I - IP - IQ …… (i) x2
Where IR is the intensity of gravitational field at a point, Total work done in raising the body from the surface of
distance x from O due to remainder sphere (exclude cavity) earth to a height R/2 is given by
4p ´ (4) 3 d R +R / 2
GM m
Mass of big sphere M =
3 W= ò x2
dx
R
4π ´ (1)3 d M
Mass of small sphere P or Q, m = \m= 3 R/2 3 R/2
3 64 æ 1ö
Gm G æMö
= GMm ò x -2 dx = -GMm ç ÷
è xøR
R
At P, Ip = 0, I Q = 2
= .ç ÷
r 4 2 è 64 ø é 2 1ù GM m g R 2m
GM GM = - GMm ê - ú = =
\ IR = - [From eq (i)] ë 3R R û 3R 3R
32 1024
31GM 1
= mg R [\ GM = gR2]
\ IR = 3
1024
Gravitation 217

SATELLITES Energy of a satellite


A satellite is any body revolving around a large body under Total energy of satellite revolving in an orbit of radius r around
the gravitational influence of the latter. the earth can be calculated as follows :
The period of motion T of an artificial satellite of earth at a (i) The gravitational potential energy of a satellite of mass m
distance h above the surface of the earth is given by, GMm
is U g = – , where r is the radius of the orbit.
r
(Re + h)3
T = 2p 1 GMm
(ii) Kinetic energy of the satellite is Ek = mv0 =
2
gRe 2
2 2r
where, Re = radius of the earth GMm
where, g = acceleration due to gravity on the surface of the earth. (iii) So, total energy of the satellite E = U g + Ek = -
2r
Re The negative sign shows that satellite cannot leave the
If Re > > h, then T = 2p ; Re = 6.4 × 106 m ;
g GMm
g = 9.8 ms–2 orbit itself. It requires an energy equal to , which is
2R
i.e. T ; 84.58 minutes » 5075sec called Binding Energy (B.E.) of satellite.
Assuming Re + h = r, the distance of the satellite from the centre (iv) Total energy of a satellite at a height equal to the radius of
the earth
of the earth, T µ (r) 3 / 2
GMm GMm 1
=– =– = mgR

.IN
Orbital velocity (v0 ) 2( R + R ) 4R 4
Let a satellite of mass m revolve around the earth in circular orbit GEO-STATIONARY SATELLITE
of radius r with speed v0. The gravitational pull between satellite AL
A satelite which appears to be stationary for a person on the
and earth provides the necessary centripetal force.
surface of the earth is called geostationary satellite.
mv 20 It is also known as parking satellite or synchronous satellite.
Centripetal force required for the motion =
N
r (i) The orbit of the satellite must be circular and in the equatorial
plane of the earth.
R

GMm
Gravitational force = (ii) The angular velocity of the satellite must be in the same

U

direction as the angular velocity of rotation of the earth i.e.,


mv02 GMm from west to east.
GM
JO

= 2 or v 02 = (iii) The period of revolution of the satellite must be equal to the


r r r
period of rotation of earth about its axis.
GM i.e. 24 hours = 24 × 60 × 60 = 86400 sec.
U

or v0 = .......... (1)
r 1 1
æ GMT 2 ö 3 æ GM R 2T 2 ö 3
ED

4p²
g GM T2 = r³ or r=ç ÷ =ç 2 ´ ÷
or v0 = R [Q g = and r = (R + h)] GM è 4p 2 ø èR 4p 2 ø
R+h R2
1
GM e
Orbital velocity, Vo = = gRe = éê9.8 ´ (6.38 ´ 106 ) 2 ´
(86400)² ù 3
Re
ë 4p ² úû
(i) Value of orbital velocity does not depend on the mass of the = 42237 × 103 m = 42,237 km. » 42000 km.
satellite. h = r – R = 42000 – 6400 = 35600 km.
(ii) Around the earth the value of orbital velocity is
(a) Height of geostationary satellite from the surface of
7.92 km/sec. the earth is nearly 35600 km.
(iii) Greater is the height of the satellite, smaller is the orbital
(b) The orbital velocity of this satellite is nearly
velocity.
3.08 km/sec.
(iv) The direction of orbital velocity is along the tangent to the
(c) The relative velocity of geostationary satellite with
path.
respect to earth is zero.
(v) The work done by the satellite in a complete orbit is zero.
This type of satellite is used for communication
Angular momentum (L) : For satellite motion, angular momentum
purposes. The orbit of a geostationary satellite is called
will be given by
‘Parking Orbit’.
GM Polar Satellite :
L = mvr = mr i.e., L = [m2GMr]1/2
r Polar satellites travel around the earth in an orbit that travels
Angular momentum of a satellite depends on both, the mass of around the earth over the poles. The earth rotates on its axis as
orbiting and central body. It also depends on the radius of the the satellite goes around the earth. Thus over a period of many
orbits it looks down on every part of the earth.
orbit.
EBD_7179
218 PHYSICS

Different orbital shapes corresponding to different velocities of so v will be minimum when v¢ ® 0,


a satellite :
2GM é GM ù
[1] When v < v0 i.e. ve = vmin = = 2 gR ê as g = 2 ú
(i) The path is spiral. The satellite finally falls on the earth R ë R û
(ii) Kinetic energy is less then potential energy • The value of escape velocity does not depend upon the mass
(iii) Total energy is negative of the projected body, instead it depends on the mass and
radius of the planet from which it is being projected.
[2] When v = v0
• The value of escape velocity does not depend on the angle
(i) The path is circular
and direction of projection.
(ii) Eccentricity is zero • The value of escape velocity from the surface of the earth is
(iii) Kinetic energy is less than potential energy 11.2 km/sec.
(iv) Total energy is negative • The minimum energy needed for escape is = GMm/R.
[3] When v0 < v < ve • If the velocity of a satellite orbiting near the surface of the
(i) The path is elliptical earth is increased by 41.4%, then it will escape away from
(ii) Eccentricity < 1 the gravitational field of the earth.
(iii) Kinetic energy is less than potential energy • If a body falls freely from infinite distance, then it will reach
(iv) Total energy is negative the surface of earth with a velocity of 11.2 km/sec.
[4] When v = ve Relation between orbital velocity (V0) and escape speed (Ve)
(i) The path is a parabola Ve = 2 gR = 2 V0
(ii) Eccentricity = 1

.IN
(iii) Kinetic energy is equal to potential energy Keep in Memory
(iv) Total energy is zero
[5] When v > ve
AL æ g ö
(i) The path is a hyperbola 1. The escape velocity on moon is low ç as gm = E ÷ hence
è 6ø
(ii) Eccentricity > 1
there is no atmosphere on moon.
N
(iii) Kinetic energy is greater than potential energy
(iv) Total energy is positive 2. If the orbital radius of the earth around the sun be one
R

If the orbit of a satellite is elliptical fourth of the present value, then the duration of the year
will be one eighth of the present value.
U

GMm 3. The satellites revolve around the earth in a plane that


(1) The energy E = - = const. with ‘a’ as semi-major axis;
2a coincides with the great circle around the earth.
JO

(2) KE will be maximum when the satellite is closest to the central


body (at perigee) and minimum when it is farthest from the KEPLER'S LAWS OF PLANETARY MOTION
central body (at apogee) [as for a given orbit L = const.,
U

i.e., mvr = const., i.e., v µ 1/r] 1. The law of orbits : Each planet revolves about the sun in
(3) PE = (E – KE) will be minimum when KE = max, i.e., the an elliptical orbit with the sun at one of the foci of the
ED

satellite is closest to the central body (at perigee) and ellipse.


y
maximum when KE = min, i.e., the satellite is farthest from
the central body (at apogee). P
ESCAPE SPEED (Ve) Perihelion Sun Aphelion
It is the minimum speed with which a body should be projected
from the surface of a planet so as to reach at infinity i.e., beyond x
S¢ S
the gravitational field of the planet.
If a body of mass m is projected with speed v from the surface of ea ea
a planet of mass M and radius R, then
1 2 GMm 1 GMm a a
as K = mv and U = - , ES = mv 2 –
2 R 2 R a = Semi-major axis
Now if v' is the speed of body at ¥, then A planet of mass m moving in an elliptical orbit around the
sun. The sun of mass M, is at one focus S¢ of the ellipse.
1 1 The other focus is S, which is located in empty space. Each
E¥ = m(v ¢) 2 + 0 = m(v ¢ ) 2 [as U ¥ = 0 ]
2 2 focus is at distance ‘ea’ from the ellipse’s centre, with ‘e’
So by conservation of energy being the eccentricity of the ellipse and ‘a’ semimajor axis
1 2 GMm 1 of the ellipse, the perihelion (nearest to the sun) distance
= mv - = m (v ¢ ) 2 rmin., and the aphelion (farthest from the sun) distance r max.
2 R 2 are also shown.
1 2 GMm 1 rmax = a + e a = (1 + e) a
i.e. mv = + m (v ¢ ) 2
2 R 2 rmin = a – e a = (1 – e) a
Gravitation 219

The distance of each focus from the centre of ellipse is ea, so that it travels in a ballistic trajectory. A ballistic trajectory is
where e is the dimensionless number between 0 to 1 called the common type of trajectory you get by throwing a rock or a
the eccentricity. If e = 0, the ellipse is a circle. baseball, neglecting air friction. At every point on the trajectory,
For earth e = 0.017. the acceleration is equal to g downward since there is no support.
2. Law of areas : An imaginary line that connects a planet to A considerable amount of experimentation has been done with
the sun sweeps out equal areas in the plane of the planet's such ballistic trajectories to practice for orbital missions where
orbit in equal times; you experience weightlessness all the time.
i.e., the rate dA/dt at which it sweeps out area A is constant. The satellite is moving in a circular orbit, it has a radial acceleration

P3
P2 v20 GM é æ GM ö ù
a= = 2 êas v0 = ç ú
r r è r ÷ø ú
P4 A2 A1 P1 ëê û

S i.e., it is falling towards earth's centre with acceleration a,


so apparent weight of the body in it Wap = m(g´ – a)
where g´ is the acceleration due to gravity of earth at the position
If t P1P 2 = t P3P4 then A1 = A2 (height) of satellite, i.e. g´ = (GM/r2), so that
é GM GM ù
Wap = m ê 2 - 2 ú = 0
dA 1 (r)(v dt) 1 ë r r û
= = rv and as L = mvr

.IN
dt 2 dt 2 i.e., the apparent weight of a body in a satellite is zero and is
independent of the radius of the orbit.

dA
AL Keep in Memory
vdt
r 1. The moon takes 27.3 days to revolve around the earth. The
radius of its orbit is 3.85 × 105 km.
N

2. Kepler's second law is based on conservation of angular


dA L
R

so = ....(1) momentum.
dt 2m
3. Perihelion distance is the shortest distance between the
U

But as L = constt., (force is central, so torque = 0 and hence sun and the planet.
angular momentum of the planet is conserved)
JO

4. Aphelion distance is the largest distance between the Sun


areal velocity (dA/dt) = constant which is Kepler's IInd law, and the planet.
i.e., Kepler's IInd law or constancy of areal velocity is a
consequence of conservation of angular momentum. Vaphelion rperihelion
U

=
3. Law of periods : The square of the period of revolution of Vperihelion raphelion
ED

any planet is proportional to the cube of the semi-major


axis of the orbit, 5. If e is the eccentricity of the orbit
i.e., T2 µ r3. raphelion
1+ e
then =
T12 r13 1 - e rperihelion
or, =
T22 r23 raphelion + rperihelion = 2r
6. If e > 1 and total energy (KE + PE) > 0, the path of the
Satellite satellite is hyperbolic and it escapes from its orbit.
r 7. If e < 1 and total energy is negative it moves in elliptical
path.
Perigee Sun a Apogee 8. If e = 0 and total energy is negative it moves in circular
KE = Max KE = Min path.
PE = Min Focus Semi major
PE = Max 9. If e = 0 and total energy is zero it will take parabolic path.
axis b
b = Semi 10. The path of the projectiles thrown to lower heights is
minor axis parabolic and thrown to greater heights is elliptical.
rmin rmax 11. Kepler’s laws may be applied to natural and artificial
satellites as well.
WEIGHTLESSNESS 12. Gravitational force does not depend upon medium so no
The “weightlessness” you may feel in an aircraft occurs any time medium can shield it or block it.
the aircraft is accelerating downward with acceleration g. It is 13. The escape velocity and the orbital velocity are independent
possible to experience weightlessness for a considerable length of the mass of the body being escaped or put into the orbit.
of time by turning the nose of the craft upward and cutting power
EBD_7179
220 PHYSICS

Example 13. Example 15.


If the earth is at 1/4 of its present distance from the sun, A spaceship is launched into a circular orbit close to earth’s
what would be the duration of the year? surface. What additional velocity has now to be imparted
Solution : to the spaceship in the orbit to overcome the gravitational
pull? Radius of earth = 6400 km and g = 9.8 m/s2.
T12 R13 Solution :
We know that, =
T22 R 32 The orbital velocity of spaceship in circular orbit
Substituting the given values, we get GM GM
v0 = =
(1) 2
R 3
1 r R
= or = (4)3 = 64 (Q spaceship is very close to earth, r = R)
T22 æRö
3
T22
ç ÷
è4ø GM
As g = hence G M = g R2
R2
1 1
T22 = or T2 = year .
64 8 \ v0 = (R g) = (6.4 ´106 ) ´ (9.8) = 7.9195 km/s
Example 14.
Two satellites A and B go round a planet P in a circular Further, v e = 2Rg = ( 2 ´ 7.9195) = 11.2 km/s
orbits having radii 4R and R respectively. If the speed of (where ve is escape velocity)

.IN
satellite A is 3v, what will be the speed of satellite B? Additional velocity required = 11.2000 – 7.0195
Solution : = 3.2805 km/s
We know that v = (G M / r) So the velocity 3.2805 km/s must be added to orbital velocity
AL of spaceship.
æGMö æ GMö
Here 3v = ç ÷ and v' = ç ÷;
N
è 4R ø è R ø
R

v' æ GM ö æ 4R ö
\ = ç ÷´ ç ÷ = 2 ; v' = 6v
U

3v è R ø è GM ø
JO
U
ED
Gravitation 221

.IN
AL
N
R
U
JO
U
ED
EBD_7179
222 PHYSICS

1. A satellite is orbiting around the earth near its surface. If its 7. Due to rotation of the earth the acceleration due to gravity
kinetic energy is doubled, then g is
(a) it will remain in the same orbit. (a) maximum at the equator and minimum at the poles
(b) it will fall on the earth. (b) minimum at the equator and maximum at the poles
(c) it will revolve with greater speed. (c) same at both places
(d) it will escape out of the gravitational field of the earth. (d) None of these
2. There is no atmosphere on the moon because 8. A planet moves around the sun. At a point P it is closest
(a) it is closer to the earth and also it has the inactive inert from the sun at a distance d1 and has a speed v1. At another
gases in it. point Q, when it is farthest from the sun at a distnace d 2 its
(b) it is too for from the sun and has very low pressure in speed will be
its outer surface.
(c) escape velocity of gas molecules is greater than their (a) d12 v1 / d 22 (b) d 2 v1 / d1
root mean square velocity.
(d) escape velocity of gas molecules is less than their
(c) d1 v1 / d 2 (d) d 22 v1 / d12

.IN
root mean square velocity. 9. The weight of an object in the coal mine, sea level and at
3. The maximum kinetic energy of a planet moving around the the top of the mountain, are respectively W1, W2 and W3
sun is at a position then
AL
B (a) W1< W2 > W3 (b) W1= W2 = W3
(c) W1< W2 < W3 (d) W1> W2 > W3
N
10. Two planets of radii r1 and r2 are made from the same
A Sun C material. The ratio of the acceleration due to gravity g1/g2
R

at the surfaces of the two planets is


(a) r1/r2 (b) r2/r1
U

D
(a) A (b) B (c) (r1/r2)2 (d) (r2/r1)2
JO

(c) C (d) D 11. What would be the length of a sec. pendulum at a planet
4. A man waves his arms while walking. This is to (where acc. due to gravity is g/4) if it’s length on earth is l
(a) keep constant velocity (a) l/2 (b) 2 l
U

(b) ease the tension (c) l/4 (d) 4 l


(c) increase the velocity 12. Time period of a simple pendulum inside a satellite orbiting
ED

(d) balance the effect of earth’s gravity earth is


5. A missile is launched with a velocity less than escape (a) zero (b) ¥
velocity. The sum of its kinetic and potential energies is (c) T (d) 2 T
(a) zero 13. The ratio of the radii of the planets R1 and R2 is k. The ratio
(b) negative of the acceleration due to gravity is r. The ratio of the escape
(c) positive velocities from them will be
(d) may be positive, negative or zero.
(a) k r (b) kr
6. Which of the following graphs represents the motion of a
planet moving about the sun ?
(c) (k / r) (d) (r / k)
(a) (b)
14. If v e and v 0 represent the escape velocity and orbital
T2 T2 velocity of a satellite corresponding to a circular orbit of
radius R, then

R3 R3 (a) ve = vo (b) ve = 2 vo

(c) (d) (c) ve = (1/ 2) vo (d) ve and vo are not related


15. The kinetic energy needed to project a body of mass m from
T2 T2
the earth surface (radius R) to infinity is
(a) mgR/2 (b) 2mgR
R3 R3 (c) mgR (d) mgR/4.
Gravitation 223

16. The escape velocity of a body depends upon mass as


T T2
(a) m0 (b) m1 (a) (b)
2 R R
(c) m (d) m3.
17. The radius of a planet is 1/4th of Re and its acc. due to T2 T2
gravity is 2g. What would be the value of escape velocity (b) (d)
on the planet, if escape velocity on earth is ve. R2 R3
ve 22. The orbital velocity of an artificial satellite in a circular orbit
(a) (b) v e 2 very close to Earth is v. The velocity of a geosynchronous
2 satellite orbiting in a circular orbit at an altitude of 6R from
ve Earth's surface will be
(c) 2 ve (d)
2 v v
18. If the gravitational force had varied as r–5/2 instead of r –2; (a) (b)
7 6
the potential energy of a particle at a distance ‘r’ from the
centre of the earth would be proportional to (c) v (d) 6v
23. Escape velocity when a body of mass m is thrown vertically
(a) r -1 (b) r -2
from the surface of the earth is v, what will be the escape
(c) r -3 / 2 (d) r -5 / 2 velocity of another body of mass 4 m is thrown vertically
19. Two satellites revolve round the earth with orbital radii 4R (a) v (b) 2v

.IN
and 16R, if the time period of first satellite is T then that of (c) 4v (d) None of these
the other is 24. The potential energy of a satellite of mass m and revolving
(a) 4 T (b) 42/3 T AL at a height Re above the surface of earth where Re = radius
(c) 8 T (d) None of these of earth, is
20. A planet revolves in an elliptical orbit around the sun. The
-m g R e
semi-major and semi-minor axes are a and b. Then the square (a) – m g Re (b)
2
N
of time period, T is directly proportional to
-m g R e -m g R e
(a) a3 (b) b3
R

(c) (d)
3 4
3 3
æa+bö æa-bö
U

25. Energy required to move a body of mass m from an orbit of


(c) ç ÷ (d) ç ÷
è 2 ø è 2 ø radius 2R to 3R is
JO

21. Which of the following quantities do not depend upon the (a) GMm/12R2 (b) GMm/3R2
orbital radius of the satellite ? (c) GMm/8R (d) GMm/6R
U
ED

1. The escape velocity from the earth's surface is 11 km/s. The (a) – 5 unit (b) + 5 unit
escape velocity from a planet having twice the radius and (c) + 10 unit (d) + 15 unit
same mean density as that of earth is 5. A planet of mass 3 × 1029 gm moves around a star with a
(a) 5.5 km/s (b) 11 km/s constant speed of 2 × 106 ms–1 in a circle of radii 1.5 × 1014
(c) 22 km/s (d) None of these cm. The gravitational force exerted on the planet by the star is
2. Two point masses each equal to 1 kg attract one another (a) 6.67 × 1022 dyne (d) 8 × 1027 Newton
with a force of 10–10 N. The distance between the two 26
(c) 8 × 10 N (d) 6.67 × 1019 dyne
point masses is (G = 6.6 × 10–11 MKS units)
(a) 8 cm (b) 0.8 cm 6. The mass of the moon is 1/81 of earth’s mass and its radius
(c) 80 cm (d) 0.08 cm 1/4 that of the earth. If the escape velocity from the earth’s
3. There are two bodies of masses 103 kg and 105 kg separated surface is 11.2 km/sec, its value from the surface of the
by a distance of 1 km. At what distance from the smaller moon will be
body, the intensity of gravitational field will be zero (a) 0.14 kms–1 (b) 0.5 kms–1
(c) 2.5 kms –1 (d) 5.0 kms–1
(a) 1/9 km (b) 1/10 km
(c) 1/11 km (d) 10/11 km 7. If the mass of earth is eighty times the mass of a planet and
4. Taking the gravitational potential at a point infinte distance diameter of the planet is one fourth that of earth, then
away as zero, the gravitational potential at a point A is –5 acceleration due to gravity on the planet would be
unit. If the gravitational potential at point infinite distance (a) 7.8 m/s2 (b) 9.8 m/s2
away is taken as + 10 units, the potential at point A is (c) 6.8 m/s 2 (d) 2.0 m/s2
EBD_7179
224 PHYSICS

8. Two bodies of masses 10 kg and 100 kg are separated by a 18. A satellite of mass m revolves around the earth of radius R
distance of 2m ( G = 6.67 × 10–11 Nm 2 kg–2 ). The at a height ‘x’ from its surface. If g is the acceleration due to
gravitational potential at the mid point on the line joining gravity on the surface of the earth, the orbital speed of the
the two is satellite is
(a) 7.3 × 10–7 J/kg (b) 7.3 × 10–9 J/kg
–9
(c) –7.3 × 10 J/kg (d) 7.3 × 10–6 J/kg gR 2 gR
(a) (b)
9. The time period of a satellite of earth is 5 hours. If the R+x R-x
separation between the earth and the satellite is increased
to 4 times the previous value, the new time period will become æ gR 2 ö 1/ 2
(a) 10 hours (b) 80 hours (c) gx (d) ç ÷
çR+x÷
(c) 40 hours (d) 20 hours è ø
10. At sea level, a body will have minimum weight at
19. The time period of an earth satellite in circular orbit is
(a) pole (b) equator
independent of
(c) 42° south latitude (d) 37° north latitude
(a) both the mass and radius of the orbit
11. The distance of neptune and saturn from the sun is nearly
1013 and 1012 meter respectively. Assuming that they move (b) radius of its orbit
in circular orbits, their periodic times will be in the ratio (c) the mass of the satellite
(a) 10 (b) 100 (d) neither the mass of the satellite nor the radius of its orbit.

.IN
(c) 10 10 (d) 1000 20. Suppose the gravitational force varies inversely as the nth
power of distance. Then the time period of a planet in circular
12. A geostationary satellite is orbiting the earth at a height of orbit of radius ‘R’ around the sun will be proportional to
5R above that surface of the earth, R being the radius of the
AL æ n -1 ö
earth. The time period of another satellite in hours at a ç ÷
height of 2R from the surface of the earth is (a) Rn (b) Rè 2 ø
N
(a) 5 (b) 10 æ n -2 ö
æ n +1 ö ç ÷
6 ç ÷
Rè 2 ø
R

(c) 6 2 (d) (c) Rè 2 ø (d)


2
21. An earth satellite of mass m revolves in a circular orbit at a
U

13. The escape velocity for a body projected vertically upwards


from the surface of earth is 11 km/s. If the body is projected height h from the surface of the earth. R is the radius of the
earth and g is acceleration due to gravity at the surface of
JO

at an angle of 45º with the vertical, the escape velocity will be


the earth. The velocity of the satellite in the orbit is given
(a) 22 km/s (b) 11 km/s
by
11
U

(a) g R2/(R + h)
(c) km/s (d) 11 2 km/s
2 (b) g R
ED

14. If the length of a simple pendulum is increased by 2%, then (c) g R/(R – h)
the time period
(a) increases by 2% (b) decreases by 2% (d) ég R 2 / (R + h) ù
ë û
(c) increases by 1% (d) decreases by 1%
22. Mass of the Earth has been determined through
15. The kinetic energy of a satellite in its orbit around the earth
is E. What should be the kinetic energy of the satellite so as T2
(a) use of Kepler's constancy law and Moon's period
to enable it to escape from the gravitational pull of the earth? R3
(a) 4 E (b) 2 E (b) sampling the density of Earth's crust and using Earth's
radius
(c) 2E (d) E
(c) Cavendish's determination of G and using Earth radius
16. The time period of a satellite in a circular orbit of radius R is and g at its surface
T, the period of another satellite in a circular orbit of radius
(d) use of periods of satellites at different heights above
4 R is
Earth's surface and known radius of Earth
(a) 4 T (b) T/4
(c) 8 T (d) T/8 23. Consider Earth to be a homogeneous sphere. Scientist A
goes deep down in a mine and scientist B goes high up in a
17. If the change in the value of g at the height h above the
balloon. The gravitational field measured by
surface of the earth is the same as at a depth ‘x’ below it,
then (both x and h being much smaller than the radius of the (a) A goes on decreasing and that by B goes on increasing
earth) (b) B goes on decreasing and that by A goes on increasing
(a) x = h (b) x = 2 h (c) each decreases at the same rate
(c) x = h/2 (d) x = h2 (d) each decreases at different rates
Gravitation 225

24. There are _______ gravitational lines of force inside a 30. If value of acceleration due to gravity changes from one
spherically symmetric shell. place to another, which of the following forces will undergo
(a) infinitely many a change ?
(b) zero (a) Viscous force (b) Buoyant force
(c) Magnetic force (d) All of the above
(c) varying number depending upon surface area
31. The amount of work done in lifting a mass ‘m’ from the
(d) varying number depending upon volume surface of the earth to a height 2R is
25. A uniform ring of mass m and radius r is placed directly (a) 2mgR (b) 3mgR
above a uniform sphere of mass M and of equal radius. The
centre of the ring is directly above the centre of the sphere 3 2
(c) mgR (d) mgR
2 3
at a distance r 3 as shown in the figure.
32. The radius of the earth is 4 times that of the moon and its
The gravitational force exerted by the sphere on the ring mass is 80 times that of the moon. If the acceleration due to
will be gravity on the surface of the earth is 10 m/s2, then on the
surface of the moon its value will be
GMm r
(a) 1 ms–2 (b) 2 ms–2
(a)
8r 2 (c) 3 ms–2 (d) 4 ms–2
3r 33. A satellite of mass ‘m’, moving around the earth in a circular
GMm 2r

.IN
(b) orbit of radius R, has angular momentum L. The areal velocity
2
4r of satellite is (Me = mass of earth)
(a) L /2m (b) L /m
GMm
(c) 3
AL (c) 2L /m (d) 2L /Me
8r 2 34. A solid sphere of uniform density and radius R applies a
gravitational force of attraction equal to F1 on a particle
N
GMm
(d) placed at A, distance 2R from the centre of the sphere.
8r 3 3
R

26. The gravitational potential difference between the surface


U

of a planet and a point 20 m above the surface is 2 joule/kg. A


If the gravitational field is uniform, then the work done in
JO

carrying a 5 kg body to a height of 4 m above the surface is


(a) 2 J (b) 20 J
R R
U

(c) 40 J (d) 10 J
27. The ratio of the kinetic energy required to be given to a A spherical cavity of radius R/2 is now made in the sphere
ED

satellite so that it escapes the gravitational field of Earth to as shown in the figure. The sphere with cavity now applies
the kinetic energy required to put the satellite in a circular a gravitational force F2 on the same particle placed at A.
orbit just above the free surface of Earth is The ratio F2/F1 will be
(a) 1 (b) 2 (a) 1/2 (b) 3
(c) 3 (d) 9 (c) 7 (d) 1/9
35. A body starts from rest from a point distance R0 from the
28. The radii of two planets are respectively R1 and R2 and
centre of the earth. The velocity acquired by the body when
their densities are respectively r1 and r2. The ratio of the
it reaches the surface of the earth will be (R represents
accelerations due to gravity at their surfaces is
radius of the earth).
r1 r2
(a) g1 : g 2 = : æ1 1 ö æ 1 1ö
R12 R 22 (a) 2 G M çç - ÷ (b) 2 G M çç - ÷÷
÷ R R
è R R0 ø è 0 ø
(b) g1 : g 2 = R1R 2 : r1r 2
æ1 1 ö æ1 1 ö
(c) G M çç - ÷ (d) 2 G M çç - ÷
÷
(c) g1 : g 2 = R1r 2 : R 2r1 ÷
è R R0 ø è R R0 ø

(d) g1 : g 2 = R1r1 : R 2r 2 36. The largest and the shortest distance of the earth from the
sun are r1 and r2. Its distance from the sun when it is at
29. An artificial satellite moving in a circular orbit around the perpendicular to the major-axis of the orbit drawn from the sun
earth has a total (K.E. + P.E.) is E0. Its potential energy is –
(a) (r1 + r2 ) / 4 (b) (r1 + r2 ) /(r1 - r2 )
(a) –E0 (b) 1.5 E0
(c) 2E0 (d) E0 (c) 2 r1 r2 /(r1 + r2 ) (d) (r1 + r2 ) / 3
EBD_7179
226 PHYSICS

37. The orbital velocity of an artificial satellite in a circular orbit 44. If there were a small gravitational effect, then which of the
just above the centre’s surface is u. For a satellite orbiting following forces will undergo a change?
at an altitude of half of the earth’s radius, the orbital velocity is (a) Viscous force (b) Electrostatic force
(c) Magnetic force (d) Archimedes' uplift
æ æ 2öö 2 45. The gravitational force of attraction between a uniform
(a) ç ç ÷ ÷v (b) v0
ç è 3ø÷ 0 3 sphere of mass M and a uniform rod of length l and mass m
è ø
oriented as shown is
3 æ 3ö
(c) v0 (d) ç ÷v0
2 è 2ø
M m
38. A planet is revolving around the sun in an elliptical orbit.
Its closests distance from the sun is rmin. The farthest
distance from the sun is rmax. If the orbital angular velocity r l
of the planet when it is nearest to the sun is w, then the
orbital angular velocity at the point when it is at the farthest GMm GM
distance from the sun is (a) (b)
r(r + l ) r2
(a) ( rmin / rmax ) w (b) ( rmax / rmin ) w
(c) Mmr 2 + l (d) (r 2 + l )mM

.IN
2 2 2 2
(c) (rmax / rmin ) w (d) (rmin / rmax ) w
39. Two spherical bodies of mass M and 5M and radii R and 2R 46. Explorer 38, a radio-astronomy satellite of mass
respectively are released in free space with initial separation 3R
between their centres equal to 12 R. If they attract each
AL 200 kg, circles the Earth in an orbit of average radius
2
other due to gravitational force only, then the distance
where R is the radius of the Earth. Assuming the gravitational
covered by the smaller body just before collision is
N
pull on a mass of 1 kg at the earth's surface to be 10 N,
(a) 2.5 R (b) 4.5 R calculate the pull on the satellite
R

(c) 7.5 R (d) 1.5 R (a) 889 N (b) 89 N


40. If the radius of the earth were to shrink by one per cent, its (c) 8889 N (d) 8.9 N
U

mass remaining the same, the acceleration due to gravity 47. Suppose, the acceleration due to gravity at the Earth's
on the earth’s surface would surface is 10 m s–2 and at the surface of Mars it is
JO

(a) decrease (b) remain unchanged 4.0 m s–2. A 60 kg pasenger goes from the Earth to the Mars
(c) increase (d) None of these in a spaceship moving with a constant velocity. Neglect all
U

41. If earth is supposed to be a sphere of radius R, if g 30 is other objects in the sky. Which part of figure best represents
the weight (net gravitational force) of the passenger as a
value of acceleration due to gravity at lattitude of 30° and g
ED

function of time?
at the equator, the value of g – g30 is
Weight
1 2 3 2
(a) w R (b) w R
4 4 600 N A

1 2
(c) w2R (d) w R B
2
200 N
42. A ball is dropped from a satellite revolving around the earth
at height of 120 km. The ball will C
(a) continue to move with same speed along a straight D t0 time
line tangentially to the satellite at that time
(b) continue to move with same speed along the original (a) A (b) B
orbit of satellite. (c) C (d) D
(c) fall down to earth gradually 48. A projectile is fired vertically from the Earth with a velocity
kve where ve is the escape velocity and k is a constant less
(d) go far away in space
than unity. The maximum height to which projectile rises,
43. Two identical geostationary satellites are moving with equal
as measured from the centre of Earth, is
speeds in the same orbit but their sense of rotation brings
them on a collision course. The debris will R R
(a) (b)
(a) fall down k k -1
(b) move up
R R
(c) begin to move from east to west in the same orbit (c) 2 (d)
(d) begin to move from west to east in the same orbit 1- k 1+ k2
Gravitation 227

49. A diametrical tunnel is dug across the Earth. A ball is 53. The percentage change in the acceleration of the earth
dropped into the tunnel from one side. The velocity of the towards the sun from a total eclipse of the sun to the point
ball when it reaches the centre of the Earth is .... (Given : where the moon is on a side of earth directly opposite to
3 GM the sun is
gravitational potential at the centre of Earth = – )
2 R
2
(a) R (b) gR Ms r2 Ms æ r2 ö
(a) ´ 100 (b) ´ 100
M m r1 M m çè r1 ÷ø
(c) 2.5gR (d) 7.1gR
50. A man of mass m starts falling towards a planet of mass M
2 2
and radius R. As he reaches near to the surface, he realizes æ r ö Mm æ r1 ö M s
that he will pass through a small hole in the planet. As he (c) 2ç 1 ÷ ´ 100 (d) ç ÷ ´ 100
è r2 ø Ms è r2 ø M m
enters the hole, he sees that the planet is really made of two
pieces a spherical shell of negligible thickness of mass 2M/
54. A satellite is revolving round the earth in an orbit of radius
3 and a point mass M/3 at the centre. Change in the force of
r with time period T. If the satellite is revolving round the
gravity experienced by the man is
earth in an orbit of radius r + D r (Dr << r) with time period T
2 GMm
(a) (b) 0 + D T then,
3 R2

.IN
DT 3 Dr DT 2 Dr
1 GMm 4 GMm (a) = (b) =
(c) 2 (d) T 2 r T 3 r
3 R 3 R2 AL
51. In a region of only gravitational field of mass 'M' a particle
DT Dr DT Dr
is shifted from A to B via three different paths in the figure. (c) = (d) =-
The work done in different paths are W1 , W2 , W 3 T r T r
N
respectively then 55. A cavity of radius R/2 is made inside a solid sphere of
radius R. The centre of the cavity is located at a distance R/
R

3
B C 2 from the centre of the sphere. The gravitational force on a
U

M particle of mass ‘m’ at a distance R/2 from the centre of the


2 sphere on the line joining both the centres of sphere and
JO

1 cavity is – (opposite to the centre of gravity)


[Here g = GM/R², where M is the mass of the sphere]
U

A
mg 3mg
ED

(a) W1 = W2 = W3 (b) W1 > W2 > W3 (a) (b)


2 8
(c) W1 = W2 > W3 (d) W1 < W2 < W3
52. Four similar particles of mass m are orbiting in a circle of mg
radius r in the same angular direction because of their mutual (c) (d) None of these
16
gravitational attractive force. Velocity of a particle is given by
56. A spherical uniform planet is rotating about its axis. The
m
velocity of a point on its equator is V. Due to the rotation of
planet about its axis the acceleration due to gravity g at
equator is 1/2 of g at poles. The escape velocity of a particle
m r on the pole of planet in terms of V is
m
(a) Ve = 2V (b) Ve = V

(c) Ve = V/2 (d) Ve = 3V


m
57. The escape velocity from a planet is ve. A tunnel is dug
1/ 2
along a diameter of the planet and a small body is dropped
é GM æ 1 + 2 2 ö ù GM into it at the surface. When the body reaches the centre of
(a) ê ç ÷ú (b) 3
êë r è 4 ø úû r the planet, its speed will be

1/ 2 (a) v e (b) ve / 2
é 1 GM æ 1 + 2 ö ù
(c)
GM
r
(
1+ 2 2 ) (d) ê ç
ëê 2 r è 2 ø úû
÷ú (c) ve/2 (d) zero
EBD_7179
228 PHYSICS

58. A (nonrotating) star collapses onto itself from an initial 63. A satellite is launched into a circular orbit of radius R around
radius Ri with its mass remaining unchanged. Which curve the earth. A second satellite is launched into an orbit of
in figure best gives the gravitational acceleration ag on the radius 1.01 R. The period of second satellite is larger than
surface of the star as a function of the radius of the star the first one by approximately
during the collapse (a) 0.5% (b) 1.0%
ag (c) 1.5% (d) 3.0%
64. If gE and gM are the accelerations due to gravity on the
surfaces of the earth and the moon respectively and if
d
Millikan’s oil drop experiment could be performed on the
b two surfaces, one will find the ratio
a electronic charge on the moon
c to be
electronic charge on the earth
R
Ri (a) gM / gE (b) 1
(a) a (b) b
(c) 0 (d) gE / gM
(c) c (d) d
59. The earth is assumed to be sphere of radius R. A platform is 65. The density of a newly discovered planet is twice that of
arranged at a height R from the surface of Earth. The escape earth. The acceleration due to gravity at the surface of the
velocity of a body from this platform is kv, where v is its planet is equal to that at the surface of the earth. If the

.IN
escape velocity from the surface of the earth. The value of radius of the earth is R, the radius of the planet would be
k is
(a) ½ R (b) 2 R
1 1
AL
(a) (b) (c) 4 R (d) 1/4 R
2 3
66. Imagine a new planet having the same density as that of
N
1 earth but it is 3 times bigger than the earth in size. If the
(c) (d) 2 acceleration due to gravity on the surface of earth is g and
2
R

that on the surface of the new planet is g’, then


60. Four equal masses (each of mass M) are placed at the corners
(a) g’ = g/9 (b) g’ = 27g
U

of a square of side a. The escape velocity of a body from


the centre O of the square is (c) g’=9g (d) g’=3g
JO

67. For a satellite moving in an orbit around the earth, the ratio
2GM 8 2 GM of kinetic energy to potential energy is
(a) 4 (b)
a a
U

1 1
(a) (b)
2 2
ED

4GM 4 2 GM
(c) (d)
a a (c) 2 (d) 2
68. The figure shows elliptical orbit of a planet m about the
61. A planet of mass m moves around the sun of mass M in an
sun S. The shaded area SCD is twice the shaded area SAB.
elliptical orbit. The maximum and minimum distance of the
If t1 is the time for the planet to move from C to D and t 2 is
planet from the sun are r1 and r2 respectively. The time
the time to move from A to B then :
period of planet is proportional to
3/ 2 v
æ ö m
(a) r12 / 5 (b) çr + r ÷
è 1 2ø B C
3/ 2
æ ö
(c) ç r1 – r2 ÷ (d) r 3/ 2 S
è ø D
62. The change in potential energy, when a body of mass m is A
raised to a height nR from the earth’s surface is (R = radius
of earth)
(a) t1 = 4t2 (b) t1 = 2t2
æ n ö
(c) t1 = t2 (d) t1 > t2
(a) mgRç ÷ (b) nmgR
ç n – 1÷ 69. The radii of circular orbits of two satellites A and B of the
è ø
earth, are 4R and R, respectively. If the speed of satellite A
æ n2 ö is 3 V, then the speed of satellite B will be
æ n ö
(c) mgR ç ÷ (d) mgR ç ÷ (a) 3 V/4 (b) 6 V
ç n 2 +1 ÷ è n +1 ø
è ø (c) 12 V (d) 3 V/2
Gravitation 229

70. A particle of mass M is situated at the centre of a spherical


(a) I (b) I
shell of same mass and radius a. The gravitational potential
a
at a point situated at distance from the centre, will be
2
3GM 2 GM r=a r
(a) - (b) - r=a r
a a
(c) I (d) I
GM 4 GM
(c) - (d) -
a a
71. A planet moving along an elliptical orbit is closest to the
sun at a distance r 1 and farthest away at a distance of r2. If r=a r
r=a r
v1 and v 2 are the linear velocities at these points
v1 Directions for Qs. (74 to 75) : Each question contains
respectively, then the ratio v is STATEMENT-1 and STATEMENT-2. Choose the correct answer
2
(a) (r1/r2)2 (b) r2/r1 (ONLY ONE option is correct ) from the following
(c) (r2/r1)2 (d) r1/r2 (a) Statement -1 is false, Statement-2 is true
72. A particle of mass m is thrown upwards from the surface of (b) Statement -1 is true, Statement-2 is true; Statement -2 is a
the earth, with a velocity u. The mass and the radius of the correct explanation for Statement-1

.IN
earth are, respectively, M and R. G is gravitational constant (c) Statement -1 is true, Statement-2 is true; Statement -2 is not
and g is acceleration due to gravity on the surface of the a correct explanation for Statement-1
earth. The minimum value of u so that the particle does not AL
return back to earth, is (d) Statement -1 is true, Statement-2 is false
74. Statement -1 : For the planets orbiting around the sun,
2GM 2GM angular speed, linear speed and K.E. changes with time,
(a) (b)
N
R R2 but angular momentum remains constant.
Statement -2 : No torque is acting on the rotating planet.
R

2GM
(c) 2gR 2 (d) So its angular momentum is constant.
R2
U

75. Statement -1 : Gravitational potential is maximum at infinity.


73. Which one of the following graphs represents correctly
JO

the variation of the gravitational field intensity (I) with the Statement -2 : Gravitational potential is the amount of work
distance (r) from the centre of a spherical shell of mass M done to shift a unit mass from infinity to a given point in
and radius a ? gravitational attraction force field.
U
ED

Exemplar Questions (c) not be true because the major gravitational force on
mercury is due to the sun
1. The earth is an approximate sphere. If the interior contained
matter which is not of the same density everywhere, then (d) not be true because mercury is influenced by forces
on the surface of the earth, the acceleration due to gravity other than gravitational force
(a) will be directed towards the centre but not the same 3. Different points in the earth are at slightly different
everywhere distances from the sun and hence experience different
(b) will have the same value everywhere but not directed forces due to gravitation. For a rigid body, we know that if
towards the centre various forces act at various points in it, the resultant motion
(c) will be same everywhere in magnitude directed towards is as if a net force acts on the CM (centre of mass) causing
the centre translation and a net torque at the CM causing rotation
around an axis through the CM. For the earth-sun system
(d) cannot be zero at any point
(approximating the earth as a uniform density sphere).
2. As observed from the earth, the sun appears to move in an
(a) the torque is zero
approximate circular orbit. For the motion of another planet
like mercury as observed from the earth, this would (b) the torque causes the earth to spin
(a) be similarly true (c) the rigid body result is not applicable since the earth
(b) not be true because the force between the earth and is not even approximately a rigid body
mercury is not inverse square law (d) the torque causes the earth to move around the sun
EBD_7179
230 PHYSICS

4. Satellites orbitting the earth have finite life and sometimes NEET/AIPMT (2013-2017) Questions
debris of satellites fall to the earth. This is because 9. A body of mass ‘m’ is taken from the earth’s surface to the
(a) the solar cells and batteries in satellites run out height equal to twice the radius (R) of the earth. The change
(b) the laws of gravitation predict a trajectory spiralling in potential energy of body will be [2013]
inwards 2
(c) of viscous forces causing the speed of satellite and (a) mgR (b) 3 mgR
3
hence height to gradually decrease
1
(d) of collisions with other satellites (c) mgR (d) mg2R
3
5. Both the earth and the moon are subject to the gravitational
force of the sun. As observed from the sun, the orbit of the 10. Infinite number of bodies, each of mass 2 kg are situated on
moon x-axis at distances 1m, 2m, 4m, 8m, ..... respectively, from the
origin. The resulting gravitational potential due to this
(a) will be elliptical
system at the origin will be [2013]
(b) will not be strictly elliptical because the total
8 4
gravitational force on it is not central (a) - G (b) - G
3 3
(c) is not elliptical but will necessarily be a closed curve
(c) – 4 G (d) – G
(d) deviates considerably from being elliptical due to 11. The radius of a planet is twice the radius of earth. Both
influence of planets other than the earth have almost equal average mass-densities. If VP and VE are

.IN
6. In our solar system, the inter-planetary region has chunks escape velocities of the planet and the earth, respectively,
of matter (much smaller in size compared to planets) called then [NEET Kar. 2013]
asteroids. They (a) VE = 1.5VP (b) VP = 1.5VE
AL
(a) will not move around the sun, since they have very (c) VP = 2VE (d) VE = 3VP
small masses compared to the sun 12. A particle of mass ‘m’ is kept at rest at a height 3R from the
surface of earth, where ‘R’ is radius of earth and ‘M’ is mass
N
(b) will move in an irregular way because of their small
masses and will drift away into outer space of earth. The minimum speed with which it should be
R

projected, so that it does not return back, is (g is acceleration


(c) will move around the sun in closed orbits but not obey
due to gravity on the surface of earth) [NEET Kar. 2013]
Kepler's laws
U

1 1
(d) will move in orbits like planets and obey Kepler's laws æ GM ö 2 æ GM ö 2
JO

(a) ç ÷ (b) ç ÷
7. Choose the wrong option. è R ø è 2R ø
(a) Inertial mass is a measure of difficulty of accelerating 1 1
a body by an external force whereas the gravitational æ gR ö 2 æ 2g ö 2
U

(c) ç ÷ (d) ç ÷
mass is relevant in determining the gravitational force è 4 ø è 4 ø
ED

on it by an external mass
13. Dependence of intensity of gravitational field (E) of earth
(b) That the gravitational mass and inertial mass are equal with distance (r) from centre of earth is correctly represented
is an experimental result by: [2014]
(c) That the acceleration due to gravity on the earth is the
same for all bodies is due to the equality of gravitational E E
mass and inertial mass O R
(a) (b) O
r R r
(d) Gravitational mass of a particle like proton can depend
on the presence of neighbouring heavy objects but
the inertial mass cannot
E E
8. Particles of masses 2M, m and M are respectively at points A,
R
1 (c) O (d) O
B and C with AB = ( BC ) × m is much-much smaller than r R r
2
M and at time t = 0, they are all at rest as given in figure.
At subsequent times before any collision takes place. 14. A black hole is an object whose gravitational field is so
strong that even light cannot escape from it. To what
A r B C
approximate radius would earth (mass = 5.98 × 1024 kg)
2M m M have to be compressed to be a black hole? [2014]
(a) m will remain at rest (a) 10– 9 m (b) 10– 6 m
(b) m will move towards M
(c) 10– 2 m (d) 100 m
(c) m will move towards 2M
(d) m will have oscillatory motion
Gravitation 231

15. Two spherical bodies of mass M and 5 M and radii R and 2 (b) the linear momentum of S remains constant in
R released in free space with initial separation between their magnitude.
centres equal to 12 R. If they attract each other due to (c) the acceleration of S is always directed towards the
gravitational force only, then the distance covered by the centre of the earth.
smaller body before collision is [2015]
(d) the angular momentum of S about the centre of the
(a) 4.5 R (b) 7.5 R earth changes in direction, but its magnitude remains
(c) 1.5 R (d) 2.5 R constant.
16. Kepler's third law states that square of period of revolution 19. The ratio of escape velocity at earth (ve) to the escape
(T) of a planet around the sun, is proportional to third power velocity at a planet (vp) whose radius and mean density are
of average distance r between sun and planet i.e. T 2 = Kr3 twice as that of earth is :
here K is constant. If the masses of sun and planet are M
(a) 1 : 2 (b) 1 : 2 2 [2016]
and m respectively then as per Newton's law of gravitation
(c) 1 : 4 (d) 1 : 2
GMm 20. At what height from the surface of earth the gravitational
force of attraction between them is F = 2 , here G is
r potential and the value of g are –5.4 × 107 J kg–1 and 6.0 ms–2
gravitational constant. The relation between G and K is
respectively ?
described as [2015]
Take the radius of earth as 6400 km : [2016]
(a) GMK = 4p2 (b) K = G
(a) 2600 km (b) 1600 km
1

.IN
(c) K = (d) GK = 4p2 (c) 1400 km (d) 2000 km
G 21. Two astronauts are floating in gravitation free space after
17. A remote - sensing satellite of earth revolves in a circular AL having lost contact with their spaceship. The two will
orbit at a height of 0.25 × 106 m above the surface of earth. (a) move towards each other. [2017]
If earth's radius is 6.38 × 106 m and g = 9.8 ms–2, then the (b) move away from each other.
orbital speed of the satellite is: [2015 RS] (c) become stationary
N
(a) 8.56 km s–1 (b) 9.13 km s–1 (d) keep floating at the same distance between them.
22. The acceleration due to gravity at a height 1 km above the
R

(c) 6.67 km s–1 (d) 7.76 km s–1 earth is the same as at a depth d below the surface of earth.
U

18. A satellite S is moving in an elliptical orbit around the earth. Then [2017]
The mass of the satellite is very small compared to the mass 3
JO

(a) d = 1 km (b) d = km
of the earth. Then, [2015 RS] 2
(a) the total mechanical energy of S varies periodically 1
(c) d = 2 km (d) d = km
U

with time. 2
ED
EBD_7179
232 PHYSICS

Hints & Solutions


EXERCISE - 1 l
1. (d) 2. (d) 3. (a) 4. (d) 12. (b) Since T = 2π
g
5. (b) 6. (c) 7. (b)
r r but inside the satellite g = 0
8. (c) In planetary motion τ ext. = 0 Þ L = constant So T = ¥
r r r r
L = r ´ p (= mv ) = mrv (Q q = 90º) 13. (b) We know that, ve = (2 g R)
So m1d1v1 = m2d2v2 (here r = d)
(ve ) P1 (2 g1 R 1 )æg ö æR ö
vd \ = = ç 1 ÷´ ç 1 ÷ = kr
Þ v2 = 1 1 (v e ) P2 (2 g 2 R 2 ) g
è 2ø è R2 ø
d2
14. (b) v e = 2vo where ve and vo are the escape velocity
d1 d2 v2 and orbital velocity respectively.
v1
sun 1 2 1
15. (c) KE = mvesc = m ( 2gR ) 2 = mgR .
2 2

.IN
9. (a) At the surface of earth, the value of g = 9.8m/sec2. If
16. (a) vesc = 2 gR , where R is radius of the planet.
we go towards the centre of earth or we go above the
surface of earth, then in both the cases the value of g AL Hence escape velocity is independent of m.
decreases. 17. (a) The escape velocity on the earth is defined as
Hence W1=mgmine, W2=mgsea level, W3=mgmoun ve = 2g e R e
So W1< W2 > W3 (g at the sea level = g at the
N
Where Re & ge are the radius & acceleration due to
suface of earth)
gravity of earth.
10. (a) According to Gravitational Law
R

Now for planet gP=2ge, RP=Re/4


GM1M 2 ve
U

F= 2 …… (i) So vP = 2g P R P = 2 ´ 2ge ´ R e / 4 =
r 2
JO

Where M1 is mass of planet & m2 is the mass of any 18. (c) The potential energy for a conservative force is defined
body. Now according to Newton’s second law, body of as
mass m2 feels gravitational acceleration g which is - dU
r r r
U

F = m2g ……(ii) F= or U = - ò F.dr …… (i)


dr
¥
ED

GM1
So from (i) & (ii), we get g = r
r2 GM1M 2 -GM1M 2
So the ratio of gravitational acceleration due to two
or U r = ò r 2
dr =
r
…… (ii)
¥
planets is (Q U¥ = 0)
If we bring the mass from the infinity to the centre of
g1 M1 r22 (4 / 3)πr13 ´ρ r22
= 2 ´ = ´ earth, then we obtain work, ‘so it has negative
g2 r1 M2 r12 (4 / 3) πr23 ´ ρ (gravitational force do work on the object) sign &
potential energy decreases. But if we bring the mass
g1 æ r1 ö from the surface of earth to infinite, then we must do
= ç ÷ (both planet have same material, so work against gravitational force & potential energy of
g 2 è r2 ø
the mass increases.
density is same)
GM1M 2
Now in equation (i) if F = instead of
l r5 / 2
11. (c) Since T(time period) = 2π
g GM1M 2
F= then
for second pendulum T=2sec r2
Now on the planet the value of acceleration due to gravity is r
g/4. GM1M 2 -2 GM1M 2
So for the planet, the length of sec. pendulam l' is
Ur = ò r 5/2
dr =
3 r 3/2
¥
2 sec l g/4 1
= ´ Þ l' = l / 4 Þ Ur µ
2 sec g l' +3/ 2
r
Gravitation 233

3/ 2 3/ 2 1 10 2
T1 æ R1 ö T æ 4R ö =
19. (c) =ç ÷ Þ =ç ÷ r 2 (1 - r) 2
T2 çè R 2 ÷ø T2 è 16R ø
1 10
= Þ 10 r = 1 - r
Þ T2 = 8T r 1- r
1
20. (a) 3 2
T 2 µ (major axis) Þ T a a
3 \ r = km
11
21. (d) According to 3rd law of Kepler 4. (b) The gravitational potential V at a point distant ‘r’ from
a body of mass m is equal to the amount of work done
T2 µ R3 in moving a unit mass from infinity to that point.
Þ T 2 = KR 3 r ur r
Vr - Vµ = - ò E.dr = -GM (1/ r - 1/ µ )
where K is a constant
µ
2
T -GM æ r -dV ö
Thus 3 does not depends on radius. = çè As E = ÷
R r dr ø
(i) In the first case
1 1
22. (a) v1 µ , v2 µ 7R - GM

.IN
R 7R when Vµ = 0, Vr = = -5 unit
r
v2 1 v v
= Þ v2 = 1 = R (ii) In the second case Vµ = + 10 unit
v1 7 7 7
AL
Vr – 10 = – 5
23. (a) Escape velocity does not depend upon the mass of or Vr = + 5 unit
N
the body. 5. (c) Gravitational force supplies centripetal force
24. (b) At a height h above the surface of earth the
mv 2 3 ´ 10 29 ´ (2 ´ 108 ) 2
R

gravitational potential energy of the particle of mass \ F= = dynes


m is r 1.5 ´ 1014
U

GM e m = 8 × 1031 dyne = 8 × 1026 N (Q 1N = 105 dyne.)


Uh = -
JO

Re + h
Me
Where Me & Re are the mass & radius of earth 2G
2GM e 81 = 2 v
respectively. 6. (c) ve = ; vm =
U

Re e
Re 9
In this question, since h = Re
4
ED

GM e m - mgRe
So U h = Re = - = = 2/9 × 11.2 kms–1 = 2.5 kms–1
2 Re 2
7. (d) Since gravitational acceleration on earth is defined as
25. (d) Energy required to move a body of mass m from an
orbit of radius 2R to 3R is GM e
ge = …… (i)
GMm -GMm GMm GMm GMm R e2
DU = - - = - =
3R 2R 2R 3R 6R
Me R
mass of planet is MP = & radius R P = e
EXERCISE - 2 80 4
1. (c) GM P
2. (c) According to Newton’s Gravitation Law So g P = …… (ii)
R 2P
GM1M 2
Fg = here Fg = 10–10 Newton, From (i) & (ii), we get
r2
m1= m2= 1kg, MP R e2 g e
G = 6.6 × 10–11 g P = ge ´ = =2m/sec2
R 2P M e 5
2 GM1 M 2 6.6 ´ 10 - 11 ´ 1 ´ 1
So r = = = 0.66 (as g=10m/sec2)
Fg 10 - 10
or r = 0.8125 metre = 81.25 cm » 80cm – 6. 67 ´ 10 - 11 ´ 10 6 .67 ´ 10 - 11 ´ 100
8. (c) Vg = -
G ´ 10 3 G ´ 10 5 1 1
3. (c) = –10
= –6.67 × 10 – 6.67 × 10 –9
(r ) 2 (1 - r ) 2 = – 6.67 × 10–10 × 11 = –7.3 × 10–9 J/kg
EBD_7179
234 PHYSICS

9. (c) T2 µ R3 1/ 2
2 π é (R + h)3 ù
2 3 16. (c) T= ê ú
æ T1 ö æ R1 ö R êë g úû
Þ çç ÷÷ = çç ÷÷
è T2 ø è R2 ø Here (R + h) changes from R to 4 R. Hence period of
3/ 2 revolution changes from T to (43)1/2 T = 8 T.
3/ 2
æ T1 ö æ R1 ö æ 1ö
Þ çç ÷÷ = çç ÷÷ =ç ÷ æ 2h ö
g h = g ç1 -
è T2 ø è R2 ø è 4ø 17. (b) ÷
è R ø
T2 æ xö
Þ = ( 4) 3 / 2 = 8 g x = g ç1 - ÷
T1
è Rø
Þ T2 = 8 ´ T1 = 8 ´ 5 = 40 hours Given that, g h = g x
10. (b) At poles, the effect of rotation is zero and also the \x=2h
distance from the centre of earth is least.
mv 2 GmM GM
18. (d) = also g= 2
11. (c) T 2 µ R 3 (According to Kepler’s law) (R + x ) (R + x ) 2
R

T12 µ (1013 )3 and T2 2 µ (1012 )3 mv 2 æ GM ö R 2


\ = mç

.IN
(R + x) è R 2 ÷ø (R + x) 2
T12 T1
\ = (10) 3 or = 10 10 .
2 T2 mv 2 R2
T2 \ = mg
AL (R + x ) (R + x ) 2
12. (c) According to Kepler’s law of period T2 µ R3
1/ 2
æ gR 2 ö
N
T12 R13 (6 R )3 gR 2
\v = Þ v = çç ÷
2
= = =8 ÷
T22 R23 (3 R )3 R+x R + x
R

è ø
24 ´ 24 mv 2
U

=8 =
GmM
T22 19. (c)
R + x (R + x ) 2
JO

24 ´ 24 x = height of satellite from earth surface


T22 = = 72 = 36 × 2
8 m = mass of satellite
U

T2 = 6 2 GM GM
Þ v2 = or v =
(R + x) R+x
ED

13. (b) Since escape velocity v e = ( 2g R e ) is independent


2p(R + x) 2p(R + x)
of angle of projection, so it will not change. T= =
14. (c) Given that l2 = 1.02 l1 v GM
R+x
ælö which is independent of mass of satellite
We know that T = 2π ç ÷ or T µ l
ègø 20. (c) F = KR - n = MRw 2 Þ w 2 = KR - (n +1)
-( n +1)
T2 æl ö (1.02 l1 )
\ = ç 2÷ = = 101 or w = K' R 2 [where K' = K1/2, a constant]
T1 è l1 ø l1
-(n +1)
Thus time period increases by 1%. 2p
aR 2
15. (b) We know that v e = 2 v0 , where v0 is orbital T
velocity. (n +1)
\T a R 2
1
K.E. in the orbit, E = M v 02 21. (d) According to Newton gravitational force
2
GM e m GM e m
1 1 = mg & = mg¢
K.E. to escape E = M v e = M (2 v 0 )
2 2
R e2 (R e + h )2
2 2
1 gR e2
=
1
M v 02 ´ 2 = 2 E Þ g' = g =
2 (1 + h / R e ) 2
(R e + h) 2
Gravitation 235

where Me is mass of earth, G is gravitational constant,


R GMm é1 1 ù
Re is radius & earth, h is height of satellite above the 35. (b) P.E. = ò dr = -GMm ê - ú
R0 2 R R
surface of earth, g is value at the surface of earth, g' is r ë 0û
value at height h above the surface of earth.
1
mv 2 mgR 2e gR 2e The K.E. acquired by the body at the surface = m v2
so ( R + h ) = = mg ¢ Þ v = 2
e (R e + h ) 2 Re + h
1 2 é1 1 ù
earth \ mv = -GMm ê - ú
h+Re 2 ë R R0 û
surface

Re æ 1 1ö
O v v = 2G M ç - ÷
R
è 0 R ø
Earth satellite
orbit 36. (c) Applying the properties of ellipse, we have
22. (c) 2 1 1 r1 + r2
23. (d) Both decreases but variation are different. = + =
R r1 r2 r1 r2
24. (b) There is no gravitational field in the shell.
25. (c) The gravitational field due to the ring at a distance 3r Instant position

.IN
is given by of satellite
Sun
Gm( 3r ) 3Gm
E= ÞE= R
[r 2 + ( 3r) 2 ]3 / 2 8r 2
AL major axis
3GmM r1 r2
Attractive force =
N
8r 2
26. (a) For uniform gravitational field
R

2 r1 r2
R=
V -2 1 r1 + r2
ms - 2
U

Eg = - =- =
r 20 10
æ GM ö
JO

1 37. (a) v= ç ÷ where r is radius of the orbit of the


Now, W = mgh = 5 ´ ´ 4 = 2J è r ø
10
satellite.
U

1 1
27. (b) E1 = mve2 , E 2 = mv20 Re 3
2 2 herer = Re + h = R e + = Re
ED

2 2
2
E1 æ v e ö
\ =ç ÷ =2 Q ve = 2v0 2GM 2
E 2 çè v 0 ÷ø So, v = = v0
3R e 3,
28. (d) g µ rR
where v0 is the orbital velocity of the satellite, which
29. (c) is moving in circular orbit of radius, r = Re
30. (b) As buoyant force involves ‘g’ in it.
31. (d) Work done = Uf – Ui 38. (d) v1 r1 = v 2 r2 or r12 ω1 = r2 2 ω 2
GMm æ Gmm ö 2 GmM 2 (Q L=mrv = constant)
=- -ç- ÷= = mgR
2R + R è R ø 3 R 3
or rmin 2 ω = rmax 2 ω¢
32. (b) 33. (a)
34. (d) The gravitational force due to the whole sphere at A \ ω¢ = (rmin 2 / rmax 2 ) ω
point is
39. (c) Distance between the surface of the spherical bodies
GM e m o , where m is the assumed rest mass at = 12R – R – 2R = 9R
F1 = 0
(2R ) 2 Force µ Mass
point A.
Acceleration µ Mass
In the second case, when we made a cavity of radius
(R/2), then gravitational force at point A is Distance µ Acceleration
GM e m o S 1
F2 =
2 \ F2/F1= 1/9 Þ a1 = M = 1 Þ 1 = Þ S 2 = 5S1
( R + R / 2) a 2 5M 5 S 2 5
EBD_7179
236 PHYSICS

S1 + S2 = 9
æ 2h ö 4 g æ 3R ö
46. (a) g = g ç1 - ÷ = çè since h = R + ÷ø
9 h è R ø h 2
Þ 6S1 = 9 Þ S1 = = 1 . 5
6
4
S2 = 1.5 × 5 = 7.5 Force on the satellite = mg h = mg
9
Note: Maximum distance will be travelled by smaller
bodies due to the greater acceleration caused by the 4
= ´ 200 ´ 10 » 889 N
same gravitational force 9
GM GM 1
40. (c) g= ; g' = 47. (c) gµ so we will not get a straight line.
2 2
R (0.99R ) R2
Also F = 0 at a point where Force due to Earth = Force
g' æ R 2 ö due to mars
\ =ç ÷ Þ g' > g 48. (c) Applying conservation of energy principle, we get
g è 0.99R2 ø
1 GMm GMm
41. (b) Acceleration due to gravity at lattitude’ l ’ is given by mk 2 v e2 - =-
2 R r
g l = g e - R e w 2 cos 2 l 1 2GM GMm GMm
Þ mk 2 - =-
At equator, l = 90° Þ cos l = cos 90° = 0 2 R R r

.IN
or g l = g e = g (as given in question) k2 1 1 1 1 k2
Þ - =- Þ = -
R R r r R R
2 2 3
At 30°, g 30 = g - Rw cos 30 = g - Rw 2
4
AL Þ
1 1
= (1 - k 2 ) Þ r =
R
r R 1- k 2
3
or, g - g 30 = Rw 2 49. (b) Loss in potential energy = Gain in kinetic energy
N
4
42. (b) The orbital speed of satellite is independent of mass GMm æ 3 GMm ö 1 2
- -ç- ÷ = mv
R

of satellite, so ball will behave as a satellite & will R è 2 R ø 2


continue to move with same speed in original orbit.
U

GMm 1 GM
43. (a) The total momentum will be zero and hence velocity Þ = mv 2 Þ v = = gR
2R 2 R
JO

will be zero just after collisiion. The pull of earth will


make it fall down. 50. (a) Change in force of gravity
44. (d) Weight of liquid displaced = mg. M
G m
U

45. (a) The force of attraction between sphere and shaded GMm
= - 32 (only due to mass M/3 due to shell
æm ö R2 R
ED

ç dx ÷ gravitational field is zero (inside the shell))


l
position dF = GM è ø
2 2GMm
x =
3R 2
r l 51. (a)
52. (a) Centripetal force = net gravitational force

m mv20 2GM 2 1 Gm 2
M = 2Fcos 45° + F1 = +
r ( 2r) 2 2 4r 2
45° 2r
x V0
F
r+l F1 F

r +l r +l r
GMm GMm 1
F= òr
lx 2
dx =
l òr
x2
dx

r +l r +l
GMm GMm é x -2+1 ù
=
l ò
r
x -2 dx =
l êë -2 + 1úû r
r+l
GMm -1 r + l GMm é 1 ù GMm 1/2
=- éx ù = - = mv20 Gm 2 æ GM(2 2 + 1ö
l ë ûr l êë x úû r r (r + l ) = 2
[2 2 + 1] Þ ç ÷
r 4r è 4r ø
Gravitation 237

53. (c) During total eclipse : 56. (a) V = wR


Total attraction due to sun and moon, g = g 0 - w 2 R [g = at equator, g0 = at poles]
GM s M e GM m M e g0 g g R
F1 = + = g0 - w 2 R ; w 2 R = 0 ; V 2 = 0
r12 r22 2 2 2
When moon goes on the opposite side of earth. Ve = 2g 0 R = 4V 2 = 2V
Effective force of attraction,
1 GMm GMm
GM s M e GM m M e 57. (b) mv e2 = ; ve = = 2gR
F2 = - 2 R R
r12 r22 In tunnel body will perform SHM at centre
Change in force, Vmax = Aw (see chapter on SHM)
2GM m M e R2p ve
DF = F1 - F2 = = = gR =
r22 2p R / g 2
Change in acceleration of earth
1
DF 2GM m 58. (b) gµ
Da = M = R2
e r22
R decreasing g increase hence, curve b represents
Fav GMs correct variation.

.IN
Average force on earth, Fav = M = 2
e r1 2GM 2GM
%age change in acceleration 59. (a) Here, v = and kv = .
R R+R
AL
Da 2GM m r12 1
= ´100 = ´ ´100 Solving k =
a av r22 GM s 2
N
2 60. (b) Potential energy of particle at the centre of square
æ r ö Mm
R

= 2ç 1 ÷ ´ 100 æ ö
è r2 ø M s
ç GMm ÷
U

= -4 ç
54. (a) Since, T 2 = kr 3 a ÷
ç ÷
JO

Differentiating the above equation è 2 ø


DT Dr DT 3 Dr
Þ2 =3 Þ = æ ö
T r
U

T 2 r GMm ÷ 1 8 2 GM
\ -4 ç + mv 2 = 0 Þ v 2 =
55. (b) Gravitational field at mass m due to full solid sphere ç a ÷ 2 a
r
ED

r rr rR è 2ø
é 1 ù
E1 = = ........ ê e0 =
3e 0 6e 0 ë 4 p G úû r1 + r2
61. (b) T2 a r3, where r = mean radius =
Gravitational field at mass m due to cavity (–r) 2
r r é GMm æ GMm ö
(-r) (R / 2)3 r ra 3 ù 62. (d) DU = U f - U i = - -ç- ÷
E2 = ........ ê using E = ú nR + h è R ø
3e 0 R 2 êë 3e 0 r 2 úû
n GMm n
= . = mgR
(-r)R 3 -rR n +1 R n +1
= - 2
=-
24e 0
24e 0 R
(R + h) 3
Net gravitational field 63. (c) T = 2p
m GM
r r r rR rR CM
E = E1 + E 2 = - R R3 (1.01R ) 3
6e0 24e 0 R/2 T1 = 2 p , T2 = 2p
GM GM
rR T2 - T1
= ´ 100 = 1.5%
8e 0 T1
r mrR 64. (b) Electronic charge does not depend on acceleration
Net force on m ® F = mE =
8e 0 due to gravity as it is a universal constant.
So, electronic charge on earth
M 1 3mg = electronic charge on moon
Here, r = & e0 = then F =
(4 / 3) pR 3 4 pG 8 \ Required ratio = 1.
EBD_7179
238 PHYSICS

GM 4 GM
65. (a) g= also M = d ´ pR 3 \ u = 2· ·R Þ 2GM
2 3
R R2 R
4 73. (d) Intensity will be zero inside the spherical shell.
\g = dpR at the surface of planet
3 1
I = 0 upto r = a and I µ when r > a
4 4
g p = ( 2d ) pR ¢ , g e = (d ) pR r2
3 3 74. (b) 75. (b)
ge = gp Þ dR = 2d R' EXERCISE - 3
Þ R' = R/2
66. (d) We know that Exemplar Questions
1. (d) Let the density of earth as a sphere is uniform, then it
æ4 ö
GM G ç pR 3 ÷r 4 can be treated as point mass placed at its centre then
g= 2 =
è3 ø = pGRr acceleration due to gravity g = 0, at the centre. But if
R R2 3
the density of earth is considered as a sphere of non-
g' R' 3R uniform then value of 'g' will be different at different
= =
g R R
=3 \ g' = 3g
points
67. (a) K.E. of satellite moving in an orbit around the earth is
æ 4 ö
çQ g = prGR ÷ . So g cannot be zero at any point.
2
1 1 æ GM ö÷ GMm
K= mv 2 = mç = è 3 ø
2 èç r ø÷

.IN
2 2r
2. (c) Force of attraction between any two objects obeys
P.E. of satellite and earth system is the inverse square law.
GMm As observed from the earth, the sun appears to move
GMm K 1
AL
U= Þ = 2r = in an approximate circular orbit. The gravitational force
r U GMm 2 of attraction between the earth and the sun always
r follows inverse square law.
N
68. (b) According to Kepler’s law, the areal velocity of a Due to relative motion between the earth and mercury,
planet around the sun always remains constant.
R

the orbit of mercury, as observed from the earth will


SCD : A1– t1 (areal velocity constant) not be approximately circular, since the major
U

SAB : A2 – t2 gravitational force on mercury is due to the sun is


A1 A 2
= , very large than due to earth and due to the relative
JO

t1 t2 motion to sun and earth with mercury.


A 3. (a) As we know that, the torque on earth due to
t1 = t2 . 1 , (given A1 = 2A2)
A2 gravitational attractive force on earth is zero.
U

2A 2 As the earth is revolving around the sun in a circular


= t2 .
ED

A2 motion due to gravitational attraction. The force of


\ t1 = 2t2 attraction will be of radial nature i.e., angle between
69. (b) Orbital velocity of a satellite in a circular orbit of radius position vector r and force F is also, zero.
a is given by
So, torque = t = r ´ F = rF sin 0° = 0
GM 1 v2 a1
v= Þ va Þ v = 4. (c) As the total (P.E.) of the earth satellite orbiting in orbit
a a 1 a2
æ -GM ö
4R is negative ç ÷ , where r is radius of the satellite
\ v2 = v1 = 2 v1 = 6V è 2r ø
R and M is mass of the earth.
70. (a) Potential at the given point = Potential at the point Due to the viscous force acting on satellite, energy
due to the shell + Potential due to the particle decreases continuously and radius of the orbit or
GM 2GM 3GM height decreases gradually.
=- - =-
a a a 5. (b) The major force acting on moon is due to gravitational
71. (b) Angular momentum is conserved force of attraction by sun and earth and moon is not
\ L1 = L2 always in line of joining sun and earth.
v1 r2
Þ mr1v1 = mr2v2 Þ r1v1 = r2v2 Þ = As observed from the sun, two types of forces are
v2 r1 acting on the moon one is due to gravitational
72. (a) The velocity u should be equal to the escape velocity. attraction between the sun and the moon and the other
That is, u = 2gR is due to gravitational attraction between the earth
and the moon. So these two force have different lines
GM
But g = of action and it will not be strictly elliptical because
R2 total force on the moon is not central.
Gravitation 239

6. (d) Asteroids are also being acted upon by central -Gm


gravitational forces, hence Asteroid will move in Gravitational potential V =
r
circular orbits like planets and obey Kepler's laws.
G´2 G´2 G´2 G´2
7. (d) Gravitational mass of proton is equivalent to its inertial V0 = – – – –
1 2 4 8
mass and is independent of presence of neighbouring
heavy objects so verifies the option (d). é 1 1 1 ù
– 2G ê1 + + + + ....¥ ú
ë 2 4 8 û
GMm
8. (c) Force of Gravitation, Fg =
r2 1 1
= – 2G × = – 2G × = – 4 G..
Let AB = r 1 1
1-
So, force on B due to A 2 2

G (2 Mm) 8
= FBA =
11. (c) Escape velocity, Ve = R pGP
towards BA. 3
( AB) 2
VP RP
G (2 Mm ) Þ Ve µ R Þ = =2
= = 2 Fg VE RE
r2
Þ VP = 2VE.
and force on B due to C
12. (b) As we know, the minimum speed with which a body is

.IN
GMm projected so that it does not return back is called
= FBC = towards BC
( BC ) 2 escape speed.
As, (BC) = 2AB
AL Ve =
2GM
=
2GM
=
2GM
GMm GMm GMm Fg r R+h 4R
Þ FBC = 2
= 2
= 2
= 1
N
(2 AB) 4( AB ) 4r 4 æ GM ö 2
=ç ÷ (Q h = 3R)
As FBA > FBC, è 2R ø
R

hence, m will move towards (BA) i.e., (2M). 13. (b) First when (r < R) E µ r and then when r > R E µ 1 .
U

r2
NEET/AIPMT (2013-2017) Questions
Hence graph (b) correctly dipicts.
JO

-GMm 14. (c) From question,


9. (a) Initial P. E., Ui = ,
R Escape velocity
U

-GMm 2GM
Final P.E., Uf = [Q R' = R + 2R = 3R] = = c = speed of light
ED

3R R
\ Change in potential energy, 2GM
Þ R=
-GMm GMm c2
DU = + 2 ´ 6.6 ´ 10 - 11 ´ 5.98 ´ 10 24
3R R = m
(3 ´ 108 )2
GMm æ 1 - 1 ö 2 GMm 2
= 10– 2 m
= ç 3÷ = = mgR
R è ø 3 R 3
15. (b) Before collision At the time of collision
5M 5M
æ GMm ö
çèQ = mgR ÷ M M
R ø 12R–3R=9R
R 2R R 2R
mgh
ALTERNATE : DU = 12R
h
1+
R Let the distance moved by spherical body of mass M
is x1 and by spherical body of mass 5m is x2
By placing the value of h = 2R we get As their C.M. will remain stationary
2 So, (M) (x1) = (5M) (x2) or, x1 = 5x2
DU = mgR. and for tauching x1 + x2 = 9R
3
So, x1 = 7.5 R
10. (c) GM
16. (a) As we know, orbital speed, Vorb =
m r
EBD_7179
240 PHYSICS

2 pr 2 pr
Time period T = = r Ve R re
vorb GM Þ = e
Squarring both sides, Vp 2R e 2r e
2
æ 2 pr r ö 4p 2 3 Ve
T2 = ç ÷ = .r \ Ratio =1 : 2 2
è GM ø GM Vp
T 2 4p 2 20. (a) As we know, gravitational potential (v) and
Þ = =K
r 3 GM acceleration due to gravity (g) with height
Þ GMK = 4p2. -GM
17. (d) Given: Height of the satellite from the earth's surface V= = –5.4 × 107 …(1)
R+h
h = 0.25 × 106m
Radius of the earth R = 6.38 × 106m GM
and g = =6 …(2)
Acceleration due to gravity g = 9.8 m/s2 ( R + h )2
Orbital velocity, V0 = ? Dividing (1) by (2)

GM GM R 2 -GM
V0 = = . R +h -5.4 ´107
(R + h) R 2 (R + h) =
GM 6

.IN
( R + h )2
9.8 ´ 6.38 ´ 6.38
=
6.63 ´106 5.4 ´ 107
AL Þ =6
(R + h )
é GM ù
= 7.76 km/s êQ 2 = g ú Þ R + h = 9000 km so, h = 2600 km
ë R û
N
21. (a) Both the astronauts are in the condition of
18. (c) The gravitational force on the satellite will be aiming weightlessness. Gravitational force between them
R

towards the centre of the earth so acceleration of the pulls towards each other. Hence Astronauts move
satellite will also be aiming towards the centre of the
U

towards each other under mutual gravitional force.


earth. 22. (c) Above earth surface Below earth surface
JO

19. (b) As we know, escape velocity,


æ 2h ö æ d ö
2GM 2G æ 4 3 ö gh = g ç1 - R ÷ gd = g ç1 - ÷
Ve = = · ç pR r ÷ µ R r è eø è R eø
R è3 ø
U

R
According to question, gh = gd
ED

Ve R e re æ 2h ö æ d ö
\ =
Vp R p rp g ç1 - ÷ = g ç1 - ÷
è Re ø è Re ø
Clearly,
d = 2h = 2 km
Mechanical
9 Properties of Solids
INTERATOMIC AND INTERMOLECULAR FORCE The force of attraction between molecules may be written as
The force between atoms of an element is called interatomic a
force. The force between molecules of a compound (or element) Fa = -
is called intermolecular force. These forces are electrical in r7
The negative sign shows that Fa is a force of attraction and ‘a’ is

.IN
nature. Depending on the distance between the atoms, this force
may be attractive or repulsive in nature. These forces are a constant which depends on the kind of attractive force between
responsible for the definite size or shape of a solid. the molecules and the structure of the molecules. The force of
attraction results from the creation of induced dipole moment in
Detailed calculations as well as deductions from experiment show
AL
one molecule by the neighbouring molecule.
that the interaction between any isolated pairs of atoms or
molecules may be represented by a curve that shows how the When the molecules are brought closer there is a force of repulsion
N
potential energy varies with separation between them as shown between them. It can be shown that the repulsive force is
in the figure.
R

b
Fr = + ,
r9
U

where b is a constant like a. The force of repulsion varies very


JO

rapidly. Fr is inversely proportional to the ninth power of distance


between the molecules. The resultant force acting on the
molecules is
U

a b
F=- 7
+
r9
ED

This curve describes the interatomic potential. The force between r


the atoms can be found from the potential energy by using the The distance between the molecules decides the sign of the
relation resultant force.
dU ELASTICITY
F =- The property of the body by virtue of which it tends to regain its
dr
original shape and size after removing the deforming force is
The resulting interatomic force curve is shown in figure. called elasticity. If the body regains its original shape and size
Force is along the line joining the atoms or molecules, and is completely, after the removal of deforming forces, then the body
shown negative for attraction and positive for repulsion. is said to be perfectly elastic.
· The property of the body by virtue of which it tends to
retain its deformed state after removing the deforming
F(R) force is called plasticity. If the body does not have any
R tendency to recover its original shape and size, it is called
O
R0 perfectly plastic.

STRESS AND STRAIN


We see that as the distance decreases, the attractive force first Stress :
increases and then decreases to zero at a separation where the When a deforming force is applied to a body, an internal restoring
potential energy is minimum. force comes into play.
The restoring force per unit area is called stress.
For smaller distance force is repulsive, because at these distance
the negative charge distribution associated with one atom begins Restoring force
Stress =
to over lap with that associated with the neighboring atom. Area
EBD_7179
242 PHYSICS

Its S.I. Unit is Nm-2 greater and in which external force is exactly balanced by
Stress is a tensor as its value changes when direction changes. internal forces. The bar is said to be stressed in this
Types of Stress : condition.
Longitudinal stress or tensile stress, volumetric stress and
tangential stress are the types of stress.
Strain : Dl
It is defined as the ratio of the change in shape or size to the
Fixed Fl
original shape or size of the body.
l0
Change in dimension
Strain =
Original dimension
Strain has no units or dimensions. Now if the solid bar obey the Hooke’ law, the Young's
Types of Strain : modulus, Y is defined as
Longitudinal strain : It is defined as the ratio of the change in Tensile stress Fl / A
length to the original length. Y= =
Tensile strain Dl / l
Dl Where Dl is change in the length of bar, when we
Longitudinal strain =
l apply Fl.
Volume strain : It is the ratio of the change in volume to the (ii) Shear modulus (h) : Shear modulus or modulus of

.IN
original volume. rigidity h is
DV Shearing stress Ft / A
Volume strain = h= =
V Shearing strain Dx / h
AL
Shearing strain : It is the angular deformation produced in a
body. Dx
N
Dx Ft
Shearing strain (q) =
h
R

HOOKE’S LAW hq
U

It is the fundamental law of elasticity given by Robert Hooke in


1679. It states that “the stress is directly proportional to strain Fixed face
JO

provided the strain is small “. (a) As we see, there is no change in volume under
stress this deformation, but shape changes.
i.e. Stress µ strain Þ = constant = E
U

strain Dx
This proportionality constant is called modulus of elasticity (b ) = tan q » q (see the figure), where q is shear
ED

h
(name given by Thomas Young) or coefficient of elasticity (E). angle.
Since stress has same dimensions as that of pressure and strain (iii) Bulk modulus (B) : The Bulk modulus B is defined
is dimensionless. So the dimensions of E is same as, that of as
stress or pressure i.e. [ML–1T–2]
The modulus of elasticity depends on the material and on the Volume stress Fn / A -DP
B= = =
nature of deformation. There are three type of deformations and Volume stain DV / V DV / V
therefore three types of modulus of elasticity. V
(i) Young’s modulus (Y) : It measures the resistance of a solid
to elongation.
(ii) Shear modulus (h) or modulus of rigidity : It measures
the resistance to motion of the plane of a solid sliding part Fn
on each other.
(iii) Bulk modulus (B) : It measure the resistance that solid or
V– D V
liquid offer to their volume change.
The stress under which the system breaks is called breaking
stress. Negative sign comes to make B positive, because with the
(i) Young's modulus (Y) : Let us consider a long bar (shown increase of pressure, the volume of body decreases or vice
in fig.) of cross-sectional area A and length lo, which is versa.
clamped at one end. When we apply external force Fl The reciprocal of the Bulk modulus is called
longitudinally along the bar, internal forces in bar resist compressibility of material
distortion, but bar attains equilibrium in which its length is i.e., Compressibility = 1/B.
Mechanical Properties of Solids 243

Poisson's Ratio (s) : The stress-strain graph for increasing and decreasing load
Lateral strain/longitudinal strain = Poisson's ratio (s). encloses a loop, as shown in figure. The area of the loop gives
The theoretical value of s lies between –1 and 0.5 and practical the energy dissipated during its deformation.
value of s lies between 0 and 0.5.
Keep in Memory
STRESS-STRAIN CURVE
F
Plastic 1. Thermal stress = = YaD T, where a is the coefficient of
region A
Breaking D linear expansion and DT is the change in temperature.
strength C E (Fracture point) 2. (i) The modulus of rigidity (h) for liquids is zero.
B
Elastic limit (ii) For a given tensile force, the increase in length is
A
Proportional limit inversely proportional to square of its diameter.
(iii) The pressure required to stop volume expansion of a
Stress piece of metal is
OO¢ Strain P=BgdT,
where g = coefficient of volume expansion = 3a
(i) Proportional limit : The limit in which Hooke’s law is valid
(iv) To compare elasticities of different materials, their
i.e, stress is directly proportional to strain is called proportion
identical small balls are made and they are dropped
limit. Stress µ strain

.IN
from same height on a hard floor. The ball which rises
(ii) Elastic limit : It is a maximum stress upto which the body
maximum after striking the floor, is most elastic. The
completely recovers its original state after the removal of
order of elasticity of different materials on this basis
the deforming forces.
AL is as follows :
(iii) Yield point : The point beyond elastic limit, at which the Yivory > Ysteel > Yrubber > Yclay
length of wire starts increasing without increasing stress,
N
is defined as the yield point. (v) For the construction of rails, bridges, girders and
machines, materials with high Young's modulus are
R

(iv) Breaking point : The position when the strain becomes so


large that the wire breaks down at last, is called breaking used so that they may not get permanently deformed.
U

point. At this position the stress acting in that wire is called YA D L


For a spring, F = = kx.
breaking stress and strain is called breaking strain. L
JO

• Breaking stress is also known as the tensile strength. YA


• Metals with small plastic deformation are called brittle. Hence spring constant k = . Here DL = x.
L
U

• Metals with large plastic deformation are called ductile.


3. When equal force is applied on identical wires of different
Elastic fatigue : This is the phenomenon of a delay in recovering materials then the wire in which minimum elongation is
ED

the original configuration by a body, if it had been subjected to produced is more elastic. For the same load, more elongation
stress for a longer time the body looses the property of elasticity is produced in rubber than in steel wire, hence steel is more
temporarily. elastic than rubber.
Elastic relaxation time : It is the time delay in regaining the
original shape after removal of deforming forces. Elastic relaxation Relation between Y, B, h and s
time for gold, silver and phosphor bronze is negligible. (i) Y = 3B(1 – 2s) (ii) Y = 2h (1 + s)
Elastic Hysteresis
3B – 2h 9 3 1
When the stress applied on a body, is decreased to zero, the (iii) s= (iv) = +
6 B + 2h Y h B
strain will not be reduced to zero immediately. For some
substances (e.g.-vulcanized rubber), the strain lags behind the Energy stored per unit volume in a strained body
stress. This lagging of strain behind stress is called elastic 1
Energy per unit volume = stress × strain
hysteresis. 2
1
= modulus of elasticity × (strain)2
2
1
Stress

=(stress)2 /modulus of elasticity..


2
Work done in stretching a wire or work done per unit volume
1 1
= ´ stress ´ strain = ´ load ´ extension KEEP I
Strain 2 2
EBD_7179
244 PHYSICS

Torsional rigidity of a cylinder : Example 1.


(i) The torsional rigidity C of a cylinder is given by, A uniform rod of mass m, length L, area of cross section A
and Young’s modulus Y hangs from a ceiling. Its
p h R4
C= elongation under its own weight will be
2l
2 mg L mg L
where h = modulus of rigidity of material of cylinder,, (a) (b)
AY AY
R = radius of cylinder, l = length of cylinder
mg L
p h R4f
(ii) Restoring couple, t = Cf = (c) 2 AY
(d) zero
2l
(iii) Work done in twisting the cylinder through an angle Solution (c) A

f , W = p h R f joule = C f
4 2 1 2 m
Mass of section BC of wire = (L - x) ;
4l 2 L x
Cantilever : m
Tension at B, T = (L - x)g
A beam fixed at one end and loaded at the other end is called a L dx at B,
Elongation of element
cantilever. B
(i) The depression y at a distance x from the fixed end is (when T dx m(L - x )g
dl = = dx (L–x)
the weight of cantilever is ineffective) A Y LAY C
mg æç lx 2 x 3 ö÷ L

.IN
y= - mg mg L
Total elongation = ò dl =
LAY ò
(L - x )dx =
YI çè 2 6 ÷ø 2YA
0
where mg = load applied, l = length of cantilever, Example 2.
AL
I = geometrical moment of inertia of its cross section. If the potential energy of the molecule is given by
(ii) Maximum depression at free end of cantilever,
A B
mgl 3 U = 6 - 12 . Then at equilibrium position its potential
N
y max = d= r r
3YI
energy is equal to
R

bd 3
(a) For rectangular beam, I = (a) –A2/4 B (b) A2/4 B
12
U

(c) 2 A/B (d) A/2 B


pr4 Solution : (b)
(b) For a circular cross section beam of radius r, I =
JO

4 dU d éA B ù é - A x 6 12 B ù
(iv) Depression produced in a beam supported at two ends F=- =- ê - ú = ê 7 + 13 ú
dr ë r 6 12
dr r û ë r r û
mgl 3
U

and loaded at the middle, d = In equilibrium position F = 0.


48YI
ED

mgl 3 6A 12 B 2B
For rectangular beam, d = so, = or r 6 =
7 13
4Ybd 3 r r A
\ Potential energy at equilibrium position
mgl 3
and for circular beam, d =
12pr 4 Y A B A 2 A2 A2
U= - = - =
(2 B / A) (2 B / A) 2 2 B 4 B 4 B
Keep in Memory Example 3.
1. Wound spring possess elastic potential energy. The normal density of gold is r and its bulk modulus is K.
2. A material which can be drawn into wires is called ductile The increase in density of a lump of gold when a pressure
and a material which can be hammered into sheet is called P is applied uniformly on all sides is
malleable. (a) K/r P (b) P/r K
3. Ductility, brittleness, malleability, etc., are not elastic (c) r P/K (d) r K/P
properties. Solution : (a)
4. The substance which breaks just beyond the elastic limit p DV p
is called brittle. K= or = ;
DV / V V K
5. (i) Breaking force of a wire M M
Also r = and r =
'
= breaking stress × area of cross section ;
V V - DV
(ii) Breaking stress does not depend on the length of
-1
wire r' V 1 æ DV ö
(iii) Breaking stress depends on the material of wire. \ = = = ç1 - ÷
r (V - DV ) (1 - DV / V) è V ø
Mechanical Properties of Solids 245

æ DV ö p r¢ p Dr / r Dr Dl
» ç1 + ÷ = 1+ or -1 = Now s = - or = -s ;
è V ø K r K Dl / l r l
Dr
pr \ = -0.5 ´ (2 ´10 -3 ) = -1´ 10-3
or r¢ - r = (Q D V << V) r
K DV
Example 4. Further, = (2 ´10 -3 ) - 2 ´ (1 ´ 10 -3 ) = 0
V
A wire 3 m in length and 1 mm in diameter at 30ºC and
kept in a low temperature at –170ºC and is stretched \ % increase in volume is 0.
by hanging a weight of 10 kg at one end. Calculate the Example 6.
change in the length of the wire. Given a = 1.2 ×10–5/ºC (a) A heavy machine is to be installed in a factory. To
and Y = 2 × 1011 N/m2. Take g = 10 m/s2. absorb vibrations of the machine, a block of rubber is
Solution : placed between the machine and floor, which of the
We know that, two rubbers A and B would you prefer to use for the
purpose ? Why ?
FL 10 ´ 10 ´ 3
l= = = 1.91 × 10–3
Y A 2 ´10 ´ 0.785 ´10 -6
11

(where A = p r2) = 0.785 × 10–6 m2


Contraction in length = –a L D T A B
Stress Stress
= – (1.2 × 10–5) (3) (–170 – 30)

.IN
= 7.2× 10–3 m
O Strain O Strain
The resultant change in length AL
= 7.2 × 10–3 – 1.91 × 10–3 = 5.29 mm
(b) Which of the two rubber materials would you choose
Example 5.
for a car tyre ?
A material has Poisson’s ratio 0.5. If a uniform rod of it
N
Solution :
suffers a longitudinal strain of 2 × 10–3 , what is the
(a) Rubber B is preferred. The area of the hysteresis loop
percentage increase in volume?
R

Solution : measures the amount of heat energy dissipated by the


material. The area of loop B is more than A. So B can
U

Here
absorb more vibrations.
DV D (p r 2 l) r 2 Dl + 2 r l Dr DV Dl Dr
JO

= = or = +2 (b) To avoid excessive heating of car tyre, rubber A would


V 2
pr l 2
r l V l r be preferred over rubber B.
U
ED
246

ED
U
JO
U
R
N
AL
.IN
PHYSICS

EBD_7179
Mechanical Properties of Solids 247

1. Elastomers are the materials which 9. The value of tan (90 – q) in the graph gives
(a) are not elastic at all
(b) have very small elastic range
(c) do not obey Hooke’s law
(d) None of these

Strain
2. The load versus elongation graph for four wires is shown.
The thinnest wire is q
Stress
Load P
Q (a) Young's modulus of elasticity
R (b) compressibility
(c) shear strain
S (d) tensile strength

.IN
10. The length of a metal is l1 when the tension in it is T1 and
Elongation is l2 when the tension is T2. The original length of the wire
AL is
(a) P (b) Q l1 + l 2 l 1T2 + l 2 T1
(c) R (d) S (a) (b)
2 T1 + T2
3. Which of the following affects the elasticity of a substance?
N
l 1T2 - l 2 T1
(a) hammering and annealing (c) T2 - T1
(d) T1T2 l 1l 2
R

(b) change in temperature


11. Uniform rod of mass m, length l , area of cross-section A
U

(c) impurity in substance


has Young’s modulus Y. If it is hanged vertically, elongation
(d) All of these under its own weight will be
JO

4. Which of the following has no dimensions ?


mgl 2mgl
(a) strain (b) angular velocity (a) (b)
2AY AY
U

(c) momentum (d) angular momentum


5. Minimum and maximum values of Possion’s ratio for a metal mgl mgY
ED

(c) (d)
lies between AY Al
(a) – ¥ to + ¥ (b) 0 to 1 12. Which one of the following affects the elasticity of a
(c) – ¥ to 1 (d) 0 to 0.5 substance ?
6. In solids interatomic forces are (a) Change in temperature
(a) totally repulsive (b) totally attractive (b) Hammering and annealing
(c) Impurity in substance
(c) both (a) and (b) (d) None of these
(d) All of the above
7. Which one of the following is not a unit of Young’s
13. According to Hooke’s law of elasticity, if stress is
modulus ?
increased, then the ratio of stress to strain
(a) Nm–1 (b) Nm–2 (a) becomes zero (b) remains constant
(d) dyne cm–2 (d) mega pascal (c) decreases (d) increases
8. Two rods A and B of the same material and length have 14. The length of an iron wire is L and area of corss-section is
their radii r1 and r2 respectively. When they are rigidly fixed A. The increase in length is l on applying the force F on its
at one end and twisted by the same couple applied at the two ends. Which of the statement is correct?
other end, the ratio (a) Increase in length is inversely proportional to its length
æ Angle of twist at the end of A ö (b) Increase in length is proportional to area of cross-
çè Angle of twist at the end of B ÷ø is section
(c) Increase in length is inversely proportional to area of
(a) r12 / r22 (b) r13 / r23
cross-section
(c) r24 / r14 (d) r14 / r24 (d) Increase in length is proportional to Young's modulus
EBD_7179
248 PHYSICS

15. A and B are two wires. The radius of A is twice that of B. 21. The diagram shown below represents the applied forces
They are stretched by the same load. Then the stress on B per unit area with the corresponding change X (per unit
is length) produced in a thin wire of uniform cross section in
(a) equal to that on A (b) four times that on A the curve shown. The region in which the wire behaves like a
(c) two times that on A (d) half that on A liquid is F
16. Hooke's law defines (a) ab d
(a) stress b
(b) bc a
(b) strain c
(c) modulus of elasticity (c) cd
(d) elastic limit
17. In case of steel wire (or a metal wire), the limit is reached (d) Oa O X
when 22. A steel wire is suspended vertically from a rigid support.
(a) the wire just break When loaded with a weight in air, it extends by la and when
(b) the load is more than the weight of wire the weight is immersed completely in water, the extension is
(c) elongation is inversely proportional to the tension reduced to lw. Then the relative density of material of the
(d) None of these weight is
18. A steel ring of radius r and cross sectional area A is fitted la
onto a wooden disc of radius R (R > r). If the Young’s (a) la / lw (b) la -lw
modulus of steel is Y, then the force with which the steel

.IN
ring is expanded is (c) l w /(l a - l w ) (d) l w / l a
(a) A Y (R/r) (b) A Y (R – r)/r 23. When an elastic material with Young’s modulus Y is
(c) (Y/A)[(R – r)/r] (d) Y r/A R AL subjected to stretching stress S, elastic energy stored per
19. Which of the following relation is true ? unit volume of the material is
9hY (a) YS / 2 (b) S2Y / 2
(a) 3Y = K (1 - s) (b) K=
Y+h 2
N
(c) S / 2Y (d) S / 2Y
05.Y - h 24. The ratio of shearing stress to the corresponding
R

(c) s = (6K + h)Y (d) s= shearing strain is called


h
(a) bulk modulus (b) Young's modulus
U

20. For a constant hydraulic stress on an object, the fractional


(c) modulus of rigidity (d) None of these
æ DV ö
JO

change in the object volume ç ÷ and its bulk modulus 25. The Young’s modulus of a perfectly rigid body is
è V ø (a) unity
(B) are related as
(b) zero
DV DV
U

1 (c) infinity
(a) µB (b) µ
V V B (d) some finite non-zero constant
ED

DV DV
(c) µ B2 (d) µ B -2
V V

1. Two wires of same material and length but cross-sections 3. The Young’s modulus of brass and steel are respectively
in the ratio 1 : 2 are used to suspend the same loads. The 1010 N/m2. and 2 × 1010 N/m2. A brass wire and a steel wire
extensions in them will be in the ratio of the same length are extended by 1 mm under the same
force, the radii of brass and steel wires are RB and RS
(a) 1 : 2 (b) 2 : 1 respectively. Then
(c) 4 : 1 (d) 1 : 4 (a) RS = 2 R B (b) RS = R B / 2

2. A body of mass 10 kg is attached to a wire of radius 3 cm. It’s (c) R S = 4R B (d) R S = R B / 4


breaking stress is 4.8 × 107 Nm–2, the area of cross-section of 4. A steel wire of length 20 cm and uniform cross-section
1 mm2 is tied rigidly at both the ends. The temperature of
the wire is 10–6 m2. What is the maximum angular velocity with
the wire is altered from 40ºC to 20ºC. Coefficient of linear
which it can be rotated in the horizontal circle ? expansion for steel a = 1.1 × 10–5/ºC and Y for steel is
(a) 1 rad sec–1 (b) 2 rad sec–1 2.0 × 1011 N/m2. The change in tension of the wire is
(a) 2.2 × 106 newton (b) 16 newton
(c) 4 rad sec–1 (d) 8 rad sec–1 (c) 8 newton (d) 44 newton
Mechanical Properties of Solids 249

5. A cube is subjected to a uniform volume compression. If 15. A massive stone pillar 20 m high and of uniform cross-
the side of the cube decreases by 2% the bulk strain is section rests on a rigid base and supports a vertical load of
(a) 0.02 (b) 0.03 5.0 × 105 N at its upper end. If the compressive stress in the
(c) 0.04 (d) 0.06 pillar is not to exceed 1.6 × 106 N m–2, what is the minimum
6. A wire suspended vertically from one of its ends is cross-sectional area of the pillar? Density of the stone
stretched by attaching a weight of 200N to the lower end. = 2.5 × 103 kg m–3. (Take g = 10 N kg–1)
The weight stretches the wire by 1 mm. Then the elastic (a) 0.15 m2 (b) 0.25 m2
(c) 0.35 m 2 (d) 0.45 m2
energy stored in the wire is
(a) 0.2 J (b) 10 J 16. A circular tube of mean radius 8 cm and thickness 0.04 cm is
(c) 20 J (d) 0.1 J melted up and recast into a solid rod of the same length.
The ratio of the torsional rigidities of the circular tube and
7. A metal rod of Young's modulus 2 × 1010 N m–2 undergoes
the solid rod is
an elastic strain of 0.06%. The energy per unit volume stored
in J m–3 is (8.02) 4 - ( 7.98) 4 (8.02) 2 - (7.98) 2
(a) 3600 (b) 7200 (a) (b)
(0.8) 4 (0.8) 2
(c) 10800 (d) 14400
8. A force of 103 newton, stretches the length of a hanging (0.8) 2 (0.8) 2
wire by 1 millimetre. The force required to stretch a wire of (c) (d)
(8.02) 4 - ( 7.98) 4 (8.02) 3 - (7.98) 2
same material and length but having four times the diameter 17. From a steel wire of density r is suspended a brass block of

.IN
by 1 millimetre is density rb. The extension of steel wire comes to e. If the
(a) 4 × 103 N (b) 16 × 103 N
1 1 brass block is now fully immersed in a liquid of density rl,
´ 103 N ´ 103 N
(c)
4
(d)
16
AL e
the extension becomes e'. The ratio will be
9. A 2 m long rod of radius 1 cm which is fixed from one end is e'
given a twist of 0.8 radian. The shear strain developed will rb r b - rl
N
(a) (b)
be r b - rl rb
R

(a) 0.002 (b) 0.004 rb - r rl


(c) 0.008 (d) 0.016 (c) (d)
rl - r r b - rl
U

10. There are two wire of same material and same length while
the diameter of second wire is two times the diameter of 18. One end of a uniform wire of length L and of weight W is
JO

first wire, then the ratio of extension produced in the wires attached rigidly to a point in the roof and W1 weight is
by applying same load will be suspended from looser end. If A is area of cross-section of
(a) 1 : 1 (b) 2 : 1 L
U

the wire, the stress in the wire at a height from the upper
(c) 1 : 2 (d) 4 : 1 4
end is
ED

11. For a given material, the Young's modulus is 2. 4 times that


of rigidity modulus. Its Poisson's ratio is W1 + W W1 + 3W / 4
(a) 2.4 (b) 1.2 (a) (b)
a a
(c) 0.4 (d) 0.2
12. A cube at temperature 0ºC is compressed equally from all W1 + W / 4 4 W1 + 3W
(c) (d)
sides by an external pressure P. By what amount should its a a
temperature be raised to bring it back to the size it had 19. A beam of metal supported at the two edges is loaded at the
before the external pressure was applied. The bulk modulus centre. The depression at the centre is proportional to
of the material of the cube is B and the coefficient of linear (a) Y 2 (b) Y
expansion is a. (c) 1/Y (d) 1/Y 2
(a) P/B a (b) P/3 B a 20. An iron rod of length 2m and cross-sectional area of 50
(c) 3 p a/B (d) 3 B/P mm2 stretched by 0.5 mm, when a mass of 250 kg is hung
13. The compressibility of water is 4 × 10–5 per unit atmospheric from its lower end. Young’s modulus of iron rod is
pressure. The decrease in volume of 100 cm3 of water under 18 2
(a) 19.6 ´ 10 20 N / m 2 (b) 19.6 ´10 N / m
a pressure of 100 atmosphere will be
(a) 0.4 cm3 (b) 4 × 10–5 cm3 (c) 19.6 ´ 1010 N / m 2 (d) 19.6 ´1015 N / m 2
(c) 0.025 cm 3 (d) 0.004 cm3 21. A wire fixed at the upper end stretches by length l by
14. For the same cross-sectional area and for a given load, the applying a force F. The work done in stretching is
ratio of depressions for the beam of a square cross-section (a) 2Fl (b) Fl
and circular cross-section is
(a) 3 : p (b) p : 3 F Fl
(c) (d)
(c) 1 : p (d) p : 1 2l 2
EBD_7179
250 PHYSICS

22. A metalic rod of length l and cross-sectional area A is made 31. Two wires A and B are of the same material. Their lengths
of a material of Young modulus Y. If the rod is elongated by are in the ratio of 1 : 2 and the diameter are in the ratio 2 : 1.
an amount y, then the work done is proportional to If they are pulled by the same force, then increase in length
1 will be in the ratio of
(a) y (b) (a) 2 : 1 (b) 1 : 4
y
(c) 1 : 8 (d) 8 : 1
1 32. Two wires are made of the same material and have the same
(c) y 2 (d) 2 volume. However wire 1 has cross-sectional area A and
y
wire 2 has cross-sectional area 3A. If the length of wire 1
23. On stretching a wire, the elastic energy stored per unit
increases by Dx on applying force F, how much force is
volume is
needed to stretch wire 2 by the same amount?
(a) Fl/2AL (b) FA/2L (a) 4 F (b) 6 F
(c) FL/2A (d) FL/2 (c) 9 F (d) F
24. When a pressure of 100 atmosphere is applied on a spherical 33. A rubber cord catapult has cross-sectional area 25 mm2
ball, then its volume reduces to 0.01%. The bulk modulus and initial length of rubber cord is 10 cm. It is stretched to 5
of the material of the rubber in dyne/cm2 is cm and then released to project a missile of mass 5 gm.
(a) 10 × 1012 (b) 100 × 1012
12 Taking Yrubber = 5 × 108 N/m2. Velocity of projected missile
(c) 1 × 10 (d) 10 × 1012
25. What per cent of length of wire increases by applying a is
(a) 20 ms–1 (b) 100 ms–1

.IN
stress of 1 kg weight/mm2 on it? –1
(Y = 1 × 1011 N/m2 and 1 kg weight = 9.8 newton) (c) 250 ms (d) 200 ms–1
34. The potential energy U between two atoms in a diatomic
(a) 0.0067% (b) 0.0098%
molecules as a function of the distance x between atoms
(c) 0.0088% (d) 0.0078%
AL
has been shown in the figure. The atoms are
26. K is the force constant of a spring. The work done in
increasing its extension from l1 to l2 will be
N
K
(a) K(l2 – l1) (b) (l2 + l1 )
R

2 O
K 2 2 U
U

(c) K (l22 - l12 ) (d) (l2 - l1 )


2
JO

27. If a rubber ball is taken at the depth of 200 m in a pool, its


volume decreases by 0.1%. If the density of the water is
1 × 103 kg/m3 and g = 10m/s2, then the volume elasticity in A B C x
U

N/m2 will be
(a) 108 (b) 2 × 108 (a) attracted when x lies between A and B and are repelled
ED

(c) 10 9 (d) 2 × 109 when x lies between B and C


28. The diagram below shows the change in the length X of a (b) attracted when x lies between B and C and are repelled
thin uniform wire caused by the application of stress F at when x lies between A and B
two different temperatures T1 and T2. The variation shown (c) are attracted when they reach B from C
suggests that (d) are repelled when they reach B from A
(a) T1 > T2 T2 35. The diagram shows a force - extension graph for a rubber
F
band. Consider the following statements :
(b) T1 < T2 T1
I. It will be easier to compress this rubber than expand it
(c) T2 > T1 II. Rubber does not return to its original length after it is
stretched
(d) T1 ³ T2 X
III. The rubber band will get heated if it is stretched and
29. A material has poisson’s ratio 0.50. If a uniform rod of it released
suffers a longitudinal strain of 2 × 10–3, then the percentage
change in volume is
(a) 0.6 (b) 0.4
(c) 0.2 (d) Zero Extension
30. A 5 metre long wire is fixed to the ceiling. A weight of 10 kg
is hung at the lower end and is 1 metre above the floor. The
wire was elongated by 1 mm. The energy stored in the wire Force
due to stretching is Which of these can be deduced from the graph?
(a) zero (b) 0.05 joule (a) III only (b) II and III
(c) 100 joule (d) 500 joule (c) I and III (d) I only
Mechanical Properties of Solids 251

36. A rod of length l and radius r is joined to a rod of length 43. When a 4 kg mass is hung vertically on a light spring that
l/2 and radius r/2 of same material. The free end of small rod obeys Hooke’s law, the spring stretches by 2 cms. The
is fixed to a rigid base and the free end of larger rod is given work required to be done by an external agent in stretching
a twist of q°, the twist angle at the joint will be this spring by 5 cms will be (g = 9.8 m/sec2)
(a) q/4 (b) q/2 (a) 4.900 joule (b) 2.450 joule
(c) 0.495 joule (d) 0.245 joule
(c) 5q/6 (d) 8q/48
44. The following four wires are made of the same material.
37. To break a wire, a force of 106 N/m2 is required. If the density Which of these will have the largest extension when the
of the material is 3 × 103 kg/m3, then the length of the wire same tension is applied ?
which will break by its own weight will be (a) Length = 100 cm, diameter = 1 mm
(a) 34 m (b) 30 m (b) Length = 200 cm, diameter = 2 mm
(c) 300 m (d) 3 m (c) Length = 300 cm, diameter = 3 mm
38. The upper end of a wire of diameter 12mm and length 1m is (d) Length = 50 cm, diameter = 0.5 mm
clamped and its other end is twisted through an angle of 45. A steel rod of radius R = 10 mm and length
30°. The angle of shear is L= 100 cm is stretched along its length by a force F = 6.28 ×
104 N. If the Young’s modulus of steel is Y = 2 ×1011 N/m2,
(a) 18° (b) 0.18°
the percentage elongation in the length of the rod is :
(c) 36° (d) 0.36° (a) 0.100 (b) 0.314
39. A steel wire of uniform cross-section of 1mm2 is heated

.IN
(c) 2.015 (d) 1.549
upto 50°C and clamped rigidly at its ends. If temperature of DIRECTIONS for Qs. 46 to 50 : These are Assertion-Reason
wire falls to 40°C, change in tension in the wire is (coefficient type questions. Each of these question contains two statements:
AL
of linear expansion of steel is 1.1 × 10–5/°C and Young's Statement-1 (Assertion) and Statement-2 (Reason). Answer
modulus of elasticity of steel is 2 × 1011 N/m2 ) these questions from the following four options.
(a) 22 N (b) 44 N (a) Statement-1 is True, Statement-2 is True; Statement-2 is a
N
(c) 88 N (d) 88 × 106 N correct explanation for Statement -1
(b) Statement-1 is True, Statement -2 is True; Statement-2 is
R

40. In Searle's experiment to find Young's modulus the diameter


NOT a correct explanation for Statement - 1
of wire is measured as d = 0.05cm, length of wire is
U

(c) Statement-1 is True, Statement- 2 is False


l = 125cm and when a weight, m = 20.0 kg is put, extension (d) Statement-1 is False, Statement -2 is True
JO

in wire was found to be 0.100 cm. Find maximum permissible


46. Statement-1 Identical springs of steel and copper are equally
mgl stretched. More work will be done on the steel spring.
error in Young's modulus (Y). Use : Y = .
( p / 4) d 2 x Statement-2 Steel is more elastic than copper.
U

(a) 6.3% (b) 5.3% 47. Statement-1 Stress is the internal force per unit area of a
ED

(c) 2.3% (d) 1% body.


41. If the ratio of lengths, radii and Young’s modulus of steel Statement-2 Rubber is less elastic than steel.
and brass wires shown in the figure are a, b, and c, 48. Statement 1 : The stress-strain graphs are shown in the
respectively. The ratio between the increase in lengths of figure for two materials A and B are shown in figure. Young's
brass and steel wires would be modulus of A is greater than that of B.
Stress

A
b2 a B
(a)
2c

bc Brass
(b)
2a 2 2kg
Strain
2 Statement 2 : The Young's modules for small strain is,
ba Steel
(c) stress
2c Y= = slope of linear portion, of graph; and slope
4kg
strain
a of A is more than slope that of B.
(d) 49. Statement 1: Young’s modulus for a perfectly plastic body
2b2c is zero.
42. A uniform cube is subjected to volume compression. If each Statement 2: For a perfectly plastic body, restoring force is
side is decreased by 1%, then bulk strain is zero.
(a) 0.01 (b) 0.06 50. Statement 1: Strain causes the stress in an elastic body.
(c) 0.02 (d) 0.03 Statement 2: An elastic rubber is more plastic in nature.
EBD_7179
252 PHYSICS

Exemplar Questions
x2 x
(a) (b)
1. Modulus of rigidity of ideal liquids is 2 L2 L
(a) infinity
x2 x2
(b) zero (c) (d)
L 2L
(c) unity
7. A rectangular frame is to be suspended symmetrically by
(d) some finite small non-zero constant value
two strings of equal length on two supports (figure). It can
2. The maximum load a wire can withstand without breaking,
be done in one of the following three ways;
when its length is reduced to half of its original length, will
(a) be double (b) be half
(c) be four times (d) remain same

.IN
3. The temperature of a wire is doubled. The Young's modulus
of elasticity AL (a) (b) (c)
(a) will also double (b) will become four times
The tension in the strings will be
(c) will remain same (d) will decrease
(a) the same in all cases (b) least in (a)
N
4. A spring is stretched by applying a load to its free end. The
(c) least in (b) (d) least in (c)
strain produced in the spring is
R

8. Consider two cylindrical rods of identical dimensions, one


(a) volumetric
U

of rubber and the other of steel. Both the rods are fixed
(b) shear
rigidly at one end to the roof. A mass M is attached to each
JO

(c) longitudinal and shear of the free ends at the centre of the rods.
(d) longitudinal (a) Both the rods will elongate but there shall be no
U

5. A rigid bar of mass M is supported symmetrically by three perceptible change in shape


wires each of length l. Those at each end are of copper and (b) The steel rod will elongate and change shape but the
ED

the middle one is of iron. The ratio of their diameters, if each rubber rod will only elongate
is to have the same tension, is equal to
(c) The steel rod will elongate without any perceptible
change in shape, but the rubber rod will elongate and
Yiron
(a) Ycopper / Yiron (b) the shape of the bottom edge will change to an ellipse
Ycopper
(d) The steel rod will elongate, without any perceptible
change in shape, but the rubber rod will elongate with
2
Yiron Yiron the shape of the bottom edge tapered to a tip at the
(c) 2 (d) Ycopper
Ycopper centre

6. A mild steel wire of length 2L and cross-sectional area A is NEET/AIPMT (2013-2017) Questions
stretched, well within elastic limit, horizontally between two 9. The following four wires are made of the same material.
pillars (figure ). A mass m is suspended from the mid-point Which of these will have the largest extension when the
of the wire. Strain in the wire is same tension is applied ? [2013]
(a) Length = 100 cm, diameter = 1 mm
2L
x (b) Length = 200 cm, diameter = 2 mm
(c) Length = 300 cm, diameter = 3 mm
m (d) Length = 50 cm, diameter = 0.5 mm
Mechanical Properties of Solids 253

10. If the ratio of diameters, lengths and Young’s modulus of 12. The approximate depth of an ocean is 2700 m. The
steel and copper wires shown in the figure are p, q and s compressibility of water is 45.4 × 10–11 Pa–1 and density of
respectively, then the corresponding ratio of increase in water is 103 kg/m3.What fractional compression of water
their lengths would be [NEET Kar. 2013] will be obtained at the bottom of the ocean ? [2015]
7q (a) 1.0 × 10–2 (b) 1.2 × 10–2
(a)
(5sp)
Steel (c) 1.4 × 10–2 (d) 0.8 × 10–2
5q 13. The Young's modulus of steel is twice that of brass. Two
(b) 2m
(7 sp 2 ) wires of same length and of same area of cross section, one
of steel and another of brass are suspended from the same
7q Copper
(c) roof. If we want the lower ends of the wires to be at the
(5sp 2 ) 5m same level, then the weights added to the steel and brass
wires must be in the ratio of : [2015 RS]
2q
(d) (5sp) (a) 2:1 (b) 4 : 1
11. Copper of fixed volume ‘V; is drawn into wire of length ‘l’. (c) 1:1 (d) 1 : 2

.IN
When this wire is subjected to a constant force ‘F’, the
14. The bulk modulus of a spherical object is 'B'. If it is subjected
extension produced in the wire is ‘Dl’. Which of the
to uniform pressure 'p', the fractional decrease in radius is
following graphs is a straight line? [2014] AL
1 B 3p
(a) Dl versus (b) Dl versus l2 (a)
3p
(b) [2017]
l B
N

1 p p
R

(c) Dl versus 2 (d) Dl versus l (c) (d)


l 3B B
U
JO
U
ED
EBD_7179
254 PHYSICS

Hints & Solutions


EXERCISE - 1 Dp 1 DV
20. (b) B= Þ µ [Dp = constant]
1. (c) 2. (b) 3. (d) 4. (a) 5. (d) DV / V B V
6. (c) 7. (a) 21. (b) The wire starts behaving like a liquid at point b. It
behaves like a viscous liquid in the region bc of the
p h r4 graph.
8. (c) Couple per unit angle of twist, C =
2l 22. (b) Let V be the volume of the load and r its relative
density
p h r 4q
\ Couple t = C q = FL VrgL
2l So, Y = = .....(1)
A la Ala
Here h, l, C & t are same. So, r 4q = constant
When the load is immersed in the liquid, then
q1 æç r24 ö÷ F¢ L (V r g - V ´ 1 ´ g ) L
\ = Y= = ....(2)
q 2 èç r14 ø÷ Alw Alw
stress (Q Now net weight = weight – upthrust)
9. (a) tan( 90 - q) = From eqs. (1) and (2), we get
strain
r (r - 1) la

.IN
10. (c) If l is the original length of wire, then change in length = or r =
la lw (l a - l w )
of first wire, Dl 1 = (l 1 - l)
23. (c) Energy stored per unit volume
change in length of second wire, Dl 2 = (l 2 - l)
AL 1
= ´ stress ´ strain
T l T l 2
Now, Y = 1 ´ = 2´ 1
A Dl1 A Dl 2 = ´ stress ´ (stress / Young ' s modulus )
N
2
T1 T T1 T2 S2
= 2 or = 1
R

or 2
Dl 1 Dl 2 l1 - l l 2 - l = ´ (stress) /(Young' s modulus) =
2 2Y
U

T2 l 1 - T1l 2 24. (c)


or T1 l 2 – T1 l = T2 l 1 – l T2 or l =
T2 - T1 25. (c) For a perfectly rigid body strain produced is zero for
JO

the given force applied, so


Fl Fl mgl
11. (c) Y= Þ Dl = = Y = stress/strain = ¥
ADl YA YA
U

12. (d) The elasticity of a material depends upon the EXERCISE - 2


temperature of the material. Hammering & annealing
ED

1. (b) Let W newton be the load suspended. Then


reduces elastic property of a substance.
13. (b) The ratio of stress to strain is always constant. If (W / A1 ) W L
Y= = ...(1)
stress is increased, strain will also increase so that (l 1 / L ) A1 l 1
their ratio remains constant. (W / A 2 ) WL
FL 1 and Y= = ....(2)
14. (c) l= Þl µ (l 2 / L ) A2 l2
YA A
Dividing equation (1) by equation (2), we get
Force 1
15. (b) Stress = \ Stress µ æl ö æ A2 ö æ l 2 ö æ2ö
Area πr 2 1 = çç 2 ÷÷ ç ÷ ç ÷
ç A ÷ =ç l ÷ ç ÷
2 è l1 ø è 1ø è 1ø è1ø
S B æ rA ö
= ç ÷ = (2)2 Þ S B = 4 S A l1 2
S A è rB ø \ = or l : l = 2 : 1
l2 1 1 2
16. (c) 2. (c) Given that F/A = 4.8 × 107 Nm–2
17. (d) According to Hooke's Law, within the elastic limits \ F = 4.8 ×107 ×A or
stress is directly proportional to strain.
18. (b) Let T be the tension in the ring, then mv2
= 4 .8 ´ 1 0 7 ´ 1 0 - 6 = 4 8
T.2 p r Tr r
Y= = Y A (R - r )
\ T= m r 2 w2 48
A. 2p (R - r ) A (R - r) r 2
or = 48 or w =
r mr
0.5Y - h
19. (d) Y = 2h(1 + s) Þ s =
h æ 48 ö
w= ç ÷ = 16 = 4 rad / sec
è 10 ´ 0.3 ø
Mechanical Properties of Solids 255

3. (b) We know that Y = F L/p r2 l or r2 = F L/(Y p l) Wl 3


14. (a) d= , where W = load, l = length of beam and I is
\ R 2B = F L /(YB p l) and R S2 = F L /(YS p l) 3 YI
geometrical moment of inertia for rectangular beam,
R 2B YS 2 ´ 1010
or = = =2 b d3
R S2 YB 1010 Ι= where b = breadth and d = depth
12
For square beam b = d
or R 2B = 2 R S2 or R B = 2 R S \
b4
\ Ι1 =
RS = R B / 2 12
4. (d) F = Y A a t = (2.0 × 1011) (10–6) (1.1 × 10–5) (20) æ πr4 ö
= 44 newton For a beam of circular cross-section, Ι2 = ç ÷
5. (d) è 4 ø
1 3 3
6. (d) Elastic energy =
2
´ F´ x \ d1 = W l ´12 = 4 W l (for sq. cross section)
F = 200 N, x = 1 mm = 10–3 m 3 Y b4 Y b4
W l3 4 W l3
1 and d 2 = =
\ E= ´ 200 ´1´10 -3 = 0.1 J 3 Y(p r / 4) 3Y (p r 4 )
4
2
(for circular cross-section)
1

.IN
7. (a) U / volume = Y ´ strain2 = 3600 J m–3 d1 3 p r 4 3pr4 3
2 Now = = =
[Strain = 0.06 × 10–2] AL d2 b 4 2 2
(p r ) p
l 2
8. (b) F = Y ´ A ´ Þ F µ r (Y, l and and L are constant) (Q b 2 = π r 2 i.e., they have same cross-sectional area)
L
If diameter is made four times then force required will Weight of load + Weight of puller
be 16 times, i.e., 16 × 103 N 15. (d) = Compressive stress
N
area
rq 1 0. 8 20A ´ d ´ 10 + 5 ´ 105
R

9. (b) f= = ´ = 0.004 radian Þ = 1.6 ´ 106


l 100 2 area
(d) Q Both wires are same materials so both will have
U

10. Where d is the density.


same Young’s modulus, and let it be Y.
20A ´ 2.5 ´ 103 ´ 10 + 5 ´ 105
JO

stress F = 1.6 ´ 106


Y= = , F = applied force A
strain A.(ΔL/L) ie, 5× 105 = 1.1× 106A or A = 0.45m2
U

A = area of cross-section of wire ph( r24 - r14 ) ph r 4


16. (a) C1 = , C2 =
FL FL 2l 2l
ED

Now, Y1 = Y2 Þ =
(A1 )(DL1 ) (A 2 )(DL 2 ) Initial volume = Final volume
Since load and length are same for both \ p[ r22 - r12 ]lr = pr 2 lr

æ DL ö æ r ö
2 Þ r 2 = r22 - r12 Þ r 2 = ( r2 + r1 )(r2 - r1 )
Þ r12 DL1 = r2 2 DL 2 , çç 1 ÷÷ = çç 2 ÷÷ = 4
è DL 2 ø è r1 ø Þ r 2 = (8.02 + 7.98)(8.02 - 7.98)
DL1 : DL2 = 4 :1 Þ r 2 = 16 ´ 0.04 = 0.64 cm Þ r = 0.8 cm
11. (d) Y = 2h(1 + s) C r 4 - r 4 [8.02] 4 - [ 7.98]4
\ 1 = 2 1 =
2.4h = 2h(1 + s) Þ 1.2 = 1 + s Þ s = 0.2
C2 r4 [0.8] 4
17. (a) Weights without and with liquid proportional to rb
-P -PV and rb – rl.
12. (b) Bulk modulus B = = ....(1)
(DV / V) DV 18. (b)

-V 1 d
and DV = g V DT = 3 a.V.T or = ...(2) 19. (c)
DV 3a. T.
For a beam, the depression at the centre is given by,
From eqs. (1) and (2), B = P /(3a.T) or T = P
3aB æ fL ö
d=ç ÷
1 DV / V è 4Ybd3 ø
13. (a) K= = . Here, P = 100 atm, [f, L, b, d are constants for a particular beam]
B P
K = 4 × 10 and V = 100 cm3.
–5
1
Hence, DV = 0.4 cm3 i.e. d µ
Y
EBD_7179
256 PHYSICS

250 ´ 9.8 28. (a) When same stress is applied at two different
-6 250 ´ 9.8 2
temperatures, the increase in length is more at higher
= 50 ´ 10
F/A = ´
20. (c) Y= ´ -6
´ 10 -3
temperature. Thus T1 > T2.
Dl / l 0.5 ´ 10 - 3 50 10 0. 5
29. (d)
2 1 1
30. (b) W = ´ F ´ l = mgl
Þ 19.6 ´ 10 N / m
10 2 2 2
21. (d) Work done by constant force in displacing the object 1
by a distance l. = ´ 10 ´ 10 ´ 1 ´ 10 -3 = 0.05 J
2
= change in potential energy 31. (c) We know that Young's modulus
1
= ´ stress × strain ×volume Y=
F
´
L
2 2 l
pr
1 F l Fl
= ´ ´ ´ A´ L = Since Y, F are same for both the wires, we have,
2 A L 2
22. (c) Volume V = cross sectional A × length l or V = Al 1 L1 1 L 2 l 1 r22 ´ L1 (D 2 / 2) 2 ´ L1
= or, = =
Elongation Y r12 l 1 r22 l 2 l 2 r12 ´ L 2 (D1 / 2) 2 ´ L 2
Strain = Original =
length l
l D2 ´ L D 22 L 1
Stress or, 1 = 22 1 = ´ 2 =
l 2 D1 ´ L 2 (2 D 2 ) 2

.IN
Young’s modulus Y = 2 L2 8
Strain
1 So, l 1 : l 2 = 1 : 8
Work done, W = ´ stress × strain × volume
2 l
32.
AL(c)
1 A
W = ´ Y ´ (strain) 2 ´ Al Y
2
N
Wire (1)
2
1 æ yö 1 æ YA ö 2
= ´Y ´ç ÷ø ´ Al = çè ÷ø y Þ W µ y
2
R

2 è l 2 l
3A Y
U

1 æ F öæ l ö Fl
23. (a) Energy stored per unit volume = ç ÷ç ÷ = –
2 è A øè L ø 2 AL l/3
JO

Wire (2)
100 As shown in the figure, the wires will have the same
24. (c) K= = 106 atm
0.01/100 Young’s modulus (same material) and the length of
U

= 1011 N/m2 = 1012 dyne/cm2 the wire of area of cross-section 3A will be l/3 (same
25. (b) Stress = 1 kg wt/mm2 = 9.8 N/mm2 volume as wire 1).
ED

= 9.8 × 106 N/m2. F/A


For wire 1, Y = ...(i)
Dl D x/l
Y = 1 ´ 10 N / m ,
11 2
´100 = ? F '/ 3 A
l For wire 2 , Y = ...(ii)
Dx /( l / 3)
Stress Stress
Y= = F l F' l
Strain Dl / l From (i) and (ii) , ´ = ´ Þ F ' = 9F
A Dx 3 A 3Dx
Dl Stress 9.8 ´ 106
\ = = 33. (c) Young’s modulus of rubber, Yrubber
l Y 1´ 1011 F l Dl
Dl = ´ Þ F = YA.
´100 = 9.8 ´ 10-11 ´ 100 ´ 106 A Dl l
l On putting the values from question,
= 9.8 × 10–3 = 0.0098 %
5 ´ 108 ´ 25 ´ 10 -6 ´ 5 ´ 10 -2
1 F=
26. (d) At extension l1, the stored energy = Kl12 10 ´ 10 - 2
2
1 2 = 25 ´ 25 ´ 10 2-1 = 6250 N
At extension l2, the stored energy = Kl2 kinetic energy = potential energy of rubber
2
Work done in increasing its extension from l1 to l2 1 1
1 mv 2 = F Dl
= K (l22 – l12 ) 2 2
2
FDl 6250 ´ 5 ´ 10-2
DP hrg 200 ´10 ´ 10 3 v= = = 62500
27. (d) K= = = = 2 ´109 m 5 ´10-3
DV / V DV / V 0.1/100 = 25 × 10 = 250 m/s
Mechanical Properties of Solids 257

34. (b) The atoms when brought from infinity are attracted l1 r Y1
due to interatomic electrostatic force of attraction. At 41. (d) Given, = a, 1 = b , =c
l2 r2 Y2
point B, the potential energy is minimum and force of
attraction is maximum. But if we bring atoms closer
than x = B, force of repulsion between two nuclei starts T T
and P.E. increases. Brass
Steel

P.E.

2g
x
C T 2g
Let Young’s modulus of steel be Y1, and that of brass
be Y2
A B
F1l1
35. (c) x \ Y1 = A Dl …(i)
1 1
F2l2
and Y2 = A Dl …(ii)
2 2
Dividing equation (i) by equation (ii), we get

.IN
Y1 F1 A2 l1Dl2
= …(iii)
Y2 F2 A1l2 Dl1
F Force on steel wire from free body diagram
Fcom Fext
AL
T = F1 = (2 g ) Newton
From the figure, it is clear that
Fcom < Fext. Force on brass wire from free body diagram
q¢ = 0
N
4 F2 = T1 = T + 2 g = 4 g Newton
phr q Now putting the value of F1, F2 in equation (iii), we
36. (d) r = C.q = = Constant
R

2L get
l/2
U

phr 4 (q - q0 ) ph(r / 2) 4 (q0 - q ') Y1 æ 2 g ö æ pr22 ö é l1 ù æ Dl2 ö 1 æ 1 ö æ Dl2 ö


Þ = = ç ÷ .ê ú . = .a
2l 2(l / 2) Y2 çè 4 g ÷ø è pr12 ø ë l2 û çè Dl1 ÷ø 2 çè b2 ÷ø çè Dl1 ÷ø
JO

q0
(q - q0 ) q0 8 dV dL
Þ = Þ q0 = q 42.
3
(d) If side of the cube is L then V = L Þ =3
2 16 9 l V L
U

\ % change in volume = 3 × (% change in length)


= 3 × 1% = 3%
ED

DV
\ Bulk strain = 0.03
q V
S 106 100
37. (a) L= = = = 34 m F 4 ´ 9.8
dg 3 ´ 103 ´ 10 3 43. (b) K = = = 19.6 ´ 102
rq 6mm ´ 30° x 2 ´ 10 -2
38. (b) rq = lf Þ f = = = 0.18°
l 1m 1
19.6 ´102 ´ (0.05)2 = 2.45 J
Work done =
Dl 2
39. (a) F = YA = YAaDq
l YA
= 2 × 1011 × (1 × 10–6) × 1.1 × 10–5 × (10) = 22 N 44. (d) F = ×l
L
mgl
40. (a) Y = ......... (1) L L
( p / 4) d 2 x So, extension, lµ µ 2 [Q F and Y are
Dm Dl Dd Dx A D
æ dY ö
çè ÷ = + +2 + constant]
Y ø max m l d x
100 200
m = 20.0 kg Þ Dm = 0.1 kg l1 µ µ 100 and l2 µ µ 50
2
l = 125cm Þ Dl = 1 cm. 1 22
d = 0.050 cm. Þ Dd = 0.001 cm 300 100 50
x = 0.100 cm. Þ Dx = 0.001 cm. l3µ µ and l4 µ µ 200
3 2 3 1
æ 0.1kg 1cm ö 4
+
æ dY ö ç 20.0kg 125cm ÷
çè ÷ø =ç ÷ ´ 100% = 6.3% L
Y max 0.001cm 0.001cm The ratio of is maximum for case (d).
ç +2 ´ + ÷ D2
è 0.05cm 0.100cm ø
Hence, option (d) is correct.
EBD_7179
258 PHYSICS

Fl L
45. (a) Percentage elongation in the wire = If F and are constants.
AY DL
6.28 ´ 104 ´ (1) 1
= = 1
2
p(0.01) ´ 2 ´ 10 11 1000 So, D µ
Y
1 1
46. (a) Work done = ´ Stress ´ Strain = ´ Y ´ (Strain)2 Hence, we can find ratio as
2 2
Since, elasticity of steel is more than copper, hence Dcopper Yiron
more work has to be done in order to stretch the steel. =
47. (b) Stress is defined as internal force (restoring force) per Diron Ycopper
unit area of a body. Also, rubber is less elastic than
6. (a) Consider the given diagram
steel, because restoring force is less for rubber than
steel.
48. (d) 49. (d) 50. (a) L L
A C B
EXERCISE - 3 90°–q 90°–q
Exemplar Questions x
1. (b) As liquid is ideal so no. frictional force exists hence,
q q
tangential forces are zero so there is no stress
developed.

.IN
O
2. (d) As we know that,
m
Breaking force
Breaking stress = ..(i)
Area of cross-section So, change in length
AL
When length of the wire changes (or by reducing half)
area of cross-section remains same. DL = ( A0 + B0 ) - ( AC + CB)
Hence, breaking force will be same because breaking = 2BO – 2AC
N
stress does not depend on length. = 2 [BO – AC] (\ AO = BO, AC = CB)
3. (d) As we know that, length of a wire when the temperature
R

increased = 2[( x2 + L2 )1/ 2 - L]


U

Lt = L0 (1 + aDT )
éæ x 2 ö1/ 2 ù
where DT is change in the temperature.
= 2 L êç1 + 2 ÷ - 1ú
JO

L0 is original length, êçè L ÷ø ú


a is coefficient of linear expansion and ë û
Lt is length at temperature T.
U

Now, DL = Lt - L0 = L0 aDT é 1 x2 ù x2
DL » 2L ê1 + 2
- 1ú = [Q x << L]
Stress ë 2L û L
ED

Now, Young's modulus (Y) =


Strain
FL0 FL0 1 DL x 2 / L x 2
= = µ Strain = = = 2
A ´ DL AL0 aDT DT 2L 2L 2L
1 7. (c) Let us consider the free body diagram of the
As, Y µ rectangular frame
DT
So, if temperature increass DT increases, hence
Young's modulus of elasticity (Y) decreases. Tsinq Tsinq
4. (c) When a spring is stretched by applying a load to its T
free end. Clearly the length and shape of the spring q q
changes. So strain produced when change in length Tcosq Tcosq
corresponds to longitudinal strain and change in shape
corresponds to shearing strain. m
5. (b) As we know that,
The Young's modulus
mg
Stress F / A F L
Y= = = ´
Strain DL / L A DL Net forces acting on frame will be zero.
F L 4 FL So, Balancing vertical forces
= ´ =
p( D / 2) 2
D L pD 2 DL 2T sinq – mg = 0 [T is tension in the string]
2T sinq = mg ...(i)
4 FL 4 FL
D2 = ÞD= Total horizontal force
pDLY pDLY
= T cos q - T cos q = 0
Mechanical Properties of Solids 259

mg 10. (c) From formula,


Now from Eq. (i), T =
2 sin q FL 4 FL
Increase in length DL = =
AY pD 2Y
mg
Tmax =
2sin qmin DLS FS æ DC ö YC LS
2
7 æ 1 ö æ1ö
2

= ç ÷ = ´ç ÷ ç ÷q
As mg is constant then DLC FC è DS ø YS LC 5 è pø èsø
1 7q
Tµ =
sin q (5sp 2 )
sin qmin = 0 Þ qmin = 0
F
No option matches with q = 0°
A Þ Dl = Fl
mg 11. (b) As Y = Dl
Tmin = AY
(since, sinqmax = 1)
2sin qmax l
V
But V = Al so A =
sin qmax = 1 Þ q = 90° l
So tension is all three cases are different rejects option Fl 2
(a) Therefore Dl = µ l2
VY

.IN
For minimum tension q must be 90° i.e. sin q = 1 Hence graph of Dl versus l2 will give a straight line.
Hence, tension is the least for the case (b). 12. (b) Compressibility of water,
8. (d) A mass M is attached at the centre or midpoint of rod AL K = 45.4 × 10–11 Pa–1
of rubber and steel. As the mass is attached to both density of water P = 103 kg/m3
the rods, both rod will be elongated as shown in depth of ocean, h = 2700 m
figures but due to different elastic properties of material DV
N
rubber changes shape also. We have to find =?
V
R

As we know, compressibility,
1 (DV / V)
U

K= = (P = rgh)
B P
M M
JO

So, (DV/V) = Krgh


= 45.4 × 10–11 × 103 × 10 × 2700 = 1.2258 × 10–2
W l
.
U

13. (a) Young's modulus Y =


(Rubber) (Steel) A Dl
W1 W2
ED

=
As the Young's modulus of rigidity for steel, is larger Y1 Y2 [Q A, l, Dl same for both brass and steel]
DL
than rubber, so strain for rubber is larger than
L
steel for same stress.
A, l, Dl l, A, Dl
NEET/AIPMT (2013-2017) Questions Y2 Y1
Brass Steel

YA L L
9. (d) F = × l! So, extension, lµ µ 2
L A D
[Q F and Y are constant] W1 Y1
= =2 [Ysteel/Ybrass = 2 given]
100 200 W2 Y2
l1 µ µ 100 and l2 µ µ 50
12 22 14. (c) Bulk modulus is given by
P DV P
300 100 50 B= or =
l3µ µ and l4 µ µ 200 æ DV ö V B
32 3 1 ç ÷
4 è V ø
DR P DR
L 3 = (here, = fractional decreases in radius)
The ratio of is maximum for case (d). R B R
D2
DR P
Hence, option (d) is correct. Þ =
R 3B
EBD_7179
260 PHYSICS

Mechanical
10 Properties of Fluids
FLUIDS Hydrostatic pressure : The hydrostatic pressure at a depth h
Fluid is something that can flow. All liquids and gases are fluids. below the surface of a fluid is given by
The force exerted normally at a unit area of the surface of a fluid P = hrg
is called fluid pressure. where r is the density of the fluid, g acceleration due to gravity

.IN
and h is the depth of the liquid column.
F Gauge pressure : The pressure at any point in a fluid is equal to
i.e., P =
a the sum of the atmospheric pressure acting on its surface and the
AL
Its S.I. unit is Nm–2 or Pascal. Its dimensions are [ML–1T–2]. hydrostatic pressure due to the weight of the fluid above that
point which is at a depth h below the surface of the fluid.
PASCAL'S LAW AND ITS APPLICATIONS
The gauge pressure is given by
N
Pascal's law : Pressure in a fluid in equilibrium is the same P = P0 + hrg
everywhere, if the effect of gravity is neglected.
R

or, P – P0 = hrg
Another form of Pascal's law : The excess pressure, applied BUOYANCY AND ARCHIMEDES' PRINCIPLE
U

anywhere in a mass of confined incompressible fluid is Buoyancy


JO

transmitted by the fluid in all directions without being If a body is partially or wholly immersed in a fluid, it experiences
diminished in magnitude. an upward force due to the fluid surrounding it. The phenomenon
Applications of Pascal's Law of force exerted by fluid on the body called buoyancy and the
U

Hydraulic lift : Its working is based on Pascal's law. A piston of force is called buoyant force or upthrust.
A body experiences buoyant force whether it floats or sinks, under
ED

small cross-sectional area (a) exerts a force (f) on the liquid.


its own weight or due to other forces applied on it.
Archimedes' Principle
Applied force = f When any body is immersed (totally or partially) in a liquid it
Area = A Area = a appears to lose part of its weight and the apparent loss of weight
is equal to the weight of liquid displaced.
Let a body of weight W is immersed in a fluid and W' is upthrust
F = pA on it then
f = pa (i) if W > W', then body will sink.
(ii) if W = W', then the body floats with whole or some part of
its volume inside the fluid.
The pressure is transmitted undiminished to the larger cylinder Let V be the volume of a body of density d and V' be the
of cross-sectional area A. volume of liquid of density r displaced. If the body floats
f F A then Vd = V¢r
P= = ÞF = f
a A a d V'
\ = gives the fraction of the volume inside the liquid
Hydraulic brakes also work on pascal's law. r V
in which the body floats.
ATMOSPHERIC, HYDROSTATIC AND GUAGE PRESSURE
Also, body immersed in a fluid experiences an upward buoyant
Atmospheric pressure : The atmosphere exerts pressure on the
force equivalent to the weight of the fluid displaced by it.
earth's surface. The atmospheric pressure at sea level is given by
The proof of this principle is very simple. Imagine a body of
P0 = 1.01 × 105 Pa arbitrary shape completely immersed in a liquid of density r as
Mechanical Properties of Fluids 261

shown in the figure (a). A body is being acted upon by the forces (ii) If two bodies have equal upthrust when just immersed
from all directions. Let us consider a vertical element of height h in a liquid, both will have the same volume.
and cross-sectional area dA (as shown in the figure (b)). (iii) If a person floats on his back on the surface of water,
the apparent weight of person is zero.
13. The hydrometer can be used to measure density of the
h1 F1
dA liquid or fluid.
h2
h
Relative Density (or Specific Gravity)
F2
Liquids may be treated as incompressible. Hence their density
may be assumed to be constant throughout.
(a) (b) Weight of substance in air
Relative density =
The force acting on the upper surface of the element is F1 Weight of equal volume of water
(downward) and that on the lower surface is F2 (upward). Since
F2 > F1, therefore, the net upward force acting on the element is Weight of substance in air
=
dF = F2 – F1 Loss of weight in water
It can be easily seen from the figure (b), that Density of substance
=
F1 = (rgh1 )dA and F2 = (rgh 2 )dA so dF = rg(h)dA Density of water at 4º C
Also, h2 – h1 = h and d (dA) = dV

.IN
Density in SI system = 1000 × density in the cgs system.
\ The net upward force is F = ò rgdV = rVg (i) The density of liquid of bulk modulus B at a depth h is
æ rgh ö
Hence, for the entire body, the buoyant force is the weight of the
AL given by rh = r0 ç 1 + where r0 is the density of
volume of the fluid displaced. è B ÷ø
The buoyant force acts through the centre of gravity of the liquid on its surface and r is the average density of liquid.
N
displaced fluid. (ii) The density of liquid changes with pressure as
æ DP ö
R

Keep in Memory Ph = P0 ç 1 + ÷
è B ø
U

1. The pressure is perpendicular to the surface of the fluid. where DP = change in pressure and B = bulk modulus of
2. The upthrust on a body immersed in a liquid does not liquid.
JO

depend on the mass, density or shape of the body. It only (iii) If two liquids of masses m1, m2 and densities r1, r2 are
depends on the volume of the body. mixed together, then the density of the mixture is given by
3. The weight of the plastic bag full of air is same as that of
m1 + m2
U

the empty bag because the upthrust is equal to the weight r=


of the air enclosed. m1 m2
+
ED

4. The cross-section of the water stream from a top decreases r1 r2


as it goes down in accordance with the equation of And if m1 = m2 but different densities are mixed together,
continuity. then the density of the mixture is harmonic mean of the
5. We cannot sip a drink with a straw on the moon, because densities.
there is no atmosphere on the moon.
6. The line joining the centre of gravity and centre of 2r1r 2 1 1é1 1 ù
i.e., r = or = ê + ú
buoyancy is called central line. r1 + r 2 r 2 ë r1 r 2 û
7. Metacenter - is a point where the vertical line passing (iv) If two drops of same volume but different densities are
through the centre of buoyancy intersects the central line. mixed together, then the density of the mixture is the
8. The floating body is in stable equilibrium when the arithmetic mean of the densities.
metacenter is above the centre of gravity (centre of gravity
is below the centre of buoyancy). r1 + r 2 r1V0 + r2V0
i.e., r = (as r = )
9. The floating body is in the unstable equilibrium when the 2 V0 + V0
metacenter lies below the centre of gravity (centre of gravity SURFACE TENSION
is above the centre of buoyancy).
It is defined as the force per unit length acting at right angles
10. The floating body is in the neutral equilibrium when centre
on either side of an imaginary line drawn on the free surface of
of gravity coincides with the metacenter (centre of gravity
the liquid.
coincides with the centre of buoyancy).
F
11. The wooden rod cannot float vertically in a pond of water i.e., S =
because centre of gravity lies above the metacenter. l
12. (i) If a body just floats in a liquid (density of the body is The surface tension is also defined as the work required to
equal to the density of liquid) then the body sinks if it increase unit area of that liquid film.
is pushed downwards. Its SI unit is N/m or J/m2 and dimensions are [ML0T–2].
EBD_7179
262 PHYSICS

Keep in Memory ANGLE OF CONTACT


1. The liquid surface always acquires minimum surface area The angle between the tangent to the liquid surface and the
due to surface tension (ST). So, the small droplet of any tangent to the solid surface at the point of contact (inside the
liquid is always spherical. liquid) is known as angle of contact.
2. The ST is a molecular phenomenon as ST is due to
'cohesion' between the molecules of a liquid.
3. The force of attraction between the molecules of the same Glass Glass
substance is called a cohesive force and that between
molecules of different substance is called adhesive force. q q
4. The molecular range is the maximum distance (10–9 m) upto Mercury
Water
which the molecules attract each other.
5. In general the ST of liquids decreases with increase in
temperature but the ST of molten Cadmium and Copper q < 90º q > 90°
increases with increase in temperature.
6. If the impurity is completely soluble then on mixing it in the
Some values of angle of contact of solid and liquid :
liquid, its surface tension increases. For example on
dissolving ionic salts in small quantities in a liquid, its
surface tension increases. On dissolving salt in water, its Pair of s urface Angle of contact

.IN
surface tension increases. Pure water and glas s 0°
7. If the impurity is partially soluble in a liquid, then its surface Silver and glas s 90°
tension decreases. For example on mixing detergent or
A lcohol and glass 138°
phenol in water its surface tension decreases.
AL
8. On increasing temperature surface tension decreases. At Normal water & glas s 8°
critical temperature and boiling point it becomes zero. Surface
Mercury & glas s 135°
N
tension of water is maximum at 4°C.
R

S.T.
U

of water
JO

S.T.=0
U

4°C Temp. C
ED

Adhesion > cohesion Adhesion = cohesion Adhesion < cohesion


1. Liquid will wet the solid Critical Liquid will not wet the solid
2. Meniscus is concave Meniscus is plane Meniscus is convex
3. Angle of contact is acute (q < 90°) Angle of contact is 90º Angle of contact is obtuse (q > 90°)
4. Pressure below the menisucs is lesser Pressure below the meniscus Pressure below the meniscus is more
2T 2T
than above it by (2T/r), i.e. p = p0 – is same as above it, i.e. p = p0 then above it by (2T/r), i.e., p = p0 +
r r
5. In capillary tube liquid will ascend. No capillary rise In capillary tube liquid will descend.

Keep in Memory 4. For all those liquids which neither rise nor get depressed in
a capillary tube, the angle of contact is right angle (q = 90°),
1. The value of angle of contact lies between 0º and 180º. For
e.g. silver and water.
pure water and glass it is 0º, for tap water and glass it is 8º
5. Angle of contact depends on impurities, water proofing
and for mercury and glass it is 135º.
agent, surface in contact and temperature. Angle of contact
2. For all those liquids which wet the solid surface and which
qC µ T where T is the temperature.
rise up in a capillary tube, the angle of contact is an acute
angle (q < 90°), e.g. water and glass.
3. For all those liquids which do not wet a solid surface and Capillarity :
which depress in a capillary tube, the angle of contact is an The phenomenon of rise or fall of liquids in capillary tube is
obtuse angle (q > 90°), e.g. glass and mercury. known as capillarity.
Mechanical Properties of Fluids 263

The rise or fall of a liquid in a capillary tube is given by Keep in Memory


r
2T cos q 2T 1. Work done in breaking a liquid drop of radius R into n
h= = q R
rrg Rrg equal small drops = 4pR 2 (n1 / 3 - 1) T ; where
T = surface tension.
2T q 2. Work done in breaking a soap bubble of radius R into n
Þ hrg =
R water equal small drops = 8pR 2 (n1 / 3 - 1) T ; where
T = surface tension.
where T = surface tension, q = angle of contact, r = density of
liquid, r = radius of capillary tube, R = radius of meniscus. Example 1.
(i) If capillary tube is of insufficient length l (i.e. l < h), then the A solid uniform ball having volume V and density r floats at
liquid rises to a full height h with radius R' such that hR = lR' the interface of two unmixable liquids as shown in fig. The
(ii) When the capillary tube is tilted from vertical by an angle densities of the upper and the lower liquids are r1 and r2
a, then the vertical height h of liquid column remains the respectively, such that r1 < r < r2. What fraction of the
same. The length of liquid in capillary increases such that volume of the ball will be in the lower liquid ?

h h
cos a = or h ¢ = . r1
h¢ cos a r
1 r2
According to Zurin's law capillary rise h µ

.IN
where r is the radius of the capillary tube. r
Keep in Memory Solution :
Let V1 and V2 be the volumes of the ball in the upper and
1. Work done in forming a liquid drop of radius R, surface
AL
lower liquids respectively. So V1 + V2 = V.
tension T is, W = 4pR2T. As ball is floating in the two liquids ; weight of the ball =
2. Work done in forming a soap bubble of radius R, surface upthrust on ball due to two liquids
N
tension T is, W = 2×4pR2T = 8pR2T. i.e., V r g = V1 r1 g + V2 r2 g ;
R

3. When n no. of smaller drops of liquid, each of radius r, or V r = V1 r1 + (V – V1) r2 ;


surface tension T are combined to form a bigger drop of
U

radius R then R = n 1/3 r . æ r - r2 ö


or V1 = çç ÷÷ V
The surface area of bigger drop = 4pR2 = 4pn2/3 r2. It is less è r1 - r 2 ø
JO

4.
than the area of n smaller drops.
V1 r - r 2
\ Fraction in the upper liquid = =
SHAPE OF LIQUID MENISCUS : V r1 - r2
U

The pressure on the concave side is always greater than the V1


Fraction in the lower liquid = 1-
ED

pressure on the convex side.


V
P0 = Atmospheric
r- r2 r1 - r
P0 = Atmospheric pressure = 1- =
r1 - r 2 r1 - r2
pressure
Example 2.
A A A piece of cork is embedded inside of block of ice which
Mercury
Water floats on water. What will happen to the level of water
when all the ice melts?
Solution :
q < 90º q > 90º Let , M = mass of the block of ice, m = mass of piece of cork
Concave meniscus Convex meniscus and V = Volume of water displaced.
PA=P0–2T/r PA=P0+2T/r Now (M + m) = V × 1 = V ...(1)
(r is radius of meniscus) When the ice melts, let it be converted into V ' c.c. of
Excess pressure of liquid drop and soap bubble : water.
(i) Excess of pressure for spher ical soap bubble is
Also M = V ' ´ 1 = V '
p = 4T/r and excess of pressure for liquid drop and air bubble
The piece of cork floats on the surface of water when all ice
in a liquid is p = 2T/r.
(ii) (a) Excess of pressure within a cylindrical liquid drop p = melts. Let the cork displaces a volume V '' c.c. of water..
T/R Then m = V" ´ 1 = V"
(b) Excess of pressure within a cylindrical soap bubble p If V1 be the volume of water displaced by melted ice and
= 2T/R cork, then
where T = surface tension, R = radius of the cylindrical
drop.
( M + m ) = V ' + V" = V1 ...(2)
EBD_7179
264 PHYSICS

From eqns. (1) and (2), V = V1 The pressure at a depth d, in liquid P = hdg. Therefore, the
Hence, no change in the level of water. total pressure inside the air bubble is
Example 3. 2T
Two substances of densities r1 and r2 are mixed in equal Pin = Patm + hdg +
r
volume and the relative density of mixture is 4. When they
are mixed in equal masses, the relative density of the or Pin = 1.013 × 105 + 10 × 10–2 × 103 × 9.8 + 1460
mixture is 3. Determine the values of r1 and r2 . = 101300 + 980 + 1460
Solution : = 103740 = 1.037 × 105 Pa.
When the substances are mixed in equal volumes, then
Example 6.
Vr1 + Vr 2 = 2 V ´ 4 ... (1) A capillary of the shape as shown is dipped in a liquid.
When the two substances are mixed in equal masses, then Contact angle between the liquid and the capillary is 0°
m m 2m and effect of liquid inside the meniscus is to be neglected. T
+ = ...(2) is the surface tension of the liquid, r is radius of the meniscus,
r1 r 2 3
g is acceleration due to gravity and r is density of the
From eq. (1), r1 + r 2 = 8 ... (3) liquid then determine the height h in equilibrium.
1 1 2 r1 + r 2 2
From eqn. (2) r + r = or =
1 2 3 r1 r 2 3

.IN
h
8 2
or = or r1 r 2 = 12 . ...(4)
r1 r 2 3 AL
2 1/ 2
Now r1 - r2 = [(r1 + r 2 ) - 4 r1 r 2 ]
= [64 - 48]1 / 2 = 4 ...(5) Solution :
N
Solving eqns. (3) and (5), we get r1 = 6 and ρ 2 = 2 ] As weight of liquid in capillary is balanced by surface tension,
R

Example 4. then T ´ 2pr = pr 2 h1rg (for a tube of uniform radius r)


r
A sealed tank containing a liquid of density r moves with
U

a horizontal acceleration a, as shown in fig. Find the


2T h1
JO

difference in pressure between the points A and B. h1 =


rrg
l
But weight of liquid in tapered tube is more than uniform tube
U

C A
a of radius r, then in order to balance h < h 1.
h
ED

B r
2T
h<
Solution : rrg
Since points A and C are in the same horizontal line but
separated by distance l and liquid tank is moving
horizontally with acceleration a, hence Example 7.
PC – PA = lra or PC = PA + lra A hydraulic automobile lift is designed to lift car with a
Points B and C are vertically separated by h maximum mass of 3000 kg. The area of cross-section of the
\ PB – PC = h r g
piston carrying the load is 425 cm2. What maximum pressure
or PB – (PA + l r a) = h r g
or PB – PA = h r g + l ra would the smaller piston have to bear ?
Solution :
Example 5.
Calculate the excess pressure within a bubble of air of Here mass of car = 3000 kg.
radius 0.1 mm in water. If the bubble had been formed 10 cm Area of cross section of larger piston
below the water surface when the atmospheric pressure = 425 cm2 = 425 × 10–4 m2.
was 1.013 × 105 Pa, then what would have been the total \ The maximum pressure that the smaller piston would
pressure inside the bubble? have to bear
Solution :
Excess pressure within air bubble Weight of car 3000 ´ 9.8
= =
Area of cross-section 425 ´ 10 -4
2T 2 ´ 73 ´ 10-3
= = = 1460 Pa
r 0.1 ´ 10 -3 = 6.92 ´ 10 5 Nm - 2
Mechanical Properties of Fluids 265

FLOW OF LIQUIDS where h is called the coefficient of viscosity.


The motion of fluids are of following four types : Its S.I. unit is poiseuille or decapoise. The C.G.S. unit is called
(i) Streamline motion : When fluid in motion, if fluid particles poise.
preceeding or succeeding a fluid particle follow the same 1 decapoise =10 poise.
path, then the path is called streamline and the motion of
Effects on Viscosity :
the fluid is called streamline motion. This type of motion
takes place in non-viscous fluids having very small speed. (1) Effect of temperature : On increasing temperature viscosity
Principle of continuity : When incompressible, non-viscous liquid of a liquid decreases.
flows in non-uniform tube then in streamline flow product of (2) Effect of pressure : On increasing pressure viscosity of a
area and velocity at any section remains same. liquid increases but viscosity of water decreases.
The mass of liquid flowing in equals the mass flowing out. Keep in Memory
i.e., m1 = m2
1. The viscosity of gases increases with increase of
or, v1A1r1 Dt = v2 A2r2Dt ..... (1) temperature, the rate of diffusion increases.
Q 2. The viscosity of liquids decrease with increase of
v2 temperature, because the cohesive force between the liquid
molecules decreases with increase of temperature.
P A2
Critical Velocity :
It is the maximum velocity of a fluid above which a stream line

.IN
v1
flow changes to a turbulent flow, i.e. it is the maximum velocity
of a liquid below which its flow remains streamline.
(i) Reynold's formula for critical velocity is
AL
A1 Nη V rr
Vc = or N = c
As we have considered the fluid incompressible thus, ρr h
N
v1 A1 = v2 A2 or Av = constant ....(2)
where N = Reynold's no., h = coefficient of viscosity,,
(Since r1 = r2)
R

r = density of liquid; r = radius of tube, N = 1000 for narrow


Equations (1) and (2) are said to be as equation of continuity. tube.
U

(ii) Steady state motion : In a liquid in motion, when liquid (ii) (a) If 0 < N < 2000 then the flow is laminar
JO

particles, crossing a point, cross it with same velocity, then (b) If 2000 < N < 3000 then flow of liquid is unstable and
the motion of the liquid is called steady state motion. This may change from laminar to turbulent
type of motion takes place in non-viscous liquids having (c) If N > 3000, then the flow is turbulent.
U

very small speed. (iii) When velocity of fluid is less than its critical velocity then
(iii) Laminar motion : Viscous liquids flow in bounded region the flow of liquid is determined by its viscosity, its density
ED

or in a pipe, in layers and when viscous liquid is in motion, has no effect on its flow.
different layers have different velocities. The layers in (iv) When the velocity of liquid is more than its critical velocity
contact with the fixed surface has least velocity and the then its flow is determined by its density, where viscosity
velocity of other parallel layers increases uniformly and has little effect on its flow. For example lava from volcano is
continuously with the distance from the fixed surface to highly thick, despite that it comes out with high speed.
the free surface of the liquid. This is called laminar motion
(v) When V £ Vc the flow of liquid is streamline and when V
of the liquid.
(iv) Turbulent motion : When the velocity of a liquid is irregular, > Vc then the flow of liquid is turbulent.
haphazard and large, i.e. Beyond a limiting value called Vc1 r 2
critical velocity the flow of liquid loses steadiness then =
Vc2 r for same liquid, which is flowing in two tubes of
1
the motion of the liquid is called turbulent motion.
radii r1 and r2 respectively.
Kh
Critical velocity Vc = (vi) The critical velocity of a liquid with high viscosity and
rr smaller radius is higher than that of a liquid with low
Here h is called coefficient of viscosity. viscosity and greater radius.
VISCOSITY Reynold's Number (N) :
The internal friction of the fluid, which tends to oppose relative It is pure number which determines the nature of flow of liquid
motion between different layers of the fluid is called viscosity. through a pipe.
The viscous force between two layers of a fluid of area A having
dv V dr V2 Inertial force/area
a velocity gradient is given by N= c = c =
dx η æ ηVc ö Viscous force/area
dv ç r ÷
F = -h A è ø
dx N is a dimensionless quantity and carries no unit.
EBD_7179
266 PHYSICS

BERNOULLI'S THEOREM STOKE’S LAW


For non-viscous, incompressible, streamline flow of fluids the When a solid moves through a viscous medium, its motion is
sum of pressure per unit volume, potential energy per unit volume opposed by a viscous force depending on the velocity and shape
and Kinetic energy per unit volume remain constant. and size of the body. The energy of the body continuously
decreases in overcoming the viscous resistance of the medium.
p 1 This is why cars, aeroplanes etc. are shaped streamline to minimize
i.e., + gh + v2 = constant
r 2 the viscous resistance on them.
where r = density of fluid. The viscous drag on a spherical body of radius r, moving with
velocity v, in a viscous medium of viscosity h is given by
p 1 2 Fviscous = 6phrv
When h = 0 then + v = constant
r 2 This relation is called Stokes' law. .
Bernoulli's theorem is strictly applicable for an ideal fluid. Importance of Stoke’s law :
An ideal fluid is one which is (a) incompressible (b) streamline (i) It is used in the determination of electronic charge with the
(c) irrotational and (d) non-viscous. help of milikan’s experiment.
Applications of Bernoulli’s Principle (ii) It accounts the formation of clouds.
(iii) It accounts why the speed of rain drops is less than that of
Dynamic lift :
a body falling freely with a constant velocity from the height
(i) Wings of aeroplane : The wings of the aeroplane are having
of clouds.
tapering. Due to this specific shape of wings when the (iv) It helps a man coming down with the help of a parachute.

.IN
aeroplane runs, air passes at higher speed over it as compared
Terminal Velocity :
to its lower surface. This difference of air speeds above and
below the wings, in accordance with Bernoulli's principle, When a spherical body is allowed to fall through viscous
medium, its velocity increases till the viscous drag plus upthrust
AL
creates a pressure difference, due to which an upward force
is equal to the weight of the body. After that body moves with
called 'dynamic lift' (= pressure difference area of wing) acts
constant velocity, called terminal velocity.
on the plane. If this force becomes greater than the weight of
2R 2
N
the plane, the plane will rise up.
The terminal velocity is given by v = ( d - s) g
v large, 9h
R

p small
where d = density of body,
s = density of medium,
U

h = coefficient of viscosity of medium,


JO

R = radius of the spherical body.


v small, p large 1
From terminal velocity, Va , i.e. greater the viscosity, smaller
(ii) Ball moving without spin: The velocity of fluid (air) above h
U

and below the ball at corresponding points is the same is the terminal speed.
ED

resulting in zero pressure difference. The air therefore, exerts Flow of liquid through tube /pipe :
no upward or downward force on the ball.
(iii) Ball moving with spin: A ball which is spinning drags air ppr 4
(i) Poiseuille's equation is Q = , where p is the pressure
along with it. If the surface is rough more air will be dragged. 8hl
The streamlines of air for a ball which is moving and spinning difference between the two ends of the tubes, r is the
at the same time. The ball is moving forward and relative to it radius, l is the length of the tube and h is the coefficient
the air is moving backwards. Therefore, the velocity of air of viscosity, Q = rate of flow of liquid.
above the ball relative to it is larger and below it is smaller. 8hl1 8hl 22
Equivalent liquid resistance = + , when tube
The stream lines thus get crowded above and rarified below. pr14 pr24
This difference in the velocities of air results in the pressure are joined in series.
difference between the lower and upper faces and their is a net It means that liquid flow through capillary tube is similar
to flow of electric current through a conductor i.e., Q (rate
upward force on the ball. This dynamic lift due to spinning is
of liquid flow) corresponds to I (rate of flow of charge),
called Magnus effect. pressure difference similar to potential difference.
Some other applications of Bernoulli’s principle : (ii) When two tubes are joined in series then the volume of
(i) The action of carburator, sprayer or atomizer based on fluid flowing through the two tubes is the same but the
Bernoulli’s principle. pressure difference across the two tubes is different. The
(ii) The action of bunsen burner, exhaust pump etc. total pressure difference, P = P1 + P2.
(iii) If two tubes are joined in parallel then the pressure
(iii) Air foil or lift on aircraft wing works on Bernoulli’s principle. difference across the two tubes is the same but the volume
(iv) Motion of a spinning ball i.e., magnus effect. of fluid flowing through the two tubes is different. The
(v) Blowing of roofs by wind storms etc. based on Bernoullis total volume of the fluid flowing through the tube in one
principle. second is Q = Q1 + Q2.
Mechanical Properties of Fluids 267

VELOCITY OF EFFLUX AND TORRICELLI'S THEOREM


(i) Torricelli's theorem : For liquid filled in a tank upto a
height H having a hole O at a depth h from free level of
=
A
a
æ 2ö
´ ç ÷´
è gø
( H1 - H 2 )
liquid through which the liquid is coming out, velocity of Example 9.
efflux of liquid v = 2gh . In a test experiment on a model aeroplane in a wind tunnel,
(ii) Time taken by the liquid in falling from hole to ground the flow speeds on the upper and lower surfaces of the
wing are 70 m/s and 63 m/s respectively. What is the lift on
2( H - h) the wing if its area is 2.5 m2 ? Take the density of air to be
level t =
g 1.3 kg m–3.
Free
surface Solution :
Let v1 be the speed and P1 be the pressure on the upper
h surface of the wing, and corresponding values on the lower
H
surface be v2 and P2 respectively.
\ v1 = 70 m/s, v2 = 63 m/s, A = 2.5 m2,
(iii) The horizontal range covered by the liquid P = 1.3 kg m–3.
x = horizontal velocity (v) ×time (t) According to Bernoulli’s theorem

.IN
2( H - h) 1 1
= 2 gh ´ = 2 h( H - h) . P1 + rv12 = P2 + rv 22
g 2 2
For maximum horizontal range, differentiating both side w.r.t. 1 1
P2 - P1 = r(v12 - v 22 ) = P2 - P1 = ´ 3 ´ (70 2 - 632 )
h and we get h = H/2, the range is maximum.
AL 2 2
Keep in Memory Force (lift) on the wing = A (P2 - P1 )
N
1. The cross-section of the water stream from a top decreases 1
= 2.5 ´ ´ 1.3 ´ (70 2 - 632 )
2
R

as it goes down in accordance with the equation of


continuity. 1
= 2.5 ´ ´ 1.3 ´ 133 ´ 7
U

2. When a hale blows over a roof, the force on the roof is 2


upwards.
JO

3. Sudden fall in atmospheric pressure predicts possibility of = 1.5 ´ 103 N


a storm. Example 10.
4. Venturimeter is a device used for the measurement of the In Millikan’s oil drop experiment, what is the terminal
U

rate of flow of incompressible fluid through a tube. Its speed of an uncharged drop of radius 2.0 × 10–5 m and
working is based on Bernoulli’s principle. density 1.2 × 103 kg m–3. Take the viscosity of air at the
ED

temperature of the experiment to be 1.8 × 10–5 Pa s. How


Example 8. much is the viscous force on the drop at that speed ? Neglect
The vessel of area of cross-section A has liquid to a height buoyancy of the drop due to air.
H. There is a hole at the bottom of vessel having area of Solution :
cross-section a. Find the time taken to decrease the level Here, r = 2.0 × 10–5 m, r = 1.2 × 103 kg m–3,
from H1 to H2. h = 1.8 × 10–5 Pa s.
Solution : From formula, terminal velocity
2 g H1 + 2 g H 2
The average velocity of efflux, v = 2 r 2 (r - s ) g
2 V=
Let t be the time taken to empty the tank from level H1 to 9 h
H2. 2 ´ (2 ´ 10 -5 ) 2 (1.2 ´ 103 - 0) ´ 9.8
Þ V=
2 g H1 + 2 g H 2 9 ´ 1.8 ´ 10-5
Then, ´ a ´ t = A [H1 - H 2 ]
2 = 5.8 ´ 10 -2 ms -1

A æ 2 ö é (H1 - H 2 )´
çç ÷÷ ´ ê
( H1 - H 2 ) ùú Now viscous force on the drop
or t = a ´
( )( H1 - H 2 )úû
F = 6phrv
è g ø êë H1 + H 2
22
Þ F = 6´ ´ (1.8 ´ 10-5 ) ´ (2 ´ 10 -5 ) ´ 5.8 ´ 10 -2
7
=
A
a
´
æ 2ö
çç ÷÷ ´ ( H1 - H 2 ) = 3.93 × 10–19 N
ègø
268

ED
U
JO
U
R
N
AL
.IN
PHYSICS

EBD_7179
Mechanical Properties of Fluids 269

1. The constant velocity attained by a body while falling (a) mx y (x – y) (b) m/(y – x)
through a viscous medium is termed as
(a) critical velocity (b) terminal velocity æ1 1ö
(c) m çç - ÷÷ (d) (y – x)/x
(c) threshold velocity (d) None of these èy xø
2. The difference between viscosity and solid friction is/are 11. Consider a 1 c.c. sample of air at absolute temperature T0 at
(a) viscosity depends on area while solid friction does not sea level and another 1 c.c. sample of air at a height where
(b) viscosity depends on nature of material but solid the pressure is one-third atmosphere. The absolute
friction does not temperature T of the sample at the height is
(c) both (a) and (b) (a) equal to T0/3
(d) neither (a) nor (b) (b) equal to 3/T0
3. Water is not used in thermometer because (c) equal to T0
(a) it sticks to glass (d) cannot be determined in terms of T0 from the above data
(b) its shows anamalous expansion 12. A small ball (menu) falling under graivty in a viscous medium

.IN
(c) both (a) and (b) experiences a drag force proportional to the instantaneous
(d) neither (a) nor (b) speed u such that Fdrag = Ku . Then the terminal speed of
4. Toricelli’s theorem is used to find
AL the ball within the viscous medium is
(a) the velocity of efflux through an orifice.
(b) the velocity of flow of liquid through a pipe. K mg
(a) (b)
N
(c) terminal velocity mg K
(d) critical velocity
R

2
5. Gases do not posses mg æ mg ö
(c) (d) ç ÷
U

(a) density (b) surface tension K è K ø


(c) volume (d) viscosity
JO

13. A cylinder is filled with non-viscous liquid of density d to a


6. Paint-gun is based on height h 0 and a hole is made at a height h 1 from the bottom
(a) Bernoulli’s theorem (b) Archimedes’ principle of the cylinder. The velocity of the liquid issuing out of the
(c) Boyle’s law (d) Pascal’s law
U

hole is
7. Water is flowing through a horizontal pipe having a
(a) 2 gh 0 (b) 2 g (h 0 - h1 )
ED

restriction, then
(a) pressure will be greater at the restriction.
(c) dgh 1 (d) dgh 0
(b) pressure will be greater in the wider portion.
(c) pressure will be same throughout the length of the pipe. 14. The terminal velocity depends upon
(d) None of these 1 1
(a)1 (b) (c ) 3 (d) r
2
8. Fevicol is added to paint to be painted on the walls, because 2
r r r
(a) it increases adhesive force between paint and wall.
15. The velocity of efflux of a liquid through an orifice in the
(b) it decreases adhesive force between paint and wall
bottom of the tank does not depend upon
molecules.
(a) size of orifice
(c) it decreases cohesive force between paint molecules.
(b) height of liquid
(d) None of these
(c) acceleration due to gravity
9. A beaker containing a liquid of density r moves up with an
(d) density of liquid
acceleration a. The pressure due to the liquid at a depth h
below the free surface of the liquid is 16. At the boiling point of a liquid, surface tension
(a) h r g (b) h r (g – a) (a) is zero
(b) is infinite
æg+aö (c) is same as that at any other temperature
(c) h r (g + a) (d) 2h r g çç ÷÷
è g -a ø (d) cannot be determined
17. The surface energy of a liquid drop of radius r is proportional
10. The density of ice is x gram/litre and that of water is y gram/
to
litre. What is the change in volume when m gram of ice
1
melts? (a) r3 (b) r2 (c) r (d)
r
EBD_7179
270 PHYSICS

18. A liquid is contained in a vessel. The liquid-solid adhesive 22. The rain drops falling from the sky neither injure us nor
force is very weak as compared to the cohesive force in the make holes on the ground because they move with
liquid. The shape of the liquid surface will be (a) constant acceleration
(a) horizontal (b) vertical (b) variable acceleration
(c) concave (d) convex (c) variable speed
19. Two liquids drops coalesce to form a large drop. Now, (d) constant terminal velocity
(a) energy is liberated 23. The lift of an air plane is based on
(b) energy is neither liberated nor absorbed (a) Torricelli's theorem
(c) some mass gets converted into energy (b) Bernoulli's theorem
(d) energy is absorbed (c) Law of gravitation
20. A man is sitting in a boat which is floating in pond. If the (d) conservation of linear momentum
man drinks some water from the pond, the level of water in 24. Surface tension may be defined as
the pond will (a) the work done per unit area in increasing the surface
(a) rise a little (b) fall a little area of a liquid under isothermal condition
(c) remain stationary (d) none of these (b) the work done per unit area in increasing the surface
21. A liquid is allowed to flow into a tube of truncated cone area of a liquid under the adiabatic condition
shape. Identify the correct statement from the following. (c) the work done per unit area in increasing the surface
(a) The speed is high at the wider end and high at the area of liquid under both isothermal and adiabatic condition

.IN
narrow end. (d) free surface energy per unit volume
(b) The speed is low at the wider end and high at the 25. A liquid does not wet the sides of a solid, if the angle of
narrow end.
AL
contact is
(c) The speed is same at both ends in a stream line flow. (a) Zero (b) Obtuse (more than 90°)
(d) The liquid flows with uniform velocity in the tube. (c) Acute (less than 90°) (d) 90° (right angle)
N
R
U
JO

1. The reading of spring balance when a block is suspended (a) 0.7 m/s (b) 0.07 m/s
from it in air is 60 newton. This reading is changed to 40 (c) 0.007 m/s (d) 0.0007 m/s
newton when the block is submerged in water. The specific 7. A cylinder of height 20m is completely filled with water. The
U

gravity of the block must be therefore velocity of efflux of water (in ms–1) through a small hole on
(a) 3 (b) 2 (c) 6 (d) 3/2 the side wall or the cylinder near its bottom is
ED

2. An ice-berg floating partly immersed in sea water of density


(a) 10 m/s (b) 20 m/s (c) 25.5 m/s (d) 5 m/s
1.03 g/cm3. The density of ice is 0.92 g/cm3. The fraction of
the total volume of the iceberg above the level of sea water 8. 8 mercury drops coalesce to form one mercury drop, the
is energy changes by a factor of
(a) 8.1% (b) 11% (c) 34% (d) 0.8% (a) 1 (b) 2 (c) 4 (d) 6
3. A boat having a length of 3 metres and breadth 2 metres is 9. Water rises to a height of 10 cm in capillary tube and mercury
floating on a lake. The boat sinks by one cm when a man falls to a depth of 3.1 cm in the same capillary tube. If the
gets on it. The mass of the man is density of mercury is 13.6 and the angle of contact for
(a) 60 kg (b) 62 kg (c) 72 kg (d) 128 kg mercury is 135°, the approximate ratio of surface tensions of
4. The excess of pressure inside a soap bubble is twice the water and mercury is
excess pressure inside a second soap bubble. The volume (a) 1 : 0.15 (b) 1 : 3 (c) 1 : 6 (d) 1.5 : 1
of the first bubble is n times the volume of the second where
n is 10. A vessel with water is placed on a weighing pan and it reads
(a) 0.125 (b) 0.250 (c) 1 (d) 2 600 g. Now a ball of mass 40 g and density 0.80 g cm–3 is
5. The level of water in a tank is 5m high. A hole of area 1 cm 2 sunk into the water with a pin of negligible volume, as shown
is made in the bottom of the tank. The rate of leakage of in figure keeping it sunk. The weighting pan will show a
water from the hole (g = 10 m/s2) is reading
(a) 10–2 m3/s (b) 10–3 m3/s (a) 600 g
(c) 10–4 m3/s (d) 103 m3/s
(b) 550 g
6. A spherical ball of iron of radius 2 mm is falling through a
column of glycerine. If densities of glycerine and iron are (c) 650 g
respectively 1.3 × 103 kg/m3 and 8 × 103 kg/m3. h for glycerine
= 0.83 Nm–2 sec, then the terminal velocity is (d) 632 g Weighing pan
Mechanical Properties of Fluids 271

11. Water flows in a stream line manner through a capillary tube 19. If a water drop is kept between two glass plates, then its
of radius a, the pressure difference being P and the rate flow shape is
a
Q. If the radius is reduced to and the pressure is increased
2
to 2P, the rate of flow becomes
(a) (b)
Q Q
(a) 4Q (b) Q (c) (d)
2 8
12. A rain drop of radius 0.3 mm has a terminal velocity in air =
1 m/s. The viscosity of air is 8 × 10–5 poise. The viscous
force on it is (c) (d) None of these
(a) 45.2 × 10–4 dyne (b) 101.73×10–5 dyne
–4
(c) 16.95 × 10 dyne (d) 16.95 × 10–5 dyne 20. The fraction of a floating object of volume V0 and density
13. A big drop of radius R is formed by 1000 small droplets of d0 above the surface of liquid of density d will be
water. The radius of small drop is d0 d - d0
(a) (b)
R R R R d - d0 d
(a) (b) (c) (d)
10 100 500 1000 d0 d 0d
(c) (d)
d + d0
14. 1 m3 water is brought inside the lake upto 200 metres depth d
from the surface of the lake. What will be change in the 21. An open vessel containing water is given a constant

.IN
volume when the bulk modulus of elastically of water is acceleration a in the horizontal direction. Then the free
22000 atmosphere? surface of water gets sloped with the horizontal at an angle
(density of water is 1 × 103 kg/m3 atmosphere pressure
AL q, given by
= 105 N/m2 and g = 10 m/s2) g a
(a) q = cos -1 (b) q = tan -1
(a) 8.9 × 10–3 m3 (b) 7.8 × 10–3 m3 a g
(c) 9.1 × 10 m–4 3 (d) 8.7 × 10–4 m3 a g
N
(c) q = sin -1 (d) q = tan -1
15. Horizontal tube of non-uniform cross-section has radii of g a
R

0.1 m and 0.05 m respectively at M and N for a streamline


22. Two drops of the same radius are falling through air with a
flow of liquid the rate of liquid flow is
U

steady velocity of 5 cm per sec. If the two drops coalesce,


N the terminal velocity would be
JO

(a) 10 cm per sec (b) 2.5 cm per sec


(c) 5 × (4)1/3 cm per sec (d) 5 ´ 3 cm per sec
23. A tank is filled with water upto a height H. Water is allowed
U

M
to come out of a hole P in one of the walls at a depth h below
ED

(a) continuously changes with time the surface of water (see fig.) Express the horizontal distance
(b) greater at M than at N X in terms H and h.
(c) greater at N than at M
(d) same at M and N
h
16. There is a hole in the bottom of tank having water. If total P
pressure at bottom is 3 atm (1 atm = 105 N/m2) then the H
velocity of water flowing from hole is
(a) 400 m/s (b) 600 m/s
x
(c) 60 m/s (d) None of these
17. In the figure, the velocity V3 will be h
(a) X = h (H - h ) (b) X= (H - h )
2
A2 = 0.2 m2
V1 = 4 ms –1
V2 = 2 ms–1 (c) X = 2 h (H - h) (d) X = 4 h (H - h)
A1 = 0.2 m2 24. A body of density r' is dropped from rest at a height h into
A3= 0.4 m 2 V3 a lake of density r where r > r' neglecting all dissipative
(a) Zero (b) 4 ms–1 (c) 1 ms–1 (d) 3 ms–1 forces, calculate the maximum depth to which the body sinks
before returning to float on the surface :
18. 1 centipoise is equal to
(a) 1 kg m–1 s–1 (b) 1000 kg m–1 s–1 h hr' hr' hr
–1 –1 (a) (b) (c) (d)
(c) 0.1 kg m s (d) 0.001 kg m–1 s–1 r - r' r r - r' r - r'
EBD_7179
272 PHYSICS

25. Two capillary of length L and 2L and of radius R and 2R are 32. Two pieces of metals are suspended from the arms of a
connected in series. The net rate of flow of fluid through balance and are found to be in equilibrium when kept
them will be (given rate to the flow through single capillary, immersed in water. The mass of one piece is 32 g and its
density 8 g cm–3. The density of the other is 5 g per cm3.
pPR 4
X= ) Then the mass of the other is
8hL
(a) 28 g (b) 35 g (c) 21 g (d) 33.6 g
8 9 5 7 33. Figure shows a weigh-bridge, with a beaker P with water on
(a) X (b) X (c) X (d) X one pan and a balancing weight R on the other. A solid ball
9 8 7 5
Q is hanging with a thread outside water. It has volume 40
26. One drop of soap bubble of diameter D breaks into 27 drops cm3 and weighs 80 g. If this solid is lowered to sink fully in
having surface tension s. The change in surface energy is water, but not touching the beaker anywhere, the balancing
(a) 2psD2 (b) 4psD2 weight R' will be
(c) psD 2 (d) 8psD2
27. In case A, when an 80 kg skydiver falls with arms and legs
fully extended to maximize his surface area, his terminal Q
velocity is 60 m/s. In Case B, when the same skydiver falls
with arms and legs pulled in and body angled downward to R
minimize his surface area, his terminal velocity increases to P

.IN
80 m/s. In going from Case A to Case B, which of the
following statements most accurately describes what the
skydiver experiences? AL
(a) Fair resistance increases and pressure P increases (a) same as R (b) 40 g less than R
(b) Fair resistance increases and pressure P decreases (c) 40 g more than R (d) 80 g more than R
N
(c) Fair resistance decreases and pressure P increases 34. Figure here shows the vertical cross section of a vessel
(d) Fair resistance remains the same and pressure P increases filled with a liquid of density r. The normal thrust per unit
R

28. A ring is cut from a platinum tube 8.5 cm internal and 8.7 cm area on the walls of the vessel at the point P, as shown, will
U

external diameter. It is supported horizontally from the pan be


of a balance, so that it comes in contact with the water in a
JO

glass vessel. If an extra 3.97. If is required to pull it away


from water, the surface tension of water is P
(a) 72 dyne cm–1 (b) 70.80 dyne cm–1
U

H h
(c) 63.35 dyne cm–1 (d) 60 dyne cm–1
ED

29. A capillary tube of radius r is immersed in a liquid. The


liquid rises to a height h. The corresponding mass is m. q
What mass of water shall rise in the capillary if the radius of
the tube is doubled? (a) hrg (b) Hrg
(a) m (b) 2m (c) 3m (d) 4m (c) (H – h) rg (d) (H – h) rg cosq
30. In a satellite moving round any planet, the gravitational 35. Two vessels A and B of cross-sections as shown in figure
force is effectively balanced. If an ice cube exists there, and contain a liquid up to the same height. As the temperature
it melts with passage of time, its shape will
rises, the liquid pressure at the bottom (neglecting expansion
(a) remain unchanged of the vessels) will
(b) change to spherical
(c) become oval-shaped with long-axis along the orbit
plane
(d) become oval-shaped with long axis perpendicular to
orbit plane A B
31. An egg when placed in ordinary water sinks but floats when
placed in brine. This is because
(a) increase in A, decrease in B
(a) density of brine is less than that of ordinary water
(b) increase in B, decrease in A
(b) density of brine is equal to that of ordinary water
(c) density of brine is greater than that of ordinary water (c) increase in both A and B
(d) None of these (d) decrease in both A and B
Mechanical Properties of Fluids 273

36. A beaker with a liquid of density 1.4 g cm–3 is in balance 42. Figure shows a capillary rise H. If the air is blown through
over one pan of a weighing machine. If a solid of mass 10 g the horizontal tube in the direction as shown then rise in
and density 8 g cm–3 is now hung from the top of that pan capillary tube will be
with a thread and sinking fully in the liquid without touching
the bottom, the extra weight to be put on the other pan for
balance will be
H
(a) 10.0 g

(b) 8.25 g

(c) 11.75 g (a) = H (b) > H (c) < H (d) zero


43. Two soap bubbles (surface tension T) coalesce to form a
(d) –1.75 g big bubble under isothermal condition. If in this process the
37. The pressure energy per unit volume of a liquid is change in volume be V and the surface area be S, then the
P r correct relation is (P is atmospheric pressure)
(a) (b) P (c) P × r (d) (a) PV + TS = 0 (b) 3PV + 4TS = 0
r P
38. A water tank of height 10m, completely filled with water is (c) 3PV + TS = 0 (d) 4PV + 3TS = 0
44. Two liquids of densities d1 and d2 are flowing in identical

.IN
placed on a level ground. It has two holes one at 3 m and the
other at 7 m from its base. The water ejecting from capillary tubes uder the same pressure difference. If t 1 and
(a) both the holes will fall at the same spot t2 are time taken for the flow of equal quantities (mass) of
liquids, then the ratio of coefficient of viscosity of liquids
(b) upper hole will fall farther than that from the lower hole
AL
(c) upper hole will fall closer than that from the lower hole must be
(d) more information is required t1 d1 t 1
d1 t 1 d2 t2
N
39. A fast train goes past way side station platform at high (a) (b) (c) (d)
d2t2 t2 d1 t 1 d2t 2
speed. A person standing at the edge of the platform is
R

(a) attracted to train 45. A tank has a small hole at its botom of area of cross-section
a. Liquid is being poured in the tank at the rate Vm3/s,
U

(b) repelled from train


(c) unaffected by outgoing train the maximum level of liquid in the container will be (Area of
JO

(d) affected only if the train's speed is more than the speed tank A)
of sound V2 V2
V V
40. Three tubes X, Y and Z are connected to a horizontal pipe (a) (b) (c) (d)
U

gaA 2gAa gAa 2gaA


in which ideal liquid is flowing. The radii of the tubes X, Y
ED

and Z at the junction are respectively 3 cm, 1 cm and 3 cm. It Directions for Qs. (46 to 50) : Each question contains
can be said STATEMENT-1 and STATEMENT-2. Choose the correct answer
(ONLY ONE option is correct ) from the following-
(a) Statement -1 is false, Statement-2 is true
X Y Z (b) Statement -1 is true, Statement-2 is true; Statement -2 is a
correct explanation for Statement-1
(c) Statement -1 is true, Statement-2 is true; Statement -2 is not
a correct explanation for Statement-1
(d) Statement -1 is true, Statement-2 is false
(a) the height of the liquid in the tube A is maximum. 46. Statement 1 : Smaller the droplets of water, spherical they
(b) the height of liquid in the tubes Aand B is same. are.
(c) the height liquid in the tubes A,B and C is same. Statement 2 : Force of surface tension is equal, and opposite
(d) the height of the liquid in the tubes A and C is the to force of gravity.
same. 47. Statement 1 : If a body is floating in a liquid, the density of
41. A sphere of brass released in a long liquid column attains a liquid is always greater than the density of solid.
terminial speed v0. If the terminal speed attained by sphere Statement 2 : Surface tension is the property of liquid
of marble of the same radius and released in the same liquid surface.
is nv0, then the value of n will be (Given : The specific 48. Statement 1 : The velocity of flow of a liquid is smaller
gravities of brass, marbles and the liquid are 8.5, 2.5 and 0.8 when pressure is larger and vice-versa.
respectively.) Statement 2 : According to Bernoulli’s theorem, for the
5 17 11 17 stream line flow of an ideal liquid, the total energy per unit
(a) (b) (c) (d) mass remains constant.
17 77 31 5
EBD_7179
274 PHYSICS

49. Statement 1 : Falling raindrops acquire a terminal velocity. can be considered to be acting at the centre of mass of the
Statement 2 : A constant force in the direction of motion object.
and a velocity dependent force opposite to the direction of Statement 2 : For a rigid body a force field distributed
motion, always result in the acquisition of terminal velocity. uniformly through its volume can be considered to be acting
50. Statement 1 : The buoyant force on a submerged rigid object at the centre of mass of the body.

Exemplar Questions 3. Along a streamline,


1. A tall cylinder is filled with viscous oil. A round pebble is (a) the velocity of a fluid particle remains constant
dropped from the top with zero initial velocity. From the plot (b) the velocity of all fluid particles crossing a given
shown in figure, indicate the one that represents the velocity position is constant
(v) of the pebble as a function of time (t) . (c) the velocity of all fluid particles at a given instant is
constant

.IN
(d) the speed of a fluid particle remains constant
4. An ideal fluid flows through a pipe of circular cross-section
AL
v v made of two sections with diameters 2.5 cm and 3.75 cm.
The ratio of the velocities in the two pipes is
N
t t (a) 9 : 4 (b) 3 : 2 (c) 3 : 2 (d) 2: 3
(a) (b)
5. The angle of contact at the interface of water-glass is 0°,
R

ethyl alcohol-glass is 0°, mercury-glass is 140° and


U

methyliodide-glass is 30°. A glass capillary is put in a trough


containing one of these four liquids. It is observed that the
JO

meniscus is convex. The liquid in the trough is


v v
(a) water (b) ethylalcohol
U

(c) mercury (d) methyliodide


t t
ED

(c) (d) NEET/AIPMT (2013-2017) Questions

2. Which of the following diagrams does not represent a 6. The wetability of a surface by a liquid depends primarily on
streamline flow? (a) surface tension [2013]
(b) density
(c) angle of contact between the surface and the liquid
(d) viscosity
7. A fluid is in streamline flow across a horizontal pipe of
variable area of cross section. For this which of the following
statements is correct? [NEET Kar. 2013]
(a) (b) (a) The velocity is minimum at the narrowest part of the
pipe and the pressure is minimum at the widest part of
the pipe
(b) The velocity is maximum at the narrowest part of the
pipe and pressure is maximum at the widest part of the
pipe
(c) Velocity and pressure both are maximum at the
narrowest part of the pipe
(d) Velocity and pressure both are maximum at the widest
(c) (d)
part of the pipe
Mechanical Properties of Fluids 275

8. A certain number of spherical drops of a liquid of radius ‘r’ (c) water does not rise at all.
coalesce to form a single drop of radius ‘R’ and volume ‘V’. (d) Water rises upto the tip of capillary tube and then starts
If ‘T’ is the surface tension of the liquid, then : [2014] overflowing like fountain.
æ1 1 ö 12. Two non-mixing liquids of densities r and nr(n > 1) are put
(a) energy = 4VT ç - ÷ is released in a container. The height of each liquid is h. A solid cylinder
è r Rø
of length L and density d is put in this container. The cylinder
æ1 1 ö floats with its axis vertical and length pL(p < 1) in the denser
(b) energy = 3VT ç + ÷ is absorbed liquid. The density d is equal to : [2016]
è r Rø
(a) {1 + (n + 1)p}r (b) {2 + (n + 1)p}r
æ1 1 ö (c) {2 + (n – 1)p}r (d) {1 + (n – 1)p}r
(c) energy = 3VT ç - ÷ is released
è r Rø
13. A U tube with both ends open to the atmosphere, is partially
(d) energy is neither released nor absorbed filled with water. Oil, which is immiscible with water, is poured
9. A wind with speed 40 m/s blows parallel to the roof of a into one side until it stands at a distance of 10 mm above the
house. The area of the roof is 250 m2. Assuming that the water level on the other side. Meanwhile the water rises by
pressure inside the house is atmospheric pressure, the force 65 mm from its original level (see diagram). The density of
exerted by the wind on the roof and the direction of the the oil is [2017]

.IN
forcewill be (rair = 1.2 kg/m3) [2015]
(a) 4.8 × 105 N, upwards
Pa Pa
(b) 2.4 × 105 N, upwards
AL F
(c) 2.4 × 105 N, downwards A
E 10 mm
(d) 4.8 × 105 N, downwards Final water level
N
10. The cylindrical tube of a spray pump has radius, R, one end 65 mm
Oil D
R

of which has n fine holes, each of radius r. If the speed of the Initial water level
liquid in the tube is V, the speed of the ejection of the liquid
65 mm
U

through the holes is : [2015 RS]


B C
JO

2 2 2 2
VR VR V R VR Water
(a) 2 (b) 3 2 (c) (d)
nr n r nr n2r2
U

11. Water rises to a height 'h' in a capillary tube. If the length of


ED

capaillary tube above the surface of water is made less than


'h' then : [2015 RS] (a) 425 kg m–3 (b) 800 kg m–3
(a) water rises upto the top of capillary tube and stays (c) 928 kg m–3 (d) 650 kg m–3
there without overflowing
(b) water rises upto a point a little below the top and stays
there
EBD_7179
276 PHYSICS

Hints & Solutions


EXERCISE - 1 W
24. (a) In Isothermal conditons T=
1. (b) 2. (a) 3. (c) 4. (a) 5. (b) DA
6. (a) 7. (b) 8. (a) where, T = surface tension, W = work done,
9. (c) When a beaker containing a liquid of density r moves DA = change in area.
up with an acceleration a, it will work as a lift moving 25. (b)
upward with acceleration a. The effective acceleration EXERCISE - 2
due to gravity in lift = (a + g)
\ Pressure of liquid of height h = h r (a + g) weight of block in air
1. (a) Specific gravity of block =
10. (c) Volume of m g of ice = m/x and loss of weight in water
volume of m g of water = m/y. So change in volume 60
= =3
60 - 40
m m æ1 1ö 2. (b) Let v be the volume of the ice-berg outside the sea
= y - x = mçç y - x ÷÷
è ø water and V be the total volume of ice-berg. Then as
per question

.IN
11. (a) PµT
0.92 V = 1.03 (V – v) or v/V = 1 – 0.92/1.03
12. (b) AL = 11/103
13. (b) Velocity of liquid flowing out of hole = 2 gh . \ (v/V) × 100 = 11 × 100 / 103 @ 11%
Here h = (h 0 – h1) 3. (a) Weight of a man = wt. of water displaced
N
2r 2g 1
14. (d) Terminal velocity, v T = (r - s) , v T µ r 2 = volume × density = 3 ´ 2 ´ ´ 103 = 60 kg
9h 100
R

4T 4T
15. (a) v = velocity of efflux through an orifice = 2gH 4. (a) Given, = 2´ or r2 = 2r1
U

r1 r2
JO

4 3 4 4
v=0 pr1 = n ´ pr23 = n ´ p( 2 r1 ) 3 or n = 1 = 0.125
3 3 3 8
H
U

5. (b) Velocity of efflux, v = 2gh ;


v
ED

volume of liquid flowing out per sec

It is independent of the size of orifice.


= v ´ A = 2gh ´ A = 2 ´ 10 ´ 5 ´10 -4 = 10 -3 m3/s
16. (a)
2 r 2 (r - r 0 )g
17. (b) Surface energy µ surface area = pr2 6. (b) Terminal velocity, v 0 =
9h
18. (d)
19. (a) When liquid drops coalesce, there is a decrease of 2 ´ ( 2 ´ 10 -3 ) 2 ´ (8 - 1.3) ´ 10 3 ´ 9.8
surface area and therefore decrease of surface energy. = = 0.07 ms–1
9 ´ 0.83
Hence, energy is liberated.
7. (b) P.E. = K.E.
20. (c)
21. (b) The theorem of continuity is valid.
\ A1v1r = A2v2r as the density of the liquid can be
taken as uniform.
20 m = h
A2
A1
v
\ A1v1 = A2v2
1
Þ Smaller the area, greater the velocity. mgh = mv2
2
22. (b)
23. (b) Apply Bernoulli’s theorem. v = 2gh = 2 ´10 ´ 20 (Here g = 10 m/s2) = 20 m/s
Mechanical Properties of Fluids 277

8. (c) Surface energy = suface tension × area of surface


2 P1 2 ´ 2 ´ 105
4
For 1 drop, volume = pR 3 if R = radius of drop. \ v= = = 400 m/s
r 103
3
4 17. (c) According to equation of continuity
Total volume of 8 drops = 8. pR 3 = p(2R )
4 3
A1V1 = A2V2 + A3V3
3 3
Þ 4 × 0.2 = 2 × 0.2 +0.4 × V3 Þ V3 = 1 m/s.
R ' = 2R , new radius of big drop
18. (d) 1 centipoise = 10–2 gcm–1 s–1 = 0.001 kg m–1s–1
New area = 4p4R2 = 4 × old area
19. (c) Angle of contact is acute.
Energy µ area 20. (b) Let x be the fraction of volume of object floating above
E1 = 4pR2 .....(1) the surface of the liquid.
E2 = 4.4pR2 .....(2) As weight of liquid displaced = weight of object
From equation (1) and (2) we get, E2 = 4E1 \ (V0 – x V0)d g = V0 d0 g
2s cos q hr d0 d - d 0
9. (c) h= Þsµ (1 – x)d = d0 or x = 1 - =
rrg cos q d d
mg g
s w h w r w cos q m 10 ´ 1 cos 135° 21. (d) tan q = = or q = tan–1 g/a
Þ = ´ = ´ ma a
s m cos q w h m r m cos 0° - 3.1 ´ 13.6 22. (c) If R is radius of bigger drop formed, then

.IN
10 ´ (-0.707 ) 1 4 4
= » pR 3 = 2 ´ pr 3 or R = 21/3 r
- 3.1 ´13.6 6 3 3

As v 0 µ r 2
40
= 50 cm3
AL
10. (c) Volume of ball =
0.8
v 01 R 2 (21 / 3 r ) 2
Downthrust on water = 50 g. \ = = = 22 / 3
N
Therefore reading is 650 g.
v0 r2 r2
R

4 or v 01 = v 0 ´ 2 2 / 3 = 5 ´ (4)1 / 3
æaö é
p(2P) ç ÷ pPa ù
4
23. (c) Vertical distance covered by water before striking
U

11. (d) è2ø = Q êQ Q = ú


Q' =
ëê 8hl úû
8hl 8 ground = (H – h). Time taken is, t = 2 ( H - h ) / g :
JO

12. (a) F = 6 p h r n Horizontal velocity of water coming out of hole at P,


= 6 × 3.14 × (8 ×10–5) × 0.03 ×100 = 4.52 ×10–3 dyne u = 2 gh
U

13. (a)
\ Horizontal range = ut = 2gh ´ 2(H - h) / g
ED

P PV
14. (c) K = \ DV =
DV / V K = 2 h (H - h )
P = hrg = 200 × 103 × 10 N/m2
æ rö
K = 22000 atm = 22000 × 105 N/m2 24. (c) The effective acceleration of the body g ' = çç1 - ÷÷g
V = 1m3 è r' ø

200 ´ 103 ´ 10 ´ 1
DV = = 9.1 ´ 10 - 4 m3
5
22000 ´ 10 h
15. (d) According to principle of continuity, for a streamline
flow of fluid through a tube of non-uniform cross-
Now, the depth to which the body sinks
section the rate of flow of fluid (Q) is same at every
point in the tube. æ u 2 ö 2gh gh h ´r '
i.e., Av = constant Þ A1v1 = A2v2 h' = ç ÷= = =
ç 2g' ÷ 2g ' g' r - r'
Therefore, the rate of flow of fluid is same at M and N. è ø

5 N 8hL
16. (a) Pressure at the bottom of tank P = hrg = 3 ´ 10 25. (a) Fluid resistance is given by R =
m 2 pr4
Pressure due to liquid column When two capillary tubes of same size are joined in
parallel, then equivalent fluid resistance is
P1 = 3 ´ 105 - 1 ´ 105 = 2 ´ 105
8h L 8h ´ 2L æ 8hL ö 9
R S = R1 + R 2 = + = çç 4 ÷÷ ´
and velocity of water v = 2 gh pR 4 p (2R) 4 è pR ø 8
EBD_7179
278 PHYSICS

Effective weight = (32 – 4) gf = 28 gf


P pPR 4 8 8 é pPR 4 ù
Rate of flow = = ´ = X ê as X = ú If m be the mass of second body, volume of second
RS 8hL 9 9 êë 8hL úû
m
body is
26. (d) Volume of bigger bubble = volume of 27 smaller 5
bubbles m
Now, 28 = m - Þ m = 35 g
4 4 D 5
Þ pD 3 = 27 ´ pd 3 Þ d = 33. (c) Upthrust = weight of 40 cm3 of water
3 3 3
= 40 g = down thrust on water
Initial surface energy Si = 4pD 2 s 34. (c) Pressure is proportional to depth from the free surface
and is same in all directions.
Final surface energy Sf = 27 ´ 4pd 2 s 35. (a) As temperature rises, the density decreases, height
increases. In A, the top cross-section is smaller.
D Therefore hA > hB.
DS = Sf - Si and using d =
3 36. (a) 10g is the force on water = extra wt. on other pan.
37. (b) Bernoulli’s theorem.
é D2 ù
DS = s ´ 4pê27 ´ - D2 ú 38. (a) Velocity of water from hole
ëê 9 ûú A = v1 = 2gh

.IN
2 2 Velocity of water from hole B
= 2D ´ 4p ´ s = 8psD
27. (d) For the first part of the question, remember that terminal = v 2 = 2g(H 0 - h)
AL
velocity means the acceleration experienced becomes
Time of reaching the ground from hole B
zero.
N
= t1 = 2( H 0 - h ) / g
Since a = 0 m/s2, then, SFy = Fair resistance - Fw = 0
Time of reaching the ground from hole A
R

Fair resistance = Fw mg
= t 2 = 2h / g
U

For the second part of the question, while the velocity


is higher, the acceleration is still zero. Therefore, the 39. (a) Apply Bernoulli’s theorem.
JO

Fair resistance is still equal to the skydiver’s weight. 40. (d) Use the equation of continuity and Bernoulli’s theorem.
F air resistance Case A = Fair resistance Case B 41. (b) For the same radius, terminal velocity is directly
proportional to density difference.
U

What has changed is the surface area of the skydiver.


Since pressure is P = F/A, as A decreases, the pressure 42. (b) Due to increase in velocity, pressure will be low above
ED

experienced increases. the surface of water.


PAAA = PB AB = mg 43. (b) 44. (a) 45. (b) 46. (c) 47. (d)
Since AA > AB , then PA < PB 48. (a) 49. (a) 50. (d)

28. (a) (2pr1 + 2pr2 )s = mg EXERCISE - 3


Exemplar Questions
é 8.7 8. 5 ù
ê2 p ´ 2 + 2p ´ 2 ús = 3.97 ´ 980 1. (c) When the pebble is dropped from the top of cylinder
ë û filled with viscous oil and pebble falls under gravity
with constant acceleration, but as it is dropped it enter
Þ s = 72 dyne cm-1 in oil. So dragging or viscous force is
29. (b) Mass of liquid which rises in the capillary, F = 6phrv
2s cos q where r is radius of the pebble, v is instantaneous speed,
m = (pr 2 )hr = pr 2 ´ ´r h is coefficient of viscosity.
rrg As the force is variable, hence acceleration is also
Þ mµr variable so v-t graph will not be straight line due to
30. (b) Because of surface tension. viscosity of oil. First velocity increases and then
31. (c) Brine due to its high density exerts an upthrust which becomes constant known as terminal velocity.
can balance the weight of the egg. 2. (d) In a streamline flow the velocity of fluid particles
remaines constant across any cross-sectional area, then
32
32. (b) Volume of first piece of metal = = 4 cm3 a point on the area cannot have different velocities at
8 the same time, hence two streamlines flow layers do
Upthrust = 4 gf not cross each other.
Mechanical Properties of Fluids 279

3. (b) In streamline flow, the speed of liquid of each particle 7. (b) According to Bernoulli’s theorem,
at a point in a particular cross-section is constant, 1
between two cross-section of a tube of flow because P + rv 2 = constant and Avv = constant
2
AV = constant (law of continuity).
If A is minimum, v is maximum, P is minimum.
4. (a) As given that,
8. (c) As surface area decreases so energy is released.
Diameter at 1st section (d1) = 2.5.
Energy released = 4pR2T[n 1/3 – 1]
Diameter at 2nd section (d2) = 3.75.
where R = n 1/3r
According to equation of continuity,
for cross-sections A1 and A2. 3 é1 1 ù é1 1 ù
= 4pR T ê - ú = 3VT ê - ú
A1 v1 = A2 v2 ër R û ër R û
2 9. (b) According to Bernoulli’s theorem,
v1 A2 p(r22 ) æ r2 ö
= = =ç ÷ P +12 r v2 = P0 + 0
v2 A1 p(r12 ) è r1 ø
1 2
So, DP = rv P
æ d2 ö
2 2
ç ÷ æd ö
2
= è ø2 = ç 2 ÷
2 1 2
é d2 d1 ù F = DPA = rv A P0
æ d1 ö è d1 ø êQ r2 = 2 , r1 = 2 ú 2
ë û

.IN
ç ÷
è 2ø 1
= × 1.2 × 40 × 40 × 250
2
2 = 2.4 × 105 N (upwards)
æ 3.75 ö

9
AL
÷ = 10. (a) Inflow rate of volume of the liquid = Outflow rate of
è 2.5 ø 4
volume of the liquid
\ v1 : v2 = 9 : 4
N
pR 2V VR 2
5. (c) We observed that meniscus of liquid is convex shape pR2V = npr2(v) Þ v = =
R

npr 2 nr 2
as shown in figure which is possible if only, the angle
11. (a) Water rises upto the top of capillary tube and stays
U

of contact is obtuse. Hence, the combination will be of


case of mercury-glass (140°). Hence verifies the option there without overflowing.
JO

(c). 12. (d) As we know,

Pressure P = Vdg d
U

convex r (1 – p)L
140°
ED

nr pL
mercury

Here, L A d g = (pL) A (nr)g + (1 – p)L A r g


NEET/AIPMT (2013-2017) Questions Þ d = (1 – p)r + pn r = [1 + (n – 1)p]r
6. (c) Wetability of a surface by a liquid primarily depends 13. (c) Here, h oil ´ roil ´ g = h water ´ rwater ´ g
on angle of contact between the surface and liquid. r0g × 140 × 10–3 = rwg × 130 × 10–3
If angle of contact is acute liquids wet the solid and 130
vice-versa. roil = ´ 103 » 928kg / m 3 [Q r = 1 kgm–3]
140 w
EBD_7179
280 PHYSICS

Thermal Properties
11 of Matter
TEMPERATURE It is the combination of following three laws :
The temperature of a body is that physical quantity which indicates (i) Boyle's law : When temperature is held constant, the pressure
how much hot or cold the body is? So temperature indicates the is inversely proportional to volume.
hotness or coldness of a body. 1
i.e., P µ ( at constant temperature)

.IN
The flow of heat is always from higher temperature to lower V
temperature. Temperature is a macroscopic concept. (ii) Charle's law : When the pressure is held constant, the volume
Two bodies are said to be in thermal equilibrium with each other, AL of the gas is directly porportional to the absolute temperature.
when no heat flows from one body to the other, that is when both i.e., V µ T (at constant pressure)
the bodies are at the same temperature. (iii) Avogadro's law : When the pressure and temperature are
Temperature measuring device : kept constant, the volume is directly proportional to the
N
Thermometer is a device to measure the temperature. number of moles of the ideal gas in the container.
i.e., V µ n (at constant pressure and temperature)
R

Thermometers used for measuring very high temperatures are


called pyrometers. Absolute Temperature
U

Different types of temperature scales The lowest temperature of –273.16 °C at which a gas is supposed
to have zero volume and zero pressure and at which entire
JO

molecular motion stops is called absolute zero temperature. A


Symbol Lower Upper No. of
Name of the new scale of temperature starting with –273.16°C by Lord Kelvin
for each fixed fixed divisions on
scale as zero. This is called Kelvin scale or absolute scale of temperature.
U

degree point point the scale


T(K) = t°C + 273.16
Reaumur ºR 0ºR 80ºR 80
ED

Thermometric property : It is the property used to measure the


Celsius ºC 0ºC 100ºC 100
temperature. Any physical quantity such as length of mercury (in
Fahrenheit ºF 32ºF 212ºF 180
a glass capillary), pressure of a gas (at constant volume), electrical
Rankine Ra 460Ra 672Ra 212 resistance (of a metal wire), thermo emf (thermo-couple
Kelvin K 273K 373K 100 thermometer) which changes with a change in temperature can be
used to measure temperature.
* Lower fixed point (LFP) is the freezing point of water. The unknown temperature t on a scale by utilising a general
* Upper fixed point (UFP) is the boiling point of water. property X of the substance is given by
TºK = (tºC + 273) or tºC = (TK – 273)
æ X - X0 ö
Relationship between different temperature scales t=ç t ÷ ´100º C
è X 100 - X 0 ø
R C F - 32 Ra - 460 K - 273
= = = = Keep in Memory
80 100 180 212 100
The zero of the Kelvin scale is called absolute zero. The Kelvin 1. The mercury thermometer with cylindrical bulb are more
scale is often termed as the absolute scale. In common use the sensitive than those with spherical bulb.
absolute zero corresponds to –273ºC. However, its exact value is 2. Alcohol thermometer is preferred to the mercury thermometer
–273.16º C. due to the larger value of the coefficient of cubical expansion.
3. Properties that make mercury the ideal thermometric
IDEAL GAS EQUATION AND ABSOLUTE TEMPERATURE
substance are :
The equation, PV = nRT
(i) It does not stick to the glass walls
where, n = number of moles in the sample of gas (ii) Available in pure form
R = universal gas constant; (its value is 8.31 J mol–1 K–1), is (iii) Low thermal conductivity and low specific heat
known as ideal-gas equation (iv) Vapour Pressure is low
Thermal Properties of Matter 281

(v) Coefficient of expansion is uniform


(vi) It shines

av(10 K ) ®
4. Gas thermometer is more sensitive than the mercury

–1
thermometer, because its coefficient of cubical expansion is

–5
much larger.
(For copper)
5. At the following temperatures different temperature scales
have the same reading :
(i) Fahreinheit and Reaumur at –25.6ºF = –25.6ºR 250 500
T (K)
(ii) Reaumur and Celsius at 0ºR = OºC
Keep in Memory
(iii) Celsius and Fahrenheit at –40ºC = –40ºF
(iv) Fahrenheit and Kelvin at 574.25ºF = 574.25K 1. During change in temperature, the mass does not change
6. The Celsius scale and the Kelvin cannot have the same but the density decreases due to increase in volume.
reading at any temperature. Therefore
7. Six’s thermometer measures minimum and maximum r0
r=
temperature during a day. It uses mercury as well as alcohol 1 + gDT
as the thermometric substances.
2. Relation between a, b and g
8. Normal human body temperature is 37ºC = 98.6ºF.

.IN
g = 3a and b = 2a
9. Clinical thermometers are much shorter than the laboratory
thermometers because they are used to measure a limited 3. Water has negative coefficient of volume expansion for
range (96º F to 100º F) of temperature. temperature range (0°C – 4°C). Therefore water contracts
AL
10. Rapidly changing temperature is measured by using thermo when the temperature rises from 0°C to 4°C and then expands
electric thermometer. as the temperature increases further.
Density of water reaches a maximum value of 1000 kg m–3 at
N
11. The temperature inside a motor engine is measured using
platinum resistance thermometer. 4°C. This is the anomalous behaviour of water.
R

12. The radiation thermometers can measure temperature from


U

a distance.
Volume

13. Adiabatic demagnetisation can be used to measure Volume


JO

temperature very near to the absolute zero.


14. The Rankine is the smallest temperature range among all the
U

scales of temperature.
4°C Temperature 4°C Temperature
15. When a substance is heated it expands along each
ED

dimension in the same proportion. (if it is uniform)


Thermal Expansion of Liquid (which is heated in a vessel)
THERMAL EXPANSION DVapp = V( g - 3a)DT
Thermal Expansion of Substances
Linear expansion : i.e., increase in length with increase in where DV app = apparent change in volume
temperature V= volume of the liquid initially
Dl = al 0 DT g = coefficient of volume expansion of liquid
a = coefficient of linear expansion of vessel
or l - l 0 = al0 DT Þ l = l 0 (1 + aDT )
where l0 is the initial length, l = final length, DT = change in It is clear from the formula that if g = 3a , DVapp = 0 but if g > 3a ,
temperature, a = coefficient of linear expansion. DVapp is positive and the liquid rises in the vessel and if g < 3a ,
Areal expansion : i.e., increase in area with increase in
temperature. DVapp is negative and liquid falls in vessel.
D A = b A0 D T where b = coefficient of area expansion Change in density of solids and liquids with temperature.
Volume expansion : i.e. increase in volume with increase in The density at t°C is given by
temperature. r0
rt =
(1 + gt)
DV = gV0 DT where g = coefficient of volume expansion. The
where r0 is the density at 0°C. If r1 and r2 are the densities at
units of a, b and g is (°C)–1 or K–1. temperature t1 and t2 respectively, then
Graph of av versus T : r -r
g= 1 2
r2 (t2 - t1 )
EBD_7179
282 PHYSICS

Thermal Stress Isothermal and adiabatic elasticities :


If a rod fixed at two ends is heated or cooled, then stress are dP
produced in the rod. (i) Isothermal elasticity, E iso º = P.
æ dV ö
-ç ÷
æ Dl ö è V ø
Thermal stress = Y ´ aDT ç = aDT = strain ÷
è l0 ø
dP P
Effect of Temperature on Pendulum Clock (since PV = RT = constant, =- )
dV V
A Pendulum clock consists of a metal bar attached with a bob at (ii) Adiabatic elasticity,
one end and fixed at the other end. Let pendulum clock read correct
Cp
time when its length is l0. The time period E adia =
dP
= g P ; where g =
æ dV ö Cv
l0 -ç ÷
t0 = 2π è V ø
g
(iii) E adia = g E iso
Suppose that the temperature is raised by DT, then new time period
is Expansion of Gases
There are two coefficients of expansion of gases.
l t l
t = 2p Þ = .....(i) (a) Pressure coefficient of expansion of gas (gP)
g t0 l0

.IN
Pt - P0
We have l = l 0 (1 + aDt) gP =
P0 t
l where P0 = Pressure at 0°C, Pt = Pressure at t°C
Þ = (1 + aDt) .....(ii)
AL
l0 (b) Volume coefficient of expansion of gas (gv)
From equations (i) and (ii), Vt - V0
gv =
N
1 V0 t
t
R

= (1 + aDt) 2 where V0 = Volume at 0°C, Vt = Volume at t°C


t0
U

HEAT
1 Heat is the form of energy which is transferred from one body to
= 1 + aDt
JO

(since a is very small)


2 another body due to temperature difference between two bodies.
t 1 Thermal energy of a body is the sum of kinetic energy of random
Þ - 1 = aDt
U

t0 2 motion of the molecules/atoms and the potential energy due to


the interatomic forces.
t - t0
ED

1
Þ = aDt Calorimeter is an instrument used to measure the heat.
t0 2 The SI unit of heat is joule and its dimensions are [ML2 T–2]. Other
so, fractional change in time units of heat are :
Dt 1 (i) Calorie: It is the amount of heat required to raise the
= aDT
t0 2 temperature of 1g of water from 14.5ºC to 15.5ºC.
1 calorie = 4.2 joule. Calorie is the unit of heat in CGS system.
1 (ii) Kilocalorie, 1Kcal = 1000 cal
Loss of time per day = aDT ´ 86400 second .
2 (iii) British Thermal Unit (BTU): It is the fps unit of heat
(Q no. of seconds in one day = 86400 sec.) = 252 cal = amount of heat required to raise the temperature
When temperature increases (during summer), the length of the of 1 lb of water through 1ºF (from 63°F to 64ºF).
pendulum increases due to which the time period increases and SPECIFIC HEAT CAPACITY
the clock loses time. On the other hand, when temperature It is the amount of heat required to raise the temperature of unit
decreases (during winter season), the length decreases and the mass of substance through 1 degree.
time period decrease, the clock in this case, gains time. Q
Specific heat capacity, c =
Error in scale reading : m ´ Dt
As the temperature changes, the length of the metal scale changes Its SI unit is J kg–1K–1
so is the difference in graduation CGS unit cal g–1°C–1
R a = R m (1 + aDT) It dimensions are [M 0 L 2 T –2 q –1 ]
where Ra = actual reading, Rm = measured reading For gases : During the isothermal process
Dt = 0 Þ c = ¥ (infinite)
Thermal Properties of Matter 283

and during the adiabatic process (ii) Latent heat of vaporisation : It is the quantity of heat required
Q = 0 Þ c = 0 (zero) to convert unit mass of liquid into vapour at its boiling
Specific heat of gas : The specific heat of a gas varies from zero to point.
infinity. It may have any positive or negative value. Latent heat of vaporisation of water is 536 cal/g.
Following of two specific heats of gas are more significant. Latent heat increases the intermolecular potential energy of the
(a) Molar specific heat at constant volume (CV) : It is the molecules while kinetic energy remains constant.
amount of heat required to raise the temperature of Different Processes of Phase Change or State Change
1 mole of a gas through 1K or 1°C at constant volume.
(i) Sublimation : It is the process of conversion of solids to
1 æ dQ ö gaseous state on heating. On cooling the vapours get
CV = ç
n è dT ÷ø converted back into solids.
(b) Molar specific heat at constant pressure (CP) : It is the (ii) Condensation : It is the process of conversion of gases/
amount of heat required to raise the temperature of 1 mole of vapours to liquid state on cooling. On heating, these liquids
a gas through 1K or 1°C at constant pressure. are converted into vapours/gases.
1 æ dQ ö (iii) Boiling : It is the process of conversion of liquid to gaseous
CP = ç ÷ state on heating. It is a cooling process.
n è dT ø
(iv) Melting : It is the process in which solid gets converted
Mayer’s relation : CP – CV = R

.IN
[where R is a gas constant. (R = 8.314 Jmol –1K–1)] into liquid on heating.
(v) Evaporation : Conversion of liquid into gaseous state at all
Why CP is greater than CV ?
When heat is supplied to a gas at constant volume entirely used temperatures is called evaporation. It is a phenomenon that
AL
to raise its temperature. When a gas is heated at constant pressure, occurs at the surface of the liquid.
some work is done in expansion of gas which is in addition to Melting and boiling occur at definite temperature called melting
N
raise its temperature and hence CP > CV. point and boiling point respectively. The liquid boils at a
THERMAL CAPACITY OR HEAT CAPACITY temperature, at which its vapour pressure is equal to the
R

It is the amount of heat required to raise the temperature of atmospheric pressure.


U

substance through 1ºC or 1K. How evaporation is different from boiling?


Thermal capacity = mass × specific heat. Evaporation is a slow process occurring at all the temperature at
JO

This is the relation between sp. heat capacity and heat capacity. the surface of liquid while boiling is a fast process involving whole
Its SI unit is JK–1 CGS unit cal/°C. Its dimensions are [ML2T– of the liquid heated to a particular temperature called boiling point
2q –1 ]
U

of the liquid.
Water Equivalent
Keep in Memory
ED

It is the mass of water in gram which would require the same


amount of heat to raise its temperature through 1ºC as the body 1. The heat capacity or thermal capacity depends on nature of
when heated through the same temperature. It is measured in the substance (specific heat capacity) and mass of quantity
gram. Hence of matter of the body.
Water equivalent of a body = mass of the body × specific heat
2. In case of melting and boiling, the temperature does not
Principle of Heat (or Calorimetry) change, only intermolecular potential energy of the system
When two bodies at different temperatures are placed in contact changes.
with each other then heat will flow from the body at higher
3. System are in thermal equilibrium when their temperature
temperature to the body at lower temperature until both reach to a
are same or average kinetic energy per molecule is same.
common temperature.
i.e. Heat lost by hot body = heat gained by cold body. 4. Internal energy is the sum of all energies in a system which
It follows the law of conservation of energy. includes energy of translational, rotational as well as
LATENT HEAT vibrational motion of the molecules.
5. The molar specific heat capacity depends on the molecular
It is defined as the amount of heat absorbed or given out by a
body during the change of state while its temperature remaining weight of the substance.
constant. 6. The temperature, volume or pressure of a system may remain
It is of two types : constant when it absorbs heat.
(i) Latent heat of fusion : It is the quantity of heat required to 7. When we press two block of ice together such that after
change unit mass of the solid into liquid at its melting point. releasing the pressure two block join this phenomenon is
Latent heat of fusion of ice is 80 cal/g. called regelation.
EBD_7179
284 PHYSICS

Phase Diagram Solution :


Let L0 and L25 be the length of pendulum at 0ºC and 25ºC
D
B respectively. We know that
P
Liquid L25 = L0(1 + aT) = L0(1 + 0.000019 × 25) = 1.000475 L0
4.58 mm C
A
If T25 and T0 be the time periods at 25ºC and 0ºC respectively,
of mercury P then
Solid
E Vapour
æ L 25 ö æ L0 ö
F T25 = 2p çè g ÷ø and T0 = 2p çè g ÷ø
O Ttr TC
273.16ºK T T25 æ L 25 ö æ 1.000475L0 ö
water \ T0 = çè L ÷ø = çè ÷ø
0 L 0
(i) The diagram shows vaporisation curve (AB), fusion curve
= (1.000475) = 1.000237
(CD), sublimation curve (EF).
(ii) For water, AB is called steam line, CD is called ice line and T25 - T0
EF is called hoar-frost line. Now T0 = 0.000237
(iii) The temperature at which all the three states of matter (solid, \ Gain in time for one vibration = 2 × 0.000237 sec.
liquid, gas) are mutually in thermal equilibrium is called the

.IN
triple point (point P in diagram) of the substance. 24 ´ 60 ´ 60
Number of vibration in one day = (T = 2 sec.)
(iv) The temperature upto which a gas can be liquified under 2
pressure alone is called the critical temperature (Tc).
AL Hence the gain in time in one day
Example 1. 24 ´ 60 ´ 60
= 2 × 0.000237 × = 20.52sec.
A solid material is supplied with heat at a constant rate. 2
N
The temperature of the material is changing with the heat Example 3.
input as shown in Fig. Study the graph carefully and answer A circular hole in an aluminium plate is 2.54 cm in diameter
R

the following : at 0ºC. What is the diameter when the temperature of the
(i) What is represented by horizontal regions AB & CD? plate is raised to 100ºC ? Given aA = 2.3 × 10– 5 (ºC) – 1
U

(ii) If CD = 2 AB. What do you infer? Solution :


Solution :
JO

Let D0 and Dt be diameters of hole at 0ºC and tºC


Y (100ºC liquid) E respectively.
Circumference of hole at 0ºC
U

Temp. (0ºC liquid)


C D
(100ºC Vapour) l0 = 2pr0 = pD0
ED

0ºC(ice) Circumference of hole at t = 100ºC


B lt = 2prt = pDt
A
From relation lt = l0 (1 + a.t), we get
Q1 Q2 Q3 Q4 pDt = pD0 (1 + 2.3 × 10–5 × 100)
X
O Heat Supplied Dt = 2.54 (1 + 0.0023) = 2.5458 cm
Example 4.
(i)
In the horizontal regions AB and CD heat is supplied
If the volume of a block of a metal changes by 0.12% when
but temp. remains constant. Therefore, they represent
it is heated through 20ºC, what is the coefficient of linear
change of state. The graph AB represents change from
expansion of metal ?
solid state to liquid state, at constant temp. = melting
Solution :
point of solid.
Coefficient of cubical expansion of metal is given by
AB = latent heat of fusion.
Again CD represents change of state from liquid to DV
vapour at boiling point of the liquid. g=
Vt
CD = latent heat of vaporisation.
DV 0.12
(ii) As CD = 2 AB Here g = = , t = 20ºC
\ Latent heat of vaporisation is twice the latent heat Vt 100t
of fusion. 0.12
Example 2. \ g= = 6.0 × 10–5 per ºC
100 ´ 20
A clock which keeps correct time at 25ºC has a pendulum
Coefficient of linear expansion
made of brass whose coefficient of linear expansion is
0.000019. How many seconds a day will it gain if the g 6.0 ´ 10-5
a= = = 2.0 × 10–5 Per ºC
temperature fall to 0ºC? 3 3
Thermal Properties of Matter 285

Example 5. Dew Point :


From what height must a block of ice be dropped into a It is the temperature at which the amount of water vapours
well of water so that 5% of it may melt? Given : both ice actually present in a certain volume of the air which is sufficient
and water are at 0°C, L = 80 cal g–1, J = 4.2 J cal–1 and to saturate that volume of air.
g = 980 cm s–2. At the dew point the actual vapour pressure becomes the
saturated vapour pressure.
Solution :
It means that vapour pressure at room temperature = saturated
Let m be the mass of ice. Let h be the height from which vapour pressure(S.V.P) at dew point
block of ice is dropped. Work done, If room teperature = dew point
W = mgh = m × 980 × h erg then Relative humidity (R.H.) = 100%
5 Keep in Memory
Mass of ice to be melted = ´m
10 0
1. Hygrometry is the branch of thermal physics that deals
5 with the measurement of amount of water vapours present
Heat required, Q =
100
× m × 80 cal or Q = 4 m cal

[Q L = 80 cal g–1] [Q W = JQ] 2.


11.1
in the atmosphere.
Hygrometer is an instrument used to determine dew point
and relative humidity.
Now, m × 980 × h = J × 4 m 3. Relative humidity is low when the air is dry.
7
4.2 ´ 10 ´ 4m 4. Saturated vapours do not obey (except Dalton’s law) gas

.IN
h= cm = 1714.3 m laws, while unsaturated vapours obey them. The S.V.P.
m ´ 980
varies linearly with temperature (keeping volume constant)
[Q J = 4.2 × 107 erg cal–1]
and equal to one atmospheric pressure at boiling
AL point.(shown in fig.(a)) S.V.P. constant at constant
SATURATED AND UNSATURATED AIR
The air is said to be saturated when the maximum possible amount temperature on increasing or decreasing volume fig. (b).
P P
N
of water vapours are present in it. The pressure of the water
1 atm V = constant
vapours in the saturated air is called saturation vapour pressure.
T = constant
R

If the air contains vapours less than the maximum possible amount (S.V.P.) (S.V.P.)
possible in the air, then it (air) is said to be unsaturated.
U

Boiling point
Vapour Pressure
T V
JO

A vapour differs from a gas in that the former can be liqufied by (a) (b)
pressure alone, wheras the latter cannot be liqufied unless it is 5. At 0ºC, the saturation vapour pessure is 4.6 mm of Hg.
first cooled. Therefore, water vapours are always present around the
U

If a liquid is kept in a closed vessel, the space above the liquid ice. It increases with the increase in temperature.
becomes saturated with vapour. The pressure exerted by this 6. The relative humidity is 100% when the temperature of the
ED

vapour is called saturated vapour pressure. Its value depends atmosphere is equal to the dew point.
only on temperature. It is independent of any external pressure. 7. Cryogenics is the study of low temperature.
8. As RH is expressed in percentage, hence the maximum
Humidity
value of RH may be 100%.
The humidity shows the presence of water vapours in the 9. The saturation vapour pressure of water at 100ºC is 760
atmosphere. The amount of water vapours present per unit volume mm of Hg at sea level.
of the air is called absolute humidity. 10. As the temperature rises, the absolute humidity may
Relative humidity (RH) is defined as the ratio of the mass of increase and the relative humidity may remain constant or
water vapour (m) actually present in the given volume of air to may even decrease.
the mass of water vapour (M) required to saturate the same 11. If the relative humidity is high then at the same temperatue
volume at the same temperature. Normally RH is expressed in one feels hotter.
percentage 12. If the air is absolutley dry, the dew point is not observed.
m 13. The relative humidity decreases with the increase in
i.e., % RH = ´100%
M temperature.
RH is also defined as the ratio of actual pressure (p) of water 14. If water is sprinkled in the room, both relative humidity as
vapour to the saturated vapour pressure (P) of the water at the well as the dew point increase.
same temperature 15. Dew appears on the leaves of the trees etc. due to the
p condensation of saturated vapours.
i.e., % RH = ×100%
P 16. The mist or fog is the small droplets of water formed due to
Also, the condensation of water vapours near the surface of
Saturated vapour pressure of water at dew point earth.
RH = 17. Mist or fog formed much higher above the surface of earth
Saturated vapour pressure of water at room temp erature
is called cloud.
EBD_7179
286 PHYSICS

18. The Relative humidity between 50% and 60% gives (b) Convection : It is the process by which heat is transferred
comfortable feeling to human beings. from one place to another in a medium by the movement of
19. When the relative humidity is 100%, the reading of the dry particles of the medium. It occurs due to density difference.
and wet bulbs is same. This phenomenon occurs in fluids.
20. Smaller the difference in the temperature of dry and wet
(c) Radiation : It is the process by which heat is transferred
bulbs, larger is the relative humidity.
from one place to another without any intervening medium.
21. Water vapours are always seen around the ice, because the
temperature around it is less than the dew point. The light reaches from Sun to Earth by radiation process.
Keep in Memory
HEAT TRANSFER
Three modes of transmission of heat :
1. The equation
ΔQ æ ΔT ö is valid for steady state
(a) Conduction (b) Convection and (c) Radiation = KA ç ÷
(a) Conduction : In thermal conduction particles of body at Δt è Δx ø
higher temperature transmit heat to the particles at lower condition. The condition is said to occur when no part of
temperature by mutual contact (collision) only and not by heat is used up in raising the temperature of any part of
the movement of the particles. All solids are heated by cross-section of the solid.
conduction. Conduction cannot take place in vacuum. 2. On comparing equation (1) with the following equation used
T1 > T2 for flowing of charge on account of potential difference.
Dx V1 V2

.IN
Dq s
A DQ
Dt
l
DQ T1
Dq DV
Dt T2
AL
We find : = sA
Dt l
Dx
Figure shows a solid of cross-section area A and thickness The role of resistance (thermal resistance) is played by
N
KA
Dx. The face are at different temperature T1 and T2 (T1 > T2) 3. Series combination of conductors
æ DQ ö
R

The rate of heat flow ç ÷ as found experimentally is T1 T T2


è Dt ø
given by
U

K1, A1 K2, A2
DQ æ DT ö
JO

µ Aç ÷ where DT = T1 - T2 l1 l2
Dt è Dx ø
Equivalent thermal conductivity :
ΔQ æ ΔT ö
U

Þ = KA ç ÷ …(1) 2 K1 K 2
Δt è Δx ø Keq =
K1 + K 2
ED

where K is proportionality constant called thermal


conductivity. A(T1 – T2 )
H=
It is a measure of how quickly heat energy can conduct l1 l
through the substance. + 2
K1 K 2
Coefficient of thermal conductivity (K) : The coefficient of thermal
conductivity, K, of a material is defined as the amount of heat that where H = heat flow per second
flowing per second through a rod of that material having unit 4. Parallel combination of conductors
length and unit area of cross-section in the steady state, when T1 T2
the difference of temperature between two ends of the rod is 1 ºC
and flow of heat is perpendicular to the end faces of the rod. K1, A1
Unit of coefficient of thermal conductivity in SI system is
watt/m-K K2, A2
Dimensions : [M L T–3 q–1] l
For a perfect conductor thermal conductivity K is infinite and
for a perfect insulator K is zero. Equivalent thermal conductivity :
In general, solids are better conductors than liquids and liquids K1 A1 + K 2 A2
are better conductors than gases. (Heat transfer through the Keq =
process of conduction is possible in liquids and gases also, if A2 + A2
they are heated from the top.) Metals are much better conductors
than non-metals. K1 K
T1 + 2 T2
A good conductor of heat is also a good conductor of electricity l1 l2
H=
The conduction of both heat and electricity is due to the movement K1 K 2
+
of free electrons. l1 l 2
Thermal Properties of Matter 287

5. Davy’s safety lamp is based on the conduction. It is used in


C C 2C 2 K1 K 2
mines to know the ignition temperature of gases. Danger of or + = or K =
explosion can be avoided. K1 K 2 K K1 + K 2
6. (i) Principle of chimney used in a kitchen or a factory is Example 8.
based on the convection. Two walls of thickness d1 and d2 and thermal conductivities
(ii) Land and sea breezes are due ot the convection. K1 and K 2 are in contact. In the steady state, if the
(iii) Temperature of the upper part of the flame is more than temperatures at the outer surfaces are T1 and T2 then find
the temperature on the sides, because the currents of the temperature at the common wall.
air carry the heat upwards. Solution :
(iv) Radiation can be detected by differential air T1 T T2
thermometer, Bolometer, thermopile, etc. K1 K2
Example 6. d1 d2
Two plates of same area are placed in contact. Their
Let the temperature of common wall be T. Then
thickness as well as thermal conductivities are in the ratio
2 : 3. The outer surface of one plate is maintained at 10ºC K1 A (T1 - T )t K 2 A (T - T2 ) t
and that of other at 0ºC. What is the temp. of the common =
d1 d2
surface?
Solution : K1 (T1 - T) t K 2 (T - T2 ) t
Let q be the temp. of common surface. or, =
d1 d2

.IN
æ DQö æ DQ ö
As çç D t ÷÷ = çç D t ÷÷
or, d 2 [ K1 ( T1 - T )] = d1 [ K 2 (T - T2 )]
è ø1 è ø2 AL or, K1T1d 2 - K1d 2 T = d1K 2 T - d1K 2 T2
(10 - q) ( q - 0) K1 x
K1 A1 = K2 A2 or (10 - θ) = 1 θ or, K1T1d 2 - K 2 d1T2 = T[K1d 2 + d1K 2 ]
x1 x2 K2 x2
é K T d - K 2 d1 T2 ù
N
Plate 1 Plate 2 \ T=ê 1 1 2 ú
ë K 1 d 2 + K 2 d1 û
R

K1 K2
10°C 8°C 0°C HEAT TRANSFER BY RADIATION AND NEWTON’S LAW
U

x1 x2 OF COOLING
JO

Now, Stefan’s Law:


x K 2 2 2 This law is also called Stefan Boltzmann law. This law states that
A1 = A 2 ; 1 = 1 = ; (10 - q) = q or q = 5º C.
x2 K2 3 3 3 the power radiated for overall wavelength from a black body is
U

Example 7. proportional to the fourth power of thermodynamic temperature T.


ED

A slab consists of two parallel layers of two different


materials of same thickness and thermal conductivities E µ T 4 Þ E = sT 4 for perfectly black body ....(1)
K1 and K2. Find the equivalent thermal conductivity of where E is energy emitted per second from unit sruface area of the
the slab. black body, T is temperature and s is Stefan’s constant, and
Solution : s = 5.67×10–8 Wm–2 K–4.
Rate of flow of heat through each layer of slab is same, If body is not perfectly black body, then
because slabs are in series. E = es T4 ....(2)
where e is emissivity. Then energy radiated per second by a body
of area A is
DT2 x E1 = es T4 ....(3)
If the body temperature is T and surrounding temperature is T 0,
DT1 x then net rate of loss of energy by body through radiation from
equation (3) is
E1' = eAs(T 4 - T04 ) ....(4)
DT D T2 K (D T1 + D T2 ) A
K1 A 1 = K 2 A = Now rate of loss of energy in terms of specific heat c is
l l 2l
dQ dT
K (D T1 + D T2 ) = mc = E1' ....(5)
or K1 D T1 = K 2 D T2 = = C , say dt dt
2
and dT = eAs (T 4 - T 4 ) ....(6)
C C 2C 0
\ D T1 = , D T2 = , D T1 + D T2 = dt mc
K1 K2 K (T- temperature, t - time in second)
EBD_7179
288 PHYSICS

So body cools by radiation and rate of cooling depends on


dT
e (emissivity of body), A (area of cross-section of body), i.e. = - K (T - T0 )
m (mass of body) and c (specific heat capacity of body). dt
A black body is defined as a body which completely absorbs all where T = temperature of body, T 0 = temperature of surrounding.
the heat radiation falling on it without reflecting and eAs
transmitting any of it. and K = where, e = emissivity of body, A = area of surface
mc
Weins Law : of body, s = Stefan’s constant, m = mass of body, c = specific heat
lmT = b, of body.
where b is the Weins constant and b = 2.898 × 10 –3mK It is a special case of Stefan’s law and above relation is applicable
and T = temperature. only when the temperature of body is not much different from the
Graph of lm versus T temperature of surroundings.
Solar constant : Solar constant is the solar radiation incident
normally per second on one square metre at the mean distance
of the earth from the sun in free space.
l m1 T2 It is given by S = 1.34 × 109 Jm–2s–1.
=
l m2 T1 Temperature of sun is given T4 = S/s(R/r)2, where R is mean
distance of the earth from the sun and r is the radius of the sun.
T Keep in Memory
Hence on increasing temperature of a body, its colour changes 1. Good absorber is a good emitter.

.IN
gradually from red to orange ® yellow ® green ® blue ® 2. Cooking utensils are provided with wooden or ebonite
violet. Thus the temperature of violet star is maximum and
handles, since wood or ebonite is a bad conductor of heat.
temperature of red star is minimum. Sun is a medium category star
3. Good conductor of heat are good conductors of electricity,
with lm = 4753Aº (yellow colour) and temperature 6000K.
AL Mica is an exception which being a good conductor of heat
Kirchoff’s Law is a bad conductor of electricity.
According to this law, the ratio of emissive power to absorptive
N
Example 9.
power is same for all surfaces at the same temperature and is Compare the rate of loss of heat from a metal sphere of
R

equal to the emissive power of a perfectly black body at the same temperature 827°C, with the rate of loss of heat from the
temperature. same sphere at 427 °C, if the temperature of surroundings
U

e is 27°C.
i.e. E = A
a
JO

where e = emissive power of a given surface Solution :


Given : T1 = 827 °C = 1100 K, T2 = 427 °C = 700 K
a = absorptive power of a given surface
and T0 = 27 °C = 300 K
E = emissive power of a perfect black body
U

According to Steafan Boltzmann law of radiation,


A = absorptive power of a perfect black body
dQ
ED

e = s Ae (T4 – T04)
(i) For a perfect black body, A = 1, \ E = dt
a
(ii) If emissive and absorptive power are considered for a
e æ dQ ö
particular wavelenth l then E l = l çè ÷ø
(T14 - T04 )
al dt 1 [(1100)4 - (300)4 ]
(iii) Since El is constant at a given temperature, according to \ = = = 6.276
æ dQ ö (T24 - T04 ) [(700)4 - (300)4 ]
this law if a surface is good absorber of a particular çè ÷ø
dt 2
wavelength then it is also a good emitter of that wavelength. æ dQ ö æ dQ ö
Fraunhoffer's Line or çè dt ÷ø : èç dt ø÷ = 6.276 :1
(i) Fraunhoffer lines are the dark lines in the spectrum of sun 1 2
and are explained on the basis of Kirchoff’s law. When white Example 10.
light emitted from central core of sun (photosphere) passes The filament of an evacuated light bulb has a length 10 cm,
through its atmosphere (chromosphere) radiation of those diameter 0.2 mm and emissivity 0.2, calculate the power it
wavelength will be absorbed by the gases present there, radiates at 2000 K. (s = 5.67 × 10–8 W/m2 K4)
which they usually emit (as good emitter is a good absorber) Solution :
resulting dark lines in the spectrum of sun. l = 10 cm = 0.1 m, d = 0.2 mm = 2 × 10–4 m
(ii) On the basis of Kirchoff’s law, Fraunhoffer identified some r = 0.1 mm = 1 × 10–4 m,
of the elements present in the chromosphere. They are e = 0.2, T = 2000 K, s = 5.67 × 10–8 W/m2 K4
hydrogen, helium, sodium, iron, calcium, etc. Fraunhoffer According to stefan's law of radiation, rate of emission of
had observed about 600 darklines in the spectrum of sun. heat for an ordinary body,
Newton's Law of Cooling E = sAeT4 = s(2 p r l) eT4 [Q A = 2prl]
The rate of loss of heat of a body is directly proportional to the = 5.67 × 10–8 × 2 × 3.14 × 1 × 10–4 × 0.1 × 0.2 × (2000)4
temperature difference between the body and the surroundings. = 11.4 W
\ Power radiated by the filament = 11.4 W
Thermal Properties of Matter 289

.IN
AL
N
R
U
JO
U
ED
EBD_7179
290 PHYSICS

1. The coefficient of thermal conductivity depends upon 10. A black body rediates energy at the rate of E watt per metre2
(a) temperature difference between the two surfaces. at a high temperature T K. when the temperature is reduced
(b) area of the plate to (T/2) K, the radiant energy will be
(c) material of the plate (a) E/16 (b) E/4
(c) E/2 (d) 2E
(d) All of these
11. The tempearture of an isolated black body falls from T1 to
2. The wavelength of radiation emitted by a body depends T2 in time t, then t is (Let c be a constant)
upon
(a) the nature of its surface æ 1 1ö æ 1 1 ö
(a) t = cç - (b) t = cç 2 - 2 ÷
(b) the area of its surface è T2 T1 ÷ø è T2 T1 ø
(c) the temperature of its surface
(d) All of the above æ 1 1 ö æ 1 1 ö
3. A vessel completely filled with a liquid is heated. If a and g (c) t = cç 3 - 3 ÷ (d) t = cç 4 - 4 ÷
è T2 T1 ø è T2 T1 ø

.IN
represent coefficient of linear expansion of material of vessel
and coefficient of cubical expansion of liquid respectively, 12. Two straight metallic strips each of thickness t and length l
then the liquid will not overflow if are rivetted together. Their coefficients of linear expansions
are a1 and a2 . If they are heated through temperature DT,
(a) g = 3 a (b) g > 3 a
AL
(c) g < 3 a (d) g £ 3 a the bimetallic strip will bend to form an arc of radius
4. The fastest mode of transfer of heat is (a) t /{α1 + α 2 ) ΔT} (b) t /{( α 2 - α1 ) ΔT}
N
(a) conduction (b) convection (c) t (α1 - α 2 ) ΔT (d) t (α 2 - α1 ) ΔT
R

(c) radiation (d) None of these


13. Two solid spheres, of radii R1 and R2 are made of the same
5. In order that the heat flows from one part of a solid to material and have similar surfaces. The spheres are raised to
U

another part, what is required? the same temperature and then allowed to cool under
(a) Uniform density
JO

identical conditions. Assuming spheres to be perfect


(b) Temperature gradient conductors of heat, their initial rates of loss of heat are
(c) Density gradient
(a) R12 / R22 (b) R1 / R 2
U

(d) Uniform temperature


6. The sprinkling of water slightly reduces the temperature (c) R 2 / R1 (d)R22 / R12
ED

of a closed room because 14. A radiation of energy E falls normally on a perfectly


(a) temperature of water is less than that of the room reflecting surface. The momentum transferred to the surface
(b) specific heat of water is high is
(c) water has large latent heat of vaporisation (a) Ec (b) 2E/c
(d) water is a bad conductor of heat
7. Which of the following is not close to a black body? (c) E/c (d) E/c 2
(a) Black board paint (b) Green leaves 15. Which of the following graph correctly represents the
(c) Black holes (d) Red roses relation between ln E and ln T where E is the amount of
8. According to Newton’s law of cooling, the rate of cooling of radiation emitted per unit time from unit area of body and T
a body is proportional to (Dq)n, where Dq is the difference of is the absolute temperature
the temperature of the body and the surroundings, and n is
equal to
(a) two (b) three (a) (b)
(c) four (d) one ln E
ln E
9. A metallic rod l cm long, A square cm in cross-section is ln T
ln T
heated through tºC. If Young’s modulus of elasticity of the
metal is E and the mean coefficient of linear expansion is a per
degree celsius, then the compressional force required to (c) (d)
prevent the rod from expanding along its length is
ln E ln E
(a) E A a t (b) E A a t/(1 + at) ln T
ln T
(c) E A a t/(1 – a t) (d) E l a t
Thermal Properties of Matter 291

16. For the construction of a thermometer, one of the essential 22 The rate of heat flow through the cross-section of the rod
requirements is a thermometric substance which shown in figure is (T2 > T1 and thermal conductivity of the
(a) remains liquid over the entire range of temperatures to material of the rod is K)
be meaured .
(b) has property that varies linearly with temperature r1 r2
(c) has a property that varies with temperature L
(d) obey Boyle's law
T1 T2
17. The temperature of stars is determined by
(a) Stefan’s law Kpr1r2 (T2 - T1 ) Kp(r1 + r2 )2 (T2 - T1 )
(b) Wien’s displacement law (a) (b)
L 4L
(c) Kirchhoff’s law
(d) Ohm’s law Kp(r1 + r1 ) 2 (T2 - T1 ) Kp(r1 + r1 ) 2 (T2 - T1 )
18. The temperature of the Sun is measured with (c) (d)
L 2L
(a) platinum thermometer
23. When a body is heated, which colour corresponds to high
(b) gas thermometer temperature
(c) pyrometer (a) red (b) yellow
(d) vapour pressure thermometer. (c) white (d) orange
19. Which of the following circular rods, (given radius r and

.IN
24. The two ends of a rod of length L and a uniform cross-
length l) each made of the same material and whose ends sectional area A are kept at two temperatures T1 and T2
are maintained at the same temperature will conduct most dQ
heat? (T1 > T2). The rate of heat transfer, through the rod in
AL dt
(a) r = 2r0; l = 2l0 (b) r = 2r0; l = l0 a steady state is given by
(c) r = r0; l = 2l0 (d) r = r0; l = l0
dQ k (T1 - T2 ) dQ
N
20. At a certain temperature for given wave length , the ratio of (a) = (b) = kLA (T1 - T2 )
dt LA dt
emissive power of a body to emissive power of black body
R

in same circumstances is known as dQ kA (T1 - T2 ) dQ kL (T1 - T2 )


(c) = (d) =
(a) relative emissivity (b) emissivity
U

dt L dt A
(c) absorption coefficient (d) coefficient of reflection 25. Two rods of the same length and areas of cross-section A1
JO

21. If a liquid is heated in weightlessness, the heat is transmitted and A2 have their ends at the same temperature. K1 and K2
through are the thremal conductivities of the two rods. The rate of
(a) conduction flow of heat is same in both rods if
U

(b) convection A1 A1
K1 K2
(c) radiation = =
ED

(a) (b)
A2 K2 A2 K1
(d) None of these, because the liquid cannot be heated in
weightlessness. (c) A1A2 = K1K2 (d) A1 K12 = A2 K22

1. The resistance of a resistance thermometer has values 2.71 4. If a bar is made of copper whose coefficient of linear
and 3.70 ohms at 10ºC and 100ºC respectively. The expansion is one and a half times that of iron, the ratio of
temperature at which the resistance is 3.26 ohm is force developed in the copper bar to the iron bar of identical
(a) 40ºC (b) 50ºC lengths and cross-sections, when heated through the same
(c) 60ºC (d) 70ºC temperature range (Young’s modulus of copper may be taken
2. The temperature of an iron block is 140ºF. Its temperature to be equal to that of iron) is
on the Celsius scale is (a) 3/2 (b) 2/3
(a) 60º (b) 160º
(c) 140º (d) 132º (c) 9/4 (d) 4/9
3. A pendulum clock is 5 seconds fast at temperature of 15ºC 5. A metallic bar is heated from 0ºC to 100ºC. The coeficient of
and 10 seconds slow at a temperature of 30ºC. At what linear expansion is 10–5 K–1. What will be the percentage
temperature does it give the correct time? (take time interval = increase in length?
24 hours) (a) 0.01% (b) 0.1%
(a) 18ºC (b) 20ºC (c) 1% (d) 10%
(c) 22ºC (d) 25ºC
EBD_7179
292 PHYSICS

6. Steam is passed into 22 gm of water at 20ºC. The mass of 14. A mountain climber finds that water boils at 80ºC. The
water that will be present when the water acquires a temperature of this boiling water is...... Fahrenheit
temperatue of 90ºC (Latent heat of steam is 540 cal/g) is (a) 50º (b) 150º
(a) 24.83 gm (b) 24 gm (c) 176º (d) 200º
(c) 36.6 gm (d) 30 gm 15. A metal cube of length 10.0 mm at 0°C (273K) is heated to
7. A cylindrical rod of aluminium is of length 20 cms and radius 200°C (473K). Given : its coefficient of linear expansion is
2 cms. The two ends are maintained at temperatures of 0ºC 2 × 10–5 K–1. The percent change of its volume is
and 50ºC the coefficient of thermal conductivity is (a) 0.1 (b) 0.2
0.5 cal (c) 0.4 (d) 1.2
. Then the thermal resistance of the rod in
cm×sec × ºC 16. No other thermometer is suitable as a platinum resistance
cal thermometer to measure temperatures in the entire range of
is
sec × ºC (a) –50ºC to + 350ºC (b) –200ºC to + 600ºC
(a) 318 (b) 31.8 (c) 0ºC to 100ºC (b) 100ºC to 1500ºC
(c) 3.18 (d) 0.318 17. Calculate the surface temperature of the planet, if the energy
8. A metal ball of surface area 200 square cm, temperature 527ºC radiated by unit area in unit time is 5.67 ×104 watt.
is surrounded by a vessel at 27ºC. If the emissivity of the (a) 1273°C (b) 1000°C
metal is 0.4, then the rate of loss of heat from the ball is (c) 727°C (d) 727K

.IN
joule 18. In a thermocouple, the temperature of the cold junction and
approximately s = 5.67 ´ 10 -8
2 2 the neutral temperature are –40°C and 275°C respectively. If
m ´ sec´ K
the cold junction temperature is increased by 60°C, the
(a) 108 joule (b) 168 joule
AL
neutral temperature and temperature of inversion
(c) 182 joule (d) 192 joule respectively become
9. The rate of radiation of a black body at 0ºC is E joule per sec.
N
(a) 275°C, 530°C (b) 355°C, 530°C
Then the rate of radiation of this black body at 273ºc will be
(c) 375°C, 590°C (d) 355°C, 590°C
R

(a) 16 E (b) 8 E
19. In a surrounding medium of temperature 10°C, a body takes
(c) 4 E (d) E
U

7 min for a fall of temperature from 60°C to 40°C. In what time


10. Solar radiation emitted by sun resembles that emitted by a the temperature of the body will fall from 40°C to 28°C?
JO

black body at a temperature of 6000 K. Maximum intensity is


(a) 7 min (b) 11 min
emitted at wavelength of about 4800 Å. If the sun were to
cool down from 6000 K to 3000 K, then the peak intensity (c) 14 min (d) 21 min
U

would occur at a wavelength 20. Two rods P and Q of same length and same diameter having
(a) 4800 Å (b) 9600 Å thermal conductivity ratio 2 : 3 joined end to end. If
ED

temperature at one end of P is 100°C and at one and of Q


(c) 2400 Å (d) 19200 Å
0°C, then the temperature of the interface is
11. A bucket full of hot water is kept in a room and it cools from
(a) 40°C (b) 50°C
75ºC to 70ºC in T1 minutes, from 70ºC to 65ºC in T2 minutes
and from 65ºC to 60ºC in T3 minutes. Then (c) 60°C (d) 70°C
(a) T1 = T2 = T3 (b) T1 < T2 < T3 21. 100 g of ice is mixed with 100 g of water at 100ºC. What will
(c) T1 > T2 > T3 (d) T1 < T2 > T3 be the final temperature of the mixture ?
12. The maximum energy in the thermal radiation from a hot (a) 13.33ºC (b) 20ºC
source occurs at a wavelength of 11 ×10–5 cm. According (c) 23.33ºC (d) 40ºC
to Wien’s law, the temperature of this source (on Kelvin 22. A crystal has a coefficient of expansion 13×10–7 in one
scale) will be n times the temperature of another source (on direction and 231 × 10–7 in every direction at right angles to
Kelvin scale) for which the wavelength at maximum energy it. Then the cubical coefficient of expansion is
is 5.5 × 10–5 cm. The value n is (a) 462 × 10–7 (b) 244 × 10–7
(a) 2 (b) 4 (c) 475 × 10 –7 (d) 257 × 10–7
(c) 1/2 (d) 1 23. A glass flask of volume 1 litre is fully filled with mercury at
13. The temperature of a furnace is 2324ºC and the intensity is 0ºC. Both the flask and mercury are now heated to 100ºC. If
maximum in its radiation spectrum nearly at 12000 Å. If the the coefficient of volume expansion of mercury is 1.82 × 10–
intensity in the spectrum of a star is maximum nearly at 4/ºC, volume coefficient of linear expansion of glass is 10 ×
4800Å, then the surface temperature of the star is 10–6/ºC, the amount of mercury which is spilted out is
(a) 8400ºC (b) 7200ºC (a) 15.2 ml (b) 17.2 ml
(c) 6492.5ºK (d) 5900ºC (c) 19.2 ml (d) 21.2 ml
Thermal Properties of Matter 293

24. A rectangular block is heated from 0ºC to 100ºC. The (a) 4 : 7 (b) 7 : 4
percentage increase in its length is 0.10%. What will be the (c) 25 : 16 (d) 16 : 25
percentage increase in its volume?
33. A partition wall has two layers A and B, in contact, each made
(a) 0.03% (b) 0.10%
of a different material. They have the same thickness but the
(c) 0.30% (d) None of these thermal conductivity of layer A is twice that of layer B. If the
25. The coefficient of apparent expansion of mercury in a glass steady state temperature difference across the wall is 60 K,
vessel is 153 × 10–6/ºC and in a steel vessel is 144 × 10–6/ºC. then the corresponding difference across the layer A is
If a for steel is 12 × 10–6/ºC, then, that of glass is
(a) 10 K (b) 20 K
(a) 9 × 10–6/ºC (b) 6 × 10–6/ºC
(c) 30 K (d) 40 K
(c) 36 × 10–6/ºC (d) 27 × 10–6/ºC
34. The spectral energy distribution of the sun (temperature
26. A beaker contains 200 gm of water. The heat capacity of the
beaker is equal to that of 20 gm of water. The initial 6050 K) is maximum at 4753 Å. The temperature of a star for
temperatue of water in the beaker is 20ºC. If 440 gm of hot which this maximum is at 9506 Å is
water at 92ºC is poured in it, the final temperature, neglecting (a) 6050 K (b) 3025 K
radiation loss, will be nearest to (c) 12100 K (d) 24200 K
(a) 58ºC (b) 68ºC 35. Two bodies A and B are placed in an evacuated vessel
(c) 73ºC (d) 78ºC maintained at a temperature of 27ºC. The temperature of A is

.IN
27. Steam at 100ºC is passed into 1.1 kg of water contained in a 327ºC and that of B is 227ºC. The ratio of heat loss from A
calorimeter of water equivalent 0.02 kg at 15ºC till the and B is about
temperature of the calorimeter and its contents rises to
80ºC.The mass of the steam condensed in kg is
AL (a) 2 : 1 (b) 1 : 2
(c) 4 : 1 (d) 1 : 4
(a) 0.130 (b) 0.065
36. The thermal capacity of 40g of aluminium (specific heat =
N
(c) 0.260 (d) 0.135
0.2 cal g–1 °C–1) is
28. A body of mass 5 kg falls from a height of 20 metres on the
R

ground and it rebounds to a height of 0.2 m. If the loss in (a) 40 cal °C–1 (b) 160 cal °C–1
U

potential energy is used up by the body, then what will be the (c) 200 cal °C–1 (d) 8 cal °C–1
temperature rise? 37. Which of the following temperatures is the highest?
JO

(specific heat of material = 0.09 cal gm–1 ºC–1)


(a) 100K (b) –13°F
(a) 0ºC (b) 4ºC
(c) –20°C (d) –23°C.
U

(c) 8ºC (d) None of these


38. 5 kg of water at 10°C is added to 10 kg of water at 40°C.
29. 80 g of water at 30ºC are poured on a large block of ice at 0ºC.
ED

Neglecting heat capacity of vessel and other losses, the


The mass of ice that melts is
equilibrium temperature will be
(a) 1600 g (b) 30 g
(a) 30°C (b) 25°C
(c) 150 g (d) 80 g
(c) 35°C (d) 33°C
30. Ice starts forming in a lake with water at 0ºC when the
atomspheric temperature is –10ºC. If the time taken for the 39. A beaker contains 200 g of water. The heat capacity of beaker
first 1 cm of ice to be formed is 7 hours, then the time taken is equal to that 20 g of water. The initial temperature of water
for the thickness of ice to change from 1 cm to 2 cm is in the beaker is 20°C . If 440 g of hot water at 92°C is poured
(a) 7 hours (b) 14 hours in, the final temperature, neglecting radiation loss, will be
(c) 21 hours (d) 3.5 hours (a) 58°C (b) 68°C
31. Two identical rods of copper and iron are coated with wax (c) 73°C (d) 78°C
uniformly. When one end of each is kept at temperature of 40. The rectangular surface of area 8 cm × 4 cm of a black body
boiling water, the length upto which wax melts are 8.4 cm at temperature 127°C emits energy E per second. If the length
amd 4.2 cm, respectively. If thermal conductivity of copper
and breadth are reduced to half of the initial value and the
is 0.92, then thermal conductivity of iron is
temperature is raised to 327°C, the rate of emission of energy
(a) 0.23 (b) 0.46 becomes
(c) 0.115 (d) 0.69
3 81
32. Two vessels of different materials are similar in size in every (a) E (b) E
8 16
respect. The same quantity of ice filled in them gets melted
in 20 min and 35 min, respectivley. The ratio of coefficients 9 81
of thermal conduction of the metals is (c) E (d) E
16 64
EBD_7179
294 PHYSICS

41. Two rods of same length and transfer a given amount of 46. A black body at 227°C radiates heat at the rate of 7 cals/
heat 12 second, when they are joined as shown in figure (i). cm2s. At a temperature of 727°C, the rate of heat radiated in
But when they are joined as shwon in figure (ii), then they the same units will be
will transfer same heat in same conditions in (a) 50 (b) 112 (c) 80 (d) 60
47. The top of an insulated cylindrical container is covered by
a disc having emissivity 0.6 and conductivity 0.167
WK–1m–1 and thickness 1 cm. The temperature is maintained
l by circulating oil as shown in figure. Find the radiation loss
Fig. (i) to the surrounding in Jm–2s–1 if temperature of the upper
surface of the disc is 27°C and temperature of the
surrounding is 27°C.
l l
Fig. (ii)
(a) 24 s (b) 13 s Oil out
(c) 15 s (d) 48 s
42. A slab consists of two parallel layers of copper and brass of
the same thickness and having thermal conductivities in Oil in

.IN
the ratio 1 : 4. If the free face of brass is at 100°C and that of
(a) 595 Jm–2s–1 (b) 545 Jm–2s–1
copper at 0°C, the temperature of interface is
(c) 495 Jm s –2 –1 (d) None of these
(a) 80°C (b) 20°C AL
48. Two marks on a glass rod 10 cm apart are found to increase
(c) 60°C (d) 40°C their distance by 0.08 mm when the rod is heated from 0°C to
43. 540 g of ice at 0°C is mixed with 540 g of water at 80°C. The 100°C. A flask made of the same glass as that of rod measures
N
final temperature of mixture is a volume of 1000 cc at 0°C. The volume it measures at 100°C
(a) 0°C (b) 40°C in cc is
R

(c) 80°C (d) less than 0°C (a) 1002.4 (b) 1004.2
U

44. Three identical rods A, B and C of equal lengths and equal (c) 1006.4 (d) 1008.2
diameters are joined in series as shown in figure. Their DIRECTIONS for Qs. 49 to 50 : These are Assertion-Reason
JO

thermal conductivities are 2k, k and k/2 respectively. The type questions. Each of these question contains two statements:
temperatures at two junction points are Statement-1 (Assertion) and Statement-2 (Reason). Answer
these questions from the following four options.
U

T1 T2
(a) Statement-1 is True, Statement-2 is True; Statement-2 is a
100°C A B C 0°C
ED

correct explanation for Statement -1


2k k 0.5°C (b) Statement-1 is True, Statement -2 is True; Statement-2 is
(a) 85.7, 57.1°C (b) 80.85, 50.3°C NOT a correct explanation for Statement - 1
(c) 77.3, 48.3°C (d) 75.8, 49.3°C (c) Statement-1 is True, Statement- 2 is False
45. Which one of the following graphs best represents the ways (d) Statement-1 is False, Statement -2 is True
in which the total power P radiated by a black body depends 49. Statement 1 : The equivalent thermal conductivity of two
upon the thermodynamic temperature T of the body? plates of same thickness in series is less than the smaller
value of thermal conductivity.
Statement 2 : For two plates of equal thickness in series the
P P
equivalent thermal conductivity is given by
1 1 1
= +
(a) (b) K K1 K 2
O T O T 50. Statement 1 : As the temperature of the black body
increases, the wavelength at which the spectral intensity
P P ( El ) is maximum decreases.
Statement 2 : The wavelength at which the spectral intensity
(c) (d) will be maximum for a black body is proportional to the fourth
O T O T power of its absolute temperature.
Thermal Properties of Matter 295

Exemplar Questions 5. As the temperature is increased, the period of a pendulum


1. A bimetallic strip is made of aluminium and steel (aAl > (a) increases as its effective length increases even though
asteel). On heating, the strip will its centre of mass still remains at the centre of the bob
(a) remain straight (b) decreases as its effective length increases even though
its centre of mass still remains at the centre of the bob
(b) get twisted
(c) increases as its effective length increases due to shifting
(c) will bend with aluminium on concave side to centre of mass below the centre of the bob
(d) will bend with steel on concave side (d) decreases as its effective length remains same but the
2. A uniform metallic rod rotates about its perpendicular centre of mass shifts above the centre of the bob
bisector with constant angular speed. If it is heated uniformly 6. Heat is associated with
to raise its temperature slightly (a) kinetic energy of random motion of molecules
(a) its speed of rotation increases (b) kinetic energy of orderly motion of molecules

.IN
(b) its speed of rotation decreases (c) total kinetic energy of random and orderly motion of
(c) its speed of rotation remains same
AL molecules
(d) kinetic energy of random motion in some cases and
(d) its speed increases because its moment of inertia
kinetic energy of orderly motion in other
increases
7. The radius of a metal sphere at room temperature T is R and
N
3. The graph between two temperature scales A and B is shown the coefficient of linear expansion of the metal is a. The
in figure between upper fixed point and lower fixed point
R

sphere heated a little by a temperature DT so that its new


there are 150 equal division on scale A and 100 on scale B. temperature is T + DT. The increase in the volume of the
U

The relationship for conversion between the two scales is sphere is approximately.
given by
JO

(a) 2pRaDT (b) pR 2 aDT


180 (c) 4pR 3 aDT / 3 (d) 4pR3 aDT
U
Temperature (°A)

8. A sphere, a cube and a thin circular plate, all of same material


DtA = 150° and same mass are initially heated to same high temperature.
ED

(a) Plate will cool fastest and cube the slowest


(b) Sphere will cool fastest and cube the slowest
DtB = 100° (c) Plate will cool fastest and sphere the slowest
O Temperature (°B) 100 (d) Cube will cool fastest and plate the slowest
Past Years (2013-2017) NEET/AIPMT Questions
t A - 180 t B t A - 30 t B
(a) = (b) = 9. A piece of iron is heated in a flame. It first becomes dull red
100 150 150 100 then becomes reddish yellow and finally turns to white hot.
The correct explanation for the above observation is
t B - 180 t A t B - 40 t A
(c) = (d) = possible by using [2013]
150 100 100 180
(a) Wien’s displacement law
4. An aluminium sphere is dipped into water. Which of the
(b) Kirchoff’s law
following is true?
(a) Buoyancy will be less in water at 0°C than that in water (c) Newton’s law of cooling
at 4°C (d) Stefan’s law
(b) Buoyancy will be more in water at 0°C than that in water 10. The density of water at 20°C is 998 kg/m3 and at 40°C 992
at 4°C kg/m3. The coefficient of volume expansion of water is
(c) Buoyancy in water at 0°C will be same as that in water [NEET Kar. 2013]
at 4°C –4
(a) 10 /°C –4
(b) 3 × 10 /°C
(d) Buoyancy may be more or less in water at 4°C depending –4
(c) 2 × 10 /°C (d) 6 × 10–4/°C
on the radius of the sphere
EBD_7179
296 PHYSICS

11. Two metal rods 1 and 2 of same lengths have same 17. A piece of ice falls from a height h so that it melts completely.
temperature difference between their ends. Their thermal Only one-quarter of the heat produced is absorbed by the
conductivities are K1 and K2 and cross sectional areas A1 ice and all energy of ice gets converted into heat during its
and A2, respectively. If the rate of heat conduction in rod 1 fall. The value of h is : [2016]
is four times that in rod 2, then [NEET Kar. 2013] 5
[Latent heat of ice is 3.4 × 10 J/kg and g = 10 N/kg]
(a) K1A1 = K2A2 (b) K1A1 = 4K2A2 (a) 34 km (b) 544 km
(c) K1A1 = 2K2A2 (d) 4K1A1 = K2A2 (c) 136 km (d) 68 km
12. Certain quantity of water cools from 70°C to 60°C in the first 18. Coefficient of linear expansion of brass and steel rods are
5 minutes and to 54°C in the next 5 minutes. The temperature
a1 and a2. Lengths of brass and steel rods are l1 and l 2
of the surroundings is: [2014]
(a) 45°C (b) 20°C respectively. If (l 2 - l1 ) is maintained same at all
(c) 42°C (d) 10°C temperatures, which one of the following relations holds
good ? [2016]
13. Steam at 100°C is passed into 20 g of water at 10°C. When
water acquires a temperature of 80°C, the mass of water (a) a1 l 22 =a2 l12 (b) a1 l 2 =a2 l1
present will be: [2014]
– 1 – 1
[Take specific heat of water = 1 cal g °C and latent heat (c) a1 l 2 =a2 l1 (d) a1l1 = a 2l 2
of steam = 540 cal g– 1] 19. A black body is at a temperature of 5760 K. The energy of

.IN
(a) 24 g (b) 31.5 g radiation emitted by the body at wavelength 250 nm is U1,
(c) 42.5 g (d) 22.5 g at wavelength 500 nm is U2 and that at 1000 nm is U3. Wien's
constant, b = 2.88 × 106 nmK. Which of the following is
14. On observing light from three different stars P, Q and R, it
AL correct ? [2016]
was found that intensity of violet colour is maximum in the
spectrum of P, the intensity of green colour is maximum in (a) U1 = 0 (b) U3 = 0
the spectrum of R and the intensity of red colour is maximum (c) U1 > U2 (d) U2 > U1
N
in the spectrum of Q. If TP, TQ and TR are the respective 20. A spherical black body with a radius of 12 cm radiates 450
R

absolute temperature of P, Q and R, then it can be concluded watt power at 500 K. If the radius were halved and the
from the above observations that [2015] temperature doubled, the power radiated in watt would be :
U

(a) TP > TR > TQ (b) TP < TR < TQ (a) 450 (b) 1000 [2017]
JO

(c) TP < TQ < TR (d) TP > TQ > TR (c) 1800 (d) 225
15. The two ends of a metal rod are maintained at temperatures 21. Two rods A and B of different materials are welded together
100°C and 110°C. The rate of heat flow in the rod is found to as shown in figure. Their thermal conductivities are K1 and
U

be 4.0 J/s. If the ends are maintained at temperatures 200°C K2. The thermal conductivity of the composite rod will be :
and 210°C, the rate of heat flow will be [2015]
ED

(a) 16.8 J/s (b) 8.0 J/s [2017]


A K1
(c) 4.0 J/s (d) 44.0 J/s T1 T2
16. The value of coefficient of volume expansion of glycerine is B K2
5 × 10-4 K-1. The fractional change in the density of glycerine
for a rise of 40°C in its temperature, is: [2015 RS]
d
(a) 0.020 (b) 0.025
(c) 0.010 (d) 0.015 3(K1 + K 2 )
(a) (b) K1 + K2
2

K1 + K 2
(c) 2 (K1 + K2) (d)
2
Thermal Properties of Matter 297

Hints & Solutions


EXERCISE - 1 l l
Also Thermal resistance, R = KA = 2
1. (c) 2. (d) 3. (d) 4. (c) 5. (b) Kpr
6. (c) 7. (a) 8. (d) Heat flow will be maximum when thermal resistance is
minimum. From given option
F / A stress
9. (a) E= = where Dl=(l'–l) = lat so F = EAat 2l l
Dl / l strain 0 0
(i) r = 2r0, l = 2l0 \ R = Kp ( 2 r ) 2 = 2 Kpr 2
10. (a) 11. (c) 0 0
12. (b) Let the angle subtended by the arc formed be q. Then l0 l0
(ii) r = 2r 0, l = l0 \ R= =
l Dl l 2 - l 1 Kp ( 2 r0 )
2
4 Kpr0
2
q = or q = =
r Dr r1 - r2
2l 0 2l 0
(iii) r = r0, l = 2l0 \ R= =
l (a 2 - a1 ) DT l l (a 2 - a1 ) DT Kp r0 2
Kpr0
2
\ q= or =
t r t
l0 l0
(iv) r = r0, l = l0 \ R= 2
= 2
t

.IN
Kp r0 Kpr0
So, r =
( a 2 - a 1 ) DT It is clear that for option (b) resistance is minimum,
hence heat flow will be maximum.
s T 4 ´ A1 ´ e R 12
13. (a) Initial rate of loss of heat = = 20.
AL
(c)
s T4 ´ A2 ´ e R 22 21. (a) In condition of weightlessness, convection is not
possible.
N
E
14. (b) Momentum of photon = 22. (a) reff = r1r2
c
R

2E dQ KA(T2 - T 1 ) Kpr1r2 (T2 - T1 )


Change in momentum = = =
U

c dt L L
= momentum transferred to the surface 23. (c) When a body is heated then relation between colours
JO

(the photon will reflect with same magnitude of and temperature is according to Prevost’s theory of
momentum in opposite direction). radiation which states that everybody emitting radiant
15. (c) We know energy in all directions at a rate depending only on the
U

nature of its surface and its temperature e.g., when a


E = eT 4 Þ ln E = ln e + 4 ln T body is placed in an enclosure (furnace) it would acquire
ED

\ ln e is negative the temperature of furnace and seem white means


radiate white light. So it becomes first dark and then
because e < 1 white.
dQ kA (T1 - T2 )
24. (c) =
dt L
[(T1–T2) is the temperature difference]
25. (b)
EXERCISE - 2
ln E
ln T 1. (c) R t = R 0 (1 + a t )
2.71 = R 0 (1 + a ´ 10 ) ... (1)
16. (c) The temperature is measured by the value of the 3.70 = R 0 (1 + a ´ 100 ) ... (2)
thermodynamic property of a substance i.e., the
property which varies linearly with the temperature. 3.26 = R 0 (1 + a t ) ... (3)
17. (b) Temperature of stars can be determined by Wein’s Solve these equations to obtain the value of t.
displacement law
140 - 32 C 700 - 160
l m = constant 2. (a) = or = C or C = 60ºC
9 5 9
18. (c)
TH - TL 1
19. (b) We know that Q = 3. (c) Dt = a DT ´ t
R 2
EBD_7179
298 PHYSICS

1
\ 5= a (T - 15 ) ´ 86400 F - 32 80
2 14. (c) =
9 5
1
and 10 = a (30 - T ) ´ 86400 F – 32 = 144 or F = 176ºF.
2 15. (d) Percentage change in volume = gt × 100
4. (a) F = Y a t A or F µ a
= 3at ´ 100 = 300at = 300 ´ 2 ´ 10 -5 ´ 200 = 1.2
(Q Y t A is same for both copper and iron) 16. (b)
or FC µ α C and FΙ µ α Ι 17. (c) According to Stefan's Boltzmann law, the energy
F 3/2 3 radiated per unit time E = sAT4.
\ C = = \ 5.67 × 104 = 5.67 × 10–8 × 1 × T4 (\ A = 1 m2)
FI 1 2
1/ 4
Δl æ 5.67 ´ 104 ö
5. (b) = α Δ T = 10 -5 ´ 100 = 10 -3 or, T = ç = 1000K
l ÷
è 5.67 ´ 10 –8 ø
Δl or , T = 1000 – 273 = 727°C.
´ 100% = 10 -3 ´ 100 = 10 - 1 = 0.1%
l 18. (a) When the temperature of cold junction is increased,
6. (a) Let m be the mass of steam condensed. Then the neutral temperature remains constant for a given
m × 540 + m × 10/2 = 22 × 70 thermocouple and it is independent of the temperature
\ m = 2.83 gm of the cold junction.

.IN
Now, total mass = 22 + 2.83 = 24.83 gm \ qn = 275°C, qi – qn = qn – qc
7. (d) Thermal resistance R =l/KA \ qi = 2qn – qc = 530°C.
Where l =20cm. & A(cylindrical rod) 19. (a) According to Newton's law of cooling,
AL
=pr2 = 40pcm2
q1 - q 2 éq + q ù
20 cal = K ê 1 2 - q0 ú
So R = = 0.318 t ë 2 û
N
0.5 ´ 40p sec ´º C
where q0 is the surrounding temperature.
8. (c) According to Stefan’s Law, the rate of loss of heat is
R

60 - 40 æ 60 + 40 ö
Q \ = Kç - 10÷
= sA (T14 - T24 ) ´ e
U

7 è 2 ø
t
JO

here s = 5.67 × 10–8J/m2 × sec.K2, 20 1


T1= 527+273 = 800K, Þ = 40K Þ K =
7 14
T2 = 27 + 273 = 300K & A = 200×10–4m2
40 - 28 é 40 + 28
U

ù 12
\ = Kê - 10ú Þ = 24K
So. Q = 5.67 ´ 10 -8 ´ 2 ´ 10 -2 t ë 2 û t
ED

t
[(800)4 - (300)4 ] ´ 0.4 12 12 ´ 14
or t = = = 7 min
@ 182 joule 24K 24
9. (a) According to Stefan’s Law 20. (a) Let q be temperature of interface.
energy radiated per sec E = sAT4 K P A(100 - q) K Q A(q - 0)
(here e = 1 for black body) \ =
l l
for first case E = sA(273)4
for second case E1=sA(546)4 so E1=16E KP q 2 q
\ K = 100 - q Þ =
10. (b) According to Wein's displacement law, λm T = b. Q 3 100 - q
where b =2.884 × 10–3 mK. or, 200 – 2q = 3q or, 5q = 200 or, q = 40°C
21. (a) Let the final temperature of mixture be T.
So l m1 T1 = l m2 T2
Then 100 × 80 + 100 (T – 0) ×½
or 4800 × 6000 = l m 2 ´ 3000 or l m2 = 9600A° (as specific heat of ice is 0.5 cal/g C° and specific heat
of water is 1 cal/gC°)
11. (b) The time of cooling increases as the difference between = 100 × 1 × (100 – T)
the temperature of body & surrounding is reduced. So Solving, we get T = 13.33ºC.
T1 < T2 < T3 (according to Newton’s Law of cooling).
22. (a) g = a1 + a 2 + a 3
12. (c) 11.0 ´ 10 -5 T1 = 5.5 ´ 10 -5 T2
T1 1 = 13 ´ 10 - 7 + 231 ´ 10 - 7 + 231 ´ 10 - 7
=
T2 2 = 475 ´ 10 - 7
13. (c)
Thermal Properties of Matter 299

DV = V0 ( g m - g g )DT 29. (b) 80 × 1 × 30 = m × 80 Þ m = 30 gm.


23. (a)
30. (c)
= 1[1 .82 ´ 10 - 4 - 3 ´ (10 ´ 10 - 6 )] 100
K1 l12
= 1[1 . 82 ´ 10 - 4 - 0 .3 ´ 10 -4 )] 100 = 15.2 ml 31. (a) Use K = 2
2 l2
24. (c) Given Δl / l = 0.10% = 0.001 and ΔT = 100 º C 32. (b) Here flow of heat, area of vessels & temperature
gradient are same so
Δl
Now = αΔT dT dT
l Q = K1A t1 = K 2 A t2
dx dx
or 0.001 = α ´ 100
K t 35 7
or α = 10 - 5 /º C so 1 = 2 = =
K2 t1 20 4
33. (b)
Further γ = 3α = 3 ´ 10-5/ºC
34. (b) λ1T1 = λ 2 T2
ΔV
\ ´ 100 = (3 ´ 10 - 3 ) (100) = 0.30% 6050 ´ 4753 = 9506 .T2
V
T2 = 3025 K
25. (a) We know that γ real = γ apparent + γ vessel

So, (γ app + γ vessel ) glass = (γ app + γ vessel ) steel E1 s (T14 - T04 ) (600 ) 4 - (300 ) 4

.IN
35. (a) = =
E 2 s( T24 - T04 ) (500 ) 4 - (300 ) 4
(Q g real is same in both cases)
36. (d) Thermal capacity = 40 × 0.2 = 8 cal °C –1
or 153 ´ 10 - 6 + (γ vessel ) glass
AL
37. (b) –13°F is (13 + 32)° below ice point on F scale.
= 144 ´ 10 - 6 + ( g vessel ) steel 38 (a) Mass ratio 1 : 2; hence DT ratio 2 : 1. Therefore
equilibrium temp is 30°C.
N
Further (γ vessel ) steel 39. (b) Heat capacity of cold : hot =1 : 2
-6 -6
R

= 3a = 3 ´ (12 ´ 10 ) = 36 ´ 10 /ºC 1
So final temp. is × (2 × 92 + 1 × 20) = 68°C.
3
\ 153 ´ 10 - 6 + (γ vessel ) glass
U

3
40. (d) E = s ´ area ´ T 4 ; T increases by a factor .
JO

= 144 ´ 10 - 6 + 36 ´ 10 - 6 2
1
Solving we get ( g vessel )glass = 27 ´ 10-6/ºC Area increases by a factor .
4
U

g glass l 2l
or a = = 9 ´ 10-6/ºC 41. (d) t µ , t ' µ
A A/2
ED

3
26. (b) Let the final temperature be T. 42. (a) 4K (100 - q) = K(q - 0) Þ 400 - 4q = q
Then 200 × 1 × (T – 20) + 20 × (T – 20)
Þ q = 80°C
= 440 (92 – T)
Solving it, we get T = 68ºC. 43. (a) 540 × 80 + 540 q = 540 (80 – q)
Þ 80 + q = 80 - q Þ 2q = 0 Þ q = 0°
27. (a) mL + m(100 - 80)
44. (a)
= 1.1´ 1´ (80 – 15) + 0.02 ´ (80 - 15) 45. (c) The total power radiated by a black body of area A at
m ´ 540 + 20 m = 71.5 + 1.30 temperature TK is given by P = AsT4
Where s = Stefan's constant
560 m = 72.80 \ m = 0.130 = 5.7 × 10–8 W m–2 K–4
28. (d) W = W1 - W2 = mgh - mg h ¢ = mg (h - h ¢) Which is best represented in graph. (c)

= 5 ´ 10 ( 20 - 0 . 2 ) = 5 ´ 10 ´ 19 .8 46. (b) According to Stefan’s law E = sT 4 ,


= 5 ´ 198 = 990 joule T1 = 500 K
T2 = 1000 K
This energy is converted into heat when the ball strikes
the earth. Heat produced is 4 4
E 2 æ T2 ö æ 1000 ö
990 =ç ÷ =ç ÷ = 16
Q= calorie E1 è T1 ø è 500 ø
4.2
\ E2 = 16 × 7 = 112 cal / cm2s
Q 99 ´ 100 11 47. (a) The rate of heat loss per unit area due to radiation
DT = = = ºC
mc 42 ´ 5000 ´ 0 .09 32 = Îs (T4–T04)
= 0.6 × 5.67 × 10–8 [(400)4–(300)4] = 595 Jm–2s–1.
EBD_7179
300 PHYSICS

V2 - V1 l 2 - l1 2. (b) On heating a uniform metallic rod its length will increase


48. (a) g= ;a= so moment of inertia of rod increased from I1 to I2.
V1 (T2 - T 1 ) l1 (T2 - T1 )
No external torque is acting on the system so angular
T1 = 0°C, T2 = 100°C momentum should be conserved.
V2 - V1 l -l L = Angular momentum = Iw = constant
g= ;a = 2 1
100V1 100l1 I1w1 = I 2 w2
l 1 = 10 cm; l 2 – l 1 = 0.08 mm = 0.008 cm
w2 I1
0.008 So, w = I < 1
a= = 8 ´ 10 -6 / °C; g = 3a = 24 × 10–6 / °C 1 2 Rod
10 ´ 100
w2 < w1
V2 - 1000
\ 24 × 10–6 = (Q Due to expansion of the rod I2 > I1)
1000 ´ 100 So, angular veloctiy decreases.
V2 – 1000 = 24 × 10–6 × 105 = 2.4 \ V2 = 1002.4 cc
3. (b) In the given graph shows lowest fixed point for scale A
49. (a) For equivalent thermal conductivity, the relation is
is 30° and lowest point for scale B is 0°. Upper fixed
1 1 1 point for the scale A is 180° and upper fixed point for
= + ; If K1 = K 2 = K
K R K1 K 2 scale B is 100°. Hence, formula is

1 1 1 2 K t A - (LEP) A t B - (LEP) B

.IN
= + = Þ KR = =
KR K K K 2 (UFP) A - (LFP) A (UFP) B - (LFP) B
Which is less than K. AL where , LFP - Lower fixed point, UFP - Upper fixed
If K1 > K 2 suppose K1 = K 2 + x point.

1 1 1 K + K1 (+A) B (+ B)
= + = 2 180
N
K K1 K 2 K1K 2 C
R
Temperature (°A)

1 K2 + K2 + x 2 DtA = 150°
Þ = Þ K = K 2 + K 2x
K (K 2 + x)K 2 2K 2 + x
U

90°–q
q
2
K + K2x
JO

Now, K 2 - K = K 2 - 2 O
2K 2 + x 30°
DtB = 100°
2K 2 2 + K 2 x - K 2 2 - K 2 x K 22 O Temperature (°B) 100
U

= = = positive
( 2K 2 + x ) 2K 2 + x t A - 30 t -0
ED

So, K2 > K, so the value of K is smaller than K2 and K1. So, = B


180 - 30 100 - 0
50. (c) From Wein's law lmT = constant i.e. peak emission
1 t A - 30 tB
wavelength lm µ . =
T 150 100
Hence as T increases lm decreases. 4. (a) As we know that, the Buoyant force (F) on a body
volume (V) and density of (r), when immersed in liquid
EXERCISE - 3
of density (rl) is = V ' rl g
Exemplar Questions
where V ' = volume of displaced liquid by dipped body
1. (d) If strips of aluminium and steel are fixed together on (V).
metallic strip and both are heated then (aAl > asteel) Let volume of the sphere is V and r is its density, then
aluminium will exapnd more because the metallic strip we can write buoyant force
with higher coefficient of linear expansion (aAl) will F = V rs G
expand more Thus it should have larger radius of
curvature. Hence, aluminium will be on convex side. F µ rl (for liquid)
Aluminium F4°C r4°C
= >1
F0°C r0°C
(Q r4°C > r0°C )
F4°C > F0°C
q Steel
Hence, buoyancy will be less in water at 0°C than that
in water at 4°C .
O
Thermal Properties of Matter 301

5. (a) As the temperature increased the length (L) of the m


pendulum increases due to expansion i.e., linear. T m
T

L m
T
Sphere Cube Plate
Pendulum As thickness of the plate is least so, surface area of the
So, Time period of pendulum plate is maximum.
We know that, according to Stefan's law of heat loss
L
T = 2p H µ AT 4
g where, A is surface area of object and T is temperature.
So, Hsphere : Hcube : Hplate
Tµ L = Asphere : Acube : Aplate
Hence on increasing temperature, time period (T) also So area of circular plate is maximum.
increases. For sphere, as the sphere is having minimum surface
6. (a) As we know that when the temperature increases area.

.IN
vibration of molecules about their mean position Hence, the sphere cools slowest and circular plate will
increases hence the kinetic energy associated with cool faster.
random motion of molecules increases.
Past Years (2013-2017) NEET/AIPMT Questions
7. (d) Let the radius of the sphere is R. As the temperature
AL
increases radius of the sphere increases as shown. 9. (a) Wein’s displacement law
According to this law
N
1
lmax µ
R

T
or, lmax × T = constant
U

R So, as the temperature increases l decreases.


10. (b) From question,
JO

dV Dr = (998 – 992) kg/m3 = 6 kg/m3


998 + 992
r= kg/m 3 = 995 kg/m3
U

4 3 2
Original volume V = pR
ED

3 m
r=
V
Coefficient of linear expansion = a
Coefficient of volume expansion = 3a Dr DV Dr DV
Þ =- Þ =
r V r V
DV
As we know that Y = \ Coefficient of volume expansion of water,
V Dt
By putting the value of V, increase in the volume 1 DV 1 Dr 6
= = » 3 ´10 -4 / °C
V Dt r Dt 995 ´ 20
Þ DV = 3V aDt = 4pR3aDt
11. (b) Q1 = 4Q2 (Given)
8. (c) Loss of heat temperature on cooling temperature
increase depend on material of object surface area K1 A1Dt K A Dt
exposed to surrounding and temperature difference Þ =4 2 2 Þ K1A1 = 4K2A2.
L L
between body and surrounding.
12. (a) Let the temperature of surroundings be q0
Let us consider the diagram where all the three objects
By Newton's law of cooling
are heated to same temperature T. As we know that
density q1 - q 2 é q + q2 ù
= kê 1 - q0 ú
mass t ë 2 û
r=
volume 70 - 60 é 70 + 60 ù
where r is same for all the three objects hence, volume Þ = kê - q0 ú
5 ë 2 û
will also be same.
Þ 2 = k [65 – q0] ...(i)
EBD_7179
302 PHYSICS

60 - 54 é 60 + 54 ù 4L 4 ´ 3.4 ´105
Similarly, =kê - q0 ú Þ h= = = 136 km .
5 ë 2 û g 10
6 18. (d) From question, (l2 – l1) is maintained same at all
Þ = k [57 – q0] ...(ii) temperatures hence change in length for both rods
5
should be same
By dividing (i) by (ii) we have i.e., Dl1 = Dl2
10 65 - q0 As we know, coefficient of linear expansion,
= Þ q0 = 45º
6 57 - q0 Dl
a=
13. (d) According to the principle of calorimetry. l 0 DT
Heat lost = Heat gained l1a1DT = l2a2DT
mLv + mswDq = mwswDq l1a1 = l2a2
Þ m × 540 + m × 1 × (100 – 80) 19. (d) According to wein's displacement law, maximum amount
= 20 × 1 × (80 – 10) b
of emitted radiation corresponding to lm =
Þ m = 2.5 g T
Therefore total mass of water at 80°C 2.88 ´106 nmK
= (20 + 2.5) g = 22.5 g lm = = 500 nm
5760K

.IN
14. (a) From Wein’s displacement law
lm × T = constant ­ U2
P – max. intensity is at violet U
AL Emitted
Þ lm is minimum Þ temp maximum
(radiation) wave length K
R – max. intensity is at green 250 nm 1000 nm
N
Þ lm is moderate Þ temp moderate 500 nm
Q – max. intensity is at red Þ lm is maximum Þ temp. From the graph U1 < U2 > U3
R

minimum i.e., Tp > TR > TQ 20. (c) Given r1 = 12 cm , r2 = 6 cm


(c) As the temperature difference DT = 10°C as well as the T1 = 500 K and T2 = 2 × 500 = 1000 K
U

15.
thermal resistance is same for both the cases, so thermal P1 = 450 watt
JO

current or rate of heat flow will also be same for both Rate of power loss P µ r 2 T 4
the cases.
16. (a) From question, P1 r12 T14
=
U

Rise in temperature Dt = 40°C P2 r22 T24


ED

Dr r22T2 4
Fractional change in the density =? P2 = P1
r0
r12T14
Coefficient of volume expansion Solving we get, P2 = 1800 watt
g = 5 × 10–4K–1 21. (d) Heat current H = H1 + H2
r = r0 (1 –gDt) K A(T1 - T2 ) K 2 A(T1 - T2 )
= 1 +
Dr d d
Þ = gDT = (5 × 10–4) (40) = 0.02
r0 K EQ 2A(T1 - T2 ) A(T1 - T2 )
= [K1 + K 2 ]
17. (c) According to question only one-quarter of the heat d d
produced by falling piece of ice is absorbed in the Hence equivalent thermal conductivities for two rods
melting of ice. k1 + k 2
of equal area is given by K EQ =
mgh 2
i.e., = mL
4
12 Thermodynamics

THERMAL EQUILIBRIUM AND ZEROTH LAW OF (c) Internal energy : The internal energy of a gas is sum of
THERMODYNAMICS internal energy due to moleculer motion (called internal
Thermal Equilibrium kinetic energy UK) and internal energy due to molecular
Two systems are said to be in thermal equilibrium with each other configuration (called internal potential energy UP.E.)
if they have the same temperature.
i.e., U = UK + UP.E. ……(1)

.IN
Zeroth Law of Thermodynamics
If objects A and B are separately in thermal equilibrium with a (i) In ideal gas, as there is no intermolecular attraction,
third object C then objects A and B are in thermal equilibrium AL hence
with each other. 3n
FIRST LAW OF THERMODYNAMICS U = UK = RT ……(2)
First law of thermodynamics gives a relationship between 2
N
heat, work and internal energy. (for n mole of ideal gas)
(a) Heat : It is the energy which is transferred from a system to (ii) Internal energy is path independent i.e., point function.
R

surrounding or vice-versa due to temperature difference (iii) In cyclic process, there is no change in internal energy
between system and surroundings. (shown in fig.)
U

(i) It is a macroscopic quantity. i.e., dU = Uf – Ui = 0


(ii) It is path dependent i.e., it is not point function.
JO

Þ Uf = Ui
(iii) If system liberates heat, then by sign convention it is
taken negative, If system absorbs heat, it is positive.
(b) Work : It is the energy that is transmitted from one system
U

to another by a force moving its points of application. The


expression of work done on a gas or by a gas is
ED

V2
W = ò dW = ò PdV
V1

where V1 is volume of gas in initial state and V2 in final


state. (iv) Internal energy of an ideal gas depends only on
(i) It is also macroscopic and path dependent function. temperature eq.(2).
(ii) By sign convention it is +ive if system does work (i.e., First law of thermodynamics is a generalisation of the
expands against surrounding) and it is – ive, if work is law of conservation of energy that includes possible
done on system (i.e., contracts). change in internal energy.
(iii) In cyclic process the work done is equal to area under First law of thermodynamics “If certain quantity of heat dQ is
the cycle and is negative if cycle is anti-clockwise and added to a system, a part of it is used in increasing the internal
+ive if cycle is clockwise (shown in fig.(a) and (b)). energy by dU and a part is use in performing external work
done dW
i.e., dQ = dU + dW Þ dU = dQ - dW
The quantity dU (i.e., dQ – dW) is path independent but dQ and
dW individually are not path independent.
Applications of First Law of Thermodynamics
(i) In isobaric process P is constant
V2
so dW = ò PdV = P(V2 - V1 )
V1
so dQ = dU + dW = n CP dT
EBD_7179
304 PHYSICS

(ii) In cyclic process heat given to the system is equal to work For ideal gas, dQ = 0
done (area of cycle). dU = mCVdT (for any process)
(iii) In isothermal process temperature T is constant and work
V2 V2 K
done is dW = òV1
PdV = ò
V1 Vg
dV
V2 V2
dW = òV1
PdV = nRT Log e
V1
(where PVg = K = constant)

Since, T = constant so for ideal gas dU = 0 K æ 1 1 ö ( P V - PV )


= ç g -1
- g -1 ÷ = 2 2 1 1
1 - g è V2 V1 ø 1- g
V2
Hence, dQ = dW = nRT Log e (for ideal gas)
V1 where PVg = constant is applicable only in adiabatic process.
(iv) In isochoric process W = 0 as V = constant Adiabatic process is called isoentropic process (in these
It means that heat given to system is used in increasing process entropy is constant).
internal energy of the gas. (iii) Isobaric process : A process taking place at constant
(v) In adiabatic process heat given or taken by system from pressure is called an isobaric process. In this process
surrounding is zero i.e., dQ = 0 dQ = n CpdT, dU = n CVdT and dW = P(V2–V1)
(iv) Isochoric process : A process taking place at constant
é nR ù é (P V - P V ) ù
dU = -dW = - ê ( T1 - T2 )ú = ê 1 1 2 2 ú volume is called isochoric process.
ë g - 1 û ë g -1 û In this process, dQ = dU =n CVdT and dW = 0

.IN
It means that if system expands dW is +ive and dU is –ive (v) Cyclic process : In this process the inital state and final
(i.e., temperature decrease) and if system contracts dW is state after traversing a cycle (shown in fig.) are same.
–ive and dU is +ive (i.e., temperature increase). In cyclic process, dU = 0 = Uf – Ui and dW = area of cycle
THERMODYNAMIC PROCESSES
AL = area (abcd)
(i) Isothermal process : If a thermodynamic system is perfectly
conducting to surroundings and undergoes a physical
N
change in such a way that temperature remains constant
R

throughout, then process is said to be isothermal process.


P
U
JO

T = constant

Slope of adiabatic and isothermal curve :


U

For isothermal process PV = constant


ED

On differentiating, we get PdV + VdP = 0


V
And slope of isothermal curves
For isothermal process, the equation of state is
PV = nRT = constant, where n is no. of moles. æ dP ö P
ç ÷ =- ... (1)
For ideal gas, since internal energy depends only on è dV øisothermal V
temperature.
V2 V2 dV P
dU = 0 Þ dQ = dW = ò PdV = nRT ò
V1 V1 V Isothermal
V2 V
or dQ = nRT log e = 2.303nRT log10 2
V1 V1
(ii) Adiabatic process : If system is completely isolated from
the surroundings so that no heat flows in or out of it, then Adiabatic
any change that the system undergoes is called an V
adiabatic process. For adiabatic process PVg
= constant
P On differentiation, we get slope of adiabatic curve
æ dP ö
ç ÷ = -g(P/V) .... (2)
è dV øadiabatic
It is clear from equation (1) and (2) that the slope of
adiabatic curve is more steeper than isothermal curve as
shown by fig by g time (g = CP/CV)
V
Thermodynamics 305

Graphs of thermodynamic processes : 4. For isochoric process, the P–V, V–T and P–T graphs :
1. In the figure (i) P–V graph the process ab is isothermal, bc
is isobaric and ca is isochoric. 2 2
P
T

P a 1 1
V V

c b
2
P
V
Fig (i)
1

The fig (ii) is the P–T diagram of fig (i) T


5. For isobaric process, the P – V, P – T and V – T graphs :
V P
P

.IN
a

P T
AL
c b
V
N
T
Fig.(ii)
R

T
U

2. Figure below shows P – V diagrams for two processes. 6. For isothermal process, the P – V, V – T and P – T graphs :
JO

P V
P I
U

T
ED

II V

The heat absorbed in process I is more than that in II. T


Because, area under process I is also more than area under Keep in Memory
process II. The work done in the process I is more than that
in II. Also, the change in internal energy is same in both 1. In thermodynamics heat and work are not state variables,
cases. whereas internal energy is a state variable.
2. For ideal-gas
3. The P–V and corresponding V–T diagram for a cyclic
(i) relation between P and V is PV g = constant
process abca on a sample of constant mass of ideal gas are
(ii) relation between V and T is TV g –1 = constant
shown below:
(iii) relation between P and T is T g P 1– g = constant
3. A quasi-static process is an infinitely slow process such
that system remains in thermal and mechanical equilibrium
a b c b with the surroundings throughout.
P V
4. Pressure, volume, temperature and mass are state variables.
Heat and work are not state variables.
a 5. A graphical representation of the state of a system with the
c help of two thermodynamical variables is called indicator
diagram.
V T
EBD_7179
306 PHYSICS

REVERSIBLE AND IRREVERSIBLE PROCESS


Reversible Process : Hot Q1 Q2 Cold
Working
A process which can proceed in opposite direction in such a Reservoir Substance Reservoir
way that the system passes through the same states as in direct T1 T2
process and finally the system and the surroundings acquire
the intial conditions.
Conditions for a process to be reversible : Work (W) = Q1 – Q2
(a) The process must be extremely slow.
(b) There should no loss of energy due to conduction, or The coefficient of performance of refrigerator or heat pump is
radiation. The dissipating forces should not be in the system. Heat extracted from cold reservoir Q2 T2
(c) The system must always be in thermal and chemical b= = = ,
Work done on refrigerator Q 1 - Q 2 T1 - T 2
equilibrium with the surroundings.
where T2 is temperature of cold body and T1 is temperature of
Examples : Fusion of ice, vaporisation of water, etc. hot body.
Irreversible Process : CARNOT ENGINE
The process which cannot be traced back in the opposite Carnot devised an ideal engine which is based on a reversible
direction is defined as irreversible process. cycle of four operations in succession : isothermal expansion,
Examples : Work done against friction, magnetic hysteresis. adiabatic expansion, isothermal compression and adiabatic

.IN
• In nature all process are irreversible, because no natural compression.
process can fulfil the requirement of a reversible process. A Isothermal
Q 1 expansion
HEAT ENGINE
AL
c o mp re s s io n
T1
Ad ia b a tic
A heat engine is a device which converts heat energy into

expa n s io n
Adia b a tic
mechanical energy. B
N
P D
T2
R

Hot Q1 Q2 Cold
Working Isothermal C
Reservoir Reservoir Q2
U

Substance compression
T1 T2 V1 V2
V
JO

Efficiency of Carnot engine,


æV ö æV ö
µRT1 In ç 2 ÷ + µRT2 In ç 4 ÷
U

W è V1 ø è V3 ø
Work (W) = Q1 – Q2 h= =
Q1 æV ö
ED

µRT1 In ç 2 ÷
Efficiency of heat engine is given by è V1 ø
The points B and C are connected by an adiabatic path as
Work done (W ) are the points D and A. Hence, using this eqn. and the
Efficiency η =
Heat taken from source (T1 ) adiabatic gas eqn.
Q2 T
T1V2(g – 1) = T2 V3(g – 1) and T1V1(g – 1) = T2 V4(g – 1).
= 1- = 1- 2 V2 V3
Q1 T1 Combination of the above eqns. gives V = V , and,
1 4
where Q2 = amount of heat rejected per cycle to the sink T1 - T2 Q1 - Q2
(of temp T2) h= =
T1 Q1
Q1 = amount of heat energy absorbed per cycle from the source
Q2 T2
(of temp T1). or, h = 1 - Q = 1 - T .
The efficiency of heat engine h is never greater than unity, 1 1

h =1 only for ideal engine & for practical heat engine h < 1. The percentage efficiency of Carnot’s engine,
T -T Q - Q2
REFRIGERATOR AND HEAT PUMP : h = 1 2 ´ 100% or, h = 1 ´ 100%
T1 Q1
Refrigerator or heat pump is a heat engine running in backward
The efficiency of a Carnot engine is never 100% because it
direction i.e. working substance (a gas) takes heat from a cold
is 100% only if temperature of sink T2 = 0 which is impossible.
body and gives out to a hotter body with the use of external
Q T Q Q
energy i.e. electrical energy. A heat pump is the same as a In a Carnot cycle, 2 = 2 or 1 = 2 .
refrigerator. Q1 T1 T1 T2
Thermodynamics 307

Carnot Theorem : No irreversible engine (I) can have Example 1.


efficiency greater than Carnot reversible engine (R) A cyclic process is shown in fig. Work done during isobaric
working between same hot and cold reservoirs. expansion is
T2 Q
i.e., hR > hI or 1 - > 1- 2
T1 Q1 2
2 × 10 A B
SECOND LAW OF THERMODYNAMICS
It states that it is impossible for a self acting machine unaided P
2 2
by any external agency, to transfer heat from a body at a lower (N/m ) 10 D C
temperature to a body at higher temperature.
It is deduced from this law that the efficiency of any heat engine
can never be 100%. 1 2 3
3
Entropy : V(m )
Entropy is a measure of disorder of the molecular motion of a
system. The greater the disorder, the greater is the entropy. The (a) 1600 J (b) 100 J
change in entropy is given by (c) 400 J (d) 600 J
Solution : (c)
Heat absorbed by the system (dQ) Isobaric expansion is represented by curve AB;
=DS

.IN
Absolute temperature (T ) Work done = area under AB
= 2 × 102 × (3 – 1) = 4 × 102 = 400 J.
dQ
S1 - S2 = ò (here T is not differentiable) Example 2.
T
AL
An ideal gas heat engine operates in carnot cycle between
Clausius inequality 227ºC and 127ºC. It absorbs 6 × 104 cal of heat at higher
N
dQ dQ temp. Amount of heat converted into work is
T Ñò
£ 0 or, dS ³ ò
T (a) 1.2 × 104 cal (b) 2.4 × 104 cal
R

or, dQ = TdS ³ dU + PdV (c) 6 × 104 cal (d) 4.8 × 104 cal
U

Also, S = K logew Solution : (a)


JO

w2 Q 2 T2
DS = K log e As = ;
w1 is the microscopic form of entropy, where Q1 T1

K is Boltzmann's constant and w respresents the number of


U

Q2 127 + 273 400 Q2 T


possible microscopic states. \ = = (As h = 1 - = 1- 2 )
6 ´ 10 4 227 + 273 500 Q1 T1
ED

Energy entering a body increases disorder.


Energy leaving a body decreases disorder. 4
When a hot body is brought into thermal contact with a Q2 = ´ 6 ´ 104 = 4.8 ´ 104 cal
cold body for a short time, then :
5
(i) Each body will experience a change in the entropy of \ W = Q 1 - Q 2 = 6 ´ 10 4 - 4.8 ´ 10 4 = 1.2 ´ 10 4 cal.
its particle.
(ii) The hot body experiences a decrease in entropy (a Example 3.
negative change) of magnitude An ideal carnot engine whose efficiency is 40% receives
heat at 500 K. If its efficiency were 50%, then what would
DQ be intake temp. for same exhaust temp ?
DS1 =
T1 Solution :
(iii) The cold body experiences an increase in entropy (a
T2 T2 40 3
positive change) of magnitude From, h = 1 - ; = 1- h = 1 - = ;
T1 T1 100 5
DQ
DS2 =
T2 3 3
\ T2 = T1 = ´ 500 = 300 K
(iv) The net change in entropy 5 5

DS = DS1 + DS2 T2 300 50 1


Again = 1 - η' or =1- =
The effect of naturally occurring processes is always to T1 ' T1 ' 100 2
increase the total entropy (or disorder) of the universe.
or T1 ' = 600K
EBD_7179
308 PHYSICS

Example 4. Example 6.
When a system is taken from state a to state b, in fig. along A Carnot engine working between 300 K and 600 K has a
the path a ® c ® b, 60 J of heat flow into the system, and work output of 800 J per cycle. What is the amount of heat
30 J of work are done : energy supplied to the engine from source per cycle?
(i) How much heat flows into the system along the path Solution :
a ® d ® b if the work is 10 J. W = 800 J, T1 = 600 K, T2 = 300 K
(ii) When the system is returned from b to a along the
curved path, the work done by the system is –20 J. T1 W 300 800 800
\ h=1– = =1- = or 0.5 =
Does the system absorb or liberate heat, and how T2 Q1 600 Q1 Q1
much?
Heat energy supplied by source,
(iii) If, Ua = 0 and Ud = 22 J, find the heat absorbed in the
process a ® d and d ® b. 800
Q= = 1600 joule per cycle
0.5
P Example 7.
c b
The temperatures T1 and T2 of the two heat reservoirs in
an ideal Carnot engine are 1500°C and 500°C respectively.
Which of the following : increasing T1 by 100°C or

.IN
decreasing T 2 by 100°C would result in a greater
a d
improvement in the efficiency of the engine?
Solution :
V
AL
T2
Solution : The efficiency of a Carnot's engine is given by h = 1 -
T1
N
For the path a ® c ® b
dU = dQ – dW = 60 – 30 = 30 J or Ub – Ua = 30 J Given T1 = 1500°C = 1500 + 273 = 1773 K and
R

T2 = 500°C = 500 + 273 = 773 K.


(i) Along the path a ® d ® b
When the temperature of the source is increased by 100°C,
U

dQ = dU + dW = 30 + 10 = 40 J
keeping T2 unchanged, the new temperature of the source
(ii) Along the curved path b – a
JO

is
dQ = (Ua – Ub) + W = (–30) + (–20) = –50 J, T´1 = 1500 + 100 = 1600°C = 1873 K. The efficiency becomes
heat flows out the system
T2 773
U

(iii) Qad = 32 J; Qdb = 8 J h´= 1 - = 1- = 0.59


T´1 1873
Example 5.
ED

Two samples of a gas initially at same temperature and On the other hand, if the temperature of the sink is decreased
pressure are compressed from a volume V to V/2. One sample by 100°C, keeping T1 unchanged, the new temperature of
is compressed isothermally and the other adiabatically. In the sink is T´2 = 500 – 100 = 400°C = 673 K. The efficiency
which sample is the pressure greater? now becomes
Solution :
T´2 673
Let initial volume, V1 = V and final volume, V2 = V/2 h´´= 1 - = 1- = 0.62
T1 1773
Initial pressure, P1 = P ; final pressure, P2 = ?
For isothermal compression Since h´´ is greater than h´, decreasing the temperature of
the sink by 100°C results in a greater efficiency than
P1V1 PV increasing the temperature of the source by 100°C.
P2V2 = P1V1 or P2 = = = 2P
V2 V/2 Example 8.
Calculate the work done when 1 mole of a perfect gas is
For adiabatic compression
compressed adiabatically. The initial pressure and volume
g g of the gas are 105 N/m2 and 6 litre respectively. The final
æV ö æ V ö
P2' V2g = P1V1g or P2' = P1 ç 1 ÷ = P ç volume of the gas is 2 litres. Molar specific heat of the gas
è V2 ø è V / 2 ÷ø
at constant volume is 3R/2. [(3)5/3 = 6.19]
or P2´ = 2gP Solution :
Since g > 1 \ 2g > 2 \ P2' > P2 For an adiabatic change PVg = constant
Pressure during adiabatic compression is greater than the P1V1g = P2V2g
pressure during isothermal compression.
Thermodynamics 309

As molar specific heat of gas at constant volume P P

3 P0
Cv = R
2
(a) P0 (b)
3 5
CP = CV + R = R + R = R ; V0/2 V0 V V0/2 V0 V
2 2 2V0 2V0

CP (5 / 2) R 5 P V
g= = =
CV (3 / 2) R 3 V0
P0

g 5/3 (c) P0/2 (d) V0/2


æV ö æ 6ö
\ From eqn. (1) P2 = ç 1 ÷ P1 = ç ÷ ´ 105 N / m 2
è V2 ø è 2ø T0/4 T0 T
T0
T

= (3)5/3 × 105 = 6.19 × 105 N/m2 Solution : (c)


Process AB is isothermal expansion,
1
[6.19 ´ 105 ´ 2 ´ 10 -3 - 10 -5 ´ 6 ´ 10 -3 ]
BC is isobaric compression and in process CA
Work done =
æ 5ö
1- ç ÷ nRT
è 3ø Pµ Þ P2 µ T

.IN
P

é 2 ´ 102 ´ 3 ù P
= -ê (6.19 - 3) ú
ëê 2 ûú
AL A
P0
= – 3 × 102 × 3.19 = – 957 joule C
B
N
P0/2
[–ve sign shows external work done on the gas]
R

Example 9.
T0/4 T0
A refrigerator is to maintain eatables kept inside at 90C. If
U

room temperature is 360C, calculate the coefficient of Example 11.


A Carnot’s heat engine works with an ideal monatomic
JO

performance.
gas, and an adiabatic expansion ratio 2. Determine its
Solution :
efficiency.
Here, T1 = 36°C = 36 + 273 = 309 K, Solution :
U

T2 = 10°C = 10 + 273 = 283 K V3


ED

Given, r = V = 2 and g for a monatomic gas = 5/3.


T2 283 283 2
COP = = = = 10.9
T1 - T2 309 - 283 26 g -1
æ 1ö
Example 10. Using, h = 1 - ç ÷
è rø
One mole of an ideal gas at pressure P0 and temperature
we have, the required efficiency
T0 is expanded isothermally to twice its volume and then
5
compressed at constant pressure to (V0/2) and the gas is -1
æ 1ö 3
brought back to original state by a process in which P µ V h = 1- ç ÷ = 1 - 0.63 = 0.37 or 37%
è 2ø
(Pressure is directly proportional to volume). The correct
representation of process is
310

ED
U
JO
U
R
N
AL
.IN
PHYSICS

EBD_7179
Thermodynamics 311

1. Which of the following is incorrect regarding first law of 10. One mole of an ideal gas at temperature T was cooled
thermodynamics? P
(a) It is a restatement of principle of conservation of isochorically till the gas pressure fell from P to . Then,
n
energy. by an isobaric process, the gas was restored to the initial
(b) It is applicable to cyclic processes temperature. The net amount of heat absorbed by the gas
(c) It introduces the concept of entropy in the process is
(d) It introduces the concept of internal energy RT
2. Choose the incorrect statement related to an isobaric (a) nRT (b)
n
process.
(c) RT (1 – n–1) (d) RT (n – 1)
V 11. Ice contained in a beaker starts melting when
(a) = constant
T (a) the specific heat of the system is zero
(b) W = PDV (b) internal energy of the system remains constant

.IN
(c) Heat given to a system is used up in raising the (c) temperature remains constant
temperature only. (d) entropy remains constant
(d) DQ > W 12. A uniform sphere is supplied heat electrically at the centre
AL
3. The internal energy of an ideal gas does not depend upon at a constant rate. In the steady state, steady temperatures
(a) temperature of the gas are established at all radial locations r, heat flows outwards
(b) pressure of the gas radial and is ultimately radiated out by the outer surface
N
isotropically. In this steady state, the temperature gradient
(c) atomicity of the gas
varies with radial distance r according to
R

(d) number of moles of the gas.


(a) r–1 (b) r–2
4. During isothermal expansion, the slope of P-V graph
U

(c) r –3 (d) r–3/2


(a) decreases (b) increases
13. For an ideal gas graph is shown for three processes. Process
JO

(c) remains same (d) may increase or decrease 1, 2 and 3 are respectively.
5. During melting of ice, its entropy
(a) increases (b) decreases Work done (magnitude)
U

(c) remains same (d) cannot say


6. Which of the following processes is adiabatic ?
ED

3
(a) Melting of ice 2
(b) Bursting of tyre
(c) Motion of piston of an engine with constant speed 1
DT
(d) None of these Temperature change
7. At a given temperature the internal energy of a substance
(a) in liquid state is equal to that in gaseous state. (a) Isobaric, adiabatic, isochoric
(b) in liquid state is less than that in gaseous state. (b) Adiabatic, isobaric, isochoric
(c) in liquid state is more than that in gaseous state. (c) Isochoric, adiabatic, isobaric
(d) is equal for the three states of matter. (d) Isochoric, isobaric, adiabatic
8. Air conditioner is based on the principle of 14. The efficiency of carnot engine when source temperature
(a) Carnot cycle is T1 and sink temperature is T2 will be
(b) refrigerator T1 - T2 T2 - T1
(c) first low of thermodynamics (a) (b)
T1 T2
(d) None of these
T1 - T2 T1
9. A mass of ideal gas at pressure P is expanded isothermally (c) (d)
T2 T2
to four times the original volume and then slowly
compressed adiabatically to its original volume. Assuming 15. In the equation PVg = constant, the value of g is unity. Then
g to be 1.5, the new pressure of the gas is the process is
(a) 2 P (b) P (a) isothermal (b) adiabatic
(c) 4 P (d) P/2 (c) isobaric (d) irreversible
EBD_7179
312 PHYSICS

16. For adiabatic processes (Letters have usual meanings) (a) D Q = D U + DW (b) DQ = DU - DW
(a) PgV = constant (b) TgV = constant
(c) DQ = DW - DU (d) DQ = – DW - DU
(c) TVg–1 = constant (d) TVg = constant
22. The work done in which of the following processes is equal
PV to the internal energy of the system?
17. The gas law = constant is true for
T (a) Adiabatic process (b) Isothermal process
(a) isothermal changes only (c) Isochoric process (d) None of these
(b) adiabatic changes only 23. Which of the following processes is reversible?
(c) both isothermal and adiabatic changes (a) Transfer of heat by conduction
(d) neither isothermal nor adiabatic change (b) Transfer of heat by radiation
18. When heat is given to a gas in an isothermal change, the (c) Isothermal compression
result will be (d) Electrical heating of a nichrome wire
(a) external work done 24. In thermodynamic processes which of the following
(b) rise in temperature statements is not true?
(c) increase in internal energy (a) In an isochoric process pressure remains constant
(d) external work done and also rise in temperature (b) In an isothermal process the temperature remains
19. Volume of one mole gas changes according to the V = a/T. constant
If temperature change is DT, then work done will be (c) In an adiabatic process PVg = constant

.IN
(a) RDT (b) – RDT
(d) In an adiabatic process the system is insulated from
R the surroundings
(c) DT (d) R (g – 1) DT
g -1 25.
AL Monatomic, diatomic and polyatomic ideal gases each
20. In changing the state of thermodynamics from A to B state, undergo slow adiabatic expansions from the same initial
the heat required is Q and the work done by the system is volume and same initial pressure to the same final volume.
N
W. The change in its internal energy is The magnitude of the work done by the environment on
the gas is
R

(a) Q + W (b) Q – W
Q -W (a) the greatest for the polyatomic gas
U

(c) Q (d) (b) the greatest for the monatomic gas


2
JO

21. If DQ and DW represent the heat supplied to the system (c) the greatest for the diatomic gas
and the work done on the system respectively, then the (d) the question is irrelevant, there is no meaning of slow
first law of thermodynamics can be written as adiabatic expansion
U
ED

1. A gas at 27ºC and pressure of 30 atm. is allowed to expand to (a) 7.5 × 105 joule (b) 7.5 × 105 erg
atmospheric pressure and volume 15 times larger. The final (c) 12 × 105 joule (d) 6 × 105 joule
temperature of the gas is
(a) – 123ºC (b) +123ºC 3. A refrigerator works between 0ºC and 27ºC. Heat is to be
(c) 273ºC (d) 373ºC removed from the refrigerated space at the rate of 50 kcal/
2. A system changes from the state (P1, V1) to (P2, V2) as minute, the power of the motor of the refrigerator is
shown in the figure. What is the work done by the system? (a) 0.346 kW (b) 3.46 kW
(c) 34.6 kW (d) 346 kW
5
6×105 (P2, V2) 4. A perfect gas goes from a state A to another state B by
2

5×105
Pressure in N/m

absorbing 8 × 105 J of heat and doing 6.5 × 105 J of external


4×105
work. It is now transferred between the same two states in
3×105
2×105 another process in which it absorbs 105 J of heat. In the
1×10 (P1, V1) second process
(a) work done by gas is 105 J
(b) work done on gas is 105 J
1 2 3 4 5 (c) work done by gas is 0.5 × 105 J
Volume in metre 3 (d) work done on the gas is 0.5 × 105 J
Thermodynamics 313

5. The temperature of 5 moles of a gas which was held at 15. The temperature at which speed of sound in air becomes
constant volume was changed from 100º to 120ºC. The double of its value at 27° C is
change in the internal energy of the gas was found to be 80 (a) 54°C (b) 327°C
joule, the total heat capacity of the gas at constant volume (c) 927°C (d) None of these
will be equal to 16. 1 gm of water at a pressure of 1.01 × 105 Pa is converted into
(a) 8 joule per K (b) 0.8 joule per K steam without any change of temperature. The volume of 1
(c) 4.0 joule per K (d) 0.4 joule per K g of steam is 1671 cc and the latent heat of evaporation is
540 cal. The change in internal energy due to evaporation
6. A polyatomic gas (g = 4/3) is compressed to 1/8th of its
of 1 gm of water is
volume adiabatically. If its initial pressure is P0, its new
(a) » 167 cal (b) » 500 cal
pressure will be
(c) 540 cal (d) 581 cal
(a) 8P0 (b) 16P0 17. An ideal refrigerator has a freezer at a temperature of 13ºC.
(c) 6P0 (d) 2P0 The coefficient of performance of the engine is 5. The
7. The efficiency of a Carnot engine operating with reservoir temperature of the air (to which heat is rejected) is
temperatures of 100ºC and –23ºC will be (a) 320ºC (b) 39ºC
(c) 325 K (d) 325ºC
100 + 23 100 - 23
(a) (b) 18. One mole of an ideal monoatomic gas is heated at a constant
100 100 pressure of one atmosphere from 0ºC to 100ºC. Then the
work done by the gas is
100 + 23 100 - 23

.IN
(c) (d) (a) 6.56 joule (b) 8.32 × 102 joule
373 373 2
(c) 12.48 × 10 joule (d) 20.8 × 102 joule
8. By what percentage should the pressure of a given mass of 19. The pressure inside a tyre is 4 times that of atmosphere. If the
AL
a gas be increased so as to decrease its volume by 10% at tyre bursts suddenly at temperature 300 K, what will be the
a constant temperature? new temperature?
(a) 8.1 % (b) 9.1 % (a) 300 (4)7/2 (b) 300 (4)2/7
N
(c) 300 (2) 7/2 (d) 300 (4)–2/7
(c) 10.1 % (d) 11.1 %
9. A gas has pressure P and volume V. It is now compressed 20. A monatomic ideal gas expands at constant pressure, with
R

heat Q supplied. The fraction of Q which goes as work


adiabatically to 1/32 times the original volume. Given that
done by the gas is
U

(32)1.4 = 128, the final pressure is (g = 1.4)


(a) P/128 (b) P/32 2
(a) 1 (b)
JO

(c) 32 P (d) 128 P 3


10. At 27ºC a gas is compressed suddenly such that its pressure 3 2
(c) (d)
becomes (1/8) of original pressure. Final temperature will 5 5
U

be (g = 5/3) 21. A carnot’s engine takes 300 calories of heat at 500 K and
rejects 150 calories of heat to the sink. The temperature of
ED

(a) 450 K (b) 300 K


the sink is
(c) –142ºC (d) 327ºC (a) 1000 K (b) 750 K
11. A diatomic gas initally at 18ºC is compressed adiabatically (c) 250 K (d) 125 K
to one eighth of its original volume. The temperature after 22. The source and sink temperatures of a Carnot engine are
compression will be 400 K and 300 K, respectively. What is its efficiency?
(a) 18ºC (b) 887ºC (a) 100% (b) 75%
(c) 327ºC (d) None of these (c) 33.3% (d) 25%
12. Absolute zero is obtained from 23. The volume of a gas is reduced adiabatically to 1/4 of its
volume at 27ºC. If g = 1.4 the new temperature is
1 graph
(a) P–V graph (b) P- (a) (300) 20.4 K (b) (300) 21.4 K
V (c) 300 (4) K0.4 (d) 300 (2)1.4 K
(c) P–T graph (d) V–T graph 24. In pressure-volume diagram, the isochoric, isothermal,
13. An ideal gas heat engine operates in Carnot cycle between isobaric and iso-entropic parts respectively, are
227°C and 127°C. It absorbs 6 × 104 cals of heat at higher
A B
temperature. Amount of heat converted to work is
(a) 4.8 × 104 cal (b) 6 × 104 cal
(c) 2.4 × 104 cal (d) 1.2 × 104 cal C
14. Three moles of an ideal gas kept at a constant temperature P
D
at 300 K are compressed from a volume of 4 litre to 1 litre.
The work done in the process is
V
(a) – 10368 J (b) –110368 J (a) BA, AD, DC,CB (b) DC, CB, BA, AD
(c) 12000 J (d) 120368 J (c) AB, BC, CD, DA (d) CD, DA, AB, BC
EBD_7179
314 PHYSICS

25. Two cylinders fitted with pistons contain equal amount of 33. Figure shows the variation of internal energy (U) with the
an ideal diatomic gas at 300 K. The piston of A is free to pressure (P) of 2.0 mole gas in cyclic process abcda. The
move, while that of B is held fixed. The same amount of heat temperature of gas at c and d are 300 K and 500 K. Calculate
is given to the gas in each cylinder. If the rise in temperature the heat absorbed by the gas during the process.
of the gas in A is 30 K, then the rise in temperature of gas in
B is U
(a) 30 K (b) 18 K a d
(c) 50 K (d) 42 K
26. A Carnot engine works first between 200°C and 0°C and
then between 0°C and –200°C. The ratio of its efficiency in
b c
the two cases is
(a) 1.0 (b) 0.577
P
(c) 0.34 (d) 0.68 P0 2P0
27. A Carnot’s engine works as a refrigerator between 250 K
and 300 K. If it receives 750 calories of heat from the reservoir (a) 400 R ln 2 (b) 200 R ln 2
at the lower temperature, the amount of heat rejected at the (c) 100 R ln 2 (d) 300 R ln 2
higher temperature is 34. The figure shows the P-V plot of an ideal gas taken through
(a) 900 calories (b) 625 calories a cycle ABCDA. The part ABC is a semi-circle and CDA is

.IN
(c) 750 calories (d) 1000 calories half of an ellipse. Then,
28. A Carnot engine is working between 127°C and 27°C. The
increase in efficiency will be maximum when the temperature
P
of
AL 3
A
(a) the source is increased by 50°C
(b) the sink is decreased by 50°C
N
2D
(c) source is increased by 25°C and that of sink is B
decreased by 25°C
R

1
(d) both source and sink are decreased by 25°C each. C
U

29. During an adiabatic process an object does 100J of work


0
and its temperature decreases by 5K. During another
JO

1 2 3 V
process it does 25J of work and its temperature decreases
by 5K. Its heat capacity for 2nd process is (a) the process during the path A ® B is isothermal
(a) 20 J/K (b) 24 J/K
U

(b) heat flows out of the gas during the path B ® C ® D


(c) 15 J/K (d) 100 J/K
(c) work done during the path A ® B ® C is zero
ED

30. The internal energy change in a system that has absorbed


(d) positive work is done by the gas in the cycle ABCDA
2 kcals of heat and done 500 J of work is
35. A thermodynamic system goes from states (i) P1, V to 2P1,
(a) 6400 J (b) 5400 J
V (ii) P, V1 to P, 2V1. Then work done in the two cases is
(c) 7900 J (d) 8900 J
(a) zero, zero (b) zero, PV1
2
31. In an adiabatic process, the pressure is increased by %. (c) PV1, zero (d) PV1, P1V1
3
36. For an isothermal expansion of a perfect gas, the value of
3
If g = , then the volume decreases by nearly DP
2 is equal to
P
4 2
(a) % (b) %
9 3 DV DV
(a) – g1/ 2 (b) –
9 V V
(c) 1% (d) %
4
DV DV
32. A closed gas cylinder is divided into two parts by a piston (c) –g (d) – g2
held tight. The pressure and volume of gas in two parts V V
respectively are (P, 5V) and (10P, V). If now the piston is left 37. One mole of an ideal gas at an initial temperature of T K
free and the system undergoes isothermal process, then does 6R joules of work adiabatically. If the ratio of specific
the volumes of the gas in two parts respectively are heats of this gas at constant pressure and at constant
(a) 2V, 4V (b) 3V, 3V volume is 5/3, the final temperature of gas will be
10 20 (a) (T – 4) K (b) (T + 2.4) K
(c) 5V, V (d) V, V
11 11 (c) (T – 2.4) K (d) (T + 4) K
Thermodynamics 315

38. If DU and DW represent the increase in internal energy If Q1, Q2, Q3 indicate the heat a absorbed by the gas along
and work done by the system respectively in a the three processes and DU1, DU2, DU3 indicate the change
thermodynamical process, which of the following is true? in internal energy along the three processes respectively,
then
(a) DU = - DW , in an adiabatic process
(a) Q1 > Q2 > Q3 and DU1 = DU2 = DU3
(b) DU = DW , in an isothermal process (b) Q3 > Q2 > Q1 and DU1= DU2 = DU3
(c) DU =DW , in an adiabatic process (c) Q1 = Q2 = Q3 and DU1 > DU2 > DU3
(d) DU = - DW , in an isothermal process (d) Q3 > Q2 > Q1 and DU1> DU2 > DU3
39. During an isothermal expansion, a confined ideal gas does 44. Choose the correct relation between efficiency h of a
–150 J of work against its surroundings. This implies that Carnot engine and the heat absorbed (q1) and released
(a) 150 J heat has been removed from the gas by the working substance (q2).
(b) 300 J of heat has been added to the gas q2 q1
(c) no heat is transferred because the process is isothermal (a) h = 1 + (b) h = 1 +
q1 q2
(d) 150 J of heat has been added to the gas
40. When 1 kg of ice at 0°C melts to water at 0°C, the
q1 q2
resulting change in its entropy, taking latent heat of ice to (c) h = 1 - (d) h = 1-
be 80 cal/°C, is q2 q1

.IN
(a) 273 cal/K (b) 8 × 104 cal/K 45. In the given (V – T) diagram, what is the relation between
(c) 80 cal/K (d) 293 cal/K pressure P1 and P2 ?
41. A mass of diatomic gas (g = 1.4) at a pressure of 2
AL
atmospheres is compressed adiabatically so that its V
temperature rises from 27°C to 927°C. The pressure of the
gas in final state is P2
N
(a) 28 atm (b) 68.7 atm P1
R

(c) 256 atm (d) 8 atm


42. A thermodynamic system is taken through the cycle ABCD q2
U

as shown in figure. Heat rejected by the gas during the q1


JO

cycle is T

(a) P2 > P1 (b) P2 < P1


D C
U

2P (c) P2 = P1 (d) Cannot be predicted


46. A system goes from A to B via two processes I and II as
ED

shown in figure. If DU1 and DU2 are the changes in internal


Pressure

energies in the processes I and II respectively, then

p II
P A B

V 3V A B
Volume I

(a) 2 PV (b) 4 PV v
1 (a) relation between DU1 and DU2 can not be determined
(c) PV (d) P V
2 (b) DU1 = DU2
43. An ideal gas goes from state A to state B via three different (c) DU1 < DU2
processes as indicated in the P-V diagram :
(d) DU1 > DU2
47. Which of the following statements about a thermodynamic
process is wrong ?
A 1
P 2 (a) For an adiabatic process DEint = – W
3 B (b) For a constant volume process DEint = + Q
(c) For a cyclic process DEint = 0
(d) For free expansion of a gas DEint > 0
V
EBD_7179
316 PHYSICS

48. In a Carnot engine efficiency is 40% at hot reservoir (b) Statement-1 is true, Statement-2 is true; Statement -2 is a
temperature T. For efficiency 50%, what will be the correct explanation for Statement-1
temperature of hot reservoir? (c) Statement-1 is true, Statement-2 is true; Statement -2 is not
a correct explanation for Statement-1
2 (d) Statement-1 is true, Statement-2 is false
(a) T (b) T
3 49. Statement-1 : At a given temperature the specific heat of a
gas at constant volume is always greater than its specific
4 6 heat at constant pressure.
(c) T (d) T
5 5 Statement-2 : When a gas is heated at constant volume
some extra heat is needed compared to that at constant
Directions for Qs. (49 to 50) : Each question contains
pressure for doing work in expansion.
STATEMENT-1 and STATEMENT-2. Choose the correct answer
(ONLY ONE option is correct ) from the following. 50. Statement -1 : If an ideal gas expands in vacuum in an
insulated chamber, DQ, DU and DW all are zero.
(a) Statement-1 is false, Statement-2 is true Statement-2 : Temperature of the gas remains constant.

.IN
Exemplar Questions AL T T
1. An ideal gas undergoes four different processes from the
same initial state (figure). Four processes are adiabatic, 2 1 1 2
isothermal, isobaric and isochoric. Out of 1, 2, 3 and 4 which
(iii) (iv)
N
one is adiabatic?
p p
R

4
(a) (iv) (b) (ii)
U

p 3 (c) (iii) (d) (i)


4. An ideal gas undergoes cyclic process ABCDA as shown
JO

2
1 in given p-V diagram. The amount of work done by the gas
is
V
U

(a) 4 (b) 3
(c) 2 (d) 1 p
ED

2. If an average person jogs, he produces 14.5 × 103 cal/min. D C


2p0
This is removed by the evaporation of sweat. The amount
of sweat evaporated per minute (assuming 1 kg requires p0
580 × 103 cal for evaporation) is A B
(a) 0.025 kg (b) 2.25 kg
(c) 0.05 kg (d) 0.20 kg V
V0 3V0
3. Consider p-V diagram for an ideal gas shown in figure.
p 1
(a) 6p0V0 (b) –2p0 V0
p=
constant (c) +2p0V0 (d) +4p0V0
V
5. Consider two containers A and B containing identical gases
2 at the same pressure, volume and temperature. The gas in
V container A is compressed to half of its original volume
Out of the following diagrams, which figure represents the isothermally while the gas in container B is compressed to
T-p diagram? half of its original value adiabatically. The ratio of final
T T pressure of gas in B to that of gas in A is
2 2
g-1
g-1
æ1ö
(a) 2 (b) ç ÷
è2ø
(i) 1 (ii) 1
2 2
æ 1 ö æ 1 ö
p p (c) ç ÷ (d) ç ÷
è 1- g ø è g -1 ø
Thermodynamics 317

6. Three copper blocks of masses M1 , M 2 and M3 kg (a) 6 J (b) 10 J


respectively are brought into thermal contact till they reach
(c) 12 J (d) 36 J
equilibrium. Before contact, they were at T1, T2, T3 (T1 > T2
> T3). Assuming there is no heat loss to the surroundings, 10. Which of the following relations does not give the equation
the equilibrium temperature T is (s is specific heat of copper) of an adiabatic process, where terms have their usual
T1 + T2 + T3 meaning?
(a) T=
3 (a) PgT1–g = constant [NEET Kar. 2013]
M1T1 + M 2T2 + M 3T3 (b) P1–g Tg = constant
(b) T =
M1 + M 2 + M 3 (c) PVg = constant
(d) TVg–1 = constant
M T + M 2T2 + M 3T3
(c) T= 11 11. Two Carnot engines A and B are operated in series. The
3( M 1 + M 2 + M 3 )
engine A receives heat from the source at temperature T1
M 1T1 s + M 2T2 s + M 3T3 s and rejects the heat to the sink at temperature T. The second
(d) T =
M1 + M 2 + M 3 engine B receives the heat at temperature T and rejects to
its sink at temperature T2 . For what value of T the
NEET/AIPMT (2013-2017) Questions
efficiencies of the two engines are equal? [NEET Kar. 2013]
A gas is taken through the cycle A ® B ® C ® A, as

.IN
7.
shown in figure. What is the net work done by the gas ? T1 + T2 T1 - T2
(a) (b)
5
2 2
P (10 Pa) [2013]
AL
(c) T1T2 (d) T1T2
7
6 B 12. A thermodynamic system undergoes cyclic process ABCDA
N
5
4 as shown in fig. The work done by the system in the cycle
R

3 A
2 C is : [2014]
1 P
U

3 3 (a) P0V0 C B
V (10 m ) 3P0
0 2 4 6 8 (b) 2P0V0
JO

(a) 1000 J (b) zero 2P0


(c) – 2000 J (d) 2000 J P0 V0 P0 D
(c) A
2
U

8. During an adiabatic process, the pressure of a gas is found


to be proportional to the cube of its temperature. The ratio (d) Zero V0 2V0 V
ED

Cp 13. A monoatomic gas at a pressure P, having a volume V


of for the gas is [2013]
Cv expands isothermally to a volume 2V and then adiabatically
5 to a volume 16V. The final pressure of the gas is : (take g =
(a) 2 (b) 5
3 ) [2014]
3
3 4 (a) 64P (b) 32P
(c) (d)
2 3
P
9. A system is taken from state a to state c by two paths adc (c) (d) 16P
and abc as shown in the figure. The internal energy at a is 64
Ua = 10 J. Along the path adc the amount of heat absorbed 14. Figure below shows two paths that may be taken by a gas
dQ1 = 50 J and the work done dW1 = 20 J whereas along the to go from a state A to a state C.
path abc the heat absorbed dQ2 = 36 J. The amount of work
done along the path abc is [NEET Kar. 2013]
P B C
d c 6×104 Pa

2×104 Pa
p A
a b

2 × 10 3 m3 4 × 10–3 m3
V V
EBD_7179
318 PHYSICS

In process AB, 400 J of heat is added to the system and in (c) Compressing the gas isothermally or adiabatically will
process BC, 100 J of heat is added to the system. The heat require the same amount of work.
absorbed by the system in the process AC will be [2015] (d) Which of the case (whether compression through
(a) 500 J (b) 460 J isothermal or through adiabatic process) requires more
(c) 300 J (d) 380 J work will depend upon the atomicity of the gas.
1 19. A refrigerator works between 4°C and 30°C. It is required to
15. A carnot engine having an efficiency of as heat engine, remove 600 calories of heat every second in order to keep
10
the temperature of the refrigerated space constant. The
is used as a refrigerator. If the work done on the system is
power required is: (Take 1 cal = 4.2 joules) [2016]
10 J, the amount of energy absorbed from the reservoir at
lower temperature is :- [ 2015, 2017] (a) 2.365 W (b) 23.65 W
(a) 90 J (b) 99 J (c) 236.5 W (d) 2365 W
(c) 100 J (d) 1 J 20. Thermodynamic processes are indicated in the following
diagram : [2017]
16. The coefficient of performance of a refrigerator is 5. If the
P
inside temperature of freezer is –20°C, then the temperature
of the surroundings to which it rejects heat is [2015 RS] IV
i f
(a) 41°C (b) 11°C I III
II f
f 700k

.IN
(c) 21°C (d) 31°C
f 500k
17. An ideal gas is compressed to half its initial volume by 300k V
means of several processes. Which of the process results Match the following
in the maximum work done on the gas? [2015 RS]
AL
Column-1 Column-2
(a) Isobaric (b) Isochoric P. Process I A. Adiabatic
N
(c) Isothermal (d) Adiabatic Q. Process II B. Isobaric
R. Process III C. Isochoric
R

18. A gas is compressed isothermally to half its initial volume.


The same gas is compressed separately through an adiabatic S. Process IV D. Isothermal
U

process until its volume is again reduced to half. Then : (a) P ® C, Q ® A, R ® D, S ® B


[2016] (b) P ® C, Q ® D, R ® B, S ® A
JO

(c) P ® D, Q ® B, R ® A, S ® C
(a) Compressing the gas isothermally will require more
work to be done. (d) P ® A, Q ® C, R ® D, S ® B
U

(b) Compressing the gas through adiabatic process will


require more work to be done.
ED
Thermodynamics 319

Hints & Solutions


EXERCISE - 1 a
V=
1. (c) 2. (c) 3. (b) 4. (a) 5. (a) T
6. (b) 7. (b) 8. (b)
a
9. (a) Let P and V be the initial pressure and volume of ideal \ dV = - dT
gas. After isothermal expansion, pressure is P/4. So volume T2
is 4 V.
Let P1 be the pressure after adiabatic compression. W = ò P dV
Then
RT 2 æ a ö
g g =ò - dT
P1 V = (P / 4) (4 V ) a çè T 2 ÷ø
P1 = ( P / 4) (4) 3 / 2 = 2 P W = – R DT
10. (c) The temperature remains unchanged therefore 20. (b) DQ = DU + DW
U f = Ui . Þ DU = DQ - DW = Q - W (using proper sign)

.IN
Also, DQ = DW .
21. (b) From FLOT DQ = DU + DW
In the first step which is isochoric, DW = 0 . AL Q Heat supplied to the system so DQ ® Positive
P and work is done on the system so DW® Negative
In second step, pressure = . Volume V is increased
n Hence +DQ = DU - DW
N
from V on nV. 22. (a) In adiabatic process
DQ = 0
R

P \ DW = – DU
\ W= (nV - V)
n 23. (c) For process to be reversible it must be quasi-static.
U

æ n - 1ö For quasi static process all changes take place


= PV ç
è n ÷ø
infinitely slowly. Isothermal process occur very slowly
JO

so it is quasi-static and hence it is reversible.


= RT(1 - n -1 ) 24. (a) In an isochoric process volume remains constant
U

whereas pressure remains constant in isobaric


11. (c) During melting temperature remains constant process.
ED

12. (b) Flow rate µ gradient × r2.


nRdT
When flow rate is constant, gradient µ r–2. 25. (a) W= g is minimum for a polyatomic gas
13. (d) Isochoric proceess dV = 0 g -1
W= 0 proceess 1 Hence, W is greatest for polyatomic gas
Isobaric : W = P DV = nRDT
EXERCISE - 2
nRDT
Adiabatic | W | = 0<g–1<1 P1 V1 P2 V2 P V T
g -1 = \ T2 = 2 2 1
1. (a) We know that T1 T2 P1 V1
As workdone in case of adiabatic process is more so
process 3 is adiabatic and process 2 is isobaric. Here P1 = 30 atm., P2 = 1 atm, V1 = V (say),
T -T V2 = 15 V, T1 = 27ºC = 27 + 273 = 300ºK and T2 =?
14. (a) Efficiency of carnot engine = h = 1 2
T1 Substituting these values, we get T2 = 150ºK
\ T2 = 150ºK = 150 – 273 = –123ºC.
where T1 = source temperature
(1´105 + 6 ´105 ) (5 - 1)
T2 = sink temperature. 2. (c) W=
2
15. (a) PV = constant represents isothermal process.
16. (c) 17. (c) 18. (a) 6 ´105 ´ 4
= = 12 ´105 joule
19. (b) PV = RT ; 2
RT RT 2 T2 Q
P= = 3. (a) = 2
V a T1 - T2 W
EBD_7179
320 PHYSICS

273 50,000 TL W
= \ 1- =
300 - 273 W TH QS
27 ´ 50, 000 æ T ö
W= cal/min
273 Þ W = QS ç1 - L ÷
è TH ø
W 4.2 ´ 27 ´ 50, 000
P= = Joule/sec
t 60 ´ 273 æ 127 + 273ö
= 6 ´ 104 ç1 -
= 346 watt = 0.346 kW è 227 + 273÷ø
4. (d) dU = dQ - dW = (8 ´ 10 5 - 6.5 ´ 10 5 ) = 1.5 ´105 J = 1.2 ´ 104 cal
14. (a) Work done in an isothermal process is given by
dW = dQ - dU == 10 5 - 1.5 ´ 10 5 = - 0.5 ´ 10 J
5

– ve sign indicates that work done on the gas is V2


W = 2.3026nRT log10
V1
0.5 ´ 10 5 J .
Here, n = 3, R = 8.31 J mol–1 degree–1
5. (c) dU = nC v dT or 80 = 5 ´ C v (120 - 100)
T = 300 K, V1 = 4 litre, V2 = 1 litre.
Cv = 4.0 joule/K
1
4/3 Hence, W = 2.3026 × 3 × 8.31 × 300 × log10
4

.IN
V
6. (b) P0 V 4/3 = P1 æç ö÷ Þ P1 = P0 84/3 = 16Po
è8ø = 17221.15(–2log102)
= – 17221.15 × 2 × 0.3010 = – 10368 J.
T2 250 123 100 + 23
7. (c) h =1- Þ h =1- = = 15.
AL(c) The speed of sound in air, v µ T
T1 373 373 373
v1 T1
P V P ¢ (90 / 100 ) V =
N
8. (d) = v2 T2
T T
R

P ¢ 100 10 v1 (27 + 273)


or = = 1+ Þ 2v =
U

P 90 90 1 T2
P ¢ - P 10
JO

or = = 11 .1 % 1 300
P 90 =
2 T2
g
æVö
U

9. (d) PVg = P1 ç ÷ T2 = 1200 K = (1200 – 273)°C = 927°C


è 32 ø
16. (b) dW = P D V = 1 .01 ´ 10 5 [1671 - 1] ´ 10 - 6 Joule
ED

g
or P1 = (32) , P = (32) 1.4
, P = 128 P
1.01 ´ 167
= cal.
10. (c) T1g P11- g = T2 g P21- g 4.2
11. (d) T1 = 18º C = (273 + 18) = 291 K = 40cal. nearly
and V2 = V1 / 8 Δ Q = mL = 1 ´ 540,
We know that T V g -1 = constant ΔQ = ΔW + ΔU
or , T2 V2 g -1 = T1 V1g -1 or D U = 540 - 40 = 500 cal.
g -1 17. (b) T2 = 273 - 13 = 260,
æV ö
\ T2 = T1 çç 1 ÷
÷ = 291 ´ (8)1 .4 -1
è V2 ø T2
K=
T1 - T2
= 668.5 K = 395 .5º C
12. (c) P µ T if V is constant, where P is pressure of certain 260
5=
amount of gas and T is absolute zero temperature. T1 - 260

L T or T1 - 260 = 52
13. (d) We know that efficiency of carnot engine = I - T
H
T1 = 312 K,
Work output
Also, Efficiency of Heat engine = T2 = 312 - 273 = 39º C
Heat input
Thermodynamics 321

25. (d) In cylinder A, heat is supplied at constant pressure


5
18. (b) dQ = n C p dT = 1 ´ R ´ 100 while in cylinder B heat is supplied at constant volume.
2
(DQ)A = nCP(DT)A
= 2077 .5 J = 20 . 8 ´ 10 2 J and (DQ)B = nCV (DT)B
Given : (DQ)A = (DQ)B
dU = n Cv dT
CP
3 \ (DT)B = ( DT)A
= 1 ´ R ´ 100 CV
2
= 1246.5 J = 1.4 ´ 30
= 42K
\ dU = dQ - dU
[Q for diatomic gas CP = 1.4 ]
= 8.31 ´ 102 J CV
19. (d) Under adiabatic change
T
1- g 26. (b) h = 1- 2
T2 æ P1 ö g T1
=ç ÷
T1 çè P2 ÷ø 273 + 0

.IN
= 1-
273 + 200
1- g
g 200
or T2 = T1 (P1 / P2 ) =
AL 473
1- ( 7 / 5 )
N
(7 / 5 )
\ T2 = 300 ( 4 / 1) ; g = 1.4 = 7 / 5 for air T
h' = 1 - 2
T1
R

or T2 = 300 (4) -2 / 7
(273 - 200)
U

20. (d) Q = nCp DT and W = PΔV = nRΔT. = 1-


273 + 0
JO

5R W 2 200
=
For monatomic gas, C p = . Þ = 473
2 Q 5
U

Q2 T Q T h 200 273
21. (c) h = 1- = 1- 2 Þ 2 = 2
= ´
ED

Q1 T1 Q1 T1 h ' 473 200


Q 2 ´ T1 150 ´ 500 = 0.577
So T2 = = = 250K
Q1 300
750 250
27. (a) =
T W 300 - 250
22. (d) Efficiency, η = 1 – 2
T1 Heat rejected = 750 + 150 = 900 cal.
T1 (source temp.) = 400 K T1 - T2
T2 (sink temp.) = 300 K 28. (b) T1
is maximum in case (b).
300 1 29. (c) For adiabatic process, dU = – 100 J
\h=1 – = = 25%
400 4 which remains same for other processes also.
(1.4 -1) Let C be the heat capacity of 2nd process then
æVö
23. (c) (300) V (1.4-1) = T2 . ç ÷ – (C) 5 = dU + dW
è4ø
= – 100 + 25 = – 75
T2 = (300 ) ( 4) 0.4 \ C = 15 J/K
24. (d) From C to D, V is constant. So process is isochoric. 30. (c) According to first law of thermodynamics
From D to A, the curve represents constant Q = DU + W
temperature. So the process is isothermal. DU = Q – W
From A to B, pressure is constant . So, the process is = 2 × 4.2 × 1000 – 500
isobaric. = 8400 –500 = 7900 J
BC represents constant entropy. 31. (a) PV3/2 = K
EBD_7179
322 PHYSICS

3 P1V1 - P2 V2 nRT1 - nRT2 nR (T1 - T2 )


log P + log V = log K 37. (a) W= = =
2 g -1 g -1 g -1
DP 3 DV
+ =0 n = 1, T1 = T Þ R (T - T2 ) = 6 R Þ Tf = ( T – 4)K
P 2 V 5 / 3 -1
DV 2 DP 38. (a) By first law of thermodynamics,
=-
V 3 P DQ = DU + DW
DV æ 2 ö æ 2ö In adiabatic process, DQ = 0
or = ç- ÷ çè ÷ø
V è 3ø 3 \ DU = -DW
In isothermal process, DU = 0
4
=- \ DQ = DW
9
39. (a, d) If a process is expansion then work done is positive
32. (a) The volume on both sides will be so adjusted that the
so answer will be (a).
original pressure × volume is kept constant as the
But in question work done by gas is given –150J so
piston moves slowly (isothermal change)
that according to it answer will be (d).
P5V = P'V' ........... (1)
40. (d) Change in entropy is given by
10PV = P'V'' ........... (2)
From (1) and (2), V'' = 2V' dQ DQ mLf

.IN
and from V' + V'' = 6V dS = or DS = =
T T 273
V' = 2V, V'' = 4V
33. (a) Change in internal energy for cyclic process (DU) = 0. 1000 ´ 80
DS = = 293cal / K.
For process a ® b, (P-constant)
AL 273
41. (c) T1 = 273 + 27 = 300K
Wa ®b = P DV
T2 = 273 + 927 = 1200K
N
= nR DT For adiabatic process,
P1–g Tg = constant
R

= - 400 R Þ P11–g T1g = P21–g T2g


U

For process b ® c, (T-constant) 1-g g


æP ö æ T1 ö
Wb®c = -2R (300) ln 2 Þç 2÷
JO

=ç ÷
è P1 ø èT ø 2
For process c ® d, (P-constant)
1-g g
Wc®d = +400R æP ö æ T2 ö
U

Þç 1÷ =ç ÷
For process d ® a, (T-constant) èT ø 2 èT ø
1
ED

Wd®a = +2R (500) ln 2


1-1.4 1.4
Net work (DW) = Wa ®b + Wb®c + Wc®d + Wd®a æ P1 ö æ 1200 ö

çè P ÷ø
2 è 300 ÷ø
DW = 400R ln 2
-0.4
\ dQ = dU + dW, first law of thermodynamics æ P1 ö
çè P ÷ø = (4)1.4
\ dQ = 400 R ln 2. 2
34. (b,d)(a) Process is not isothermal.
(b) Volume decreases and temperature decreases 0.4
æ P2 ö
DU = negative, çè P ÷ø = 41.4
So, DQ = negative 1
(c) Work done in process A ® B ® C is positve æ 1.4 ö æ7ö
(d) Cycle is clockwise, so work done by the gas is positive çè ÷ çè ÷ø
0.4 ø 2
P2 = P1 4 = P1 4
35. (b) (i) Case ® Volume = constant Þ ò PdV = 0 = P1 7
(2 ) = 2 × 128 = 256 atm
(ii) Case ® P = constant 42. (a) Q Internal energy is the state function.
2V1 2V1 \ In cyclie process; DU = 0
Þ òV1 PdV = P ò
V1
dV = PV1 According to 1st law of thermodynamics
36. (b) Differentiate PV = constant w.r.t V DQ = DU + W
Þ P DV + V DP = 0 So heat absorbed
DP DV DQ = W = Area under the curve
Þ =– = – (2V) (P) = – 2PV
P V
So heat rejected = 2PV
Thermodynamics 323

43. (a) Initial and final condition is same for all process EXERCISE - 3
DU1 = DU2 = DU3
Exemplar Questions
from first law of thermodynamics
DQ = DU + DW 1. (c) For the straight line in the graph denoted by 4, that
Work done shows pressure is constant, so curve 4 represents an
DW1 > DW2 > DW3 (Area of P.V. graph) isobaric process.
So DQ1 > DQ2 > DQ3
W
44. (d) Efficiency h = and W = q1 – q2 4
q1
p 3
q – q2 q
\ h= 1 = 1- 2 2
q1 q1 1
V
45. (b) P1 > P2
For the straight line in graph denoted by 1, that shows
P2 volume is constant, so curve 1 represents isochoric
process. Out of curves 3 and 2, curve 2 is steeper.

.IN
P1 Hence, curve 2 is adiabatic and curve 3 is isothermal.
V
2.
AL (a) As we know that amount of sweat evaporated/minute
q2 Sweat produced/ minute
=
q1 T Number of calories required for evaporation/kg
N
As V = constant Þ Pµ T Amount of heat produced per minute in jogging
=
Hence from V–T graph P1 > P2 Latent heat (in cal/kg)
R

46. (b) Change in internal energy do not depend upon the 580 × 10 3 calories are needed to convert
path followed by the process. It only depends on initial
U

1 kg H2O into stream.


and final states i.e.,
1 cal. will produce sweat = 1 kg/ 580 × 103
JO

DU1 = DU2
14.5 × 103 cal will produce (sweat)
47. (a) For adiabatic process Q = 0.
By first law of thermodynamics, 14.5 ´ 103 145
= kg= kg/m
U

Q = DE + W 580 ´ 103 580


Þ DEint = – W.
ED

= 0.025 kg.
48. (d) The efficiency of carnot's heat engine 3. (c) According to given P–V diagram that
T2 pV = constant
h = 1–
T1 So we can say that the gas is going through an
isothermal process.
where T2 is temperature of sink, and T 1 is temperature
of hot reservoir or source. If pressure (P) increase at constant temperature volume
V decreases, the graph (iii) shows that pressure (P) is
T2 smaller at point 2 and larger at point 1 point so the gas
When efficiency is 40% i.e. h = 40/100 = 1 –
T1 expands and pressure decreases. Hence verifies option
(c).
2 T
or = 1 – 2 [T1 = T(given)] 4. (b) According to the given p-V diagram.
5 T
Work done in the process ABCD
3 = (AB) × BC = (3V0 – V0) × (2p0 – p0)
\ T2 = T
5 = 2V0 × p0 = 2p0V0
Now, when efficiency is 50% Here the direction of arrow is anti-clockwise, so work
done is negative.
3
T Hence, work done by the gas = –2p0V0
50 6
h= = 1 – 5 \ T1 = T That shows external work done on the system.
100 1 5
T
5. (a) Let us consider the p-V diagram for container A
49. (a) 50. (c) (isothermal) and for container B (adiabatic).
EBD_7179
324 PHYSICS

8. (c) According to question P µ T3


2 2 But as we know for an adiabatic process the
p p g
g -1
pressure P µ T .
p0 1 p0 1 g 3 Cp 3
V0 2V0 V0 2V0 So, =3Þg= or,, =
V V g -1 2 Cv 2
Container A Container B 9. (a) From first law of thermodynamics
(Isothermal) (Adiabatic) Qadc = DUadc + Wadc
In both process compression of the gas. 50 J = DUadc + 20 J
For isothermal process (gas A) during 1® 2 DUadc = 30 J
p1V1 = p2V2 (QV1 = 2V0 , V2 = V0 ) Again, Qabc = DUabc + Wabc
Wabc = Qabc – DUabc
p0 (2V0 ) = p2 (V0 )
= Qabc – DUadc
p2 = 2 p0 = 36 J – 30 J
For adiabatic process, (gas B) during (1® 2) = 6J
10. (a) Adiabatic equations of state are

.IN
p1V1g = p2V2g (Q V1 = 2V0 , V2 = V0 )
PVg = constant
p0 (2V0 )g = p2 (V0 ) g TVg–1 = constant
P1–gTg = constant.
æ 2V ö
g
AL
p2 = ç 0 ÷ p0 = (2) g p0 T
èV ø 0 11. (d) Efficiency of engine A, h1 = 1 - ,
T1
N
So, ratio of final pressure
R

æ ( p ) ö (2) g p0 T2
= ç 2 B÷ = = 2g -1 Efficiency of engine B, h2 = 1 -
è ( p1 ) A ø 2 p0 T
U

Here, h1 = h2
where, g is ratio of specific heat capacities for the gas.
JO

Hence, verifies the option (a). T T2


\ T = T Þ T = T1T2
6. (b) Consider the equilibrium temperature of the system is 1
T.
U

12. (d) Work done by the system in the cycle


Let us consider, T1, T2 < T < T3. = Area under P-V curve and V-axis
ED

As given that, there is no net loss to the surroundings.


1
Heat lost by M3 = Heat gained by M1 = (2P0 - P0 )(2V0 - V0 ) +
2
+ Heat gained by M2
M 3s (T3 - T ) = M1s(T - T1 ) + M 2 s (T - T2 ) é æ 1ö ù
ê - çè 2 ÷ø (3P0 - 2P0 )(2V0 - V0 ) ú
ë û
M 3 sT3 - M 3 sT = M 1 sT - M 1 sT1
+ M 2 sT - M 2 sT2 P0 V0 P0 V0
= - =0
(where, s is specified heat of the copper material) 2 2
13. (c) For isothermal process P1V1 = P2V2
T [ M1 + M 2 + M 3 ] = M 3T3 + M1T1 + M 2T2
P
M T + M 2T2 + M 3T3 Þ PV = P2(2V) Þ P2 =
T= 11 2
M1 + M 2 + M 3
For adiabatic process
NEET/AIPMT (2013-2017) Questions g g
P2 V2 = P3V3
7. (a) Wnet = Area of triangle ABC
1 æ Pö g g
= AC × BC Þ çè ÷ø (2v) = P316v)
2 2

1 5/3
= × 5 × 10–3 × 4 × 105 = 1000 J 3 æ 1ö P
2 Þ P3 = ç ÷ =
2 è 8ø 64
Thermodynamics 325

14. (b) In cyclic process ABCA


Qcycle = Wcycle P
QAB + QBC + QCA = ar. of DABC
1
+ 400 + 100 + QC®A = (2 × 10–3) (4 × 104) Isothermal Adiabatic
2
Þ QC ® A = – 460 J
Þ QA ® C = + 460 J Isobaric

1
15. (a) Given, efficiency of engine, h =
10 V
work done on system W = 10J
Coefficient of performance of refrigerator 18. (b) Wext = negative of area with volume-axis
W(adiabatic) > W(isothermal)
1 9
1-
Q 1- h 10 = 10 P
b= 2 = =
W h 1 1 =9
10 10 Adiabatic

.IN
Energy absorbed from reservoir
Q2 = bw
Isothermal
Q2 = 9 × 10 = 90 J AL O
16. (d) Coefficient of performance, V0 2V0 V
T2 19. (c) Coefficient of performance of a refrigerator,
Cop =
N
T1 - T2
Q2 T2
b= W = (Where Q2 is heat removed)
R

273 - 20 253 T1 - T2
5= =
T1 - (273 - 20) T1 - 253
U

Given: T2 = 4°C = 4 + 273 = 277 k


5T1 – (5 × 253) = 253 T1 = 30°C = 30 + 273 = 303 k
JO

5T1 = 253 + (5 × 253) = 1518 600 ´ 4.2 277


\ b= =
1518 W 303 - 277
\ T1 = = 303.6
U

5 Þ W = 236.5 joule
or, T1 = 303.6 – 273 = 30.6 @ 31°C
ED

W 236.5 joule
17. (d) Since area under the curve is maximum for adiabatic Power P = = = 236.5 watt.
t 1sec
process so, work done (W = PdV) on the gas will be
maximum for adiabatic process 20. (a) Process I volume is constant hence, it is isochoric
In process IV, pressure is constant hence, it is isobaric
EBD_7179
326 PHYSICS

13 Kinetic Theory

BEHAVIOUR OF GASES A relation connecting macroscopic properties P, V and T of


Ideal gas : In an ideal gas, we assume that molecules are point a gas describing the state of the system is called equation
masses and there is no mutual attraction between them. The ideal of state.
gas obeys following laws : The equation of state for an ideal gas of n mole is
(i) Boyle’s law : According to Boyle’s law for a given mass of

.IN
PV
ideal gas, the pressure of a ideal gas is inveresly proportional = nR
T
to the volume at constant temperature AL where R is universal gas constant whose value is
1 R = 8.31 J/mol K and R = NAk, where k is Boltzmann’s
i.e., Pµ Þ PV = constant
V constant (NA is Avagadro number). n is the number of moles
(ii) Charle’s law : For a given mass, the volume of a ideal gas of a gas and
N
is proportional to temperature at a constant pressure
m N
V n= = , NA is Avagadro's number
R

i.e., V µ T Þ = constant M NA
T
where m is the mass of a gas, N is the number of molecules
U

(iii) Gay-Lussac’s law : For a given mass of ideal gas, the and M is the molecular weight of a gas.
pressure is proportional to temperature at constant volume
JO

P Equation of Real Gas :


i.e., P µT = constant The real gas follows Vander Wall’s law. According to this:
T
U

(iv) Avogadro’s law : According to Avogadro’s law, the number æ n2a ö


of molecules of all gases are same at same temperature, ç p + 2 ÷ (V - nb ) = nRT;
è V ø
ED

pressure and volume


i.e., N A1 = N A2 for same P, V and T. here a and b are Vander Wall’s constant
The value of Avagadro number is 6.02 × 1023 molecules. Critical Temperature, Volume and Pressure :
(v) Graham’s law : (a) The temperature at or below which a gas can be liquefied
(a) At constant temperature and pressure, the rms speed by applying pressure alone is called critical temperature.
of diffusion of two gases is inversely proportional to
the square root of the relative density It is given by Tc = 8a
27 Rb
1 ( vrms )1 d2
i.e., vrms a Þ = (b) The volume of gas at a critical temperature TC is called
d ( vrms )2 d1
critical volume VC, where VC = 3b
(b) According to Graham’s law, the rate of diffusion of a
(c) The pressure of gas at a critical temperature TC is called
gas is inversely proportional to the square root of its
density, provided pressure and temperature are a
critical pressure PC, where PC =
constant 27b2
1 Gas Equation
r (rate of diffusion) µ
ρ (i) The gases found in nature are real gases.
(vi) Dalton’s law: The pressure exerted by a gaseous mixture is (ii) The real gas do not obey ideal gas equation but they obey
equal to sum of partial pressure of each component gases vander wall's gas equation
present in the mixture, æ a ö
i.e., P = P1 + P2 + P3+.................. Pn çè P + 2 ÷ø ( V - b ) = RT
V
Kinetic Theory 327

(a) 'a' depends upon the intermolecular force and the


1 R
nature of gas. mC 2 = T = kT ( k = R is Boltzmann constant)
(b) 'b' depends upon the size of the gas molecules and 3 N N
represents the volume occupied by the molecules of
1 3
the gas. or mC 2 = kT ....(vi)
(iii) The molecules of real gas have potential energy as well as 2 2
kinetic energy. 1 2 3
(iv) The real gas can be liquefied and solidified. or mvr .m.s = kT
2 2
(v) The real gases like CO2, NH3, SO2 etc. obey Vander Wall's
equn at high pressure and low temperature. 3kT
or vr2.m.s = ....(vii)
KINETIC THEORY OF AN IDEAL GAS m
The basic assumptions of kinetic theory are :
(i) A gas consist of particles called molecules which move 3nkT 3RT
or vr.m.s = =
randomly in all directions. nm M
(ii) The volume of molecule is very small in comparison to the
(a) It is clear from eq.(vi) that at a given temperature, the average
volume occupied by gas i.e., the size of molecule is
translational kinetic energy of any gas molecules are equal
infinitesimely small.
(iii) The collision between two molecules or between a molecule i.e., it depends only on temperature.
and wall are perfectely elastic and collision time (duration (b) From eqn. (vii) It is clear that

.IN
of collision) is very small. (a) vr.m.s µ T
(iv) The molecules exert no force on each other or on the walls
of containers except during collision. 1
AL (b) vr.m.s µ , where M is moleculer mass of the gas.
(v) The total number of molecules are large and they obey M
Newtonian mechanics.
DEGREE OF FREEDOM
N
1
pV = mn C 2 ....(i) The degree of freedom of a particle is the number of independent
3
R

modes of exchanging energy or the number of idependent motion,


where C 2 is called mean square velocity and C 2 is called which the particle can undergo.
U

For monatomic (such as helium, argon, neon etc.) gas the


root mean square velocity
JO

molecules can have three independent motion i.e., it has


C12 + C22 + ...............Cn2 3 degree of freedom, all translational.
i.e, C2 = ....(ii)
n
U

and n r.m.s. = C 2 ....(iii)


ED

where m = mass of one molecule and n = number of X


molecules
M 2 1 2 Z
ÞP= C = ρv r.m.s ....(iv)
3V 3
For a diatomic gas molecules such as H2, O2, N2, etc. it has two
2 é1 2 ù æ Mö independent rotational motion besides of three independent
or P= rv r.m.s ú çè where M = m ´ n and r = ÷ø
3 êë 2 û V translational motion, so it has 5 degree of freedom.

2 Y Y
or P= E ....(v)
3
where E is translational kinetic energy per unit volume of
the gas. It is clear that pressure of ideal gas is equal to 2/3 X X
of translational kinetic energy per unit volume.
Z Z
Diatomic Triatomic
Kinetic interpretation of temperature : From eqn. (iv), we get
1 In polyatomic gas molecules such as CO2, it can rotate about any
PV = MC 2 of three coordinate axes. It has six degree (three translational
3
+three rotational) of freedom. At high temperature the molecule
For 1 mole of a gas at temperature T :
can vibrate also and degree of freedom due to vibration also
1 arises, but we neglect it.
PV = RT so mC 2 = RT
3
EBD_7179
328 PHYSICS

MEAN FREE PATH


æ ¶Q ö æ ¶U ö
CV = ç ÷ =ç ÷ ....(v)
The distance covered by the molecules between two successive è ¶T ø V è ¶T ø V
collisions is called the free path.
Nf K f R
The average distance covered by the molecules between two so CV = = ....(vi)
successive collisions is called the mean free path 2 2
where R is universal gas constant.
1 K BT Now by Mayer’s formula,
i.e., l= =
2 . pnd 2
2pd 2 P æ fö
CP - CV = R Þ CP = R ç1 + ÷ ... (vii)
where, n = number of molecules per unit volume è 2ø
d = diameter of each molecule CP 2 C
So ratio of specific heat g = is g = 1 + = P ...(viii)
KB = Boltzmann’s constant CV f CV
T = temperature where f is degree of freedom of one molecule.
P = pressure 3 5
(a) For monatomic gas, f = 3, Cv = R , Cp = R
Mean free path depends on the diameter of molecule (d) and the 2 2
number of molecules per unit volume n.
2
At N.T.P., l for air molecules is 0.01 µm. so g = 1 + = 1.67

.IN
3
LAW OF EQUIPARTITION OF ENERGY
5 7
If we dealing with a large number of particles in thermal (b) For diatomic gas, f = 5, Cv = R , Cp = R
2 2
equilibrium to which we can apply Newtonian mechanics, the
AL
energy associated with each degree of freedom has the same 2
g = 1+ = 1.4
1 5
N
average value (i.e., kT ), and this average value depends on
2
temperature. (c) For Polyatomic gas, f = 6, Cv = 3R , C p = 4 R
R

From the kinetic theory of monatomic ideal gas, we have 2


g = 1+ = 1.33
U

1 3 6
mC 2 = kT ....(i)
JO

2 2
Keep in Memory
or
Energy of molecule
U

1. Real gases behave like perfect gas at high temperature


1
= (number of degree of freedom) × kT ....(ii) and low pressure.
ED

2 In real gas, we assume that the molecules have finite size


So it is clear from equation (i) and (ii) that monatomic gas has ant intermolecular attraction acts between them.
three degree of freedom and energy associated per degree of 2. Real gases deviate most from the perfect gas at high pressure
1 and low temperature.
freedom is kT (where k is Boltzmann’s constant)
2 3. Gaseous state of matter below critical temperature is called
Since we know that internal energy of an ideal gas depends only vapours. Below critical temperature gas is vapour and above
on temperature and it is purely kinetic energy. Let us consider an critical temperature vapour is gas.
1 mole ideal gas, which has N molecules and f degree of freedom, 4. Random motion of the consituents of the system involving
then total internal energy exchange of energy due to mutual collisions is called
U = (total number of molecules) thermal motion.
× (degree of freedom of one molecule) 5. Total kinetic energy or internal energy or total energy does
× (energy associated with each degree of freedom) not depend on the direction of flow of heat. It is determined
by the temperature alone.
1
=(N) (f) × ( kT ) 6. The internal energy of a perfect gas consists only of kinetic
2
energy of the molecules. But in case of the real gas it
1 consists of both the kinetic energy and potential energy of
U = Nf kT ....(iii)
2 inter molecular configuration.
Differentiating eq.(iii) w.r.t T, at constant volume, we get,
Distribution of Molecular Speeds :
æ ¶U ö NfK The speed of all molecules in a gas is not same but speeds of
ç ÷ = ....(iv)
è ¶T øV 2 individual molecules vary over a wide range of magnitude .
Maxwell derived the molecular distribution law (by which we
Now molar, specific heat at constant volume is defined as
can find distribution of molecules in different speeds) for sample
Kinetic Theory 329

of a gas containing N molecules, which is Root Mean Square Speed


3/ 2 In this case we multiply the number of particles in each speed
æ m ö 2 / 2 kT
interval by v2 characterstic of that interval; sum of these products
N( v) = 4 pNç ÷ v 2 e - mv ...(1)
è 2 pkT ø over all speed interval and divide by N
where N(v)dv is the number of molecules in the gas sample having ¥
speeds between v and v + dv. i.e., v 2 ò
= o
N (v)v 2 dv
....(2)
where T = absolute temperature of the gas N
m = mass of molecule Root mean square speed is defined as
k = Boltzmann’s constant
3kT kT
The total number of molecules N in the gas can be find out by vr.m.s = v 2 = = 1.73
integrating eqation (1) from 0 to ¥ i.e., m m
¥ Most Probable Speed
N = ò N (v)dv ....(2)
0 It is the speed at which N(v) has its maximum value (or possessed
by large number of molecules), so
No. of molecules per

T1 T2 > T1 d 2kT kT
Ζ N(v)∴ < 0 Þ vp <
unit speed interval

< 1.41 ....(3)


T2 du m m
It is clear from eqn. (1), (2) and (3)

.IN
vr .m.s > v > v p
The root mean square velocity of a particle in thermal
system is given by
v
AL
vp
v
v rms 3RT 3kT
Crms = =
Speed (m/sec) M m
N
where R - universal gas constant
Figure shows Maxwell distribution law for molecules at two
R

T - temperature of gas
different temperature T1 and T2(T2 > T1). m - mass of the gas
U

The number of molecules between v1 and v2 equals the area M - molecular weight of gas or mass of one mole
under the curve between the vertical lines at v1 and v2 and the of a gas.
JO

total number of molecules as given by equation (2) is equal to The average speed of the gas molecules is given by
area under the distribution curve.
8kT 8 RT
The distribution curve is not symmetrical about most probable Cav = =
U

pm pM
speed, vP,(vP is the speed, which is possesed in a gas by a large
where M is the mass of one mole and m is the mass of one
ED

number of molecules) because the lowest speed must be zero,


whereas there is no limit to the upper speed a molecule can attain. particle.
Most probable speed is that with which the maximum number
It is clear from fig.1 that
of molecules move. It is given by
vrms(root mean square) > v (average speed of molecules)
> (most probable speed) vP 2 2RT 2 kT
Vp = crms = =
3 M m
AVERAGE, ROOT MEAN SQUARE AND MOST PROBABLE
SPEED The most probable speed, the average speed as well as
root mean square speed increases with temperature.
Average speed Example 1.
To find the average speed v , we multiply the number of particles The root mean square velocity of the molecules in a sample
in each speed interval by speed v characteristic of that interval. of helium is 5/7th that of the molecules in a samzple of
hydrogen. If the temperature of the hydrogen gas is 0ºc,
We sum these products over all speed intervals and divide by
then find the temperature of helium sample.
total number of particles Solution :

ò
¥
N (v)vdv (v r.m.s )He (3KTHe / m He ) THe m H
i.e., v= o (where summation is replaced by = = ´
N (v r.m.s )H 3KTH / m H TH m He
integration because N is large)
THe ´ 1 5 1 THe
= Þ =
8kT kT 273 ´ 4 7 2 273
hv = = 1.59 ....(1)
pm m or THe = 557.14º K = 284.14º C
EBD_7179
330 PHYSICS

Example 2. Solution : (b)


Let v, v rms and v P respectively denote the mean speed, The Vander Wall’s equation of state is
r.m.s. speed, and most probable speed of the molecules in æ a ö RT a
an ideal monatomic gas at absolute temperature T. The çç P + ÷÷ ( V - b ) = RT ; or P = - 2
è V2 ø V-b V
mass of a molecule is m, then which of the following is
correct ? At the critical point, P = Pc, V = Vc, and T = Tc ;

(a) No molecule can have a speed greater than 2 v max RTc a


\ Pc = - 2 ... (i)
Vc - b Vc
(b) No molecule can have a speed less than vp / 2
dPc
At the critical point on the isothermal, =0
(c) v p < v < v r.m.s. dVc
(d) None of these - RTc 2a RTc 2a
\ 0= - ; or = ...(ii)
Solution : (c) (Vc - b) 2
Vc3 (Vc - b) 2
Vc3
According to kinetic theory of gases, a molecule of a gas
can have speed such that 0< v < ¥ , so the alternatives (a) 2
and (b) can never be correct, Since Also at critical point, d Pc = 0

.IN
dVc2
3k T 8k T 2kT
vr.m.s. = ; vav = ; and vmp =
m 3m m 2 RTc
AL \ 0= -
6a
; or
2 RTc
=
6a
...(iii)
3
(Vc - b) Vc4 (Va - b) 3
Vc4
so vr.m.s. > vav > v p i.e. v p < v < vr.m.s. ;
N
Example 3. Dividing eqn. (iii) by (ii), we get,
A gas cylinder of 50 litres capacity contained helium at
R

1 1
80 atmospheric pressure. Due to slow leakage it was found (Vc - b) = Vc or Vc = 3b ...(iv)
2 3
U

after a while that the pressure had dropped to 40


atmosphere. Find the proportion of the gas that has Putting this value in eqn. (ii), we get,
JO

escaped and also the volume the escaped gas would RT 2a 8a


occupy at atmospheric pressure. = or Tc = ...(v)
4 b2 27 b 3 27 b R
U

Solution :
Volume of the remaining gas Putting the values of Vc and Tc in eqn. (i), we get,
ED

= volume of cylinder = 50 l at 40 atmospheric pressure


The volume of this gas at 80 atmospheric pressure, using R æ 8a ö a a
Pc = çç ÷÷ - =
2
Boyle’s law p1V1 = p2V2 2 b è 27 b R ø 9 b 27 b 2

40 ´ 50 = 25 l Keep in Memory
=
80
Volume of the gas (at 80 atmospheric pressure) which has 1. Brownian motion, provides a direct evidence for the
escaped = 50 – 25 = 25 l existance of molecules and their motion. The zig-zag motion
of gas molecules is Brownian motion.
Mass of gas escaped 25 1
\ Original mass of gas = 50 = 2 2. Average speed v =
8k B T
=
8 RT
=
8PV
πm πM πM
Again using Boyle’s law, 3. Root mean square speed,
1 × V = 80 × 25 or V = 2000 litre
3k BT 3RT 3PV
Example 4. vrms = = =
m M M
In Vander Wal’s equation the critical pressure Pc is given
by 2k BT 2 RT 2 PV
4. Most probable speed Vmp = = =
m M M
a 27a b2
(a) 3b (b) 2
(c) (d) 5. Vrms : V : Vmp = 1.73 : 1.60 : 1.41
27 b b2 a
Vrms > V > Vmp.
Kinetic Theory 331

.IN
AL
N
R
U
JO
U
ED
EBD_7179
332 PHYSICS

1. The kinetic theory of gases 1 2


(c) the average rotational K.E. of a molecule is mv
(a) explains the behaviour of an ideal gas. 4
(b) describes the motion of a single atom or molecule. 5 2
(d) the average K.E. of a molecule is mv
(c) relates the temperature of the gas with K.E. of atoms 6
of the gas 9. The adjoining figure shows graph of pressure and volume
(d) All of the above of a gas at two tempertures T1 and T2. Which of the following
2. The ratio of principal molar heat capacities of a gas is inferences is correct?
maximum for P
(a) a diatomic gas
(b) a monatomic gas
(c) a polyatomic gas having linear molecules.
(d) a polyatomic gas having non-linear molecules. T2
T1
3. The correct statement of the law of equipartition of energy
V
is
(a) T1 > T2 (b) T1 = T2

.IN
(a) the total energy of a gas is equally divided among all
the molecules. (c) T1 < T2 (d) None of these
(b) The gas possess equal energies in all the three 10.
AL At constant pressure, the ratio of increase in volume of an
directions x,y and z-axis. ideal gas per degree rise in kelvin temperature to its original
(c) the total energy of a gas is equally divided between volume is (T = absolute temperature of the gas) is
kinetic and potential energies. (a) T 2 (b) T (c) 1/T (d) 1/T 2
N
(d) the total kinetic energy of a gas molecules is equally 11. The K.E. of one mole of an ideal gas is E = (3/2) RT. Then Cp
divided among translational and rotational kinetic will be
R

energies. (a) 0.5 R (b) 0.1 R


U

4. The internal energy of an ideal gas is (c) 1.5 R (d) 2.5 R


(a) the sum of total kinetic and potential energies. 12. The root mean square speed of the molecules of a diatomic
JO

(b) the total translational kinetic energy. gas is v. When the temperature is doubled, the molecules
(c) the total kinetic energy of randomly moving molecules. dissociate into two atoms. The new root mean square speed
(d) the total kinetic energy of gas molecules.
of the atom is
U

5. A fixed mass of gas at constant pressure occupies a volume


V. The gas undergoes a rise in temperature so that the root (a) 2v (b) v (c) 2v (d) 4v
ED

mean square velocity of its molecules is doubled. The new 13. Which of the following formula is wrong?
volume will be
R gR
(a) V/2 (b) V / 2 (a) CV = (b) CP =
g –1 g –1
(c) 2 V (d) 4 V
6. In kinetic theory of gases, it is assumed that molecules (c) Cp / CV = g (d) Cp – CV = 2R
(a) have same mass but can have different volume 14. For a gas if ratio of specific heats at constant pressure and
(b) have same volume but mass can be different volume is g then value of degrees of freedom is
(c) have different mass as well as volume 3g –1 2 9 25
(d) have same mass but negligible volume. (a) (b) (c) ( g – 1) (d) (g – 1)
2g –1 g –1 2 2
7. Gases exert pressure on the walls of the container because 15. In the kinetic theory of gases, which of these statements
the gas molecules is/are true ?
(a) possess momentum
(i) The pressure of a gas is proportional to the mean speed
(b) collide with each other
of the molecules.
(c) have finite volume
(d) obey gas laws. (ii) The root mean square speed of the molecules is
8. Let v denote the rms speed of the molecules in an ideal proportional to the pressure.
diatomic gas at absolute temperature T. (iii) The rate of diffusion is proportional to the mean speed
The mass of a molecule is ‘m’ Neglecting vibrational energy of the molecules.
terms, the false statement is (iv) The mean translational kinetic energy of a gas is
proportional to its kelvin temperature.
(a) a molecule can have a speed greater than 2v
(a) (ii) and (iii) only (b) (i),(ii)and (iv) only
(b) v is proportional to T (c) (i) and (iii) only (d) (iii) and (iv) only
Kinetic Theory 333

16. A gas mixture consists of molecules of type 1, 2 and 3, with (a) 2 P (b) P
molar masses m1 > m2 > m3. vrms and K are the r.m.s. speed (c) P/2 (d) 4 P
and average kinetic energy of the gases. Which of the 21. The perfect gas equation for 4 gram of hydrogen gas is
following is true? (a) PV = RT (b) PV = 2RT
(a) (vrms)1 < (vrms)2 < (vrms)3 and ( K )1 = ( K ) 2 = ( K )3 1
(c) PV= T
RT (d) PV = 4RT
2
(b) (vrms)1 = (vrms)2 = (vrms)3 and ( K )1 = ( K ) 2 > ( K )3
22. Maxwell's laws of distribution of velocities shows that
(c) (vrms)1 > (vrms)2 > (vrms)3 and ( K )1 < ( K ) 2 > ( K )3 (a) the number of molecules with most probable velocity
is very large
(d) (vrms)1 > (vrms)2 > (vrms)3 and ( K )1 < ( K ) 2 < ( K ) 3 (b) the number of molecules with most probable velocity
17. A sample of an ideal gas occupies a volume of V at a pressure is very small
(c) the number of molecules with most probable velocity
P and absolute temperature. T. The mass of each molecule
is zero
is m. The equation for density is (d) the number of molecules with most probable velocity
(a) m k T (b) P/k T is exactly equal to 1
(c) P/(k T V) (d) P m/k T 23. According to kinetic theory of gases, which one of the
following statement(s) is/are correct?
18. The value of critical temperature in terms of van der Waal’s (a) Real gas behave as ideal gas at high temperature and

.IN
constant a and b is given by low pressure.
8a a (b) Liquid state of ideal gas is impossible
(a) Tc = (b) Tc = (c) At any temerature and pressure, ideal gas obeys
27 R b 27 bR
AL Boyle's law and charles' law
a –a (d) The molecules of real gas do not exert any force on
(c) Tc = (d) Tc = one another.
2R b Rb
N
24. For hydrogen gas Cp – C v = a and for oxygen gas
19. At room temperature, the rms speed of the molecules of a Cp – Cv = b. So, the relation between a and b is given by
R

certain diatomic gas is found to be 1930 m/s. The gas is (a) a = 16 b (b) 16 a = b
U

(a) H2 (b) F2 (c) a = 4 b (d) a = b


(c) O2 (d) Cl2 25. The relation between the gas pressure P and average kinetic
JO

20. Two gases A and B having the same temperature T, same energy per unit volume E is
pressure P and same volume V are mixed. If the mixture is at 1
the same temperature T and occupies a volume V, the (a) P = E (b) P = E
2
U

pressure of the mixture is 3 2


(c) P = E (d) P = E
ED

2 3

1. If the critical temperature of a gas is 100ºC, its Boyle 5. If one mole of a monatomic gas (g = 5/3) is mixed with one
temperature will be approximately mole of a diatomic gas (g = 7/3), the value of g for the mixture
(a) 337.5ºC (b) 500ºC is
(c) 33.3ºC (d) 1000ºC (a) 1.40 (b) 1.50
2. The r.m.s. velocity of oxygen molecule at 16ºC is
(c) 1.53 (d) 3.07
474 m/sec. The r.m.s. velocity in m/s of hydrogen molecule
at 127ºC is 6. The molecules of a given mass of gas have a root mean
(a) 1603 (b) 1896 square velocity of 200m s–1 at 27°C and 1.0 × 105 N m–2
(c) 2230.59 (d) 2730 pressure. When the temperature is 127°C and the pressure
3. The gases are at absolute temperature 300ºK and 350ºK 0.5 × 105 Nm–2, the root mean square velocity in ms–1, is
respectively. The ratio of average kinetic energy of their
molecules is 400
(a) 7 : 6 (b) 6 : 7 (a) (b) 100 2
3
(c) 36 : 49 (d) 49 : 36
4. The total degree of freedom of a CO2 gas molecule is
100 2 100
(a) 3 (b) 6 (c) (d)
(c) 5 (d) 4 3 3
EBD_7179
334 PHYSICS

7. If masses of all molecule of a gas are halved and their speed 18. The temperature at which the root mean square velocity of
doubled then the ratio of initial and final pressure will be the gas molecules would become twice of its value at 0°C is
(a) 2 : 1 (b) 1 : 2 (a) 819°C (b) 1092°C
(c) 4 : 1 (d) 1 : 4 (c) 1100°C (d) 1400°C
8. On a particular day, the relative humidity is 100% and the 19. At what temperature is the r.m.s. velocity of a hydrogen
room temperature is 30ºC, then the dew point is molecule equal to that of an oxygen molecule at 47ºC
(a) 70ºC (b) 30ºC
(a) 80 K (b) –73 K
(d) 100ºC (d) 0ºC
9. The velocity of the molecules of a gas at temperature 120 K (c) 3 K (d) 20 K
is v. At what temperature will the velocity be 2 v? 20. One litre of oxygen at a pressure of 1 atm, and 2 litres of
(a) 120 K (b) 240 K (c) 480 K (d) 1120 K nitrogen at a pressure of 0.5 atm. are introduced in the vessel
10. The density of a gas is 6 × 10–2 kg/m3 and the root mean of 1 litre capacity, without any change in temperature. The
square velocity of the gas molecules is 500 m/s. The total pressure would be
pressure exerted by the gas on the walls of the vessel is (a) 1.5 atm. (b) 0.5 atm.
(a) 3 2 (b) –4 2
5×10 N /m 1.2×10 N/m (c) 2.0 atm. (d) 1.0 atm.
21. The air in a room has 15 gm of water vapours per cubic
(c) 0.83×10 –4 N /m 2 (d) 30 N / m 2
metre of its volume. However for saturation one cubic metre

.IN
11. The temperature of the mixture of one mole of helium and of volume requires 20 gm of water vapour then relative
one mole of hydrogen is increased from 0°C to 100°C at
humidity is
constant pressure. The amount of heat delivered will be
(a) 50% (b) 75%
(a) 600 cal (b) 1200 cal
AL
(c) 1800 cal (d) 3600 cal (c) 20% (d) 25%
12. Helium gas is filled in a closed vessel (having negligible 22. A vessel contains air at a temperature of 15ºC and 60% R.H.
N
thermal expansion coefficient) when it is heated from 300 K What will be the R.H. if it is heated to 20ºC? (S.V.P. at 15ºC is
to 600 K, then average kinetic energy of helium atom will be
R

12.67 & at 20ºC is 17.36mm of Hg respectively)


(a) 2 times (b) 2 times
U

(a) 262% (b) 26.2%


(c) unchanged (d) half
13. One mole of a gas occupies 22.4 lit at N.T.P. Calculate the (c) 44.5% (d) 46.2%
JO

difference between two molar specific heats of the gas. 23. To what temperature should be the hydrogen at 327°C be
J = 4200 J/kcal. cooked at constant pressure so that the root mean square
velocity of its molecules becomes half of its previous value
U

(a) 1.979 k cal/kmol K (b) 2.378 k cal/kmol K


(c) 4.569 kcal/kmol K (d) 3.028 k cal/ kmol K (a) –123°C (b) 120°C
ED

14. Four molecules have speeds 2 km/sec, 3 km/sec, 4 km/sec (c) –100°C (d) 0°C
and 5 km/sec. The root mean square speed of these 24. If pressure of a gas contained in a closed vessel is increased
molecules (in km/sec) is by 0.4% when heated by 1ºC, the initial temperature must
(a) 54 / 4 (b) 54 / 2 be

(c) 3.5 (d) 3 3 (a) 250 K (b) 250ºC

15. The density of air at pressure of 105 Nm–2 is 1.2 kg m–3. (c) 2500 K (d) 25ºC
Under these conditions, the root mean square velocity of 25. N molecules, each of mass m, of gas A and 2 N molecules,
the air molecules in ms–1 is each of mass 2 m, of gas B are contained in the same vessel
(a) 500 (b) 1000 which is maintained at a temperature T. The mean square of
(c) 1500 (d) 3000 the velocity of molecules of B type is denoted by v2 and
16. How many degrees of freedom are associated with 2 grams the mean square of the X component of the velocity of A
of He at NTP ? type is dentoed by w2, w2/v2 is
(a) 3 (b) 3.01 × 1023
23 (a) 2 (b) 1
(c) 9.03 × 10 (d) 6
17. If 2 mole of an ideal monatomic gas at temperature T0 is (c) 1/3 (d) 2/3
mixed with 4 moles of another ideal monatomic gas at 26. At what temperature, pressure remaining constant, will the
temperature 2T0, then the temperature of the mixture is r.m.s. velocity of a gas be half of its value at 0ºC?
5 3 4 5 (a) 0ºC (b) –273ºC
(a) T0 (b) T0 (c) T0 (d) T0
3 2 3 4 (c) 32ºC (d) –204ºC
Kinetic Theory 335

27. Graph of specific heat at constant volume for a monatomic 34. Two monatomic ideal gases 1 and 2 of molecular masses
gas is m1 and m2 respectively are enclosed in separate containers
kept at the same temperature. The ratio of the speed of
Y Y sound in gas 1 to that in gas 2 is given by
3R
m1 m2
(a) cv (b) cv (a) (b)
m2 m1
X O X
O T
T
m1 m2
(c) m2
(d)
m1
Y 3 Y
—R
2 35. If the potential energy of a gas molecule is
(c) cv (d) cv U = M/r6 – N/r12, M and N being positive constants, then
X X the potential energy at equilibrium must be
O O
T T (a) zero (b) M2/4N
28. The order of magnitude of the number of nitrogen molecules (c) N2/4M (d) MN2/4
in an air bubble of diameter 2 mm under ordinary conditions is 36. Air is pumped into an automobile tube upto a pressure of
(a) 105 (b) 109 200 kPa in the morning when the air temperature is 22°C.

.IN
(c) 1013 (d) 1017 During the day, temperature rises to 42°C and the tube
29. At identical temperatures, the rms speed of hydrogen AL expands by 2%. The pressure of the air in the tube at this
molecules is 4 times that for oxygen molecules. In a mixture temperature, will be approximately
of these in mass ratio H2 : O2 = 1:8, the rms speed of all molecules (a) 212 kPa (b) 209 kPa (c) 206 kPa (d) 200 kPa
is n times the rms speed for O2 molecules, where n is 37. Work done by a system under isothermal change from a
N
(a) 3 (b) 4/3 volume V1 to V2 for a gas which obeys Vander Waal's
R

(c) (8/3) 1/2 (d) (11)1/2


æ an 2 ö
30. In Vander Waal’s equation the critical pressure Pc is given by equation (V - bn) ç P + ÷ = nRT is
U

ç V ÷
è ø
a
JO

(a) 3 b (b)
27 b 2 æ V - nb ö 2 æ V1 - V2 ö
(a) nRT log e ç 2 ÷ + an ç ÷
è V1 - nb ø è V1V2 ø
b2
U

27 a
(c) (d)
b2 a æ V - nb ö 2 æ V1 - V2 ö
ED

(b) nRT log10 ç 2 ÷ + an ç ÷


31. 1 mole of a gas with g = 7/5 is mixed with 1 mole of a gas with è V1 - nb ø è V1V2 ø
g = 5/3, then the value of g for the resulting mixture is
(a) 7/5 (b) 2/5 æ V - nb ö 2 æ V1 - V2 ö
(c) nRT loge ç 2 ÷ + bn ç ÷
(c) 24/16 (d) 12/7 è V1 - nb ø è V1V2 ø
32. N molecules, each of mass m, of gas A and 2 N molecules,
each of mass 2 m, of gas B are contained in the same vessel æ V - nb ö 2 æ V1V2 ö
which is maintained at a temperature T. The mean square (d) nRT log e ç 1 ÷ + an ç ÷
V
è 2 - nb ø è V1 – V2 ø
velocity of molecules of B type is denoted by V2 and the
38. A gas mixture consists of 2 moles of oxygen and 4 moles of
mean square velocity of A type is denoted by V1, then V1 Argon at temperature T. Neglecting all vibrational moles,
V2
is the total internal energy of the system is
(a) 2 (b) 1 (a) 4 RT (b) 15 RT
(c) 1/3 (d) 2/3 (c) 9 RT (d) 11RT
33. An ideal gas is found to obey an additional law VP2 = 39. The molar heat capacities of a mixture of two gases at
constant. The gas is initially at temperature T and volume constant volume is 13R/6. The ratio of number of moles of
V. When it expands to a volume 2 V, the temperature becomes the first gas to the second is 1 : 2. The respective gases may
be
(a) T/ 2 (b) 2 T
(a) O2 and N2 (b) He and Ne
(c) 2T 2 (d) 4 T (c) He and N2 (d) N2 and He
EBD_7179
336 PHYSICS

40. A graph is plotted with PV/T on y-axis and mass of the gas
PV PV
along x-axis for different gases. The graph is
(a) a straight line parallel to x-axis for all the gases
(a) (b)
(b) a straight line passing through origin with a slope
having a constant value for all the gases V V
(c) a straight line passing through origin with a slope
PV PV
having different values for different gases
(d) a straight line parallel to y-axis for all the gases
(c) (d)
41. Four mole of hydrogen, two mole of helium and one mole of
water vapour form an ideal gas mixture. What is the molar V V
specific heat at constant pressure of mixture? 46. The molar specific heat at constant pressure of an ideal gas
is (7/2)R. The ratio of specific heat at constant pressure to
16 7 23 that at constant volume is
(a) R (b) R (c) R (d) R
7 16 7 (a) 5/7 (b) 9/7 (c) 7/5 (d) 8/7
42. An ideal gas is taken through a process in which the pressure DIRECTIONS for Qs. 47 to 50 : Each of these question
and the volume are changed according to the equation contains two statements: STATEMENT-1 (Assertion) and

.IN
P = kV. Molar heat capacity of the gas for the process is STATEMENT-2 (Reason). Answer these questions from the
R following four options.
(a) C = Cv + R (b) C = Cv + (a) Statement-1 is True, Statement-2 is True; Statement-2 is a
2
R
AL correct explanation for Statement -1
(c) C = Cv – (d) C = Cv + 2R (b) Statement-1 is True, Statement -2 is True; Statement-2 is
2
43. A vessel has 6g of hydrogen at pressure P and temperature NOT a correct explanation for Statement - 1
N
500K. A small hole is made in it so that hydrogen leaks out. (c) Statement-1 is True, Statement- 2 is False
R

How much hydrogen leaks out if the final pressure is P/2 (d) Statement-1 is False, Statement -2 is True
and temperature falls to 300 K ?
U

C
47. Statement 1 : The ratio V for a monatomic gas is less
(a) 2g (b) 3g (c) 4g (d) 1g CP
JO

than for a diatomic gas.


44. Figure shows a parabolic graph between T and 1/V for a
Statement 2 : The molecules of a monatomic gas have more
mixture of a gas undergoing an adiabatic process. What is
degrees of freedom than those of a diatomic gas.
U

the ratio of Vrms of molecules and speed of sound in mixture?


48. Statement 1 : Air pressure in a car tyre increases during
T driving.
ED

(a) 3/ 2 Statement 2 : Absolute zero temperature is not zero energy


temperature.
(b) 2 2T0 49. Statement 1 : The total translational kinetic energy of all
T0 the molecules of a given mass of an ideal gas is 1.5 times the
product of its pressure and its volume.
(c) 2/3 Statement 2 : The molecules of a gas collide with each other
1/V
1/V0 4/V0
and the velocities of the molecules change due to the
collision.
(d) 3
50. Statement 1 : Mean free path of a gas molecules varies
inversely as density of the gas.
45. Which one the following graphs represents the behaviour
Statement 2 : Mean free path varies inversely as pressure
of an ideal gas ?
of the gas.
Kinetic Theory 337

Exemplar Questions (a) both p and V of the gas will change


1. A cubic vessel (with face horizontal + vertical) contains an (b) only p will increase according to Charles' law
ideal gas at NTP. The vessel is being carried by a rocket (c) V will change but not p
which is moving at a speed of 500 m s–1 in vertical direction. (d) p will change but not V
The pressure of the gas inside the vessel as observed by 5. Volume versus temperature graphs for a given mass of an
us on the ground ideal gas are shown in figure. At two different values of
(a) remains the same because 500 ms–1 is very much constant pressure. What can be inferred about relation
smaller than vrms of the gas between p1 and p2?
(b) remains the same because motion of the vessel as a
whole does not affect the relative motion of the gas V(l) p2
molecules and the walls
2 40
(c) will increase by a factor equal to (vrms + (500)2 ) / vrms
2

where vrms was the original mean square velocity of 30

.IN
the gas p1
20
(d) will be different on the top wall and bottom wall of the
vessel 10
AL
2. 1 mole of an ideal gas is contained in a cubical volume V,
ABCDEFGH at 300K (figure). One face of the cube (EFGH) 100 200 300 400 500
T(K)
N
is made up of a material which totally absorbs any gas
molecule incident on it. At any given time,
(a) p1 > p2 (b) p1 = p2
R

B (c) p1 < p2 (d) Data is insufficient


C
U

6. 1 mole of H2 gas is contained in a box of volume V = 1.00 m3


A D at T =300 K. The gas is heated to a temperature of T = 3000
JO

G K and the gas gets converted to a gas of hydrogen atoms.


F
The final pressure would be (considering all gases to be
E
H ideal)
U

(a) the pressure on EFGH would be zero (a) same as the pressure initially
ED

(b) the pressure on all the faces will the equal (b) 2 times the pressure initially
(c) the pressure of EFGH would be double the pressure (c) 10 times the pressure initially
on ABCD (d) 20 times the pressure initially
(d) the pressure on EFGH would be half that on ABCD 7. A vessel of volume V contains a mixture of 1 mole of
3. Boyle's law is applicable for an hydrogen and 1 mole oxygen (both considered as ideal).
(a) adiabatic process Let f1(v)dv, denote the fraction of molecules with speed
between v and (v + dv ) with f2(v)dv, similarly for oxygen.
(b) isothermal process
Then,
(c) isobaric process
(d) isochoric process (a) f1 (v) + f 2 (v ) = f (v ) obeys the Maxwell's distribution
4. A cylinder containing an ideal gas is in vertical position law
and has a piston of mass M that is able to move up or down (b) f1(v), f2(v) will obey the Maxwell's distribution law
without friction (figure). If the temperature is increased separately
(c) neither f 1(v), nor f 2 (v) will obey the Maxwell's
M distribution law
(d) f2(v) and f1(v) will be the same
8. An inflated rubber balloon contains one mole of an ideal
gas, has a pressure p, volume V and temperature T. If the
temperature rises to 1.1 T, and the volume is increased to
1.05 V, the final pressure will be
(a) 1.1 p (b) p
(c) less than p (d) between p and 1.1
EBD_7179
338 PHYSICS

NEET/AIPMT (2013-2017) Questions The change in internal energy of the gas during the
9. In the given (V – T) diagram, what is the relation between transition is: [2015]
pressure P1 and P2 ? V [2013] (a) – 20 kJ (b) 20 J
(a) P2 > P1 P2 (c) –12 kJ (d) 20 kJ
(b) P2 < P1 P1 15. Two vessels separately contain two ideal gases A and B at
(c) Cannot be predicted q2
the same temperature. The pressure of A being twice that of
q1 B. Under such conditions, the density of A is found to be
(d) P2 = P1 T 1.5 times the density of B. The ratio of molecular weight of
10. The amount of heat energy required to raise the temperature A and B is : [2015 RS]
of 1g of Helium at NTP, from T1K to T2K is [2013]
3
3 (a) (b) 2
(a) N k (T – T1) 4
2 a B 2

3 1 2
(b) N k (T – T1) (c) (d)
4 a B 2 2 3
16. 4.0 g of a gas occupies 22.4 litres at NTP. The specific heat
3 T
(c) Nk 2 capacity of the gas at constant volume is 5.0JK–1. If the

.IN
4 a B T1
speed of sound in this gas at NTP is 952 ms–1, then the
heat capacity at constant pressure is (Take gas constant R
3
= 8.3 JK–1 mol–1)
(d) N k (T – T1)
AL [2015 RS]
8 a B 2
(a) 7.5 JK–1 mol–1
11. In a vessel, the gas is at a pressure P. If the mass of all the
(b) 7.0 JK–1 mol–1
N
molecules is halved and their speed is doubled, then the
resultant pressure will be [NEET Kar. 2013] (c) 8.5 JK–1 mol–1
R

(a) 4P (b) 2P (d) 8.0 JK–1 mol–1


U

(c) P (d) P/2 17. The molecules of a given mass of a gas have r.m.s. velocity
12. The mean free path of molecules of a gas, (radius ‘r’) is of 200 ms–1 at 27°C and 1.0 × 105 Nm–2 pressure. When the
JO

inversely proportional to : [2014] temperature and pressure of the gas are respectively, 127°C
(a) r3 (b) r2 and 0.05 × 105 Nm–2, the r.m.s. velocity of its molecules in
ms–1 is :
U

[2016]
(c) r (d) r
ED

400
Cp (a) 100 2 (b)
13. The ratio of the specific heats C = g in terms of degrees 3
v

of freedom (n) is given by [2015] 100 2 100


(c) (d)
3 3
æ nö æ 2ö
(a) ç1 + 3 ÷ (b) ç1 + ÷
nø 18. A gas mixture consists of 2 moles of O2 and 4 moles of Ar at
è ø è
temperature T. Neglecting all vibrational modes, the total
æ nö æ 1ö internal energy of the system is :- [2017]
(c) ç1 + 2 ÷ (d) ç1 + ÷

è ø è (a) 15 RT (b) 9 RT
14. One mole of an ideal diatomic gas undergoes a transition (c) 11 RT (d) 4 RT
from A to B along a path AB as shown in the figure.
A
5

P(in kPa) 2 B

4 6
V(in m3)
Kinetic Theory 339

Hints & Solutions


EXERCISE - 1 17. (d) We know that P V = n R T = (m/M) R T
where M = Molecular weight.
1. (d) 2. (b) 3. (b) 4. (d)
5. (d) Since vrms is doubled by increasing the temp. so by æmö æ m ö
Now P ´ ç ÷ = ç ÷ R T
3KT è d ø èMø
vrms = , the temp. increase by four times.
m where d = density of the gas
V1 V2 P R T k NAT
Now for constant pressure = = = ... (2)
T1 T2 d M M
V1=V, T1=TºK, T2 = 4TºK, V2= ? where R = k NA, k is Boltzmann constant.
V2 = 4V
M
6. (d) 7. (a) 8. (c) But = m = mass of each molecule so
9. (c) For a given pressure, volume will be more if NA
temperature is more (Charle's law)
P´m
d=

.IN
P kT
18.
AL (a)
Constant 3RT
pressure 19. (a) For one mole of gas v rms =
T1 M
V
N
V1 V2 3 ´ 8.31 ´ 300
1930 = Þ M = 2×10–3 kg = 2 g
R

M
From the graph it is clear that V2 > V1 Þ T1 > T2
So that di-atomic gas is hydrogen.
U

DV DT 20. (a) Here, initially P1 = P, V1 = V + V = 2 V ;


10. (c) At constant pressure V µ T Þ =
V T
JO

Finally, P2 = P ; V2 = V
Hence ratio of increase in volume per degree rise in
kelvin temperature to it’s origianl volume P1V1 P´2V
As P1V1 = P2V2 or P2 = = = 2P
V V
U

(DV / DT ) 1
= = 4
ED

V T 21. (b) n= =2
2
3 22. (a) Based on Maxwell's velocity distribution curve.
11. (d) Cv = dE / dT = R
2 23. (a, b, c)
3 5 24. (d) Both are diatomic gases and C p – Cv = R for all gases.
Cp = R + R = R = 2.5 R
2 2 2
25. (d) P= E
3
3RT
12. (c) vrms = . According to problem T will become
M EXERCISE - 2
2T and M will becomes M/2 so the value of vrms will
1. (a) Boyle temperature is defined as
increase by 4 = 2 times i.e. new root mean square
a 27 æ 8a ö 27
velocity will be 2v. TB = = ç ÷= Tc = 3.375 Tc
Rb 8 è 27Rb ø 8
13. (d) The difference of CP and CV is equal to R, not 2R.
= 3.375 × 100 = 337.5ºC
2 2 f 1 2
14. (b) g = 1 + , Þ g –1 = Þ = Þ f =
f f 2 g –1 g –1 3R ´ 289 æ 3RT ö
2. (c) v oxg . = çè vrms = M ÷ø
15. (d) 32
1
16. (a) vrms µ Þ (vrms)1 < (v rms)2 < (vrms)3 also in 3R ´ 400
M vH = so vH = 2230.59 m/sec
2
mixture temperature of each gas will be same, hence
3. (b) 4. (c)
kinetic energy also remains same.
EBD_7179
340 PHYSICS

5. (c) Let g1 and g2 denote the ratio of two specific heats of 13. (a) V = 22.4 litre = 22.4 × 10–3 m3, J = 4200 J/kcal
gas 1 and gas 2 respectively when they are mixed in by ideal gas equation for one mole of a gas,
the ratio of n1 and n2 mole. The resultant value of g is
the weighted mean of g1 and g2 i.e., PV 1.013 ´ 105 ´ 22.4 ´ 10 -3
R= =
T 273
n1 g1 + n 2 g 2 1(5 / 3) + 1 (7 / 5)
g= =
n1 + n 2 1+1 R 1.013 ´ 105 ´ 22.4
Cp - C v = = = 1.979 kcal/kmol K
J 273 ´ 4200
25 + 21 46
= = = 1 .53 1/ 2
15 ´ 2 30 é (2) 2 + (3) 2 + (4) 2 + (5) 2 ù é 54 ù
14. (a) vrms =ê ú = ê 4ú
4 ë û
c2 400 2 2 400 ëê ûú
6. (a) = = Þ c2 = ´ 200 = ms -1
c1 300 3 3 3
3P 3 ´ 105
15. (a) c= = = 500 ms -1
1 mNv 2rms r 1.2
7. (b) Since P =
3 V
2 1
since m is halved & speed is doubled so pressure 16. (c) Moles of He = =
4 2
become twice.

.IN
1
S.V.P at dew point Molecules = ´ 6.02 ´ 1023 = 3.01 × 1023
8. (b) Relative humidity (R.H) = 2
S.V.P at room temp. AL As there are 3 degrees of freedom corresponding to 1
since here R.H is 100% molecule of a monatomic gas.
Þ room temp. = dew point = 30ºC \ Total degrees of freedom = 3 × 3.01 × 1023
= 9.03 × 1023
N
v' T' 17. (a) Let T be the temperature of the mixture, then
9. (c) vµ T \ =
v T
R

U = U1 + U2
f
U

2 T' Þ (n1 + n 2 ) RT
Given v' =2 v or, = 2
1 T
JO

\ T' = 4 T = 4 × 120K = 480 K f f


= (n1 ) (R) (T0 ) + (n 2 ) (R) (2T0 )
2 2
1 1
r u 2 = ´ ( 6 ´ 10 - 2 ) ´ (500) 2
U

10. (a) P= Þ (2 + 4)T = 2T0 + 8T0 (Q n1 = 2, n2 = 4)


3 3
5
ED

= 5´ 10 3 N / m 2 \ T= T0
3
m1C p1 + m 2 C p2 ct 273 + t
11. (b) (C p )mix = 18. (a) = Þ 4 ´ 273 - 273 = t Þ t = 819°C
m1 + m 2 c0 273
19. (d) The r.m.s. velocity of the molecule of a gas of molecular
5 7
(C p1 ( He) = R and C p2 ( H 2 ) = R) weight M at Kelvin temperature T is given by,
2 2
3RT
5 7 v r.m.s. =
1´ R + 1 ´ R M
2 2
(Cp)mix = = 3R = 3 × 2 = 6 cal/mol.°C Let MO and MH are molecular weight of oxygen and
1+1
hydrogen and TO and TH the corresponding Kelvin
\ Amount of heat needed to raise the temperature temperature at which vr.m.s. is same for both,
from 0°C to 100°C
3RTO 3RTH
(DQ ) p = mC p DT = 2 ´ 6 ´ 100 = 1200 cal v r.m.s. = =
MO MH
1 2 3
12. (b) Average K.E. = mc = kT µ T TO T
2 2 Hence, = H
MO MH
(Av.K.E.)600 K 600
\ = =2 To = 273º + 47º = 320 K, Mo = 32, MH = 2
(Av.K.E.)300 K 300
2
TH = ´ 320 = 20 K
\ At 600K it will be 2 times than that at 300K. 32
Kinetic Theory 341

20. (c) According to Dalton’s Law but ( v r.ms ) x = w


P = P1 + P2.....................
here 1 litre of orygen at a pressure of 1atm, & 2 litre of so ( v r.ms ) y = ( v r.ms ) z = w
nitrogen at a pressure of 5 atm are introduced in a
vessel of 1 litre capacity. 1
Total K.E. of B type molecules = ´ 2 m v2 = m v2
2
V1P1 + P2 V2 1 ´ 1 + 2 ´ .5
so P = = = 2atm
volume of vessel 1 3 2 2 2
Now, ´ m ω 2 = m v 2 or ( ω / v ) =
2 3
m
21. (b) Relative humidity = ´ 100 %
M 26. (d) vrms µ T
where m is the mass of vapour actually present in
At 0ºC, vrms µ 273
certain volume of air & M is the mass required to
saturate the air fully of that volume v rms
At temp. T, µ T
so R .H = 15 ´ 100 = 75 % 2
20
1 T 273
22. (c) By definition \ = or T = = 68 .2 K ~
- 69 K
2 273 4

.IN
V.P at room temp. 60
Þ R.H = ´100; T = 69 - 273 = -204 º C
S.V.P at room temp. 100
27. AL(c) For a monatomic gas
(V.P.)15
= 3
12.67 Cv = R
2
(V.P)15 = 7.6mm of Hg.
N
So correct graph is
Now since unsaturated vapour obeys gas equation &
R

mass of water remains constant so


3 / 2R
U

­
æ nRT ö
P =ç ÷ Þ PαT Cv
è V ø
JO

(V.P)15 273 + 15
= Þ (V.P) 20 =7.73 mm of Hg T®
(V.P) 20 273 + 20
U

28. (d) Molar volume = 22400 cm3


(V.P) 20
ED

So (R.H) 20 = ´ 100 = 44.5%


(S.V.P) 20 4 d3 4
Bubble volume = p = p10-3
3 8 3
( v rms ) 2 t + 273 1 t + 273
23. (a) = or =
( v rms )1 327 + 273 2 600 6 ´ 1023 ´ 4p10 -3
No. of molecules = » 1017
or t = – 123ºC 22400 ´ 3

P1V1 2 1
24. (a) = constant for one mole 29. (d) Molecule number ratio is H 2 : O 2 = : .
T1 3 3

P1 2 æ 2ö + 1 æ 1ö
& = constant for constant volume That gives (c rms ) = 16 times the
T1 è 3ø è 3ø
value for O2.
P1 1.004P1
so = Þ T1 = 250K 30. (b) The Vander waal’s equation of state is
T1 (T1 + 1)
æ a ö
çç P + ÷÷( V - b ) = RT or P = RT - a
3 V2
25. (d) Total K.E. of A type molecules = mw 2 è ø V - b V2
2
At the critical point, P = Pc, V = Vc, and T = Tc
Total K.E. of A type of molecule is
RTc a
K.E A =
1
2
[
m ( v r.ms ) 2x + ( v r.ms ) 2y + ( v r.ms ) 2z ] \ Pc = -
Vc - b Vc2
....(i)
EBD_7179
342 PHYSICS

dPc dU d éM N ù é -6M 12N ù


At the critical point on the isothermal, dV = 0 F= =- ê - = -ê +
dr ë r 3 R12 úû ú
35. (b)
c dr ë r2 r13 û
- RTc 2a RTc 2a In equilibrium position, F = 0
\ 0= + or = ....(ii)
(Vc - b)2 Vc3 (Vc - b) 2 Vc3 6M 12N
- 13 = 0 or, r 6 =
2N
\ 2
r r M
d 2 Pc \ Potential energy at equilibrium position
Also at critical point, =0
dVc2
M N M2 M2 M2
U= = = - =
2 RTc 6a 2 RTc 6a (2N / M) (2N / M) 2 2N 4N 4N
\ 0= 3
- or = ..(iii)
(Vc - b) Vc4 (Vc - b) 3
Vc4
P1 V1 P2 V2
36. (b) = Here, P1 = 200kPa
Dividing (ii) by (iii), we get T1 T2
1 1 T1 = 22°C = 295 K T2 = 42°C = 315K
(Vc - b) = Vc or V = 3 b ....(iv)
2 3 c
2
Putting this value in (ii), we get V2 = V1 + V1 = 1.02V1
100

.IN
RTc 2a 8a
= or Tc = ...(v) \ P2 =
200 ´ 315V1
4 b2 27 b3
27 b R = 209.37kPa
AL 295 ´1.02V1
Putting the values of Vc and Tc in (i), we get 37. (a) According to given Vander Waal's equation
R æ 8a ö a a nRT an 2
Pc = çç ÷÷ - = P= - 2
N
2
2 b è 27 b R ø 9 b 27 b 2 V - nb V
g for resulting mixture should be in between 7/5 and
R

31. (c) V2 V2 V
2
dV dV
5/3. Work done, W = ò PdV = nRT ò - an 2 ò 2
U

V1 V
V - nb V V
3KT 1 1
JO

32 (b) For 1 molecule of a gas, vrms =


m V
é1ù 2
= nRT [ loge (V - nb)]V2 + an2 ê ú
V
where m is the mass of one molecule
1 ëV ûV1
U

For N molecule of a gas, v1= 3KT ´ N


ED

m æ V - nb ö 2 éV1 - V2 ù
= nRT loge ç 2 ÷ + an ê ú
è V1 - nb ø ë V1V2 û
3KI ´ N
For 2N molecule of a gas v2 = ´N 38. (d) Internal energy of 2 moles of oxygen
(2m)
æ5 ö 5
v Uo 2 = µ ç RT÷ = 2. RT = 5RT
\ 1 =1 è2 ø 2
v2
Internal energy of 4 moles of Argon.
33. (a) Since VP2 = constant,
æ3 ö 3
VP2 = 2VP'2 U Ar = µ ç RT÷ = 4. RT = 6RT
è2 ø 2
P \ Total internal energy
\P' =
2 U = U O2 + U Ar = 11RT
P
As = constant or T α P, thus T becomes T/Ö2 n1Cv1 + n 2 Cv2
T 39. (c) Cv mix =
n1 + n 2
æ gRT ö
34. (a) We know that, C rms = ç ÷ 13R n1Cv1 + 2n1Cv2 é n1 1 ù
è M ø Þ = êQ n = 2 ú
6 n1 + 2n1 ë 2 û
Here C µ 1 C rms1 æ m2 ö
;\ = 13R
rms
M C rms 2 çè m ÷ø Þ = Cv1 + 2C v2
1 2
Kinetic Theory 343

3R 5R 3P Pg
Possible values are, C v1 = , Cv2 = Also v rms = and vsound =
2 2 r r
\ Gases are monatomic (like He) and diatomic
(like N2)
v rms 3
Þ = = 2
PV æ mö PV æ R ö vsound g
40. (c) = nR = ç ÷ R or =ç ÷ m
T è M ø T è Mø
45. (b) For an ideal gas PV = constant i.e. PV doesn’t vary
PV with V.
i.e. versus m graph is straight line passing
T 7
through origin with slope R/M, i.e. the slope depends 46. (c) Molar specific heat at constant pressure CP = R
2
on molecular mass of the gas M and is different for
different gases. 7 5
Since, CP – CV = R Þ CV – CP – R = R–R= R
5 2 2
41. (d) Cv for hydrogen = R
2 CP (7 / 2) R 7
\ = = .
3R CV (5 / 2) R 5
Cv for helium =

.IN
2 47. (c) For monatomic gas, f = 3,
6R Cv 3
Cv for water vapour = = 3R 3R 5R
; C =5
2
AL Cv = , Cp =
2 2 p
5 3
4 ´ R + 2 ´ R + 1 ´ 3R For diatomic gas, f = 5
N
2 2 16
\ (Cv)mix = = R
4 + 2 +1 7 Cv 5
R

5R 7R
\ Cp = Cv + R
Cv = , Cp = ; C =7
2 2 p
U

16 23 48. (b) When a person is driving a car then the temperature


Cp = R + R or Cp = R
JO

7 7 of air inside the tyre is increased because of motion.


42. (b) P= kV Þ PV–1 = k From the Gay Lussac’s law,
It is polytropic process (PVn = constant) PµT
U

So n = – 1 Hence, when temperature increases the pressure also


ED

increase.
R R
So, C = CV + = CV +
1- n 2 3 3
49. (b) Total translational kinetic energy = nRT = PV
2 2
m
43. (d) PV = RT In an ideal gas all molecules moving randomly in all
M
direction collide and their velocity changes after
6 collision.
Initially, PV = R ´ 500
M 50. (a) The mean free path of a gas molecule is the averge
distance between two successive collisions. It is
P (6 - x)
Finally, V= R ´ 300 (if x g gas leaks out) represented by l.
2 M
1 kT m
6 5 l= and l =
Hence, 2 = ´ \ x = 1 gram 2
2 ps P 2 × ps2 d
6-x 3
44. (b) From graph, T2V = const. .....(1) Here, s = 0 diameter of molecule and
As we know that TVg–1 = const k = Boltzmann’s constant.
Þ l µ 1 / d, l µ T and l µ 1 / P.
1
g-1 Hence, mean free path varies inversely as density of
Þ VT = cons. ....(2)
the gas. It can easily proved that the mean free path
On comparing (1) and (2), we get varies directly as the temperature and inversely as the
Þ g = 3/2 pressure of the gas.
EBD_7179
344 PHYSICS

EXERCISE - 3 T V æ 1ö
Vµ as n, R are constant µ ç ÷
Exemplar Questions p T è pø
1. (b) As the relative velocity of molecule with respect to Slope of the V – T graph,
the walls of container does not change in rocket, due
dV nR
to the mass of a molecule is negligible with respect to m= = [m = slope of V – t graph]
the mass of whole system and system of gas moves dT p
as a whole and (g = 0) on molecule energy where.
dV 1 1
Hence pressure of the gas inside the vessel, as µ or m µ [Q nR = constant]
dT p p
observed by us, on the ground remain the same.
2. (d) Pressure on the wall due to force exerted by molecule 1
on walls due to its rate of transfer of momentum to So, p µ
m
wall.
In an ideal gas, when a molecule collides elastically p1 m2
with a wall, the momentum transferred to each molecule
hence, =
p2 m1
will be twice the magnitude of its normal momentum is
2 mv. For the wall EFGH, absorbs those molecules. where, m1 is slope of the graph corresponding to p1
Which strike to it so rate of change in momentum to it and similarly m2 is slope corresponding to p2. So slope
become only mv so the pressure of EFGH would half of p1 is smaller than p2. Hence, (p1 > p2).
6. (d) Pressure exerted by gas is due to rate of change of

.IN
of ABCD.
3. (b) Boyle's law is applicable at constant temperature and momentum (p) imparted by particles to wall.
temperature remains constant in isothermal process. When the molecules breaks into atoms, the number of
moles would become twice.
For ideal gas, pV = nRT = constant
AL
So, pV = constant (at constant temperature) From ideal gas equation,
1 pV = nRT

N
where, p = Pressure of gas, n = Number of moles
V
So, this process can be called as isothermal process. R = Gas constant, T = temperature
R

4. (c) Let us consider the given diagram where an ideal gas As gases breaks number of moles becomes twice of
is contained in a cylinder, having a piston of mass M. initial, so n2 = 2n1
U

The pressure on gas does not change. So, p µ nT


JO

M p2 n2T2 (2n1 )(3000)


pa or p = n T = n (300) = 20
1 1 1 1
U

So, p2 = 20p1
pa Mg/A Hence, final pressure of the gas would be 20 times to
ED

p the initial pressure.


7. (b) For a function f1(v) the number of molecules (n) which
A p will have their speeds between v and v + dv.
For each function f1(v) and f2(v) number of molecules
The pressure inside the gas will be remain same 1 mole each but due to mass difference
p = pa + Mg /A their speed will be different.
where, pa = atmospheric pressure Hence both gases of each function f1(v) and f2(v) will
A = area of cross-section of the piston. obey the Maxwell's distribution law separately.
Mg = weight of piston 8. (d) As we know that an ideal gas equation,
Hence, p = constant. pV = nRT
As the piston and cylinder is frictionless so the where, n = Number of moles, p = Pressure,
equation for ideal gas V = Volume, R = Gas constant,
pV = nRT, volume (V) increases at constant pressure. T = Temperature
as p, R, n are constant so, pV
V µT n=
RT
so on increasing temperature of system its volume If n, R are constant for the system or as number of
increased but p will remain constant. moles of the gas remains fixed, hence, we can write
5. (a) As we know that an ideal gas equation,
as the pressure and quantity of gas in system are pV
= constant
constant T
p1V1 p2V2
æ nR ö or =
pV = nRT Þ V = ç ÷T RT1 RT2
è p ø
Kinetic Theory 345

æ T ö (As gas is diatomic \ f = 5)


p2 = ( p1V1 ) ç 2 ÷
è V2T1 ø 5
= {2 × 103 × 6 – 5 × 103 × 4}
( p )(V )(1.1 T ) 2
= [p1 = p, V2 = 1.05 V
(1.05)V (T ) 5
and T2 = 1.1 T] = {12 – 20} × 103 J = 5 × (–4) × 103 J
2
æ 1.1 ö DU = –20 KJ
= p ´ç ÷
è 1.05 ø 15. (a) From PV = nRT
= p (1.0476) r M r M
So, final pressure p2 will lies between p and 1.1p. PA = A A and PB = B B
RT RT
NEET/AIPMT (2013-2017) Questions From question,
9. (b) P1 > P2 PA rA M A M 3
= =2 A =
PB rB M B MB 2
P2
MA 3
So, M = 4
P1 B
V
16. (d) Molar mass of the gas = 4g/mol

.IN
q2 Speed of sound
q1 T
gRT g ´ 3.3 ´ 273
As V = constant Þ Pµ T V= Þ 952 =
AL m 4 ´ 10-3
Hence from V–T graph P1 > P2
10. (d) From first law of thermodynamics 16 8
Þ g = 1.6 = =
N
3 1 10 5
DQ = DU + DW = . R (T2 – T1) + 0
2 4
R

CP 8
Also, g = C = 5
3 R
U

V
= N K (T – T1) [Q K = ]
8 a B 2 N 8´ 5
JO

So, CP = = 8JK–1mol–1 [CV = 5.0 JK–1 given]


1 mn 2 5
11. (b) Q P = Vrms
3 V 17. (b) Here v1 = 200 m/s;
U

When mass is halved and speed is doubled then temperature T1 = 27°C = 27 + 273 = 300 k
temperature T2 = 127° C = 127 + 273 = 400 k, V = ?
ED

1 m n
Resultant pressure, Pt = ´ ´ (2vrms )2 R.M.S. Velocity, V µ T
3 2 V
= 2 P. v 400
Þ =
1 200 300
12. (b) Mean free path lm =
2p d 2 n 200 ´ 2 400
Þ v= m/s Þv= m/s
where d = diameter of molecule and d = 2r 3 3
1 18. (c) Internal energy of the system is given by
\ lm µ 2
r
13. (b) Let ‘n’ be the degree of freedom f
U= nRT
2
æn ö Degree of freedom
Cp çè + 1÷ø R
2 æ 2ö Fdiatomic = 5
g= = = ç1 + ÷
Cv æ ö
n è nø fmonoatomic = 3
çè ÷ø R
2 and, number of moles
14. (a) Change in internal energy from A ® B n(O2) = 2
n(Ar) = 4
f f
DU = nRDT = nR (Tf – Ti) 5 3
2 2 Utotal = (2)RT + (4)RT = 11RT
T
2 2
5
= {P V – P V }
2 f f i i
EBD_7179
346 PHYSICS

14 Oscillations

PERIODIC AND OSCILLATORY MOTION ¬F


Periodic Motion B O C A
When a body repeats its motion after regular interval of time, it
i.e., F µ – x x®
is said to be in periodic motion. The path of periodic motion may
r
or, F = -kxr

.IN
be rectilinear, open/ closed curvilinear.
Example : (i) Motion of moon around earth. where k is called force constant or spring constant
(ii) Motion of a piston in a cylinder. AL r r
(iii) Motion of a simple pendulum etc. d 2 x kx
or, m + =0
dt 2 m
Oscillatory Motion
(b) Angular S.H.M.
N
If during a periodic motion, the particle moves to and fro on the
The restoring torque is proporational to the angular
same path, the motion is vibratory or oscillatory.
R

displacement from the mean position.


Example : (i) The motion of a ball in bowl r r
t = -Cq where C is called torsional rigidity
U

Ball
r
Bowl d 2q r
JO

or, I 2 = -Cq
dt
r
d 2q C r
U

(ii) The needle of a sewing machine or, + q = 0.


(iii) Vibrations of prongs of tuning fork etc. dt 2 I
ED

Terms Related to S.H.M.


(i) Amplitude : The maximum displacement of the oscillating
(i) All oscillatory motion are periodic but all periodic particle on either side of its mean position is called its
motioniiare not oscillatory motion. amplitude. It is denoted by A.
(ii) The oscillatory motion which can be expressed in terms of (ii) Time period : The time taken by a oscillating particle to
sine and cosine function, is said to be harmonic motion. complete one oscillation is called its time period. It is
denoted by T.
SIMPLE HARMONIC MOTION (S.H.M.) (iii) Frequency : It is the number of oscillations completed in
“If a particle moves up and down (back and forth) about a one second. It is denoted by u.
mean position (also called equilibrium position) in such a way 1
u=
that a restoring force/ torque acts on particle, which is T
proportional to displacement from mean position, but in The S.I. unit of frequency is s–1 or Hz.
opposite direction from displacement, then motion of the particle (iv) Angular frequency
is called simple harmonic motion. If displacement is linear, it is
2p
called linear S.H.M. and if displacement is angular, it is called an w = 2pu =
angular S.H.M. T
Example : (i) Motion of a body suspended by spring. The S.I. unit of angular frequency is rad/sec.
(ii) Oscillations of simple pendulum. (v) Phase : The parameter, by which the position of particle
from its mean position is represented, is known as phase.
Equations of S.H.M. The phase at any instant tells the state of position &
(a) Linear S.H.M. direction of motion at that instant. The phase at time t = 0
The restoring force is proportional to the displacement from is known as the initial phase or epoch (e).
mean position.
Oscillations 347

(vi) Total phase angle : The total angle (wt + q) is known as total (ii) Potential energy : A particle in S.H.M. possesses potential
phase angle. energy due to its displacement from the mean position.
Characteristics of S.H.M. 1 2 1
P.E . =ky = mw2 y 2
(i) Displacement : The displacement of a particle in S.H.M. is 2 2
given by (iii) Total mechanical energy
y = A sin(wt + f ) E = K.E. + P.E.
where A is amplitude, w is angular frequency and (wt + f) is 1 1
called the phase of the particle at any instant t. = mw 2 (A 2 - y2 ) + mw 2 y2
2 2
1 1
E= mw2 A2 = kA 2
+A 2 2
(iv) The curves representing KE, PE and total energy are shown
Displacement

3T/4 T
(0,0) in figure.
T/4 T/2
Total energy
–A E = K.E. + P.E.
Time (t)
P.E.

Energy
(ii) Velocity : The velocity of a particle in S.H.M. is given by

.IN
dy K.E.
= Aw cos(w t + f) or, v = w A 2 - y 2
dt –A O +A
AL Displacement
At y = 0 (at mean position), vmax = w A
Keep in Memory
N

1. Restoring force F = – Mw2x


R

2. Kinetic energy = (1/2) Mw2(A2 – x2)


Velocity

T/2
U

(0) 3. Potential energy = 1/2 Mw2x2


T/4 3T/4 T 4. Total energy of SHM = 1/2 Mw2A2
JO

Time (t) Equation a = – w2y shows that if body perform S.H.M. then
acceleration of the body is proportional to displacement,
(iii) Acceleration : The acceleration of a particle in S.H.M. is but in the opposite direction of displacement. It is an
U

given by essential requirement for any motion to be S.H.M.


ED

5. The kinetic and potential energy of SHM varies sinusoidally


dv
a= = -w2 A sin(w t + f) with a frequency twice that of SHM.
dt
1
or, a = – w2y 6. Total energy = mw 2 A 2 = 2p 2 mA 2 n 2
The negative sign indicates that the acceleration is directed 2
towards the mean position where n = frequency of vibration.
At y = A (at extreme position),
7. a = -w2 x where w is constant v = w A 2 - x 2
amax = – w2A
a v
Acceleration

T/4 x x
T/2 3T/4 T

Time (t)
8. Geometrically the projection of the body undergoing
uniform circular motion on the diameter of the circle is
Energy in S.H.M. : SHM.
(i) Kinetic energy : A particle in S.H.M. possesses kinetic l
energy by virtue of its motion. 9. In a non-inertial frame. T = 2p
g eff
1 2 1 g eff = (g - a y ) 2 + a 2x
K .E. = mv = mw2 ( A2 - y 2 )
2 2
EBD_7179
348 PHYSICS

SOME SYSTEMS EXECUTING S.H.M. Let a bob of mass m is displaced from its, equilibrium position
Case 1 Spring mass system : and released, then it oscillates in a vertical plane under gravity.
(i) When two springs having force constants k1 and k2 Let q be the angular displacement at any time t, then
connected in parallel, then corresponding linear displacement along the arc is
x = l q.
It is clear from the diagram that mg sinq, is the restoring force
k1 k2 acting on m tending to return it to mean position. So from
Newton’s second law

M md 2 x
F= = - mg sin q ...(i)
dt 2
The force constant of the combination is
where negative sign indicates that restoring force mg sin q (= F)
k = k1 + k2 and hence T = 2p[M/(k1 + k2)]1/2
is opposite to displacement q. If q is very small, then
(ii) When two springs of force constants k 1 and k2 are
sin q » q, so from equation (i)
connected in series, then
md 2 x x
= - mg sin q = - mg or ...(ii)
k1 dt 2 l
where w2 = g/l.
This is the equation of S.H.M. of the bob with time period

.IN
k2
2p ælö
M T= = 2p ç ÷ OM
w ègø
AL
The force constant of the combination is How to find the time period of a body undergoing S.H.M.?
1/k = 1/k1 + 1/k2. i.e., Step 1 : First, find the equilibrium position. Equilibrium position
N
m will be one for which SF = 0 and St = 0
k = k1k2/(k1 + k2) Hence T = 2p
R

K Step 2 : Displace the body , from the equilibrium position by x.


(iii) If two mass M1 and M2 are connected at the two ends of Find the restoring force acting on the body F = –kx (for translation)
U

the spring, then their period of oscillation is given by Find the restoring torque acting on the body t = - kq (for
JO

M1 M2 rotational)

md 2 x
M1M 2 Step 3 : Since F = ma =
T = 2p[m/k)]1/2 where m = is the reduced mass. dt 2
U

M1 + M 2
md 2 x
ED

(iv) When the length of spring increases, spring constant \ Use = - kx ... (i) for translational
decreases. If the length of spring becomes n times, its spring dt 2
1 md 2 x
constant becomes times and therefore time period will = - kq … (ii) for rotational
n dt 2
be increased by n times.
m
(v) If we divide the spring into n equal parts, the spring Step 4 : T = 2p (for translational)
k
constant of each part becomes n k. Hence time period when
the same mass is suspended from each part is: I
T = 2p (for rotational) where, I = moment of inertia
1/ 2 k
éM ù
T = 2p ê ú
ë nk û COMMON DEFAULT
Case 2 Simple pendulum : A simple pendulum consists of a
Incorrect. The time period of spring mass-system is
point mass suspended by a weightless inextensible cord from a
rigid support. dependent on the value of g.
Correct. Time period of spring-mass system shifts only the
equilibrium position. It does not change the time period.
q Because of this reason, time period of spring mass system
l l remains same on plains / mountains / in satellites.
Incorrect. The time taken to cover half the amplitude form
x =lq
T
equilibrium position is .
mg sinq mg mg cosq 8
Oscillations 349

T Case 7: The time period of a ball oscillating in the neck of a


Correct. The actual time taken is . chamber
12
Incorrect. In a spring mass system, mass oscillate about 2p mV
the end of a spring when the spring is in unstretched T=
A B
condition. Case 8 : If a dipole of dipole moment p is suspended in a uniform
Correct. The mass oscillates about the equilibrium position electric field E then time period of oscillation
which may or may not be at the unstretched length.
I
Case 3 Liquid in U-tube : A U-tube of uniform cross-sectional T = 2p
area A has been set up vertically with open ends facing up. PE
Keep in Memory
1. In S.H.M. the phase relationship between displacement,
x velocity and accleration, is as follows :
x
(a) The velocity is leading the displacement by a phase
p
radian
2
(b) The acceleration is leading the displacement by a
phase p radian
[Restoring force = –2Adg x] p

.IN
(c) The acceleration is leading the velocity by a phase
If m gm of a liquid of density d is poured into it then time period 2
of oscillation. AL radian.
m A
T = 2p 2. (a) When x = , then velocity V = 0.86Vmax.
2Adg 2
(b) When V = Vmax/2, the displacement x = 0.87A.
N
Case 4 Rectangular block in liquid :
A
(c) When x = , the kinetic energy of S.H.M. is 75% of
R

2
the total energy and potential energy 25% of the total
U

h
h' energy.
d' (d) When the kinetic energy of S.H.M. is 50% of the total
JO

energy, the displacement is 71% of the amplitude.


d 3. The time period of a simple pendulum of length l which is
comparable with radius of earth.
U

h' d' 1/2 1/2


Rectangular block floating in a liquid, T = 2p ; where é R ù é lR ù
ED

gd T = 2p ê ú = 2p ê ú
d = density of liquid, d¢ = density of block, h = height of block
æ R ö
ê ç1 + ÷ g ú ë (l + R)g û
ëê è l ø ûú
Case 5 : Vibration of gas system in a cylinder with frictionless
piston. where R = radius of the earth and g is the acceleration due
to gravity on the surface of the earth.
l
A (a) When l << R, then T = 2p
g
m 1/ 2
h é R ù
p (b) When l = R, we find T = 2pê ú » 1 hour
ëê 2g úû
(c) When l = ¥ , then
mh
Time period, T = 2p 1/ 2
AP é Rù
T = 2p ê ú = 84.6 minutes. Thus maximum of T
where m = mass of gas, A = cross sectional area of piston êë g úû
P = pressure exerted by gas on the piston, h = height of piston
Case 6: If a tunnel is dig in the earth diametrically or along a is 84.6 minutes.
(d) Under weightlessness or in the freely falling lift
R
chord then time period, T = 2p = 84 min 36 sec for a particle 1/ 2
él ù
g T = 2pê ú = ¥ . This means, the pendulum does
released in the tunnel. ë0û
not oscillate at all.
EBD_7179
350 PHYSICS

4. Time period of a simple pendulum in a train accelerating or 11. If a ball of radius r oscillates in a bowl of radius R, then its
retarding at the rate a is given by 1/ 2
éR - rù
1/ 2 time period is given by : T = 2pê ú
é ù ë g û
l
T = 2pê ú
ê 2 2 ú 12. If a disc of radius r oscillates about a point at its rim, then its
ë g +a û
1/ 2
5. If a simple pendulum whose bob is of density do is made to érù
time period is given by: T = 2pê ú
oscillate in a liquid of density d, then its time period of ëgû
vibration in liquid will increase and is given by It behaves as a simple pendulum of length r.
13. The graph between the length of a simple pendulum and its
l
T = 2p (where d0 > d) time period is a parabola.
æ d ö 14. The graph between the length of a simple pendulum and
çç1 - ÷÷g
è d o ø the square of its time period is a straight line.
15. The graph between l & T and between l & T2 intersect at
6. The time period of a simple pendulum in a vehicle moving T = 1 second.
along a circular path of radius r and with constant velocity 16. The time period of the mass attached to spring does not
1/ 2 change with the change in acceleration due to gravity.
é l ù
V is given by, T = 2p ê 17. If the mass m attached to a spring oscillates in a non viscous

.IN
4 ú
ê V + g2 ú liquid density s, then its time per iod is given
ê r2 ú
ë û -1 / 2
é m æ s öù
7. If T1 and T2 are the time periods of a body oscillating under
AL by T = 2 p ê çç1 - ÷÷ú
the restoring force F1 and F2 then the time period of the ë k è r øû
r r r
body under the influence of the resultant force F = F1 + F2 where k = force constant and r is density of the mass
N
suspended from the spring.
R

T1 T2 18. The length of second pendulum (T = 2 sec) is 99 cm


will be T =
T12 + T2 2
U

Physical Pendulum
8. (a) The percentage change in time period of simple
JO

Trestoring = – mgd sin q


pendulum when its length changes is
d
If q is small, sin q » q
DT 1 æ Dl ö q
´ 100 = ç ÷ ´ 100% \ Trestoring = – mgdq cm
U

T 2è l ø
mg
ED

(b) The percentage change in time period of simple I


And T = 2p
pendulum when g changes but l remains constant is mgd
DT 1 æ Dg ö Let a test-tube of radius r, carrying lead shots of mass m is held
´100 = çç ÷ ´100 % vertically when partly immersed in liquid of density r. On pushing
T 2 è g ÷ø
the tube little into liquid and let it executes S.H.M. of time period
(c) The percentage change in time peirod of simple
pendulum when both l and g change is m
T = 2p
p r 2r g
DT 1 æ Dl Dg ö
´ 100 = çç + ÷ ´100% Conical Pendulum
T 2è l g ÷ø When the bob of a simple pendulum moves in a horizontal circle
9. If a wire of length l , area of cross-section A, Young’ss it is called as conical pendulum.
modulus Y is stretched by suspending a mass m, then the
S
TA
mass can oscillate with time period: T = 2p q
ml T
10. If a simple pendulum is suspended from the roof of
compartment of a train moving down an inclined plane of r
inclination q, then the time period of oscillations
1/ 2 If l is the length of the pendulum and the string makes an angle
é l ù
T = 2pê ú l cos q
ë g cos q û q with vertical then time period, T = 2p
g
Oscillations 351

Torsional Pendulum In this case electric force qE and gravity force are
It is an arrangement which consists of a heavy mass suspended opposite.
from a long thin wire whose other and is clamped to a rigid support. 5. A pendulum clock slows down in summer and goes faster
Time period in winter.
I 6. Potential energy of a particle executing S.H.M. is equal to
T = 2p average force × displacement.
C
where I = moment of inertia of body about the suspension wire 0+ F æ 0 + mw2 x ö
as axis of rotation. i.e., U P = éê ù ç
úx = ç
÷ x = 1 mw2 x 2 .
ë 2 û 2 ÷ 2
C = restoring couple per unit thirst. è ø
7. If the total energy of a particle executing S.H.M. is E, then
Keep in Memory its potential energy at displacement x is
1. The displacement, velocity and acceleration of S.H.M. vary x2 æ x 2 ö÷
simple harmonically with the same time period and UP = E and kinetic energy U k = ç1 - E
A2 ç A2 ÷
frequency. è ø
2. The kinetic energy and potential energy vary periodically
but not simple harmonically. The time period of kinetic
energy or potential energy is half that of displacement, FREE, DAMPED, FORCED OSCILLATIONS AND
velocity and acceleration. RESONANCE
3. The graph between displacement, velocity or acceleration Free Oscillation

.IN
and t is a sine curve. But the graph between P.E. or K.E. of
S.H.M. and time t is parabola. If a system oscillates on its own and without any external
influence then it is called as free oscillation.
AL
dx
x=A sin wt v= = Aw cos wt Frequency of free oscillation is called natural frequency. The
Displacement

dt
equation for free S.H.M. oscillation
Velocity

t
md 2 x
N
t
= Frestoring force = –kx, where k is constant.
dt 2
R

d2 x The differential equation of harmonic motion in absence of


a= = -w2 x
U
Acceleration

dt 2 damping and external force is


JO

d2x
t + w02 x = 0 , where w0 is natural frequency of body..
dt 2
The time period is
U

2p m
ED

T= = 2p
K.E., P.E. E
2 2
w0 k
E=½ mw A
1
P.E. = mw2 x 2 Damped Oscillation
2 ½E
1
K.E. = mw2 (A 2 - x 2 )
Oscillation performed under the influence of frictional force is
x 2 called as damped oscillation.
–A O +A
(a) In case of damped oscillations the amplitude goes on
4. (a) If the bob of simple pendulum is -vely charged and a decreasing and ultimately the system comes to a rest.
+vely charged plate is placed below it, then the
effective acceleration on bob increases and x
consequently time period decreases.
l
Time period, T = 2p qE
t
g+
m
In this case electric force q E and gravity force act in
same direction. (b) The damping force (Fdamping µ – v Þ Fdamping = – bv) is
(b) If the bob of a simple pendulum is -vely charged and proportional to the speed of particle. Hence the equation of
is made to oscillate above the -vely charged plate, dV
then the effective acceleration on bob decreases and motion m = -kx - bv
dt
l where b is positive constant and is called damping
the time period increases. T = 2 p
qE coefficient. Then the differential equation of a damped
g-
m harmonic oscillation is
EBD_7179
352 PHYSICS

d2x dx f0
+ 2C + w02 x = 0 ...(i) x= sin(pt - q)
2 dt
dt é(w 20 - p 2 ) + 4c 2 p 2 ùû
ë
k
where 2 C = b/m (C is damping constant) & = w20 , the
m 2cp
natural frequency of oscillating particle i.e., its frequency where tan q = & p is the frequency of external periodic
in absence of damping. w - p2
2

In case of over damping the displacement of the particle is force.


x = A0 exp(-t / 2t) sin (wt + f) ...(ii)
Þ x = A sin (wt + f) where A0 = max. amplitude of the oscillator. Example 1.
A spring is stretched by 0.20 metre when a mass of 0.50 kg
m is suspended. Calculate the period of the spring when a
w = w02 - c 2 and t = (relaxation time)
b mass of 0.25 kg is suspended and put to oscillation.
It is clear from the fig & eqn. (ii) that the amplitude of damped (g = 10 m/sec2).
harmonic oscillator decreases with time. In this case, the motion Solution :
does not repeat itself & is not periodic in usual sense of term. The force constant K of the spring is given by
2p 2p
However it has still a time period, T = = , which is F 0.5 kg Wt

.IN
w w0 - c 2
2 K= =
y 0.20 m
the time interval between its successive passage in same direction 0.5 ´ 10 newton
=
passing the equilibrium point.
AL 0.20 m
Displacement

–ct = 25 newton / metre


A=A0e
N

t
R

æmö æ 0.25 ö
Now T = 2 p ç ÷ = 2 ´ 3.14 ç ÷ = 0.628 second.
A= –A0e
–c t èKø è 25 ø
U

Example 2.
JO

Forced Oscillation and Resonance : What will be the force constant of the spring system shown
The oscillation of a system under the action of external periodic in fig?
force is called forced oscillation.
U

k1
External force can maintain the amplitude of damped oscillation. (a) + k2
2
ED

When the frequency of the external periodic force is equal to the


natural frequency of the system, resonance takes place.
-1 k1 k1
The amplitude of resonant oscillations is very very large. In the é 1 1 ù
absence of damping, it may tend to infinity. (b) ê + ú
ë 2k1 k 2 û
At resonance, the oscillating system continuously absorbs
energy from the agent applying external periodic force.
é 1 1 ù k2
In case of forced oscillations, the total force acting on the system
(c) ê 2k + k ú
bdx ë 1 2û
is F = - kx - + F0 sin pt …… (i)
Re storting dt Ext periodic
force Damping force -1
force é2 1 ù
Then by Newton’s second law : (d) ê + ú
k
ë 1 k 2û
d2x dx
Þ m = - kx - b + F0 sin pt Solution : (b)
2 dt
dt Two springs of force constants k1 and k2 are in parallel.
d 2x dx Hence
or + w20 x = f 0 sin b t
+ 2C …… (ii)
2 dt
dt k ¢ = k1 + k1 = 2 k1
b 2 k F
where 2C = , w0 = , f 0 = 0 The third spring is in series with spring of force constant
m m m
The equation (ii) is the differential equation of motion of forced k¢.
harmonic oscillator.
The amplitude at any time t is
Oscillations 353

-1 æAxö æ Ax ö 1 æ PA ö
1 é 1 1 ù é 1 1 ù F = -P ç ÷ \ a = -P ç ÷ = -çç ÷÷ x
\ =ê + ú or k = ê + ú è h ø è h ø M è Mh ø
k ë 2k1 k 2 û ë 2 k1 k 2 û
PA 2p æ M hö
Example 3. This gives, w 2 = or T = = 2p
Mh w çè P A ÷ø
A particle starts with S.H.M. from the mean position as
shown in figure below. Its amplitude is A and its time period Example 5.
is T. At one time, its speed is half that of the maximum A simple harmonic oscillator has an amplitude A and time
speed. What is this displacement at that time ?
period T. Determine the time required by it to travel from
2A A
(a) x = A to x = .
3 2
3A Solution :
(b)
2 æ 2p ö
2A For S.H.M., x = A sin ç t÷
è T ø
(c)
3
æ 2p ö
3A When x = A, A = A sin ç .t ÷
(d) è T ø

.IN
2
Solution : (b) æ 2p ö æ 2p ö æpö
\ sin ç .t ÷ = 1 Þ sin ç .t ÷ = sin ç ÷ Þ t = (T/4)
è T ø è T ø è2ø
We know that v = ω[A 2 - x 2 ]1/ 2
AL A A æ 2p ö
1/ 2 When x = and = A sin ç .t ÷
é v2 ù 2 2 è T ø
\ x = ê A2 - ú
ω2 ûú
N
ëê or sin
p æ 2p ö
= sin ç t ÷ or t = (T/12)
6 è T ø
R

v max Aω
Given that v = = . Now, time taken to travel from
2 2
U

x = A to x = A/2 = T/4 – T/12 = T/6


1/ 2
é 2 A 2 w2 ù 3 Example 6.
JO

so, êA - 2
ú = .A Calculate the increase in velocity of sound for 1ºC rise of
ëê 4 w ûú 2
temperature, if the velocity of sound at 0ºC is 332 m/sec.
Example 4. Solution :
U

A cylindrical piston of mass M slides smoothly inside a


long cylinder closed at one end, enclosing a certain mass v1 æT ö æ 273 + t ö
= ç t÷ = ç
ED

;
of gas. The cylinder is kept with its axis horizontal. If the v0 è T0 ø è 273 ÷ø
piston is disturbed from its equilibrium position, it
1/2
oscillates simple harmonically. The period of oscillation æ t ö æ t ö
\ v1 = v0 ç1 + = v0 ç 1 +
will be è 273÷ø è 2 ´ 273 ÷ø
h
æ t ö æ t ö
= v0 ç1 + ÷ = v0 + v0 ç
M è 546 ø è 546 ÷ø
P A
æ t ö
\ vt - v0 = v0 ç = 0.61 m/sec.
è 546 ÷ø
æ Mh ö æ MA ö (As v0 = 332 m/sec and Dt = 1ºC)
(a) T = 2π ç ÷ (b) T = 2π ç ÷
è PA ø è Ph ø Example 7.
Which one of the following equations does not represent
æ M ö S.H.M.; where x = displacement and t = time. Parameters
(c) T = 2π ç ÷ (d) T = 2π (M P h A)
è PAh ø a, b and c are the constants of motion
Solution : (a) (a) x = a sin bt
We know that P V = R T (b) x = a cos bt + c
\ VDP + P DV = 0 or DP = P (DV / V) (c) x = a sin bt + c cos bt
F æ Ax ö æxö (d) x = a sec bt + c cosec bt
or A = -P çç ÷÷ = -P ç ÷ Solution : (d)
è Ah ø èhø
Sec bt is not defined for bt = p/2.
EBD_7179
354 PHYSICS

a sin bt + c cos bt v = [9 - 2 g l (1 - cos q )]1/2


x = a sec bt + c cosec bt =
sin bt cos bt = [9 - 2 ´ 10 ´ 0.5 (1 - 1/ 2)]1/ 2
This equation cannot be modified in the form of simple = (9 – 5)1/ 2 = 2 m / s
equation of S.H.M. Example 9.
i.e. x = a sin (wt + f). A small spherical steel ball is placed a little away from the
so, it cannot represent S.H.M. centre of a large concave mirror whose radius of curvature
R = 2.5 cm. When the ball is released, it begins to oscillate
Example 8. about the centre. Show that the motion of the ball is simple
If length of pendulum is increased, time period T increases. harmonic and find the period of motion. Neglect friction
So, frequency decreases or angular velocity decreases. As and take g = 10 m/sec2.
Solution :
maximum velocity v max = Aω , so max. velocity will also
F = – mg sin q = – mg q ( Q q is small)
decrease. (where A is amplitude of bob). = – mg (x/R)
A pendulum bob has a speed of 3 m/s at its lowest position. \ F = – kx, where k = (mg/R)
The pendulum is 0.5 m long. What will be the speed of the Now, a = F/m = – kx/m = – w 2x
bob, when the length makes an angle of 60º to the vertical? Hence, motion is simple harmonic motion and
(g = 10 m/s2)
2p æ Rö æ 2.5 ö
Solution : T= = 2p ç ÷ = 2 × 3.14 × çè ÷ = 3.142 sec.
w è g ø 10 ø
Here, r w = 3 m/s; l = 0.5 m; K.E. at the lowest point

.IN
Example 10.
1 1
= m r 2 w2 = m (3) 2 = m
9 A spring of stiffness constant k and natural lengthl is cut
2 2 2 into two parts of length 3l / 4 and l / 4 respectively, and an
AL arrangement is made as shown in the figure. If the mass is
Let v be the velocity of the bob at B when ÐOAB = 60° then,
slightly displaced, find the time period of oscillation.
N
A (3/4)l l/4
m
R

60º
Solution :
l
U

The stiffness of a spring is inversely proportional to its


C B length. Therefore the stiffness of each part is
JO

O 4
k1 = k and k2 = 4k
3
OC = h = (l – l cos q) = (l – l cos 60º) = l /2
U

k1 = (4/3)k k2 = 4k
If v is the velocity of bob at position B, then using law of
m
ED

conservation of energy, we have,


1 1 m
m ´ 9 = m v 2 + mg l (1 - cos θ) Time period, T = 2p
2 2 k1 + k 2
1 1 3m p 3m
or m v 2 = m ´ 9 - mg l (1 - cos θ) or T = 2p =
2 2 16k 2 k
Oscillations 355

.IN
AL
N
R
U
JO
U
ED
EBD_7179
356 PHYSICS

1. A child swinging on swing in sitting position stands up. (a) velocity


The time period of the swing will (b) potential energy
(a) increase (c) phase difference between acceleration and displacement
(b) decrease (d) difference between kinetic energy and potential energy
(c) remain same 8. A simple pendulum has a metal bob, which is negatively
(d) increase if the child is tall and decrease if the child is charged. If it is allowed to oscillate above a positively charged
short. metallic plate, then its time period will
(a) increase
2. The graph of time period (T) of simple pendulum versus its
(b) decrease
length (l) is
(c) become zero
T T (d) remain the same
9. In the fig. S1 and S2 are identical springs. The oscillation
frequency of the mass m is f. If one spring is removed, the
frequency will become

.IN
(a) (b) m
S1 S2
O l O l (a) f (b) f × 2
AL
T T
(c) f ´ 2 (d) f / 2
10. A pendulum is undergoing S.H.M. The velocity of the bob
N
in the mean position is v. If now its amplitude is doubled,
keeping the length same, its velocity in the mean position
R

(c) (d) will be


U

(a) v/2 (b) v


O l (c) 2 v (d) 4 v
O
JO

l
11. The potential energy of a particle (Ux) executing S.H.M. is
3. Which of the following is a simple harmonic motion?
given by
(a) Particle moving through a string fixed at both ends.
U

k
(b) Wave moving through a string fixed at both ends. (a) Ux = (x - a) 2 (b) U x = k 1x + k 2 x 2 + k 3 x 3
(c) Earth spinning about its axis. 2
ED

(d) Ball bouncing between two rigid vertical walls. (c) U x = A e - bx (d) Ux = a constant
4. A rod is hinged vertically at one end and is forced to oscillate 12. A particle moves such that its acceleration ‘a’ is given by a
in a vertical plane with hinged end at the top, the motion of = – bx where x is the displacement from equilibrium position
the rod: and b is constant. The period of oscillation is
(a) is simple harmonic
(a) 2 p/b (b) 2 π / b
(b) is oscillatory but not simple harmonic
(c) is pericolic but not oscillatory (c) 2p/ b (d) 2 p / b
(d) may be simple harmonic 13. A particle executes S.H.M. having time period T, then the
5. The graph plotted between the velocity and displacement time period with which the potential energy changes is
from mean position of a particle executing SHM is (a) T (b) 2 T
(c) T/2 (d) ¥
(a) circle (b) ellipse
14. An instantaneous displacement of a simple harmonic
(c) parabola (d) straight line oscillator is x = A cos (wt + p/4). Its speed will be maximum
6. Acceleration of a particle executing SHM, at it’s mean at time
position is (a) p/4 w (b) p/2 w
(a) infinity (b) variable (c) p/w (d) 2 p/w
(c) maximum (d) zero 15. The tension in the string of a simple pendulum is
7. A vertical mass-spring system executes simple harmonic (a) constant
oscillations with a period of 2s. A quantity of this system (b) maximum in the extreme position
which exhibits simple harmonic variation with a period of (c) zero in the mean position
1 s is (d) None of these
Oscillations 357

16. A rectangular block of mass m and area of cross-section A (a)


Acceleration = – k(x + a)
floats in a liquid of density r . If it is given a small vertical (b)
Acceleration = k(x + a)
displacement from equilibrium. It undergoes oscillations (c)
Acceleration = kx
with a time period T then Acceleration = – k0x + k1x2
(d)
(a) T µ m (b) T µ r where k, k0, k1 and a are all postive.
22. Out of the following functions, representing motion of a
(c) T µ 1/ A (d) T µ 1 / r particle, which represents SHM?
17. If the magnitude of displacement is numerically equal to (A) y = sin wt - cos wt
that of acceleration, then the time period is (B) y = sin3 wt
(a) 1 second (b) p second æ 3p ö
(c) 2p second (d) 4p second (C) y = 5cos ç - 3wt ÷
è 4 ø
18. The graph shown in figure represents
(D) y = 1 + wt + w 2 t 2
Velocity
(a) Only (A)
(b) Only (D) does not represent SHM
(c) Only (A) and (C)
Time (d) Only (A) and (B)
23. The total mechanical energy of a spring-mass system in

.IN
1
(a) motion of a simple pendulum starting from mean simple harmonic motion is E = mw2 A 2 . Suppose the
2
position
oscillating particle is replaced by another particle of double
(b) motion of a simple pendulum starting from extreme
AL
the mass while the amplitude A remains the same. The new
position
mechanical energy will
(c) simple pendulum describing a horizontal circle
N
(a) become 2E (b) become E/2
(d) None of these (c) become 2 E (d) remain E
R

19. A particle executing simple harmonic motion along y-axis


24. The displacement of aparticle along the x-axis is given by x
U

has its motion described by the equation y = A sin(wt ) + B . = a sin2 wt. The motion of the particle corresponds to:
The amplitude of the simple harmonic motion is (a) simple harmonic motion of frequency w / p
JO

(a) A (b) B (b) simple harmonic motion of frequency 3w / 2 p


(c) A + B (d) A+ B (c) non simple harmonic motion
U

20. Resonance is an example of (d) simple harmonic motion of frequency w / 2 p


(a) tuning fork (b) forced vibration 25. The period of oscillation of a mass M suspended from a
ED

(c) free vibration (d) damped vibration spring of negligible mass is T. If along with it another mass
M is also suspended, the period of oscillation will now be
21. Which one of the following equations of motion represents
simple harmonic motion? (a) T (b) T / 2
(c) 2T (c) 2T

1. A simple pendulum performs S.H.M. about x = 0 with an (a) 5 (b) 5 2


amplitude a, and time period T. The speed of the pendulum
at x = a/2 will be (c) 5 3 (d) 10 2
3. A mass M is suspended from a spring of negligible mass.
a 3 pa 3 The spring is pulled a little and then released so that the
(a) p (b)
T 2T mass executes SHM of time period T. If the mass is increased
pa 3 p2 a by m, the time period becomes
5T
. Then the ratio of
m
is
(c) (d)
T T 3 M
2. A body of mass 5 gram is executing S.H.M. about a fixed 25 16
point O. With an amplitude of 10 cm, its maximum velocity (a) (b)
9 9
is 100 cm/s. Its velocity will be 50 cm s–1 at a distance 5 3
(in cm) (c) (d)
3 5
EBD_7179
358 PHYSICS

4. A tunnel has been dug through the centre of the earth and 13. Three masses of 500 g, 300 g and 100 g are suspended at
a ball is released in it. It executes S.H.M. with time period the end of a spring as shown, and are in equilibrium. When
(a) 42 minutes (b) 1 day the 500 g mass is removed, the system oscillates with a
(c) 1 hour (d) 84.6 minutes period of 2 second. When the 300 g mass is also removed,
5. A particle of mass 1 kg is moving in S.H.M. with an it will oscillate with a period of
amplitude 0.02 and a frequency of 60 Hz. The maximum
force acting on the particle is (a) 2 s
(a) 144 p2 (b) 188 p2
(c) 288 p 2 (d) None of these
(b) 4 s
6. The length of a second’s pendulum at the surface of earth
is 1 m. The length of second’s pendulum at the surface of
moon where g is 1/6th that at earth’s surface is (c) 8 s
(a) 1/6 m (b) 6 m 500 g
(c) 1/36 m (d) 36 m 300 g
(d) 1 s 100 g
7. The displacement of a S.H.M. doing particle when
K.E. = P.E. (amplitude = 4 cm) is 14. If the mass shown in figure is slightly displaced and then
(a) 2 2 cm (b) 2 cm let go, then the system shall oscillate with a time period of

1 m
(c ) cm (d) (a) 2p
2 cm

.IN
3k
2 k
8. The equation of SHM of a particle is A + 4p2x = 0 where a is 3m
instantaneous linear acceleration at displacement x. The (b) 2p
2k
frequency of motion is
AL
(a) 1 Hz (b) 4p Hz 2m k k
(c) 2p
3k
N
1
(c) Hz (d) 4 Hz
4 3k
R

(d) 2p m
é pt ù m
9. y = 2 (cm) sin ê + fú what is the maximum acceleration of
U

ë2 û 15. Two oscillators are started simultaneously in same phase.


the particle doing the S.H.M. After 50 oscillations of one, they get out of phase by p, that
JO

is half oscillation. The percentage difference of frequencies


p p2
(a) cm/s2 (b) cm/s2 of the two oscillators is nearest to
2 2
(a) 2% (b) 1%
U

p2 p
(c) cm/s2 (d) cm/s2 (c) 0.5% (d) 0.25%
ED

4 4
10. A simple pendulum has time period 't'. Its time period in a 16. Two wires are kept tight between the same pair of supports.
lift which is moving upwards with acceleration 3 ms–2 is The tensions in the wires are in the ratio 2 : 1, the radii are in
the ratio 3 : 1 and the densities are in the ratio 1 : 2. The ratio
9. 8 12.8 of their fundamental frequencies is
(a) t (b) t
12.8 9. 8 (a) 2 : 3 (b) 2 : 4
9.8 6. 8 (c) 2 : 5 (d) 2 : 6
(c) t (d) t
6. 8 9.8 17. The time period of the oscillating system (see figure) is
11. Two particles are oscillating along two close parallel straight m
lines side by side, with the same frequency and amplitudes. (a) T = 2p
k1k 2 k1
They pass each other, moving in opposite directions when
their displacement is half of the amplitude. The mean
m
positions of the two particles lie on a straight line (b) T = 2p
k1 + k 2 k2
perpendicular to the paths of the two particles. The phase
difference is
(a) 0 (b) 2p/3 (c) T = 2p mk1k 2 M
(c) p (d) p/6 (d) None of these
12. If a simple pendulum of length l has maximum angular
18. A particle of mass 10 gm is describing S.H.M. along a straight
displacement q, then the maximum K.E. of bob of mass m is
line with period of 2 sec and amplitude of 10 cm. Its kinetic
1 energy when it is at 5 cm from its equilibrium position is
(a) ml / g (b) mg / 2l
2 (a) 37.5p 2 erg (b) 3.75p 2 erg
(c) mgl (1 - cos q) (d) mgl sin q/2 (c) 375p 2 erg (d) 0.375p 2 erg
Oscillations 359

19. A boy is executing simple Harmonic Motion. At a pendulum of shorter length has completed oscillations
displacement x its potential energy is E 1 and at a [nT1=(n–1)T2, where T1 is time period of shorter length &
displacement y its potential energy is E2. The potential T2 be time period of longer wavelength and n are no. of
energy E at displacement (x + y) is oscillations completed]
(a) 5 (b) 1
(a) E = E1 - E2 (b) E = E1 + E2
(c) 2 (d) 3
(c) E = E1 + E2 (d) E = E1 - E2 28. A clock which keeps correct time at 20ºC, is subjected to
40ºC. If coefficient of linear expansion of the pendulum is
20. A particle undergoes simple harmonic motion having time
period T. The time taken in 3/8th oscillation is
12 × 10–6 per ºC. How much will it gain or loose in time ?
(a) 10.3 seconds/day (b) 20.6 seconds/day
3 5 (c) 5 seconds/day (d) 20 minutes/day
(a) T (b) T
8 8 29. A mass is suspended separately by two different springs
5 7 in successive order then time periods is t 1 and t 2
(c) T (d) T respectively. It is connected by both springs as shown in
12 12
fig. then time period is t0, the correct relation is
21. Two particles are executing S.H.M. of same amplitude and
frequency along the same straight line path. They pass
each other when going in opposite directions, each time
their displacement is half of their amplitude. What is the k1 k2
phase difference between them ?

.IN
(a) 5 p/6 (b) 2 p/3
m
(c) p/3 (d) p/6 AL
22. Lissajous figure obtained by combining x = a sin wt and (a) t 02 = t 12 + t 22 (b) t 0-2 = t 1-2 + t 2-2
y = a sin (wt + p/4) will be a/an
(c) t 0-1 = t1-1 + t -2 1 (d) t 0 = t1 + t 2
(a) ellipse (b) straight line
N
30. When an oscillator completes 100 oscillations its amplitude
(c) circle (d) parabola
1
R

23. A simple spring has length l and force constant K. It is cut


reduces to of its initial value. What will be its amplitude,
into two springs of lengths l1 and l2 such that l1 = n l2 3
U

(n = an integer). The force constant of spring of length l1 is when it completes 200 oscillations ?
(a) K (1 + n) (b) (K/n) (1 + n) 1 2
JO

(c) K (d) K/(n + 1) (a) (b)


8 3
24. The spring constant from the adjoining combination of
springs is 1 1
U

(c) (d)
6 9
ED

2K 31. A particle of mass m is fixed to one end of a light spring of


force constant k and unstretched length l. The system is
m rotated about the other end of the spring with an
angular velocity w, in gravity free space. The increase in
K K length of the spring will be
w
(a) K (b) 2 K k
(c) 4 K (d) 5 K/2 m
25. A Second’s pendulum is placed in a space laboratory
orbiting around the earth at a height 3 R from the earth’s
surface where R is earth’s radius. The time period of the mw2 l mw2 l
pendulum will be (a) (b)
k k - mw2
(a) zero (b) 2 3 mw 2 l
(c) (d) None of these
(c) 4 sec (d) infinite k + mw2
26. A simple pendulum attached to the roof of a lift has a time 32. Frequency of oscillation is proportional to
period of 2s in a stationary lift. If the lift is allowed to fall k 2k
freely the frequency of oscillations of pendulum will be m
(a) zero (b) 2 Hz
(c) 0.5 Hz (d) infinity 3k k
(a) (b)
27. Two simple pendulums of length 0.5 m and 20 m respectively m m
are given small linear displacement in one direction at the
same time. They will again be in the phase when the 2k m
(c ) (d)
m 3k
EBD_7179
360 PHYSICS

33. A pendulum bob is raised to a height h and released from


1
rest. At what height will it attain half of its maximum speed? (a) n (b)
n
3h h (c) n –1/2 (d) 1
(a) (b)
4 2 39. A uniform pole of length l = 2 L is laid on smooth horizontal
table as shown in figure. The mass of pole is M and it is
h
(c) (d) 0.707 h connected to a frictionless axis at O.A spring with force
4 constant k is connected to the other end. The pole is
34. A mass m fall on spring of spring constant k and negligible displaced by a small angle q0 from equilibrium position and
mass from a height h. Assuming it sticks to the pan and released such that it performs small oscillations. Then
executes simple harmonic motion, the maximum height upto
which the pan will rise is
mg
(a) M x =0
k O

mg é 2kh ù m 2L
(b) ê 1+ - 1ú
k ë mg û h M k
(a) w0 = (b) w 0 =
3k 3M

.IN
mg é 2kh ù
(c) ê 1+ + 1ú
k ë mg û k 3k k
(c) w0 = (d) w 0 =
M 2M
AL
mg é kh ù 40. A particle of mass is executing oscillations about the origin
(d) ê 1+ - 1ú
k ë mg û on the x-axis. Its potential energy is V(x) = k | x |3, where k is
N
a positive constant. If the amplitude of oscillation is a, then
35. The graph shown in figure represents
its time period T is
Velocity
R

(a) S.H.M.
1
U

(b) circular motion (a) proportional to (b) proportional to a


a
JO

(c) rectillinear motion 3


–a O +a Displacement
(d) uniform circular motion (c) independent a 2 (d) None of these
41. A circular hoop of radius R is hung over a knife edge. The
U

36. The time period of a simple pendulum of infinite length is period of oscillation is equal to that of a simple pendulum
of length
ED

(Re = radius of Earth)


(a) R (b) 2R
Re 2R e
(a) T = 2p (b) T = 2p 3R
g g (c) 3R (d)
2
42. In the figure shown, the spring is light and has a force
Re
(c) T = 2p (d) T = ¥ constant k. The pulley is light and smooth and the strring
2g is light . The suspended block has a mass m. On giving a
37. A block rests on a horizontal table which is executing SHM slight displacement vartically to the block in the downward
in the horizontal plane with an amplitude 'a'. If the coefficient direction from its equilibrium position the block executes
of friction is 'm', then the block just starts to slip when the S.H.M. on being released with time period T. Then
frequency of oscillation is
m
(a) T = 2p
1 mg mg k
(a) (b)
2p a a
m
(b) T = 2p
a 2k
a
(c) 2p (d) mg
mg 2m
(c) T = 2p
38. On Earth, a body suspended on a spring of negligible mass k
causes extension L and undergoes oscillations along length m
of the spring with frequency f. On the Moon, the same m
(d) T = 4p
quantities are L/n and f ' respectively. The ratio f '/f is k
Oscillations 361

43. A body of mass m falls from a height h onto a pan (of (a) 1.05 s (b) 0.52 s
negligible mass) of a spring balance as shown. The spring (c) 0.25 s (d) 0.03 s
also possesses negligible mass and has spring constant k.
47. A forced oscillator is acted upon by a force F = F0 sin wt.
Just after striking the pan, the body starts socillatory motion
in vertical directioin of amplitude A and energy E. Then 55
The amplitude of oscillation is given by .
mg 2
A= m 2w - 36w + 9
(a)
k The resonant angular frequency is
mg 2kh (a) 2 unit (b) 9 unit
(b) A= 1+ h
k mg (c) 18 unit (d) 36 unit
48. A straight rod of negligible mass is mounted on a frictionless
1 pivot and masses 2.5 kg and 1 kg are suspended at
(c) E = mgh + kA 2
2 distances 40 cm and 100 cm respectively from the pivot as
2 shown. The rod is held at an angle q with the horizontal
æ 2mg ö and released. Then
(d) E = mgh + ç
è 2k ÷ø
44. A coin is placed on a horizontal platform which undergoes
vertical simple harmonic motion of angular frequency w. 100cm

.IN
The amplitude of oscillation is gradually increased. The 40cm
coin will leave contact with the platform for the first time q
(a) at the mean position of the platform AL 1 kg

g
(b) for an amplitude of
w2
2.5kg
N
2
g
(c) for an amplitude of
R

w2 /////////////////////////////////////////////////////////////
U

(d) at the highest position of the platform


45. A point particle of mass 0.1 kg is executing S.H.M. of (a) the rod executes periodic motion about horizontal
JO

amplitude of 0.1 m. When the particle passes through the position after the release
mean position, its kinetic energy is 8 × 10–3 Joule. Obtain (b) the rod remains stationary after the release.
the equation of motion of this particle if this initial phase of
(c) the rod comes to rest in vertical position with 2.5 kg
U

oscillation is 45º.
mass at the lowest point
ED

æ pö (d) the rod executes periodic motion about vertical


(a) y = 0.1sin ç ±4t + ÷
è 4ø position after the release
DIRECTIONS for Qs. (49-50) : Each question contains
æ pö
(b) y = 0.2sin ç ±4t + ÷ STATEMENT-1 and STATEMENT-2. Choose the correct answer
è 4ø (ONLY ONE option is correct ) from the following-
æ pö (a) Statement -1 is false, Statement-2 is true
(c) y = 0.1sin ç ±2t + ÷
è 4ø (b) Statement -1 is true, Statement-2 is true; Statement -2 is a
correct explanation for Statement-1
(d) æ pö
y = 0.2sin ç ±2t +÷ (c) Statement -1 is true, Statement-2 is true; Statement -2 is not
è 4ø a correct explanation for Statement-1
46. A body of mass 0.01 kg executes simple harmonic motion (d) Statement -1 is true, Statement-2 is false
about x = 0 under the influence of a force as shown in
figure. The time period of S.H.M. is 49. Statement 1 : The graph between velocity and displacement
for a harmonic oscillator is an ellipse.
F(N)
Statement -2 : Velocity does not change uniformly with
displacement in harmonic motion.
80 50. Statement -1 : If the amplitude of a simple harmonic
0.2 x(m) oscillator is doubled, its total energy becomes four times.
–0.2
Statement -2 : The total energy is directly proportional to
–80 the square of the amplitude of vibration of the harmonic
oscillator.
EBD_7179
362 PHYSICS

Exemplar Questions elastic support as shown in figure. A and C are of the same
length, while B is smaller than A and D is larger than A. If A
1. The displacement of a particle is represented by the is given a transverse displacement,
æp ö (a) D will vibrate with maximum amplitude
equation y = 3cos ç - 2wt ÷ . The motion of the particle
è4 ø (b) C will vibrate with maximum amplitude
is
(c) B will vibrate with maximum amplitude
(a) simple harmonic with period 2p/w
(d) All the four will oscillate with equal amplitude
(b) simple harmonic with period p/w
(c) periodic but not simple harmonic 8. Figure shows the circular motion of a particle. The radius of
(d) non-periodic the circle, the period, sense of revolution and the initial
position are indicated on the figure. The simple harmonic
2. The displacement of a particle is represented by the
equation y = sin3 wt. The motion is motion of the x-projection of the radius vector of the rotating
particle P is
(a) non-periodic y

.IN
(b) periodic but not simple harmonic æ 2 pt ö p(t = 0)
(c) simple harmonic with period 2p/w (a) x(t ) = B sin ç ÷ T = 30s
è 30 ø
(d) simple harmonic with period p/w B
æ pt ö x
3. The relation between acceleration and displacement of four
AL x ( t ) = B cos
(b) ç ÷
particles are given below è 15 ø
(a) ax = +2x (b) ax = +2x2 æ pt p ö
N
(c) ax = –2x 2 (d) ax = –2x (c) x(t ) = B sin ç + ÷
è 15 2 ø
R

Which, one of the particle is exempting simple harmonic


motion? æ pt p ö
(d) x(t ) = B cos ç + ÷
U

4. Motion of an oscillating liquid column in a U-tube is è 15 2 ø


JO

(a) periodic but not simple harmonic 9. The equation of motion of a particle is x = a cos(at)2. The
(b) non-periodic motion is
(c) simple harmonic and time period is independent of the (a) periodic but not oscillatory
U

density of the liquid (b) periodic and oscillatory


(d) simple harmonic and time period is directly proportional
ED

(c) oscillatory but not periodic


to the density of the liquid
(d) neither periodic nor oscillatory
5. A particle is acted simultaneously by mutually perpendicular
simple harmonic motion x = a cos wt and y = a sin wt. The 10. A particle executing SHM maximum speed of 30 cm/s and a
trajectory of motion of the particle will be maximum acceleration of 60 cm/s2. The period of oscillation
(a) an ellipse (b) a parabola is
(c) a circle (d) a straight line p
6. The displacement of a particle varies with time according to (a) p sec (b) sec
2
the relation y = a sin wt + b cos wt.
p
(a) The motion is oscillatory but not SHM (c) 2p sec (d) sec
(b) The motion is SHM with amplitude a + b t
(c) The motion is SHM with amplitude a2 + b2
Past Years (2013-2017) NEET/AIPMT Questions
(d) The motion is SHM with amplitude a +b 2 2
11. A particle of mass m oscillates along x-axis according to
7. Four pendulums A, B, C and D are suspended from the equation x = a sin wt. The nature of the graph between
same momentum and displacement of the particle is
[NEET Kar. 2013]
G G
(a) straight line passing through origin
(b) circle
(c) hyperbola
C B A (d) ellipse
D
Oscillations 363

12. The oscillation of a body on a smooth horizontal surface is 14. When two displacements represented by y1 = asin(wt) and
represented by the equation, y2 = b cos(wt) are superimposed the motion is: [2015]
X = A cos (wt) a
where X = displacement at time t (a) simple harmonic with amplitude
b
w = frequency of oscillation
Which one of the following graphs shows correctly the (b) simple harmonic with amplitude a2 + b2
variation of ‘a’ with ‘t’? [2014]
(a + b)
(c) simple harmonic with amplitude
a 2
(a) O (d) not a simple harmonic
T t 15. A particle is executing a simple harmonic motion. Its maximum
acceleration is a and maximum velocity is b.
Then its time period of vibration will be : [2015 RS]
a
a b2
(b) O (a) (b)
T t b a

2pb b2
(c) (d)

.IN
a a2
a
(c) 16. A particle executes linear simple harmonic motion with an
O
T t amplitude of 3 cm. When the particle is at 2 cm from the
AL
mean position, the magnitude of its velocity is equal to that
of its acceleration. Then its time period in seconds is [2017]
N
a 5 4p
(a) (b)
(d)
R

O 2p 5
T t
U

2p 5
(c) (d)
13. A particle is executing SHM along a straight line. Its 3 p
JO

velocities at distances x1 and x2 from the mean position 17. A spring of force constant k is cut into lengths of ratio 1 : 2
are V1 and V2, respectively. Its time period is [2015]
: 3. They are connected in series and the new force constant
U

is k'. Then they are connected in parallel and force constant


x 22 – x12 V12 + V22
(a) 2p (b) 2p is k¢¢ . Then k' : k¢¢ is [2017]
ED

V12 – V22 x12 + x 22 (a) 1 : 9 (b) 1 : 11


(c) 1 : 14 (d) 1 : 6
V12 – V22 x12 – x 22
(c) 2p (d) 2p
x12 – x 22 V12 – V22
EBD_7179
364 PHYSICS

Hints & Solutions


EXERCISE - 1 20. (b)
21. (a) a = – kX, X = x + a.
1. (b) 2. (a) 3. (b) 4. (d) 5. (b)
In simple harmonic motion acceleration is directly
6. (d) 7. (b) 8. (b) proportional to the displacement from the mean
9. (d) Here effective spring factor, keff = 2 k ; position. Also the acceleration is in the opposite
direction of displacement.
1 2k
Frequency of oscillation, f = ; when one 22. (c) Only functions given in (A) & (C) represent SHM.
2p m
1 k 2 1
spring is removed, then spring factor = k ; 23. (d) E= m A Þ E = kA 2
2 m 2
1 k f Þ E does not depend on m
New frequency of oscillation f ¢ = =
2p m 2
a
24. (a) x = a sin2 wt = (1 –cos 2wt)
10. (c) v max = a ω ; v max
¢ = 2 a w = 2 v max = 2v 2

.IN
(since vmax = v) dx a
= 2 w sin 2 wt
dt 2
displacement x
12. (b) T = 2π = 2p b x = 2p/ b
AL d2x 4w2 a
acceleration Þ = × cos 2 wt
dt 2 2
11. (a) P.E. of body in S.H.M. at an instant,
N
w
This represents an S. H. M. of frequency =
1 1 p
R

U= m w 2 y 2 = ky 2
2 2
m
U

If the displacement, y = (a – x) then 25. (d) T = 2 p


K
JO

1 1
U= k(a - x)2 = k(x - a) 2 T1 M1
2 2 \ =
T2 M2
U

13. (c) P.E. changes from zero to maximum twice in each


vibration so its time period is T/2 M2 2M
ED

dx \ T2 = T1 M1 = T 1
(a) Velocity, v = M
14. = -A w sin (w t + p / 4)
dt
T2 = T1 2 = 2 T (where T1 =T)
Velocity will be maximum, when
EXERCISE - 2
wt + p/4 = p/2 or wt = p/2 – p/4 = p/4 or t
= p/4w 1. (a) v = w a 2 - y2
15. (d) Tension is maximum at the mean position.
At x= 0, v = w a 2 - o2 = wa.
16. (a) A rectangular block of mass m and area of cross-
section 2
a æ aö 3a 2
At x = , v ' = w a2 - ç ÷ = w
A floats in a liquid of density r. If it is given a small 2 è 2ø 4
vertical displacement from equilibrium. It undergoes
oscillations with a time period T then : T µ m. v' 3
\ =
v 2
4p 2
17. (c) &&
y = w2 y, w2 = 1, = 1 or T 2 = 4p 2 3 3pa 3pa æQ w = 2p ö
T2 or v' = wa = wa = ç ÷
2 2 T è Tø
or T = 2p second.
18. (a) At displacement ± a, the velocity is zero. At zero 2. (c) v max = 100 = a w ; w = 100/a = 100/10 = 10 rad/s
displacement, velocity is maximum. v2 = w2 (a2 – y2) or 502
19. (a) The amplitude is a maximum displacement from the = 102 (102 – y2) or 25 = 100 – y2
mean position.
or y = 75 = 5 3 cm.
Oscillations 365

M 1 1
3. (b) T = 2p
k
10. (a) tµ
9.8
, t' µ
12.8
(Q g ' = 9.8 + 3 = 12.8)

M+m t' 9.8 9.8


T¢ = 2 p k \ = Þ t' = t
k t 12.8 12.8
5 M M+m M 11. (b) Equation of SHM is given by
Þ 2p = 2p
3 k k x = A sin (wt + d)
(wt + d) is called phase.
25 M M + m m 16
Þ = Þ = A
9 k k M 9 When x = , then
2

R 64 ´ 106 22 8 ´ 10 3 1
4. (d) T = 2p = 2p = 2 ´ ´ sin (wt + d) =
g 9.8 7 7´ 2 2
p
Þ wt + d =
2 ´ 22 ´ 8 ´ 1000 6
= min = 84.6 min
49 ´ 60 p
or f1 =
5. (c) Max. force = mass × max. acceleration 6

.IN
= m 4 p2 n2 a = 1 × 4 ×p2 × (60)2 × 0.02 = 288 p2 For second particle,

l
; 2 = 2p l = 2p l¢
6. (a) T = 2p
AL 1
A
g g (g / 6 )
A
Time period will remain constant if on moon, 2 x = A/2
N
l' = l/6 = 1/6 m x=0
R

7. (a) Kinetic energy,


p 5p
( )
f2 = p - =
U

1 1
K.E = mv2 = mw2 A 2 - x 2 6 6
2 2
\ f = f2 - f1
JO

Potential energy,
4p 2p
1 1 = =
P.E = kx 2 = mw 2 x 2 6 3
U

2 2 12. (c) When the bob moves from maximum angular


displacement q to mean position, then the loss of
ED

putting, K.E = P.E


gravitational potential energy is mgh
1
2
( 1
)
mw2 A 2 - x 2 = mw2 x 2 Þ A 2 = 2x 2
2
where h = l (1 - cos q)

A 400 100
x= = 2 2 cm. 13. (d) T = 2p , T ' = 2p
2 k k

8. (a) Comparing A + w2x = 0 T' 100 1 T 2


\ = = Þ T' = = = 1 s
2 2
w = 4 p Þ w = 2p , 2pn = 2p, n = 1Hz T 400 2 2 2
14. (b) The equivalent situation is a series combination of
æ pt ö two springs of spring constants k and 2k.
9. (b) y = 2 sin ç + f ÷
è2 ø If k' is the equivalent spring constant, then
(k )(2 k ) 2 k
dy p æ pt ö k' = =
velocity of particle = 2 ´ cos ç + f ÷ 3k 3
dt 2 è 2 ø
3m
d2y p2 æ pt ö Þ T = 2p
acceleration =- sinç + f ÷ 2k
dt 2 è 2 ø 15. (b) Phase change p in 50 oscillations.
Phase change 2p in 100 oscillations.
p2
Thus a max = So frequency different ~ 1 in 100.
2
EBD_7179
366 PHYSICS

1 2T 1 T 1 (n + 1)
n= , n' = \ = or k = (n + 1) K
16. (a) l (3D) pr lD p(2r) K k
The spring of length l1 (= n l2) will be equivalent to n
n 2 2 springs connected in series where spring constant
\ = ´ 2=
n' 3 3 k
k¢ = = (n + 1) K / n & spring constant of length l2
17. (b) n
1 is K(n+1).
2 2 2
18. (c) Kinetic energy K = mw (a - y ) 24. (c) Here all the three springs are connected in parallel to
2
mass m. Hence equivalent spring constant
1 æ 2p ö
2 k = K + K + 2 K = 4 K.
= ´ 10 ´ ç ÷ [102 - 52 ] = 375 p 2 erg 25. (d) The second pendulum placed in a space laboratory
2 è 2ø
orbiting around the earth is in a weightlessness state.
1 2 2 E1 1 2 E2 Hence g = 0 so T = ¥
19. (b) E1 = kx Þ x = , E2 = ky 2 Þ y = 26. (a) When lift is falling freely, the effective acceleration
2 k 2 k
due to gravity inside the lift is zero i.e. g' = g – g = 0.
1 2E Therefore time period will be infinity and frequency is
and E = k ( x + y )2 Þ x + y =
2 k zero
27. (b) Let T1, T2 be the time period of shorter length and

.IN
2 E1 2 E2 2E
Þ + = Þ E1 + E2 = E longer length pendulums respectively. As per question,
k k k
n T1 = (n – 1) T2 ;
T
20. (c) Time to complete 1/4th oscillation is s. Time to
4
AL 0 .5 20
so n 2 p = ( n - 1) 2 p
1 g g
complete th vibration from extreme position is
8
N
obtained from or n = (n - 1) 40 » (n - 1) 6
R

a 2p T Hence n = 6/5 » 1
y = = a cos w t = a cos t or t = s
2 T 6
U

So time to complete 3/8th oscillation l DT 1


28. (a) T = 2p ; = 2 p ´ (l / g) -1/ 2 ´ Dl / l
g T 2
JO

T T 5T
= + =
4 6 12
l + Dl
21. (b) y = a sin (wt + f) ; when y = a/2, (Q T + DT = 2p )
g
U

a
then= a sin (w t + f ) DT 1 Dl 1
ED

2 \ = = a Dq
5p T 2 l 2
1 p
or sin (w t + f) = = sin or sin
2 6 6 1 -6 -5
= ´12 ´ 10 ´ (40 - 20) = 12 ´10
So phase of two particles is p/6 and 5 p/6 radians 2
Hence phase difference = (5 p/6) – p/6 = 2 p/3 –5
22. (a) sin wt = x/a and D T = T ´ 12 ´ 10 -5 = 24 × 60 × 60 × 12 × 10
= 10.3 s/day
cos wt = 1 - sin 2 wt = 1 - x 2 / a 2
m 4 p2m 4 p2 m
y = sin (wt + p/4) 29. (b) t 1 = 2p or t12 = or k1 = 2
k1 k1 t1
= sin wt cos p/4 + cos wt sin p/4
x 1 æ ö 1
= ´ + ç 1- x2 / a2 ÷´ 4 p2m 4p 2 m
a 2 è ø 2 Similarly, k 2 = and (k1 + k 2 ) =
t 22 t 02
2 2
or 2 y= x+ a -x
4 p2m 4 p 2m 4 p2m 1 1 1
or 2 y 2 = x 2 + (a 2 - x 2 ) + 2 x a 2 - x 2 \ = + or = +
t 02 t12 t 22 t 02 t12 t 22
2 2 2
= a +2x a - x 30. (d) It is a damped oscillation, where amplitude of
It is an equation of an ellipse.
23. (b) Let k be the force constant of spring of length l2. oscillation at time t is given by A = a 0 e - γ t
Since l1 = n l2, where n is an integer, so the spring is where a0 = initial amplitude of oscillation
made of (n + 1) equal parts in length each of length l2. g = damping constant.
Oscillations 367

a0
As per question, = a 0 e - γ 100 / ν …… (i) mg m 2g 2 2mgh
3 or x= ± +
(where n is the frequency of oscillation) k k2 k

and A = a 0 e - γ 200 / ν …… (ii)


mg m 2g 2 2 mgh mg
x = A– = + -
a k k 2 k k
From (i) ; o = a0e-γ ´100/ ν …… (iii)
3
Dividing equation (ii) by (iii), we have mg é 2kh ù
- γ 200 / ν
= ê 1+ - 1ú
A e 1 k êë mg úû
= - γ ´ 100 / ν
= e =
a 0 (1/ 3) e - γ ´ 100 / ν 3 35. (a) t = 0, v maximum. The motion begins from mean
1 1 1 position. So it represents S.H.M.
or A = a 0 ´ ´ = a 0 36. (a) The both of the pendulum will more along a straight
3 3 9
31. (b) Let x = the increase in the length of spring. Then the line AB.
particle moves along a circular path of radius (l + x),
and the spring force = kx = centripetal force

mw2 l
\ mw2 (l + x) = kx or x =

.IN
k - mw2 A
x
B
q q
k 2k
32. (a)
AL mg
x
Re
N
Let mass is displaced towards left by x then force on
mass = – kx – 2kx = – 3kx
R

[negative sign is taken because force is opposite to The direction of the Earth's gravitational field is radial
the direction of motion]
U

GM e m
Now, F = = mg
3k R e2
Þ F = - 3kx = -mw x Þ w =
2 .
JO

m
x GM e m
w 1 3k Fx = -F cos q = -F =- x = –kx
f= = . Re R 3e
U

2 p 2p m
Thus it is propotional to 3k / m GM e m
ED

where k =
33. (a) Maximum velocity can be found out by energy R e3
conservation. Time period of a simple harmonic oscillator,
1
mgh = mv 2 or v max = 2gh ; m m
2 T = 2p = 2p
v max gh
k GM e m / R 3e
= v' =
2 2
Re
1
mgh = m( v ' ) 2 + mgh ' ; GM e Re
2 or T = 2p = 2p
1 mgh R e2 g
mgh = + mgh '
2 2
37. (a) For the block is about to slip, mg = w 2 a
3mgh 3h
Þ = mgh ' or h ' =
4 4 mg
34. (b) Loss in PE of mass = gain in PE of the spring or Þ w =
a
1 2
\ mg (h + x) = kx
2 mg 1 mg
Þ 2pν = Þν=
a 2p a
kx 2 – 2mgx – 2mgh = 0
38. (d) Oscillations along spring length are independent of
2mg ± 4 m 2 g 2 - 4k (-2mgh) gravitation.
Þ x=
2k
EBD_7179
368 PHYSICS

1 4 æ xö
39. (c ) I = M(2L) 2 = ML2 Þ 2T = k ç x 0 ÷
3 3 è 2ø
Force applied by the spring is F = – kx
kx
Þ F = – k (2Lq) Þ (T - T0 ) =
4
(q is the angular displacement from the equilibrium
position). Further m m
r r Þ Time period = 2 p = 4p
k k
t =| l ´ F |= 4L2 k sin q = -4L2 kq
4
..
Also, t = Ia = I q = -4L2 kq 1 2
43. (b) mg(h + y) = ky
..
2
3k
Þ q+ q=0
M mg mg 2kh
Þ y= ± 1+
k k mg
3k
Þ w0 = At equilibrium
M

.IN
mg = ky0
40. (a) V(x) = k | x |3
mg
dV(x) Þ y0 =
since, F = - = -3k | x |2 …… (i)
dx
AL k

x = a sin (wt) mg 2kh


Þ Amplitude A = y – y0 = 1+
N
This equation always fits to the differential equation k mg
R

Energy of oscillation is
d2 x 2
= -w2 x or m d x = -mw 2 x
U

dt 2 dt 1 æ mg ö
2
E = kA 2 = mgh + ç
è 2k ÷ø
JO

Þ F = – mw2x …… (ii) 2
Equations (i) and (ii) give 44. (b) For block A to move in S.H.M.
U

2 2
-3k | x | = - mw x N
ED

A
3kx 3ka
Þ w= = [sin(wt)]1/ 2
m m
mg x
mean
Þ wµ a position

1 mg – N = mw2x
Þ Tµ
a where x is the distance from mean position
For block to leave contact N = 0
2R l
41. (b) T = 2p = 2p
g g g
Þ mg = mw2 x Þ x =
w2
42. (d) Let the extension in the spring be x0 at equilibrium. If
F0 be the tension in the string then F0 = kx0. Further if 45. (a) The displacement of a particle in S.H.M. is given by
T0 is the tension in the thread then T0 = mg and y = a sin (wt + f)
2T0 = kx0.
dy
Let the mass m be displaced through a slight velocity = = wa cos (wt + f)
dt
displacement x downwards. Let the the new tension in
the string and spring be T and F respectively. The velocity is maximum when the particle passes
through the mean position i.e.,
æ xö æ dy ö
Þ F = ç x 0 ÷ and F = 2T çè ÷ø
è 2ø dt max = w a
Oscillations 369

The kinetic energy at this instant is given by A simple harmonic motion is always periodic. So motion
2 is periodic simple harmonic.
1 æ dy ö 1
mç ÷ = mw2 a2 = 8 × 10–3 joule From the given equation,
2 è dt ø max 2
æp ö
1 y = 3cos ç - 2wt ÷
or × (0.1) w2 × (0.1)2 = 8 × 10–3 è4 ø
2
Solving we get w = ± 4 Compare it by standard equation
Substituting the values of a, w and f in the equation of y = a cos(wt + f)
S.H.M., we get
y = 0.1 sin (± 4t + p /4) metre. So, w ' = 2w
80 2p 2p p
46. (d) Slope of F - x curve = – k = - Þ k = 400 N/m, = 2w Þ T ' = =
0.2 T' 2w w
m p
Time period, T = 2p = 0.0314 sec. Hence, the motion is SHM with period .
k w
47. (b) At resonance, amplitude of oscillation is maximum
2. (b) A motion will be harmonic if a µ displacement and a
Þ 2w2 – 36w + 9 is minimum
simple harmonic motion is always periodic but all
Þ 4w – 36 = 0 (derivative is zero)
Þ w= 9 simple harmonic motion are periodic but all periodic

.IN
are not harmonic.
48. (b) Torque about hinge
As given equation of motion is
2.5 g × 0.40 cos q – 1g × 1 cos q = 0 AL
49. (c) 50. (b) y = sin 3 wt

EXERCISE - 3 = (3sin wt - 4sin 3wt ) / 4


N
Exemplar Questions [Q sin 3q = 3sin q - 4sin 3 q]
R

Differentiating both side w.r.t. t


æp ö
(b) As given that, y = 3cos ç - 2wt ÷
U

1. dy é d d ù
è 4 ø So, v = dt = ê dt (3sin wt ) - dt (4 sin 3wt ) ú / 4
ë û
JO

Velocity of the particle


dy d é æp öù 3 é (3sin q - sin 3q) ù
v= = 3cos ç - 2wt ÷ ú or sin q = ê ú
dt dt êë è4 øû ë 4 û
U

é æp öù dy
ED

= 3(-2w) ê - sin ç - 2wt ÷ú 4 = 3w cos wt - 4 ´ [3w cos 3wt ]


ë è4 øû dt
Again, differentiating both side w.r.t. t
æp ö
= 6w sin ç - 2wt ÷
è4 ø d2y
4´ = -3 w 2 sin wt + 36 w sin 3wt
So, acceleration, dt 2
dv d é æp öù
a= = 6w sin ç - 2wt ÷ ú d 2 y -3 w 2 sin wt + 36 w sin 3wt
dt dt êë è 4 øû a= =
dt 2 4
æp ö
= (6w) ´ (-2w) cos ç - 2wt ÷ æ d2y ö
è4 ø So, çç 2 ÷÷ is not directly proportional to y.
è dt ø
é æp öù
= -4w2 ê3cos ç - 2wt ÷ú So motion is not harmonic.
ë è4 øû
y(t ) = sin 3 wt ,
a = -4w2 y
In simple harmonic motion acceleration (or force) is y (t + T ) = sin 3 (wt + T )
directly proportional to the negative of displacement
of particle é 2p ù
Þ as acceleration, a µ - y = sin 3 ê (t + T ) ú
ëT û
Hence, due to negative sign motion is simple harmonic
motion (SHM.) = sin3 (2p + wt ) = sin 3 wt
EBD_7179
370 PHYSICS

3. (d) For motion to be SHM acceleration of the particle must


y ' = a 2(cos wt cos 45° + sin wt sin 45°)
be opposite of restoring force and proportional to
negative of displacement. So, F = ma = m(–2x) = a 2 cos(wt - 45°)
i.e., F = –2mx, so F µ - x So, the displacement is not straight line and not
Hence, ax = –2x parabola also.
We should be clear that x has to be linear. Now, squaring and adding eqs. (i) and (ii),
4. (c) Consider a U-tube filled in which a liquid column x2 + y 2 = a 2 [Q cos 2 wt + sin 2 wt = 1]
oscillates. When liquid column lifted upto height y
This is the equation of a circle, so motion in circular
from A to B in arm Q. The liquid level in arm P becomes
(independent of time).
at C', so the difference between the height of two
Clearly, the locus is a circle of constant radius a.
columns are = AB + A'C = y + y = 2y.
6. (d) As given that, the displacement is
In this case, restoring force acts on the liquid due to
gravity. Acceleration of the liquid column, can be y = a sin wt + b cos wt
calculated in terms of restoring force. Let a = A sin f ...(i)
P Q
and b = A cos f ...(ii)
B' B Squaring and adding (i) and (ii)
y

.IN
A' A a 2 + b2 = A2 sin 2 f + A2 cos 2 f
h C
C'
AL A = a 2 + b2 (amplitude)
Equilibrium level y = A sin f× sin wt + A cos f cos wt
Restoring force,
y = A cos (wt - f)
f = Weight of liquid column of height 2y
N

f = -( A ´ 2 y ´r) ´ g = -2 Argy dy
= - A w sin(wt - f)
R

dt
[Q h = rv]
U

As restoring force at A opposite to gravitational force d2y


2
= - Aw 2 cos (wt - f) = - w 2 y
JO

as liquid is lifted against wt (mg )mk = 2 Arg dt

Then f µ - y so motion is SHM. d2y


Thus, µ (- y ) so, motion is SHM.
dt 2
U

Time period
Hence, it is an equation of SHM with amplitude
ED

m A ´ (2h) ´ r h
T = 2p = 2p = 2p A = a2 + b2 .
k 2 Arg g
7. (b) When pendulum vibrate with transverse vibration then
h l
T = 2p T = 2p
g g
Time period is independent of the density of the liquid l = length of pendulum.
and motion is harmonic.
5. (c) The resultant-displacement can be find by adding x Elastic support
G G
and y-components.
According to variation of x and y, trajectory will be
predicted, so resultant displacement is y' = (x + y) e
B
As given that, A
C
x = a cos wt ...(i) D
y = a sin wt ...(ii) Through the elastic rigid support the disturbance is
transferred to all the pendulum A and C are having
So, y ' = (a cos wt + a sin wt )
length and hence same frequency. They will be in
Þ y ' = a(cos wt + sin wt ) resonance, because their time period of oscillation.
So, a periodic force of period (T) produces resonance
é cos wt sin wt ù in A and C and they will vibrate with maximum amplitude
y' = a 2 ê + ú
ë 2 2 û as in resonance.
Oscillations 371

8. (a) As the particle (P) is executing circular motion with Eqs. (i) and (ii), we get
radius B. w(wa) = 60 Þ w(30) = 60
Let particle P is at Q at instant any (t), foot of w = 2 rad/s
perpendicular on x axis is at R vector OQ makes Ðq ,
with its zero position not P displacement of particles 2p
= 2 rad/s
for O to R. T
Consider angular velocity of the particle executing T = p sec
circular motion is w and when it is at Q makes and Past Years (2013-2017) NEET/AIPMT Questions
angle q as shown in the diagram.
v2 y2
y 11. (d) As + = 1 This is the equation of ellipse.
a 2w 2 a 2
p(t=0) Hence the graph is an ellipse. P versus x graph is
r Q similar to V versus x graph.
q q
90–q
x 12. (c) Displacement, x = A cos (wt) (given)
O
BR dx
= - Aw sin (wt)
Velocity, v =
dt
dv
Acceleration, a = = - Aw 2 cos wt
Clearly, q = wt dt
Hence graph (c) correctly dipicts the variation of a

.IN
Now, we can write with t.
OR = OQ cos(90 – q) (Q OR = X ) 13.
AL (a) As we know, for particle undergoing SHM,

x = OQ sin q = OQ sin wt V = w A2 – X2
V12 = w2 (A 2 – x12 )
= r sin wt [Q OQ = r ]
N
V22 = w2 (A 2 – x 22 )
2p æ 2p ö
R

\ x = B sin t = B sin ç t ÷ Substracting we get,


T è 30 ø
V12 V22
U

2
æ 2p ö + x1 = + x 22
x = B sin ç t ÷ w2 w2
JO

è 30 ø
V12 – V22
Hence, this equation represents SHM. Þ 2
= x 22 – x12
w
9. (c) According to the question,
U

V12 – V22
Þ w =
x = a cos(at )2 x 22 – x12
ED

is a cosine function, so it is an oscillatory motion. x 22 – x12


Þ T = 2p
Now, at, t = (t + T). The equation of motion of particle V12 – V22
14. (b) The two displacements equations are y1 = a sin(wt)
x(t + T ) = a cos[a(t + T )]2 [Q x(t ) = a cos(at )2 ]
æ pö
2 2 and y2 = b cos(wt) = b sin ç wt + ÷
= a cos[at + aT + 2atT ] ¹ x (t ) è 2ø
yeq = y1 + y2
where, T is supposed as period of the function w(t).
Hence, it is oscillatory but not periodic. æ pö
= a sinwt + b coswt = a sinwt + b sin ç wt + ÷
10. (a) Let us consider a equation of an SHM is represented è 2ø
by y = a sin wt Since the frequencies for both SHMs are same,
resultant motion will be SHM.
dy
v= = aw cos wt p
dt Now Aeq = a 2 + b 2 + 2ab cos
2
(v )max = aw = 30 cm/sec (given) ...(i)
b Þ Aeq = a 2 + b2
dv
Acceleration (a) = = - aw 2 sin wt
dt

amax = w 2 a = 60 (given) ...(ii)


a
EBD_7179
372 PHYSICS

15. (c) As, we know, in SHM 17. (b) Let l be the complete length of the spring.
Maximum acceleration of the particle, a = Aw2 l l l
Maximum velocity, b = Aw Length when cut in ratio, 1 : 2 : 3 are , and
6 3 2
a 1
Þ w=
b Spring constant (k) µ
length (l)
2p 2pb é 2p ù Spring constant for given segments
Þ T=
w
=
a êëQ w = T úû
k1 = 6k, k2 = 3k and k3 = 2k
16. (b) Given, Amplitude A = 3 cm When they are connected in series
Which particle is at x = 2 cm 1 1 1 1
= + +
According to question, magnitude of velocity k ' 6k 3k 2k
= acceleration
1 6
Þ =
w A 2 - x 2 = xw2 k ' 6k
\ Force constant k' = k
2 2 æ 2p ö And when they are connected in parallel
(3) - (2) = 2 ç ÷
è T ø k" = 6k + 3k + 2k

.IN
4p 4p Þ k" = 11k
5= Þ T= Then the ratios
T 5 AL k' 1
= i.e., k' : k" = 1 : 11
k " 11
N
R
U
JO
U
ED
15 Waves

WAVE MOTION (Compression) l


Wave motion is a type of motion in which the disturbance travels C C C C
from one point of the medium to another but the particles of the
medium do not travel from one point to another.
R R R

.IN
For the propagation of wave, medium must have inertia and (Rarefaction)
elasticity. These two properties of medium decide the speed of
wave.
AL l
Logitudinal waves propagate through medium with the help of
There are two types of waves compressions and rarefactions.
1. Mechanical waves: These waves require material medium Equation of a Harmonic Wave
N
for their propagation. For example : sound waves, waves in Harmonic waves are generated by sources that execute simple
stretched string etc. harmonic motion.
R

2. Non-mechanical waves or electromagnetic waves : These A harmonic wave travelling along the positive direction of x-axis
U

waves do not require any material medium for their is represented by


y = A sin(wt - kx )
JO

propagation. For example : light waves, x-rays etc.


There are two types of mechanical waves ì æ t x öü
= A sin í 2 p ç - ÷ ý
(i) Transverse waves : In the transverse wave, the î è T l øþ
U

particles of medium oscillate in a direction


ì 2p ü
ED

perpendicular to the direction of wave propagation. = A sin í ( vt - x ) ý


Waves in stretched string, waves on the water surface îl þ
are transverse in nature. where,
Transverse wave can travel only in solids and surface of liquids. y = displacement of the particle of the medium at a location
x at time t
Transverse waves propagate in the form of crests and troughs.
A = amplitude of the wave
l = wavelength
Wave motion
Crest T = time period
v = ul, wave velocity in the medium
2p
w= , angular frequency
T
l
2p
Trough k = , angular wave number or propagation constant.
Wavelength l
All electromagnetic waves are transverse in nature. If the wave is travelling along the negative direction of
x-axis then
(ii) Longitudinal waves : In longitudinal waves particles
of medium oscillate about their mean position along y = A sin(wt + Kx)
the direction of wave propagation.
ì æ t xöü
Sound waves in air are longitudinal. These waves can travel in = A sin í 2 p ç + ÷ ý .
î è T løþ
solids, liquids and gases.
EBD_7179
374 PHYSICS

Differential equation of wave motion : (ii) The speed of transverse waves on stretched string is given
by
d2y 1 d2 y
= T
dx2 v 2 dt 2 v=
m
Relation between wave velocity and particle velocity :
where T is the tension in the string and m is the mass per
The equation of a plane progressive wave is unit length of the string.
y = A sin(wt - kx ) ... (i) Speed of Longitudinal Waves :
The particle velocity The speed of longitudinal waves in a medium of elasticity E and
dy density r is given by
vp = = Aw cos(w t - kx) ... (ii)
dt E
v=
Slope of displacement curve or strain r
dy For solids, E is replaced by Young's modulus (Y)
= - Ak cos( w t - kx ) ... (iii)
dx Y
vsolid =
Dividing eqn. (ii) by (iii), we get r
dy dt w For liquids and gases, E is replaced by bulk modulus of
= elasticity (B)

.IN
-dy dx k
B
vp æ w ö vliquid/gas =
Þ =v çè since = v, wave velocity÷ r
-dy dx k ø
AL
The density of a solid is much larger than that of a gas but the
elasticity is larger by a greater factor.
dy
Þ v p = - v. vsolid > vliquid > vgas
N
dx
Speed of Sound in a Gas :
R

i.e., Particle velocity = – wave velocity × strain. Newton's formula.


Particle velocity changes with the time but the wave velocity is
U

constant in a medium. P
v=
Relation between phase difference, path difference and time r
JO

difference : where P is the atmospheric pressure and r is the density of air at


• Phase difference of 2p radian is equivalent to a path STP.
difference l and a time difference of period T. Laplace's correction
U

2p gP
ED

• Phase difference = × path difference v=


l r
2p l where g is the ratio of two specific heats Cp and Cv
f= ´x Þ x = ´f
l 2p Power and Intensity of Wave Motion :
If a wave is travelling in a stretched string, energy is transmitted
2p
• Phase difference = × time difference along the string.
T
Power of the wave is given by
2p T
1 2 2
f= ´t Þ t = ´f
P= mw A v where m is mass per unit length.
T 2p
2
T Intensity is flow of energy per unit area of cross section of the
• Time difference = × path difference string per unit time.
l
T l 1 2 2
t= ´ x Þ x = ´t Intensity I = rw A v
l T 2
Principle of Superposition of Waves :
Speed of Transverse Waves :
If two or more waves arrive at a point simultaneously then the
(i) The speed of transverse waves in solid is given by
net displacement at that point is the algebraic sum of the
h displacement due to individual waves.
v=
r y = y1 + y2 + ............... + yn.
where y1, y2 .......... yn are the displacement due to individual
where h is the modulus of rigidity of the solid and r is the
waves and y is the resultant displacement.
density of material.
Waves 375

INTERFERENCE OF WAVES When sound wave is reflected from a rigid boundary or denser
When two waves of equal frequency and nearly equal amplitude medium, the wave suffers a phase reversal of p but the nature
travelling in same direction having same state of polarisation does not change i.e., on reflection the compression is reflected
in medium superimpose, then intensity is different at different back as compression and rarefaction as rarefaction.
points. At some points intensity is large, whereas at other points When sound wave is reflected from an open boundary or rarer
it is nearly zero. medium, there is no phase change but the nature of wave is
Consider two waves changed i.e., on reflection, the compression is reflected back as
y1 = A1sin (wt – kx) and rarefaction and rarefaction as compression.
y2 = A2 sin (wt – kx + f)
By principle of superposition Keep in Memory
y = y1 + y2 = A sin (wt – kx + d) (i) For a wave, v = f l
where, A2 = A12 + A22 + 2A1A2 cos f, (ii) The wave velocity of sound in air
A2 sin f E Yr gRT
and tan d = v= = =
A1 + A2 cos f r r M
As intensity I µ A2 dx
(iii) Particle velocity is given by v p = . It changes with time.
So, resultant intensity I = I1 + I2 + 2 I1 I 2 cos f dt
The wave velocity is the velocity with which disturbances
For constructive interference (maximum intensity) :
w

.IN
Phase difference, f = 2np travel in the medium and is given by v w = .
k
and path difference = nl
(iv) When a wave reflects from denser medium the phase change
where n = 0, 1, 2, 3, ...
is p and when the wave reflects from rarer medium, the
AL
Þ Amax = A1 + A2 and Imax = I1 + I2 + 2 I1I2 phase change is zero.
(v) In a tuning fork, the waves produced in the prongs is
= ( I1 + I 2 ) 2 transverse whereas in the stem is longitudinal.
N
For destructive interference (minimum intensity) : (vi) A medium in which the speed of wave is independent of the
R

Phase difference, f = (2n + 1)p, frequency of the waves is called non-dispersive. For
example air is a non-dispersive medium for the sound waves.
U

l
and path difference = ( 2n - 1) ; where n = 0, 1, 2, 3, ... (vii) Transverse waves can propagate in medium with shear
2 modulus of elasticity e.g., solid whereas longitudinal waves
JO

Þ Amin = A1 – A2 and Imin = I1 + I2 – 2 I1I 2 need bulk modulus of elasticity hence can propagate in all
media solid, liquid and gas.
= ( I1 – I 2 ) 2
U

Energy Transported by a Hormonic Wave Along a String :


Results : Kinetic energy of a small element of length dx is
ED

(1) The ratio of maximum and minimum intensities in any 2


interference wave form. 1 æ ¶y ö
dk = (m dx )ç ÷ where m = mass per unit length
2 2
2 è ¶t ø
I max æ I1 + I 2 ö æ A1 + A2 ö
=ç ÷ =ç 1
I min è I1 - I 2 ø è A1 - A2 ÷ø dk = ém w2 A2 cos 2 (kx - wt ) ù dx
2ë û
(2) Average intensity of interference in wave form : and potential energy stored
I max + I min 1 æ ¶y ö 1
2
Iav =
2 dU = (T dx) ç ÷ = m w2 A2 cos 2 (kx - wt )dx
2 è ¶x ø 2
Put the value of Imax and Imin Example 1.
or Iav = I1 + I2 The displacement y (in cm) produced by a simple harmonic
If A = A1 = A2 and I1 = I2 = I wave is given by y = (10/p) sin (2000 pt – px /17). What will
then Imax = 4I, Imin = 0 and Iav = 2I be the periodic time and maximum velocity of the particles
(3) Condition of maximum contrast in interference wave form in the medium?
A1 = A2 and I1 = I2 Solution :
then Imax = 4I and Imin = 0
10 æ px ö 10 æ x ö
For perfect destructive interference we have a maximum y= sin ç 2000pt - ÷ or y = sin 2 pç1000 t - ÷
contrast in interference wave form. p è 17 ø p è 34 ø
The standard equation of S.H.M. is,
Reflection of Waves :
A mechanical wave is reflected and refracted at a boundary é t xù
y = a sin 2 p ê - ú ;
separating two media according to the usual laws of reflection ëT l û
and refraction.
EBD_7179
376 PHYSICS

\ By comparison of the standard equation to above given Solution :


equation, The velocity of sound in air is given by
1 gP
we get T = = 10 -3 sec . E g RT
1000 v= = =
r r M
Particle velocity
(a) In terms of density and pressure
dy 10 æ px ö vH PH rair rair
= = ´ 2000 p cos ç 2000 p t - ÷
dt p 17 ø ´
è vair = rH Pair = rH [as Pair = PH]

æ dy ö rair
Þç ÷ = 20,000 cm / s = 200 m / s (as vmax = Aw) or vH = vair ´ = 332 ´
16
= 1328 m/s
è dt ø max rH 1
Example 2. (b) In terms of temperature and molecular weight
A progressive wave of frequency 500 Hz is travelling with v Ne g Ne M W
a velocity of 360 m/s. How far apart are two points 60o out = ´ [as TN = TW]
vW M Ne g W
of phase?
Now as neon is monatomic (g = 5/3) while water vapours
Solution :
polyatomic (g = 4/3) so
We known that for a wave v = f l
v 360 v Ne ( 5 / 3) ´ 1.8 ´ 10-2 5 1.8

.IN
so l = = ´
f 500
= 0.72 m vW = ( 4 / 3) ´ 2.02 ´ 10-2
=
4 2.02
= 1.05

Now as in a wave path difference is related to phase BEATS


AL
difference by the relation. When two wave trains slightly differing in frequencies travel
Phase difference Df = 60o = (p/180) x 60 = (p/3) rad along the same straight line in the same direction, then the
l 0.72 p resultant amplitude is alternately maximum and minimum at a
N
Path difference Dx = (Df) = = 0.12 m point in the medium. This phenomenon of waxing and waning
2p 2p 3
of sound is called beats.
R

Example 3.
Determine the change in volume of 6 litres of alcohol if
U

the pressure is decreased from 200 cm of Hg to 75 cm.


[velocity of sound in alcohol is 1280 m/s, density of
JO

alcohol = 0.81 gm/cc, density of Hg = 13.6 gm/cc and


g = 9.81 m/s2].
Solution :
U

For propagation of sound in liquid


ED

v= ( B / r) , i.e., B = v2r

DP
But by definition B= –V
DV

DP V( -DP)
so –V = v2r, i.e., DV =
DV rv 2
Here DP = H2rg – H1rg = (75 – 200) ´ 13.6 ´ 981
= –1.667 ´ 106 dyne/cm2

s o DV =
(6 ´ 10 )(1.667 ´ 10 ) = 0.75 cc
3 6 Let two sound waves of frequencies n 1 and n2 are propagating
simultaneously and in same direction. Then at x=0
0.81 ´ (1.280 ´ 10 ) 5 2
y1 = A sin 2p n1t, and y2 = A sin 2p n2t,
For simplicity we take amplitude of both waves to be same.
Example 4. By principle of superposition, the resultant displacement at any
(a) Speed of sound in air is 332 m/s at NTP. What will be instant is
the speed of sound in hydrogen at NTP if the density y = y1 + y2 = 2A cos 2p nAt sin 2p navt
of hydrogen at NTP is (1/16) that of air? n1 - n 2
n + n2
(b) Calculate the ratio of the speed of sound in neon to that where n av = 1 , nA =
2 2
in water vapour at any temperature. [Molecular weight of
Þ y = Abeat sin 2p navt ..................(i)
neon = 2.02 ´ 10–2 kg/mol and for water vapours = 1.8 ´
10–2 kg/mol] It is clear from the above expression (i) that
Waves 377

(i) Abeat = 2A cos 2p nAt, amplitude of resultant wave varies Example 6.


periodically as frequency Two tuning forks A and B sounded together give 6 beats
æ n - n2 ö per second. With an air resonance tube closed at one end,
nA = ç 1 ÷ the two forks give resonance when the two air columns are
è 2 ø
24 cm and 25 cm respectively. Calculate the frequencies of
A is maximum when | cos q |max = 1 | A beat | max = 2A forks.
A is minimum when | cos q |min = 0 | A beat | min = 0 Solution :
(ii) Since lntensity is proportional to amplitude i.e., IaA 2beat Let the frequency of the first fork be f1 and that of second
be f2.
For Imax cos 2p nAt = ± 1 For Imin
We then have,
i.e., 2p nAt = 0,p, 2p 2p nAt = p/2, 3p/2
v v
i.e., t = 0, 1/2nA, 2/2nA t = 1/4nA, 3/4nA....... f1 = and f2 =
4 ´ 24 4 ´ 25
So time interval between two consecutive beat is We also see that
æ 1 ö f1 > f2
Dt = t n - t n -1 = çç ÷
÷
è 2n A ø \ f1 – f2= 6 …(i)
Number of beats per sec is given by f1 24
and = …(ii)

.IN
1 2( n1 - n2 ) f 2 25
nbeat = = 2n A - = n1 - n2
Dt 2 Solving eqns. (i) and (ii), we get
So beat frequency is equal to the difference of frequency of two f1 = 150 Hz and f2 = 144 Hz
AL
interferring waves. DOPPLER EFFECT
To hear beats, the number of beats per second should not be When a source of sound and an observer or both are in motion
N
more than 10. (due to hearing capabilities of human beings) relative to each other there is an apparent change in frequency
Filing/Loading a Tuning Fork of sound as heard by the observer. This phenomenon is called
R

On filing the prongs of tuning fork, raises its frequency and on the Doppler's effect .
U

loading it decreases the frequency. Apparent change in frequency :


(i) When a tuning fork of frequency n produces Dn beats per (a) When source is in motion and observer at rest
JO

second with a standard tuning fork of frequency n0, then (i) when source moving towards observer
n = n 0 ± Dn æ V ö
n1 = n 0 ç ÷
If the beat frequency decreases or reduces to zero or remains è V - VS ø
U

the same on filling the unknown fork, then (ii) when source moving away from observer
ED

n = n 0 - Dn æ V ö
n1¢ = n0 ç ÷
(ii) If the beat frequency decreases or reduces to zero or remains
è V + VS ø
the same on loading the unknown fork with a little wax, Here V = velocity of sound
then n =n 0 + Dn VS = velocity of source
If the beat frequency increases on loading, then
n0 = source frequency.
n = n 0 - Dn
(b) When source is at rest and observer in motion
Example 5.
Tuning fork A has frequency 1% greater than that of (i) when observer moving towards source
standard fork B, while tuning fork C has frequency 2%
æ V + V0 ö
smaller than that of B. When A an C are sounded together, n2 = ç ÷ n0
the number of beats heard per second is 5. What is the è V ø
frequency of each fork? (ii) when observer moving away from source and
Solution : V0 = velocity of observer.
Let the frequencies of forks be n 1, n2 and n3 respectively.
æ V - V0 ö
Then n ¢2 = ç ÷ n0
n1 = n2 (1 + 0.01) = 1.01 n2 and n3 = n2 (1 - 0.02) = 0.98 n2 è V ø
Further, n 1 – n3 = 5 (c) When source and observer both are in motion
Substituting the values, we get (1.01 n2 – 0.98 n2) = 5 (i) If source and observer both move away from each
\ n2 = 166.7 Hz other.
Now n1 = 1.01 × 166.7 = 168.3 Hz æ V - V0 ö
and n3 = 0.98 × 166.7 = 163.3 Hz. n3 = ç ÷ n0
è V - Vs ø
EBD_7179
378 PHYSICS

(ii) If source and observer both move towards each other. Solution :
In this case, we can assume that both the source and
æ V + V0 ö observer are moving towards each other with velocity v. If
n3¢ = ç ÷ n0 ;
è V - Vs ø c be the velocity of signal, then
æc+vö æ (c + v) (c - v) ö
When the wind blows in the direction of sound, then in all above n¢ = ç ÷n or n¢ = ç ÷n
formulae V is replaced by (V + W) where W is the velocity of wind. èc-vø ç (c - v) 2 ÷
è ø
If the wind blows in the opposite direction to sound then V is replaced
by (V – W). (c2 - v2 ) c2
or n ¢ = Þ n¢ = n; (as c >> v)
Keep in Memory c 2 + v2 - 2c v c 2 - 2cv
æ c ö
1. The motion of the listener causes change in number of \ n ¢ = çç ÷÷ n
waves received by the listener and this produces an apparent è c- 2v ø
change in frequency. STATIONARY OR STANDING WAVES
2. The motion of the source of sound causes change in When two progressive waves having the same amplitude, velocity
wavelength of the sound waves, which produces apparent and time period but travelling in opposite directions
change in frequency. superimpose, then stationary wave is produced.
3. If a star goes away from the earth with velocity v, then the Let two waves of same amplitude and frequency travel in opposite
frequency of the light emitted from it changes from n to n'. direction at same speed, then

.IN
n' = n (1–v/c), where c is the velocity of light and y1 = A sin (wt –kx) and
y2 = A sin (wt + kx)
Dn v Dl v By principle of superposition
= or = where Dl is called Doppler’s shift.
AL
n c l c y = y1 + y2 = (2A cos kx) sin wt ...(i)
If wavelength of the observed waves decreases then the y = A s sin ωt
N
object from which the waves are coming is moving towards
It is clear that amplitude of stationary wave As vary with position
the listener and vice versa. (a) As = 0, when cos kx = 0 i.e., kx = p/2, 3p/2............
R

Example 7. i.e., x = l/4, 3l/4...................[as k = 2p/l]


U

Two engines cross each other travelling in opposite These points are called nodes and spacing between two
direction at 72 km/hour. One engine sounds a whistle of nodes is l/2.
JO

frequency 1088 cps. What are the frequencies as heard by (b) As is maximum, when cos kx is max
an observer on the other engine before and after crossing. i.e., kx = 0, p , 2p, 3p i.e., x = 0, ll/2, 2l/2....
Take the speed of sound as 340 m/s. It is clear that antinode (where As is maximum) are also
U

Solution : equally spaced with spacing l/2.


The apparent frequency before crossing, (c) The distance between node and antinode is l/4 (see figure)
ED

Antinode Node Antinode


æ v + v0 ö
n ¢ = n çç ÷ 2A
÷
è v - vs ø
segment 1 segment 2 segment 3
Here Vo = 72 km/hour = 20 m/s x
o
vs = 72 km/hour = 20 m/s
l /2
æ 340 + 20 ö l /4
360
\ n ¢ = 1088 ´ ç ÷ = 1088 ´ = 1224 Hz
è 340 - 20 ø 320 Keep in Memory
The apparent frequency after crossing 1. When a string vibrates in one segment, the sound produced
is called fundamental note. The string is said to vibrate in
æ v - v0 ö æ 340 - 20 ö
n '' = n ç ÷ = 1088 ´ ç ÷ fundamental mode.
è v + v s ø è 340 + 20 ø 2. The fundamental note is called first harmonic, and is given

32 v
= 1088 ´ = 967.11 Hz by n 0 = , where v = speed of wave.
36 2l
Example 8. 3. If the fundamental frequency be n 0 then 2 n 0 , 3 n 0 , 4 n 0
A rocket is going towards moon with a speed v. The ... are respectively called second third, fourth ... harmonics
astronaut in the rocket sends signals of frequency n respectively.
towards the moon and receives them back on reflection 4. If an instrument produces notes of frequencies
from the moon. What will be the frequency of the signal
n1 , n 2 , n 3 , n 4 .... where ν1 < ν 2 < ν3 < ν 4 ....., then n 2 is
received by the astronaut? (Take v << c)
Waves 379

called first overtone, n3 is called second overtone, n 4 is Similarly the frequency of third harmonic or first overtone (IInd
called third overtone ... so on. harmonic absent) is (see fig. b)
5. Harmonics are the integral multiples of the fundamental
3v
frequency. If n0 be the fundamental frequency, then nn 0 is n c2 =
4l
the frequency of nth harmonic.
6. Overtones are the notes of frequency higher than the 5v 7v
Similarly n c3 = , n c4 = ........
fundamental frequency actually produced by the instrument. 4l 4l
7. In the strings all harmonics are produced. End Correction
Stationary Waves in an Organ Pipe : It is observed that the antinode actually occurs a little above the
In the open organ pipe all the harmonics are produced. open end. A correction is applied for this which is known as end
In an open organ pipe, the fundamental frequency or first correction and is denoted by e.
v (i) For closed organ pipe : l is replaced by l+ e where
harmonic is n0 = , where v is velocity of sound and l is the
2l e = 0.3D, D is the diameter of the tube.
length of air column [see fig. (a)] (ii) For open organ pipe: l is replaced by l + 2e where
(a) (b) e = 0.3D
In resonance tube, the velocity of sound in air given by

.IN
l l v = 2ν ( l2 - l1 )

where n = frequency of tuning fork, ll = 1st resonating


AL
length, l2 = 2nd resonating length.
l 2l 2l 2l
l= , l= l= , l= Resonance Tube :
2 1 2 2
N
(c) It is used to determine velocity of sound in air with the help of a
R

tuning fork of known frequency.


l
U

n n
JO

3l 3L e e
l= ,l= A
2 3
Similarly the frequency of second harmonic or first overtone is l1
U

2v
[see fig (b)], n01 =
ED

2l l2
Similarly the frequency of third harmonic and second overtone
3v
is, [(see fig. (c)] n02 =
2l
4v 5v
Similarly n03 = , n04 = ..................
2l 2l
In the closed organ pipe only the odd harmonics are produced. In
a closed organ pipe, the fundamental frequency (or first Let l1 and l2 are lengths of first and second resonances then
harmonic) is (see fig. a)
l 3l
v l1 + e = and l 2 + e =
nc = 4 4
4l

(a) (b) (c) l


Þ l 2 - l1 = Þ l = 2(l 2 - l1)
2
l l l Speed of sound in air is
v = ul where u is the frequency
v = 2 u(l 2 - l1 )
l 3l 5l
l= l= l=
4 4 4
EBD_7179
380 PHYSICS

For vibrating strings/ open organ pipe.


Mode of Harmonic Tone Frequency Wavelength Shape for string
vibration

v
First or First Fundamental n= 2l A
2l N N
Fundamental tone

Second Second First 2n N A A N


2l N
overtone 2

Second 3n 2l
Third Third overtone 3

.IN
For closed organ pipe.

Mode of Harmonic Tone Frequency


AL Wavelength Shape for string
vibration
N

First or v
n=
R

First Fundamental 4l 4l
Fundamental tone
U
JO

Second Third First 3n 4l


U

overtone 3
ED

Second 5n 4l
Third fifth overtone 5

Comparision of Progressive (or travelling) and Stationary (or standing) Wave:

SI. Progressive Wave Stationary Wave


1. The wave advances with a constant speed The wave does not advance but remains confined
in a particular region.
2. The amplitude is the same for all the particles in The amplitude varies according to position, being
the path of the wave zero at the nodes and maximum at the antinodes.
3. All particles within one wavelength have different Phase of all particles between two adjacent nodes is
phases. the same. Particles in adjacent segments of length
p/2 have opposite phase.
4. Energy is transmitted in the direction of propagation Energy is associated with the wave, but there is no
of the wave transfer of energy across any section of the medium.
Waves 381

Comparative Study of Interference, Beats and Stationary Wave:


Interference Beats Stationary wave
Superposition of two waves Superposition of two waves Superposition of two waves
of nearly same amplitude or of same amplitude having of same amplitude having a
same amplitude having a zero phase difference constant phase difference (or
constant phase difference (or moving in the same direction no phase difference) moving
no phase difference) having small difference in in opposite direction having
travelling in the same frequency (less than 10 Hz) same frequency give rise to
direction and with the same show the phenomenon of stationary waves.
wavelength show the beats.
phenomenon of interference.
Maxima and minima are Maxima and minima vary Maxima and minima are
fixed at their locations. periodically with time at fixed at their locations.
every location.

CHARACTERISTICS OF SOUND The time gap between the initial direct note and the reflected note
1. A musical sound consists of quick, regular and periodic upto the minimum audibility level is called reverberation time.
succession of compressions and rarefactions without a Sabine Reverberation Formula for Time

.IN
sudden change in amplitude. Sabine established that the standard period of reverberation viz.,
2. A noise, consists of slow, irregular and a periodic succession the time that the sound takes to fall in intensity by 60 decibels or
to one millionth of its original intensity after it was stopped, is
of compressions and rarefactions, that may have sudden
AL
changes in amplitude. 0.05V
given by T =
3. (i) Pitch, (ii) loudness and (iii) quality are the characteristics å a i Si
N
of musical sound.
4. Pitch depends on frequency, loudness depends on intensity where V = volume of room, å a i Si = a1 S1 + a2 S2 + ....
R

and, quality depends on the number and intensity of S1, S2 .... are different kinds of surfaces of room and
a1 , a2 .... are their respective absorption coefficient.
U

overtones.
The above formula was derived by Prof C. Sabine.
5. The ratio of the frequencies of the two notes is called the
JO

interval between them. For example interval between two Shock waves : The waves produced by a body moving with a
notes of frequencies 512 Hz and 1024 Hz is 1 : 2 (or 1/2). speed greater than the speed of sound are called shock waves.
6. Two notes are said to be in unison if their frequencies are These waves carry huge amount of energy. It is due to the shock
U

wave that we have a sudden violent sound called sonic boom


equal, i.e., if the interval between them is 1 : 1.
when a supersonic plane passes by.
ED

Some other common intervals, found useful in producing


The rate of speed of the source to that of the speed of sound is
musical sound are the following:
called mach number.
Octave (1 : 2), majortone (8 : 9), minortone (9 : 10) and
Intensity of sound : The sound intensities that we can hear range
semitone (15 : 16)
from 10–12 Wm–2 to 103 Wm–2. The intensity level b, measured
7. Major diatonic scale : It consists of eight notes. The
consecutive notes have either of the following three I
in terms of decibel (dB) is defined as b = 10 log
intervals. They are 8 : 9 ; 9 : 10 and 15 : 16. I0
Acoustics : The branch of physics that deals with the process of
where I = measured intensity, I0 = 10–12 Wm–1
generation, reception and propagation of sound is called
At the threshold b = 0
acoustics.
Acoustics may be studied under the following three subtitles. 1
At the max b = 10 log = 120 dB
(a) Electro acoustics. This branch deals with electrical sound 10 -12
production with music. Lissajous Figures: when two simple harmonic waves having
(b) Musical acoustics. This branch deals with the relationship vibrations in mutually perpendicular directions superimpose
of sound with music. on each other, then the resultant motion of the particle is along
(c) Architectural acoustics. This branch deals with the design a closed path, called the Lissajous figures. These figures can be
and construction of buildings. of many shapes depending on
REVERBERATION (i) ratio of frequencies or time periods of two waves
Multiple reflections which are responsible for a series of waves (ii) ratio of amplitude of two waves
falling on listener’s ears, giving the impression of a persistence (iii) phase difference between two waves.
or prolongation of the sound are called reverberations.
EBD_7179
382 PHYSICS

Example 9. Example 11.


The equation of stationary wave in stretched string is The fundamental frequency of an open organ pipe is 300
px Hz. The first overtone of this pipe has same frequency as
given by y = 5 sin cos 40 p t where x and y are in cm first overtone of a closed organ pipe. If speed of sound is
3
330 m/s, then find the length of closed organ pipe.
and t in second. Find the separation between two adjacent
nodes. Solution :
Solution : v
For open pipe, n =
2l
Given that y = 5 sin p x cos 40 p t
3
v 330 11
2p 2p \ l= = =
We know that y = a sin x cos vt 2 n 2 ´ 300 20
l l
As frequency of first overtone of open pipe = frequency of
2p p 1st overtone of closed pipe
\ = or l = 6
l 3
v v
Now distance between two adjacent nodes \ 2 =3
2l 4 l¢
l 6
= = = 3 cm

.IN
2 2 3l 3 11
l¢ = = ´ = 41.25 cm
Example 10. 4 4 20
For a certain organ pipe, three successive resonance Example 12.
AL
frequencies are observed at 425, 595 and 765 Hz, String wires of same material of length l and 2 l vibrate
respectively. Taking the speed of sound in the air to be 340 with frequencies 100 and 150 respectively. Determine the
metre per second, (i) explain whether the pipe is closed at
N
ratio of their tensions.
one end or open at both the ends and (ii) determine the Solution :
R

fundamental frequency and the length of the pipe.


Solution : 1 T
U

(i) Given ratio of successive frequencies As n =


2l m
= 425 : 595 : 765 = 5 : 7 : 9.
JO

So, the harmonics are odd harmonics. Hence the pipe


1 T1 1 T2
is closed at one end. \ 100 = and 150 =
2l m 4l m
U

(ii) Fundamental frequency n is given by, n = 425 = 85


5
ED

100 4 l T1 2 T
Divide = or =2 1
v v 150 2 l T2 3 T2
We known that n = or l = for closed organ
4l 4n
pipe at one end. T1 1 T1 1
\ = or = =1:9
T2 3 T2 9
340
\ Length of the pipe l = = 1 metre
4 ´ 85
Waves

ED
U
JO
U
R
N
AL
.IN
383
EBD_7179
384 PHYSICS

1. Frequencies of sound produced from an organ pipe open 12. What is the effect of increase in temperature on the
at both ends are frequency of sound produced by an organ pipe?
(a) only fundamental note (b) only even harmonics (a) increases (b) decreases
(c) only odd harmonics (d) even and odd harmonics (c) no effect (d) erratic change
2. The fundamental frequency of an organ pipe is 512 Hz. If its 13. Shock waves are produced by objects
length is increased, then frequency will (a) carrying electric charge and vibrating
(a) decrease (b) increase (b) vibrating with frequency greater than 20000 Hz
(c) remains same (d) cannot be predicted
(c) vibrating with very large amplitude
3. The property of a medium necessary for wave propagation is
(a) inertia (b) elasticity (d) moving with a speed greater than that of sound in the
(c) low resistance (d) All of the above medium
4. Doppler’s effect is not applicable for 14. The speed of sound in a medium depends on
(a) audio waves (b) electromagnetic waves (a) the elastic property but not on the inertia property
(c) shock waves (d) None of these (b) the inertia property but not on the elastic property
5. Two sound waves of equal intensity I produce beats. The

.IN
(c) the elastic property as well as the inertia property
maximum intensity of sound produced in beats will be
(d) neither the elastic property nor the inertia property
(a) I (b) 2I (c) 3I (d) 4I
15. Consider the three waves z 1, z2 and z3 as
6. Two sinusoidal plane waves of same frequency having
AL z1 = A sin (kx – wt)
intensities I0 and 4 I0 are travelling in the same direction. z2 = A sin (kx + wt)
The resultant intensity at a point at which waves meet with z3 = A sin (ky – wt)
N
a phase difference of zero radian is Which of the following represents a standing wave?
(a) I0 (b) 5 I0 (a) z1 + z2 (b) z2 + z3
R

(c) 9 I0 (d) 3 I0 (c) z3 + z1 (d) z1 + z2 + z3


16. Each of the properties of sound listed in column A primarily
U

7. If the intensities of two interfering waves be I1 and I2, the


depends on one of the quantitites in column B. Choose the
contrast between maximum and minimum intensity is
JO

matching pairs from two columns


maximum, when Column A Column B
(a) I1 > > I2 (b) I1 << I2 Pitch Waveform
(c) I1 = I2 (d) either I1 or I2 is zero
U

Quality Frequency
8. Sound waves of length l travelling with velocity v in a Loudness Intensity
ED

medium enter into another medium in which their velocity (a) Pitch-wave form; Quality-frequency; Loudness-
is 4 v. The wavelength in 2nd medium is intensity
(a) 4λ (b) l (b) Pitch-frequency; Quality-wave form; Loudness-
(c) l/4 (d) 16 l intensity
(c) Pitch-intensity; Quality-wave form; Loudness-
9. Two periodic waves of intensities I1 and I2 pass through a
frequency
region at the same time in the same direction. The sum of (d) Pitch-wave form; Quality-intensity; Loudness-
the maximum and minimum intensities is frequency
(a) 2 (I1 + I2) (b) I1 + I2 17. Three transverse waves are represented by
(c) ( I1 + I 2 ) 2 (d) ( I1 - I 2 ) 2 y1 = A cos ( kx - w t )
y 2 = A cos ( kx + w t )
10. An open and closed organ pipe have the same length. The
y 3 = A cos (ky - wt )
ratio of pth mode of frequency of vibration of two pipes is
(a) 1 (b) p The combination of waves which can produce stationary
waves is
2p (a) y1 and y2 (b) y2 and y3
(c) p (2p + 1) (d)
( 2 p - 1) (c) y1 and y3 (d) y1, y2 and y3
11. Sound waves are not transmitted to long distances 18. Which of the following changes at an antinode in a
because, stationary wave?
(a) they are absorbed by the atmosphere (a) Density only
(b) they have constant frequency (b) Pressure only
(c) the height of antenna required, should be very high (c) Both pressure and density
(d) velocity of sound waves is very less (d) Neither pressure nor density
Waves 385

19. Sound waves are travelling in a medium whose adiabatic 2p


elasticity is E and isothermal elasticity E'. The velocity of 23. The equation y = a sin (vt - x ) is expression for
l
sound waves is proportional to
(a) stationary wave of single frequency along x-axis
(a) E' (b) E (b) a simple harmonic motion
(c) a progressive wave of single frequency along x-axis
E
(c) E' (d) (d) the resultant of two SHMs of slightly different
E' frequencies
20. The equation of a cylindrical progressive wave is 24. A whistle S of frequency f revolves in a circle of radius R at
a constant speed v. What is the ratio of largest and smallest
(a) y = a sin wt (b) y = a sin (wt - kr )
frequency detected by a detector D at rest at a distance 2R
a from the centre of circle as shown in figure ?
a
(c) y= sin (wt - kr) (d) y = sin (w t - kr ) (take c as speed of sound)
r r
21. A wave y = a sin (wt – kx) on a string meets with another
wave producing a node at x = 0. Then the equation of the D S
R
unknown wave is
(a) y = a sin (wt + kx) (b) y = –a sin (wt + kx)
(c) y = a sin (wt – kx) (d) y = –a sin (wt – kx) 2R
æ c + vö æ c + vö

.IN
22. A sound wave of frequency f travels horizontally to the (a) çè ÷ 2ç (b)
right. It is reflected from a large vertical plane surface moving c - vø è c - v ÷ø
to left with a speed v. The speed of sound in medium is c. (c + v)
(c) (d)
Then
AL 2 c 2
(a) the number of wave striking the surface per second is 25. When a string is divided into three segments of length l1,
l2, and l3 the fundamental frequencies of these three
(c + v)
N
f segments are v1 , v 2 and v3 respectively. The original
c fundamental frequency (v) of the string is
R

c(c + v)
(b) the wavelength of reflected wave is (a) v = v1 + v2 + v3
f (c + v)
U

(c - v) (b) v = v1 + v2 + v3
(c) the frequency of the reflected wave is f
JO

(c - v) 1 1 1 1
(c) = + +
v v1 v2 v3
(d) the number of beats heard by a stationary listener to
U

vf 1 1 1 1
the left of the reflecting surface is (d) = + +
c-v v v1 v2 v3
ED

1. The equation of a plane progressive wave is 2. If two waves of same frequency and same amplitude, on
é xù superposition, produce a resultant disturbance of the same
y = 0 .9 sin 4 p ê t - ú . When it is reflected at a rigid
ë 2û amplitude, the wave differ in phase by
2 (a) p (b) 2 p/3
support, its amplitude becomes of its previous value.
3 (c) Zero (d) p/3
The equation of the reflected wave is
3. Two tones of frequencies n 1 and n 2 are sounded together.
é xù
(a) y = 0 .6 sin 4 p ê t + ú The beats can be heard distinctly when
ë 2û
(a) 10 < (n1 – n 2) < 20 (b) 5 < (n1 – n 2) > 20
é xù (c) 5 < (n1 – n 2) < 20 (d) 0 < (n1 – n 2) < 10
(b) y = - 0.6 sin 4 p ê t + ú
ë 2û 4. The equation of a spherical progressive wave is
é xù (a) y = a sin wt (b) y = a sin (wt - kr )
(c) y = - 0.9 sin 8p ê t - ú
ë 2û
a a
(c) y= sin (wt - kr ) (d) y = sin (w t - kr )
é xù 2 r
(d) y = - 0.6 sin 4 p ê t + ú
ë 2û
EBD_7179
386 PHYSICS

5. The velocity of sound in air is 330 m/s. The r.m.s. velocity (a) 117.7 cm (b) 92.9 cm
of air molecules (g = 1.4) is approximately equal to (c) 115.5 cm (d) 113.5 cm
(a) 400 m/s (b) 471.4 m/s 17. A fork of frequency 256 Hz resonates with a closed organ
(c) 231 m/s (d) 462 m/s pipe of length 25.4 cm. If the length of pipe be increased by
6. The velocity of sound in hydrogen is 1224 m/s. Its velocity 2 mm, the number of beats/sec. will be
in a mixture of hydrogen and oxygen containing 4 parts by (a) 4 (b) 1
volume of hydrogen and 1 part oxygen is (c) 2 (d) 3
(a) 1224 m/s (b) 612 m/s 18. A distant star which is moving away with a velocity of 106
(c) 2448 m/s (d) 306 m/s m/sec is emitting a red line of frequency 4.5 × 1014 Hz. The
7. A person standing symmetrically between two cliffs claps observed frequency of this spectral line is
his hands and starts hearing a series of echoes at intervals (a) 4.5 × 108 Hz (b) 4.485 × 1014 Hz
of 1 sec. If speed of sound in air is 340 m/s, the distance 14
(c) 4.515 × 10 Hz (d) 4.5 × 1014 Hz
between the parallel cliffs must be 19. The speed of sound in air under ordinary conditions is
(a) 340 m (b) 680 m around 330 m s–1. The speed of sound in hydrogen under
(c) 1020 m (d) 170 m similar conditions will be (in m s–1) nearest to
8. An echo repeats two syllables. If the velocity of sound is (a) 330 (b) 1200
330 m/s, then the distance of the reflecting surface is (c) 600 (d) 900
(a) 66.0 m (b) 33.0 m 20. A wave disturbance in a medium is described by
(c) 99.0 m (d) 16.5 m

.IN
9. At room temperaturre, velocity of sound in air at 10 æ pö
y( x , t ) = 0.02 cos ç 50pt + ÷ cos(10px ) where x and y are
atmospheric pressure and at 1 atmospheric pressure will be è 2ø
in the ratio
in metre and t is in second. Which of the following is correct?
(a) 10 : 1 (b) 1 : 10
AL
(a) A node occurs at x = 0.15 m
(c) 1 : 1 (d) cannot say
10. A series of ocean waves, each 5.0 m from crest to crest, (b) An antinode occurs at x = 0.3 m
(c) The speed wave is 5 ms–1
N
moving past the observer at a rate of 2 waves per second,
what is the velocity of ocean waves? (d) The wavelength is 0.3 m
R

(a) 2.5 m/s (b) 5.0 m/s 21. Two waves represented by y1 = a sin wt and
(c) 8.0 m/s (d) 10.0 m/s
U

11. If the ratio of maximum to minimum intensity in beats is 49, p


y 2 = a sin(wt + f) with f = are superposed at any point
JO

then the ratio of amplitudes of two progressive wave trains 2


is at a particular instant. The resultant amplitude is
(a) 7 : 1 (b) 4 : 3 (a) a (b) 4a
(c) 49 : 1 (d) 16 : 9
U

(c) 2a (d) zero


12. There are three sources of sound of equal intensities and
ED

frequencies 400, 401 and 402 vibrations per second. The 22. If the speed of a transverse wave on a stretched string of
number of beats/sec is length 1 m is 60 m/s, what is the fundamental frequency of
(a) 0 (b) 1 vibration?
(c) 3 (d) 2 (a) 10 Hz (b) 30 Hz
13. A fork of unknown frequency gives four beats/sec when (c) 40 Hz (d) 70 Hz
sounded with another of frequency 256. The fork is now 23. The equation of a progressive wave is
loaded with a piece of wax and again four beats/sec are
heard. Then the frequency of the unknown fork is é t x ù
y = 0.02 sin 2pê - ú
(a) 256 Hz (b) 252 Hz ë 0.01 0.30 û
(c) 264 Hz (d) 260 Hz Here x and y are in metre and t is in second. The velocity of
14. If there are six loops for 1 m length in transverse mode of propagation of the wave is
Melde’s experiment., the no. of loops in longitudinal mode (a) 300 m s–1 (b) 30 m s–1
under otherwise identical conditions would be (c) 400 m s–1 (d) 40 m s–1
(a) 3 (b) 6 24. The equation Y = 0.02 sin (500pt) cos (4.5 x) represents
(c) 12 (d) 8 (a) progressive wave of frequency 250 Hz along x-axis
15. The apparent wavelength of the light from a star moving (b) a stationary wave of wavelength 1.4 m
away from the earth is 0.2% more than its actual wavelength. (c) a transverse progressive wave of amplitude 0.02 m
Then the velocity of the star is (d) progressive wave of speed of about 350 m s–1
(a) 6 × 107 m/sec (b) 6 × 106 m/sec 25. The intensity level of sound wave is said to be 4 decibel. If
5
(c) 6 × 10 m/sec (d) 6 × 104 m/sec the intensity of wave is doubled, then the intensity level of
16. In a resonance column, first and second resonance are sound as expressed in decibel would be
obtained at depths 22.7 cm and 70.2 cm. The third resonance (a) 8 (b) 16
will be obtained at a depth (c) 7 (d) 14
Waves 387

semi circular track. The driver blows a whistle of frequency


26. The equation y = A sin 2 (kx - wt ) represents a wave with
200 Hz. Velocity of sound is 340 m/sec Then the apparent
(a) amplitude A, frequency w / 2 p frequency as observed by a passenger in the middle of the
(b) amplitude A/2, frequency w / p train, when the speed of the train is 30 m/sec is
(a) 181 Hz (b) 200 Hz
(c) amplitude 2A, frequency w / 4p
(c) 188 Hz (d) 210 Hz
(d) it does not represent a wave motion 38. The displacement of a particle varies according to the
27. A toothed wheel is rotating at 240 rpm and a post card is
relation x = 4(cos πt + sin πt). The amplitude of the
held against the teeth. If the pitch of the tone is 256 Hz,
particle is
then the number of teeth on the rotating wheel is
(a) – 4 m (b) 4 m
(a) 256 (b) 128
(c) 64 (d) 32 (c) 4 2 m (d) 8 m
28. When a tuning fork vibrates with 1.0 m or 1.05 m long wire 39. Two waves of wavelengths 99 cm and 100 cm both travelling
of a sonometer, 5 beats per second are produced in each with velocity 396 m/s are made to interfere. The number of
case. What will be the frequency of the tuning fork? beats produced by them per second is
(a) 195 (b) 200 (a) 1 (b) 2
(c) 205 (d) 210 (c) 4 (d) 8
29. The speed of sound in oxygen (O2) at a certain temperature 40. A whistle of frequency 1000 Hz is sounded on a car travelling
is 460 ms–1. The speed of sound in helium (He) at the same towards a cliff with velocity of 18 m s–1 normal to the cliff.
temperature will be (assume both gases to be ideal) If c = 330 m s–1, then the apparent frequency of the echo as
(a) 460 ms–1 (b) 500 ms–1

.IN
heard by the car driver is nearly
(c) 650 ms–1 (d) 330 ms–1 (a) 1115 Hz (b) 115 Hz
30. A closed pipe and an open pipe have their first overtones (c) 67 Hz (d) 47.2 Hz
identical in frequency. Their lengths are in the ratio 41. An organ pipe open at one end is vibrating in first overtone
(a) 1 : 2 (b) 2 : 3
AL and is in resonance with another pipe open at both ends
(c) 3 : 4 (d) 4 : 5 and vibrating in third harmonic. The ratio of length of two
31. The threshold of hearing for the human ear is 10–12 W m–2. pipes is
N
This is taken as the standard level. The intensity of sound (a) 1 : 2 (b) 4 : 1
is 1 Wm–2. It has intensity (in dB). (c) 8 : 3 (d) 3 : 8
R

(a) 1012 dB (b) 12 dB 42. An organ pipe, open from both end produces 5 beats per
(c) 240 dB (d) 120 dB
U

second when vibrated with a source of frequency 200 Hz.


32. In expressing sound intensity, we take 10–12 W m–2 as the The second harmonic of the same pipes produces 10 beats
JO

reference level. For ordinary conversation, the intensity per second with a source of frequency 420 Hz. The
level is about 10–6 W m–2. Expressed in decibel, this is fundamental frequency of organ pipe is
(a) 106 (b) 6 (a) 195 Hz (b) 205 Hz
(c) 60 (d) loge (106) (c) 190 Hz (d) 210 Hz
U

33. If two tuning forks A and B are sounded together, they 43. A progressive sound wave of frequency 500 Hz is travelling
through air with a speed of 350 ms–1 . A compression
ED

produce 4 beats per sec. A is then slightly loaded with wax


and same no. of beats/sec. are produced again. If frequency maximum appears at a place at a given instant. The minimum
of A is 256, the frequency of B would be time interval after which the rarefraction maximum occurs at
(a) 250 (b) 262 the same point, is
(c) 252 (d) 260 1
34. A uniform wire of length 20 m and weighing 5 kg hangs (a) 200 s (b) s
250
vertically. If g = 10 m/s2, then the speed of transverse waves
in the middle of the wire is 1 1
(c) s (d) s
(a) 10 m/s (b) 10 2 m/s 500 1000
44. When a sound wave of frequency 300 Hz passes through a
(c) 4 m/s (d) zero
medium, the maximum displacement of a particle of the
35. The fundamental frequency of an open organ pipe is 300
medium is 0.1 cm. The maximum velocity of the particle is
Hz. The first overtone of this pipe has same frequency as
equal to
first overtone of a closed organ pipe. If speed of sound is
(a) 60p cms–1 (b) 30p cms–1
330 m/s, then the length of closed organ pipe is
(a) 41 cm (b) 37 cm (c) 30 cms–1 30 cm s -1 (d) 60 cms–1
(c) 31 cm (d) 80 cm v
36. A pipe closed at one end produces a fundamental note of 45. An engine running at speed sounds a whistle of
10
412 Hz. It is cut into two pieces of equal length. the frequency 600 Hz. A passenger in a train coming from the
fundamental frequencies produced by the two pieces are
v
(a) 206 Hz, 412 Hz (b) 824 Hz, 1648 Hz opposite side at speed experiences this whistle to be of
(c) 412 Hz, 824 Hz (d) 206 Hz, 824 Hz 15
37. A train has just completed U-curve in a track which is a semi- frequency f. If v is speed of sound in air and there is no
circle. The engine is at the forward end of the semi-circular wind, f is near to
part of the track while the last carriage is at the rear end of the (a) 710 Hz (b) 630 Hz
(c) 580 Hz (d) 510 Hz
EBD_7179
388 PHYSICS

46. A source of sound produces waves of wavelength 60 cm (a) 7.5 units (b) 1.5 units
when it is stationary. If the speed of sound in air is 320 m s– (c) 22.5 units (d) 30 units
1 and source moves with speed 20 m s–1, the wavelength of 54. A wave travelling along the x-axis is described by the
sound in the forward direction will be nearest to equation y(x, t) = 0.005 cos (a x – bt). If the wavelength and
(a) 56 cm (b) 60 cm the time period of the wave are 0.08 m and 2.0s, respectively,
(c) 64 cm (d) 68 cm then a and b in appropriate units are
47. A person carrying a whistle emitting continuously a note 0.08 2.0
of 272 Hz is running towards a reflecting surface with a (a) a = 25.00 p , b = p (b) a = ,b =
speed of 18 km h–1. The speed of sound in air is 345 m s–1. p p
The number of beats heard by him is 0.04 1.0 p
(c) a = ,b = (d) a = 12.50p, b =
(a) 4 (b) 6 p p 2.0
(c) 8 (d) zero 55. Where should the two bridges be set in a 110cm long wire
48. In the sonometer experiment, a tuning fork of frequency so that it is divided into three parts and the ratio of the
256 Hz is in resonance with 0.4 m length of the wire when frequencies are 3 : 2 : 1 ?
the iron load attached to free end of wire is 2 kg. If the load (a) 20cm from one end and 60cm from other end
is immersed in water, the length of the wire in resonance (b) 30cm from one end and 70cm from other end
would be (specific gravity of iron = 8) (c) 10cm from one end and 50cm from other end
(a) 0.37 m (b) 0.43 m (d) 50cm from one end and 40cm from other end
(c) 0.31 m (d) 0.2 m 56. A source X of unknown frequency produces 8 beats per
49. A longitudinal wave is represented by second with a source of 250 Hz and 12 beats per second
æ xö with a source of 270 Hz. The frequency of the source X is

.IN
x = x 0 sin 2pç nt - ÷ (a) 242 Hz (b) 258 Hz
è lø
(c) 282 Hz (d) 262 Hz
The maximum particle velocity will be four times the wave 57. An isotropic point source S of sound emits constant power.
velocity if
AL Two points A and B separated by a distance r are situated
px 0
(a) l = (b) l = 2px 0 near the source as shown in figure. The difference of the
4 intensity level of sound at the points A and B is about
px
N
(c) l = 0 (d) l = 4px 0 r r
2
R

50. Two strings A and B, made of same material, are stretched


S A B
by same tension. The radius of string A is double of radius
(a) 3 dB (b) 2 dB
U

of B. A transverse wave travels on A with speed vA and on


B with speed vB. The ratio vA / vB is (c) 6 dB (d) 12 dB
JO

(a) 1/2 (b) 2 58. A thick uniform rope of length L is hanging from a rigid
(c) 1/4 (d) 4 support. A transverse wave of wavelength l0 is set up at
51. A sonometer wire of length l vibrates in fundamental mode the middle of rope as shown in figure. The wavelength of
the wave as it reaches to the topmost point is
U

when excited by a tuning fork of frequency 416 Hz. If the


length is doubled keeping other things same, the string will /////////////////
ED

(a) vibrate with a frequency of 416 Hz


L
(b) vibrate with a frequency of 208 Hz
(c) vibrate with a frequency of 832 Hz
(d) stop vibrating
l0
100 (a) 2l0 (b) 2l 0 (c) (d) l0
52. A source of sound is travelling at m s–1 along a road, 2
3 59. Wave pulse on a string shown in figure is moving to the
towards a point A. When the source is 3 m away from A, a
person standing at a point O on a road perpendicular to the right without changing shape. Consider two particles at
track hears a sound of frequency n'. The distance of O from positions x1 = 1.5m and x2 = 2.5m. Their transverse velocities
A at that time is 4 m. If the original frequency is 640 Hz, then at the moment shown in figure are along directions
y
the value of n' is (given : velocity of sound = 340 m s–1)
100 ms –1
(a) 620 Hz A 3m 3 S
q
(b) 680 Hz v
4m
(c) 720 Hz
x(m)
(d) 840 Hz 1 2 3 4 5 6
O
53. The equation of a stationary wave is : (a) positive y-axis and positive y-axis respectively
æ px ö (b) negative y-axis and positive y-axis respectively
y = 4 sinç ÷ cos (96pt ) . The distance between a node
è 15 ø (c) positive y-axis and negative y-axis respectively
and its next antinode is (d) negative y-axis and negative y-axis respectively
Waves 389

60. The transverse wave represented by the equation (a) decreases by a factor 10
(b) increases by a factor 20
æpö
y = 4 sin ç ÷ sin (3x - 15 t ) has (c) increases by a factor 10
è6ø (d) decreases by a factor 20
(a) amplitude = 4 69. Two identical piano wires kept under the same tension T
have a fundamental frequency of 600 Hz. The fractional
p
(b) wavelength = 4 increase in the tension of one of the wires which will lead to
3 occurrence of 6 beats/s when both the wires oscillate
(c) speed of propagation = 5 together would be
p (a) 0.02 (b) 0.03 (c) 0.04 (d) 0.01
(d) period = 70. Velocity of sound waves in air is 330 m/s. For a particular
15
sound wave in air, a path difference of 40 cm is equivalent
61. An organ pipe P1 closed at one end vibrating in its first to phase difference of 1.6p. The frequency of this wave is
overtone and another pipe P2 open at both ends vibrating
(a) 165 Hz (b) 150 Hz
in third overtone are in resonance with a given tuning fork.
(c) 660 Hz (d) 330 Hz
The ratio of the length of P1 to that of P2 is
71. Two sources of sound placed close to each other are
(a) 8/3 (b) 3/8
emitting progressive waves given by y1 = 4 sin 600 pt and
(c) 1/2 (d) 1/3
y2 = 5 sin 608 pt. An observer located near these two sources
62. A car is moving towards a high cliff. The car driver sounds of sound will hear

.IN
a horn of frequency f. The reflected sound heard by the (a) 4 beats per second with intensity ratio 25 : 16 between
driver has as frequency 2f. If v be the velocity of sound, waxing and waning.
then the velocity of the car, in the same velocity units, will
(b) 8 beats per second with intensity ratio 25 : 16 between
be
AL waxing and waning
(a) v /2 (b) v /Ö2
(c) 8 beats per second with intensity ratio 81 : 1 between
(c) v /3 (d) v /4
waxing and waning
N
63. The phase difference between two waves, represented by
(d) 4 beats per second with intensity ratio 81 : 1 between
y1 = 10–6 sin{100 t + (x/50) + 0.5} m waxing and waning
R

y2 = 10–6 cos{100 t + (x/50)} m Directions for Qs. (72 to 75) : Each question contains
where x is expressed in metres and t is expressed in
U

STATEMENT-1 and STATEMENT-2. Choose the correct answer


seconds, is approximately (ONLY ONE option is correct ) from the following-
JO

(a) 1.5 radians (b) 1.07 radians (a) Statement -1 is false, Statement-2 is true
(c) 2.07 radians (d) 0.5 radians (b) Statement -1 is true, Statement-2 is true; Statement -2 is a
64. Two vibrating tuning forks produce progressive waves correct explanation for Statement-1
U

given by Y1 = 4 sin 500 pt and Y2 = 2 sin 506 pt. Number of (c) Statement -1 is true, Statement-2 is true; Statement -2 is not
beats produced per minute is a correct explanation for Statement-1
ED

(a) 360 (b) 180 (d) Statement -1 is true, Statement-2 is false


(c) 60 (d) 3 72. Statement 1 : All points on a wavefront vibrate in same
65. A transverse wave is represented by y = A sin ( w t – kx). phase with same frequency.
For what value of the wavelength is the wave velocity equal Statement 2 : Two sources are said to be coherent if they
to the maximum particle velocity? produce waves of same frequency with a constant phase
pA difference.
(a) (b) p A (c) 2pA (d) A 73. Statement 1 : A transverse waves are produced in a very
2
66. A tuning fork of freqqency 512 Hz makes 4 beats per second long string fixed at one end. Only progressive wave is
with the vibrating string of a piano. The beat frequency observed near the free end.
decreases to 2 beats per sec when the tension in the piano Statement 2 : Energy of reflected wave does not reach the
string is slightly increased. The frequency of the piano free end.
string before increasing the tension was 74. Statement 1 : Two waves moving in a uniform string having
(a) 510 Hz (b) 514 Hz (c) 516 Hz (d) 508 Hz uniform tension cannot have different velocities.
67. Two waves are represen ted by the equations Statement 2 : Elastic and inertial properties of string are
y1 = a sin (wt + kx + 0.57) m and y2 = a cos (wt + kx) m, where same for all waves in same string. Moreover speed of wave
x is in meter and t in sec. The phase difference between in a string depends on its elastic and inertial properties only.
them is
(a) 1.0 radian (b) 1.25 radian 75. Statement 1 : Doppler formula for sound wave is symmetric
(c) 1.57 radian (d) 0.57 radian with respect to the speed of source and speed of observer.
68. Sound waves travel at 350 m/s through a warm air and at Statement 2 : Motion of source with respect to stationary
3500 m/s through brass. The wavelength of a 700 Hz observer is not equivalent to the motion of an observer
acoustic wave as it enters brass from warm air with respect to stationary source.
EBD_7179
390 PHYSICS

Exemplar Questions æ xö
1. Water waves produced by a motorboat sailing in water are (c) y = 0.4sin 2p ç t + ÷
è 2ø
(a) neither longitudinal nor transverse
æ xö
(b) both longitudinal and transverse (d) y = -0.4 sin 2p ç t - ÷
(c) only longitudinal è 2ø
(d) only transverse 9. A string of mass 2.5 kg is under tension of 200 N. The
2. Sound waves of wavelength l travelling in a medium with a length of the stretched string is 20.0 m. If the transverse
speed of v m/ s enter into another medium where its speed jerk is struck at one end of the string, the disturbance will
in 2v m/s. Wavelength of sound waves in the second reach the other end in 200 N = T
medium is
l (a) 1 s
(a) l (b) (b) 0.5 s
2
(c) 2l (d) 4 l (c) 2 s 20 m
3. Speed of sound wave in air (d) data given is insufficient

.IN
(a) is independent of temperature 10. A train whistling at constant frequency is moving towards
(b) increases with pressure a station at a constant speed v. The train goes past a
(c) increases with increase in humidity stationary observer on the station. The frequency n' of the
(d) decreases with increase in humidity
AL sound as heard by the observer is plotted as a function of
4. Change in temperature of the medium changes time t (figure). Identify the expected curve.
(a) frequency of sound waves n n
N
(b) amplitude of sound waves
(c) wavelength of sound waves
R

(d) loudness of sound waves


U

5. With propagation of longitudinal waves through a medium, (a) t (b) t


the quantity transmitted is
n n
JO

(a) matter
(b) energy
(c) energy and matter
U

(d) energy, matter and momentum


(c) t (d) t
6. Which of the following statements are true for wave motion?
ED

(a) Mechanical transverse waves can propagate through Past Years (2013-2017) NEET/AIPMT Questions
all mediums
11. If we study the vibration of a pipe open at both ends, then
(b) Longitudinal waves can propagate through solids only
which of the following statements is not true ? [2013]
(c) Mechanical transverse waves can propagate through
solids only (a) Odd harmonics of the fundamental frequency will be
(d) Longitudinal waves can propagate through vacuum generated
7. A sound wave is passing through air column in the form of (b) All harmonics of the fundamental frequency will be
compression and rarefaction. In consecutive compressions generated
and rarefactions, (c) Pressure change will be maximum at both ends
(a) density remains constant (d) Antinode will be at open end
(b) Boyle's law is obeyed 12. A source of unknown frequency gives 4 beats/s, when
(c) bulk modulus of air oscillates sounded with a source of known frequency 250 Hz. The
(d) there is no transfer of heat second harmonic of the source of unknown frequency gives
8. Equation of a plane progressive wave is given by five beats per second, when sounded with a source of
æ xö frequency 513 Hz. The unknown frequency is [2013]
y = 0.6sin 2p ç t - ÷ . On reflection from a denser medium
è 2ø (a) 246 Hz (b) 240 Hz
2 (c) 260 Hz (d) 254 Hz
its amplitude becomes of the amplitude of the incident
3 13. A wave travelling in the +ve x-direction having displacement
wave. The equation of the reflected wave is along y-direction as 1m, wavelength 2p m and frequency
æ xö 1
(a) y = 0.6sin 2p ç t + ÷ Hz is represented by [2013]
è 2ø p
æ xö (a) y = sin (2px – 2pt) (b) y = sin (10px – 20pt)
(b) y = -0.4sin 2p ç t + ÷ (c) y = sin (2px + 2pt) (d) y = sin (x – 2t)
è 2ø
Waves 391

14. The length of the wire between two ends of a sonometer is (a) 103 Hz
100 cm. What should be the positions of two bridges below
the wire so that the three segments of the wire have their (b) 106 Hz
fundamental frequencies in the ratio of 1 : 3 : 5?
1500 2000 (c) 97 Hz
(a) cm, cm [NEET Kar. 2013] 60°
23 23 O
(d) 100 Hz S
1500 500
(b) cm, cm 21. A string is stretched between two fixed points separated
23 23 by 75.0 cm. It is observed to have resonant frequencies of
1500 300 420 Hz and 315 Hz. There are no other resonant frequencies
(c) cm, cm
23 23 between these two. The lowest resonant frequency for this
string is : [2015 RS]
300 1500
(d) cm, cm (a) 205 Hz (b) 10.5 Hz
23 23
15. Two sources P and Q produce notes of frequency 660 Hz (c) 105 Hz (d) 155 Hz
each. A listener moves from P to Q with a speed of 1 ms–1. 22. A siren emitting a sound of frequency 800 Hz moves away
If the speed of sound is 330 m/s, then the number of beats from an observer towards a cliff at a speed of 15ms–1. Then,
heard by the listener per second will be [NEET Kar. 2013] the frequency of sound that the observer hears in the echo
(a) zero (b) 4 reflected from the cliff is : [2016]

.IN
(c) 8 (d) 2 (Take velocity of sound in air = 330 ms–1)
16. If n1, n2 and n3 are the fundamental frequencies of three
(a) 765 Hz (b) 800 Hz
segments into which a string is divided, then the original
(c) 838 Hz (b) 885 Hz
fundamental frequency n of the string is given by : [2014]
AL
23. A uniform rope of length L and mass m 1 hangs vertically
1 1 1 1 from a rigid support. A block of mass m2 is attached to the
(a) = + +
N
n n1 n 2 n 3 free end of the rope. A transverse pulse of wavelength l1 is
1 1 1 1 produced at the lower end of the rope. The wavelength of
R

(b) = + + the pulse when it reaches the top of the rope is l2 the ratio
n n1 n2 n3
l2/l1 is
U

[2016]
(c) n = n1 + n 2 + n 3
JO

m1 m1 + m 2
(d) n = n1 + n2 + n3 (a) m2 (b) m2
17. The number of possible natural oscillation of air column in
U

a pipe closed at one end of length 85 cm whose frequencies


lie below 1250 Hz are : (velocity of sound = 340 ms–1) m2 m1 + m 2
(c) (d)
ED

(a) 4 (b) 5 [2014] m1 m1


(c) 7 (d) 6 24. An air column, closed at one end and open at the other,
18. A speeding motorcyclist sees trafic jam ahead of him. He resonates with a tuning fork when the smallest length of
slows down to 36 km/hour. He finds that traffic has eased
the column is 50 cm. The next larger length of the column
and a car moving ahead of him at 18 km/hour is honking at
resonating with the same tuning fork is : [2016]
a frequency of 1392 Hz. If the speeds of sound is 343 m/s,
the frequency of the honk as heard by him will be : (a) 66.7 cm (b) 100 cm
(a) 1332 Hz (b) 1372 Hz [2014] (c) 150 cm (d) 200 cm
(c) 1412 Hz (d) 1464 Hz 25. The two nearest harmonics of a tube closed at one end and
19. The fundamental frequency of a closed organ pipe of length open at other end are 220 Hz and 260 Hz. What is the
20 cm is equal to the second overtone of an organ pipe fundamental frequency of the system? [2017]
open at both the ends. The length of organ pipe open at (a) 20 Hz (b) 30 Hz
both the ends is [2015] (c) 40 Hz (d) 10 Hz
(a) 100 cm (b) 120 cm 26. Two cars moving in opposite directions approach each other
(c) 140 cm (d) 80 cm with speed of 22 m/s and 16.5 m/s respectively. The driver
20. A source of sound S emitting waves of frequency 100 Hz of the first car blows a horn having a frequency 400 Hz. The
and an observor O are located at some distance from each frequency heard by the driver of the second car is [velocity
other. The source is moving with a speed of 19.4 ms–1 at an of sound 340 m/s] :- [2017]
angle of 60° with the source observer line as shown in the
(a) 361 Hz (b) 411 Hz
figure. The observor is at rest. The apparent frequency
observed by the observer is (velocity of sound in air (c) 448 Hz (d) 350 Hz
330 ms–1) [2015 RS]
EBD_7179
392 PHYSICS

Hints & Solutions


EXERCISE - 1 I = P/S, where S is surface area.
1. (d) 2. (a) 3. (d) 4. (c) 5. (d)
6. (c) Let I1 = I0 and I2 = 4 I0
Resultant intensity, I = I 1 + I 2 + 2 I1I 2 cos f r l

= I 0 + 4 I 0 + 2 I 0 4 I 0 cos0º = 9 I 0
When f = p, I = I0
When f = p/2, I = 5 I0 (i) For spherical wave front Þ S = 4pr2(surface)
7. (c) The contrast will be maximum, when I1 = I2 i.e. æ P öæ 1 ö 1
a = b. In that event, Imin = (a – b)2 = 0, where a and b are So I = ç ÷ ç ÷ Þ I µ
2
è 4π ø è r ø r2
the amplitudes of interfering waves.
8. (a) From v = n l, we find l µ v because freq. n is constant. 1
Since I a (Amplitude A)2 Þ A µ

.IN
Therefore, new wavelength = 4 l. r
9. (a) Here, I max = I1 + I 2 + 2 I1 I 2 cos 0º AL
I min = I1 + I 2 + 2 I1 I 2 cos 180º P r

\ I max + I min = 2 (I1 + I 2 )


N

v (ii) For cylindrical wave front S = 2prl


R

10. (d) For open pipe, n = p


2l
æ P öæ 1 ö 1 1
U

So I = ç ÷ç ÷ Þ I µ So A µ
v n 2p è 2πl øè r ø r r
JO

For closed pipe n ¢ = ( 2p - 1) \ ¢=


4l n 2 p -1 21. (b) Equation of a wave
11. (a) Because they are absorbed by the atmosphere. y1 = a sin (wt – kx) ....(i)
U

12. (a) Let equations of another wave may be,


uµ T
y2 = a sin (wt + kx) ....(ii)
ED

13. (d)
14. (c) Speed of sound is determined by elasticity and inertia. y3 = –a sin (wt + kx) ....(iii)
15. (a) Standing waves are produced when two waves If Eq. (i) propagate with Eq. (ii), we get
propagate in opposite direction y = 2a cos kx sin wt
As z 1 & z 2 are propagating in +ve x-axis & If Eq. (i), propagate with Eq. (iii), we get
–ve x-axis y = –2a sin kx cos wt
so, z1 + z2 will represent a standing wave. At x = 0, y = 0, wave produce node
16. (b)
So, Eq.(iii) is the equation of unknown wave
17. (a) For producing stationary waves, the two transverse
waves must be travelling in opposite directions. 22. (a) Frequency of reflected wave is,
Therefore, y1 and y2 is the only combination. æc+vö
18. (d) f' =ç ÷f
è c ø
gP E This is the number of wave striking the surface per
19. (b) v= =
r r second.
20. (c) The surface area of cylindrical surface is 2prl, where r 23. (c) The equation of progressive wave propagating in the
is radius of base, l is length of cylinder. positive direction of X-axis is
We know that with increasing distance from source, 2p
the total energy or power transmitted remains same, y = a sin ( nt - x ) or
l
but intensity decreases. For any source of power P,
intensity I at distance r from it will be y = a sin (wt – kx)
Waves 393

c 7. (a) Let x be distance of person from each cliff


S
v \ x + x = v × t = 340 × 1 = 340
24. (a) D x = 170 m
D
Distance between two cliffs = 2 x 170 = 340 m
c S v 2
8. (a) Time taken for two syllables t = sec.
Largest frequency (f1) Lowest frequency (f2) 5
2
Largest frequency will be detected when the source x + x = v ´ t = 330 ´ \ x = 66 m
5
approaches detector along the line joining and the 9. (c) At a given temperature, velocity of sound is not
smallest frequency will be detected when the source affected by pressure.
recedes the detector along the line joining them
10. (d) Here, l = 5.0 m, n = 2
æ c ö \ v = n l = 2 × 5.0 = 10.0 m/s
f
f1 çè c - v ÷ø c+v
= =
f2 æ c ö c- v I max (a + b) 2 a+b
çè ÷f 11. (b) = = 49 \ =7
c + vø I min (a - b) 2 a-b
25. (c) Fundamental frequency is given by
a 8 4
1 T 1 1 7a – 7b = a + b or 6a = 8b or = =
v= Þ vµ Þ P µ b 6 3

.IN
2l m l v
12. (b) No. of beats/sec = 402 – 401 = 401 – 400 = 1
Since, P divided into l1 , l2 and l3 segments 13.AL (d) Freq. of unknown fork = 256 ± 4 = 260 or 252
Here l = l1 + l2 + l3 As frequency decreases on loading, therefore, orginal
1 1 1 1 freq. of unknown fork = 260 Hz
So = + +
N
v v1 v2 v3 6
14. (a) No. of loops in longitudinal mode = =3
EXERCISE - 2 2
R

2 Dl 0.2
U

1. (b) Amplitude of reflected wave = ´ 0 .9 = 0 .6 15. (c) v= ´c = ´ 3 ´ 10 8 = 6 × 105 m/s


3 l 100
JO

It would travel along negative direction of x-axis, and


l
on reflection at a rigid support, there occurs a phase 16. (a) l1 + x = = 22.7 equation (1)
change of p. 4
U

2. (b) As R 2 = a 2 + b 2 + 2 a b cos f 3l
l2 + x = = 70.2 equation (2)
ED

4
1 2p
\ a 2 = a 2 + a 2 + 2 a 2 cos f , cos f = - ,f= 5λ
2 3 l3 + x = equation (3)
4
3. (d) As number of beats/sec = diff. in frequencies has to
be less than 10, therefore 0 < (n 1 –n2) < 10 From equation (1) and (2)
4. (d) In the spherical source, the amplitude A of wave is
l 2 - 3 l 1 70 .2 - 68 .1 2 .1
x= = = = 1 .05 cm
1 2 2 2
inversely proportional to the distance r i.e., A µ
r
Where r is distance of source from the point of l3 + x
From equation (2) and (3) =5
consideration. l1 + x

3 3 l 3 = 5 l 1 + 4x = 5 × 22.7 + 4 × 1.05 =117.7 cm


5. (b) c r.m.s. = v = 330 ´ » 471.4 m/s
g 1.4 v v
17. (c) n 1 = 256 = =
4 ´ 1 + 1 ´ 16 4 l 1 4 ´ 25 . 4
6. (b) If rH = 1, then r mix = ( 4 + 1)
=4
\ v = 256 × 101.6 cm/s
v mix rH 1 1 v 256 ´ 101.6
= = = n2 = = = 254 Hz
vH r mix 4 2 4l2 4 ´ 25.6
v H 1224 No. of beats/sec = n 1 – n2 = 256 – 254 = 2
v mix = = = 612 m / s
2 2
EBD_7179
394 PHYSICS

30. (c)
v 4.5 ´ 1014 ´ 10 6
18. (b) D n = n ´ = = 1.5 × 1012 Hz
c 3 ´ 108 æ I ö
31. (d) 10 logçç I ÷÷ = 10 ´ 12 log10 10 = 120 dB
\ n¢ = n - Dn = 4.5 × 1014 – 1.5 × 1012 è 0ø
= 4.485 × 1014 Hz
I 10-6
v1 r 32. (c) 10log = 10log = 10log106 = 60dB
19. (b) = 2 I0 10-12
v2 r1
20. (c) Comparing it with y (x, t) = A cos (wt + p/2) cos kx. 33. (c) Two possible frequencies of B are 256 ± 4 = 260 or
If kx = p/2, a node occurs ; \ 10 px = p/2 Þ x = 0.05 m 252. When A is loaded its frequency reduces from 256
If kx = p, an antinode occurs Þ 10px = p to 248 say which would produce 4 beats/sec only if
Þ x = 0.1 m freq. of B is 252
50p 1
Also speed of wave w / k = = 5m / s and 5
10p 34. (a) Here, m = kg/m = kg/m
20 4
l = 2p / k = 2p / 10p = 0.2 m Tension in the middle of wire
5 5
21. (c) Resultant amplitude is a 2 + a 2 i.e. 2a T = weight of half the wire = ×g= × 10 N
2 2

.IN
v 60 = 25 N
22. (b) n= = = 30 Hz
2l 2 ´ 1 AL 25
As v = T / m \ v = = 10 m / s
2p 2p 1/ 4
23. (b) w= and k =
0.01 0.30 v
35. (a) For open pipe, n = , where n0 is the fundamental
N
w 2p 0.30 2l
v= = ´ = 30 m s -1
R

k 0.01 2p frequency of open pipe.


24. (b) Equation is of stationary wave. Comparing with the v 330 11
U

standard equation \ l = 2 n = 2 ´ 300 = 20


æ 2p ö æ 2p ö
JO

y = 2A sin çè ÷ø t cos çè ÷ø x As freq. of 1st overtone of open pipe = freq. of 1st


T l
overtone of closed pipe
2p 2p
U

= 4.5 or l = = 1.4m v v 3l 3 11
l 4.5 \ 2 =3 Þ l' = = ´ = 41 .25 cm
2l 4 l¢ 4 4 20
ED

25. (c) Intensity doubles, dB level increases by 3 dB.


v
26. (b) y = A sin 2 (kx - wt ) 36. (b) = 412 , when cut into two equal pieces, frequency
4l
A of closed pipe of half the length
Þy= [1 - cos 2(kx - wt )]
2
v 2v
w = = = 2 ´ 412 = 824 Hz
w' = 2w, 2pn ' = 2w, n ' = 4 (l / 2) 4 l
p
frequency of open pipe of half the length
240
27. (c) 256 = n ´ = 4n Þ n = 64 v
60 = 2 (l / 2) = 4 ´ 412 = 1648 Hz
28. (c)
37. (b) As passenger is a part of moving train, there is no relative
gRT motion between source and listener. Therefore, n' = n =
29. (N) The speed of sound in a gas is given by v =
M 200 Hz
vO2 g O2 M He 1.4 4 p
\ = ´ = ´ = 0.3237 38. (c) x = 4(cos pt + sin pt) = 4[sin( - pt )] + sin pt ]
v He M O2 g He 32 1.67 2

vO2 é æ p ö æ p öù
\ v He =
460
= = 1421 m / s ê ç pt - - pt ÷ ç pt - + pt ÷ ú
= 4 ê 2 ´ sin ç 2 ÷ cos ç 2 ÷ú
0.3237 0.3237
ê ç 2 ÷ ç 2 ÷ú
None of the option is correct. ç ÷ ç ÷
êë è ø è ø úû
Waves 395

Case II : frequency of 2nd harmonic of organ pipe = 2f


é p æ p öù
= 8 êsin . cos ç - + pt ÷ ú (as is clear from the second figure)
ë 4 è 4 øû
2f = 420 ± 10 or f = 210 ± 5 or f = 205 or 215
[since cos (–q) =cosq].
Hence fundamental frequency of organ pipe = 205 Hz
8 é pù é pù
= . cos ê p t - ú = 4 2 cos ê p t - ú 1 T 1
2 ë 4û ë 4û 43. (d) T= s; t = = s
500 2 1000
Comparing it with standard equation
x = A cos ( wt – kx) Þ A = 4 2
44. (a) aw = 0.1´ 2p´ 300 = 60p cm s-1

39. (c) Velocity of wave v = nl é v + v0 ù é v + v /15 ù


45. (a) n' = nê ú = 600 ê ú
where n = frequency of wave Þ n =
v ë v - vs û ë v - v /10 û
l
é16 10 ù 400 ´16
v
n2 = 2 =
396
= 396 Hz = 600 ê ´ ú = » 711
l 2 100 ´ 10-2 ë15 9 û 9
no. of beats = n 1 –n2 = 4 46. (a) Doppler effect.

.IN
40. (a) By the concept of accoustic, the observer and source 47. (c) Velocity of source = 18 km h–1 = 5 m s–1
are moving towards each other, each with a velocity (i) S moves towards listener (vS)
of 18 m s–1. AL (ii) listener moves towards source (vL)
330 + 18
\n ' = ´1000 » 1115 Hz v + vL
330 - 18 n' = n = 280 Hz , Beats = n' – n = 8.
v - vS
N
41. (a) For 3rd harmonic/2nd over tone of organ pipe open at
ends
R

48. (a) T / l = constant; Tension decreases by a factor


l2
(8 – 1) / 8, length decreases by square root of this i.e.
U

0.77.
JO

49. (c) Particle velocity

l/ 4 l l/ 4 d é æ x öù
v = dt ê x 0 sin 2pç nt - l ÷ú
U

3ν ë è øû
Þ n2 =
ED

2l 2
For 1st overtone of organ pipe open at one end æ xö
= 2 pnx 0 cos 2pç nt - ÷
l1 è lø

\ Maximum particle velocity = 2pnx 0

l
l/ 2 l/ 4 Wave velocity = = nl
T

Þ n1 =
4l1 2pnx 0 px 0
Given, 2pnx 0 = 4nl Þ l = =
4n 2
3n 3n l 1
Given n1 = n 2 Þ = or 1 =
2l 2 4l1 l2 2 vA D B 1
50. (a) = =
42. (b) Let the fundamental frequency of organ pipe be f v B DA 2
Case I : f = 200 ± 5 = 205 Hz or 195 Hz 51. (a) Since Tension and mass per unit length remains
unchanged, the frequency will be obtained in different
mode.
52. (b) Effective value of velocity of source,

100 100 3
vs = cos q = ´ = 20 m s -1
3 3 5
EBD_7179
396 PHYSICS

P
100 m s–1 57. (c) Intensity at A, IA = ;
3 3 4pr 2
q P
intensity at B, IB =
4p (2r) 2
4
5 IA
Sound level at A, SA = 10log ;
I0

IB
Sound level at B, SB = 10 log
I0
v
n' = n Difference of sound level at A and B is
v - vs
IA I æI ö
340 SA - SB = 10log - 10 log B = 10log ç A ÷
Þ n' = ´ 640 Hz = 680 Hz I0 I0 è IB ø
340 - 20
= 10 log 4 = 20 log 2 » 6 dB
p 2p p 58. (b) Speed of pulse at a distance x /////////////////
53. (a) k = Þ = Þ l = 30 units v
15 l 15

.IN
from bottom, v = gx .
Distance between node and next antinode
While traveling from mid point x
l 30 to the top, frequency remains
= = = 7.5 units
4 4
AL unchanged.
54. (a) y(x, t) = 0.005 cos (ax - bt) (Given) v1 v2 g (L / 2) gL
= = Þ l 2 = 2l 0
l1 l 2 Þ
N
Comparing it with the standard equation of wave l0 l2
y(x, t) = a cos (kx - wt) we get
R

59. (b) y
k=a and w = b
U

2p 2p
JO

But k = and w =
l T
2p 2p
Þ =a and =b
U

l T
x(m)
Given that l = 0.08 m and T = 2.0s
ED

2p 2p Dotted shape shows pulse position after a short time


\ a= = 25p and b = =p interval. Direction of the velocities are decided
0.08 2
according to direction of displacements of the
l1 l2 l3
55. (a) particles.
60. (c) Compare the given eqn. with the standard form
110 cm
n1 : n2 : n3 = 3 : 2 : 1 é 2 px 2 pt ù
y = a 0 sin ê -
1 ë l T úû

l 2p
2p
1 1 1 = 3 , l = 2p / 3 and T = 15
l1 : l 2 : l 3 = : : = 2 : 3 : 6 l
3 2 1
l1 + l 2 + l 3 = 110 T = 2 p/15
Þ 2x + 3x + 6x = 110 Þ x = 10 l 2p/3
Speed of propagation, v = = =5
\ The two bridges should be set at 2x i.e, 20 cm from T 2 p / 15
one end and 6x i.e, 60 cm from the other end.
56. (b) nX = 258 Hz or 242 Hz v v l c 3v 2 3
61. (b) 3 ´ = 4´ or = ´ =
nX = 258 Hz or 282 Hz 4l c 2l0 l0 4 4v 8
Þ frequency is 258 Hz 62. (c) Let f ' be the frequency of sound heard by cliff.
The total momentum will be zero and hence velocity
will be zero just after collision. The pull of earth will νf
\f¢= ......(1)
make it fall down. v - vc
Waves 397

Now for the reflected wave cliff. acts as a source 68. (c) We have, v = nl
f ¢ (v + v c ) Þ v µ l (as n remains constant)
\2 f ¢ = ......(2)
Thus, as v increases 10 times, l also increases 10 times.
v
69. (a) For fundamental mode,
(v + v c ) f
2f = v
Þ 2v – 2 vc = v + vc or = v c
v - vc 3
1 T
f=
–6 2l m
63. (b) y1 = 10 sin (100 t + x/50 + 0.5)m
= 10–6 cos (100 t + x/50 – p/2 + 0.5)m Taking logarithm on both sides, we get
y2 = 10–6 cos (100 t + x/50)m æ 1ö æ Tö
log f = log ç ÷ + log ç ÷
\ f = p/2 – 0.5 = 1.07 rad è 2l ø è mø
64. (b) Equation of progressive wave is given by
Y = A sin2pf t æ1 ö 1 æTö
= log çè ÷ø + log ç ÷
Given Y1 = 4sin500 pt and Y2 = 2sin506pt. 2l 2 èmø
Comparing the given equations with equation of
æ1ö 1
progressive wave, we get or log f = log ç ÷ + [log T - log m]
è 2l ø 2
2f1 = 500, Þ f1 = 250

.IN
2f2 = 506 Þ f2 = 253 Differentiating both sides, we get
Beats = f2– f1 = 253 – 250 = 3 beats/sec df 1 dT
AL = (as l and m are constants)
= 3 × 60 = 180 beats/minute. f 2 T
65. (c) y = A sin ( w t–kx) dT df
Particle velocity, Þ = 2´
T f
N
dy Here df = 6
vp = = A w cos ( w t – kx)
R

dt f = 600 Hz
\ vp max = A w dT 2 ´ 6
U

\ = = 0.02
w T 600
JO

wave velocity =
k D x 2 p (0.4)
l
w 70. (c) From Dx = Df , l = 2 p = = 0. 5 m
\ Aw = 2p Df 1 .6 p
U

k
2p
ED

1 v 330
i. e., A = But k = n= = = 660 Hz
k l l 0 .5
\ l=2pA 71. (d) 2p f1 = 600 p
66. (d) The frequency of the piano string = 512 + 4 = 516 or f1 = 300 ... (1)
508. When the tension is increased, beat frequency 2p f2 = 608 p
decreases to 2, it means that frequency of the string is f2 = 304 ...(2)
508 as frequency of string increases with tension. |f1 – f2| = 4 beats
67. (a) Here, y1 = a sin (wt + kx + 0.57)
and y2 = a cos (wt + kx) I max ( A1 + A2 )2 (5 + 4)2 81
= = =
I m.n ( A1 + A2 )2 (5 - 4)2 1
ép ù
= a sin ê + (wt + kx) ú
ë2 û where A1, A2 are amplitudes of given two sound wave.
Phase difference, Df = f2– f1 72. (c) 73. (b)
74. (d) Two waves moving in uniform string with uniform
p
= - 0.57 tension shall have same speed and may be moving in
2 opposite directions. Hence both waves may have
velocities in opposite direction. Hence statement-1 is
3.14
= - 0.57 false.
2
75. (d) Statement-2 is true, Statement-1 is false. In doppler for
= 1.57 – 0.57 sound wave effect due to observer and source motion
= 1 radian are different.
EBD_7179
398 PHYSICS

EXERCISE - 3 5. (b) Only energy is transmitted from one point to another


and during propagation of any longitudinal waves in
Exemplar Questions a medium transmission of energy through the medium
1. (b) As the water waves are produced by a motor boat on without matter being transmitted.
surfaces of water as well as inside the water are both 6. (c) Mechanical transverse wave can propagates through
longitudinal and transverse, because the waves, a solid medium, the constituent of the medium oscillate
produce transverse as well as lateral vibrations in the perpendicular to wave motion causing change in
particles of the medium. shape. Thus each, element of the medium is subjected
2. (c) Let the frequency in the first medium is v and in the to shearing stress. Solids and strings have shear
second medium is v'. modulus, so sustain shearing stress.
As we know that, Fluids have no shape of their own, they yield to
When waves passes from one medium to another its shearing stress. Hence, transverse waves are possible
frequency does not change but its velocity and
in solids and strings but not in fluids.
wavelength changes.
7. (d)
v v' (a) The density of medium changes due to compression
Hence, v = v ' Þ =
l l' and rarefraction. At compressed regions density is
(frequency are same for both medium) maximum and at rarefactions density is minimum.
æv'ö (b) As density is changing very rapidly so temperature of
l ' = ç ÷l

.IN
medium increases. Hence Boyle's law is not obeyed.
èvø
(c) Bulk modulus of air remains same or constant.
l and l', are wavelengths and v and v' speeds in first
and second medium respectively.
AL (d) The time of compressions and rarefraction is very small
i.e., we can assume adiabatic process, i.e. no transfer
æ 2v ö
So, l ' = ç ÷ l = 2l of heat from surroundings.
è v ø 8. (b) As given that
N
3. (c) The speed of sound (longitudinal) wave in air is
Amplitude of reflected wave
pp
R

v= . 2 æ2 ö
r Ar = ´ Ai = ç ´ 0.6 ÷ = 0.4 units
U

The density of water vapours is small (rises up) than 3 è 3 ø


the air, so on increasing humidity the density of Given incident wave
JO

medium increase the speed of sound in air.


For air l and p are constants, æ xö
yi = 0.6sin 2p ç t - ÷
1 è 2ø
U

vµ , where r is density of air..


r As we know that the reflected wave equation at denser
ED

v2 r2 medium where phase difference is p


=
v1 r1 æ x ö
yr = Ar sin 2p ç t + + p ÷
where r1 is density of dry air and r2 is density of è 2 ø
moist air. The positive sign is due to reversal of direction of
v2 propagation
As r2 < r1 = > 1 Þ v2 > v1
v1
æ xö
Hence, speed of sound wave in air increases with So, yr = -0.4sin 2p ç t + ÷ [Q sin(p + q) = - sin q]
è 2ø
increase in humidity.
4. (c) As we know that, 9. (b) As given that, Mass M = 2.5 kg
vt = v0 (1 + 0.61t) (Mass per unit length)
Speed of sound wave in a medium v µ T M 2.5 kg 125
m= = = = 0.125 kg/m
(where T is temperature of the medium) l 20 10
Clearly, when temperature increase then the speed also
increase as frequency does not change during T 200
Speed v = =
propogation of wave by formula. m 0.125
As, v = nl [speed of transverse waves in any string]
where n is frequency and l is wavelength. Distance l = v ´ t
Frequency (n) remains fixed,
v µ l or l µ v 200
Þ 20 = ´t
So velocity v and wavelength (l) both increases. 0.125
Waves 399

125 25 ´ 5 2p 2p
t = 20 ´ = 20 ´ k= = =1 [Q l = 2 p]
2 ´ 105 2 ´105 l 2p
\ Y = 1 sin (2t – x + f) [Q A = 1 m]
1
= 20 ´ 25 ´ 1 T
0.4 ´ 105 14. (a) From formula, f =
x m
1 20 ´ 5 1
= 20 ´ 5 = Þ µl
4 ´ 10 4 2 ´102 f
1 1 1 1
t= = 0.5 sec. \ l1 : l2 : l3 = : :
2 f1 f 2 f3
10. (c) When observer is at rest and source of sound is moving = f2 f3 : f1 f3 : f1 f2 [Given: f1 : f2 : f3 = 1 : 3 : 5]
towards observer then observed frequency n'. = 15 : 5 : 3
Let the original frequency of the source is n0. Therefore the positions of two bridges below the wire are
Let the speed of sound wave in the medium is v. 15 ´ 100 15 ´100 + 5 ´100
cm and cm
As observer is stationary 15 + 5 + 3 15 + 5 + 3

i.e., 1500 cm, 2000 cm

.IN
v 23 23
v0
Df n
15. (b) =
AL f C
Observer (Beats) 2 n
Þ =
N
f C
æ v ö
( n ') = ç
R

Apparent frequency ÷ n0 2 fv
è v - vs ø Þ Beats = = 4.
C
U

(when train is approaching) 16. (a) Total length of string l = l1 + l2 + l3


(As string is divided into three segments)
JO

æ v ö
=ç ÷ n0 ( n ' > n0 )
è v - vs ø 1 æ 1 Tö
But frequency µ çQ f =
ç ÷
length è 2l m ÷ø
U

When the train is moving away from the observer.


ED

æ v ö 1 1 1 1
Apparent frequency n '' = ç ÷ n0 (n '' < n0 ) so = + +
è v + vs ø n n1 n 2 n 3
Hence, frequencies in both cases are same and n' > n''. 17. (d) In case of closed organ pipe frequency,
so graph (c) verifies. v
fn = (2n + 1)
Past Years (2017-2013) NEET/AIPMT Questions 4l
for n = 0, f0 = 100 Hz
11. (c) Pressure change will be minimum at both ends. In fact, n = 1, f1 = 300 Hz
pressure variation is maximum at l/2 because the dis-
n = 2, f2 = 500 Hz
placement node is pressure antinode.
n = 3, f3 = 700 Hz
12. (d) When sounded with a source of known frequency
n = 4, f4 = 900 Hz
fundamental frequency
n = 5, f5 = 1100 Hz
= 250 ± 4 Hz = 254 Hz or 246 Hz
n = 6, f6 = 1300 Hz
2nd harmonic if unknown frequency (suppose) 254
Hz = 2 × 254 = 508 Hz Hence possible natural oscillation whose frequencies
< 1250 Hz = 6(n = 0, 1, 2, 3, 4, 5)
As it gives 5 beats
18. (c) According to Doppler's effect
\ 508 + 5 = 513 Hz
Apparent frequency
Hence, unknown frequency is 254 Hz
13. (d) As Y = A sin (wt – kx + f) æ v + v0 ö æ 343 + 10 ö
n¢ = n ç ÷ = 1392 ç ÷.
2p 1 è v + v s ø è 343 + 5 ø
w = 2pf = =2 [Q f = ]
p p = 1412 Hz
EBD_7179
400 PHYSICS

19. (b) Fundamental frequency of closed organ pipe 22. (c) According to Doppler's effect in sound
V
Vc =
4lc Observer source 15 m/s
Fundamental frequency of open organ pipe
V Apparent frequency,
V0 = 2l
0 v
n' = v - v n 0
3V s
Second overtone frequency of open organ pipe = 2l
0
=
330
(800 ) = 330 ´ 800 = 838 Hz
From question, 330 - 15 315
The frequency of sound observer hears in the echo
V 3V
= reflected from the cliff is 838 Hz.
4lc 2l0 23. (b) From figure, tension T1 = m2g
Þ l0 = 6lc = 6 × 20 = 120 cm T2 = (m1 + m2)g
Rigid support
20. (a) Here, original frequency of sound, f0 = 100 Hz As we know
Speed of source Vs = 19.4 cos 60° = 9.7 Velocity µ T So, T2
19.4 lµ

.IN
T L m1
l1 T
Þ = 1 T1
l2
AL T2
m2
l2 m1 + m2
Þ =
N
60° l1 m2
S O
R

19.4 cos 60° = 9.7 24. (c) For a closed organ pipe first minimum resonating
length
U

From Doppler's formula


l
L1 = = 50 cm
JO

æ V - V0 ö 4
f1 = f0 ç ÷ \ Next or second resonating length, L2
è V - Vs ø
3l
= = 150 cm
U

æ V-0 ö 4
f1 = 100 ç ÷
è V - ( +9.7) ø
ED

25. (a) Difference in two successive frequencies of closed


V 100 pipe
f1 = 100 =
æ 9.7 ö æ 9.7 ö 2v
V ç1 - ÷ ç1 - ÷ = 260 – 220 = 40 Hz
è V ø è 330 ø 4l
= 103Hz 2v
Apparent frequency f1 = 103 Hz or = 40 Hz
4l
21. (c) In a stretched string all multiples of frequencies can
be obtained i.e., if fundamental frequency is n then v
higher frequencies will be 2n, 3n, 4n ... Þ = 20Hz
4l
75 cm Which is the fundamental frequency of system of
closed organ pipe.
26. (c) As we known from Doppler's Effect

So, the difference between any two successive é v + v0 ù é 340 + 16.5 ù


fapprent = f0 ê v - v ú = 400 ê ú
frequencies will be 'n' ë sû ë 340 - 22 û
According to question, n = 420 – 315 fapprent = 448 Hz
= 105 Hz
vs = 22 m/s v0= 16.5 m/s
So the lowest frequency of the string is105 Hz. A B
f0= 400 Hz
Electric Charges
16 and Fields
ELECTRIC CHARGE METHODS OF CHARGING
Charge is something associated with matter due to which it (i) By friction : By rubbing two suitable bodies, given in box
produces and experiences electric and magnetic effects. one is charged by +ve and another by –ve charge in equal
There are two types of charges : amount.

.IN
(i) Positive charge and (ii) Negative charge +ve –ve
Positive and negative charges : Positive charge means the
Glass rod Silk
deficiency of electons while negative charge means excess of
Fur Ebonite rod
electrons. In any neutral body the net charge is equal to zero
AL
i.e., the sum of positive charges is equal to the sum of negative Dry hair Comb
charges.
N
Wool Amber
+ Mass M. – – Mass M''
Mass M' –
R

+
Electron – Electron – Note : Electric charges remain confined only to the
+ Electron
< Proton – > Proton
U

= Proton – rubbed portion of a non-conductor but in case of a conductor,


+ – –
+ – they spread up throughout the conductor.
JO

– – –
Positively charged Uncharged body Negatively charged
body M' < M M body M'' > M (ii) By conduction : Charging a neutral body by touching it
Charge is a scalar quantity and its SI unit is coulomb (C). with a charged body is called charging by conduction.
U

CONDUCTORS AND INSULATORS · It is important to note that when the bodies are charged
by conduction, a charged and an uncharged bodies
ED

The materials which allow electric charge (or electricity) to


are brought into contact and then seperated, the two
flow freely through them are called conductors. Metals are very
good conductors of electricity. Silver, copper and aluminium are bodies may or may not have equal charges.
some of the best conductors of electricity. Our skin is also a · If the two bodies are identical the charges on the two
conductor of electricity. Graphite is the only non-metal which is a will be equal.
conductor of electricity. · If the two bodies are not identical, the charges will be
All metals, alloys and graphite have 'free electrons', which can different.
move freely throughout the conductor. These free electrons make · The potential of the two bodies will always be the same.
metals, alloys and graphite good conductor of electricity. (iii) By induction : Charging a body without bringing it in
Aqueous solutions of electrolytes are also conductors. contact with a charged body is called charging by
induction.
The materials which do not allow electric charge to flow through
them are called nonconductors or insulators. A +++ – – B ++
+ – +
For example, most plastics, rubber, non-metals (except graphite), –
dry wood, wax, mica, porcelain, dry air etc., are insulators. + ++ –– ++
e–
Insulators can be charged but do not conduct electric charge.
Insulators do not have 'free electrons' that is why insulators do
not conduct electricity.
First rearrangement of charge takes place in metal rod B.
Induced charge can be lesser or equal to inducing charge (but When the rod B is connected to earth, electrons flow from
never greater) and its max. value is given by
earth to the rod B thus making it -vely charged
Q' = – Q (1 – 1/k), where 'Q' is inducing charge and 'K' is the
The magnitude of elementary positive or negative charge
dielectric const. of the material of the uncharged body.
(electron) is same and is equal to 1.6 × 10–19 C.
For metals k = ¥ Þ Q' = – Q.
EBD_7179
402 PHYSICS

Properties of Electric Charge Quantisation of electric charge is a basic (unexplained) law


(i) Similar charges repel and dissimilar charges attract each of nature. It is important to note that there is no analogous
other. law of quantisation of mass.
Recent studies on high energy physics have indicated the
presence of graphs with charge 2e/3, e/3. But since these
F
cannot be isolated and are present in groups with total charge,
+ + + + + + + + therefore the concept of elementary charge is still valid.
F Attractive
COULOMB’S LAW
ive

+ F force
F
The force of attraction or repulsion between two point charges
Repuls

+
force

+ (q1 and q2) at finite separation (r) is directly proportional to


+ the product of charges and inversely proportional to the square
of distance between the charges and is directed along the line
joining the two charges.
In rare situation you may find similar charged bodies qq 1 q1q2 q1 q2
attracting each other. Suppose a big positive charged body i.e., F µ 1 2 or F = . 2
r2 4pe r r
is placed near a small positively charged body then because
where e is the permittivity of medium between the charges.
of induction, opposite charge produced on the small body
makes it to attract the other body. If e 0 is the permittivity of free space, then relative permittivity of
(ii) A charged body attracts light uncharged bodies, due to medium or dielectric constant (K), is given by

.IN
polarisation of uncharged body. e
Wall er ( or K ) =
e0

-- +
AL
The permittivity of free space
e 0 = 8.86 ´ 10-12 C 2 N -1m -2
--
Positively +
charged + 1 1
N
balloon and = = 9 × 109 Nm2 C –2.
+ 4pe 0 4 ´ 3.14 ´ 8.86 ´ 10- 2
R

Fig : When a positively charged balloon is placed in 1


contact with the wall, an opposite charge is induced with Also = 1 in CGS system of unit.
U

4 pe 0
the wall, the balloon stick to the wall due to electrostatic Coulomb’s law may also be expressed as
JO

attraction
1 qq
(iii) Charge is conserved i.e., the charge can neither be created F= . 1 2
nor be destroyed but it may simply be transferred from one 4pe0 e r r 2
Let F0 be the force between two charges placed in vacuum then
U

body to the other.


Thus we may say that the total charge in the universe is 1 q1q2
F0 = .
ED

constant or we may say that charges can be created or 4pe 0 r 2


destroyed in equal and opposite pair. For example
r
® e-
g (Energy ³ 1.02MeV) ¾¾ + e+
Electron Positron
(Pair-production process) Medium with dielectric
· Positron is an antiparticle of electron. It has same mass • •
+q1 constant K –q2
as that of electron but equal negative charge.
e- + e+ ¾
¾® g (Pair-annihilation process) æ 1 q1q2 e ö
= K çèQ F = 4pe . r 2 and e = K ÷ø
F0
(iv) Charge is unaffected by motion. This is also called charge Hence
F 0
invariance with motion Therefore we can conclude that the force between two
Mathematically, (q)at rest = (q)in motion charges becomes 1/K times when placed in a medium of
(v) Quantisation of charge. A charge is an aggregate of small dielectric constant K.
unit of charges, each unit being known as fundamental or The value of K for different media
elementary charge which is equal to e = 1.6 × 10–19 C. This
principle states that charge on any body exists as integral Medium Dielectric Cons tant (K)
mutliple of electronic charge. A ir 1.006
i.e. q = ne where n is an integer. Vacuu m 1.00
According to the concept of quantisation of charges, the
W ater 1.00026
charge q cannot go below e. On macroscopic scale, this is
as good as taking limit q0 ® 0. M ica 3 to 6
M etals ¥
Electric Charges and Fields 403

Dielectric : A dielectric is an insulator. It is of two types - r r 1 q1dq q1 dq


(i) Polar dielectric and i.e., F1 = ò dF1 = ò 2
rˆ = ò rˆ
(ii) Non-polar dielectric. 4p Î0 r 4p Î0 r
Significance of Permittivity Constant or Dielectric Constant : where r̂ is a variable unit vector which points from each dq,
Permittivity constant is a measure of the inverse degree of towards the location of charge q1 (where dq is a small charge
permission of the medium for the charges to interact. element)
Dielectric strength : The maximum value of electric field that can Types of Charge Distribution
be applied to the dielectric without its electric breakdown is called
(i) Volume charge distribution : If a charge, Q is uniformly
its dielectric strength.
distributed through a volume V, the charge per unit volume
Difference between electrostatic force and gravitational force : r (volume charge density) is defined by
Electrostatic force Gravitational force Q
r= ; r has unit coulomb/m3.
1. Much stronger 1. Much weaker as compared V
to electrostatic force . . . .
. .
. .. dq
2. Can be attractive 2. Only attractive
or repulsive
dQ = rdV . .. .
. .. ..
Total charge is
3. Depends on the nature of 3. Does not depend on the . uniformly distributed
. .V
in a cube
medium between charges nature of medium

.IN
between masses

Both electric and gravitational forces follow inverse (ii)


AL Surface charge distribution : If a charge Q is uniformly
distributed on a surface of area A, the surface charge density
square law.
s , is defined by the following equation
Vector Form of Coulomb’s Law : Q
s = , s has unit coulomb / m2
N
uuur 1 q1q 2 uur 1 q1q 2 A
F12 = r12 = rˆ12
4pe 0 k r3 4pe 0 k r 2
R

Total charge Q, which


U

SUPERPOSITION PRINCIPLE FOR DISCRETE CHARGE


is uniformly
DISTRIBUTION : FORCE BETWEEN MULTIPLE dQ = s dA distributed over disc
JO

CHARGES
r13
U

r12 q2 q3
q5 (iii) Linear charge distribution : If a charge q is uniformly
ED

q4 distributed along a line of length l, the linear charge density


q1
q6 l, is defined by
O q7 Q
Discrete charge l = , l has unit coloumb/m.
r17 l
r distribution
The electric force F1 on q1 due to a number of charges placed in dQ = ldl
air or vacuum is given by
r 1 n q1 q i 1 é q1 q 2 q1 q 3 ù dl
F1 = å 2 rˆ1i = ê 2 rˆ12 + 2 rˆ13 + ......ú
4pe 0 i =1 r1i 4pe 0 ëê r 12 r13 ûú
Total charge Q uniformly
Q distributed in a tube

Coulomb's law is valid if r ³ 10 -15 m and if charges


are point charges l If the charge is non uniformly distributed over a volume,
surface, or line we would have to express the charge
FORCE FOR CONTINUOUS CHARGE DISTRIBUTION densities as
A small element having charge dq is considered on the body. The
dQ dQ dQ
force on the charge q1 is calculated as follows : r= , s= , l= ,
dV dA dl
ur 1 q1dq
d F1 = . rˆ where dQ is the amount of charge in a small volume, surface
4pÎ0 r 2 or length element.
r r l In general, when there is a distribution of direct and
Now the total force F1 is calculated by integrating dF1 under continuous charge bodies,we should follow the following
proper limits. steps to find force on a charge q due to all the charges :
EBD_7179
404 PHYSICS

(1) Fix the origin of the coordinate system on charge q. Keep in Memory
(2) Draw the forces on q due to the surrounding charges
considering one charge at a time. 1. When the distance between the two charges placed in
(3) Resolve the force in x and y-axis respectively and vacuum or a medium is increased K-times then the force
between them decreases K2-times. i.e., if F0 and F be the
find SFx and SFy initial and final forces between them, then
Fo
(4) The resultant force is F = (SFx )2 + (SFy )2 and the F=
K2
SFx 2. When the distance between the two charges placed in
direction is given by tan q = .
SFy vacuum or a medium is decreased K-times then the force
between them increases K2-times. i.e., if Fo and F be the
Calculation of electric force in some situations : initial and final forces then
(a) Force on one charge due to two other charges : F = K2Fo
Resultant force on q due to q1 and q2 are obtained by 3. When a medium of dielectric constant K is placed between
vector addition of individual forces the two charges then the force between them decreases by
K-times. i.e., if Fo and F be the forces in vacuum and the
q1
medium respectively, then
Fo
F=

.IN
K
F1 F 4. When a medium of dielectric constant K between the
charges is replaced by another medium of dielectric constant
q
AL K' then the force decreases or increases by (K/K') times
+q
F2 q2 according as K' is greater than K or K' is less than K.
r r r
N
Example 1.
F = F1 + F2
If we supply a charge to a soap bubble then it will expand.
R

r r r Why?
F = | F1 |2 + | F2 |2 Solution :
U

r Since we know like charges repel and try to get away from
JO

|F | each other which is at outer surface of the conductor. So a


The direction of F is given by tan q = r1
| F2 | soap bubble expand.
(b) Force due to linear charge distribution :
U

Example 2.
Let AB is a long (length l) thin rod with uniform
Calculate the net charge on a substance consisting of
ED

distribution of total charge Q.


(a) 5 × 1014 electrons (b) a combination of 7 × 1013
dx a protons and 4 × 1013 electrons.
•q Solution :
A dQ B
(a) The charge of one electron is –1.6×10–19C. So net
x
charge on a substance consisting of 5 × 1014 electrons
We calculate force of these charges i.e. Q on q which is
is situated at a distance a from the edge of rod AB. 5 × 1014 × (–1.6 × 10–19C) = -8 ´ 10 -5 C = -80mC.
Let, dQ is a small charge element in rod AB at a distance (b) Similarly the net charge on a substance consisting of a
x from q . combination of 7 × 1013 protons and 4 × 1013 electron
The force on q due to this element will be is
q dQ q mdx [7 ´ 1013 ´ (1.6 ´ 10 -19 C)] + [ 4 ´ 1013 (-1.6 ´ 10 -19 - C]
dF = =
4pe o x 2 4pe o x 2 = + 4.8 mC.
(Q the charge on one proton is + 1.6 × 1019C)
where m is linear charge density i.e., m = Q / l.
Example 3.
l+ a l+a Two protons in a molecule is separated by a distance
q µ
so, F = ò dF = ò . dx
4p Îo x 2
3 × 10–10 m. Find the electrostatic force exerted by one
a a proton on the other.
l+ a
Solution :
qµ 1
=
4p Î0 ò 2
dx newton
3 ´ 10 -10 m
a x
–19 –19
1.6 × 10 C 1.6 × 10 C
Electric Charges and Fields 405

According to coulomb’s law, the electrostatic force F Example 6.


between two charges q1 and q2, which are seperated by Four identical point charges each of magnitude q are
distance r is placed at the corners of a square of side a. Find the net
q q electrostatic force on any of the charge.
1
´ 1 2
F= Solution :
4pe o r2 Let the concerned charge be at C then charge at C will
Here, q1 = q2 = 1.6 × 10–19C , r = 3 × 10–10 m experience the force due to charges at A, B and D. Let these
r r r
(1.6 ´ 10-19 ) 2 forces respectively be FA , FB and FD thus forces are given as
so, F = 9 ´ 109 = 2.56 ´ 10 -9 C (Repulsive)
9 ´ 10 -20 A Bq
Example 4. q y
When a piece of polythene is rubbed with wool, a charge
of 2 × 10–7 C is developed on polythene. What is the
amount of mass, which is transferred to polythene?
Cq
Solution : D FD x
q q FA
No. of electrons transferred, n = FB
e
æ qö r 1 q2 q2 æ î ĵ ö÷
Mass transferred = m e ´ n = m e ´ ç ÷ FA = along AC = ç -
è eø 4pe 0 AC 2 4pe 0 2a 2 ç 2 2 ÷ø
è

.IN
æ 2 ´10 -7 ö r
= 9.1´10 -31 ´ ç ÷ 1 q2 q2
ç 1.6 ´10 -19 ÷ FB = along BC = ( - ĵ)
è ø 4pe 0 BC 2 4πε 0 a 2
AL
= 11.38 ´10 -19 kg r 1 q2 q2
Example 5. FD = along DC = (î )
4pe 0 DC 2
4πε 0 a 2
N
Two negative charges of unit magnitude each and a
positive charge q are placed along a straight line. At what r r r r
Fnet = FA + FB + FD
R

position and for what value of q will the system be in


equilibrium? Check whether it is stable, unstable or q2 éæ 1 ö æ 1 öù
U

neutral equilibrium? = êî çç + 1÷÷ - ĵ çç + 1÷÷ú


4 πε 0 a 2
ëê è 2 2 ø è2 2 øúû
Solution :
JO

Let the charge + q be held at a distance x1 from unit negative r æ 1 ö q 2


æ1 ö q
2
Fnet = 2ç + 1÷ = ç + 2 ÷
charge at A, and at a distance x2 from unit negative charge ç ÷ 4pe a 2 è2 ø 4pe 0 a 2
at B.
è2 2 ø 0
U

Example 7.
–1 +q –1
Electric force between two point charges q and Q at rest is
ED

A B F. Now if a charge – q is placed next to q what will be the


x1 x2 (a) force on Q due to q (b) total force on Q ?
q (-1) q (-1) Solution :
For equilibrium of q, =
4 p e o x12 4 p e o x 22 (a) As electric force between two body interaction, i.e., force
\ x1 = x2 i.e. q must be equidistant from A and B. between two particles, is independent of presence or
For equilibrium of unit negative charge at B. absence of other particles, the force between Q and q
Force on B due to charge at A + force on B due to q = 0 will remain unchanged, i.e., F.
(b) An electric force is proportional to the magnitude of
(-1) (-1) q (-1) charges, total force on Q will be given by
2
+ =0
4 p e o ( x1 + x 2 ) 4 p e o x 22 F¢ Qq ¢ q ¢ 0
= = = = 0 [as q' = q + (– q) = 0]
1 -(-q) F Qq q q
Þ = (Q x1 = x 2 )
2 i.e., the resultant or total force on Q will be zero.
4 p e o (2 x 2 ) 4 p e o x 22
1 1 ELECTRIC FIELD
q= i.e. th of the magnitude of either unit charge.
4 4 The space around an electric charge, where it exerts a force on
Stability : If q is displaced slightly towards A, force of another charge is an electric field.
attraction due to A exceeds the force of attraction due to B. Electric force, like the gravitational force acts between the bodies
Therefore, q will get displaced further towards A. Hence that are not in contact with each other. To understand these
the equilibrium of q is unstable. forces, we involve the concept of force field. When a mass is
However, if q is displaced in a direction ^ to A, net force present somewhere, the properties of space in vicinity can be
would bring q back to its normal position. Therefore, the considered to be so altered in such a way that another mass
equilibrium will be stable.
EBD_7179
406 PHYSICS

brought to this region will experience a force there. The space


where alteration is caused by a mass is called its Gravitational
field and any other mass is thought of as interacting with the
field and not directly with the mass responsible for it. +2q –q
Similarly an electric charge produces an electric field around it
so that it interacts with any other charges present there. One
reason it is preferable not to think of two charges as exerting
forces upon each other directly is that if one of them is (iii) The number of lines that originate from or terminate on a
changed in magnitude or position, the consequent change in charge is proportional to the magnitude of charge.
the forces each experiences does not occur immediately but | q1 | N1
takes a definite time to be established. This delay cannot be i.e., | | =
q2 N2
understood on the basis of coulomb law but can be explained (iv) Two electric lines of force never intersect each other.
by assuming (using field concept) that changes in field travel (v) They begin from positive charge and end on negative charge
with a finite speed. (» 3 × 108 m / sec). i.e., they do not make closed loop (while magnetic field
Electric field can be represented by field lines or line of force. lines form closed loop).
The direction of the field at any point is taken as the direction of
the force on a positive charge at the point.

.IN
Electric field intensity due to a charge q at any position + –

r F
( r ) from that charge is defined as
ur
uur F
AL
(vi) Where the electric lines of force are
q0 (a) close together, the field is strong (see fig.1)
E =
q0 r (b) far apart, the field is weak (see fig.2)
ur
N
uur F 1 q
E (r) = Limit = ˆr Electric field lines
q0 ® 0 q 0 4pe r 2
R

q
O Strong field
U

where F is the force experienced by a small positive test charge Fig. 1


JO

q0 due to charge q. Its SI unit is NC–1.


It is a vector quantity.
If there are more charges responsible for the field, then Weak field
uur uur uur uur Fig. 2
U

E = E1 + E2 + E3 + ........
ED

where E1 , E 2 , E 3 ,........ are the electric field intensities due to (vii) Electric lines of force generate or terminate at charges
/surfaces at right angles.
charges q1, q2, q3 ......respectively.
ELECTRIC LINES OF FORCE -
- -
These are the imaginary lines of force and the tangent at any -
point on the lines of force gives the direction of the electric field
- -
at that point. - -
Properties of Electric Lines of Froce -
- -
(i) The lines of force diverge out from a positive charge and -
converge at a negative charge. i.e. the lines of force are
always directed from higher to lower potential. d
Fixed point charge
near infinite metal plate

Electric Field for Continuous Charge Distribution :


+ – If the charge distribution is continuous, then the electric field
strength at any point may be calculated by dividing the charge
into infinitesimal elements. If dq is the small element of charge
r
(ii) The electric lines of force contract length wise indicating within the charge distribution, then the electric field dE at point
unlike charges attract each other and expand laterally P at a distance r from charge element dq is
indicating like charges repel each other. uur 1 dq ^
dE = r;
4pe r 2
Electric Charges and Fields 407

Non conducting sphere (dq is small charge element) (ii) A circular ring of radius R with uniformly distributed charge
r Qx
E =k Q +
2 2 3/2
(R + x )
R P R E
Continuous Q x P
charge When x >> R, E = k
distribution x2
[The charge on ring behaves +
dq = ldl (line charge density) as point charge]
= s ds (surface charge density) R Q
= rdv (volume charge density) E is max when x = ± . Also Emax =
2 6 3pe 0 R 2
The net field strength due to entire charge distribution is given
by (iii) A circular disc of radius R with uniformly distributed
uur 1 dq ^ charge with surface charge density s
E =
4pe ò r2
r

where the integration extends over the entire charge distribution. R


Electric field intensity due to a point charge q, at a E

.IN
Q x
distance (r1 + r2) where r1 is the thickness of medium of dielectric p
constant K1 and r2 is the thickness of medium of dielectric constant AL
K2 as shown in fig. is given by
r1 r2
2kQ é x ù s é x ù
E= ê1 - ú= ê1 - ú
N
q
R ëê x 2 + R2 úû 2e0 ëê x 2 + R2 ûú
K2 2
K1
R

1 q
E= (iv) An infinite sheet of uniformly distributed charges with
(
4 p Î0 r K + r K
)
2
surface charge density s
U

1 1 2 2
JO

CALCULATION OF ELECTRIC FIELD INTENSITY FOR


A DISTRIBUTION OF DIRECT AND CONTINOUS
CHARGE
U

E
1. Fix origin of the coordinate system where electric field p
ED

intenstiy is to be found.
2. Draw the direction of electric field intensity due to the
surrounding charges considering one charge at a time.
3. Resolve the electric field intensity in x and y-axis s
E=
respectively and find SEx and SEy 2e 0
4. The resultant intensity is E = (SE x ) 2 + (SE y ) 2 (v) A finite length of charge with linear charge density l
SE y ur
and tan q = where q is the angle between E and +
SE x
+ Ey
x-axis. +
5. To find the force acting on the charge placed at the origin, + b
the formula F = qE is used. + x
+ a P
Energy density : Energy in unit volume of electric field is called Ex
1 +
energy density and is given by u = eo E 2 , +
2 +
where E = electric field and eo= permitivity of vacuum
Electric Field due to Various Charge Distribution : kl kl
(i) Electric Field due to an isolated point charge Ex = [sin a + sin b] and E y = [cos a - cos b]
x x
q x E
E=k
2
x +q P
EBD_7179
408 PHYSICS

Special case :
E' K'
=
p E K
For Infinite length of charge, a = b =
2 4. The electric field intensity at a point due to a ring with
uniform charge distribution doesn't depend upon the radius
2 kl
\ Ex = and E y = 0 of the ring if the distance between the point and the centre
x of the ring is much greater than the radius of the ring. The
(vi) Due to a spherical shell of uniformly distributed charges ring simply behaves as a point charge.
with surface charge density s 5. The electric field intensity inside a hollow sphere is zero
æ KQ ö
+

Q +
+

but has a finite value at the surface ç = and outside it


+ E è R 2 ÷ø
+

+ KQ
+ (= ; x being the distance of the point from the centre of
+ R x2
+ the sphere).
R x
+
+ 6. The electric field intensity at a point outside a hollow sphere
+

Ein = 0 (x < R) (or spherical shell) does not depend upon the radius of the
sphere. It just behaves as a point charge.
Q s
Esurface = k = (x = R) 7. The electric field intensity at the centre of a non-conducting
R 2 e0

.IN
solid sphere with uniform charge distribution is zero. At
other points inside it, the electric field varies directly with
Q
Eout = k the distance from the centre (i.e. E µ x; x being the distance
x2
ALof the point from the centre). On the surface, it is constant
(vii) Due to a solid non conducting sphere of uniformly
but varies inversely with the square of the distance from
distributed charges with charge density r
1
the centre (i.e. E µ 2 ). Note that the field doesn't depend
N
E
x
on the radius of the sphere for a point outside it. It simply
R

R behaves as a point charge.


U

8. The electric field intensity at a point on the axis of non-


conducting solid cylinder is zero. It varies directly with the
JO

x
Qx distance from the axis inside it (i.e. E µ x). On the surface, it
Ecentre = 0 Ein = k is constant and varies inversely with the distance from the
R3
U

Q Q 1
Esurface = k Eout =k 2 axis for a point outside it (i.e. E µ ).
2 x
R x
ED

(viii) Due to a solid non-conducting


cylinder with linear charge denisty l MOTION OF A CHARGED PARTICLE IN AN ELECTRIC
2l x FIELD
Eaxis = 0, E in = k 2 ,
R Let a charged particle of mass m and charge q be placed in an
2l 2l R uniform electric field E , then electric force on the charge particle
E surface = k , E out = k is
R x uur uur
F = qE
uur
ur qE
1 \ acceleration, a = (constant)
In above cases, k = m
4p Î0 (a) The velocity of the charged particle at time t is,
Keep in Memory qE
v = u + at = at = t (Particle initially at rest)
m
1. If the electric lines of force are parallel and equally spaced, qE
the field is uniform. or v = t
m
2. If E0 and E be the electric field intensity at a point due to a 1 1 qE 2
point charge or a charge distribution in vacuum and in a (b) Distance travelled by particle is S = at 2 = t
2 2 m
medium of dielectric constant K then
E = KE0 1 q 2 E 2t 2
3. If E and E' be the electric field intensity at a point in the two (c) Kinetic energy gained by particle, K = mv 2 =
2 2m
media having dielectric constant K and K' then
Electric Charges and Fields 409

If a charged particle is entering the electric field in perpendicular Example 8.


direction. Point charge q moves from point P to point S along the
Y path PQRS (as shown in fig.) in a uniform electric field
E pointing co-parallel to the positive direction of X-axis.
The coordinates of the points P, Q, R and S are (a, b, 0),
E (2 a, 0, 0), (a, –b, 0) and (0, 0, 0) respectively.
P (x, y) Y

O X P
U
E
S Q
Let E = E ˆj and the particle enters the field with speed u along
x-axis. R
qE
Acceleration along Y-axis, a y = The workdone by the field in the above case is given by
m
the expression
The initial component of velocity along y-axis is zero. Hence the
deflection of the particle along y-axis after time t is (a) q E A (b) – q E A
1 qE 2 1 2 2
y = u yt + . t = 0 + a yt2 ; (c) q E A 2 (d) q E [(2 a) + b ]

.IN
2 m 2
Solution : (b)
\y = .
1 qE 2
t …… (i) r r r r
2 m
AL W = F.S = q E i .S
Distance covered by particle in x-axis, r r r
x = ut …… (ii) (Q acceleration ax = 0) Now WP®Q= q E i · (a i + b j ) = q E a
Eliminating t from equation (i) & (ii), r r r
N
WQ®R = q E i · (– a i + b j ) = – q E a
qE x 2
y= . i.e. y µ x2. (workdone against field is taken as negative)
R

2m u 2 r r r
This shows that the path of charged particle in perpendicular WR®S = q E i · (– a i + b j ) = – q E a
U

field is parabola.
If the width of the region in which the electric field exists be l W = WP® Q + WQ + R + WR ®S
JO

then = q E a - q E a - q E a = -q E a
(i) the particle will leave the field at E vy
a distance from its original path q Example 9.
U

in the direction of field, given by vx Calculate the electric field strength required to just
y
support a water drop of mass 10–7 kg and having charge
ED

2
qE l u
y= . 1.6 × 10–19 C.
2m u 2 l
Solution :
(ii) The particle will leave the region in the direction of the
tangent drawn to the parabola at the point of escape. Here, m = 10–7 kg, q = 1.6 × 10–19 C
(iii) The velocity of the particle at the point of escape is given Step 1 : Let E be the electric field strength required to
by support the water drop.
v = v 2x + v 2y , where v x = u and Force acting on the water drop due to electric field E is
F = qE = 1.6 × 10–19 E
qE
vy = u y + a yt = 0 + t Weight of drop acting downward,
m
W = mg = 10–7 × 9.8 newton.
qEt qEl æ lö
= = çèQ t = ÷ø Step 2 : Drop will be supported if F and W are equal and
m mu u opposite.
æ qE l ö
2 i.e., 1.6 × 10–19 E = 9.8 × 10–7
\ v = u2 + ç ÷
è mu ø 9.8 ´ 10 -7
(iv) The direction of the particle in which it leaves the field is or E=
given by 1.6 ´ 10 -19
vy qEl qEl = 6.125 × 1012 N C–1.
tan q = = = Example 10.
vx mu.u mu 2
Can a metal sphere of radius 1cm hold a charge of
æ qE l ö
Þ q = tan -1 ç ÷ 1 coulomb.
è mu 2 ø
EBD_7179
410 PHYSICS

Solution : Electric Field due to an Electric Dipole


Electric field at the surface of the sphere. (a) Along the axial line (or end-on position)
KQ 9 ´ 109 ´ 1 V 2l
P
E= 2 = -2 2 = 9 × 1013 E
R (1 ´ 10 ) m –q +q
p x
This field is much greater than the dielectric strength of air
(3 × 106 v/m), the air near the sphere will get ionised and p and E are parallel
charge will leak out. Thus a sphere of radius 1 cm cannot
1 2 px
hold a charge of 1 coulomb in air. E ax = . when x >> l
4 p Î0 ( x 2 - l 2 )2
ELECTRIC DIPOLE
Two equal and opposite charges separated by a finite distance (b) Along equatorial line (or broad-side on position)
constitute an electric dipole. If –q and +q are charges at distance p
E
2l apart, then dipole moment, x
2l
p = q ´ 2l –q l l +q
–q +q
Its SI unit is coulomb metre. 1 p
Its direction is from –q to +q. It is a vector quantity. Eeq = . 2 when x >>l
4p Î0 ( x + l 2 )3 / 2
The torque t on a dipole in uniform electric field as shown in r
uur uur r

.IN
When p and E are anti parallel then,
figure is given by, t = qE ´ 2l sin q = p ´ E Eax = 2 Eeq
So t is maximum, when dipole is ^ to field & minimum (=0) when (c)
AL At any point (from the dipole)
dipole is parallel or antiparallel to field.
Ey E Ex
If p = p x î + p y ĵ + p z k̂ and E = E x î + E y ˆj + E z k̂ b
N
x p
+q
R

qE q
î ˆj k̂ 2l –q p +q
U

Then t = p x py pz
qE q
1 p 1
JO

Ex Ey Ez E= 3cos 2 q + 1 ; tan b = tan q


–q E
4p Î0 x 3 2
The work done in rotating the dipole from equilibrium through Electric field intensity due to a point charge varies inversely as
U

an angle dq is given by cube of the distance and in case of quadrupole it varies inversely
dW = td q = pE sin q d q as the fourth power of distance from the quadrupole.
ED

and from q1 ® q2, Electric Force between Two Dipoles


The electrostatic force between two dipoles of dipole moments
q2 q2
p1 and p2 lying at a seperation r is
W= ò dW = ò pE sin q dq 1 6 p1 p2
q1 q1 F= when dipoles are placed co-axially
4p Î0 r 4
If q1 = 0 i.e., equilibrium position, then
1 3 p1 p2
q2
F= when dipoles are placed perpendicular to
4p Î0 r 4
W= ò pE sin qdq = pE(1 - cos q2 ) each other.
0
Keep in Memory
Workdone in rotating an electric dipole in uniform electric field
from q1 to q2 is W = pE (cosq1 – cosq2) 1. The dipole moment of a dipole has a direction from the
negative charge to the positive charge.
Potential energy of an electric dipole in an electric field is,
2. If the separation between the charges of the dipole is
increased (or decreased) K-times, the dipole moment
U = – p .E i.e. U = –pE cosq
increases (or decreases) by K-times.
r r 3. The torque experienced by a dipole placed in a uniform
where q is the angle between E and p .
electric field has value always lying between zero and pE,
We can also write where p is the dipole moment and E, the uniform electric
U = p x E x + p y E y + pz E z field. It varies directly with the separation between the
charges of the dipole.
Electric Charges and Fields 411

4. The work done in rotating a dipole in a uniform electric field Since net force is zero, the centre of mass will not move but
varies from zero (minimum) to 2pE (maximum). Also , it varies the dipole will rotate about the centre of mass due to torque
directly with the separation between the charges of the t = qE (a / 4) sin q + qE (3a / 4) sin q
dipole.
t = q Ea sin q …… (1)
5. The potential energy of the dipole in a uniform electric field
always lies between +pE and –pE. d 2q
6. The electric field intensity at a point due to an electric dipole Also, t = Icma = – I cm …… (2)
dt 2
varies inversely with the cube of the distance of the point
from its centre if the distance is much greater than the length For small oscillations sin q = q
of the dipole. d 2q
7. The electric field at a point due to a small dipole in end-on Þ I cm = - qEa sin q = –qEaq;
position is double of its value in broad side-on position, dt 2
i.e. EEnd-on = 2EBroad side-on From eq. (1) & (2),
8. For a small dipole, the electric field tends from infinity at a
point very close to the axis of the dipole to zero at a point at d 2q æ qEa ö
= -ç q
infinity. dt2
è Icm ÷ø
9. The force between two dipoles increases (or decreases) by
K4 -times as the distance between them decreases (or æ aö
2
æ 3a ö
2

increases) by K-times. Icm = 3M ç ÷ + M ç ÷


è 4ø è 4ø

.IN
10. Time period of a dipole in uniform electric field is
3Ma 2 + 9Ma 2 12Ma 2 3
I = = = Ma 2
T = 2p AL 16 16 4
pE
where I = moment of inertia of the dipole about the axis of qEa 4qE 4qE
\ w2 = = or w =
rotation. 3 2 3Ma 3Ma
Ma
N
Example 11. 4
R

Calculate the electric field intensity due to a dipole of


length 10 cm and having a charge of 500 m C at a point on ELECTRIC FLUX
U

the axis distant 20 cm from one of the charges in air. Electric flux is a measure of the number of electric field lines
Solution : passing through the surface. If surface is not open & encloses
JO

Given : q = 500 × 10–6C, a = 10 cm or a/2 = 5 × 10–2 m, some net charge, then net number of lines that go through the
r = (20 + 5) cm = 25 × 10–2 m, surface is proportional to net charge within the surface.
p = q × a = (500 × 10–6) × (10 × 10–2) = 5 × 10–5 C-m For uniform electric field when the angle between area vector
U

The electric intensity on the axial line of the dipole is given ur ur


by ( ) ( )
A and electric field E has the same value throughout the
ED

1 2r p uur ur
E axial = ´ area, f = E × A Þ f = EA cos q
4p e o é 2ù
2
2 æaö
êr - ç ÷ ú
êë è 2 ø úû ^n
q E

9 2 ´ (25 ´ 10 -2 ) ´ (5 ´ 10 -5 )
Eaxial = (9 ´ 10 ) ´
10 -8 [(25) 2 - (5) 2 ]2
For uniform electric field when the angle between the area vector
= 3.25 ´ 10 7 NC -1 and electric field is not constant throughout the area
Example 12. n^
Consider two objects of masses M and 3M seperated by a E
distance l . Charge q and –q are placed on them dA
respectively and they are lying in an electric field E.
Find the angular frequency of oscillation (S.H.M.)
Solution :
(3M)
qE uur uur uur uur
d f = E × dA Þ f = ò E × dA Þ f = ò EdA cos q = E ò dA cos q
a/4
3Ma + 0 3 q
x cm = = a O
4M 4 3a Keep in Memory
4

qE 1. The electric flux is a scalar although it is a product of two


(M)
ur ur
(Q xcm is the position of centre of mass of system) vectors E and A (because it is a scalar product of the two).
EBD_7179
412 PHYSICS

2. The electric flux has values lying between –EA and +EA,
where E and A are the electric field and the area of cross- A
section of the surface. +q
–q
GAUSS'S LAW
It states that, the net electric flux through a closed surface in
vacuum is equal to 1/eo times the net charge enclosed within Q in
the surface. i.e., f = ò E. dS = = 0 because Qin = – q + q = 0
eo
r uur rr Qin
i.e., j = Ñ
ò E.dA = Ñò E.ndA = e o
Applications of Gauss’s Law :
s (i) To determine electric field due to a point charge :

where Qin represents the net charge inside the gaussian surface S.
. .. . . r
........................................ E
.. .................. ......... .... Q dA
.. ..dA.... .............. ..
n

.. . .. .. . .... . E
Gaussian
surface
........ ........... .. ...

.IN
.. ........ .. . . The point charge Q is at the centre of spherical surface
. .......... Gaussian
surface
AL shown in figure. Gaussian surface and E is parallel to d A
(direction normal to Gaussian surface ) at every point on
Closed surface of irregular shape which enclosed total charge Qin the Gaussian surface.
ur ur Q
In principle, Gauss's law can always be used to calculate the so, fc = Ñò E.dA = Ñò EdA = eo
N
electric field of a system of charges or a continuous distribution
R

of charge . But in practice it is useful only in a limited number of


( )
Q
situation, where there is a high degree of symmetry such as ÞE= Q A = 4pr 2
4pe o r 2
U

spherical, cylindrical etc.


(i) The net electric flux through any closed surface depends (ii) To determine electric field due to a cylindrically symmetric
JO

only on the charge inside that surface. In the figures, the charge distribution :
net flux through S is q1/eo, the net flux through S’ is (q2 +q3 We calculate the electric
)/eo and the net flux, through S" is zero. field at a distance r from a Gaussian
U

uniform positive line + surface


+
ED

charge of infinite length + E


q2 whose charge per unit l + dA
q3 length is l = constant. The +
q1 flux through the plane +
S" + E
S S' surfaces of the Gaussian +
(a) (b) (c) r
(ii) A point charge Q is located outside a closed surface S. In cylinder is zero, since E is dA
this case note that the number of lines entering the surface r
parallel to the plane of end surface ( E is perpendicular to
equals to the number of lines leaving the surface. In other r
words the net flux through a closed surface is zero, if there dA ). The total charge inside the Gaussian suface is l l,
is no charge inside. where l is linear charge density and l is the length of cylinder.

ld Now applying Gauss’s law and noting E is parallel to d A


c fie
tri s
ec line everywhere on cylindrical surface, we find that
El
ur ur
fc = Ñ
ò E.dA = (flux)both ends + (flux)cylindrical surface
Gaussian
Q surface

= 0 + E . 2A
S
Qin ll l
(iii) The net flux across surface A is zero 2E . p r l = = or E =
eo eo 2pe o r
Electric Charges and Fields 413

Keep in Memory

1. The closed imaginary surfaces drawn around a charge are –2q


called Gaussian surfaces.
2. If the flux emerging out of a Gaussian surface is zero then it +q
is not necessary that the intensity of electric field is zero.
3. In the Gauss's law, S
ur uuur Q Solution :
Ñò E. dA = Îin0 According to Gauss’s law, the net flux is
ur
E is the resultant electric field due to all charges lying 1 -2q
f= × net charge enclosed by closed surface =
inside or outside the Gaussian surface, but Qin is the charge eo eo
lying only inside the surface. (Because +q is outside the surface S, so net flux due to +q
4. The net flux of the electric field through a closed surface is zero)
due to all the charges lying inside or outside the surface is Example 15.
equal to the flux due to the charges only enclosed by the In which of the following figures, the electric flux is
surface. maximum?

.IN
5. The electric flux through any closed surface does not
depend on the dimensions of the surface but it depends
only on the net charge enclosed by the surface. q q
q
AL
Example 13.
A B C
N
A charge q is enclosed in a cube. What is the electric flux
Solution :
associated with one of the faces of cube?
R

According to Gauss’s law, the electric flux linked with a


Solution :
closed surface depends only on net charge enclosed by
U

According to Gauss’s theorem, that surface. It does not depend on the shape and size of
1 that closed surface. Hence electric flux linked in above three
JO

Total electric flux f= × total enclosed charge figures are same i.e., fA = fB = fC.
eo
Example 16.
1
= ´q.
U

A point charge +q is placed at mid point of a cube of side


eo ‘L’. What is the electric flux emerging from the cube ?
ED

Since cube has six faces, hence electric flux linked with Solution :
each face = (1/6f) = q/6eo.
According to Gauss’s law,
Example 14.
q
The following figure shows a surface S which is enclosing fnet closed surface =
eo
–2q charge. The charge +q is kept outside the surface S.
Calculate the net outward/ inward flux from the surface Net charge enclosed by closed surface
=
S. eo
CONCEPT MAP

414
Surface charge
charge
density =
area By induction
Coulomb’s Law
Linear charge Volume charge 1 q1q2
Distribution F= Methods of charging By conduction
charge charge 4p Î0 r 2 By friction
density = of charge density =
length volume 1
= 9 ´ 109 Nm2 C -2
4p Î0

Positive charge Conservation


ED
Deficiency of Quantization
Q = ± ne Neither created Invariant
electrons U nor destroyed

Neutral Charge Due to which matter


No of electrons Types of charge produces and experiences Basic properties of electric charge
= no. of protons electric and magnetic effects.
JO
Negative charge
Excess of electron
U Transferable can
be transfered from
Associated with mass
Mass of electron
one body to another –31
ELECTRIC CHARGES
R Me= 9.1 × 10kg
AND FIELDS N
Due to descrete
At axial position distribution of charge Torque
1 2P t = PE sin q Applicaion of Gauss’s
E= rˆ ur n r
4p Î0 r 3 E = å Ei Electric field Space surrounding t =P× E theorem due to infinitely
AL
i =1 long straight wire
a charge in which its electrostatic
Electric field Torque and potential l
force can be experienced by any E=
due to dipole energy of a dipole 2 pe0 r
.IN
F 1 q
Due to continuous test charge E = = × rˆ Infinite plane sheet
q 4p Î0 r 2
At equatorial distribution of charge Potential energy s
position 1 dq r E=
E= r U = –PE cos q 2e0
1 -p 4p Î0 ò r 3 U = – P.E.
E= .rˆ
4 p Î0 r 3
Never intersect Gauss’s theorem
each other Total flux over a
Electric field lines Imaginary closed surface is
Electric flux 1
Never form Properties of line along which a positive test times the total
closed loops electric field lines f = E.A cos q
charge will move if left free Î0 enclosed charg
f = E. A
qenclosed
Come out of Always normal f = ò E.ds =
e0
positive charge and to conducting
go into negative surface
PHYSICS

charge

EBD_7179
Electric Charges and Fields 415

1. The electric field strength at a distance r from a charge q is 10. Identify the wrong statement in the following. Coulomb's
E. What will be electric field strength if the distance of the law correctly describes the electric force that
observation point is increased to 2 r? (a) binds the electrons of an atom to its nucleus
(a) E/2 (b) E/4 (b) binds the protons and neutrons in the nucleus of an
(c) E/6 (d) None of these atom
2. The surface density on the copper sphere is s. The electric (c) binds atoms together to form molecules
field strength on the surface of the sphere is (d) binds atoms and molecules together to form solids
(a) s (b) s/2 11. An infinite parallel plane sheet of a metal is charged to
(c) Q / 2eo (d) Q / eo charge density s coulomb per square metre in a medium of
3. What is the angle between the electric dipole moment and dielectric constant K. Intensity of electric field near the
the electric field due to it on the axial line? metallic surface will be
(a) 0º (b) 90º s s
(a) E = (b) E =
(c) 180º (d) None of these eo K 2e o

.IN
4. In a region of space having a uniform electric field E, a
s Ks
hemispherical bowl of radius r is placed. The electric flux f (c) E= (d) E=
through the bowl is
AL 2e o K 2e o
(a) 2pRE (b) 4pR2E r
2
12. In a medium of dielectric constant K, the electric field is E .
(c) 2pR E (d) pR2E
If Î0 is permittivity of the free space, the electric
5. A cylinder of radius R and length l is placed in a uniform
N
electric field E parallel to the axis of the cylinder. The total displacement vector is
r r
R

flux over the curved surface of the cylinder is


KE E
(a) zero (b) pR2E (a) (b)
U

Î0 K Î0
(c) 2pR2E (d) E / pR2
r
JO

6. An electric dipole when placed in a uniform electric field E Î0 E r


will have minimum potential energy if the dipole moment (c) (d) K Î0 E
makes the following angle with E. K
13. Two conducting spheres of radii r 1 and r2 are charged to
U

(a) p (b) p/2


the same surface charge density. The ratio of electric fields
(c) zero (d) 3 p/2
ED

near their surface is


7. At the centre of a cubical box + Q charge is placed. The
value of total flux that is coming out a wall is r22 / r12
(a) r12 / r22 (b)
(a) Q / eo (b) Q / 3eo
(c) Q / 4eo (d) Q / 6eo (c) r1 / r2 (d) 1 : 1
8. If a charge is moved against the coulomb force of an electric 14. A charge q is placed at the centre of the open end of a
field, then cylindrical vessel. The flux of the electric field through the
(a) work is done by the electric field surface of the vessel is
(b) energy is used from some outside source
(c) the strength of the field is decreased q
(d) the energy of the system is decreased
9. The charge given to any conductor resides on its outer
surface, because
(a) the free charge tends to be in its minimum potential (a) zero (b) q/e o
energy state (c) q/2eo (d) 2q/eo
(b) the free charge tends to be in its minimum kinetic energy 15. Two thin infinite parallel sheets have uniform surface
state densities of charge + s and – s. Electric field in the space
(c) the free charge tends to be in its maximum potential between the two sheets is
energy state. (a) s/eo (b) s/2eo
(d) the free charge tends to be in its maximum kinetic energy (c) 2s/eo (d) zero
state
EBD_7179
416 PHYSICS

16. The E-r curve for an infinite linear charge distribution will 21. When air is replaced by a dielectric medium of force
be constant K, the maximum force of attraction between two
charges, separated by a distance
(a) decreases K-times (b) increases K-times
(a) E (b) E
1
(c) remains unchanged (d) becomes times
r r K2
22. A point Q lies on the perpendicular bisector of an electrical
dipole of dipole moment p. If the distance of Q from the
(c) E (d) E dipole is r (much larger than the size of the dipole), then
the electric field at Q is proportional to
r r (a) p–1 and r–2 (b) p and r–2
r 2 –3
17. If a dipole of dipole moment p is placed in a uniform electric (c) p and r (d) p and r–3
r 23. A particle of mass m and charge q is placed at rest in a
field E , then torque acting on it is given by
r rr r r r uniform electric field E and then released. The kinetic energy
(a) t = p.E (b) t = p ´ E attained by the particle after moving a distance y is
r r r r r r
(c) t = p + E (d) t = p - E (a) q E y2 (b) q E2 y
18. A charge Q is enclosed by a Gaussian spherical surface of

.IN
(c) q E y (d) q2 E y
radius R. If the radius is doubled, then the outward electric
24. Intensity of an electric field (E) depends on distance r, due
flux will
to a dipole, is related as
(a) increase four times (b) be reduced to half
(c) remain the same (d) be doubled
AL
1 1
19. What is the value of E in the space outside the sheets? (a) Eµ (b) Eµ
(a) s/eo (b) s/2eo r r2
N
(c) E ¹ 0 (d) 2s/eo
1 1
Eµ Eµ
R

20. Two charges are at a distance d apart. If a copper plate of (c) (d)
3
d r r4
thickness is kept between them, the effective force will
U

2 25. The formation of a dipole is due to two equal and dissimilar


be point charges placed at a
JO

(a) F/2 (b) zero (a) short distance (b) long distance
(c) 2F (d) 2F (c) above each other (d) none of these
U
ED

1. The electric field intensity just sufficient to balance the (a) 3 newton per coulomb
earth’s gravitational attraction on an electron will be: (given (b) 4 netwon per coulomb
mass and charge of an electron respectively are (c) 5 newton per coulomb
9.1 ´10-31 kg and 1.6 ´ 10-19 C .) (d) None of these
4. From a point charge, there is a fixed point A. At A, there is
(a) –5.6 × 10–11 N/C (b) –4.8 × 10–15 N / C an electric field of 500 V/m and potential difference of 3000
(c) –1.6 × 10–19 N/C (d) –3.2 × 10–19 N / C V. Distance between point charge and A will be
2. The insulation property of air breaks down when the electric (a) 6 m (b) 12 m
field is 3 × 106 Vm–1. The maximum charge that can be (c) 16 m (d) 24 m
given to a sphere of diameter 5 m is approximately 5. If electric field in a region is radially outward with magnitude
E = Ar, the charge contained in a sphere of radius r centred
(a) 2 × 10–2 C (b) 2 × 10–3 C at the origin is
(c) 2 × 10–4 C (d) 2 × 10–5 C
1
3. There is an electric field E in X-direction. If work done in (a) A r3 (b) 4peoAr3
4 p eo
moving a charge 0.2 C through a distance of 2m along a line
making an angle of 60 degree with X-axis is 4.0 joule, what 1 A 4 p eo A
(c) (d)
is the value of E? 4 p e o r3 r3
Electric Charges and Fields 417

6. A hollow insulated conduction sphere is given a positive 14. Two particle of equal mass m and charge q are placed at a
charge of 10 mC. What will be the electric field at the centre distance of 16 cm. They do not experience any force. The
of the sphere if its radius is 2 metres? q
(a) Zero (b) 5 mCm–2 value of is
m
(c) 20 mCm –2 (d) 8 mCm–2
7. An electric dipole has the magnitude of its charge as q and pe0
(a) l (b)
its dipole moment is p. It is placed in uniform electric field E. G
If its dipole moment is along the direction of the field, the G
force on it and its potential energy are respectively. (c) (d) 4pe0 G
4pe 0
(a) q.E and max. (b) 2 q.E and min.
(c) q.E and min (d) zero and min. 15. The spatial distribution of electric field due to charges
8. If the dipole of moment 2.57 × 10–17 cm is placed into an (A, B) is shown in figure. Which one of the following
electric field of magnitude 3.0 × 104 N/C such that the fields statements is correct ?
lines are aligned at 30° with the line joining P to the dipole,
what torque acts on the dipole?
(a) 7.7 × 10–13 Nm (b) 3.855 × 10–13 Nm A B
(c) 3.855 × 10 Nm –15 (d) 7.7 × 10–15 Nm
9. An electric dipole is placed at an angle of 30° with an electric
field of intensity 2 × 105 NC–1, It experiences a torque of (a) A is +ve and B –ve, |A| > |B|

.IN
4 Nm. Calculate the charge on the dipole if the dipole length (b) A is –ve and B +ve, |A| = |B|
is 2 cm. (c) Both are +ve but A > B
(a) 8 mC (b) 4 mC AL (d) Both are –ve but A > B
(c) 8 mC (d) 2 mC 16. A drop of oil of density r and radius r carries a charge q
10. Charge Q is distributed to two different metallic spheres when placed in an electric field E, it moves upwards with a
having radii R and 2R such that both spheres have equal velocity v. If r0 is the density of air, h be the viscosity of
N
surface charge density, then charge on large sphere is the air, then which of the following force is directed
upwards?
R

4Q Q
(a)
5
(b)
5 (a) q E (b) 6π ηr v
U

4 3
3Q 5Q (c) p r (r - r 0 )g (d) None of these
JO

(c) (d) 3
5 4 17. ABC is an equilateral triangle. Charges +q are placed at
11. Two point charges q1 = 4mC and q2 = 9 mC are placed 20 cm each corner as shown as fig. The electric intensity at centre
apart. The electric field due to them will be zero on the line
U

O will be
joining them at a distance of
+q A
ED

(a) 8 cm from q1 (b) 8 cm form q2


80 80
(c) cm from q1 (d) cm from q 2
13 13 r
12. Three charge q, Q and 4q are placed in a straight line of O
1 r r
length l at points distant 0, and l respectively from one
2 +q +q
end. In order to make the net froce on q zero, the charge Q B C
must be equal to 1 q 1 q
(a) –q (b) – 2q (a) (b)
4 p Îo r 4 p Îo r 2
-q
(c) (d) q 1 3q
2 (c) (d) zero
4 p Îo r 2
13. Among two discs A and B, first have radius 10 cm and charge
18. An electric dipole is placed along the x-axis at the origin O.
10–6 µC and second have radius 30 cm and charge 10–5µC.
A point P is at a distance of 20 cm from this origin such that
When they are touched, charge on both q A and q B
OP makes an angle p/3 with the x-axis. If the electric field at
respectively will, be
P makes an angle q with the x-axis, the value of q would be
(a) q A = 2.75 μC, q B =3.15 μC æ 3ö
p p
(a) (b) + tan -1 ç ÷
(b) q A = 1.09 μC, q B = 1.53μC 3 ç ÷
3 è 2 ø
(c) q A = q B = 5.5 μC æ 3ö
(d) tan -1çç ÷
2p
(c) ÷
(d) None of these 3 è 2 ø
EBD_7179
418 PHYSICS

19. A simple pendulum has a length l , mass of bob m. The bob 24. An electric dipole of moment P is placed in a uniform
is given a charge q. The pendulum is suspended between
the vertical plates of the charged parallel plate capacitor. If electric field E such that P points along E . If the dipole
E is the field strength between the plates, then time period is slightly rotated about an axis perpendicular to the plane
T equal to
containing E and P and passing through the centre of
+ -
the dipole, the dipole executes simple harmonic motion.
Consider I to be the moment of inertia of the dipole about
the axis of rotation. What is the time period of such
oscillation?
(a) (pE / I) (b) 2p ( I / pE)
+ -
(c) 2p ( I / 2pE) (d) None of these
25. There exists a non-uniform electric field along x-axis as
l l shown in the figure below. The field increases at a uniform
(a) 2p (b) 2p
g qE rate along +ve x-axis. A dipole is placed inside the field as
g+
m shown. Which one of the following is correct for the dipole?

l l

.IN
(c) 2p (d) 2p –q
qE 2 a
g- 2 æ qE ö
m g +ç ÷ +q x-axis
èmø
AL
20. Two identical thin rings, each of radius a meter, are coaxially
placed at a distance R meter apart. If Q1 coulomb and Q2 (a) Dipole moves along positive x-axis and undergoes a
N
coulomb are respectively the charges uniformly spread on clockwise rotation
the two rings, the work done in moving a charge q coulomb (b) Dipole moves along negative x-axis and undergoes a
R

from the centre of one ring to that of the other is clockwise rotation
(c) Dipole moves along positive x-axis and undergoes a
U

q(Q1 - Q2 )( 2 - 1) anticlockwise rotation


(a) Zero (b) (d) Dipole moves along negative x-axis and undergoes a
JO

4 2 pe0 a
anticlockwise rotation
q(Q1 + Q 2 )( 2 + 1) 26. The electric dipole is situated is an electric field as shown
q 2(Q1 + Q 2 )
(c) (d) in fig 1. The dipole and electric field are both in the plane of
U

4pe 0a 4 2pe 0 a the paper. The dipole is rotated about an axis perpendicular
ED

21. A metallic sphere is placed in a uniform electric field. The to plane of paper passing through A in anticlockwise
line of force follow the path (s) shown in the figure as direction. If the angle of rotation (q) is measured with respect
to the direction of electric field, then the torque (t)
1 1 experienced by the dipole for different values of the angle
2 2
of rotation q will be represented in fig. 2, by
3 3
4 4 +q

(a) 1 (b) 2 q E
(c) 3 (d) 4
22. A soap bubble is given negative charge, its radius will
(a) increase (b) decrease –q
(c) remain unchanged (d) fluctuate
23. A and B are two identically spherical charged bodies which Fig. 1
repel each other with force F, kept at a finite distance. A t (1) (2) (3) (4)
third uncharged sphere of the same size is brought in contact
with sphere B and removed. It is then kept at mid point of A
3p
and B. Find the magnitude of force on C. O p 2p
F F
(a) (b)
2 8 Fig. 2
(c) F (d) Zero (a) curve (1) (b) curve (2)
(c) curve (3) (d) curve (4)
Electric Charges and Fields 419

27. Force between two identical charges placed at a distance (a) For spherical region r £ R0 total electrostatic energy
of r in vacuum is F. Now a slab of dielectric of dielectric
stored is zero
contrant 4 is inserted between these two charges. If the
thickness of the slab is r/2, then the force between the (b) Within r = 2R0 total charge is q.
charges will become
(c) There will be no charge anywhere except at r = R0
3
(a) F (b) F (d) None of these
5
32. A solid sphereical conductor of radius R has a spherical
4 F cavity of radius a (a < R) at its centre. A charge + Q is kept
(c) F (d)
9 2 at the centre. The cahrge at the inner surface, outer and at a
28. A charge +q is at a distance L/2 above a square of side L. position r (a < r < R) are respectively
Then what is the flux linked with the surface? (a) + Q, – Q, 0 (b) – Q, + Q, 0
q 2q (c) 0, – Q, 0 (d) + Q , 0, 0
(a) 4e 0 (b) 3e 0 33. A glass rod rubbed with silk is used to charge a gold leaf
electroscope and the leaves are observed to diverge. The
q 6q
(c) (d) electroscope thus charged is exposed to X-rays for a short
6e 0 e0 period. Then
29. Positive and negative point charges of equal magnitude (a) the divergence of leaves will not be affected

.IN
æ aö -a ö (b) the leaves will diverge further
æ
are kept at ç 0, 0, ÷ and ç 0, 0, ÷ respectively. The work (c) the leaves will collapse
è 2 ø è 2 ø
AL (d) the leaves will melt
done by the electric field when another positive point charge
is moved from (–a, 0, 0) to (0, a, 0) is 34. A solid conducting sphere of radius a has a net positive
N
(a) positive charge 2Q. A conducting spherical shell of inner radius b
(b) negative and outer radius c is concentric with the solid sphere and
R

(c) zero has a net charge – Q. The surface charge density on the
(d) depends on the path connecting the initial and final inner and outer surfaces of the spherical shell will be
U

positions
r 2Q Q
JO

30. A short electric dipole of dipole moment p is placed at a (a) - 2


,
4pb 4pc 2
distance r from the centre of a solid metallic sphere of radius
a (<< r) as shown in the figure. The electric field intensity at a
Q Q
U

the centre of sphere C due to induced charge on the sphere (b) - , b


4pb2 4pc 2 c
is
ED

–q O +q Q
a (c) 0,
C 4pc 2
(d) None of these
35. An uncharged sphere of metal is placed in between charged
r plates as shown. The lines of force look like

1 2p +++++++ +++++++
(a) zero (b) along CO
4pe 0 r 3

1 2p 1 p
(c) along OC (d) along CO – –––– –– – –––– ––
4pe 0 r 3 4pe0 r3 A B
31. The electrostatic potential (fr) of a spherical symmetric
system kept at origin, is shown in the figure, and given as +++++++ +++++++
q
fr = (r ³ R0 ) fr
4pe 0 r
– –––– –– – –––– ––
q C D
fr = (r £ R0 )
4pe 0 R0 R0 r (a) A (b) B
Which of the following option(s) is/are incorrect (c) C (d) D
EBD_7179
420 PHYSICS

36. If a charge q is placed at the centre of the line joining two 42. Two metallic spheres of radii 1 cm and 3 cm are given
equal charges Q such that the system is in equilibrium then charges of –1×10–2 C and 5×10–2 C, respectively. If these
the value of q is are connected by a conducting wire, the final charge on the
(a) Q/2 (b) –Q/2 bigger sphere is
(a) 2 × 10–2 C (b) 3 × 10–2 C
(c) Q/4 (d) –Q/4 (c) 4 × 10 C –2 (d) 1 × 10–2 C
37. The electric intensity due to a dipole of length 10 cm and 43. An electric dipole of moment ´p´ is placed in an electric
having a charge of 500 mC, at a point on the axis at a distance field of intensity ´E´. The dipole acquires a position such
20 cm from one of the charges in air, is that the axis of the dipole makes an angle q with the direction
(a) 6.25 × 107 N/C (b) 9.28 × 107 N/C of the field. Assuming that the potential energy of the dipole
to be zero when = 90°, the torque and the potential energy
(c) 13.1 × 1011 N/C (d) 20.5 × 107 N/C
of the dipole will respectively be
38. Two positive ions, each carrying a charge q, are separated (a) p E sin q, – p E cos q (b) p E sin q, –2 p E cos q
by a distance d. If F is the force of repulsion between the (c) p E sin q, 2 p E cos q (d) p E cos q, – p E cos q
ions, the number of electrons missing from each ion will be 44. Figure shows two charges of equal magnitude separated by
(e being the charge of an electron) a distance 2a. As we move away from the charge situated at
x = 0 to the charge situated at x = 2a, which of the following
4pe 0 Fd 2 4pe0 Fe 2 graphs shows the correct behaviour of electric field ?
(a) (b) +q +q
e2 d2
x=0 x=a x = 2a

.IN
4pe0 Fd 2 4pe 0 Fd 2
(a) y (b) y
(c) (d) ®
e2 q2 ®
AL E E
39. A square surface of side L meter in the plane of the paper is
placed in a uniform electric field E (volt/m) acting along the
O x
same plane at an angle q with the horizontal side of the a O x
N
2a a 2a
square as shown in Figure. The electric flux linked to the
R

surface, in units of volt. m, is


U

E
y
(c) y
JO

(d)
q ®
® E
E
U

O x O x
a 2a a 2a
ED

(a) EL2 (b) EL2 cos q


(c) EL2 sin q (d) zero
40. The electric potential V at any point (x, y, z), all in meters in DIRECTIONS (for Qs. 45 to 50) : Each question contains
STATEMENT-1 and STATEMENT-2. Choose the correct answer
space is given by V = 4x2 volt. The electric field at the point
(ONLY ONE option is correct ) from the following-
(1, 0, 2) in volt/meter is
(a) Statement-1 is true, Statement-2 is true; Statement -2 is a
(a) 8 along positive X-axis (b) 16 along negative X-axis correct explanation for Statement-1
(c) 16 along positive X-axis (d) 8 along negative X-axis (b) Statement-1 is true, Statement-2 is true; Statement -2 is
41. Three charges, each +q, are placed at the corners of an NOT a correct explanation for Statement-1
isosceles triangle ABC of sides BC and AC, 2a. D and E are (c) Statement-1 is true, Statement-2 is false
the mid points of BC and CA. The work done in taking a (d) Statement-1 is false, Statement-2 is true
charge Q from D to E is 45. Statement-1 : Consider two identical charges placed
A distance 2d apart, along x-axis.
3qQ
(a) 8p Î0 a
E
qQ
(b) 4 p Î0 a
(c) zero B C
D The equilibrium of a positive test charge placed at the
3qQ point O midway between them is stable for displacements
(d) along the x-axis.
4 p Î0 a
Statement-2: Force on test charge is zero.
Electric Charges and Fields 421

46. Statement-1 : A deuteron and an a-particle are placed in an 48. Statement-1 : The positive charge particle is placed in front
electric field. If F1 and F2 be the forces acting on them and of a spherical uncharged conductor. The number of lines of
a1 and a2 be their accelerations respectively then, a1 = a2. forces terminating on the sphere will be more than those
Statement-2 : Forces will be same in electric field. emerging from it.
47. Statement-1 : Four point charges q1, q2, q3 and q4 are as Statement-2 : The surface charge density at a point on the
shown in figure. The flux over the shown Gaussian surface sphere nearest to the point charge will be negative and
depends only on charges q1 and q2. maximum in magnitude compared to other points on the
sphere.
49.. Statement-1 : A uniformly charged disc has a pin hole at its
centre. The electric field at the centre of the disc is zero.
Statement-2 : Disc can be supposed to be made up of many
rings. Also electric field at the centre of uniformly charged
ring is zero.
50. Statement-1 : When a conductor is placed in an external
electrostatic field, the net electric field inside the conductor
becomes zero after a small instant of time.
Statement-2 : Electric field at all points on Gaussian surface Statement-2 : It is not possible to set up an electric field
depends only on charges q1 and q2. inside a conductor.

.IN
AL
N
Exemplar Questions 3. The electric flux through the surface
R

1. In figure two positive charges q2 and q3 fixed along the y- s s


axis, exert a net electric force in the + x-direction on a charge
U

q1 fixed along the x-axis. If a positive charge Q is added at +q +q


(x, 0), the force on q1
JO

y y
(i) (ii) (i) (ii)
q2 q2
U

Q s
x x
ED

q1 q1 O (x, 0)

q3 q3 +q
+q
(a) shall increase along the positive x-axis
(b) shall decrease along the positive x-axis
(iii) (iv)
(c) shall point along the negative x-axis
(a) in Fig. (iv) is the largest
(d) shall increase but the direction changes because of
the intersection of Q with q2 and q3 (b) in Fig. (iii) is the least
2. A point positive charge is brought near an isolated conducting (c) in Fig. (ii) is same as Fig. (iii) but is smaller than Fig. (iv)
sphere (figure). The electric field is best given by (d) is the same for all the figures
4. Five charges q1, q2, q3,q4, and q5 are fixed at their positions
+q +q
as shown in Figure, S is a Gaussian surface. The Gauss’ law
+q +q q
is given by ò E.dS = . Which of the following statements
s e0
is correct?

q1
q2
q3
q5 q4
(a) (b) (c) (d)
EBD_7179
422 PHYSICS

(a) E on the LHS of the above equation will have a


contribution from q1, q5 and q1, q5 and q3 while q on
the RHS will have a contribution from q2 and q4 only
(b) E on the LHS of the above equation will have a y
contribution from all charges while q on the RHS will
y/2
have a contribution from q2 and q4 only
(c) E on the LHS of the above equation will have a
contribution from all charges while q on the RHS will æ r ö æ 2r ö
have a contribution from q1, q3 and q5. (a) ç3 ÷ (b) ç ÷
è 2ø è 3ø
(d) Both E on the LHS and q on the RHS will have
contributions from q2 and q4 only 2
æ 2r ö æ r ö
5. Figure shows electric field lines in which an electric dipole (c) ç 3 ÷ (d) ç ÷
P is placed as shown. Which of the following statements is è ø è 2ø
correct? 9. A charge ‘q’ is placed at the centre of the line joining two
equal charges ‘Q’. The system of the three charges will be
in equilibrium if ‘q’ is equal to [NEET Kar. 2013]
(a) Q/2 (b) – Q/4

.IN
–p p +p (c) Q/4 (d) – Q/2
10. An electric dipole of dipole moment p is aligned parallel to
a uniform electric field E. The energy required to rotate the
(a) The dipole will not experience any force
AL dipole by 90° is [NEET Kar. 2013]
(b) The dipole will experience a force towards right (a) pE 2 (b) p E2
N
(c) The dipole will experience a force towards left (c) pE (d) infinity
(d) The dipole will experience a force upwards 11. The electric field in a certain region is acting radially outward
R

and is given by E = Aa. A charge contained in a sphere of


6. A point charge + q is placed at a distance d from an isolated
radius 'a' centred at the origin of the field, will be given by
U

conducting plane. The field at a point P on the other side of


the plane is (a) A e0 a2 (b) 4 pe0 Aa3 [2015]
JO

(c) e0 Aa 3 (d) 4 pe0 Aa 2


(a) directed perpendicular to the plane and away from
12. Two identical charged spheres suspended from a common
the plane
point by two massless strings of lengths l, are initially at a
U

(b) directed perpendicular to the plane but towards the distance d (d << l ) apart because of their mutual repulsion.
plane The charges begin to leak from both the spheres at a
ED

(c) directed radially away from the point charge constant rate. As a result, the spheres approach each other
(d) directed radially towards the point charge. with a velocity v. Then v varies as a function of the distance
7. A hemisphere is uniformely charged positively. The electric x between the spheres, as : [2016]
field at a point on a diameter away from the centre is directed 1
(a) perpendicular to the diameter (a) vµ x2 (b) vµx
(b) parallel to the diameter 1
- -1
(c) at an angle tilted towards the diameter (c) v µ x 2 (d) v µ x
(d) at an angle tilted away from the diameter 13. Suppose the charge of a proton and an electron differ
slightly. One of them is – e, the other is (e + De). If the net of
NEET/AIPMT (2013-2017) Questions
electrostatic force and gravitational force between two
8. Two pith balls carrying equal charges are suspended from hydrogen atoms placed at a distance d (much greater than
a common point by strings of equal length. The equilibrium atomic size) apart is zero, then De is of the order of [Given
separation between them is r. Now the strings are rigidly mass of hydrogen mh = 1.67 × 10–27 kg] [2017]
clamped at half the height. The equilibrium separation (a) 10–23C (b) 10–37 C
between the balls now become [2013] (c) 10–47 C (d) 10–20 C
Electric Charges and Fields 423

Hints & Solutions


EXERCISE - 1 Q1 Q2
\ = Q1 Q2
2 or =
1. (b) As new distance = 2 r and electric field due to single 4 p r1 4 p r2 2 4 p e o r12 4 p e o r22
1
charge, E µ , \ E1 = E 2 or E1 / E 2 = 1 Þ E1 : E 2 = 1:1
2
r 14. (a) The flux is zero according to Gauss’ Law because it is
therefore, new intensity = E/4. a open surface which enclosed a charge q.
2. (d) According to Gauss's theorem,
s æ -s ö
15. (a) E = E1 - E 2 = -ç ÷ = s / eo
q / 4 pR 2 2 e o çè 2 e o ÷ø
\E = [Q q / 4pR 2 = s]
Î0 The field intensity in between sheets having equal
and opposite uniform surface densities of charge
or E = s / e o becomes constant. ie, an uniform electric field is
r produced and it is independent of the distance between

.IN
3. (a) Electric field E is along the dipole axis. the sheets.
\ q = 0º. 16. (c) The field due to infinite linear charge distribution

f = E (ds) cos q = E (2p r 2 ) cos 0º = 2p r 2 E . 1 dq


4. (c)
AL E= ò ÞEµ
1
So hyperbola .
4pe 0 r r
5. (a) For the curved surface, q = 90º
r
17. (b) Given : Dipole moment of the dipole = p and uniform
N
\ f = E ds cos90º = 0 .
®
R

U p = –p • E = –p E cos q electric field = E .Torque (t) = Either force ×


6. (c)
perpendicular distance between the two forces =
r r r
U

(Up)minimum = – pE, when q = 0°


qaEsinq or t = pE sin q or t = p ´ E (vector form)
7. (d) According to Gauss’ Law
JO

Qin
Q enclosed by closed surface 18. (c) By Gauss’s theorem, f =
Î0
ò E.ds = eo
= flux
Thus, the net flux depends only on the charge
U

so total flux = Q/eo enclosed by the surface. Hence, there will be no effect
ED

on the net flux if the radius of the surface is doubled.


Since cube has six face, so flux coming out through
19. (c) Inside a charged conducting surface E = 0, but on or
one wall or one face is Q/6eo.
outside the surface E ¹ 0.
8. (b) To overcome electrostatic repulsion forces, work will
\ Electric intensity is discontinuous across a charged
have to be done by external agency.
conducting surface.
9. (a) 20. (b) The dielectric constant for metal is infinity, the force
10. (b) Nuclear force binds the protons and neutrons in the between the two charges would be reduced to zero.
nucleus of an atom.
1 q1q2
11. (c) Electric field intensity due to thin infinite plane sheet 21. (a) In air, Fair =
4 pe 0 r 2
of charge,
s s 1 q1q2
E= = In medium, Fm =
2 e 2 e0K 4 pe 0 Kr 2

e Fm 1 F
[Where dielectric constant, K = ] \ = Þ Fm = air (decreases K-times)
e0 Fair K K

12. (d) Electric displacement vector, D = eE p


22. (d) E=
4 pe 0 .r 3
As, e = e 0 K \ D = e0K E
1
13. (d) As s1 = s 2 Apparently, E µ p and E µ µ r -3 .
r3
EBD_7179
424 PHYSICS

23. (c) K.E. acquired = work done


= force × distance = q E × y = q E y 8. (b) ( æ
è
Nö æ 1ö
t = 2.57 ´ 10-17 Cm ç 3.0 ´ 104 ÷ ç ÷
C ø è 2ø
)
24. (c) Intensity of electric field due to a Dipole
= 3.855 ´ 10 -13 Nm.
p 1 r r r
Eµ 3 9. (d) Torque, t = p ´ E = pE sin q
E=
4pe 0 r 3 3cos 2 q + 1 Þ r 4 = p × 2 × 105 × sin 30°
25. (a) Dipole is formed when two equal and unlike charges
4
are placed at a short distance. or, p = = 4 ´ 10-5 Cm
5
2 ´ 10 ´ sin 30°
EXERCISE - 2 Dipole moment, p = q × l
1. (a) – eE = mg
p 4 ´10-5
uur -31 q= = = 2 ´ 10-3 C = 2mC
9.1 ´ 10 ´ 10 -11 l 0.02
E =- = -5.6 ´10 N / C
1.6 ´ 10 -19 10. (a) Let q and q' be the charges on spheres of radii R and
2R respectively.
kQ E ´ r 2 3 ´ 106 ´ (2.5) 2 Given q + q' = Q …(i)
2. (b) E= ÞQ= =
r2 k 9 ´ 109 Surface charge densities are

.IN
= 2 ´ 10-3 C q q'
s= and s =
3. (d) Work done, W = F s cosq = (q E) s cos q 4pR 2 4p (2R) 2
AL
4. (a) Given : Electric field (E) = 500 V/m Given s = s'
and potential difference (V) = 3000 V.
q q'
N
We know that electric field \ = or, q' = 4q
2
4 pR 4 p(2R) 2
R

V 3000 From eq. (i), q' = Q – q or, 4q = Q – q


(E) = 500 = or d = = 6m
d 500
U

or, Q = 5q …(ii)
[where d = Distance between point charge and A]
Q 4Q
JO

q q \ q' = Q – q = Q - = .
5. (b) E = Þ Ar = Þ q = 4pe o Ar3 5 5
2
4 pe o r 4p Î0 r 2 11. (a) EP = 0
U

6. (a) Charge resides on the outer surface of a conducting


hollow sphere of radius R. We consider a spherical 4 9 20 - x 3
Þ = Þ =
ED

2 2 x 2
surface of radius r < R. x (20 - x)
By Gauss theorem Þ 40 – 2x = 3x Þ x = 8 cm
+++ +++
+
+ + 12. (a) (Fnet )q = 0
+
+ +
+ R
+ +
+ + Qq 4q 2
+
O S + Þ k +k =0
+ r + ælö
2
l2
+ E + ç ÷
+
+ +
+ è2ø
++ + +
+ +
l/2 l/2
rr 1 1
ò E.ds = ´ charge enclosed or E ´ 4pr 2 = ´0 q Q 4q
s e0 e0
ÞE = 0 1
where k =
i.e electric field inside a hollow sphere is zero. 4pe0
7. (d) When the dipole is in the direction of field then net Þ 4Qq + 4q2 = 0 Þ Q = – q
force is qE + (–qE) = 0
13. (c) The charge on disc A is 10–6 mC. The charge on
E
disc B is 10 × 10–6 mC. The total charge on both =
–q a q
11 mC. When touched, this charge will be distributed
and its potential energy is minimum = – p.E. equally i.e. 5.5 mC on each disc.
= – qaE
Electric Charges and Fields 425
r r Surface tension decreases after electrification of
14. (d) They will not experience any force if | FG |=| Fe |
æ 1ö
m2 1 q2 q bubble and therefore pressure decreases. ç P µ ÷
ÞG = . Þ = 4pe 0 G è rø
-2 2 -
4 pe0 (16 ´ 10 )2 2 m
(16 ´ 10 )
means radius increases. In equation (A ), q 2 is not
15. (a) Since lines of force starts from A and ends at B, so A is
+ve and B is –ve. Lines of forces are more crowded affected by positive or negative charge hence, whether
near A, so A > B. it is given a positive or a negative charge it always
expands in radius.
16. (a) Force due to electric field (F = q E) acts upwards.
23. (c) Initial force between the two spheres carrying charge
17. (d) Unit positive charge at O will be repelled equally by
(say q) is
three charges at the three corners of triangle.
r
By symmetry, resultant E at O would be zero. 1 q2
F= (r is the distance between them)
p 4pe 0 r 2
18. (b) From figure, q = + a , where
3 Further when an uncharged sphere is kept in touch
with the sphere of charge q, the net charge on both
E 2 æ p sin q ö æ 4p Î0 r 3 ö 1
tan a = =ç ÷ç ÷ = tan q q+0 q
E1 è 4 p Î0 r 3 ø è 2p cos q ø 2 become = . Force on the 3rd charge, when
2 2

.IN
æ pö 1 p 3 placed at center of the 1st two
tan ç q - ÷ = tan =
è 3 ø 2 3 2 AL r/2 r/2

p æ 3ö
or q- = tan -1 ç ÷ 1 3 2
3 è 2 ø q q/2 q/2
N
æ 3ö
or a = tan -1 ç ÷
R

ç 2 ÷ æqö æqö
2
è ø qç ÷ ç ÷
1 è2ø - 1 è2ø
U

æ 3ö F3 =
p 4pe 0 æ r ö 2 4pe 0 æ r ö 2
or q = + tan -1ç ÷
JO

3 ç 2 ÷ ç ÷ ç ÷
è ø è2ø è2ø

E E
U

a 1 1 q2 1 q2
E2 p/3 = [2 - 1] = =F
4pe 0 r 2 4pe 0 r 2
ED

P
24. (b) The dipole experiences a torque pE sin q tending to
p /3 bring itself back in the direction of field.
–q O
+q Therefore, on being released (i.e. rotated) the dipole
oscillates about an axis through its centre of mass and
19. (d) 20. (b) 21. (d)
perpendicular to the field. If I is the moment of inertia
22. (a) For a soap bubble (electrified) of the dipole about the axis of rotation, then the
4T equation of motion is I.d 2 q / dt 2 = -pE sin q
excess pressure = P = Pi - P0 =
r
T = surface tension, r = radius. For small amplitude sin q » q
Force due to excess pressure balances surface tension. Thus d 2 q / dt 2 = -(pE / I).q = -w 2 q
When bubble is charged
4T where w = ( pE / I) .
Pexcess = Pelectrosta tic +
r This is a S.H.M., whose period of oscillation is
s2 1 q2 T = 2p / w = 2p ( I / pE) .
Pelectrostatic = - =-
2e 0 2e 0 16p 2 r 4 25. (d) The dipole is placed in a non-uniform field, therefore a
force as well as a couple acts on it. The force on the
4é q2 ù negative charge is more (F µ E) and is directed along
Pexcess = êT - ú ....... ( A ) negative x-axis. Thus the dipole moves along negative
r ëê 128p 2 e 0 r 3 ûú
x-axis and rotates in an anticlockwise direction.
EBD_7179
426 PHYSICS

E 1q 30. (b) a
–q O
–q q C
a
E

+q E2q
r
Electric field at C due to electric dipole
26. (b) t = pE sin q , this is given by the second curve.
1 2p
= along OC
1 q2 4pe 0 r 3
27. (c) In vacuum, F = …(i)
4pe0 r 2
Electric field at C due to induced charge must be equal
Suppose, froce between the chrages is same when and opposite to electric field due to dipole as net field
charges are r¢ distance apart in dielectric. at C is zero.
31. (a, b, c, d)
1 q2 The potential shown is for charged spherical
\ F' = …(ii)
4pe0 kr '2 conductor.
From (i) and (ii), kr'2 = r2 or, r = kr ' 32. (b) The charge at the inner surface, outer surface and
inside the conductor at P = (– Q, + Q, 0) as shown in

.IN
In the given situation, force between the charges
the figure
would be
+Q +
q2 4 q2
1 4F
AL + r
F' = = = +
4pe0 æ r rö
2 9 4pe0 r 2 9 –Q P
ç + 4 ÷ + +
è2 2ø
N
+Q

q +
R

+
L/2 +
U

q 33. (b) Charge on glass rod is positive, so charge on gold


L L/2 leaves will also be positive. Due to X-rays, more
JO

28. (c)
electrons from leaves will be emitted, so leaves
L L
becomes more positive and diverge further.
L
U

The given square of side L may be considered as one Charge


of the faces of a cube with edge L. Then given charge 34. (a) Surface charge density (s) =
Surface area
ED

q will be considered to be placed at the centre of the


– Q + 2Q = Q
cube. Then according to Gauss's theorem, the -2Q
magnitude of the electric flux through the faces (six) So sinner = –2Q
4pb2
of the cube is given by a b
f = q/e0 Q
and sOuter = c
Hence, electric flux through one face of the cube for +2Q
4pc 2
the given square will be
1 q
f' = f = 35. (c) Electric lines of force never intersect the conductor.
6 6e 0
They are perpendicular and slightly curved near the
29. (c) The charges make an electric dipole. A and B points surface of conductor.
lie on the equitorial plane of the dipole. There fore 36. (d) Let q charge is situated at the mid position of the line
potential at A = potential at B = 0 AB. The distance between AB is x. A and B be the
W = Q (VA – VB) = q × 0 = 0 positions of charges Q and Q respectively.
Y A C B
B (0, a, 0) –Z Q Q
–q x q x
(0, 0,–a/2) 2 2
A X x
(–a, 0, 0) +q
(0, 0, a/2) x x
Let AC = , BC =
Z 2 2
Electric Charges and Fields 427

The force on A due to charge q at C, 41. (c) AC = BC qA


® 1 Q.q VD = VE
F CA = . along AC
4pe 0 ( x / 2) 2 We have, E

The force on A due to charge Q at B W = Q (VE – VD) q


q
ÞW=0 B D C
® 1 Q2 ®
F AB = . 2 along BA 42. (b) At equilibrium potential of both sphere becomes same
4pe0 x if charge of sp here one x and other sphere
The system is in equilibrium, then two oppositely Q – x then
directed force must be equal, i.e., total force on A is where Q = 4 × 10–2 C
equal to zero. v1 = v2
® ® ® ® kx k (Q - x)
F CA + F AB = 0 Þ F CA = - F AB =
1cm 3cm
3x = Q – x Þ 4x = Q
1 4Q.q –1 Q 2 Q
. 2 = . Þq=-
4 pe 0 x 4 pe 0 x 2 4 Q 4 ´ 10-2
x= = C = 1 ´ 10-2
4 4

.IN
37. (a) Given : Length of the dipole (2l) = 10cm = 0.1m or
l = 0.05 m Q¢ = Q – x = 3 × 10– 2C
43. (a) The torque on the dipole is given as
Charge on the dipole (q) = 500 mC = 500 × 10–6 C and
t = PE sin q
distance of the point on the axis from the mid-point of
AL
the dipole (r) = 20 + 5 = 25 cm = 0.25 m. The potential energy of the dipole in the electric field
is given as
We know that the electric field intensity due to dipole
N
U = – PE cos q
on the given point (E)
44. (a) For the distances close to the charge at x = 0 the field
R

1 2(q.2l )r is very high and is in positive direction of x-axis. As


= ´ we move towards the other charge the net electric
U

4pe 0 (r 2 - l 2 ) 2
field becomes zero at x = a thereafter the influence of
JO

charge at x = 2a dominates and net field increases in


2(500 ´ 10 -6 ´ 0.1) ´ 0.25 negative direction of x-axis and grows unboundedly
= 9 ´109 ´
[(0.25) 2 - (0.05) 2 ]2 as we come closer and closer to the charge at x = 2a.
U

45. (b) If +ve charge is displaced along x-axis, then net force
3 will always act in a direction opposite to that of
225 ´ 10
ED

= = 6.25 ´ 10 7 N / C (k = 1 for air) displacement and the test charge will always come
-3
3.6 ´ 10 back to its original position.
38. (c) Let n be the number of electrons missing. 46. (c) qd = e, md = 2mp = 2m
qa = 2e, ma = 4mp = 4m
1 q2 F1 = Fa = eE, F2 = Fa = 2eE ¹ F1
F= × 2
4pe 0 d 2 Þ q = 4pe 0 d F = ne F1 eE
Further, a1 = =
2m 2m
4pe 0 Fd 2 F2 2eE eE
\ n= and a 2 = = = = a1
e2 2m 4m 2m
47. (c) Electric field at any point depends on presence of all
39. (d) Electric flux, f = EA cos q , where q
charges.
= angle between E and normal to the surface. 48. (d) No. of lines entering the surface = No. of lines leaving
p the surface.
Here q =
2 49. (a) The electric field due to disc is superposition of electric
field due to its constituent ring as given in statement-
Þ f=0 2. Statement-1 is true, statement-2 is true, statement-2
r é dv dv dv ˆ ù is a correct explanation for statement-1.
40. (d) E = - ê iˆ + ˆj + k ú = –8 ´ iˆ volt/meter 50. (c) Statement-1 is correct. The induced field cancels the
ë dx dy dz û
external field. Statement-2 is false. When a current is
r
\ E (1,0,2) = -8iˆ V / m set up in a conductor, there exists an electric field inside
it.
EBD_7179
428 PHYSICS

EXERCISE - 3 3. (d) By Gauss’s law : The total of the electric flux out of a
closed surface is equal to the charge enclosed devided
Exemplar Questions
Q
1. (a) The force on q1 depend on the force acting between by the permittivity i.e., f = .
e0
q1 and q2 and q1 and q3 so that the net force acting
on q1 by q2 and q1 by q3 is along the + x-direction, so Thus, electric flux through a surface doesn’t depend
the force acting between q1, q2 and q1, q3 is attractive on the shape, size or area of a surface but it depends
force as shown in figure : on the number of charges enclosed by the surface. So
+q2
all the given figures have same electric flux as all of
them also has same single positive charge.
q1 x 4. (b) Gauss's law states that total electric flux of an enclosed
+q1 q
surface is given by, Ñò E.dS = , includes the sum of
+q3 s
e0

The attractive force between these charges states that all charges enclosed by the surface.
q1 is a negative charge (since, q2 and q3 are positive). The charges may be located anywhere inside the
Then the force acting between q 1 and charge Q surface, and out side the surface. Then, the electric
(positive) is also know as attractive force and then the

.IN
field on the left side of equation is due to all the charges,
net force on q1 by q2, q3 and Q are along the same both inside and outside S.
direction as shown in the figure.
So, E on LHS of the above equation will have a
AL contribution from all charges while q on the RHS will
+q2
have a contribution from q2 and q4 only.
N
+Q 5. (c) The electric field lines, are directed away from
x
–q1 (x, 0) positively charged source and directed toward
R

+q3 negatively charged source. In electric field force are


U

directly proportional to the electric field strength hence,


The figure shows that the force on q1 shall increase higher the electric field strength greater the force and
JO

along the positive x-axis due to the positive charge Q. vice-versa.


2. (a) If a positive point charge is brought near an isolated The space between the electric field lines is increasing,
conducting sphere without touching the sphere, then
U

from left to right so strength of electric field decreases


the free electrons in the sphere are attracted towards with the increase in the space between electric field
ED

the positive Charge and electric field passes through


lines. Then the force on charges also decreases from
a charged body. This leaves an excess of positive
charge on the (right) surface of sphere due to the left to right.
induction process. Thus, the force on charge – q is greater than force on
Both type of charges are bound in the (isolated charge +q in turn dipole will experience a force towards
conducting) sphere and cannot escape. They, left.
therefore, reside on the surface. 6. (a) When a positive point charge +q is placed near an
Thus, the left surface of sphere has an excess of isolated conducting plane, some negative charge
negative charge and the right surface of sphere has developes on the surface of the plane towards the
an excess of positive charge as shown in figure.
charge and an equal positive charge developes on
attracted negative opposite side of the plane. This is called induction
charge process and the electric field on a isolated conducting
+
+ plane at point is directly projected in a plane
+
+q + perpendicular to the field and away from the plane.
+
7. (a) Consider a point on diameter away from the centre of
An electric field lines start from positive charge and
ends at negative charge. hemisphere uniformly positively charged, then the
electric field is perpendicular to the diameter and the
Also, electric field line emerges from a positive charge,
in case of single charge and ends at infinity shown in component of electric intensity parallel to the diameter
figure (a). cancel out.
Electric Charges and Fields 429

NEET/AIPMT (2013-2017) Questions 11. (b) Net flux emmited from a spherical surface of radius a
according to Gauss’s theorem
8. (a) q
fnet = in
e0 qin
or, (Aa) (4pa2) = e
0
So, qin = 4pe0 A a3
Fe O
2
12. (c) From figure tan q = mg ; q
kq
Fe r/2 q
2 l
From figure, tan q = Þ = r kq 2 x
mg y
mg =
x mg 2l
2
q q
kq 2 or x3 µ q2 …(1)
[Q F = from coulomb’s law] x
r2 or x3/2 µ q …(2)
Differentiating eq. (1) w.r.t. time
y r' 1
Þ r3 µ y Þ r'3 µ Þ = 1/3 dx dq dq
2 r 2 3x2 µ 2q but is constant
dt dt dt

.IN
r so x2(v) µ q Replace q from eq. (2)
Þ r' = 3
2 x2(v) µ x3/2 or v µ x–1/2
9. (b) The system of three charges will be in equilibrium. 13. (b) According to question, the net electrostatic force (FE)
q
AL = gravitational force (FG)
A B
r O r FE = FG
Q Q
N
For this, force between charge at A and B + force 1 De 2 Gm2
between charge at point O and either at A or B is zero. or =
4pe0 d 2 d2
R

KQ 2 KQq
U

i.e., + =0 G æ 1 9ö
r 2
(r / 2) 2 Þ De = m ç 4pe = k = 9 ´10 ÷
K è ø
JO

0
By solving we get,

q= -
Q 6.67 ´ 10-11
. = 1.67 × 10–27
U

4 9 ´ 109
10. (c) When electric dipole is aligned parallel
ED

De » 1.436 × 10–37 C
q = 0° and the dipole is rotated by 90° i.e.,
q = 90°.
Energy required to rotate the dipole
W = Uf – Ui = (–pE cos 90°) – (–pE cos 0°)
= pE.
EBD_7179
430 PHYSICS

Electrostatic Potential
17 and Capacitance
ELECTROSTATIC POTENTIAL
Electric potential at a point in an electric field is defined as the r
amount of work done in bringing a unit positive test charge P
from infinity to that point along any arbitrary path. (Infinity is

.IN
taken as point of zero potential). It is denoted by V ; dq
R
work done in bringing unit positive
W test charge from infinity to some point
V= =
AL dq dq
q0 unit positive test charge i.e. V = ò dV = ò where dV =
4pe o r 4pe o r
Its SI unit is JC–1 or volt. It is a scalar quantity.
N
Potential due to a System of Charges
Also, electric potential at any point in an electric field is defined The electric potential due to a system of charges q 1, q2, ...qn is
R

r V = V1 + V2 + ... + Vn
as the negative line integral of the electric field vector E from a
U

point infinitely away from all charges to that point 1 æ q1 q2 q ö 1 qi


= .ç +
4pe è r1 r2
+ ... + n ÷ =
rn ø 4pe
å
JO

r uur uur ri
i.e. V = - ò E .dr P +lC where ri is the point from charge qi and e is the permitivity of
¥
medium in which the charges are situated.
U

Potential due to a Point Charge Potential at any point P due to a point chage q at a distance
(r1 + r2) where r1 is the thickness of medium of dielectric constant
ED

The electric potential due to a point charge q at separation r is


given by x1 and r 2 is the thickness of the medium of dielectric constant k2
r1 r2
1 q
V= .
4pe r
q K1 P
(Please note that we have to write q with its sign in this formula) K2
4F potential difference between two points is the work done in
bringing unit positive charge from one point to another. q 1
V =k where k =
r1 K1 + r2 K2 4 p Î0
Q B A
Relation between electric field and potential
rB
The relation between electric field (E) and potential (V) is
rA
dV
E=-
B
-Q æ l l ö dr
VAB = VB – VA = - ò E.dr = . ç - ÷ J/C For 3-D we can write
4peo è rB rA ø
A
¶V ¶V ¶V
Ex = - , Ey = - and E z = -
Electric Potential due to Continuous Charge ¶x ¶y ¶z
Distribution So electric field is equal to negative potential gradient.
The potential due to a continuous charge distribution is the sum In this relation negative sign indicates that in the direction of
of potentials of all the infinitesimal charge elements in which the electric field, potential decreases. Consider two points A and B
distribution may be divided. situated in a uniform electric field at a distance d then,
Electrostatic Potential and Capacitance 431

If A is at infinity then at infinity since potential is zero


d we assume infinity as reference point,VA = 0
A B E B

The potential difference between A and B is Þ VB = - q E.dlò


¥
VAB = Ed
Potential energy of a system of two charges Q 1 and Q 2 is,
Conservative nature of electric field
Q1 Q2
The electric field is conservative in nature. In figure the work,
WAB has the same value whatever path is taken in moving the r
test charge. 1 Q1 Q2
U=
4p Î0 r 2
Terminal
point [Please note that in this formula we have to write charges with
Qo B
2 sign]
Vb
Potential energy of a system of three charges Q1, Q2 and Q3
Q1
Qo
1

A r3 r1

.IN
Starting
point
Q3 Q2
W
AL r2
so, VAB = VB - VA = AB 1 é Q1 Q 2 Q 2 Q3 Q 3 Q1 ù
Q U= ê + + ú
4p Î0 ë r1 r2 r3 û
N
has the same value for any path between A and B and V B and VA
are unique for the points A and B. Keep in Memory
R

We cannot find the absolute value of potential 1. For an assembly of n charges [Total number of intersection
U

therefore conventionally, we take infinity as the point of zero n n (n - 1)


potential. If need arises, we can assume any point to be the point C2 = ] the potential energy is
JO

2
of zero potential and find the potential of other points on this
basis. é ù
1ê q iq j ú
n

å
U

U = êk ú
POTENTIAL ENERGY OF A SYSTEM OF CHARGES 2 ê i , j rij ú
ED

Potential energy can be defined only for those forces, which are êë i ¹ j úû
conservative, such as gravitational and electrostatic forces. The 2. For a system of two charges.
potential energy of a charge between two points is defined as
If Usystem = –ve, then there is net force of attraction between
the amount of work done in bringing the charge from one point
the charges of the system.
to another.
If Usystem = +ve, then there is net force of repulsion between
B
Wext the charges of the system
i.e. VB - VA =
q ò
= - E.dr
A
Usystem = max for unstable equilibrium
Usystem = min for stable equilibrium

A dU
Also F = - =0
dx
1 3. The energy required to take away the charges of a dipole at
2
dl q2
infinite distance U = k
2l
4. The work done when a charge q is moved across a potential
Q difference of V volt is given by W = qV
B 5. When one electronic charge (1.6×10–19 coulomb i.e., charge
of electron) is moved across one volt the work done is
Calculation of external work done against the field and a point charge called one electron volt (eV). Thus
Q in moving a test charge q from A to B. For a conservative field the 1eV = (1 volt) × (1.6×10–19 coulomb) = 1.6×10–19 joule.
work done by any path is same. The sectional force is – qE.
EBD_7179
432 PHYSICS

EQUIPOTENTIAL SURFACE (iv) A finite length of charge with linear charge density l
It is that surface where the potential at any point of the surface
has the same value. The electric lines of force and the x
+ + + + + + + æ x + Lö
equipotential surface are mutually perpendicular to each other. V = k l loge ç
No work is done in moving a charge from one point to other on an L P è x ÷ø
equipotential surface. Work is done in moving a charge from one
equipotential surface to another. (v) Due to a spherical shell of uniformly distributed charge
with surface charge density s

V1 Q ++ + +
V1 > V2 r1 + +
V2 V +
+
o +q +
+ +
r2 R
+ +
x=R x + +
+ + +
Spherical equipotential surface
for point charge
Q Q Q
Vin = k , Vsurface = k , Vout = k
R R x

.IN
V = V2
V = V1 (vi) Due to a solid sphere of uniformly distributed charge with
AL volume charge density r.

Q
V
N
R
R

Plane equipotential surface for uniform field


x=R x
· Equipotential surface do not cut each other.
U

· The density of the equipotential lines gives an idea of the 3Q kQ(3R2 - r 2 )


Vcentre = k , Vin =
JO

strength of electric field at that point. Higher the density, 2R 2 R3


larger is the field strength.
Potential Due to Various Charge Distribution Q Q
Vsurface = k , Vout = k
U

(i) Electric potential due to isolated point charge R x


ED

Potential due to Electric dipole


x ( + q) (a) Along axial line :
V =k
+q x
P x
(ii) A circular ring of radius R with uniformly distributed charge Q –q +q
2l P
Q
++ + +
+ p
+ + Vaxial = k
+ R + kQ x - l2
2
+ + V=
+ + x P R2 + x2 kp
+ + when x > > l Vaxial =
+
+ + ++ x2
(b) Along equatorial line :
· Potential V does not depend on the way of charge
distriubution on the ring (uniform / non-uniform). Veq = zero
(iii) A circular disc of radius R with uniformly distributed charge (c) At any point from the dipole :
with surface charge density s P
Q x
+ + p cos q q
+ R ++ s é 2 V =k
+ V= R + x2 - xù ( x 2 - l 2 cos 2 q) –q +q
+ +++ x P 2e 0 êë úû
++ p
Electrostatic Potential and Capacitance 433

Keep in Memory Solution : (d)


1. Electric field inside a charged conductor is zero As electric field represents the direction of motion of positive
charge, which is from higher potential to lower potential,
+ + +
+ + + + + therefore, from fig, we find VA = VB and VA > VC
+ + + +
E = 0 + +
+ in + + Ein = 0 Y
+ + +
+ + + + + + + E E
Spherical conductor Irregular shaped conductor
C
But in both the cases the potential at all the points of the
surface will remain the same. But charges will have same
X
distribution on spherical conductor and in case of r A B
irregularly shaped conductor the charge distribution will E
be non-uniform. At sharp points, charge density has Z
greatest value.
2. Electronic lines of force are always perpendicular to the Example 2
equipotential surfaces. A uniform electric field pointing in positive x-direction
3. The work done in moving a charge from a point to the exists in a region. Let A be the origin, B be the point on the

.IN
other on an equipotential surface is zero as the potential x-axis at x = + 1 cm and C be the point on the y-axis at
difference between the two points is zero. y = + 1 cm. Then the potential at the points A, B and C
4. The electric potential at a point due to a point charge satisfy
decreases (or increases) by K-times if the distance between
AL
(a) VA < VB (b) VA > VB
the charge and the point increases (or decreases) by (c) VA < VC (d) VA > VC
K-times.
N
Solution : (b)
5. A ring with a charge distribution behaves as a point charge uur
R

for the points very far from its centre. As E is directed along +ve direction of X-axis, therefore,
6. The electric potential is constant inside a hollow charged VA > VB
U

sphere and it is also equal to its value on the surface but it C is vertically above A. Therefore, VA = VC.
JO

varies inversely with the distance outside the sphere.


7. The electric potential at points inside a solid sphere has a V
non-zero value and decreases as we go from the centre
U

outwards. It behaves as a point charge for the points C


outside the sphere. E
ED

8. The electric potential at a point due to a dipole varies


directly with the dipole moment.
X
A B
COMMON DEFAULT
Incorrect : Where electric field is zero, electric potential is
also zero. Example 3
Correct : It is not always correct, for example in a charged Calculate the maximum voltage upto which a sphere of
conducting shell, electric field inside the shell E = 0 but radius 2 cm can be charged in air under normal conditions,
potential is not zero. assuming that maximum electric intensity in air can be
3 × 106 volt/m. Also, find the charge required to be given
Incorrect : Where electric potential is zero, electric field is
to the sphere.
also zero.
Solution :
Correct : It is not always correct. In the case of equitorial We know that
plane of an electric dipole the electric potential is zero but
Electric Potential (V)
the electric field is non-zero. Electric intensity (E) =
Distance (r)
Example 1
A uniform electric field pointing in positive y-direction Þ V = Er …… (1)
Given, E = 3 × 106 volt/m, r = 2 cm = 2 × 10–2 m.
exists in a region. Let A be the origin, B be the point on the
Substituting the above values in eq. (1), we get
x-axis at x = 2 cm and C be the point on the y-axis at y = 2
V = (3 × 106 volt/m) × 2 × 10–2 m = 6 × 104 volt.
cm. Then the potential at the points A, B and C satisfy:
Þ V = 60 kV
(a) VA < VB (b) VA > VB
Also, we know that the electric intensity on the surface of
(c) VA < VC (d) VA > VC a charged sphere is given by
EBD_7179
434 PHYSICS

1 Q The S.I. unit of capacitance is coulomb per volt or farad (F).


E= . ; Q = charge on the sphere Further more, the value of capacitance depends on size, shape,
4p Î0 r 2
relative positions of plate, and the medium between the plates.
Q The value of C does not depend on the charge of the plate or p.d.
Þ 3 ´ 10 6 = (9 ´ 10 9 ) ´ between the plates.
( 2 ´ 10 - 2 ) 2
ENERGY STORED IN A CAPACITOR
Þ Q = 1.33 ´ 10 -7 coulomb. If Q is charge, V is p.d, C is the capacitance of the capacitor then
ELECTROSTATICS OF CONDUCTORS 1 1 Q2
Conductor is a substance that can be used to carry or conduct the energy stored is U = CV 2 = QV =
2 2 2C
electric charges. Metals like silver. Copper, aluminium etc. are
good conductors of electricity. Sharing of Charges
Regarding electrostatics of conductors following points are worth When the two charged conductors of capacitances C1 and C2 at
noting. potentials V1 and V2 respectively, are connected by a conducting
(i) Inside a conductor, electric field is zero. wire, the charge flows from higher to lower potential, until the
(ii) The interior of a conductor can have no excess charge in potentials of the two conductors are equal.
static situation.
(iii) Electric field at the surface of a charged conductor is V1 V2
r s C1 C2
E= nˆ
Î0

.IN
where, s = surface charge density Q1' Q2'
The common potential after sharing of charges,
n̂ = unit vector normal to the surface in the outward
Net charge Q + Q 2 C1V1 + C 2 V2
direction.
AL
V= = 1 =
(iv) Electric field just outside a charged conductor is Net capacitance C1 + C 2 C1 + C 2
perpendicular to the surface of the conductor at every The charges after sharing on two conductors will be
N
point.
Q1 C1 Q1¢
(v) Electrostatic potential is constant throughout the volume Q1' = C1V and Q '2 = C 2 V i.e., = =
Q 2 C2 Q¢2
R

of the conductor and has the same value as on its surface.


(vi) Surface density of charge is different at different points. There is a loss of energy during sharing, converted to heat given
U

CAPACITORS AND CAPACITANCE by


JO

A capacitor or condenser is a device that stores electrical é1 2 1 2ù 1 2


energy. It generally consists of two conductors carrying equal DU = Uinitial - Ufinal = ê C1V1 + C 2 V2 ú - (C1 + C 2 )V
ë2 2 û 2
but opposite charges.
C1C2 (V1 - V2 )2
U

The ability of a capacitor to hold a charge is measured by a


or, Du =
quantity called the capacitance. Let us consider two uncharged 2(C1 + C2 )
ED

identical conductors X and Y and create a P.D. (Potential


Difference) V between them by connecting with battery B as PARALLEL PLATE CAPACITOR
shown in figure. It consists of two parallel metallic plates of any shape, each of
area A and at a distance d apart.
+Q –Q
Y
X The capacitance of the capacitor is given by C = e 0 A
A d
Effect of Dielectric on Capacitance
+qp
d When a dielectric slab is placed +
between the plates of a parallel plate – + –
+
B capacitor, the charge induced on its – E + –
+ 0
– + –
Fig- A capacitor consists of electrically insulated conductors carrying plates due to polarisation of dielectric +
– Ep + –
equal positive and negative charge + –
After connection with the battery, the two conductors X and Y æ 1ö
is q p = Qç1 - ÷ + + –
have equal but opposite charges. Such a combination of charged è K ø + – E = E 0 – E p+ –

conductors is a device called a capacitor. The P.D. between X where K = dielectric constant. + Q – qp –Q
and Y is found to be proportional to the charge Q on capacitor.
The capacitance C, of a capacitor is defined as the ratio of the When an electric field is applied across a dielectric, induced
magnitude of the charge on either conductor to the magnitude charges appear on the surface of dielectric which is shown in the
of P.D. between them. above figure. These induced charges produce their own field
which acts in the opposite direction of the applied field. Hence,
Q
Vµ Q Þ = constant = (Capacitance). total field is reduced, i.e., E 0 - E p = E , where E0 is the applied
V
Capacitance is always a positive quantity. field, Ep is the induced field and E is the resultant field.
Electrostatic Potential and Capacitance 435

E0 By introducing a slab of thickness t, the new capacitance


E is given by , where K is the dielectric constant. C´ becomes
K
If medium between the plates is having a dielectric of dielectric e0 A
C' = ... (2)
æ K eo A ö d'- t (1 - 1/K)
constant K then the capacitance is given C = ç ÷
è d ø The charge (Q = CV) remains the same in both the cases.
If the space between the plates is partly filled with dielectric Hence
then the capacitance of the capacitor will be given by, eo A e oA æ 1ö
t = or d = d'- t ç 1 - ÷
d d'- t (1 - 1/K) è Kø
P1 + – P2
+ – d ' = d + 2.4 ´ 10-3 m, t = 3 mm = 3 × 10–3 m.
+ K – Substituting these values, we have
+ – æ 1ö
A + – d = d + ( 2.4 ´ 10 -3 ) - 3 ´ 10 -3 ç1 - ÷
+ – è Kø
+ – æ 1ö
or ( 2.4 ´ 10 -3 ) = 3 ´ 10 -3 ç1 - ÷
è Kø
d Solving it, we get K = 5.
SPHERICAL CAPACITOR B
e0 A e0 A

.IN
C= = , It consists of two concentric
t æ 1ö ++
d -t + d - t ç1 - ÷ spherical conductors of radii R1 ++ ++
K è Kø R2 A

++ ++

++ ++
and R2 . The space between two
AL

++

++
where t is the thickness of the dielectric with dielectric constant K.
conductors is filled by a dielectric R O
1

of dielectric constant K.
++ ++
Keep in Memory ++
N
1. The unit farad is quite a big unit for practical purposes. (a) When outer conductor is earthed,
R

Even the capacitance of a huge body like earth is 711 mF. Capacitance of spherical capacitor,
2. A capacitor is a device which stores charges and produces 4p Î0 R1R2
U

electricity whenever required. C= (without dielectric)


R2 - R1
JO

3. If the two plates of a capacitor is connected with a


(4pe o K ) R1R2
conducting wire, sparking takes place which shows that C= (with dielectric)
electrical energy is converted into heat and light energy. ( R2 - R1 )
(b) When inner sphere is earthed,
U

4. A capacitor allows A.C. but doesn’t allow D.C. to pass


through it. 4pe o K R1R2
C= + 4pe o R2
ED

5. The capacitance of a capacitor increases with insertion of a R2 - R1


dielectric between its plates and decreases with increase in This is because the combination behaves as two capacitors
the separation between the plates. in parallel, one is a capacitor formed by two concentric
6. The capacitance of a capacitor increases K times if a medium spherical shells and the other is an isolated spherical shell
of dielectric constant K is inserted between its plates. of radius R2.
7. The energy of a capacitor for a particular separation between CYLINDRICAL CAPACITOR
the plates is the amount of work done in separating the two
It consists of two-coaxial cylindrical conductors of radii R1 and R2,
plates to that separation if they are made to touch to each
the outer surface of outer conductor being earthed. The space
other.
between the two is filled with a dielectric of dielectric constant K.
8. The loss of energy when the two charged conductors are
connected by a wire doesn’t depend on the length of the R1
wire.
Example 4
–+ +–
A parallel plate capacitor is maintained at a certain potential –+ +–
difference. When a 3 mm slab is introduced between the plates, –+ A +–
K B
l –+ +–
in order to maintain the same potential difference, the distance –+ +–
between the plates is increased by 2.4 mm. Find the dielectric –+ +–
–+ +–
constant of the slab. –+ +–
Solution : –+ +–
The capacity of a parallel plate capacitor in air is given by
e A
C= 0 ... (1) R2
d
EBD_7179
436 PHYSICS

The capacitance of cylindrical condenser of length l t1 t2


2p Î0 l
C= (without dielectric)
æR ö
log e ç 2 ÷
è R1 ø + K1 K2

2p Îo K l
C=
æR ö
log e ç 2 ÷ (with dielectric)
è R1 ø
2. When a number of dielectric slabs of same thickness (d)
COMBINATION OF CAPACITORS and different areas of cross-section A1, A2, A3 ... having
Series Combination dielectric constants K1, K2, K3, .... respectively are placed
(i) In this combination, the positive plate of one capacitor is between the plates of a parallel plate capacitor then the
connected to the negative plate of the other. capacitance is given by
+
Q Q Q Electric slabs which fill

Area A1
Area A2
Area A3
the whole space of parallel
A B plate capacitor
C1 C2 C3
d –
V1 V2 V3 e o (K 1 A 1 + K 2 A 2 + K 3 A 3 + ....)

.IN
C=
V d
(ii) The charges of individual capacitor are equal. 3. When five capacitors are connected in wheatstone bridge
(iii) The potential difference is shared by the capacitors in the
AL C1 C3
inverse ratio of their capacities arrangement as shown, such that = , the bridge is
i.e. Q = C1V1 = C2 V2 = C3 V3 C2 C4
N
Hence V = V1 + V2 + V3 balanced and C5 becomes ineffective. No charge is stored
on C5. Therefore C1, C2 and C3, C4 are in series. The two
(iv) The equivalent capacitance (C) between A and B is
R

series combinations are in parallel between A and C. Hence


1 1 1 1 1 equivalent capacitance can be calculated.
U

= + + + ..... + B
C C1 C 2 C 3 Cn
JO

Parallel Combination C1 C2
q1
(i) In this arrangement, +ve plates + – c1
of all the condensers are +–
U

A C5 C
connected to one point and q2
negative plates of all the A c
ED

+– 2 B
condensers are connected to Q + – C3 C4
the other point. q3
(ii) The Potential difference +– c D
across the individual capacitor +– 3
is same.
V RELATION BETWEEN THREE ELECTRIC VECTORS
(iii) The total charge shared by the individual capacitor is in
direct ratio of their capacities
ur ur r
D , P AND E
q q q
i.e. V = 1 = 2 = 3 If an electric field E is applied across a parallel plate capacitor
C1 C 2 C3 filled with a dielectric of dielectric constant K (or permittivity e),
Hence, Q = q1 + q2 + q3 then
(iv) The equivalent capacitance between A and B is Polarisation P = induced charge per unit area (opposite to free
Ceq = C1 + C2 + C3 + ........+ Cn q'
charge) =
Keep in Memory A
Electric displacement D = eE = eo E + P
1. The capacitance of a parallel plate capacitor having a
i.e. Polarisation P = (e – eo) E = (Keo – eo) E
number of slabs of thickness t1, t2, t3 .... and dielectric
constant K1, K2, K3 .... respectively between the plates is Electric susceptibility, c e = P / E
Relation between dielectric constant K and electric susceptibility
eo A ce is
C=
æ t1 t t ö
çç + 2 + 3 + .....÷÷ ce
K K K K = 1+
è 1 2 3 ø e0
Electrostatic Potential and Capacitance 437

Effect of filling dielectric With battery connected current in the circuit.


When there is no dielectric 4. As the charge on the capacitor increases, more energy is
stored in the capacitor.
Ae 0
Capacitance C 0 = 5. When the capacitor is fully charged, potential difference
d
across the capacitor is equal to the potential difference of
Potential difference between the plates V the source and the transient current tends to zero.
Charge on a plate Q = CV
If V0 = constant potential difference of the source
1 R = pure resistance in the circuit
Energy E 0 = C0V 2
2 C = capacity of the capacitor
V V Q0 = final charge on the capacitor, when fully charged
Electric field E 0 = q = charge on the capacitor at time ‘t’ from the starting of
d
the charging
When dielectric is inserted
V = potential difference across the capacitor at time ‘t’
KA e 0
C= = KC 0 Source V0 Key
d
Q = K C0 V = KQ0 V
·
( )

1
U= KC 0 V 2 = KE 0

.IN
2
V R C
E= = E0
d Q0 q
AL Then = =C
Effect of filling a dielectric in a capacitor after V0 V
disconnection of battery dq
N
and i = current in the circuit at time ‘t’ =
+Q –Q dt
+Q –Q
R

At time ‘t’ by Kirchhoff’s law


V dq q dq 1
U

V0 - R - =0 i.e. = dt
Capacitance C0 C = K C0 dt C CV0 - q CR
JO

Integrating and putting in the initial condition q = 0 at t = 0,


Charge Q0 Q = Q0 we get
é 1 ù
U

Q0 Q 0 V0 – t
P.D V0 = V= = q = CV0 ê1 – e CR ú
C0 C K ê ú
ED

ë û
1 2 2
1 V U0
Potential energy U0 = C0V0 U= KC 0 0 = Special cases :
2 2 K K
(i) At t = 0, q = 0.
(ii) When t increases, q increases.
CHARGING AND DISCHARGING A CAPACITOR
é -
1 ù
t
Charging a Capacitor (iii) As t ® ¥ , q ® Q = CV \ q = Q ê 1 - e CR ú
0 0 0
When an uncharged capacitor is connected across a source of ëê ûú
constant potential difference such as a cell, it takes a finite time (iv) At t = CR [‘CR’ has dimensions of time]
to get fully charged, although this time interval may be small.
é 1ù
This time-interval depends on the capacity of the capacitor and q = Q 0 ê1 - ú = 0.631Q 0
the resistance in the circuit. ë eû
This value of t = CR is called the ‘time constant’ of the (CR)
During the period of charging :
circuit.
1. The charge on the capacitor increases from ‘zero’ to the
final steady charge. Discharging of a Capacitor
2. The potential difference developed across the capacitor If after charging the capacitor, the source of constant potential
opposes the constant potential difference of the source. difference is disconnected and the charged capacitor is shorted
through a resistance ‘R’, then by Kirchhoff’s law, at time ‘t’ from
3. The charge on the capacitor ‘grows’ only as long as the
the instant of shorting,
potential difference of source is greater than the potential
difference across the capacitor. This transport of the charge q dq
+R =0
from the source to the capacitor constitutes a transient C dt
EBD_7179
438 PHYSICS

Putting, In equilibrium, Pi + Pe = PO + PT
(i) the initial condition, q = Q0 at t = 0 and or, Pi – PO = PT – Pe
(ii) the final condition, q = 0 at t ® ¥ , or, Pexcess = PT – Pe
the solution to the above equation is
4T s 2
\ Pexcess = -
Q– 0 r 2e o
– 1 t Where T = surface tension of soap solution,
q = Q 0 e CR q s = surface charge density of bubble.
If Pi = PO then Pi – PO = PT – Pe = 0 or PT = Pe
2
t 4T s 2 1 æ q ö
= = ç ÷
Keep in Memory r 2e o 2e o çè 4 pr 2 ÷ø
Hence for maintaining the equilibrium of charged soap
1. If n small drops each having a charge q, capacity ‘C’ and
8e o T
potential V coalesc to form a big drop, then bubble, s =
(i) the charge on the big drop = nq r
(ii) capacity of big drop = n1/3 C q = 8p r 2e o rT
(iii) potential of big drop = n2/3 V 3. Force of attraction between the plates of a parallel plate
(iv) potential energy of big drop = n5/3 U

.IN
(v) surface density of charge on the big drop = n1/3 × q2
capacitor =
surface density of charge on one small drop. 2e o AK
2. Charged soap bubble : Four types of pressure act on a where, A = area of the plates of capacitor and
charged soap bubble.
AL K = dielectric constant of the medium filled between the
(i) Pressure due to air outside the bubble PO, acting plates.
inwards. In terms of electric field, the force of attraction
N
(ii) Pressure due to surface tension of soap solution PT, 1
F = e 0 KE 2
R

acting inwards.
2
(iii) Pressure due to air inside the bubble, P i, acting 4. Uses of capacitor :
U

outwards. = In LC oscillators = As filter circuits


JO

s2 = Tuner circuit in radio etc.


(iv) Electric pressure due to charging, Pe = , acting 5. The total energy stored in an array of capacitors
2e o
(in series or in parallel) is the sum of the individual energies
outwards.
U

stored in each capacitor.


ED

COMBINATION OF CAPACITOR : EQUIVALENT CAPACITANCE


Ae 0 Ae
+

C1 = K1 C1 = K1 0 1 1 1
K1 K1 d/2 d/2 = +
(i) d º Ae 0 Ae 0 Ceq C1 C 2
K2 K2 C2 = K 2 C2 = K 2
d/2 d/2
– –
é d d ù
êë d = 2 + 2 and area of each plate = A úû

++
+ æA ö
K K1 Kç æ Aæç ÷Aöeeö÷0e æA ö
1 ç12 ÷ 0 0
K2ç ÷e 0
(ii) d K1 K2 K2
Cº1 C=C1 1== è è è2 ø2ø ø C 2 = è2 ø
; C eq= C +1 C 2
dd d d
– ––
+
+ K 2 (A / 2)e 0
C2 =
K2 K (A / 2)e 0 d/ 2 C 2 C3
(iii) K1 º C1 = 1 Ceq = C1 +
d d K 3 (A / 2)e 0 C 2 C3
K3 C3 =
d/ 2
– –
Electrostatic Potential and Capacitance 439
+ Ae 0
C1 = K1
+ d/2
K1 1 1 1
d K 2e 0 (A / 2) K 3e 0 (A / 2) = +
(iv) K2 K3 = º C2 = C3 = Ceq C1 C 2 + C 3
d/2 d/2


C
1 2
1
+ 2 A+ 3C 2 –B
(v) ºA º

3 B
C
3 4
4
Ceq = C + C + C = 3C
+– +– +– +– 1 C
2 1 2

.IN
– A + –
(vi) A Bº
= =º A
B B
C
AL 3
4 4 3
+ + + +
N
Ceq = C + C = 2C
R

2 3
C
U

(+) A A + –
– – – –1 º B
A
JO

+ + + + C C
+ + + +2
(vii) º 2 1 4 3
U

– – – –3
– – – – 1 1 3C
= C+ or, C eq =
1 1
ED

+ + + + C eq 2
(–) B 4 +
B C C
SOME ME THODS OF FINDING EQUIVALENT Method 2 : Using Symmetry
CAPACITANCE C

Method 1 : Successive Reduction C


C
This method is applicable only when the capacitor can be clearly C
identified as in series or in parallel. X C E C Y
A B
2 mF 2 mF
A A C
C C
2 mmF 2 mF 2 mF 2 mF
2 mF 2 mF A
D
2 mF Þ The above circuit is symmetrical about XAEBY axis. This is
2 mF 2 mF 4 mF because the upper part of the circuit is mirror image of lower part.
B B Therefore VC = VE = VD. The circuit can be redrawn as
C C
Þ

C
2
C C
2 mF C
C C
A X Y = 2
2 mF 3 mF A B A B
A B Þ
A B Þ
C C C
26 13 mF 4
mF mF 2
19 3 3 D DC
B
2
EBD_7179
440 PHYSICS

3C Principle : When a charged conductor is placed in contact with


2 the inside of a hollow conductor, all of the charge of first
= conductor is transferred to the hollow conductor. i.e., the charge
A B
Method 3 : Wheatstone bridge on hollow conductor or its potential can be increased by any
C limit by repeating that processes.
The basic fact of Van de Graaf generator is described in fig.
C1 (Charge is delivered continuously to a high voltage electrode on
C2
a moving belt of insulating material).

B
A C5 High voltage electrode

B
C4 +
C3

D Belt
C1 C 2
If C = C then the wheatstone bridge is balanced. In this case
3 4
there will be no charge accumulation in C5 when battery is

.IN
Insulator
attached across A and B. Therefore the equivalent circuit is the
A
capacitance C1 and C2 are in series. Similarly C3 and C4 is in
series. Therefore the equivalent capacitance occurs between A
AL
and B is
1 1 1 Grounded grid
= +
N
C eq C 1 +C 2 C 3 + C 4
R

Schematic diagram of a Van de Graaf generator. Charge is transferred


The other forms of wheatstone bridge are : to hollow conductor at the top by means of a rotating belt. The charge is
C1
U

deposited on the belt at point A and is transferred to hollow conductor


C2 B at point B.
JO

A C5 The high voltage electrode is a hollow conductor mounted on an


C3
insulating medium. The belt is charged at A by means of corona
C4
discharge between comb-like metallic needles and a grounded
or
U

grid. The needles are maintained at a positive potential of typically


104 eV. The positive charge on the moving belt is transferred to
ED

C5 C1 the high voltage electrode by second comb of needles at B.


C2
Since the electric field inside the hollow conductor is negligible,
C4 the positive charge on the belt easily transfers to the high- voltage
electrode, regardless of its potential. We can increase the potential
B A
of the high voltage electrode until electrical discharge occur
C3 through the air . The “ breakdown” voltage of air is about 3 × 10 6 V/m.
Method 4 : If none of the above method works, then we can use Example 5
the method of Kirchhoff’s laws - junction law and loop law.
Obtain equivalent capacitance of the following network
SHARP POINT ACTION (CORONA DISCHARGE)
as shown in fig. For a 300 volt supply, determine the
When the electric field (s / e 0 ) on a point on the surface of a charge and voltage across each capacitor.
conductor exceeds the electric strength of air, then the air becomes
conducting and the surface of conductor loses charge. This action C1
occurs usually at the sharp points of a conductor as here s is
high, thus creating high electric field. This phenomenon is also C2 100 pF C3
called corona discharge.
VAN DE GRAAF GENERATOR 200 pF 200 pF +
300 V
C4
R.J. Van de Graff in 1931 designed an electrostatic generator –
capable of generating very high potential of the order of 5 × 106 V,
which was then made use of an accelerating charged particles so 100 pF
as to carry out nuclear reactions.
Electrostatic Potential and Capacitance 441

Solution :
As it is clear from fig, C2 and C3 are in series. total ch arg e q 20 p R 2 s 5 Rs
V= = = =
total capacity C 12 p e o R 3 eo
1 1 1 1 1
\ = + = + Now, charge on bigger sphere,
Cs C2 C3 200 200
5 Rs 40 p R 2 s
2 1 q2' = C2 V = 4 p e o (2 R ) ´ =
= = Þ Cs = 100pF 3e o 3
200 100
Now, Cs and C1 are in parallel. \ Surface density of charge
Cp = Cs + C1 = 100 + 100 = 200 pF.
q '2 40 p R 2 s 5
Again, Cp and C4 are in series. Their combined capacitance s '2 = = = s.
surface area 3 (4 p) (2 R ) 2 6
C is
1 1 1 1 1 3 Example 7
= + = + =
C C p C 4 200 100 200 Two insulated metal spheres of radii 10 cm and 15 cm
charged to a potential of 150 V and 100 V respectively,
200 are connected by means of a metallic wire. What is the
C= = 66.7 pF = 66.7 ´ 10 -12 F
3 charge on the first sphere?

.IN
As Cp and C4 are in series. Solution :
\ Vp + V4 = 300 volt. Here, r1 = 10 cm, r2 = 15cm
Charge on C4, V1 = 150 V, V2 = 100 V
AL
200 Common potential
q 4 = CV = ´ 10 -12 ´ 300 = 2 ´ 10 -8 C.
3
N
Potential difference across C4: C1V1 + C 2 V2
V=
C1 + C 2
R

q4 2 ´10 -8
V4 = = = 200 volt. 4 p Î0 (r1 V1 + r2 V2 )
C 4 100 ´10 -12
U

=
4 p Î0 (r1 + r2 )
JO

V p = 300 - V 4 = 300 - 200 = 100 volt . = 120 volt


Potential difference across C1 = V1 = Vp = 100 volt.
Charge on
U

C1, q1 = C1 V1 = 100 × 10–12 × 100 = 10–8 C q1 = C1V


Potential difference across C2 and C3 in series = 100 volt
ED

V2 = V3 = 50 volt = 4 p Î0 r1 V
Charge on 10-1
C2 = q2 = C2V2 = 200 × 10–12 × 50 = 10–8 C = ´ 12C
9 ´ 109
Charge on
12
C3 = q3 = C3V3 = 200 × 10–12 × 50 = 10–8 C = ´ 3 ´ 109 esu = 4esu
9 ´ 109
Example 6
Two isolated metallic solid spheres of radii R and 2 R are Example 8
charged such that both of these have same charge density Consider a parallel plate capacitor of capacity 10 mF
s. The spheres are located far away from each other, and with air filled in the gap between the plates. Now one half
connected by a thin conducting wire. Find the new charge of the space between the plates is filled with a dielectric of
density on the bigger sphere. dielectric constant K = 4 as shown in fig.
Solution :
Charge on smaller sphere, q1 = 4pR2s
Charge on bigger sphere, q2 = 4p(2R)2s = 1pR2s K=4
\ Total charge, q = q1 + q2 = 20pR2s
Combined capacity of two spheres,
C = C1 + C2 = 4peoR + 4peo (2R) = 12peoR
The capacity of the capacitor changes to
After contact, charge is exchanged and a common potential (a) 25 mF (b) 20 mF
V is reached. (c) 40 mF (d) 5 mF
EBD_7179
442 PHYSICS

Solution : (a) As C1, C2 are in series,


The arrangement is equivalent to three capacitors in parallel
1 1 1 C + C1
e A / 4 10 \ = + = 2 ;
C1 = o = = 2.5 mF ; Cs C1 C 2 C1 C 2
d 4
C1 C 2 2 K1 C 0 2 K 0 C 0 2 K1 K 2 C 0
Ke o A / 2 10 \ Cs = = =
C2 = = 4´ = 20 mF ; C1 + C 2 2 C0 (K1 + K 2 ) K1 + K 2
d 2
e A / 4 10 Cs 2 K1 K 2
C3 = o = = 2.5 mF =
d 2 C 0 K1 + K 2
\ Ceq = C1 + C2 + C3 = 2.5 + 20 + 2.5 = 25mF Example 10
A capacitor of capacity 1 mF is connected in closed series
Example 9 circuit with a resistance of 107 W, an open key and a cell
A parallel plate capacitor is filled with dielectric as shown of 2 V with negligible internal resistance.
in fig. Its capacitance has ratio with that and without of (i) When the key is switched on at a time t = 0, find :
dielectric as (a) the time constant for the circuit.
(b) the charge on the capacitor at steady state.
+
(c) time taken to deposit charge equalling half that
at steady state.

.IN
K2 (ii) If after fully charging the capacitor, the cell is shorted
d by zero resistance at time t = 0, find the charge on the
d
K1 capacitor at t = 50 s.
2
AL
– Solution :
(i) (a) Time constant = RC = (107) (10–6) = 10 s.
N
(b) Q0 = charge on capacitor at steady state
æ K1 + K 2 ö = V0C = 2 × 10–6 = 2 mC
R

(a) (K1 + K2) (b) çç K – K ÷÷ 1


è 1 2 ø 1 - t
(c) q = Q0 = Q0 [1 - e CR ]
U

2
æ 2K 1 K 2 ö æ K1 +K 2 ö
JO

(c) çç K + K ÷÷ (d) çç K K ÷÷ t
è 1 2 ø è 1 2 ø Þ e 10 = 2 or, t = 10 ´ 2.306 ´ log10 2 = 6.94 s.
Solution : (c) 1
U

- t
eo A (ii) q = Q0 e CR

Without dielectric, C 0 =
ED

d 50
- æ 1ö
With dielectric as shown, = (2 ´ 10-6 )(e 10 ) = (2 ´ 10-6 ) ç 5 ÷
èe ø
K1e o A K 2e o A
C1 = = 2K1C 0 ; C2 = = 2K 2C 0 ; = 1.348 × 10–8 C (Q e = 2.718)
d/2 d/2
CONCEPT MAP

At a point outside On axial line Equipotential surface


the spherical shell 1 p Imaginary surface joining
V= On equatorial line
1 q 4p e 0 r 2 V=0 the points of same potential
V= (r > R) in an electric field
4p Î0 r

Electrostatic potential
At a point on the surface Electric potential due to a point charge
Electric potential due to a dipole q
or inside the spherical shell due to a charged V= K
1 q conducting spherical r
ED
V= (r £ R)
Electrostatic Potential and Capacitance

4p Î0 R shell Electrostatic potential


U (V)0 = work done w ¥
charge (q0)
Electric potential due to a Electrostatic potential
JO
charged non-conducting due to a system of charges
sphere V= V1 + V2 + V3 ... + Vn
ELECTROSTATIC
U AND CAPACITANCE
n
q
R V = Kå i
i =1 ri
At a point on the surface At a point outside the N
or inside the sphere non-conducting sphere
Capacitance of a Capacitance
1 q 1 q
V= (r £ R) V= (r > R) spherical capacitor (c) = Charge (Q) Combination of capacitors
4p Î0 R 4p Î0 r C = 4rpÎ0
AL
Potential (V)
.IN
Energy stored in a capacitor Capacitance of a
parallel plate capacitor
1 Q2
u= CV 2 = C = KAe0 k = dielectric
2 2C Series grouping of capacitors Parallel grouping of
d constant
Equivalent capacitance capacitors Equivalent
capacitance
1 1 1 1
Energy loss when two = + +.... + Cp= C1+ C 2+ ... Cn
Capacitance when Cs C1 C2 Cn
isolated charged conductors metallic slab inserted
are connected to each other between the plates
1 C1C2 (V1 - V2 )2 æ e Aö
= C = ç 0 ÷ t = thickness
2 C1 + C2 è d tø of slab
443
EBD_7179
444 PHYSICS

1. A parallel plate capacitor is charged to a certain voltage. 10. A positive point charge q is carried from a point B to a point
Now, if the dielectric material (with dielectric constant k) is A in the electric field of a point charge + Q at O. If the
removed then the permitivity of free space is e0, the work done in the process
is given by
(a) capacitance increases by a factor of k
(b) electric field reduces by a factor k qQ æ1 1 ö qQ æ1 1 ö
(a) ç + ÷ (b) ç - ÷
(c) voltage across the capacitor decreases by a factor k 4 p eo è a b ø 4 p eo è a b ø
(d) None of these qQ æ 1 1 ö qQ æ 1 1 ö
çç - ÷ çç + ÷÷
(c)
4 p eo 2 2÷ (d)
4 p eo è a 2
2. Two identical conducting balls having positive charges q1 èa b ø b2 ø
and q2 are separated by a distance r.If they are made to 11. Two concentric, thin metallic spheres of radii R1 and R2
touch each other and then separated to the same distance, (R1 > R2) bear charges Q1 and Q2 respectively. Then the
the force between them will be potential at distance r between R 1 and R 2 will be
(a) less than before (b) same as before

.IN
æ 1 ö
(c) more than before (d) zero çè k = 4pe ÷ø
0
3. A sphere of radius R has uniform volume charge density. AL
The electric potential at a points (r <R) is æ Q + Q2 ö æQ Q ö
(a) kç 1 ÷ø (b) kç 1 + 2 ÷
(a) due to the charge inside a sphere of radius r only è r è r R2 ø
(b) due to the entire charge of the sphere
N
æQ Q ö æQ Q ö
(c) due to the charge in the spherical sheel of inner and (c) kç 2 + 1÷ (d) kç 1 + 2 ÷
R

outer radii r and R, only è r R1 ø è R1 R 2 ø


12. Force between two plates of a capacitor is
U

(d) independent of r
4. Eight drops of mercury of equal radius and possessing equal Q Q2
JO

charge combine to form a big drop. The capacitance of bigger (a) (b)
eoA 2 eo A
drop as compared to each small drop is
(a) 16 times (b) 8 times
Q2
U

(c) 4 times (d) 2 times (c) (d) None of these


eoA
ED

5. The capacitance of a metallic sphere is 1mF , then it’s radius


13. An alpha particle is accelerated through a potential
is nearly
differenceof 106 volt. Its kinetic energy will be
(a) 1.11 m (b) 10 m (a) 1 MeV (b) 2 MeV (c) 4 MeV (d) 8 MeV
(c) 9 km (d) 1.11 cm 14. Two capacitors of capacitances C1 and C2 are connected in
6. Three charges 2 q, – q and – q are located at the vertices of parallel across a battery. If Q1 and Q2 respectively be the
an equilateral triangle. At the centre of the triangle
Q1
(a) the field is zero but potential is non-zero charges on the capacitors, then will be equal to
Q2
(b) the field is non-zero, but potential is zero
(c) both field and potential are zero
C2 C1 C12 C 22
(d) both field and potential are non-zero (a) (b) (c) (d)
7. Two conducting spheres of radii r 1 and r 2 are equally C1 C2 C 22 C12
charged. The ratio of their potentials is 15. A system of two parallel plates, each of area A, are separated
(a) r1 / r2 (b) r2 / r1 (c) r12 / r22 (d) r22 / r12 by distances d1 and d2. The space between them is filled
with dielectrics of permittivities e1 and e2. The permittivity
8. The electric potential due to a small electric dipole at a large
of free space is e0. The equivalent capacitance of the system
distance r from the centre of the dipole is proportional to
is
(a) r (b) 1/r (c) 1/r2 (d) 1/r3
9. An electron of mass m and charge e is accelerated from rest e1e2 A e1e 2 e0 A
through a potential difference V in vacuum. Its final speed (a) e2d1+e1d2 (b) e1d1+e 2d 2
will be
2e V eV e0 A e0 A
(a) (b) (c) e V/2m (d) e V/m (c) e1d1+e2d2 (d) e1d 2 +e2d1
m m
Electrostatic Potential and Capacitance 445

16. A large insulated sphere of radius r charged with Q units of 21. A parallel plate condenser with oil between the plates
electricity is placed in contact with a small insulated (dielectric constant of oil K = 2) has a capacitance C. If the
uncharged sphere of radius r´ and is then separated. The oil is removed, then capacitance of the capacitor becomes
charge on smaller sphere will now be
(a) Q (r' + r) (b) Q (r + r') C C
(a) 2C (b) 2 C (c) (d)
Q Qr' 2 2
(c) (d)
r '+ r r '+ r 22. An air capacitor C connected to a battery of e.m.f. V acquires
17. The capacitance of the capacitor of plate areas A1 and A2 a charge q and energy E. The capacitor is disconnected
(A1 < A2) at a distance d, as shown in figure is from the battery and a dielectric slab is placed between the
Î0 (A1 + A 2 ) plates. Which of the following statements is correct ?
(a) A2 (a) V and q decrease but C and E increase
2d A1
(b) V remains unchange, but q, E and C increase
Î0 A 2 (c) q remains unchanged, C increases, V and E decrease
(b)
d (d) q and C increase but V and E decrease.
Î0 A1A 2 23. Two parallel metal plates having charges + Q and – Q face
(c) each other at a certain distance between them. If the plates
d
are now dipped in kerosene oil tank, the electric field
Î0 A1 d between the plates will

.IN
(d)
d (a) remain same (b) become zero
18. When air is replaced by a dielectric medium of force (c) increases (d) decrease
constant K, the maximum force of attraction between two 24. A parallel plate condenser has a uniform electric field E(V/
AL
charges, separated by a distance m) in the space between the plates. If the distance between
(a) decreases K-times (b) increases K-times the plates is d(m) and area of each plate is A(m2) the energy
(joules) stored in the condenser is
N
1
(c) remains unchanged (d) becomes times
K2 1
(a) E2Ad/ Î0 Î0 E 2
R

(b)
19. A conductor carries a certain charge. When it is connected 2
to another uncharged conductor of finite capacity, then the
U

1
energy of the combined system is (c) Î0 EAd (d) Î E 2 Ad
2 0
JO

(a) more than that of the first conductor


(b) less than that of the first conductor 25. Which of the following figure shows the correct
(c) equal to that of the first conductor equipotential surfaces of a system of two positive charges?
U

(d) uncertain
20. The magnitude of the electric field E in the annular region (a) (b)
ED

of a charged cylindrical capacitor + + + +


(a) is same throughout
(b) is higher near the outer cylinder than near the inner
cylinder
(c) (d)
1
(c) varies as , where r is the distance from the axis + +
r + +

1
(d) varies as 2 , where r is the distance from the axis
r

1. The positive terminal of 12 V battery is connected to the (a) zero (b) 10 V (c) 4 V (d) 10/3 V
ground. Then the negative terminal will be at 3. Find the dipole moment of a system where the potential
(a) – 6 V (b) + 12 V (c) zero (d) – 12 V 2.0 × 10–5 V at a point P, 0.1m from the dipole is 3.0 × 104.
2. A hollow metal sphere of radius 5 cm is charged such that (Use q = 30°).
the potential on its surface is 10 V. The potential at a distance (a) 2.57 × 10–17 Cm (b) 1.285 × 10–15 Cm
of 2 cm from the centre of the sphere is (c) 1.285 × 10–17 Cm (d) 2.57 × 10–15 Cm
EBD_7179
446 PHYSICS

4. A battery of e.m.f. V volt, resistors R1 and R2, a condenser 10. Two capacitors when connected in series have a
C and switches S1 and S2 are connected in a circuit shown. capacitance of 3 mF, and when connected in parallel have a
The condenser will get fully charged to V volt when capacitance of 16 mF. Their individual capacities are
(a) 1 mF, 2 mF (b) 6 mF, 2 mF
(c) 12 mF, 4 mF (d) 3 mF, 16 mF
R1 S1 S2 R2
11. The capacity of a parallel plate condenser is 10 mF, when
the distance between its plates is 8 cm. If the distance
between the plates is reduced to 4 cm, then the capacity of
V C this parallel plate condenser will be
(a) 5 mF (b) 10 mF (c) 20 mF (d) 40 mF
12. The capacitor, whose capacitance 6mF 6mF 3mF
is 6, 6 and 3mF respectively are
(a) S1 and S2 are both closed connected in series with 20 volt
(b) S1 and S2 are both open line. Find the charge on 3mF.
(c) S1 is open and S2 is closed (a) 30 mc
(b) 60 mF
(d) S1 is closed and S2 is open
(c) 15 mF 20mF
5. The electric potential at the surface of an atomic nucleus (d) 90 mF
(Z = 50) of radius of 9 × 10–15 m is 13. Four metallic plates each with a surface area of one side A,
(a) 80 V (b) 8 × 106 V (c) 9 V (d) 9 × 105 V

.IN
are placed at a distance d from each other. The two outer
6. Three point charges +q , + 2q and – 4q where q = 0.1 mC, are plates are connected to one point A and the two other inner
placed at the vertices of an equilateral triangle of side 10 cm plates to another point B as shown in the figure. Then the
as shown in figure. The potential energy of the system is
AL capacitance of the system is
– 4q
N
A B
10
cm

R
cm

e0A 2e 0 A 3e 0 A 4e 0 A
10

(a) (b) (c) (d)


U

d d d d
+q 10 cm +2q 14. Two spherical conductors A and B of radii a and b (b>a) are
JO

placed concentrically in air. The two are connected by a


(a) 3 × 10–3 J (b) –3 × 10–3 J copper wire as shown in figure. Then the equivalent
(c) 9 × 10 J–3 (d) –9 × 10–3 J capacitance of the system is
U

7. The four capacitors, each of 25 m F are connected as shown ab


4pe 0
ED

in fig. The dc voltmeter reads 200 V. The charge on each (a)


b-a b B
plate of capacitor is A
(b) 4pe 0 (a + b) a
V
(c) 4pe 0 b
– + – +
(d) 4pe 0a
– + – + 15. A ball of mass 1 g carrying a charge 10–8 C moves from a
point A at potential 600 V to a point B at zero potential. The
change in its K.E. is
(a) ± 2 ´ 10 -3 C (b) ± 5 ´ 10 -3 C (a) – 6 × 10–6 erg (b) – 6 × 10–6 J
(c) 6 × 10 J–6 (d) 6 × 10–6 erg
(c) ± 2 ´ 10 -2 C (d) ± 5 ´ 10 -2 C
16. Two capacitors C1 and C2 in a circuit are joined as shown in
8. An air capacitor of capacity C = 10 mF is connected to a figure. The potentials of points A and B are V1 and V2
constant voltage battery of 12 volt. Now the space between respectively; then the potential of point D will be
the plates is filled with a liquid of dielectric constant 5. The
A B
(additional) charge that flows now from battery to the
capacitor is V1 D V2
C1 C2
(a) 120 m C (b) 600 m C (c) 480 m C (d) 24 m C
9. A capacitor is charged to store an energy U. The charging (V1 + V2 ) C 2 V1 + C1V2
(a) (b)
battery is disconnected. An identical capacitor is now 2 C1 + C 2
connected to the first capacitor in parallel. The energy in
each of the capacitors is C1V1 + C 2 V2 C 2 V1 + C1V2
(c) (d)
(a) 3 U/2 (b) U (c) U/4 (d) U/2 C1 + C 2 C1 + C 2
Electrostatic Potential and Capacitance 447

17. A parallel plate capacitor with air between the plates is 26. Capacitance (in F) of a spherical conductor with radius 1 m
charged to a potential difference of 500V and then insulated. is
A plastic plate is inserted between the plates filling the (a) 1.1 × 10–10 (b) 106
whole gap. The potential difference between the plates now (c) 9 × 10–9 (d) 10–3
becomes 75V. The dielectric constant of plastic is 27. Two metal pieces having a potential difference of 800 V are
(a) 10/3 (b) 5 (c) 20/3 (d) 10 0.02 m apart horizontally. A particle of mass 1.96 × 10–15 kg
18. The plates of a parallel plate capacitor have an area of is suspended in equilibrium between the plates. If e is the
90 cm2 each and are separated by 2.5 mm. The capacitor is elementary charge, then charge on the particle is
charged by a 400 volt supply. How much electrostatic (a) 8 (b) 6 (c) 0.1 (d) 3
energy is stored by the capacitor? 28. Identical charges – q each are placed at 8 corners of a cube
(a) 2.55 × 10–6 J (b) 1.55 × 10–6 J of each side b. Electrostatic potential energy of a charge
(c) 8.15 × 10–6 J (d) 5.5 × 10–6 J + q which is placed at the centre of cube will be
19. From a supply of identical capacitors rated 8 mF, 250V, the - 4 2 q2 - 8 2 q2
minimum number of capacitors required to form a composite (a) (b)
16 mF, 1000V is p eo b p eo b
(a) 2 (b) 4 (c) 16 (d) 32
- 4 q2 - 8 2 q2
20. Calculate the area of the plates of a one farad parallel plate (c) (d)
capacitor if separation between plates is 1 mm and plates 3 p eo b p eo b
are in vacuum 29. A charge +q is fixed at each of the points x = x0, x = 3x0,
(a) 18 × 108 m2 (b) 0.3 × 108 m2 x = 5x0, .... upto ¥ on X-axis and charge –q is fixed on each

.IN
(c) 1.3 × 10 m 8 2 (d) 1.13 × 108 m2 of the points x = 2x0, x = 4x0, x = 6x0, .... upto ¥ . Here x0 is
21. A one microfarad capacitor of a TV is subjected to 4000 V a positive constant. Take the potential at a point due to a
potential difference. The energy stored in capacitor is AL Q
(a) 8 J (b) 16 J charge Q at a distance r from it to be . Then the
4pe 0 r
(c) 4 × 10–3 J (d) 2 × 10–3 J potential at the origin due to above system of charges will be
22. Two capacitors, C1 = 2mF and C2 = 8 mF are connected in q
N
series across a 300 V source. Then (a) zero (b)
8pe 0 x 0 log e 2
(a) the charge on each capacitor is 4.8×10–4 C
R

(b) the potential difference across C1 is 60 V q log e 2


(c) infinity (d)
(c) the potential difference across C2 is 240 V 4pe 0 x 0
U

(d) the energy stroed in the system is 5.2 × 10–2 J 30. Two equally charged spheres of radii a and b are connected
JO

23. Two capacitors C1 and C2 = 2C1 are Q C1 together. What will be the ratio of electric field intensity on
connected in a circuit with a switch between their surfaces?
them as shown in the figure. Initially the
a a2 b b2
U

switch is open and C1 holds charge Q. The R


(a) (b) (c ) (d)
switch is closed. At steady state, the charge b b2 a a2
ED

on each capacitor will be 31. In a hollow spherical shell, potential (V) changes with respect
Q 2Q
C2= 2C1 to distance (s) from centre as
(a) Q, 2Q (b) ,
3 3 (a) (b)
3Q 2Q 4Q
(c) ,3Q (d) , V V
2 3 3
24. Two capacitors of capacitance C are connected in series. If
one of them is filled with dielectric substance k, what is the S S
effective capacitance ?
(c) (d)
kC
(a) (b) C(k + 1)
(1 + k ) V V

2kC
(c) (d) None of these S S
1+ k 32. A parallel plate capacitor of capacitance C is connected to
25. The potential at a point x (measured in m m) due to some a battery and is charged to a potential difference V. Another
charges situated on the x-axis is given by V(x) = 20/(x2 – 4) capacitor of capacitance 2C is similary charged to a potential
volt difference 2V. The charging battery is now disconnected
The electric field E at x = 4 m m is given by and the capacitors are connected in parallel to each other in
(a) (10/9) volt/ m m and in the +ve x direction such a way that the positive terminal of one is connected to
(b) (5/3) volt/ m m and in the –ve x direction the negative terminal of the other. The final energy of the
configuration is
(c) (5/3) volt/ m m and in the +ve x direction
3 25 9
(d) (10/9) volt/ m m and in the –ve x direction (a) zero (b) CV 2 (c) CV 2 (d) CV 2
2 6 2
EBD_7179
448 PHYSICS

33. A solid conducting sphere having a charge Q is surrounding 40. The capacitance of a parallel plate capacitor is Ca (Fig. a). A
by an uncharged concentric conducting hollow spherical dielectric of dielectric constant K is inserted as shown in
shell. Let the potential difference between the surface of fig (b) and (c). If Cb and Cc denote the capacitances in fig
the solid sphere and that of the outer surface of the hollow (b) and (c), then
shell be V. If the shell is now given a charge of – 3Q, the new d/2
potential difference between the same two surfaces is d
(a) V (b) 2 V (c) 4 V (d) – 2 V K
34. In the electric field of an point charge A Ca Cb
(a) (b)
q, a certain charge is carried from point
A to B, C, D and E. Then the work +
done is q d K (c)
(a) least along the path AB B E
(b) least along the path AD C D Cc
(c) zero along any one of the path AB, AC, AD andAE (a) both Cb, Cc > Ca (b) Cc > Ca while Cb > Ca
(d) least along AE (c) both Cb, Cc < Ca (d) Ca = Cb = Cc
35. A circuit is connected as shown in the figure with the switch 41. In the circuit shown, which of the following statements is
S open. When the switch is closed, the total amount of true if V1 (potential across C1) is 30 V and V2 (potential
charge that flows from Y to X is across C2) is 20 V?
3mF

.IN
6mF V1 = 30V V2 = 20V
X
S1 S3 S2
AL C1 = 2pF C 2 = 3pF
S

3W 6W
N
Y (a) With S1 closed, V1 = 15 V, V2 = 25 V
(b) With S3 closed, V1 = V2 = 25 V
R

9V (c) With S1 and S2 closed, V1 = V2 = 0


(a) 0 (b) 54 mC (c) 27mC (d) 81 mC (d) With S1 and S3 closed, V1 = 30 V, V2 = 20 V
U

36. If a slab of insulating material 4 × 10–5 m thick is introduced 42. A parallel plate capacitor is located horizontally such that
between the plates of a parallel plate capacitor, the distance one of the plates is submerged in a liquid while the other is
JO

between the plates has to be increased by 3.5 × 10–5 m to above the liquid surface. When plates are charged the level
restore the capacity to original value. Then the dielectric of liquid
constant of the material of slab is
U

(a) rises
(a) 8 (b) 6 (c) 12 (d) 10 (b) falls
ED

37. Three capacitors each of capacity 4mF are to be connected


(c) remains unchanged
in such a way that the effective capacitance is 6 mF. This can
(d) may rise or fall depending Changed Liquid
be done by
(a) connecting two in parallel and one in series on the amount of charge
43. Two small conductors A and B are given charges q 1 and
(b) connecting all of them in series
q2 respectively. Now they are placed inside a hollow metallic
(c) connecting them in parallel conductor C carrying a charge Q. If all the three
(d) connecting two in series and one in parallel conductors A, B and C are connected by a conducting wire
38. If we increase ‘d’ of a parallel plate condenser to ‘2d’ and fill as shown, the charges on A, B and C will be respectively
wax to the whole empty space between its two plate, then
Q
capacitance increase from 1pF to 2pF. What is the dielectric
A
constant of wax?
(a) 2 (b) 4 (c) 4 (d) 8 q1 C
39. Two spherical conductors A and B of radii a and b (b > a)
are placed concentrically in air. B is given charge +Q and A q2 B
is earthed. The equivalent capacitance of the system is
ab q1 + q 2 q1 + q 2
+ + B (a) , ,Q
4pe 0
+
(a) +
+
2 2
b-a
+
+

b Q + q1 + q 3 Q + q1 + q 2 Q + q1 + q 2
+ + + +

A– (b) , ,
+ + + +

(b) 4pe 0 (a + b) – –
3 3 3
(c) 4pe 0 b – a O –
– – (c) q1 + q 2 + Q , q1 + q 2 + Q , 0
– 2 2
æ b2 ö
+

4pe 0 ç ÷ (d) 0, 0, Q + q1 + q2
+

(d)
+

ç b -a ÷ + +
+ +
+
è ø
Electrostatic Potential and Capacitance 449

44. Between the plates of a parallel plate capacitor dielectric 47. Two vertical metallic plates carrying equal and opposite
plate is introduced just to fill the space between the plates. charges are kept parallel to each other like a parallel plate
The capacitor is charged and later disconnected from the capacitor. A small spherical metallic ball is suspended by a
battery. The dielectric plate is slowly drawn out of the long insulated thread such that it hangs freely in the centre
capacitor parallel to plates. The plot of the potential of the two metallic plates. The ball, which is uncharged, is
difference V across the plates and the length of the dielectric taken slowly towards the positively charged plate and is
plate drawn out is made to touch that plate. Then the ball will
(a) stick to the positively charged plate
(a) V (b) V (b) come back to its original position and will remain there
(c) oscillate between the two plates touching each plate
in turn
(d) oscillate between the two plates without touch them
48. Two parallel plate capacitors of capacitances C and 2C are
connected in parallel and charged to a potential difference V.
x x
The battery is then disconnected and the region between the
plates of the capacitor C is completely filled with a material fo
(c) V (d) V dielectric constant K. The potential difference across the
capacitors now becomes
3V V 3

.IN
(a) (b) KV (c) (d)
K+2 K KV
49. A parallel plate capacitor is connected to a battery. The
x x
AL quantities charge, voltage, electric field and energy
associated with this capacitor are given by Q0, V0, E0, and
45. Three capacitors C1, C2 and C3 are connected to a battery
U0 respectively. A dielectric slab is now introduced to fill
as shown. With symbols having their usual meanings, the
N
the space between the plates with the battery still in
correct conditions are
connection. The corresponding quantities now given by
R

V Q, V, E and U are related to the previous ones as


Q2 2 C 2
(a) Q > Q0 (b) V > V0 (c) E > E0 (d) U < U0
U

V1
Q1 C1 50. The effective capacitance of combination of combination of
JO

Q3 C3
equal capacitors between points A and B shown in figure is

V3 C C
U

V
C C C
ED

(a) Q1 = Q2 = Q3 and V1 = V2 = V C C
A B
(b) V1 = V2 = V3 = V
(c) Q1 = Q2 + Q3 and V = V1 = V2 C C C
(d) Q2 = Q3 and V2 = V3
46. Figure (i) shows two capacitors connected in series and
connected by a battery. The graph (ii) shows the variation C
of potential as one moves from left to right on the branch (a) C (b) 2C (c) 3C (d)
2
AB containing the capacitors. Then 51. A parallel plate capacitor of plate area A and plate separation
d is charged to potential difference V and then the battery
C1 C2 is disconnected. A slab of dielectric constant K is then
V
A B inserted between the plates of capacitor so as to fill the
space between the plates. If Q, E and W denote respectively,
the magnitude of charge on each plate electric field between
the plates (after the slab is inserted), and work done on the
E system, in question, in the process of inserting the slab,
(i) (ii) then which is wrong ?
e 0 AV e 0 KAV
(a) C1 = C2 (a) Q= (b) Q=
(b) C 1 < C2 d d
(c) C1 > C2 V e 0 AV 2 æ 1ö
(d) C1 and C2 cannot be compared (c) E= (d) W= çè1 - ÷ø
Kd 2d K
EBD_7179
450 PHYSICS

52. In the circuit given below, the charge in mC, on the capacitor 59. In the given circuit with steady current, the potential drop
having 5 mF is across the capacitor must be
A V R B
3mF 2mF
e d
C
5mF V
f c
4mF
a + b 2V 2R
6V

(a) 4.5 (b) 9 (c) 7 (d) 15 2V V V


(a) (b) (c) (d) V
53. If a charge – 150 nC is given to a concentric spherical shell 3 3 2
and a charge +50 nC is placed at its centre then the charge 60. A uniformly charged thin spherical shell of radius R carries
on inner and outer surface of the shell is uniform surface charge density of s per unit area. It is made
(a) –50 nC, –100 nC (b) +50 nC, –200 nC of two hemispherical shells, held together by pressing them
(c) –50 nC, –200 nC (d) 50 nC, 100 nC with force F (see figure). F is proportional to
54. A battery is used to charge a parallel plate capacitor till the
potential difference between the plates becomes equal to

.IN
the electromotive force of the battery. The ratio of the F F
energy stored in the capacitor and the work done by the
battery will be
(a) 1/2 (b) 1 (c) 2 (d) 1/4
AL
55. Four point charges q, q, q and – 3q are placed at the vertices 1 2 2 1 2
of a regular tetrahedron of side L. The work done by electric (a) s R (b) s R
e0 e0
N
force in taking all the charges to the centre of the tetrahedron
R

1 1 s2 1 s2
is (where k = )
4pe 0 (c) (d)
e0 R e0 R 2
U

6kq 2 -6kq 2 12kq2 61. A dielectric slab of thickness d is inserted in a parallel plate
JO

(a) (b) (c) (d) zero capacitor whose negative plate is at x = 0 and positive plate
L L L
is at x = 3d. The slab is equidistant from the plates. the
56. Two identical particles each of mass m and having charges
U

– q and +q are revolving in a circle of radius r under the capacitor is given some charge. As one goes from 0 to 3d
influence of electric attraction. Kinetic energy of each (a) the magnitude of the electric field remains the same
ED

(b) the direction of the electric field remains the same


æ 1 ö (c) the electric potential decreases continuously
particle is ç k =
è 4pe 0 ÷ø (d) the electric potential increases at first, then decreases
and again increases
(a) kq2/4r (b) kq2/2r (c) kq2/8r (d) kq2/r
62. In the given circuit if point C is connected to the earth and
57. Figure shows three circular arcs, each of radius R and total
charge as indicated. The net electric potential at the centre a potential of +2000V is given to the point A, the potential
of curvature is at B is
Q +Q
(a) 2pe R 10mF 10mF
0
5mF
Q 45° C
(b) A
4pe 0 R 30° B
–2Q
2Q
(c) 10mF
pe0 R R
Q +3Q (a) 1500V (b) 1000 V (c) 500 V (d) 400 V
(d) pe0 R 63. A 4 mF capacitor, a resistance of 2.5 MW is in series with
58. If the potential of a capacitor having capacity 6 mF is increased 12V battery. Find the time after which the potential difference
from 10 V to 20 V, then increase in its energy will be across the capacitor is 3 times the potential diference across
(a) 4 × 10–4 J (b) 4 × 10–4 J the resistor. [Given In (b) = 0.693]
(c) 9 × 10–4 J (d) 12 × 10–6 J (a) 13.86s (b) 6.93 s (c) 7s (d) 14 s
Electrostatic Potential and Capacitance 451

64. If a capacitor 900 µF is charged to 100 V and its total energy 70. A network of four capacitors of capacity equal to C1 = C,
is transferred to a capacitor of capacitance 100 µF then its C2 = 2C, C3 = 3C and C4 = 4C are conducted to a battery as
potential is shown in the figure. The ratio of the charges on C2 and C4 is
(a) 200 V (b) 30 V (c) 300 V (d) 400 V
65. What is the effective capacitance between points X and Y?
C1 = 6mF
(a) 24 mF
(b) 18 mF C3 = 6m F C5 = 20m F C2 = 6m F
X Y
(c) 12 mF A C B D
C4 = 6m F
(d) 6 mF
66. In a parallel plate capacitor, the distance between the plates
is d and potential difference across plates is V. Energy
stored per unit volume between the plates of capacitor is (a) 4/7 (b) 3/22 (c) 7/4 (d) 22/3
71. A series combination of n1 capacitors, each of value C1, is
Q2 1 V2
(a) (b) e0 charged by a source of potential difference 4 V. When
2V 2 2 d2 another parallel combination of n2 capacitors, each of value
1 V2 C2, is charged by a source of potential difference V, it has
1 V2
(c) (d) e0 the same (total) energy stored in it, as the first combination
2 e 0d 2 2 d has. The value of C2 , in terms of C1, is then

.IN
67. A capacitor C1 is charged to a potential difference V. The
2 C1 n2 n2 16 C1
charging battery is then removed and the capacitor is (a) (b) 16 n C1 (c) 2 C1 (d)
connected to an uncharged capacitor C2. The potential
AL n1 n2 1 n1 n1 n2
difference across the combination is 72. A condenser of capacity C is charged to a potential
VC1 æ C ö difference of V1. The plates of the condenser are then
(a) (b) Vçç1 + 2 ÷÷ connected to an ideal inductor of inductance L. The current
N
(C1 + C 2 ) è C1 ø
through the inductor when the potential difference across
R

æ C ö VC 2 the condenser reduces to V2 is


(c) Vçç1 + 1 ÷÷ (d)
è C2 ø (C1 + C 2 )
U

1/ 2 1/ 2
68. As per this diagram a point charge +q is placed at the origin æ C(V12 - V22 ) ö æ C (V1 - V2 ) 2 ö
(a) çè ÷ø (b) çè ÷ø
JO

O. Work done in taking another point charge L L


– Q from the point A [coordinates (0, a)] to another point B
[coordinates (a, 0)] along the straight path AB is C(V12 - V22 ) C (V1 - V2 )
(c) (d)
U

(a) zero L L
æ - qQ 1 ö DIRECTIONS (for Qs. 73 to 75) : Each question contains
ED

ç ÷
(b) ç 4 pe 2 ÷ 2 a y STATEMENT-1 and STATEMENT-2. Choose the correct answer
è 0 a ø
A (ONLY ONE option is correct ) from the following-
æ qQ 1 ö a (a) Statement -1 is false, Statement-2 is true
ç ÷
(c) ç 4 pe 2 ÷. 2 (b) Statement -1 is true, Statement-2 is true; Statement -2 is a
è 0 a ø
correct explanation for Statement-1
æ qQ 1 ö O x (c) Statement -1 is true, Statement-2 is true; Statement -2 is not
ç ÷ B
(d) ç . 2 a
2÷ a correct explanation for Statement-1
è 4 pe 0 a ø
(d) Statement -1 is true, Statement-2 is false
69. Two charges q1 and q2 are placed 30 cm apart, as shown in
73. Statement 1 : Each of the plates of a parallel-plate capacitor
the figure. A third charge q3 is moved along the arc of a
is given equal positive charge Q. The charges on the facing
circle of radius 40 cm from C to D. The change in the
surfaces will be same.
q3
potential energy of the system is k , , where k is Statement 2 : A negative charge (–Q) will be induced on
4p Î0
each of the facing surfaces.
q3 74. Statement 1 : Electric potential and electric potential energy
C
are different quantities.
Statement 2 : For a system of positive test charge and
40 cm point charge electric potential energy = electric potential.
q2
75. Statement I : Two equipotential surfaces cannot cut each
q1
other.
A 30 cm B D Statement II : Two equipotential surfaces are parallel to
(a) 8q1 (b) 6q1 (c) 8q2 (d) 6q2 each other.
EBD_7179
452 PHYSICS

Exemplar Questions 4. The electrostatic potential on the surface of a charged


1. A capacitor of 4 µF is connected as shown in the circuit. conducting sphere is 100V. Two statements are made in this
The internal resistance of the battery is 0.5W. The amount regard S1 at any point inside the sphere, electric intensity
of charge on the capacitor plates will be is zero. S2 at any point inside the sphere, the electrostatic
potential is 100V. Which of the following is a correct
4 mF
10W statement?
(a) S1 is true but S2 is false
(b) Both S1 and S2 are false
(c) S1 is true, S2 is also true and S1 is the cause of S2
2.5 V (d) S1 is true, S2 is also true but the statements are
independant
5. Equipotentials at a great distance from a collection of
2W

.IN
charges whose total sum is not zero are approximately
(a) 0 µC (b) 4 µC (c) 16 µC (d) 8 µC (a) spheres (b) planes
2. A positively charged particle is released from rest in an uniform (c) paraboloids (d) ellipsoids
electric field. The electric potential energy of the charge
AL
6. A parallel plate capacitor is made of two dielectric blocks in
(a) remains a constant because the electric field is uniform series. One of the blocks has thickness d1 and dielectric
(b) increases because the charge moves along the electric field
N
constant K1 and the other has thickness d2 and dielectric
(c) decreases because the charge moves along the electric constant K2 as shown in figure. This arrangement can be
R

field thought as a dielectric slab of thickness d (= d1 + d2) and


(d) decreases because the charge moves opposite to the effective dielectric constant K. The K is
U

electric field
JO

3. Figure shows some equipotential lines distributed in space. d1 K1


A charged object is moved from point A to point B.
(a) The work done in Fig. (i) is the greatest d2 K2
U

(b) The work done in Fig. (ii) is least


(c) The work done is the same in Fig. (i), Fig.(ii) and Fig. (iii) K1 d1 + K 2 d 2 K1d1 + K 2 d 2
ED

(a) (b)
(d) The work done in Fig. (iii) is greater than Fig. (ii) but d1 + d 2 K1 + K 2
equal to that in
K1 K 2 ( d1 + d 2 ) 2K1K 2
(c) (d)
20V 40V
( K1d1 + K 2 d 2 ) K1 + K 2

NEET/AIPMT (2013-2017) Questions


A B A B 7. A, B and C are three points in a uniform electric field. The
electric potential is [2013]

10V 20V 30V 40V 50V 10V 30V 30V A


Fig. (i) Fig. (ii) B ®
E
C
30V
(a) maximum at B
(b) maximum at C
A B (c) same at all the three points A, B and C
(d) maximum at A
8. Two thin dielectric slabs of dielectric constants K1 and K2
10V 20V 40V 50V (K1 < K2) are inserted between plates of a parallel plate
Fig. (iii) capacitor, as shown in the figure. The variation of electric
Electrostatic Potential and Capacitance 453

field ‘E’ between the plates with distance ‘d’ as measured 12. If potential (in volts) in a region is expressed as V(x, y, z) =
from plate P is correctly shown by : [2014] 6 xy – y + 2yz, the electric field (in N/C) at point (1, 1, 0) is :
[2015 RS]
P + – Q
+ –
+ – (a) - (6iˆ + 5jˆ + 2k)
ˆ (b) - (2iˆ + 3jˆ + k)
ˆ
+ –
+ –
+
+

– (c) - (6iˆ + 9ˆj + k)
ˆ (d) -(3iˆ + 5jˆ + 3k)
ˆ
+ –
+ – 13. A parallel plate air capacitor has capacity 'C' distance of
K1 K2 separation between plates is 'd' and potential difference 'V'
is applied between the plates. Force of attraction between
(a) (b) the plates of the parallel plate air capacitor is : [2015 RS]
E C2 V 2 C2 V 2
E CV 2 CV 2
(a) (b) (c) (d)
2d d 2d 2 2d 2
0 14. 1 2
d 0
d

E V
E

.IN
2m F 8m F
(c) (d)
0
0
d d
AL A capacitor of 2mF is charged as shown in the diagram.
When the switch S is turned to position 2, the percentage
9. A conducting sphere of radius R is given a charge Q. The of its stored energy dissipated is : [2016]
electric potential and the electric field at the centre of the
N
(a) 0% (b) 20%
sphere respectively are: [2014] (c) 75% (d) 80%
R

Q 15. A capacitor is charged by a battery. The battery is removed


(a) Zero and
4p e 0 R 2 and another identical uncharged capacitor is connected in
U

Q parallel. The total electrostatic energy of resulting system:


(b) and Zero
JO

4p e 0 R (a) decreases by a factor of 2 [2017]


(b) remains the same
Q Q (c) increases by a factor of 2
and
U

(c) (d) increases by a factor of 4


4p e 0 R 4p e 0 R 2 16. The diagrams below show regions of equipotentials. [2017]
ED

(d) Both are zero 20V 40V 20V 40V


10. In a region, the potential is represented by
V(x, y, z) = 6x – 8xy – 8y + 6yz, where V is in volts and x, y, z
are in metres. The electric force experienced by a charge of 2 (a) A B (b) A B
coulomb situated at point (1, 1, 1) is : [2014]
(a) 6 5 N (b) 30 N 10 V 30V
10 V 30V
10V 30V 40V
(c) 24 N (d) 4 35 N
20V
11. A parallel plate air capacitor of capacitance C is connected
to a cell of emf V and then disconnected from it. A dielectric (c) A B (d) A B
slab of dielectric constant K, which can just fill the air gap
of the capacitor, is now inserted in it. Which of the following 10V
is incorrect ? [2015] 20V 40V 30V
(a) The energy stored in the capacitor decreases K times. A positive charge is moved from A to B in each diagram.
(a) In all the four cases the work done is the same
1 æ1 ö
(b) The chance in energy stored is CV 2 ç – 1÷ (b) Minimum work is required to move q in figure (a)
2 èK ø
(c) The charge on the capacitor is not conserved. (c) Maximum work is required to move q in figure (b)
(d) The potential difference between the plates decreases
K times. (d) Maximum work is required to move q in figure (c)
EBD_7179
454 PHYSICS

Hints & Solutions


EXERCISE - 1 15. (a)
1. (d) 2. (c) 3. (a) 4. (d) 5. (c)
6. (b) Potential at the centre of the triangle,
A e1 e2 A
åq 2q -q - q
V= = =0
4 p e0 r 4 p e0 r
Obviously, E ¹ 0
d1 d2
Q 1
7. (b) As V = i.e. V µ 1 1 1
4 p e0r r = +
Cs C1 C2
V1 r2
\ = 1 1
V2 r1 Þ =
Cs e1A e 2A
+
1 d1 d 2

.IN
8. (c) Due to small dipole, V µ 2 .
r 1 1 æ d1 d 2 ö Ae1e 2
9. (a) K.E. = Work done = eV Þ = ç + ÷ Þ Cs = e d + e d
Cs A è e1 e 2 ø 2 1 1 2
AL
1 2 eV Q 0
m n2 = eV \ n= 16. (d) Common potential, V = +
2 m 4 pe 0 r 4 pe 0 r
N
10. (b) WBA = q (VA - VB ) Q+0
=
R

é1 1ù 4 pe 0 (r + r ')
é Q Q ù = qQ
=q ê - ú êa - bú \ charge on smaller sphere of radius r' is
U

4 p e0 ë û
ë 4 p e0 a 4 p e 0 b û
Q r¢
4 pe 0 r¢ ´ V =
JO

r + r¢
Q2 Q1 Î0 A
11. (c) R1 17. (d) C=
R2
U

d
r A ® common area, Here A = A1
ED

1 q1q 2
18. (a) In air Fair =
4pe 0 r 2
Q2 Q1
Vr = + 1 q1q 2
4pe 0 r 4pe 0 R1
In medium Fm =
4pe 0 Kr 2
1 æ Q 2 Q1 ö
Vr = ç + ÷ Fm 1 F
4pe0 è r R1 ø \ = Þ Fm = air (decreases K-times)
Fair K K
12. (b) The magnitude of electric field by any one plate is
19. (b) Energy will be lost during transfer of charge (heating
Q –Q effect).
s Q
or E l 1
2e o 2Ae o 20. (c) Eµ hence E µ
2pe 0 r r
Q2 21. (d) When oil is placed between space of plates
Now force magnitude is Q E i.e. F = 2 Ae 0
2Ae o C= ... (1)
d
13. (b) Charge on a particle, q = 2 e. é KAe 0 ù
K.E. = work done = q × V = 2e × 106 V = 2 MeV. êQ C = d , where K = 2ú
ë û
14. (b) In parallel, potential is same, say V
Ae 0
Q1 C1V C1 When oil is removed C' = ............. (2)
= = d
Q 2 C2V C2 on comparing both equation, weget C' = C/2
Electrostatic Potential and Capacitance 455

22. (c) When a battery across the plates of capacitor is


disconnected and dielectric slab is placed in between 9 ´ 109 ´ 10 ´ (0.1 ´10 -6 )2
Þ U=-
the plates, then æ 10 ö
(i) capacity C increases ç ÷
è 100 ø
(ii) charge q remains unchanged Þ U = – 9 × 10–3J
(iii) potential V decreases 7. (b) Charge on each plate of each capacitor
(iv) energy E decreases
s Q Q = ±CV = ±25 ´ 106 ´ 200 = ±5 ´ 10 -3 C
23. (d) Electric field E = = 8. (c) q1 = C1V = 10 ´12 = 120 m C
e Ae
e of kerosine oil is more than that of air.. q 2 = C 2 V = KC1 ´ V = 5 ´10 ´12 = 600 m C
As e increases, E decreases. Additional charge that flows
1 = q 2 - q1 = 600 - 120 = 480 m C.
24. (d) U = CV 2
2 9. (c) As battery is disconnected, total charge Q is shared
1 æ A Î0 ö 2 1 2
equally by two capacitors.
U= ç ÷ (Ed) = A Î0 E d
2è d ø 2 2 2
Energy of each capacitor = (Q / 2) = 1 Q = 1 U.
25. (c) Equipotential surfaces are normal to the electric field 2C 4 2C 4
lines. The following figure shows the equipotential
surfaces along with electric field lines for a system of C1 C 2
10. (c) Cs = =3
C1 + C 2

.IN
two positive charges.
C p = C1 + C 2 = 16 \ C1 C 2 = 48
AL C1 - C 2 = (C1 + C 2 ) 2 - 4 C1 C 2

= 16 2 - 4 ´ 48 = 64 = 8
N
C1 + C2 = 16 mF
R

C1 - C2 = 8 mF
EXERCISE - 2 Þ 2C1 = 24mF Þ C1 = 12mF
U

1. (d) When negative terminal is grounded, positive terminal 48


JO

of battery is at +12 V. When positive terminal is \ C2 = = 4 mF


12
grounded, the negative terminal will be at –12 V. 11. (c) C = 10 mF ; d = 8 cm
2. (b) Potential at any point inside the sphere = potential at C' = ? ; d' = 4 cm
U

the surface of the sphere = 10V.


A Î0 1
Þ Ca
( )
2.0 ´10-5 V ( 0.1m)
C=
ED

2
Vr 2 d d
3. (a) = qd = = 2.57 ´10 -17 Cm. If d is halved then C will be doubled.
k cos q ì ü
ïï Hence C' = 2C = 2 × 10 mF = 20 mF
9 m ïï æ 3 ö
í9.0 ´10 ý
ï æ C ö çè 2 ÷ø 1
=
1
+
1
+
1
çè ÷ø ï 12. (a) In series
C C1 C 2 C3
and charge on each
ïî V ïþ
Note that the units cancel to leave units appropriate capacitor is same.
for a dipole moment. 13. (b) It consists of two capacitors in parallel, therefore, the
4. (d) When S1 is closed and S2 is opened, the capacitor will 2 Î0 A
get charged to a potential difference of V volts. total capacitance is =
d
q (Ze)
5. (b) V= = + + + +
4 p e o r 4p Î0 r + + + +
– – – –
– – – –
(50 ´ 1.6 ´ 10-19 ) +A
9 ´ 109 = 8 ´ 10 6 V –




– –
– –B
9 ´ 10-15 + + + +
+ + + +
1 æ 2q 2 8q 2 4q 2 ö (The plates of B, having negative charge do not
6. (d) U= ç - - ÷ constitute a capacitor).
4pe0 çè a a a ÷ø
14. (c) All the charge given to inner sphere will pass on to
the outer one. So capacitance that of outer one is
1 æ 10q 2 ö
Þ U = 4pe çç - a ÷÷ 4p Î0 b .
0è ø 15. (c) As work is done by the field, K.E. of the body increases
by
EBD_7179
456 PHYSICS

K.E. = W = q (VA - VB ) 4.8 ´ 10-4


V1 = = 240V
= 10 -8 (600 - 0) = 6 ´10 -6 J 2 ´ 10-6
16. (c) Consider the potential at D be ‘V’.
4.8 ´ 10 -4
Potential drop across C1 is (V – V1 ) and C2 is V2 = = 60V
(V2 – V) 8 ´ 10-6
\ q 1 = C1(V - V1 ), q 2 = C2 (V2 - V)
Q2 Q2
As q1 = q2 [capacitors are in series] U= +
2C1 2C2
\ C1 (V - V1) = C2 (V2 - V)

v=
C1V1 + C2 V2
ÞU=
( 4.8 ´ 10 ) -4 2
æ 1 ö
C1 + C2 çè ÷
2 1.6 ´ 10 -6 ø
q Þ U = 3 × 2.4 × 10–2 J
17. (c) V0 =
C0 Þ U = 7.2 × 10–2 J
23. (b) In steady state, both the capacitors are at the same
q V C0 potential,
V= Þ =
C V0 C
Q1 Q 2 Q Q
i.e., = or 1 = 2 or Q2 = 2Q1
C0 500 20 C1 C2 C 2C

.IN
Þ = =
C 75 3 Also Q1 + Q2 = Q
20
By definition, C = kC0 Þ k = \ Q1 =
Q
, Q2 =
2Q
3
AL 3 3
18. (a) Here, A = 90 cm² = 90 × 10–4 m2;
d = 2.5 mm = 2.5 × 10–3 m; V = 400 volt 24. (a)
N
20
e 0 A 8.854 ´ 10-12 ´ 90 ´ 10-4 25. (a) Here, V(x) = 2 volt
C= = x -4
R

d 2.5 ´ 10 -3
–11
= 3.187 × 10 F dV d æ 20 ö
U

We know that E = - =- ç ÷
1 1 dx dx è x 2 - 4 ø
W = CV 2 = ´ 3.187 ´ 10-11 ´ ( 400 )
2
JO

2 2
= 2.55 × 10–6 J 40x
or, E = +
19. (d) Let ‘n’ such capacitors are in series and such ‘m’ such (x - 4) 2
2
U

branch are in parallel.


At x = 4 mm ,
\ 250 × n = 1000 \ n = 4 … (i)
ED

40 ´ 4 160 10
8
´ m = 16 E= + =+
= + volt / mm.
Also
n (42 - 4) 2 144 9
r
16 ´ n Positive sign indicates that E is in +ve x-direction.
m= =8 … (ii) 26. (a) Capacitance of spherical conductor = 4pe0a
8
\ No. of capacitor = 8 ´ 4 = 32 where a is radius of conductor.
1 1
e0 A Therefore, C = ´1 = ´10 -9
20. (d) For a parallel plate capacitor C= 9 ´ 10 9 9
d
Cd 1 ´ 10 -3 = 0.11´10 -9 F = 1.1 ´ 10-10 F
\ A= e = = 1.13 × 108 m2
0 8.85 ´ 10 -12 V
This corresponds to area of square of side 10.6 km 27. (d) In equilibrium, F = q E = ( n e) = mg
d
which shows that one farad is very large unit of
capacitance. mg d 1.96 ´ 10 -15 ´ 9.8 ´ 0.02
n= = =3
21. (a) E=
1 1
CV 2 = ´ 1 ´ 10-6 ´ (4000) 2 = 8 J. eV 1.6 ´ 10 -19 ´ 800
2 2
28. (c) Length of body diagnonal = 3b
2´8
22. (a) Cs = = 1.6mF \ Distance of centre of cube from each corner,
2+8
Since, Q = Cs V = 1.6 × 10–6 × 300 3
Q = 4.8 × 10–4 C r= b
2
Total P.E. of charge + q at the centre
Electrostatic Potential and Capacitance 457

8 q ( -q ) - 8q 2 - 4 q2 33. (a) V2
= = =
4 pe o r 4 pe o ( 3 b / 2) pe o 3 b Q r1 V1
29. (d) Potential at origin r2
= (V1 + V3 + V5 + .....) – (V2 + V4 + V6 + .....)
Situation 1 :
q é 1 1 1 ù
Þ ê - + .....¥ú Q Q é1 1 ù
4pe 0 ë x 0 2x 0 3x 0 û V1 - V2 = V = K - K = KQ ê - ú
r1 r2 ë r1 r2 û
q é 1 1 1 ù Situation 2 :
Þ ê1 - 2 + 3 - 4 .....¥ ú
4 pe 0 x 0 ë û
é KQ 3KQ ù é KQ 3KQ ù
V '1 - V '2 = V ' = ê - ú-ê - ú
q q
Þ log e (1 + 1) Þ log e 2 ë r1 r2 û ë r2 r2 û
4pe 0 x 0 4pe 0 x 0
é1 1 ù
= KQ ê - ú = V
ë r1 r2 û
a b
30. (c) 34. (c) Since, all A, B, C, D and E lie on an equipotential surface
so,
W= 0
35. (c) When steady state is reached, the current I coming

.IN
Let charge on each sphere = q
when they are connected together their potential will from the battery is given by
be equal . AL 3mF 6mF
Now let charge on a = q1 and on b = 2q - q1 X
+Q1 –Q1 +Q2 –Q2
1 q1 1 2q - q1
Þ Va = Vb or = S
4peo a 4pe o
N
b 3W 6W
q1 a
R

Þ = Y
2q - q1 b
U

1. q1 9V
9 = I (3 + 9) Þ I = 1A
a 2 æ q1 ö b 2
JO

Ea 4peo
= =ç ÷ a b2 b Þ potential difference across 3 W resistance = 3V
Eb 1 q 2 è 2q - q1 ø a 2 = . 2 = and potential difference across 6 W resistance
b a a
4peo b2 = 6V
U

Þ p.d. across 3 mF capacitor = 3V


= b:a and p.d. across 6mF capacitor = 6V
ED

31. (b) In shell, q charge is uniformly distributed over its \ Charge on 3 mF capacitor, Q1 = 3 × 3 = 9 mC
surface, it behaves as a conductor. Charge on 6mF capacitor, Q2 = 6 × 6 = 36 mC
+
+ Þ Charge (–Q1) is shifted from the positive plate of
+ q 6mF capacitor. The remaining charge on the positive
R plate of 6 mF capacitor is shifted through the switch.
+ +
\ Charge passing through the switch
+ +
= 36 – 9 = 27 mC
+
q æ 1ö
V= potential at surface = and inside 36. (a) As x = t ç1 - ÷ , where x is the addition distance of
4pe 0 R è Kø
plate, to restore the capacity of original value.
q
V= æ 1ö
4pe 0 R \ 3.5 ´ 10 -5 = 4 ´ 10 -5 ç1 - ÷ .
è K ø
Because of this it behaves as an equipotential surface.
32. (b) Energy stored, Solving, we get, K = 8.
1 1 3 C1 ´ C 2 4´ 4
U= Caq V 2net = (3C)V 2 = CV 2 37. (d) For series C' = = = 2mF
2 2 2 C1 + C 2 4 + 4
+ C
– For parallel Ceq = C'+ C2 = 2 + 4 = 6mF
V

A B
2V
– +
2C
EBD_7179
458 PHYSICS

Ke o A q qd
38. (b) C= V= =
d C b Î0 [l + x(K - 1)]
1´ Î0 A as x decreases, V increases.
1 ´ 10-12 = .......... (1)
d 45. (c) C2 and C3 are parallel so V2 = V3
K ´ Î0 A C1 and combination of C2 & C3 is in series.
2 ´ 10 -12 = .......... (2)
2d So, V = V2 + V1 or V = V3 + V1
(2) K and also Q1 = Q2 + Q3
Þ = 2 or K = 4
(1) 2 46. (c) Since, potential difference across C2 is greater than C1.
39. (d) The charge Q given to outer sphere distributes as é q ù
Q1 outside and Q2 inside which induces charge – Q2 Þ C1 > C2 êQ V = and q is same in series ú
on outside of inner sphere, + Q2 on inside of inner ë C û
sphere which is earthed. 47. (c) The ball on touching plate A will
The inside of outer and the inner sphere constitute a get positively charged. It will be A B
ab repelled by A and get attracted
spherical condenser having capacitance 4p Î0 + –
towards B. After touching B it +
b-a + +
+ + –

and the outside of the outer constitutes an isolated will get negatively charged. It + + +

will now be repelled by B and + –
sphere of capacitance 4p Î0 b . get attracted towards A. + –

.IN
\ the effective capacitance is Thus it will remain oscillating + –
ab and at the extreme position touch the plates.
4p Î0 + 4 p Î0 b
b-a 48. (a) Initial charge on first capacitor is CV = Q1.
AL
é a ù éa + b - a ù Initial charge on second capacitor is 2CV = Q2.
= 4 p Î0 b ê + 1ú = 4 p Î0 bê ú
ë b - a û ë b-a û Final capacitance of first capacitor is KC
N
If V' is the common potential then
b2
C = 4 p Î0
R

b-a Q1 + Q 2 CV + 2CV 3V
V¢ = V¢ = =
C1¢ + C 2 Þ KC + 2C 2+ K
U

Î A Î0 A 2 Î0 A (1 + K )
40. (a) Ca = 0 and Cb = =
d d d d 49. (a) Since battery is still in connection, so,
JO

+
2 2K V = V0
A A Þ Q0 = C0V0 and Q = kC0V0
Î0 Î0 K Î A
2 + 2
U

and C c = = 0 (1 + K) Þ Q = kQ0
d d 2d
Since, k > 1
ED

Î0 A Þ Q > Q0
or C b = 2(1 + K ) > C a
d
1
Î0 A 1 + K Also, U 0 = Q 0 V0 and
or C c = > Ca \ C b and C c > C a . 2
d 2
41. (d) When S1 and S3 is closed V1 = 30 V and potential 1
U= QV = kU0 {\ Q = kQ0 and V = V0 }
drop across C2 becomes 20 V. 2
42. (a) The molecules of liquid will convert into induced
Hence, U > U0
dipole, get oriented along the electric field produced
between the plates and rise due to force of attraction.
C C
43. (d) Charge given to a hollow conductor resides only on
the outer surface.
(l - x )b Î0 Kxb Î0 50. (b) A C C B
44. (b) C = C1 + KC2 = +
d d C C
b d
b Î0
= [l - x + Kx]
d C C
b Î0 The figure shows two independent balanced
C= [l + x(K - 1)] l
d wheatstone Brides connected in parallel each having
x K a capacitance C. So,
Cnet = CAB = 2C
Electrostatic Potential and Capacitance 459

51. (b) Ket C0 be the capacitance initially and C be the


1 2
Ce
e0A
capacitance finally. The C 0 = = 2 . (Q V = e)
d Ce 2

e 0 AV 1
=
Since, Q = C0 V Þ Q = 2
d
é (-3q) q (q) ´ (q) ù 2
V E V 55. (b) Ui = k ê ´ 3+ ´ 3ú = -6kq
Further, E 0 = and E = 0 Þ E = ë L L û L
d K Kd
Uf = 0
1 2 Work done by electric field = – Change in potential energy
Also, if Ui is the initial energy, then Ui = C 0 V
2
-6kq 2
After the introduction of slab if Uf be the final energy, = Ui – Uf =
then L
2 56. (c)
1 1 æ Vö
2
U f = CVslab = ( KC0 ) ç ÷ r
2 2 è Kø –q +q
O

.IN
1 C0 V 2
Þ Uf = Þ DU = U 2 - U1
2 K
mv2 kq 2 kq 2
1 æ1 ö = ; mv2 =
Þ DU = C V 2 ç - 1÷
AL r (2r) 2
4r
2 0 èK ø
Kinetic energy of each particle
Since, work done = Decrease in Potential Energy
N
Þ W = -DU 1 kq2
= mv2 =
R

2 8r
1 e 0 AV 2 æ 1ö
ÞW= çè1 - ÷ø 1 æ -2Q ö
U

2 d K 1 Q
57. (a) V = V1 + V2 + V3 = . + ç ÷
52. (b) Potential difference across the branch de is 6 V. Net 4p Î0 R 4p Î0 è R ø
JO

capacitance of de branch is 2.1 µF 1 æ 3Q ö 1 æ 2Q ö


So, q = CV + çè ÷ø = ç ÷
4p Î0 R 4p Î0 è R ø
Þ q = 2.1 × 6 µC
U

Þ q = 12.6 µ C 58. (c) Capacitance of capacitor (C) = 6 mF = 6 × 10–6 F;


Initial potential (V1) = 10 V and final potential
ED

Potential across 3 µF capacitance is


(V2) = 20 V. The increase in energy (DU)
12.6
V= = 4.2 volt 1 1 -6 2 2
3 = C( V22 - V12 ) = ´ (6 ´ 10 ) ´ [( 20) - (10) ]
2 2
Potential across 2 and 5 combination in parallel is
6 – 4.2 = 1.8 V = (3 ´10 -6 ) ´ 300 = 9 ´10 - 4 J .
So, q' = (1.8) (5) = 9 µC 59. (b) As the capacitors offer infinite resistance to steady
53. (a) Whenever a charge (+50 nC) is kept inside a hollow current so, the equivalent circuit is
metallic spherical shell, it induces an equal and
V R
opposite charge on the inner surface and an equal A B
and same type of charges on the outer surface.
\ Inside, induced charge is – 50 nC and outside, +50 V VC
nC – 150 nC already present. 2R A B
54. (a) Required ratio
2V I
1
Energy stored in capacitor CV 2 Using ohm’s law, current in circuit is
= = 2 ,
Workdone by the battery Ce 2 V
2V – V = I (2R + R) Þ I =
where C = Capacitance of capacitor 3R
V = Potential difference, The voltage drop across
e = emf of battery V 4
VAB = 2V - ´ 2R = V
3R 3
EBD_7179
460 PHYSICS

4 C1 = 6mF
VAB = V = V + VC 65. (d)
3
C3 C5 6mF
1 X A Y
Þ Voltage drop across C = V.. 6mF 20mF C2
3
C4
60. (a) The electrostatic pressure at a point on the surface of
6mF
s2 Equivalent circuit
a uniformly charged sphere =
2 Î0
6mF C2
s2 6mF
\ The force on a hemispherical shell = ´ pR 2 C1
m
6F
2 Î0
X C5 20mmF
6F Y
61. (c) Even after introduction of dielectric slab, direction of
electric field will be perpendicular to the plates and C3 6mF
directed from positive plate to negative plate. 6mF C4

s
Further, magnitude of electric field in air =
e0 C C
1 2
As C = C
s

.IN
3 4
Magnitude of electric field in dielectric =
K e0 Hence no charge will flow through 20mF
Similarly electric lines always flows from higher to lower AL C1 C2 C'
potential, therefore, electric potential inceases
continuously as we move from x = 0 to x = 3d.
X Y ÞX Y
(–) (+)
N

C3 C4 C''
R

C1 and C2 are in series, also C3 and C4 are in series.


U

Hence C' = 3 mF, C'' = 3 mF


C' and C'' are in parallel hence net capacitance
JO

x =0 x = d x=2d x =3d = C' + C'' = 3 + 3 = 6 mF


62. (c) The given circuit can be redrawn as follows 66. (c) Energy stored per unit volume
5mF 15mF
U

2
A C 1 1 æVö 1 V2 æ Vö
B = e0 E 2 = e0 ç ÷ = e0 çQ E = ÷
2 2 èdø 2 d2 è dø
ED

67. (a) Charge Q = C1V


Total capacity of combination (parallel) C = C1+ C2
2000V
Q C1V
æ 15 ö P.D. = =
(VA – VB) = ç ÷ ´ 2000 Þ V A – VB = 1500V C C1 + C 2
è 5 + 15 ø
Þ 2000 – VB = 1500V Þ VB = 500V 68. (a) We know that potential energy of two charge system


t t
is given by U = 1 q1q 2
V 1
63. (a) VR = 0 = V0 e RC Þ =e 10 4 p Î0 r
4 4
According to question,
t
t 1 ( +q )( -Q) 1 Qq
Þ4 =e Þ log e 4 = Þ t = 10log 4 = 13.86 s
10
UA = 4 p Î =-
10 0 a 4 pe 0 a
(RC = 2.5 × 106 × 4 × 10–6 = 10)
1 ( +q )( -Q) 1 Qq
1 1 and UB = 4 p Î =-
64. (c) C1V12 = C 2 V22 0 a 4 pe 0 a
2 2
DU = UB–UA = 0
because total energy is transferred (given). When known that for conservative force,
1 1 W = –DU = 0
\ ´ 900 ´ 10 -6 ´ 1002 = ´ 100 ´ 10 -6 ´ V 2 69. (c) We know that potential energy of discrete system of
2 2
\ V2 = 90000 Þ V = 300 V.. charges is given by
Electrostatic Potential and Capacitance 461

72. (a) q = CV1 cos wt


1 æ q1q 2 q2 q3 q3q1 ö
U= + +
4p Î0 çè r12 r23 r31 ÷ø Þi =
dq
= –w Cv1 sin w t
dt
According to question, 1
Also, w 2 = and V = V1 cos w t
1 æ q1q 2 q 2q3 q3q1 ö LC
Uinitial = + + At t = t1 , V = V2 and i = -wCV1 sin wt1
4p Î0 çè 0.3 0.5 0.4 ÷ø
V2
1 æ q1q 2 q 2q3 q3q1 ö \ cos w t1 = V (–ve sign gives direction)
Ufinal = + + 1
4p Î0 çè 0.3 0.1 0.4 ÷ø
1/ 2 1/ 2
æ V22 ö
C æ C (V12 - V22 ö
Hence, i = V1 ç 1 - = çè ÷ø
1 æ q 2 q 3 q 2 q3 ö L 2÷ L
Ufinal – Uinitial = - è V1 ø
4p Î0 çè 0.1 0.5 ÷ø
73. (d) The charge on each of two facing surfaces will be zero.
74. (d) Potential and potential energy are different quantities
1 q3 and cannot be equated.
= é10q q - 2q q ù = (8q 2 )
4p Î0 ë 2 3 2 3 û 4p Î0 75. (d) Two equipotential surfaces are not necessarily parallel
to each other.
70. (b)

.IN
EXERCISE - 3
AL Exemplar Questions
1. (d) As capacitor offers infinite resistance in dc-circuit.
So, current flows through 2W resistance from left to
right, given by
N
V 2.5V 2.5
I= = = =1A
R+r 2 + 0.5
R

2.5
Equivalent capacitance for three capacitors So, the potential difference across 2W resistance
U

(C1, C2 & C3) in series is given by


V = IR = 1 × 2 = 2 volt.
1 1 1 1 C 2C 3 + C 3C1 + C1C 2 Since, capacitor is in parallel with 2W resistance, so it
JO

= + + =
C eq. C1 C 2 C3 C1C 2C 3 also has 2V potential difference across it.
C1C 2 C 3 As current does not flow through capacitor branch so
Þ Ceq. = no potential drop will be accross 10W resistance.
U

C1C2 + C2 C 3 + C 3C1
The charge on capacitor
C(2C)( 3C) 6
ED

Ceq. = = C q = CV = (4 µF) × 2V = 8 µC
Þ C(2C) + (2C)(3C) + (3C) C 11 2. (c) The direction of electric field is always perpendicular
Þ Charge on capacitors (C1, C2 & C3) in series to the direction of electric field and equipotential
6C surface maintained at high electrostatic potential to
= Ceq V = V other equipotential surface maintained at low
11
Charge on capacitor C4 = C4V = 4C V electrostatic potential.
The positively charged particle experiences the
6C
V electrostatic force in the direction of electric field i.e.,
Charge on C2 11 6 1 3
= = ´ = from high electrostatic potential to low electrostatic
Charge on C4 4CV 11 4 22 potential. Thus, the work done by the electric field on
the positive charge, so electrostatic potential energy
C1 of the positive charge decreases because speed of
71. (d) In series, Ceff =
n1 charged particle moves in the direction of field due to
\ Energy stored, r
force qE.
1 1 C1 2 C1 3. (c) The work done (in displacing a charge particle) by a
ES = Ceff VS2 = 16V 2 = 8V electric force is given by W12 = q(V2 – V1). Here initial
2 2 n1 n1
In parallel, Ceff = n2 C2 and final potentials are same in all three cases are
equal (20V) and same charge is moving from A to B, so
1 work done is (DVq) same in all three cases.
\ Energy stored, Ep = n C V2
2 2 2 4. (c) As we know that the relation between electric field
2
8V C1 1 intensity E and electric potential V is
16C1
\ = n2C2V 2 Þ C2 = dV
n1 2 n1n2 E =-
dr
EBD_7179
462 PHYSICS

dV capacitor
Electric field intensity E = 0 then =0 K K (d + d )
dr K= 1 2 1 2
This imply that V = constant K1d 2 + K 2 d1
Thus, E = 0 inside the charged conducting sphere NEET/AIPMT (2013-2017) Questions
then the constant electrostatic potential 100V at every
where inside the sphere and it verifies the shielding 7. (a) Potential at B, VB is maximum
effect also. VB > VC > VA
5. (a) Here we have to findout the shape of equipotential As in the direction of electric field potential decreases.
surface, these surface are perpendicular to the field 1
lines, so there must be electric field which can not be 8. (c) Electric field, E µ
K
without charge.
As K1 < K2 so E1 > E2
So, the collection of charges, whose total sum is not
Hence graph (c) correctly dipicts the variation of
zero, with regard to great distance can be considered
as a point charge. The equipotentials due to point electric field E with distance d.
charge are spherical in shape as electric potential due 9. (b) Due to conducting sphere
to point charge q is given by At centre, electric field E = 0
q Q
V= Ke And electric potential V =
r 4p Î0 R
This suggest that electric potentials due to point charge r ¶V $ ¶V $ ¶V $

.IN
is same for all equidistant points. The locus of these 10. (d) E=- i- j- k
equidistant points which are at same potential, form ¶x ¶y ¶z
spherical surface.
= -[(6 - 8y)iˆ + (-8x - 8 + 6z)ˆj + (6y)k]
ˆ
The lines of field from point charge are radial. So the
AL r
equipotential surface perpendicular to field lines from At (1, 1, 1), E = 2i$ + 10j$ - 6k$
a sphere. r
N
6. (c) The capacitance of parallel plate capacitor filled with Þ (E) = 22 + 102 + 62 = 140 = 2 35
r
dielectric of thickness d1 and dielectric constant K1 is \ F = qE = 2 ´ 2 35 = 4 35
R

KeA
C1 = 1 o Q
U

d1 11. (c) Capacitance of the capacitor, C =


Similarly, capacitance of parallel plate capacitor filled V
JO

with dielectric of thickness d2 and dielectric constant After inserting the dielectric, new capacitance
K2 is C1 = K.C
K 2 eo A New potential difference
U

C2 =
d2 V
V1 =
ED

Since both capacitors are in series combination, then K


the equivalent capacitance is
1 1 1 1 2 Q2
= + ui = cv = (Q Q = cv)
C C1 C 2 2 2C
K1e 0 A K 2 e 0 A
Q 2 Q 2 C2 V 2 æ ui ö
C1C2 d1 d2 uf = = = =ç ÷
or C= = Ke A K e A 2f 2kc 2KC è k ø
C1 + C2 1 0 + 2 0
d1 d2 1 2ì1 ü
Du = uf – ui = cv
í – 1ý
K1K 2 e 0 A 2 îk þ
C= ... (i)
K1d 2 + K 2d1 As the capacitor is isolated, so change will remain
So multiply the numerator and denominator of equation conserved p.d. between two plates of the capacitor
(i) with (d1 + d2) Q V
K1K 2 e 0 A (d + d ) L= =
C= ´ 1 2 KC K
( K1d 2 + K 2d1 ) ( d1 + d 2 ) 12. (a) Potential in a region
V = 6xy – y + 2yz
K1K 2 ( d1 + d 2 ) e0A
= ´ ... (ii) As we know the relation between electric potential
( K1d 2 + K 2d1 ) ( d1 + d 2 )
ur -dV
So, the equivalent capacitances is and electric field is E =
dx
Ke 0 A
C= ... (iii)
(d1 + d 2 ) ur æ ¶V ˆ ¶V ˆ ¶V ˆ ö
Comparing, (ii) and (iii), the dielectric constant of new E = ç ¶x i + ¶y j + ¶z k ÷
è ø
Electrostatic Potential and Capacitance 463

ur 15. (a) When battery is replaced by another uncharged


E = éë (6yiˆ + (6x - 1 + 2z)ˆj + (2y)kˆ ùû capacitor
® C q
E (1,1,0) = -(6iˆ + 5jˆ + 2k)
ˆ
13. (a) Force of attraction between the plates,
F = qE
s q
= q´ =q
C
2 Î0 2A Î0
As uncharged capacitor is connected parallel
q2 c2 v2 cv2 So, C' = 2C
= = =
æÎ A ö 2cd 2d
2ç 0 ÷´ d q1 + q 2
è d ø and Vc =
C1 + C2
Î0 A
Here, c = , q = cv, A = area q+0 V
d Vc = Þ Vc =
14. (d) When S and 1 are connected C+C 2
The 2mF capacitor gets charged. The potential 1

.IN
difference across its plates will be V. Initial Energy of system, Ui = CV 2 … (i)
2
The potential energy stored in 2 mF capacitor
1 1 2
æVö
U i = CV 2 = ´ 2 ´ V 2 = V 2
AL 1
Final energy of system, Uf = (2C) ç ÷
2 2 2 è2ø
When S and 2 are connected
N
The 8mF capacitor also gets charged. During this 1 æ1ö
= CV 2 ç ÷ …(ii)
charging process current flows in the wire and some 2 è2ø
R

amount of energy is dissipated as heat. The energy


loss is From equation (i) and (ii)
U

1 C1 C2 2 1
DU = 2 C + C (V1 - V2 ) Uf = Ui
JO

1 2 2
Here, C1 = 2mF, C2 8 mF, V1= V, V2 = 0 i.e., Total electrostatic energy of resulting system
decreases by a factor of 2
U

1 2´8 4
\ DU = ´ (V - 0) 2 = V 2 16. (a) As the regions are of equipotentials, so Work done
2 2+8 5
ED

W = qDV
The percentage of the energy dissipated DV is same in all the cases hence work - done will also
4 2 be same in all the cases.
DU V
= ´ 100 = 5 ´ 100 = 80%
Ui V2
EBD_7179
464 PHYSICS

18
ELECTRIC CURRENT
Current Electricity

Its SI unit is Am–2


It is the rate of flow of charge through any cross section. Current density can also be related to electric field as
dq r
i.e. I = r r E
dt j = sE =
Conventionally, the direction of flow of positive charge is taken r

.IN
as the direction of electric current. It is a scalar quantity and its where s is conductivity of the substance & r is specific
S.I. unit is ampere (A). resistance of the substance.
Keep in Memory
AL r
J is a vector quantity and its direction is same as that of E .
(i) Current carriers in conductor are electrons, (valence e–s) Dimensions of J are [M°L–2T°A]
ions in electrolytes, electrons & holes in semiconductor
Keep in Memory
N
and positive ions /electrons in gases.
(ii) Charge of electron = 1.6 × 10–19c (i) Electric current is a macroscopic physical quantity where
R

(iii) 1 ampere = 6.25 × 1018 electrons/sec as current density is a microscopic physical quantity.
(iv) Though direction is associated with current (opposite to
U

the motion of electrons), but it is not a vector quantity as it (ii) For a given conductor current does not change with change
does not follow rules of vector addition. in cross-sectional area.
JO

(v) For a current to flow through a cross-section, there must be uuur


a net flow of charge through that cross-section.
In a metal like copper there are around 1028 free electrons
DRIFT VELOCITY Vd ( )
U

per m3 moving randomly in all direction with speeds of the When the ends of a conductor are connected to the two terminals
order of 106 m/s even in the absence of electric field. But of a battery, an electric field is set up in the conductor from the
ED

since the number of electrons passing through a cross- positive terminal to the negative terminal. The free electrons in
section from left to right is equal to the number of electrons
the conductor experiences a force opposite to the direction of
passing from right to left in a given time, therefore the net
charge flow is zero and hence the electric current is zero. the electric field and hence get accelerated. However this process
(vi) A conductor remains uncharged when current flows in it. of acceleration is soon interrupted by collision with ions of solid.
i.e. Net charge in a current carrying conductor is zero. The average time for which each electron is accelerated before
suffering a collision is called the mean free time or mean
CURRENT DENSITY
relaxation time.
Current density at a point inside a conductor is defined as the Thus, the free electrons within the metal, in addition to its random
amount of current flowing per unit cross sectional area around motion acquire a small velocity towards the positive end of
that point of the conductor, provided the area is held in a
conductor. This velocity is called drift velocity. It is given by
direction normal to the direction of current.
ur
I r eEt
i.e. Current density, J = vd = – ,
A m
If area is not normal to current, then area normal to current is A' = where e is the charge and m is the mass of electron.
A cos q (see the figure) ur
A' E is the electric field established in conductor and t is the
A average relaxation time.
r r
q Negative sign is because the directions of E and vd (for electron)
q J are opposite.
V
E=
I r r r rr l
J= or I = J A cos q or I = J . A = ò J . A
A cos q
Current Electricity 465

4. Variation of drift velocity :


l
+ E – E 1 V
Vd = = ; Vd µ E
Vd e– I ne ne l
J when length is doubled, vd becomes half and
when V is doubled, vd becomes twice.
Example 1.
The current in a wire varies with time according to the
+V – relation i = 4 + 2t2
where V is the potential difference across ends of the conductor How many coulomb of charge pass a cross-section of wire
of length l. The uniform current I, flowing through the conductor in time interval t = 5s to t = 10s?
is given by Solution :
I = n e A vd
dq
where n = number of free electrons per unit volume, i= Þ dq = i dt Þ dq = (4 + 2 t2 ) dt
A = area of cross-section, vd = drift velocity dt
ur r 10
In vector form, Þ J = - nev d
10 é 3ù
The negative sign is because the direction of drift velocity of t
r On integrating q = ò (14 + 2t 2 )dt = ê14t + 2 ú
electron is opposite to J . êë 3 úû
5 5

.IN
Mobility - Drift velocity per unit electric field is called mobility. = 603.33 C
It is denoted by µ. Example 2.
r
Vd r æ 4 A ö ˆ
m=
AL The current density at a point is J = çè 2 ´ 10 2 ÷ø j . Find
E m
Its S.I. unit is m2/volt-sec. r
the rate of charge flow through a cross-sectional area S
N
Keep in Memory such that
r
R

1. Drift velocity is very small, it is of the order of 10–4m/s (i) S = (2cm 2 )ˆj ,
which is negligible as compared to thermal speed of e–s at
U

r r
room temperature (;105 m/s) (ii) S = (4cm 2 ) ˆi and (iii) S = (2iˆ + 3jˆ cm 2 )
JO

2. The drift velocity is given by


Solution :
J The rate of charge flow = current
vd = ,
ne r r
i = JdS
U

I
where, J = current density =
A (i) Current i = (2 ´ 104 A / m 2 ) ˆj.(2 ´ 10 -4 m 2 )ˆj = 2A
ED

e = electronic charge = 1.6 × 10–19 C


n = the number of free electrons per unit volume [using ˆj.jˆ = 1]
3. The number of free electrons per unit volume (n) can be
determined by the following relation : (ii) Current i = (2 ´ 10 4 A / m 2 ) ˆj.(4 cm 2 ) iˆ = 0
No. of free e – per atom M [using ˆj.iˆ = 0]
n= and volume= .
Volume N0 d
ˆ ´ 10 -4 m 2 = 6A
(iii) Current i = (2 ´ 104 A / m2 ) ˆj.(2iˆ + 3j)
N d
n = 0 ´ x,
M
where N0 = Avogadro number OHM’S LAW AND ELECTRICAL RESISTANCE
d = density of the metal When a potential difference is applied across the ends of a
conductor, a current I is set up in the conductor.
M = molecular weight
According to Ohm’s law “Keeping the given physical conditions
and x = number of free electrons per atom such as temperature, mechanical strain etc. constant, the current
3. For steady current : (I) produced in the conductor is directly proportional to the
I I I potential difference (V) applied across the conductor”.
J= ; E= ; Vd = i.e., I µ V or I = KV ... (1)
A sA neA
where K is a constant of proportionality called the conductance
This means that for a given material and steady current in
of the given conductor.
case of non-uniform cross-section of material
Alternatively, V µ I or V = RI ... (2)
1 1 1 where the constant R is called the electrical resistance or simply
J µ ; E µ ; Vd µ
A A A resistance of the given conductor.
EBD_7179
466 PHYSICS

From above two eqs. it is clear that R = 1/K. (ii)Resistance is the property of object while resistivity is
If a substance follows Ohm’s law, then a linear relationship exists the property of material.
between V & I as shown by figure 1. These substance are called Materials and their resistivity
Ohmic substance. Some substances do not follow Ohm’s law,
Material Resistivity (r) (at 0°c)
these are called non-ohmic substance (shown by figure 2) (in W m)
Diode valve, triode valve and electrolytes, thermistors are some
(i) Silver 1.6 × 10–8
examples of non-ohmic conductors.
(ii) Copper 1.7 × 10–8
V V (iii) Aluminium 2.8 × 10–8
(iv) Tungsten 5.2 × 10–8
I (v) Platinum 10.6 × 10–8
(vi) Manganin 42 × 10–8
q (vii) Carbon 35 × 10–6
I (viii) Germanium .46
Ohmic conductor Non-linear conductor
or linear conductor or non-ohmic conductor (ix) Silicon 2300
Fig. 1 Fig. 2 (x) Glass ~ 1013
(xi) Mica ~ 2 × 1015
Slope of V-I Curve of a conductor provides the resistance of the
conductor COMMON DEFAULT

.IN
l
V Since R = r Þ Rµl
slope = tan q = A
I It is incorrect to think that if the length of a resistor is doubled
AL
The SI unit of resistance R is volt/ampere = ohm (W) its resistance will become twice.
Electrical Resistance If you look by an eye of physicist you will find that when l
On application of potential difference across the ends of a change, A will also change. This is discussed in the following
N
conductor, the free e–s of the conductor starts drifting towards article.
Case of Reshaping a Resistor
R

the positive end of the conductor. While drifting they make


collisions with the ions/atoms of the conductor & hence their On reshaping, volume of a material is constant.
U

motion is obstructed. The net hindrance offered by a conductor i.e., Initial volume = final volume
to the flow of free e–s or simply current is called electrical or, Ai li = Af lf ... (i)
JO

resistance. where li, Ai are initial length and area of cross-section of resistor
It depends upon the size, geometry, temperature and nature of and lf, Af are final length and area of cross-section of resistor.
the conductor. If initial resistance before reshaping is Ri and final resistance
U

after reshaping is Rf then


Resistivity : For a given conductor of uniform cross-section A
and length l, the electrical resistance R is directly proportional to l
r i
ED

length l and inversely proportional to cross-sectional area A Ri Ai l A


= = i ´ f ... (ii)
Rf l l f Ai
l rl RA r f
i.e., R µ or R = or ρ = Af
A A l 2
r is called specific resistance or electrical resistivity. R i æ li ö
From eqs. (i) and (ii), =ç ÷ Þ R µ l2
R f çè lf ÷ø
m
Also, r = This means that resistance is proportional to the square of the
ne 2t
length during reshaping of a resistor wire.
The SI unit of resistivity is ohm - m. 2
R i æ Af ö 1
1 Also from eqs. (i) and (ii), = çç ÷÷ Þ R µ
Conductivity(s) : It is the reciprocal of resistivity i.e. s = . R f è Ai ø A2
r
This means that resistance is inversely proportional to the square
The SI unit of conductivity is Ohm–1m–1 or mho/m. of the area of cross-section during reshaping of resistor.
Ohm’s law may also be expressed as, J = sE Since A = p r2 (for circular cross-section)
where J = current density and E = electric field strength 1
\R µ 4
ne 2 t r
Conductivity, s = where n is free electron density, t is where r is radius of cross section.
m
Effect of Temperature on Resistance and Resistivity
relaxation time and m is mass of electron.
Resistance of a conductor is given by Rt = R0 (1 + aDt)
(i) The value of r is very low for conductor, very high for
insulators & alloys, and in between those of conductors & Where a = temperature coefficient of resistance and Dt = change
insulators for semiconductors. in temperature
Current Electricity 467

For metallic conductors : If r1 and r2 be resistivity of a SERIES AND PARALLEL COMBINATION OF RESISTORS
conductor at temperature t1 and t2, then
Resistances in Series
r2 = r1 (1 + a D T) R1 R2
where a = temperature coefficient of resistivity and When a number of resistances are
where DT = t2 – t1 = change in temperature joined end to end so that same current
The value of a is positive for all metallic conductors. flows through each, resistor when some I V1 V2
\ r2 > r1 potential difference is applied across V
In other words, with rise in temperature, the positive ions of the the combination, the conductor are
metal vibrate with higher amplitude and these obstruct the path said to be connected in series.
of electrons more frequently. Due to this the mean path decreases The equivalent resistance in series is given by
and the relaxation time also decreases. This leads to increase in (Req)s = R1 + R2 + ...+ Rn
resistivity.
Equivalent resistance of same
1 -1 R1 R2
Please note that the value of a for most of the metals is K resistances connected in series
273
is always greater than the V1 V2
For alloys : In case of alloys, the rate at which the resistance
greatest of individual resistance.
changes with temperature is less as compared to pure metals.
For example, an alloy manganin has a resistance which is Potential division rule in series
30-40 times that of copper for the same dimensions. combination :
V

.IN
Also the value of a for manganin is very small » 0.00001°C–1.
VR1 VR 2
Due to the above properties manganin is used in preparing wires V1 = ; V2 =
for standard resistance (heaters), resistance boxes etc. AL R1 + R 2 R1 + R 2
Please note that eureka and constantan are other alloys for which
Resistances in Parallel
r is high. These are used to detect small temperature, protect
picture tube/ windings of generators, transformers etc. Two or more resistors are said to be connected in parallel if the
N
For semiconductors : The resistivity of semi-conductors same potential difference exits across all resistors.
I1 R1
decreases with rise in temperature. For semi conductor the value
R

I
of a is negative. R2
U

m I2
r=
JO

ne 2 t
With rise in temperature, the value of n increases. Please note
that t decreases with rise in temperature. But the value of increase V
U

in n is dominating for the value of r in this case. The equivalent resistance is given by
For electrolytes : The resistivity decreases with rise in
ED

1 1 1 1
temperature. This is because the viscosity of electrolyte decreases = + + ... +
with increase in temperature so that ions get more freedom to (R eq )p R1 R 2 Rn
move.
The equivalent resistance in a parallel combination is always less
For insulators : The resistivity increases nearly exponentially
than the value of the least individual resistance in the circuits.
with decrease in temperature. Conductivity of insulators is almost
zero at 0 K. Current division rule in parallel combination
Superconductors : There are certain materials for which the IR 2 IR1
resistance becomes zero below a certain temperature. This I1 = ; I2 =
R1 + R 2 R1 + R 2
temperature is called the critical temperature. Below critical
temperature the material offers no resistance to the flow of e–s. In a given combination of resistors, when you want to detect
The material in this case is called a superconductor. The reason whether the resistances are in series or in parallel then see that if
for super conductivity is that the electrons in superconductors the same current flows through two resistors then these are in
are not mutually independent but are mutually coherent. This series and if same potential difference is there across two resistors
coherent cloud of e–s makes no collision with the ions of super- then these are in parallel potential diff across each resistor is the
conductor and hence no resistance is offered to the flow of e–s
same & is equal to the applied potential difference.
For example, R = 0 for Hg at 4.2 K and R = 0 for Pb at 7.2 K. These
substances are called superconductors at that critical temperature. HOW TO FIND EQUIVALENT RESISTANCE ?
Super conductors ar e used (a) in making very strong Successive Reduction
electromagnets, (b) to produce very high speed computers This method is applicable only when the resistors can be clearly
(c) in transmission of electric power (d) in the study of high
identified as in series or parallel. Let us take some example to find
energy particle physics and material science.
resistance between ends A and B
EBD_7179
468 PHYSICS

3W 3W 33 W 3W

6W 3W
Ex. (i) 3W = 6W 6W 3W = 6W
6W
3W 33 W 3W
6W 66 W 6W

A 3W B AA 33 W B
B 3W B A 3W B
6W
3W 3W 2W
= =
A 3W B A 3W B
7
10 10
5
Ex. (ii) 3 5 5
5 5
10 = 10 = A B
= =
A B A B A B
10 A B 10
10 10

.IN
Ex. (iii) =
AL = =
N
R
U

Ex. (iv) =
JO

= =
U

Please note that all points on the circumference are at same potential as there is no resistance on circumference.
ED

Ex. (vi) Infinite series : one unit to the sum of infinite units, then it will be
approximately the same.
r r
X´r
B \ X = 2r +
to ¥ r r X+r
r r A Solve the equation as a normal algebraic equation to find
X.
Using Symmetry of the Circuit
We observe that there is a repetitive unit extending to infinity Axis symmetry :
on left hand side. We assume that the equivalent resistance
Ex.(i) The circuit shown in figure is symmetrical about XAEBY
of all the unit except one (shown dotted) is equal to X ohm.
axis. This is because the upper part of the axis is the mirror
The equivalent circuit will be as shown below.
image of lower part (resistors and current direction both)
B C
r
r
r r r
X r r
X A E B Y
r r
r A r
The equivalent resistance across A and B is D
X´r \ IAC = IAD;ICB = IDB ;
R AB = r + +r IAE = IEB ( Q VC < VE < VD wheatstone
X+r
Please note that RAB can be taken as X because if you add bridge principle)
Current Electricity 469

Þ ICE = IED = 0 The diagram given above is symmetrical but the positions
Therefore the circuit can be redrawn. It is now easier to find of the resistances are shifted. Let I be the current in the
resistance between X and Y. circuit from A. The same leaves the circuit at C. Let current
in AB, AD and AE be I1 , I2 and I3 respectively. Since the
C
same current flows in AE and EC, the detached equivalent
r r circuit can be drawn as
r r B

X A E B Y 10 W 5W
I1 2.5 W

r r X I A I2 E D Y
2.5 W I1
5W 10 W
D I3
Ex.(ii) The circuit shown is symmetrical about axis XY. Therefore C
VB = VH ; VC = VI = VG; VD = VF
5W 5W
C
Keep in Memory
r r D
B Equivalent resistance between A and B of the resistors connected
r r as shown in the figure
r r

.IN
X A r r I r r EY
R1 R2
H r r F AL A B
G
Therefore the circuit can be redrawn as R2 R1
N
r r
r Wheatstone bridge
r
R

r r
R 1 (R1 + 3R 2 )
B C D
I R AB =
R 2 + 3R 1
U

X H r
r G r F r Y
JO

r r
3. Path symmetry :
Ex.(iii) The circuit is asymmetric about the dotted line All paths from one point to another which have the same
\ IBG = IGC; IFG = IGE and IAG = IGB setting of resistances have the same amount of currents.
U

Example :
B C C
ED

B
r I1 I1 I
I1
A r r r
r D I A I1 D

X r r r r Y
r G I – 2I1

F r E I – 2I1
F
Therefore the equivalent circuit is G

B r C E H
r Twelve wires each having resistance r are joined to form a cube.
A r r r We have to find the equivalent resistance across A and B.
D
X r r r r Y By path symmetry, IAB = IBC = IAD = IDC = I
r \ IAE = I– 2I1 Þ IGC = I– 2I1,
F E Since current in AB = current in BC
2. Shifted symmetry : Þ IBF = 0
Also IAD = IDC Þ IDH = 0
B The equivalent circuit will be as shown. The resistance now clearly
visible as in series and in parallel.
R1 R2
2.5 W r
I1 I2 r I
I3 5 W 5W r
I r
X I A E C Y r
2.5 W I1
I2 r
R2 R1 r
r
r
D r
EBD_7179
470 PHYSICS

4. Star -delta connection : IA = IB = IC = ID = 1 amp


B VA = VB = +5V
r1 B
VC = VD = 0V
Rb This means that a potential drop of 5V takes place across
A r2 =
A Ra the resistor
r3 Rc 3. O Incorrect : If two resistances are not in series then it
C is in parallel and vice-versa.
C
r1r3 r1r2 r2 r3 P Correct : The above thinking is incorrect. We may
Ra = ; Rb = ; Rc = ; have resistances which are neither in series nor in
r1 + r2 + r3 r1 + r2 + r3 r1 + r2 + r3
parallel.
B
B r
Rb 1
Colour Coding for Carbon Resistor and their Standard
Ra
= A r2 Values –
A (i) It is a system of colour coding used to indicate the values
Rc r3 of resistors.
C (ii) For the fixed, moulded composition resistor, four colour
C
bands are printed on one end of the outer casing as shown
R R + R b R c + R cR a
r1 = a b ; below.

.IN
Rc
4
R a R b + R b R c + R cR a 3
r2 = ; 1
2
Ra
AL
R a R b + R b R c + R cR a
r3 = ;
Rb
N
Using delta to star conversion Tolerence
R

3W 2W 2W Multiplier
U

= Significant
2W 2W 9W 9W
digits
3W 3W 9W
JO

2W 2W (iii) The colour bands are always read left to right from the end
X Y X Y that has the bands closest to it.
If none of the above method works then we may use Kirchhoff’s (iv) (a) The first and second colour bands, represent the first
U

method which will be discussed later and second significant digits respectively, of the
resistance value.
ED

COMMON DEFAULTS
(b) The third colour band is for the number of zeros that
1. Resistors are not just in series or in parallel if they look so follow the second digit.
geometrically, e.g. the resistors in the diagram are not in (c) In case the third band is gold or silver, it represents a
parallel but in series. multiplying factor of 0.1 or 0.01.
A (d) The fourth band represents the manufacture’s
tolerance. It is a measure of the precision with which
the resistor was made.
(e) If the fourth band is not present, the tolerance is
assumed to be ± 20%.
B
These resistors across A and B are in series, as same current (v) Standard value of colour codes for carbon resistors
passes through them.
Colour Digit Multiplier Tolerance
2. This is a common thinking that current which comes out
from the positive terminal of a battery is used up till it reaches Black 0 100 = 1
the negative terminal. But infact the current remains the Brown 1 101 = 10
same in a branch. In fact a potential drop takes place across Red 2 102
a resistor. Orange 3 103
B 5W C
Yellow 4 104
Green 5 105
1 amp 1 amp
Blue 6 106
Violet 7 107
A +5V 0V D
Current Electricity 471

Grey 8 108 ELECTRIC POWER


It is defined as the rate at which work is done in maintaining
White 9 109
the current in electric circuit.
Gold – 0.1 ± 5% Electric power, P = VI = I2 R = V2 / R watt or joule/second.
Silver – 0.01 ± 10% Electric energy : The electric energy consumed in a circuit is
No colour – – ± 20% defined as the total work done in maintaining the current in an
electric circuit for a given time.
To learn the above table of colour codes of resistors let us Electric energy = VIt = Pt = I2 Rt = V2 t / R
learn this interesting sentence : The S.I. unit of electric energy is joule (denoted by J)
BB ROY of Great Britain has a Very Good Wife. where 1 joule = 1 watt × 1 second = 1 volt × 1 ampere × 1 sec.
In the above sentence the capital letters have the following In household circuits the electrical appliances are connected in
meaning : parallel and the electrical energy consumed is measured in kWh
(kilo watt hour).
B Þ Black B Þ Brown 1 kWh (1 B.O.T. unit) = 1000 Wh = 3.6 × 106 J
R Þ Red O Þ Orange
Keep in Memory
Y Þ Yellow G Þ Green
V Þ Violet G Þ Grey For Series Combination :
W Þ White 1. If resistances (or electrical appliances) are connected in
Remember the colour in the above order and the series, the current through each resistance is same. Then
corresponding digits from 0 to 9 and also the multiplier with power of an electrical appliance

.IN
the power to 10 from 0 to 9. P µ R and V µ R (Q P = i 2 Rt )
Commercial resistors are of two types It means in series combination of resistances, the potential
(a) Wire round resistor made by winding of wires of an difference and power consumed will be more in larger
alloy manganins, constantan and nichrome.
AL
resistance.
(b) Carbon resistors have low cost and are compact. 2. (i) When the appliances of power P 1, P2, P3 ... are
THERMISTOR connected in series, the effective power consumed
N
A thermistor is a heat sensitive resistor usually made up of (P) is
R

semiconductor. The oxides of various metals such as mickel, 1 1 1 11


iron, copper etc. temperature coefficient of thermistor is –ve but = + +
...+ i.e., the effective power is
U

P P1 P2 P3 Pn
is usually large, of the order of 0.04/ºC.
The V–I curve of thermistor is as shown.
JO

less than the power of individual appliance.


(ii) If n appliances, each of equal resistance R, are
connected in series with a voltage source V, the power
U

dissipated Ps will be
I
V2
ED

Ps = ... (i)
nR
3. When two lamps of different wattage are connected in series
in a house the lamp of lower wattage glows more brightly.
V For Parallel Combination :
1. If resistances (i.e. electrical appliances) are connected in
Thermistors are used for resistance thermometer in very low parallel, the potential difference across each resistance is
temperature measurement of the order of 10K and to safeguard
same. Then P µ 1 / R and I µ 1 / R .
electronic circuits against current jumps because initially
It means in parallel combination of resistances the current
thermistors has high resistance when cold and its resistance
and power consumed will be more in smaller resistance.
drops appreciably when it heats up. 2. When the appliances of power P1, P2, P3.... are in parallel,
JOULE’S LAW OF HEATING the effective power consumed (P) is
It states that the amount of heat produced in a conductor is directly P = P1 + P2 + P3 ... +Pn
proportional to the i.e. the effective power of various electrical appliances is
(i) square of the current flowing through the conductor, more than the power of individual appliance.
(q, T – constt) i.e. H µ i2 3. If n appliances, each of resistance R, are connected in parallel
(ii) resistance of the conductor (i, T – constt.) with a voltage source V, the power dissipated Pp will be
i.e. HµR V2 nV 2
PP = = ... (ii)
(iii) time for which the current is passed (i, R, – constt) (R / n ) R
i.e., Hµt From eqns. (i) and (ii),
i 2 RT PP
Thus H = i2 RT joule = cal = n 2 or PP = n2 PS
4.2 PS
EBD_7179
472 PHYSICS

This shows that power consumed by n equal resistances in 14. A lead-acid secondary cell is discharged if the relative
parallel is n2 times that of power consumed in series if density of electrolyte drop to 1.18 and e.m.f. of 1.8V.
voltage remains same. 15. Hot wire galvanometer is based on heating effect of current.
4. In parallel grouping of bulbs across a given source of voltage, Its deflection q is directly proportional to the heating effect
the bulb of greater wattage will give more brightness and will (i.e. I2). It works on A.C. as well as D.C.
allow more current through it, but will have lesser resistance 16. Two wires of same material and same length but having
and same potential difference across it. different diameters connected in parallel produce more heat
5. For a given voltage V, if resistance is changed from R to than when connected in series. i.e., Hparallel>Hseries.
R/n, power consumed changes from P to nP. 17. If t1and t2 be the time taken by two heaters to boil a given
P = V2 / R; when R´ = R/n, mass of a liquid, then the time taken to boil the same amount
then P´ = V2 / (R/n) = n V2 / R = nP. of the liquid, when both heaters are connected in parallel is
6. Filament of lower wattage bulb is thinner than that of higher given by
wattage bulb i.e. filament of 60 watt bulb is thinner than
that of 100 watt bulb. t1t2
tp <
7. If I is the current through the fuse wire of length l, radius r, t1 ∗ t2 .
specific resistance r and Q is the rate of loss of heat per 18. If t1and t2 be the time taken by two heaters to boil a given
unit area of a fuse wire, then at steady state, amount of a liquid, then the time taken to boil the same
I 2rl amount of the liquid, when both heaters are connected in
I2R = QA or = Q ´ 2prl series is given by

.IN
pr 2
ts = t1 + t2 .
2p 2 Q 3 19. If I be the current at which a fuse wire of radius R blows,
or I2 = r Þ I a r 3/ 2
r
AL I2
Hence current capacity of a fuse is independent of its length then 3 constant. i.e., for two fuse wires of radii R1and R2
and varies with its radius as r3/2. R
N
8. If t1 and t2 are the time taken by two different coils for and maximum bearing current I1, and I2, we have
producing same heat with same supply, then
R

(i) If they are connected in series to produce same heat, I12 I22
3
< .
time taken, t = t1 + t2 R1 R23
U

(ii) If they are connected in parallel to produce same heat,


20. If the two resistors R1and R2 are first connected in series
JO

t1 t 2 and then parallel then the ratio of heat produced in the two
time taken is, t =
t1 + t 2 cases (series to parallel) is given by
9. When a bulb glows the temperature of the filament is of the
U

Hs (R1 ∗ R2 )2
order of 3000K. <
To avoid lengthy calculations use P = I2R in series Hp R1 R2
ED

10.
V2 21. If two resistances R1 and R2 are connected in parallel and a
combination and P = in parallel combination and current is passed in them such that heat produced in them
R
is H1 and H2 respectively, then
P = VI when we want to find power of a device and V and I
H1 R
are known. < 2.
11. The resistance of an appliance, can be formed by rated H2 R1
2
Vrated Example 3.
voltage and power is R =
Prated The temperature coefficient of resistance of a wire is
Now this resistance does not change (remember resistance 0.00125ºC–1.At 300 K its resistance is one ohm. At what
depends only on the parameters of resistors and not on the temperature the resistance of the wire will be 2 ohm?
voltage across it or the current flowing through it, if we Solution :
neglect the changes occurring due to change in From formula Rt = R0 (1 + µDt)
temperature). R 300 = R 0 (1 + a ´ 27) = 1
12. The maximum current that can be safely passed through an
Prated
R t = R 0 (1+ µ D ´ t) = 2
appliance (a resistor) is I = .
Vrated 1 + 27 a 1
\ =
13. Bulbs get fused sometimes when switched on. This is 1 + at 2
because with the rise in temperature the resistance increases or 2 + 54a = 1 + at
and power decreases (P = V2 / R). Therefore the bulb glows Þ 2 + 54(0.00125) = 1 + (0.00125)t
brighter in the beginning and get fused. \ t = 854ºC = 1127 K
Current Electricity 473

Example 4. Solution :
A cylindrical copper rod is reformed to twice its original 8W, 16W, and 16W resistances are connected in parallel.
length with no change in volume. The resistance between Their equivalent resistance is given by R’
its ends before the change was R. Now its resistance will 1 1 1 1 2 +1+ 1 4
be = + + = =
(a) 8 R (b) 6 R R ' 8 16 16 16 16
(c) 4 R (d) 2 R \ R’ = 4W.
Solution : (c) This is in series with 20W. So upper part of AB has a resistance
4 + 20 = 24 W ...(1)
r l1 ...(1) For lower part, 9W and 18W resistances are connected in
R=
pr12 parallel. Their equivalent resistance R” is given by
Now the rod is reformed such that 1 1 1 2 +1 3
= + = = Þ R ¢¢ = 18 / 3 = 6W
l2 = 2l1 and pr12l 1 = pr22l 2 (no change in volume) R ¢¢ 9 18 18 18
This is in series with 6W. So the resistance of lower part of
or (r12 / r22 ) = (l 2 / l 1 ) AB is 6 + 6 = 12W ...(2)
The upper and lower parts of AB are in parallel. Hence the
rl2 equivalent resistance between A and B is given by
\ R2 = ...(2)
pr22 1 1 1 1+ 2 3
= + = = Þ R = 24/3 = 8W
From eqns. (1) and (2), we get R 24 12 24 24
l r2 l l

.IN
R 1 1
< 1 ≥ 22 < 1 ≥ 1 < ≥ Example 7.
R2 l 2 r1 l2 l2 2 2 A wire has a resistance of 10W. It is stretched by one-tenth
\ of its original length. Then its resistance will be
R2 = 4 R
AL
Example 5. (a) 10W (b) 12.1W
A copper wire is stretched to make 0.1% longer. What is (c) 9W (d) 11W
the percentage change in its resistance?
N
Solution : (b)
Solution : Here volume remains constant. Thus
R

rl V1 = V2 Þ l1 ´ A1 = l 2 ´ A2
The resistance R of a wire is given by R = , where
A
U

r = specific resistance 102


(pr12 )
pr12 l = pr22 (11l / 10) or pr2 =
Let d and m be the density and mass of the wire, respectively
JO

11
then A l d = m or A = m/ld {Q When wire is stretched by 1/10 of its original length,
r l ´l d r l2 d æ r d ö 2 the new length of wire becomes (11l/10)}
\ R= = =ç ÷´ l
U

m m èmø l2
Let the new resistance be R2. Then R2 = r .
A2
ED

Taking log, of both sides, we get


æ rd ö
loge R = log e ç ÷ + 2e loge l æ 11 ö
r.ç l ÷
è mø
10 ø (11 / 10)rl
R2 = è =
dR 2 dl æ rd ö pr22 (10 / 11)pr12
Differentiating = çèQ = constant ÷
ø
R l m
dR æ dl ö (11 / 10) é rl ù (11 / 10) 121
´100 = 2ç ´100 ÷% = 2 ´ (0.1) = (10 / 11) ê 2 ú = (10 / 11) ´10 = 10 = 12.1W
R è l ø p
ëê 1 ûú
r
\ Percentage change in resistance = 2×(0.1) % = 0.2%
So the resistance increases by 0.2%. Example 8.
What will be the equivalent resistance between the two
Example 6.
Determine the equivalent resistance of the arrangement points A and D of fig?
10W 10W 10W
of resistances shown in fig between the points A and B. A C
8W R1 R2
16W R6 10W R3 10 W
R5 R4
16W 20W B D
A B 10W 10W 10W
Solution :
9W Resistances R2 and R3 are in series.
6W
18W
\ R' = R2 + R3 = 10 + 10 = 20 W
EBD_7179
474 PHYSICS

Similarly, R5 and R6 are in series. Using r = r0 [1 + a (Dt)] , the resistivity r at a temperature


\ R'' = R5 + R6 = 10 + 10 = 20 W
80°C will be
Further, R' and R'' are in parallel
r = 1.6 ´ 10-8 W - m [1 + (4.1 ´ 10 -3 °C -1 ) (80°C)]
R ¢R ¢¢ 20 ´ 20
\ R ¢¢¢ = = = 10W
R ¢ + R ¢¢ 20 + 20 = 2.1 ´ 10 -8 W - m
Resistance between A and D = R1 + R'''+ R4 =
10 + 10 + 10 = 30W Example 12.
Example 9. A bulb has voltage rating of 220 V and power rating of
A wire 1 m long has a resistance of 1 W. If it is uniformly 40 W. How can this bulb be made to glow with normal
stretched, so that its length increases by 25%, then its brightness if a voltage source of e.m.f. 330 V is available?
resistance will increase by Solution :
Here V = 220 V; P = 40 W
(a) 25% (b) 50%
(c) 56.25% (d) 77.33% V 2 (220) 2
Resistance of the bulb, R = = = 1210 W
Solution : (c) P 40
330
New length, l¢ = l + 25 l = 125 l ; Current in the circuit, I =
1210 + S
100 100 Potential difference across the bulb,
Let new area of cross-section = A ¢ . Then

.IN
æ 330 ö
IR = ç 1210
A l d = A ¢ l¢ d or A¢ = A l / l¢ (Volume is constant) è 1210 + S ÷ø
330 ´ 1210
As per question, 220 =
æ 125 ö 100
AL
or A¢ = A ´ l / ç ´ l÷ = A 1210 + S
è 100 ø 125 On solving, we get S = 605 W
Hence to glow bulb, a resistance of 605W should be
N
rl r l¢ connected in series.
R= and R ¢ =
A A¢ Example 13.
R

If two bulbs of wattage 25 and 30 W, each rated at 220


volts are connected in series with a 440 volt supply, which
U

æ 125 ö
rç ÷ l r l 125 2 bulb will fuse?
è 100 ø æ ö Solution :
JO

R'= = ç ÷ = 1.5625 R
æ 100 ö A è 100 ø (220) 2
ç ÷ A Resistance of 25 W bulb, R1 =
è 125 ø W;
25
U

% increase in resistance 25
Current I1 = A
ED

220
æ R¢ - R ö æ 1.5625 - 1 ö
=ç ÷ ´ 100 = ç ÷ ´ 100 = 56.25% (220) 2
è R ø è 1 ø Resistance of 30 W bulb, R 2 = W,
30
Example 10 : 30
A wire has a resistance of 16.0 ohm. It is melted and drawn Current I 2 = A
220
to a wire half its initial length. What will be the new When bulbs are connected in series, effective resistance is
resistance of the wire ?
é1 1ù 11
Solution : R = R 1 + R 2 = ( 220) 2 ê + ú = 220 ´ 220 ´ W
ë 25 30 û 150
l¢ 1 When supply voltage is 440 V, then current is
The factor by which the length is changed is n = =
l 2 440 440 ´150 27.27
I¢ = = = A
The new resistance R' is given by R 220 ´ 220 ´ 11 220
2 As I¢ > I1 but less than I 2 , hence the bulb of 25 watt will
æ 1ö
R ¢ = R (n 2 ) = 16 W ç ÷ = 4W fuse.
è 2ø Example 14.
Example 11 : What will happen when 40 W, 220 V lamp and 100 W,
The resistivity of silver at 0°C is 1.6 × 10–8 W-m. If its 220 V lamp are connected in series across 440V lamp?
temperature coefficient of resistance is 4.1 × 10–3 °C–1, Solution :
find the resistivity of silver at 80°C. Resistance of first lamp,
Solution : R1 = 220 × 220 / 40 = 1210 W.
Resistance of second lamp,
Here, r 0 = 1.6 × 10–8 W-m, a = 4.1 × 10–3 °C–1 and R2 = 220 × 220 /100 = 484 W.
Dt = 80°C Total resistance in series = 1210 + 484 = 1694 W.
Current Electricity 475

Current in the circuit when supply voltage is 440 V will be,


440
I= = 0.26 A + –
1694 +
H.P L.P
Voltage drop across R1 = 0.26 × 1210 = 314.6 V
Voltage drop across R2 = 0.26 × 484 = 112.84V
Therefore 40 W bulb will fuse because lamp can tolerate +
only 220 V. H.P L.P
Example 15.
For this to happen, work is done by some agency in the emf
A fuse wire with a radius of 1 mm blows at 1.5 A. If the fuse
device. The energy required to do this work is chemical energy
wire of the same material should blow at 3.0 A, the radius
of the wire must be (as in a battery), mechanical energy (as in electric generator),
temperature difference (as in a thermopile).
(a) 41/3 mm (b) 2 mm The emf is thus given by the formula
(c) 0.5 mm (d) 8.0 mm.
Solution : (a) dW
E=
The temperature of the wire increases to such a value at dq
which, the heat produced per second equals heat lost per
J
second due to radiation i.e. The S.I unit of emf is < volt(V)
C
æ rl ö

.IN
I 2 çç 2 ÷÷ = H ´ 2 p r l ,
Keep in Memory
è pr ø
where H is heat lost per second per unit area due to radiation. 1. AL Electromotive force is not a force but a potential difference.
I 12 r13 2. E.m.f. can be defined as the work done in moving a charge
Hence, I 2 µ r 3 so = once around a closed circuit.
I 22 r 23
N
Internal Resistance (r)
or r 2 = r1 (I 2 / I 1 ) 2/ 3 < 1≥(3.0/1.5)2/3 < 41/3 mm.
The potential difference across a real source of emf is not equal
R

to its emf. The reason is that the charge which is moving inside
Example 16
U

the emf device also suffers resistance. This resistance is called


A fuse wire with a circular cross section and having internal resistance of the emf device.
JO

diameter of 0.4 mm blows with a current of 3 amp. The


value of the current for which another fuse wire made of E
the same material but having circular cross-section with + r –
diameter of 0.06 mm will blow is
U

I
(a) 3 amp. (b) 3 ´ (3/2) amp.
ED

(c) 3×(3/2) amp. (d) 3 ´ (3/2) 3/ 2 amp.


I R
Solution : (d) p.d V
2 3
For a fuse wire I µ r
E = IR + Ir = V + Ir
I12 r13 Þ V = E – Ir
\ =
I 22 r23
Keep in Memory
0.04 0.06
where r1 = = 0.02 cm and r2 = = 0.03 cm.
2 2 1. For a cell
3 3
Circuit Mode Expression
(3) 2 æ 0.02 ö æ 2ö
\ =ç ÷ =ç ÷
I 22 è 0. 03 ø è 3ø (i) Discharging V = E – Ir \V <E
3 3/ 2
æ 3ö æ 3ö
or I 22 = (3) 2 ç ÷ or I 2 = 3 ´ ç ÷ (ii) No current V= E
è 2ø è 2ø
ELECTROMOTIVE FORCE AND INTERNAL RESISTANCE
OF A CELL (iii) I
Charging V = E + Ir \V >E
An emf (electromotive force) device has a positive terminal (at
high potential) and a negative terminal (at low potential). This 2. Emf is the property of a cell but terminal potential difference
device is responsible for moving positive charge within itself depends on the current drawn from the cell.
from negative terminal to positive terminal.
EBD_7179
476 PHYSICS

Short Circuiting 1 2 n
1
When the terminals of an emf device are connected with a E r E r E r

conducting path without any external resistance then


2
A E r E r E r B

I
m
r r r
r E E E

Equivalent emf EAB = nE


E
nr
Equivalent resistance =
m
E Where n = no. of cells in a row. and
E = Ir Þ I=
r M = no. of rows
Since internal resistance has a very small value, therefore a very If this equivalent cell is attached to an external resistance R
high current flows in the circuit producing a large amount of then
heat. This condition is called short circuiting. nr
m

.IN
During short circuiting, the terminal potential nE
difference is zero. AL
COMBINATION OF CELLS
Series Combination of Cells R
N
æ nr ö
E1 E2 En nE = I ç R + ÷ Þ I = nmE
è mø mR + nr
R

A r1 r2 rn B
Keep in Memory
U

Equivalent Emf EAB = E1 + E2 + ... + En


Equivalent internal resistance, RAB = r1 + r2 + ....... + rn (i) The condition for maximum current through external resistance
JO

R
Parallel Combination of Cells
m r
nr = mR Þ = Þ R = nr/m
n R
U

E1
In other words, when external resistance is equal to total
r1 internal resistances of all the cells.
ED

E2 nE mE
The maximum current Imax = or
A r2 B 2R 2r
(ii) Maximum power dissipation for the circuit shown in fig.
En
E r
rn

Equivalent emf

E1 E 2 E
+ + ........+ n
r1 r2 rn R
E AB =
1 1 1 E 2R
2
Power P = I 2 R = æç
+ + ..... E ö
r1 r2 rn ÷ R=
èR+rø (R + r ) 2
Equivalent internal resistance dP
For maximum power across the resistor, =0
1 1 1 1 dR
= + + .........+ On solving, we get R = r
R AB r1 r2 rn
This is the condition for maximum power dissipation.
Mixed Grouping of Cells : (iii) If identical cells are connected in a loop in order, then emf
between any two points in the loop is zero.
If the cells are connected as shown below then they are said to (iv) If n identical cells are connected in series and m are wrongly
be in mixed grouping. connected then Enet = nE – 2mE
Current Electricity 477

FARADAY’S LAW OF ELECTROLYSIS Example 18.


(i) 1st law : The mass of the substance liberated or deposited Two cells P and Q connected in series have each an emf of
at an electrode during electrolysis is directly proportional 1.5 V and internal resistances 1.0 Wand 0.5W respectively.
to the quantity of charge passed through the electrolyte. Find the current through them and the voltages across
i.e., mass m µ q = Zq = Z It, their terminals.
where Z = electrochemical equivalent (E.C.E.) of substance. Solution :
(ii) 2nd law : When the same amount of charge is passed For a single closed loop, consisting of cells and resistors
through different electrolytes, the masses of the substance the current i flowing through it is given by
liberated or deposited at the various electrodes are E1=1.5V r1=1.0
proportional to their chemical equivalents P
m1 E
i.e. = 1
m 2 E2
SE i i
where m1 and m2 are the masses of the substances liberated i=
or deposited on electrodes during electrolysis and E1 and Sr + Sri
Q
E2 are their chemical equivalents.
Faraday's Constant E2=1.5V r2=0.5
Faraday constant is equal to the amount of charge required to 1.5 + 1.5V
\ i= = 2.0 A
liberate the mass of a substance at an electrode during electrolysis, 1.0 + 0.5W
equal to its chemical equivalent in gram (i.e. one gram equivalent) The voltage across the cell P is

.IN
One faraday (I F) = 96500 C/gram equivalent. Vp = E1 - ir1 = 1.5V - 1.5 (1.0)V = zero
Keep in Memory ALand across cell Q is
VQ = E 2 - ir2 = 1.5V - 1.5 (0.5)V = 0.75V
1. If r is the density of the material deposited and A is the area
of deposition, then the thickness (d) of the layer deposited Example 19.
If electrochemical equivalent of hydrogen is ZH kg/coul-
N
m ZIt equivalent and chemical equivalent of copper is W, then
in electroplating process is d = = .
rA rA determine the electrochemical equivalent of copper.
R

2. The back e.m.f. for water voltameter is 1.67 V and it is 1.34 V Solution :
U

for CuCl2 electrolytes voltameter with platinum electrodes. Z Cu W


3. 96500 C are required to liberate 1.008 g of hydrogen. =
ZH WH
JO

4. 2.016 g of hydrogen occupies 22.4 litres at N.T.P.


5. E.C.E. of a substance = E.C.E. of hydrogen × chemical We know that, WH = 1
equivalent of the substance. \ ZCu = W.ZH
U

Example 17. Example 20.


Upon a six fold increase in the external resistance of a circuit, Find the mass of silver liberated in a silver voltameter
ED

the voltage across the terminals of the battery has increased carrying a current of 1.5A, during 15 minutes. The electro
from 5 V to 10 V. Find the e.m.f. of battery. chemical equivalent of silver is 1.12× 10–6 kg/C.
Solution : Solution :
V = E – Ir. Here, m = ?, i = 1.5 A, Z = 1.12× 10–6 kg/C
and t = 15 × 30s
E Using m = Zit
V =E- ´r
(R + r) Mass of silver liberated is
E æ kg ö æ 1.5C ö
E- ´r =5 ...(1) m = ç1.12 ´ 10-6 ÷ ç ÷ (450s)
(R + r ) è Cøè s ø
= 7.6 × 10–4 kg = 0.76 g
æ R ö
or E ç ÷=5 ...(2) Example 21.
èR+rø In a water voltameter, the act of passing a certain amount of
E current for a certain time produces of 1.2 g H2 at STP. Find
E- ´ r = 10 ...(3) the amount of O2 liberated during that period.
( 6R + r )
Solution :
æ 6R ö Since the same current flows through both the electrodes of
or E ç ÷ = 10 ...(4) a water voltameter, so the amount of oxygen and hydrogen
è 6R + r ø liberated (for the same charge) will be directly proportional
Dividing eqn. (2) by eqn. (4), we get 2r = 3R to their respective equivalent weights.
E´ r By Faradays’ second law of electrolysis,
Putting values in eqn. (3), we get E - = 10
4r + r mO 2 8
i.e., = or mO 2 = çæ 8 ÷ö m H = 8 ´ 1.2g = 9.6g
Solving we get E = 12.5 V mH2 1 è 1ø 2
EBD_7179
478 PHYSICS

SEEBECK/THERMOELECTRIC EFFECT PELTIER EFFECT


When an electric circuit is composed of two dissimilar metals It states that if current is passed through a junction of two
and the junctions are maintained at different temperature, then different metals the heat is either evolved or absorbed at that
an emf is set up in the circuit. This effect is known as junction. It is the reverse of seebeck effect.
thermoelectric or seebeck effect. The quantity of heat evolved or absorbed at a junction due to
Thermocouple: It is a device in which heat energy is converted Peltier effect is proportional to the quantity of charge crossing
that junction.
into electrical energy. Its working is based on seebeck effect. It
Peltier Coefficient (p) :
has two junctions of two dissimilar metals.
It is defined as the amount of heat energy evolved or absorbed
Cu
per second at a junction of two different metals when a unit
current is passed through it.
T1 T2 The Peltier heat evolved or absorbed at a junction of a
Fe thermocouple = pI t
where I = current passing through the junction for time t.
Some of the elements forming thermo-electric series dE
\ p =T
Sb, Fe, Zn, Cu, Au, Ag, Pb, Al, Hg, Pt, Ni, Bi dT
(i) Lead (Pb) is thermo-electrically neutral where T and (T + dT) are the temperature of cold and hot junctions

.IN
(ii) At the cold junction, current flows from the element occuring of a thermocouple and dE is the thermo emf produced.
earlier into the element occuring later in the series.
p dE
For example: In Cu–Fe thermo–couple, current flows from \ = = S (Seebeck coefficient)
T dT
Cu to Fe at hot junction.
AL
Neutral Temperature (Tn) THOMSON EFFECT
It is that temperature of hot junction for which the thermo emf If a metallic wire has a non uniform temperature and an electric
N
produced in a thermocouple is maximum. current is passed through it, heat may be absorbed or produced
in different sections of the wire. This heat is over and above the
R

It depends upon the nature of the material of thermocouple but is


joule heat I2Rt and is called Thomson heat. The effect is called
independent of temperature of cold junction.
U

Thomson effect.
Temperature of Inversion (Ti) If a charge DQ is passed through a small section of given wire
JO

It is that temperature of hot junction for which the thermo emf having temperature difference DT between the ends,
becomes zero and beyond this temperature, the thermo emf in a Thomson heat, DH = s DQ DT
thermocouple reverses its direction. where s is constant for a given metal at a given temperature.
U

It depends upon the nature of the material of thermocouple and Thomson emf, s DT, is defined as s DT = DH/DQ.
temperature of cold junction
ED

s is positive if heat is absorbed when a current is passed


Let To, Tn, T i be the temperature of cold junction, neutral
temperature and temperature of inversion then from low temp. to high temperature. s is numerically equal to P.D.
developed between two points of the conductor differing in temp.
Tn - T0 = Ti - Tn or Tn = (Ti + To ) / 2
by 1ºC.
Keep in Memory
1. The actual emf developed in a thermocouple loop is the
Thermo emf

algebraic sum of the net Peltier emf and the net Thomson
emf developed in the loop.
sA(T–T0)
O To Tn Ti A
Temp. difference (pAB)T
With temperature difference T between hot and cold junctions,
the thermo-e.m.f. is given by B (pAB)T0
E = aT + bT2
where a and b are Seebeck co-efficients T sB(T–T0) T0
At Tn, (dE/dT) = 0 EAB = (pAB )T - (pAB )To + s A (T - To ) - s B (T - To )
\ Tn = – a/2b and Ti = – a/b, when To = 0 2. If S, p and s are the Seebeck coefficient, Peltier coefficient,
S = dE/dT is called thermo-electric power. and Thomson coefficient respectively then it is found that
Current Electricity 479

Solution :
dE p
(i) S= =
dT T é T 2 T Tr T Tr Tr2 ù
E = k êT T0 - - - Tr T0 + - ú
ëê 2 2 2 2 úû
d2E d æ dE ö TdS
(ii) s = -T = -T ç ÷=-
dT 2 dT è dT ø dT dE é T T ù
Hence = k êT0 - T - r + r ú = k ( T0 - T )
3. For Peltier effect or Thomson effect, the heat evolved or dT ë 2 2û
absorbed is directly proportional to current. But for Joule's At temperature T = T0 /2,
law of heating, the heat produced is directly proportional to Thermoelectric power = k (T0 - T0 / 2) = k T0 / 2.
the square of the current flowing through it.
KIRCHOFF’S LAWS AND ELECTRICAL CIRCUIT
4. Thermo-emf set up in a thermocouple when its junctions
Many practical combination of resistors cannot be reduced to
are maintained at temperature T1 and T3 (i.e. E TT3 ) is equal simple series, parallel combinations. For example the resistors in
1
the figure are neither in series nor in parallel.
to the sum of the emfs set up in a thermocouple when its
junctions are maintained first at temperature T1 and T2 (i.e. e
a R1
T T
E T2 ) and then at T2 & T3 (i.e. E T32 ) i.e. E T3 = E T2 + E T3 R5 R1
1 T1 T1 T2
f d
It is called law of intermediate temperature. E R2
b R2

.IN
Example 22. I2 I1
The temperature of inversion of a thermocouple is 620ºC c
and the neutral temperature is 300ºC. What is the AL I
temperature of cold junction ? The use of Ohm’s law is not sufficient to solve such problems.
Solution : Kirchoff’s laws are used in such cases.
Let Ti,Tn and Tc be the temperature of inversion, neutral
We will often use the term junction and loop, so let us first
N
temperature and temperature of cold junction
respectively, then understand the meaning of these words. A junction in a circuit
is a point where three or more conductors meet. A loop is a
R

Ti – Tn = Tn – Tc
\ 620 – 300 = 300 – Tc Þ 320 = 300 – Tc closed conducting path. In the above figure e, f, d, c are junctions.
U

\ Tc = 300 – 320 = – 20ºC a, b, are not junctions. The various loops are efde, cdfc, eabcf
Example 23. and eabcde.
JO

The temperature of cold junction of a thermocouple is 0ºC (i) Kirchoff’s junction law : (Based on conservation of
and the temperature of hot junction is TºC. The thermo charge) At any junction, the sum of currents entering the
e.m.f. is given by E = 16 T – 0.04 T2 m volt junction must be equal to the sum of currents leaving it.
U

Find (a) the neutral temperature and (b) the If this is not so, charges will accumulate at the junction.
temperature of inversion This cannot happen as this would mean high/low potential
ED

Solution : maintained at a point in a wire without external influence.


Given that E = 16T – 0.04T2
When we apply this rule at junction c, we get I = I 1 + I2
dE (ii) Kirchoff’s loop law : (Based on energy conservation)
\ = 16 - 2 ´ 0.04T
dT The algebraic sum of changes in potential around any
(a) At neutral temperature, dE/dT = 0 closed loop of a circuit must be zero.
\ 16 – 2×0.04 Tn = 0 or 16 = 0.08 Tn Sign convention for using loop law. If we move a loop element
16 (resistor, emf device, capacitor, inductor etc.) in the direction of
or Tn =
= 200º C increasing potential, we take the potential difference positive
0.08
(b) At the temperature of inversion, E = 0 and vice-versa.
\ 16 Ti – 0.04 Ti2 = 0 or 16 – 0.04 Ti = 0 Travel
or 16 = 0.04 Ti – + p.d = +E
E
16
\ Ti = = 400º C Travel
0.040 p.d = –E
Example 24. +E –
One junction of a certain thermocouple is at a fixed H.P. L.P.
temperature Tr and the other junction is at a temperature p.d = – IR
T. The electromotive force for this is expressed by, I R
Travel
é 1 ù
E = k (T - Tr ) êT0 - (T + Tr ) ú . L. P. H.P
ë 2 û p.d = +IR
R I
At, temperature T = T0/2. Determine the thermoelectric
power. Travel
EBD_7179
480 PHYSICS

Problem Solving Tactic for Using Kirchoff’s Law + E2 – I1R4 – (I1 + I2) R3 = 0
(i) Draw a circuit diagram large enough to show all resistors, For loop afeda, moving the loop in clockwise direction we get –
emf device, capacitors, currents clearly. E1 – I2 R1 – I2R2 – (I1 + I2)R3 = 0
(ii) Take into account the resistance of voltmeter/ammeter/
Node method to apply Kirchoff’s law (Open loop method)
internal resistance of a cell (if given).
(iii) Assume the direction of current in all branches. It may be Step 1 : We select a reference node and assume its potential
noted here that one branch has only one direction of to be (zero/x)V
current. Step 2 : We calculate the voltage of other selected points w.r.t.
It is best to use junction law simultaneously while drawing the reference node
currents. This helps to reduce the number of unknown Step 3: We find some independent node (whose voltage is
quantities. not known). We apply Kirchoff’s law to find the
relevant values.
E2 a E1 Example 25.
b f
R2 I2 Find current through branch BD
loop
loop R1 A 1W B 2W
n R3 C
direction
dir
I1+ I2 5V
15V
c d I2 R e
3W
R4 I1

.IN
2
Fig 1

D
b
E2 a E1
AL
Solution :
f Let VD = 0V
I1 I2
\ VA = +5V and VC = + 15 V
N
R3 R1 Let the voltage of B = VB
Applying Kirchhoff’s junction law at B
R

I3 5 - VB 15 - VB 0 - VB
+ + = 0 Þ VB = 6.82 Volt
U

c R4 I1 d R2 I2 e 1 2 3
Fig 2 6.82 - 0
JO

Current through BD = = 2.27 A


3
In the above circuits we arbitrarily assumed the direction Example 26.
of current I1 in branch abcd as anti-clockwiswe and the Calculate the currents I1, I2 and I3 in the circuit shown in
U

direction of current I2 in branch afed as clockwise. figure.


In figure 1 we have two unknown currents (I1, I2) whereas
ED

figure 2 we have three unknown currents (I1, I2 and I3).


The first figure is a better option for solving problems. In
figure 1 we used junction rule at d simultaneously while
labelling currents.
(iv) In a branch containing a capacitor, the current is zero when
d.c is applied and steady state conditions are achieved.
(v) Now we need as many independent equations as there are
conditions unknowns. If we have to find a particular Solution :
unknown, we should ensure that, the unknown appears in Junction rule at C yields
one of the equations made by us. I1 + I2 – I3 = 0 i.e., I1 + I2 = I3 .... (1)
while loop for meshes a and b yields respectively :
(vi) For making equations choose the loop and travel the loop –14 – 4I2 + 6I1 – 10 = 0
completely. We may travel the loop in clockwise or anti- i.e., 3I1 – 2I2 = 12 .... (2)
clockwise direction. While using second law use sign and, 10 – 6I1 – 2I3 = 0
conventions properly. i.e., 3I1 + I3 = 5 .... (3)
(vii) Solve the equations formed to find the unknown quantities. Substituting I3 from equation (1) in (3)
If any value of current comes out to be negative then that 4I1 + I2 = 5
particular current is in the opposite direction to that Solving equations (2) and (4) for I1 and I2, we find
assumed. I1 = 2A and I2 = –3A
And hence equation (1) yields, I3 = –1A
Applications The fact that I2 and I3 are negative implies that actual
Let us use second law in the loop abcda of figure 1 taking the direction of I2 and I3 are opposite to that shown in the
loop in anti-clockwise direction starting from a. circuit.
Current Electricity 481

WHEATSTONE BRIDGE 1
The condition for balanced wheatstone bridge D
EAB = (pAB )T - (pAB )To + s A (T - To ) - s B (T - To )
A r B r C r

B 2

P Q
P Correct In the diagram, the three resistors are in parallel.
A G C The potential at A is equal to the potential at C.
Current flows in wire 1 but there is no potnetial
R S drop across A and C.
O Incorrect If potential difference between the points is zero,
D there is zero current between the two points.
P Coorect There is no p.d. between A and C still current
E flows in segment 1.
P R Example 27.
P Q
= also = Calculate the effective resistance between A and B in the
Q S R S following network.
Note that when battery and galvanometer of a Wheatstone bridge 3W
is interchanged, the balance position remains undisturbed, while
sensitivity of the bridge changes. 2W D

.IN
In the balanced condition, the resistance in the branch BD may A B
be neglected C 7W 6W C
Example : Resistance connected to BC may be neglected. AL
Solution : 4W
2W
2W The circuit can be redrawn as
A
B 2W C 2W D;
N
C
2W
R

P=2W Q=3W
U

2W 2W
A 7W B
2W
JO

2W A B
R=4W S=6W
2W
U

D
In a Wheatstone bridge, the deflection in a
ED

galvanometer does not change, if the battery and the P R 2


galvanometer are interchanged Here = = so bridge is balanced
Q S 3
Measuring temperature with the help of Wheatstone bridge So the resistance between c and d is non useful.
P Q Equivalent resistance = (P + Q) (R + S)
At balancing =
R + DR S(1 + aDT ) (P + Q)(R + S) (2 + 3)(4 + 6)
R eq = =
P+Q+ R +S 2 + 3+ 4+ 6
P Q 5 ´ 10 10
= = W
15 3
G
Even if not able to observe balanced wheatstone bridge try
to observe symmetry in network and use plane cutting
R
S

method.
DR Example 28.
When P = Q then DR = S a DT [Q R » S] Find the potential difference between the points A and B
in fig.
DR 5W 5W B
\ DT =
Sa
COMMON DEFAULT
O Incorrect If the current flows in a wire, there has to be a 5W + –2V 5W
potential difference. The potential drop takes
place only when current passes through a
resistor. A 5W 5W
EBD_7179
482 PHYSICS

Solution : POTENTIOMETER :
The upper three resistances of the cell are in series. Their Principle : The p.d. across a resistance wire is directly
equivalent resistance is 15W. Similarly lower three proportional to its length provided I, r and A are constant.
resistances are in series. Their equivalent resistance is also
l
15W. The upper and lower equivalent resistances are V = IR = I r
A
connected in parallel. So, resultant resistance of the circuit
is given by Þ V a l [ I, r and A are constant]
Working : PQ is the resistance wire of potentiometer generally
1 1 1 2 15 made up of constantan or nichrome. One end P is connected to
= + = or R = W
R 15 15 15 2 the positive terminal of the battery B while negative terminal is
connected to Q through a Rheostat (Rh) and key (K). This is the
2 4 main circuit.
Current from the cell, i = = amp.
(15 / 2) 15 A cell whose emf has to be measured is also connected to the
potential wire in such a way that the positive terminal is connected
In order to calculate potential difference between points A
with P and negative terminal is connected to a galvanometer and
and B, see fig.
then to a jockey (J) which is free to slide along the wire
B K Rh
2V

.IN
5W 5W 5W
E C A
A
F D
5W 5W B 5W
AL
A C
P
IB J Q
Half of the current goes to each part i.e., current in each
N
part is (2/15) amp. Consider the loop AEFDCA, we have IE
E
There is a potential drop along PQ.
R

2 2 2
VAB = - ´ 5 + ´ 5 + ´ 5 = VA - VB The potential drop per unit length along PQ is called potential
15 15 15
U

gradient.
2 2 2 4 When the jockey is pressed on some point, current flows from E
JO

= ´ 5 = volt or VA = 2 - ´ 5 = volt ,
15 3 15 3 to P (Þ). Also current that comes from B after reaching P divides
into two parts. One part moves towards A and the other towards
2 2 2 E (®). Three cases may arise.
U

VB = 2 - ´ 10 = volt so VAB = VA - VB = volt


15 3 3 (a) IB > IE. This happens when VPC > E. One side deflection in
ED

galvanometer
METER BRIDGE OR SLIDE WIRE BRIDGE
(b) IB = IE. This happens when VPC = E, Zero deflection in
Principle: Based on balanced Wheatstone bridge principle.
galvanometer
Use : To find unknown resistance
(c) IB < IE . This happens when VPA < E. Other side defection
Working : Let P be the unknown resistance.
in galvanometer

P Q
• At null point since no current flows through E therefore it is
said to be in the condition of open circuit.
• More is the length of potentiometer, higher is the sensitivity of
100– l potentiometer and smaller is the potential gradient.
l
• Potentiometer will work only when B > E. Also the positive
G
terminal of the batteries is connected at P. If any of the above
conditions is not followed, we do not get a null point.

E1 l1
Uses : (i) Comparison of emfs of cells E = l
2 2
At balance point
P Q ælö
= (ii) To find internal resistance of a cell r = ç ÷ R
l 100 - l
è l' ø
Q is known and l can be calculated. emf can be measured by potentiometer and not voltmeter.
Current Electricity 483

Example 29.
Copper strip
A 10 m long wire AB of uniform area of cross-section and
20W resistance is used as a potentiometer wire. This wire 100 x
is connected in series with a battery of 5V and a resistor
of 480W. An unknown emf is balanced at 600 cm of the G Copper
wire as shown in the figure. A B strip
1 2
5V 480 W

r
E
600 cm Sol. Q Wheatstone bridge is in balanced condition
A J B
100

G 100 x
E

Calculate :
(i) The potential gradient for the potentiometer wire
(ii) The value of unknown emf E

.IN
1 2
Solution :
(i) V = 5V 480 W I 100x
AL 100 100 + x l1
so = and Q l = 2
I l1 l2 2
N
A B Þ x = 100 W
R

Applying Ohm’s law MEASURING INSTRUMENTS


V = I (RAB + 480)
U

Galvanometer
5 It is an instrument used to detect small currents in a circuit.
JO

5 = I (20 + 480) [RAB = 20 W (given)] Þ I = = 0.01A


The current required for full scale deflection in the galvanometer
500
Potential gradient of potentiometer wire is called full scale deflection current and is denoted by Ig.
Current sensitivity of a galvanometer.
U

VAB 0.2
< < < 0.02Vm,1 It is defined as the deflection produced in the galvanometer,
l AB 10
ED

when unit current flows through it.


(ii) With reference to the current given in question, the emf E
and the potential drop across AJ should be equal for the q NBA
Current sensitivity, IS = = and its unit is rad A–1
balancing : E = (potential gradient of potentiometer wire) I C
× balancing length = 0.02 × 6 = 0.12 V Current sensitivity can be increased either by decreasing C i.e.
Example 30.
restoring torque per unit twist or increasing B.
A uniform potential gradient is established across a
potentiometer wire. Two cells of emf E1 and E2 connected Voltage sensitivity of a galvanometer
to support and oppose each other are balanced over It is defined as the deflection produced in the galvanometer when
l1 = 6m and l2 = 2m. Find E1/E2. a unit voltage is applied across the two terminals of the
Solution : galvanometer.
E1 + E2 = xl1 = 6x and E1 – E2 = 2x
q q NBA
E1 + E 2 6 E1 2 Voltage sensitivity VS = = = , its unit is rad V–1
= or = V IR CR
E1 - E 2 2 E2 1
Example 31. Ammeter
In a practical wheatstone bridge circuit as shown, when Ammeter is used to measure current in a circuit. Ammeter is
one more resistance of 100 W is connected is parallel with always connected in series in the circuit as shown.
unknown resistance ‘x’, then ratio l1 / l 2 become ‘2’. l1 I
A
is balance length. AB is a uniform wire. Find the value of' R ext
x'.
EBD_7179
484 PHYSICS

Conversion of a galvanometer into an ammeter When ammeter/voltmeter is connected in the circuit,


A the current or voltage indicated by these is less than the actual
values in their absence.
Ig
Example 32.
G
I I A galvanometer of coil resistance 20 ohm, gives a full
scale deflection with a current of 5 mA. What arrangements
I – Ig S should be made in order to measure currents upto 1.0 A ?
For this, we connect a small resistance S (called shunt) in Solution :
parallel with the galvanometer. The upper limiting value of current to be measured is to be
Mathematically, Ig × G = (I – Ig) S increased by a factor
where I is the maximum current which ammeter can measure. G is 1.0A
the resistance of galvanometer and Ig is the current of full scale n= = 200
5A
deflection in galvanometer. S is shunt.
\ Resistance of the shunt required will be
The resistance of the ammeter will be
G 20W
1 1 1 G´S S= = » 0.1 W
= + RA = n - 1 200 - 1
RA G S G+S
Hence, a shunt of resistance 0.1 W should be connected

.IN
Since shunt is a small resistance. Therefore the resistance of
in parallel across the galvanometer coil.
ammeter is very small.
Example 33.
The above arrangement is made so that when we connect
A voltmeter having 100 W resistance can measure a
ammeter in series to measure current, it does not change the
AL potential difference of 25V. What resistance R is required
original current to a large extent. The change is infact very small.
Also since galvanometer is a sensitive device and cannot take to be connected in series, to make it read voltage upto
large currents, this arrangement serves the purpose. Most of 250 V ?
N
the current entering the ammeter passes through the shunt as Solution :
The upper limiting value of voltage is to be increased by a
R

current always prefer low resistance path.


An ideal ammeter is one which has zero resistance. factor
U

The range of ammeter can be increased but cannot be decreased 250V


below Ig. n= = 10
JO

25V
Voltmeter
\ Resistance, R = (n – 1) G = (10 – 1) 100 = 900 W
Voltmeter is used to measure potential difference across a
Keep in Memory
resistor. Voltmeter is always connected in parallel across a
U

resistor. 1. In order to increase the range of a voltmeter n times, its


total resistance should also be increased by n times. So
ED

Conversion of a galvanometer into a voltmeter


the resistance to be connected in series is
Voltmeter (V) R = (n – 1) G.
V
2. In order to increase the range of an ammeter n times, the
G value of shunt resistance to be connected in parallel is
R
Ig S = G / (n – 1)
3. For galvanometers with their coil in uniform magnetic field B

I a Rext b Cq
I=
V NAB sin a
For this, we connect a large resistance R in series with the C = torsional rigidity of suspension wire.
galvanometer. For galvanometer with concave pole-pieces – radial
The potential difference which has to be measured is across the magnetic field is produced, a = 90°
external resistance i.e. across points a and b.
I = (C / NAB)q or I = Kq
Let it be V. Then
V = Ig (G + R) All galvanometers used in practice have concave
where V is the maximum potential difference that the voltmeter pole-pieces, for making the radial magnetic field.
can measure and R is the large resistance connected in series
with the galvanometer Example 34.
The resistance of the voltmeter will be RV = G + R A galvanometer has a resistance of 30 W and current of
Since R is a large resistance. Therefore resistance of 2 mA is needed to give full scale deflection. What is the
voltmeter is very large. resistance needed and how is it to be connected to convert
An ideal voltmeter is one which has infinite resistance. the galvanometer
The range of voltmeter can be increased and decreased. (a) into an ammeter of 0.3 ampere range
(b) into voltmeter of 0.2 volt range
Current Electricity 485

Solution : The voltmeter gives full scale deflection for potential


difference V. Its resistance is G
æ S ö
(a) We know that i g = ç ÷i
èS+G ø Hence Ιg = (V / G)
Substituting the given values, we get
nV
\ R= - G = n G - G = (n - 1) G
-3 æ S ö (V / G)
2 ´10 = ç ÷ ´ 0.3
è S + 30 ø Example 36.
Solving it, we get S = (30/149) W In the circuit shown in fig., Find the reading of voltmeter.
So, a resistance of (30/149) W must be connected in + –
parallel with the galvanometer.
2V,r = 0
æ V ö V 80W
(b) In this case, i g = ç ÷ or R = - G V
èR+Gø ig
20W 80W
0.2
\ R= - 30 = 100 - 30 = 70 W Solution :
2 ´10 -3 The equivalent resistance of the circuit is given by
So, a resistance of 70 W must be connected in series
with the galvanometer. 654 80 ´ 80
R eq. = 20 + = 20 + 40 = 60W

.IN
80 ´ 80
Example 35.
A voltmeter has a resistance of G ohm and range V volt.
V
What will be the value of resistance used in series to Current through the circuit I = =(2/60) = (1/30) amp
convert it into voltmeter of range nV volt?
AL R
Solution : Reading of voltmeter
= current × RT
N
V
We know that, R = -G
Ιg 1 æ 80 ´ 80 ö 40
R

= ´ç ÷= = 1.33 V
30 è 80 + 80 ø 30
U
JO
U
ED
CONCEPT MAP
486

Colour coding of
On length ()l and Resistance R = AB
area of cross-section (A) Resistance (R)Obstruction × C ± D% A, B – First
to flow offered of electrons
R µ lü two significant figures
ï l of resistance C-multiplier
1 ýR = r
µ ï A D-tolerance
Aþ Ohm's law if the physical
r = resistivity Dependence of conditions remain same,
resistance Grouping of resistance
current I µ V Þ V = IR
R-electric resistance
ED
On temperature
Rt = R 0(1 + µt) U Series grouping of Parallel grouping of
Conductivity (s) resistances Equivalent resistances Equivalent
Reciprocal of resistance Current density (J) Electric Current (I) The time resistance, Rs= R 1 + resistance,
JO
1 Current per unit cross rate of flow of charge (Q) R2 +...+ Rn 1 1 1 1
s= sectional area (A) = + +...+
r through any cross-section
U R P R1 R 2 Rn
r I E Q
J= =r I= t
A R Drift velocity (Vd ) Mobitity (m)
2nd law Loop
N Average uniform velocity Drift velocity per
lst law Junction law rule Algebraic CURRENT ELECTRICITY acquired by free electrons unit electric field
Algebraic sum of sum of changes in i V
Vd = m= d
all the current potential around neA E
AL
meeting at a junction any closed loop is
is zero i.e. S I = 0 zero.
Electric cell source of energy that
Groupings of cells
.IN
maintains continuous flow of charge
in a circuit

Kirchhoff's laws Potentiometer used to


Cells in series Cells in parallel Cells in series
(i) Compare emfs Current in thecircuit and parallel i.e.
Current in the
E1 l1
= circuit, I = ne current I = e mixed Current inne
E 2 l2 R + nr r the circuit, I =
Balanced condition R+ nr
of wheatstone bridge (ii) Find internal m +R
P R resistance of cell m
Electrical energy
=
Q S æE ö H µ I2 ü Jule's heating law
r = ç –1÷ S
èV ø ï 2
µ R ý H = I Rt; Electrical
Meter Bridge Based on V2
µ t ïþ power P =
P R l R R
Wheatstone bridge = Þ =
PHYSICS

Q S 100 – l S

EBD_7179
Current Electricity 487

1. The emf developed by a thermocouple is measured with (c) the potential draws no current during measurement.
the help of a potentiometer and not by a moving coil (d) range of the voltmeter is not as wide as that of the
millivoltmeter because potentiometer.
(a) the potentiometer is more accurate than the voltmeter 8. The infinity resistance plug in a post-office box has
(b) the potentiometer is more sensitive than voltmeter (a) an air gap only
(c) the potentiometer makes measurement without (b) a resistance coil of infinite resistance
drawing any current from the thermocouple (c) largest resistance available in box
(d) measurement using a potentiometer is simpler than (d) resistance of the coil 5000 W
with a voltmeter 9. In an household electric circuit, which of the following is/
2. Three copper wires of lengths and cross sectional areas are are correct?
(l, A), (2 l, A/2) and (l/2, 2A). Resistance is minimum in (A) All electric appliances drawing power are joined in
(a) wire of cross-sectional area A/2 parallel
(B) A switch may be either in series or in parallel with the
(b) wire of cross-sectional area A appliance which it controls

.IN
(c) wire of cross-sectional area 2A (C) If a switch is in parallel with an appliance, it will draw
(d) same in all the three cases power when the switch is in the ‘off’ position (open)
3. When a current I is set up in a wire of radius r, the drift (D) If a switch is in parallel with an appliance, the fuse will
blow (burn out) when the switch is put ‘on’ closed.
velocity is vd. If the same current is set up through a wire of
AL
(a) A, D (b) A, C, D
radius 2 r, the drift velocity will be (c) B, C, D (d) A, B, D
(a) 4 vd (b) 2 vd 10. Two identical fuses are rated at 10 A. If they are joined
N
(c) vd/2 (d) vd/4
(A) in parallel, the combination acts as a fuse of rating 20 A
4. In the given circuit, as the sliding contact C is moved from
R

(B) in parallel, the combination acts as a fuse of rating 5 A


A to B
(C) in series, the combination acts as a fuse of rating 10 A
U

V (D) in series, the combination acts as a fuse of rating 20 A


Select the correct options.
JO

C
A B (a) A, B (b) A, C
(c) B, D (d) B, C, D
U

A
11. The resistance of the coil of an ammeter is R. The shunt
required to increase its range n-fold should have a
ED

(a) the readings of both the ammeter and the voltmeter


remain constant resistance
(b) the reading of both the ammeter and the voltmeter R R
increase (a) (b)
n n -1
(c) the reading of the ammeter remains constant but
that of the voltmeter increases R
(c) (d) nR
(d) the reading of the ammeter remains constant but n +1
that of the voltmeter decreases 12. A cell of internal resistance r is connected across an
5. Potentiometer measures potential more accurately because external resistance nr. Then the ratio of the terminal voltage
(a) it measures potential in open circuit to the emf of the cell is
(b) it uses sensitive galvanometer for null deflection
(c) it uses high resistance potentiometer wire 1 1 n n -1
(a) (b) (c) (d)
(d) it measures potential in closed circuit n n +1 n +1 n
6. Coils in the resistance boxes are made from doubled up 13. A battery of e.m.f. 10 V and internal resistance 0.5 W is
insulated wires connected across a variable resistance R. The value of R
(a) to cancel the effect of self induction for which the power delivered in it is maximum is given by
(b) to nullify the heating effect (a) 0.5 W (b) 1.0 W (c) 2.0 W (d) 0.25 W
(c) to nullify the Peltier effect
14. An ammeter has a resistance of G ohm and a range of I amp.
(d) to reduce effective length of the wire The value of resistance used in parallel to convert it into an
7. For measuring voltage of any circuit, potentiometer is ammeter of range nI amp is
preferred to voltmeter because
(a) nG (b) (n – 1)G
(a) the potentiometer is cheap and easy to handle.
(b) calibration in the voltmeter is sometimes wrong . (c) G/n (d) G/(n – 1)
EBD_7179
488 PHYSICS

15. A current source drives a current in a coil of resistance R1 21. A 4 ohm resistance wire is bent through 180º at its mid point
for a time t. The same source drives current in another coil and the two halves are twisted together. Then the resistance
of resistance R2 for same time. If heat generated is same, is
find internal resistance of source. (a) 1 W (b) 2 W (c) 5 W (d) 8 W
22. Is it possible that any battery has some constant value of
R1R 2 e.m.f but the potential difference between the plates is zero?
(a) (b) R1 + R 2
R1 + R 2 (a) No
(b) Yes, if another identical battery is joined in series.
(c) zero (d) R 1R 2 (c) Yes, if another identical battery is joined in
opposition in series.
16. Two electric bulbs whose resistance are in the ratio 1 : 2 are (d) Yes, if another similar battery is joined in parallel.
arranged in parallel to a constant voltage source. The 23. Which of the following in electricity is analogous to
powers dissipated in them have the ratio momentum mv in dynamics ?
(a) 1 : 2 (b) 1 : 1 (c) 2 : 1 (d) 1 : 4 (a) IV (b) I L (c) QL (d) IQ
17. Who among the following scientists made the statement ? 24. You are given a resistance coil and a battery. In which of
“Chemical change can produce electricity”. the following cases is largest amount of heat generated ?
(a) Galvani (b) Faraday (a) When the coil is connected to the battery directly
(b) When the coil is divided into two equal parts and
(c) Coulomb (d) Thompson
both the parts are connected to the battery in parallel
18. Which of the following is not reversible ? (c) When the coil is divided into four equal parts and all

.IN
(a) Joule effect (b) Peltier effect the four parts are connected to the battery in parallel
(c) Seebeck effect (d) Thomson effect (d) When only half the coil is connected to the battery
19. In which of the following the power dissipation is 25.
AL The electric resistance of a certain wire of iron is R. If its
proportional to the square of the current ? length and radius are both doubled, then
(a) Peltier effect (b) Joule's effect (a) the resistance and the specific resistance, will both
remain unchanged
(c) Thomson effect (d) None of the above
N
(b) the resistance will be doubled and the specific
20. When current is passed through a junction of two dissimilar resistance will be halved
R

metals, heat is evolved or absorbed at the junction. This (c) the resistance will be halved and the specific
process is called resistance will remain unchanged
U

(a) Seebeck effect (b) Joule effect (d) the resistance will be halved and the specific
resistance will be doubled
JO

(c) Petlier effect (d) Thomson effect


U
ED

1. A given resistor has the following colour scheme of the 5. A primary cell has an e.m.f. of 1.5 volt. When short-circuited
various strips on it : Brown, black, green and silver. Its it gives a current of 3 ampere. The internal resistance of the
value in ohm is cell is
(a) 1.0 ´ 10 4 ± 10% (b) 1.0 ´ 105 ± 10% (a) 4.5 ohm (b) 2 ohm
(c) 0.5 ohm (d) (1/4.5) ohm
(c) 1.0 ´ 10 6 ± 10% (d) 1.0 ´ 10 7 ± 10% 6. Two wires A and B of the same material, having radii in the
2. If R1 and R2 are respectively the filament resistances of a ratio 1 : 2 and carry currents in the ratio 4 : 1. The ratio of
200 watt bulb and a 100 watt bulb designed to operate on drift speed of electrons in A and B is
the same voltage (a) 16 : 1 (b) 1 : 16
(a) R1 is two times R2 (b) R2 is two times R1 (c) 1 : 4 (d) 4 : 1
(c) R2 is four times R1 (d) R1 is four times R2 7. A torch bulb rated as 4.5 W, 1.5 V is connected as shown in
3. A conductor carries a current of 50 m A. If the area of cross- fig. The e.m.f. of the cell, needed to make the bulb glow at
section of the conductor is 50 mm2, then value of the current full intensity is
density in Am–2 is 4.5 W,
(a) 0.5 (b) 1 1.5V
(c) 10–3 (d) 10–6 2E/9
4. A cell when balanced with potentiometer gave a balance
length of 50 cm. 4.5 W external resistance is introduced in E/9
the circuit, now it is balanced on 45 cm. The internal 0.33 W
resistance of cell is E/3
(a) 0.25 W (b) 0.5 W E, r = 2.67 W
(a) 4.5 V (b) 1.5 V
(c) 1.0 W (d) 1.5 W (c) 2.67 V (d) 13.5 V
Current Electricity 489

8. A flow of 107 electrons per second in a conducting wire 19. If the resistance of a conductor is 5W at 50º C & 7W at
constitutes a current of 100º C, then mean temperature coefficient of resistance (of
material) is
(a) 1.6 ´ 10 -26 A (b) 1.6 ´ 10 26 A
(a) 0.013/ ºC (b) 0.004/ ºC
(c) 1.6 ´ 10 -12 A (d) 1.6 ´ 1012 A (c) 0.006/ ºC (d) 0.008/ ºC
9. A 100-W bulb and a 25-W bulb are designed for the same 20. If negligibly small current is passed though a wire of length
voltage. They have filaments of the same length and 15 m & resistance of 5W, having uniform cross section of
material. The ratio of the diameter of the 100-W bulb to that 6 × 10–7 m2, then coefficient of resistivity of material is
of the 25-W bulb is (a) 1×10–7W–m (b) 2×10–7W–m
(a) 4 : 1 (b) 2 : 1 –7
(c) 3×10 W–m (d) 4×10–7W–m
(c) (d) 1 : 2 21. A potentiometer consists of a wire of length 4m and
2 :1
resistance 10W. It is connected to a cell of e.m.f. 3V. The
10. In a metre bridge, the balancing length from the left end potential gradient of wire is
(standard resistance of one ohm is in the right gap) is found to (a) 5V/m (b) 2V/m
be 20 cm. The value of the unknown resistance is
(c) 5V/m (d) 10V/m
(a) 0.8 W (b) 0.5 W
22. The four wires from a larger circuit intersect at junction A
(c) 0.4 W (d) 0.25 W as shown. What is the magnitude and direction of the
11. An electric fan and a heater are marked as 100 W, 220 V and current between points A and B ?
1000 W, 220 V respectively. The resistance of heater is 4A

.IN
(a) equal to that of fan (b) lesser than that of fan
(c) greater than that of fan (d) zero A B
5A
]
12. In a neon gas discharge tube Ne+ ions moving through a
AL
cross-section of the tube each second to the right is 2.9 × 6A
1018, while 1.2 × 1018 electrons move towards left in the (a) 2 A from A to B (b) 2A from B to A
N
same time; the electronic charge being 1.6 × 10–19 C, the (c) 3A from A to B (d) 2A from B to A
23. Potentiometer wire of length 1 m is connected in series with
R

net electric current is


(a) 0.27 A to the right (b) 0.66 A to the right 490W resistance and 2 V battery. If 0.2 mV/cm is the potential
gradient, then resistance of the potentiometer wire is
U

(c) 0.66 A to the left (d) zero


13. A capacitor of 10m F has a pot. difference of 40 volts across (a) 4.9 W (b) 7.9 W (c) 5.9 W (d) 6.9 W
JO

it. If it is discharged in 0.2 second, the average current 24. The deflection in a galvanometer decreases from 25 divisions
to 5 divisions when a resistor of 20W is connected in series.
during discharge is
Find resistance of galvanometer.
(a) 2 m A (b) 4 m A
U

(a) 4 W (b) 5 W (c) 6 W (d) 7 W


(c) 1 m A (d) 0.5 m A
25. A galvanometer of 50 ohm resistance has 25 divisions. A
14. The amount of charge Q passed in time t through a cross-
ED

current of 4 × 10–4 ampere gives a deflection of one division.


section of a wire is Q = 5 t2 + 3 t + 1. To convert this galvanometer into a voltmeter having a range
The value of current at time t = 5 s is of 25 volts, it should be connected with a resistance of
(a) 9 A (b) 49 A (a) 2450 W in series. (b) 2500 W in series.
(c) 53 A (d) None of these (c) 245 W in series. (d) 2550 W in series.
15. A wire X is half the diameter and half the length of a wire Y 26. In the equation AB = C, A is the current density, C is the
of similar material. The ratio of resistance of X to that of Y is electric field, Then B is
(a) 8 : 1 (b) 4 : 1 (c) 2 : 1 (d) 1 : 1 (a) resistivity (b) conductivity
16. 2, 4 and 6 S are the conductances of three conductors. (c) potential difference (d) resistance
When they are joined in series, their equivalent conductance 27. A dynamo develops 0.5 A at 6 V. The energy which is
will be
generated in one second is
(a) 12 S (b) (1/12) S
(a) 0.083 J (b) 3 J
(c) (12/11) S (d) (11/12) S
17. 2, 4 and 6 S are conductances of three conductors. When they (c) 12 J (d) None of these
are joined in parallel, their equivalent conductance will be 28. In an electroplating experiment, m g of silver is deposited
(a) 12 S (b) (1/12) S when 4 A of current flows for 2 minutes. The amount in g of
(c) (12/11) S (d) (11/12) S silver deposited by 6 A of current for 40 seconds will be
18. Two identical cells connected in series send 1.0A current (a) 4 m (b) 2 m (c) m/2 (d) m/4
through a 5 W resistor. When they are connected in parallel, 29. A steady current of 5 A is maintained for 45 minutes. During
they send 0.8 A current through the same resistor. What is this time it deposits 4.572 g of zinc at the cathode of voltameter.
the internal resistance of the cell? E.C.E. of zinc is
(a) 0.5 W (b) 1.0 W (c) 1.5 W (d) 2.5 W (a) 3.387 × 10–4 g/C (b) 3.387 × 10–4 kg/C
–4
(c) 3.384 × 10 kg/C (d) 3.384 × 10–3 kg/C
EBD_7179
490 PHYSICS

30. What is the equivalent resistance between the points A (a) 8 minutes (b) 10 minutes
and D in given figure? (c) 12 minutes (d) 15 minutes
40. An electric lamp is marked 60 W, 220 V. The cost of kilo
watt hour of electricity is Rs. 1.25. The cost of using this
lamp on 220 V for 8 hours is
(a) Re 0.25 (b) Re 0.60
(c) Re 1.20 (d) Re 4.00
(a) 10 W (b) 20 W 41. If current flowing in a conductor changes by 1% then power
consumed will change by
(c) 30 W (d) 40 W
(a) 10% (b) 2% (c) 1% (d) 100%
31. Two wires of same metal have the same length but their
cross-sections are in the ratio 3 : 1. They are joined in series. 42. If nealy 105 coulomb are liberated by 1gm equivalent of
The resistance of the thicker wire is 10 W. The total aluminium, then amount of aluminium (equivalent weight 9)
resistance of the combination is deposited through electrolysis in 20 minutes by a current
(a) 5/2 W (b) 40/3 W of 50 ampere will be :
(c) 40 W (d) 100 W (a) 0.6 gm. (b) 0.09 gm (c) 5.4 gm (d) 10.8 gm
32. The resistance of a wire at room temperature 30°C is found 43. Three equal resistors, connected across a source of e.m.f.
to be 10 W. Now to increase the resistance by 10%, the together dissipate 10 watt of power. What will be the power
temperature of the wire must be [ The temperature coefficient dissipated in watts if the same resistors are connected in

.IN
of resistance of the material of the wire is 0.002 per °C] parallel across the same source of e.m.f.
(a) 36°C (b) 83°C (c) 63°C (d) 33°C (a) 10 (b) 10/3 (c) 30 (d) 90
33. A potentiometer wire, 10 m long, has a resistance of 40W. It 44. A wire of radius r and another wire of radius 2r, both of
AL
is connected in series with a resistance box and a 2 V storage same material and length are connected in series to each
cell. If the potential gradient along the wire is 0.1 m V/cm, other. The combination is connected across a battery. The
the resistance unplugged in the box is ratio of the heats produced in the two wires will be
N
(a) 260 W (b) 760 W (a) 4.00 (b) 2.00 (c) 0.50 (d) 0.25
R

(c) 960 W (d) 1060 W 45. The electrochemical equivalent of a metal is 3.3 × 10–7 kg
34. If R1 and R2 are the filament resistances of 200 W and a 100 per coulomb. The mass of the metal liberated at the
U

W bulb respectively both designed to run at the same cathode when a 3 A current is passed for 2 seconds will
JO

voltage, then be
(a) R2 is four times of R1 (b) R1 is four times of R2 (a) 19.8 × 10–7 kg (b) 9.9 × 10–7 kg
(c) R2 is two times of R1 (d) R1 is two times of R2 (c) 6.6 × 10 kg–7 (d) 1.1 × 10–7 kg
U

35. Two heating wires of equal length are first connected in 46. An electrical cable of copper has just one wire of radius 9
series and then in parallel to a constant voltage source.
ED

mm. Its resistance is 5 ohm. This single copper wire of the


The rate of heat produced in two cases is (parallel to series) cable is replaced by 6 different well insulated copper wires
(a) 1 : 4 (b) 4 : 1 (c) 1 : 2 (d) 2 : 1 each of radius 3 mm. The total resistance of the cable will
36. Two identical batteries each of e.m.f. 2 V and internal now be equal to
resistance 1 W are available to produce heat in an external
(a) 7.5 ohm (b) 45 ohm
resistance by passing a current through it. The maximum
power that can be developed across R using these batteries (c) 90 ohm (d) 270 ohm
is 47. In an experiment to measure the internal resistance of a cell,
(a) 3.2 W (b) 2.0 W (c) 1.28 W (d) 8/9 W by a potentiometer, it is found that the balance point is at a
37. A heater of 220 V heat a volume of water in 5 minutes time. length of 2 m, when the cell is shunted by a 5 W resistance
A heater of 110 V heats the same volume of water in and is at a length of 3 m when the cell is shunted by a 10 W
(a) 5 minutes (b) 8 minutes resistance. The internal resistance of the cell is then
(c) 10 minutes (d) 20 minutes (a) 1.5 W (b) 10 W
38. The internal resistance of a primary cell is 4W. It generates (c) 15 W (d) 1 W
a current of 0.2 A in an external reistance of 21 W. The rate 48. To get maximum current in a resistance of 3 ohms, one can
of chemical energy consumed in providing the current is use n rows of m cells (connected in series) connected in
(a) 0.42 J s–1 (b) 0.84 J s–1 parallel. If the total number of cells is 24 and the internal
(c) 1 J s–1 (d) 5 J s–1 resistance of a cell is 0.5 ohms then
39. Water boils in the electric kettle in 15 minutes after switching (a) m = 12, n = 2 (b) m = 8, n = 3
on. If the length of heating wire is decreased to 2/3 of its (c) m = 2, n = 12 (d) m = 6, n = 4
initial value, then the same amount of water will boil with 49. Each of the resistance in the network shown in fig. is equal
the same supply voltage in to R. The resistance between the terminals A and B is
Current Electricity 491

56. A 4 m long wire of resistance 8 W is connected in series


L with a battery of e.m.f. 2 V and a resistor of 7 W. The internal
resistance of the battery is 1 W. What is the potential
gradient along the wire?
R R R (a) 1.00 V m–1 (b) 0.75 V m–1
A (c) 0.50 V m–1 (d) 0.25 V m–1
R

R B 57. The length of a given cylindrical wire is increased by 100%.


K M Due to the consequent decrease in diameter the change in
the resistance of the wire will be
(a) R (b) 5 R (a) 100% (b) 50% (c) 300% (d) 200%
(c) 3 R (d) 6 R 58. Two resistances R1 and R2 are made of different materials.
50. Four resistors are connected as shown in fig. A 6 V battery The temperature coefficient of the material of R1 is a and
of negligible resistance is connected across terminal AC. that of material of R2 is – b . The resistance of the series
The potential difference across terminals B, D will be combination of R1 and R2 will not change with temperature
6V R1
+ – if equal to
R2
B C D E
A a a +b a 2 + b2 b
5W 15 W 30 W 10 W (a) (b) (c) (d)
b a -b 2ab a
59. Current I1 in the following circuit is

.IN
(a) 0 V (b) 1.5 V 30W
(c) 2 V (d) 3 V
51. A non-conducting ring of radius R has charge Q distributed
AL I1 40W 40V
unevenly over it. If it rotates with an angular velocity w, the
equivalent current will be I3
I2
N
(a) zero (b) Qw
w w 40W
R

80V
(c) Q (d) Q (a) 0.4A (b) – 0.4 A (c) 0.8 A (d) – 0.8 A
2p 2 pR
60. Null point with 1V cell comes out to be 55 cm and with
U

52. All the edges of a block with parallel faces are unequal. Its R = 10 W it is 50 cm. What is the internal resistance of the
JO

longest edge is twice its shortest edge. The ratio of the cell ?
maximum to minimum resistance between parallel faces is 2V
(a) 2 (b) 4
100 cm
U

(c) 8
(d) indeterminate unless the length of the third edge is 1V
ED

specified
R
53. In the network shown below, the ring has zero resistance.
The equivalent resistance between the point A and B is (a) 0.5W (b) 0.4W (c) 1W (d) 0.2W
(a) 2R 61. Three resistances are connected to form a T-shape as shown
3R in the figure. Then the current in the 4K resistor is:
(b) 4R 3R A +2V 2K 8K – 4V
B
(c) 7R R
3R
4K
(d) 10R
54. The belt of an electrostatic generator is 50 cm wide and – 8V
travels at 30 cm/sec. The belt carries charge into the sphere (a) 0.93mA (b) 1.42mA
at a rate corresponding to 10–4 ampere. What is the surface (c) 2.5mA (d) 1.57mA
density of charge on the belt. 62. The equivalent resistance between A and B is
A
(a) 6.7 ´10 -5 C m -2 / s (b) 6.7 ´10 -4 C m -2 / s R R

(c) 6.7 ´10 -7 C m -2 / s (d) 6.7 ´10 -8 C m -2 / s B R


55. A wire has a resistance 12 W. It is bent in the form of a
circle. The effective resistance between two points on any R R
diameter is
8R 5R 3R 7R
(a) 6 W (b) 3 W (c) 12 W (d) 24 W (a) (b) (c) (d)
5 8 8 8
EBD_7179
492 PHYSICS

63. Determine the current in 2W resistor. 68. The numerical value of charge on either plate of capacitor C
2W shown in figure is
3W (a) CE
1W CER1
(b)
R1 + r
6V 2.8 W
C R2
CER 2 ||
(c)
(a) 1 A (b) 1.5 A (c) 0.9 A (d) 0.6 A R2 + r
R1
64. Twelve resistors each of resistance 16 W are connected in
the circuit as shown. The net resistance between AB is CER1
(d) E
R2 + r r

69. In the circuit shown, the internal resistance of the cell is


negligible. The steady state current in the 2W resistor is

(a) 0.6 A

.IN
(b) 0.9 A
(a) 1 W (b) 2 W (c) 3 W (d) 4 W
65. A battery of e.m.f E and internal resistance r is connected AL (c) 1.2 A
to a variable resistor R as shown. Which one of the
following is true ?
E r (d) 1.5 A
N
70. In the network shown, each resistance is equal to R. The
R equivalent resistance between adjacent corners A and D is
R
U

(a) R
(a) Potential difference across the terminals of the battery
JO

is maximum where R = r 2
(b) R
(b) Power delivered to resistor is maximum when 3
R= r
U

(c) Current in the circuit is maximum when R = r 3


(d) Current in the circuit is maximum when R >> r (c) R
7
ED

66. There is an infinite wire grid with cells in the form of


equilateral triangles. The resistance of each wire between
neighbouring joint connections is R0. The net resistance 8
(d) R
of the whole grid between the points A and B as shown is 15
71. Resistances 1 W, 2 W and 3 W are connected to form a
A B triangle. If a 1.5 V cell of negligible internal resistance is
connected across the 3 W resistor, the current flowing
through this resistor will be
R0 R0 R0
(a) R0 (b) (c) (d) (a) 0.25 A (b) 0.5 A
2 3 4 (c) 1.0 A (d) 1.5 A
67. Twelve indentical resistors each of value 1 W are connected
72. Seven resistances, each of value 20 W, are connected to a 2
as shown. Net resistance between C and D (R) is
V battery as shown in the figure. The ammeter reading will
C
be
7
(a) R= W
6 G
4 2V
(b) R= W A E
3 B
F A
(c) R =1 W
H
3 (a) 1/10 A (b) 3/10 A
(d) R= W
4 (c) 4/10 A (d) 7/10 A.
D
Current Electricity 493

73. In the circuit shown below, if the resistance of voltmeter is 80. The thermo e.m.f. E in volts of a certain thermocouple is
4 kW, then the error in the reading of voltmeter will be found to vary with temperature T of hot junction while cold
junction is kept at 0ºC
T2
E = 40 T -
20
The neutral temperature of the couple is
(a) 100ºC (b) 200ºC (c) 400ºC (d) 800ºC
81. Two bulbs of 500 W and 200 W are manufactured to operate
on 220 V line. The ratio of heat produced in 500 W and 200
(a) 50% (b) 68% (c) 17% (d) 33.3% W, in two cases, when firstly they are connected in parallel
74. In the circuit , the galvanometer G shows zero deflection. If and secondary in series will be
the batteries A and B have negligible internal resistance,
the value of the resistor R will be 5 2 5 5 2 5 2 2
(a) : (b) : (c) : (d) :
2 5 2 2 5 2 5 5
500 W
G 82. A wire of resistance 20 W is covered with ice and a voltage
2V of 210 V is applied across the wire, then rate of melting the
12V R
ice is
B A
(a) 0.85 g/s (b) 1.92 g/s

.IN
(c) 6.56 g/s (d) All of these
(a) 100 W (b) 200W 83. Two identical batteries, each of e.m.f. 2 volt and internal
(c) 1000 W (d) 500 W
AL resistance 1.0 ohm are available to produce heat in a
resistance R = 0.5 W, by passing a current through it. The
75. A fuse wire with a radius of 1 mm blows at 1.5 A. If the fuse
maximum power that can be developed across R using these
wire of the same material should blow at 3.0 A, the radius of
batteries is
N
the wire must be
(a) 41/3 mm (b) 2 mm 1W 2V
R

(c) 0.5 mm (d) 8.0 mm


U

76. A 4 m F conductor is charged 50 volts and then its plates 1W 2V


are joined through a resistance of 1 k W. The heat produced
JO

in the resistance is
(a) 0.16 J (b) 1.28 J (c) 0.64 J (d) 0.32 J
77. The thermo e.m.f. of a thermocouple is given by
U

E = 2164 t – 6.2 t2. The neutral temperature and a temperature 0.5W


of inversion are
ED

(a) 1.28 W (b) 2.0 W (c) 8/9 W (d) 3.2 W


(a) 349, 174.5 (b) 174.5, 349
84. The thermo e.m.f. of a thermocouple is 25mV/ºC at room
(c) 349, 698 (d) 698, 349
78. One junction of a certain thermocouple is at a fixed temperature. A galvanometer of 40 ohm resistance, capable
temperature Tr and the other junction is at a temperature T. of detecting current as low as 10–5 A, is connected with the
The electormotive force for this is expressed by, thermocouple. The smallest temperature difference that can
be detected by this system is
é 1 ù (a) 12ºC (b) 0ºC (c) 20ºC (d) 16ºC
E = k (T - Tr ) êT0 - (T + Tr ) ú
ë 2 û 85. Two different metals are joined end to end. One end is kept
At temperature T = T0/2, the thermo electric power is at constant temperature and other end is heated to a very
(a) k T0/2 (b) k T0 high temperature. The graph depicting the thermo e.m.f. is

k T02 1 2
(c) (d) k (T0 - Ti )
2 2
79. The e.m.f. developed in a thermo-couple is given by (a) (b)
1
E = a T + bT2 T T
2
where T is the temperature of hot junction, cold junction
being at 0ºC. The thermo electric power of the couple is
b
(a) a+ T (b) a + b T (c) (d)
2
a T 2 b T3
(c) + (d) a / 2 b
2 6
EBD_7179
494 PHYSICS

86. The cold junction of a thermocouple is maintained at 10ºC. 96. In the circuit shown in figure, the 5W resistance
No thermo e.m.f. is developed when the hot junction is develops 20.00 cal/s due to the current flowing through it.
maintained at 530ºC. The neutral temperature is The heat developed in 2 W resistance (in cal/s) is
(a) 260ºC (b) 265ºC (c) 270ºC (d) 520ºC
87. Three equal resistors connected across a source of e.m.f.
together dissipate 10 watt of power. What will be the power
dissipated in watts if the same resistors are connected in
parallel across the same source of e.m.f.?
(a) 23.8 (b) 14.2 (c) 11.9 (d) 7.1
10 97. A heating coil is labelled 100 W, 220 V. The coil is cut in half
(a) 10 (b) (c) 30 (d) 90
3 and the two pieces are joined in parallel to the same source.
88. An electric heating element in vacuum is surrounded by a The energy now liberated per second is
cavity at constant temperature of 227ºC; it consumes 60W (a) 25 J (b) 50 J (c) 200 J (d) 400 J
of power to maintain a temperature of 727ºC. What is the 98. Five resistances have been connected as shown in the
power consumed by the element to maintain a temperature figure. The effective resistance between A & B is
of 1227º C?
(a) 101 W (b) 304 W (c) 90 W (d) 320 W 3W 4W
89. Silver and copper voltameters are connected in parallel A 7W B
with a battery of e.m.f 12 V. In 30 minute 1 g of silver and
6W 8W

.IN
1.8 g of copper are liberated. The energy supplied by the
battery is (a) 14/3W (b) 20/3W
[ ZAg = 11.2 × 10–4gc–1; ZCu = 6.6 × 10–4 gc–1] (c) 14W (d) 21W
If specific resistance of a potentiometer wire is 10–7Wm
(a) 720 J (b) 2.41 J
AL
99.
(c) 24.12 J (d) 4.34 × 104 J current flowing through it, is 0.1 amp and cross sectional
90. A 5–ampere fuse wire can withstand a maximum power area of wire is 10–6 m2, then potential gradient will be
N
of 1 watt in the circuit. The resistance of the fuse wire is (a) 10–2 volt/m (b) 10–4 volt/m
–6
(c) 10 volt/m (d) 10–8 volt/m
(a) 0.04 W (b) 0.2 W (c) 5 W (d) 0.4 W
R

91. In the Seebeck series Bi occurs first followed by Cu and Fe 100. If 25W, 220 V and 100 W, 220 V bulbs are connected in
among other. The Sb is the last in the series. If z1 be the series across a 440 V line, then
U

thermo emf at the given temperature difference for Bi – Sb (a) only 25W bulb will fuse
JO

thermocouple and z, be that for Cu-Fe thermocouple, which (b) only 100W bulb will fuse
of the following is true? (c) both bulbs will fuse
(a) z1 = z2 (b) z1 < z2 (c) z1 > z2 (d) None of these
U

(d) Data is not sufficient to predict it. 101. An electric kettle has two heating coils. When one of the
92. 50 electric bulbs are connected in series across a 220 V coils is connected to an a.c. source, the water in the kettle
ED

supply and the illumination produced is I1. 5 bulbs are boils in 10 minutes. When the other coil is used the water
fused. If the remaining 45 are again connected in series, boils in 40 minutes. If both the coils are connected in parallel,
the illumination produced is I2. Which of the following is the time taken by the same quantity of water to boil will be
true ? (a) 15 min (b) 8 min (c) 4 min (d) 25 min
(a) I1 = I2 (b) I1 < I2 (c) I1 > I2 102. Two 220 volt, 100 watt bulbs are connected first in series
(d) It will depend on the resistance of each bulb. and then in parallel. Each time the combination is connected
93. A leclanche cell supplies a current of one ampere for ten to a 220 volt a.c. supply line. The power drawn by the
minutes. The electreochemical equivalent of hydrogen = combination in each case respectively will be
0.00001014 gram per coulomb. The mass of hydrogen (a) 50 watt, 200 watt (b) 50 watt, 100 watt
liberated is : (c) 100 watt, 50 watt (d) 200 watt, 150 watt
(a) 0.00625 g (b) 0.01248 g 103. The internal resistance of a 2.1 V cell which gives a current
(c) 0.01872 g (d) 0.02496 g. of 0.2 A through a resistance of 10 W is
94. Three equal resistors connected across a source of e.m.f. (a) 0.5 W (b) 0.8 W
together dissipate 10 watt of power. What will be the power (c) 1.0 W (d) 0.2 W
dissipated in watt if the same resistors are connected in parallel 104. Five equal resistances each of resistance R are connected
across the same source of e.m.f.? as shown in the figure. A battery of V volts is connected
(a) 10/3 (b) 10 (c) 30 (d) 90 between A and B. The current flowing in AFCEB will be
95. Two 1000 W heaters when connected in parallel across C
2V 3V
220 V supply produced heat QP in time t. If they are (a) (b)
connected in series across the same power supply the heat R R
R
produced in the same time is QS. What is QP/QS? V V R F
R
(a) 4 (b) 2 (c) 0.5 (d) 0.25. (c) (d) A
R 2R R B
D E
R
Current Electricity 495

105. A battery is charged at a potential of 15V for 8 hours when (c) (d)
the current flowing is 10A. The battery on discharge e V
supplies a current of 5A for 15 hour. The mean terminal V
voltage during discharge is 14V. The “Watt-hour”
efficiency of the battery is
(a) 87.5% (b) 82.5% (c) 80% (d) 90% 0
R 0
106. When three identical bulbs of 60 watt, 200 volt rating are R
connected in series to a 200 volt supply, the power drawn 113. A milli voltmeter of 25 milli volt range is to be converted
by them will be into an ammeter of 25 ampere range. The value (in ohm) of
(a) 20 watt (b) 60 watt (c) 180 watt (d) 10 watt necessary shunt will be
107. In India electricity is supplied for domestic use at 220 V. It (a) 0.001 (b) 0.01
is supplied at 110 V in USA. If the resistance of a 60 W (c) 1 (d) 0.05
bulb for use in India is R, the resistance of a 60 W bulb for 114. In the circuit shown the cells A and B have negligible
use in USA will be resistances. For VA = 12V, R1 = 500W and R = 100W the
(a) R/2 (b) R (c) 2R (d) R/4 galvanometer (G) shows no deflection. The value of VB is
108. For the network shown in the figure the value of the current
R1
i is
G
9V
(a)
35
18V VA R VB
(b)

.IN
5
5V
(c) (a) 4 V (b) 2 V
9
5V
AL (c) 12 V (d) 6 V
(d) 115. If voltage across a bulb rated 220 Volt-100 Watt drops by
18
2.5% of its rated value, the percentage of the rated value by
109. In producing chlorine by electrolysis 100 kW power at 125
N
which the power would decrease is
V is being consumed. How much chlorine per minute is
(a) 20% (b) 2.5%
liberated? (E.C.E. of chlorine is 0.367×10–6 kg / C)
R

(a) 1.76 × 10–3 kg (b) 9.67 × 10–3 kg (c) 5% (d) 10%


(c) 17.61 × 10 kg–3 (d) 3.67 × 10–3 kg 116. A wire of resistance 4 W is stretched to twice its original
U

110. In the circuit shown in the figure, if potential at point A is length. The resistance of stretched wire would be
(a) 4 W (b) 8 W
JO

taken to be zero, the potential at point B is


(c) 16 W (d) 2 W
R1 D 2V
117. The resistance of the four arms P, Q, R and S in a
1A B
Wheatstone’s bridge are 10 ohm, 30 ohm, 30 ohm and 90
U

ohm, respectively. The e.m.f. and internal resistance of the


2A cell are 7 volt and 5 ohm respectively. If the galvanometer
R2
ED

resistance is 50 ohm, the current drawn from the cell will be


(a) 0.2 A (b) 0.1 A
A (c) 2. 0 A (d) 1. 0 A
1V 1A C 2A
(a) –1V (b) + 2V Directions for Qs. (118 to 125) : Each question contains
(c) –2V (d) + 1V STATEMENT-1 and STATEMENT-2. Choose the correct answer
111. The power dissipated in the circuit shown in the figure is 30 (ONLY ONE option is correct ) from the following-
Watts. The value of R is (a) Statement -1 is true, Statement-2 is true; Statement -2 is a
R
(a) 20 W correct explanation for Statement-1
(b) Statement -1 is true, Statement-2 is true; Statement -2 is not
(b) 15 W
5W a correct explanation for Statement-1
(c) 10 W (c) Statement -1 is true, Statement-2 is false
(d) 30 W 10V (d) Statement -1 is false, Statement-2 is true
112. Cell having an emf e and internal resistance r is connected 118. Statement 1 : When a battery is short-circuited, the terminal
across a variable external resistance R. As the resistance R voltage is zero.
is increased, the plot of potential difference V across R is Statement 2 : In the situation of a short-circuit, the current
given by
is zero
(a) (b) 119. Statement 1 : A current flows in a conductor only when
e e
V there is an electric field within the conductor.
V
Statement 2 : The drift velocity of electron in presence of
electric field decreases.
0 0
R R
EBD_7179
496 PHYSICS

120. Statement 1 : Voltmeter is much better than a potentiometer 123. Statement 1 : Resistance of 50W bulb is greater than that
for measuring emf of cell. of 100 W.
Statement 2 : A potentiometer draws no current while Statement 2 : Resistance of bulb is inversely proportional
measuring emf of a cell. to rated power.
121. Statement 1 : When current through a bulb decreases by 124. Statement 1 : 40 W tube light give more light in comparison
0.5%, the glow of bulb decreases by 1%. to 40 w bulb.
Statement 2 : Glow (Power) which is directly proportional Statement 2 : Light produced is same from same power.
to square of current. 125. Statement 1: In a simple battery circuit, the point of the
122. Statement 1 : Long distance power transmission is done lowest potential is negative terminal of the battery.
at high voltage. Statement 2: The current flows towards the point of the
Statement 2 : At high voltage supply power losses are higher potential, as it does in such a circuit from the negative
less. to the positive terminal.

Exemplar Questions (c) The first portion of 50 cm of wire itself should have a

.IN
1. Consider a current carrying wire (current I) in the shape of potential drop of 10 V
a circle. (d) Potentiometer is usually used for comparing resistances
(a) source of emf
AL and not voltages
(b) electric field produced by charges accumulated on the 5. A metal rod of length 10 cm and a rectangular cross-section
surface of wire 1
of 1cm × cm is connected to a battery across opposite
(c) the charges just behind a given segment of wire which 2
N
push them just the right way by repulsion faces. The resistance will be
(d) the charges ahead (a) maximum when the battery is connected across 1 cm ×
R

2. Two batteries of emf e1 and e2(e2 > e1 ) and internal 1


U

resistances r1 and r2 respectively are connected in parallel cm faces


as shown in figure. 2
JO

(a) Two equivalent emf eeq of the two cells is between e1 (b) maximum when the battery is connected across 10 cm ×
and e2, i.e., e1 < eeq < e2 1 cm faces
(b) The equivalent emf eeq is smaller than e1 (c) maximum when the battery is connected across 10 cm ×
U

(c) The eeq is given by eeq = e1 + e2 always 1


(d) eeq is independent of internal resistances r1 and r2 cm faces
2
ED

e1 (d) same irrespective of the three faces


r1
6. Which of the following characteristics of electrons
A B determines the current in a conductor?
r2 (a) Drift velocity alone
e2
(b) Thermal velocity alone
3. A resistance R is to be measured using a meter bridge, (c) Both drift velocity and thermal velocity
student chooses the standard resistance S to be 100W. He (d) Neither drift nor thermal velocity
finds the null point at l1 = 2.9 cm. He is told to attempt to
improve the accuracy. NEET/AIPMT (2013-2017) Questions
Which of the following is a useful way? 7. A wire of resistance 4 W is stretched to twice its original
(a) He should measure I1 more accurately length. The resistance of stretched wire would be [2013]
(b) He should change 5 to 1000W and repeat the experiment (a) 4 W (b) 8 W
(c) He should change S to 3W and repeat the experiment (c) 16 W (d) 2 W
(d) He should given up hope of a more accur ate
8. The internal resistance of a 2.1 V cell which gives a current
measurement with a meter bridge
of 0.2 A through a resistance of 10 W is [2013]
4. Two cells of emfs approximately 5 V and 10 V are to be
accurately compared using a potentiometer of length 400 (a) 0.5 W (b) 0.8 W
cm. (c) 1.0 W (d) 0.2 W
(a) The battery that runs the potentiometer should have 9. The resistance of the four arms P, Q, R and S in a
voltage of 8V Wheatstone’s bridge are 10 ohm, 30 ohm, 30 ohm and 90
(b) The battery of potentiometer can have a voltage of 15 ohm, respectively. The e.m.f. and internal resistance of the
V and R adjusted so that the potential drop across the cell are 7 volt and 5 ohm respectively. If the galvanometer
wire slightly exceeds 10 V resistance is 50 ohm, the current drawn from the cell will be
Current Electricity 497

(a) 0.2 A (b) 0.1 A [2013] 15. Two cities are 150 km apart. Electric power is sent from one
(c) 2. 0 A (d) 1. 0 A city to another city through copper wires. The fall of
potential per km is 8 volt and the average resistance per km
10. A 12 cm wire is given a shape of a right angled triangle ABC
is 0.5 W. The power loss in the wires is : [2014]
having sides 3 cm, 4 cm and 5 cm as shown in the figure.
(a) 19.2 W (b) 19.2 kW
The resistance between two ends (AB, BC, CA) of the
(c) 19.2 J (d) 12.2 kW
respective sides are measured one by one by a multi-meter.
16. The resistances in the two arms of the meter bridge are 5W
The resistances will be in the ratio of [NEET Kar. 2013]
and RW, respectively. When the resistance R is shunted
A with an equal resistance, the new balance point is at 1.6 l1.
The resistance ‘R’ is : [2014]
3 cm 5 cm

5W RW
B C
4 cm
(a) 3 : 4 : 5 (b) 9 : 16 : 25
(c) 27 : 32 : 35 (d) 21 : 24 : 25 G
11. Two rods are joined end to end, as shown. Both have a
cross-sectional area of 0.01 cm2. Each is 1 meter long. One A B
l1 100 – l1
rod is of copper with a resistivity of 1.7 × 10–6 ohm-centimeter,

.IN
the other is of iron with a resistivity of 10–5 ohm-centimeter. (a) 10W (b) 15W
How much voltage is required to produce a current of 1 (c) 20W (d) 25W
ampere in the rods? [NEET Kar. 2013] 17. Across a metallic conductor of non-uniform cross section
AL
V a constant potential difference is applied. The quantity
which remains constant along the conductor is : [2015]
(a) current (b) drift velocity
N
(c) electric field (d) current density
18. A potentiometer wire has length 4 m and resistance 8W.
R

The resistance that must be connected in series with the


Cu Fe wire and an accumulator of e.m.f. 2V, so as to get a potential
U

(a) 0.117 V (b) 0.00145 V gradient 1 mV per cm on the wire is [2015]


(d) 1.7 × 10–6 V (a) 40 W (b) 44 W
JO

(c) 0.0145 V
12. Ten identical cells connected in series are needed to heat a (c) 48 W (d) 32 W
wire of length one meter and radius ‘r’ by 10ºC in time ‘t’. 19. A, B and C are voltmeters of resistance R, 1.5 R and 3R
respectively as shown in the figure. When some potential
U

How many cells will be required to heat the wire of length


two meter of the same radius by the same temperature in difference is applied between X and Y, the voltmeter readings
ED

time ‘t’? [NEET Kar. 2013] are VA, VB and VC respectively. Then [2015]
(a) 10 (b) 20
B
(c) 30 (d) 40
13. A potentiometer circuit has been set up for finding the A
X Y
internal resistance of a given cell. The main battery used C
across the potentiometer wire, has an emf of 2.0 V and a
negligible internal resistance. The potentiometer wire itself (a) VA ¹ VB = VC (b) VA = VB ¹ VC
is 4m long, When the resistace R, connected across the (c) VA ¹ VB ¹ VC (d) VA = VB = VC
given cell, has values of 20. A circuit contains an ammeter, a battery of 30V and a
(i) infinity (ii) 9.5W resistance 40.8W all connected in series. If the ammeter has
The balancing lengths’, on the potentiometer wire are found a coil of resistance 480W and a shunt of 20W, the reading in
to be 3 m and 2.85 m, respectively. The value of internal the ammeter will be: [2015 RS]
resistance of the cell is [2014] (a) 0.25 A (b) 2A
(a) 0.25W (b) 0.95W (c) 1 A (d) 0.5 A
21. Two metal wires of identical dimension are connected in
(c) 0.5W (d) 0.75W
series. If s1 and s2 are the conductivities of the metal wires
14. In an ammeter 0.2% of main current passes through the respectively, the effective conductivity of the combination
galvanometer. If resistance of galvanometer is G, the is : [2015 RS]
resistance of ammeter will be : [2014]
s1 + s 2 s1 + s 2
1 499 (a) (b)
(a) G (b) G 2s1s2 s1s 2
499 500
1 500 s1s 2 2s1s2
(c) G (d) G (c) (d) s + s
500 499 s1 + s 2 1 2
EBD_7179
498 PHYSICS

22. A potentiometer wire of length L and a resistance r are 24. A potentiometer wire is 100 cm long and a constant potential
connected in series with a battery of e.m.f. E0 and a resistance difference is maintained across it. Two cells are connected
in series first to support one another and then in opposite
r1. An unknown e.m.f. E is balanced at a length l of the direction. The balance points are obtained at 50 cm and 10
potentiometer wire. The e.m.f. E will be given by: [2015 RS] cm from the positive end of the wire in the two cases. The
ratio of emf's is : [2016]
E0r l E 0l (a) 5 : 1 (b) 5 : 4
(a) . (b)
(r + r1 ) L L (c) 3 : 4 (d) 3 : 2
25. A potentiometer is an accurate and versatile device to make
LE 0 r LE 0 r electrical measurements of E.M.F. because the method
(c) (d) involves [2017]
(r + r1 )l lr1
(a) Potential gradients
23. The charge flowing through a resistance R varies with time (b) A condition of no current flow through the
galvanometer
t as Q = at – bt2, where a and b are positive constants. The
(c) A combination of cells, galvanometer and resistances
total heat produced in R is: [2016] (d) Cells
26. The resistance of a wire is 'R' ohm. If it is melted and
a 3R a 3R stretched to 'n' times its original length, its new resistance
(a) (b)
6b 3b will be :- [2017]

.IN
R
a 3R a 3R (a) (b) n2R
(c) (d) n
2b b AL R
(c) (d) nR
n2
N
R
U
JO
U
ED
Current Electricity 499

Hints & Solutions


EXERCISE - 1 Þ R 2 ( R 1 - R 2 ) = R 1R 2 ( R 1 - R 2 )
1. (c)
l Þ R= R 1R 2
2. (c) Rµ ;
A P1 R 2 2
So, the resistance of the wire will be minimum when 16. (c) P = V 2 / R or P µ 1 / R \ = = .
P2 R 1 1
the area of cross-section is maximum and length is
17. (a) Galvani made the statement “Chemical change can
minimum.
produce electricity”.
3. (d) I = n A e vd or vd µ 1/ p r 2 18. (a) Joule effect H = I2RT
4. (c) As we more from A to B the potential difference across When current flows heat is produced. But, by heating
AC increases and hence the reading of voltmeter also conduction current cannot produced.
increases. But currnet flowing through ammeter 19. (b)
remains almost constant. 20. (c)
5. (a) r l ¢ r ( l / 2) 1
6. (a) 7. (c) 21. (a) R = r l / A; R ¢ = = = R.
A¢ 2A 4

.IN
8. (a)
9. (b) 10. (b) 22. (c)
23. (b) m @ L, v @ I
Ig R IgR
11. (b) S= ÞS= R=
AL
nI g - I g (n - 1)I g n -1 rl
24. (c) R=
A
12. (c) Internal resistance = r, External resistance = nr.
N
Let terminal voltage = V When wire is cut into 4 pieces and connected in
parallel.
R

Er
then V = E - Ir Þ V = E - R
U

(n + 1)r R eff. = Þ PC = 16P


16
JO

nE V n
V= Þ = V 2 V2 V2 V2
n +1 E n +1 PA : PB : PC : PD :: : : :
13. (a) According to maximum power theorem, the power in R R / 4 R / 16 R / 2
U

the circuit is maximum if the value of external resistance


is equal to the internal resistance of battery. r l1
R= , now l2 = 2l1
ED

25. (c)
A1
Ig I G
14. (d) S= ´G = ´G = A 2 = p(r2)2 = p (2r1)2 = 4p r12 = 4A1
I - Ig ( nI - I ) ( - 1)
n
r(2 l1 ) r l R
15. (d) Let internal resistance of source = R \ R2 = = =
4A1 2A 2
V
Current in coil of resistance R1 = I1 = \ Resistance is halved, but specific resistance
R + R1
remains the same.
V
Current in coil of resistance R 2 = I 2 = EXERCISE - 2
R + R2
1. (c) Numbers attached for brown, black, green and silver
Further, as heat generated is same, so are 1, 0, 5, ± 10%. Therefore the resistance of given
resistor
I12 R1t = I 2 2 R 2 t
= 10 ´ 10 5 W ± 10% = 1.0 ´ 10 6 W ± 10%.
2 2
æ V ö æ V ö V2 V2
or çç ÷÷ R1 = çç ÷÷ R 2 R1 = and R 2 =
2. (b)
è R + R1 ø è R + R2 ø P1 P2

Þ R1 (R + R 2 ) 2 = R 2 (R + R1 ) 2 R 2 P1 200
\ = = =2 ( Q V = constant)
R1 P2 100
2
Þ R R1 + R 1R 22 + 2RR 1R 2
R 2 = 2 R1
2 2
= R R 2 + R 1 R 2 + 2RR 1 R 2 ?
EBD_7179
500 PHYSICS

3. (b) Current density J = I/A


1 1 1
-6 -6 -2 16. (c) R1 = W; R 2 = W; R 3 = W
= 50 ´16 / 50 ´10 = 1 Am 2 4 6
4. (a)
5. (c) r = E / I = 1.5 / 3 = 0.5 ohm. In series; R s = R1 + R 2 + R 3 = 1 + 1 + 1 = 11
6. (a) Current flowing through the conductor, 2 4 6 12
I = n e v A. Hence
1 æ 12 ö
2 \ ss = =ç ÷
4 nevd1 p(1) vd 4 ´ 1 16 R s è 11 ø
= or 1 = = .
1 nevd p(2) 2 vd 2 1 1
2 17. (a) R1 = 1 / 2 W, R 2 = 1 / 4 W; R 3 = 1 / 6 W

(1.5) 2 1 1 1 1
7. (d) Resistance of bulb R b = = 0.5 W In parallel; = + + = 2 + 4 + 6 = 12
4.5 R p R1 R 2 R 3
E E
Current drawn from battery = = 1
2.67 + 0.33 3 \ sp = = 12 S
Rp
2 E 2E
Share of bulb = ´ = 18. (d) Case (I) : E + E = (r + r + 5) 1 or 2 E = 2 r + 5 ...(i)
3 3 9

.IN
æ r´r ö ær ö
æ 2E ö
2 Case (II) : E = ç + 5 ÷ ´ 0.8 or E = ç + 5 ÷ 0.8
\ ç ÷ ´ 0.5 = 4.5 or E = 13.5 V.. è r + r ø è 2 ø
è 9 ø
AL or E = 0.4 r + 4.0 ...(ii)
8. (c) I = q / t = 10 7 ´1.6 ´10 -19 / 1 = 1.6 ´ 10 -12 A Multiplying (ii) by 2 and equating with (i), we get
N
3
R1 V 2 / P1 P2 2 r + 5 = 0.8 r + 8 or 1.2 r = 3 or r = = 2.5
9. (b) = = 1.2
R

R 2 V 2 / P2 P1
19. (a) [Hint Þ Rt = Ro (1 + a t)]
U

5W = R0 (1 + a × 50) and 7W = R0 (1 + a × 100)


rl / p(d1 / 2) 2 d 2 100W
= = 2 = 5 1 + 50a
JO

2
rl / p(d 2 / 2) 2 d12 25W or = or a = = 0.0133/ °C
7 1 + 100a 150
d 2 10 2
Þ = = (b) (Hint Þ r = R .A = Coefficient of resistivity)
U

d1 5 1 20.
l
ED

P l
10. (d) = or P = l ´ Q = 20 ´ 1 = 0.25W. 21. (a) Hint : Potential gradient =
Pot.Difference VA - VB
=
Q (100 - l ) 100 - l 80 length of wire l
11. (b) As R µ V2/P or R µ 1/P, so resistance of heater is less 22. (c) Kirchhoff’s junction rule states that the algebraic sum
than that of fan. of all currents into and out of any branch point is zero :
SI = 0. By convention, the sign of current entering a
12. (b) Current, I = (2.9 ´ 1018 + 1.2 ´1018 ) × 1.6 × 10–19 junction is positive and current leaving a junction is
towards right. negative.
4A + 5 A – 6A + IAB = 0, therefore IAB = – 3A. The wire
q CV (10 ´ 106 ) ´ 40 between points A and B carries a current of 3A away
13. (a) Current I = = = = 2 ´ 10 -3 A
t t 0. 2 from the junction.
23. (a) Pot. gradient = 0.2mV/cm
dQ
14. (c) I = = 10t + 3
dt 0.2 ´ 10 - 3
= = 2 ´ 10 - 2 V / m
At t = 5s, I = 10 × 5 + 3 = 53 A 10 - 2

rl l Emf of cell = 2×10–2×1m = 2 ´ 10 - 2 V = 0.02 V


15. (c) R= or R µ .
( p D 2 / 4) D2 As per the condition of potentiometer
0.02 (R + 490) = 2 (R) or 1.98 R = 9.8
Rx l D y2 ly / 2 D y2 2 9.8
= x2 ´ = ´ = Þ R= = 4.9 W
R y Dx ly (D y / 2)2 l y 1 1.98
Current Electricity 501

24. (b) Case (I) : When resistor is not connected 32. (b) Rt = R0 (1 + at)
Using V = IR Þ V = 25 (R G ) .............. (i) Initially, R0 (1 + 30a) = 10 W
Finally, R0 (1 + at) = 11 W
Case (II) : When resistor is connected
11 1 + at
V = 5(20 + R G ) = 100 + 5 R G ............. (ii) \ =
10 1 + 30a
From (i) and (ii), 20 R G = 100 Þ R G = 5 W or, 10 + (10 × 0.002 × t) = 11 + 330 × 0.002
25. (a) Rg = 50W, Ig = 25 × 4 × 10–4W = 10–2 A
1.66
Range of V = 25 volts or, 0.02t = 1 + 0.66 = 1.066 or t = = 86°C.
0.02
V = Ig(HR + Rg)
33. (b) Potential gradient along wire
V
\ HR = - R g = 2450W potential difference along wire
Ig =
length of wire
R
A B I ´ 40
or, 0.1 ´10 -3 = V / cm
1000
Ig HR Rg
1
26. (a) J = sE Þ J r = E or, Current in wire, I= A
400

.IN
J is current density, E is electric field
so B = r = resistivity. 2 1
or, = or R = 800 - 40 = 760 W
27. (b) Energy produced = V I t = 6 × 0.5 × 1 = 3.0 J. 40 + R 400
28. (c) Mass of substance deposited
AL
34. (c) As R= V2/P or R µ 1/P
m = Z I t = Z ´ 4 ´ (2 ´ 60) = 480 Z and so R2/R1 = P1 /P2 = 200/100 = 2
N
m ¢ = Z ´ 6 ´ 40 = 240 Z or m ¢ = m / 2. or R2 = 2 R1.
29. (a) Electrochemical equivalent, 35. (b) If R is the resistance of each wire, total resistance in
R

m 4.572 series = R + R = 2 R; and total resistance in parallel


Z= = = 3.387 ´ 10 -4 g/C.
U

Ι t 5 ´ 45 ´ 60 R´R R
= = .
30. (c) The effective circuit will be as shown in the figure. R +R 2
JO

Heat produced per second (= V2/R) will be four times


in parallel than in series.
36. (b) For maximum current, the two batteries should be
U

connected in series. The current will be maximum when


ED

external resistance is equal to the total internal


resistance of cells i.e. 2 W. Hence power developed
across the resistance R will be
Effective resistance of R2 and R4 in series, 2
æ 2E ö 2´2 ö
R' = 10 + 10 = 20 W. 2
= I R = çç ÷÷ R = æç ÷ ´ 2 = 2 W.
Effective resistance of R3 and R5 in series, è R + 2 r ø è +2ø
2
R'' = 10 + 10 = 20 W
Net total resistance of R' and R'' in parallel is V2t
37. (d) Heat produced, H = . When voltage is halved,
20 ´ 20 R
R1 = = 10W.
20 + 20 the heat produced becomes one fourth. Hence time
\Total resistance between A and D taken to heat the water becomes four time.
= 10 + 10 + 10 = 30 W. 38. (c) Chemical energy consumed per sec
31. (c) Resistance of a wire = rl / A. = heat energy produced per sec.
For the same length and same material,
= Ι2 (R + r) = (0.2)2 (21 + 4) = 1Js -1 .
R 2 A1 3
= = or, R 2 = 3R1
R1 A 2 1 V2 V2
39. (b) H= ´ 15 ´ 60 = ´t
The resistance of thick wire, R1 = 10 W R (2/3)R
The resistance of thin wire = 3R1 = 3 × 10 = 30 W.
Total resistance = 10 + 30 = 40W. 2
or t = ´ 15 ´ 60 = 600 s = 10 minutes.
3
EBD_7179
502 PHYSICS

40. (b) Energy consumed per day = P × t = 60 × 8 = 480 watt mr


hour = 480/1000 = 0.48 kWh or unit of electricity. It is minimum for R = ...(2)
n
Hence the cost = 0.48 × 1.25 = Re 0.60. So maximum current in external circuit is
41. (b) As P = Ι 2 R, so P1 = (1.01 Ι)2 R = 1.02 I 2 R = 1.02 P. mE ...(3)
I=
It means % increase in power 2R
m
æP ö here R =3, r = 0.5 so equation (2) become =6
= ç 1 - 1÷ ´ 100 = 2%. n
è P ø so n = 2, m = 12
42. (c) (Hint : m = Z it, where Z is electro chemical equivalent) 49. (a) The equivalent circuit is shown in fig. Since the
Wheatstone’s bridge is balanced, therefore no current
43. (d) (Hint : P = V2/R)
will flow through the arm KL. Equivalent resistance
l between
44. (a) H = I2 Rt. Here R1 = r and AKM = R + R = 2 R
p r2
Equivalent resistance between ALM = R + R = 2 R
The two resistances are in parallel. Hence equivalent
l H1 resistance between A and B is given by
R 2 =r That is, R1 = 4R2. Hence, =4
p ( 2r )
2 H2

.IN
K

45. (a) m = ZIt.


R R
46. (a) Let the resistance of single copper wire be R1. If r is
the specific resistance of copper wire, then
AL
A R M B

r´l r´l
N
R1 = = 2 ...(1)
A1 p r1 R R
R

When the wire is replaced by six wires, let the


L
U

resistance of each wire be R2. Then


JO

r´l r´l 1 1 1 2 1
R2 = = 2 = + = =
A2 ...(2) R¢ 2 R 2 R 2 R R
p r2
i.e., R ¢ = R
U

From eqs. (1) and (2), we get 50. (a) The given figure is a circuit of balanced Wheatstone’s
ED

bridge. Point B and D would be at the same potential


R1 r2 2 5 (3 ´103 ) 2 i.e., potential difference between these points is zero.
= or =
R 2 r12 R 2 (9 ´10 -3 ) 2 ; R 2 = 45 W 51. (c) With each rotation, charge Q crosses any fixed point
P near the ring. Number of rotations per second = w/2p.
These six wires are in parallel. Hence the resistance of
Qw
the combination would be R 2 = 7.5 W \ charge crossing P per second = current =
2p
47. (b) In case of internal resistance measurement by 52. (b) Let the edges be 2l, a, and l, in decreasing order.
potentiometer,
V1 l1 {E R 1 /(R1 + r )} R 1 (R 2 + r) 2l 2r l r
= = = R max = r = , R min = r = ;
V2 l 2 {E R 2 /(R 2 + r )} R 2 (R 1 + r) al a 2 la 2 a

Here l 1 = 2 m, l 2 = 3 m, R1 = 5 W and R 2 = 10 W R max


= 4.
2 5 (10 + r ) R min
\ = or 20 + 4 r = 30 + 3 r or r = 10 W
3 10 (5 + r ) 53. (a) As the ring has no resistance, the three resistances of
48. (a) Let, we connect 24 cells in n rows of m cells, then if I is 3R each are in parallel.
the current in external circuit then
1 1 1 1 1
mE
...(1) Þ = + + = Þ R¢ = R
I= R ¢ 3R 3R 3R R
mr / n + R
For I to be maximum, (mr + nR) should be minimum. \ between point A and B equivalent resistance
= R+ R = 2R
Current Electricity 503

-30I1 + 40 - 40I1 - 40I 2 = 0


54. (b) J = I / A = 10-4 /(0.30 ´ 0.50).
or , - 70I1 - 40I 2 = -40
-4 -2 -4 -2
= 6.7 ´ 10 C m / s = 6.7 ´10 Am or, 7I1 + 4I2 =4 … (iv)

55. (b) Resistance of the wire of a semicircle = 12/2 = 6W and 40I1 + 40I 2 - 40 - 80 + 40I 2 = 0
For equivalent resistance between two points on any or , 40I1 + 80I 2 = 120 or , 4I1 + 8I 2 = 12
diameter, 6W and 6W are in parallel. or , 2I1 + 4I 2 = 6..............(v )
or
(iv ) - (v ) gives 5I1 = -2 I1 = -0.4 A
If a wire of resistance R is bent in the form of a circle,
60. (c) Length of wire = 100cm.
the effective resistance between the ends of a diameter
= R/4. 2V = voltage of battery, 1V = voltage of a cell
When current flows along wire electric potential falls
2 1 continuously along wire.
56. (d) Current in the potentiometer, I = = A.
8 + 7 +1 8
V 2V
1 100cm
Voltage drop across potentiometer wire = ´ 8 = 1 V
8 B

.IN
A
\ Potential gradient of potentiometer wire E 1V

=
1
= 0.25 V / m
AL
4

E
N
R1 æ l1 ö
2
R æ 1ö
2 Potential gradient = fall in potential length, K =
57. (c) = Þ = ç ÷ l
R 2 èç l 2 ø÷ R ¢ è 2ø
R

because in null point position, E = V


U

Þ R ¢ = 4R or DR = 3R E1 l1 R1
\ = = ..............(1)
JO

\ % change in resistance of wire E 2 l2 R 2

3R For internal resistance, E = E1, r = E 2


= ´100 = 300%,
U

R E = iR + ir
ED

58. (d) R1 + R2 = Constant, R1 will increase, R2 will decrease. æE ö


E - V = ir (Q i = V / R ) r = ç - 1÷ R
èV ø
R1aD T - RbDT = 0 Þ R1aDT = R 2bD T
æl ö æ 55 ö æ 5 ö
R b Þ r = çç 1 - 1÷÷R . = 10ç - 1÷ = 10ç ÷ = 1W
\ 1 = l
è 2 ø è 50 ø è 50 ø
R2 a
61. (d) Let potential of X be x.
59. (b) 30W Now current from AX and BX will pass through XC.
A B
I1 40W 40V
C +2V 2K X 8K –4V
F
I2 I3 A B
40W
E D
80V 4K
Applying 1st law of Kirchoff’s, I3 = I1 + I2 … (i)
–8V
Applying 2nd law of Kirchhoff C
in Mesh ABCFA
-30I1 + 40 - 40I3 = 0 … (ii) so 2 - x + -4 - x = x + 8
2 8 4
in Mesh FCDEF
40I3 - 40 - 80 + 40I2 = 0 … (iii) -4 - x x x x
2- x + = + 4 - x -1 - = + 2
4 2 4 2
putting I3 from (i) in (ii) and (iii) we get
EBD_7179
504 PHYSICS

5x x 7x 12
- - =3 - = 3 Þ or x = - R/3
4 2 4 7

12 R/3 R/3
- +8
7 44 11
So current in 4K will be = = = 1.57 A
4 28 7
A R/3 B
62. (b) The equivalent circuit can be redrawn as 3R R
´
R2
R R Rnet between AB = 3 3 = = 4W
3R R 4R
R R +
3 3
A· · B º ·
R R 2R
R R E E
65. (b,c) I = =
R + r ( R - r )2 + 2 R r
R R
5R / 8
º º º I is maximum when R = r
2R / 3
R 5R / 3 P = I 2 R , when I is max, P is also max.

.IN
63. (c) At steady state the capacitor will be fully charged and Pmax = I 2max R .
thus there will be no current in the 1W resistance. So 66. (c) By principle of symmetry and superposition,
the effective circuit becomes R
AL 2´
I
´ R 0 = I R eq. Þ R eq. = 0
2W 6 3
I
N
I1
A B (Current in AB is due to division in current entering
6
I2 3W
R

I
at A and current is due to current returning from
U

I 6
infinity of grid).
JO

6V 2.8W 67. (d) Current in arm AE and FB is zero.


So,
Net current from the 6V battery,
U

A
6 6 3
I= = = = 1.5A
ED

æ 2 ´ 3 ö 2.8 1.2 + 2.8 2


ç ÷+ E
è 2+3ø 1

Between A and B, voltage is same in both resistances, C G H D


2I1 = 3I 2 where I1 + I 2 = I = 1.5 F

Þ 2I1 = 3(1.5 - I1 ) Þ I1 = 0.9A B


64. (d) R
68. (b) Current in branch of capacitor is zero.
Rnet = R + r
R
E
I= potential across capacitor is equal to
R+r
R potential across resistance R1.
R R
R R E
R R V1 = R 1 So charge = CV = ECR1
R R1 + r 1 R+r
69. (b) Given Capacitance of the capacitor, C = 0.2 µF and
R
e.m.f. of cell, E = 6V.
B Reactance of a capacitor for a cell, which is a DC source,
A
is infinity. Therefore, no current flows in 4W resistance.
Current Electricity 505

Resistances 2 W and 3 W (both in upper arm) are


1 1 1 2
connected in parallel. Therefore, their equivalent = + = or R = 1.5 W.
R 3 3 3
resistance ( R ¢ ) = 2 ´ 3 = 1.2W \ Current in the circuit is I = V/R = 1.5/1.5 = 1A.
2+3 Since the resistance in arm ACB = resistance in arm AB
Now, R ¢ and 2.8 W are in series combination. = 3 W, the current divides equally in the two arms. Hence
Therefore, equivalent resistance of the circuit, the current through the 3 W resistor = I/2 = 0.5 A.
R = R ¢ + 2.8 = 1.2 + 2.8 = 4 W 72. (c) 73. (d)

E 6 74. (a) 500W


Current drawn in the circuit, I = = = 1.5A A
R 4
i
Therefore, potential difference across 2 W resistance, 2V
V = IR = 1.5 × 1.2 = 1.8 V R
12V
V 1.8
Thus, current in 2 W resistance ( I1 ) = = = 0.9A
2 2
70. (d) The equivalent circuit is as shown in figure.
The resistance of arm AOD (= R + R) is in parallel to 10 1
12 – 2 = (500W) i Þ i = =
the resistance R of arm AD. 500 50

.IN
12 1
Again, i = =
AL 500 + R 50
Þ 500 + R = 600 Þ R = 100 W
75. (a) The temperature of the wire increases to such a value
at which, the heat produced per second equals heat
N
lost per second due to radiation i.e.
R

æ rl ö
I 2 çç 2 ÷÷ = H ´ 2 p r l , where H is heat lost per
U

è pr ø
JO

Their effective resistance R1 = 2R ´ R = 2 R second per unit area due to radiation. Hence, I 2 µ r 3
2R ´ R 3
I12 r13
U

The resistance of arms AB, BC and CD is


so = or r2 = r1 (I 2 / I1 )2 / 3
2 8 I 22 r23
ED

R2 = R + R + R = R
3 3 = 1´ (3.0 / 1.5) 2 / 3 = 41/ 3 mm.
The resistance R1 and R2 are in parallel. The effective
1
resistance between A and D is 76. (d) The energy stored in the capacitor = CV 2 ; This
2
2 8 energy will be converted into heat in the resistor.
R´ R
R1 ´ R 2 3 3 = 8 R. 77. (b) At neutral temperature, dE/dt = 0;
R3 = =
R1 + R 2 2 8 15 so 2164 – 6.2 × 2 × tn = 0
R+ R
3 3 or tn = 174.5ºC.
71. (b) Equivalent resistance between A and B = series At temperature of inversion, E = 0
combination of 1 W and 2 W in parallel with 3 W Þ 2164 ti – 6.2 t i2 = 0 Hence ti = 349ºC.
resistor..
é T 2 TTr TT T 2 ù
78. (a) E = k ê TT0 - - - Tr T0 + r + r ú
ë 2 2 2 2 û

dE é T T ù
Hence = k êT0 - T - r + r ú = k (T0 - T )
dT ë 2 2û
At temp.T = T0 /2,
thermoelectric power = k (T0 – T0 /2) = k T0 /2.
EBD_7179
506 PHYSICS

dE R
79. (b) Thermo-electric power S = = a + bT \ Equivalent resistance R ¢ =
dT 3

V 2 30R
T2 \ Power = = = 90 W
80. (c) Given E = 40T – . At neutral temperature, R¢ R / 3
20
H
dE T 88. (d) P= = s (T14 - T2 4 )
= 0 or 40 - = 0 or T = 400o C. t
dT 10
T1 = 727 + 273 = 1000 K, T2 = 227 + 273 = 500 K,
H P t P 500 5 P = 60 watt
81. (a) In parallel, 1 = 1 = 1 = =
H 2 P2 t P2 200 2 In the second case, T1' = 1227 + 273 = 1500 K,
T2 ' = 500 K, P' = ?
H1 I 2 R 1 t R 1 V 2 /P1
In series, = = =
H 2 I 2 R 2 t R 2 V 2 /P2 P¢ (1500)4 - (500) 4 (500)4 [3 4 - 1] 80
= =
P (1000)4 - (500)4 (500)4 [2 4 - 1] Þ 15
P2 200 2
= = = .
80
P1 500 5 Þ P' = 60 ´ = 320 watt.
15

.IN
V2 t 89. (d) mCu = ZCuICut
82. (c) H = m L = V 2 t / RJ or m =
JRL mAg = ZAgIAgt
m Cu m Ag
=
(210) 2 ´ 1
= 6.56 g / s.
AL I = I Cu + I Ag = +
4.2 ´ 20 ´ 80 Z Cu t Z Ag t

1´1
N
83. (b) Total internal resistance of two cells = = 0.5 W . ém m Ag ù
1+1 Pt = Energy = VIt = V ê Cu + ú
R

Since internal resistance of coil is equal to external ëê Z Cu Z Ag ûú


resistance (= 0.5 W), hence power developed is
U

maximum by cells in circuit. é 1.8 1 ù


= 12 ê -4
+ -4 ú
ë 6.6 ´ 10 11.2 ´ 10 û
JO

2
Current through R = = 2A
0.5 + 0.5 =12 × 104 [0.362] = 4.34 × 104J.
U

Power = (2) 2 ´ 0.5 = 2 W. 90. (a) P 1


R = 2 = = 0.04W
I 25
ED

Vg 4 ´10 -4
84. (d) Vg = R g I g = 40´10-5 V ; q = = = 16º C. 91. (c)
V 25 ´10 - 6
92. (b) Resistance of 45 bulbs in series is less than that of 50
85. (d) At the heighest point O bulbs. Since illumination is proportional to the heat
æ V2 t ö
produced çç R ÷÷ , therefore it will be more with 45
E­ è ø
bulbs.
B A 93. (a) m = ZIt = 0.0000104 × 1 × 600 g
temperature
E= maximum and A shows temperature of inversion 94. (d) Rs = 3R and R P =
R
at which emf changes in sign. 3
86. (c) Neutral temperature,
q + q 0 530 + 10 V2
qn = i = = 270º C. PS = = 10W and
2 2 3R
87. (d) In series, Equivalent resistance = 3R PP = V2 /(R / 3) = 3V2/R = 9PS = 90 W.
95. (a) PP = P1 + P2 and PS = P1 P2 / (P1 + P2)
V2 V2
Power = Þ 10 = Þ V2 = 30R Heat produced = Pt.
3R 3R
Q P Pp 2000 W
In parallel,
1 1 1 1
= + + =
3 Hence, = = = 4.
R¢ R R R R QS PS 500 W
Current Electricity 507

96. (b) Let I1 be the current throug 5 W resistance, I2 through 103. (a)
(6 + 9) W resistance. Then as per question, 104. (d) A balanced Wheatstone’s bridge exists between
A & B. Req = R
I12 ´ 5 = 20 or, I1 = 2A.
current through circuit is V/R
Potential difference across C and D = 2 × 5 = 10V current through AFCEB = V/2R
10 2 105. (a) Efficiency is given by
Current I 2 = = A.
6+9 3 output Power 5 ´ 15 ´ 14
h= = = 8.75 or 87.5 %
Heat produced per second in 2 W input Power 10 ´ 8 ´ 15

2
æ8ö 1 1 1 1 1 3
= I2 R ç ÷ ´ 2 = 14.2cal / s. 106. (a) = + + or = Þ Peq = 20 watt.
è 3ø Peq P1 P2 P3 Peq 60
97. (d) Power of heating coil = 100 W and voltage
(V) = 220 volts. When the heating coil is cut into two V2 V 2 (220) 2 4(110) 2
107. (d) P= ÞR = = = ;
equal parts and these parts are joined in parallel, then R P 60 60
the resistance of the coil is reduced to one-fourth of (110) 2 R
the previous value. Therefore energy liberated per R¢ = =
60 4
second becomes 4 times i.e., 4 × 100 = 400 J.
108. (d) It is balanced Wheatstone bridge. Hence bridge 4W

.IN
98. (a) Hint : The wheatstone bridge is balanced, when can be eliminated.
3 4 V 5V
P/Q = R/S, In the this case= , so bridge is balanced 6 ´ 9 18
\ I = R = 18
6 8
AL \ R eq =
6+9 5
=
eq
& 7W resistance is not effective)
P 100 ´103 105
N
V - VB i ´ r 0.1 ´ 10-7 109. (c) I = = A= A
99. (a) Potential gradient = A = = V 125 60
l A 10-6
R

E.C.E. = 0.367 × 10–6 kg C –1


-2 Charge per minute
= 10
U

V/m
100. (a) As for an electric appliance R = (Vs2 /P) , so for same 105 ´ 60 6 ´106
JO

= (I × 60) C = C= C
specified voltage Vs , 125 125

R 25 100 6 ´ 106
U

= = 4 i.e, R = 4R with R
25 100 = R \ Mass liberated = ´ 0.367 ´10-6
R100 25 125
ED

Now, in series potential divides in proportion to


resistance. 6 ´ 1000 ´ 0.367 ´10-3
=
125
R1 4
So, V1 = V i.e., V25 = 5 ´ 440 = 352V = 17.616 × 10–3 kg
(R1 + R 2 ) 110. (d) Current from D to C = 1A
\ VD – VC = 2 × 1 = 2V
R2 1
and, V2 = V i.e., V100 = ´ 440 = 88V VA = 0 \ VC = 1V, \ VD – VC = 2
(R 1 + R 2 ) 5
Þ VD – 1 = 2 \ VD = 3V
From this it is clear that voltage across 100 W bulb \ VD – VB = 2 \ 3 – VB = 2 \ VB = 1V
(= 88 V) is lesser than specified (220 V) while across 25 111. (c) The power dissipated in the circuit.
W bulb (=352 V) is greater than specified (220 V), so
25 W bulb will fuse. V2
P= ...(i)
10 ´ 40 400 Req
101. (b) Time = = = 8 min
10 + 40 50 v = 10 volt
1 1 1 1 5+ R
102. (a) Power µ = + =
Resistance Req R 5 5R
In series combination, Resistance doubles
Hence, Power will be halved. æ 5R ö
Req = çè ÷
In parallel combination, resistance halved 5 + Rø
Hence, power will be double. P = 30 W
EBD_7179
508 PHYSICS

Substituting the values in equation (i) Since deflection in galvanometer is zero so current
will flow as shown in the above diagram.
(10) 2
30 = VA 12 12
æ 5R ö current I = = =
çè ÷ R1 + R 500 + 100 600
5 + Rø
15 R 12
= 10 Þ 15R = 50 + 10R So VB = IR = ´ 100 = 2V
5+ R 600
5R = 50 Þ R = 10 W 115. (c) Resistance of bulb is constant
112. (c) The current through the resistance R
V2 Dp 2DV DR
æ e ö P= Þ = +
I =ç R p V R
è R + r ÷ø
The potential difference across R Dp
= 2 × 2.5 + 0 = 5%
p
æ e ö
V = IR = ç R
è R + r ÷ø
rl
116. (c) Resistance R =
I e r A
Q l¢ = 2 l

.IN
A
\ A¢ =
AL 2

R 2l
\ R¢ r A = 4R = 4 × 4 W = 16 W
N
2
e e
R

V= V Therefore the resistance of new wire becomes 16 W


æ rö 117. (a) Given : V = 7 V
çè1 + ÷ø
U

P Q
R r = 5W
0
JO

R
when R = 0, V = 0, 40 ´ 120
R = ¥, v = e Req = W 5W
40 + 120
U

Thus V increases as R increases upto certain limit, but


it does not increase further. 7V
ED

V 7
113. (a) Galvanometer is converted into ammeter, by connected I= =
R 40 ´ 120
a shunt, in parallel with it. 5+
40 + 120
G
I 7 1
= = = 0.2 A.
5 + 30 5
118. (c) In the case of a short-circuited battery, the current

S E (emf of the battery)


I= ¹0
r (internal resistance)
GS VG 25 ´ 10-3 Terminal voltage V = IR = I (i) = I (0) = 0
==
G+ S I 25 where R = external resistance = 0
GS 119. (c) Before the presence of electric field, the free electrons
= 0.001W move randomly in the conductor, so their drift velocity
G+S
Here S << G so S = 0.001 W is zero and therefore there is no current in the
conductor. In the presence of electric field, each
114. (b) R1 electron in the conductor experience a force in a
G
direction opposite to the electric field. Now the free
R VB electrons are accelerated from negative and to the
VA
positive end of the conductor and hence a current
starts to flow from the conductor.
Current Electricity 509

120. (d) Potentiometer is preferred for measurement of emf of


é 2.9 ù
cell. Potentiometer draws no current from cell while R = Sê
ë 97.1 úû
voltmeter draws some current, therefore emf measured
by voltmeter is slightly less than actual value of emf So, here, R : S = 2.9 : 97.1 implies that the S is nearly 33
of cell. Further the potentiometer is used with a times to that of R. In orded to make this ratio 1 : 1 it is
galvanometer which is set to null reading when the 1
necessary to reduce the value of S nearly times
experiment is performed. The method of null reading 33
avoids many errors. i.e., nearly 3 W,
121. (a) Glow = Power (P) = I2R 4. (b) The potential drop across wires of potentiometer
should be more than emfs of primary cells. Here,
dP æ dI ö values of emfs of two cells are given as 5V and 10V, so
\ = 2ç ÷ = 2 ´ 0.5 = 1%
P è I ø the potential drop along the potentiometer wire must
be more than 10V. So battery should be of 15V and
2
æ Pö about 4V potential is droped by using variable
122. (a) Power loss = i²R = ç ÷ R resistance.
è Vø
[P = Transmitted power] l
5. (a) As we known that the resistance of wire is R = r
A
V2 1 For maximum value of R, l must be higher and A should
123. (a) P= ; Rµ (same rated voltage)

.IN
R P be lower and it is possible only when the battery is
124. (c) In tube light majority portion of radiation comes under æ1ö
connected across area of cross section = 1cm ´ ç ÷cm.
visible region while bulb radiation consists of visible, è2ø
ultraviolet, infrared radiation giving less visible part.
AL
6. (a) We know that the relationship between current and
125. (c) Positive terminal of a battery is point of highest drift speed is
potential and current flows from highest to lowest I = ne Avd
N
potential i.e. from +ve to –ve potential. Where, I is the current and Vd is the drift velocity.
R

So, I µ Vd
EXERCISE - 3 Hence, only drift velocity determines the current in a
U

Exemplar Questions conductor.


JO

NEET/AIPMT (2013-2017) Questions


1. (b) As we know, electric current per unit area
I rl
I/A, is called current density j i.e., j = 7. (c) Resistance R =
A
U

A
The SI units of the current density are A/m2. Q l¢ = 2 l
ED

The current density is also directed along E and is A


also a vector and the relationship is \ A¢ =
2
j = sE
Current density changes due to electric field produced 2l
\R= r = 4R = 4 × 4 W = 16 W
by charges accumulated on the surface of wire. A
2. (a) As we know the equivalent emf (eeq) in the parallel 2
combination Therefore the resistance of new wire becomes 16 W
e r +e r 8. (a) Given : emf e = 2.1 V
eeq = 2 1 1 2 I = 0.2 A, R = 10W
r1 + r2
Internal resistance r = ?
So according to formula the equivalent emf eeq of the
From formula.
two cells in parallel combination is between e1 and e2.
e – Ir = V = IR
Thus (e1 < eeq < e2). 2.1 – 0.2r = 0.2 × 10
3. (c) Adjusting the blance point near the middle of the 2.1 – 0.2 r = 2 or 0.2 r = 0.1
bridge, i.e.. when l1 is close to 50 cm. requires a suitable
choice of S, R is unknown resistance : 0.1
Þ r= = 0.5 W
0.2
R Rl1
Since, = e 2.1
S R (100 - l1 ) ALTERNATE : i = Þ 0.2 =
r+R r + 10
R l1 é l ù
= or R = S ê 1 ú 1
S 100 - l 100 - l Þ 2.1 = 0.2 r + 2 Þ r = = 0.5 W
1 ë 1û 2
EBD_7179
510 PHYSICS

9. (a) Given : V = 7 V 14. (c) As 0.2% of main current passes through the
r = 5W
998
P Q galvanometer hence I current through the
1000
shunt.
998I
1000 S
5W

7V

40 ´ 120 G
Req = W I 2I
40 + 120 1000
V 7
I= = æ 2I ö æ 998I ö G
R 5+
40 ´ 120 ç ÷G = ç ÷S Þ S =
è 1000 ø è 1000 ø 499
40 + 120
Total resistance of Ammeter
7 1
= = = 0.2 A. æ G ö
5 + 30 5 ç ÷G
SG è 499 ø G

.IN
10. (c) Resistance is directly proportional to length R= = =
S+G æ G ö 500
1 1
= +
1 (4 + 5) + 3 ç ÷+G
= è 499 ø
R AB 3 4 + 5 (3)(4 + 5)
AL
15. (b) Total resistance R = (0.5 W/km) × (150 km)
3 ´ (4 + 5) 27 = 75 W
RAB = =
3 + (4 + 5) 12 Total voltage drop = (8 V/km) × (150 km)
N
Similarly, = 1200 V
R

4 ´ (3 + 5) 32 (DV)2 (1200) 2
RBC = = Power loss = = W
U

4 + (3 + 5) 12 R 75
= 19200 W = 19.2 kW
JO

5 ´ (3 + 4) 35 16. (b) This is a balanced wheatstone bridge condition,


RAC = =
5 + (3 + 4) 12
5 l1 5 1.6l1
\ RAB : RBC : RAC = 27 : 32 : 35 = and =
R 100 - l1 R / 2 100 - 1.6l1
U

11. (a) Copper rod and iron rod are joined in series.
Þ R = 15 W
ED

l æ lö 17. (a) Here, metallic conductor can be considered as the


\ R = RCu + RFe = (r1 + r2) çèQ R = r ÷ø
A A combination of various conductors connected in
From ohm’s law V = RI series. And in series combination current remains
= (1.7 × 10–6 × 10–2 + 10–5 × 10–2) ¸ 0.01 × 10–4 volt same.
= 0.117 volt (Q I = 1A)
12. (b) Resistance is directly proportionl to length of the wire.
As length is doubled so mass is doubled and i
resistance is doubled.
We have
V
(10E )2 (nE )2 t 18. (d) Total potential difference across potentiometer wire
= t mS DT , Now= (2m)S DT
R 2R = 10–3 × 400 volt = 0.4 volt
1mv
n2 E 2t 102 E 2t potential gradient =
Þ =2 cm
2R R v
Þ n = 20 = 10–3 v/cm = 10–1 m
13. (c) Internal resistance of the cell,
Let resistance of RW connected in series.
æ E-Vö æ l1 - l 2 ö
r= ç ÷R = ç ÷R 2V
è V ø è l2 ø
i
æ 3 - 2.85 ö +0.4V
=ç ÷ ´ (9.5) W = 0.5 W
è 2.85 ø RW 8W
Current Electricity 511

2 10 –1 ´ 4 1 V
So, = = 22. (a) EMF, E = Kl where K = potential gradient
R +8 8 20 L
Þ R + 8 = 40 or, R = 32 W
V iR æ E 0 r ö l
19. (d) Effective resistance of B and K= = =ç
L L è r + r1 ÷ø L
R B × R C 1.5R ´ 3R 4.5R 2
= = = =R
R B + R C 1.5R + 3R 4.5R E 0 rl
So, E = Kl =
i.e., equal to resistance of voltmeter A. (r + r1 )L
1.5R 23. (a) Given: Charge Q = at – bt2

A B ¶Q a
\ Current i= = a – 2bt {for i = 0 Þ t= }
¶t 2b
R C From joule's law of heating, heat produced
3R dH = i2Rdt
In parallel potential difference is same so, a /2b
VB = VC and in series current is same H= ò ( a - 2bt )2 Rdt

.IN
So, VA = VB = VC 0
20. (d) From circuit diagram
a
20W
AL ( )3 2b a 3R
H = a - 2bt R =
-3 ´ 2b 0 6b
N
A 24. (d) When two cells are connected in series i.e., E 1 + E2
480W the balance point is at 50 cm. And when two cells are
R

40.8W connected in opposite direction i.e., E1 – E2 the balance


U

point is at 10 cm. According to principle of potential


JO

E1 + E 2 50
=
E = 30V E1 - E 2 10
U

480 ´ 20 2E1 50 + 10 E1 3
Resistance of ammeter = = 19.2W. = =
480 + 20 Þ Þ
ED

2E 2 50 - 10 E2 2
Total resistance R = 40.8 + 19.2 = 60W
25. (b) Reading of potentiometer is accurate because during
V
Reading in the ammeter i = taking reading it does not draw any current from the
R
circuit.
30
= = 0.5A rl
40.8 + 19.2 26. (b) We know that, R =
21. (d) In figure, two metal wires of identical dimension are A
connected in series
rl 2
or R = Þ R µ l2
A s2 Volume
s1
According to question l2 = nl1
l l 2 2
R2 n l1
l l leq = 2
Req = + = R1 l1
s1A s2 A seq Aeq
R2
= n2
2l l æ s1 + s2 ö or,
R1
= ç ÷
seq A A è s1s 2 ø
Þ R2 = n2R1
2s1s2
\ seq = s + s
1 2
EBD_7179
512 PHYSICS

Moving Charges
19 and Magnetism
OERSTED EXPERIMENT Comparison between electric field and magnetic field
In 1802 Gian Domenico Romagnosi, an Italian lawyer and judge, Electric field Magnetic field
found that a steady electric current flowing in wire affected a 1. Source is an electric 1. Source is a current element
uur
magnetic needle placed near it. He published his observation in a charge (q). ( Idl ) .

.IN
local newspaper (called Gazetta di Trentino). But nobody noticed 2. Isolated charge exists 2. Isolated poles do not exist.
this phenomenon. 3. Electric field at a point 3. Magnetic field at a point due to
due to a point charge is a current element is perpendicular
In 1820 Hans Christian Oersted (a Danish Physicist) rediscovered
AL
in the plane containing to the plane containing the point
this phenomenon. He noted that a magnetic compass needle, the point and the charge. and the current element.
brought close to a straight wire carrying a steady electric current, 4. It obeys inverse square 4. It also obeys inverse square law
N
law (a long range force). (a long range force).
aligned
r itself perpendicular to the wire i.e., the direction of magnetic 5. It obeys principle of 5. It also obeys principle of
field B is tangential to a circle which has the wire as centre, and
R

superposition as the field superposition.


which has its plane perpendicular to the wire (Fig 1-a). Oersted is linear related to charge.
U

also noticed that on reversing the direction of current; the direction 6. Angle dependence is not 6. Angle dependence is present.
present.
of magnetic field is reversed.
JO

7. Line of electric lines of force 7. Lines of magnetic lines of


do not form closed loops. force form closed loops.
8. Electric field changes 8. Magnetic field does not
U

kinetic energy of a charged change kinetic energy of a


i particle. charged particle.
ED

9. A charged particle whether 9. A charged particle at rest


B at rest or in motion in an do not experience force
electric field experiences due to magnetic field.
B a force due to electric field.
i

Clockwise Anticlockwise MAGNETIC FIELD DUE TO CURRENT CARRYING


Fig. 1(a) Fig. 1(b) CONDUCTOR, BIOT-SAVART’S LAW
uuur
The magnetic induction dB at any point outside
the current path due to a small current element of A
In first case when current is in upward direction, magnetic field is uur
length dl (in the direction of the current) is given
clockwise (Fig 1-a) and when the current is downward, magnetic i P
by Biot-Savart's law q
field is anticlockwise (Fig. 1-b).
uur ur dl r
MAGNETIC FIELD uuur m I (d l ´ r )
dB = 0
It is a region of space around a magnet or current carrying 4p r3
B
conductor or a moving charge in which its magnetic effect can
m I dl sin q
be felt. or, | dB | = 0
4p r2
The conductor carrying current is electrically neutral but a
ur ur
magnetic field is associated with it. uur m q(v ´ r )
Also, dB = 0 where v is the drift velocity of charge
The SI unit of magnetic field induction is tesla (T) or weber/m2 4p r 3
and cgs unit is gauss. 1 gauss = 10– 4 T
where m0 = 4p × 10–7 TmA–1.
Moving Charges and Magnetism 513
uuur Magnetic field due to an infinitely long conductor :
Direction of dB
uur uur r
The direction of dB is perpendicular to both dl and r , governed
uur r
by the right hand thumb rule of the cross-product of dl and r .
ur m I ˆ m 2I a
The magnetic fields going into the page and coming out of the B = 0 (-k) ÞB= 0 I
page are represented as follows : (at P) 2pa 4p a P

Elucidation
Magnetic field in the case of infinitely long wire
Magnetic field Magnetic field
going into the page coming out of the page. p/2
m0I
Magnetic Field due to Various Current Carrying
dB = ò 4pa
cos qdq I
-p / 2
Conductors
Magnetic field due to finite sized conductor : r m I( - kˆ) a
ÞB= 0
2 pa
^ Magnetic field near the end of a long conductor :
j
I

.IN
q1 ^
a i
O q2 P m0I
B = I
4pa
AL (at P)
^
k a
P
N
r m I Elucidation
B(at P) = 0 (sin q1 + sin q 2 ) ( - k)
ˆ
r = a cosec q, l = a cot q, A
R

2 pa 2
dl = a cosec q dq
ur q
m 0 2I
U

ÞB= (sin q1 + sin q 2 ) r m 0 dl ´ r r


4p a dB = I
JO

3
4pr
Elucidation l 90°
m Ia cosec 2 qdqˆj ˆj)
ur m I r r = 0 3 ´ (aj – a cot q P
( )
dB = 0 3 d l ´ r r = a sec q, l = a tanq 4pa cosec2 q O a
U

4pr m I cos ec q uur


r = 0 dq( -k)ˆ Þ dB = m 0 I sin qdq ( -k) ˆ
ED

Þ dl = a sec2 q dq Þ d l = a sec2 qdqˆj 4pa cos ec q 3


4pa
ur uur m 0 I
r = -a tan qˆj + aiˆ B = ˆ = æ m 0 I - 0ö ( - k)
[ - cos q]pp /2 (- k) ˆ
çè ÷ø
4 pa 4 pa
ur m Ia sec 2 qdq m0I
Þ dB = 0 3 3 ˆj ´ ( -a tan qˆj + ai)
ˆ ÞB = (- k̂ )
4pa sec q 4pa
Magnetic field due to a current carrying coil :
m 0 I sec 2 qdq m 0 I cos qdq (i) Magnetic field at a point on the axis of symmetry of a
= (-k̂) = (-k̂)
4pa sec q 3 4pa circular coil, at a distance “x” from its centre :
q2 q2 B = m0NI a2/2(a2 + x2)3/2
m 0I m I
\B = ò 4 pa
cos qdq = 0
4pa ò cos qdq or, B =
mo 2pNIa2 a
- q1 ( - q1 ) 4p (a2 + x2 )3/ 2 I
x
m0I N = total number of turns
ÞB= [sin q 2 + sin q1 ] (Pointing into the plane of paper)
4 pa a = coil radius
Magnetic field near the end of a finite sized conductor : r
The direction of B is given by Right hand screw rule.
Right hand screw rule : If direction of rotation of right handed
ur screw-head is the directon of current in a circular conductor then
mI
B = 0 sin q I the direction of its advance is the direction of magnetic field. This
(at P) 4pa
is applicable even if the current, magnetic field are interchanged, as
a q
in case of current flowing through a straight conductor.
P
EBD_7179
514 PHYSICS

Elucidation Magnetic field inside a current carrying solenoid


y
(i) Finite size solenoid
dl q2
m0 nI q1
r B = (cos q1 - cos q2 )
a ( at P ) 2 P
q q x
x
(ii) Near the end of a finite solenoid
m0 nI
B= cos q ; (q1 = q & q 2 = p / 2)
2
z
(iii) In the middle of a very long solenoid, B = m0 n I
Let for a particular angle, position of small length element dl is
given by its coordinates as (iv) Near the end of a very long solenoid
z = -a cos q, y = a sin q .
m0 n I q
r B=
Now, a = -a cos qkˆ + a sin qˆj , 2
P

r r
r = - a + xiˆ = xiˆ - a sin qˆj + a cos qkˆ n is the number of turns per unit length of solenoid.
Also we have l = rq Þ dl = rdq . (v) Magnetic field in the endless solenoid (toroid) is same

.IN
throughout and is m0nI.
Now d l ^ a at any instant
(vi) Magnetic field outside a solenoid or toroid is zero.
uur m0I ur r
( )
\ d l = dl(sin qk̂ + cos qˆj) , dB = dl ´ r
AL
AMPERE’S CIRCUITAL LAW
4pr3
uur m Irdq The line integral of magnetic field across a closed loop is equal to
Þ dB = 0 3 (sin qkˆ + cos qˆj) ´ (xiˆ - a sin qˆj + a cos qk) ˆ
N
40 times the net correct inside the loop
4pr uur uur
Ñò B .dl = m0 I
R

m 0 Irdq i.e.,
= 2 2 3/2
(x sin qˆj + a sin 2 qˆi + x cos qkˆ + a cos 2 qˆi)
4p(x + a ) where I is the net current inside the loop.
U

m 0 Irdq (1) The direction of the magnetic field at a point on one side of
= [x(sin qˆj + cos qk)ˆ + ai]ˆ
JO

4p(x 2 + a 2 )3/2 a conductor of any shape is equal in magnitude but opposite


uur uur in direction of the field at an equidistant point on the other
Þ B = ò dB
side of the conductor.
U

m 0 Ia 2p (2) If the magnetic field at a point due to a conductor of any


= [x | - cos qˆj + sin qkˆ |0 +a(2p - 0)i]
ˆ
ED

2 2 3/2
4p(x + a ) shape is Bo if it is placed in vacuum then the magnetic field
uur ˆ
m I2pa(i) at the same point in a medium of relative permeability mr is
0
B = given by B = mr Bo .
4p(x 2 + a 2 )3/2
If number of turns of coil are N, then (3) If the distance between the point and an infinitely long
conductor is decreased (or increased) by K-times then the
r m0 I2pNa (i) ˆ ˆ
m 0 INa(i)
|B|= = magnetic field at the point increases (or decreases) by K-
2 2 3/2 2 2 3/2
4p(x + a ) 2(x + a ) times.
m 2pNIa (4) The magnetic field at the centre of a circular coil of radius
Þ B= 0 2
4p (x + a 2 )3/2 smaller than other similar coil with greater radius is more
than that of the latter.
m0 2pNI
(ii) At the centre of a circular coil, B = m0NI / 2a = (5) For two circular coils of radii R1 and R2 having same current
4p a
and same number of turns, we have
(iii) Magnetic field at the centre of a circular arc carrying
current B1 R2
= , where B1 and B2 are the magnetic fields at their
B2 R1
ur m I q m Iq q centres.
B = 0 ´ (- kˆ) Þ B = 0
( at P ) 2a 360 4p a P (6) The magnetic field at a point outside a thick straight wire
a
carrying current is inversely proportional to the distance
where q is in radian. r but magnetic field at a point inside the wire is directly
In this case the direction of magnetic field B is into the proportional to the distance.
page.
Moving Charges and Magnetism 515

Keep in Memory m 0 2 Ix
B inside = for x < R
4p R 2
1. If in a coil the current is clockwise, it acts as a South-pole. If R
m 0 2I
the current is anticlockwise, it acts as North-pole. B surface = for x = R
4p R
x P
m 0 2I
B outside = for x > R
4p x
ur
4. Graph of magnetic field B versus x
2. No magnetic field occurs at point P, Q and R due to a thin
current element Idl . B

Idl
R P Q
x
3. Magnetic field intensity in a thick current carrying conductor
at any point x is 5. Magnetic field is zero at all points inside a current carrying
hollow conductor.

.IN
Magnitude and direction of magnetic field due to different configuration of current carrying conductor.

Configuration of Point of observation Magnetic field


S.No. current carrying conductor Magnitude Direction
AL
1. Two long linear and parallel At P, the mid point between B=0 Normal to the plane of
current carrying conductors the two wires. paper, inwards.
N
Wire 1 Wire 2 The distance of P from each B = m 0 2Ié 1 + 1 ù
wire is r/2. 4p êë x r + x úû
R

At P', distant x from wire 2 as


I I P' shown. m0 é 1 1ù Normal to the plane of
U

P P'' At P'', distant x' from wire 1 B = 2Iê - ú paper, outwards.


x‘ x
as shown. 4p ë r - x' x ' û
JO

r
2. Square loop At the centre. é m0 I ù Normal to the plane of
I
U

ê
B = 4 4p a / 2 ú paper, inwards.
ê ú
ëê (sin 45° + sin 45° )ûú
ED

O I
I
45°45°
a/2
aI
3. Two concentric circular coils At the centre. m0 én n ù n1 n 2
B= 2p I ê 1 - 2 ú If >
having turns n1 and n2 4p ëa bû a b
n2 Perpendicular to the
plane of paper
b inwards.
Oa
n1

4. Straight wire and loop At the centre.


B=0

I O I
EBD_7179
516 PHYSICS

5. Straight wire & semi-circular loop At the centre. m 0 pI Normal to the plane of paper,
B= inwards.
4p a
a
I O I
6. Circular loop At the centre of loop. m 0 é 2pI 2I ù Normal to the plane of
B= - ú
I 4p êë a aû paper, inwards.

O
a
I I
7. Two concentric circular arcs At the common centre. m0 é1 1ù Normal to the plane of paper,
B= Iq -
4p êë a b úû outwards.

.IN
a
q
O
AL
8. Semi-circular area and straight At the centre of the semi- m 0 pI m 0 I Normal to the plane of paper,
B= +
conductors circle. 4p r 4p r outwards.
N

I
R
U

r
I O
JO

I
9. Two concentric coils mutually At the common centre. According to law of vectors
B = B12 + B 22
U

normal to each other. addition.


B1
ED

where
I2 m 2p n1I
B B1 = 0
4p a

a B2 m 0 2p n 2 I
O B2 =
4p b
n1 I1
b

FORCE ON A CONDUCTOR t = BINA


The force on a conductor is given by where N is the number of turns in the coil, A is the area and I is the
F = BIl sin a current.
where l is the length of the conductor in meter; B is the flux
density of field in tesla (Wb/m2); I is the current in ampere and If the plane of the coil makes an angle a with the direction of the
a is the angle which the conductor makes with the direction of the field, then
field. t = BINA cos a.
Special case : Example 1.
If a = 90°, then F = BIl
The direction of the force is given by Fleming's left hand rule. The field normal to the plane of a wire of n turns and radius
r which carries a current i is measured on the axis of the
TORQUE ON A COIL
coil at a small distance h from the centre of the coil. By
The torque acting on a rectangular coil placed with its plane parallel
what fraction this is smaller than the field at the centre?
to a uniform magnetic field of flux density B is given by
Moving Charges and Magnetism 517

Solution : Solution :
The magnetic field on the axis of a current i carrying coil of Magnetic induction at O due to coil Y is given by
turns n, radius r and at a distance h from the centre of the
m0 2p I ( 2 r ) 2
coil BY = ´ ...(1)
4p [(2r ) 2 + (d) 2 ]3 / 2
m 2pnir 2
B= 0 ´ .....(1) Similarly, the magnetic induction at O due to coil X is given
4p ( r 2 + h 2 ) 3 / 2 by
The field at the centre is given by
m0 2p I ( r ) 2
m 2pi ´ n BX = ´ ...(2)
B centre = 0 ´ ....(2) (Q at centre h = 0) 4p [(r) 2 + (d / 2) 2 ]3 / 2
4p r
BY 1
B r3 From eqs. (1) and (2), =
= 2 BX 2
B centre (r + h 2 ) 3/ 2
Example 4.
r3 1 A cell is connected between two points of a uniformly thick
= 3/2 = circular conductor. i1 and i2 are the currents flowing in
é h2 ù æ 3 h2 ö
r 3 ê1 + 2 ú ç1 + ÷ two parts of the circular conductor of radius a. What will
r û ç 2 r2 ÷
ë è ø be the magnetic field at the centre of the loop?
æ 3 Solution :
h 2 ö÷
B ç1 + = Bcentre Let l1, l2 be the lengths of the two parts PRQ and PSQ of the

.IN
or
ç 2 r 2 ÷ø
è conductor and r be the resistance per unit length of the
conductor. The resistance of the portion PRQ will be R1 = l1r
3 h2
\ (B centre - B) / B =
AL
2 r2 i2
Example 2.
S O Q
In fig., there are two semi-circles of radii r1 and r2 in which
N
a current i is flowing. Find the magnetic induction at i1
R

P
centre O. R
U

r1 i
JO

The resistance of the portion PSQ will be R2 = l2r


O Pot. diff. across P and Q = i1R1 = i2R2
r2
Solution : or i1l1r = i2l2r or i1l1 = i2l2 …… (i)
U

Magnetic field induction at the centre O due to currents


m0 p i m 0 p i
ED

B = B1 + B2 = ´ + ´ through circular conductors PRQ and PSQ will be


4p r1 4p r2
m0 i1l1 sin 90º m 0 i 2 l 2 sin 90º
B1 – B2 = - =0
m 0 i é 1 1 ù m 0 i é r1 + r2 ù 4p r2 4p r2
= 4 êr + r ú = 4 ê r r ú
ë 1 2û ë 12 û Example 5.
Example 3. A current passing through a circular coil of two turns
Two circular coils X and Y having equal number of turns and produces magnetic field B at its centre. The coil is then
carry equal currents in the same sense and subtend same rewound so as to have four turns and the same current is
solid angle at point O. If the smaller coil X is midway between passed through it.The magnetic field at its centre now is
O and Y, then if we represent the magnetic induction due to B B
(a) 2 B (b) (c) (d) 4 B
bigger coil Y at O as BY and due to smaller coil X at O as BX 2 4
Solution : (d)
BY
then find . m 0 2 pnI n
BX B= i.e. B µ ;
4p r r
d
Given, L = 2pr1 × 2 = 2pr2 ×4 ; r1/r2 = 4/2 = 2
d/2
2r B2 n 2 r1 4
r = ´ = ´2 = 4 or B2 = 4B1 = 4B.
B1 n1 r2 2
O
Y X
Example 6.
Compute the flux density in air at a point of 9 cm from the
long straight wire carrying a current of 6A.
EBD_7179
518 PHYSICS

Solution : At P, the earth's horizontal magnetic flux density,


Given : a = 9 cm = 9 × 10–2 m, I = 6A Be = 4 × 10–5T (from South to North)
The direction of B is from north to south.
m o I 4p ´ 10 -7 ´ 6
B= = = 1.33 × 10–5 T \ Resultant intensity at
2pa 2p ´ 9 ´ 10 -2 P = 4 × 10–5 – 1 × 10–5 T
Example 7. = 3 × 10–5 T (From south to north)
Calculate the flux density at a distance of 1 cm from a very For a point 10 cm , north of X the flux density due to the
long straight wire carrying a current of 5A. At what distance current in X = 1 ×10–5T (due east)
from the wire will the field flux density neutralize that due
(b) The flux density due to the horizontal component of
to the earth's horizontal component flux density 2 × 10–5
the earth's field = 4× 10–3T (due north)
T ? (m0 = 4p × 10–7 Hm–1)
Solution :
m o I 4p ´ 10-7 ´ 5
B= = = 10-4 T Be BR
2pa 2p ´ 1 ´ 10 -2
For the second part,
m o I 4p ´ 10-7 ´ 5
a= = = 5 ´ 10 -2 = 5 cm B
2pB 2p ´ 2 ´ 10 -5 \ Resultant Intensity

.IN
Example 8.
A wire 28m long is bent into N turns of circular coil of BR = B2e + B 2 . = (4 ´ 10-5 )2 + (1 ´ 10-5 )2
diameter 14 cm forming a solenoid of length 60 cm. AL
Calculate the flux density inside it when a current of 5 amp –5
= 17 ´ 10-5 = 4.1 × 10 T
passed through it. (m0 = 12.57 × 10–7 m–1)
which is greater than the flux density at P.
Solution :
N
Given : d = 14cm = 0.14m l = 60cm = 0.6 m Example 10.
By the question, N × pd = 28 m. A horizontal wire, of lenth 5 cm and carrying a current of
R

N × p × 0.14 = 28 2A placed in the middle of a long solenoid and right angles


to its axis. The solenoid has 1000 turns per metre and carries
U

28 a steady current I. Calculate I if the force on the wire is


\ N= = 63.66 turns.
0.14 ´ p
JO

vertically downwards and equal to 10–4 N.


N 63.66 Solution :
B < λ o nI < λ o I < 12.57 ´ 10,7 ´ ´5 l = 5cm = 5 × 10–2m, I wire = 2A, n = 1000 m–1, F = 10–4 N
l 0.6
U

= 6.67 ×10–4 T If I be the current through the solenoid, then


ED

Example 9. B = m0 nI
A vertical conductor X carries a downward current of 5A.
(a) What is the flux density due to the current alone at a Force = BIwire × l or 10–4 = m 0 NI ´ I wire ´ l
point P 10 cm due east of X?
(b) If the earth's horizontal magnetic flux density has a or 10 -4 = 4 p ´ 10 -7 ´ 1000 ´ I ´ 2 ´ 5 ´ 10 -2 = 4p ´ 10 -5 I
value 4 × 10–5 T, calculate the resultant flux density
at P. 10-4 10
\ I= -5
= = 0.8A
Is the resultant flux density at a point 10cm due north 4p ´ 10 4p
of X greater or less than at P? Explain your answer.
Example 11.
Two long parallel conductors carry currents of 12A and 8A
N respectively in the same direction. If the wires are 10cm
apart, find where the third parallel wire also carrying a
current must be placed so that the force experienced by it
Be W E shall be zero.
B Solution :
S For the force on the third conductor to be zero, the direction of
I the flux density due to the current flowing in the two wires
Solution : must be opposite in the position of the wire.
(a) I = 5A, a = 10cm = 0.1 m \ Third wire must be placed between the wire. Let the third
wire placed at a distance x m from the wire carrying 12A,
m o I 4p ´ 10-7 ´ 5
B= = = 1 ´ 10-5 T then, B1 = B2
2pa 2p ´ 0.1
Moving Charges and Magnetism 519

(b) v sin a : The force acting on charged particle due to this


component is q(vsin a )Bsin 90° = qvBsin a . This acts
12A X 8A
as the centripetal force and moves the particle in a circular
path.
0.lm The combined effect of these two is a helical path.
A charged particle entering a uniform magnetic field at an angle
m o I1 m o I2 executes helical path.
= .
2px 2p(0.1 - x) mv sin a
Radius of the helix, R =
12 8 3 2 qB
or = or =
x 0.1 - x x 0.1 - x Angular frequency of rotation, w = (2p / T ) = qB / m
0.3 2pmv cos a
or 0.3 = 5x = = 0.06 m Pitch of the helix = (v cos a ) T = .
5 qB
FORCE ACTING ON A CHARGED PARTICLE MOVING ur
Direction of force F :We can use the rule of cross product. The
IN A UNIFORM MAGNETIC FIELD
The force acting on a particle having a charge q and moving with direction of F is perpendicular to the plane containing v and B
r r and can be found by right hand thumb rule. It is important to note
velocity v in a uniform magnetic field B is given by
ur ur ur

.IN
that if q is positive, we will get the correct direction of F by right
F = q(v ´ B) Þ F = qvB sin q ,
r r hand thumb rule. But if q is negative, we have to reverse the
where q is the angle between v and B direction of force.
AL
Case (i) If q = 0, F = 0. Also if q = 180°, F = 0 Flemings left hand rule : It states that if the fore finger, the central
If a charged particle enters a uniform magnetic field in the direction finger and the thumb of the left hand are stretched at right angles
of magnetic field or in the opposite direction of magnetic field, the to each other then if the central finger represents the direction of
N
force acting on the charged particle is zero. current and fore finger represents field, the thumb will represent
the direction of motion or force experienced by the current
R

Case (ii) If q = 90°, F = qvB carrying conductor.


In this case the force acting on the v × × × ×
U

FORCE BETWEEN TWO PARALLEL CURRENTS


particle is maximum and this force acts × × × × B
When a current flows in a conductor, the free charges (electrons
as centripetal force which makes the v
JO

× × × × in case of a metal wire) move. Each free charge movement generates


charged particle move in a circular path. F
× × × × a force which adds up to give the force on the conductor.
mv2 × × × × Force between infinitely long conductors placed parallel to each
\ F = qvB =
U

q × × × × other at distance d.
r v
mv × × × ×
ED

Þ r= B1
qB
where r is the radius of the circular path. F
I2
It is important to note that this force cannot change the speed of
I1
the charged particle and hence its kinetic energy. But it changes d
the velocity of charged particle (due to change in direction) and
hence also causes a change in momentum. F m0 2 I1 I 2
Force per unit length = =
Also the work done by the force is zero as the force is acting l 4p d
where l is the length of wire.
perpendicular to the direction of motion. If currents are pointing in same direction, the force is of attractive
Case (iii) If q = a is any other angle then the path taken is helical. nature and if currents are oppositely directed the force is of
The velocity of the charged particle can be split into two parts for repulsive nature.
better understnading. Lorentz Force Equation
For a charged particle q moving in a region of simultaneously
B uur uur
applied electric field E and magnetic field B , the force
v experienced by it is given by
v sin a a uur uur ur uur
vcosa F = q [ E + (v ´ B )]
uur
Torque on a current loop in uniform magnetic field B is given
ur uuur uur uur
(a) v cos a : The force on charged particle due to this component is by t = (M ´ B ) where M is the magnetic moment of coil.
zero. This component is responsible in moving the charged
uuur r
M = N I Anˆ where n is the unit vector normal to the plane of
particle uniformly in the direction of B . the loop.
EBD_7179
520 PHYSICS

Keep in Memory T1
Torsional
head
1. No force acts on a charged particle if it enters a magnetic Phosphor
field in a direction parallel or antiparallel to the field. bronze wire
2. A finite force acts on a charged particle if it enters a uniform M Mirror
magnetic field in a direction with finite angle with the field. Soft iron core
3. If two charged particles of masses m1and m2 and charges P Q
q1 and q2 are projected in a uniform magnetic field with
same constant velocity in a direction perpendicular to the L N S
N S
field then the ratio of their radii (R1: R2) is given by S R
R1 m1 q 2
= ´ Spring Showing radial
R 2 m 2 q1 field
4. The force on a conductor carrying current in a magnetic T2
field is directly proportional to the current, the length of It essentially consists of a rectangular coil PQRS or a cylindrical
conductor and the magnetic field. coil of large number of turns of fine insulated wire wound over a
5. If the distance between the two parallel conductors is non-conducting frame of ivory or bamboo. This coil is suspended
decreased (or increased) by k-times then the force between by means of phosphor bronze wire between the pole pieces of a
them increases (or decreases) k-times.
powerful horse shoe magnet NS. The poles of the magnet are

.IN
6. The momentum of the charged particle moving along the
curved to make the field radial. The lower end of the coil, is attached
direction of magnetic field does not change, since the force
to a spring of phosphor-bronze wire. The spring and the free ends
acting on it due to magnetic field is zero.
of phosphor bronze wire are joined to two terminals T 2 and T1
7. Lorentz force between two charges q1 and q2 moving with
AL
velocity v1, v2 separated by distance r is given by: respectively on the top of the case of the instrument. L is a soft
iron core. A small mirror M is attached on the suspension wire.
m (q v ) (q v )
Fm = 0 . 1 1 2 2 2
N
Using lamp and scale arrangement, the deflection of the coil can
4p r
8. If the charges move, the electric as well as magnetic fields be recorded. The whole arrangement is enclosed in a non-metallic
R

are produced. In case the charges move with speeds case.


U

comparable to the speed of light, magnetic and electric force Theory : Let the coil be suspended freely in the magnetic field.
between them would become comparable. Suppose, n = number of turns in the coil
JO

9. A current carrying coil is in stable equilibrium if the magnetic A = area of the coil
r r B = magnetic field induction of radial magnetic field in which
dipole moment M , is parallel to B and is in unstable the coil is suspended.
r r
U

equilibrium when M is antiparallel to B . Here, the magnetic field is radial, i.e., the plane of the coil always
10. Magnetic moment is independent of the shape of the loop. remains parallel to the direction of magnetic field, and hence the
ED

It depends on the area of the loop. torque acting on the coil


11. A straight conductor and a conductor of any shape in the t = niAB … (1)
same plane and between the same two end points carrying Due to this torque, the coil rotates. As a result, the suspension
equal current in the same direction, when placed in the same wire gets twisted. Now a restoring torque is developed in the
magnetic field experience the same force. suspension wire. The coil will rotate till the deflecting torque acting
12. There is net repulsion between two similar charges moving on the coil due to flow of current through it is balanced by the
parallel to each other inspite of attractive magnetic force restoring torque developed in the suspension wire due to twisting.
between them. This is because of electric force of repulsion Let C be the restoring couple for unit twist in the suspension wire
which is much more stronger than the magnetic force. and q be the angle through which the coil has turned. The couple
13. The speed of the charged particle can only be changed by for this twist q is Cq.
an electric force. In equilibrium, deflecting couple = restoring couple
\ ni AB = Cq or i = Cq/ (nAB)
MOVING COIL GALVANOMETER
or i = Kq (where C/nAB = K) … (2)
The moving coil galvanometer was first devised by Kelvin and K is a constant for galvanometer and is known as
later on modified by D’Arsonaval. This is used for detection and galvanometer constant.
measurement of small electric current. Hence i µ q
The principle of a moving coil galvanometer is based on the fact
Therefore, the deflection produced in the galvanometer is directly
that when a current carrying coil is placed in a magnetic field, it
proportional to the current flowing through it.
experiences a torque. Current sensitivity of the galvanometer : The current sensitivity
Construction: A moving coil ballistic galvanometer is shown in of a galvanometer is defined as the deflection produced in the
figure. galvanometer when a unit current is passed through it.
Moving Charges and Magnetism 521

19.1
We know that, niAB = Cq Explanation : Let us consider a conductor carrying current in +X
q nAB direction. The magnetic field is applied along +Y direction.
\ Current sensitivity, is = = Consider two points P1 and P2 on the conductor and connect a
i C
where C = restoring couple per unit twist voltmeter between these points. If no magnetic field is applied
The SI unit of current sensitivity is radian per ampere or deflection across the conductor, then the points P1 and P2 will be at same
per ampere. potential and there will be no deflection in the galvanometer.
Voltage sensitivity of the galvanometer : The voltage sensitivity However, if a magnetic field is applied as shown in the figure,
of the galvanometer is defined as the deflection produced in the then the Lorentz force acts on electrons as shown in the figure.
galvanometer when a unit voltage is applied across the terminals
® ®
of the galvanometer. The Lorentz force on electrons Fm = –e (vd ´ B) acts in the
q
\ Voltage sensitivity, Vs = downwards direction.
V Now there may be two cases:
If R be the resistance of the galvanometer and a current is passed
Case I : If the charge particles are negatively charged, then
through it, then
these negative charges will accumulate at the point P2 and
V = iR
therefore P2 will be at lower potential than P1.
q nAB
\ Voltage sensitivity, Vs = i R = CR Case II : If the charged particles are positively charged, then the
point P2 will be at higher potential than P1.

.IN
The SI. unit of voltage sensitivity is radian per ampere or deflection
Magnitude of hall voltage :
per ampere.
Conditions for sensitivity : A galvanometer is said to be more Let w be width and A be the cross-sectional area of the conductor.
AL
sensitive if it shows a large deflection even for a small value of If e is magnitude of charge or the current carrier (electron or
current. hole).
The force on the current carrier due to magnetic field B,
N
nAB
We know that, q= i Fm = Bevd
C
R

Here, vd is drift velocity of the current carries.


For a given value of i, q will be large if (i) n is large, (ii) A is large,
Due to the force Fm, the opposite charges build up at the points
U

(iii) B is large, and (iv) C is small.


P1 and P2 of the conductor.
Regarding above factors, n and A cannot be increased beyond a
JO

certain limit. By increasing n, the resistance of the galvanometer If VH is Hall voltage developed across the two faces, then the
will increase and by increasing A, the size of the galvanometer will strength of electric field due to Hall voltage is given by
increase. So, the sensitivity will decrease. Therefore, B is increased.
U

VH
The value of B can be increased by using strong horse shoe EH = .
w
ED

magnet. Further, the value of C can be decreased. The value of C


for quartz and phosphor-bronze is very small. So, the suspension Here w = P1P2.
wire of quartz or phosphor-bronze is used. The value of C is further This electric field exerts an electric force on the current carries in
decreased if the wire is hammered into a flat strip. a direction opposite to that of magnetic force. The magnitude of
HALL EFFECT VH
When a current passes through a slab of material in the presence this force is Fe = E H e = e
w
of a transverse magnetic field, a small potential difference is
In equilibrium condition, Fe = Fm,
established in a direction perpendicular to both, the current
flow and the magnetic field. This effect is called Hall effect VH
The voltage thus developed is called Hall voltage. or e = B e vd, or VH = B vd w
w

Z j
P1 V Now, drift velocity of current carrier is given by, vd =
ne

Fm X where n is the number of current carries per unit volume of the


strip.
B P2
Hall resistance, RH = H = æç
V Bwj ö 1
Y
I è ne ÷ø I
Hall effect enables us to :
(i) Determine the sign of charge carriers inside the conductor.
(ii) Calculate the number of charge carriers per unit volume. \ Hall voltage VH = Bwj
ne
EBD_7179
522 PHYSICS

Keep in Memory B
1. Hall effect can determine nature of current (charge) carriers in Q R
the material. i.e. whether the charge is +ve or –ve.

I2 = 15A

I1 = 15A
I1 = 25A
bBI
2. Hall voltage VH =
neA
where n is the density of charge carriers r1
Z P S
r2
A
PS = 10 cm = 10 × 10–2 m
I Distance of PQ from AB,
b I O Y r1 = 2.0 cm = 2.0 × 10–2 m
VH
Distance of RS from AB,
r2 = 2.0 + 10 = 12.0 cm = 12.0 × 10–2 m
X B
Current through long wire AB, I1 = 25 A
b = thickness of plate, B = magnetic field, I = current flowing Current through rectangular loop, I2 = 15 A
through plate, A = area of cross-section of plates
m 0 2 I1 I 2
Example 12. \ Force on the arm PQ, F1 = ´ length PQ

.IN
4p r1
Two particles X and Y having equal charges, after being
accelerated throught the same potential difference, enter
10 -1 ´ 2 ´ 25 ´15 ´ 25 ´10 -2
a region of uniform magnetic field and describe circular F1 =
paths of radii R1 and R2 respectively. Find the ratio of
AL 2.0 ´ 10 -2
mass of X to that of Y. = 9.375 × 10–4 (towards AB)
Solution : Force on the arm RS,
N

1 m 2I I
mv 2 = qV F2 = 0 1 2 ´ length RS
R

2 4 p r2
U

\v = (2qV / m)1 / 2 10-7 ´ 2 ´ 25 ´ 15 ´ 25 ´ 10-2


=
JO

(where V is the potential difference)


12 ´ 10-2
mv2
Centripetal force < qvB = 1.563×10-4 (away from AB)
R
U

\ Effective force on the loop,


æ qB ö
\v = ç
ED

÷R F = F1 - F2 = 9.375´10-4 - 1.563´10-4 = 7.812 ´104 N


èmø
1/ 2 1/ 2
(towards AB)
æ 2qV ö æ qB ö æ 2mV ö 1 Example 14.
Hence ç ÷ =ç ÷ R or R = çç ÷÷ ´
è m ø è mø è q ø B An electron beam moving with a velocity of 106 ms–1
through a uniform magnetic field of 0.1T, which is
Here V, q and B are constant. Hence R µ m
perpendicular to the direction of the beam. Calculate the
m1 æ R 1 ö
2 force on an electron if the electron charge is 1.6 ×10–19C.
\ =ç ÷
m 2 çè R 2 ÷ø Solution :
v=10–6ms –1, B = 0.IT, q = 1.6 × 10 – 19C
Example 13.
F = Bqv = 0.1 × 106 × 1019 = 1.6 × 10–14 N
A rectangular loop of sides 25 cm and 10 cm carrying a
current of 15 A is placed with its longer side parallel to a Example 15.
long straight conductor 2.0 cm apart carrying a current of A narrow vertical rectangular coil is suspended from the
25 A. What is the net force on the loop? middle of its upper side with its plane parallel to a uniform
Solution : horizontal magnetic field of 0.02 T. The coil has 10 turns
Consider a rectangular loop PQRS placed near a long straight and the lengths of its vertical and horizontal sides are 0.1
conductor AB as shown in Fig. Due to the interaction of m and 0.05 m respectively. Calculate the torque on the coil
currents, the arm PQ of the loop will get attracted while arm when a current of 5A is passed into it.
RS will get repelled. Forces on the arms QR and SP will be
What would be the new value of the torque if the plane of
equal and opposite and hence cancel out.
Here, PQ = 25 cm = 25 × 10–2 m, the vertical coil was initially at 60o to the magnetic field
and a current of 5A was passed into the coil.
Moving Charges and Magnetism 523

Solution : Solution :
B = 0.02 T, N = 10 turns n = 10 × 1029 m–3, A = 2.0 mm2 = 2 × 10–6 m2
I = 5.0A, B = 0.15 T, F = ?.
A = 1 × b= 0.1 × 0.05 = 0.005 m2
I
I = 5A I = nevA or v =
nev
Torque = BINA = 0.02 × 5 × 10 × 0.005
Be ´ I BI 0.15 ´ 5
= 0.005 Nm = 5 × 10–3Nm F = Bev = = =
neA nA 1.0 ´ 10 29 ´ 2 ´ 10 -6
New value of the torque when the plane of the vertical coil –24
= 3.75×10 N
was at 60° to the magnetic field. Example 18.
= BINA cos q = 5 × 10–3 cos 60° If the coil of a moving coil galvanometer having 10 turns
and of resistance 4W is removed and is replaced by a second
1 coil having 100 turns and of resistance 160W. Calculate
= 5× 10–3 × = 2.5 × 10–3 Nm.
2 (a) the factor by which the current sensitivity changes and
Example 16. (b) the factor by which the voltage sensitivity changes.
A rectangular coil of 50 turns hungs vertically in a uniform Solution :
magnetic field of magnitude 10–2 T, so that the plane of the Given : N1 = 10 turns, R = 4W
coil is parallel to the field, the mean height of the coil is N2 = 100 turns, R2 = 160W
5cm and its mean width is 2cm. Calculate the strength of æ q ö N AB

.IN
the current the must pass through the coil in order to (a) Current sensitivity with the Ist coil, ç ÷ = 1
è I ø1 C
deflect it 30° if the torsional constant of the suspension is
10–9 Nm per degree. æ qö N AB
AL Current sensitivity with the 2nd coil ç ÷ = 2
Solution : è Iø2 C
N = 50 turns, B = 10–2T, π = 30° æ qö
N
C = 10–9 Nm per degree, çè ÷ø
I 2 N 2 100
A = 5 × 2 cm2 = 10 × 10–4 m2 = 10–3 m2 \ = = = 10
R

æ qö N1 10
Torque = BINA cos q = Cq çè ÷ø
I 1
U

Cq 10 -9 ´ 30 æ q ö N AB
\ I= = -2 = 6.9 × 10 -5 (b) Voltage sensitivity with the Ist coil ç ÷ = 1
JO

BNA cos q 10 ´ 50 ´ 10 -3 cos 30o è V ø 1 CR1


A = 69mA æ qö N AB
Voltage sensitivity with the 2nd coil ç ÷ = 2
Example 17. è Vø 2
U

CR 2
A copper wire has 1.0 × 1029 free electrons per cubic metre,
æ qö
ED

a cross sectional area of 2.0 mm2 and carries a current of çè ÷ø


v 2 N 2 R1 100 4 1
5.0A. Calculate the force acting on each electron if the \ = = ´ =
wire is now placed in a magnetic field of flux density 0.15 æ qö N1 R 2 10 160 4
çè ÷ø
T which is perpendicular to the wire (e=1.6 × 1019C) v 1
524
CONCEPT MAP

Galvanometer to ammeter Magnetic field due to Direction of magnetic field-Depends Magnetic field due to
conversion : Low resistance a straight current carrying upon the direction of current. Right a solenoid. Inside a long
or shunt connected in parallel conductor of infinite lenght hand thumb rule-Thumb points in the solenoid B = µnI
0
æ Ig ö m I direction of current, curling of fingers At a point on one end
S =ç ÷G B= 0 represents direction of magnetic field . m nI
ç I – Ig ÷ 2pR B= 0
è ø
ED 2

Biot-savart's law Magnetic field Space in


Galvanometer to voltmeter the surrounding of a magnet
magnetic field due Ampere's circuital law
conversion : High resistance U or any current carrying conductor
to current carrying
in series m Idl sin q in which its magnetic influence ò Bdl = m0 I
V element, dB= 0
R = -G 4p r 2 can be experienced
JO
Ig
U Magnetic field due to
MOVING CHARGES a toroid inside the turns
Magnetic field due
B = nI
to a current carrying
R AND MAGNETISM m0
circular loop N
Motion of a charged
particle in a uniform Force acting on a
AL
magnetic field follows charged particle
On the axis of At the centre
a circular path, radius moving in a uniform
circular loop of circular loop
magnetic field
mNI
.IN
m 0 NIa 2 MV sin q
B= B= r= F = qVB sin q= q(V × B)
2(r 2 + a 2 )3/2 2R Bq

Force between two Force on a conductor


Torque experienced by parallel current carrying carrying current in a
a current carrying loop conductors uniform magnetic field,
in a uniform magnetic m 2I I F = I Blsin q Lorentz force
field T = MB sin q n̂ F= 0. 12 F = I (B × l)
4p r F = q (E × V × B)
=M×B
PHYSICS

EBD_7179
Moving Charges and Magnetism 525

1. A current carrying coil is subjected to a uniform magnetic 9. In cyclotron the gyro radius is
field. The coil will orient so that its plane becomes (a) proportional to momentum
(a) inclined at 45° to the magnetic field (b) proportional to energy
(b) inclined at any arbitrary angle to the magnetic field
(c) inversely proportional to momentum
(c) parallel to the magnetic field
(d) inversely proportional to energy
(d) perpendicular to the magnetic field
10. The current sensitivity of a moving coil galvanometer
2. An electron enters a region where magnetic field (B) and
depends on
electric field (E) are mutually perpendicular, then
(a) it will always move in the direction of B (a) the number of turns in the coil
(b) it will always move in the direction of E (b) moment of inertia of the coil
(c) it always possesses circular motion (c) current sent through galvanometer
(d) it can go undeflected also (d) eddy current in Al frame
3. A current carrying conductor placed in a magnetic field 11. Current i is flowing in a coil of area A & number of turns N,

.IN
experiences maximum force when angle between current and then magnetic moment of the coil is M is equal to
magnetic field is
(a) NiA (b) Ni/A (c) Ni / A (d) N2Ai
(a) 3 p/4 (b) p/2 (c) p/4 (d) zero
4. Two concentric circular coils of ten turns each are situated
AL –2
12. 1 Wbm is equal to
in the same plane. Their radii are 20 and 40 cm and they carry (a) 104 G (b) 102 G (c) 10– 2 G (d) 10–4 G
respectively 0.2 and 0.4 ampere current in opposite direction. 13. The radius of motion of a charged particle oscillating in a
N
The magnetic field in weber/m2 at the centre is magnetic field is
R

(a) m0/80 (b) 7m0/80 (c) (5/4) m0 (d) zero mB mv


5. A wire of length L metre carrying a current I ampere is bent (a) (b)
qv qB
U

in the form of a circle. Its magnitude of magnetic moment


will be mq qv
JO

(c ) (d)
(a) IL/4p (b) I2L2/4p (c) IL2/4p (d) IL2/8p vB mB
6. Two straight long conductors AOB and COD are 14. Magnetic effect of current was discovered by
(a) Faraday (b) Oersted
U

perpendicular to each other and carry currents I1 and I2.


The magnitude of the magnetic induction at a point P at a (c) Kirchhoff (d) Joule
ED

distance a from the point O in a direction perpendicular to 15. In cyclotron the charged particle may be accelerated upto
the plane ABCD is energies
m0 m0 (a) Several eV (b) MeV
(a) ( I1 + I 2 ) (b) (I1 - I 2 )
2pa 2pa (c) BeV (d) Kev
1 16. In cyclotron the resonance condition is
m0 2 2 2 m 0 I1 I 2
(c) (I1 + I 2 ) (d) (a) the frequency of revolution of charged particle is
2pa 2 p a I1 + I 2 equal to the frequency of A.C. voltage sources
7. A proton, deutron and an a-particle enter a magnetic field (b) the frequency of revolution of charged particle is
perpendicular to field with same velocity. What is the ratio of equal to the frequency of applied magnetic field
the radii of circular paths? (c) the frequency of revolution of charged particle is
(a) 1 : 2 : 2 (b) 2 : 1 : 1 equal to the frequency of rotation of earth
(c) 1 : 1 : 2 (d) 1 : 2 : 1 (d) the frequency of revolution of charged particle,
8. If an electron and a proton having same momenta enter frequency of A.C. source and frequency of magnetic
perpendicular to a magnetic field, then field are equal
(a) curved path of electron and proton will be same 17. Two parallel wires carrying currents in the same direction
(ignoring the sense of revolution) attract each other because of
(b) they will move undeflected (a) mutual inductance between them
(c) curved path of electron is more curved than that of the (b) potential difference between them
proton (c) electric forces between them
(d) path of proton is more curved
(d) magnetic forces between them
EBD_7179
526 PHYSICS

18. If we double the radius of a coil keeping the current through 22. A long solenoid carrying a current produces a magnetic
it unchanged, then the magnetic field at any point at a large field B along its axis. If the current is double and the number
distance from the centre becomes approximately of turns per cm is halved, the new value of the magnetic
(a) double (b) three times field is
(c) four times (d) one-fourth (a) 4 B (b) B/2
19. To convert a galvanometer into an ammeter, one needs to (c) B (d) 2 B
connect a
23. The total charge induced in a conducting loop when it is
(a) low resistance in parallel
moved in a magnetic field depends on
(b) high resistance in parallel
(c) low resistance in series (a) the rate of change of magnetic flux
(d) high resistance in series. (b) initial magnetic flux only
20. If a current is passed through a spring then the spring will (c) the total change in magnetic flux
(a) expand (b) compress (d) final magnetic flux only
(c) remains same (d) None of these. 24. Energy in a current carrying coil is stored in the form of
21. A charged particle moves through a magnetic field in a (a) electric field (b) magnetic field
direction perpendicular to it. Then the
(c) dielectric strength (d) heat
(a) velocity remains unchanged
25. Tesla is the unit of
(b) speed of the particle remains unchanged

.IN
(c) direction of the particle remains unchanged (a) magnetic flux (b) magnetic field
(d) acceleration remains unchanged (c) magnetic induction
AL (d)magnetic moment
N
R

1. A portion of a conductive wire is bent in the form of a 4. An electron (mass = 9 × 10–31 kg, charge = 1.6 × 10–19 C)
U

semicircle of radius r as shown below in fig. At the centre of moving with a velocity of 106 m/s enters a magnetic field. If it
JO

semicircle, the magnetic induction will be describes a circle of radius 0.1m, then strength of magnetic
field must be
i (a) 4.5 × 10–5 T (b) 1.4 × 10–5 T
U

r (c) 5.5 × 10–5 T (d) 2.6 × 10–5 T


i 5. An electron moving with kinetic energy 6×10–16 joules
ED

O
enters a field of magnetic induction 6 × 10–3 weber/m2 at
(a) zero (b) infinite right angle to its motion. The radius of its path is
(a) 3.42 cm (b) 4.23 cm
μ0 π i μ0 πi
(c) . gauss (d) . tesla (c) 5.17 cm (d) 7.7 cm
4π r 4π r 6. An electron moves in a circular arc of radius 10 m at a contant
2. A helium nucleus makes a full rotation in a circle of radius speed of 2 × 107 ms–1 with its plane of motion normal to a
0.8 meter in 2 sec. The value of the magnetic field induction magnetic flux density of 10–5 T. What will be the value of
B in tesla at the centre of circle will be specific charge of the electron?
(a) 2 × 104 C kg–1 (b) 2 × 105 C kg–1
(a) 2 ´ 10 -19 m 0 (b) 10 -19 / m 0 (c) 5 × 106 C kg–1 (d) 2 × 1011 C kg–1
7. A current of 3 A is flowing in a linear conductor having a
(c) 10 -19 m 0 (d) 2 ´ 10 -20 / m 0 length of 40 cm. The conductor is placed in a magnetic field
of strength 500 gauss and makes an angle of 30º with the
3. A solenoid of length 1.5 m and 4 cm diameter possesses 10
direction of the field. It experiences a force of magnitude
turns per cm. A current of 5A is flowing through it, the
magnetic induction at axis inside the solenoid is (a) 3 × 10–4 N (b) 3 × 10–2 N
(c) 3 × 102 N (d) 3 × 104 N
(m 0 = 4 p ´ 10 - 7 weber amp -1 m - 1 ) 8. A cathode ray beam is bent in a circle of radius 2 cm by a
magnetic induction 4.5 × 10–3 weber/m2. The velocity of
(a) 4 p ´ 10 - 5 gauss (b) 2 p ´ 10 -5 gauss electron is
(a) 3.43 × 107 m/s (b) 5.37 × 107 m/s
(c) 4 p ´ 10 -5 tesla (d) 2 p ´ 10 -5 tesla (c) 1.23 × 107 m/s (d) 1.58 × 107 m/s
Moving Charges and Magnetism 527

9. Two long parallel wires P and Q are held perpendicular to m0 Iq m0 Iq m0 Iq m0 Iq


the plane of paper with distance of 5 m between them. If P (a) (b) (c) (d)
4 pr 2 pr 2r 4r
and Q carry current of 2.5 amp. and 5 amp. respectively in 16. A uniform electric field and a uniform magnetic field exist in
the same direction, then the magnetic field at a point half- a region in the same direction. An electron is projected with
way between the wires is velocity pointed in the same direction. The electron will
(a) m 0 / 17 (b) 3 m0 / 2 p (a) turn to its right
(b) turn to its left
(c) m0 / 2 p (d) 3 m0 / 2 p (c) keep moving in the same direction but its speed will
10. A charged particle with velocity 2 × 103 m/s passes increase
undeflected through electric and magnetic field. Magnetic (d) keep moving in the same direction but its speed will
field is 1.5 tesla. The electric field intensity would be decrease
(a) 2 × 103 N/C (b) 1.5 × 103 N/C 17. A current of I ampere flows along an infinitely long straight
3
(c) 3 × 10 N/C (d) 4/3 × 10–3 N/C thin walled hollow metallic cylinder of radius r. The magnetic
11. If in a circular coil A of radius R, current I is flowing and in field at any point inside the cylinder at a distance x from the
another coil B of radius 2R a current 2I is flowing, then the axis of the cylinder is
ratio of the magnetic fields BA and BB, produced by them m0 I m0 I
will be (a) ¥ (b) (c) (d) zero
2p r 2p x
(a) 1 (b) 2 (c) 1/2 (d) 4

.IN
12. A circular loop of area 0.02 m2 carrying a current of 10A, is 18. Two particles X and Y having equal charge, after being
held with its plane perpendicular to a magnetic field accelerated through the same potential difference enter a
induction 0.2 T. The torque acting on the loop is
AL region of uniform magnetic field and describe circular paths
of radii R1 and R2 respectively. The ratio of the mass of X to
(a) 0.01 Nm (b) 0.001 Nm
that of Y is
(c) zero (d) 0.8 Nm
N
2
13. Through two parallel wires A and B, 10A and 2A of currents R1 æ R2 ö
are passed respectively in opposite directions. If the wire A (a) R2 (b) ç ÷
è R1 ø
R

is infinitely long and the length of the wire B is 2m, then


U

force on the conductor B, which is situated at 10 cm distance 2


æ R1 ö R2
from A, will be (c) ç ÷ (d)
JO

è R2 ø R1
(a) 8 × 10–7 N (b) 8 × 10–5 N
(c) 4 × 10 N –7 (d) 4 × 10–5 N 19. A square coil of side a carries a current I. The magnetic field
at the centre of the coil is
U

14. The magnetic flux density B at a distance r from a long


straight wire carrying a steady current varies with r as
ED

(a) B (b) B

a O

r r
(c) B (d) B m0 I 2m 0 1
(a) (b)
ap ap
m 01 2 2 m0 I
(c) (d)
2 ap ap
20. Protons and a-particles of equal momenta enter a uniform
r r
magnetic field normally. The radii of their orbits will have
15. A current of I ampere flows in a wire forming a circular arc of
the ratio.
radius r metres subtending an angle q at the centre as shown.
(a) 1 (b) 2 (c) 0.5 (d) 4
The magnetic field at the centre O in tesla is
21. Under the influence of a uniform magnetic field a charged
I particle is moving in a circle of radius R with constant speed
v. The time period of the motion
q
(a) depends on both R and v
(b) is independent of both R and v
O
(c) depends on R and not v
(d) depends on v and not on R
EBD_7179
528 PHYSICS

22. What is cyclotron frequency of an electron with an energy 31. In fig, what is the magnetic field induction at point O
of 100 e V in the earth's magnetic field of 1 × 10–4 weber / m2 m0 i
if its velocity is perpendicular to magnetic field? (a) i
4pr
(a) 0.7 MHz (b) 2.8 MHz
(c) 1.4 MHz (d) 2.1 MHz m0 i m0 i
(b) +
23. A circular loop of area 0.02 m2 carrying a current of 10A, is 4r 2pr
held with its plane perpendicular to a magnetic field m0 i m0 i
induction 0.2 T. The torque acting on the loop is (c) + r
4r 4pr
(a) 0.01 Nm (b) 0.001 Nm O
(c) zero (d) 0.8 Nm m0 i m0 i
24. Two thin, long, parallel wires, separated by a distance ‘d’ (d) 4 r - 4 p r
carry a current of ‘i’ A in the same direction. They will 32. The field B at the centre of a circular coil of radius r is p times
2 that due to a long straight wire at a distance r from it, for
(a) repel each other with a force of m 0 i /(2pd) equal currents. Fig. shows three cases:
(b) attract each other with a force of m 0 i 2 /(2pd)
P
(c) repel each other with a force of m 0 i 2 /(2pd 2 ) P P
(a) (b) (c)
(d) attract each other with a force of m 0 i 2 /(2pd 2 )
in all cases the circular part has radius r and straight ones are

.IN
25. A horizontal overhead powerline is at height of 4m from infinitely long. For same current the field B at the centre P in
the ground and carries a current of 100A from east to west. cases 1, 2, 3 has the ratio
The magnetic field directly below it on the ground is
æ p ö p æ 3p 1 ö
(m0 = 4p × 10–7 Tm A–1)
AL ç- ÷: : ç - ÷
(a)
(a) 2.5×10–7 T southward (b) 5 × 10–6 T northward è 2ø 2 è 4 2ø
(c) 5 × 10–6 T southward (d) 2.5 × 10–7 T northward
æ p ö æ p ö æ 3p 1 ö
N
26. If an electron describes half a revolution in a circle of radius (b) ç - + 1÷ : ç + 1÷ : ç + ÷
r in a magnetic field B, the energy acquired by it is è 2 ø è2 ø è 4 2ø
R

1 p p 3p
(a) zero (b) mv 2 (c) - : :
U

2 2 2 4
1 æ p ö æ p 1 ö æ 3p 1 ö
JO

(c) mv 2 (d) p r ´ Bev


4 (d) çè - - 1÷ø : çè + ÷ø : çè + ÷ø
2 4 4 4 2
27. The orbital speed of electron orbiting around a nucleus in a 33. An infinite straight conductor carrying current 2 I is split
circular orbit of radius 50 pm is 2.2 × 106 ms–1. Then the
U

into a loop of radius r as shown in fig. The magnetic field at


magnetic dipole moment of an electron is the centre of the coil is
ED

(a) 1.6 × 10–19 Am2 (b) 5.3 × 10–21 Am2


(c) 8.8 × 10 Am–24 2 (d) 8.8 × 10–26 Am2 m 0 2 (p + 1) I
(a)
28. A deutron of kinetic energy 50 keV is describing a circular 4p r
orbit of radius 0.5m, in a plane perpendicular to magnetic m 0 2 ( p - 1) 2I 2I
r (b) O
field B . The kinetic energy of a proton that discribes a 4p r
circular orbit of radius 0.5m in the same plane with the same
r m 0 ( p + 1)
magnetic field B is (c)
4p r I
(a) 200 keV (b) 50 keV (c) 100 keV (d) 25 keV (d) zero
29. A proton and an a-particle enter a uniform magnetic field 34. A long wire is bent into shape ABCDE as shown in fig., with
perpendicularly with the same speed. If proton takes 25 m BCD being a semicircle with centre O and radius r metre. A
second to make 5 revolutions, then the time period for the current of I amp. flows through it in the direction
a-particle would be A ® B ® C ® D ® E. Then the magnetic induction at the
(a) 50 m sec (b) 25 m sec point O of the figure in vacuum is
(c) 10 m sec (d) 5 m sec
30. A cell is connected between two points of a uniformly thick é I I ù
(a) m0 ê + ú B
circular conductor and i1 and i2 are the currents flowing in ë 2 p r 4 rû A
two parts of the circular conductor of radius a. The magnetic I
I
field at the centre of the loop will be é I I ù B × O r C
m0 (b) m 0 ê - ú
(I1 - I 2 ) ë 2 p r 4 rû I
(a) zero (b) E
4p D
(c) m0 I / 4 r
m0 m0
(c) ( I1 + I 2 ) (d) ( I1 + I 2 ) (d) m0 I / p r
2a a
Moving Charges and Magnetism 529

35. Three wires are situated at the same distance. A current of


1A, 2A, 3A flows through these wires in the same direction.
What is ratio of F1/F2, where F1 is force on 1 and F2 on 2? Q S
P R
(a) 7/8
(b) 1
(c) 9/8 (a) P (b) Q
(c) R (d) S
(d) None of these 1A 2A 3A 40. A charged particle (charge q) is moving in a circle of radius
36. A conducting circular loop of radius r carries a constant R with uniform speed v. The associated magnetic moment µ
r is given by
current i. It is placed in a uniform magnetic field B0 such
(a) qvR2 (b) qvR2/2 (c) qvR (d) qvR/2
r 41. A straight wire of diameter 0.5 mm, carrying a current of 1A
that B0 is perpendicular to the plane of the loop. The
is replaced by another wire of 1mm diameter carrying the
magnetic force acting on the loop is same current. The strength of magnetic field far away is
(a) ir B0 (b) 2p ir B0 (a) unchanged
(c) zero (d) p ir B0 (b) quarter of its earlier value
37. An electron traveling with a speed u along the positive (c) half of the earlier value
x-axis enters into a region of magnetic field where B = –B0 (d) twice the earlier value
42. Two equal electric currents are flowing perpendicular to each

.IN
k̂ ( x > 0). It comes out of the region with speed v then other as shown in figure. AB and CD are perpendicular to
each other and symmetrically placed with respect to the
×B currents. Where do we expect the resultant magnetic field
y
AL to be zero? I B
C
e– u (a) on AB
x® (b) on CD I
N
O
(c) on both AB & CD
(d) on both OD & BO A D
R

43. The magnetic field (dB) due to small element (dl) at a distance
r
U

(a) v = u at y > 0 (b) v = u at y < 0 ( r ) from element carrying current i, is


JO

(c) v > u at y > 0 (d) v > u at y < 0 æ ® rö æ ® rö


38. A wire ABCDEF is bent in the form as shown in figure. The m 0 ç dl ´ r ÷ m dl´ r
(a) dB = i (b) dB = 0 i2 ç 2 ÷
wire carries a current I and is placed in a uniform magnetic 4p è r ø 4p è r ø
U

field of induction B parallel to positive Y-axis. If each side is æ ® rö


æ ® rö
of length L, the force experienced by the wire will be m 0 2 ç dl´ r ÷ m 0 ç dl´ r ÷
(d) dB =
ED

(c) dB = i i
4p è r 2 ø 4p è r3 ø
Z 44. A 10eV electron is circulating in a plane at right angles to a
D uniform field at a magnetic induction 10–4 Wb/m2 (= 1.0
L gauss). The orbital radius of the electron is
B (a) 12 cm (b) 16 cm (c) 11 cm (d) 18 cm
C L 45. A coil of one turn is made of a wire of certain length and
then from the same length a coil of two turns is made. If the
I
E F same current is passed in both the cases, then the ratio of
L L the magnetic inductions at their centres will be
Y (a) 2 : 1 (b) 1 : 4 (c) 4 : 1 (d) 1 : 2
O
46. A very long straight wire carries a current I. At the instant
B L A ®
when a charge + Q at point P has velocity v , as shown,
X the force on the charge is
y
(a) IBL along the positive Z-direction
(b) IBL along the negative Z-direction
Q
(c) 2IBL along the positive Z-direction
(d) 2IBL along the negative Z-direction P®
v
39. Four wires, each of length 2.0 m, are bent into four loops P, I o x
Q, R and S and then suspended in a uniform magnetic field.
If the same current is passed in each, then the torque will be (a) along oy (b) opposite to oy
maximum on the loop (c) along ox (d) opposite to ox
EBD_7179
530 PHYSICS

47. A square current carrying loop is suspended in a uniform 52. Charge q is uniformly spread on a thin ring of radius R. The
magnetic field acting in ther plane of the loop. If the force on ring rotates about its axis with a uniform frequency f Hz.
one arm of the loop is F , the net force on the remaining The magnitude of magnetic induction at the centre of the
three arms of the loop is ring is
r r m 0 qf m0 q m 0q m 0 qf
(a) 3 F (b) – F (a) (b) (c) (d)
r 2f R 2pf R 2 pR
r 2R
(c) – 3 F (d) F 53. Two similar coils of radius R are lying concentrically with
48. A current loop consists of two identical semicircular parts their planes at right angles to each other. The currents
each of radius R, one lying in the x-y plane and the other in flowing in them are I and 2 I, respectively. The resultant
x-z plane. If the current in the loop is i., the resultant magnetic magnetic field induction at the centre will be
field due to the two semicircular parts at their common centre is
m0 i m 0i 5m 0 I 3m0 I m0I m0I
(a) (b) (c) (d)
(a) (b) 2R 2R 2R R
2R 2 2R
54. An alternating electric field, of frequency v, is applied across
m 0i m 0i the dees (radius = R) of a cyclotron that is being used to
(c) (d)
2R 4R accelerate protons (mass = m). The operating magnetic field
49. A current carrying loop in the form of a right angle isosceles (B) used in the cyclotron and the kinetic energy (K) of the

.IN
triangle ABC is placed in a uniform magnetic field acting proton beam, produced by it, are given by
along AB. If the magnetic force on the arm BC is F, what is
the force on the arm AC? (a) mn and K = 2mp2n2R2
r A
AL B=
(a) - 2 F e
r
(b) - F 2pm n
N
(b) B= and K = m2pnR2
r e
(c) F
R

r B C
(d) 2F (c) 2pmn and K = 2mp2n2R2
U

B=
50. A uniform electric field and uniform magnetic field are acting e
JO

along the same direction in a certain region. If an electron is


projected in the region such that its velocity is pointed along mn
(d) B= and K = m2pnR2
the direction of fields, then the electron e
U

(a) will turn towards right of direction of motion 55. A proton carrying 1 MeV kinetic energy is moving in a
(b) speed will decrease
ED

circular path of radius R in uniform magnetic field. What


(c) speed will increase should be the energy of an a-particle to describe a circle of
(d) will turn towards left direction of motion same radius in the same field?
51. A square loop, carrying a steady current I, is placed in a (a) 2 MeV (b) 1 MeV
horizontal plane near a long straight conductor carrying a
(c) 0.5 MeV (d) 4 MeV
steady current I1 at a distance d from the conductor as
shown in figure. The loop will experience 56. A magnetic needle suspended parallel to a magnetic field
requires 3 J of work to turn it through 60°. The torque
I1
needed to maintain the needle in this position will be
d I
3
(a) 2 3J (b) 3J (c) 3J (d) J
2
57. A current loop in a magnetic field
I (a) can be in equilibrium in one orientation
(b) can be in equilibrium in two orientations, both the
(a) a net repulsive force away from the conductor
equilibrium states are unstable
(b) a net torque acting upward perpendicular to the
horizontal plane (c) can be in equilibrium in two orientations, one stable
(c) a net torque acting downward normal to the horizontal while the other is unstable
plane (d) experiences a torque whether the field is uniform or
(d) a net attractive force towards the conductor non-uniform in all orientations
Moving Charges and Magnetism 531

58. A charged particle moves through a magnetic field in a (b) start moving in a circular path Y–Z plane
direction perpendicular to it. Then the (c) retard along X-axis
(a) velocity remains unchanged (d) move along a helical path around X-axis
(b) speed of the particle remains unchanged 66. The magnetic field at a distance r from a long wire carrying
(c) direction of the particle remains unchanged current i is 0.4 tesla. The magnetic field at a distance 2r is
(a) 0.2 tesla (b) 0.8 tesla
(d) acceleration remains unchanged
(c) 0.1 tesla (d) 1.6 tesla
59. Electron move at right angle to a magnetic field of 1.5 × 10–
2 tesla with speed of 6 × 107 m/s. If the specific charge of the
67. A straight wire of length 0.5 metre and carrying a current of
1.2 ampere is placed in uniform magnetic field of induction 2
electron is 1.7 × 1011 C/kg. The radius of circular path will be tesla. The magnetic field is perpendicular to the length of
(a) 3.31 cm (b) 4.31cm the wire. The force on the wire is
(c) 1.31 cm (d) 2.35 cm (a) 2.4 N (b) 1.2 N
60. A conducting circular loop of radius r carries a constant (c) 3.0 N (d) 2.0 N
current i. It is placed in a uniform magnetic field B such that 68. A moving coil galvanometer has a resistance of 900 W. In
B is perpendicular to the plane of the loop. The magnetic order to send only 10% of the main current through this
force acting on the loop is : galvanometer, the resistance of the required shunt is
(a) ir B (b) 2priB (a) 0.9 W (b) 100 W (c) 405 W (d) 90 W

.IN
(c) zero (d) priB 69. A uniform magnetic field acts at right angles to the direction
61. An infinitely long straight wire contains a uniformly of motion of electron. As a result, the electron moves in a
continuous current of 10A. The radius of the wire is 4× 10 – circular path of radius 2cm. If the speed of electron is
2 m. The magnetic field at 2 × 10 –2 m from the centre of the
AL doubled, then the radius of the circular path will be
wire will be: (a) 2.0 cm (b) 0.5 cm
N
(a) 0 (b) 2.5 × 10 – 5 T (c) 4.0 cm (d) 1.0 cm
70. An electron moves in a circular orbit with a uniform speed
(c) 5.0 × 10 – 5 T (d) none of these.
R

v. It produces a magnetic field B at the centre of the


62. A proton moving vertically downward enters a magnetic
circle. The radius of the circle is proportional to
U

field pointing towards north. In which direction proton will


deflect?
JO

B B
(a) East (b) West (a) (b)
v v
(c) North (d) South
r
U

63. When a charged particle moving with velocity v is subjected v v


(c) (d)
ur B B
ED

to a magnetic field of induction B , the force on it is non-


zero. This implies that 71. The magnetic induction at a point P which is at a distance of
r ur 4 cm from a long current carrying wire is 10 –3 T. The field of
(a) angle between v and B is necessarily 90° induction at a distance 12 cm from the current will be
r ur
(b) angle between v and B can have any value other (a) 3.33 × 10–4 T (b) 1.11 × 10–4 T
than 90° (c) 3 × 10–3 T (d) 9 × 10–3 T
r ur
(c) angle between v and B can have any value other Directions for Qs. (72 to 75) : Each question contains
than zero and 180° STATEMENT-1 and STATEMENT-2. Choose the correct answer
r ur (ONLY ONE option is correct ) from the following.
(d) angle between v and B is either zero or 180° (a) Statement -1 is false, Statement-2 is true
64. A coil carrying electric current is placed in uniform magnetic (b) Statement -1 is true, Statement-2 is true; Statement -2 is a
field, then correct explanation for Statement-1
(a) torque is formed (c) Statement -1 is true, Statement-2 is true; Statement -2 is not
(b) e.m.f is induced a correct explanation for Statement-1
(c) both (a) and (b) are correct (d) Statement -1 is true, Statement-2 is false
(d) none of the above 72. Statement 1 : If the current in a solenoid is reversed in
65. A charge moving with velocity v in X-direction is subjected direction while keeping the same magnitude, the magnetic
to a field of magnetic induction in negative X-direction. As field energy stored in the solenoid decreases.
a result, the charge will Statement 2 : Magnetic field energy density is proportional
(a) remain unaffected to square of current.
EBD_7179
532 PHYSICS

73. Statement 1 : If a charged particle is released from rest in a


region of uniform electric and magnetic fields parallel to I1
each other, it will move in a straight line. I I
Statement 2 : The electric field exerts no force on the particle q
but the magnetic field does.
74. Statement 1 : A cyclotron cannot accelerate neutrons.
Statement 2 : Neutrons are neutral.
I2
75. Statement 1 : The magnetic field at the centre of the circular
coil in the following figure due to the currents I1 and I2 is
Statement 2 : I1 = I2 implies that the fields due to the current
zero.
I1 and I2 will be balanced.

Exemplar Questions 5. In a cyclotron, a charged particle

.IN
1. Two charged particles traverse identical helical paths in a (a) undergoes acceleration all the time
completely opposite sense in a uniform magnetic field (b) speeds up between the dees because of the magnetic field
B = B0 kˆ .
AL
(c) speeds up in a dees
(a) They have equal z-components of momenta (d) slows down within a dee and speeds up between dees
(b) They must have equal charges
N
(c) They necessarily represent a particle, anti-particle pair NEET/AIPMT (2013-2017) Questions
R

(d) The charge to mass ratio satisfy 6. A current loop in a magnetic field [2013]
U

æeö æeö (a) can be in equilibrium in one orientation


ç ÷ +ç ÷ = 0
è m ø1 è m ø2 (b) can be in equilibrium in two orientations, both the equi-
JO

2. Biot-Savart law indicates that the moving electrons (velocity librium states are unstable
v) produce a magnetic field B such that (c) can be in equilibrium in two orientations, one stable
U

(a) B is perpendicular of while the other is unstable


(b) B is parallel to v
ED

(d) experiences a torque whether the field is uniform or


(c) it obeys inverse cube law non-uniform in all orientations
(d) it is along the line joining the electron and point of 7. When a proton is released from rest in a room, it starts with
observationt.
an initial acceleration a0 towards west. When it is projected
3. A current carrying circular loop of radius R is placed in the x- towards north with a speed v0 it moves with an initial
y plane with centre at the origin. Half of the loop with x > 0 is
acceleration 3a0 towards west. The electric and magnetic
now bent so that it now lies in the y-z plane.
fields in the room are respectively [2013]
(a) The magnitude of magnetic moment now diminishes
(b) The magnetic moment does not change ma0 2ma0
(c) The magnitude of B at (0, 0, z), z > R increases (a) west, down
e ev0
(d) The magnitude of B at (0, 0, z), z >> R is unchanged
4. An electron is projected with uniform velocity along the ma0 3ma0
axis of a current carrying long solenoid. Which of the (b) east, up
e ev0
following is true?
(a) The electron will be accelerated along the axis 3ma0
ma0
(b) The electron path will be circular about the axis (c) east, down
e ev0
(c) The electron will experience a force at 45° to the axis and
hence execute a helical path
ma0 2ma0
(d) The electron will continue to move with uniform velocity (d) west, up
e ev0
along the axis of the solenoid
Moving Charges and Magnetism 533

8. A long straight wire carries a certain current and produces a Z


weber
magnetic field of 2 × 10–4 at a perpendicular distance
m2
of 5 cm from the wire. An electron situated at 5 cm from the
I
wire moves with a velocity 107 m/s towards the wire along
perpendicular to it. The force experienced by the electron R
Y
O
will be [NEET Kar. 2013]
(charge on electron =1.6 × 10–19 C)
I
(a) Zero (b) 3.2 N
X
(c) 3.2 × 10–16 N (d) 1.6 × 10–16 N
ur
9. A circular coil ABCD carrying a current i is placed in a uniform
magnetic field. If the magnetic force on the segment AB is
(a)
m I $
B= – 0
4p R
mi ´ 2k$ ( )
r
F , the force on the remaining segment BCDA is ur
[NEET Kar. 2013] (b)
m I $
B= – 0
4p R
pi + 2k$ ( )
ur m0 I
A
i
(c) B=
4p R
(
p$i – 2k$ )

.IN
ur m0 I
D B AL (d) B=
4p R
(
p$i + 2k$ )
13. A proton and an alpha particle both enter a region of uniform
C magnetic field B, moving at right angles to field B. If the
radius of circular orbits for both the particles is equal and
N
r r the kinetic energy acquired by proton is 1 MeV the energy
(a) F (b) -F
R

acquired by the alpha particle will be: [2015 RS]


r r
(c) 3F (d) -3F (a) 0.5 MeV (b) 1.5 MeV
U

10. Two identical long conducting wires AOB and COD are (c) 1 MeV (d) 4 MeV
JO

placed at right angle to each other, with one above other 14. A rectangular coil of length 0.12 m and width 0.1 m having 50
such that ‘O’ is their common point for the two. The wires turns of wire is suspended vertically in a uniform magnetic field
carry I1 and I2 currents respectively. Point ‘P’ is lying at of strength 0.2 weber/m2. The coil carries a current of 2A. If
U

distance ‘d’ from ‘O’ along a direction perpendicular to the the plane of the coil is inclined at an angle of 30° with the
direction of the field, the torque required to keep the coil in
ED

plane containing the wires. The magnetic field at the point


stable equilibrium will be : [2015 RS]
‘P’ will be : [2014]
(a) 0.20 Nm (b) 0.24 Nm
m 0 æ I1 ö m0 (c) 0.12 Nm (d) 0.15 Nm
(a) (b) (I1 + I2 )
2pd çè I2 ÷ø 2pd 15. A square loop ABCD carrying a current i, is placed near and
coplanar with a long straight conductor XY carrying a
current I, the net force on the loop will be : [2016]
m0 2 m0 2 2 1/ 2
(c) (I1 - I22 ) (d) (I1 ´ I 2 ) Y B C
2 pd 2 pd
11. An electron moving in a circular orbit of radius r makes n
rotations per second. The magnetic field produced at the 1 L
I
centre has magnitude: [2015]
m0 n 2 e
(a) Zero (b) X A D
r
m0 ne m0 ne
(c) (d) L/2 L
2r 2pr
12. A wire carrying current I has the shape as shown in adjoining 2m 0 Ii m 0 Ii
(a) (b)
figure. Linear parts of the wire are very long and parallel to 3p 2p
X-axis while semicircular portion of radius R is lying in Y-Z 2m 0 IiL m 0 IiL
plane. Magnetic field at point O is : [2015] (c) (d)
3p 2p
EBD_7179
534 PHYSICS

16. A long straight wire of radius a carries a steady current I. (a) 4.55 mJ (b) 2.3 mJ
The current is uniformly distributed over its cross-section. (c) 1.15 mJ (d) 9.1 mJ
The ratio of the magnetic fields B and B', at radial distances 19. An arrangement of three parallel straight wires placed
a perpendicular to plane of paper carrying same current 'I
and 2a respectively, from the axis of the wire is :[2016] along the same direction is shown in fig. Magnitude of force
2
per unit length on the middle wire 'B' is given by [2017]
1 1
(a) (b)
4 2 B d C
(c) 1 (d) 4 90°
17. A long solenoid has 1000 turns. When a current of 4A flows
through it, the magnetic flux linked with each turn of the d
solenoid is 4 × 10–3 Wb. The self inductance of the solenoid
is : [2016]
A
(a) 4H (b) 3H
(c) 2H (d) 1H
2m 0 i 2 2m0 i 2
18. A 250-turn rectangular coil of length 2.1 cm and width 1.25 (a) (b)
pd pd
cm carries a current of 85 mA and subjected to magnetic field

.IN
of strength 0.85 T. Work done for rotating the coil by 180º m0 i 2 m0i2
against the torque is [2017] (c) (d)
2pd 2 pd
AL
N
R
U
JO
U
ED
Moving Charges and Magnetism 535

Hints & Solutions


EXERCISE - 1 1 df
Total charge induced = ò i dt = ò .dt
1. (d) 2. (d) R dt
3. (b) F = iB l sin q. This is maximum when sin q = 1 f
1 2 1
R fò
or q = p/2. = d f = (f 2 - f1 )
R
m 0 2pni1 m 0 2pni 2 m é ni ni ù 1

4. (d) B= . - . = 0 ê 1- 2ú 24. (b) Energy is stored in magnetic field.


4p r1 4p r2 2 ë r1 r2 û 25. (b) Tesla is the unit of magnetic field.
5. (c) If r is the radius of the circle,
L EXERCISE - 2
then L = 2pr or, r =
2p 1. (d) The straight part will not contribute magnetic field at the
Area = pr 2 = pL2 / 4p 2 = L2 / 4 p centre of the semicircle because every element of the
6. (c) The point P is lying symmetrically w.r.t. the two long straight part will be 0º or 180º with the line joining the
straight current carrying conductors. The magnetic centre and the element
fields at P due to these current carrying conductors are 1 m0i m 0 i

.IN
mutually perpendicular. Due to circular portion, the field is =
2 2r 4r
mv m
7. (a) r= or, r µ for the same value of v and B. m0 i
Bq q Hence total field at O =
AL 4r
tesla
8. (a) r = mv/Bq is same for both.
mv2 Þ r = mv m 0 2pi
9. (a) Bqv = 2. (c) B= where
N
r Bq 4p r
nBA 2e 2 ´ 1.6 ´10 -19
R

10. (a) Current sensitivity = i= = = 1.6 ´ 10 -19 A


K t 2
U

where K is constant of torsional rigidity.


m i m ´ 1.6 ´ 10 -19
11. (a) \ B= 0 = 0 = m 0 ´ 10 -19 T
JO

12. (a) 2r 2 ´ 0.8


13. (b) 14. (b) 3. (d) B = m 0 nI = 4p ´10 -7 ´ 10 ´ 5 = 2p ´ 10 -5 T.
15. (b) In cyclotron, energy with which aceleration takes place
m v2 (9 ´ 10 -31 ) ´ 10 6
U

mv
is in term of MeV. 4. (c) Bqv = or B = =
16. (a) r rq 0.1 ´ (1.6 ´ 10 -19 )
ED

17. (d) = 5.5 ´10 -5 T


æ m 0 NI ö 2 1
18. (c) Baxis = çç ÷÷R 5. (a) Ek = mv2 or mv = 2 E k m and
è 2x 3 ø 2
B µ R2 mv 2 Ek m
r= =
So, when radius is doubled, magnetic field becomes Bq Bq
four times. 6. 2
(d) Bqv = mv /r or q/m = v /rB.
19. (a) To convert a galvanometer into an ammeter, one needs 7. (b) F = I l B sin q = 3 ×0.40 × (500 × 10–4) × sin 30º
to connect a low resistance in parallel so that maximum = 3 × 10–2 N.
current passes through the shunt wire and ammeter remains
protected. Bqr 4.5 ´10 -3 ´ 1.6 ´10 -19 ´ 2 ´10 -2
20. (b) It will compress due to the force of attraction between 8. (d) v= =
m 9.1´10 -31
two adjacent coils carrying current in the same
= 1.58 ´107 m / s
direction.
21. (b) Magnetic force acts perpendicular to the velocity. m0 2 i2 m 2 i1 m 4
9. (c) B= - 0 = 0 (i 2 - i1 )
Hence speed remains constant. 4 p (r / 2) 4 m (r / 2) 4 p r
æN ö m0 4 m
22. (c) B = m0 N0i ; B1 = (m 0 ) ç 0 ÷ (2 i) = m 0 N 0 i = B = (5 - 2.5) = 0 .
è 2 ø 4p 5 2p
Þ B1 = B 10. (c) E = vB = 2 ´ 103 ´ 1.5 = 3 ´ 103 V / m.
df e 1 df
23. (c) e= ;i= = m 2pI I
dt R R dt 11. (a) In coil A, B = 0 . \B µ ;
4p R R
EBD_7179
536 PHYSICS

B1 I1 R 2 2
Hence, B = R × I = 2 = 1 F m 0 i1 i 2 m 0i 2
= =
2 1 2 l 2 pd 2 pd
12. (c) Torque on loop t = nIAB cos q; Here q = 90º
\ t = 0. (attractive as current is in the same direction)
25. (c) The magnetic field is
m 2I I 10-7 ´ 2 ´10 ´ 2
13. (b) F= 0 1 2 ´l = ´ 2 = 8 ´10-5 N m0 2I 2 ´ 100
4p r 0.1 B= = 10-7 ´ = 5 × 10–6 T
14. (c) 4p r 4

µ0 I q m Iq W N
15. (a) B= ´ = 0
2r 2p 4 pr 100A
16. (d) No magnetic force acts on the electron and force due
to electric field will act opposite to its initial direction
of motion. Hence its velocity decreases in magnitude. 4m
17. (d) Since no current is enclosed inside the hollow S E
conductor. Hence Binside = 0. Ground
1 2mV B
18. (c) r= According to right hand palm rule, the magnetic field is
B q
directed towards south.

.IN
26. (a) Since magnetic force is always perpendicular to the
R1 mx
= velocity of electron, so it can only change the direction
R2 my AL of velocity of electron, but it (the magnetic force) cannot
accelerate or deaccelerate the electron.
2 27. (c) Magnetic dipole moment
m x æ R1 ö
Þ =ç ÷ e e erv
N
my è R2 ø m = iA = ´ pr 2 = ´ pr 2 = .
T (2 pr / v) 2
R

19. (d) Btotal = 4Bside


1.6 ´ 10-19 ´ 50 ´ 10-12 ´ 2.2 ´ 106
m0 I é p pù =
U

Btotal = 4 ê sin + sin ú 2


æa ö 4 4û
2p ç ÷ ë = 8.8 ´ 10-24 Am2 .
JO

è2ø
2m d E d 2m p E p
2 2m0 I 28. (c) So rdeutron = ; rproton =
Btotal =
U

Bq Bq
ap
For same radius, B and q
ED

mv p rp q a 2e 2
r= = Þ = = = md 2
20. (b)
qB qB ra q p e 1 mp Ep = mdEd Þ E p = E d = ´ 50 = 100keV
mp 1
21. (b) In a uniform magnetic field, a charged particle is moving 29. (c) Time taken by proton to make one revolution
in a circle of radius R with constant speed v. 25
= = 5 m sec .
mv2 mv 5
\ = Bqv or, R = .....(1)
R Bq 2 pm T m q
As T = ; so 2 = 2 ´ 1
qB T1 m1 q 2
2pR 2pmv 2pm m 2 q1 5 ´ 4 m1 q
Time period, T = = = .....(2) or T2 = T1 = ´ = 10 m sec.
v Bqv Bq m1 q 2 m1 2q
Time period T does not depend on both R and v 30. (a) Let l1, l2 be the lengths of the two parts PRQ and PSQ of
because when v is changed, R is also changed the conductor and r be the resistance per unit length of
proportionately and for period, it is R/v that is taken. the conductor. The resistance of the portion PRQ will be
22. (b) 23. (c) R1 = l1 r
24. (b) I2
i i
F
S O Q
R
P I1
Moving Charges and Magnetism 537

The resistance of the portion PSQ will be R2 = l2 r


Pot. diff. across P and Q = I1 R1 = I2 R2 é I I ù
= m0 ê + ú
or I1 l1 r = I2 l2 r or I1 l1 = I2 l2 ...(1) ë 2 p r 4 rû
Magnetic field induction at the centre O due to currents r
through circular conductors PRQ and PSQ will be (The direction of B is into the page.)
m 0 I1l1 sin 90º m 0 I 2 l 2 sin 90º 35. (a) Due to flow of current in same direction at adjacent
= B1 – B2 = - = 0. side, an attractive magnetic force will be produced.
4p r2 4p r2 36. (c) The magnetic field is perpendicular to the plane of the
31. (c) B at O will be due to the following portions paper. Let us consider two diametrically opposite
(i) Vertical straight portion. This is zero. elements. By Fleming's Left hand rule on element AB
(ii) Circular portion. This is given by the direction of force will be Leftwards and the
1 m0 i m0 i magnitude will be
Bcircular = =
2 2r 4r dF = Idl B sin 90° = IdlB
(iii) Straight horizontal portion. This is given by
m i
x x x x x x x
Bstraight = 0
4pr x xB x x Cxdl x x
dF dl dF
m0 i m 0 i A D
\ BTotal = +
4r 4pr x x x x Ix x x

.IN
32. (a) For case (a) magnetic field due to straight portions is x x x x x x x
cancelled & the magnetic field due to semi circular arc
of radius r at P is On element CD, the direction of force will be towards
right on the plane of the papper and the magnitude will
AL
m o i ´ p æ m oi ö be dF = IdlB.
Ba = =ç ÷´ p
4p r è 4pr ø 37. (b) From Lorentz equation
N
It is in upward direction & we take upward direction F = -eu î ´ B0 (-k̂ ) = -euB 0 ˆj
r
R

æm iö hence it will complete a semicircular arc and comes out


negative, So B a = -ç o ÷.p
è 4 pr ø of the region at a position y, such that y < 0
U

38. (a)
For case (b) Due to straight portion the magnetic field
39. (d) For a given perimeter the area of circle is maximum. So
JO

is zero so the magnetic field due to semi circular arc is


magnetic moment of (S) is greatest.
r æm iö 40. (d) Magnetic moment µ = IA
B b = ç o ÷ ´ p (in down wards direction so +ive sign)
è 4 pr ø
U

2pR
Since T =
For case (c) Magnetic field due to straight portion is v
ED

mo i q qv
=- (upward direction) Also, I = =
4p r T 2pR
Magnetic field due to circular arc which substand an
angle 3p/2 at centre is æ qv ö
\ m=ç
è 2 pR ø
( )
2
÷ pR =
qvR
2
.
æ m i ö 3p
= ç o ÷´ (down ward direction)
è 4pr ø 2 m 0i
41. (a) [Hint Þ B = , where r is distance of point from the
2pr
r m i 3p
so Bc = æç o ö÷ æç - 1ö÷ wire, where we want to calculate the magnetic field. It is
è 4 pr ø è 2 ø clear from expression that B is independent of thickness
r r r æ 3p ö
of wire.]
so Ba : B b : B c = -p : p : ç - 1÷ 42. (a) The direction of the magnetic field due to a current is
è 2 ø
given by right hand curled fingers rule. Therefore at
æ -p ö p æ 3p 1 ö AB axis, the components of magnetic field will cancel
= ç ÷ : :ç - ÷ each other and the resultant magnetic field will be zero
è 2 ø 2 è 4 2ø
on AB.
33. (d) Here, the wire does not produce any magnetic field at O 43. (d)
because the conductor lies on the line of O. Also, the
loop does not produce magnetic field at O. mv 2
44. (b) [Hint Þ = qvB ].
m 0 é Ι pI I ù m 0 é 2 I p I ù r
34. (a) B= + + ê + r ú 45. (b) Let l be length of wire
4 p êë r r r úû 4 p ë r û
EBD_7179
538 PHYSICS

l where B is the magnitude of the magnetic force.


Ist case : l = 2pr Þ r = r
2p The direction of F will be in the direction perpendicular
to the plane of the paper and going into it.
m 0I m0I By Pythagorus theorem,
B= =
2 pr l
AC = x 2 + x 2 = 2x
l
2nd Case : l = 2(2pr ¢) Þ r ¢ =
4p \ Magnitude of force on AC = i 2 x B sin 45°

m 0 In 2m I 1 r
B¢ = = 0 (where n = 2) = i 2 xB´ = ixB = | F |
l l 2
2p
4p 2 The direction of the force on AC is perpendicular to
the plane of the paper and going out of it. Hence, force
æ m0I ö r
on putting the value of B Þ B¢ = 4ç ÷ = 4B on AC = - F
è l ø r
r
Ù 50. (b) v and B are in same direction so that magnetic force
46. (a) The direction of B is along ( - k ) on electron becomes zero, only electric force acts. But
\ The magnetic force force on electron due to electric field is opposite to the
direction of velocity.
F = Q( v ´ B) = Q( v î ) ´ B(- k̂ ) = QvBĵ

.IN
Þ along OY.. I1
47. (b) The force on the two arms parallel to the field is zero. 51. (d)
AL F1
< I
<

<

F F3 F4
N
B
R

–F
F2
U

<
\ Force on remaining arms = – F z 1
JO

48. (b) Magnetic fields due to the two i F1 > F2 as F µ, and F3 and F4 are equal and opposite.
parts at their common centre are d
respectively, Hence, the net attraction force will be towards the
conductor.
U

m i m i i
B y = 0 and Bz = 0 y 52. (a) Magnetic field at the centre of the ring is
4R 4R
ED

m 0 qf
i
Resultant field = B y2 + Bz2 2R

2
2 53. (a)
æ m iö æ m iö m 0i m i
= ç 0 ÷ +ç 0 ÷ = 2. = 0 B2
è 4R ø è 4R ø 4R 2 2 R
49. (b) Let a current i be flowing in the loop ABC in the
B1
direction shown in the figure. If the length of each of
the sides AB and BC be x then
r
| F| = i x B

A The magnetic field, due the coil, carrying current I


Ampere
m0 I
B1 =
2R
The magnetic field due to the coil, carrying current 2I
Ampere
m 0 (2I )
B C B2 =
2R
Direction of The resultant B
magnetic field
Moving Charges and Magnetism 539

58. (b) Magnetic force acts perpendicular to the velocity. Hence


Bnet = B12 + B22 + 2B1B2 cos q, q = 90° speed remains constant.
59. (d) B = 1.5 × 10–2 T, q = 90°, sin q = 1
m (2 I ) 5 m0 I
Bnet = B12 + B22
= 0 1+ 4 = e
2R 2R v = 6 × 107 m/s, = 1.7 × 1011 C/kg
m
54. (c) Time period of cyclotron is
mv 6 ´ 107
1 2pm 2pm mu p r= =
T= = ; B= u; R = = Be 1.5 ´ 10-2 ´ 1.7 ´ 1011
u eB e eB eB
= 2.35 × 10–2m = 2.35 cm
2pmu 60. (c) The force on each point on loop is radially outward and so
Þ P = eBR = e ´ R = 2pmuR net force = 0
e

p 2 (2pmuR )2 m 0 ir -7 2 ´ 10 -2
K.E. = = = 2p2mu2R2 61. (b) B= . 2 = 2 ´ 10 ´ 10 ´
2m 2m 2p R (4 ´ 10 - 2) 2
55. (b) According to the principal of circular motion in a
magnetic field = 2.5 ´ 10-5 T
62. (a) Proton will represent the direction of current, so the direction
mv 2 of current is vertically downward. By Fleming's left hand
Fc = Fm Þ = qVB rule, force acting on the proton in east direction.
R r urur

.IN
63. (c) As F = qVBsin q
mv P 2 m.k
Þ R= = = F is zero for sin 0° or sin 180° and is non-zero for angle
ur ur
qB qB qB
AL between V and B any value other than zero and 180°.
64. (a) A current carrying coil has magnetic dipole moment.
Ra =
2(4 m) K '
r r
2qB Hence, a torque pm ´ B acts on it in magnetic field.
N
65. (a) The force acting on a charged particle in magnetic field
R

R K is given by
= r r r
Ra K' F = q ( v ´ B ) or F = qvB sin q,
U

When angle between v and B is 180°,


but R = Ra (given)
F=0
JO

Thus K = K¢ = 1 MeV
56. (b) According to work energy theorem mi 1
66. (a) B = 0 or B µ
W =Ufinal – Uinitial = MB (cos 0 – cos 60°) 2pr r
U

When r is doubled, the magnetic field becomes half,


MB
W= = 3J ...(i) i.e., now the magnetic field will be 0.2 T.
ED

2 67. (b) F = Bil = 2 ×1.2 × 0.5 = 1.2 N


r r æ ö 68. (b) Ig = 0.1I, Is = 0.9 I ; S = Ig R g / Is
t = M ´ B = MB sin 60° = ç MB 3 ÷ ...(ii)
è 2 ø
= 0.1 ´ 900 / 0.9 = 100 W.
From equation (i) and (ii) mv
69. (c) r = or r µ v
2 3´ 3 qB
t= = 3J
2 As v is doubled, the radius also becomes double.
57. (c) A current loop in a magnetic field is in equilibrium in Hence, radius = 2 × 2 = 4 cm
two orientations one is stable and another unstable. mv v
r uur ur 70. (d) r= Þrµ
Q t = M ´ B = M B sin q qB B
If q = 0° Þ t = 0 (stable) m0 I 1
If q = p Þ t = 0 (unstable) 71. (a) B= ÞBµ
2 pr r
As the distance is increased to three times, the magnetic
induction reduces to one third. Hence,
1
B = ´ 10 -3 tesla = 3.33 ´ 10 -4 tesla
3
72. (a) Reversing the direction of the current reverses the
direction of the magnetic field. However, it has no effect
on the magnetic-field energy density, which is
Do not experience a torque in some orientations proportional to the square of the magnitude of the
Hence option (c) is correct. magnetic field.
EBD_7179
540 PHYSICS
r r 0 is now bent so that it now lies in the y-z plane, the
73. (c) Due to electric field, the force is F = qE in the direction
magnitudes of magnetic field moment of each
r r r
of E . Since E is parallel to B , the particle velocity vr semicircular loop of radius R lie in the x-y plane and the
r r r y-z plane is M' = I(pr2)/4 and the direction of magnetic
(acquired due to force F ) is parallel to B . Hence B
r field moments are along z-directlon and x-direction
will not exert any force since vr ´ B = 0 and the motion respectively.
r Then resultant is :
of the particle is not affected by B .
74. (b) Neutrons are neutral. Mnet = M ¢ 2 + M ¢ 2 = 2 M¢ = 2 I ( pr 2 ) 4
I1 2p - q So, Mnet < M or M diminishes.
75. (d) = Þ I1q = I2 (2p - q) ........... (1)
I2 q Hence, the magnitude of magnetic moment is now
diminishes.
q m 0 I1 2p - q m 0 I 2
B1 = . and B2 = . 4. (d) Magnetic Lorentz force :
2p 2R 2p 2R
F = qVB sin q
Using (1), we get B1 = B2.
Magnetic Lorentz force electron is projected with
EXERCISE - 3 uniform velocity along the axis of a current carrying
long solenoid F = –qvB sin 180° = 0(q = 0°) as magnetic

.IN
Exemplar Questions
field and velocity are parallel and electric field is zero (E
1. (d) As we know that the uniqueness of helical path is AL = 0) due to this magnetic field (B) perpendicular to the
determined by its pitch direction of motion (V). So it will not affect the velocity
of moving charge particle. So the electron will continue
2p mv cos q
P(Pitch) = to move with uniform velocity along the axis of the
N
Bq solenoid
Where q is angle of velocity of charge particle with x-
R

5. (a) There is crossed electric and magnetic field between


axis dees so the charged particle accelerates by electric field
U

For the given pitch d correspond to charge particle, we between dees towards other dees.
JO

have So, the charged particle undergoes acceleration as


q 2pv cos q (i) speeds up between the dees because of the oscillating
= = constant electric field.
m BP
U

If motion is not helical, (q = 0) (ii) speed remain the same inside the dees because of the
ED

As charged particles traverse identical helical paths in magnetic field but direction undergoes change
a completely opposite direction in a same magnetic continuously.
field B, LHS for two particles should be same and of Hence, the charge particle accelerates inside and
opposite sign. between Dees always.

æeö æeö NEET/AIPMT (2013-2017) Questions


\ ç ÷ +ç ÷ = 0
è m ø1 è m ø2 6. (c) A current loop in a magnetic field is in equilibrium in
2. (a) By Biot-Savart law two orientations one is stable and another unstable.
r uur ur
Q t = M ´ B = M B sin q
Idl sin q æ I ´ dl ö
dB = =ç ÷ If q = 0° Þ t = 0 (stable)
r2 è r ø
If q = p Þ t = 0 (unstable)
In Biot-Savat’s law, magnetic field B||idl × r and idl due
to flow of electron is in opposite direction of v and by
direction of cross product of two vectors
B^V
So, the magnetic field is ^ to the direction of flow of
charge.
3. (a) As the direction of magnetic moment of circular loop
of radius R placed in the x-y plane is along z-direction
Do not experience a torque in some orientations
and given by M = I (pr2), when half of the loop with x >
Hence option (c) is correct.
Moving Charges and Magnetism 541

12. (b) Magnetic field due to segment ‘1’


uur m I
$
B1 = 0 [sin 90° + sin 0°] (–k)
4 pR
7. (a)
– m0 I ur
=
4pR
( k$ ) = B 3

Magnetic field due to segment 2


When moves with an acceleration a0 towards west,
electric field
B2 =
4R
–i =( )
m0 I $ – m0 I $
4pR
pi ( )
F ma 0 Z
E= = (West)
q e
When moves with an acceleration 3a0 towards east, 2
magnetic field
2ma 0 Y
B= (downward) O
ev0 1
I I
8. (c) Given: 3

.IN
Magnetic field B = 2 × 10–4 weber/m2
Velocity of electron, v = 107 m/s X
Lorentz force F = qvB sin q ur
\ B at centre
= 1.6 × 10–19 × 107 × 2 × 10–4 (Q q = 90°)
AL
ur ur ur ur
= 3.2 × 10–16 N
r r r Bc = B1 + B2 + B3 =
– m0I $
4pR
pi + 2k$ ( )
N
9. (b) Here, FAB + FBCDA = 0
r r r mv 2
13. (c) As we know, F = qvB =
R

Þ FBCDA = - FAB = - F R
r mv 2m(kE)
U

(Q FAB = F ) \R= =
qB qB
JO

10. (d) Net magnetic field, B = B12 + B22 q2


Since R is same so, KE µ
m
2 2 Therefore KE of a particle
æm I ö æm I ö æ m 0 I1 m I ö
U

= ç 0 1 ÷ + ç 0 2 ÷ çQ B1 = and B2 = 0 2 ÷ q2
2
( 2 ) = 1 MeV
è 2pd ø è 2pd ø è 2 p d 2 pd ø = =
ED

m 4
m0 14. (a) Here, number of turns of coil, N = 50
= I12 + I22 Current through the coil I = 2A
2 pd Area A = l × b = 0.12 × 0.1m2
11. (c) Radius of circular orbit = r r
Magnetic field B = 0.2 w/m2
No. of rotations per second = n
1
i.e., T =
n

30° B
O r 60°

Magnetic field at its centre, Bc =? M


As we know, current
e e Torque required to keep the coil in stable equilibrium.
i= = = en = equivalent current r r
T (1 / n) t = M ´ B = MB sin 60° = Ni AB sin 60°
Magnetic field at the centre of circular orbit, 3
= 5 0 × 2 × 0 .1 2 × 0 . 1 × 0. 2 ×
m i m ne 2
Bc = 0 = 0
2r 2r = 12 3 ´ 10 -2 = 0.20784 Nm
EBD_7179
542 PHYSICS

15. (a) The direction of current in conductor 17. (d) Here, number of turns n = 100; current through the
Y FBC solenoid i = 4A; flux linked with each turn = 4 × 10–3
B C Wb
FAB FCD
\ Total flux linked, and total
I i L = 1000[4 × 10–3] = 4 Wb
A L D ftotal = 4 Þ L i = 4
X Þ L= 1H
FAD
XY and AB is same 18. (d) Work done, W = MB(cosq1–cosq2)
\ FAB = ilB (attractive) When it is rotated by angle 180° then
W = MB (cos0° – cos 180°) = MB (1 + 1)
m0 I m iI
(¬) = 0 ( ¬) W = 2MB
FAB = i(L). 2p æç ö÷
L p
è 2ø W = 2 (NIA) B
= 2 × 250 × 85 × 10–6[1.25 × 2.1 × 10–4] × 85 × 10–2
FBC opposite to FAD
FBC (­) and FAD (¯) = 9.1 mJ
Þ cancels each other 19. (c) Force per unit length between two parallel current car-
FCD = ilB (repulsive) rying conductors,

.IN
m0 I m iI m 0 i1i 2
FCD = i(L) (®) = 0 ( ®) F=
æ 3L ö 3p 2pd
2p ç ÷
AL
è 2 ø Since same current flowing through both the wires
Therefore the net force on the loop
ii = i2 = i
Fnet = FAB + FBC + FCD + FAD
N

mo iI mo iI 2mo iI m0i2
R

Þ Fnet = - = so F1 = = F2
p 3p 3p 2pd
U

16. (c) For points inside the wire i.e., (r < R)


F1[due to wire A]
JO

m0 Ir
Magnetic field B =
2pR 2
For points outside the wire (r > R)
U

m0I F2[due to wire C]


ED

Magnetic field, B' =


2 pR
\ Magnitude of force per unit length on the middle
m0 I ( a / 2 ) wire 'B'

B 2 pa 2 = 1:1 m0 i 2
\ = m0 I Fnet = F12 + F22 =
B' 2pd
2 p ( 2a )
Magnetism and
20 Matter
NATURAL MAGNET MAGNETIC FIELD
A natural magnet is an ore of iron (Fe3 O4) which The space around a magnet within which its influence can be
(i) attracts small pieces of iron, cobalt and nickel towards it. experienced is called its magnetic field.
(ii) when suspendeded freely, comes to rest along north-south Uniform magnetic field : A uniform magnetic field is one where the

.IN
direction. strength of the magnetic field is the same at all points of the field. In
The magnets which are obtained artificially are called artificial a uniform field, all the magnetic lines of force are parallel to one
magnets, e.g. a bar magnet, a magnetic needle, horse shoe magnet another. But in non-uniform magnetic field the strength of
AL
etc. magnetic field is not same at all points of the field and also the
BAR MAGNET magnetic lines of force are not parallel.
A bar magnet consists of two equal and opposite magnetic poles Atom as a Magnetic Dipole
N
separated by a small distance. Poles are not exactly at the ends. Every atom of a magnetic material behaves as a magnetic dipole,
R

The shortest distance between two poles is called effective length because electrons in the atom revolve round the nucleus. The
(Le) and is less than its geometric length (Lg). magnetic moment M associated with an atomic dipole as
U

For bar magnet Le = 2l and Le =(5/6) Lg. neh


M= = nm B
JO

For semicircular magnet Lg = pR and Le = 2R. 4pm


eh
where n = 1, 2, 3 ..... denotes the no. of orbits and m B = .
4pm
U

–24
Least value of dipole moment of atom = 9.27 × 10 Am . 2
S N
Le =2l m B is called Bohr magneton.
ED

Lg Most of the magnetic moment is produced due to electron spin,


Bar magnet
the contribution of the orbital revolution is very small.
Properties of Magnets MAGNETIC LINES OF FORCE
(i) Attractive property : A magnet attracts small pieces of iron, Magnetic line of force is an imaginary curve the tangent to which
cobalt, nickel, etc. and other magnetic subsances. at a point gives the direction of magnetic field at that point or
(ii) Directive property : A freely suspended magnet aligns itself the magnetic field line is the imaginary path along which an
nearly in the geographical north-south direction. isolated north pole will tend to move if it is free to do so.
(iii) Law of magnetic poles : Like magnetic poles repel, and unlike Properties of Magnetic Lines of Force
magnetic poles attract each other. (i) Magnetic lines of force are hypothetical lines use to depict
According to Gauss’s theorem in magnetism, surface magnetic field in a region and understand certain
integral of magnetic field intensity over a surface (closed or phenomenon in magnetism.
uur ur uur ur
open) is always zero i.e. Ñò B .d s (or ò B .d s ) = 0. Direction of magnetic
lines outside the body
This theorem establishes that the poles always exist in equal of magnet (from north
and unlike pairs. pole to south pole)
(iv) Magnetic poles exist in pairs : Isolated magnetic poles do
not exist. If we break a magnet into two pieces, we get two N S Direction of magnetic
lines inside the body
smaller dipole magnets. Magnetic field lines of magnet (from south
(v) Repulsion is a sure test of magnetism. in a bar magnet pole to north pole)
EBD_7179
544 PHYSICS

(ii) Tangent to field line at a point gives us the direction of Magnetic length : The shortest distance between the two poles
uur
magnetic field intensity B at that point. No two magnetic of a magnet is called its magnetic length. It is less than the
lines of force can intersect each other because magnetic geometrical length of the magnet. This magnetic length is also
field will have two directions at the point of intersection. called an effective length.
(iii) Magnetic lines of force are continuous curve from north to
south, outside the body of the magnet and from south to Geometrical length
north inside the body of the magnet.
Magnetic length
(iv) The number of lines originating or terminating on a pole is = 0.84 S N
Geometrical length
proportional to its pole strength.
Magnetic flux = number of magnetic lines of force Magnetic length
= µ0 × m
Where µ0 is number of lines associated with unit pole. MAGNETIC MOMENT
(v) The number of lines of force per unit area at a point gives The magnetic moment of a magnet in magnitude is equal to the
magnitude of field at that point. The crowded lines show a product of pole strength with effective length (i.e. magnetic
strong field while distant lines represent a weak field.
length). Its direction is along the axis of magnet from south pole
(vi) The magnetic lines of force have a tendency to contract
to north pole.
longitudinally like a stretched elastic string producing
r r r

.IN
attraction between opposite pole. M = m ´ 2 l ´ (n) Þ | M |= 2ml
If the same bar magnet is bent in a semicircle then
AL
N S 2l
pr = 2l Þ r =
p
N

Longitudnal contraction (attraction)


R

(vii) The magnetic lines of force have a tendency to repel each r


U

other laterally resulting in repulsion between similar poles. m m


JO

Net magnetic moment

2l 4l 2 M
M ¢ = m ´ 2r = m ´ 2 ´ = m´ =
U

S N X N p p p
r
ED

Where m is pole strength, 2l is effective length and n is unit


vector having a direction from S-pole to N-pole.
n (attraction) Lateral expansion (repulsion)
r
The SI unit of M is A m2, which is equivalent to J/T..
(viii) The region of space with no magnetic field has no lines of
Circular current loop as a magnet : A small plane loop of current
force. At neutral point where resultant field is zero there
cannot be any line of force. behaves as a magnet with a definite dipole moment given by
uuur
SOME TERMS RELATED TO MAGNETISM M = I Anˆ
Magnetic poles : These are the regions of apparently concentrated
where A is the area of the loop, I the current in the loop and n̂ is
magnetic strength where the magnetic attraction is maximum.
It means that pole of a magnet is located not at a point but over a a unit vector perpendicular to the plane of the loop, and its direction
region. Magnetic poles exist in pairs. An isolated magnetic pole is decided by the sense of flow of current I using the Fleming’s
(north or south) does not exist. If a magnet is cut into two pieces, right hand rule.
then instead of obtaining separate N-pole and S-pole, each of the Relation between magnetic moment and angular momentum
two parts are found to behave as complete magnets.
Magnetic axis : The line passing through the poles of a magnet is uuur q uur
M = L
called its magnetic axis. 2m
Magnetic equator : The line passing through the centre of the
Where q is the total charge on a body of mass m rotaing about a
magnet and at right angles ot the magnetic axis is called the
fixed axis.
magnetic equator of the magnet.
Magnetism and Matter 545

Magnetic moment, pole strength and effective length when a magnet is cut
Magnet placed after Pole strength Effective length Magnetic moment
cutting after cutting

N S Breadth
m 2l M = 2ml
Length

N S m M
m/2 2l M1 = .2l =
X X´ 2 2

M
S N m l M 2 = ml =
2

.IN

Y AL
N S ml M
m/2 l M3 = =
X X´ 2 4
N
R


U

2l
m 2l M = m ´ 2l
JO

m m

COULOMB’S LAW OF MAGNETIC FORCE A magnet of dipole moment M suspended freely in a magnetic
U

It states that : field B experiences a torque t given by


ur uuur uur
(i) The force of attraction or repulsion between two magnetic
ED

t = M ´ B ; t = MB sin q
poles is directly proportional to the product of their pole uur ur
strengths. where q is the angle between M and B
r
(ii) The force of attraction or repulsion between two magnetic It is clear from the expression that | tmax |= MB
poles is inversely proportional to the square of the distance i.e., when dipole is perpendicular to field the torque is maximum
between them. This law is also known as inverse square law. and when they are parallel, the torque is minimum
r
1 m m m (for q = 0 or 180º ® | t | = 0 ).
i.e., F µ m1m2 and F µ r or, F = 0 . 1 2 The net force acting on a bar magnet placed
4p r 2 ® in a uniform magnetic field is zero
where m1 and m2 are the pole strengths of the two magnetic ® in a non-uniform magnetic field is non-zero
poles, r is the distance between them and m0 is the Let the length of a bar magnet be 2l and pole strength be m,
permeability of free space. the magnetic field is B, and the angle between B and bar
Unit magnetic pole : A unit magnetic pole may be defined as the magnet is q. Force on north pole is mB along the field and
pole which when placed in vacuum at a distance of one metre from that on south pole is mB opposite to the field.
an identical pole, repels it with a force of 10–7 newton. B mB
TORQUE ON A MAGNET IN A MAGNETIC FIELD N
q
m(North pole)
N mB
B O
Bar magnet at lsinq
q
an angle q. with S
S the magnetic field B
mB m (South pole) 2l
mB
EBD_7179
546 PHYSICS

The torque of these two forces about O is GAUSS'S LAW IN MAGNETISM


t = 2mBlsinq = MB sinq r
uur uur The surface integral of magnetic field B over a closed surface S
t = M ´B is always zero.
® ®
where M is the magnetic moment of the magnet. Mathematically Ñò B . da = 0
(Q M = 2ml) S
This torque tries to align the magnet with the field. (1) Isolated magnetic poles do not exist is a direct consequence
Work Done by External Agent in Rotating the Magnet of gauss's law in magnetism.
If an external agent rotates the magnet slowly, the agent has to (2) The total magnetic flux linked with a closed surface is always
exert a torque MBsinq opposite to that exerted by the field. zero.
Work done by the agent in changing the angle from q to q + dq is (3) If a number of magnetic field lines are leaving a closed
dW = (MBsinq) dq surface, an equal number of field lines must also be entering
the surface.
q
Magnetic Flux
Wext = ò (MB sin q)dq The magnetic flux through a given area may be defined as the
q0 total number of magnetic lines of force passing through this
area. It is equal to the product of the normal components of the
Wext = MB(cos q 0 - cos q)
magnetic field B and the area over which it is uniform. In general,
Wext is stored as potential energy of the field-magnet system. uur uuur
Magnetic flux, f = ò B .dA = ò BdA cos q , where q is angle

.IN
Thus
A A
U(q) - U(q0 ) = MB(cos q0 - cos q)
between normal to the area dA with magnetic field B.
If we take U(q 0 ) = 0 for q 0 = 90o , then
AL
Potential energy
uur uur
Magnetic flux linked with a closed surface is zero i.e., ò B .dA = 0
s
U (q) = U ( q) - U (90°) = - MB cos q = – M . B
N
(i) When q = 0, U = – MB (minimum PE) The S.I. Unit of magnetic flux is weber (Wb) : If a magnetic field of
1 tesla passes normally through a surface of area 1 square metre,
(ii) When q = 90º , U = 0
R

then the magnetic flux linked with this surface is said to be 1


(iii) When q = 180º , U = MB (maximum PE) weber.
U

M B Oscillations of a Bar Magnet in a Magnetic Field


A freely suspended magnet of magnetic moment M and of moment
JO

N B
of inertia I oscillates simple harmonically in a magnetic field B with
frequency
q = 90° S N
U

S M
ED

U = – MB (min)
U=0 q = 0° +m
q = 90° Stable equilibrium N mB
q
B B
mB S
–m
Freely suspended bar
N M S magnet, at an angle q
with the magnetic field B

1 MB I
n= , \ Time period , T = 2 p
U = + MB (max) 2p I MB
q = 180° MAGNETIC FIELD DUE TO A BAR MAGNET
Unstable equilibrium (i) Magnetic field intensity B1 due to a bar magnet at any point
Work done in Rotating a Uniform Magnetic Dipole
m0 2 Md
in a Magnetic Field on the axial line of the magnet is B1 =
4 p (d 2 - l 2 ) 2
Work done in deflecting the dipole through an angle q is,
W = MB (1 – cos q) B1
S N
If q = 0, cos q = 1 then W = MB (1 – 1) = 0
P
If q = 90°, cos q = 0 then W = MB
If q = 180°, cos q = –1, then W = 2MB 2l
d
Magnetism and Matter 547

where d = distance of the point from the centre of the magnet. MAGNETIC POTENTIAL
The direction of B1 is along SN. The magnetic potential at a point is defined as the work done in
m m carrying a unit N-pole from infinity to that point against the
BN = 0 field. It may also be defined as the quantity whose space rate of
4 p (d - l ) 2 where, m is pole strength.
variation in any direction gives the intensity of the magnetic field
BN is magnetic field due to north pole, it is directed away from the
magnet. dV B
i.e., B=-
m m dx
BS = 0
4p (d + l) 2 it is directed towards the magnet. (i) Magnetic potential due to a point dipole, at a distance r
from the pole of strength m is given by
m0m é 1 1 ù m m
\ B = BN - BS Þ B = ê - ú V B = 0 . (joule/Wb)
4 p êë (d - l) 2
(d + l) 2 úû 4p r
B due to a pole of Pole strength m at a distance r is given by
m 0 m (4ld ) m0 2 Md
= = [Q M = m(2L)]
4p ( d 2 - l 2 ) 2 4p ( d 2 - l 2 ) 2 m0 m
B=
4p r 2
0 m 2M
If d > > l, B = 4p 3 Now V0 at A is the work done is bringing a unit pole from
d
infinity to A.

.IN
(ii) Magnetic field intensity (B 2) due to A bar magnet at any
point on the equatorial line of the bar magnet is r
m 0m
\ VA - V¥ = ò a dr Þ V A = m 0m
m M 2
B2 = 0
AL ¥
4pr 4 pr
4 p ( d 2 + l 2 )3 / 2
The direction of B2 is along a line parallel to NS. (ii) Potential due to a magnetic dipole at a point in end-side on
N
m0 æ m ö
P position is VB = .ç ÷ , where M = 2ml.
R

B 4p è r 2 - l 2 ø
U

S N
d
O
JO

S r
N
U

2l
ED

m m
m m NP m m PS
If l2 < < r2 then VB = 0
BN = 0 , BS = 0 4p r 2
4p NP 3 4p PS3
(iii) Potential due to a magnetic pole at a point in the broad side-
Now, NP = PS = (d2 + l2 )3/ 2 on position. Net potential at P = 0. The potential at any point
uur uuur uuur lying on the magnetic equator of a magnet is zero in CGS
Resultant field at P is, B = BN + BS and MKS system.
m m m m P
ÞB = 0. ( NP + PS) = 0 . ( NS)
4p ( d 2 + l 2 ) 3 / 2 4 p (d 2 + l 2 ) 3 / 2
r
m 2ml m M
| B |= 0 = 0
4 p (d + l )
2 2 3 / 2 4p ( d + l 2 ) 3 / 2
2
N S
uur m 0 M
If d > > l, | B |= 4p 3 2l
d
(iv) The magnetic potential at a point lying on a line passing
The magnetic field at any point having polar coordinates (r, q) through the centre and making angle q with the axis
relative to centre of magnet or loop
m 0 M cos q
m0 M VB = and for small dipole, r >> l
B= (1 + 3 cos 2 q ) and direction is given by 4p (r 2 - l 2 )
4p r 3
1 m0 M cos q
tan a = tan q VB =
2 4p r2
EBD_7179
548 PHYSICS

Example 1. Example 5.
Two identical thin bar magnets each of length l and pole The time period of oscillation of a magnet in a vibration
strength m are placed at right angles to each other, with magnetometer is 1.5 sec. What will be the time period of
north pole of one touching south pole of the other, then oscillation of another magnet similar in size, shape and
find the magnetic moment of the system. mass but having 1/4 magnetic moment than that of the 1st
N1 magnet oscillating at the same place?
Solution :

T2 M1 M1
= = =2; \ T2 = 2 T1 = 3 s .
2 T1 M2 1
M1
4
Example 6.
S1 The time period of oscillation of a magnet is 2 sec. When it
N2 S2 is remagnetised so that its pole strength is 4 times, what
will be its period?
Solution : Solution :
Initial magnetic moment of each magnet = m × l.
As is clear from fig., S1 and N2 neutralize each other. T2 M1 m1 ´ 2 l T2 m1 1
= = = =
Effective distance between T1 M2 m 2 ´ 2 l ; T1 4 m1 2 ; T2 = 1 sec.

.IN
N1 and S2 = l 2 + l2 = l 2 \ M ¢ = ml 2 . Example 7.
Example 2.
AL A thin rectangular magnet suspended freely has a period
A steel wire of length l has a magnetic moment M. It is then of oscillation of 4s. If it is broken into two equal halves,
bent into a semi-circular arc. Find the new magnetic what will be the period of oscillation of each half ?
moment. Solution :
N
Solution : For each half, mass is half and length is half;
Let d be the diameter of semi-circle.
R

\ l = (p d/2) or d = (2 l/p) m l2
As M.I. = \ M.I. becomes 1/8th.
New magnetic moment = m × d = m × (2 l/p)
U

12
= 2m l/p = (2 M/p) Also M becomes 1/2
JO

Example 3.
Work done in turning a magnet of magnetic moment M by an As T = 2 p M.I. / MB
angle 90º from the magnetic meridian is n times the
corresponding work done to turn through an angle of 60º, 1/ 8 1
U

\ T becomes times = times


where n is 1/ 2 2
ED

(a) 1/2 (b) 2


(c) 1/4 (d) 1 1
New time period = ´ 4s = 2s
Solution : (b) 2
W1 = – MB (cos 90º – cos 0º) = MB EARTH’S MAGNETISM
W2 = – MB (cos 60º – cos 0º)
Magnetic field of earth extends nearly upto five times the radius
æ1 ö 1 1 of earth i.e., 3.2 × 104 km.
= - MB ç - 1÷ = MB = W1
è2 ø 2 2 The magnetic field of earth is fairly uniform and can be represented
As W1 = n W2 ; \ n =2 by equidistant parallel lines.
Example 4. NG
In a hydrogen atom, an electron revolves with a frequency Magnetic axis Sm
Magnetic equator
of 6.8 × 109 megahertz in an orbit of diameter 1.06 Å. What
will be the equivalent magnetic moment? Geographic equator
Solution : Earth's magnetic
n = 6.8 × 109 MHz = 6.8 × 1015 Hz, Imaginary lines of force
bar magnet in
1.06 the core of Earth
r= = 0.53 Å = 0.53 × 10–10 m
2 to depict earth's
magnetic field
æ e ö 2 2 SG Nm
M = IA = ç ÷pr = enpr
è 1/ n ø Geographic axis
22 Geographic meridian : The geographic meridian at a place is the
= (1.6 ´ 10 -19 ) (6.80 ´ 1015 ) ´ (0.53 ´10 -10 ) 2
7 vertical plane passing through the geographic north & south at
= 9.7 ´10 - 24 A m 2 that place.
Magnetism and Matter 549

Magnetic meridian : The magnetic meridian at a place is the vertical BV¢ BV


plane passing through the magnetic axis of a freely suspended tan d¢ = =
BH¢ BH cos q
small magnet. The earth’s magnetic field acts in the plane of
magnetic meridian. where d¢ = apparent dip
Magnetic meridian d = true dip
If the dip circle is rotated by 90°, the new apparent dip d¢¢ and d¢
S N
N Vertical plane and d are related as cot 2 d = cot 2 d¢ + cot 2 d¢¢
S (iii) Horizontal component of earth’s magnetic field (B H) : It is
passing through
the magnetic axis uur
the component of the earth’s total magnetic field B in the
horizontal diection and is given by, BH = B cos d
Elements of Earth’s Magnetic Field
The earth’s magnetic field at a place can be completely described Keep in Memory
by three independent parameters called elements of earth’s
magnetic field. These are : 1. Dip circle is an instrument used to measure angle of dip at a
Geographic
place.
BH
meridian a d 2. At poles total magnetic intensity is 0.66 oersted and at
–d
Bcos d d 90
° equator it is 0.33 oersted
BI Bv The total magnetic intensity at a particular latitude is

.IN
Magnetic
meridian B
Bv
I = I 0 1 + 3 sin 2 l
Magnetic
where l is the angle of latitude
meridian
AL
Magnetic 90°
meridian d=
N
Magnetic equator
S Horizontal
R

line
d 0° Magnetic axis
Axis of a freely d=
U

N suspended magnet
90°
JO

(i) Magnetic declination (q) : The angle between the d=


geographic meridian and the magnetic meridian at a place The angle of dip d = 0° at magnetic equator
is called the magnetic declination at that place. and d = 90° at magnetic poles.
U

(ii) Angle of dip (d) : The angle made by the earth’s total 3. A spectacular effect due to earth's magnetism is observed
uur near the magnetic poles of earth. This effect is called aurora-
magnetic field B with the horizontal is called angle of
ED

borealis in the north and aurora - austorlis in south. It is


dip at any place.
shown by patterns of coloured lights.
B
tan d = H
BV Magnetic Maps
True and apparent dip Magnetic surveys all over the earth have been carried out and
When the plane of the dip circle is in the magnetic meridian, then magnetic maps have been prepared which show the values of
the needle stops in the actual direction of the Earth's magnetic magnetic element throughout the world. Lines can be drawn to
field. The angle made by the needle with the horizontal in this join places having the same value of a particular magnetic element.
condition is called true dip. (i) Isogonic lines : These join places of equal declination. A
In case, the plane of the dip circle is not in the magnetic meridian line joining places of zero declination is called agonic line.
then the angle made by the needle with the horizontal is called
(ii) Isoclinic lines : These join places of equal dip.
apparent dip. In this case the vertical component of earth's magnetic
field remains the same but the effective horizontal component A line joining the places of zero-dip is called aclinic line.
B'H = BHcos q (iii) Isodynamic lines : These join places of equal horizontal
B'H component.
Plane in q Shielding from magnetic fields : For shielding a certain region of
which the dip space from magnetic field, we surround the region by soft iron
BH
circle is rings. Magnetic field lines will be drawn into the rings and the
present space enclosed will be free of magnetic field.
Neutral Points
Magnetic
meridian Neutral points are the points where the net field intensity due to
the field of the bar magnet and field of earth is zero. When magnet
EBD_7179
550 PHYSICS

is placed with its north pole towards geographic north, neutral Example 9.
points lie on equatorial line of the magnet. At each neutral point,
If q1 and q2 are angles of dip in two vertical planes at
m0 M
B2 = =H right angle to each other and q is true dip then prove
4p (d 2 + l 2 )3/ 2
cot2q = cot2q1 + cot2q2.
where H = horizontal component of earth’s magnetic field.
Solution :
When the bar magnet is placed with its north pole towards
If the vertical plane in which dip is q1 subtends an angle a
geographic south, the neutral points lie on the axial line of the
with meridian than other vertical plane in which dip is q2 and
magnet. At each neutral point,
is perpendicular to first will make an angle of 90° – a with
m M
B1 = 0 =H magnetic meridian. If q1 and q2 are apparent dips then
4p ( d - l 2 ) 2
2

Relation between the units of quantities associated with magnetic BV BV BV


tan q1 = ; tan q 2 = =
field : BH cos a BH cos(90 - a ) B H sin a
1A = 1JT–1 m–2 = 1J Wb–1 1 1
1T = 1JA–1 m–2 = 1Wb m–2 cot 2 q1 + cot 2 q 2 = +
2
(tan q1 ) (tan q 2 ) 2
1Wb = 1JA–1 = 1Tm2
[B] = NA–1 m–1 = T = Wb m–2 2
B2H cos 2 a + B2H sin 2 a B2H æ B cos q ö
[M] = A m2 = JT–1 = J m2 Wb–1 = = =ç = cot 2 q
B2V B2V è B sin q ÷ø
[m0] = NA–2 = T2 m2 N–1 = Wb2 J–1 m–1

.IN
Example 8. s o cot2q1 + cot2q2 = cot2q
A circular coil of radius 0.157 m has 50 turns. It is placed TERMS RELATED TO MAGNETISM
AL
such that its axis is in magnetic meridian. A dip needle is
supported at the centre of the coil with its axis of rotation Magnetic intensity (H ) : When a magnetic material is placed in
horizontal and in the plane of the coil. The angle of dip is
a magnetic field, it becomes magnetised. The capability of the
N
30º, when a current flows through the coil. The angle of
dip becomes 60º on reversing the current. Find the current magnetic field to magnetise a material is expressed by means of
uur
R

in the coil assuming that magnetic field due to the coil is a magnetic vector H , called the `magnetic intensity’ of the field.
smaller than the horizontal component of earth’s field. Take
U

H = 3 × 10–5 T. The relation between magnetic induction B and magnetising field


JO

Solution : B
H H = , m being permeability of medium.
If H is horizontal component and V is vertical component at m
the place, then true value of dip (d) at the place is given by, Intensity of magnetisation (I) : When a material is placed in a
U

V magnetising field, it acquires magnetic moment M. The intensity


tan d = of magnetisation is defined as the magnetic moment per unit
H
ED

If B is magnetic field intensity at the centre of coil due to current M


volume i.e., I m =
V
V
I in circular coil and B is along H, then tan 30º = . V being volume of mateiral. If the material is in the form of a bar of
H+B cross-sectional area A, length 2l and pole strength m, then
On reversing the direction of current, the direction of B is
reversed. M m.2l m
M = m × 2l; V = A × 2l \
\ Im = = =
V V A.2 l A
\ tan 60º = Magnetic susceptibility (c) : The magnetic susceptibility is defined
H-B
as the intensity of magnetisation per unit magentising field.
tan 60º H + B 3
Dividing, we get = = =3 Im
tan 30º H - B 1 / 3
i.e., c =
H
3 ´ 10 -5
\ B = H/2 Þ B = = 1.5 ´ 10 -5 T Magnetic permeability (m) : The magnetic permeability of a material
2 is the measure of degree to which the material can be permited by
m o nI a magnetic field and is defined as the ratio of magnetic induction
For a circular coil, B =
2r B
-7 (B) in the material to the magnetising field i.e. m =
\ 1.5 × 10–5 = (4 p ´ 10 ) ´ 50 ´ I H
2 ´ 0.157
Relation between Magnetic Susceptibility and
\ I= 1.5 ´ 10 -5 ´ 2 ´ 0.157 Permeability
= 0.75 A .
4p ´ 10 - 7 ´ 50 We have magnetic induction in mateiral, B = mH
Also B = B0 + Bm
Magnetism and Matter 551

where B0 = magnetic induction in vacuum produced by The characteristics of paramagnetic substances are
magnetising field (a) They are attracted by a strong magnet
Bm = magnetic induction due to magnetisation of material. (b) Their susceptibility is positive but very small ( c > 0)
But B0 = m0H and Bm = m0 I m Þ B = m 0 [H + I m ] (c) Their relative permeability is slightly greater than unity.
(m > 1)
Im
\ B = m 0 H [1 + ] = B0 [1 + c] ; \ B / B0 = [1 + c] (d) Their susceptibility and permeability do not change
H with the variation of magnetising field.
mH (e) Their susceptibility is inversely proportional to
\ B / B0 = = m / m0 = µr, the relative magnetic permeability
temperature, æç i.e. c a ö÷ .
m0 H 1
è Tø
\ m r =1 + c (f) They are found in those material which have atoms
This is required relation. containing odd number of electrons
3. Ferromagnetic Substances : These are the substances which
CLASSIFICATION OF MATERIALS are strongly magnetised by relatively weak magnetising
According to the behaviour of substances in magnetic field, they field in the same sense as the magnetising field. The
are classified into three categories: examples are Ni, Co, iron and their alloys.
1. Diamagnetic Substances : These are the substances which The characteristics of ferromagnetic substances are
when placed in a strong magnetic field acquire a feeble (a) They are attracted even by a weak magnet.

.IN
magnetism opposite to the direction of magnetising field. (b) The susceptibility is very large and positive.
The examples are copper, gold, antimony, bismuth, alcohol, ( c >> 0)
water, quartz, hydrogen, etc.
(c) The relative permeability is very high (of the order of
AL
hundreds and thousands). (m >> 1)
N S (d) The intensity of magnetisation is proportional to the
magnetising field H for smaller values, varies rapidly
N
N S
for moderate values and attains a constant value for
R

larger values of H.
N S (e) The susceptibility of a ferromagnetic substance is
U

Bo
inversely proportional to temperature i.e., c µ1/ T
Diamagnetic
JO

C
B<B0 Þ c = ; C = curie constant .
T
This is called Curie law. At a temperature called curie
r
U

temperature, ferromagnetic substance becomes


Behaviour of diamagnetic substance in an external magnetic field Bo paramagnetic. The curie temperatures for Ni, Fe and
ED

The characteristics of diamagnetic substances are Co are 360ºC, 740ºC and 1100ºC respectively.
(a) They are feebly repelled by a strong magnet (f) They are found in those material which have domains
(b) Their susceptibility is negative (i.e. c < 0) and can be converted into strong magnets
(c) Their relative permeability is less than 1. (i.e. mr < 1) Keep in Memory
(d) Their susceptibility is independent of magnetising field
and temperature (except for Bismuth at low 1. Diamagnetism is universal. It is present in all materials. But
it is weak and hard to detect if substance is para or
temperature)
feromagnetic.
2. Paramagnetic Substances : These are the materials which 2. I – H curve for different materials
when placed in a strong magnetic field acquire a feeble
magnetism in the same sense as the applied magnetic field. I Ferromagnetic
The examples are platinum, aluminium, chromium, Paramagnetic
manganese, CuSO4, O2, air, etc.
D iama
g netic H
N S
N S 3. Curve for magnetic susceptibility and temperature for a
paramagnetic and ferromagnetic material.
X
N S
Bo 1
Paramagnetic cµ
B > B0 T

r T
Behaviour of paramagnetic substance in an external field Bo
EBD_7179
552 PHYSICS

HYSTERESIS Keep in Memory


When a bar of ferromagnetic material is magnetised by a varying
magnetic field and the intensity of magnetisation I m induced is 1. By alloying soft-iron with 4% silicon ‘transformer steel’ is
measured for different values of magnetising field H, the graph of produced. It has a higher relative permeability and is an ideal
I versus H is as shown in fig material for cores of transformers. Alloys of iron and nickel
called ‘permalloys’, also have very large permeabilities.
2. Energy spent per unit volume of specimen is complete cycle
Im
of magnetisation is numerically equal to area of I – H loop
B A
Perme- Suscep- Intensity of Reten- Coerc- Hysteresis
ability tibility magnetisation tivity ivity loss
C O G H Soft iron high high high high low low
Steel low low low low high high
F
D Þ Steel is most suitable for making parmanent magnet
The graph shows : Þ Soft iron is most suitable for making core of an
(i) When magnetising field is increased from O the intensity of electromagnet.
magnetisation I m increases and becomes maximum (i.e.
point A). This maximum value is called the saturation value. Example 10.
(ii) When H is reduced, I m reduces but is not zero when H = 0. A magnetising field of 1600 Am–1 produces a magnetic
flux of 2.4 × 10–5 weber in a bar of iron of cross section 0.2

.IN
The remainder value OB of magnetisation when H = 0 is
called the residual magnetism or retentivity. OB is retentivity. cm2. Calculate permeability and susceptibility of the bar.
(iii) When magnetic field H is reversed, the magnetisaiton Solution :
decreases and for a particular value of H, it becomes zero Here, H = 1600 Am–1, f = 2.4 ×10–5 Wb.
AL
i.e., for H = OC, I = 0. This value of H is called the coercivity. A = 0.2 cm2 = 0.2 × 10–4 m2, m = ? cm = ?
(iv) When field H is further increased in reverse direction, the
intensity of magnetisation attains saturation value in reverse f 2.4 ´ 10-5
N
direction (i.e., point D). B= = = 1.2 weber / m 2 ;
A 0.2 ´ 10-4
(v) When H is decreased to zero and changed direction in steps,
R

we get the part DFGA. B 1. 2


m= = = 7.5 ´ 10 - 4 TA -1m ;
U

Properties of Soft Iron and Steel H 1600


For soft iron, the susceptibility, permeability and retentivity are [Q As m = m 0 (1 + c m ) ]
JO

greater while coercivity and hysteresis loss per cycle are smaller
than those of steel. m 7.5 ´10 -4
\ cm = -1 = -1 ;
B B m0 4p ´10 -7
U

Hysteresis
curve 7.5 ´103
ED

cm = - 1 = 597.1 - 1 = 596.1
H 4p
Hysteresis H
curve Soft magnetic Hard magnetic Example 11.
material material A solenoid of 500 turns/m is carrying a current of 3A. Its
core is made of iron which has a relative permeability of
PERMANENT MAGNETS AND ELECTROMAGNETS
5000. Determine the magnitude of magnetic intensity,
Permanent magnets are made of steel and cobalt while
magnetisation and magnetic field inside the core.
electromagnets are made of soft iron.
Solution :
An electromagnet is made by inserting a soft iron core into the
interior of a solenoid. Soft iron does not retain a significant Magnetic intensity
permanent magnetization when the solenoid’s field is turned off– H = ni = 500 × 3 = 1500 A/m
soft iron does not make a good permanent magnet. When current µr = 1 + cm so cm = µr – 1 = 4999 » 5000
flows in the solenoid, magnetic dipoles in the iron tend to line up Intensity of magnetisation
with the field due to the solenoid. The net effect is that the field I = cH = 5000 × 1500 = 7.5 × 106 A/m
inside the iron is intensified by a factor known as the relative Magnetic field B = µr µ0 H = 5000 × 4p × 10–7 × 1500
permeability. The relative permeability is analogous in magnetism
to the dielectric constant in electricity. However, the dielectric = 9.4 tesla.
constant is the factor by which the electric field is weakened, TANGENT GALVANOMETER
while the relative permeability is the factor by which the magnetic It is an instrument used for measuring small current. It is based on
field is strengthened. The reactive permeability of a ferromagnet
tangent law. It is a moving magnet and fixed coil type galvanometer.
can be in the hundreds or even thousands–the intensification of
the magnetic field is significant. Not only that, but in an Tangent Law : If a small magnetic needle is under the influence
electromagnet the strength and even direction of the magnetic of two crossed magnetic fields (B) and (H) and suffers a deflection
field can be changed by changing the current in the solenoid. q from field H, then by tangent law, B = H tan q.
Magnetism and Matter 553

Formula for current : If a current passing through the coil of n and net M.Ι ¢ = M.Ι + M.Ι = 2M.Ι
turns and mean radius r of a tangent galvanometer placed in
magnetic meridian causes a deflection q in the magnetic needle 2 M.I. 2 M.I.
kept at the centre of the coil, then \ T ¢ = 2p = 2´ ....(1)
M 2H MH
æ 2rH ö 2rH When one of the magnets is taken away,
I=ç tan q ; I = K tanq where K = and is called the
è m 0 n ÷ø m0 n M² = M, MI² = MI,
reduction factor. M.Ι
\ T¢¢ = 2p ...(2)
MH
DEFLECTION MAGNETOMETER
Divide eqn. (2) by (1),
It's working is based on the principle of tangent law.
(a) Tan A Position : In this position the magneto- meter is set T ¢¢ 1 T¢ 4
= or T ¢¢ = = = 3 .34 s
perpendicular to magnetic meridian so that, magnetic field T ¢ ( 2)1 / 4 ( 2) 1/ 4
2 1/ 4
due to magnet is in axial position and perpendicular to earth’s
field and hence Examples 13.
The magnetic needle of an oscillation magnetometer makes
m0 2Md
= H tan q 10 oscillations per minute under the action of earth’s
4p (d 2 - l2 )2 magnetic field alone. When a bar magnet is placed at some
where d = distance of needle from centre of magnet and distance along the axis of the needle, it makes 14

.IN
2l = length of magnet. oscillations per minute. If the bar magnet is turned so that
(b) TanB position : The arms of magnetometer are set in its poles interchange their positions, then what will be the
magnetic meridian, so that the field is at equatorial position new frequency of oscillation of the needle?
AL
m0 M 1 MH
and hence, H tan q = Sol. 10 = ...(i)
4p ( d 2 - l 2 )3 / 2 2p I
N

Magnetic field of earth extends nearly upto five times the


1 M (H + F)
R

radius of earth i.e., 3.2 × 104 km. 14 = ...(ii)


2p I
U

The magnetic field of earth is fairly uniform and can be


represented by equidistant parallel lines.
1 M (H - F)
JO

n= ...(iii)
VIBRATION MAGNETOMETER 2p I
It is an instrument for comparing the magnetic moments of two
14 H+F F 7
U

magnets and for comparing their magnetic fields. Divide eqs. (ii) by (i), = = 1+ =
10 H H 5
The time period of a bar magnet vibrating in the vibration
ED

magnetometer kept in magnetic meridian is given by F 24


\ =
4p I2 H 25
I
T = 2p \ M = n H-F F 1
MH T 2H Divide eqs. (iii) by (i), = = 1- =
10 H H 5
æ m(l 2 + b2 ) ö
where I = ç 10
÷ is the moment of inertia of the vibrating or, n= = 2 vibs /minute.
è 12 ø 5
magnet, m = mass of magnet, l = length of magnet, b = breadth of Example 14.
magnet. The period of oscillation of a magnet in a vibration
Example 12. magnetometer is 2 sec. What will be the period of oscillation
A vibration magnetometer consists of two identical bar of a magnet whose magnetic moment is four times that of
magnets, placed one over the other, such that they are the first magnet?
mutually perpendicular and bisect each other. The time
æ Ι ö
period of oscillation in a horizontal magnetic field is 4 Sol. T = 2p ç
second. If one of the magnets is taken away, find the period è M BH ÷ø
of oscillation of the other in the same field.
æ Ι ö 1é Ι ù
Sol. For a vibration magnetometer, we know that T = 2p I / MH T ' = 2p ç ÷ = ê2p ú
è 4M BH ø 2 ë (M BH ) û
Let M be the magnetic moment and M.I, moment of inertia
of each magnet, 1
= ´ 2 = 1 second.
\ M¢ = M2 + M2 = M 2 2
EBD_7179
554 PHYSICS

Example 15.
A magnet is suspended in such a way that it oscillates in æ Ι ö
T1 = 3 = 2p ç
the horizontal plane. It makes 20 oscillations per minute è M B1 cos 30 ÷ø
at a place where dip angle is 30º and 15 oscillations per
minute at a place where dip angle is 60º. What will be the
æ Ι ö
ratio of the total earth’s magnetic field at the two places? and T2 = 4 = 2p ç
Sol. Let the total magnetic fields due to earth at the two places è M B2 cos 60º ÷ø
be B1 and B2. If horizontal components be (BH)1 and (BH)2
respectively, then 1/ 2
3 æ B 2 cos 60 ö B1 16 cos 60
\ =ç ÷ or = ´
(BH)1 = B1 cos 30º and (BH)2 = B2 cos 60º 4 çè B1 cos 30 ÷ø B2 9 cos 30
Here T1 = 3 sec. and T2 = 4 sec.
B1 16 1 2 16
or = ´ ´ = or B1 : B2 = 16 : 9 3
B2 9 2 3 9 3

.IN
AL
N
R
U
JO
U
ED
CONCEPT MAP

Directive
Magnet field due A freely suspended magnet always
to a bar magnet points in north-south direction
Force between two
Magnetism and Matter

magnetic poles
At a point on axial At a point on equatorial
m 2M m M m 0 m1m2
line B = 0 3 line B = 0 2 2 F=
4p d 4p (r + l )3 / 2 4p r 2

Properties of magnet Pole of a magnet


ED
Repulsive Like poles always exist in pair
always repel one U
another. It is sure Magnetism Property of Magnetic dipole
test of magnet attracting a piece of iron, moment M = NIA
cobalt, nickel or steel
JO
U Angle of dip or
inclination ()d
Magnetic Magnetic Magnetic R Angle made by
intensity permeability susceptibility MAGNETISM AND Earth's magnetic
direction of earth's
M MATTER N elements
B µ = µ0(1 + )cm I magnetic field
H= 0 cm = I=
m H V with the horizontal
dequator = 0; dpole = 90º
Paramagnetics Magnetist Magnetic field lines
AL
in the direction of magnetic Imaginary lines in a Horizontal component
field e.g., Al, Mn µr, I, c>m1 magnetic field which In a uniform magnetic BV= B sin q
.IN
Magnetic continuously represent field time period of
Materials the direction of magnetic oscillation of a freely BH= B cos q
Ferromagnetics Strongly
magnetised in the direction field suspended magnet
of magnetic field e.g., Fe, Co, I Angle of declination
Ni µr, I,c>> T = 2p Angle between magnetic
m 1 MB
Properties of magnetic meridian and geographic
field lines meridian

Diamagnetics Magnetised Form continuous closed


in a direction opposite to the loops start from N-pole Tangent to the field
direction of magnetic field end S-pole outside the Come out of Magnetic field line at a given point
e.g., Bi, Cu, Hg µr, I and cm magnet and its opposite surface at any lines do not intersect represents the direction
555

are negative inside the magnet angle each other of the net magnetic field
EBD_7179
556 PHYSICS

1. The main difference between electric lines of force and (a) 2 : 1 (b) 1 : 2
magnetic lines of force is (c) 1 : 1 (d) None of these
(a) electric lines of force are closed curves whereas magnetic 11. A watch glass containing some powdered substance is
lines of force are open curves placed between the pole pieces of a magnet. Deep concavity
(b) electric lines of force are open curves whereas magnetic is observed at the centre. The substance in the watch glass
lines of force are closed curves is
(c) magnetic lines of force cut each other whereas electric (a) iron (b) chromium
lines of force do not cut (c) carbon (d) wood
(d) electric lines of force cut each other whereas magnetic 12. Needles N1, N2 and N3 are made of a ferromagnetic, a
lines of force do not cut paramagnetic and a diamagnetic substance respectively. A
2. Current i is flowing in a coil of area A and number of turns N, magnet when brought close to them will
then magnetic moment of the coil, M is (a) attract N1 and N2 strongly but repel N3
(b) attract N1 strongly, N2 weakly and repel N3 weakly
Ni Ni (c) attract N1 strongly, but repel N2 and N3 weakly
(a) NiA (b) (c) (d) N2Ai

.IN
A A (d) attract all three of them
3. Nickel shows ferromagnetic property at room temperature. If 13. A bar magnet is oscillating in the Earth’s magnetic field
the temperature is increased beyond Curie temperature, then
AL with a period T. What happens to its period and motion if
it will show its mass is quadrupled?
(a) anti ferromagnetism (b) no magnetic property (a) Motion remains S.H. and period remains nearly
(c) diamagnetism (d) paramagnetism constant
N
4. The line on the earth surface joining the point where the field T
(b) Motion remains S.H. with time period =
2
R

is horizontal, is called
(a) magnetic equator (b) magnetic line (c) Motion remains S.H. with time period = 2T
(d) Motion remains S.H. with time period = 4T
U

(c) magnetic axis (d) magnetic inertia


5. When a ferromagnetic material is heated to temperature above 14. Two magnets of magnetic moments M and 2M are placed
JO

its Curie temperature, the material in a vibration magnetometer, with the identical poles in the
same direction. The time period of vibration is T1. If the
(a) is permanently magnetized
magnets are placed with opposite poles together and vibrate
(b) remains ferromagnetic
U

with time period T2, then


(c) behaves like a diamagnetic material
(a) T2 is infinite (b) T2 = T1
(d) behaves like a paramagnetic material
ED

6. The force between two short bar magnets with magnetic (c) T2 > T1 (d) T2 < T1
moments M1 and M2 whose centres are r metres apart is 8 N
when their axes are in same line. if the separation is increased 15. If horizontal and vertical components of earths magnetic
to 2 r, the force between them in reduced to field are equal, then angle of dip is
(a) 4 N (b) 2 N (c) 1 N (d) 0.5 N (a) 60° (b) 45° (c) 30° (d) 90°
7. The magnet of pole strength m and magnetic moment M is 16. The magnetic materials having negative magnetic
cut into two pieces along its axis. Its pole strength and susceptibility are
magnetic moment now becomes (a) non-magnetic (b) para magnetic
(c) dia magnetic (d) ferro magnetic
m M M m 17. For protecting a sensitive equipment from the external
(a) , (b) m, (c) , M (d) m, M
2 2 2 2 electric arc, it should be
8. A bar magnet of magnetic moment M and length L is cut (a) wrapped with insulation around it when passing
into two equal parts each of length L/2. The magnetic moment current through it
of each part will be (b) placed inside an iron can
(c) surrounded with fine copper sheet
(a) M (b) M/4 (c) 2M (d) M/2 (d) placed inside an aluminium can
9. A superconductor exhibits perfect : 18. If a diamagnetic substance is brought near north or south
(a) ferrimagnetism (b) ferromagnetism pole of a bar magnet, it is
(c) paramagnetism (d) diamagnetism (a) attracted by the poles
10. In end on and broadside on position of a deflection (b) repelled by the poles
magnetometer, if q1 and q2 are the deflections produced by (c) repelled by north pole and attracted by the south pole
(d) attracted by the north pole and repelled by the south
short magnets at equal distances, then tan q1/ tanq2 is
pole
Magnetism and Matter 557

19. A bar magnet, of magnetic moment M , is placed in a (c) unity


magnetic field of induction B . The torque exerted on it is (d) between zero and one
24. There are four light-weight-rod samples A,B,C,D separately
(a) M.B (b) – M . B (c) M ´ B (d) B´M suspended by threads. A bar magnet is slowly brought near
20. Current i is flowing in a coil of area A and number of turns each sample and the following observations are noted
N, then magnetic moment of the coil M is (i) A is feebly repelled
Ni Ni (ii) B is feebly attracted
(a) NiA (b) (c) (d) N2Ai (iii) C is strongly attracted
A A
21. A diamagnetic material in a magnetic field moves (iv) D remains unaffected
(a) perpendicular to the field Which one of the following is true ?
(b) from stronger to the weaker parts of the field (a) B is of a paramagnetic material
(c) from weaker to the stronger parts of the field (b) C is of a diamagnetic material
(d) None of these (c) D is of a ferromagnetic material
22. According to Curie’s law, the magnetic susceptibility of a (d) A is of a non-magnetic material
substance at an absolute temperature T is proportional to
25. If the magnetic dipole moment of an atom of diamagnetic
1 1 material, paramagnetic material and ferromagnetic material
(a) T 2 (b) (c) T (d) are denoted by md, mp and mf respectively, then
T2

.IN
T
(a) md = 0 and mp ¹ 0 (b) md ¹ 0 and mp = 0
23. The magnetic moment of a diamagnetic atom is
(a) equal to zero (c) mp = 0 and mf ¹ 0
AL (d) md ¹ 0 and mf ¹ 0
(b) much greater than one
N
R
U

1. A bar magnet 8 cms long is placed in the magnetic merdian (a) 1 × 10–5 N m (b) 1.5 × 10–5 N m
JO

with the N-pole pointing towards geographical north. Two –5


(c) 2 × 10 N m (d) 2.5 × 10–5 N m
netural points separated by a distance of 6 cms are obtained 6. When 2 ampere current is passed through a tangent
on the equatorial axis of the magnet. If horizontal component
galvanometer, it gives a deflection of 30º. For 60º deflection,
U

of earth’s field = 3.2 × 10–5 T, then pole strength of magnet


the current must be
is
ED

(a) 5 ab-amp × c (b) 10 ab-amp × cm (a) 1 amp. (b) 2 3 amp.


(c) 2.5 ab-amp × cm (d) 20 ab-amp × cm
2. Two tangent galvanometers having coils of the same radius (c) 4 amp. (d) 6 amp.
are connected in series. A current flowing in them produces 7. A curve between magnetic moment and temperature of
deflections of 60º and 45º respectively. The ratio of the number magnet is
of turns in the coils is
3 +1 3 +1 3 (a) M (b) M
(a) 4/3 (b) (c) (d)
1 3 -1 1
3. Two identical magnetic dipoles of magnetic moments
1.0 A-m2 each, placed at a separation of 2 m with their axis
perpendicular to each other. The resultant magnetic field at
point midway between the dipole is T T
O O
(a) 5 × 10–7 T (b) 5 × 10–7 T
(c) 10 T–7 (d) 2 × 10–7 T
4. Two isolated point poles of strength 30 A-m and 60 A-m are (c) M (d) M
placed at a distance of 0.3m. The force of repulsion is
(a) 2 × 10–3 N (b) 2 × 10–4 N
(c) 2 × 10 N 5 (d) 2 × 10–5 N
5. The magnetic moment of a magnet is 0.1 amp × m2. It is
suspended in a magnetic field of intensity 3 × 10–4 weber/m2. T T
The couple acting upon it when deflected by 30º from the O O
magnetic field is
EBD_7179
558 PHYSICS

8. The variation of magnetic susceptibility (x) with temperature 12. A thin bar magnet of length 2 l and breadth 2 b pole strength
for a diamagnetic substance is best represented by m and magnetic moment M is divided into four equal parts
with length and breadth of each part being half of original
(a) (b) magnet. Then the pole strength of each part is
(a) m (b) m/2 (c) 2 m (d) m/4
13. In the above question, magnetic moment of each part is
O T O T (a) M/4 (b) M (c) M/2 (d) 2 M
14. Two points A and B are situated at a distance x and 2x
respectively from the nearer pole of a magnet 2 cm long. The
ratio of magnetic field at A and B is
(c) (d) (a) 4 : 1 exactly (b) 4 : 1 approximately
(c) 8 : 1 approximately (d) 1 : 1 approximately
15. If a magnet is suspended at angle 30º to the magnetic meridian,
the dip needle makes an angle of 45º with the horizontal.
The real dip is
T O T
O (a) tan -1 ( 3 / 2 ) (b) tan -1 ( 3 )
9. At a temperatur of 30°C, the susceptibility of a ferromagnetic
(c) tan -1 ( 3 / 2) (d) tan -1 (2 / 3 )

.IN
material is found to be c . Its susceptibility at 333°C is
(a) c (b) 0.5 c (c) 2 c (d) 11.1 c 16. Two bar magnets of the same mass, same length and breadth
but having magnetic moments M and 2M are joined together
10. Of the following fig., the lines of magnetic induction due to
AL pole for pole and suspended by a string. The time period of
a magnet SN, are given by
assembly in a magnetic field of strength H is 3 seconds. If
now the polarity of one of the magnets is reversed and
N
N N
combination is again made to oscillate in the same field, the
time of oscillation is
R

(a) 3 sec (b) 3 3 sec


U

(c) 3 sec (d) 6 sec


JO

17. A compass needle placed at a distance r from a short magnet


S S in Tan A position shows a deflection of 60º. If the distance is
(1) (2) increased to r (3)1/3, then deflection of compass needle is
U

N N 1
ED

(a) 30º (b) 60´ 3 3


2 3
(c) 60´ 3 3 (d) 60´ 3 3
18. Two short magnets have equal pole strengths but one is
S S twice as long as the other. The shorter magnet is placed 20
(3) (4)
cms in tan A position from the compass needle. The longer
(a) 1 (b) 2 (c) 3 (d) 4 magnet must be placed on the other side of the magnetometer
11. The B – H curve (i) and (ii) shown in fig. associated with for no deflection at a distance equal to
(a) 20 cms (b) 20 (2)1/3 cms
B
(i) (c) 20 (2)2/3 cms (d) 20 (2)3/3 cms
(ii) 19. A dip needle lies initially in the magnetic meridian when it
shows an angle of dip q at a place. The dip circle is rotated
H through an angle x in the horizontal plane and then it shows
an angle of dip q'.
tan q '
Then is
(a) (i) diamagnetic and (ii) paramagnetic substance tan q
(b) (i) paramagnetic and (ii) ferromagnetic substance
1 1 1
(c) (i) soft iron and (ii) steel respectively (a) (b) (c) (d) cos x
cos x sin x tan x
(d) (i) steel and (ii) soft iron respectively
Magnetism and Matter 559

20. A dip circle is so set that its needle moves freely in the 29. The net magnetic moment of two identical magnets each of
magnetic meridian. In this position, the angle of dip is 40º. magnetic moment M0, inclined at 60° with each other is
(a) M 0 N
Now the dip circle is rotated so that the plane in which the
needle moves makes an angle of 30º with the magnetic
(b) 2 M0
meridian. In this position, the needle will dip by an angle
(a) 40º (b) 30º (c) 3 M0
60°
(c) more than 40º (d) less than 40º (d) 2M0 S N
21. Work done in turning a magnet of magnetic moment M by 30. A magnetic needle vibrates in a vertical plane parallel to the
an angle of 90º from the mgnetic meridian is n times the magnetic meridian about a horizontal axis passing through
corresponding work done to turn through an angle of 60º, its centre. Its frequency is n. If the plane of oscillation is
where n is turned about a vertical axis by 90°C, the frequency of its
oscillation in vertical plane will be
(a) 1/2 (b) 2 (c) 1/4 (d) 1
(a) n (b) zero
22. Two magnets are held together in a vibration magnetometer (c) less than n (d) more than n
and are allowed to oscillate in the earth’s magnetic field with 31. A thin rectangular magnet suspended freely has a period of
like poles together. 12 oscillations per minute are made but oscillation of 4 s. If it is broken into two halves (each having
for unlike poles together only 4 oscillations per minute are half the original length) and one of the pieces is suspended
executed. The ratio of their magnetic moments is similarly. The period of its oscillation will be

.IN
(a) 3 : 1 (b) 1 : 3 (c) 3 : 5 (d) 5 : 4 (a) 4 s (b) 2 s
(c) 0.5 s (d) 0.25 s
23. At a certain place, horizontal component is 3 times the
32. A steel wire of length l has a magnetic moment M. It is bent
AL
vertical component. The angle of dip at this place is in L-shape (Figure). The new magnetic moment is
(a) 0 (b) p/3 (a) M
(c) p/6 (d) None of these
M
N
24. A freely suspended magnet oscillates with period T in earth’s (b) l
horizontal magnetic field. When a bar magnet is brought 2 2
R

near it, such that the magnetic field created by bar magnet is
M
l
U

in same direction as earth’s horizontal magnetic field, the (c)


2 2
T
JO

period decreases to . The ratio of the field of the magnet (d) 2M


2
F to the earth’s magnetic field (H) is 33. The time period of oscillation of a magnet in a vibration
(a) 1 : 3 (b) 1 : 1 (c) 3 : 1 (d) 9 : 1 magnetometer is 1.5 sec. The time period of oscillation of
U

25. If relative permeability of iron is 2000. Its absolute another magnet similar in size, shape and mass but having
permeability in S.I. units is 1/4 magnetic moment than that of the 1st magnet oscillating
ED

(a) 8p × 10–4 (b) 8p × 10–3 at the same place will be


(c) 800/p (d) 8p × 109/p (a) 0.75 sec (b) 1.5 sec
26. A steel wire of length l has a magnetic moment M. It is then (c) 3.0 sec (d) 6.0 sec
bent into a semicircular arc. The new magnetic moment is 34. Time periods of vibation of two bar magnets in sum and
M 2M 3M 4M difference positions are 4 sec and 6 sec respectively. The
(a) (b) (c) (d)
p p p p ratio of their magnetic moments M1 / M2 is
27. The magnetic moment of atomic neon is equal to (a) 6 : 4 (b) 30 : 16
1 3 (c) 2 . 6 : 1 (d) 1 . 5 : 1
(a) zero (b) µB (c) µB (d) µB 35. Horizontal component of earth's field at a height of 1 m from
2 2
the surface of earth is H. Its value at a height of 10 m from
28. Torques t1 and t 2 are required for a magnetic needle to
surface of earth is
remain perpendicular to the magnetic fields at two different
places. The magnetic fields at those places are B1 and B2 (a) H/10 (b) H/9
(c) H/100 (d) H
B1
respectively; then ratio is 36. If a toroid uses bismuth for its core, the field in the core
B2
compared to that in empty core will be slightly
t2 t1 (a) greater (b) smaller
(a) (b)
t1 t2 (c) equal (d) None of these
37. The relative permeability of a medium is 0.075. What is its
t1 + t 2 t1 - t 2 magnetic susceptibility?
(c) (d)
t1 - t 2 t1 + t 2 (a) 0.925 (b) – 0.925 (c) 1.075 (d) –1.075
EBD_7179
560 PHYSICS

38. Relative permittivity and permeability of a material e r and


P
m r , respectively. Which of the following values of these D
quantities are allowed for a diamagnetic material?
(a) e r = 0.5, m r = 1.5 (b) e r = 1.5, m r = 0.5
O
(c) e r = 0.5, m r = 0.5 (d) e r = 1.5, m r = 1.5 S N N S
39. The moment of a magnet (15 cm × 2 cm × 1 cm) is 1.2 A-m2.
What is its intensity of magnetisation? d
(a) 4 × 104 A m–1 (b) 2 × 104 A m–1
(c) 104 A m–1 (d) None of these The force on the charge Q is
40. The work done in turning a magnet of magnetic moment M (a) directed perpendicular to the plane of paper
by an angle of 90° from the meridian, is n times the (b) zero
corresponding work done to turn it through an angle of
60°. The value of n is given by (c) directed along OP

(a) 2 (b) 1 (c) 0.5 (d) 0.25 (d) directed along PO


41. At a certain place, the angle of dip is 30º and the horizontal 46. A short bar magnet of magnetic moment 0.4J T–1 is placed in

.IN
component of earth’s magnetic field is 0.50 oerested. The a uniform magnetic field of 0.16 T. The magnet is in stable
equilibrium when the potential energy is
earth’s total magnetic field (in oerested) is
(a) – 0.064 J (b) zero
1 1
AL
(a) 3 (b) 1 (c) (d) (c) – 0.082 J (d) 0.064 J
3 2
42. A coil in the shape of an equilateral triangle of side l is DIRECTIONS (Qs. 47 to 50) : Each question contains
N
suspended between the pole pieces of a permanent magnet STATEMENT-1 and STATEMENT-2. Choose the correct answer
®
R

(ONLY ONE option is correct ) from the following-


such that B is in plane of the coil. If due to a current i in
U

the triangle a torque t acts on it, the side l of the triangle is (a) Statement -1 is false, Statement-2 is true
(b) Statement -1 is true, Statement-2 is true; Statement -2 is a
JO

1 1
correct explanation for Statement-1
2 æ t ö2 æ t ö2
(a) ç ÷ (b) 2çç ÷
÷ (c) Statement -1 is true, Statement-2 is true; Statement -2 is not
3 è B.i ø è 3B.i ø
U

a correct explanation for Statement-1


(d) Statement -1 is true, Statement-2 is false
ED

2 æ t ö 1 t
(c) ç ÷ (d)
3 è B.i ø 3 B.i 47. Statement-1 : The ferromagnetic substance do not obey
Curie’s law.
43. Iron is ferromagnetic
Statement-2 : At Curie point a ferromagnetic substance start
(a) above 770°C (b) below 770°C
behaving as a paramagnetic substance.
(c) at all temperature (d) above 1100°C
44. A vibration magnetometer placed in magnetic meridian has 48. Statement-1 : Magnetism is relativistic.
a small bar magnet. The magnet executes oscillations with a Statement-2 : When we move along with the charge so that
time period of 2 sec in earth's horizontal magnetic field of 24 there is no motion relative to us, we find no magnetic field
microtesla. When a horizontal field of 18 microtesla is associated with the charge.
produced opposite to the earth's field by placing a current 49. Statement-1 : A paramagnetic sample display greater
carrying wire, the new time period of magnet will be magnetisation (for the same magnetic field) when cooled.
(a) 1 s (b) 2 s (c) 3 s (d) 4s Statement-2 : The magnetisation does not depend on
45. Two identical bar magnets are fixed with their centres at a temperature.
distance d apart. A stationary charge Q is placed at P in 50. Statement-1 : Electromagnetic are made of soft iron.
between the gap of the two magnets at a distance D from
the centre O as shown in the figure. Statement-2 : Coercivity of soft iron is small.
Magnetism and Matter 561

Exemplar Questions NEET/AIPMT (2013-2017) Questions


1. A toroid of n turns, mean radius R and cross-sectional radius 6. A bar magnet of length ‘l’ and magnetic dipole moment ‘M’
a carries current I. It is placed on a horizontal table taken as is bent in the form of an arc as shown in figure. The new
xy-plane. Its magnetic moment m magnetic dipole moment will be [2013]
(a) is non-zero and points in the z-direction by symmetry
(b) points along the axis of the toroid (m = mf)
1
(c) is zero, otherwise there would be a field falling as at
r3
large distances outside the toroid
(d) is pointing radially outwards
2. The magnetic field of the earth can be modelled by that of a 3 2
point dipole placed at the centre of the earth. The dipole axis (a) M (b) M
p p

.IN
makes an angle of 11.3° with the axis of the earth. At Mumbai,
M
declination is nearly zero. Then, (c) (d) M
2
(a) the declination varies between 11.3° W to 11.3° E 7. A bar magnet of magnetic moment M is placed at right angles
AL
(b) the least declination is 0° to a magnetic induction B. If a force F is experienced by
(c) the plane defined by dipole axis and the earth axis passes each pole of the magnet, the length of the magnet will be
through Greenwich [NEET Kar. 2013]
N
(d) declination averaged over the earth must be always (a) F/MB (b) MB/F
negative (c) BF/M (d) MF/B
R

3. In a permanent magnet at room temperature. 8. Following figures show the arrangement of bar magnets in
U

different configurations. Each magnet has magnet ic dipole


(a) magnetic moment of each molecule is zero r
moment m . Which configuration has highest net magnetic
(b) the individual molecules have non-zero magnetic
JO

dipole moment ? [2014]


moment which are all perfectly aligned
(c) domains are partially aligned N
U

(d) domains are all perfectly aligned


4. Consider the two idealised systems (i) a parallel plate N S
ED

capacitor with large plates and small separation and (ii) a A. B. S N


long solenoid of length L >> R, radius of cross-section. In S S N
(i) E is ideally treated as a constant between plates and zero
outside. In (ii) magnetic field is constant inside the solenoid
and zero outside. These idealised assumptions, however, N
N
contradict fundamental laws as below
(a) case (i) contradicts Gauss’ law for electrostatic fields C. 30º D.
S 60º
(b) case (ii) contradicts Gauss’ law for magnetic fields N
S N
(c) case (i) agrees with Ñò E.dl = 0. (a) A (b) B
(c) C (d) D
(d) case (ii) contradicts Ñò H.dl = I en 9. The magnetic susceptibility is negative for : [2016]
(a) diamagnetic material only
5. A paramagnetic sample shows a net magnetisation of 8 Am–1
(b) paramagnetic material only
when placed in an external magnetic field of 0.6 T at a
(c) ferromagnetic material only
temperature of 4 K. When the same sample is placed in an
(d) paramagnetic and ferromagnetic materials
external magnetic field of 0.2 T at a temperature of 16 K, the
10. If q1 and q2 be the apparent angles of dip observed in two
magnetisation will be vertical planes at right angles to each other, then the true
32 2 angle of dip q is given by :- [2017]
(a) Am -1 (b) Am -1
3 3 (a) tan2q = tan2q1 + tan2q2
(b) cot2q = cot2q1 – cot2q2
(c) 6Am-1 (d) 2.4Am -1 (c) tan2q = tan2q1 – tan2q2
(d) cot2q = cot2q1 + cot2q2
EBD_7179
562 PHYSICS

Hints & Solutions


EXERCISE - 1 18. (b) Diamagnetic substances do not have any unpaired
electron. and they magnetised in direction opposite to
1. (b) 2. (a) 3. (d) 4. (a)
that of magnetic field. Hence when they are brought to
5. (d) North or South pole of Bar magnet, they are repelled
by poles.
1
6. (d) As F µ 4 and r becomes twice, therefore, F becomes 19. (c) We know that when a bar magnet is placed in the
r
magnetic field at an angle q, then torque acting on the
1 1 bar magnet
4
= times
2 16
(t) = MB sin q = M ´ B
1 Note : This torque t has a tendency to make the axis of
\ ´ 8 = 0 .5 N .
16 the magnet parallel to the direction of the magnetic
field.
7. (a) When cut along the axis, area of cross-section

.IN
becomes half. Therefore, pole strength is halved and 20. (a) Magnetic moment linked with one turn = iA
M = m (2 l), is also halved. Magnetic moment linked with N turn
8. (d) As magnetic moment = pole strength x length and AL = iNA amp-m2.
length is halved without affecting pole strength,
Here A = Area of current loop.
therefore, magnetic moment becomes half.
9. (d) A superconductor exhibits perfect diamagnetism. 21. (b) A diamagnetic material in a magnetic field moves, from
N
stronger to the weaker parts of the field.
tan q1 2
=
R

10. (a) 1
tan q 2 1
22. (b) According to Curie’s law, cm µ
T
U

11. (a) Iron is ferromagnetic.


12. (b) Ferromagnetic substance has magnetic domains 23. (a) The magnetic moment of a diamagnetic atom is equal
JO

whereas paramagnetic substances have magnetic to zero.


dipoles which get attracted to a magnetic field.
Diamagnetic substances do not have magnetic dipole 24. (a) A ® diamagnetic B ® paramagnetic
U

but in the presence of external magnetic field due to C ® Ferromagnetic D ® Non-magnetic


their orbital motion these substance are repelled. 25. (a) The magnetic dipole moment of diamagnetic material is
ED

zero as each of its pair of electrons have opposite spins,


13. (c) T µ I ; I a M Þ Ta M i.e., md = 0.
T1 M1 Paramagnetic substances have dipole moment > 0, i.e.
= mp ¹ 0, because of excess of electrons in its molecules
T2 M 2 Þ T2 = 2T1 = 2T
spinning in the same direction.
K1 + K 2
Ferro-magnetic substances are very strong magnets
K
14. (c) T1 = 2p = 2p and they also have permanent magnetic moment, i.e.
(M + 2M) H 3MH
mf ¹ 0.

T2 = 2p
K
= 2p
K EXERCISE - 2
(2M - M) H MH
6
Obviously T2 > T1 1. (a) Here, 2l = 8 cm , l = 4 cm , d = = 3 cm .
2
At neutral point,
B
15. (b) tan q = V = 1, BV = BH m0 M
BH H =B=
4 p (d 2 + l 2 )3 / 2
q = tan -1 (1) = 45°
M M
= 10 - 7 =
16. (c) cm is negative for diamagnetic materials. (5 ´ 10 -2 3
) 1250
17. (b) The iron can produces a magnetic screening for the
equipment as lines of magnetic force can not enter iron \ M = 1250 H = 1250 ´ 3.2 ´ 10 -5 Am 2
enclosure.
Magnetism and Matter 563

14. (c) Taking distances from the centre of the magnet,


M 1250 ´ 3.2 ´ 10 -5
m= = A m.
2l 8 ´ 10 -2 3 3
B1 æ x 2 ö æ 2 x +1ö
=ç ÷ =ç ÷ = 8 : 1, approximately.
1 B2 çè x1 ÷ø è x +1 ø
= 0.5 Am = 0.5 ´ ab amp ´ 100 cm
10
15. (d) Angle of dip, d = 45°
= 5 ab-amp cm.
2. (d) In series, same current flows through two tangent tan 45 1 2
\ tan d ¢ = tan d = = =
galvanometers. cos q cos 30º 3/2 3
3. (b) As the axes are perpendicular, mid point lies on axial
line of one magnet and on equatorial line of other \ Real dip d ¢ = tan -1 (2 / 3)
magnet.

m 0 2 M 10-7 ´ 2 ´ 1 T22 2M + M
\ B1 = = = 2 ´ 10-7 16. (b) = =3 \ T2 = T1 3 = 3 3 s.
4 p d3 13 T12 2M - M

m M
and B2 = 0 3 = 10-7 tan q 2 d13 r3 1
4p d 17. (a) = 3 = =
tan q1 d 2 [r (3)1/ 3 ]3 3

.IN
\ Resultant field = B12 + B 22 = 5 ´ 10 -7 T
1 tan 60 3 1
tan q 2 = tan q1 = = = \ q 2 = 30º
m m m 30 ´ 60
AL 3 3 3 3
4. (a) F = 0 1 2 = 10 -7 ´ = 2 ´ 10 -3 N.
4p r 2
(0.3) 2
18. (b) Here, d1 = 20 cm, M2 = 2 M1, d2 = ?
5. (b) t = MB sin q = 0.1 × 3 × 10–4 sin 30º
N

or t = 1.5 × 10–5 N–m. M 2 d 32 1/ 3


= 2 Þ d2 = 2 d1 = 20 (2)1/ 3 cm
R

=
i tan q M1 d13
2 2
(d) As i = tan q
U

6.
1 1
V V tan q¢ 1
JO

7. (c) 8. (b) 19. (a) tan q = , tan q¢ = ; =


H H cos x tan q cos x
m0C
9. (b) According to Curie's law, cm =
T 20. (d) d1 =40º , d 2 =30º , d =?
U

where C is Curie constant, T = temperature


ED

1 cot d = cot 2 d1 + cot 2 d 2


\ cma
T
= cot 2 40º + cot 2 30º
cm1 T2 273 + 333 606
= = = =2
c m2 T1 273 + 30 303 cot d = 1.192 + 3 =2.1
\ d =25º i.e. d < 40º.
(
\ c m2 = c m1 / 2 = 0.5c m = 0.5 c. Q c m1 = c
1 )
21. (b) W1 = – MB (cos 90º – cos 0º) = MB
10. (a) As lines of magnetic induction B are continuous curves,
they run continuously through the bar and outside, as W2 = – MB (cos 60º – cos 0º)
shown in Fig. (1).
æ1 ö 1 1
11. (c) The loop (i) is for soft iron and the loop (ii) is for steel = - MBç - 1÷ = MB = W1
è2 ø 2 2
in Fig.
12. (b) As breadth of each part is half the original breadth, As W1 = n W2 \ n =2
therefore, pole strength becomes half (i.e. m/2).
60 60
13. (a) As length of each part also becomes half, therefore 22. (d) Here, T1 = = 5 s, T2 = = 15 s
12 4
magnetic moment M = pole strength × length

1 1 1 M1 T22 + T12 152 + 5 2 250 5


Þ ´ = th i.e. M/4. = = = =
2 2 4 M 2 T22 - T12 15 2 - 52 200 4
EBD_7179
564 PHYSICS

V V 1 Therefore distance between poles


23. (c) tan d = = =
H 3V 3
l
\ d = 30º = p / 6 radian = ( l 2 ) 2 + ( l 2 )2 =
2

ml M
I , So, M' = =
24. (c) T1 = 2p 2 2
HM

I T2 M1 M1
T2 = 2p 33. (c) = = =2
(H + F)M T1 M2 1
M1
4
T12 H+F T2 4 \ T2 = 2T1 = 3 s
= = = Þ H + F = 4H
T22 H 2
T /4 1
M1 T22 + T12 62 + 42 52
Þ 3H = F 34. (c) = = = = ( 2.6 ) :1
M 2 T22 - T12 62 - 42 20
25. (a) m = m0 mr = (4 p´10-7 ) ´ 2000 = 8 p´10 -4 S.I. units
35. (d) The value of H is fairly uniform.
26. (b) Let pole strength = m 36. (b) Field in the core with Bismuth will be smaller because

.IN
So, M = ml
bismuth is diamagnetic.
When wire is in form of arc, then the distance between
2l 37.
AL (b) From m r = 1 + c m ;
poles =
p
Magnetic suscaptibility, c m = m r - 1
m 2l 2M
N
So, M ' = = cm = 0.075 - 1 = - 0.925.
p p
38. (b) For a diamagnetic material, the value of µr is less than
R

27. (a) Magnetic moment is cancelled and mnet = 0.


28. (c) t = MB sin q (q = 90°) one. For any material, the value of Îr is always greater
U

than 1.
B1 t1
JO

t = MB Þ = (since magnetic moment is same) 39. (a) Intensity of magnetisation


B2 t2
M 1.2
29. (c) Mnet = M 20 + M02 + 2M 02 cos 60 ° Ιm = = = 4 ´ 104 A m -1
U

V (15 ´ 2 ´ 1)10-6
ED

= 3M 02 = 3M 0 40. (a) Magnetic moment = M; Initial angle through which


magnet is turned (q1) = 90º and final angle which
1 MB magnet is turned (q2)= 60º. Work done in turning the
30. (c) n=
2p I magnet through
When it is turned by an angle 90° the effective field is 90º(W1) = MB (cos 0º – cos 90º)= MB (1–0) = MB.
vertical = V and B > V Similarly, W2 = MB (cos 0º – cos 60º)
So, new frequency < n.
æ 1 ö MB
= MBç1 - ÷ = .
I è 2ø 2
31. (b) T = 2p
MB \ W1 = 2W2 or n = 2.
2
mælö I B=
H
=
0.50
=
0.50 ´ 2
= 1/ 3
I= ç ÷ Þ I' = 41. (c)
2 è2ø 8 cos q cos 30º 3
42. (b) t = MB sinq, t = iAB sin90º
M A
M' =
2

I T
So, T ' = 2p Þ T' = = 2 sec .
4MB 2
32. (b) Magnetic moment, M = ml
D
M B C
= m , where m is the polestrength. l
l
Magnetism and Matter 565

t EXERCISE - 3
\ A= = 1/2 (BC) (AD)
iB Exemplar Questions
2
1 1 ælö 3 2 1. (c) Toroid is a hollow circular ring on which a large number
But (BC)(AD) = (l ) l 2 - ç ÷ = l
2 2 è 2ø 4 of turns of a wire are closely wound. Thus, in this case
magnetic field is only confined inside the body of toroid.
3 2 t So no magnetic field outside the toroid and magnetic
Þ (l ) =
4 Bi field only inside the toroid.
1 In case of toroid, the magnetic field is in the form of
æ t ö2 concentric magnetic lines of force and there is no
\ l = 2ç ÷
è 3 B.i ø magnetic field outside the body of toroid. This is
because the loop encloses no current. Thus, the
43. (b) magnetic moment of toroid is zero.
44. (d) Time period of a vibration magnetometer, In other case, if we take r as a large distance outside
1
1 T1 B2 the toroid, then mµ 3 . Which is not possible.
Tµ Þ = r
B T2 B1 2. (a) Magnetic declination is an angle between angle of
magnetic meridian and the geographic meridian.

.IN
B1 As the earth’s magnetism, the magnetic field lines of
Þ T2 = T1 the earth resemble that of a hypothetical magnetic
B2
dipole located at the centre of the earth.
AL
The axis of the dipole does not coincide with the axis of
24 ´10-6 rotation of the earth but is presently tilted by 11.3°
=2 = 4s
N
6 ´ 10-6 (approx) with respect to geographical of axis earth. This
results into two situations as given in the figure.
R

45. (b) Force on a charged particle is given by F = qvB. Here N N


v = 0 and also resultant B is zero.
U

11
.3

º
.3
\ Force = 0 S º S

11
JO

W E W E
46. (a) For stable equilibrium N
11.3º N
U = –MB = – (0.4) (0.16) = – 0.064 J
U

S S
47. (c) The susceptibility of ferromagnetic substan ce
decreases with the rise of temperature in a complicated So, the declination varies between 11.3° W to 11.3° E .
ED

manner. After Curies point in the susceptibility of 3. (d) We know that a permanent magnet is a substance which
ferromagnetic substance varies inversely with its at room temperature retain ferromagnetic property for a
absolute tempearture. Ferromagnetic substance obey’s long period of time. The individual atoms in a
Curie’s law only above its Curie point. ferromagnetic material possess a dipole moment as in a
paramagnetic material. However, they interact with one
48. (b) A magnetic field is produced by the motion of electric
another in such a way that they spontaneously align
charge. Since motion is relative, the magnetic field is themselves in a common direction over a macroscopic
also relative. volume i.e., domain.
49. (d) A paramagnetic sample display greater magnetisation Hence, in a permanent magnet at room temperature,
when cooled, this is because at lower temperature, the domains are all perfectly aligned.
tendency to disrupt the alignment of dipoles (due to 4. (b) The electric field lines, do not form a continuous path
magnetising field) decreases on account of reduced while the magnetic field lines form the closed paths.
random thermal motion.
q
50. (b) Electromagnets are magnets, which can be turnd on
and off by switching the current on and off.
Gauss’s law states that, Ñò E.ds = e
s 0
for electrostatic

As the material in electromagnets is subjected to cyclic field. So, it does not contradict for electrostatic fields as
changes (magnification and demangetisation), the the electric field lines do not form closed continuous
hysteresis loss of the material must be small. The path.
material should attain high value of I and B with low According to Gauss’ law in magnetic field,

Ñò E.ds = 0
value of magnetising field intensity H. As soft iron has
small coercivity, so it is a best choice for this purpose. s
EBD_7179
566 PHYSICS

It contradicts for magnetic field, because there is a pr 3l


magnetic field inside the solenoid and no field outside l= or r=
3 p
the solenoid carrying current but the magnetic field lines
form the closed path. m ´ 3l 3
so, M' = m × r = = M
5. (b) According to the Curie law, the intensity of p p
magnetisation (I) is directly proportional to the magnetic MB
7. (b) FL = MB (= Torque) Þ L =
field induction and inversely proportional to the F
temperature (t) in kelvin. q
8. (c) Net magnetic dipole moment = 2 Mcos
So, I magnetisation 2
q
B (magnetic field induction) As value of cos is maximum in case (c) hence net
2
µ magnetic dipole moment is maximum for option (c).
t(temperature in kelvin)
9. (a) Magnetic susceptibility c for dia-magnetic materials
I2 B2 t1 only is negative and low |c| = –1; for paramagnetic
Þ = ´ ... (i)
I1 B1 t 2 substances low but positive |c| = 1 and for
As given that :I 1 = 8 Am–1, I2 = ? ferromagnetic substances positive and high |c| = 102.
10. (d) If q1 and q2 are apparent angles of dip
B1 = 0.6 T, t1 = 4K Let a be the angle which one of the plane make with
B2 = 0.2 T, t2 = 16K the magnetic meridian.

.IN
by putting the value of B1, B2, t1, t2 I1 in equation (i) v
tan q1 =
0.2 4 I2 H cos a
So, ´ =
0.6 16 8
AL v
1 i.e., cos a = …(i)
We get, I2 = 8 ´ H tan q1
12
N
v
2 tan q2 = ,,
I2 = A/m H sin a
R

3
v
NEET/AIPMT (2013-2017) Questions i.e., sin a = H tan q
U

…(ii)
2
6. (a) Magnetic dipole moment
JO

Squaring and adding (i) and (ii), we get


M=m×l M' = m × r
2
From figure æV ö æ 1 1 ö
cos 2 a + sin 2 a = ç ÷ ç + ÷
U

è H ø è tan q1 tan 2 q2 ø
2
l
ED

V2 é 2
ëcot q1 + cot q2 ùû
2
i.e., 1=
r sin 30º r sin 30º H2

H2
r 30º 30º or = cot 2 q1 + cot 2 q 2
60º V2
i.e., cot 2 q = cot 2 q1 + cot 2 q2
Electromagnetic
21 Induction
THE EXPERIMENTS OF FARADAY AND HENRY When the bar magnet is pushed towards the coil, the pointer in
The discovery and understanding of electromagnetic induction the galvanometer G deflects.
are based on a long series of experiments carried out by Faraday Current is induced in coil C1 due to motion of the current carrying
and Henry. These experiments are illustrated by the following coil C2
figures.

.IN
AL
N
R
U
JO
U

S. No. Experiment Observation


1. Place a magnet near a conducting No current flows through the galvanometer.
ED

loop with a galvanometer in the


circuit.

2. Move the magnet towards the The galvanometer register a current.


loop.

3. Reverse the direction of motion The galvanometer deflection reverses.


of the magnet.
4. Reverse the polarity of the The galvanometer deflection reverses.
magnet and move the magnet
towards the loop.

5. Keep magnet fixed and move The galvanometer register a


the coil towards the magnet. current.
EBD_7179
568 PHYSICS

6. Increases the speed of the magnet. The deflection in the galvanometer increases.
7. Increase the strength of the magnet. The deflection in the galvanometer increases.
8. Increase the diameter of the coil. The deflection in the galvanometer increases.
9. Fix the speed of the magnet but The deflection in the galvanometer increases.
repeat the experiment with the
magnet closer to the coil.
10. Move the magnet at an angle to Deflection decreases, it is maximum when the magnet moves perpendicular to the
the plane of the coil. plane of the coil and is zero when the magnet moves parallel to the plane of the coil.
11. Increase the number of turns of Magnitude of current increases.
the coil.

MAGNETIC FLUX Lenz’s law is in accordance with the principle of conservation


The number of magnetic lines of force crossing a surface is called of energy. Infact, work done in moving the magnet w.r.t. the coil
magnetic flux linked with the surface. changes into electric energy producing induced current.
There is also another law for finding the direction of induced
It is represented by f .
current. This is Fleming’s right hand rule. According to this
B rule, if we stretch the right-hand thumb and two nearby fingers
perpendicular to one another such that the first finger points in

.IN
the direction of magnetic field and the thumb in the direction of
q
motion of the conductor, then the middle finger will point in the
direction of the induced current.
^n
AL
N
Direction of induced
current inwards
N
ur ur Thumb Direction of motion
Magnetic flux f = B. A = BA cos q (motion) of the conductor
R

First finger
where B is strength of magnetic field, A is area of the surface and (field)
U

q is the angle which normal to the area (unit area vector) makes
Central finger
S B
with the direction of magnetic field.
JO

(current)
A Application of Fleming’s
The S.I. unit of magnetic flux is weber which is the amount of
right-hand rule
magnetic flux over an area of 1 m2 held normal to a uniform (a)
magnetic field of one tesla. (b)
U

Total flow of charge due to change of flux (Df):


The c.g.s. unit of f is maxwell.
ED

1 weber = 108 maxwell. (No. of turns ´ change in magnetic flux)


Q = NDf / R =
FARADAY’S LAW OF ELECTROMAGNETIC Resistance
INDUCTION METHODS OF INDUCING E.M.F.
Whenever the number of magnetic lines of force (flux) linked As is known, e.m.f. is induced in a circuit only when amount of
with any closed circuit change, an induced current flows through magnetic flux linked with the circuit changes. As f = BA cos q,
the circuit which lasts only so long as the change lasts. An therefore three methods of producing induced e.m.f. :
increase in the number of lines of force produces an inverse (i) By changing B, (ii) By changing A and, (iii) By changing q
current, while a decrease of such lines produces a direct current. (orientation of the coil). When a conductor of length l moves
with a velocity v in a magnetic field of strength B so that magnetic
The induced emf is equal to the negative rate of change of
flux linked with the circuit changes, the e.m.f. induced (e) is given
magnetic flux. by
- df e = B l v.
i.e. e=
dt Induced e.m.f. and its direction
The -ve sign shows that the induced emf opposes the change in
Case (i) In conducting rod: The induced e.m.f. is generated
magnetic flux (Lenz’s law).
because of rotation of a conducting rod in a
LENZ’S LAW perpendicular magnetic field
The direction of induced e.m.f. is given by Lenz’s law. According Bl 2w
to this law, the direction of induced e.m.f. in a circuit is always e =- also, e = – BAf
2
such that it opposes the every cause which produces it. where f = frequency of rotation and
-d f A = pr2, where r is the radius of circle in which this rod
Thus, e= moves, hence r = l. w = angular velocity, l = length of
dt
conducting rod.
Electromagnetic Induction 569

Case (ii) In disc: Induced e.m.f generated in a disc rotating with (b) the direction of induced current in the loop will
a constant angular velocity in a perpendicular magnetic be anti-clockwise so that it may oppose the
field increase of magnetic flux in the loop in upward
Br 2 w direction.
e = –BAf = -Bpr 2 f = - A B
2
where A = area of disc = pr2, r = radius of disc, P
w = angular velocity of disc.
Case (iii) In two coils: When two coils are arranged as shown in
O
the figure
P
Q Case (vi) Magnet dropped freely in long vertical copper tube:
The resistance of copper tube is quite negligible and
+ - hence maximum induced current are generated in it
K due to the motion of the magnet. Due to these induced
current the motion of magnet is opposed to maximum.
(a) if key K is closed then current in P will flow in Consequently the acceleration of the magnet will be
clockwise direction and consequently induced zero (a = g – g = 0).
current in Q will flow in anticlockwise direction.
S
(see fig. a)
(b) when key K is opened then current in P falls from

.IN
maximum to zero and consequently induced
current in Q will flow in clockwise direction. (see N
g
fig. b)
AL g
P P a= g–g =0
Q Q
N
O Case (vii) Magnet dropped freely into a long solenoid of copper
O wire: The resistance of copper solenoid is much higher
R

than that of copper tube. Hence the induced current


in it, due to motion of magnet, will be much less than
U

(a) (b)
that in the tube. Consequently the opposition to the
JO

Case (iv) In three coils arranged coaxially : Three coils P, Q motion of magnet will be less and the magnet will fall
and R are arranged coaxially as shown in figure. Equal with an acceleration (a) less than g. (i.e. a < g).
currents are flowing in coils P and R . Coils Q and R are S
U

fixed. Coil P is moved towards Q. The induced current


in Q will be in anti-clockwise direction so that it may
ED

oppose the approach of P according to Lenz’s law. As


the face of P towards Q is a south pole hence plane of
Q towards P will also be a south pole. N
Induced current
a <g
P Q R Axis
Observer
coils Case (viii) Motional EMF: Induced emf in a conducting rod moving
As there is no relative motion between Q and R, hence perpendicular through a uniform magnetic field as
no current is induced in Q due to R. shown
Case (v) Current increases in straight conductor : When × ++× × ×
current in the straight conductor is increased then
(a) the direction of induced current in the loop will
be clockwise so that it may oppose the increase of ×l × × B
×
magnetic flux in the loop in downward direction. v

P
× –– × × ×
The induced emf produced across the rod
O l
r r
e = Blv = ò(v ´ B).dl
0

This is also called motional emf and it develops when


A B a metal rod cuts magnetic lines of force.
EBD_7179
570 PHYSICS

Special case : If the rod moves in the magnetic field making an The induced emf or current I is shown which is in
angle q with it, then induced emf e = Bn vl = Bvl sin q . accordance to Lenz’s law. Here the varying magnetic field
at the location of loop (due to the movement of magnet)
COMMON DEFAULT
creates an electric field.
û Incorrect. When there is no change in magnetic flux no
induced current is produced.
We should remember certain points regarding the induced
electric field produced due to changing magnetic field.
ü Correct. Consider the case (viii) discussed above. There is
no change in the magnetic flux throgh the rod, still induced · Induced electric field lines form closed loops (different
emf is produced. from the electric field lines used to depict electric field
Case (ix) A straight conductor (slider) moving with velocity v produced due to charges)
on a U shaped wire placed in a uniform magnetic field. · Induced electric field is non-conservative in nature
Blv (again a difference from the electric field produced by
The induced current produced is I = electric charges)
R
Case (x) When a rectangular loop perpendicular to the magnetic
df
field is pulled out, then forces F1 and F3 being equal Mathematically, e = ò E .dl = - ¹0
dt
and opposite cancel out.

B 1. An emf is induced in a circuit where the magnetic flux is


I
> changing even if the circuit is open. But obviously no

.IN
F1
F2 l F v current will flow. If we close the circuit, the current will start
3
flowing.
2. In a loop moving in a uniform magnetic field, when the loop
AL
remains in the field, the net emf induced is zero.
æ Blv ö B2 l 2 v
F2 = BI l = B ç l =
N
è R ÷ø
v B
R l Bl v Bl v
R

Power required to move the loop out


B2 l 2 v 2 Example 1.
U

P= F2 × v =
R A copper rod of length l is rotated about one end
JO

Case (xi) The magnet is stationary and the loop is moving perpendicular to the uniform magnietic field B with
towards the magnet. constant angular velocity w. What will be the induced
e.m.f. between two ends ?
U

Solution :
I Consider a small element of the rod of length dx at a distance
ED

S N v x from the centre O.


Stationary magnet
w
dx
Moving loop
The induced emf or current I is shown which is in x
l
accordance to Lenz's law. In this case the magnetic
force causes the charge to move. We know that if a O
charged particle is in motion in a field it experiences a
magnetic force. This is because when charged particle
moves it creates its own magnetic field which interacts
with the existing magnetic field.
Let v be the linear velocity of the element at right angles to
Case (xii) The magnet is moving towards the loop which is the magnetic field B. The e.m.f. developed across the
stationary. element is d e = B v dx = B (w x) dx (Q v = w x)
The e.m.f. across the entire rod of length l is given by
v l
I l é x2 ù
S N ò
e = de = B w ò
0
x dx = B w ê ú
êë 2 úû 0
Moving magnet

2 1
= B w (l / 2 ) = B wl2
Stationary loop 2
Electromagnetic Induction 571

Example 2. b
A conductor of length 10 cm is moved parallel to itself with
a speed of 10 m/s at right angles to a uniform magnetic
induction 10–4 Wb/m². What is the induced e.m.f. in it?
Solution : c a
Given : l = 10 cm = 0.1 m, v = 10 m/s
B = 10–4 Wb/m2 Fig. 3
e.m.f. induced in conductor
e = B l V = 10–4 × 0.1 × 10 = 10–4 V
Example 3.
A metal rod of length 1 m is rotated about one of its ends in
a plane right angles to a field of inductance 2.5 × 10–3 a b
Fig. 4
Wb/m². If it makes 1800 revolutions/min. Calculate induced
e.m.f. between its ends.
Solution :
Given : l = 1m, B = 5 × 10–3 Wb/m2
1800
f= = 30 rotations/sec
60 Decreasing at
I a stead rate
In one rotation, the moving rod of the metal traces a circle of

.IN
radius r = l Fig.5
\ Area swept in one rotation = pr2 Solution :
df dA Bpr 2
d
= ( BA ) = B. = = B f p r2
AL Applying Lenz’s law
dt dt dt T Fig. (1) along a ® b
= (5 × 10–3) × 3.14 × 30 × 1 = 0.471 V Fig. (2) along b ® a
N
\ e.m.f. induced in a metal rod = 0.471 V Fig. (3) along c ® a
Example 4. Fig. (4) along a ® b
R

A coil having 100 turns and area 0.001 metre2 is free to Fig. (5) no induced current since field lines lie in the plane
rotate about an axis. The coil is placed perpendicular to a
U

of the loop.
magnetic field of 1.0 weber/metre2. If the coil is rotate
JO

rapidly through an angle of 180°, how much charge will


flow through the coil? The resistance of the coil is 10 ohm. EDDY CURRENTS
Solution : The induced circulating currents produced in a metal itself due
U

The flux linked with the coil when the plane of the coil is to change in magnetic flux linked with the metal are called
perpendicular to the magnetic field is eddy currents. These currents were discovered by Foucault, so
ED

f = nAB cos q = nAB. they are also known as Foucault Currents.


Change in flux on rotating the coil by 180° is The direction of eddy currents is given by Lenz’s law.
df = nAB – (–nAB) = 2nAB Eddy currents produced in a metallic block moving in a
non-uniform magnetic field is shown in fig.
df
\ induced charge =
R
2nAB 2 ´ 100 ´ 0.001 ´ 1
= =
dt 10
= 0.01 coulomb
Example 5.
Predict the direction of induced current in the situations
described by the following fig. (1) to (5).

Applications of Eddy Current


Like friction, eddy currents are helpful in some fields and have to
a b N S be increased, while in some other fields they are undesirable and
Fig -1 have to be minimised.
a b c d (1) Dead beat galvanometer. (2) Energy meter.
N S
(3) Speedometer. (4) Electric brakes.
(5) Single phase AC motor. (6) Induction furnace.
Fig.2 (7) Diathermy
EBD_7179
572 PHYSICS

In a moving coil galvanometer, damping is necessary 2. The self inductance is a measure of the coil to oppose the
to avoid oscillation of display needle. This is brought into practice flow of current through it. The role of self-inductance in an
with the help of eddy currents. The winding of the coil of electrical circuit is the same as that of the inertia in
galvanometer is done on a metallic frame. When the coil rotates mechanics. Therefore it is called electrical inertia.
the magnetic flux linked with the metallic frame changes due to 3. The magnetic energy density (energy stored per unit
which eddy currents are developed which oppose the rotation of B2
the coil. This is called dead beat galvanometer. volume) in a solenoid =
2m 0
SELF INDUCTANCE AND MUTUAL INDUCTANCE
Self Inductance Mutual Inductance
The property of a coil by virtue of which the coil opposes any Mutual induction is the property of two coils by virtue of which
change in the strength of the current flowing through it, by each opposes any change in the strength of current flowing
inducing an e.m.f. in itself is called self inductance. through the other by developing an induced e.m.f.

Coil Second
First coil coil
Direction of
induced e.m.f. Mutual
(e)
K induction

.IN
When a current I flows through a coil, the magnetic flux f linked N1 N2
I1
with the coil is f = LI, where L is coefficient of self inductance of
AL
the coil. Coefficient of mutual inductance (M) of two coils is said to be
On differentiating, we get one henry, when a current change at the rate of 1 ampere/sec. in
N
df dI one coil induces an e.m.f. of one volt in the other coil. The value
= L = -e of M depends on geometry of two coils, distance between two
R

dt dt
coils, relative placement of two coils etc.
If dI / dt = 1; L = – e.
The coefficient of mutual inductance of two long co-axial
U

Hence coefficient of self inductance of a coil is equal to e.m.f.


solenoids, each of length l, area of across section A, wound on
induced in the coil when rate of change of current through the
JO

same coil is unity. Coefficient of self induction of a coil is also m N N A


an air core is M = 0 1 2 ] … (1)
defined as the magnetic flux linked with a coil when 1 ampere l
current flows through the same coil.
U

where N1 and N2 are total number of turns of the two solenoids.


The value of L depends on geometry of the coil and is given by The mutual inductance M is defined by the equation
ED

N2f2 = MI1
m N2A
L= 0 . where I1 is the current in coil 1, due to which flux f2 is linked
l with each turn of secondary coil.
where l is length of the coil (solenoid), N is total number of turns Now we can calculate, e.m.f. e2 induced in secondary by a
of solenoid and A is area of cross section of the solenoid. changing current in first coil. From Faraday‘s law
The S.I. unit of L is henry. Coefficient of self induction of a coil is
said to be one henry when a current change at the rate of 1 d dI
e2 = - ( N 2 f2 ) = - M 1
ampere/sec. in the coil induces an e.m.f. of one volt in the coil. dt dt
Keep in Memory dI1
If = 1 Þ e 2 = -M ...(2)
dt
1 2
1. Energy stored in a coil (inductor) = Li The two definitions for M defined by equations (1) and (2) are
2
equivalent. We can express these two equations in words as :
where L is the self-inductance and i current flowing through
(i) M is numerically equal to the flux-linkage in one circuit,
the inductor.
when unit current flows through the other. (we use this
The energy stored in the magnetic field of the coil.
definition to calculate M)
2
1 2 1 æ B ö (ii) M is numerically equal to the e.m.f. induced in one circuit,
E= Li = (m 0 n 2 Al) ç
è m 0 n ÷ø
2 2 when the current changes in the other at the rate of one
ampere in each second. (it is used to describe the mutual
behavior of two circuits).
æ B2 ö æ B2 ö
=ç ÷ Al = ç ÷ ´ volume For a pair of coils, M12 = M21 = m0 N1 N2 A/l, when wound on
è 2m0 ø è 2m0 ø one another.
Electromagnetic Induction 573

Keep in Memory When the armature coil ABCD rotates in the magnetic field
provided by the strong field magnet, it cuts the magnetic lines of
1. Coefficient of self inductance of two coils in series : force. Thus the magnetic flux linked with the coil changes and
hence induced emf is set up in the coil. The direction of the
L1 L2 Ls = L1 + L2 induced emf or the current in the coil is determined by the
The effective self inductance is Ls = L1 + L2 Fleming’s right hand rule.
If M is the coefficient of mutual inductance between the The current flows out through the brush B1 in one direction of
two coils when they have flux linkage in the same sense,then half of the revolution and through the brush B2 in the next half
L = L1 + L2 + 2M revolution in the reverse direction. This process is repeated.
Therefore, emf produced is of alternating nature.
L1
Ndf
e=- = NBAw sin wt = e 0 sin wt , where e0 = NBAw
dt
L1 L2 e e
L2 I = = 0 sin wt = I0 sin wt , R ® resistance of the circuit
R R
And for flux linkage in opposite direction
DC MOTOR
L = L1 + L2 – 2M
2. Coefficient of self inductance of two coils in parallel : A D.C. motor converts direct current energy from a battery into
mechanical energy of rotation.
L1
Principle : It is based on the fact that when a coil carrying current
Lp is held in a magnetic field, it experiences a torque, which rotates

.IN
the coil.
Working :
L2 AL
1 1 1 B C C B
= +
L p L1 L 2
N
(i) The coefficient of coupling between two coils having N S N S
self inductance L1 & L2 and coefficient of mutual
R

inductance M is A D D A
B1 B2
U

B1 B2
±M
K= R1 R2 R2 R1
L1 L 2
JO

(ii) Generally the value of K is less than 1. V V


(iii) If K is 1, then the coupling of two coils is tight while if
The battery sends current through the armature coil in the
U

K < 1, then coupling is loose.


· Inductance is pure geometrical factor,and is direction shown in fig. Applying Fleming’s left hand rule, CD
ED

experiences a force directed inwards and perpendicular to the


independent of current or applied e.m.f.
plane of the coil. Similarly, AB experiences a force directed
· If the angle between the axis of two closely placed outwards and perpendicular to the plane of the coil. These two
coil is q then M µ cos q . forces being equal, unlike and parallel form a couple. The couple
AC GENERATOR/DYNAMO/ALTERNATOR rotates the armature coil in the anticlockwise direction. After the
coil has rotated through 180°, the direction of the current in AB
An electrical machine used to convert mechanical energy into and CD is reversed, fig. Now CD experiences an outward force
electrical energy is known as AC generator/alternator or and AB experiences an inward force. The armature coil thus
dynamo. continues rotating in the same i.e., anticlockwise direction.
Principle : It works on the principle of electromagnetic induction, Efficiency of the d.c. motor : Since the current I is being supplied
i.e., when a coil is rotated in uniform magnetic field, an induced to the armature coil by the external source of e.m.f. V, therefore,
emf is produced in it.
Input electric power = VI
Working : According to Joule’s law of heating,
B C Power lost in the form of heat in the coil = I2 R
If we assume that there is no other loss of power, then
Power converted into external work
N S i.e., Output mechanical power = VI – I2 R = (V – IR) I = EI
A D
\ Efficiency of the d.c. motor
B1
R1 Output mechanical power
h=
RL Output Input electric power
R2 EI E Back e.m. f .
B2 or h= = =
VI V Applied e.m. f .
EBD_7179
574 PHYSICS

Uses of D.C Motor Example 8.


1. The D.C. motors are used in D.C. fans (exhaust, ceiling or A small coil of radius r is placed at the centre of a large
table) for cooling and ventilation. coil of radius R, where R >>r. The two coils are coplanar.
2. They are used for pumping water. The mutual induction between the coils is proportional to
3. Big D.C. motors are used for running tram-cars and even (a) r/R (b) r2/R
2
(c) r /R 2 (d) r/R2
trains.
Solution : (b)
Example 6.
Let I be the current flows in the large coil.
Two coils are wound on the same iron rod so that the flux
generated by one also passes through the other. The m0I
\ Mag. field at the centre of coil B =
primary has 100 loops and secondary has 200 loops. When 2R
a current of 2 A flows through the primary the flux in it is
25 x 10–4 Wb. Determine value of M between the coils. Mag. flux linked with smaller coil
Solution : æm Iö
f = pr 2 B = pr 2 ç 0 ÷
df s di p è 2R ø
es = Ns and e s = M ;
dt dt
di p f pr 2 m0
dfs But f = MI \ M= = or M µ r 2 / R
\ Ns =M or I 2R
dt dt

.IN
df s 200 ( 2.5 ´10 -4 - 0) Example 9.
M = Ns = The mutual inductance of a pair of coils is 0.75 H. If current
di p ( 2 - 0)
in the primary coil changes from 0.5 A to zero in 0.01 s find
= 2.5×10–2 = 25 mH
AL
average induced e.m.f. in secondary coil.
Example 7. Solution :
A long solenoid of length L, cross section A having N1
N
turns has wound about its centre is small coil of N2 turns dI 0.5 - 0
Given : M = 0.75 H and = = 50 A / s
as shown in fig. Then find the mutual inductance of two
R

dt 0.01
circuits. \ Average induced e.m.f. in secondary coil,
U

L
dI
1 N2 N1 e=M = 0.75 ´ 50 = 37.5 V
JO

dt
Example 10.
2 Find the self inductance of a coil in which an e.m.f. of 10 V is
U

Solution : induced when the current in the circuit changes uniformly


ED

Magnetic flux at the centre of solenoid B1 = m 0 ( N1 / L)i1 from 1 A to 0.5 A in 0.2 sec.
Magnetic flux through each turn of the coil of area A, Solution :
m Ni dI 1 - 0.5 0.5
f1 = B1A = 0 1 1 ´ A Given : e = 10 V and = = = 2.5 A / s
L dt 0.2 0.2
Magnetic flux linked with th e coil of turns N 2 ,
m N N i A e 10
f 2 = f1 ´ N 2 = 0 1 2 1 Self inductance of coil L = = =4 H
L dI / dt 2.5
According to the definition of mutual inductance f 2 = Mi1 é dI ù
êQ e = L dt (Considering Magnitude only) ú
m 0 N1 N 2 i1 m N N A ë û
\ Mi1 = A or M = 0 1 2
L L
CONCEPT MAP

Direction of induced current


Fleming’s Right Hand Rule: df
Motional emf e = – = –Blv
Thumb, forefinger, central dt
finger of right hand stretched lst law When magnetic 2nd law Induced emf 1 2
Across the end of rod e = Bwl
Electromagnetic Induction

perpendicular to each other flux linked with the µ rate of change of –df
2
then if thumb ® direction circuit changes an emf is magnetic flux e = dt
of motion; forefinger ® induced in the circuit
direction of magnetic field Magnetic flux
then central finger ®induced fB = B.A=BA cos q
ED
current
Lenz's law Direction
U of induced emf or current
Faraday's laws of is always in such a way
electromagnetic that it opposes cause due
JO
Induction Ac Generator or Dynamo
Induced current in a coil rotated Produces electrical energy to which it is produced. It
in uniform magnetic field
U from mechanical energy. It is in accordance with
works on EMI principle conservation of energy
NBA w sin wt R
I=
R ELECTROMAGNETIC N
INDUCTION (EMI)
Generation of current or emf
by changing magnetic field
AL
Eddy current Induced, Mutual Inductance Induced
when magnetic flux linked emf in a circuit due to change
with the conductor changes in magnetic flux in its
.IN
Inductance A measure
neighbouring circuit. Coefficients
of the ratio of the flux
of mutual inductance
to the current f
M=
I

® Electromagnetic Coefficient of mutual


damping Self inductance Inertia Self inductance of inductance between two
® Induction furnace of electricity. Coefficient a long solenoid long solenoids
® Magnetic braking fB m N2 A m NN A
® Electric power meter of self inductance L = L= 0 M= 0 1 2
i l l
575
EBD_7179
576 PHYSICS

1. Eddy currents are produced when


wBA wBA wBA 2wBA
(a) a metal is kept in varying magnetic field (a) (b) (c) (d)
p 2p 4p p
(b) a metal is kept in steady magnetic field
(c) a circular coil is placed in a magnetic field 10. According to Faraday’s law of electromagnetic induction
(d) through a circular coil, current is passed (a) electric field is produced by time varying magnetic flux.
2. An inductor may store energy in (b) magnetic field is produced by time varying electric flux.
(a) its electric field (c) magnetic field is associated with a moving charge.
(b) its coils (d) None of these
(c) its magnetic field 11. Two solenoids of same cross-sectional area have their
(d) both in electric and magnetic fields lengths and number of turns in ratio of 1 : 2. The ratio of
3. If N is the number of turns in a coil, the value of self self-inductance of two solenoids is
inductance varies as (a) 1 : 1 (b) 1 : 2 (c) 2 : 1 (d) 1 : 4
(a) N0 (b) N (c) N2 (d) N–2 12. The back e.m.f. in a d.c. motor is maximum, when

.IN
4. A coil having an area A0 is placed in a magnetic field which (a) the motor has picked up max speed
changes from B0 to 4 B0 in time interval t. The e.m.f. induced (b) the motor has just started moving
in the coil will be (c) the speed of motor is still on the increase
AL (d) the motor has just been switched off
(a) 3 A 0 B0 / t (b) 4 A 0 B0 / t
13. The mutual inductance between two coils depends on
(c) 3 B0 / A 0 t (d) 4A 0 / B0 t (a) medium between the coils
N
5. An electron moves along the line PQ which lies in the same (b) separation between the two coils
R

plane as a circular loop of conducting wire as shown in figure. (c) orientation of the two coils
What will be the direction of the induced current in the loop ? (d) All of the above
U

(a) Anticlockwise 14. If coefficient of self induction of a coil is 1 H, an e.m.f. of 1V


loop is induced, if
JO

(b) Clockwise
(c) Alternating (a) current flowing is 1A
P Q
(d) No current will be induced (b) current variation rate is 1 As–1
(c) current of 1A flows for one sec.
U

6. Induced emf in the coil depends upon


(d) None of these
(a) conductivity of coil
ED

(b) amount of flux ML2


15. Which of the following units denotes the dimension ,
(c) rate of change of linked flux Q2
(d) resistance of coil where Q denotes the electric charge?
7. Two identical coaxial circular loops carry current i each (a) Wb/m2 (b) henry (H)
circulating in the clockwise direction. If the loops are (c) H/m2 (d) weber (Wb)
approaching each other, then 16. In an AC generator, a coil with N turns, all of the same area
(a) current in each loop increases A and total resistance R, rotates with frequency w in a
(b) current in each loop remains the same magnetic field B. The maximum value of emf generated in
(c) current in each loop decreases the coil is
(d) current in one-loop increases and in the other it decreases (a) N.A.B.R.w (b) N.A.B.
8. The mutual inductance of a pair of coils, each of N turns, is (c) N.A.B.R. (d) N.A.B.w
M henry. If a current of I ampere in one of the coils is 17. A metal rod moves at a constant velocity in a direction
brought to zero in t second, the emf induced per turn in the perpendicular to its length. A constant uniform magnetic
other coil, in volt, will be field exists in space in a direction perpendicular to the rod
MI NMI MN MI as well its velocity. Select correct statements (s) from the
(a) (b) (c) (d) following.
t t It Nt
9. A rectangular coil of single turn, having area A, rotates in (a) The entire rod is at the same potential
a uniform magnetic field B with an angular velocity w about (b) There is an electric field in the rod
an axis perpendicular to the field. If initially the plane of (c) The electric potential is highest at the centre
the coil is perpendicular to the field, then the average (d) The electric potential is lowest at its centre and
increases towards its ends
induced emf when it has rotated through 90° is
Electromagnetic Induction 577

18. A small square loop of wire of side l is placed inside a 21. If the number of turns per unit length of a coil of solenoid is
large square loop of side L (L >> l ). The loop are coplanar doubled, the self-inductance of the solenoid will
and their centres coincide. The mutual inductance of the (a) remain unchanged(b) be halved
system is proportional is (c) be doubled (d) become four times
22. The total charge induced in a conducting loop when it is
l l2 L L2 moved in a magnetic field depend on
(a) (b) (c) (d)
L L l l (a) the rate of change of magnetic flux
19. As a result of change in the magnetic flux linked to the (b) initial magnetic flux only
closed loop shown in the figure, an e.m.f. V volt is induced (c) the total change in magnetic flux
in the loop. (d) final magnetic flux only
23. Lenz’s law is consequence of the law of conservation of
(a) energy (b) momentum
(c) charge (d) mass
24. If rotational velocity of a dynamo armature is doubled, then
induced e.m.f. will become
(a) half (b) two times
The work done (in joule) in taking a charge Q coulomb once (c) four times (d) unchanged
along the loop is

.IN
25. Choke coil works on the principle of
(a) QV (b) 2QV (c) QV/2 (d) zero (a) transient current (b) self induction
20. A wire loop is rotated in a uniform magnetic field about an AL (c) mutual induction (d) wattless current
axis perpendicular to the field. The direction of the current
induced in the loop reverses once each
(a) quarter revolution (b) half revolution
N
(c) full revolution (d) two revolutions
R
U
JO

1. A current i = 2 sin (pt/3) amp is flowing in an inductor of 2 4. A generator has an e.m.f. of 440 Volt and internal resistance
U

henry. The amount of work done in increasing the current of 4000 hm. Its terminals are connected to a load of 4000
from 1.0 amp to 2.0 amp is ohm. The voltage across the load is
ED

(a) 1 J (b) 2 J (c) 3 J (d) 4 J (a) 220 volt (b) 440 volt
2. Fig shown below represents an area A = 0.5 m2 situated in a (c) 200 volt (d) 400 volt
uniform magnetic field B = 2.0 weber/m2 and making an 5. When the current in a coil changes from 2 amp. to 4 amp. in
angle of 60º with respect to magnetic field. 0.05 sec., an e.m.f. of 8 volt is induced in the coil. The
coefficient of self inductance of the coil is
(a) 0.1 henry (b) 0.2 henry
B (c) 0.4 henry (d) 0.8 henry
60 6. A copper disc of radius 0.1 m rotated about its centre with
10 revolutions per second in a uniform magnetic field of 0.1
tesla with its plane perpendicular to the field. The e.m.f.
induced across the radius of disc is
The value of the magnetic flux through the area would be p 2p
equal to (a) volt (b) volt
10 10
(a) 2.0 weber (b) 3 weber (c) p ´10 -2 volt (d) 2p ´ 10 -2 volt
(c) 3 / 2 weber (d) 0.5 weber
7. A coil has 200 turns and area of 70 cm2. The magnetic field
3. In a coil of area 10 cm2 and 10 turns with magnetic field
perpendicular to the plane of the coil is 0.3 Wb/m2 and take 0.1
directed perpendicular to the plane and is changing at the
sec to rotate through 180º.The value of the induced e.m.f. will
rate of 108 Gauss/second. The resistance of the coil is 20W.
be
The current in the coil will be
(a) 0.5 A (b) 5 A (a) 8.4 V (b) 84 V
(c) 50 A (d) 5 × 108 A (c) 42 V (d) 4.2 V
EBD_7179
578 PHYSICS

8. If a current increases from zero to one ampere in 0.1 second The current induced in the loop is
in a coil of 5 mH, then the magnitude of the induced e.m.f.
will be Bl v Bl v
(a) clockwise (b) anticlockwise
(a) 0.005 volt (b) 0.5 volt R R
(c) 0.05 volt (d) 5 volt
9. A 100 millihenry coil carries a current of 1 ampere. Energy 2Blv
(c) anticlockw ise (d) zero
stored in its magnetic field is R
(a) 0.5 J (b) 1 J (c) 0.05 J (d) 0.1 J 19. The two rails of a railway track, insulated from each other
10. The armature of a dc motor has 20W resistance. It draws a and the ground, are connected to millivoltmeter. What is
current of 1.5 A when run by a 220 V dc supply. The value the reading of the millivoltmeter when a train passes at a
of the back emf induced in it is speed of 180 km/hr along the track, given that the vertical
(a) 150 V (b) 170 V (c) 180 V (d) 190 V component of earth’s magnetic field is 0.2 × 10–4 wb/m2 and
11. In the figure the flux through the loop perpendicular to the rails are separated by 1 metre
plane of the coil and directed into the paper is varying (a) 10–2 volt (b) 10mV
according to the relation f = 6t2 + 7t + 1 where f is in
(c) 1 volt (d) 1mV
milliweber and t is in second. The magnitude of the emf
induced in the loop at t = 2 s and the direction of induce 20. A long solenoid having 200 turns per cm carries a current of
current through R are Ä Ä Ä Ä Ä
1.5 amp. At the centre of it is placed a coil of 100 turns of
cross-sectional area 3.14 × 10–4 m2 having its axis parallel to

.IN
Ä Ä Ä Ä Ä
(a) 39 mV; right to left the field produced by the solenoid. When the direction of
Ä Ä Ä Ä Ä
(b) 39 mV; left to right Ä Ä Ä Ä Ä current in the solenoid is reversed within 0.05 sec, the induced
(c) 31 mV; right to left Ä Ä Ä Ä Ä e.m.f. in the coil is
AL
(d) 31 mV; left to right (a) 0.48 V (b) 0.048 V
R
12. A coil having 500 square loops each of side 10 cm is placed (c) 0.0048 V (d) 48 V
N
normal to a magnetic field which increases at the rate of 21. Two coils have a mutual inductance 0.005H. The current
1 Wb/m2. The induced e.m.f. is changes in first coil according to equation I = I0 sin wt
R

(a) 0.1 V (b) 5.0 V (c) 0.5 V (d) 1.0 V where I0 = 10A and w = 100p radian/sec. The max. value of
e.m.f. in second coil is
U

13. A circular coil and a bar magnet placed nearby are made to
move in the same direction. If the coil covers a distance of (a) 2p (b) 5p
JO

1 m in 0.5. sec and the magnet a distance of 2 m in 1 sec, the (c) p (d) 4p
induced e.m.f. produced in the coil is 22. A metal conductor of length 1 m rotates vertically about one of
(a) zero (b) 0.5 V (c) 1 V (d) 2 V. its ends at angular velocity 5 radians per second. If the
U

14. Magnetic flux f in weber in a closed circuit of resistance horizontal component of earth’s magnetic field is
10W varies with time f (sec) as f = 6t2 – 5t + 1. The magnitude 0.2 × 10–4T, then the e.m.f. developed between the two ends of
ED

of induced current at t = 0.25s is the conductor is


(a) 0.2 A (b) 0.6 A (c) 1.2 A (d) 0.8 A (a) 5 mV (b) 50 mV
15. The current in a coil of L = 40 mH is to be increased uniformly (c) 5 mV (d) 50mV
from 1A to 11A in 4 milli sec. The induced e.m.f. will be
23. Two identical induction coils each of inductance L are
(a) 100 V (b) 0.4 V (c) 440 V (d) 40 V jointed in series are placed very close to each other such
16. The self inductance of the motor of an electric fan is 10 H. that the winding direction of one is exactly opposite to that
In order to impart maximum power at 50 Hz, it should be of the other, what is the net inductance?
connected to a capacitance of
(a) L2 (b) 2 L
(a) 8 mF (b) 4 mF (c) 2 mF (d) 1 mF (c) L /2 (d) zero
17. The flux linked with a coil at any instant 't' is given by 24. A thin circular ring of area A is held perpendicular to a
f = 10t2 – 50t + 250. The induced emf at t = 3s is uniform magnetic field of induction B. A small cut is made in
(a) –190 V (b) –10 V (c) 10 V (d) 190 V the ring and a galvanometer is connected across the ends
18. A conducting square loop of side L and resistance R moves such that the total resistance of the circuit is R. When the
in its plane with a uniform velocity v perpendicular to one ring is suddenly squeezed to zero area, the charge flowing
of its side. A magnetic induction B constant in time and through the galvanometer is
space, pointing perpendicular and into the plane of the
loop exists everywhere. BR AB
(a) (b)
x x x x x x A R
x x x x x x x x
x x x x x vx B B2 A
x x x x x (c) ABR (d)
x x x x x x R2
Electromagnetic Induction 579

25. Consider the situation shown. The wire AB is sliding on 31. Two identical circular loops of metal wire are lying on a
fixed rails with a constant velocity. If the wire AB is replaced table without touching each other. Loop A carries a current
by semi-circular wire, the magnitude of induced e.m.f. will which increases with time. In response the loop B
Ä Ä Ä A Ä Ä (a) remains stationary
Ä Ä Ä Ä Ä (b) is attracted by loop A
Ä Ä vÄ Ä Ä R (c) is repelled by loop A
Ä Ä Ä Ä Ä
(d) rotates about is CM with CM fixed
32. A square loop of side a is rotating about its diagonal with
r
Ä Ä Ä B Ä Ä
angular velocity w in a perpendicular magnetic field B . It
(a) increase has 10 turns. The emf induced is
(b) decrease
(c) remain the same
(d) increase or decrease depending on whether the semi-
circle buldges towards the resistance or away from it. × × × ×B
26. A coil is wound on a frame of rectangular cross-section. If
all the linear dimensions of the frame are increased by a
factor 2 and the number of turns per unit length of the coil a

.IN
remains the same, self-inductance of the coil increases by
(a) B a2 sin wt (b) B a2 cos wt
a factor of
(a) 4 (b) 8 (c) 12 (d) 16 AL (c) 5 2 B a2 (d) 10 B a2 sin wt
27. A horizontal telegraph wire 0.5 km long running 33. In fig., final value of current in 10W resistor, when plug of
east and west in a part of a circuit whose resistance is 2.5 key K is inserted is
1H
W. The wire falls to g = 10.0 m/s2 and B = 2 × 10–5 weber/
N
3
m 2 , then the current induced in the circuit is (a) A
10
R

(a) 0.7 amp (b) 0.04 amp


3 10 W
(c) 0.02 amp (d) 0.01 amp
U

(b) A
28. A conductor of length 0.4 m is moving with a speed of 20 30 W
JO

7 m/s perpendicular to a magnetic field of intensity 3


(c) A
0.9 Wb/m2. The induced e.m.f. across the conductor is 11 3V K
(a) 1.26 V (b) 2.52V (c) 5.04 V (d) 25.2 V (d) zero
U

29. The inductance between A and D is 34. In a circuit given in figure 1 and 2 are ammeters. Just after
ED

(a) 3.66 H key K is pressed to complete the circuit, the reading is


(b) 9 H (a) zero in both 1 and 2 C R1
(b) maximum in both 1 and 2 1
(c) 0.66 H A 3H 3H L R2
3H D (c) zero in 1 and maximum in 2
(d) 1 H 2
(d) maximum in 1 and zero in 2
30. A square metal loop of side 10 cm and resistance 1 W is K
+ –
moved with a constant velocity partly inside a uniform
35. A solenoid has 2000 turns wound over a length of
magnetic field of 2 Wbm–2, directed into the paper, as shown
0.3 m. Its cross-sectional area is 1.2 × 10–3 m2. Around its
in the figure. The loop is connected to a network of five
central section a coil of 300 turns is wound. If an initial
resistors each of value 3W. If a steady current of 1 mA
current of 2 A flowing in the solenoid is reversed in 0.25 s,
flows in the loop, then the speed of the loop is
the emf induced in the coil will be
v (a) 2.4 × 10–4 V (b) 2.4 × 10–2 V
Ä Ä Ä Ä Ä (c) 4.8 × 10 V–4 (d) 4.8 × 10–2 V
Ä Ä Ä Ä Ä 36. Two coaxial solenoids are made by winding thin insulated
Ä Ä Ä Ä Ä
wire over a pipe of cross-sectional area A = 10 cm2 and
length = 20 cm. If one of the solenoid has 300 turns and the
Ä Ä Ä Ä Ä
other 400 turns, their mutual inductance is
Ä Ä Ä Ä Ä (m0 = 4p × 10 –7 Tm A–1)
(a) 0.5 cms–1 (b) 1 cms–1 (a) 2.4p × 10–5 H (b) 4.8p × 10–4 H
(c) 2 cms–1 (d) 4 cms–1 (c) 4.8p × 10–5 H (d) 2.4p × 10–4 H
EBD_7179
580 PHYSICS

37. A varying current in a coil change from 10A to zero in 0.5 43. A coil of resistance 400W is placed in a magnetic field. If
sec. If the average e.m.f induced in the coil is 220V, the self- the magnetic flux f (wb) linked with the coil varies with time
inductance of the coil is t (sec) as f = 50t2 + 4. The current in the coil at t = 2 sec is
(a) 5 H (b) 6 H (c) 11 H (d) 12 H (a) 0.5 A (b) 0.1 A (c) 2 A (d) 1 A
38. In an inductor of self-inductance L = 2 mH, current changes 44. A coil of self-inductance L is connected in series with a
with time according to relation i = t2e–t. At what time emf is bulb B and an AC source. Brightness of the bulb decreases
zero? when
(a) 4s (b) 3s (c) 2s (d) 1s (a) number of turns in the coil is reduced
39. The magnetic flux through a circuit of resistance R changes (b) a capacitance of reactance XC = XL is included in
by an amount Df in a time Dt. Then the total quantity of the same circuit
electric charge Q that passes any point in the circuit during (c) an iron rod is inserted in the coil
the time Dt is represented by (d) frequency of the AC source is decreased
Df 1 Df 45. A magnetic field of 2 × 10–2 T acts at right angles to a coil of
(a) Q = R. (b) Q= .
Dt R Dt area 100 cm2, with 50 turns. The average e.m.f. induced in
Df Df the coil is 0.1 V, when it is removed from the field in t sec.
(c) Q= (d) Q =
R Dt The value of t is

.IN
40. A conducting circular loop is placed in a uniform magnetic (a) 10 s (b) 0.1 s
field, B = 0.025 T with its plane perpendicular to the loop. AL (c) 0.01 s (d) 1 s
The radius of the loop is made to shrink at a constant rate 46. A rectangular coil of 20 turns and area of cross-section 25
of 1 mm s–1. The induced e.m.f. when the radius is 2 cm, is sq. cm has a resistance of 100W. If a magnetic field which is
perpendicular to the plane of coil changes at a rate of 1000
N
(a) 2pm V (b) pm V
tesla per second, the current in the coil is
p
R

(c) mV (d) 2 m V (a) 1 A (b) 50 A


2
U

41. The current i in a coil varies with time as shown in the (c) 0.5 A (d) 5 A
figure. The variation of induced emf with time would be DIRECTIONS for Qs. (47 to 50) : Each question contains
JO

STATEMENT-1 and STATEMENT-2. Choose the correct answer


i
(ONLY ONE option is correct ) from the following.
(a) Statement -1 is false, Statement-2 is true
U

(b) Statement -2 is true, Statement-2 is true; Statement -2 is a


ED

correct explanation for Statement-1


0 t (c) Statement -1 is true, Statement-2 is true; Statement -2 is not
T/4 T/2 3T/4 T
a correct explanation for Statement-1
emf emf
(d) Statement -2 is true, Statement-2 is false
T/4 47. Statement 1 : An induced emf appears in any coil in which
(a) 0 t (b) 0 t
T/2 3T/4 T T/4 T/2 3T/4 T the current is changing.
Statement 2 : Self induction phenomenon obeys Faraday's
emf law of induction.
emf
48. Statement 1 : Lenz's law violates the principle of
T/4 T/2 3T/4 T T/2 3T/4 T t conservation of energy.
(c) 0 t (d) 0
T/4 Statement 2 : Induced emf always opposes the change in
magnetic flux responsible for its production.
42. In a coil of resistance 10 W, the i(amp) 49. Statement 1 : When number of turns in a coil is doubled,
induced current developed by coefficient of self-inductance of the coil becomes 4 times.
changing magnetic flux through 4 Statement 2 : This is because L µ N2.
it, is shown in figure as a 50. Statement 1 : An induced current has a direction such that
function of time. The the magnetic field due to the current opposes the change in
magnitude of change in flux t(s) the magnetic flux that induces the current.
0 0.1
through the coil in weber is Statement 2 : Above statement is in accordance with
(a) 8 (b) 2 (c) 6 (d) 4 conservation of energy.
Electromagnetic Induction 581

Exemplar Questions (a) constant current clockwise


1. A square of side L metres lies in the xy-plane in a region, (b) varying current clockwise
(c) varying current counter clockwise
where the magnetic field is given by B = B (2iˆ + 3jˆ + 4k)
0
ˆ T,,
where B0 is constant. The magnitude of flux passing through (d) constant current counter clockwise
the square is
A
(a) 2B0L2Wb (b) 3B0L2Wb w B

(c) 4B0L2Wb (d) 29B0 L2 Wb


2. A loop, made of straight edges has six corners at A (0, 0, 0),
B (L, 0, 0), C(L, L, 0), D (0, L, 0), E(0, L, L) and F (0, 0, L). A
6. The self inductance L of a solenoid of length l and area of
magnetic field B = B0 ( ˆi + kˆ ) T is present in the region. The
cross-section A, with a fixed number of turns N increases

.IN
flux passing through the loop ABCDEFA (in that order) is as
(a) B0L2 Wb (b) 2B0L2Wb
(a) l and A increase
(c) Ö2B0L2 Wb (d) 4B0L2Wb
AL (b) l decreases and A increases
3. A cylindrical bar magnet is rotated about its axis. A wire is
(c) l increases and A decreases
connected from the axis and is made to touch the cylindrical
N
surface through a contact. Then, (d) both l and A decrease
(a) a direct current flows in the ammeter A
R

NEET/AIPMT (2013-2017) Questions


(b) no current flows through the ammeter A
U

7. A wire loop is rotated in a magnetic field. The frequency of


(c) an alternating sinusoidal current flows through the change of direction of the induced e.m.f. is [2013]
JO

2p
ammeter A with a time period T = (a) twice per revolution
w
(b) four times per revolution
(d) a time varying non-sinusoidal current flows through
U

the ammeter A. (c) six times per revolution


ED

4. There are two coils A and B as shown in figure a current (d) once per revolution
starts flowing in B as shown, when A is moved towards B 8. A current of 2.5 A flows through a coil of inductance 5 H.
and stops when A stops moving. The current in A is counter The magnetic flux linked with the coil is [NEET Kar. 2013]
clockwise. B is kept stationary when A moves. We can infer
(a) 2 Wb (b) 0.5 Wb
that
(c) 12.5 Wb (d) Zero
(a) there is a constant current in the clockwise direction
in A 9. A thin semicircular conducting ring (PQR) of radius ‘r’ is
falling with its plane vertical in a horizontal magnetic field
(b) there is a varying current in A
B, as shown in figure. The potential difference developed
(c) there is no current in A across the ring when its speed is v, is : [2014]
(d) there is a constant current in the counter clockwise
direction in A B
Q
A B

v
r
P R

5. Same as problem 4 except the coil A is made to rotate about


(a) Zero
a vertical axis (figure). No current flows in B if A is at rest.
The current in coil A, when the current in B (at t = 0) is (b) Bvpr2 /2 and P is at higher potnetial
counter-clockwise and the coil A is as shown at this instant, (c) prBv and R is at higher potnetial
t = 0, is (d) 2rBv and R is at higher potential
EBD_7179
582 PHYSICS

10. A conducting square frame of side ‘a’ and a long staight a


wire carrying current I are located in the same plane as
shown in the figure. The frame moves to the right with a
b d
constant velocity ‘V’. The emf induced in the frame will be
proportional to [2015]
X c
X electron Y
l (a) adcb
V (b) The current will reverse its direction as the electron
goes past the coil
(c) No current induced
a
(d) abcd

1 1 12. A long solenoid of diameter 0.1 m has 2 × 104 turns per


(a) (b) meter. At the centre of the solenoid, a coil of 100 turns and
(2x – a)2 (2x + a)2
radius 0.01 m is placed with its axis coinciding with the

.IN
solenoid axis. The current in the solenoid reduces at a
1 1
(c) constant rate to 0A from 4 A in 0.05 s. If the resistance of the
(2x – a)(2x + a) (d) x 2 AL coil is 10p2W. the total charge flowing through the coil during
11. An electron moves on a straight line path XY as shown. this time is :- [2017]
The abcd is a coil adjacent to the path of electron. What (a) 16 mC (b) 32 mC
will be the direction of current if any, induced in the coil? 16 p mC (d) 32 p mC
N
(c)
[2015 RS]
R
U
JO
U
ED
Electromagnetic Induction 583

Hints & Solutions


EXERCISE - 1 17. (b) Due to shifting of electrons, one end of the rod becomes
positive and the other end negative. This developes a
1. (a) 2. (c) 3. (c) electric field in the rod.
df dB A dB 18. (b)
4. (a) Induced e.m.f. e = = = A0
dt dt dt
W
æ 4 B 0 - B0 ö 19. (a) V = Q Þ W = Q V
= A0 ç ÷ = 3 A 0 B0 / t
è t ø 20. (b) It is because after every 1/2 revolution the current
5. (a) 6. (c) 7. (c) becomes zero and mode of change in flux changes
d dI NM I thereafter (If before the current becomes zero, the mode
8. (a) E = ( NMI ) Þ E = NM Þ E=
of flux change was from left to right then after the current
dt dt t
becomes zero the mode of flux change becomes right to
E MI
emf induced per unit turn = = left).
N t
9. (d) Initially flux, f = BA cos 0 = BA mn 2 A

.IN
21. (d) Self inductance of a solenoid =
After rotating through an angle 90°. l
Flux through the coil is zero. AL So, self induction µ n2
So, Df = BA So, inductance becomes 4 times when n is doubled.
2p
Angular speed = w, so, time period = =T 1 1 æ -d f ö 1
q = ò idt = ÷ dt = R ò d f

w 22. (c) edt = ò ç
N
R è dt ø
T
is time taken to rotate 90°. (taking only magnitude of e)
R

4
Δf BA 2BAω Hence, total charge induced in the conducting loop
U

So, = = depends upon the total change in magnetic flux.


Δt T /4 π
23. (a)
JO

10. (a) Farady's law states that time varying magnetic flux can
induce an e.m.f. 24. (b) e µ w
m0 N 2A N2 25. (b)
U

11. (b) From L = a EXERCISE - 2


l l
ED

L1 (1 / 2) 1. (c)
2
1
we get, = =
L2 1/ 2 2 2.0 ´ 0.5
2. (d) f = BA cos q = 2.0 ´ 0.5 ´ cos 60º = = 0.5 weber.
2
12. (a) The back e.m.f. in a motor is induced e.m.f., which is
maximum, when speed of rotation of the coil is maximum. df dB
13. (d) Mutual inductance between two coils depends on all the three 3. (b) e = =nA
dt dt
factors given here.
8 4
e 1 -1
\ e = 10 ´ (10 ´ 10-4 ) (104 ) (10 Gauss/sec=10 T/s)
14. (b) From e = LdI / dt, dI / dt = = = 1As
L 1 = 100 V.

f BA Ι = (e /R) = (100/ 20) = 5amp.


15. (b) Mutual inductance = = 4. (d) Total resistance of the circuit = 4000 + 400 = 4400 W
I I
V 440
[MT -1Q -1L2 ] Current flowing i = = = 0.1 amp.
[Henry] = = ML2 Q - 2 R 4400
[QT -1 ] Voltage across load = R i = 4000 × 0.1 = 400 volt.
ur ur
df d(NB.A) di é ( 4 - 2) ù
16. (d) e=- =- 5. (b) e = M or 8 = M ê ú
dt dt dt ë 0.05 û
d 8 ´ 0.05
= -N (BA cos wt ) = NBAw sin wt \ M= = 0.2 henry
dt 2
Þ e max = NBAw
EBD_7179
584 PHYSICS

18. (d) Since the magnetic field is uniform the flux f through the
1 1
6. (c) e.m.f. induced = BR 2 w = B R 2 (2 p n) square loop at any time t is constant, because
2 2 f = B × A = B × L2 = constant
1 df
= ´ (0.1) ´ (0.1) 2 ´ 2 p ´ 10 = (0.1)2 p volts \ e =- = zero
2 dt
7. (a) Change in flux = 2 B A N
19. (d) ε=B l v = (0.2 ´10-4 ) (1) (180 ´ 5 / 18) = 10-3 V = 1 mV
-4
2 ´ 0.3 ´ 200 ´ 70 ´10
\ Induced e.m.f. =
0.1 20. (b) B = m 0 n i = (4 p ´ 10-7 ) (200 ´10 -2 ) ´1.5
= 3.8 × 10–2 Wb / m2
8. (c) e = (5 ´10 -3 ) (1 / 0.1) = 0.05 V .
Magnetic flux through each turn of the coil
9. (c) Energy stored U is given by
f = BA = (3.8 × 10–2) (3.14 × 10–4) = 1.2 × 10–5 weber
1 2 1
U= L i = ´ (100 ´ 10 -3 ) (1) 2 = 0.05 J. When the current in the solenoid is reversed, the
2 2 change in magnetic flux
10. (d)
= 2 ´ (1.2 ´ 10-5 ) = 2.4 ´10 -5 weber
2 df
11. (d) f = 6t + 7 t + 1 Þ = 12t + 7
dt df 2.4 ´ 10 -5
Induced e.m.f. = N = 100 ´ = 0.048 V.

.IN
At time, t = 2 sec. dt 0.05

df M
= 24 + 7 = 31 volt 21. (b) ε=
AL dI=0.005×I 0 cos ωt× ω
dt dt
Direction of current is from left to right according to and e max = 0.005 × I0 × w = 5 p
Flemmings right hand rule.
22. (b) l = 1m, w = 5 rad/s, B = 0.2 ´ 10 -4 T
N
df d dB
12. (b) e= = ( NBA ) = NA = 500×10–2 × 1 = 5.0 V -4
R

dt dt dt Bωl 0.2 ´ 10 ´ 5 ´1
e= = = 50mV
2 2
U

1
13. (a) Vel. of coil = = 2m / s 23. (d) When two inductance coil are joined in series, such that the
0.5
JO

winding of one is exactly opposite to each other the emf


2 produced in the two coils are out of phase such that they
velocity of magnet = = 2m / s.
cancel out.
1
U

As they are made to move in the same direction, their - df


relative velocity is zero. Therefore, induced e.m.f. = 0. 24. (b) The individual emf produced in the coil e =
ED

dt

14. (a) e=
- df -d 2
dt
=
dt
( )
6t - 5t + 1 = -12t + 5 \ The current induced will be i =
|e|
R
Þi=
1 df
R dt
e = – 12 (0.25) + 5 = 2 volt
dq dq 1 df 1 BA
e 2 But i = Þ = Þ ò dq = ò df Þ q =
i = = = 0.2A. dt dt R dt R R
R 10 25. (c) E.m.f. will remain same because change in area per unit
-3 time will be same in both cases.
LdI 40 ´10 (11 - 1)
15. (a) e= = = 100V 2 2
dt 4 ´10-3 26. (b) Self inductance = m 0 n AL = m 0 n (l ´ b) ´ L
16. (d) For maximum power, X L = X C , which yields n = Total number of turns/length
L = Length of inductor
1 1 l = Length of rectangular cross section
C= =
2 2 b = breadth of rectangular cross-section
( 2pn ) L 4 p ´ 50 ´ 50 ´ 10
So, when all linear dimensions are increased by a factor
\ C = 0.1´ 10 -5 F = 1mF of 2. The new self inductance becomes L' = 8L.
17. (b) f = 10t 2 - 50t + 250 e 1 df
27. (c) i = =
R R dt
df
e=- = - (20 t - 50)
dt Here df = B × A = (2 ´ 10 -5 ) ´ (0.5 ´ 10+3 ´ 5)
e t =3 = -10 V dt = time taken by the wire to fall at ground
Electromagnetic Induction 585

= (2 h / g)1/ 2 = (10 /10)1/ 2 = 1sec. 220


or L= =11 H
20
1 é (2 ´ 10 -5 ) ´ (0.5 ´ 103 ´ 5) ù
\ i= ê ú = 0.02 amp. (where L = Self inductance of coil)
2.5 ëê 1 ûú 38. (c) L = 2mH, i = t2e–t
28. (b) Length of conductor (l) = 0.4 m; Speed (v) = 7 m/s and di
E= -L = - L[ - t 2 e - t + 2 te - t ]
magnetic field (B) = 0.9 Wb/ m 2. Induced e.m.f. dt
(e) = Blv cos q = 0.9 × 0.4 × 7 × cos 0º = 2.52 V. when E = 0
29. (d) The given circuit clearly shows that the inductors are in –e–t t2 + 2te–t = 0
1 1 1 1 2t e–t = e–t t2
parallel we have, = + + or L= 1H t = 2 sec.
L 3 3 3
Df Df
e = Blv = 2 ´ 10 -1 ´ v = 0.2 v
39. (c) = e = iR Þ Df = (iDt )R = QR Þ Q =
30. (c) Dt R
e 0.2v (b) Magnetic flux linked with the loop is f = Bpr 2
I= = 10 -3 Þ = 10 -3 40.
R 4
df dr
6´6 | e |= = Bp × 2 r
[Since effective resistance R of bridge is R = = 3W

.IN
dt dt
6+6
dr
so total resistance = 1 + 3 = 4W] When r = 2 cm, = 1 mm s–1
dt
Þ v = 2 cm s–1
AL
e = 0.025× p ×2 ×2 ×10–2×10–3
31. (c) An opposite current induced in B in accordance to Lenz's
= 0.100 × p × 10–5 = p × 10–6 V = pmV
law. So the two loops repel each other.
N
32. (d) f = n BA cos q = 10 B a2 cos wt di
41. (a) e = -L
R

dt
e=-
df
=-
d
( )
10 B a 2 cos wt = 10 B a 2 sin wt ( w ) . T di
U

dt dt During 0 to , = const.
4 dt
JO

33. (d) As resistance of 1 H coil is zero, the entire current flows \ e = – ve


through the coil. Current through 10W resistance is zero.
T T di
34. (c) Capacitor is a dc blocking element and hence no current During to , = 0
U

flow in (1). 4 2 dt
\e=0
ED

An inductor offers a zero resistance path to flow of dc


and hence maximum current flows through (2). T 3T di
During to , = const.
2 4 dt
N 2000 20000
35. (b) n= = = \ e = +ve
l 0.3 3
Thus graph given in option (a) represents the variation
d dB of induced emf with time.
x= ( NBA ) = NA 42. (b) The charge through the coil = area of current-time
dt dt
(i – t) graph
Since B = µ0nI
1
dt q = ´ 0.1 ´ 4 = 0.2 C
Þ x = ( mNAn ) Þ x = 0.024V 2
dt
Df
q= Q Change in flux (Df) = q × R
m N N A 4p´ 10-7 ´ 300 ´ 400 ´ 100 ´ 10-4 R
36. (d) M= 0 1 2 =
l 0.2
Df
q = 0.2 =
-4 10
= 2.4p ´ 10 H
Df = 2 weber
37. (c) Initial current (I1) = 10 A; Final current (I2)= 0; Time (t) 43. (a) According, to Faraday’s law of induction
= 0.5 sec
and induced e.m.f. (e) = 220 V. df
Induced e.m.f. e = - = - (100t )
(I - I ) dt
dI (0 - 10)
-L = -L 2 1 = -L = 20L Induced current i at t = 2 sec.
dt t 0.5
EBD_7179
586 PHYSICS

e 100 ´ 2 and B = B0 ( ˆi + kˆ )
= =+ = + 0.5Amp
R 400
Now, f = B.A = B0( iˆ + kˆ ) × ( L2 kˆ + L2ˆi )
44. (c) By inserting iron rod in the coil,
= 2 B0L2 Wb
L ­ z ­ I ¯ so brightness ¯ 3. (b) Induced current flow only when circuit is complete
-(f 2 - f1 ) -(0 - NBA) NBA and there is a variation about circuit this problem is
45. (b) e= = =
t t t associated with the phenomenon of electromagnetic
induction.
NBA 50 ´ 2 ´ 10 –2 ´ 10 –2
t= = = 0.1 s If there is a symmetry in magnetic field of cylindrical
e 0.1 bar magnet is rotated about its axis, no change in flux
nAdB linked with the circuit takes place, consequently no
e emf induces and hence, no current flows in the ammeter
46. (c) i = = dt
R R (A).

20 ´ (25 ´10 -4 ) ´1000


= = 0.5A Axis
100
A
47. (b) N
48. (a) Lenz's law (that the direction of induced emf is always

.IN
Bar
such as to oppose the change that cause it) is direct w
magnet
consequence of the law of conservation of energy.
49. (b) 50. (b)
AL
EXERCISE - 3
N
Exemplar Questions S
w
R

1. (c) As we know that, the magnetic flux linked with uniform


surface of area A in uniform magnetic field is
U

f = B.A 4. (d) When the coil A stops moving the current in B b ecome
The direction of A is perpendicular to the plane of ze ro, it possible only if the current in A is constant. If
JO

square and square line in x-y plane in a region. the current in A would be variable, there must be an
A = L2k induced emf (current) in B even if the A stops moving.
So there is a constant current in same direction or
As given that, B = B0 ( 2iˆ + 3jˆ + 4kˆ )
U

counter clockwise direction in A as in B by lenz's law.


5. (a) By Lenz's law, at (t = 0) the current in B is counter-
f = B.A= B0 ( 2iˆ + 3jˆ + 4kˆ ) .L kˆ
ED

2
So, clockwise and the coil A is considered above to it. The
= 4B0 L2 Wb counterclockwise flow of the current in B is equivalent
to north pole of magnet and magnetic field lines are
2. (b) The loop can be considered in two planes, Plane of emanating upward to coil A.
ABCDA lies x-y plane whose area vector A1 = |A| k̂ , When coil A start rotating at t = 0, the current in A is
A1 = L2 k̂ constant along clockwise direction by Lenz’s rule. As
whereas plane of ADEFA lies in y-z plane whose area flux changes across coil A by rotating it near the N-
vector A2 = |A| î , A2 = L2 î . pole formed by flowing current in B, in anticlockwise.
Then the magnetic flux linked with uniform surface of 6. (b) The self-inductance of a long solenoid of cross-
area A in uniform magnetic field is sectional area A and length l, having n turns per unit
length, filled the inside of the solenoid with a material
Y of relative permeability is given by
(0,L,0) D (L,L,0) L = µrµ0n2 Al
C
E L \ n = N/l
(0,L,L) L
L
A B (L,0,0)
X é N 2 .A ù
L = µrµ0 ê ú .l
F
(0,0,0)
ë l.l û
Z (0,0,L)
æ 1ö
f = B.A L = µrµ0 [N2A/l] çè L µ A, L µ ÷ø
l
A = A1 + A2 = ( L2 kˆ + L2 iˆ ) As µr and N are constant here so, to increase L for a
coil, area A must be increased and l must be decreased.
Electromagnetic Induction 587

NEET/AIPMT (2013-2017) Questions e = B1Vl – B2Vl


7. (a) This is the case of periodic EMI m0 I m0 I
= 2p (x – a / 2) lv – 2 p (x + a/ 2) lv
E
1
or, e µ
t (2x – a)(2 x + a)
11. (b) Current will be induced,
when e– comes closer the induced current will be
anticlockwise
From graph, it is clear that direction is changing when e– comes farther induced current will be
1 clockwise
once in cycle.
2
8. (c) Given: current I = 2.5 A
Inductance, L = 5H
Magnatic flux, f = ? e– e–
We know, f = LI Þ 5 × 2.5 Wb = 12.5 Wb
9. (d) Rate of decreasing of area of semicircular ring
12. (b) Given, no. of turns N = 100
dA
= = (2r)V radius, r = 0.01 m

.IN
dt resistance, R = 10p2 W, n = 2 × 104
From Faraday’s law of electromagnetic induction As we know,
dq dA
e= - = -B = - B(2rV)
AL df
dt dt e = -N
dt
N
e N df
=-
R R dt
R

N df
DI = -
U

As induced current in ring produces magnetic field in R dt


upward direction hence R is at higher potential.
JO

Dq N Df
=-
10. (c) Emf induced in side 1 of frame e1 = B1Vl Dt R Dt
mo I
U

B1 = é N æ Df ö ù
2p (x – a/ 2) Dq = - ê ç ÷ ú Dt
ë R è Dt ø û
ED

Emf induced in side 2 of frame e2 = B2 Vl


'–' ve sign shows that induced emf opposes the
moI
B2 = change of flux.
2p (x + a/ 2)
é æ Di ö ù 1 m nNpr 2 Di
x Dq = ê m0 nNpr 2 ç ÷ ú Dt = 0
ë è Dt ø û R R
I
1 2
4p ´ 10-7 ´ 100 ´ 4 ´ p ´ (0.01)2 ´ 2 ´ 104
a v Dq =
x– 10p2
2
a Dq = 32mC
a
x+
2
Emf induced in square frame
EBD_7179
588 PHYSICS

22 Alternating Current

ALTERNATING AND DIRECT CURRENT The root mean square (rms) value of AC is
An alternating current (A.C.) is one which periodically changes I0
in magnitude and direction. It increases from zero to a maximum I rms =
value, then decreases to zero and reverses in direction, increases 2

.IN
to a maximum in this direction and then decreases to zero. where I0 is the peak or maximum value of alternating current.
The source of alternating emf may be a dynamo or an electronic The rms value of alternating current can also be defined as the
oscillator. direct current which produces the same heating effect in a given
AL
The alternating emf E at any instant may be expressed as resistor in a given time as is produced by the given A.C. flowing
E = E0 sinwt through same resistor for the same time. Due to this reason the
where w is the angular frequency of alternating emf and E0 is the rms value of current is also known as effective or virtual value of
N
peak value of emf. current.
R

I I0 I0
(A.C.) \ Ieffective = I virtual = Irms =
U

2
Similarly the rms value of alternating voltgae is called the effective
JO

T=0 T T 3T t or virtual value of alternating voltage (or emf).


— –—
2 2
E0
\ E effective = E virtual = E rms =
U

Direct current (D.C.) is that current which may or may not 2


ED

change in magnitude but it does not change its direction. Keep in Memory
(1) Time period : The time taken by A.C. to go through one
cycle of changes is called its period. It is given as T = p
I 2
(D.C.) w
(2) Phase : It is that property of wave motion which tells us
the position of the particle at any instant as well as its
t
direction of motion. It is measured either by the angle which
Advantages of A.C. over D.C. the particle makes with the mean position or by fraction of
(i) The generation of A.C. is cheaper than that of D.C. time period.
(ii) Alternating voltage can be easily stepped up or stepped (3) Phase angle : Angle associated with the wave motion (sine
down by using a transformer. or cosine) is called phase angle.
(iii) A.C. can be easily converted into D.C. by rectifier. D.C. is (4) Lead : Out of the current and emf the one having greater
converted to A.C. by an inverter. phase angle will lead the other e.g., in equation
(iv) A.C. can be transmitted to a long distance without æ pö
i = i0 sin ç wt + ÷ and e = e0 sin wt,
appreciable loss. è 2ø
AVERAGE AND RMS VALUE OF ALTERNATING p
the current leads the emf by an angle .
CURRENT 2
The average value of AC over one full cycle is zero since there are (5) Lag : Out of current and emf the one having smaller phase
equal positive and negative half cycles. angle will lag the other. In the above equations, the emf
The average current for half cycle is 2I0 /p where I0 is the peak p
lags the current by .
value of current. 2
Alternating Current 589

RESISTANCE OFFERED BY VARIOUS ELEMENTS Graph of emf or current versus wt :


(INDUCTOR, RESISTOR AND CAPACITOR) TO A.C.
Alternating current in a circuit may be controlled by resistance,
inductance and capacitance, while the direct current is controlled E
or E
only by resistance.
I I
(i) Impedance (Z) : In alternating current circuit, the ratio of
emf applied and consequent current produced is called wt
the impedance and is denoted by Z,
E E Circuit Contianing only Inductor (L)
i.e., Z = E = 0 = rms
I I0 I rms Consider a pure inductor (zero ohmic resistance) of inductance L
Physically impedance of ac circuit is the hindrance offered connected to an alternating source of emf E = E0 sin wt.
L
by resistance along with either inductance or capacitance or
both in the circuit to the flow of ac through it. Its unit is ohm.
(ii) Reactance (X) : The hindrance offered by inductance or
capacitance or both to the flow of ac in an ac circuit is
called reactance and is denoted by X. Thus when there is E = E0 sin w t
no ohmic resitance in the cirucit, the reactance is equal to Then current I in the circuit is
impedance. The reactance due to inductance alone is called æ pö E0

.IN
inductive reactance and is denoted by X L, while the I = I 0 sin ç wt - ÷ where I 0 =
è 2ø wL
reactance due to capacitance alone is called the capacitive
reactance and is denoted by XC. Its unit is also ohm. Comparing this with standard equation, we get
AL
(iii) Admittance (Y) : The inverse of impedance is called the Z = w L and phase difference f = p/2.
1
Hence we conclude that in a purely inductive circuit the current
admittance and is denoted by Y, i.e., Y = lags behind the applied voltage by an angle p/2 and the
N
Z
impedance to the circuit is wL and this is called as inductive
Its SI unit is ohm–1.
R

reactance.
IMPEDANCES AND PHASES OF AC CIRCUIT
Graph of emf or current versus wt
U

CONTAINING DIFFERENT ELEMENTS


JO

As already pointed out that in an ac circuit the current and applied E


emfs are not necessarily in same phase. The applied emf (E) and E I
current produced (I) may be expresed as or
I wt
E = E0 sin wt and I = I0 sin (wt + f) with I0 = E0 / Z
U

where E0 and I 0 are peak values of alternating emf and current.


ED

Circuit Containing only Resistor (R) Phasor diagram Graph between XL and f
Consider a pure ohmic resistor (zero inductance) of resistance R O E
connected to an alternating source of emf E = E0 sinwt.
X XL
90º
R
I
f
Y
E = E0 sin w t Circuit Containing only Capacitor
Consider a capacitor of capacitance C connected to an alternating
Then current I in the circuit is
source of emf, E = E0 sin wt.
E E 0 sin wt C
I= = = I0 sin wt , where I0 = E0 / R
R R
Comparing this with standard equation, we get that VC
impedance of circuit, Z = R and phase difference between current
& emf = 0.
Hence we conclude that in a purely resistive ac circuit the current E = E0 sin w t
and voltage are in same phase and impedance of circuit is equal Then the current through capacitor is given by,
to the ohmic resistance.
æ pö
Phasor diagram : X I = I0 sin ç wt + ÷
I E è 2ø
EBD_7179
590 PHYSICS

Comparing this with standard equation, we find that capacitive E


\ = é R 2 + (wL )2 ù
reactance XC = 1/wC and phase difference f = + p/2 I ë û
Phasor diagram Graph between XC and f \ Impedance of R – L circuit,
IC E
XC Z = = ( R 2 + X L2 ) where XL = wL
p /2 I
It is obvious that the current lags behind the emf by angle f
given by,
VC
Hence we conclude that in a purely capacitive circuit the current
f
f = tan
–1 æ VL ö
è VR ø ( )
= tan –1 X L = tan –1 æç wL ö÷
R è ø R
leads the applied emf by an angle p/2 and the impedance of the
Circuit Containing Resistance and Capacitance in
circuit is 1/ wC and this is known as capacitive reactance
Series (C–R Series Circuit)
1 . Consider a circuit containing resistance R and capacitance C in
Z = XC =
wC series having an alternating emf E = E0 sin wt.
Graph of emf or current versus wt
R C

E E VR VC
I
or

.IN
I 3p
p/2 2p wt
AL E = E0 sin w t
Circuit Containing Resistance and Inductance in Let I be the current flowing in the circuit, VR the potential
Series (LR Series Circuit) difference across resistance and VC the potential difference across
Consider a circuit containing resistance R and inductance L in capacitance.
N
series having an alternating emf E = E0 sin wt. Phasor diagram
R

R L VR
I
f
U

VR VL
JO

E = E0 sin w t E
VC
Let I be the current flowing in the circuit and VR (= IR) the potential
U

difference across resistance and V L (= wL.I) the potential From phasor diagram the resultant emf is given by
ED

difference across inductance. E = ( VR2 + VC2 ) = ( RI ) 2 + ( X C I ) 2 ]


The current I and the potential difference VR are always in phase
but the potential difference VL across inductance leads the current \ Impedance, Z = E / I = (R 2 + X 2C ) , where
I by an anlgle p/2. æ 1 ö
XC = ç ÷
Phasor diagram è wC ø
E The potential difference VR and current I are in same phase and
E= V 2R + V 2L
the potential difference VC lags behind the current I (and hence
VL VR) by angle p/2
The current leads the applied emf by an angle f given by
f V X I X
tan j = C = C = C
I VR RI R
VR
From phasor diagrom, resultant voltage is given by, æX ö æ 1/ wC ö æ 1 ö
or tan f = ç C ÷ Þ f = tan -1 ç = tan -1 ç
è R ø è R ÷ø è wCR ÷ø
2 2 2 2
E= (VR + VL ) = (RI ) + (wL.I ) Graph of emf or current versus wt
Graph of emf or current versus wt emf
emf
Current E
E
or
or I
I wt
wt
Current
Alternating Current 591

Circuit Containing Inductance and Capacitance in Phasor diagram :


Series (Series LC Circuit)
Consider a circuit containing inductance L and capacitance C in VL
series having an alternating emf
E = E0 sin wt. VR
f I
VC E VC – VL (if VC > VL)
L C

VL VC

The p.d. VR is in phase with current I . The p.d. VC lags behind


the current by angle p/2. The p.d. VL leads the current by angle p/
E = E0 sin w t 2.

Let I be the current flowing in circuit, VL the potential difference \ Resultant applied emf, E = [VR2 + ( VC - VL ) 2 ]
across inductance L and VC the p.d. across capacitance C.
Phasor diagram :
i.e., E= {(R I) 2
+ (I X C - I X L ) 2 }
Y O X VL

{ }

.IN
VL > VC VC – VL E
E p/2 \ Impedance, Z = = R2 + ( X C - X L ) 2
E=VC–VL I I
p/2
VL – VC The phase leads of current over applied emf is given by
VC > VL
X Y' VC
AL
O VC - VL I X C - I X L X C - X L
The p.d. VC lags behind the current by angle p/2 and the p.d. VL tan j = = =
VR RI R
N
leads the current by angle p/2.
\ Resultant applied emf, E = VC – VL = XCI – XLI æ X - XL ö
i.e., j = tan -1 ç C ÷ø
R

\ Reactance of circuit, è R
U

æ 1 ö It is concluded that :
X = E / I = XC - X L = ç - wL÷
è wC ø (a) If XC > XL, the value of f is positive, i.e., current leads the
JO

The current leads applied emf by f = p / 2 . applied emf.


(b) If XC < XL, the value of f is negative, i.e., current lags
1 1
U

In case of XC = XL, Z = 0, then = wL or w = behind the applied emf.


wC ( LC ) (c) If XC = XL, the value of f is zero, i.e., current and emf are in
ED

1 same phase. This is called the case of resonance and


\ Frequency f = w / 2p == resonant frequency for condition XC = XL, is given by :
2p ( LC )
At certain frequency the impedance of the circuit is minimum and Z
E/R
the current is maximum.
Current
This frequency is called the resonant frequency.
E
Circuit Containing Resistance, Inductance and
2R
Capacitance in Series (Series LCR Circuit)
Consider a circuit containing a resistance R, inductance L and R
f
capacitance C in series having an alternating emf fo O fo
E = E0 sin wt. Frequency
R L C 1 1
= wL i.e., w =
wC LC
VR VL VC 1
\ fo = w / 2p = 2 p ( LC ) .
Thus the resonant frequency depends on the product of L
E = E0 sin w t and C and is independent of R.
At resonance, impedance is minimum, Zmin = R and current
Let I be the current flowing in circuit. VR, VL and VC are respective E E
potential differences across resistance R, inductance L and is maximum I max = =
capacitance C. Z min R
EBD_7179
592 PHYSICS

Z R 2 + w 2 L2 L
The impedance at resonance, Z = =
R RC
In parallel resonant circuit the impedance is maximum and the
current is minimum.
Zmin = R 1
R If R ® 0 , then f r = and Z ® ¥ .
2p (LC)
w Q - FACTOR
w =w 0=2p f0
Resonance frequency The sharpness of tuning at resonance is measured by
Circuit impedence in series RLC circuit Q-factor or quality factor of the circuit and is given by
Rapid fall of current 1 L
in A in comparison Q=
R C
to B curve
I Higher the value of Q-factor, sharper is the resonance i.e. more
Imax rapid is the fall of current from maximum value (I0) with slight
Small R higher change in frequency from the resonance value.
A Q i.e., sharper resonance
B It is clear from the figure that at low value of q, the resonance is
High R, small Q i.e., poor. However the bandwidth increases
no sharp resonance
I High Q

.IN
w
w =w0=2pf0
Resonance frequency AL
Current amplitude in series RLC circuit Low Q
It is interesting to note that before resonance the current
leads the applied emf, at resonance it is in phase, and after
w0
N
resonance it lags behind the emf. LCR series circuit is also w
called as acceptor circuit and parallel LCR circuit is called The figure given below explains the concept of bandwidth and
R

rejector circuit. cut-off frequency.


Imax
U

COMMON DEFAULT
O Incorrect. Adding impedances / reactances /resistors
JO

0.707 Imax 0.707 Imax


algebrically. Band width

P Correct. For these physical quantities, vector additon must


be done
Lower cut off
frequency
Upper cut -off frequency
U

O Incorrect. Kirchoff's laws are applicable in D.C. circuit only


P
ED

Correct. Kirchoff's laws are applicable in A.C. circuit also


(which may include inductor and capacitor). w0 w
PARALLEL RESONANT CIRCUIT when w < w0 XC < XL
A parallel resonant circuit consists of an inductance L and a when w > w0 XL > XC
capacitance C in parallel as shown in fig. when w = w0 XC = X L
R L
Bandwidth : It is the band of allowed frequencies and is defined
C as the difference between upper and lower cut-off frequencies,
the frequency at which power becomes half of maximum value
and current becomes Imax / 2 .
POWER IN AN A.C. CIRCUIT
E=E0 sin w t
The condition of resonance is again that the current and applied The power is defined as the rate at which work is being done in
emf must be in same phase. The condition gives angular resonant the circuit. In ac circuit, the current and emf are not necessarily
frequency. in the same phase, therefore we write
E = E0 sin wt & I = I0 sin (wt + f).
1 R2 The instaneous power, P = EI
wr = -
LC L2 = E0 sin wt I0 sin (wt + f),
wr 1 1 R2 The average power Pav = Erms Irms cos f
\ Resonant frequency f r = = -
2p 2p LC L2 E0 I 0
\ Pav = cos f
2 2
Alternating Current 593

In this expression cos f is known as power factor. The value of 2. For resonance to occur, the presence of both L and C
cos f depends on the nature of the circuit. For L, C and elements in the circuit is a must.
L-C circuit, the power factor is zero ( Q f = 90º); for R-circuit 3. In series resonant circuit, current is maximum at resonance.
cos f = 1 (Q f = 0) and for all other circuit cos f = R/Z, where
In a parallel resonant circuit, current is minimum (or zero)
Z = impedance.
at resonance but p.d across the combination is maximum.
If R = 0, cos f = 0 and Pav = 0 i.e., in a circuit with no resistance, the
power loss is zero. Such a circuit is called the wattless circuit 4. To depict oscillatory motion mathematically we may use
and the current flowing is called the wattless current. sines, cosines or their linear combination. This is because
Power is of two types changing the zero position transforms one into another.
5. While adding voltage across different elements in an a.c.
(i) Reactive power Preactive = Vrms I rms sin f circuit we should take care of their phases.
This is also called wattless power. 6. The average current over a complete cycle in an a.c circuit
It is not read by energy meter is zero but the average power is not zero.
(ii) Active power Pactive = Vrms I rms cos f 7. An inductor offers negligibly low resistance path to d.c.
It is read by energy meter and a resistive path for a.c.
8. A capacitor acts as a block for d.c and a low resistance path
Half Power Points to a.c.
The values of w at which the average power is half of its maximum

.IN
value (at resonant frequency) are called half power points.
9.
AL Inductive reactance Capacitive reactance
X L = wL = 2pfL 1 1
Pav XC = =
Þ XL µ f wC 2pfC
Small R 1
N
higher Q Þ XC µ
Dw f
R

Current through pure Current through pure


Large R,
inductor lags behind capacitor leads
U

low Q
emf by 90° emf by 90°
JO

w 1 w0= wr w 2 w For d.c f = 0 \ X L = 0 For d.c f = 0 Þ X C = ¥


For a.c as f increases For a.c as f increases
Plot of average power versus frequency for a series RLC circuit.
U

X L increases XC decreases
The upper curve is for a small R & lower broad curve is for large
ED

value of R.
It is clear from the figure that for smaller R, value of Q0 is high (Q0
is Quality factor of circuit) & hence sharper resonance i.e. greater
rate of fall of average power maximum average power Pav changes 10. Series Resonant circuit Parallel resosant circuit
with slight change in frequency from resonant frequency. 1 1
XL = XC =
w0 X L XC
The Quality factor, Q0 is defined as, Q0 =
Dw 1 1 1 R2
nr = nr = -
Where Dw = w2 – w1 and w2 & w1 are half power points. 2 p LC 2 p LC L2
2
R w0 L 1 1 æ 1 ö
Now, since Dw » ; so Q 0 » Z = R 2 + (X L - X C )2 = + Cw -
2 ç ÷
L R Z R è Lw ø
wo w
Whereas w1 = w0 - ; w2 = w0 + 0
Q0 Q0 11. The principle of electric meter is heating effect of current.
These meters give the reading of Irms. It is important to note
æ 1 ö that these meters can measure D.C. as well as A.C.
In concise term, we can write as, w = w r çç1 ± ÷
÷
è Q0 ø 12. D.C. flows through the cross-section of the conductor
whereas A.C. flows mainly along the surface of the
Keep in Memory conductor. This is also known as Skin Effect. The skin
effect is directly proportional to the frequency.
1. Unless mentioned otherwise, all a.c. currents and voltages
are r.m.s. values.
EBD_7179
594 PHYSICS

Example 1. Solution : (d)


Calculate the r.m.s. value of e.m.f. given by
E = 8 sin w t + 6 sin 2 w t volts. P 10 1 V 2 60 ´ 60
I= = = A; R= = = 360 W ;
Solution : V 60 6 P 10

The mean square value is given by E = E 2 V 100


Z= = = 600W
Ι 1/ 6
\ E 2 = (8 sin w t + 6 sin 2 w t ) 2
X 2L = Z 2 - R 2 = 6002 - 360 2 = (600 + 360) (600 - 360)
2 2
= 64sin w t + 96sin w t .sin 2 w t + 36sin 2 w t
X L = 960 ´ 240 = 240 ´ 2 = 480 W
1 1
We know that sin 2 w t = , sin 2 2 w t = , and wL = 2pnL = X L = 480
2 2
XL 480
sin w t . sin 2 w t = 0 L= = = 1.28 H
2pn 120 p
1 1
\ E 2 = 64 ´ + 96 ´ 0 + 36 ´ = 32 + 18 = 50 Example 5.
2 2 An a.c. circuit consists of only an inductor of inductance
2H. If the current is represented by a sine wave of ampli-
or E r.m.s. = (E 2 ) = 50 = 7.07 volt tude 0.25 amp. and frequency 60 Hz, calculate the effective
Example 2. potential difference across the inductor.

.IN
When 100 volt D.C. is applied across a solenoid a current Solution :
of 1.0 amp flows in it. When 100 volt A.C. is applied across The effective potential difference across the inductor is
the same coil, the current drops to 0.5 amp. If the frequency given by
of the A.C. source is 50 Hz, then determine the impedance
AL
I0
and inductance of the solenoid. Veff = Ieff. XL = .2 p f L; Veff = Vrms
Solution : 2
N
In case of D.C., w = 0 and hence Z = R Given that I0 = 0.25 amp, f = 60 Hz, L = 2H
R

E 100 0.25
\ Z= R= = = 100 Ω \ Veff = × 2 × 3.14 × 60 × 2 = 133.2 Volt
I 1 2
U

1/2 Example 6.
For A.C., Z = éëR 2 + (2 p n L) 2 ùû
JO

If a domestic appliance draws 2.5 A from a 220-V, 60- Hz


2 2 1/ 2
A.C. power supply, then find
= [(100) + (100 p L) ] (a) the average current
(where w = 2pn & n is frequency of AC source)
U

(b) the average of the square of the current


(c) the current amplitude
\ 200 = [(100) 2 + (100 p L) 2 ]1 / 2
ED

(d) the supply voltage amplitude.


ì 100 ü Solution :
íQ Z = = 200 W ý Solving we get L = 0.55 henry (a) The average of sinusoidal AC values over any whole
î 0.5 þ number of cycles is zero.
Example 3. (b) RMS value of current = Irms = 2.5 A
A coil has an inductance of 0.7 henry and is joined in
series with a resistance of 220 W. When an alternating \ (I2 )av = (I rms )2 = 6.25 A 2
e.m.f. of 220 V at 50 cycles per second, is applied to it, then
Im
what will be the wattless component of current in the (c) Irms =
circuit? 2
Solution :
\ Current amplitude = 2Irms = 2(2.5A) = 3.5 A
Here, XL = wL = 2 p n L = 2 p × 50 × 0.7 × = 220 W
R = 220 W Vm
(d) Vrms = 220V =
Z = R 2 + X 2L = 220 2 + 220 2 = 220 2 ohm. 2
\ Supply voltage amplitude
\ wattless component of current is
EO 220 1 Vm = 2(Vrms ) = 2(220V ) = 311 V..
I= = = = 0.707A Example 7.
Z 220 2 2
A 100 mF capacitor in series with a 40 W resistance is
Example 4. connected to a 110 V, 60 Hz supply.
A 60 volt-10 watt bulb is operated at 100 volt-60 Hz a.c. (a) What is the maximum current in the circuit?
The inductance required is (b) What is the time lag between current maximum and
(a) 2.56 H (b) 0.32 H voltage maximum?
(c) 0.64 H (d) 1.28 H
Alternating Current 595

Solution :
E
(a) Here, C = 100 mF = 100 × 10–6 F, R = 40 W,
Vrms = 110 V, f = 60 Hz K
Peak voltage, V0 = 2 . Vrms = 100 2 = 155.54 V
I
1
Circuit impedance, Z = R2 + a
b L
c
2 2
w C R
dI
2 1 so if rate of change of current with time =
= 40 + dt
(2 ´ p ´ 60 ´ 100 ´ 10 -6 ) 2 then due to phenomenon of self induction, induced emf across
dI
= 1600 + 703.60 = 2303.60 = 48 W inductance = - L
dt
Hence, maximum current in coil, Potential difference across the resistance = IR
During growth of current in L-R circuit, if we applying Kirchhoff’s
V0 155.54 loop rule then
I0 = = = 3.24 A
Z 48 æ dI ö
(b) Phase lead angle (for current), E + ç - L ÷ = IR
è dt ø
On solving it we get the value of current at any time t during

.IN
1 1
q = tan -1 = tan -1 æ - tö
R
wCR 2 ´ 3.14 ´ 60 ´ 100 ´ 10-6 ´ 40
growth of current in LR-circuit. I = I0 ç1 - e L ÷
= tan–1 0.66315 = 33° 33’ (taken 33.5°) çè ÷ø
AL
q q 33.5 Graph showing how current varies with time
Time lead, t = = = = 0.001551 sec I
w 2 p n 360 ´ 60
N
I0= E/R
= 1.551 × 10–3 sec
R

Voltage will lag current by = 1.551 ms. 0.63 I0


U

Example 8.
JO

30.0 µF capacitor is connected to a 220 V, 50 Hz source.


Find the capacitive reactance and the current (rms and t
tL
peak) in the circuit. If the frequency is doubled, what
Time Constant
U

happens to the capacitive reactance and the current?


L
Solution : has dimensions of time. It is called inductive time constant of
ED

R
1 LR-circuit.
The capacitive reactance is XC = = 106W R L
2 pfC L - . æ e - 1ö
At t = ; I = I0 (1 - e L R ) = I0 (1 – e–1) = I0 ç
Vrms R è e ÷ø
The rms current is Irms = = 2.08A
XC æ 2.71 - 1ö
= I0 çè ÷ = 0.632 I0
2.71 ø
The peak current is Im = 2Irms = 2.96A The inductive time constant of an LR-circuit is the time in which
the current grows from zero to 0.632 (or 63.2%) of its maximum
This current oscillates between 2.96A and – 2.96A and is value. When t ® ¥.
ahead of the voltage by 90º.
æ - .¥ ö
R
If the frequency is doubled, the capacitive reactance is
halved and consequently, the current is doubled. è ø
(
I = I0 ç1 - e L ÷ = I0 I - e -¥ = I 0 (1 - 0 ) )
VARYING CURRENT Potential difference across resistance :
æ - tö
R
dI
When the key in a D.C. circuit (containing a D.C. source of emf, VR = E ç1 - e L ÷ ; VL = L E
inductance coil, resistance and capacitor) is closed or opened, è ø dt
the current in the circuit varies. This is known as varying current -
R
t VL
as it varies w.r.t. time and takes a final value after a short while. I = I0 - I0 e L ;
Growth of Current R
dI - t æ Rö t
If K is closed at t = 0 so at t = 0, current in the circuit I = 0 = 0 – I0 e L ç - ÷
dt è Lø
After closing the key K at time t let current in the circuit = I R
- t
and for small time in the circuit, current varies with time, VL = E e L
EBD_7179
596 PHYSICS

Initially, an inductor acts to oppose changes in the current through R dI R


it. A long time later, it acts like ordinary connecting wire. or V = e- L t VL = L I = I0 -
L
t
R dt e
DECAY OF CURRENT R
- t
dI æ Rö R
Let the current has reached its steady state value I0 through = I0 e L çè - ÷ø or V = - E e- L t
dt L L
inductor. Now switch K in the circuit shown in fig. has been
closed. 0 t

VL

Let this time is t = 0.


Let at t = 0 current in the circuit (which is maximum) = I0
LC OSCILLATIONS
After time t current in the circuit = I
If a charged capacitor C is short-circuited though an inductor L,
Applying Kirchhoff’s loop rule to this circuit
the charge and current in the circuit start oscillating simple
æ dI ö harmonically. If the resistance of the circuit is zero, no energy is
0 + ç -L ÷ = IR (since there is no source of e.m.f.)

.IN
è dt ø dissipated as heat. Assume an idealized situation in which energy
dI dI R is not radiated away from the circuit. With these idealizations-
or L = - IR or = - dt zero resistance and no radiation, the oscillations in the circuit
dt I L
AL
persist indefinitely and the energy is transferred from the
The eqn. gives the value of current at any time t during decay of
capacitor’s electric field to the inductor’s magnetic field back and
current in LR-circuit.
forth. The total energy associated with the circuit is constant.
N
I
This is analogous to the transfer of energy in an oscillating
I0 = E/R mechanical system from potential energy to kinetic energy and
R

back, with constant total energy.


U

Let us now derive an equation for the oscillations in an L-C circuit.


JO

0.37 I0
U

tL t
ED

L
Again, dimensions of are same as that of time
R
The inductive time constant of the LR-circuit can also be defined Refer figure (a) : The capacitor is charged to a potential difference
by using equation V such that charge on capacitor q0 = CV
L Here q0 is the maximum charge on the capacitor. At time t = 0, it is
Setting t = in equation., we get connected to an inductor through a switch S. At time t = 0, switch
R
R L
S is closed.
- . 1
I = I0 e LR = I 0 e -1 = I0 or I @ 0.37 I0. Refer figure (b) : When the switch is closed, the capacitor starts
e discharging. Let at time t charge on the capacitor is q (< q0) and
As t ® ¥, I®0 since, it is further decreasing there is a current i in the circuit in
VR = IR the direction shown in figure.
The potential difference across capacitor = potential difference
E across inductor,
q æ di ö
or Vb – Va = Vc – Vd \ = Lç ÷ ...(i)
C è dt ø
Now, as the charge is decreasing,
VR æ -dq ö di d 2q
i=ç =- 2
è dt ÷ø or
dt dt
0 t
Alternating Current 597

di Comparative study of step-up transformer and step-down


Substituting this value of in equation (i), we get transformer.
dt
q æ d2q ö d2qæ 1 ö
Step - up transformer Step - down transformer
= -L ç 2 ÷ = -ç q
è LC ÷ø
C or ...(ii) 1. E s > E P 1. Es < E p
è dt ø dt 2

This is the standard equation of simple harmonic motion 2. Ns > N p 2. Ns < N p


æ d 2x ö 3. IS < Ip 3. Is > I p
2
ç 2 = -w x ÷ 4. Zs < Zp 4. Zs < Z p
è dt ø
5. k > 1 5. k < 1
1 1
Here w = or f = ...(iii) Power losses in a transformer :
LC 2p LC
(a) Copper loss. This is due to resistance of the winding of
The general solution of equation (ii),
primary and secondary coil (I2 R)
is q = q0 cos(wt ± f) (b) Iron loss or Eddy current loss.
In case f = 0 as q = q0 at t = 0. (c) Loss due to leakage of magnetic flux.
Thus, we can say that charge in the circuit oscillates with angular (d) Hysteresis : Due to repeated magnetisation and
frequency given by equation (iii). Thus, demagnetisation of iron core.
di (e) Humming loss : Due to vibration.

.IN
In L-C oscillations, q, i and all oscillate harmonically with
dt Inspite of all these losses, we have transformers with efficiency
same angular frequency w. But the phase difference between q
of 70% – 90%.
di
and i or between i and is p/2. Their amplitudes are q0, q0w and
AL
Example 9.
dt
w2q0 respectively. So An ideal choke takes a current of 10 ampere when connected
q = q0coswt, then to an A.C. supply of 125 volt and 50 Hz. A pure resistor
N
under the same conditions takes a current of 12.5 ampere.
dq di
i= = - q 0 w sin wt ; = -q 0 w 2 cos wt
R

dt dt If the two are connected to an A.C. supply of 100 2 volt


Similarly potential energy across capacitor (UC) and across and 40 hertz, then find the current in a series combination
U

inductor (UL) also oscillate with double the frequency 2w. of the above resistor and inductor.
JO

TRANSFORMER Solution :
A transformer is a device for converting high voltage into low
For series combination, Z = [R 2 + (X L ) 2 ]
voltage and vice versa, without change in power.
U

There are two types of transformers. 125


R= = 10 W, w L = 2 p f L = V/I
ED

(a) Step up transformer : It converts low voltage into high 12.5


voltage.
(b) Step down transformer : It converts high voltage into low \ 2 p×50×L = 125/10 = 12.5 or 2 p L = 0.25
voltage. For 40 Hz frequency, XL = 2 p L×f = 0.25 × 40 = 10 W
The principle of a transformer is based on mutual
Now Z = [(10) 2 + (10) 2 ] = 10 2 ;
induction and a transformer always works on AC. The input
is appleid across primary terminals and output is obtained
across secondary terminals. I 0 100 2
Current = = = 10 A
The ratio of number of turns in secondary and primary is Z 10 2
called the turn ratio Example10.
nS A low loss transformer has 230 V applied to primary and
i.e., = turn ratio K.
nP gives 4.6 V in secondary. The secondary is connected to a
If EP and ES are alternating voltages, IP and IS the alternating load which draws 5 A current. Find the current in primary.
currents across primary and secondary terminals Solution :
E n I Assuming no loss of power Ep Ip = EsIs
respectively then, S = S = K = P .
E P nP IS
Es Is 5
Efficiency of transformer, \ Ip = = 4.6 ´ = 0.1 A
Ep 230
Output power Pout ES I S
h= = =
Input power Pin EP I P
CONCEPT MAP
598

RC Circuit RMS value of alternating Mean or average value of Peak current (I)0 and voltage V :0 The
Current: I = I0sin (tw+ ) f current and voltage alternating current and voltage
maximum value of current and voltage
Voltage: V= VR2 +VC2 I V 2I 2V0
I rms = 0 Vrms = 0 Imean = 0 Vmean = I0= 2 Irms V0 = 2 Vrms
2 2 π π
Impedance: Z= R 2 +XC2
– 1 Step-up transformer
Phase difference: tan 1
wCR N E Ip
R2 K > 1 K= s = s =
Power factor: cos f = N p E p Is
2
R +X2C Transformer Device Changes
Alternating current(I)
ED
Leading quantity: Current a low voltage of high current
and alternating voltage (V)
U
I = I0sin t;wV = V sin0 t w into a high voltage of low
current and vice-versa
Step-down transformer
LC Circuit N p E p Is
JO K<1 K=
æ pö = =
Current: I = Isin
0 çè wt ± ÷ø N s Es I p
2 ALTERNATING CURRENT
U
Voltage: V= VL– V C Direction of current
Power in an Ac. Circuit LCR series Circuit
Impedance : Z = XL– X C Changes alternatively and
R
its magnitude changes Pav =Vrmsrms
I cos q f
Current : I = I0sin (t w± ): Voltage:
Phase difference: f= 90º
continuously
Power factor : cos f = 0
Leading quantity:
N V= R 2 +(VL –VC )2
;
Either voltage or current Impedance Z = R 2 +(X L – XC )2
X L –XC
AL
Phase difference
R
LR Circuit AC Circuit Þ =R
At resonance XL= X CZmin
.IN
Current: I = Isin
0 (tw+ ) f R Quality factor
Band-width: Dw = ;
Voltage : V = V 2 + V2 L
R L 1 L
Q=
2 R C
Impedance: Z= R +X2L
wL
Phase difference: f = tan –1 Inductive (L) Circuit Capacitive (C) Circuit
R Resistive (R) Circuit
R æ pö Current : I = I0sin (tw+ /2)p Current : I = I0sin t w
Power factor : cos f = Current : I = I0sin ç wt – ÷
è 2ø Phase difference between V and Phase difference between V and I :
R 2 +X 2L Phase difference between
I : f= 90º or – /2pPower factor : f = 0º Power factor: cos f= 1
Leading quantity : voltage V and I : f= 90º or /2p
cos f= 0 Power : P = 0 Power : P = V0 I0
Power factor : cos f= 0
Phasor : Current leads the voltage 2
Power : P = 0 Phasor : Current and voltage
Phasor : Voltage leads the by p /2
both in same phase
current by p/2
PHYSICS

EBD_7179
Alternating Current 599

1. The resistance of a coil for dc is in ohms. In ac, the resistance 11. Which of the following will have the dimensions of time
will (a) LC (b) R/L (c) L/R (d) C/L
(a) be zero (b) decrease 12. In an oscillating LC circuit the max. charge on the capacitor
(c) increase (d) remain same is Q. The charge on capacitor when the energy is stored
equally between electric and magnetic field is
2. In an a.c. circuit, the r.m.s. value of current, Irms is related to
the peak current, I0 by the relation (a) Q/2 (b) Q/ 3 (c) Q / 2 (d) Q
(a) I rms = 2 I 0 (b) I rms = p I 0 13. The power factor of an AC circuit having resistance (R) and
inductance (L) connected in series and an angular velocity
1 1 w is
(c) I rms = I0 (d) I rms = I0 (a) R/wL (b) R/(R2 + w2L2)1/2
p 2
(c) wL/R (d) R/(R2 – w2L2)1/2
3. In a RLC circuit capacitance is changed from C to 2 C. For 14. A.C. power is transmitted from a power house at a high
the resonant frequency to remain unchanged, the voltage as
inductance should be changed from L to

.IN
(a) the rate of transmission is faster at high voltages
(a) 4 L (b) 2 L (c) L/2 (d) L/4 (b) it is more economical due to less power loss
4. An LCR series circuit, connected to a source E, is at (c) power cannot be transmitted at low voltages
(d) a precaution against theft of transmission lines
resonance. Then the voltage across
AL
(a) R is zero 15. In a pure capacitive A.C. circuit current and voltage differ in
(b) R equals applied voltage phase by
N
(a) 0° (b) 45° (c) 90° (d) 180°
(c) C is zero
16. Which of the following statement is incorrect ?
(d) L equals applied voltage
R

(a) In LCR series ac circuit, as the frequency of the source


5. In a LCR circuit at resonance which of these will effect the increases, the impedence of the circuit first decreases
U

current in circuit and then increases.


(a) R only (b) L and R only (b) If the net reactance of an LCR series ac circuit is same
JO

(c) R and C only (d) all L, C and R as its resistance, then the current lags behind the voltage
6. Fleming's left and right hand rules are used in by 45°.
(c) At resonance, the impedence of an ac circuit becomes
U

(a) DC motor and AC generator


(b) DC generator and AC motor purely resistive.
ED

(c) DC motor and DC generator (d) Below resonance, voltage leads the current while above
it, current leads the voltage.
(d) Both rules are same, any one can be used
17. Resonance frequency of LCR series a.c. circuit is f0. Now
7. The time taken by the current to rise to 0.63 of its maximum the capacitance is made 4 times, then the new resonance
value in a d.c. circuit containing inductance (L) and frequency will become
resistance (R) depends on (a) f0/4 (b) 2f0 (c) f0 (d) f0/2.
L 18. A capacitor has capacitance C and reactance X, if
(a) L only (b) R only (c) (d) LR capacitance and frequency become double, then reactance
R
will be
8. A bulb and a capacitor are connected in series to a source of
(a) 4X (b) X/2 (c) X/4 (d) 2X
alternating current. If its frequency is increased, while
19. In a series resonant circuit, having L,C and R as its elements,
keeping the voltage of the source constant, then bulb will
the resonant current is i. The power dissipated in circuit at
(a) give more intense light resonance is
(b) give less intense light
(c) give light of same intensity before i2R
(a) (b) zero
(d) stop radiating light (wL -1/ wC)
9. In LCR circuit if resistance increases quality factor (c) i2 wL (d) i2 R.
(a) increases finitely (b) decreases finitely Whereas w is angular resonant frequency
(c) remains constant (d) None of these 20. An inductance L having a resistance R is connected to an
10. In an A.C. circuit with phase voltage V and current I, the alternating source of angular frequency w. The Quality factor
power dissipated is Q of inductance is
(a) VI (b) V2I (c) VI2 (d) V2I2 (a) R/ wL (b) (wL/R)2 (c) (R /wL)½ (d) wL/R
EBD_7179
600 PHYSICS

21. The core of any transformer is laminated so as to (a) contact is made or broken
(a) reduce the energy loss due to eddy currents (b) contact is made
(b) make it light weight (c) contact is broken
(c) make it robust and sturdy (d) won't become bright at all
(d) increase secondary voltage 24. Energy in a current carrying coil is stored in the form of
22. The time constant of C–R circuit is (a) electric field (b) magnetic field
(a) 1/CR (b) C/R (c) dielectric strength (d) heat
(c) CR (d) R/C 25. In a circuit L, C and R are connected in series with an
alternating voltage source of frequency f. The current leads
23. In the circuit of Fig, the bulb will become suddenly bright if the voltage by 45°. The value of C is
L B
1 1
(a) (b)
pf ( 2pfL - R ) 2pf (2pfL - R)

+ 1 1
– (c) (d)
K pf (2pfL + R) 2pf (2pfL + R)
B

.IN
AL
1. In an A.C. circuit, the current flowing in inductance is the power dissipated in the circuit is
N
I = 5 sin (100 t – p/2) amperes and the potential difference is (a) 104 watt (b) 10 watt (c) 2.5 watt (d) 5.0 watt
R

V = 200 sin (100 t) volts. The power consumption is equal to 7. The primary winding of a transformer has 100 turns and its
(a) 1000 watt (b) 40 watt secondary winding has 200 turns. The primary is connected
U

(c) 20 watt (d) Zero to an A.C. supply of 120 V and the current flowing in it is 10
A. The voltage and the current in the secondary are
JO

2. If resistance of 100W, and inductance of 0.5 henry and


(a) 240 V, 5 A (b) 240 V, 10 A
capacitance of 10 × 106 farad are connected in series through
(c) 60 V, 20 A (d) 120 V, 20 A
50 Hz A.C. supply, then impedance is
8. A step down transformer is connected to 2400 volts line and
U

(a) 1.8765 W (b) 18.76 W 80 amperes of current is found to flow in output load. The
(c) 187.6 W (d) 101.3 W ratio of the turns in primary and secondary coil is 20 : 1. If
ED

3. Using an A.C. voltmeter the potential difference in the transformer efficiency is 100%, then the current flowing in
electrical line in a house is read to be 234 volt. If the line the primary coil will be
frequency is known to be 50 cycles/second, the equation (a) 1600 amp (b) 20 amp
for the line voltage is (c) 4 amp (d) 1.5 amp
(a) V = 165 sin (100 p t) (b) V = 331 sin (100 p t) 9. In the circuit shown in fig, the resonant frequency is
(c) V = 220 sin (100 p t) (d) V = 440 sin (100 p t) (a) 75 kc/s 5mF
4. An inductance of negligible resistance whose reactance is 22 (b) 750 kc/s
W at 200 Hz is connected to 200 volts, 50 Hz power line. The (c) 7.5 kc/s 0.1H 5W
value of inductance is (d) 75 mc/s
(a) 0.0175 henry (b) 0.175 henry
(c) 1.75 henry (d) 17.5 henry 10. An alternating voltage E (in volts) = 200 2 sin 100 t is
5. An inductive circuit contains resistance of 10 ohms and an connected to one micro farad capacitor through an a.c.
inductance of 2 henry. If an A.C. voltage of 120 Volts and ammeter. The reading of the ammeter shall be
frequency 60 Hz is applied to this circuit, the current would (a) 100 mA (b) 20 mA (c) 40 mA (d) 80 mA
11. The r.m.s value of an a.c. of 50 Hz is 10 amp. The time taken
be nearly
by the alternating current in reaching from zero to maximum
(a) 0.32 Amp (b) 0.16 Amp
value and the peak value of current will be
(c) 0.48 Amp (d) 0.80 Amp (a) 2 × 10–2 sec and 14.14 amp
6. In an a.c. circuit V and I are given by (b) 1 × 10–2 sec and 7.07 amp
V = 100 sin (100 t) volts (c) 5 × 10–3 sec and 7.07 amp
I = 100 sin (100 t + p/3) mA (d) 5 × 10–3 sec and 14.14 amp
Alternating Current 601

12. The frequency of A.C. mains in India is 22. An alternating voltage V = V0 sin wt is applied across a
(a) 30 c/s (b) 50 c/s circuit. As a result, a current I = I0 sin (wt – p/2) flows in it.
(c) 60 c/s (d) 120 c/s The power consumed per cycle is
13. A 12 W resistor and a 0.21 henry inductor are connected in (a) zero (b) 0.5 V0I0
series to an a.c. source operating at 20 volt, 50 cycle. The (c) 0.707 V0I0 (d) 1.414 V0I0
phase angle between the current and source voltage is 23. In an A.C. circuit, a resistance of R ohm is connected in
(a) 30º (b) 40º (c) 80º (d) 90º series with an inductance L. If phase angle between voltage
and current be 45°, the value of inductive reactance will be
14. A step down transformer reduces 220 V to 110 V. The primary
(a) R/4
draws 5 ampere of current and secondary supplies 9 ampere.
The efficiency of transformer is (b) R/2
(c) R
(a) 20% (b) 44% (c) 90% (d) 100%
(d) cannot be found with given data
15. An alternating current is given by
24. The ratio of mean value over half cycle to r.m.s. value of
i = i1 coswt + i2 sinwt
A.C. is
The rms current is given by
(a) 2 : p (b) 2 2 : p (c) 2 :p (d) 2 :1
i1 + i 2 i1 + i 2
(a) (b) 25. For the circuit shown in the fig., the current through the
2 2 inductor is 0.9 A while the current through the condenser is
C
0.4 A. Then

.IN
i12 + i 22 i12 + i 22
(c) (d) (a) current drawn from
2 2 generator I = 1.13 A
L
16. The impedance in a circuit containing a resistance of 1 W (b) w = 1/(1.5 L C)
AL
and an inductance of 0.1 H in series, for AC of 50 Hz, is (c) I = 0.5 A
(d) I = 0.6 A
~
(a) 100 10 W (b) 10 10 W V = V0 sin wt
N
26. In series combination of R, L and C with an A.C. source at
(c) 100W (d) 10W resonance, if R = 20 ohm, then impedence Z of the
R

17. The primary winding of transformer has 500 turns whereas combination is
U

its secondary has 5000 turns. The primary is connected to (a) 20 ohm (b) zero (c) 10 ohm (d) 400 ohm
an A.C. supply of 20 V, 50 Hz. The secondary will have an 27. In an LR circuit f = 50 Hz, L=2H, E=5 volts, R=1 W then
JO

output of energy stored in inductor is


(a) 2 V, 5 Hz (b) 200 V, 500 Hz (a) 50 J (b) 25 J
(c) 2V, 50 Hz (d) 200 V, 50 Hz
U

(c) 100 J (d) None of these


18. Determine the rms value of the emf given by 28. A capacitor in an ideal LC circuit is fully charged by a DC
ED

E (in volt) = 8 sin ( w t) + 6sin (2 w t) source, then it is disconnected from DC source, the current
in the circuit
(a) 5 2 V (b) 7 2 V (c) 10 V (d) 10 2 V
(a) becomes zero instantaneously
19. A transformer is used to light a 140 W, 24 V bulb from a (b) grows , monotonically
240 V a.c. mains. The current in the main cable is 0.7 A. The
(c) decays monotonically
efficiency of the transformer is
(d) oscillate infinitely
(a) 63.8 % (b) 83.3 % (c) 16.7 % (d) 36.2 %
29. In a circuit inductance L and capacitance C are connected
20. In the given circuit, the current drawn from the source is as shown in figure. A1 and A2 are ammeters.
A1 C
V = 100x sin(100pt )

R1
X C = 20W
X L =10W
R = 20W

L A2
~
R2
K

Battery
(a) 20 A (b) 10 A (c) 5 A (d) 5 2 A When key K is pressed to complete the circuit, then just
after closing key (K), the readings of A1 and A2 will be
21. An AC voltage source has an output of V = 200sin 2pft .
(a) zero in both A1 and A2
This source is connected to a 100 W resistor. RMS current
(b) maximum in both A1 and A2
in the resistance is
(c) zero in A1 and maximum in A2
(a) 1.41 A (b) 2.41 A (c) 3.41 A (d) 0.71 A
(d) maximum in A1 and zero in A2
EBD_7179
602 PHYSICS

30. The tuning circuit of a radio receiver has a resistance of 36. In the question 86, if the switch is opened after the capacitor
50 W , an inductor of 10 mH and a variable capacitor. A has been charged, it will discharges with a time constant
1 MHz radio wave produces a potential difference of 1
(a) RC (b) 2RC (c) RC (d) RC ln 2
0.1 mV. The values of the capacitor to produce resonance is 2
(Take p2 = 10) 37. An alternating voltage of 220 V, 50 Hz frequency is applied
(a) 2.5 pF (b) 5.0 pF (c) 25 pF (d) 50 pF across a capacitor of capacitance 2 µF. The impedence of
31. Which one of the following curves represents the variation the circuit is
of impedance (Z) with frequency f in series LCR circuit? p 1000 5000
Z Z (a) (b) (c) 500 p (d)
5000 p p
(a) (b) 38. An inductive coil has a resistance of 100 W. When an a.c.
signal of requency 1000 Hz is fed to the coil, the applied
voltage leads the current by 45°. What is the inductance of
the coil ?
f f
Z (a) 10 mH (b) 12 mH (c) 16 mH (d) 20mH.
Z
39. The primary of a transformer has 400 turns while the
(c) (d) secondary has 2000 turns. If the power output from the
secondary at 1000 V is 12 kW, what is the primary voltage?

.IN
(a) 200 V (b) 300 V (c) 400 V (d) 500 V
40. An inductor of self inductance 100 mH and a resistor of
f f
resistance 50W, are connected to a 2 V battery. The time
32. Two coils A and B are connected in series across a 240 V, 50
AL required for the current to half its steady value is
Hz supply. The resistance of A is 5 W and the inductance of (a) 2 milli second (b) 2 ln (0.5) milli second
B is 0.02 H. The power consumed is 3 kW and the power (c) 2 ln (3) milli second (d) 2 ln (2) milli second
N
factor is 0.75. The impedance of the circuit is 41. The instantaneous voltage through a device of impedance
(a) 0.144 W (b) 1.44 W (c) 14.4 W (d) 144 W 20 W is e = 80 sin 100 pt. The effective value of the current is
R

33. In LCR series circuit fed by a DC source, how does the (a) 3 A (b) 2.828 A (c) 1.732 A (d) 4 A
amplitude of charge oscillations vary with time during 42. A transformer has an efficiency of 80%. It works at 4 kW
U

discharge ? and 100 V. If secondary voltage is 240 V, the current in primary


q q
JO

coil is
(a) 0.4 A (b) 4 A (c) 10 A (d) 40 A
(a) qo (b) 43. The primary winding of transformers has 500 turns whereas
U

its secondary has 5000 turns. The primary is connected to


an A.C. supply of 20 V, 50 Hz. The secondary will have an
ED

t t output of
O O
(a) 2V, 5Hz (b) 200 V, 500 Hz
q q
(c) 2V, 50 Hz (d) 200 V, 50Hz
qo 44. A step up transformer operates on a 230 V line and supplies
(c) qo (d) a current of 2 ampere. The ratio of primary and secondary
winding is 1:25 . The current in primary is
(a) 25 A (b) 50 A (c) 15 A (d) 12.5 A
t O t 45. A step-up transformer has transformation ratio of 3 : 2. What
O
34. A steady potential difference of 10 V produces heat at a rate is the voltage in secondary, if voltage in primary is 30 V?
x in a resistor. The peak value of the alternating voltage (a) 45 V (b) 15 V (c) 90 V (d) 300 V
x 46. In an experiment, 200 V A.C. is applied at the ends of an
which will produce heat at a rate in the same resistor is LCR circuit. The circuit consists of an inductive reactance
2
(XL ) = 50 W, capacitive reactance (XC ) = 50 W and ohmic
(a) 5 V (b) 5 2 V (c) 10 V (d) 10 2 V resistance (R) = 10 W. The impedance of the circuit is
35. In the circuit shown, when the switch is closed, the capacitor (a) 10W (b) 20W (c) 30W (d) 40W
charges with a time constant C R 47. In a region of uniform magnetic induction
(a) RC B = 10–2 tesla, a circular coil of radius 30 cm and resistance
(b) 2RC p2 ohm is rotated about an axis which is perpendicular to the
1 direction of B and which forms a diameter of the coil. If the
(c) RC coil rotates at 200 rpm the amplitude of the alternating current
2 +
induced in the coil is
(d) RC ln 2 B
(a) 4p2 mA (b) 30 mA (c) 6 mA (d) 200 mA
Alternating Current 603

48. In the given circuit the reading of voltmeter V1 and V2 are (b) Statement -1 is true, Statement-2 is true; Statement -2 is a
300 volt each. The reading of the voltmeter V3 and ammeter correct explanation for Statement-1
A are respectively (c) Statement -1 is true, Statement-2 is true; Statement -2 is not
L C R = 100 W a correct explanation for Statement-1
(d) Statement -1 is true, Statement-2 is false
V1 V2 V3 49. Statement - 1 : A capacitor blocks direct current in the steady
A state.
~ Statement - 2 : The capacitive reactance of the capacitor is
220 V, 50 Hz inversely proportional to frequency f of the source of emf.
50. Statement - 1 : In the purely resistive element of a series
(a) 150 V and 2.2 A (b) 220 V and 2.2 A
LCR, AC circuit the maximum value of rms current increases
(c) 220 V and 2.0 A (d) 100 V and 2.0 A
with increase in the angular frequency of the applied emf.
Directions for Qs. (49 to 50) : Each question contains 2
STATEMENT-1 and STATEMENT-2. Choose the correct answer e max æ 1 ö
Statement - 2 : Imax = , z = R 2 + ç wL - ÷ ,
(ONLY ONE option is correct ) from the following- z è wC ø
(a) Statement -1 is false, Statement-2 is true where Imax is the peak current in a cycle.

.IN
AL
Exemplar Questions (c) R =15W, L = 3.5 H, C = 30 µF
(d) R = 25W, L = 1.5 H, C = 45 µF
N
1. If the rms current in a 50 Hz AC circuit is 5 A, the value of
the current 1/300 s after its value becomes zero is 6. An inductor of reactance 1W and a resistor of 2W are
R

(a) 5Ö2A (b) 5Ö3/2 A connected in series to the terminals of a 6V (rms) AC source.
(c) 5/6 A (d) 5/Ö2A The power dissipated in the circuit is
U

2. An alternating current generator has an internal reactance (a) 8 W (b) 12 W


JO

Rg and an internal reactance Xg. It is used to supply power (c) 14.4 W (d) 18 W
to a passive load consisting of a resistance Rg and a
7. The output of a step-down transformer is measured to be
reactance XL. For maximum power to be delivered from the
24 V when connected to a 12 W light bulb. The value of the
U

generator to the load, the value of XL is equal to


peak current is
(a) zero (b) Xg
ED

(c) –Xg (d) Rg (a) 1 / 2 A (b) 2A


3. When a voltage measuring device is connected to AC mains, (c) 2 A (d) 2 2 A
the meter shows the steady input voltage of 220 V. This
means NEET/AIPMT (2013-2017) Questions
(a) input voltage cannot be AC voltage, but a DC voltage 8. A coil of self-inductance L is connected in series with a
(b) maximum input voltage is 220 V bulb B and an AC source. Brightness of the bulb decreases
(c) the meter reads not v but < v2 > and is calibrated to read when [2013]
< v2 > (a) number of turns in the coil is reduced
(d) The pointer of the meter is stuck by some mechanical (b) a capacitance of reactance XC = XL is included in the
defect same circuit
4. To reduce the resonant frequency in an L-C-R series circuit (c) an iron rod is inserted in the coil
with a generator (d) frequency of the AC source is decreased
(a) the generator frequency should be reduced 9. The primary of a transformer when connected to a dc battery
(b) another capacitor should be added in parallel to the of 10 volt draws a current of 1 mA. The number of turns of
first the primary and secondary windings are 50 and 100
(c) the iron core of the inductor should be removed respectively. The voltage in the secondary and the current
(d) dielectric in the capacitor should be removed drawn by the circuit in the secondary are respectively
5. Which of the following combinations should be selected
(a) 20 V and 0.5 mA [NEET Kar. 2013]
for better tuning of an L-C-R circuit used for communication?
(b) 20 V and 2.0 mA
(a) R = 20 W, L =1.5 H, C = 35µF
(c) 10 V and 0.5 mA
(b) R=25W, L = 2.5 H, C = 45 µF
(d) Zero and therefore no current
EBD_7179
604 PHYSICS

10. A transformer having efficiency of 90% is working on 200V 13. An inductor 20 mH, a capacitor 50 mF and a resistor 40W are
and 3kW power supply. If the current in the secondary coil connected in series across a source of emf V = 10 sin 340 t.
is 6A, the voltage across the secondary coil and the current The power loss in A.C. circuit is : [2016]
in the primary coil respectively are : [2014] (a) 0.51 W (b) 0.67 W
(a) 300 V, 15A (b) 450 V, 15A (c) 0.76 W (d) 0.89 W
14. A small signal voltage V(t) = V0 sin wt is applied across an
(c) 450V, 13.5A (d) 600V, 15A
ideal capacitor C : [2016]
11. A resistance 'R' draws power 'P' when connected to an AC (a) Current I (t), lags voltage V(t) by 90°.
source. If an inductance is now placed in series with the (b) Over a full cycle the capacitor C does not consume
resistance, such that the impedance of the circuit becomes any energy from the voltage source.
'Z', the power drawn will be [2015] (c) Current I (t) is in phase with voltage V(t).
R æRö (d) Current I (t) leads voltage V(t) by 180°.
(a) P (b) Pç ÷ 15. Figure shows a circuit that contains three identical resistors
Z èZø
with resistance R = 9.0 W each, two identical inductors with
2
æRö inductance L = 2.0 mH each, and an ideal battery with emf e
(c) P (d) P ç ÷ = 18 V. The current 'i' through the battery just after the
èZø
12. A series R-C circuit is connected to an alternating voltage switch closed is [2017]
source. Consider two situations: [2015 RS]
(A) When capacitor is air filled. R R

.IN
+ L
(B) When capacitor is mica filled. e–
Current through resistor is i and voltage across capacitor is
R L C
V then :
AL
(a) Va > Vb (b) ia > ib
(a) 0.2 A (b) 2A
(c) Va = Vb (d) Va < Vb
N
(c) 0 (d) 2 mA
R
U
JO
U
ED
Alternating Current 605

Hints & Solutions


EXERCISE - 1 1 1 1
1. (c) The coil has inductance L besides the resistance R. Lw = Þ w2 = \ w= = 2pf0
Cw LC LC
Hence for ac its effective resistance R2 + X L 2 will 1 1
\ f0 = or f 0a
be larger than its resistance R for dc. 2p LC C
2. (d) When the capacitance of the circuit is made 4 times,
1 its resonant frequency become f 0'
3. (c) We know that f = ,
2 p (LC)
f0' C f0
when C is doubled, L should be halved so that resonant \ = or f 0' =
f0 4C 2
frequency remains unchanged.
4. (b) 1
18. (c) The reactance of capacitor X = where w is
wC
1 frequency and C is the capacitance of capacitor.

.IN
5. (a) At resonance, wL =
wC 19. (d) At resonance wL= 1/wC
Hence the impedance of the circuit would be just equal AL and i = E/R , So power dissipated in circuit is P = i2R.
to R (minimum). In other words, the LCR-series circuit
will behave as a purely resistive circuit. Due to this Potential drop across capacitor or inductor
20. (d) Q=
the current is maximum. This condition is known as Potential drop across R.
N
resonance
wL
=
V R
R

\ Z = R, Current =
R
21. (a)
U

6. (c) 7. (c) 8. (a) 9. (b) 10. (a) 22. (c) The time constant for resonance circuit,
11. (c)
JO

= CR
12. (c) In the case of maximum charge on capacitor, the whole Growth of charge in a circuit containing capacitance
energy is, stored in capacitor in the form of electric and resistance is given by the formula,
U

field which is
q = q0 (1 - e -t / CR )
1 Q2 CR is known as time constant in this formula.
ED

U=
2 C 23. (c) When a circuit is broken, the induced e.m.f. is largest.
So the answer is (c).
When energy in distributed equally between electric
and magnetic field , then energy stored in electric field 1 2
24. (b) Energy stored in a coil = Li
2
U æç 1 Q 2 ö÷ 1 where, L is the self-inductance and i the current flowing
i.e. in capacitor is U1 = =
2 çè 2 C ÷ø 2 through the inductor. Thus, energy is stored in the
magnetic field of the coil.
At that time if charge on capacitor is Q1, then 25. (d) From figure,
1 Q12 U 1 Q 2 1
U1 = = = Þ Q1 = Q / 2 - wL 1
- wL

2 C 2 4 C tan 45º = wC wC
R
13. (b) 14. (b) 15. (c)
16. (d) Option (d) is false because the reason why the voltage 1
1
Þ - wL = R 45º
> Lw and if the wC
leads the current is because O R
Cw
voltage lags, the inductive reactance is greater than 1
Þ = R + wL
the capacitive reactance. wC
17. (d) In LCR series circuit, resonance frequency f0 is given 1 1
by ÞC= = 2 pf ( R + 2pfL)
w ( R + w L)
EBD_7179
606 PHYSICS

EXERCISE - 2 w L 2 p ´ 50 ´ 0.21
tan f = = = 5. 5
1. (d) Power, P = Ι r.m.s ´ Vr.m.s ´ cos f R 12

In the given problem, the phase difference between f = tan -1 5.5 = 80º
voltage and current is p/2. Hence
Es I s 110 ´ 9
P = Ι r.m.s ´ Vr.m.s ´ cos(p / 2) = 0. 14. (c) h= \h= = 0.9 ´ 100% = 90%
Ep I p 220 ´ 5
2
æ 1 ö 15. (c) 16. (b)
2. (c) Z = R 2 + çç w L - ÷
è w C ÷ø 17. (d) The transformer converts A.C. high voltage into A.C.
Here R = 100 W, L = 0.5 henry, C = 10 × 106 farad low voltage, but it does not cause any change in
frequency.
w = 2 p p = 100 p.
3. (b) V= V0 sin w t E s Ns N 5000
= Þ Es = s Ep = ´ 20 = 200V
Voltage in r.m.s. value Ep Np Np 500

V0 = 2 ´ 234 V = 331 volt Thus output has voltage 200 V and frequency 50 Hz.
18. (a) E = 8 sin wt + 6 sin 2wt
and w t = 2 p n t = 2 p ´ 50 ´ t = 100 p t

.IN
Thus, the equation of line voltage is given by Þ E peak = 8 2 + 6 2 = 10 V
V = 331 sin (100 p t) AL
4. (a) XL = w L = 2 p n L 10
E rms = =5 2 V
2
XL 22 ´ 7
\ L= = H = 0.0175H
N
2 p n 2 ´ 22 ´ 200 19. (b) Power of source = EI = 240 × 0.7 = 166
R

5. (b) 140
Þ Efficiency = Þ h = 83.3%
166
U

1
6. (c) P = Vr.m.s ´ I r.m.s ´ cos f = V0 I 0 cos f 20. (d) 21. (a)
2
JO

22. (a) The phase angle between voltage V and current I is


1
= ´ 100 ´ (100 ´ 10 - 3 ) cos p / 3 = 2.5 W p/2. Therefore, power factor cos f = cos (p/2) = 0. Hence
2
the power consumed is zero.
U

Es n æn ö wL X L
= s or E s = E p ´ ç s ÷
ED

7. (a) 23. (c) tan f = =


Ep np ç np ÷ R R
è ø
Given f = 45°. Hence XL = R.
æ 200 ö
\ E s = 120 ´ ç ÷ = 240 V 24. (b) We know that Ι r m s = Ι 0 / 2 and Ι m = 2 Ι 0 / p
è 100 ø
Ιm 2 2
Ιp n æ np ö æ 100 ö \ =
= s or Ι s = Ι p ç ÷ \ Is = 10 ç = 5 amp p
Ιs n p è sø
n è 200 ÷ø Ιrms
25. (c) The current drawn by inductor and capacitor will be in
Ιs n p 80 20 opposite phase. Hence net current drawn from
8. (c) = ; = or Ι p = 4 amp.
Ιp ns Ιp 1 generator
9. (a) = IL – IC = 0.9 – 0.4 = 0.5 amp.
26. (a)
E æE ö
10. (b) Ι= = E w C = çç 0 ´ w C ÷÷
XC è 2 ø
27. (d) L = 2H , E = 5 volts, R = 1W

æ 200 ö 64 447
Z 4 4 48
\ I = 120 ´ ç = 240V = 20 ´10 -3 amp.
è 100 ÷ø
R L
11. (d) 12. (b)
13. (c) The phase angle is given by E
Alternating Current 607

1 2 E 35. (a) The resistance in the middle plays no part in the charging
Energy in inductor = LI I = process of C, as it does not alter either the potential
2 Z
difference across the RC combination or the current
5 5 through it.
I= =
R 2 + ( wL ) 1 + 4p 2 ´ 50 2 ´ 4
2 36. (b) C discharges through both resistance in series.
37. (d) Impedence of a capacitor is XC = 1/wC
5 5
= =
200p 1 1 5000
1 + ( 200p )
2
XC = = -
= .
2pfC 2p´ 50 ´ 2 ´ 10 6 p
1 5´5 38. (c)
Energy = ´2´ = 6.33 × 10–5 joules
2 200 ´ 200 p 2 39. (a) NP = 400, NS = 2000 and VS = 1000 V.
28. (a) In ideal condition of LC circuit R = 0 and LC oscillation VP N P VS ´ N P 1000 ´ 400
= of, VP = = = 200V.
continue indefinitely. Energy being shunted back and VS NS NS 2000
forth between electric field of capacitor and magnetic
field of inductor. As capacitor is fully charged current 40. (d) The time constant of the circuit is

1 q0 2 L 100 ´10-3
in L is zero and energy is stored in electric t= = = 2 ´10-3 s= 2 milli second.
2 C R 50

.IN
field. Then capacitor begins to discharge through L Current at time t is given by I = I 0e–t/t
causing a current to flow and build up a magnetic where I0 is the steady current. Therefore, time for I to
field, around L. Therefore, energy stored.
fall to I0/2 is
AL
1 2
Now in L = LI 0 when C is fully discharged, V 1
e – t/ t =
2 or, et/t = 2, t = tln(2) = 2ln(2) milli second.
across the plate reduces to zero. 2
N
41. (b) Given equation, e = 80 sin 100pt …(i)
\ Electric field energy is transferred to magnetic
R

field and vice-versa. Standard equation of instantaneous voltage is given


29. (d) Initially there is no D.C. current in inductive circuit by e = em sinwt …(ii)
U

and maximum D.C.current is in capacitive current. Compare (i) and (ii), we get em = 80 V
JO

Hence, the current is zero in A2 and maximum in A1. where em is the voltage amplitude.
30. (a) L = 10 mHz = 10–2 Hz em
f = 1MHz = 106 Hz Current amplitude Im = where Z = impendence
Z
U

1 1 = 80/20 = 4 A.
ED

f= ; f2 = 2
2p LC 4p LC 4 4 2
I r.m.s = = = 2 2 = 2.828 A.
2 2

10 -12
Pi 4000
ÞC=
1
=
1
= = 2.5 pF
42. (d) As E p Ι p = Pi \ Ιp = = = 40 A.
Ep 100
4p 2 f 2 L 4 ´ 10 ´ 10 - 2 ´ 1012 4
31. (c) Impedance at resonant frequency is minimum in series 43. (d)
LCR circuit. np Ep 1
44. (b) = =
2 ns Es 25
æ 1 ö
So, Z = R 2 + ç 2 pfL - ÷ \ E s = 25E p
è 2pfC ø
E S ´ IS
When frequency is increased or decreased, Z But Es Is = E p I p Þ I p = Þ I p = 50A
increases. Ep
NS
E 2v cos f 45. (a) Transformation ratio k =
32. (c) P= NP
Z
VS NS N
Since = \ VS = S ´ VP
(240) 2 (0.75) VP N P NP
P = 3000 = d Þ Z = 14.4W
Z
3
33. (c) 34. (c) VS = ´ 30 = 45V
2
EBD_7179
608 PHYSICS

46. (a) Given : Supply voltage (Vac) = 200 V So, Xint = Xext
Inductive reactance (XL) = 50 W Xg = (XL) = –XL
Capacitive reactance (XC) = 50 W Hence, XL = –Xg (Reactance in external
circuit)
Ohmic resistance (R) = 10 W.
3. (c) As we know that,
We know that impedance of the LCR circuit The voltmeter in AC reads rms values of voltage
(Z) = {(X L - X C ) 2 + R 2 } = {(50 - 50) 2 + (10) 2 } = 10 W Irms = 2I0 and Vrms = 2v0
The voltmeter in AC circuit connected to AC mains
E0 nBAw
47. (c) I0 = = reads mean value (<v2>) and is calibrated in such a
R R way that it gives rms value of <v2>, which is multiplied
Given, n = 1, B = 10–2 T, by form factor Ö2 to give rms value Vrms.
A = p(0.3)2m2, R = p2 4. (b) As we know that,
f = (200/60) and w = 2p(200/60) The resonant frequency in an L-C-R series circuit is
Substituting these values and solving, we get 1
n0 =
I0 = 6 × 10–3 A = 6mA 2p LC
48. (b) As VL = VC = 300 V, resonance will take place So, to reduce n0 either increase L or increase C.

.IN
\ VR = 220 V To increase capacitance, another capacitor must be
connect in parallel with the first capacitor.
220 5. (c) As we know that, Quality factor (Q) of an
= 2.2 A
Current, I =
100
AL
L-C-R circuit must be higher so Q is
\ reading of V3 = 220 V 1 L
N
and reading of A = 2.2 A Q=
R C
R

49. (b) 50. (c) where R is resistance, L is inductance and C is


capacitance of the circuit.
U

EXERCISE - 3 So, for higher Q, L must be large, and C and R should


JO

Exemplar Questions be low.


Hence, option (c) is verify.
1. (b) As given that, v = 50 Hz, Irms = 5A
6. (c) As given that,
U

1 XL = 1W, R = 2W, Erms = 6V, Pav = ?


t= s
300 The average power dissipated in the L, R, series circuit
ED

I0 with AC source
As we know that Irms = Then Pav= Erms Irms cos f ... (i)
2
I0 E rms
I0 = Peak value = 2.Irms = 2 ´5 Irms = =
2 Z
I0 = 5 2A
1 Z= R 2 + X L2 = 4 +1 = 5
at, t = sec , I = I0 sinwt = 5 2 sin 2pvt
300
6
1 Irms = A
= 5 2 sin 2 p ´ 50 ´ 5
300
p 3 R 2
I = 5 2 sin = 5 2´ = 5 3 2 Amp cos f = =
3 2 Z 5
æ p 3ö By putting the value of Irms, Erms, cos f in equation (i),
çè\ sin = ÷ then,
3 2 ø
æ 3ö 6 2 72
I = ç 5 ÷ Amp Pav = 6 ´ ´ =
ç 2÷ 5 5 5 5
è ø
2. (c) To deliver maximum power from the generator to the 72
load, total internal reactance must be equal to conjugate = = 14.4 watt
5
of total external reactance.
Alternating Current 609

7. (a) As given that, Xc = 1/cw


Secondary voltage (VS) is :
VS = 24 Volt
R
Power associated with secondary is : C
PS = 12 Watt
As we know that PS = VSIS ~
PS 12 1 AC Source
IS = = = A = 0.5 Amp V V
VS 24 2 Current i = =
Zc 2
æ 1 ö
Peak value of the current in the secondary R2 + ç ÷
è Cw ø
I0 = IS 2 = 0.5 2 V 1
Vc = iXc = ´
2 Cw
5 é 1 ù æ 1 ö
= 2 Þ ê I0 = Ampú R2 + ç ÷
10 ë 2 û è Cw ø
NEET/AIPMT (2013-2017) Questions V
Vc =
8. (c) By inserting iron rod in the coil, (RCw)2 + 1
L ­ z ­ I ¯ so brightness ¯ If we fill a di-electric material like mica instead of air

.IN
9. (d) A transformer is essentially an AC device. DC source then capacitance C­ Þ Vc¯
so no mutual induction between coils So, Va > Vb
Þ E2 = 0 and I2 = 0 13.
AL(a) Given: L = 20 mH; C = 50 mF; R = 40 W
Vs Is V (6) V = 10 sin 340 t
10. (b) Efficiency h = Þ 0.9 = s 10
Vp Ip 3 ´ 103 \ Vruns =
2
N
Þ Vs = 450 V
As VpIp = 3000 so 1 1
= = 58.8 W
R

XC =
3000 3000 wC 340 ´ 50 ´10-6
Ip = = A = 15A XL = wL = 340 × 20 × 10–3 = 6.8 W
U

Vp 200
R 2 + (XC - X L )
2
Impedance, Z =
JO

11. (d)
Pure resistor L-R series circuit 2
= 40 2 + ( 58.8 - 6.8 ) = 4304 W
Power loss in A.C. circuit,
U

R R L
2
æV ö
ED

2
V V P = i rms R = ç rms ÷ R
Vs V è Z ø
2
æ 10 / 2 ö 50 ´ 40
Phasor diagram =ç ÷ ´ 40 = ; 0.51 W
R è 4304 ø 4304
q 14. (b) As we know, power P = Vrms · Irms cosf
as cosf = 0 (Q f = 90°)
R \ Power consumed = 0 (in one complete cycle)
XL cos q =
Z Z
15. (b)
Z = impedance
For pure resistor circuit, power
At t = 0, no current flows through R1 and R3
V2
P= Þ V 2 = PR
R
For L-R series circuit, power
V2 V 2 R PR
2 \
æ Rö
P1 = cos q = . = .R =Pç ÷
Z Z Z Z2 è Zø
12. (a) For series R – C circuit, capacitive reactance,
Current through battery just after the switch closed is
æ 1 ö
2 e 18
R2 + ç i= = = 2A
Zc = ÷ R2 9
è Cw ø
EBD_7179
610 PHYSICS

Electromagnetic
23 Waves
DISPLACEMENT CURRENT (ii) Gauss's law for magnetism : It states that the net magnetic
It is that current which comes into play in a region, where electric flux through a closed surface is zero. It means that number
field (and hence the electric flux) is changing with time. of magnetic field lines that enter a closed volume must be
df equal to number of field lines leaving that volume
The displacement current is given by, I D = eo E

.IN
r r
Ñò
dt
where e o = absolute permittivity (or permitivity of free space) and i.e., B.dA = 0
dfE It means that magnetic mono pole cannot exists in nature.
r r
dt
= rate of change of electric flux.
AL The differential form of this law is Ñ.B = 0
In case of a steady electric flux linked with a region, the (iii) Faraday’s law : This law states that the line integral of the
displacement current is maximum.
N
electric field around any closed path (which equals the e.m.f.)
The current in the electric circuit which arises due to flow of
equals the rate of change of magnetic flux through any
electrons in the connecting wires of the circuit, in a defined
R

surface area bounded by that path.


closed path is called conduction current.
r r d fm
U

MAXWELL’S MODIFICATION OF AMPERE’S CIRCUITAL


LAW
i.e., Ñò E.d l = dt
JO

The consequence of this law is that, if we keep any


In 1864, Maxwell showed that Ampere’s circuital law is logically conducting loop in a time varying magnetic field, then an
inconsistent for non-steady currents. He modified Ampere’s law induced current, flows in that conducting loop. The
ur uuur
U

æ df ö r r
as
Ñò B.d l = m 0 ç I + e 0 E ÷ (Sum of conduction current and
è dt ø differential form of this law is Ñ ´ E = -
dB
ED

displacement current) dt
This law decribes a relationship between an electric field
The term I d = e 0 (dfE / dt) is displacement current. It is that and a changing magnetic flux.
current which comes into existence, in addition to the conduction (iv) Modified Ampere-Maxwell law : It states that “ the line
current, whenever the electric field and hence the electric flux integral of magnetic field around any closed path is
changes with time. determined by the sum of the net conduction current through
Maxwell’s Equation that path and the rate of change of electric flux through any
Maxwell found that all the basic principles of electromagnetism can surface bounded by that path
r r df
be formulated in terms of four fundamental equations, called
Maxwell’s equation. These are :
i.e., Ñò B.dS = m 0 I + e om 0 e
dt
(i) Gauss's law for electrostatics : According to Gauss's law It describes the relationship between magnetic and electric
the total electric flux through any closed surface is equal to fields and electric current. The differential form of this law is
the net charge inside that surface divided by eo r r dE
Ñ ´ B = m0 J + m0 E0
r uur Q dt
i.e., Ñò E.dA =
eo ELECTROMAGNETIC WAVES
This law relates the electric field to charge distribution, Maxwell-on the basis of four basic equations of electromagnetism
whereas electric field lines orignate from positive (+ive) theoretically predicted the existence of electromagnetic waves.
charge and terminate on negative (–ive) charge. The An electromagentic wave is the one constituted by oscillating
r r r electric and magnetic fields which oscillate in two mutually
differential form of this law is Ñ.E = perpendicular planes. The wave itself propagates in a direction
e0
where r is volume charge density and eo is the permittivity perpendicular to both of the directions of oscillations of electric
of free space. and magnetic fields.
Electromagnetic Waves 611

Properties of Electromagnetic Waves


1 1 1
(i) The direction of oscillations of E and B fields are uE = e o E 2 = e o ( E0 / 2) 2 = e o E02
2 2 4
perpendicular to each other as well as to the direction of
The average magnetic energy density is given by
propagation.Electromagnetic waves are transverse in nature.
(ii) The electric and magnetic field oscillate in same phase. B 2 ( B0 / 2) 2 B2
(iii) The electromagnetic waves travel through vacuum with the uB = = = 0
2m 0 2m 0 4m 0
same speed of light

1 1 2 1 2 1 2 2
c= = 3 ´ 108 ms -1 Also, u E = e o E0 = e o (c B0 ) = e o c B0
4 4 4
m0 eo

1 e o B02 B2
1 2 = = 0 = uB
(iv) The energy density of electric field is e o E and that of 4 e o m 0 4m 0
2
\ Total average energy density
1 B2
magnetic field is , so the energy density of the
2 m0 1 B2
= uE + uB = 2uE = 2uB = e o E02 = 0
2 2m 0

.IN
1é 2 B

5. Poynting vector: When an electromagnetic wave advances,
electromagnetic wave is u = êe o E + ú
2 ëê m 0 ûú
the electromagnetic energy flows in the direction E ´ B .
where E and B are the instantaneous values of the electric
AL
The total energy flowing perpendicularly per second per
and magnetic field vectors. unit area into the space in free space is called a poynting
ur
N
E 1 vector S where S = ce o (E ´ B ) = ( E ´ B ) / m 0 .
(v) The ratio = c = .
R

B m0 eo
uur 1 EB
S =|S | = E B sin 90° =
U

Production of Electromagnetic Waves m0 m0


JO

An accelerated charge emits electromagnetic waves. An oscillating


The SI unit of S is watt/m2
charge, as in an LC-circuit has non-zero acceleration, it continues
to emit electromagnetic waves. The frequency of electromagnetic 6. The intensity of a sinusoidal plane electromagnetic wave is
defined as average value of poynting vector taken over one
U

waves is the same as that of the oscillating charge.


Hertz’s experiment : In 1888, Hertz succeeded in experimentally cycle. Thus,
ED

confirming the existence of electromagnetic waves. By using


1 æ E0 ö æ B0 ö E0 B0 E 2 c B0 2
oscillator LC-circuits, he not only produced and detected I = Sav = ç ÷ ç ÷ = = 0 =
m0 è 2 ø è 2 ø 2 m0 2 m 0c 2 m 0
electromagnetic waves, but also demonstrated their properties of
reflection, refraction and interference and established beyond 7. Radiation pressure : The pressure exerted by electromagnetic
doubt that light radiation has wave nature. waves is called as radiation pressure (P). When an
r
Keep in Memory electromagnetic wave with Poynting vector S is incident on a
1. The amplitudes of electric and magnetic fields in free space, perfectly absorbing surface, then radiation pressure on surface
r
E0 S
in electromagnetic waves are related by E0 = cB0 or C = is P = .
B0 c

2. The velocity of electromagnetic wave does not depend on ELECTROMAGNETIC SPECTRUM


amplitude of field vectors. All the known radiations form a big family of electromagnetic
3. The electric vector of an electromagnetic wave is responsible waves according to frequency or wavelength. We call this family
for the optical effects & is called a light vector. as the complete electromagnetic spectrum. It includes: Gamma
4. In a plane electromagnetic wave, the average energy rays, X-rays, ultraviolet light, visible light, infrared light,
densities of electric and magnetic fields are equal. microwaves and radio waves.
In vacuum, the average electric energy density is given by,
EBD_7179
612 PHYSICS

Wavelength range Frequency range


S.No. Name Source Detection
(in m) (in Hz)
1 Gamma rays 6 × 10
–14
to 1 × 10
–11 22
5 × 10 to 3 × 10
19 Nuclear origin Photographic film,
Geiger tubes,
ionisation chamber
2 X-rays 1 × 10
–11 –8
to 3 × 10
19
3 × 10 to 1 × 10
16 Sudden deceleration of Photographic film,
high energy electron Geiger tubes,
(collision of electrons ionisation chamber
with target)
3 Ultra-violet –8
6 × 10 to 4 × 10
–7 17
5 × 10 to 8 × 10
14 Excitation of atom, Photocells,
spark and arc lamp photographic film

4 Visible light –7
4 × 10 to 7 × 10
–7 14
8 × 10 to 4 × 10
14 Excitation of valency The eye, photocells,
electron photographic film
5 Infrared –7
7 × 10 to 3 × 10
–5 14
4 × 10 to 1 × 10
13 Excitation of atoms and Thermopiles, bolometer
molecules
6 Heat radiations –5
10 to 10
–1 13
3 × 10 to 3 × 10
9 Hot bodies
7 Micro-waves –3
10 to 0.3
11
3 × 10 to 1 × 10
9 Oscillating current in Point contact diodes

.IN
special vacuum tube
8 Ultra high –1
1 × 10 to 1
9 8
3 × 10 to 3 × 10 Oscillating circuit
frequency AL
9 Very high radio 1 to 10 8 7
3 × 10 to 3 × 10 Oscillating circuit
frequency
10 Radio frequency 10 to 104 7 4
3 × 10 to 3 × 10 Oscillating circuit Receiver's aerial
N
11 Power 5 × 106 to 6 × 106 60 to 50 Weak radiations from
R

frequency a.c. circuit


U

Example 1. 2 pc 2p ´ 3 ´ 108
In a plane electromagnetic wave the electric field varies w = 2pn = = = p ´ 1011 rad / s.
l 6 ´ 10 -3
JO

with time having an amplitude 1 Vm–1. The frequency of The maximum magnetic field,
wave is 0.5 × 1015 Hz. The wave is propagating along
E 33
Z-axis. What is the average energy density of (i) electric B0 = 0 = = 11´ 10 -8 T
U

c 8
field (ii) magnetic field (iii) total field. (iv) What is amplitude 3 ´ 10
The equation for the electric field, along y-axis in the
of magnetic field?
ED

electromagnetic wave is
Solution :
(i) Average energy density of electric field æ xö
E y = E 0 sin wç t - ÷ = 33 sin p ´ 1011 ( t - x / c)
2
é E0 ù è cø
1 1 æ E0 ö
êQ E =
2 ç ÷
U E = eoE = eo ç ÷ ú The equation for the magnetic field along z-axis in the
2 2 è 2ø ë 2û electromagnetic wave is
1 1
= e o E 02 = ´ (8.854 ´10 -12 ) ´12 = 2.21´ 10 -12 Jm -3 . æ xö
4 4 B z = B 0 sin wç t - ÷ = 11´ 10 -8 sin p ´ 1011 ( t - x / c)
(ii) Average energy density of magnetic field, è cø
UB = Average energy density of electric field Example 3.
= 2.21 × 10–12 Jm–3 If ε o and m0 represent the permittivity and permeability of
(iii) Total average energy density U = UE + UB = 2 UE vacuum and ε and m represent the permittivity and
= 2 × 2.21 ×10–12 = 4.42 × 10–12 Jm–3 permeability of medium, then refractive index of the medium
(iv) Amplitude of magnetic field, B0 = E0/c is given by
= 1/(3 × 108) = 3.33 × 10–9 T μ 0ε o με ε μ 0ε o
Example 2. (a) (b) (c) (d)
με μ 0ε o μ 0ε o μ
A plane electromagnetic wave propagating in the
Solution : (b)
x-direction has a wavelength of 6.0 mm. The electric field
is in the y-direction and its maximum magnitude is 33 Vm–1. 1
Q Velocity of light in medium c =
Write suitable equations for the electric and magnetic fields me
as a function of x and t. c 0 1/ m0 eo me
Solution : \ Refractive index = = =
c 1 / me m 0e o
Here, l = 6.00 mm = 6 × 10–3 m; E0 = 33 V/m.
Electromagnetic Waves 613

Example 4. The study of X-rays has revealed the atomic structure and
The magnetic field in a plane electromagnetic wave is given crystal structure.
by B = 2 × 10–7 sin (0.5× 103 x +1.5 × 1011t) T. 7. The study of g-rays provides us valueable information
(a) What is the wavelength and frequency of the wave? about the structure of the atomic nuclei
(b) Write an expression for the electric field. 8. Super high frequency electromagnetic waves (3000 to
Solution : 30,000 MHz) are used in radar and satellite communication.
(a) Comparing the given equation with 9. Electromagnetic waves (frequency 50 to 60 Hz) are ued for
é æ x t öù lighting. These are weak waves having wavelength
B = B0 sin ê 2p ç + ÷ ú 5 × 106 to 6 × 106 m and can be produced from A.C. circuits.
ë è l Tøû
2p Keep in Memory
we get, l = m = 1.26 cm
0.5 ´ 103 1. Green house effect : It is the phenomenon which keeps the
1
and = n = (1.5 ´ 1011 ) / 2p = 23.9 GHz earth’s surface warm at night. The earth absorbs solar radiation
T and reflects back only infrared rays due to its low temperature.
(b) E0 = B0c = 2 × 10–7 T × 3 × 108 m/s = 6 × 101 V/m These rays are reflected back by the clouds and the gas molecules
The electric field component is perpendicular to the of the lower atmosphere. This keeps the earth’s surface warm at
direction of propagation and the direction of magnetic night.
field. Therefore, the electric field component along the z-

.IN
axis is obtained as PROPAGATION OF RADIO WAVES THROUGH THE
Ez = 60 sin (0.5 × 103x + 1.5 × 1011 t) V/m ATMOSPHERE
Example 5. It takes place in three ways :
AL
A plane electromagnetic wave of frequency 25 MHz travels (i) Ground wave propagation,
in free space along the x-direction. At a particular point in
r (ii) Sky wave propagation and
r
space and time, E = 6.3 ˆj V / m . What is B at this point ? (iii) Space wave propagation.
N
Solution : (i) Ground wave propagation : When the radio wave travel
r directly from one point to another following the surface of
R

The magnitude of B is
the earth, it is called ground or surface wave. This type of
U

E 6.3 V / m transmission is possible only with waves of wavelengths


B= = = 2.1 ´ 10 -8 T
c 3 ´ 108 m / s above 200 m or frequencies below 1500 k Hz.
JO

r (ii) Sky wave propagation : When a radiowave is directed


To find the direction, we note that E is along y-direction towards the sky and is reflected by the ionosphere towards
and the wave propagates along x-axis. Therefore, B should desired location on the earth, it is called sky wave. This
U

be in a direction perpendicular to both x- and y-axes. Using method is useful for the transmission of waves of
r r
ED

vector algebra, E ´ B should be along x-direction. wavelengths less than 200 m or frequencies above 1500 k
r r Hz upto 30 MHz.
ˆ = i.B
Since, (+ ˆj) ´ (+ k) ˆ , B is along the z-direction.
(iii) Space wave propagation : For the transmission of television
r signals (frequencies in the range 100-200 M Hz), space
Thus, B = 2.1 ´ 10 -8 kT
ˆ
wave propagation method is used, in which the wave travels
USES OF ELECTROMAGNETIC WAVES
directly from a high transmitting antenna to the receiving
The following are some of the uses of electromagnetic waves
antenna. The relation between the height ‘h’ of the
1. Radio waves are used in radio and T.V. communication
transmitting antenna above the ground level and the distance
systems.
2. Microwaves are used in microwave oven. ‘d’ upto which TV signal can be received is d = 2hR
3. Infrared radiations are used (a) in revealing the secret = range of a TV tower where R is the radius of the earth.
writings on the ancient walls (b) in green houses to keep the Example 6.
plants warm (c) in warfare, for looking through haze, fog or A TV tower has a height of 100 m. How much population
mist as these radiations can pass through them. is covered by the TV broadcast if the average population
4. Ultraviolet radiations are used in the detection of invisible density around the tower is 1000 km–2 ?(radius of the
writing, forged documents, finger prints in forensic earth = 6.37 × 106 m)
laboratory and to preserve the food stuffs. Solution :
5. The study of infrared, visible and ultraviolet radiations help Height of twoer h = 100 m
us to know through spectra, the structure of the molecules Radius of the earth R = 6.37 × 106m
and arrangement of electrons in the external shells.
6. X-rays can pass through flesh and blood but not through d = 2 hR = 2 ´ 100 ´ 6.37 ´ 106
bones. This property of X-rays is used in medical diagnosis, Population covered = p d 2 × population density
after X-rays photographs are made. Solving we get population covered = 40 lakh.
614
CONCEPT M AP

Radio waves
Radiation pressure exerted Energy associated Do not require
Wavelength > 0.1 m
by an electromagnetic wave with an electromagnetic any material
Uses: in telecommu-
energy associated wave medium for
nication
with em waves (u) 1 1 B2 propagation
P= u= e0 E 2 +
speed of light in vacuum(c) 2 2 m0 Produced
Microwave
Wavelength 0.1 m to1 mm by accelerated
charge
ED
Uses : in microwave
oven, RADAR U Travels with
speed of light
in free space
Infra -red Characteristics
Different types of 1
Wavelength 1 mm to 700 mm of electromagnetic C=
JO
electromagnetic waves
Uses : treat muscular strain waves m0 e 0
U = 3 ´108 m/s

Visible R In free space;


Wavelength : magnitude of
700 nm to 400 nm N electric field (E)
Uses : to see objects ELECTROMAGNETIC WAVE S magnetic field (B)
Constituted by mutually perpendicular =C (speed of light
Ultra-violet in vacuum)
time varying electric and magnetic fields
AL
Wavelength 400 nm
to 1nm Transverse in
Uses : Preserve food nature
.IN
purifying water

g-rays Oscillating
–3
Wavelength : 1 nm to 10nm electric and magnetic
Uses : Medical diagnosis fields are in phase and
Conduction current Displacement current (I)D their magnitudes
g-rays Arises due to flow of Due to time varying bear constant ratio
–3 electrons in a definite electric field
Wavelength : < 10nm C=
E0
Uses : in medical science closed path df B0
ID = e0 E
information on nuclear dt
structure
PHYSICS

EBD_7179
Electromagnetic Waves 615

1. The electromagnetic radiation used in food processing 12. The ionosphere


sterilizing agent is (a) reflects back radiowaves in the AM band
(a) microwaves (b) UV rays (b) reflects back radiowaves in the FM band
(c) gamma rays (d) radio waves
(c) absorbs radiowaves in the AM band
2. An electromagnetic wave is made up of joint electric (E) and
magnetic (B) fields. Its direction of propagation is (d) absorbs radiowaves in the FM band
(a) parallel to E 13. The frequency 2 MHz belongs to
(b) perpendicular to E but parallel to B (a) visible light (b) X-rays
(c) parallel to B (c) microwaves (d) radiowaves
(d) perpendicular to both E and B
14. Radio waves and visible light in vacuum have
3. Intensity of electromagnetic wave will be
(a) same velocity but different wavelength
(a) I = cm 0 B20 / 2 (b) I = ce 0 B20 / 2 (b) continuous emission spectrum

.IN
(c) I = B20 / cm 0 (d) I = E 02 / 2ce 0 (c) band absorption spectrum
(d) line emission spectrum
4. The frequency modulated waves are
(a) reflected by atmosphere 15. 10 cm is a wavelength corresponding to the spectrum of
AL
(b) absorbed by atmosphere (a) infrared rays (b) ultra-violet rays
(c) bend by atmosphere (c) microwaves (d) g -rays
N
(d) radiowaves
16. The ozone layer absorbs radiation of wavelengths
4
If l = 1 ´ 10 Å then it corresponds to (a) less than 3 × 10–7 m
R

5.
(a) infrared (b) microwaves (b) more than 3 × 10–7 m
U

(c) ultraviolet (d) X-rays (c) less than 3 × 10–5 m


6. 10 cm is a wavelength corresponding to the spectrum of (d) more than 3 × 10–5 m
JO

(a) infrared rays (b) ultra-violet rays


17. The wave impendance of free space is
(c) microwaves (d) g -rays
7. In an electromagnetic wave (a) zero (b) 376.6 W
U

(a) power is transmitted along the magnetic field (c) 33.66 W (d) 3.76 W
ED

(b) power is transmitted along the electric field 18. The electromagnetic waves travel with a velocity
(c) power is equally transferred along the electric and (a) equal to velocity of sound
magnetic fields
(b) equal to velocity of light
(d) power is transmitted in a direction perpendicular to
both the fields (c) less than velocity of light
8. Electromagnetic waves are transverse in nature is evident (d) None of these
by 19. The speed of electromagnetic wave in vacuum depends
(a) polarization (b) interference upon the source of radiation. It
(c) reflection (d) diffraction (a) increases as we move from g-rays to radio waves
9. If l = 10 Å then it corresponds to
(b) decreases as we move from g-rays to radio waves
(a) infrared (b) microwaves
(c) is same for all of them
(c) ultraviolet (d) X-rays
10. Intensity of electromagnetic wave will be (d) None of these
20. Maxwell in his famous equation of electromagnetism
(a) I = cm 0 B20 / 2 (b) I = ce 0 B20 / 2
introduced the concept of
(c) I = B20 / cm 0 (d) I = E 02 / 2ce 0 (a) a.c. current
(b) d.c. current
11. An electromagnetic wave passes through space and its
equation is given by E = E0 sin (wt – kx) where E is electric (c) displacement current
field. Energy density of electromagnetic wave in space is (d) impedance
21. Which of the following shows green house effect ?
1 1
(a) e 0 E 02 (b) e 0 E 02 (c) e 0 E 20 (d) 2e 0 E 02 (a) Ultraviolet rays (b) Infrared rays
2 4 (c) X-rays (d) None of these
EBD_7179
616 PHYSICS

22. Radio waves diffract around building although light waves 24. Approximate height of ozone layer above the ground is
do not. The reason is that radio waves (a) 60 to 70 km (b) 59 km to 80 km
(a) travel with speed larger than c
(c) 70 km to 100 km (d) 100 km to 200 km
(b) have much larger wavelength than light
(c) are not electromagnetic waves 25. Biological importance of ozone layer is
(d) None of these (a) it stops ultraviolet rays
23. All components of the electromagnetic spectrum in vacuum (b) ozone layer reduce green house effect
have the same
(c) ozone layer reflects radio waves
(a) energy (b) velocity
(c) wavelength (d) frequency (d) ozone layer controls O2 / H2 radio in atmosphere

1. If a source is transmitting electromagnetic wave of frequency 7. An electromagnetic wave is propagating along Y-axis. Then
8.2 × 106 Hz, then wavelength of the electromagnetic waves (a) oscillating electric field is along X-axis and

.IN
transmitted from the source will be oscillating magnetic field is along Y-axis
(a) 36.6 m (b) 40.5 m (b) oscillating electric field is along Z-axis and
(c) 42.3 m (d) 50.9 m oscillating magnetic field is along X-axis
AL
2. In an apparatus, the electric field was found to oscillate with (c) both oscillating electric and magnetic fields are
an amplitude of 18 V/m. The magnitude of the oscillating along Y-axis, but phase difference between them
N
magnetic field will be is 90°
(a) 4 × 10–6 T (b) 6 × 10–8 T (d) both oscillating electric and magnetic fields are
R

(c) 9 × 10–9 T (d) 11 × 10–11 T mutually perpendicular in arbitrary direction


U

3. A TV tower has a height of 100m. How much population is 8. The frequency of electromagnetic wave, which best suited
covered by the TV broadcast if the average population to observe a particle of radii 3 × 10–4 cm is of the order of
JO

density around the tower is 100 km–2 (radius of the earth (a) 1015 (b) 1014
= 6.37 × 106 m) (c) 1013 (d) 1012
(a) 4 lakh (b) 4 billion
U

9. If e0 and m0 are the electric permittivity and magnetic


(c) 40,000 (d) 40 lakh permeability in vacuum, e and m are corresponding quantities
ED

4. In a plane electromagnetic wave propagating in space has in medium, then refractive index of the medium is
an electric field of amplitude 9 × 103 V/m, then the amplitude
of the magnetic field is e e 0m e 0m 0 em
(a) (b) (c) (d)
(a) 2.7 × 1012 T (b) 9.0 × 10–3 T e0 em 0 em e 0m 0
(c) 3.0 × 10–4 T (d) 3.0 × 10–5 T 10. A wave is propagating in a medium of electric dielectric
5. The ratio of electric field vector E and magnetic field vector constant 2 and relative magnetic permeability 50. The wave
impedance of such a medium is
æEö
H i.e., çç H ÷÷ has the dimensions of (a) 5 W (b) 376.6 W (c) 1883 W (d) 3776 W
è ø
11. The frequency of electromagnetic wave, which best suited
(a) resistance to observe a particle of radii 3 × 10–4 cm is of the order of
(b) inductance
(a) 1015 (b) 1014
(c) capacitance
(c) 1013 (d) 1012
(d) product of inductance and capacitance
12. The relation between electric field E and magnetic field H in
6. The energy of electromagnetic wave in vacuum is given by an electromagnetic wave is
the relation
µ0
E2 B2 1 1 (a) E = H (b) E= H
(a) + (b) e 0E 2 + µ0B2 e0
2e 0 2µ 0 2 2

E 2 + B2 1 B2 µ0 e0
(c) (d) e0E 2 + (c) E= H (d) E= H
c 2 2µ0 e0 µ0
Electromagnetic Waves 617

13. To double the covering range of a T.V. transmitter tower, its 23. Which of the following has/have zero average value in a
height should be made plane electromagnetic wave ?
(a) 2 times (b) 4 times (a) Both magnetic and electric field
(c) 2 times (d) 8 times (b) Electric field only
14. The transmitting antenna of a radiostation is mounted (c) Magnetic energy
vertically. At a point 10 km due north of the transmitter the (d) Electric energy
peak electric field is 10–3 Vm–1. The magnitude of the radiated
magnetic field is 24. For sky wave propagation of a 10 MHz signal, what should
be the maximum electron density in ionosphere?
(a) 3.33 × 10–10 T (b) 3.33 × 10–12 T
(c) 10–3 T (d) 3 × 105 T (a) ~1.2 × 1012m–3 (b) ~106m–3
15. It is possible to take pictures of those objects which are not (c) ~1014 m–3 (d) ~1022m–3
fully visible to the eye using camera films sensitive to 25. A new system of unit is evolved in which the values of µ0
(a) ultraviolet rays (b) infrared rays
and Î0 are 2 and 8 respectively. Then the speed of light in
(c) microwaves (d) radiowaves
this system will be
16. A plane electromagnetic wave is incident on a material
surface. If the wave delivers momentum p and energy E, (a) 0.25 (b) 0.5 (c) 0.75 (d) 1
then 26. Light wave is travelling along y-direction. If the
r

.IN
(a) p = 0, E = 0 (b) p ¹ 0, E ¹ 0 corresponding E vector at any time is along the x-axis, the
r
(c) p ¹ 0, E = 0 (d) p = 0, E ¹ 0 direction of B vector at that time is along
AL y
17. An electromagnetic wave, going through vacuum is (a) y-axis
described by E = E 0 sin(kx - wt) . Which of the following (b) x-axis x
is independent of wavelength ?
N
(c) + z-axis
(a) k (b) w
(d) – z-axis z
R

(c) k/w (d) kw


18. In an electromagnetic wave, the electric and magnetic fields 27. An electromagnetic wave of frequency n = 3.0 MHz
U

are 100 V m–1 and 0.265 A m–1. The maximum energy flow is passes from vacuum into a dielectric medium with
JO

(a) 26.5 W/m2 (b) 36.5 W/m2 permittivity Î = 4.0 . Then


(c) 46.7 W/m 2 (d) 765 W/m2
(a) wavelength is halved and frequency remains
19. In which one of the following regions of the electromagnetic
U

unchanged
spectrum will the vibrational motion of molecules give rise
to absorption ? (b) wavelength is doubled and frequency becomes half
ED

(a) Ultraviolet (b) Microwaves (c) wavelength is doubled and the frequency remains
(c) Infrared (d) Radio waves unchanged
20. The oscillating electric and magnetic vectors of an (d) wavelength and frequency both remain unchanged
electromagnetic wave are oriented along 28. In an electromagnetic wave
(a) the same direction but differ in phase by 90° (a) power is transmitted along the magnetic field
(b) the same direction and are in phase (b) power is transmitted along the electric field
(c) mutually perpendicular directions and are in phase (c) power is equally transferred along the electric and
(d) mutually perpendicular directions and differ in phase magnetic fields
by 90° (d) power is transmitted in a direction perpendicular to
both the fields
21. An electromagnetic wave going through vacuum is
described by E = E0sin(kx – wt); B = B0 sin (kx – wt). Which 29. A plane electromagnetic wave is incident on a plane surface
of the following equations is true? of area A, normally and is perfectly reflected. If energy E
strikes the surface in time t then average pressure exerted
(a) E0 k = B0w (b) E0 w = B0 k on the surface is (c = speed of light)
(c) E 0 B0 = w k (d) None of these (a) zero (b) E/Atc
22. A plane electromagnetic wave travels in free space along (c) 2E/Atc (d) E/c
x-axis. At a particular point in space, the electric field along r r
30. If E and B represent electric and magnetic field vectors of
y-axis is 9.3V m–1. The magnetic induction (B) along z-axis is
the electromagnetic waves, then the direction of
(a) 3.1 × 10–8 T (b) 3 × 10–5 T
–6
propagation of the waves will be along
(c) 3 × 10 T (d) 9.3 × 10–6 T
EBD_7179
618 PHYSICS
r r r r r r 39. The decreasing order of wavelength of infrared, microwave,
(a) B ´ E (b) E (c) B (d) E ´ B ultraviolet and gamma rays is
31. Which of the following statement is false for the properties (a) microwave, infrared, ultraviolet, gamma rays
of electromagnetic waves? (b) gamma rays, ultraviolet, infrared, micro-waves
(a) Both electric and magnetic field vectors attain the (c) microwaves, gamma rays, infrared, ultraviolet
maxima and minima at the same place and same time. (d) infrared, microwave, ultraviolet, gamma rays
(b) The energy in electromagnetic wave is divided equally 40. Which of the following are not electromagnetic waves?
between electric and magnetic vectors (a) cosmic rays (b) g-rays
(c) Both electric and magnetic field vectors are parallel to (c) b-rays (d) X-rays.
each other and perpendicular to the direction of 41. Which of the following radiations has the least wavelength ?
propagation of wave (a) g -rays (b) b -rays
(d) These waves do not require any material medium for
(c) a -rays (d) X -rays
propagation.
42. The frequency of electromagnetic wave, which is best suited
32. The electric and the magnetic field associated with an E.M.
to observe a particle of radius 3 × 10–4 cm is of the order of
wave, propagating along the +z-axis, can be represented by
(a) 1015 (b) 1014
r r r r r 13
ˆ B = B ˆjù
(a) éë E = E 0 i, (b) éë E = E 0 k, B = B0ˆi ùû (c) 10 (d) 1012
0 û
43. Pick out the longest wavelength from the following types of
r r r r radiation.
(c) éë E = E 0 ˆj, B = B0 iˆ ùû (d) éë E = E 0 ˆj,B = B0 kˆ ùû (a) blue light (b) gamma rays

.IN
33. The electric field associated with an e.m. wave in vacuum is (c) X-rays (d) red light
r DIRECTIONS for Qs. (44 to 50) : Each question contains
given by E = iˆ 40 cos (kz – 6 × 108t), where E, z and t are in
STATEMENT-1 and STATEMENT-2. Choose the correct answer
volt/m, meter and seconds respectively. The value of wave
AL
(ONLY ONE option is correct ) from the following-
vector k is (a) Statement -1 is false, Statement-2 is true
(a) 2 m–1 (b) 0.5 m–1 (b) Statement -1 is true, Statement-2 is true; Statement -2 is a
–1 (d) 3 m–1
N
(c) 6 m correct explanation for Statement-1
34. The ratio of amplitude of magnetic field to the amplitude of (c) Statement -1 is true, Statement-2 is true; Statement -2 is not
R

electric field for an electromagnetic wave propagating in a correct explanation for Statement-1
vacuum is equal to (d) Statement -1 is true, Statement-2 is false
U

(a) the speed of light in vacuum 44. Statement 1 : When variable frequency a.c. source is
(b) reciprocal of speed of light in vacuum
JO

connected to a capacitor, displacement current increases


(c) the ratio of magnetic permeability to the electric with increase in frequency.
susceptibility of vacuum Statement 2 : As frequency increases conduction current
(d) unity
U

also increases.
35. The condition under which a microwave oven heats up a 45. Statement 1 : Short wave bands are used for transmission
ED

food item containing water molecules most efficiently is of radio waves to a large distance.
(a) the frequency of the microwaves has no relation with
Statement 2 : Short waves re reflected by ionosphere.
natural frequency of water molecules.
(b) microwaves are heat waves, so always produce heating. 46. Statement 1 : Television signals are received through sky-
(c) infra-red waves produce heating in a microwave oven. wave propagation.
(d) the frequency of the microwaves must match the Statement 2 : The ionosphere reflects electromagnetic waves
resonant frequency of the water molecules. of frequencies greater than a certain critical frequency.
36. The frequencies of X-rays, g-rays and ultraviolet rays are 47. Statement 1 : Ultraviolet radiations of higher frequency
respectively a, b and c then waves are dangerous to human being.
(a) a < b, b > c (b) a > b, b > c
Statement 2 : Ultraviolet radiation are absorbed by the
(c) a > b, b < c (d) a < b, b < c
atmosphere.
37. If l v , l r and l m represent the wavelength of visible light 48. Statement 1 : Environmental damage has increased the
x-rays and microwaves respectively, then amount of ozone in the atmosphere.
(a) lm > l x > l v (b) l v > lm > l y Statement 2 : Increase of ozone increases the amount of
ultraviolet radiation on earth.
(c) l m > l v > l x (d) l v > l x > l m
49. Statement 1 : The earth without atmosphere would be
38. The electric and magnetic field of an electromagnetic wave
inhospitably cold.
are in
(a) phase and parallel to each other Statement 2 : All heat would escape in the absence of
(b) opposite phase and perpendicular to each other atmosphere.
(c) opposite phase and parallel to each other 50. Statement 1 : Radio waves can be polarised.
(d) phase and perpendicular to each other Statement 2 : Sound waves in air are longitudinal in nature.
Electromagnetic Waves 619

Exemplar Questions 8. An electromagnetic wave travels in vacuum along z-direction


1. One requires 11 eV of energy to dissociate a carbon monoxide E = ( E1iˆ + E 2 ˆj) cos ( kz - wt ) . Choose the correct options
molecule into carbon and oxygen atoms. The minimum from the following
frequency of the appropriate electromagnetic radiation to
(a) The associated magnetic field is given as
achieve the dissociation lies in
B = ( E1iˆ - E 2 ˆj) cos ( kz - wt )
(a) visible region 1
(b) infrared region c
(c) ultraviolet region (b) The associated magnetic field is given as
(d) microwave region
B = ( E1iˆ - E 2 ˆj) cos ( kz - wt )
1
2. A linearly polarised electromagnetic wave given as
c
E = E ˆi cos ( kz - wt ) is incident normally on a perfectly
0
(c) The given electromagnetic field is circularly polarised
reflecting infinite wall at z = a. Assuming that the material of (d) The given electromagnetic wave is plane polarised
the wall is optically inactive, the reflected wave will be given

.IN
NEET/AIPMT (2013-2017) Questions
as
9. An electromagnetic wave of frequency n = 3.0 MHz passes
(a) E = E ˆi ( kz - wt )
r 0 from vacuum into a dielectric medium with relative
AL
(b) Er = E0 iˆ cos ( kz + wt ) permittivity e = 4.0. Then [NEET Kar. 2013]
(a) wavelength is doubled and frequency is unchanged
(c) E = -E iˆ cos ( kz + wt )
r 0 (b) wavelength is doubled and frequency becomes half
N
(d) Er = E 0ˆisin ( kz - wt ) (c) wavelength is halved and frequency remains
unchanged
R

3. Light with an energy flux of 20 W/cm2 falls on a non-reflecting


surface at normal incidence. If the surface has an area of 30 (d) wavelength and frequency both remain unchanged
U

cm2, the total momentum delivered (for complete absorption) 10. Light with an energy flux of 25 × 104 Wm– 2 falls on a
during 30 min is perfectly reflecting surface at normal incidence. If the surface
JO

(a) 36 × 10–5 kg-m/s (b) 36 × 10–4 kg-m/s area is 15 cm2, the average force exerted on the surface is :
4
(c) 108 × 10 kg-m/s (d) 1.08 × 107 kg-m/s [2014]
4. The electric field intensity produced by the radiations coming (a) 1.25 × 10– 6 N (b) 2.50 × 10– 6 N
U

from 100 W bulb at a 3 m distance is E. The electric field (c) 1.20 × 10– 6 N (d) 3.0 × 10– 6 N
intensity produced by the radiations coming from 50 W bulb 11. A radiation of energy ‘E’ falls normally on a perfectly
ED

at the same distance is reflecting surface. The momentum transferred to the surface
E is (C = Velocity of light) [2015]
(a) (b) 2E
2
2E 2E
E (a) (b)
(c) (d) 2 E C C2
2
5. If E and B represent electric and magnetic field vectors of E E
(c) 2 (d)
the electromagnetic wave, the direction of propagation of C C
electromagnetic wave is along 12. The energy of the em waves is of the oder of 15 keV. To
(a) E (b) B which part of the spectrum does it belong? [2015 RS]
(c) B × E (d) E × B (a) Infra-red rays (b) Ultraviolet rays
6. The ratio of contributions made by the electric field and (c) g-rays (d) X-rays
magnetic field components to the intensity of an EM wave is
13. Out of the following options which one can be used to
(a) c : 1 (b) c2 : 1
produce a propagating electromagnetic wave ? [2016]
(c) 1 : 1 (d) c :1 (a) A charge moving at constant velocity
7. An EM wave radiates outwards from a dipole antenna, with (b) A stationary charge
E0 as the amplitude of its electric field vector. The electric (c) A chargeless particle
field E0 which transports significant energy from the source (d) An accelerating charge
falls off as 14. In an electromagnetic wave in free space the root mean
1 1 square value of the electric field is Erms = 6V/m. The peak
(a) 3 (b) 2
r r value of the magnetic field is :- [2017]
1 (a) 2.83 × 10–8 T (b) 0.70 × 10–8 T
(c) (d) remains constant
r (c) 4.23 × 10–8 T (d) 1.41 × 10–8 T
EBD_7179
620 PHYSICS

Hints & Solutions


EXERCISE - 1 7. (b)
8. (a) Size of particle = 2 ×(3 × 10–4) = 6 × 10–4 m. To observe a
1. (b) 2. (a) 3. (b) 4. (c)
particle, the wavelength of electromagnetic waves must
5. (a) 6. (c) 7. (d) 8. (a)
be of the size of particle.
9. (d) l = 10Å = 10 ´ 10 –10 m = 10 –9 m 9. (d) We know that velocity of electromagnetic wave in
X- ray wavelength is of the order of IÅ 1
10. (b) vacuum (v0 ) = and velocity of electromagnetic
11. (a) Energy density (EM waves) m 0e 0
2 1
2 æ E0 ö 1 wave in medium is (v) = .
= e0 E rms = e 0 ç = e0 E02 me
è 2 ÷ø 2
Therefore refractive index of the medium
12. (a) 13. (d)
14. (a) In vacuum velocity of all EM waves are same but their vel. of E.M.wave in vacuum (v 0 )
(m ) =
wavelengths are different. vel. of E.M. wave in medium (v)
15. (c) Microwave region wavelength = 10–3 m to 1m
16. (a) 1/ m0 e0 me

.IN
= = m e
m0 1/ me 0 0
17. (b) Wave impedance = Z = = 376.6 W 10. (c) 11. (a) 12. (c)
e0 AL
18. (b) Velocity of EM waves 13. (b) It is in accordance with relation = 2 hR
[where h is height of antenna]
1 So, it must be made 4 times, to make coverage distance 2
= = 3 ´ 108 m/s = velocity of light
N
m 0 Î0 times.
E0
R

1 14. (b) B0 =
19. (c) Speed of EM waves in vacuum = = constant c
m 0 Î0
U

E0 - Electric field, c - speed of light, B0 - Magnetic Field.


20. (c) 10 -3
JO

21. (b) Infrared radiations reflected by low lying clouds and B0 = = 3.33 ´ 10 -12 T
8
keeps the earth warm. 3 ´ 10
22. (b) 1
15. (b) It is given by relation =
U

23. (b) All components of electromagnetic spectrum travel in


m0e0
vacuum with velocity 3 × 108 m/s.
ED

24. (a) 25. (a) 16. (b) EM waves carry momentum and hence can exert
pressure on surfaces. They also transfer energy to the
surface so p ¹ 0 and E ¹ 0.
EXERCISE - 2
2p
c 3 ´108 17. (c) The angular wave number k = ; where l is the
1. (a) Here, l = = = 36.6 m. l
n 8.2 ´ 106 wave length. The angular frequency is w = 2pn .
2. (b) Here, E0 = 18 V/m; B0 = ?
k 2p / l 1 1
E 18 The ratio = = = = constant
B0 = 0 = = 6 ´10 -8 T w 2pn nl c
c 3 ´ 108 18. (a) Here, amplitude of electric field, E0 = 100 V/m; amplitude
3. (d) d = 2 h R . of magnetic field, H0 = 0.265 A/m. We know that the
Population covered = p d2 × population density maximum rate of energy flow,
S = E0 × H0 = 100 × 0.265 = 26.5 W/m2.
E 0 9 ´ 10 3 19. (b) Molecular spectra due to vibrational motion lie in the
4. (d) B 0 = = = 3 ´ 10 -5 T.
c 3 ´ 10 8 microwave region of EM-spectrum. Due to Kirchhoff’s
5. (a) law in spectroscopy the same will be absorbed.
r r
1 20. (c) E and B are mutually perpendicular to each other and
6. (d) e 0 E 02 is electric energy density.. are in phase i.e., they become a zero and minimum at
2
the same place and at the same time.
B2
is magnetic energy density.. E0 2p
2m 0 21. (a) = c . also k = and w = 2pv
B0 l
1 B2 These relation gives E0 k = B0w
So, total energy = e 0 E 02 + 0
2 2m 0
Electromagnetic Waves 621
22. (a) Velocity of light According to conservation of energy UE = UB
E E 9.3 B2
C= ÞB= = = 3.1 ´ 10-8 T e 0m 0 =
B C 3 ´ 108
E2
23. (a) Both magnetic and electric fields have zero average
value in a plane e.m. wave. B 1
24. (a) If maximum electron density of the ionosphere is Nmax = e 0m0 =
E C
per m 3 then the critical frequency fc is given by 35. (d) Required condition : Frequency of microwaves =
fc = 9(Nmax)1/2 Resonant frequency of water molecules.
Þ 10 ´ 106 = 9(N)1/ 2 Þ N = 1.2 ´ 1012 m -3 36. (a) Vg- rays > Vx -rays > VUV -rays
1 1 1 37. (c) l m > l v > l x
25. (a) The speed of light C = = = = 0.25
m0 e 0 2´8 4 38. (d)
39. (a) The decreasing order of the wavelengths is as given
26. (c) Light wave is an electromagnetic wave in which E below :
microwave, infrared, ultraviolet, gamma rays.
and B are at right angles to each other as well as at
right angles to the direction of wave propagation. 40. (c) b -rays are the beam of fast moving electrons.
27. (a) Frequency remains constant during refraction l decreasin g
41. (a) ¾¾¾¾¾¾
®
1 c RMIVUXGC
vmed = =

.IN
µ0 Î0 ´4 2
R ® Radio waves M ® Micro waves
l med vmed c / 2 1
= = = I ® Infra red rays V ® Visible rays
l air vair c 2
\ wavelength is halved and frequency remains
AL U ® Ultraviolet rays X ® x rays
unchanged. G ® g rays C ® Cosmic rays
28. (d)
Þ g rays has least wavelength
N
E
29. (c) Incident momentum, p = c
R

c 42. (b) Size of particle = l =


For perfectly reflecting surface with normal incidence n
U

2E Dp 2E
Dp = 2p = ; F= = 3 ´ 108
c Dt ct 3 ´ 10 -6 =
JO

F 2E n
P= =
A ctA n = 1014 Hz
30. (d) Direction of propagation of Electro-magnetic waves is
U

perpendicular to electric field and magnetic field. Hence However, when frequency is higher than this, wavelength
direction is given by pointing is still smaller. Resolution becomes better.
ED

E´B
43. (d) Wavelength of red light is longest.
vector S = E ´ H = . 44. (b)
m0
45. (c) Short wave (wavelength 30 km to 30 cm). These waves
31. (c) Electromagnetic waves are the combination of mutually are used for radio transmission and for general
perpendicular electric and magnetic fields. So, option
communication purpose to a longer distance from
(c) is false.
ionosphere.
32. (a) E.M. wave always propagates in a direction
46. (a) In sky wave propagation , the radio waves having
perpendicular to both electric and magnetic fields. So,
frequency range 2 MHz to 30 MHz are reflected back
electric and magnetic fields should be along + X and +
by the ionosphere. Radio waves having frequency
Y- directions respectively. Therefore, option (a) is the
nearly greater than 30 MHz penetrates the ionosphere
correct option.
and is not reflected back by the ionosphere. The TV
33. (a) On comparing the given equation to
r signal having frequency greater than 30 MHz therefore
E = a0iˆ cos (wt – kz), w = 6 × 108z, cannot be propagated through sky wave propagation.
In case of sky wave propagation, critical frequency is
2p w w 6 ´ 108 defined as the highest frequency is returned to the
k= = or k = = = 2 m -1
r c c 3 ´ 108 earth by the considered layer of the ionosphere after
34. (b) The average energy stored in the electric field having sent straight to it. Above this frequency, a wave
will penetrate the ionosphere and not reflected by it.
1
UE = e0 E 2 47. (c) The wavelength of these wave ranges between 4000 Å
2 to 100 Å that is smaller wavelength and higher
The average energy stored in the magnetic field frequency. They are absorbed by atmosphere and
1 B2 convert oxygen into ozone. They cause skin diseases
= UB = , and they are harmful to eye and cause permanent
2 m0
blindness.
EBD_7179
622 PHYSICS

48. (a) Ozone layer in the stratosphere helps in protecting life Momentum of the incident light
of organism form ultraviolet radiation on earth. Ozone
U
layer is depleted due to of several factors like use of = (Q c = 3 × 108)
chlorofluoro carbon (CFC) which is the cause of c
environmental damages. Momentum of incident light
49. (b) In the absence of atmosphere, all the heat will escape
20 ´ 30 ´ ( 30 ´ 60)
from earth’s surface which will make earth in hospitably = = 36 × 10–4 kg-ms–1
cold. 3 ´ 108
50. (c) Radio waves can be polarised because they are As no reflection from the surface and for comidete
transverse in nature. Sound waves in air are longitudinal absorption.
in nature. Momentum of the reflected light = 0
Hence, momentum delivered to the surface
EXERCISE - 3 = Charge in momentum. = (pf – fi)
Exemplar Questions = 36 × 10–4 – 0 =36 × 10–4 kg-ms–1
1. (c) As we know that, 4. (c) As we know that the electric field intensity on a surface
due to incident radiation is,
E = hn
Iav µ E02
As given that h = 6.62 × 10–34 J-s
E = 11 eV =11 × 1.6 × 10–19 Pav
µ E02 (A is constant)

.IN
n= ? A
11 eV = hn Here, Pav µ E02
E0 µ
11 ´ 1.6 ´ 10 -19 11´1.6 ´10-19
AL
So, Pav
So, n = J = J
h 6.62 ´ 10-34
= 2.65 × 1015 Hz
( E0 )1 ( Pav )1
N
\ =
So, that frequency radiation belongs to ultraviolet region. ( E 0 )2 ( Pav )2
R

2. (b) The type of wave doesn't change when a wave is


reflected from denser medium but only its phase changes ( E0 )1 100 é 2ù
U

= =ê ú
by 180°. As E is along positive x-axis so reflected ray will be ( E 0 )2 50 ë 1 û
along negative x-axis and its component will also be opposite
JO

to earlier in (–z) direction and phase will change. ( E0 )2 = ( E0 )1 / 2


ˆ ˆi = - ˆi and additional phase
For the reflected wave zˆ = - z, 5. (d) The direction of propagation of electromagnetic wave
U

of p in the incident wave. is perpendicular to both electric field E and magnetic field B,
As given that the incident electromagnetic wave is, i.e., in the direction of E × B by right thumb rule.
ED

The diagram given below


E = E ˆi cos ( kz - wt )
0
x
So, the reflected electromagnetic wave is
E(x) B(–y) E(x) B(–y)
Er = E 0 ( -ˆi ) cos ( k ( -z ) - wt + p )

= - E 0 ˆi cos ( - ( kz + wt ) + p )
z

y B (y) E(–x)
= - E 0 ˆi cos ( p - ( kz + wt ) ) E(–x) B(y)

So, electromagnetic wave is along the z-direction which is


= E0 iˆ cos ( kz + wt ) give the cross product of E and B direction is perpendicular
3. (b) As we know that r r
to E and B from E to B . i.e., (E × B) in z-direction.
the momentum of incident light
6. (c) Intensity in terms of electric field
U(total energy)
= 1
c e 0 E 02
Uav =
2
As given that the energy flux f = 20W/cm2
Intensity in terms of magnetic field
Surface are A = 30 cm2
Time for total momentum delivered 1 B20
Uav =
t = 30 min =30 × 60 sec 2 m0
So, total energy falling in time t sec is We also know that the relationship between E and B is
U = fAt =20 × 30 × (30 × 60) J E0 = cB0
Electromagnetic Waves 623

So the average energy by electric field is NEET/AIPMT (2013-2017) Questions


1 1 9. (c) Given: frequency f = 2MHz, relative permittivity Îr = 4
e E 2 = e E ( cB0 )
2
(Uav) =
2 0 0 2 0 0 From formula,
c c l
1 æ 1 ö velocity n = = Þ l'=
= e0 ´ c B çQ c =
2 2
÷ Îr 2 2
2 è m 0 e0 ø
[Since frequency remains unchanged]
1 1 Dp 2IA
\ (Uav)Electric field = e0 ´ B20 10. (b) Average force Fav = = (Q Power = F.V)
2 m0e0 Dt c

1 B20 2 ´ 25 ´ 104 ´ 15 ´ 10 - 4
= = (Uav)Magnetic field =
2 m0 3 ´ 108

So, the energy in electromagnetic wave is divided equally = 2.50 × 10– 6 N


between electric field vector and magnetic field vector. E
11. (a) Momentum of light falling on reflecting surface p =
Then, the ratio of contributions by the electric field and C
magnetic field components to the intensity of an As surface is perfectly reflecting so momentum reflect p1 =
electromagnetic wave is E

.IN

C
( U av )electric field
Ratio = =1: 1
( U av )Magnetic field
AL E E
=P
7. (c) As we know that, the electric field is inversly C C
N
æ 1ö
proportional to r, so ç E 0 µ ÷ So, momentum transferred
è rø
R

From a diode antenna, an electromagnetic waves are radiated E æ E ö 2E


– –
C çè C ÷ø
= P – P1 = =
U

outwards from dipole antenna with the amplitude of electric C


field vector (E0) which transports significant energy from 12. (d) Energy of x-ray is (100 ev to 100 kev)
JO

the source falls off intensity inversely as the distance (r) Hence energy of the order of 15 kev belongs to x-rays.
from the antenna, i.e., 13. (d) To generate electromagnetic waves we need
U

accelerating charge particle.


radiated energy æç E 0 µ 1 ö÷ 14. (a) Given, Erms = 6 V/m
ED

è rø
E rms
8. (d) In electromagnetic wave, =c
Brms
the electric field vector is
E rms
E = ( E1iˆ + E 2 ˆj) cos ( kz - wt ) Þ Brms =
c
...(i)
and the associated magnetic field vector,
B0
Brms = Þ B0 = 2 Brms
E E ˆi + E 2 ˆj 2
B= = 1 cos ( kz - wt )
c c
E rms
So, E and B are perpendicular to each other and the B0 = 2 ´ From equation (i)
C
propagation of electromagnetic wave is perpendicular
to E as well as B, so the electromagnetic wave plane 2 ´6
polarised. = = 2.83 × 10–8 T
3 ´ 108
EBD_7179
624 PHYSICS

Ray Optics and


24 Optical Instruments

REFLECTION OF LIGHT (iii) If two plane mirrors are inclined to each other at 90º, the
It is the turning back of light in the same medium. emergent ray is always antiparallel to incident ray if it suffers
one reflection from each whatever be the angle of incidence.
Normal The same is found to hold good for three-plane mirrors
forming the corner of a cube if the incident light suffers one

.IN
reflection from each of them.
(iv) If there are two plane mirrors inclined to each other at an
Incident ray angle q, the no. of images of a point object formed are
r
Reflected ray
AL determined as follows :
i
360°
(a) If is even integer (say m), no. of images formed
q
N
Reflecting = (m – 1), for all positions of object.
Surface
R

i = angle of incidence æ 360° ö


(b) If çè ÷ is odd integer (say m), no. of images formed,
r = angle of reflection q ø
U

n = m, if the object is not on the bisector of mirrors and


Laws of Reflection n = (m – 1), if the object is on the bisector of mirrors.
JO

(i) Angle of incidence (i) = angle of reflection (r)


æ 360° ö
(ii) The incident ray, reflected ray and normal are always in (c) If çè ÷ is a fraction, the no. of images formed will
same plane. q ø
U

be equal to its integral part.


REFLECTION FROM PLANE SURFACE
ED

A plane mirror always forms a virtual image if object is


Plane mirror has infinitely large radius of curvature. It produces
real and forms a real image if the object is virtual.
virtual image of same size but laterally inverted. Image is as much
behind the mirror as much is the object in front of it. MIRROR FORMULA
(i) If the direction of the incident ray is kept constant and the 1 1 1
+ =
mirror is rotated through an angle q about an axis in the v u f
plane mirror, then the reflected ray rotates through an where, u = distance of the object from the pole of mirror
angle 2q. v = distance of the image from the pole of mirror
f = focal length of the mirror.
Mirror formula is valid only when the following
q conditions are satisfied :
(a) Object is placed very near to the principal axis.
90° – q
M2 (b) Object is placed far from the mirror.
Magnification
q
v v- f f I évù
m= = = or m = = – ê ú
u f u- f O ëu û
M1 where, I = size of the image and O = size of the object and negative
(ii) If an object moves towards (or away from) a plane mirror at
sign implies that image is inverted w.r.t the object.
a speed v, the image will also approach (or recede) at the
The above formulae are applicable only for paraxial rays (the rays
same speed v, i.e. the speed of image relative to object will
which makes very small angle with the principal axis).
be v – (–v) = 2v.
Ray Optics and Optical Instruments 625

Areal magnification : When a two dimensional object is placed 1. Rays retrace their path when their direction is reversed.
with its plane perpendicular to the principal axis, its magnification 2. Focal length of a mirror depends only on the curvature of
called superficial magnification or aerial magnification and is given æ Rö
by the mirror ç f = ÷ . It does not depend on the material of
è 2ø
2 the mirror or on the wavelength of incident light.
Area of image æ vö 3. Focal length of concave mirror is always negative.
Ms = =m2 =ç ÷
Area of object è uø Focal length of convex mirror is always positive.
1 1
Sign Conventions for Mirror and Lenses 4. The graph of versus for a concave mirror, if real
v u
New cartesian sign conventions : image is formed.
1. All the distances are measured from pole of spherical mirror
and optical centre in case of lenses. 1/v
2. The distances measured in a direction opposite to the direction
of incident light is taken as negative and vice-versa.
3. The heights measured upward and perpendicular to the 1/f
principal axis of mirror are taken as positive and vice -versa.
4. Angle measured from the normal in the anticlockwise
1/f 1/u
direction is positive and vice-versa.

.IN
5. The graph shows variation of v with change in u for a
mirror.
v
Incident ray AL
– ve +ve +ve
Object For plane mirror
P
N
– ve For
(Pole) 2f spherical mirror
R

f
U

f 2f u
6. A person needs a plane mirror of minimum half of his height
JO

Reflecting or refracting surface to see his full image.


IMAGE FORMED BY CONCAVE AND CONVEX MIRROR 7. A person standing in the middle of room can see complete
1
U

Image Formed by Concave Mirror wall behind him if the mirror in front of him is rd of height
3
of wall.
ED

Pos ition of Pos ition of


Magnification Nature of image 8. A convex mirror is used as a rear view mirror (called driver
object image
mirror).
Between P Behind the
+ve, m > 1 Virtual and erect 9. If two or more optical components produce magnification,
and F mirror
then overall magnification (m) is the product of magnification
– ve, Highly
At F At infinity Real and inverted due to each component,
magnified
i.e., m = m1 × m2× . . .
Between F • If m is negative, the image is inverted
Beyond C – ve, M agnified Real and inverted
and C
• If m is positive, the image is erect.
At C At C m = –1 Real and inverted 10. When an object moves with constant speed towards a
between F concave mirror from infinity to focus, the image will move
Beyond C Dis mished Real and inverted
and C away from the mirror slower in the beginning and with the
Highly speed of the object when it is at centre of curvature C and
At infinity At F Real and inverted
diminshed faster later on.
11. Concave mirrors are used as reflectors, as objective in
Image Formed by Convex Mirror
reflecting telescope and by doctors (ENT) to examine ears,
Position of Position of nose and throat. It is also used as shaving mirrors.
Magnification Nature of image 12. The inability of a spherical mirror (or lens) of large aperture
object image
Infront of Between P to focus the paraxial rays and marginal rays to the same
m < +1 Virual and erect
mirror and F point on the principal axis is called spherical aberration.
A t infinity At F m < < +1 Virtual and erect Due to this defect the image formed is blurred. This defect
can be removed by using parabolic mirror.
Keep in Memory 13. Chromatic aberration is absent in mirrors but present in
EBD_7179
626 PHYSICS

lenses. This is because the focal length of mirror is Example 3.


æ Rö An object of length 2.5 cm is placed at a distance of 1.5 f
independent of wavelength of light ç f = ÷ but that of from a concave mirror where f is the magnitude of the
è 2ø
lens is dependent on wavelength. focal length of the mirror. The length of the object is
14. Different colour rays travel with different velocity in a perpendicular to the principal axis. Find the length of the
medium but velocity of all coloured rays is same in vacuum image. Is the image erect or inverted?
(and air). Solution :
15. If a hole is formed in a mirror, then also we will get full image The given situation is shown in figure.
with no hole in the image. The hole will only reduce the
intensity of rays forming the image.
Newton’s Formula f
F
In case of spherical mirrors if object distance (x1) and image O
distance (x 2 ) are measured from focus instead of pole, 1.5f

u = (f + x1) and v = (f + x2), the mirror formula 1 + 1 = 1 The focal length f = –f and u = –1.5f, we have
v u f
1 1 1 1 1 1
+ = or - + =-
1 1 1 u v f 1.5f v f
reduces to + =
( f + x 2 ) ( f + x1 ) f

.IN
1 1 1 -1
or = - = or v = –3f
which on simplification gives x1 x2 = f 2 v 1.5f f 3f
v 3f h
Example 1. = -2 or 2 = -2
AL Now m = - =
u - 1.5f h1
The focal length of a concave mirror is 30 cm. Find the
position of the object in front of the mirror, so that the image or h 2 = -2 h1 = -5.0 cm
N
is three times the size of the object. The image is 5.0 cm long. The minus sign shows that it is
Solution : inverted.
R

Here image can be real or virtual. If the image is real


f = –30, u = ?, m = –3 REFRACTION
U

Whenever a wave is bounced back into same medium at an


f -30 interface reflection is said to have occurred. Transmission of a
JO

m= Þ -3 = Þ u = –40 cm.
f -u -30 - u wave into the second medium at an interface is called refraction.
If the image is virtual When a ray of light is passing from denser to rarer medium, it
bends away from the normal and when passing from rarer to
U

f -30
m= Þ 3= Þ u = –20 cm. denser medium, it bends towards the normal.
f -u -30 - u
ED

• When a ray of light passing from one medium to another


Example 2. medium frequency and ph ase do not change while
A square ABCD of side 1mm is kept at distance 15 cm infront wavelength and velocity changes.
of the concave mirror as shown in the figure. The focal • Twinkling of stars, appearance of sun before actual sunrise
length of the mirror is 10 cm. Find the perimeter of its image. and after actual sunset etc. are due to atmospheric refraction.
Laws of Refraction
/ / ///

(i) Snell's Law : When a light ray is incident on a surface


/ ///////

B C separating two transparent media, the ray bends at the time


///////////////////////

of changing the medium.


A D
sin i v1 m 2
15cm. i.e. sin r = v = m = 1m 2 ,
2 1
where i = angle of incidence
// / / /

Solution : r = angle of refraction


v v1 = vel. of light in 1st medium
v = – 30, m = - =–2
u v2 = vel. of light in 2nd medium
\ A ¢B ¢ = C ¢ D ¢ = 2 ´ 1 = 2mm 1
or 1m2 = refractive index of 2nd medium w.r.t. the 1st medium.
2m
B ¢C ¢ A ¢D ¢ v 2 m1 = refractive index of 1st medium w.r.t vacuum (or air)
Now = = 2 = 4 Þ B ¢C ¢ = A ¢D ¢ = 4 mm m2 = refractive index of 2nd medium w.r.t vacuum (or air)
BC AD u
(ii) The incident ray, the normal and the refracted ray at the
\ Perimeter = 2 + 2 + 4 + 4 = 12 mm
interface all lie in the same plane.
Ray Optics and Optical Instruments 627

Refractive Index of the Medium Transmission of Wave


(a) Refractive index of second medium w.r.t. first medium (i) The equation of the wave refracted or transmitted to the
m 2 c / v2 v1 ( )
next medium is given by : y = A´´ sin wt - k '0 x . This is
1m2 = = = independent of the nature (rarer/denser) of the medium.
m1 c / v1 v2
The wave is not inverted.
Velocity of light in first medium (ii) The amplitude (A´´) of the transmitted wave is less than
= that (A) of the incident wave.
Velocity of light in second medium
(iii) The angular frequency remains unchanged. However the
(b) Absolute refractive index of medium (n or µ)
wave number changes. Note that the phase of the
Velocity of light in vacuum c sin i
= =
Velocity of light in medium v sin r
= n=µ ( '
)
transmitted wave is wt - k 0 x and that of the incident
wave is (wt – kx).
Refractive index is the relative property of two media. If the (iv) The compression or rarefaction are transmitted as such and
first medium carrying the incident ray is a vacuum, then the same is the case with the crest or trough.
Sin i The wave velocity (vp), the angular frequency (w) and the wave
ratio is called the 'absolute refractive index of the number (k) are related as vp = w/k = nl. Let the wave velocity in
Sin r
second medium'. The relative refractive index of any two the medium to which the wave is transmitted be v'p = w/k´ = nl'.
media is equal to the ratio of their absolute refractive indices. (i) If second medium is denser, in comparison to first medium
Therefore, if the absolute refractive index of medium 1 and 2 (i.e. m2 > m1), then from Snell’s law
sin i m 2 v 2

.IN
be n1 and n2 respectively, then the refractive index of medium = =
2 with respect to medium 1 is sin r m1 v1 here m2 >m1 so v1 > v 2
n2 Sin i Þ k1 < k2 and l1 > l2.
1 n2 = n12 = = It means that if ray goes from rarer medium to denser
n1 Sin r
AL
medium (i.e. from first medium to second medium), then
n1 sin i = n2 sin r wave number increases & wavelength decreases.
According to cauchy’s formula (ii) If second medium is rarer in comparison to first medium,
N
B then from Snell’s law
m = A+ 2
R

l sin i m 2 v1
= =
where, A and B are cauchy’s constant. sin r m1 v 2 here m2 < m1 so v1 < v 2
U

lred > lviolet Þ k1 > k2 and l1 < l2.


JO

so, mred < mviolet It means that when ray goes from denser to rarer medium,
then wave number decreases & wavelength increases.
1
(c) 1n2 = (iii) No change in wave number k occurs on reflection.
2 n1
U

Image due to refraction at a plane surface, Apparent shift


(d) For three mediums 1, 2 and 3 due to successive refraction. O
ED

(Denser)
1n 2 × 2n 3 × 3n 1 = 1 Medium 1
t i m1
n 2 n 3 n1 I
´ ´ =1 r
B
n1 n 2 n3
A
(e) For two mediums, n1 and n2 are refractive indices with respect r m2 = 1
to vacuum, the incident and emergent rays are parallel then
n1 sinf1 = n2 sinf2. Medium 2
(Rarer)

f1 Medium (a) Here O = position of object


1 I = position of image
f2 Apparent depth 1 m 2 AI 1 t
= = = AI =
Medium
2 Real depth m m1 i.e. AO m Þ m
f2 (b) The image shifts closer to eye by an amount
f1 Medium æ 1ö
1 OI = AO – AI or Dt = ç 1 - ÷ t where t = thickness of
f1 è mø
medium over the object and Dt = apparent shift in its
Factors affecting refractive index : position towards the observer.
(i) Nature of the medium When an object in denser medium is seen through rarer
(ii) Wavelength medium, then apparent depth is less than real depth. But
(iii) Temperature of the medium-with increase in temperature, when an aeroplane or bird flying is seen by an observers in
refractive index of medium decreases. denser medium, the apparent height is more by (m – 1)t
EBD_7179
628 PHYSICS

Lateral shift by a slab of uniform thickness t, is • It is important to note that the above relationship is
x=
t
cos r
sin(i - r )
2.
24.1 valid only when boundaries undeviated.
In case of refraction, if i = 0 then r = 0. This means that the
ray which strikes to a boundary at 90° passes through the
i boundary undeviated.
3. If an object moves towards a denser medium with a
velocity v then the image moves faster with speed of mv
r as seen by the observer in denser medium.
t
m mv Denser medium
I Ov
x
If an object moves towards a rarer medium with a velocity
The apparent shift through a glass slab is in the direction of light v then the image moves slower with a speed v/m as seen
t by the observer in rarer medium.
v/m Denser medium
O
æ 1ö I v
I I ¢ = S = t ç1 - ÷
m I I¢ è mø 4. Denser the medium, smaller is the wavelength.

.IN
5. When light travels from one medium to another the
wavelength and velocity changes proportionally but
TOTAL INTERNAL REFLECTION (TIR) ALfrequency of rays remains the same
When the object is placed in an optically denser medium and if m2 a
mc
b
2m = cm= a
1
the incident angle is greater than the critical angle then the ray 6. and (‘a’ for air/vacuum)
m1 mb
of light gets reflected back to the originating medium. This
7. When a parallel compound slab consists of two media of
N
phenomenon is called total internal reflection.
Critical angle (ic) : When a ray passes from an optically denser equal thickness and refractive indices m1 and m 2 then
R

medium to an optically rarer medium, the angle of refraction r is 2m1m 2


the equivalent refractive index m =
m1 + m 2
U

greater than the corresponding angle of incidence i. From Snell’s law


r = 90° COMMON DEFAULT
JO

û Incorrect : If a mirror or a lens is painted black on one


half, then half of image will be formed.
i ic i > ic ü Correct : If half of the mirror or lens is blackened, we get
U

full image but with half the intensity.


ED

REFRACTION AT A SPHERICAL SURFACE


sin i m 2 For any curved spherical surfaces. Relation between u and v in
=
sin r m1 terms of refractive indices of the mediums and the radius of
Let m1 = m and m2 = 1 and let for i = ic, r = 90º then sin i c = 1/ m curvature of the curved spherical surface.
m 2 m1 m 2 - m1
1 - =
\ ic = sin -1 ; ic is called the critical angle v u R
m
This phenomenon takes place in shining of air bubble, sparkling P
of diamond, mirage and looming, in optical communication using i
optical fibre.
r
Keep in Memory
b g
a
1. On travelling through a series of parallel layers, light follows
the following formula
O m1 P m2 C I (Image)
(Object)
m sin q = constant = m1 sin q1 = m 2 sin q 2 = m 3 sin q 3

q1
Q
l1 Spherical surface separating two media
q2 (i) The lateral magnification in case of refraction from curved
l2 m æ vö
q3 surfaces m = 1 ç ÷
l3 m2 è u ø
m 2
(ii) Longitudinal magnification m ' = 2 m
m1
Ray Optics and Optical Instruments 629

m1 is refractive index of medium 1 through which light It is important that in the above formula, we cannot apply the
sign conventions of cartesian system rather following sign
passes first before meeting the interface and m2 is the
conventions are followed.
refractive index of medium 2 to which light encounters after
Focal length of a converging lens / mirror is taken as positive
it passes through the interface. and focal length of diverging lens/mirror is taken as negative.
REFRACTION BY A LENS Focal Length by Displacement Method
The focus point of a lens is the point where image of an object
placed at infinity is formed. And its distance from optical centre D2 - d 2
f =
of the lens is called focal length. 4D
Focal length of convex lens is +ve, and of concave lens is –ve. where D = distance between an object and screen
(i) Lens formula or thin lens formula and d = distance between two positions of lens.
1 1 1
- = d
v u f
(ii) Lens maker's formula,
1 æ m2 ö æ 1 1ö é1 1 ù
=ç - 1÷ ç - ÷ = (m - 1) ê - ú I1
f è m1 ø è R1 R2 ø ë R1 R2 û
m2
where 12 m =

.IN
D I2
m1
In the above formula m2 is refractive index of lens whereas m1 is Aperture of a lens : With reference to a lens, aperture means the
the refractive index of surrounding medium. effective diameter of its light transmitting area. So the brightness
R1 is the radius of curvature of the lens reached first by light and
AL
i.e. intensity of image formed by a lens which depends on the
R2 is the radius of curvature of the other surface. light passing through the lens will depend on the square of
aperture i.e. I µ (aperture)2
N
Magnification : m = v/u
This relation holds for both convex and concave lenses for real COMBINATION OF LENSES
R

as well as virtual images. (i) If a lens of focal length f is cut in two equal parts as shown in
Power of a lens, P = reciprocal of focal length expressed in metres. figure, each part will have focal length = f
U

1
i.e., P = . Its unit : dioptre(D). L L¢
JO

f (in metre)
(ii) If the above parts of lens are put in contact as shown then
To solve numerical problems use sign conventions
the resultant focal length will be,
while substituting values in above equations.
U

Equivalent focal length of two lenses separated by distance d


ED

1 1 1 2 f
d = + = i.e. F = L
F f f f 2 L¢

1 1 1 d (iii) If the two parts are put as shown, then L will behave as
= + -
F f1 f 2 f1 f 2 convergent lens of focal length f while the other (L´)
divergent of same focal length,
f1 f2 1 1 1
\ = + or F =¥
Equivalent focal length of lens - mirror combination : F +f -f
In such a case, the ray of light suffers two refraction from the lens L L¢
and one reflection from the mirror. The combination behaves as a \ P=0
mirror whose focal length is given by
(iv) If a lens of focal length f is divided into equal parts as shown,
then each part will have focal length f',

1 1 1
i.e. = + or f ´= 2 f
f f´ f´
1 2 1
= + i.e., each part will have focal length 2f.
F fl fm
(v) If these parts are put as shown, then the resultant focal
f l = focal length of lens , fm = focal length of mirror length of the combination will be
EBD_7179
630 PHYSICS

Keep in Memory
or 1. A ray entering a prism of angle A will not emerge out of
prism if A > 2qc where qc = critical angle
1 1 1
= + or F = f i.e. initial value. 2. Maximum deviation through a prism will occur when angle
F 2f 2f of incidence is 90°.
REFRACTION THROUGH A PRISM For this prism d = (m - 1)A
Prism is a transparent medium whose refracting surfaces are This shows that for a small angled prism, deviation is
inclined to each other. independent of angle of incidence.
(i) The angle of deviation is given by d = i + i¢ – A 3. Angle of emergence of a prism is 90° (called grazing
where A= angle of prism. For d to be minimum, i = i¢ and emergence) when angle of incidence
r = r¢
dm = A(m – 1) i = sin -1[ (m 2 - 1) sin A - cos A]
A
4. A single prism produces deviation and dispersion
U simultaneously.
T 5. Dispersion without deviation : When white light is incident
d on a combination of two prisms of different materials and
i i´ of suitable angles placed opposite to each other, the
r r´

.IN
Q R emergent light may have only dispersion without any
P
deviation ( of mean colour yellow).
S
A
B C
AL Flint
v Y
æ A + dm ö e ligh
t
sin ç Whit
è 2 ÷ø R
N
Refractive index of prism, m = Crown
æ Aö A¢
sin ç ÷
R

è 2ø For this to happen the conditions is


U

where dm = minimum angle of deviation A (m¢ - 1)


If angle of prism A is small, than dm is also small. =- ( For thin lenses)
A¢ (m - 1)
JO

( A + dm ) / 2 The net angular dispersion produced


\ m=
A/2 q = (w - w¢)d ( For thin lenses)
U

Plot of angle of deviation (d) versus angle of incidence (i) 6. Deviation without dispersion
for a triangular prism.
ED

d
t R
te ligh
Whi
v

dm A¢ -(m v - m r )
For this to happen A = (m ¢ - m ¢) .... (1)
v r
i i = i¢ i¢ i
æ wö
Net deviation d net = dç1 - ÷
Dispersion è w¢ ø
It is the breaking up of white ray of light into its constituents Equation (1) is said to be the condition of achromatism for
colours VIBGYOR. The band of seven constituents colours is combination of two prisms.
called spectrum. 7. Variation of refractive index of a medium with wavelength
Angular dispersion : It is defined as the difference of deviations causing incident light to split into constituent colours is
suffered by the extreme colours. dispersion.
i.e., q = dv - d r = (mv - m r ) A [For thin prism] b c
Cauchy’s equation : m = a + + , where a, b and c are
l 2 l 4
Dispersive power : It is defined as the ratio of angular dispersion constants.
to the mean deviation produced by the prism. 8. Rayleigh scattering law explains blue colour of sky. Intensity
d - d r mv - mr of scattered light is proportional to 1/l4. Hence the red
i.e., w = v = [For thin prism] light having highest value of lR scatters less.
d m
Ray Optics and Optical Instruments 631

9. Rainbow can be observed if light source is behind and the


COMMON DEFAULT
droplets are in front of the observer, i.e. when the back of a
û Incorrect : Using thin lens formula while the lens given in
person is towards the sun.
the numerical problem is thick.
It is a consequence of dispersion of sunlight by water
æ1 1 1ö
droplets due to a combinations of refraction and total internal ü Correct : The lens formula ç - = ÷ and lens maker’ss
reflection. If the rainbow is formed after one internal èv u f ø
reflection in the droplets, it is called a primary rainbow. In é1 æ 1 1 öù
formula ê = (m - 1)çç - ÷ú are valid only for thin
÷
this the violet ray emerges at an angle of 40.8º and red rays
ëê f è R 1 R 2 øûú
at an angle of 42.8º. If the rainbow is formed after two internal
reflections, it is called a secondary rainbow. In this the violet lenses.
rays emerge at 54º and red at 51º, i.e. the order of colours is Example 4.
reversed. The primary rainbow is brighter than the Consider the situation shown in figure. Find maximum
secondary. angle for which the light suffers total internal reflection
10. When a point source of light is placed at a depth h below at the vertical surface.
the surface of water of refractive index m , then radius of
bright circular patch on the surface of water is given by q
m = 1.0
h
R=
m2 -1 q’

.IN
m = 1.25
q’’
AL
h
Solution :
B The critical angle for this case is
N
1 4 4
11. When a lens made up of glass is immersed in water, its q ' ' = sin -1 = sin -1 or sin q ' ' =
R

1 .25 5 5
focal length changes.
p 3
U

Since q '' = - q ' , we have sin q ' = cos q '' =


1
= ( a
g m -1) æçè R1 - R1 ö÷ø 1
= ( w
) æçè R1 - R1 ö÷ø
g m -1
2
From Snell's law,
5
JO

f 1 2 f¢ 1 2
sin q 3 3
= 1.25 or sin q = 1.25 ´ sin q' = 1.25 ´ =

a
g m -1 g m / ma -1 ( g m - a m)m w sin q' 5 4
U

= w
= =
g m / mw -1 ( g m - w m)m a
f g m -1
3
or q = sin -1
ED

4
12. For achromatic combination of these lenses in contact, the
If q² is greater than the critical angle, q will be smaller than
w1 w2 this value. Thus, the maximum value of q¢ for which total
necessary condition is + =0
f1 f 2 internal reflection takes place at the vertical surface is
sin–1 (3/4).
13. For two lenses separated by distance d, spherical aberration
is minimum when d = f1 – f2. BO 1 AB + AO 1 t + mx
= m or, = m or =m
14. A convex lens forms a real image when the object is placed BI BI BI
beyond focus. When the object is placed between optical t
centre and focus, convex lens forms a virtual image. or BI = x +
m
15. A concave lens always form a virtual image for a real object.
The net shift is
16. A lens is called thin when the thickness of the lens is small
compared to the object distance, image distance, radii of æ tö æ 1ö
OI = OB - BI = ( x + t ) - çç x + ÷÷ = t çç1 - ÷÷
curvatures of the lens. è m ø è m ø
In the case of thick lens, the problem has to be solved
which is independent of x.
using formula for each interface one by one.
Example 5.
17. Real image (inverted) Virtual image (erect) A tank is filled with water to a height of 12.5 cm. The
Real image is formed by Virtual image is formed by actual apparent depth of a needle lying at the bottom of full tank
intersection of rays. extending the rays in the is measured by a microscope to be 9.4 cm. What is the
back direction. refractive index of water? If water is replaced by a liquid
This image can be brought This image cannot be of refractive index 1.63 upto the same height, by what
on a screen. brought on screen.
distance would the microscope have to be moved to focus
EBD_7179
632 PHYSICS

the needle again? separating surface at a distance of 100 cm from it.


Solution :
Example 8.
Here real depth =12.5 cm : apparent depth = 9.4 cm; m = ?
One end of a horizontal cylindrical glass rod (m = 1.5) of
real depth 12 .5 radius 5.0 cm is rounded in the shape of a hemisphere. An
m= = = 1 .33
Apparent depth 9 .4 object 0.5 mm high is placed perpendicular to the axis of
Now in second case, m = 1.63, real depth = 12.5 cm , apparent the rod at a distance of 20.0 cm from the rounded edge.
depth = ? Locate the image of the object and find its height.
Solution :
12 . 5 12 . 5
1 . 63 = or y = = 7 . 67 cm . Taking the origin at the vertex, u = –20.0 cm and R = 5.0 cm.
y 1 .63
Distance through which microscope has to be moved up m 2 m1 m 2 - m1
We have, - =
= 9.4 – 7.67 = 1.73 cm. v u R
Example 6. 1 .5 1 0 .5 1
or = + = or v = 30 cm
A converging lens has a focal length of 20 cm in air. It is v - 20 .0 cm 5.0 cm 20 cm
made of a material of refractive index 1.6. If the lens is
immersed in a liquid of refractive index 1.3. What will be
the new focal length of the lens?
Solution :

.IN
1 2
= (m1 - 1) m1 = 1.6, f1 = 20
f1 R 5.0cm
AL 20.0cm 25 cm
1 2 The image is formed inside the rod at a distance of 30 cm
= (1.6 - 1)
f1 R from the vertex.
0 .6 ´ 2 0 .6 ´ 2 ´ 20 m1 v 30 cm
N
1 The magnification is m = = = -1
or = \R = = 24 cm
20 R 10 m 2 u - 1.5 ´ 20 cm
R

1 2 1 1. 6 2 Thus, the image will be of same height (0.5 mm) as the


= (m1 - 1) ´ \ =( - 1) = object but it will be inverted.
U

f2 R f2 1. 3 R
Example 9.
JO

1 1 . 6 - 1 .3 2 0. 3 1 1 A convex lens focuses a distant object on a screen placed


=( ) = ´ =
f2 1 .3 24 1. 3 12 52 10 cm away from it. A glass plate (m = 1.5) of thickness 1.5
f2 = 52 cm. cm is inserted between the lens and the screen. Where
U

should the object be placed so that its image is again


Example 7. focused on the screen.
ED

Locate the image formed by refraction in the situation Solution :


shown in figure. The point C is the centre of curvature.

m=1.5
m=1.0

C
25 cm 20 cm The focal length of the lens is 10 cm. The situation with the
glass plate inserted is shown in figure. The object is placed
at O. The lens would form the image at I1 but the glass plate
intercepts the rays and forms the image at I on the screen.
Solution : The shift

m 2 m1 m 2 - m1 æ 1ö æ 1 ö
We have, - = ....(1) I1I = t çç1 - ÷÷ = (1 .5 cm ) ç1 - ÷ = 0 .5 cm .
v u R è m ø è 1 .5 ø
Here u = –25 cm, R = 20 cm, m1 = 1.0 and m2 = 1.5 Thus, the lens forms the image at a distance of 9.5 cm from
Putting the values in (1), itself. Using
1 .5 1 .0 1 .5 - 1 .0 1. 5 1 1 1 1 1 1 1 1 1 1
+ = or, = - - = Þ = - = -
v 25 cm 20 cm v 40 cm 25 cm v u f u v f 9.5 cm 10 cm
or, v = –100 cm. Þ u = 190 cm
As v is negative, the image is formed to the left of the
Ray Optics and Optical Instruments 633

Thus, the object should be placed at a distance of 190 cm \ Distance of image from the fish is mH + H +H/2 = H
from the lens. (m + 3/2) below the fish.
(b) Here we have to find the images of fish as seen by the
Example 10. eye.
A lens is cut into two equal pieces and they are placed as Let happarent be the apparent distance of the fish from
shown in figure. An object is placed at a distance of 12 cm the surface
left from the first half lens. The focal length of original
lens was 30 cm. Find the position of final image. h app. 1 mH
\ = ; h app =
h real m 2m
\ Image formed is (H + H/2m) below the, eye,
æ 1ö
O
i.e, H ç1 + ÷ below the eye.
è 2m ø
Also image of fish, formed by plane mirror is H/2 below
the mirror.
Solution : \ h real = H + H / 2 = 3H / 2
Focal length of each lens is 30 cm
(Q hreal is distance of fish image formed by the mirror
For first lens u = – 12, f = 30 cm
from the surface)
1 1 1 1 1 1 1 1

.IN
- = Þ = + =- + h app 1 3H
v1 u f v1 u f 12 30 Now h = Þ h app =
real. m 2m
or, v1 = – 20 cm. AL
For the second half lens image formed at v1 acts as object. æ 3Hö
Therefore object distance from second lens is = 20 + 20 = 40 \ Image formed is ç H + ÷ below the eye i.e,
è 2mø
cm or v1' = –40 cm
æ 3ö
N
f of this lens = 30 cm H ç1 + ÷ below the eye.
è 2m ø
1 1 1 1 1 1 1 1 1 1
R

\ - = Þ = + Þ =- + = Example 12.
v v'1 f v v '1 f v 40 30 120 A lens has a power of +5 diopter in air. What will be its
U

or v = 120 cm. power if completely immersed in water ?


Final image is 120 cm right from second lens.
JO

3 4
Example 11. Given m g = ; mw =
2 3
Consider the situation in figure. The bottom of the plot is Solution :
U

a reflecting plane mirror. S is a small fish and T is a human Let fa and fw be the focal lengths of the lens in air water
eye, refractive index of water is m (a) At what distance (s) respectively, then
ED

from itself, will the fish see the image (s) of the eye (b) At
what distance (s) from itself will the eye see the image (s) 1 mw
Pa = and Pw = f ; fa = 0.2 m = 20 cm
of the fish. fa w
Using lens maker’s formula
H
1 é 1 1 ù
Pa = = (mg - 1) ê - ú ...(i)
fa ë R1 R 2 û
H
S 1 æ mg öé 1 1 ù
H/2 =ç - 1÷ ê - ú
f w è m w ø ë R1 R 2 û
Solution :
(a) We have the formula mw é 1 1 ù
h app Þ Pw = f = (m g - m w ) ê R - R ú ...(ii)
m w ë 1 2û
= = h app = mH (from the surface of water)
h real 1 Dividing equation (ii) by equation (i), we get,
Now distance of fish from surface is H/2
Pw (m g - m w ) 1 P + 5 10
\ Image of eye as seen by fish is = H(m + ½) above the = = or Pw = a + = D
Pa (m g - 1) 3 3 3 3
fish.
also the apparent image of eye, again makes an image
with the plane mirror, the apparent distance of eye is OPTICAL INSTRUMENTS
mH + H from the plane mirror (i) Simple Microscope
\ Now image formed is mH + H below the plane mirror. It is known as simple magnifier & consist of a convergent
Distance of fish from the mirror is H/2
EBD_7179
634 PHYSICS

lens with object between its focus & optical centre & eye h
close to it. where q 0 = , q = h1
D ue
h1 D æ h1 öæ D ö
so M.P. = ´ = ç ÷ç ÷
u e h è h øçè u e ÷ø
T v h1 v
(since for objective m = = Þ = - , as u is –ive)
o u h u
-v æ D ö
so M .P. =
u çè ue ÷ø
(a) When image is formed at least distance of distinct
v0 æ Dö
vision, M =ç1 + ÷ = M 0 ´ M e
u0 è fe ø
(b) When the final image is formed at infinity
-v0 æ D ö
M = ç ÷
u0 è f e ø

.IN
The magnifying power of a simple microscope (M.P.) is (iii) Astronomical Telescope
visual angle with instrument q (a) If the final image is formed at a distance D,
M.P. = =
max visual angle for unaided eye q0
AL - f0 æ f ö
M = ç 1+ e ÷
fe è Dø
h h h q h D D
Here q 0 = , q = 1 = so M.P. = = ´ =
N
D v u q0 v h u fe D
and length of tube is L = f 0 +
(a) If image is at infinity [far point] then from lens formula
R

fe + D
1 1 1 1 1 1 D (b) If the final image is formed at infinity then
U

- = Þ - = i.e., u = f & M.P. =


v u f a -u f f f0
M = and the length of tube is l = f 0 + f e
JO

In this case M.P. is minimum if eye is least strained. fe


(b) If image is at D [near point] then u = –D
1 1 1
(iv) Galilean Telescope
U

and from lens formula - =


v u f f0
M = ; Length of tube L = fo – fe
ED

D D æ Dö fe
we get = (1 + ) so, M .P = ç1 + ÷
u f è fø
(v) Terrestrial Telescope
In this case M.P. is maximum and as final image is
close to eye, eye is under maximum strain. f0
M = ;
(ii) Compound Microscope fe
Length of tube L = fo + fe+ 4f, where f is the focal length of
erecting lens, which is used in this telescope.

Resolving Power of Microscope and Telescope


2 m sin q
(i) The resolving power of microscope is R =
l
where m = refractive index of medium between object and
objective of microscope and q = angle subtended by a
radius of the objective on one of the objects. (When both
objects are not self illuminous).
a
(ii) The resolving power of a telescope is R = where
1.22l
a = diameter of objective of telescope.

M.P. of compound microscope is defined as


visual angle with the instrument q
M.P = =
max. visual angle for unaided eye q 0
Ray Optics and Optical Instruments 635

Keep in Memory Example 13.


A small telescope has an objective of focal length 140cm
1. Refracting type Refl ecting type telescope and an eye piece of focal length 5.0cm. What is the
telescop e (use of lenses) (use of mi rrors) magnifying power of the telescope for viewing distant
objects when
1 It suffers from spherical 1 It is alm ost free from (a) the telescope is in normal adjustment?
aberration and chromatic spherical aberration and (b) the final image is formed at the least distance of
aberration. asbolutely free from distinct vision?
chromatic aberration. Solution :
2 T he lenses used have 2 The aperature of m irror s
small aperature and used is l arge and therefore
Here, f0 = 140 cm, fe = 5.0 cm
therefore light gathering light gathering power is (a) The magnifying power in normal adjustment is given
power is small. large. f 140
by m = 0 = = 28
fe 5
2. (b) When image is formed at least distance of distinct
vision
f0 é fe ù é 5ù 6
m= ê1 + D ú = 28ê1 + 25 ú = 25 ´ 5 = 33.6
fe ë û ë û
where D = 25cm

.IN
Example 14.
A compound microscope has an objective of focal length
1 cm and an eyepiece of focal length 2.5 cm. An object has
AL
to be placed at a distance of 1.2 cm away from the objective,
for normal adjustment. Find (a) the angular magnification
and (b) the length of the microscope tube.
N
Solution :
R

3. (a) If the first image is formed at a distance v from the


objective, then we have
U

1 1 1
- = or v = 6 cm.
v (-1.2 cm) 1 cm
JO

The angular magnification in normal adjustment is,


v D 6 cm 25 cm
U

m= =- . = -50.
u fe 1.2 cm 2.5 cm
ED

(b) For normal adjustment, the first image must be in the


4. Some common eye defects are myopia, hypermetropia, focal plane of the eyepiece.
astigmatism and presbyopia. The length of the tube is, therefore,
L = v + f e = 6 cm + 2.5 cm = 8. 5 cm
5. M yopia (or short H ypermetropia (or long -
sightedness) sightedness) Example 15.
1 Eye can see near objects 1 Eye can see far off objects A person cannot see objects in nearer than 500 cm from
clearly but cannot see far clearly but cannot see near the eye. Determine the focal length and the power of
objects clearly because objects clearly because the glasses which enable him to read a book 25 cm from his
the li ght from the for off light from the near by eye.
object arri ving the eye object arriving the eye lens Solution :
lens may get converged in may get converged at a Let f be the focal length of the glass. Then for the glass,
front of retina. point behind the retina. u = 25 cm, v = –500 cm
2 It can be corrected by 2 It can be corrected by 1 1 1
Q = +
concave lens (power of convex lens (Power of f u v
concave lens is –ve). convex lens is +ve).
1 1 1 20 - 1
or, = - =
6. Astigmatism : It is due to different curvature of cornea in f 25 500 500
horizontal and vertical plane. It is corrected by using
500
cylindrical lens. or, f = = 26.3 cm = 0.263 m
7. Presbyopia : The eye with this defect cannot see near 19
objects as well as far off objects clearly. 1 1
P(power) = = = 3.8 dioptre
f 0.263
EBD_7179
636 PHYSICS

Example 16. For the spectacle lens,


What is the power of the spectacles required (a) by a u = 25 cm, f = 100 cm, v = ?
hypermetropic eye whose near point is 125 cm (b) by a 1 1 1
myopic eye whose far point is 50 cm ? Q + =
u v f
Solution :
(a) u = 25 cm, v = –125 cm 1 1 1 u -f
or = - =
1 1 1 v f u uf
= +
f u v uf 25 ´ 100
or v = = = -33.33cm
1 1 1 5 -1 u - f 25 - 100
or, = - =
f 25 125 125 i.e., the near point is 33.33 cm from the eye.
Example 19.
125 A certain person can see clearly at distance between 20 cm
or, f= = 31.25 cm = 0.3125 m
4 and 200 cm from his eye. What sepctacles are required to
1 1 enable him to see distant objects clearly and what will be
P= = = 3.2 dioptre his least distance of distinct vision when he is wearing them?
f 0.3125 Solution :
(b) u = ¥ , v = –50 cm For seeing distant objects
1 1 1 u = ¥ , v = –200 cm, f = ?
Q = +

.IN
f u v 1 1 1
Q = +
1 1 1 f u v
= +
f ¥ v
AL 1 1
= -
1
=-
1
or f = -200cm
or,
1 1 1 1 f ¥ 200 200
or, = - =- For finding the least distance of distinct vision
f ¥ 50 50
N
u = ?, v = –20 cm, f = –200cm.
Þ f = –50 cm = – 0.5 m
R

vf -20(-200) (20 ´ 200)


1 1 u= = = = 22.2cm
P= =- = -2 dioptre v - f -20 - (-200) 180
U

f 0.5
i.e., his least distance of distinct vision is 22.2 cm when he
Example 17.
JO

is wearing spectacles.
A person with normal vision has a range of
Example 20.
accommodation from 25 cm to infinity. Over what range
An elderly person cannot see clearly, without the use of
would he be able to see objects distinctly when wearing spectacles, objects nearer than 200 cm. What spectacles
U

the spectacles of a friend whose correction is +4 dioptres. will he need to reduce this distance to 25 cm ? If his eyes
Solution :
ED

can focus rays which are converging to points not less


1 than 150 cm behind them, calculate his range of distinct
P = 4 dioptres, \ f = m = 25 cm
4 vision when using the spectacles.
For near point, v = –25 cm, Solution :
Here u = 25 cm, v = – 200 cm
vf -25 ´ 25 1 1 1
u= = = 12.5 cm Q = +
v - f -25 - 25 f u v
For far point, v = ¥
1 1 1 8 -1
1 1 1 or, = - =
Q + = f 25 200 200
u v f 200
Þf= = 28.6 cm
1 1 1 7
+ = i.e., he should use the converging lens of focal length
u ¥ 25
or u = 25 cm 28.6 cm.
Let x be the object distance for v = 150 cm, then
Hence, the range of distinct vision is from 12.5 cm to 25 cm.
Example 18. 1 1 1 7
+ = =
Where is the near point of an eye for which a spectacle x 150 f 200
lens of +1 dioptre is prescribed ? 1 7 1 21 - 4
Solution : Þ = - =
x 200 150 600
P = +1 dioptre. 600
Þ x= = 35.3cm
1 1 17
\f = = = 1 m = 100 cm
P 1 \ Range of distinct vision is 25 cm to 35.3 cm.
Ray Optics and Optical Instruments 637

Example 21. DA
An angular magnification (magnifying power) of 30 × is
desired using an objective of focal length 1.25 cm. and an w
eyepiece of focal length 5 cm. in a compound microscope.
What is the separation between objective and the eyepiece ? o r
Solution :
Let final image be formed at lease distance of distinct vision
Isotropic point source
æ 25 ö
For eyepiece, Me = ç1 + ÷ = çæ1 + ÷ö = 6
25 f
è fe ø è 5ø so I= or f = 4p. I
4p
Now, M = M0 × Me Solid angle : We know that arc of a circle subtends an angle q on
M -30 the centre of circle O
For objective, M 0 = = = -5 ,
Me 6
For objective, if u0 = – x cm, v0 = 5x cm.
o q s
1 1 1 1 1 1 r
Again, - = or - =
v u f 5x - x 1.25
On simplification, x = 1.5 S Arc of circle
i.e., q = = ...(i)
\ u0 = – 1.5 cm, v0 = 7.5 cm. r Radius of circle

.IN
v (a) The unit of q (plane angle) is radian.
For eyepiece, Me = e
ue Similarly in the case of a sphere, the surface area of sphere
v -25 subtends an angle on the centre of sphere O, which is called
or u e = e = = - 4.17 cm.
Me 6
AL solid angle & is denoted by w.
Distance between objective and eyepiece Let radius of sphere is r and a small area DA on its surface
subtends a solid angle w at the centre then
= vobjective + | u eyepiece | = 7.5 + 4.17 = 11.67 cm.
N
DA
Example 22. w= = constant ..(ii)
R

A small telescope has an objective lens of focal length r2


U

144 cm. and eye-piece of focal length 6.0 cm. What is the
magnifying power of the telescope ? What is the separation
JO

between objective and eye-piece ? Assume normal


adjustment.
Solution : (b)
U

f0 144
M= = = 24
ED

fe 6
The unit of solid angle is steradian. If in eq (ii)
L = f0 + fe = (144 + 6) cm. = 150 cm.
DA = r2 , then w = 1 steradian
If DA = 4pr2 (total surface area of sphere)
PHOTOMETRY
then w = 4p steradian.
Ray optics is based on the assumption that light travels along
straight line. (iii) Illuminance (E) of a surface is the luminous flux incident
(i) Luminous flux (f) of a source of light = amount of visible normally on unit area of the surface.
light energy emitted per second from the source. Its unit is lux.
The SI unit of luminous fulx (f) is lumen. Df
(ii) Luminous intensity (I) of a light source = luminous flux E=
DA
emitted per unit solid angle in any direction.
Its SI unit is candela. For point source, the total normal area will be 4pr2,

Df f f 1
Luminous intensity, I = so E= = 2
Þ Eµ 2
Dw A 4pr r
For isotropic point source, (iv) Luminance or brightness of a surface is the luminous flux
Solid angle, reflected into our eyes from unit area of the surface.
The unit of Brightness is lambert.
DA(surface area) 4pr 2
w= = = 4p steradian (v) Inverse square law for illuminance : Let S is a unidirectional
r2 r2 point source, whose luminous intensity is I. It has some
( where DA = 4pr2 = total surface area of sphere of radius r ) surface DA at distance r from source S.
EBD_7179
638 PHYSICS

For any given source (I constant) & at a fixed distance (r constant)


E µ cos q
i.e., the, intensity of illumination of a surface is proportional to the
cosine of angle of the inclination of the surface. This is called
Lambert’s cosine law. As q increases, cos q decreases &
consequently E decreases.
q is the angle between normal to the area and direction of light
Let central ray of source s falls perpendicularly on surface propagation.
DA, then luminous flux Df is given by
Example 23.
Df = I×Dw ...(i)
What is the effect on the intensity of illumination on a
DA table if a lamp hanging 2 m directly above it is lowered by
where Dw =
r2 0.5 m?
DA Solution :
Þ Df = I ´ 2 ...(ii)
r I I
E1 = 2 and E 2 = 2
Df I I r1 r2
or = or E = 2 ...(iii)
DA r 2 r 2 2
E 2 æ r1 ö æ 2 ö æ 4 ö
\ = çç ÷÷ = ç ÷ =ç ÷
where E is called illuminance or intensity of illumination. E1 è r2 ø è 1. 5 ø è 2.25 ø

.IN
I Fractional increase in the intensity
If in eq. (iii) I is constant for a given source then E µ 2 .
r
So intensity of illumination of any source is inversely E 2 - E1 æ E 2 ö æ 4 ö
AL = = çç - 1÷÷ = ç - 1÷ ´ 100 = 78%
proportional to square of the distance between light source E1 è E1 ø è 2.25 ø
& surface. This is called inverse square law.
Example 24.
N
Lambert's Cosine Law for Illuminance A lamp of power P is suspended at the centre of a circular
R

Let S is unidirectional point source & its luminous intensity is I. table of radius r. What should be the height of the lamp
above the table so that maximum intensity is produced at
U

There is a surface of area DA at distance r from S, which is kept in


the edge?
such a way that light from S falls obliquely on it and central ray
JO

Solution :
makes an angle q with normal to DA.
See figure, the intensity of illumination at edge
Then by fig. DA¢ = DA cos q
(i.e., at point A)
U

I cos q I1 cos q
E= =
ED

2
(LA ) (h 2 + r 2 )

h
From figure. cos q =
(h + r 2 )
2

Ih
According to definition of luminous intensity : \E =
Df = I × Dw
DA' DAcos q
Þ Df =
I ´ DA cos q 24.2 (h + r 2 )3
2

where Dw = 2 = For maximum intensity, dE/dh = 0


r r2 r2
Df I cos q Applying this condition, we get h = r 2
or E= =
DA r2
CONCEPT MAP

Necessary conditions The incident ray


The angle of incidence The incident ray Snell’s law
for TIR (i) ray of light the normal and the
(I)is always equal to the normal and the
must travel from denser refracted ray sin i
angle of reflection (r) reflected ray all lie µ=
to rarer medium all lie in the same sin r
i.e., Ði = Ðr in the same plane
(ii) Ði > Ðc for two media plane

Critical angle(c) Angle iin


Mirror formula Total internal
denser medium for which
Reflection Ray totally Laws of refraction
1 1 1 Laws of reflection angle of refraction in rarer
= + reflected back to denser
f u v 1 medium
medium is 90° µ = Refractive
sin C
c
Ray Optics and Optical Instruments

index, µ =
v
ED
Reflection of Ray optics rear depth
U Refraction of =
Relation between light Turning back Optics - branch of study of apparent depth
R of light in the same light (EM waves wavelength light Bending of light
f and R f = ray while passing
2 medium after striking 400 nm to 750 nm). The path
from one medium to
JO
the reflecting surface of light (always travel in straight Refraction at
or mirror line is ray of light another medium
U a single spherical
surface
Magnification
RAY OPTICS AND
R Refraction by lens
m 2 m1 m 2 - m1
v
-
u
=
R
v height of image OPTICAL INSTRUMENTS
N
m= =
u height of object
f f -v Focal length of lens-lens Lens formula
m= = Optical Instruments
f -u f maker’s formula
AL
1 1 1
1 æ 1 1 ö = -
= (m - 1) ç - ÷ f v u
Telescope f è R1 R 2 ø
Magnification Microscope
.IN
provide angular
produced by Forms large
magnification of
simple microscope image of tiny objects Power of a lens
distant objects
1
Refraction through Prism P=
f (in metre)
Image formed Image formed Magnification by When image When image
at near point at infinity compound microscope at near point at infinity
D D f æ f ö f
M = 1+ M= M = 0 ç1 + e ÷ m=- 0
f f fe è D ø fe
Angle of deviation Prism Formula Dispersive power
When image When final image d = A(µ – 1) æ A + dm ö
sin ç ÷ æ m – mr ö
at near point at in finity µ= è 2 ø w=ç v ÷
sin A / 2 è m -1 ø
v æ Dö æv Dö
639

M = 0 ç1 + ÷ M=ç 0 ´ ÷
m0 è fe ø è u0 fe ø
EBD_7179
640 PHYSICS

1. What will be the colour of the sky as seen from the earth if (c) the eye is not able to decrease the distance between
there were no atmosphere? the eye-lens and the retina beyond a limit
(a) Black (b) Blue (d) the eye is not able to decrease the focal length beyond a limit
(c) Orange (d) Red 9. The one parameter that determines the brightness of a light
2. Monochromatic light of wavelength l1 travelling in a medium source sensed by an eye is
of refractive index m1 enters a denser medium of refractive (a) energy of light entering the eye per second
index m2. The wavelength in the second medium is (b) wavelength of the light
(a) l1 (m1/m2) (b) l1 (m2/m1) (c) total radiant flux entering the eye
(c) l1 (m2 – m1)/m2 (d) l1 (m2 – m1)/m1 (d) total luminous flux entering the eye
3. A vessel of depth 2d cm is half filled with a liquid of refractive 10. In vacuum the speed of light depends upon
index m1 and the upper half with a liquid of refractive index m2.
(a) frequency
The apparent depth of the vessel seen perpendicularly is
(b) wavelength
æ m1 m 2 ö æ 1 1 ö

.IN
ç ÷d ç ÷ (c) velocity of light sources
(a) çm +m ÷ (b) ç m + m ÷d (d) None of these
è 1 2 ø è 1 2ø
11. The intensity produced by a long cylindrical light source at
æ 1 1 ö æ 1 ö
AL
a small distance r from the source is proportional to
ç ÷ ç ÷
(c) ç m + m ÷ 2d (d) ç m m ÷ 2d
è 1 2ø è 1 2ø 1 1
N
4. In a room containing smoke particles, the intensity due to a (a) 2 (b)
r r3
source of light will
R

(a) obey the inverse square law 1


(c) (d) None of these
U

(b) be constant at all distances r


(c) increase with distance from the source than the 12. The refractive index of a piece of transparent quartz is the
JO

inverse fourth power law greatest for


(d) fall faster with distance from the source than the
(a) violet light (b) red light
U

inverse fourth power law


5. What causes chromatic aberration?
ED

(c) green light (d) yellow light


(a) Non - paraxial rays
(b) Paraxial rays 13. Light travels through a glass plate of thickness t and having
refractive index m. If c be the velocity of light in vacuum, the
(c) Variation of focal length with colour
time taken by the light to travel this thickness of glass is
(d) Difference in radii of curvature of the bounding surfaces
of the lens t mt tc
6. Which of the following is not the case with the image formed (a) (b) t mc (c) (d)
mc c m
by a concave lens?
14. A convex mirror of focal length f produces an image
(a) It may be erect or inverted
(1/n)th of the size of the object. The distance of the object
(b) It may be magnified or diminished from the mirror is
(c) It may be real or virtual (a) (n – 1) f (b) f/n
(d) Real image may be between the pole and focus or beyond (c) (n + 1) f (d) nf
focus
15. Amount of light entering into the camera depends upon.
7. Critical angle of light passing from glass to water is minimum
for (a) focal length of objective lens
(a) red colour (b) green colour (b) product of focal length and diameter of the objective lens
(c) yellow colour (d) violet colour (c) distance of object from camera
8. A normal eye is not able to see objects closer than 25 cm (d) aperture setting of the camera
because 16. In optical fibres, propagation of light is due to
(a) the focal length of the eye is 25 cm (a) diffraction (b) total internal reflection
(b) the distance of the retina from the eye-lens is 25 cm (c) reflection (d) refraction
Ray Optics and Optical Instruments 641

17. Rectilinear motion of light in a medium is caused due to (a) remain unchanged
(a) high frequency (b) become zero
(c) become infinite
(b) short wavelength (d) become small, but non-zero
(c) velocity of light 22. Two thin lenses of focal lengths f1 and f2 are in contact and
(d) uniform refractive index of the medium coaxial. Its power is same as power of a single lens given by
18. Resolving power of a telescope increases with f1 + f 2 æ f1 ö æ f2 ö f1 + f 2
(a) (b) ç ÷ (c) ç ÷ (d)
(a) increase in focal length of eye-piece f1f 2 çf ÷ çf ÷ 2
è 2ø è 1ø
(b) increase in focal length of objective
23. For the angle of minimum deviation of a prism to be equal to
(c) increase in aperture of eye piece
its refracting angle, the prism must be made of a material
(d) increase in apeture of objective whose refractive index
19. The distance between an object and its real image formed (a) lies between 2 and 1
by a convex lens cannot be
(a) greater than 2 f (b) less than 2 f (b) lies between 2 and 2
(c) greater than 4 f (d) less than 4 f (c) is less than 1
(d) is greater than 2
20. An electromagnetic radiation of frequency n, wavelength l,
24. Which of the following is not due to total internal reflection?
travelling with velocity v in air enters in a glass slab of

.IN
(a) Working of optical fibre
refractive index (m). The frequency, wavelength and velocity
(b) Difference between apparent and real depth of pond
of light in the glass slab will be respectively
(c) Mirage on hot summer days
l v v
AL (d) Brilliance of diamond
(a) n, and (b) n, 2l and
m m m 25. An astronomical telescope has a large aperture to
n l v 2p l (a) reduce spherical aberration
N
(c) , and (d) , and v
m m m m m (b) have high resolution
R

21. A convex lens is dipped in a liquid whose refractive index is (c) increase span of observation
equal to the refractive index of the lens. Then its focal length will (d) have low dispersion
U
JO
U
ED

1. A lamp of 250 candle power is hanging at a distance of 6 m 5. The critical angle for light going from medium X into medium
from a wall. The illuminance at a point on the wall at a Y is q. The speed of light in medium X is v, then speed of
minimum distance from lamp will be light in medium Y is
(a) 9.64 lux (b) 4.69 lux (a) v(1 – cos q) (b) v/sin q
(c) 6.94 lux (d) None of these
(c) v/cos q (d) v cos q
2. An electric bulb illuminates a plane surface. The intensity
of illumination on the surface at a point 2 m away from the 6. A man wants to see two poles, separately, situated at 11 km.
bulb is 5 × 10–4 phot (lumen/cm2). The line joining the bulb The minimum distance (approximately) between these poles
to the point makes an angle of 60º with the normal to the will be
surface. The luminous intensity of the bulb in candela (a) 5 m (b) 0.5 m
(candle power) is
(c) 1 m (d) 3 m
(a) 40 3 (b) 40 7. The index of refraction of diamond is 2.0. The velocity of
(c) 20 (d) 40 × 10–4 light in diamond is approximately
3. If two mirrors are kept at 60º to each other, then the number (a) 1.5 × 1010 cm/sec (b) 2 × 1010 cm/sec
of images formed by them is
(c) 3.0 × 1010 cm/sec (d) 6 × 1010 cm/sec
(a) 5 (b) 6 (c) 7 (d) 8
8. The luminous intensity of 100 W unidirectional bulb is 100
4. Wavelength of light used in an optical instrument are
candela. The total luminous flux emitted from bulb will be
l1 = 4000Å and l2 = 5000 Å, then ratio of their respective
resolving powers (corresponding to l1 and l2) is (a) 100 p lumen (b) 200 p lumen
(a) 16 : 25 (b) 9 : 1 (c) 4 : 5 (d) 5 : 4 (c) 300 p lumen (d) 400 p lumen
EBD_7179
642 PHYSICS

9. The refractive index of water is 1.33. What will be speed of 18. An achromatic convergent doublet of two lenses in contact
light in water ? has a power of + 2D. The convex lens has power + 5D. What
(a) 3 × 108 m/s (b) 2.25 × 108 m/s is the ratio of dispersive powers of convergent and divergent
lenses ?
(c) 4 × 108 m/s (d) 1.33 × 108 m/s
(a) 2 : 5 (b) 3 : 5 (c) 5 : 2 (d) 5 : 3
10. A concave mirror of focal length f produces an image n
19. The dispersive power of material of a lens of focal length 20
times the size of object. If image is real, then distance of
cm is 0.08. What is the longitudinal chromatic aberration of
object from mirror, is
the lens ?
(a) (n – 1) f (b) { (n – 1)/n} f
(a) 0.08 cm (b) 0.08/20 cm
(c) { (n + 1)/n} f (d) (n + 1) f
11. In a concave mirror, an object is placed at a distance x1 from (c) 1.6 cm (d) 0.16 cm
focus, and image is formed at a distance x2 from focus. Then 20. The magnifying power of a telescope is 9. When it is
focal length of mirror is adjusted for parallel rays, the distance between the objective
and the eye piece is found to be 20 cm. The focal length of
x1 - x 2
(a) x1 x 2 (b) lenses are
2
(a) 18 cm, 2 cm (b) 11 cm, 9 cm
x1 + x 2 x1 (c) 10 cm, 10 cm (d) 15 cm, 5 cm
(c) (d) x
2 2 21. The focal length of the objective of a telescope is 60 cm. To
12. A convex lens of focal length f1 and a concave lens of focal obtain a magnification of 20, the focal length of the eye

.IN
length f2 are placed in contact. The focal length of the piece should be
combination is (a) 2 cm (b) 3 cm (c) 4 cm (d) 5 cm
(a) (f1 + f2) (b) (f1 – f2)
AL
22. The focal lengths of objective and eye lens of an
f1f 2 f1f 2 astronomical telelscope are respectively 2 meter and 5 cm.
(c) f 2 - f1 (d) f1 + f 2 Final image is formed at (i) least distance of distinct vision
N
(ii) infinity Magnifying power in two cases will be
13. A lens of power + 2 diopter is placed in contact with a lens (a) – 48, – 40 (b) – 40, – 48
R

of power – 1 diopter. The combination will behave like


(c) – 40, + 48 (d) – 48, + 40
(a) a convergent lens of focal length 50 cm
U

23. We wish to see inside an atom. Assume the atom to have a


(b) a divergent lens of focal length 100 cm
diameter of 100 pm. This means that one must be able to
JO

(c) a convergent lens of focal length 100 cm


resolve a width of say 10 pm. If an electron microscope is
(d) a convergent lens of focal length 200 cm
14. Light takes t1 sec to travel a distance x in vacuum and the used the energy required should be
(a) 1.5 keV (b) 50 keV
U

same light takes t2 sec to travel 10 cm in a medium. Critical


angle for corresponding medium will be (c) 150 keV (d) 1.5 MeV
ED

24. Which of the following is false ?


æ 10 t 2 ö -1 æç t 2x ö (a) Convex lens always forms image with m < 1
(a) sin -1 çç ÷÷ (b) sin ç 10 t ÷÷
è t1x ø è 1ø (b) A simple mirror produces virtual, erect and same-sized
image
æ 10 t 1 ö æ t x ö (c) A concave mirror produces virtual, erect and magnified
(c) sin -1 çç ÷÷ (d) sin -1 çç 1 ÷÷
image
è t 2x ø è 10 t 2 ø
(d) A convex lens can produce real and same-sized image
15. A double convex lens of focal length 6 cm is made of glass 25. A plane mirror reflects a beam of light to form a real image.
of refractive index 1.5. The radius of curvature of one surface The incident beam is
is double that of other surface. The value of small radius of
(a) parallel (b) convergent
curvature is
(c) divergent (d) any one of the above
(a) 6 cm (b) 4.5 cm (c) 9 cm (d) 4 cm 26. An object is placed at a distance 2f from the pole of a convex
16. A prism has a refracting angle of 60º. When placed in the mirror of focal length f. The linear magnification is
position of minimum deviation, it produces a deviation of
30º. The angle of incidence is 1 2 3
(a) (b) (c) (d) 1
(a) 30º (b) 45º (c) 15º (d) 60º 3 3 4
17. A ray of light passes through an equilateral prism such that 27. A beam of light consisting of red, green and blue colours is
the angle of incidence is equal to the angle of emergence incident on a right-angled prism as shown. The refractive
and the latter is equal to 3/4th of the angle of prism. The index of the material of the prism for the above red, green
angle of deviation is and blue wavelengths are 1.39, 1.44 and 1.47 respectively.
(a) 45º (b) 39º (c) 20º (d) 30º The prism will
Ray Optics and Optical Instruments 643

35. Why is refractive index in a transparent medium greater than


B one ?
90° (a) Because the speed of light in vaccum is always less
than speed in a transparent medium
(b) Because the speed of light in vaccum is always greater
45° 45° than speed in a transparent medium
A C
(c) Frequency of wave changes when it crosses medium
(a) separate part of the red colour from the green and (d) None of these
blue colours.
36. Two convex lenses of focal lengths 0.3 m and 0.05 m are used
(b) separate part of the blue colour from the red and to make a telescope. The distance kept between the two in
green colours. order to obtain an image at infinity is
(c) separate all the three colours from one another. (a) 0.35 m (b) 0.25 m (c) 0.175 m (d) 0.15 m
(d) not separate even partially any colour from the other
37. The refractive indices of glass and water with respect to air
two colours. are 3/2 and 4/3 respectively. Then the refractive index of
28. A concave mirror of focal length f. in vacuum is placed in a glass with respect to water is
medium of refractive index 2. Its focal length in the medium is (a) 8/9 (b) 9/8 (c) 7/6 (d) 2
f 38. The wavelength of a monochromatic light in vacuum is l. It
(a) (b) f (c) 2 f (d) 4 f

.IN
2 travels from vacuum to a medium of absolute refractive index
29. The maximum and minimum distance between a convex lens µ. The ratio of wavelength of the incident and refracted
and an object, for the magnification of a real image to be ALwave is
greater than one are (a) µ2 : 1 (b) 1 : 1 (c) µ : 1 (d) 1 : µ
(a) 2f and f (b) f and zero 39. An object is placed at a distance of 40 cm in front of a
(c) ¥ and 2f (d) 4f and 2f concave mirror of focal length 20 cm. The image produced is
N
30. A plane convex lens of focal length 16 cm, is to be made of (a) real, inverted and smaller in size
glass of refractive index 1.5. The radius of curvature of the
R

(b) real, inverted and of same size


curved surface should be
(c) real and erect
U

(a) 8 cm (b) 12 cm (c) 16 cm (d) 24 cm


(d) virtual and inverted
31. A real image is formed by a convex lens. If we put a concave
JO

lens in contact with it, the combination again forms a real 40. The frequency of a light wave in a material is 2 × 1014 Hz and
image. The new image wavelength is 5000 Å. The refractive index of material will
be
(a) is closer to the lens system.
U

(a) 1.50 (b) 3.00 (c) 1.33 (d) 1.40


(b) is farther form the lens system.
41. A ray incident at 15° on one refracting surface of a prism of
ED

(c) is at the original position.


angle 60° suffers a deviation of 55°. What is the angle of
(d) may be anywhere depending on the focal length of
emergence ?
the concave lens.
(a) 95° (b) 45°
32. A plano-convex lens of focal length 30 cm has its plane
surface silvered. An object is placed 40 cm from the lens on (c) 30° (d) None of these
the convex side. The distance of the image from the lens is 42. A man’s near point is 0.5 m and far point is 3 m. Power of
(a) 18 cm (b) 24 cm (c) 30 cm (d) 40 cm spectacle lenses required for (i) reading purposes, (ii) seeing
33. Two convex lenses of focal lengths f1 and f2 are mounted distant objects, respectively, are
coaxially separated by a distance. If the power of the (a) –2 D and + 3 D (b) +2 D and –3 D
combination is zero, the distance between the lenses is (c) +2 D and –0.33 D (d) –2 D and + 0.33 D
(a) | f1 - f 2 | (b) f1 + f2 43. Two light sources with equal luminous intensity are lying at
a distance of 1.2 m from each other. Where should a screen
f1f 2 f1f2 be placed between them such that illuminance on one of its
(c) (d)
| f1 - f2 | f1 + f2 faces is four times that on another face ?
34. If D is the deviation of a normally falling light beam on a thin (a) 0.2 m (b) 0.4 m (c) 0.8 m (d) 1.6 m
prism of angle A and d is the dispersive power of the same 44. A lamp is hanging along the axis of a circular table of radius
prism then r. At what height should the lamp be placed above the table,
(a) D is independent of A. 1
so that the illuminance at the edge of the table is of that
(b) D is independent of refractive Index. 8
at its centre?
(c) d is independent of refractive index.
(d) d is independent of A. (a) r/2 (b) r/ 2 (c) r/3 (d) r/ 3
EBD_7179
644 PHYSICS

45. A rectangular block of glass is placed on a mark made on the 55. Two lenses in contact form an achromatic lens. Their focal
surface of the table and it is viewed from the vertical position lengths are in the ratio 2 : 3. Their dispersive powers must
of eye. If refractive index of glass be m and its thickness d, be in the ratio of
then the mark will appear to be raised up by (a) 1 : 3 (b) 2 : 3 (c) 3 : 2 (d) 3 : 1
(m + 1)d (m - 1)d 56. A combination is made of two lenses of focal length f and f'
(a) (b) in contact, the dispersive powers of the materials of the
m m
lenses are w and w'. The combination is achromatic, when
(m + 1) (m - 1) m (a) w = w0, w' = 2w0 f ¢ = 2f
(c) (d)
md d (b) w = w0, w' = 2w0 f ¢ = f/2
46. Light passes through a glass plate of thickness d and (c) w = w0, w' = 2w0 f ¢ = –f/2
refractive index m. For small angle of incidence i, the lateral (d) w = w0, w' = 2w0 f ¢ = –2f
displacement is 57. An achromatic convergent lens of focal length 20 cms is
(a) id (b) id (m – 1) made of two lenses (in contact) of materials having dispersive
i d (m - 1) id m powers in the ratio of 1 : 2 and having focal lengths f1 and f2.
(c) (d) Which of the following is true ?
m m –1
(a) f1 = 10 cms, f2 = –20 cms
47. A glass slab of thickness 4 cm contains the same number of
(b) f1 = 20 cms, f2 = 10cms

.IN
waves as 5 cm of water when both are traversed by the same
monochromatic light. If the refractive index of water is 4/3, (c) f1 = –10 cms, f2 = –20 cms
what is that of glass? AL (d) f1 = 20 cms, f2 = –20 cms
(a) 5/3 (b) 5/4 (c) 16/15 (d) 1.5 58. A simple telescope, consisting of an objective of focal length
48. An air bubble in glass slab (m = 1.5) from one side is 6 cm 60 cm and a single eye lens of focal length 5 cm is focussed on
and from other side is 4 cm. The thickness of glass slab is a distant object in such a way that parallel rays emerge from
N
(a) 10 cm (b) 6.67 cm the eye lens. If the object subtends an angle of 2º at the
objective, the angular width of the image is
R

(c) 15 cm (d) None of these


49. A vessel is half filled with a liquid of refractive index m. The (a) 10º (b) 24º (c) 50º (d) (1/6)º
U

other half of the vessel is filled with an immiscible liquid of 59. An astronomical telescope has an angular magnification of
magnitude 5 for distant objects. The separation between
JO

refrative index 1.5 m. The apparent depth of the vessel is


50% of the actual depth. Then m is the objective and the eye-piece is 36 cms and the final image
(a) 1.4 (b) 1.5 (c) 1.6 (d) 1.67 is formed at infinity. The focal length f0 of the objective and
U

50. A man 160 cm high stands in front of a plane mirror. His eyes are fe of the eye piece are
at a height of 150 cm from the floor. Then the minimum length of (a) f0 = 45 cm and f2 = –9 cm
ED

the plane mirror for him to see his full length image is (b) f0 = 50 cm and fe = 10 cm
(a) 85 cm (b) 170 cm (c) 80 cm (d) 340 cm (c) f0 = 7.2 cm and fe = 5 cm
51. It is desired to photograph the image of an object placed at (d) f0 = 30 cm and fe = 6 cm
a distance of 3 m from plane mirror. The camera, which is at 60. Two lens of focal length f1 and f2 are kept in contact coaxially.
a distance of 4.5 m from mirror should be focussed for a The resultant power of combination will be
distance of
f1f 2 f1 + f 2
(a) 3 m (b) 4.5 m (c) 6 m (d) 7.5 m (a) (b)
52. Two thin lenses are in contact and the focal length of the f1 - f 2 f1f 2
combination is 80 cm. If the focal length of one lens is 20 cm, f1 f 2
(c ) f1 + f 2 (d) +
then the power of the other lens will be f 2 f1
(a) 1.66 D (b) 4.00 D 61. When white light enters a prism, its gets split into its
(c) – 100 D (d) – 3.75 D constituent colours. This is due to
53. A thin convergent glass lens (mg = 1.5) has a power of (a) high density of prism material
+ 5.0 D. When this lens is immersed in a liquid of refractive (b) because m is different for different wavelength
index m, it acts as a divergent lens of focal length 100 cm. (c) diffraction of light
The value of m must be (d) velocity changes for different frequency
(a) 4/3 (b) 5/3 (c) 5/4 (d) 6/5 62. A pencil of light rays falls on a plane mirror and form a real
54. A ray of light passes through an equilateral prism image, so the incident rays are
(m = 1.5). The angle of minimum deviation is (a) parallel (b) diverging
(a) 45º (b) 37º 12' (c) 20º (d) 30º (c) converging (d) statement is false
Ray Optics and Optical Instruments 645

63. Astronauts look down on earth surface from a space ship 71. The layered lens as shown is made of two types of
parked at an altitude of 500 km. They can resolve objects of transparent materials-one indicated by horizontal lines and
the earth of the size (It can be assumed that the pupils the other by vertical lines. The number of images formed of
diameter is 5mm and wavelength of light is 500 nm) an object will be
(a) 0.5 m (b) 5 m (c) 50 m (d) 500 m
64. Spherical aberration in a thin lens can be reduced by
(a) using a monochromatic light
(b) using a doublet combination
(c) using a circular annular mark over the lens
(d) increasing the size of the lens
65. A lens produces an image of an object on a screen. If a slab
(a) 1 (b) 2 (c) 3 (d) 6
of refractive index n is placed in between lens and screen,
72. In the displacement method, a concave lens is placed in
the screen has to be moved by distance d behind. The
between an object and a screen. If the magnification in the
thickness of slab is
two positions are m1 and m2 (m1 > m2), and the distance
n -1 (n - 1) d nd between the two positions of the lens is x, the focal length
(a) nd (b) (c) (d) of the lens is
nd n n -1

.IN
66. An object is moved along the principal axis of a converging x x
(a) (b)
lens from a position 5 focal lengths from the lens to a position m1 + m 2 m1 - m 2
that is 2 focal lengths from the lens. Which statement about AL
the resulting image is most accurate? x x
(c) (d)
(a) The image increases in size and decreases in distance (m1 + m 2 ) 2 (m1 - m 2 ) 2
from the lens
N
73. A thin lens has focal length f, and its aperture has diameter
(b) The image increases in size and increases in distance D. It forms an image of intensity I. If the central part of the
R

from the lens D


(c) The image decreases in size and decreases in distance aperture, of diameter , is blocked by an opaque paper,,
U

2
from the lens the focal length of the lens and the intensity of image will
JO

(d) The image decreases in size and increases in distance become


from the lens 3f I 3I
f I I
67. An object is placed upright on the axis of a thin convex lens (a) , (b) f, (c) , (d) f,
U

2 2 4 4 2 4
at a distance of four focal lengths (4f) from the center of the
74. The graph shows the variation of magnification m produced
ED

lens. An inverted image appears at a distance of 4/3 f on the


by a convex lens with the image distance v. The focal length
other side of the lens. What is the ratio of the height of he
of the lens is
image of the height of the object?
m
(a) 1/3 (b) 3/4 (c) 4/3 (d) 3/1 ( a + c, b )
68. A paper, with two marks having separation d, is held normal
to the line of sight of an observer at a distance of 50m. The
diameter of the eye-lens of the observer is 2 mm. Which of b
the following is the least value of d, so that the marks can be
a c v
seen as separate ? (The mean wavelength of visible light
(a , b)
may be taken as 5000 Å)
(a) 1.25 m (b) 12.5 cm (c) 1.25 cm (d) 2.5 mm b c ab
(a) (b) (c) b (d)
69. A diver inside water sees the setting sun at c b c
(a) 41° to the horizon (b) 49° to the horizon 75. A ray of light traveling in water is incident on its surface
(c) 0° to the horizon (d) 45° to the horizon open to air. The angle of incidence is q, which is less than
70. A concave mirror forms the image of an object on a screen. the critical angle. Then there will be
If the lower half of the mirror is covered with an opaque (a) only a reflected ray and no refracted ray
card, the effect would be to make the (b) only a refracted ray and no reflected ray
(a) image less bright. (c) a reflected ray and a refracted ray and the angle between
(b) lower half of the image disappear. them would be less than 180°–2q
(c) upper half of the image disappear. (d) a reflected ray and a refracted ray and the angle between
(d) image blurred. them would be greater than 180°–2q
EBD_7179
646 PHYSICS

76. Air has refractive index 1.0003. The thickness of air column, 84. Light enters at an angle of incidence in a transparent rod of
which will have one more wavelength of yellow light (6000 refractive index n. For what value of the refractive index of
Å) than in the same thickness of vacuum is the material of the rod the light once entered into it will not
(a) 2 mm (b) 2 cm (c) 2 m (d) 2 km. leave it through its lateral face whatsoever be the value of
77. The position of final image formed by the given lens angle of incidence?
combination from the third lens will be at a distance of
(f1 = + 10 cm, f2 = – 10 cm and f3 = + 30 cm). (a) n > 2 (b) n = 1 (c) n = 1.1 (d) n = 1.3
85. The radius of curvature of a thin plano-convex lens is 10 cm
(of curved surface) and the refractive index is 1.5. If the
plane surface is silvered, then it behaves like a concave
mirror of focal length
(a) 10 cm (b) 15 cm (c) 20 cm (d) 5 cm
30 cm 5 cm 10 cm 86. An air bubble in a glass slab (m = 1.5) is 5 cm deep when
viewed from one face and 2 cm deep when viewed from the
opposite face. The thickness of the slab is
(a) 15 cm (b) infinity (c) 45 cm (d) 30 cm (a) 7.5 cm (b) 10.5 cm (c) 7 cm (d) 10 cm
78. A ray of light is travelling from glass to air. (Refractive
87. A light ray falls on a rectangular glass slab as shown. The

.IN
index of glass = 1.5). The angle of incidence is 50°.
index of refraction of the glass, if total internal reflection is
The deviation of the ray is
to occur at the vertical face, is
(a) 0° (b) 80° AL 45º
-1 é sin 50° ù -1 é sin 50° ù
(c) 50° - sin ê 1.5 ú (d)
sin ê ú - 50°
ë û ë 1.5 û
N
79. If fV and fR are the focal lengths of a convex lens for violet
Glass
R

and red light respectively and FV and FR are the focal lengths
of concave lens for violet and red light respectively, then we
U

have
( 3 + 1) ( 2 + 1)
JO

(a) fV < fR and FV > FR (b) fV < fR and FV < FR


(a) 3 / 2 (b) (c) (d) 5 /2
(c) fV > fR and FV > FR (d) fV > fR and FV < FR 2 2
80. A ray is incident at an angle of incidence i on one surface of
88. An equiconvex lens is cut into two halves along (i) XOX'
U

a prism of small angle A and emerges normally from the


and (ii) YOY' as shown in the figure. Let f, f', f'' be the focal
opposite surface. If the refractive index of the material of
ED

lengths of the complete lens, of each half in case (i), and of


prism is m, the angle of incidence i is nearly equal to
each half in case (ii), respectively
A A
(a) (b)
m 2m
mA
(c) m A (d)
2
81. One face of a rectangular glass plate 6 cm thick is silvered. An
object held 8 cm in front of the first face, forms an image 12 cm
behind the silvered face. The refractive index of the glass is
(a) 0.4 (b) 0.8 (c) 1.2 (d) 1.6
82. A convex lens of focal length 80 cm and a concave lens of
focal length 50 cm are combined together. What will be their
resulting power?
(a) + 6.5 D (b) –6.5 D (c) + 7.5 D (d) –0.75 D Choose the correct statement from the following
83. A luminous object is placed at a distance of 30 cm from the (a) f ' = 2f, f '' = 2f (b) f ' = f, f '' = 2f
convex lens of focal length 20 cm. On the other side of the (c) f ' = 2f, f '' = f (d) f ' = f, f '' = f
lens, at what distance from the lens a convex mirror of radius 89. A plano-convex lens is made of material of refractive index
of curvature 10 cm be placed in order to have an upright 1.6. The radius of curvature of the curved surface is 60 cm.
image of the object coincident with it? The focal length of the lens is
(a) 12 cm (b) 30 cm (c) 50 cm (d) 60 cm
(a) 50 cm (b) 100 cm (c) 200 cm (d) 400 cm
Ray Optics and Optical Instruments 647

90. The refractive index of the material of a prism is Ö2 and its Directions for Qs. (97 to 100) : Each question contains
refracting angle is 30º. One of the refracting surfaces of the STATEMENT-1 and STATEMENT-2. Choose the correct answer
prism is made a mirror inwards. A beam of monochromatic (ONLY ONE option is correct ) from the following-
light enters the prism from the mirrored surface if its angle
(a) Statement -1 is false, Statement-2 is true
of incidence of the prism is
(a) 30º (b) 45º (c) 60º (d) 0º (b) Statement -1 is true, Statement-2 is true; Statement -2 is a
91. A telescope has an objective lens of 10 cm diameter and is correct explanation for Statement-1
situated at a distance of one kilometer from two objects. (c) Statement -1 is true, Statement-2 is true; Statement -2 is not
The minimum distance between these two objects, which a correct explanation for Statement-1
can be resolved by th e telescope, when the mean
(d) Statement -1 is true, Statement-2 is false
wavelength of light is 5000 Å, is of the order of
(a) 5 cm (b) 0.5 m (c) 5 m (d) 5mm 97. Statement 1: Two convex lenses joined together cannot
produce an achromatic combination.
92. The refractive index of the material of the prism is 3 ; then
Statement 2 : The condition for achromatism is
the angle of minimum deviation of the prism is
w1 w2
(a) 30º (b) 45º (c) 60º (d) 75º + = 0 where symbols have their usual meaning.
f1 f 2
93. A ray of light travelling in a transparent medium of refractive
index m , falls on a surface separating the medium from air at 98. Statement 1: Critical angle is minimum for violet colour.

.IN
an angle of incidence of 45°. For which of the following
value of m the ray can undergo total internal reflection? -1 æ 1 ö
Statement 2 : Because critical angle qc = sin ç ÷ and
(a) m = 1.33 (b) m = 1.40 (c) m = 1.50 (d) m = 1.25 èmø
94. A biconvex lens has a radius of curvature of magnitude 20
AL
cm. Which one of the following options best describe the 1
mµ .
image formed of an object of height 2 cm placed 30 cm from l
N
the lens?
99. Statement 1: Optical fibres are used to transmit light without
R

(a) Virtual, upright, height = 1 cm


(b) Virtual, upright, height = 0.5 cm any appreciable loss in its intensity over distance of several
U

(c) Real, inverted, height = 4 cm kilometers.


(d) Real, inverted, height = 1cm Statement 2 : Optical fibres are very thick and all the light is
JO

95. A thin prism of angle 15º made of glass of refractive index passed through it without any loss.
µ1 = 1.5 is combined with another prism of glass of refractive 100. Statement 1 : If P1 and P2 be the powers of two thin lenses
index µ2 = 1.75. The combination of the prism produces
U

located coaxially in a medium of refractive index µ at a


dispersion without deviation. The angle of the second prism
distance d, then the power P of the combination is
ED

should be
(a) 7° (b) 10° (c) 12° (d) 5° P = P1 + P2 – P1P2d/µ
96. A person is six feet tall. How tall must a vertical mirror be if Statement 2 : Because for above given system equivalent
he is able to see his entire length?
f1f 2 1
(a) 3 ft (b) 4.5 ft (c) 7.5 ft (d) 6 ft focal length is given by F = and P = .
f1 + f2 - d / m F

Exemplar Questions (a) blue (b) green


1. A ray of light incident at an angle q on a refracting face of a (c) violet (d) red
prism emerges from the other face normally. If the angle of 3. An object approaches a convergent lens from the left of the
the prism is 5° and the prism is made of a material of refractive lens with a uniform speed 5 m/s and stops at the focus. The
index 1.5, the angle of incidence is image
(a) 7.5° (b) 5° (a) moves away from the lens with an uniform speed 5 m/s
(c) 15° (d) 2.5° (b) moves away from the lens with an uniform acceleration
2. A short pulse of white light is incident from air to a glass
(c) moves away from the lens with a non-uniform acceleration
slab at normal incidence. After travelling through the slab,
the first colour to emerge is (d) moves towards the lens with a non-uniform acceleration
EBD_7179
648 PHYSICS

4. A passenger in an aeroplane shall 9. The optical density of turpentine is higher than tnat of water
(a) never see a rainbow while its mass density is lower. Figure shows a layer of
(b) may see a primary and a secondary rainbow as turpentine floating over water in a container. For which one
concentric circles of the four rays incident on turpentine in figure, the path
(c) may see a primary and a secondary rainbow as shown is correct?
concentric arcs (a) 1 (b) 2 (c) 3 (d) 4
(d) shall never see a secondary rainbow 1 2 3 4
5. You are given four sources of light each one providing a
light of a single colour - red, blue, green and yellow. Suppose Air
the angle of refraction for a beam of yellow light T
corresponding to a particular angle of incidence at the interface
of two media is 90°. Which of the following statements is W G
correct if the source of yellow light is replaced with that of
other lights without changing the angle of incidence? 10. A car is moving with at a constant speed of
60 km h–1 on a straight road. Looking at the rear view mirror,
(a) The beam of red light would undergo total internal
the driver finds that the car following him is at a distance of
reflection
100 m and is approaching with a speed of 5 kmh –1.
(b) The beam of red light would bend towards normal while
In order to keep track of the car in the rear, the driver begins
it gets refracted through the second medium

.IN
to glance alternatively at the rear and side mirror of his car
(c) The beam of blue light would undergo total internal
after every 2 s till the other car overtakes. If the two cars
reflection were maintaining their speeds, which of the following
(d) The beam of green light would bend away from the
AL
statement (s) is/are correct?
normal as it gets refracted through the second medium
(a) The speed of the car in the rear is 65 km h –1
6. The radius of curvature of the curved surface of a plano-
(b) In the side mirror, the car in the rear would appear to
N
convex lens is 20 cm. If the refractive index of the material of
approach with a speed of 5 kmh –1 to the driver of the
the lens be 1.5, it will leading car
R

(a) act as a convex lens only for the objects that lie on its
(c) In the rear view mirror, the speed of the approaching car
curved side
U

would appear to decrease as the distance between the


(b) act as a concave lens for the objects that lie on its curved cars decreases
JO

side
(d) In the side mirror, the speed of the approaching car would
(c) act as a convex lens irrespective of the side on which appear to increase as the distance between the cars decreases
the object lies
11. There are certain material developed in laboratories which
U

(d) act as a concave lens irrespective of side on which the


have a negative refractive index figure. A ray incident from
object lies
ED

air (Medium 1) into such a medium (Medium 2) shall follow a


7. The phenomena involved in the reflection of radiowaves by path given by
ionosphere is similar to
(a) reflection of light by a plane mirror
i 1
(b) total internal reflection of light in air during a mirage
(a)
(c) dispersion of light by water molecules during the
formation of a rainbow r 2
(d) scattering of light by the particles of air
8. The direction of ray of light incident on a concave mirror is
i
shown by PQ while directions in which the ray would travel 1
after reflection is shown by four rays marked 1, 2, 3 and 4 (b)
(figure). Which of the four rays correctly shows the direction r
2
of reflected ray? 1
Q
2 4
i r 1

(c)
C F 2

3 P 1

(d)
(a) 1 (b) 2 (c) 3 (d) 4
2
Ray Optics and Optical Instruments 649

NEET/AIPMT (2013-2017) Questions The focal length of the combination is [2015]


(a) –25 cm (b) –50 cm
12. A plano convex lens fits exactly into a plano concave lens. (c) 50 cm (d) –20 cm
Their plane surfaces are parallel to each other. If lenses are 20. A beam of light consisting of red, green and blue colours is
made of different materials of refractive indices m1 and m2 incident on a right angled prism. The refractive index of the
and R is the radius of curvature of the curved surface of the material of the prism for the above red, green and blue
lenses, then the focal length of the combination is [2013] wavelengths are 1.39, 1.44 and 1.47, respectively. [2015 RS]
R R
(a) (b)
2 ( m1 - m 2 ) (m1 - m 2 )
Blue
2R R Green
(c) (d)
(m 2 - m1 ) 2 ( m1 + m 2 ) Red

13. For a normal eye, the cornea of eye provides a converging 45°
power of 40D and the least converging power of the eye
lens behind the cornea is 20D. Using this information, the The prism will:
(a) separate all the three colours from one another
distance between the retina and the eye lens of the eye can
(b) not separate the three colours at all
be estimated to be [2013]
(c) separate the red colour part from the green and blue
(a) 2.5 cm (b) 1.67 cm

.IN
colours
(c) 1.5 cm (d) 5 cm (d) separate the blue colour part from the red and green
14. Two plane mirrors are inclined at 70°. A ray incident on one colours
mirror at angle q after reflection falls on second mirror and is
AL
21. In an astronomical telescope in normal adjustment a straight
reflected from there parallel to first mirror. The value of q is black line of lenght L is drawn on inside part of objective
[NEET Kar. 2013] lens. The eye-piece forms a real image of this line. The length
N
(a) 50° (b) 45° of this image is l. The magnification of the telescope is :
(c) 30° (d) 55° L L+I
R

(a) -1 (b) [2015 RS]


15. The reddish appearance of the sun at sunrise and sunset is I L-I
U

due to [NEET Kar. 2013]


L L
(a) the colour of the sky (c) (d) +1
JO

I I
(b) the scattering of light
22. The angle of incidence for a ray of light at a refracting surface
(c) the polarisation of light of a prism is 45° . The angle of prism is 60°. If the ray suffers
(d) the colour of the sun
U

minimum deviation through the prism, the angle of minimum


16. If the focal length of objective lens is increased then deviation and refractive index of the material of the prism
ED

magnifying power of : [2014] respectively, are : [2016]


(a) microscope will increase but that of telescope decrease.
1
(b) microscope and telescope both will increase. (a) 45°, (b) 30°, 2
(c) microscope and telescope both will decrease 2
(d) microscope will decrease but that of telescope increase. 1
17. The angle of a prism is ‘A’. One of its refracting surfaces is (c) 45°, 2 (d) 30°,
2
silvered. Light rays falling at an angle of incidence 2A on
the first surface returns back through the same path after 23. A astronomical telescope has objective and eyepiece of focal
suffering reflection at the silvered surface. The refractive lengths 40 cm and 4 cm respectively. To view an object 200
index m, of the prism is : [2014] cm away from the objective, the lenses must be separated
by a distance : [2016]
(a) 2 sin A (b) 2 cos A
(a) 37.3 cm (b) 46.0 cm
1 (c) 50.0 cm (d) 54.0 cm
(c) cos A (d) tan A 24. Match the corresponding entries of column-1 with column-2
2
18. The refracting angle of a prism is ‘A’, and refractive index of (Where m is the magnification produced by the mirror):
the material of the prism is cot(A/2). The angle of minimum Column-1 Column-2 [2016]
(A) m = –2 (a) Convex mirror
deviation is : [2015]
(a) 180° – 2A (b) 90° – A 1
(B) m = - (b) Concave mirror
(c) 180° + 2A (d) 180° – 3A 2
19. Two identical thin plano-convex glass lenses (refractive (C) m = +2 (c) Real image
index 1.5) each having radius of curvature of 20 cm are placed
with their convex surfaces in contact at the centre. The 1
(D) m = + (d) Virtual image
intervening space is filled with oil of refractive index 1.7. 2
EBD_7179
650 PHYSICS

(a) A ® b and c, B ®b and c, C ® b and d,


D ® a and d. y x
(a) (b)
2y
(b) A ® a and c, B ® a an d d, C ® a and b, x
D ® c and d
(c) A ® a and d, B ® b and c, C ® b and d, x y
(c) (d)
D ® b and c y 2x
(d) A ® c and d, B ® b and d, C ® b and c, 26. A thin prism having refracting angle 10° is made of glass of
D ® a and d refractive index 1.42. This prism is combined with another
25. A beam of light from a source L is incident normally on a thin prism of glass of refractive index 1.7. This combination
plane mirror fixed at a certain distance x from the source. produces dispersion without deviation. The refracting angle
The beam is reflected back as a spot on a scale placed just of second prism should be [2017]
above the source I. When the mirror is rotated through a (a) 6° (b) 8°
small angle q, the spot of the light is found to move through (c) 10° (d) 4°
a distance y on the scale. The angle q is given by [2017]

.IN
AL
N
R
U
JO
U
ED
Ray Optics and Optical Instruments 651

Hints & Solutions


EXERCISE - 1 Resolving power for telescope
1. (a) As no scattering of light occurs, sky appears dark. 1 d d
= = = 0
c v c c limit of resolution 1.22l d l
2. (a) l a = or l m = = (Q m m = )
n n mm n v by increasing the aperture of objective resolving power
c c can be increased.
\ l1 = and l 2 = 19. (d) Least distance is 4f when object is at radius of curvature,
m1n m 2n
and greatest is infinity.
or l1 m1 = l 2 m 2 or l 2 = l1 (m1 / m 2 ) 20. (a) When electromagnetic wave enters in other medium,
3. (b) Apparent depth = d/m1 + d/m2
frequency reamains unchanged while wavelength and
4. (d) 5. (c) 6. (d) 7. (d)
8. (d) Because, the focal length of eye lens can not decrease 1
velocity become times.
beyond a certain limit. m
9. (d) It is the total luminous flux. so, e.m. entering from air to glass slab (m), frequency
10. (d)
11. (c) The intensity of cylindrical source at small distance r is l

.IN
remains n, wavelength l' =
inversely proportional to r. m
v
1 1 velocity of light in medium v' =
I µ (since A µ & I µ A2 ) m
r r
AL
1
21. (c) 1
f
= (l mg - 1)æçç R1 - R1 ö
÷÷ where l m g = 1 is given, we get
12. (a) m µ , lred > lviolet è 2ø
N
l
1 æ 1 1 ö
velocity of light in vacuum Þ = (1 - 1)çç - ÷÷ = 0 or Þ f = ¥
R

13. (c) m= f R
è 1 R 2ø
velocity of light in glass plate
U

c c 1 1 1 f 2 + f1 1 f1 + f 2
or m = or c ¢ = 22. (a) = + = ; P= =
F f1 f 2 f1f 2 F f1f 2
JO

c¢ m
mt
Time taken = distance/velocity = t /( c / m ) = 23. (b) A Prism angle
c A
U

Angle of
1 -v Smin
u minimum
14. (a) m= = Þv=
ED

n (-u ) deviation
n
Incident C e Angle of
1 1 1 n 1 1 r1 r2
As + = \ - = angle emergence
v u f u u f
u = (n – 1) f
15. (d) Amount of light entering into the camera depends upon
B C
aperture setting the camera.
16. (b) Optical fibre is a device which transmits light introduced The angle of minimum deviation is given as
at one end to the opposite end, with little loss of the
light through the sides of the fibre. It is possible with the d min = i + e–A
help of total internal reflection. for minimum deviation
d min = A then
2A = i + e
in case of d min i = e
A
2A = 2i r1 = r2 =
2
i = A = 90°
17. (d) If the medium is heterogeneous having a gradient of from smell’s law
refractive index. Then light rays will not follow a 1 sin i = n sin r1
rectilinear (straight line path). A
sin A = n sin
18. (d) Resolving power = l plane transmission grating 2
dl
EBD_7179
652 PHYSICS

A A A un
2 sin cos = n sin use f = and solve to get f = x 1x 2
2 2 2 u+n
A
2 cos = n 1 1 1 f -f f1f 2
2 12. (c) = + = 2 1; f=
when A = 90° = imin f f1 - f 2 f1f 2 f 2 - f1
then nmin = 2 13. (c) P = P1 + P2 = + 2 – 1 = + 1 dioptre, lens behaves as
convergent
i = A = 0 nmax = 2 1 1
F= = = 1 m = 100 cm
24. (b) Difference between apparent and real depth of a pond P 1
is due to the refraction of light, not due to the total
x 10
internal reflection. Other three phenomena are due to 14. (c) c= , v=
the total internal reflection. t1 t2
25. (b) Large aperture increases the amount of light gathered
1 v 10 t 1 æ 10 t 1 ö
by the telescope increasing the resolution. sin i c = = = ´ ; i c = sin -1 çç ÷
÷
m c t2 x è t 2x ø
EXERCISE - 2 15. (b) If R1 = R, R2 = –2 R
1. (c) At minimum distance, incidence is normal. Therefore, 1 æ 1 1 ö
= (m - 1) çç - ÷
÷
I 250 f R
è 1 R 2 ø

.IN
E= = 2 = 6.94 lux
r2 6 1 æ1 1 ö 0 .5 ´ 3
= (1.5 - 1) çç + ÷=
2. (b) r = 2 m = 200 cm 6 è R 2 R ÷ø 2R
E = 5 × 10–4 phot, q = 60º
AL R = 4.5 cm
From,
A + d m 60 + 30
I cos q E r 2 5 ´ 10 -4 (200)2 16. (b) i= = = 45 º
N
E= , I= = = 40 C.P 2 2
r2 cos q cos 60
3
R

17. (d) i1 = i 2 = A
360 º 4
3. (a) Number of images ( n 1 ) = -1
U

q As A + d = i1 + i 2
where q = angle between mirrors 3 3 A 60 º
JO

Here, q = 60º \ d = i1 + i 2 - A = A+ A-A = = = 30 º


4 4 2 2
360 º 18. (b) Here, P1 = 5 D
So, number of images n 1 = -1 = 5 P2 = P – P1 = 2 – 5 = –3 D
U

60 º
w1 f - P2 3
1 =- 1 = =
ED

4. (d) Resolving power of an optical instrument µ w2 f2 P1 5


l 19. (c) Longitudinal chromatic aberration = w f
Resolving power at λ1 l = 0.08 × 20 = 1.6 cm
= 2
Resolving power at l 2 l1 f0
20. (a) =9, \ f0 = 9 fe
æ 1 ö fe
çç Limit of resolution µ ÷
è resolving power ø÷ Also f0 + fe = 20 (Q final image is at infinity)
9 fe + fe = 20, fe = 2 cm, \ f0 = 18 cm
5000 5
\ Ratio of resolving power = = =5:4 21. (b) In normal adjustment,
4000 4 f 60
f
5. (b) M = 0 = 20 , f e = 0 = = 3 cm
fe 20 20
6 (d) 7. (a)
8. (d) f = 4 p I = 4 p (100) = 400 p lumen. f 0 æ f e ö - 200 æ 1 + 5 ö = - 48
22. (a) (i) M = - ç1 + ÷ = ç ÷
fe è dø 5 è 25 ø
v2 m 1 v 8
9. (b) = 1 = or v 2 = 1 = 2.25 ´ 10 m / s (since least distance d = 25cm)
v1 m 2 1 .33 1.33
f0 200
1 1 1 (ii) M = - =- = -40
10. (c) Given v = nu As + = fe 5
v u f
23. (b)
1 1 1 24. (a) Convex lens can form image with m < 1, m > 1 and
\ + = or u = ( n + 1) f
nu u f n m = 1 depending upon the position of the object.
11. (a) Here, u = f + x1, n = f + x2 Convex lens forms magnified image (m > 1) when the
Ray Optics and Optical Instruments 653

object is pole and 2f, same size as the object (m = 1)


when the object is at 2f and smaller image (m < 1), when Magnification, m =
-v
=-
( -40 ) = -1
the object is beyond 2f. u ( -40 )
25. (b) Virtual object forms real image on a plane mirror, so The image is of the same size and inverted.
rays convergent.
velocity of light in vacuum (c)
1 1 1 1 3 2 40. (b) m=
26. (a) - = Þ = Þv= f velocity of light in medium (v)
v 2f f v 2f 3
But v = ul = 2 × 1014 × 5000 × 10–10
v 2 f 1
\m = = = In the medium, v = 108 m/s.
u 3 2f 3
27. (a) Difference in refractive indices of blue and green colour v vac 3 ´108
\m= = = 3.
are less so they are seen together and red is seen separate vmed 108
because deviation depends on refractive index. 41. (d) Here, i1 = 15°, A = 60°, d = 55°, i2 = e = ?
28. (b) Medium doesn't effect focal length of a mirror.
As i1 + i2 = A + d
29. (a)
i2 = A + d – i1 = 60° + 55° – 15° = 100°.
1 æ 1 1 ö 42. (c) For reading purposes :
30. (a) = (m - 1)çç - ÷÷ u = – 25 cm, v = – 50 cm, f = ?
f è R1 R 2 ø
1 1 1 1 1 1 100
1 æ1 1ö = - =- + = ; P= = +2 D
= (1.5 - 1)ç - ÷ f v u 50 25 50 f
16 èR ¥ø

.IN
For distant vision, f' = distance of far point = –3 m
1 1 1 1
Þ = 0.5 ´ Þ R = 8 cm P= = - D = -0.33 D
16 R AL f¢ 3
31. (b) When we bring in contact a concave lens the effective 43. (c) E2 = 4 E1. If x is distance from 1st source,
focal length of the combination decreases.
Ι 4Ι 1 2
1 1 1 1 1 1 then, = or =
N
- = Þ = + (1.2 - x) 2
x 2 1 .2 - x x
v u f v u f
3x = 2.4, x = 0.8 m
R

according to above relation as f reduces, v increases.


32. (b) 33. (b) 34. (d) 35. (b) 1 h 1 1
U

1 ´ =
36. (a) In a telescope, to obtain an image at infinity or in normal 44. (d) E2 = E1 or, ( r 2 + h 2 ) 2
adjustment, the distance between two convex lenses 8 r2 + h2 8 h
JO

one called objective (greater focal length) and the other (by Lambert's cosine law)
called eye piece (shorter focal length) is L. or, ( r 2 + h 2 ) 3 / 2 = (2 h ) 3 or, (r 2 + h 2 )1 / 2 = 2 h
L = f0 + fe = 0.3 + 0.05 = 0.35 m. or, r2 + h2 = 4 h2
U

37. (b) given : a mg = 3 ,a m w = 4 h = r/ 3


ED

2 3
Apparent depth 1
Q a m w ´ w m g = a mg 45. (b) Since =
Realdepth m
a
mg Þ Apparent depth = d/m
w 3/ 2 9 So mark raised up = Real depth – Apparent depth
\ mg = = = .
a 4/3 8
mw
d æ 1 ö æ m -1 ö
38. (c) = d- = d çç1 - ÷÷ = çç ÷÷d
m è mø è m ø
39. (b) Object distance u = – 40 cm
Focal length f = – 20 cm 46. (c) From figure
According to mirror formula i
x B
1 1 1 1 1 1 = sin (i - r) » (i - r ) ...(1)
BC (i–r)
+ = or = - r
u v f v f u d d m
Further, = cos r » 1 D
1 1 1 1 1 BC x
or + - = +
v -20 ( -40 ) -20 40 (When i is small r is small) C
\ BC » d
1 -2 + 1 1
= =- or v = -40 cm. x
v 40 40 From eq. (1), » (i - r) or x » d (i - r)
Negative sign shows that image is infront of concave d
mirror. The image is real.
EBD_7179
654 PHYSICS

æ rö f1 w 2
or x » i d ç1 - ÷ \ = - 1 = (leaving sign)
è iø f2 w2 3
56. (d) The necessary condition is
sin i i æ 1ö
Now sin r = r = m \ x » i d çç1 - ÷÷ w f
è mø = - which is satisfied by (d)
w¢ f¢
5 4
47. (a) Given that w mg = and a m w = f1 w 1
4 3 57. (a) =- 1 =- \ f 2 = -2 f1
f2 w2 2
5 4 5
\ a mg = w mg ´a m w = ´ =
4 3 3 1 1 1
As = +
real depth F f1 f 2
48. (c) We know that m =
apparent depth 1 1 1 1
\ = - = \ f1 = 10 cm
Let the thickness of the slab be t and real depth of the 20 f1 2 f1 2 f1
bubble from one side be x. Then f2 = –20 cm
x (t - x) x t-x b f0 f0 60
m= = or 1 .5 = = . 58. (b) M= = \ b= µ= ´ 2° = 24°
6 4 6 4 µ fe fe 5

.IN
t-9 f0
This gives x = 9 and 1 . 5 = or t = 15 cm 59. (d) M= = 5 , L = f0 + fe = 36
4 fe
49. (d) Let d be the depth of two liquids.
Then apparant depth
AL \ fe = 6 cm, f0 = 30 cm
60. (b) When two lenses are placed coaxially then their
( d / 2) ( d / 2) d equivalent focal length F is given as
+ = or 1 + 2 = 1
N
m 1. 5 m 2 m 3m 1 1 1 ff
Solving we get m = 1.671 = + ÞF= 1 2
R

F1 f1 f 2 f1 + f 2
50. (c) The minimum length of the mirror is half the length of
f1 f2
U

the man. This can be proved from the fact that Ði = Ðr.
51. (d) Distance of image from plane mirror = 3 m at the back.
JO

To photograph the image, camera must be focussed


for a distance of 4.5 + 3 = 7.5 m.
100 100
U

52. (d) P2 = P - P1 = - = - 3 .75 D 1 f +f


80 20 Now power = = 1 2
ED

focal length f1f 2


æ mg ö 61. (b) Refractive index of medium is given by
ç - 1÷÷
ç
Pa è m a +5
ø= B
53. (b) =
P1 æ m g ö - 100 / 100
= -5 m =A+ 2 ( where A and Bareconstant ) .
ç ÷ l
ç m - 1÷ Light has seven different colour, so its each colour has
è 1 ø
different wavelength and so different refractive index.
æ m g ö mg Due to difference in refractive index different refractive
- 5 çç - 1÷÷ = -1
è m1 ø m a æ sin i ö
1 .5 -1 angle ç m = ÷.
1 .5 5 è sin r ø
-1 = (1 .5 - 1) = -0 .1 ; m1 = =
m1 5 0.9 3 So this is due to dependence on wavelength of
refractive index.
sin (A + d m ) / 2 62. (c) When a object is real plane mirror form a virtual image
54. (b) m=
sin A / 2 and when object is virtual image will be real. Thus in this
question object is virtual. Virtual object means object is
sin ( A + d m ) at infinity. So rays (incident) converge on the mirror.
Þ = m sin A / 2` = 1.5 × sin 30º = 0.75
2
A + dm
= sin -1 (0 .75 ) = 48 º 3 6 ¢ \ dm = 37º12'
2
w1 w 2
55. (b) Condition for achromatism is + =0
f1 f2
Ray Optics and Optical Instruments 655

Diameter of eye lens = 2 mm = 2 ´ 10 -3 m


From eq. (1) minimum separation is
5 ´10 -7 ´ 50
y= = 12.5 ´10 -3 m = 12.5 cm
-3
2 ´10
69. (a)
70. (a) Due to covering the reflection from lower part is not
63. (c) Resolving power of eye = l / a there so it makes the image less bright.
71. (b) Due to difference in refractive indices images obtained
500 ´ 10 -9
= = 10 -4 radians will be two. Two mediums will form images at two
5 ´ 10 -3 different points due to difference in focal lengths.
72. (b) 73. (d)
Now,\ arc = angle´ radius = 10–4 × (500 × 103)m = 50m v
64. (c) Spherical abberation occurs due to the inability of a 74. (b) m =
u
lens to coverage marginal rays of the same wavelength
to the focus as it converges the paraxial rays, This defect 1 1 1
- =
can be removed by blocking marginal rays. This can be v u f
done by using a circular annular mask over the lens. Multiply the equation by v
65. (d) Shift in the position of image after introducing slab,
v v v v
æ 1ö 1- = Þ = 1-

.IN
(d) = t ç1 - ÷ u f u f
è nø
nd v
nd = t (n – 1) Þ t = \ m = 1-
(n - 1) f
66. (b) The easiest way to answer this question is with a fast
AL
sketch. For a given object’s position, draw two rays 1 b c
Slope = - = Þf =
from the top of the object. One ray is parallel to the f c b
N
principal axis and passes through the focal point on 75. (c) The ray is partly reflected and partly refracted.
the opposite side of the lens. The other ray passes
ÐMOB = 180 – 2q
R

through the center of the lens. The top of the image


appears where these two rays intersect. But the angle between refracted and reflected ray is
U

ÐPOB. Clearly ÐPOB is less than ÐMOB.


Object A Object B
JO

F Image A Image B

F
U
ED

Change the object’s position and repeat the process.


You will observe that as the object approaches the lens
while remaining beyond the focal length, the image
76. (a)
produced on the opposite side of the lens moves away
from the lens and increases in size. As an aside, the 77. (d) For 1st lens, u1 = – 30, f1 = + 10cm,
image is real and inverted. 1 1 1
67. (a) The ratio of object to image distance equals the ratio of Formula of lens, + =
v1 30 10
object to image height. The ratio of image to object height
is found by rearranging the ratios to give 4f /(4/3)f = 1/3. or, v1 = 15 cm at I1 behind the lens.
The image is demagnified by a factor of 3. Thus, answer The images I1 serves as virtual object for concave lens.
choice A is the best answer. For second lens, which is concave, u2 = (15 – 5) = 10
l cm. I1 acts as object. f2 = – 10 cm.
68. (b) Angular limit of resolution of eye, q = , where d is
d The rays will emerge parallel to axis as the virtual object
diameter of eye lens. is at focus of concave lens, as shown in the figure. Image
Also if y is the minimum just resolution separation of I1 will be at infinity. These parallel rays are incident on
between two objects at distance D from eye then the third lens viz the convex lens, f3 = + 30 cm. These
y parallel rays will be brought to convergence at the focus
q=
D of the third lens.
y l lD \ Image distance from third lens = f3 = 30 cm
Þ = Þy= ...............(1)
D d d
Here : wavelength l = 5000Å = 5 ´ 10 -7 m
D = 50m
EBD_7179
656 PHYSICS

1 1 1 1 1 1
- = ; - = Þ v = 60 cm
v u f v - 30 20
Coincidence is possible when the image is formed at
the centre of curvature of the mirror. Only then the rays
refracting through the lens will fall normally on the
convex mirror and retrace their path to form the image
at O. So the distance between lens and mirror = 60 – 10
= 50 cm.
84. (a) Let a ray of light enter at A and refracted beam is AB.
This is incident at an angle q. For no refraction at the
78. (b) aµ = 1.5
g lateral face q > C
\ 1.5 = cosecC or C = 42°. Critical angle for glass sin q > sin C But q + r = (90°)
= 42°. Hence a ray of light incident at 50° in glass B
medium undergoes total internal reflection. d denotes
the deviation of the ray. q
\ d = 180° – (50° + 50°) or d = 80°. A r
i
1 æ 1 1 ö
79. (b) = (m - 1)çç - ÷÷
f è R1 R 2 ø

.IN
B C \ sin (90° – r) > sin C or cos r > sin C ..(1)
According to Cauchy relation m = A + + ......
l
2
l
4
sin i sin i
From Snell’s law n = Þ sin r =
Hence f µ l . sin r n
AL
Hence red light having maximum wavelength has
maximum focal length. æ sin 2 i ö
\ cos r = 1 - sin 2 r = ç1 - 2 ÷
fV < fR and also FV < FR è n ø
N
80. (c) As refracted ray emerges normally from oppostite
R

surface, r2 = 0 sin 2 i
\ equation (1) gives, 1 – > sin C
As A = r1 + r2 \ r1 = A n2
U

sin i1 i1 i sin 2 i
Now, m = sin r = r = A ; i = mA
JO

Þ1– > sin 2 C


1 1 2
n
(where t = thickness of glass plate)
1
81. (c) Thickness of glass plate (t) = 6 cm; Also sin C =
U

Distance of the object (u) = 8 cm. and n


distance of the image (v) = 12 cm. sin 2 i sin 2 i
ED

1 1
Let x = Apparent position of the silvered surface in cm. \1 – > or 1 > +
Since the image is formed due to relfection at the silvered n 2
n 2
n 2 n2
face and by the property of mirror image distance of object
1
from the mirror = Distance of image from the mirror or 2
(sin 2 i + 1) < 1 or n 2 > sin 2 i + 1
or x + 8 = 12 + 6 – x or x = 5 cm. n
Therefore refractive index of glass Maximum value of sin i = 1
Re al depth 6 \ n2 > 2 Þ n > 2
= Apparent depth = 5 = 1.2 .
85. (c) The silvered plano convex lens behaves as a concave
n mirror; whose focal length is given by
å fi
1 1
82. (d) We know that f = 1 2
= +
1
i =1 F f1 f m
1 1 1 If plane surface is silvered
= +
f f1 f 2 R2 ¥
fm = = =¥
f1 = 80 cm, f2 = –50 cm 2 2
1 1 1 1 1 æ 1 1 ö
= - Þ P = = 1.25 – 2 = – 0.75D \ = (m – 1)çç – ÷
÷
f 80 50 f f1 è 1
R R 2ø
100 100
æ 1 1 ö m –1
f = 20 cm = (m – 1)ç – ÷ =
83. (c) èR ¥ø R
l
O

30 cm 10 cm
60 cm
Ray Optics and Optical Instruments 657

1 2(m – 1) 1 2(m – 1) 1 1 1
\ = + = = +
F R ¥ R f f1 f 2
R This is combination of two lenses
F=
2(m – 1) of equal focal length
Here R = 20 cm, m = 1.5 é1 1 1 2 ù
20 \ ê = ¢ + ¢ = ¢ ú Þ f '' = 2f
\ F= = 20cm ëf f f f û
2(1.5 – 1) 89. (b) R1 = 60 cm, R2 = µ , m = 1.6
Re al depth (R1 ) 1 æ 1 1 ö
86. (b) 1.5 = = (m - 1)çç - ÷
Apparent depth (5 cm) f R R ÷
è 1 2ø
\ R1 = 1.5 × 5 = 7.5 cm
1 æ 1 ö
= (1.6 - 1)ç ÷ Þ f = 100 cm.
R1 5 cm f è 60 ø
90. (b) The angle must be equal to the critical angle,
air bubble
æ1ö æ 1 ö
R2 C = sin -1 çç ÷÷ = sin -1 çç ÷÷ = 45º
2 cm m
è ø è 2ø

.IN
R2 x 1.22l 1.22 ´ 5 ´10 3 ´ 10 -10 ´ 10 3
For opposite face, 1.5 = Þ R2 = 3.0 cm 91. (d) Here = or x =
2 AL 1000 D 10 ´ 10 - 2
\ Thickness of the slab = R1 + R2 = 7.5 + 3 or x = 1.22 × 5 × 10–3 m = 6.1 m
= 10.5 cm x is of the order of 5 mm.
sin 45 1 92. (c) Angle of minimum deviation
(a) For point A, a m g =
N
sin r Þ sin r = 2 m
87.
a g æ A + dm ö æ 60° + d m ö
R

sin ç sin ç
for point B, sin (90 – r) = gma è 2 ÷ø è 2 ø
÷
45º Air m= Þ 3=
U

(90 – r) is critical angle. æ Aö æ 60° ö


sin ç ÷ sin ç
90 – r r A è 2ø è 2 ÷ø
JO

1
\ cos r = g m a = 90 – r
a mg B æ d ö 3 d
Þ sin ç 30° + m ÷ = Þ 30° + m = 60°
U

1 è 2 ø 2 2
Þ a mg = Glass
ED

cos r Þ dm = 60°.
1 1 93. (c) For total internal reflection,
= =
1 - sin r 2 1 1
1- m³ >1.414
2 amg2 sin C ³ 2
Þ µ = 1.50
2 1 2 a m 2g
Þ a mg = = 94. (c) R = 20 cm, h 0 = 2, u = –30 cm
1 2 a m 2g - 1
1- 1 æ 1 1 ö
2 a m 2g We have, = (m - 1) ç -
f è R1 R 2 ÷ø
2 3
Þ 2 a mg - 1 = 2 Þ a mg = æ 3 ö é 1 æ 1 öù
2 = çè - 1÷ø ê - çè - ÷ø ú
2 ë 20 20 û
1 æ 1 1 ö
88. (b) = (m - 1)çç - ÷
÷
f è R1 R 2 ø 1 æ3 ö 2
Þ = ç - 1÷ ´
in this case, f è 2 ø 20
R1 and R2 are unchanged
\ f = 20 cm
So, f will remain unchanged for both pieces of the
lens 1 1 1 1 1 1
\ f= f' = - Þ = +
f v u 20 v 30
1 1 1 10
= - =
v 20 30 600
v = 60 cm
EBD_7179
658 PHYSICS

hi v 5. (c) Among all given sources of light, the bule light have
m= = Þ hi = v ´ h 0 = 60 ´ 2 = – 4 cm smallest wavelength.According to Cauchy
h0 u u 30 relationship, smaller the wavelength higher the
So, image is inverted. refractive index and consequently smaller the critical
95. (b) Deviation = zero
1
So, d = d1 + d2 = 0 Þ (m1 – 1)A1 + (m2 – 1) A2 = 0 angle as µ = .
sin c
Þ A2 (1.75 – 1) = – (1.5 – 1) 15°
Hence, corresponding to blue colour, the critical angle
0.5 is least which facilitates total internal reflection for the
Þ A2 = - ´ 15° or A2 = – 10°.
0.75 beam of blue light and the beam of green light would
Negative sign shows that the second prism is inverted also undergo total internal reflection.
with respect to the first. 6. (c) Using lens maker’s formula for plano-convex lens, so
96. (a) To see his full image in a plane mirror a person requires focal length is
a mirror of at least half of his height.
( m 2 - m1 ) æç
1 1 1 ö
= - ÷
H f R
è 1 R 2 ø
M
If object on curved suface
E H
H 2 so, R2 = ¥

.IN
M' R1
then, f =
L ( m 2 - m1 )
Lens placed in air, µ1 = 1.
97. (b) 98. (b) 99. (d) 100. (b)
AL
(As given that, R = 20cm, µ2 = 1.5, on substituting the
EXERCISE - 3 values in)
N
Exemplar Questions R1
f=
R

m -1
1. (a) As we know that the deviation
U

20
d = (µ – 1) A ..... (i) =
1.5 - 1
JO

By geometry, the angle of refraction by first surface is = 40 cm


5° and given µ = 1.5
So, f is converging nature, as f > 0. Hence, lens will
So, d = (1.5 – 1) × 5°
U

always act as a convex lens irrespective of the side on


= 2.5° which the object lies.
ED

also, d = q – r, ..... (ii) 7. (b) The reflection of radiowaves by ionosphere is similar


to total internal reflection of light in air during a mirage
By putting the value of d and r in equation (ii)
because angle of incidence is greater than critical angle
2.5° = q – 5° so that internal reflection of radio wave, take place.
So, q = 5 + 2.5 = 7.5° 8. (b) The incident PQ ray of light passes through focus F on
2. (d) As we know that when light ray goes from one medium the concave mirror, after reflection should become
to other medium, the frequency of light remains parallel to the principal axis, i.e., ray-2.
unchanged. 9. (b) As we know, when the ray goes from rarer medium air
And, c = nl to optically denser turpentine, then it bends towards
So, c µ l the light of red colour is of highest wavelength the normal i.e., i > r whereas when it goes from optically
and therefore of highest speed. Thus, after travelling denser medium turpentine to rarer medium water, then
through the slab, the red colour emerge first, it bends away from normal i.e., i < r.
3. (c) According to the question, when object is at different So, the path of ray 2 is correct.
position, and if an object approaches towards a
convergent lens from the left of the lens with a uniform 10. (d) As we know that, the image formed by convex mirror
speed of 5 m/s, the image move away from the lens to does not depend on the relative position of object w.r.t.
infinity with a non-uniform acceleration. mirror.
4. (b) When a passenger in an aeroplane then he may see So, when the car approaches in the rear side, initially it
primary and secondary rainbow such as concentric appear at rest as images is formed at focus. Hence the
circles. speed of the image of the car would appear to increase
as the distance between the cars decreases.
Ray Optics and Optical Instruments 659

11. (a) When the negative refractive index materials are those Distance between retina and cornea-eye lens
in which incident ray from air (Medium 1) to them refract 5
or bend differently to that of positive refractive index = = 1.67 m
3
medium. 14. (a)

Incident ray
i Air 20°
20°
1

r
2 q q
70° 40°
r
glass

From fig. 40° + q = 90° \ q = 90° – 40° = 50°


1
15. (b) It is due to scattering of light. Scattering µ . Hence
NEET/AIPMT (2013-2017) Questions l4
the light reaches us is rich in red.
12. (b) 16. (d) Magnifying power of microscope

.IN
LD 1
= µ
AL f 0f e f0
Combination
Hence with increase f0 magnifyig power of microscope
decreases.
N
f0
Plano-convex Magnifying power of telescope = µ f0
Plano-concave
R

fe
1 1 1 Hence with increase f0 magnifying power of telescope
U

= +
f f1 f2 increases.
JO

17. (b)
æ1 1 ö æ 1 1 ö
= (m1 – 1) ç - ÷ + (m2 – 1) ç - ÷
è ¥ -R ø è¥ R ø
U

( m1 - 1) ( m 2 - 1) 1 m - m2
= – Þ = 1
ED

R R f R
R
Þf=
m1 - m 2
sin i
According to Snell’s law m =
R sin r
Hence, focal length of the combination is .
m1 - m 2 Þ (1) sin 2A = (m) sin A Þ m = 2 cos A
13. (b) Pcornea = + 40 D 18. (a) As we know, the refractive index of the material of the
Pe = + 20 D prism
Total power of combination = 40 + 20 = 60 D
æ d + Aö
1 sin ç m
Focal length of combination = ´ 100 cm è 2 ÷ø
60 m=
sin (A/ 2)
5
= cm æ A + dm ö
3 sin ç
For minimum converging state of eye lens, è 2 ÷ø cos (A/ 2)
cot A/2 = =
sin A / 2 sin (A / 2)
5
u = -¥ v =? f= [Q µ = cot (A/2)]
3
From lens formula, æ dm + A ö
Þ Sin çè ÷ = sin(90° + A/2)
1 1 1 5 2 ø
= – Þ v = cm Þ dmin = 180° – 2A
f v u 3
EBD_7179
660 PHYSICS

19. (b) Using lens maker’s formula, f


m=
1 æ 1 1 ö f +u
= (m – 1) ç – ÷
f è R1 R 2 ø I fe f I fe
- = = - e or, =
L fe + [ -(f0 + f e )] f0 L f0
1 æ 1.5 öæ 1 1 ö
=ç – 1÷ç – ÷ f0 L
f1 è 1 øè ¥ –20 ø Magnification, M = f = I
n = 1.5 n = 1.5 e
Þ f1 = 40cm n = 1.7
22. (b) Given: Angle of incidence
1 æ 1.7 öæ 1 1 ö angle of prism,
=ç – 1÷ç – ÷ i = 45°;
f2 è 1 øè –20 +20 ø
A = 60°;
100 Angle of minimum deviation,
Þ f2= – cm dm = 2i – A = 30°
7 Refractive index of material of prism.
1 æ 1.5 öæ 1 1 ö æ A + dm ö
and =ç – 1÷ç – ÷ sin ç ÷
f3 è 1 øè ¥ –20 ø è 2 ø
m=
Þ f3 = 40 cm sin A / 2

.IN
1 1 1 1 sin 45° 1 2
= + + = = · = 2
f eq f1 f 2 f 3 sin 30° 2 1
1 1 1 1
AL
23. (d) Given: Focal length of objective, f0 = 40cm
Þ = + + Focal length of eye – piece fe = 4 cm
f eq 40 –100 / 7 40
image distance, v0 = 200 cm
\
N
feq = –50 cm
Therefore, the focal length of the combination is – 50 Using lens formula for objective lens
R

cm. 1 1 1 1 1 1
20. (c) For total internal reflection, incident angle (i) > critical - = Þ = +
U

angle (ic) v0 u 0 f 0 v0 f 0 u 0
JO

1 1 1 +5 - 1
Þ = + =
v0 40 -200 200
U

Þ v0 = 50 cm
i=45° Tube length l = |v0| + fe = 50 + 4 = 54 cm.
ED

24. (a) Magnitude m = +ve Þ virtual image


m = –ve Þ real image
45°
magnitude of magnification,
So, sin i > sin ic | m | > 1 Þ magnified image
1 | m | < 1 Þ diminished image
sin 45° > Þ m > 2 Þ 1.414
m 25. (d) When mirror is rotated by angle q reflected ray will be
Since refractive index m of green and voilet are greater rotated by 2q.
than 1.414 so they will total internal reflected. But red light
colour will be refracted. y spot q
= 2q
21. (c) Objective lens x y
Eye-piece
L y source 2q
Mirror
Þq= (L) x
2x
26. (a) For dispersion without deviation
(m - 1)A1 + (m '- 1)A 2 = 0
d=f0+fe (m - 1)A1 = (m '- 1)A 2
Magnification by eye piece (1.42–1) × 10° = (1.7–1)A2
4.2 = 0.7A2
A2 = 6°
25 Wave Optics

WAVEFRONT (iii) The forward envelope of the secondary wavelets at any


The locus of all particles of the medium vibrating in the same instant gives the new wavefront.
phase at a given instant is called a wavefront. Depending on A² A A¢
the shape of source of light, wavefront can be of three types. Primary A² A A¢ Secondary
wavefront wavefront

.IN
(i) Spherical wavefront: A spherical wavefront is produced by a
point source of light. This is because the locus of all such
points which are equidistant from the point source will be a AL
sphere. Spherical wavefronts are further divided into two
headings: (i) converging spherical and (ii) diverging spherical
wavefront.
N
Secondary
wavelets
B² B B¢
R

B² B B¢
O O O
U

(iv) In a homogeneous medium the wavefront is always


perpendicular to the direction of wave propagation.
JO

With the help of Huygen’s wave theory, law of


Spherical Converging Diverging
reflection and refraction, total internal reflection and dispersion
wavefront spherical wavefront spherical wavefront can be explained easily. This theory also explain interference,
U

(ii) Cylindrical wavefront: When the O¢ diffraction and polarization successfully.


ED

source of light is linear in shape Drawbacks of Huygens Wave Theory


such as a slit, the cylindrical (a) This theory cannot explain photo-electric effect, compton,
wavefront is produced. This is S and Raman effect.
because all the points equidistant (b) Hypothetical medium in vacuum is not true imagination.
from a line source lie on the surface (c) The theory predicted the presence of back wave, which
of a cylinder. O proved to be failure.
Cylindrical wavefront REFLECTION AND REFRACTION OF PLANE WAVES
USING HUYGENS PRINCIPLE
(iii) Plane wavefront: A small part Reflection on the Basis of Wave Theory
of a spherical or cylindrical According to Huygens principle, every point on AB is a source
wavefront due to a distant of secondary wavelets. Let the secondary wavelets from B strike
source will appear plane and reflecting surface M1M2 at A¢ in t seconds.
hence it is called plane wave- \ BA ' = c ´ t … (i)
front. The wavefront of parallel where c is the velocity of light in the medium.
rays is a plane wavefront.
Plane wavefront
HUYGENS WAVE THEORY 3
B¢ 1¢
(Geometrical method to find the secondary wavefront) B
(i) Each point source of light is a centre of disturbance from 2
N 2¢
which waves spread in all directions. D D¢
(ii) Each point on primary wavelets acts as a new source of 1 i r r 3¢
i
distrubance and produces secondary wavelets which travel A P A¢
in space with the speed of light. M1 M2
EBD_7179
662 PHYSICS

The secondary wavelets from A will travel the same distance c × sin i c1
t in the same time. Therefore, with A as centre and c × t as radius, \ = =µ [using (iv)]
sin r c2
draw an arc B¢, so that
AB¢ = c × t … (ii) sin i
Hence m= … (vi)
A¢B¢ is the true reflected wavefront. sin r
angle of incidence, i = ÐBAA ' which proves Snell’s law of refraction.
and angle of reflection, r = ÐB ' A ' A It is clear from fig. that the incident rays, normal to the interface
XY and refracted rays, all lie in the same plane (i.e., in the plane of
In Ds AA¢B and AA¢B¢,
the paper). This is the second law of refraction.
AA¢ is common, BA ' = AB ' = c ´ t , and ÐB = ÐB ' = 90° Hence laws of refraction are established on the basis of wave
\ Ds are congruent \ ÐBAA ' = ÐB ' A ' A, i.e., Ði = Ðr … (iii) theory.
Which is the first law of reflection. Keep in Memory
Further, the incident wavefront AB, the reflecting surface M1M2
and the reflected wavefront A¢B¢ are all perpendicular to the plane 1. In 1873, Maxwell showed that light is an electromegnetic
of the paper. Therefore, incident ray, normal to the mirror M1M2 wave i.e. it propagates as transverse non-mechnical wave
and reflected ray all lie in the plane of the paper. This is second at speed c in free space given by
law of reflection. 1
Refraction on the Basis of Wave Theory c= = 3 ´ 108 ms -1
XY is a plane surface that separates a denser medium of refractive m 0e 0

.IN
index µ from a rarer medium. If c1 is velocity of light in rarer 2. There are some phenomenon of light like photoelectric
medium and c2 is velocity of light in denser medium, then by effect, Compton effect, Raman effect etc. which can be
definition. AL explained only on the basis of particle nature of light.
c1 3. Light shows the dual nature i.e. particle as well as wave
µ= c … (iv) nature of light. But the wave nature and particle nature
2 both cannot be possible simultaneously.
N
4. Interference and diffraction are the two phenomena that can
3 be explained only on the basis of wave nature of light.
R

B INTERFERENCE OF LIGHT WAVES AND YOUNG'S


U

2 DOUBLE SLIT EXPERIMENT


Rarer-C1 The phenomenon of redistribution of light energy in a medium
JO

D N on account of superposition of light waves from two coherent


1 i
sources is called interference of light waves.
i P A¢ Young performed the experiment by taking two coherent sources
U

X A r Y
of light. Two source of light waves are said to be coherent if the
ED

D¢ r
initial phase difference between the waves emitted by the source
3¢ remains constant with time.
Denser-C2
B¢ (i) The rays of light from two coherent sources S1 and S2

1¢ superpose each other on the screen forming alternately
maxima and minima (constructive and destructive
AB is a plane wave front incident on XY at ÐBAA ' = Ði . 1, 2, 3 interference).
are the corresponding incident rays normal to AB.
According to Huygens principle, every point on AB is a source P
S1 y
of secondary wavelets. Let the secondary wavelets from B strike M1
XY at A¢ in t seconds. S d O
\ BA¢ = c1 × t … (v)
The secondary wavelets from A travel in the denser medium with S2 M2
a velocity c2 and would cover a distance (c2 × t) in t seconds.
D Screen
A¢B¢ is the true refracted wavefront. Let r be the angle of refraction.
As angle of refraction is equal to the angle which the refracted
plane wavefront A¢B¢ makes with the refracting surface AA¢, (ii) Let the equation of waves travelling from S1 & S2 are
therefore, ÐAA ' B ' = r . y1 = A1Sin wt ...(1)
Let ÐAA ' B ' = r , angle of refraction. y 2 = A 2Sin wt ...(2)
BA ' c1 ´ t where A1 & A2 are amplitudes of waves starting from S1
In DAA¢B, sin i = = & S2 respectively. These two waves arrive at P by traversing
AA ' AA '
different distances S2P & S1P. Hence they are superimposed
AB ' c2 ´ t with a phase difference (at point P) given by
In D AA¢B¢, sin r = =
AA ' AA '
Wave Optics 663

(B) Position of fringe:


2p
d ( phase difference ) = ´ D ( path difference ) (i) If D = S2P – S1P = nl, then we obtain bright fringes at
l poin t P on the screen and it corresponds to
2p constructive interference. So from equation (4) the
= (S2 P - S1P). ....(3) position of n th bright fringe
l
yd
2 D = S 2 P - S1P = nl =
æ dö 2 D
where S2 P (from fig) = D + ç y + ÷
è 2ø æ nD ö
or y = ç ÷l ...(7)
2 è d ø th
1 ( y + d / 2) (Position of n bright fringe)
» D+ [\ D >> ( y + d)]
2 2D l
(ii) If D = S2 P - S1P = (2n + 1) , then we obtain dark
( y – d / 2) 2 2
Similarly, S1P » D +
4D fringe at point P on the screen and corresponds to
destructive interference. So from equation(4), the
yd position of, n th dark fringe is
so, S2 P - S1P = ....(4)
D
l yd
(A) Conditions for maximum & minimum intensity : D = S2 P - S1P = (2n + 1) =
2 D
(i) Conditions for maximum intensity or constructive

.IN
interference : If phase difference (2n + 1) Dl
d = 0, 2p, 4p – – – 2np or y = ...(8)
2d
or, path difference D = S2 P - S1P = 0, l, 2l - - - nl (Position of nth dark fringe)
AL
(C) Spacing or fringe width :
then resultant intensity at point P due two waves
emanating from S1 & S2 is Let yn and yn+1 are the distance of n th and (n+1)th bright
fringe from point O then
N
I = A2 = A12 + A22 + 2 A1 A2 cos d (\ I µ A 2 ) Dnl D( n + 1)l
yn = & y n +1 =
R

or I = ( A1 + A2 ) 2 d d
So spacing b between nth and (n+1)th bright fringe is
U

or I = I1 + I 2 + 2 I1I 2 ....(5)
Dl
JO

It means that resultant intensity is greater than the b = y n +1 - y n = ...(9)


d
sum of individual intensity ( where A is the amplitude
of resultant wave at point P). Since it is independent of n, so fringe width or spacing
between any two consecutive bright fringes is same.
U

(ii) Conditions for minimum intensity or destructive


interference : If phase difference, Similarly the fringe width between any two consecutive
ED

dark fringe is
d = p, 3p, 5p - - - -(2n + 1)p
or, path difference Dl
b= ...(10)
d
l 3l l
D = S2 P - S1P = , - - - (2n + 1) (D) Conditions for sustained interference:
2 2 2 (i) The two sources should be coherent i.e they should
then resultant intensity at point P is have a constant phase difference between them.
I = A2 = A12 + A22 – 2 A1 A2 cos d (ii) The two sources should give light of same frequency
(or wavelength).
or I = ( A1 - A2 ) 2 (iii) If the interfering waves are polarized, then they must
be in same state of polarization.
or I = I1 + I 2 - 2 I1 I2 ...(6) (E) Conditions for good observation of fringe:
It means that resultant intensity I is less than the sum (i) The distance between two sources i.e. d should be
of individual intensities. Now as the position of point small.
P on the screen changes, then the path difference at (ii) The distance of screen D from the sources should be
point P due to these two waves also changes & quite large.
intensity alternately becomes maximum or minimum.
(iii) The two interfering wavefronts must intersect at a very
These bright fringes ( max. intensity) & dark fringes
small angle.
(min. intensity) make an interference pattern.
It must be clear that there is no loss of energy (F) Conditions for good contrast of fringe :
( at dark fringe) & no gain of energy ( at bright fringe), (i) Sources must be monochromatic i.e they emit waves
but, only there is a redistribution of energy. of single wavelength.
The shape of fringe obtained on the screen is (ii) The amplitude of two interfering waves should be
approximately linear. equal or nearly equal.
EBD_7179
664 PHYSICS

(iii) Both sources must be narrow.


For neon l = 6238 Å , t c » 10-10 sec. and L = 0.03 m.
(iv) As Intensity I is directly proportional to the square of
amplitude, hence Intensity of resultant wave at P,
For cadmium l = 6238 Å , t c = 10-9 and L = 0.3 m
I = I1 + I2 + 2 I1 I 2 cos f; if I1 = I 2 = I 0 . , then
For Laser t c = 10-5 sec and L = 3 km.
I = 4I 0 cos 2 f ( 2) (iii) The spectral lines width Dl is related to coherence
length L and coherence time tc.
( )
2
(v) Imax = I1 + I2 .
l2 l2
Dl » or Dl »
If I1 = I2 = I0, then Imax = 4I0 ct c L
( )
2
Imin = I1 - I 2 , if I1 = I2 = I0, then Imin = 0 2. Spatial coherence : Two points in space are said to be
spatially coherence if the waves reaching there maintains a
2 constant phase difference. Points P and Q are at the same
I æ I1 + I2 ö
(vi) max = ç ÷ . distance from S, they will always be having the same phase.
Imin è I1 - I2 ø Points P and P¢ will be spatially coherent if the distance
between P and P¢ is much less than the coherence length i.e.
b l
(vii) Angular fringe-width q0 = = PP ¢ << ct c
D d

.IN
(viii) The width of all interference fringes are same. Since
fringe width b is proportional to l, hence fringes with

red light are wider than those for blue light.
AL P
(ix) If the interference experiment is performed in a medium
of refractive index m instead of air, the wavelength of
l Q
N
light will change from l to .
m
R

Dælö b Monochromatic
i.e. b´= ç ÷=
U

d èmø m source of light


(x) If a transparent sheet of refractive index m and
JO

thickness t is introduced in one of the paths of Methods of Obtaining Coherent Sources


interfering waves, then due to its presence optical path Two coherent sources are produced from a single source of light
will become mt instead of t. Due to this a given fringe by two methods :
U

from present position shifts to a new position. So the (i) By division of wavefront and (ii) By division of amplitude.
lateral shift of the fringe,
ED

(i) Division of wavefront : The wavefront emitted by a narrow


D b source is divided in two parts by reflection, refraction or
y0 = (m - 1)t = (m - 1)t
d l diffraction. The coherent sources so obtained are imaginary.
(xi) In Young’s double slit experiment (coherent sources Example : Fresnel’s biprism, Llyod’s mirror, Young’s double
in phase): Central fringe is a bright fringe. It is on the slit, etc.
prerpendicular bisector of coherent sources. Central (ii) Division of amplitude : In this arrangement light wave is
fringe position is at a place where two waves having partly reflected (50%) and partly transmitted (50%) to
equal phase superpose. produced two light rays. The amplitude of wave emitted by
(xii) Young’s experiment with the white light will give white an extended source of light is divided in two parts by partial
central fringe flanked on either side by coloured bands. reflection and partial refraction. The coherent sources
COHERENCE obtained are real and are obtained in Newton’s rings,
The phase relationship between two light waves can very from Michelson’s interferometer, etc.
time to time and from point to point in space. The property of Incoherence of Two Conventional Light Sources
definite phase relationship is called coherence.
1. Temporal coherence : A light wave (photon) is produced Let two conventional light sources L1 and L2 (like two sodium
when an excited atom goes to the ground state and emits lamps or two monochromatic bulbs) illuminate two pin holes S1
light. and S2. Then we will find that no interference pattern is seen on
(i) The duration of this transition is about 10–9 to 10–10 the screen.
sec. Thus the emitted wave remains sinusoidal for this The reason is as follows : In conventional light source, light
much time. This time is known as coherence time ( t c ). comes from a large number of independent atoms, each atom
(ii) Definite phase relationship is maintained for a length emitting light for about 10–9 seconds i.e., light emitted by an
atom is essentially a pulse lasting for only 10–9 seconds.
L = ct c called coherence length.
Wave Optics 665

Interference in Thin Films


We are familiar with the colours produced by a thin film of oil on
the surface of water and also by the thin film of a soap bubble.
L1 S1 Hooke observed such colours in thin films of mica and similar
thin transparent plates. Young was able to explain the phenomenon
on the basis of interference between light reflected from the top
S2 and bottom surface of a thin film. It has been observed that
L2 interference in the case of thin films takes place due to
(i) reflected light and (ii) transmitted light.
Interference due to reflected light
Screen From the figure, the optical path difference between the reflected
Even if all the atoms were emitting light pulses under similar ray (AT) from the top surface and the reflected ray (CQ) from the
conditions, waves from different atoms would differ in their initial bottom surface can be calculated. Let it be x, then
phases. Consequently light coming out from the holes S1 and S2 T
S
will have a fixed phase relationship only for 10–9 sec. Hence any N Q
interference pattern formed on the screen would last only for i Air
i
10–9 sec. (a billionth of a second), and then the pattern will A C
change. The human eye can notice intensity changes which last t r m
M

.IN
at least for a tenth of a second and hence we will not be able to
B Air
see any interference pattern. Instead due to rapid changes in the r F
pattern, we will only observe a uniform intensity over the screen. AL P
LIoyd’s Mirror x = m(AB + BC) - AN
The two sources are slit S ( parallel to mirror ) and its virtual image On simplification, we get
N
S'. x = 2mt cos r
l
R

Super-position 1. If 2mt cos r = (2n + 1) , where n = 0,1,2, ..............then


Screen 2
U

occurs in this region constructive interference takes place and the film appears
S
bright.
JO

d O 2. If 2mt cos r = nl , where n = 0, 1, 2, 3,............ then destructive


Flat black interference takes place and the film appears dark.
S'
U

glass Interference due to transmitted light


D P
ED

S r
Lloyd mirror arrangement
i
C Air
(i) If screen is moved so that, point O touches the edge of A
glass plate, the geometrical path difference for two wave t r m
rr M
trains is zero. The phase change of p radian on reflection at r D
i Air
denser medium causes a dark fringe to be formed. B
· The fring width remains unchanged on introduction N Q
R
of transparent film. The optical path difference between the reflected ray (DQ) and
· If the film is placed in front of upper slit S1, the fringe the transmitted ray (NR) is given by
pattern will shift upwards. On the other hand if the x = m(BC + CD) - BN
film is placed in front of lower slit S2, the fringe pattern
On simplification, we get
shifts downwards.
(ii) This interference pattern is frequently seen in a ripple tank x = 2mt cos r
when one uses a wave train to demonstrate the law of 1. If 2mt cos r = nl , where n = 0, 1, 2, 3, .............then
reflection. constructive interference takes place and the film appears
Dl bright.
(iii) In this case, fringe width b =
d l
2. If 2mt cos r = (2n + 1) , n = 0, 1, 2, ....... then destructive
Optical path : (Equivalent path in vacuum or air) In case of medium 2
of refractive index m and thickness t, the optical path = mt. interference takes place and the film appears dark.
EBD_7179
666 PHYSICS

Newton's Rings Therefore,


Newton observed the formation of interference rings when a
For bright rings, 2mt cos q = nl, n = 0,1, 2,....
plano-convex lens is placed on a plane glass plate. When viewed
with white light, the fringes are coloured while with monochromatic l
light, the fringes are bright and dark. These fringes are produced For dark rings, 2mt cos q = (2n - 1) , n = 0,1, 2,......
due to interference between the light reflected from the lower 2
surface of the lens and the upper surface of the glass plate. Proceeding further, we get Radius of bright ring,
Interference can also take place due to transmitted light.
(2n - 1)lR
r=
2
Air film

(i) The centre is bright and alternately bright and dark


Newton's rings by reflected light : rings are obtained.
Here, interference takes place due to reflected light. Therefore, (ii) The ring pattern due to reflected light is just opposite
for bright rings, to that of transmitted light.
l
2 mt cos q = (2n - 1) where n = 1, 2, 3, ...... Keep in Memory
2

.IN
And for dark rings, 2 mt cos q = nl , n = 1, 2, 3, ...... 1. If Dn and Dn + m be the diameters of n th and (n + m)th dark
AL rings then the wavelength of light used is given by

(D n + m ) 2 - (Dn )2
l=
4mR
N
L where, R is the radius of curvature of the lens.
2. If Dn = diameter of nth dark ring when air is present betwen
R

Air film
the glass plate and the lens
G
U

Dn+m = diameter of (n+m)th dark ring when air is present


between the glass plate and the lens
JO

Proceeding further, we get the radius of rings as follows:


D¢n = diameter of n th dark ring when a liquid is poured
(2n - 1)lR between the plate and the lens
For bright rings, r =
2 D¢n+m =diameter of (n+m)th dark ring when a liquid is
U

poured between the plate and the lens


For dark rings, r = nlR , where R = radius of curvature of
ED

Then the refractive index of the liquid is given by


lens.
(D n + m )2 - (Dn )2
m= or,,
(i) The centre is dark and alternately dark and bright rings (D n¢ + m ) 2 - (D n¢ )2
are produced.
(ii) While counting the order of the dark rings 1, 2, 3, etc. 4mlR
the central ring is not counted. Therefore, m=
(D n¢ + m )2 - (D n¢ )2
for 1st dark ring, n = 1 and r1 = lR
Example 1.
for 2nd dark ring, n = 2 and r2 = 2lR In Young’s expt., two coherent sources are placed 0.90 mm
apart and fringes are observed one metre away. If it
Newton's rings by transmitted light produces second dark fringe at a distance of 1 mm from
Here, interference takes place due to transmitted light. central fringe, what would be the wavelength of
monochromatic light used?
Solution :
lD
For dark fringes, x = (2 n - 1)
L 2d

\ l=
2xd 2 ´ 10 -3 ´ 0.9 ´ 10 -3
=
(2 n - 1) D (2 ´ 2 - 1) ´1
G
or, l = 0.6 ´ 10 -6 m = 6 ´10 -5 cm.
Wave Optics 667

Example 2. By comonendo and dividendo,


Two beam of light having intensities I and 4 I interfere to I1 3+1 I1 4
produce a fringe pattern on a screen. The phase difference = = =4
between the beams is p/2 at point A and p at point B. Then I2 3 - 1 i.e., I2 1
find the difference between the resultant intensities at A (b) Now as for a wave I µ A 2 ,
and B.
2 2
Solution : I1 é A1 ù é A1 ù A
=ê ú , ê ú = 4, i.e., 1 = 2
Here, I1 = I; I 2 = 4 I; q1 = p / 2, q2 = p I2 ë A 2 û ë A 2 û A2
I A = I1 + I 2 + 2 I1 I 2 cos q1 Example 5.
In a Young’s double slit experiment the angular width of a
= I + 4 I + 2 I ´ 4 I cos p / 2 = 5 I fringe formed on a distant screen is 1°. The wavelength of
the light used is 6280 Å. What is the distance between the
I B = I1 + I 2 + 2 I1 I 2 cos q 2
two coherent sources ?
= I + 4 I + 2 I 4 I cos p = 5 I - 4 I = I ; Solution :
\ IA - IB = 5 I - I = 4 I l
The angular fringe width is given by a =
d
where l is wavelength and d is the distance between two
Example 3.
l

.IN
In a biprism experiment, 5th dark fringe is obtained at a coherent sources. Thus d =
point. If a thin transparent film is placed in the path of one a
of waves, then 7th bright fringe is obtained at the same p
Given, l = 6280 Å, a = 1° = radian
point. Determine the thickness of the film in terms of
AL 180
wavelength l and refractive index m .
6280 ´ 10-10
Solution : Thus d = ´ 180 = 3.6 ´ 10-5 m = 0.036 mm
N
3.14
l D 9l D
For 5th dark fringe, x1 = (2 n - 1) = DIFFRACTION
R

2 d 2d
When a wave is obstructed by an obstacle, the rays bend round
D 7lD
U

For 7th bright fringe, x 2 = n l the corner. This phenomenon is known as diffraction.
=
d d Fraunhoffer Diffraction by Single Slit
JO

D lD é 9ù D In Fraunhoffer diffraction experiment, the source and the screen


but x 2 - x1 = (m - 1) t ; 7 - ú = (m - 1) t
d d êë 2û d are effectively at infinite distance from the diffracting element.
U

2.5 l In single slit diffraction, imagine aperature to be divided into two


\ Thickness, t = equal halves. Secondary sources in these two halves give first
(m - 1)
ED

minima at b sin q = l
Example 4.
In Young’s experiment, the interference pattern is found to
have an intensity ratio between the bright and dark fringes P
q
as 9. What is the ratio of (a) intensities (b) amplitudes of
q
the two interfering waves ? b P0
q
Solution :

In case of interference, I = I1 + I2 + 2 ( )
I1I 2 cos f
D
(a) For I to be maximum and minimum cos f is 1 and –1
l
respectively, i.e., In general, b sin q = nl for minima and, b sin q = ( 2n + 1) for
2
I max = I1 + I 2 + 2 I1I 2 = ( I1 + I 2 ) 2 and maxima.
(i) The points of the maximum intensity lie nearly midway
I min = I1 + I 2 - 2 I1I 2 = ( I1 ~ I 2 ) 2 between the successive minima. The amplitude E0' of the
According to given problem, electric field at a general point P is

( ) =9, sin b p b sin q


2
Imax I1 + I2 E0 ' = E0 where b = and
= b l
( I2 )
I min 2 1
I1 - E0 = amplitude at the point P0 i.e. at q = 0
Imax I1 + I2 3 sin 2 b
i.e., I = = The intensity at a general point P is given as I = I 0
min I1 - I2 1 b2
EBD_7179
668 PHYSICS

Difference between Interference and Diffraction of light


(ii) The graph for the variation of intensity as a function of
Interference Diffraction
sinq is as follows :
1. Interference is due to the 1. Diffraction is due to the
superposition of two superposition of two
I
wavefronts originating secondary wavelets
I0 from two coherent originating from the
sources. different points of the
same wavefront.
2. In Interference pattern, 2. In diffraction pattern, the
I0 / 22 I0 / 62.5 all the maxima i.e. bright bright fringes are of
fringes are of the same varying intensity.
-3l -2l -l O l 2l 3l sinq intensity.
b b b b b b 3. In Interference pattern, the 3. In diffraction pattern, the
dark fringes are usually dark fringes are not
almost perfectly dark. perfectly dark.
2lD ö
(iii) The width of the central maxima is æç ÷ and angular
4. In Interference pattern, the 4. In diffraction pattern, the
è b ø width of fringes (bright widths of fringes are not
and dark) is equal. equal.

.IN
æ 2l ö 5. In Interference, bands are 5. In diffraction, bands are
width of central maxima is ç ÷ .
è b ø AL large in number. a few in number.
Fraunh offer Diffraction by a Circular Aperture 6. In Interference, bands are 6. In diffraction, bands are
equally spaced. unequally spaced.
(i) The 1st dark ring is formed by the light diffracted from the
N
circular aperture at an angle q with the axis where Example 6.
In a single slit diffraction experiment, the angular position
R

1.22l of the first (secondary) maximum is found to be 5.2°, when


sin q » where l = wavelength of light used,
b the slit width is 0.01 mm. If sin 52° = 0.0906, then find the
U

b = diameter of circular aperture wavelength of light used.


Solution :
JO

For single-slit diffraction, the angular position of the first


maximum is determined from the relation
Circular aperture
U

3l
a sin q1¢ =
2
ED

b q
It is given that a = 0.01 mm
= 1 × 10–5, q1¢ = 5.2°, sin q1¢ = 0.0906 . Therefore,
D 2 2
Screen l= a sin q1¢ = ´ 10 -5 ´ 0.0906 = 6040 Å
3 3
Example 7.
(ii) If the screen is at a distance D (D >> b) from the circular In Fraunhaufer diffraction from a single slit of width 0.3
aperture, the radius of the 1st dark ring is, mm the diffraction pattern is formed in the focal plane of a
lens of focal length 1m. If the distance of third minimum
1.22lD from the central maximum is 5mm, then find the wavelength

b of light used.
(iii) If the light transmitted by the hole is converged by a Solution :
converging lens at the screen placed at the focal plane of The distance of n th minimum from the central maximum is
nlf
given by X n =
the lens, the radius of the 1st dark ring is R = 1.22lf a
b
where it is given that
This radius is also called the radius of diffraction disc. a = 0.3 × 10–3 m, n = 3, f = 1m, Xn = 5 × 10–3 m
For plane transmission diffraction grating Therefore,
(a + b) sin qn = nl for maxima, where a = width of transparent
aX n 0.3 ´ 10-3 ´ 5 ´ 10 -3
portion, b = width of opaque portion. l= = = 5 ´ 10-7 m = 500 nm
nf 3 ´1
Wave Optics 669

POLARISATION I = I0 cos 2 q where I0 is the intensity when the incident electric


An ordinary source such as bulb consists of a large number of vector is parallel to the transmission axis.
waves emitted by atoms or molecules in all directions · Polarization can also be achieved by scattering of light
symmetrically. Such light is called unpolarized light (see fig - a) r
· (a) Plane polarized : oscillating E field is in a single plane.
Y Source r
(b) Circularly polarized : tip of oscillating E field describes
a circle.
Direction of r
X wave motion (c) Elliptically polarized : tip of oscillating E field
describes an ellipse.
Z Example 8 :
Fig (a) Unpolarised light The intensity of the polarised light becomes 1/20th of its
initial intensity after passing through the analyser. What
is the angle between the axis of the analyser and the initial
amplitude of the light beam ?
Solution :

1
Here I = I0 = 0.05 I 0

.IN
Fig (b) Polarised light 20
If we confine the direction of wave vibration of electric vector in
Using I = I0 cos2 q , we get 0.05 I0 = I0 cos 2 q
one direction perpendicular to direction of wave propagation,
AL
then such type of light is called plane polarised or linearly
polarised (with the help of polaroids or Nicol prism). The Þ cos 2 q = 0.05 or cos q = 0.05 = 0.2236
N
phenomenon by which, we restrict the vibrations of wave in a
\ q = cos-1 (0.2236) = 76°9 ¢
particular direction (see fig-b) ^ to direction of wave
R

propagation is called polarization. Example 9 :


The plane of vibration is that which contains the vibrations of
U

A beam of polarised light makes an angle of 60° with the


r
electric vector E and plane of polarisation is perpendicular to axis of the polaroid sheet. How much is the intensity of
JO

the plane of vibration light transmitted through the sheet ?


· Tourmaline and calcite polarizes an e.m. wave passing Solution :
through it. Here q = 60°,
U

Polarization by Reflection (Brewster’s Law)


Using I = I0 cos2 q , we get
ED

During reflection of a wave, we obtain a particular angle called


angle of polarisation, for which the reflected light is completely 1 æ 1ö
plane polarised. I = I0 (cos 60°) 2 = I0 çQ cos 60° = ÷
4 è 2ø

Reflected light 1
is polarised. \ Intensity of transmitted light = ´ 100 = 25%
4
ip Thus, the intensity of the transmitted light is 25% of the
ip+rp=90º
Rarer intensity of incident light.
90º
Denser Example 10:
rp
A ray of light strikes a glass plate at an angle of 60° with
the glass surface. If the reflected and refracted rays are at
right angles to each other, find the refractive index of the
glass.
Solution.
m = tan (ip) When the reflected and refracted rays are at right angle to
where, ip = angle of incidence, such that the reflected and refracted each other, the angle of incident is known as angle of
waves are perpendicular to each other. polarisation (ip).
Law of Malus : If the electric vector is at angle q with the Here, q = 60°, Using m = tan ip, we get
transmission axis, light is partially transmitted. The intensity of
transmitted light is m = tan 60° = 3 = 1.732
EBD_7179
670 PHYSICS

Example 11 : Example 12 :
A beam of light AO is incident on a glass slab (m = 1.54) in
The time period of rotation of the sun is 25 days and its
a direction as shown in fig. The reflected ray OB is passed
through nicol prism. On viewing through a nicol prism, radius is 7 × 108 m. What will be the Doppler shift for the
we find on rotating the prism that light of wavelength 6000 Å emitted from the surface of the
sun?
Solution :
A N B v ælö æ 2p ö æ l ö
Doppler’s shift d l = ´ l = R wç ÷ = R ç ÷ç ÷
57º c c
è ø è T øècø
33º 33º 7 ´ 108 ´ 2 ´ 22 6000
O = ´ Å = 0.04 Å
25 ´ 24 ´ 60 ´ 60 ´ 7 3 ´ 108
Example 13 :
How far in advance can one detect two headlights of a car
(a) the intensity is reduced down to zero and remains if they are separated by a distance of 1.57 m ?
zero Solution :
(b) the intensity reduces down somewhat and rises again The human eye can resolve two objects when the angle
(c) there is no change in intensity between them is 1 minute of arc. Thus, we have
(d) the intensity gradually reduces to zero and then again x
D=

.IN
increases q
Solution : (d) 1 p
For complete polarisation of reflected light Here x = 1.57 m, q = 1¢ = ´ rad ,
60 180
AL
m = tan f (f = Brewster’s angle)
\ f = tan–1 m = tan–1 (1.54) = 57º 1.57 10800 ´ 1.57
Thus D = = = 5400 m = 5.4 km
1 p 3.14
N
From fig, angle of incidence = 90° – 33º = 57º ´
Hence the reflected light is completely polarised. When the 60 180
R

plane polarised light is viewed through a rotating nicol prism, Example 14 :


the intensity gradually reduces to zero and then again The numerical aperture of a microscope is 0.12, and the
U

increases. wavelength of light used is 600 nm. Then find its limit of
RESOLVING POWER OF AN OPTICAL INSTRUMENT resolution.
JO

The resolving power of an optical instrument, is its ability to Solution :


distinguish between two closely spaced objects. The limit of resolution of a microscope is given by
Diffraction occurs when light passes through the circular, or
U

nearly circular, openings that admit light into cameras, telescopes, 0.61 l
x=
microscopes, and human eyes. The resulting diffraction pattern m sin q
ED

places a natural limit on the resolving power of these instruments.


For example, for normal vision, the limit of resolution of normal It is given that l = 6 ´ 10 -7 m , and the numerical aperture
human eye is ~0.1 mm from 25 cm. (i.e., distances less than 0.1 mm m sin q = 0.12 .
cannot be resolved). For optical microscope the limit of resolution
~ 10–5 cm and for electron microscope ~5 Å or less. 0.61 ´ 6 ´ 10-7
Therefore, x = = 3.05 ´ 10-6 m » 3 µm
0.61l 0.12
The limit of resolution of a microscope x = where a is the
a Example 15 :
aperture of the microscope. A person wants to resolve two thin poles standing near
DOPPLER’S EFFECT FOR LIGHT WAVES each other at a distance of 1 km. What should be the
(a) When the source moves towards the stationary observer minimum separation between them?
or the observer moves towards the source, the apparent Solution :
frequency. Angular limit of resolution of eye q = 1 minute of arc
= 1/60 degree.
æ vö
n´= n ç1 + ÷ ( Blue shift ) Therefore, the minimum separation should be such that
è cø x = Dq
(b) When the source moves away from the stationary observer 1 p
with D = 1 km. = 103 m and q = ´ radian
æ vö 60 180
or vice-versa, n´= n ç1 - ÷ (Red shift)
è cø
10 3 ´ 3.14 31.4
where n´ = apparent frequency, n = active frequency Thus x = = = 0.29 m
60 ´ 180 108
v = velocity of source, c = velocity of light
or x » 30 cm.
But in both cases, the relative velocity v is small.
CONCEPT MAP

Huygens’ principle Constructive interference Destructive interference


Wave Optics

Cylindrical wavefront
Linear light source Each point on the Phase diff. d= 2n p Phase diff. d= (2n – 1) p
Effective distance -finite primary wavefront x =2n (/2)
Path diff. D l
1 is the source of a Path diff. D
x = (2n – 1)/2l
Intensity I µ Resultant amplitude A = a1 + a 2
r secondary wavelets Resultant amplitude A = a1 – a 2
Resultant intensity I = 2
1 Resultant intensity I = ( I1 – I2 )
Amplitude A µ ( I1 + I 2 )2
r

Plane wavefront Doppler effect


Light source at large in light Interference
ED
Wavefront of light Redistribution
distance: Effective Forms of Dv vradial
Locus of all particles = of energy due to super
distance infinite wavefront
U
vibrating in same phase v c position of waves
Intensity and
amplitude independent
of distance
JO
Spherical wavefront Superposition of waves. Coherent sources of light
Point light source WAVE OPTICS
U Sources of light, emitting
Describes the connection When two similar waves
Effective distance-finite propagate simultaneously light of same wavelength
1 between waves and R same frequency having
Intensity I µ rays of light then resultant
r2 N displacement y = y1+ y 2 a zero or constant phase
Brewster’s law 1 difference.
Amplitude A µ
µ = tan qp r
qp = angle of polarisation Diffraction
AL
Bending of light waves Young’s double slit
around the corners of an Experiment YDSE
obstacle
.IN
Polarisation
Restricting the vibration of
light in a particular direction Linear width of
perpendicular to the direction central maximum
of propagation of wave 2Dl l Distance between Fringe width ()b
= 2+ Distance between central fringe and
a a central fringe and Distance between
th
Law of Malus th n dark fringe centres of two
n bright fringe consecutive bright
I = Icos 2q (2n - 1)lD
0 nlD Xn =
Xn = or dark fringes
I = intensity of For secondary maximum For secondary minimum 2d
d D
transmitted light (2n + 1)l path difference = nl D = distance between b= l
Path diff. = d
from analyser 2 nDl source and screen
Linear distance Linear distance = d = distance between
(2n + 1)Dl a
two slits
671

=
2a
EBD_7179
672 PHYSICS

1. Which one of the following phenomena is not explained by 13. The phenomenon of interference is shown by
Huygens construction of wavefront? (a) longitudinal mechanical waves only
(a) Refraction (b) Reflection (b) transverse mechanical waves only
(c) Diffraction (d) Origin of spectra (c) non-mechanical transverse waves only
2. Which of the following phenomena is not common to sound (d) All of the above
and light waves ? 14. The transverse nature of light is shown by
(a) Interference (b) Diffraction (a) interference of light (b) refraction of light
(c) Coherence (d) Polarisation (c) polarization of light (d) dispersion of light
3. Interference is possible in 15. If the intensities of the two interfering beams in Young’s
(a) light waves only double-slit experiment are I1 and I2, then the contrast
(b) sound waves only between the maximum and minimum intensities is good
(c) both light and sound waves when
(d) neither light nor sound waves (a) | I1 and I2 | is large (b) | I1 and I2 | is small
4. A single slit diffraction pattern is obtained using a beam of (c) either I1 or I2 is zero (d) I1 = I2
red light. If the red light is replaced by the blue light, then 16. The idea of the quantum nature of light has emerged in an

.IN
attempt to explain
the diffraction pattern
(a) interference
(a) remains unchanged (b) becomes narrower
(b) diffraction
(c) becomes broader (d) will disappear
AL (b) polarization
5. In Young's double slit experiment, if the slit widths are in the (d) radiation spectrum of a black body
ratio 1 : 2, the ratio of the intensities at minima and maxima 17.. The contrast in the fringes in an interference pattern
will be
N
depends on
(a) 1 : 2 (b) 1 : 3 (c) 1 : 4 (d) 1 : 9 (a) fringe width
R

6. If a wave can be polarized, it must be (b) wavelength


(a) a transverse wave (b) a longitudinal wave (c) intensity ratio of the sources
U

(c) a sound wave (d) a stationary wave (d) distance between the slits
7. To demonstrate the phenomenon of interference, we require 18. Polarisation of light establishes
JO

two sources which emit radiation of (a) corpuscular theory of light


(a) nearly the same frequency (b) quantum nature of light
(b) the same frequency (c) transverse nature of light
U

(c) different wavelengths (d) all of the three


(d) the same frequency and having a definite phase 19. Huygens concept of wavelets is useful in
ED

relationship (a) explaining polarisation


8. Angular width (B) of central maxima of a diffraction pattern (b) determining focal length of the lenses
of a single slit does not depend upon (c) determining chromatic aberration
(a) distance between slit and source (d) geometrical reconstruction of a wavefront
(b) wavelength of the light used 20. When a compact disc is illuminated by small source of white
(c) width of slit light, coloured bands are observed. This is due to
(a) dispersion (b) diffraction
(d) frequency of light used
(c) interference (d) reflection
9. The phenomenon by which stars recedes from each other
21. A nicol prism is based on the action of
is explained by (a) refraction (b) double refraction
(a) black hole theory (b) neutron star theory (c) dichroism (d) both (b) and (c)
(c) white dwarf (d) red shift 22. The deflection of light in a gravitational field was predicted
10. Which of the following does not support the wave nature first by
of light? (a) Einstein (b) Newton
(a) Interference (b) Diffraction (c) Max Planck (d) Maxwell
(c) Polarisation (d) Photoelectric effect. 23. When light passing through rotating nicol is observed,
11. The colours seen in the reflected white light from a thin oil no change in intensity is seen. What inference can be
film are due to drawn ?
(a) diffraction (b) interference (a) The incident light is unpolarized.
(c) polarisation (d) dispersion (b) The incident light is circularly polarized.
12. Which of the following cannot be polarised ? (c) The incident light is unpolarized or circularly
(a) Radio waves (b) b rays polarized.
(d) The incident light is unpolarized or circularly polarized
(c) Infrared rays (d) g rays or combination of both.
Wave Optics 673

24. In refraction, light waves are bent on passing from one 25. Interference was observed in an interference chamber when
medium to the second medium, because, in the second air was present. Now, the chamber is evacuated and if the
medium same light is used, a careful observation will show
(a) the frequency is different (a) no interference
(b) the coefficient of elasticity is different (b) interference with bright bands
(c) the speed is different (c) interference with dark bands
(d) the amplitude is smaller (d) interference in which breadth of the fringe will be
slightly increased

1. The width of a slit is 0.012 mm. Monochromatic light is 7. In Young’s double slit expt. the distance between two
incident on it. The angular position of first bright line is sources is 0.1 mm. The distance of the screen from the
5.2º. The wavelength of incident light is source is 20 cm. Wavelength of light used is 5460 Å. The
[sin 5.2º = 0.0906]. angular position of the first dark fringe is
(a) 6040 Å (b) 4026 Å (a) 0.08º (b) 0.16º
(c) 5890 Å (d) 7248 Å (c) 0.20º (d) 0.32º

.IN
2. A ray of light is incident on the surface of a glass plate at an 8. The separation between successive fringes in a double slit
angle of incidence equal to Brewster’s angel f. If m represents arrangement is x. If the whole arrangement is dipped under
the refractive index of glass with respect to air, then the angle AL water what will be the new fringe separation? [The
between the reflected and the refracted rays is wavelenght of light being used is 5000 Å]
(a) 90° + f (b) sin–1(m cos f) (a) 1.5 x (b) x
(c) 0.75 x (d) 2 x
æ sin f ö
(d) 90 º - sin -1 çç
N
(c) 90º ÷÷ 9. Light of wavelength 6328 Å is incident normally on a slit
è m ø having a width of 0.2 mm. The angular width of the central
R

3. Light of wavelength 6.5 × 10–7 m is made incident on two maximum measured from minimum to minimum of diffraction
slits 1 mm apart. The distance between third dark fringe and pattern on a screen 9.0 metres away will be about
U

fifth bright fringe on a screen distant 1 m from the slits will (a) 0.36 degree (b) 0.18 degree
JO

be (c) 0.72 degree (d) 0.09 degree


(a) 0.325 mm (b) 0.65 mm 10. A plane wave of wavelength 6250 Å is incident normally on
(c) 1.625 mm (d) 3.25 mm a slit of width 2 × 10–2 cm. The width of the principal maximum
U

4. The max. intensity produced by two coherent sources of on a screen distant 50 cm will be
intensity I1 and I2 will be (a) 312.5 × 10–3 cm (b) 312.5 × 10–3 m
ED

(c) 312.5 × 10 m –3 (d) 312 m


(a) I1 + I 2 (b) I12 + I 22 11. A ray of light strikes a glass plate at an angle of 60º. If the
reflected and refracted rays are perpendicular to each other,
(c) I1 + I 2 + 2 I1 I 2 (d) zero
the index of refraction of glass is
5. The path difference between two wavefronts emitted by
coherent sources of wavelength 5460Å is 2.1 micron. The 1 3 3
(a) (b) (c) (d) 1.732
phase difference between the wavefronts at that point is 2 2 2
(a) 7.692 (b) 7.692 p 12. The wavelength of Ha line in hydrogen spectrum was found
7.692 7.692 to be 6563 Å in the laboratory. If the wavelength of same
(c) (d) line in the spectrum of a milky way is observed to be 6568Å,
p 3p
d
then recession velocity of milky way will be
6. In Young’s expt., the distance between two slits is and (a) 105 m/s (b) 1.05 × 106 m/s
3 6
(c) 10.5 × 10 m/s (d) 0.105 × 106 m/s
the distance between the screen and the slits is 3 D. The 13. A star is receding away from earth with a velocity of
1 105 m/s. If the wavelength of its spectral line is 5700 Å, then
number of fringes in m on the screen, formed by
3 Doppler shift will be
monochromatic light of wavelength 3l, will be (a) 200 Å (b) 1.9 Å
(c) 20 Å (d) 0.2 Å
d d
(a) (b) 14. A slit of width a is illuminated by red light of wavelength
9Dl 27 D l 6500 Å. If the first minimum falls at q = 30°, the value of a is
(a) 6.5 × 10–4 mm (b) 1.3 micron
d d
(c) (d) (c) 3250 Å (d) 2.6 × 10–4 cm
81 D l Dl
EBD_7179
674 PHYSICS

15. Two beams of light of intensity I1 and I2 interfere to give 24. Two sources of light of wavelengths 2500 Å and 3500 Å are
an interference pattern. If the ratio of maximum intensity to used in Young’s double slit expt. simultaneously. Which
that of minimum intensity is 25/9, then I1/I2 is orders of fringes of two wavelength patterns coincide?
(a) 5/3 (b) 4 (a) 3rd order of 1st source and 5th of the 2nd
(c) 81/625 (d) 16 (b) 7th order of 1st and 5th order of 2nd
(c) 5th order of 1st and 3rd order of 2nd
16. The condition for obtaining secondary maxima in the
(d) 5th order of 1st and 7th order of 2nd
diffraction pattern due to single slit is
25. A radar sends radiowaves of frequency v towards an
l aeroplane moving with velocity va. A change Dn is observed
(a) a sin q = nl (b) a sin q = ( 2n - 1) in the frequency of reflected waves which is higher than
2
original frequency. The velocity of aeroplane is (va << c)
nl c Dn Dn
(c) a sin q = ( 2n - 1) l (d) a sin q = c Dn 2 c Dn
2 (a) (b) (c) (d)
n Dn 2n 2cn
17. Light from two coherent sources of the same amplitude A
26. In Young’s double slit experiment, we get 10 fringes in the
and wavelength l illuminates the screen. The intensity of
field of view of monochromatic light of wavelength 4000Å.
the central maximum is I0. If the sources were incoherent, If we use monochromatic light of wavelength 5000Å, then
the intensity at the same point will be the number of fringes obtained in the same field of view is
(a) 4I0 (b) 1I0 (c) I0 (d) I0/2 (a) 8 (b) 10 (c) 40 (d) 50

.IN
18. In Young's double slit experiment with sodium vapour lamp 27. With a monochromatic light, the fringe-width obtained in a
of wavelength 589 nm and the slits 0.589 mm apart, the half Young’s double slit experiment is 0.133 cm. The whole set-
angular width of the central maximum is
AL up is immersed in water of refractive index 1.33, then the
(a) sin–1 (0.01) (b) sin –1 (0.0001) new fringe-width is
(c) sin –1 (0.001) (d) sin–1 (0.1) (a) 0.133 cm (b) 0.1 cm
19. In Young's double slit experiment with sodium vapour lamp 1.33
N
(c) 1.33 × 1.33 cm (d) cm
of wavelength 589 nm and the slits 0.589 mm apart, the half 2
R

angular width of the central maximum is 28. A slit of width a is illuminated by white light. The first
(a) sin –1 0.01 (b) sin–10.0001 minimum for red light (l = 6500 Å) will fall at q = 30º when a
U

–1
(c) sin 0.001 (d) sin–1 0.1 will be
(b) 6.5 × 10–4 cm
JO

20. When the light is incident at the polarizing angle on the (a) 3250 Å
transparent medium, then the completely polarized light is (c) 1.3 micron (d) 2.6 × 10–4 cm
(a) refracted light 29. The Fraunhoffer ‘diffraction’ pattern of a single slit is formed
U

(b) reflected light in the focal plane of a lens of focal length 1 m. The width of
(c) refracted and reflected light slit is 0.3 mm. If third minimum is formed at a distance of 5
ED

mm from central maximum, then wavelength of light will be


(d) neither reflected nor refracted light
(a) 5000 Å (b) 2500 Å
21. In the phenomena of diffraction of light, when blue light is
(c) 7500 Å (d) 8500 Å
used in the experiment in spite of red light, then 30. Two points separated by a distance of 0.1 mm can just be
(a) fringes will become narrower inspected in a microscope, when light of wavelength 600Å
(b) fringes will become broader is used. If the light of wavelength 4800 Å is used, the limit
(c) no change in fringe width of resolution will become
(d) None of these (a) 0.80mm (b) 0.12 mm (c) 0.10 mm (d) 0.08 mm
22. The wavefronts of a light wave travelling in vacuum are 31. Unpolarised light of intensity 32 W m–2 passes through
given by x + y + z = c. The angle made by the direction of three polarizers such that the transmission axis of the last
propagation of light with the X-axis is polarizer is crossed with that of the first. The intensity of
(a) 0º (b) 45º final emerging light is 3 W m–2. The intensity of light
transmitted by first polarizer will be
(c) 90º (d) cos –1 (1/ 3) (a) 32 W m–2 (b) 16 W m–2
(c) 8 W m –2 (d) 4 W m–2
23. In Fresnel’s biprism expt., a mica sheet of refractive index
32. A parallel beam of monochromatic unpolarised light is
1.5 and thickness 6 × 10–6 m is placed in the path of one of
incident on a transparent dielectric plate of refractive index
interfering beams as a result of which the central fringe
gets shifted through 5 fringe widths. The wavelength of 1
. The reflected beam is completely polarised. Then the
light used is 3
(a) 6000 Å (b) 8000 Å angle of incidence is
(c) 4000 Å (d) 2000 Å (a) 30º (b) 60º
(c) 45º (d) 75º
Wave Optics 675

33. Two nicols are oriented with their principal planes making (a) 4400 Å (b) 4100 Å
an angle of 60º. Then the percentage of incident unpolarised (c) 4642.8 Å (d) 9100 Å
light which passes through the system is 43. When the angle of incidence is 60° on the surface of a
(a) 100 (b) 50 (c) 12.5 (d) 37.5 glass slab, it is found that the reflected ray is completely
34. A beam of unpolarised light passes through a tourmaline polarised. The velocity of light in glass is
crystal A and then through another such crystal B oriented
so that the principal plane is parallel to A. The intensity of
(a) 2 ´ 108 ms -1 (b) 3 ´ 108 ms -1
emergent light is I0. Now B is rotated by 45º about the ray. (c) 2 ´ 108 ms -1 (d) 3 ´ 108 ms -1
The emergent light will have intensity 44. A lens having focal length f and aperture of diameter d
(a) I 0 / 2 (b) I 0 / 2 (c) I 0 2 (d) 2I 0 d
forms an image of intensity I. Aperture of diameter in
35. A rocket is receding away from earth with velocity 0.2 c. 2
The rocket emits signal of frequency 4 × 107 Hz. The central region of lens is covered by a black paper. Focal
apparent frequency observed by the observer on earth will length of lens and intensity of image now will be
be respectively
(a) 4 × 106 Hz (b) 3.2 × 107 Hz I 3f I
6
(c) 3 × 10 Hz (d) 5 × 107 Hz (a) f and (b) and
4 4 2
36. The heavenly body is receding from earth, such that the
fractional change in l is 1, then its velocity is 3I f I
(c) f and (d) and
4 2 2
3c c 2c

.IN
(a) c (b) (c) (d) 45. In Young’s double slit experiment, the slits are 2 mm apart and
5 5 5 are illuminated by photons of two wavelengths l1 = 12000Å
37. Fluorescent tubes give more light than a filament bulb of AL and l2 = 10000Å. At what minimum distance from the
same power because common central bright fringe on the screen 2 m from the slit
(a) the tube contains gas at low temperature will a bright fringe from one interference pattern coincide
(b) ultraviolet light is converted into visible light by with a bright fringe from the other ?
N
fluorescence (a) 6 mm (b) 4 mm (c) 3 mm (d) 8mm
(c) light is diffused through the walls of the tube 46. A parallel beam of fast moving electrons is incident normally
R

(d) it produces more heat than bulb


on a narrow slit. A fluorescent screen is placed at a large
38. In young’s double-slit experiment, the intensity of light at a
distance from the slit. If the speed of the electrons is
U

point on the screen where the path difference is l is I, l increased, which of the following statements is correct ?
being the wavelength of light used. The intensity at a point
JO

(a) The angular width of the central maximum of the


l diffraction pattern will increase.
where the path difference is will be
4 (b) The angular width of the central maximum will decrease.
I I
U

(a) (b) (c) I (d) zero (c) The angular width of the central maximum will be
4 2 unaffected.
ED

39. Aperture of the human eye is 2 mm. Assuming the mean (d) Diffraction pattern is not observed on the screen in
wavelength of light to be 5000 Å, the angular resolution case of electrons.
limit of the eye is nearly DIRECTIONS for Qs. (47 to 50) : Each question contains
(a) 2 minute (b) 1 minute STATEMENT-1 and STATEMENT-2. Choose the correct answer
(c) 0.5 minute (d) 1.5 minute (ONLY ONE option is correct ) from the following-
40. If the polarizing angle of a piece of glass for green light is (a) Statement -1 is false, Statement-2 is true
54.74°, then the angle of minimum deviation for an
(b) Statement -1 is true, Statement-2 is true; Statement -2 is a
equilateral prism made of same glass is
correct explanation for Statement-1
[Given, tan 54.74° = 1.414]
(c) Statement -1 is true, Statement-2 is true; Statement -2 is not
(a) 45° (b) 54.74°
a correct explanation for Statement-1
(c) 60° (d) 30°
(d) Statement -1 is true, Statement-2 is false
41. In Young's double slit experiment, the fringes are displaced
by a distance x when a glass plate of refractive index 1.5 is 47. Statement 1 : In YDSE, if a thin film is introduced in front of
introduced in the path of one of the beams. When this the upper slit, then the fringe pattern shifts in the downward
plate is replaced by another plate of the same thickness, direction.
the shift of fringes is (3/2) x. The refractive index of the Statement 2 : In YDSE if the slit widths are unequal, the
second plate is minima will be completely dark.
(a) 1.75 (b) 1.50 Imax
(c) 1.25 (d) 1.00 48. Statement 1 : In YDSE, if I1 = 9I0 and I2 = 4I0 then = 25.
I min
42. A single slit Fraunhoffer diffraction pattern is formed with
white light. For what wavelength of light the third Statement 2 : In YDSE Imax = ( I1 + I 2 ) 2 and
secondary maximum in the diffraction pattern coincides
with the second secondary maximum in the pattern for red I min = ( I1 - I 2 ) 2 .
light of wavelength 6500 Å?
EBD_7179
676 PHYSICS

49. Statement 1 : In Young’s double slit experiment if 50. Statement 1 : In YDSE number of bright fringe or dark fringe
wavelength of incident monochromatic light is just doubled, can not be unlimited
number of bright fringe on the screen will increase. Statement 2 : In YDSE path difference between the
Statement 2: Maximum number of bright lunge on the screen superposing waves can not be more than the distance
is inversely proportional to the wavelength of light used between the slits.

Exemplar Questions 4pd æ


12
p
1 2 ö
(c) ç1 - sin q ÷ +
1. Consider a light beam incident from air to a glass slab at l è n2 ø 2
Brewster’s angle as shown in figure. 12
A polaroid is placed in the path of the emergent ray at point 4pd æ 1 2 ö
(d) ç1 - sin q ÷ + 2p
P and rotated about an axis passing through the centre and l è n2 ø
perpendicular to the plane of the polaroid. 4. In a Young’s double-slit experiment, the source is white
light. One of the holes is covered by a red filter and another

.IN
by a blue filter. In this case,
(a) there shall be alternate interference patterns of red and
AL blue
(b) there shall be an interference pattern for red distinct
from that for blue
(c) there shall be no interference fringes
N
(d) there shall be an interference pattern for red mixing with
one for blue
R

P 5. Figure shows a standard two slit arrangement witn slits S1,


S2, P1, P2 are the two minima points on either side of P (figure).
U

(a) For a particular orientation, there shall be darkness as


JO

observed through the polaroid Screen


(b) The intensity of light as seen through the polaroid shall S1 P1
be independent of the rotation
U

(c) The intensity of light as seen through the polaroid shall S


P
go through a minimum but not zero for two orientations S3
ED

of the polaroid S2 P2 S4 Second


(d) The intensity of light as seen through the polaroid shall screen
go through a minimum for four orientations of the At P2 on the screen, there is a hole and behind P2 is a
polaroid second 2-slit arrangement with slits S3, S4 and a second
2. Consider sunlight incident on a slit of width 104 Å. The screen behind them.
image seen through the slit shall (a) There would be no interference pattern on the second
(a) be a fine sharp slit white in colour at the centre screen but it would be lighted
(b) a bright slit white at the centre diffusing to zero (b) The second screen would be totally dark
intensities at the edges (c) There would be a single bright point on the second
(c) a bright slit white at the centre diffusing to regions of screen
different colours (d) There would be a regular two slit pattern on the second
(d) only be a diffused slit white in colour screen
3. Consider a ray of light incident from air onto a slab of glass
NEET/AIPMT (2013-2017) Questions
(refractive index n) of width d, at an angle q. The phase
difference between the ray reflected bv the top surface of 6. In Young’s double slit experiment, the slits are 2 mm apart and
the glass and the bottom surface is are illuminated by photons of two wavelengths l1 = 12000Å
and l2 = 10000Å. At what minimum distance from the com-
12
2pd æ 1 2 ö mon central bright fringe on the screen 2 m from the slit will
(a) ç1 - 2 sin q ÷ +p a bright fringe from one interference pattern coincide with a
l è n ø
bright fringe from the other ? [2013]
4 pd æ 1 2 ö
12
(a) 6 mm (b) 4mm
(b) ç1 - sin q ÷ (c) 3 mm (d) 8mm
l è n2 ø
Wave Optics 677

7. A parallel beam of fast moving electrons is incident nor- 14. At the first minimum adjacent to the central maximum of a
mally on a narrow slit. A fluorescent screen is placed at a single-slit diffraction pattern, the phase difference between
large distance from the slit. If the speed of the electrons is the Huygen's wavelet from the edge of the slit and the
increased, which of the following statements is correct ? wavelet from the midpoint of the slit is : [2015 RS]
[2013] p
(a) The angular width of the central maximum of the dif- (a) radian (b) p radian
2
fraction pattern will increase. p p
(b) The angular width of the central maximum will decrease. (c) radian (d) radian
8 4
(c) The angular width of the central maximum will be un- 15. Two slits in Young’s experiment have widths in the ratio 1 :
affected. 25. The ratio of intensity at the maxima and minima in the
(d) Diffraction pattern is not observed on the screen in
case of electrons. Imax
interference pattern, I is: [2015 RS]
8. In Young’s double slit experiment the distance between the min
slits and the screen is doubled. The separation between 121 49
the slits is reduced to half. As a result the fringe width (a) (b)
49 121
(a) is doubled [NEET Kar. 2013]
4 9
(b) is halved (c) (d)
9 4
(c) becomes four times 16. In a diffraction pattern due to a single slit of width 'a', the
(d) remains unchanged first minimum is observed at an angle 30° when light of

.IN
9. A parallel beam of light of wavelength l is incident normally wavelength 5000 Å is incident on the slit. The first
on a narrow slit. A diffraction pattern is formed on a screen secondary maximum is observed at an angle of : [2016]
placed perpendicular to the direction of the incident beam.
AL æ 1ö -1 æ 2 ö
At the second minimum of the diffraction pattern, the phase (a) sin -1 ç ÷ (b) sin çè ÷ø
difference between the rays coming from the two edges of è 4ø 3
slit is [NEET Kar. 2013]
N
æ 1ö -1 æ 3 ö
(a) pl (b) 2p (c) sin -1 ç ÷ (d) sin çè ÷ø
è 2ø 4
R

(c) 3p (d) 4p
10. A beam of light of l = 600 nm from a distant source falls on 17. The intensity at the maximum in a Young's double slit
U

a single slit 1 mm wide and the resulting diffraction pattern experiment is I0. Distance between two slits is d = 5l, where
l is the wavelength of light used in the experiment. What
JO

is observed on a screen 2 m away. The distance between


first dark fringes on either side of the central bright fringe will be the intensity in front of one of the slits on the screen
is: [2014] placed at a distance D = 10 d ? [2016]
U

(a) 1.2 cm (b) 1.2 mm I0


(c) 2.4 cm (d) 2.4 mm (a) I0 (b)
4
ED

11. In the Young’s double-slit experiment, the intensity of light 3 I0


at a point on the screen where the path difference is l is K, (c) I0 (d)
4 2
(l being the wave length of light used). The intensity at a
18. The ratio of resolving powers of an optical microscope for
point where the path difference is l/4, will be: [2014] two wavelengths l1 = 4000 Å and l2 = 6000 Å is [2017]
(a) K (b) K/4 (a) 9 : 4 (b) 3 : 2
(c) K/2 (d) Zero (c) 16 : 81 (d) 8 : 27
12. In a double slit experiment, the two slits are 1 mm apart and 19. Young's double slit experment is first performed in air and
the screen is placed 1 m away. A monochromatic light then in a medium other than air. It is found that 8th bright
wavelength 500 nm is used. What will be the width of each fringe in the medium lies where 5th dark fringe lies in air.
slit for obtaining ten maxima of double slit within the central The refractive index of the medium is nearly [2017]
maxima of single slit pattern ? [2015] (a) 1.59 (b) 1.69
(a) 0.1 mm (b) 0.5 mm (c) 1.78 (d) 1.25
(c) 0.02 mm (d) 0.2 mm 20. Two Polaroids P1 and P2 are placed with their axis
13. For a parallel beam of monochromatic light of wavelength perpendicular to each other. Unpolarised light I0 is incident
'l', diffraction is produced by a single slit whose width 'a' is on P1. A third polaroid P3 is kept in between P1 and P2 such
of the wavelength of the light. If 'D' is the distance of the that its axis makes an angle 45° with that of P1 . The intensity
screen from the slit, the width of the central maxima will be: of transmitted light through P2 is [2017]
Dl Da I0 I0
(a) (b) [2015] (a) (b)
a l 4 8
2Da 2Dl I0 I0
(c) (d) (c) (d)
l a 16 2
EBD_7179
678 PHYSICS

Hints & Solutions


EXERCISE - 1 EXERCISE - 2
1. (d) 2. (d) 3. (c) 4. (b) 1. (d) It is a one of Fraunhoffer diffraction from single slit.
5. (d) 6. (a) 7. (d) so for bright fringe where a is the width of slit.
8. (a) For single slit diffraction pattern e sin q = l Angular l
a sin q = (2 n + 1)
width, e = slit width 2

ælö 2 a sin q 2 ´1.2 ´10 -5 ´ 0.0906


\ q = sin -1 ç ÷ l= =
2 n +1 2 ´1 + 1
èeø
It is independent of D, distance between screen and = 7248 ´10 -10 m = 7248Å.
slit. 2. (c) Q i p = f, therefore, angle between reflected and
9. (d) Doppler effect in light explains the phenomenon of refracted rays is 90º.
receding of stars and approaching of star by red shift
and blue shift respectively. D 5 ´ 6.5 ´ 10 -7 ´ 1
3. (c) x5 = n l = = 32.5 ´ 10 -4 m

.IN
10. (d) Photoelectric effect does not support the wave nature d 10 -3
of light.
11. (b) 1 Dl 5 ´ 6.5 ´10 -7 ´1
x 3 = (2 n - 1) =
AL 2 d 2 ´ 10 -3
12. (b) Longitudinal waves cannot be polarised.
13. (d) = 16.25 ´ 10 -4 m
N
14. (c) 15. (d) 16. (d)
17. (c) 18. (c) 19. (d) x 5 - x 3 = 16.25 ´ 10-4 m = 1.625 mm.
R

20. (b) The line rulings, each of 0.5 m m width,on a compact 4. (c) As R2 = a2 + b2 + 2 ab cos f
U

disc function as a diffraction grating. When a small \ I max = I1 + I 2 + 2 I1 I 2 cos 0º


source of light illuminates a disc, diffracted light forms
JO

coloured ‘lanes’ that are the composite of the = I1 + I 2 + 2 I1 I 2


diffraction patterns from the ruling.
21. (d) When a ray of light enters nicol prism, it splits into 2p
U

5. (b) Phase diff. = x


two plane polarised light in mutually perpendicular l
ED

direction. One of this light undergoes total reflection


and absorption whereas other comes out as a plane 2 p ´ 2.1´10 -6
Path difference = = 7.692 p radian.
polarised light. 5460 ´ 10 -10
22. (b) Newton first predicted deflection of light by
gravitational field. l¢ D¢ 3 l 3 D lD
6. (c) b= = = 27 .
d ¢ d/3 d
23. (c)
24. (c) Speed of light is different in different media and different No. of fringes = 1 / 3 = d .
medium has different refractive index. b 81l D
Speed of light in medium 1 7. (b) The position of n th dark fringe. So position of first
1m =
2 Speed of light in medium 2 dark fringe in x1 = lD / 2d .
d = 20 cm, D = 0.1mm, l = 5460 Å, x1 = 0.16
D 8. (c) When the arrangement is dipped in water;
25. (d) As fringe width b = l
d
x 3
b¢ =b / m = = x = 0.75x
B C 4/3 4
m = A + 2 + 4 + ... 9. (a) The angular width of central maxi. is
l l
mVacuum < mAir so lVacuum > lAir l 2 ´ 6328 ´ 10 -10
2q = 2 = radian .
Therefore when chamber is evacuated fringe width b a 2 ´ 10 - 4
slightly increases.
180
= 6328 ´ 10-6 ´ degree = 0.36º
p
Wave Optics 679

10. (a) Width of central maximum


l 589 ´ 10-9 1
2 l D 2 ´ 6250 ´ 10 -10
´ 0.5 18. (c) sin q = = -3
= 10-3 = = 0.0001
= = d 0.589 ´ 10 1000
a - 4
2 ´ 10 19. (c) In Young's double slit experiment, half angular width
-6
= 3125 ´ 10 m = 312.5 ´ 10 cm. -3 (q) is given by
l
sin q =
d
y2sinq = 2l
a
y1sinq = l/a 589 ´ 10 –9
a q = = 10 –3
o sinq = 0 0.589 ´ 10 –3
Intensity y1sinq = – l/a \ q = sin –1 0.001
D y2sinq = – 2l 20. (b) When the light is incident at the polarising angle on
a
the transparent medium, the reflected light is
Screen position of various minima for Fraunhoffer completely polarised.
diffraction pattern of a single slit of width a. 21. (a)
r
11. (d) As reflected and refracted rays are perpendicular to 22. (d) Let any R its components are
each other, therefore, ip = i = 60º, where ip is called r r r r

.IN
angle of polarisation. R = Rx + Ry + Rz

m = tan ip = tan 60º = 3 = 1.732. r r r r


with | R |= R = | R |= R = Rx 2 + Ry 2 + Rz

AL
Dl ( 6586 - 6263)
12. (b) u= ´c = ´ 3 ´ 108 Rx Ry Rz
l 6563 & cosq x = , cosq y , cosq z =
R R R
N

= 1.05 ´ 106 m / s. there cosqx, cosqy and cosqz one called direction
R

cosines.
u 10 5 r
U

13. (b) Dl = ´l = ´ 5700 = 1.9 Å . Hence x + y + z = c (= R )


c 3 ´ 108
JO

14. (b) According to principle of diffraction, a sin q = nl 2 2 2


So, magnitude of c = I + I + I = 3
where, n = order of secondary minimum
or, a sin 30° = 1 × (6500 × 10–-10) 1
and cos q x =
U

or, a = 1.3 × 10–6 m, or, a = 1.3 micron. 3


ED

2 23. (a) Where n is equivalent number of fringe by which the


Imax 25 æa +a ö 25 centre fringe is shifted due to mica sheet
15. (d) = or ç 1 2 ÷ =
Imin 9 a -
è 1 2ø
a 9
(m - 1) t (1.5 - 1) 6 ´10 -6
where a denotes amplitude. l= =
n 5
a1 + a 2 5
= or 5a1 - 5a 2 = 3a1 + 3a 2 = 6 ´ 10 -7 m = 6000 Å
a1 - a 2 3
24. (b) Let nth fringe of 2500 Å coincide with (n – 2)th fringe
or, 5a1 – 5a2 = 3a1 + 3a2 or 2a1 = 8a2
of 3500Å.
a1 æa ö I1
2 \ 3500 (n – 2) = 2500 × n
or, = 4 or ç 1 ÷ = 16 = . 1000 n = 7000, n = 7
a2 è a2 ø I2
\ 7th order fringe of 1st source will coincide with 5th
16. (b) order fringe of 2nd source.
17. (d) For two coherent sources, I1 = I2 25. (c) In Radar, the source & receiver are together, the
receiver being turned for frequencies other than the
( )
2
Imax = (A1 + A2)2 = I1 + I 2 source or radar frequency.
This is given as I0 for maximum and zero for minimum. To measure the speed of helicopter
If there are two noncoherent sources, there will be no (i) The moving object of speed va receive a frequency
maximum and minimum intensities. Instead of all the æ v ö
intensity I0 at maximum and zero for minimum, it will n ' = nç 1 + a ÷
è c ø
be just I0/2.
EBD_7179
680 PHYSICS

(ii) The object which receives n’ frequency now acts as a


1
moving source. The detector observes a frequency but (cos2q)av =
n0 2
1 32
æ ö æ v ö So I1 = I0 = = 16Wm -2
ç ÷ ç1+ a ÷ 2 2
1 c
n 0 = n' ç ÷ =ç ÷ 32. (a) When angle of incidence i is equal to angle of
ç va ÷ ç va ÷
ç1- ÷ ç1- ÷ polarisation i.e, then reflected light is completely plane-
è c ø è c ø
polarised whose vibration is perpendicular to plane of
incidence.
va
Þ (n 0 - n ) = Dn = (n 0 + n ) 33. (c) Suppose intensity of unpolarised light = 100.
c
\ Intensity of polarised light from first nicol prism
Dnc æ \n0 »n ö Ι 1
or va = ç Þn +n»2n ÷ = 0 = ´ 100 = 50
2n è 0 ø 2 2
According to law of Malus,
26. (a) As b µ l
2
æ1ö
5 Ι = Ι0 cos 2 q = 50 (cos 60 º )2 = 50 ´ ç ÷ = 12.5
\ fringe width becomes times, è2ø

.IN
4
34. (a) According to law of Malus
4 2
No, of fringes = ´10 = 8 æ 1 ö Ι
5 Ι = Ι 0 cos 2 q = Ι 0 (cos 45º ) 2 = Ι 0 çç ÷÷ = 0
AL
è 2ø 2
b 0.133
27. (b) b¢ = = = 0.1 cm
m 1.33 u 0.2 c
N
35. (b) Dn = ´n = ´ 4 ´ 10 7 = 0.8 ´ 10 7 Hz
28. (c) The position of n th dark fringe in Fraunhoffer c c
R

Diffraction from a single slit is 7


n ¢ = n - Dn = 4 ´ 10 7 - 0.8 ´ 10 7 = 3.2 ´10 Hz.
a sin q = n l
U

36. (a) When planet or stars are receding away from earth. If
-7 f is frequency of vibration.
JO

n l 1´ 6.5 ´ 10
a= = , (for first min. n = 1)
sin q sin 30º c
Then , f =
l
U

6.5 ´10 -7 If v = velocity of body moving away


= = 13 ´10 -7 m = 1.3 m m.
ED

1/ 2 l' = apparent wavelength to an observer on the earth


29. (a) a sin q = nl
l' =
(c + v )
(c and v are in opposite to each other)
ax f
= 3l
f æc+vö æ vö
=ç ÷l l ' = ç1 + ÷ l
(since q is very small so sin q » tan q » q = x / f ) è c ø è cø

a x 0.3 ´10 -3 ´ 5 ´10 -3 l '- l v


=
or l = =
3f 3 ´1 l c
Fractional change in wavelength
= 5 ´ 10 -7 m = 5000 Å. v
\ 1=
Þ v =c
30. (d) Limit of resolution L.R µ l c
37. (b) The fluorescent material present in the tube converts
l¢ 4800
\ L.R ¢ = L.R ´ = 0.1 ´ = 0.08 mm. u.v. rays into visible light.
l 6000 38. (b) For path difference l, phase
\ Resolution improves when light of lower wavelength
is used. æ 2p 2p ö
difference = 2p ç Q = x = .l = 2p ÷
31. (b) Intensity of polarised light transmitted from 1st è l l ø
polariser, Þ I = I0 + I0 + 2I0 cos 2p
I1 = I0 cos2q Þ I = 4I0 (\ cos 2p = 1)
Wave Optics 681

l p 44. (c) By covering aperture, focal length does not change.


For x = , phase difference = 1
4 2 But intensity is reduced by times, as aperture
4
p d
\ I' = I1 + I 2 + 2 I1 I 2 cos diameter is covered.
2 2
I I I 3I
If I1 = I2 = I0 then I ' = 2I0 = 2. = \ I' = I - =
4 2 4 4
39. (b) If the angular limit of resolution of human eye is R 3I
then \ New focal length = f and intensity = .
4
1.22l 1.22 ´ 5 ´ 10 -7 nlD
R= = rad 45. (a) Q y =
a 2 ´ 10 -3 d
\ n1 l1 = n2l2
1.22 ´ 5 ´ 10 -7 180 Þ n1 × 12000 × 10–10 = n2 × 10000 × 10–10
= ´ ´ 60 mi nute = 1 minute
2 ´ 10 -3 p or, n (12000 × 10–10) = (n + 1) (10000 × 10–10)
40. (d) By principle of polarization, µ = tanqp Þn=5
or µ = tan 54.74° or µ = 1.414
(Q l 1 = 12000 ´ 10 -10 m; l 2 = 10000 ´ 10-10 m )

.IN
For an equilateral prism, ÐA = 60°

æ A + dö æ 60° + d ö
sin ç sin ç
è 2 ÷ø è 2 ÷ø
AL
\ m= =
sin ( A / 2) sin ( 60° / 2)
N
1.141 ´ 1 æ 60° + d ö é
or, = sin ç Q1.414 = 2 ùû
è 2 ÷ø ë
R

2
U

2 æ 60° + d ö 1 æ 60° + d ö
or, = sin ç or = sin ç
è 2 ÷ø è 2 ÷ø
JO

2 2
nl1D
Hence, ycommon =
æ 60° + d ö æ 60° + d ö d
or, sin 45° = sin çè ÷ø or 45° = çè ÷
U

2 2 ø
=
(
5 12000 ´ 10-10 ´ 2 ) (Q d = 2 mm and D = 2m )
ED

41. (a) -3
2 ´ 10
( 2n + 1) l D = 5 × 12 × 10–4 m
42. (c) x=
2a = 60 × 10–4 m
= 6 × 10–3m = 6 mm
( 4 + 1) D
For red light, x = ´ 6500Å
2a

( 6 + 1) D Y
For other light, x = ´ lÅ S1
2a
46. (b) d q
x is same for each. D
S2
5
\ 5 ´ 6500 = 7 ´ l Þ l = ´ 6500 = 4642.8 Å.
7
Y nlD é Dl ù
êQ Y = d ú
43. (b) aµ = tan q where q = polarising angle.
g P P Angular width, q = =
D dD ë û
or, aµg = tan 60°
l
c so, q = , v ­l ¯ q ¯ [For central maxima n = 1]
or, = 3 d
vg
Hence, with increase in speed of electrons angular
width of central maximum decreases.
c 3 ´ 108
or, vg = = = 3 ´ 108 ms -1 47. (a) 48. (b) 49. (a) 50. (b)
3 3
EBD_7179
682 PHYSICS

EXERCISE - 3 N
P P¢
Exemplar Questions q
q
1. (c) Let us consider the diagram shown below the light beam
O
incident from air to the glass slab at Brewster’s angle
(ip). The angle between reflected ray BE and BC is 90°. r r r¢ P¢ d
Then only reflected ray is plane polarised represented N¢
by arrows. q
P
As the emergent and incident ray are unpolarised, so,
polaroid rotated in the way of CD then the intensity nd
\ Dt =
cannot be zero but varies in one complete rotation. æ sin 2 q ö
12

N c ç1 - 2 ÷
Polarised è n ø
A E
iP -1 2
nd æ sin 2 q ö
= ç1 - 2 ÷ø
90º
c è n
B
2p
r Phase difference = Df = ´ Dt
T
C

.IN
-1 2
2pd æ sin 2 q ö
= 1 çè1 - ÷
n2 ø
Polaroid D × nl
n
(P)
AL
2. (a) As given that the width of the slit -1 2
2pd é sin 2 q ù
=104 Å = 10000 Å Df = 1- 2 ú
l êë
N
= 104×10–10 m = l0–6 m = 1 µm n û
Wavelength of visible sunlight varies from 4000 Å to \ Hence the net phase difference = Df + p
R

8000 Å. -1 2
2 pd æ 1 2 ö
Thus the width of slit is 10000 Å comparable to that of ç1 - sin q ÷ + p
U

=
wavelength visible light i.e., 8000 Å. So diffraction occurs l è n2 ø
JO

with maxima at centre. Hence at the centre all colours 4. (c) For sustained interference pattern to be formed on the
appear i.e., mixing of colours form white patch at the screen, the sources must be coherent and emits lights
centre. of same frequency and wavelength.
U

3. (a) Let, us consider the diagram, the ray (P) is incident at In a Young’s double-slit experiment, when one of the
an angle q and gets reflected in the direction P' and holes is covered by a red filter and another by a blue
ED

refracted in the direction P' through O'. Due to reflection filter. In this case due to filteration only red and blue
from the glass medium there is a phase change of p. lights are present which has different frequency. In this
The time difference between two refracted ray OP' and monochromatic light is used for the formation of fringes
O'P'' is equal to the time taken by ray to travel along on the screen. So, in that case there shall be no
OO'. interference fringes.
5. (d) Consider the given figure there is a hole at point P2. By
OO¢ d cos r nd
Dt = = = Huygen’s principle, wave will propagates from the
Vg cn c cos r sources S1 and S2. Each point on the screen will acts as
sin q sources of secondary wavelets.
From Snell's law,n = Wavefront starting from P2 reaches at S3 and S4 which
sin r
will again act as two monochromatic or coherent sources.
sin q Hence, there will be always a regular two slit pattern on
sin r =
n the second screen.
As we know that,
Past Years (2013-2017) NEET/AIPMT
cos r = 1 - sin 2 r ,
so by putting sin r value in that relation. nlD
6. (a) Q y =
So, cos r = 1 - sin r 2 d
\ n1 l1 = n2l2
sin 2 q Þ n1 × 12000 × 10–10 = n2 × 10000 × 10–10
cos r = 1 -
n2 or, n (12000 × 10–10) = (n + 1) (10000 × 10–10)
Þ n=5
Wave Optics 683

(Q l )
Width of central bright fringe (= 2b)
1 = 12000 ´ 10 -10 m; l 2 = 10000 ´ 10-10 m
2lD 2 ´ 600 ´ 10 - 6 ´ 2
= = m
d 1 ´ 10 - 3
= 2.4 × 10– 3 m = 2.4 mm
11. (c) For path difference l, phase difference = 2p rad.
l p
For path difference , phase difference = rad.
4 2
As K = 4I0 so intensity at given point where path
l
difference is
4
nl1D
Hence, ycommon =
d 2 æ pö æ p ö
K¢ = 4I0 cos ç ÷ ç cos = cos 45º÷
è 4ø è ø
( )
4
5 12000 ´ 10-10 ´ 2
= K
-3 = 2I0 =
2 ´ 10
2
(Q d = 2 mm and D = 2m ) 12. (d) Here, distance between two slits,

.IN
= 5 × 12 × 10–4 m d = 1mm = 10–3m
= 60 × 10–4 m distance of screen from slits, D = 1 m
wavelength of monochromatic light used,
= 6 × 10–3m = 6 mm
AL
l = 500nm = 500 × 10–9m
width of each slit a = ?
N
2lD
Width of central maxima in single slit pattern =
a
R

Y
lD
Fringe width in double slit experiment b =
U

d d
7. (b)
JO

10 l D 2 l D
So, required condition =
d a
U

d 1
Þ a= = ´ 10–3 m = 0.2 mm
Y nlD é Dl ù 5D 5
Q Y=
ED

Angular width, q = = ê
D dD ë d úû 13. (d) Linear width of central maxima y

l 2D l l
so, q = , v ­l ¯ q ¯ = D(2q) = 2Dq = \ q=
d a a
[For central maxima n = 1]
Hence, with increase in speed of electrons angular
width of central maximum decreases.
lD
8. (c) Fringe width b = ;
d
q y
d q
From question D¢ = 2D and d ¢ =
2
lD1
\b ' = = 4b D
d1
9. (d) Conditions for diffraction minima are
Path diff. Dx = nl and Phase diff. df = 2np 14. (b) For first minima at P
Path diff. = nl = 2l AP – BP = l
Phase diff. = 2np = 4p (Q n = 2) l
AP – MP =
10. (d) Given: D = 2m; d = 1 mm = 1 × 10– 3 m 2
l = 600 nm = 600 × 10– 6 m
EBD_7179
684 PHYSICS

Phase difference,
2p l p
P Df = ´ =
l 4 2
So, resultant intensity at the desired point 'p' is
f p I0
A f I = I0cos2 = I0cos2 4 = 2
2
M f O
2m sin q
18. (b) Resolving power of a microscope =
B l
1
2p l i.e., R µ
So phase difference, f = ´ = p radian l
l 2
R1 l 2
1 or, R = l
15. (d) The ratio of slits width = (given) 2 1
25
Given that the two wavelengths,
I1 25
\ l1 = 4000Å
I2 = 1
l 2 = 6000Å

.IN
I A 2 25 A 5
I µ A2 Þ 1 = 12 = or 1 = R1 6000Å 3
I2 A 2 1 A2 1 \ = =
AL R2 4000Å 2
A max A1 + A 2 5 + 1 6 3 19. (c) According to question
= = = = 8th bright fringe in medium = 5th dark fring in air
A min A1 - A 2 5 - 1 4 2
lD
N
2
Imax A max æ3ö 9
2 Y8th bright = 8
= 2 =ç ÷ = md
R

\
Imin A min è 2 ø 4
lD 9 lD
=
U

16. (d) For the first minima, Y5th dark = (2 × 5 – 1)


2d 2 d
hl l 1
JO

q= Þ sin30° = = 9 lD lD
a a 2 Þ =8
2 d md
First secondary maxima will be at
16
U

3l 3 æ 1 ö or, refractive index m = = 1.78


æ 3ö 9
sin q = = ç ÷ Þ q = sin–1 ç ÷
ED

2a 2 è 2 ø è 4ø
17. (d) Let P is a point infront of one slit at which intensity is
to be calculated. From figure,
20. (b)
S1 P

d O B

S2
D According to malus law, I = I0cos2 q
Path difference = S2P – S1P I0
I1 =
æ 1d ö 2 2
= D2 + d 2 - D = D ç1 + ÷-D
è 2 D2 ø I0 I0 1 I0
I2 = cos2 45° = ´ =
2 2 2 4
é d2 ù d2
= D ê1 + - 1ú = I0
ë 2D 2 û 2D I3 = cos2 45°
4
d2 d 5l l I0
Dx = = = = I3 =
2 ´10d 20 20 4 8
Dual Nature of
26 Radiation and Matter

CATHODE RAYS F vis co u s 1 = 6 r 1 –qE

Discharge Tube Experiments


When a very strong potential difference is applied across the two
n2

.IN
n1 E
electrodes in a discharge tube and the pressure of the air is lowered
gradually, then a stage is reached at which the current begins to
mg
flow through the air with cracking noise. The potential at which
this happens is called sparking potential.
AL mg

Fviscous 2 = 6 phrv2
N
Aston’s Cathode glow (a) (b)
dark space Faraday’s dark space Fviscous 1 = mg [from Stoke’s law Fviscous = 6phrv]
R

+ or 6p h rv1 = mg …(1)

where h is coefficient of viscocity of air, r is radius of drop and v1
U

is the terminal velocity of drop.


JO

Negative glow Positive striation In fig. (b) we consider a single drop of mass m, radius r carrying a
negative charge –q in the presence of electric field acting
· As pressure is lowered to 0.1 m.m. Hg – cathode glow, downward. Then by free body diagram (fig. (b)), we get
U

Crooke’s dark space, negative glow, Faraday’s dark space (-q )E = mg + Fviscous 2 = mg + 6phrv2 …(2)
and striations are observed.
ED

where v2 is the terminal speed in this case. Then from eqn (1), we
· At a pressure 0.01 m.m. Hg entire tube is dark (Crooke’s dark have.
space) except the glass wall behind anode. Colour is 6phr ( v1 + v 2 )
( -q ) = …(3)
yellowish-green for soda glass and greyish-blue for lead E
glass. and radius of drop from equation (1)
· The luminous streaks travelling from cathode to anode, 4 3 9hv1
6 phrv1 = mg = pr r g or r = …(4)
below the pressure 0.01 m.m. Hg, are called cathode rays. 3 2r g
Properties of Cathode Ray : where r is density of drop.
(i) Emitted perpendicularly to cathode, (ii) Travel in straight lines Millikan repeated these measurements on thousands of drops
and he found that the charge q calculated for each drop was some
(iii) Carry energy (iv) Possesses momentum (v) Deflected by
integral multiple of an elementary charge e. (e = 1.6 × 10–19C).
electric and magnetic fields (vi) Excite fluroescence (vii) Ionise
Hence, q = ne, n = 0, ±1, ±2 ...(5)
gas (viii) Produce highly penetrating secondary radiation when
This experiment gives the evidence of quantisation of charge.
suddenly stopped (ix) Effect photographic plate etc.
EMISSION OF ELECTRON
J.J. Thomson’s e/m value of electron :
Electrons from the metal surface are emitted by anyone of the
E2 æeö following physical processes :
e/m = = 1.76 ´ 1011 C kg –1 . This value of ç m ÷ is for
2VB 2 è ø (i) Thermionic emission : The emission of electrons by suitably
electron. heating the metal surface.
Millikan’s oil drop method for e/ m : (ii) Field emission : The emission of electrons by applying very
In fig.(a) we consider a single drop of mass m carrying a negative strong field of the order of 108 Vm–1 to a metal.
charge –q in the absence of electric field. Then
EBD_7179
686 PHYSICS

(iii) Photo-electric emission : The emission of electrons when (1) No photoelectrons are emitted, if the frequency of incident
light of suitable frequency illuminates metal surface. light is less than some cut-off frequency (i.e., threshold
PHOTOELECTRIC EFFECT (EINSTEIN’S PHOTOELECTRIC frequency) n0. It is inconsistent with the wave theory of
EQUATION light, which predicts that photoelectric effect occurs at any
In 19th century, experiments showed that when light is incident frequency provided intensity of incident light is sufficiently
on certain metallic surfaces, electrons are emitted from the high.
surfaces. This phenomenon is known as the photoelectric effect (2) The maximum kinetic energy of the photoelectrons is
& emitted electrons are called photoelectrons. The first independent of light intensiy, but increases with increasing
discovery of this phenomenon was made by Hertz. the frequency of incident light.
(3) Electrons are emitted from surface almost instantaneously
C
de Light (less than 10–9 sec after the surface illumination), even at
atho low intensity of incident light(classicaly we assume that the
C
C
Anode A electrons would require some time to absorb the incident
light before they acquire enough kinetic energy to escape
G from metal).
V These above points were explained by Einstein in 1905 by treating
the light as stream of particles.
Taking Max Planck assumptions, Einstein postulated that a beam
of light consists of small packets of energy called photons or

.IN
quanta. The energy E of a photon is equal to a constant h times
its frequency n
AL
i.e., hc ...(2)
Battery E = hn =
l
When light strikes the cathode C (metallic surface), photo
N
where h is a universal constant called Planck’s constant &
electrons are ejected. Electrons are collected at anode A,
constituting a current in the circuit. (Photoelectric effect) numerical value of h = 6.62607 × 10–34 J.s
R

Fig. shows, when light strikes the cathode C, electrons are emitted When a photon arrives at surface, it is absorbed by an electron.
& they are collected on anode A due to potential difference provided This energy transfer is an All-or-None process, in contrast to
U

by battery and constitutes the current in the circuit (observed by continuous transfer of energy in classical theory; the electrons
JO

Galvanometer G.) get all photon’s energy or none at all. If this energy is greater than
A plot of photoelecric current versus the potential difference V between the work function (f) of the metal (f is the minimum energy
cathode & anode is shown in fig below. required to remove the electron from metal surface), the electron
U

may escape from the surface. Greater intensity at a particular


frequency means greater number of photons per second absorbed
ED

& consequently greater number of electrons emitted per second


& so greater current.
1 2
To obtain maximum kinetic energy K max = mvmax
2
for an emitted electron, applying law of conservation of energy.
According to it
1 2
K max = mvmax = hn - f [f = hn0 ]
Photoelectric current versus voltage for two light intensities. 2
At a voltage less than –V0 the current is zero.
K max = h( n - n0 ) ...(3)
It is clear from fig. that photoelectric current increases as we
increase the intensity of light & obtain saturation value at larger or eV0 = Kmax = h( n - n0 ) ...(4)
value of potential difference V between cathode & anode. If V is
negative then, photoelectrons are repelled by negative cathode 1 2 æ1 1 ö
or, mvmax = h(v - v0 ) = hc ç - ÷ = evs
and only those electrons reaches anode, who have energy equal 2 è l l0 ø
to or greater than eV. But if V is equal to V0, called stopping
potential (i.e., cut off. potential), no electrons will reach the anode This is the Einstein’s photoelectric equation.
i.e., Maximum kinetic energy of electron = eV0 where V0 = cut-off potential
or Kmax = eV0 ...(1) nmax = maximum velocity obtained by photoelectrons
–19
where e is charge of electron (e = 1.6 × 10 coulomb). n = frequency of incident light i.e., photon
But some features of photoelectric effect cannot be explained by n0 = cut off frequency or threshold frequency.
classical physics & the wave theory of light.
Dual Nature of Radiation and Matter 687

n0 is different for different metallic surfaces. For most metals the Keep in Memory
threshold frequency is in ultarviolet region of spectrum.
(Corresponding to l between 200 & 300 nm), but for potassium & 1. Mass spectrograph is an appratus used to determine the
cesium oxides, it is in the visible spectrum (l between 400 & 700 nm). mass or the specific charge (e/m) of positive ions. Examples
are (a) Thomson mass spectrograph (b) Bain bridge mass
Work Function (f) of Some Elements Given in Brackets :
spectrograph (c) Aston mass spectrograph (d) Dempster
Al (4.3eV) Ni (5.1 eV) mass spectrograph etc.
C (5.0 eV) Si (4.8 eV) 2. In photoelectric effect all photoelectrons do not have same
Cu (4.7 eV) Ag (4.3 eV) kinetic energy. Their KE ranges from zero to E max which
Au (5.1eV) Na (2.7 eV) depends on frequency of incident radiation and nature of
cathode.
where 1 eV = 1.602 × 10–19 joule. 3. The photoelectric effect takes place only when photons
Within the framework of photon theory of light (Quantum theory strike bound electrons because for free electrons energy
of light) we can explain above failures of classical physics. and momentum conservations do not hold together.
(1) It is clear from eq. (3) that if energy of photon is less than 4. Cesium is the best photo sensitive material.
the work function of metallic surface, the electrons will never 5. Efficiency of photoemission,
be ejected from the surface regardless of intensity of incident
light. Number of photoelectrons emitted
per unit area per unit time n
(2) Kmax is independent of intensity of incident light, but it h= = e

.IN
Number of photons incident np
depends on the frequency of incident light i.e., Kmax µ n per unit area per unit time
(frequency of light). Intensity of emitted electrons I e
(3) Electrons are emitted almost instantaneously consistent with h= =
AL Intensity of incident radiation I p
particle view of light in which incident energy is concentrated ne I
in small packets (called photons) rather than over a large Therefore, h = = e
np I p
N
area (as in wave theory).
6. Maximum velocity of emitted electrons
Various Graphs Related to Photoelectric Effect
R

1. Photocurrent versus intensity of light graph 2h(v - v0 ) 2hc(l 0 - l ) 2evs


vmax = = =
U

I m mll 0 m
(mA) n > n0
7. Stopping potential
JO

h(v - v0 ) hc(l - l0 )
Vs = =
e ell 0
U

de-BROGLIE EQUATION (DUAL NATURE OF MATTER)


ED

In 1924, Louis de Broglie, wrote a doctoral dissertation in which


Intensity he proposed that since photons have wave and particle
2
(W/m ) characterstics, perhaps all forms of matter have wave as well as
2. Photocurrent versus potential graph particle properties.
n2 > n 1 > n 0 This is called dual nature of matter. According to which A matter
I I1 = I2 particle moving with a velocity v can be treated as a wave of
I n1 = n2 > n0 I2 > I1 (mA) wavelength l. This l is called de-Broglie wavelength & it is defined
(mA)
I2 as :
I1 h h
l= = ...(1)
P mv
where m is the mass of matter particle & these waves are called
–V0 V –V02 –V0 V matter waves.
1

3. Maximum kinetic energy versus potential graph. Further with the analogy of photon, the frequency of matter waves
is
E ...(2)
n01 for metal 1 n02for metal 2 n=
1 mv2 h
max
2 The dual nature of matter is quite apparent in these two equations
(equations (1) & (2)). i.e., each equation contains both particle
concepts (mv & E) & wave concepts (l & n). It is clear from next
topic that Compton effect confirm the validity of p = h/l for
photons, and the photoelectric effect confirms the validity and
n01 n02 n
E = hn for photons.
EBD_7179
688 PHYSICS

de-Broglie wavelength associated with electron accelerated under Explanation of Bohr’s quantum condition :
12.27 (a) According to Bohr’s quantum conditions :
a potential difference V volt is given by l = Å
V h
de-Broglie wave is not an electromagnetic wave but the matter Angular momentum, mvrn = n
2p
wave.
(b) Matter waves associated with the electrons moving in an
Wavelength of matter waves associated with accelerated charged
orbit are stationary waves.
particles :
(c) For the production of stationary waves in the orbit the
If V is the accelerating voltage applied then :
circumference of the orbit should be integral multiple of
(a) For the charged particle :
wavelength of waves associated with the electron,
Energy E = qV ;
h
i.e., 2prn = nl, where l =
2qV 2E mv
Velocity v = =
m m nh
\ mvrn =
2p
Momentum p = 2mE = 2mqV ;
COMPTON EFFECT
h h Further experimental proof for photon concept(i.e., particle nature
Wavelength l = =
2 mqV 2mE of light) was discovered in 1923 by American Physicist, A.H.
Compton. According to which, when a monochromatic beam of
X-rays (wavelength l0) strikes the electron in a carbon target, two

.IN
12.27
(b) For electron le = Å types of X-rays are scattered. The first type of scattered wave has
V
same wavelength (l0) as the incoming X-rays, while second type
0.286
(c) For proton lp = Å
AL
has a longer wavelength (l) than incident rays (First type of X-
V rays are called unmodified x-rays, while second type of X-rays are
called modified X-rays.) This change in wavelength i.e. Dl = l –
N
0.101 l0 is called Compton shift & this effect is called Compton effect”.
(d) For alpha particle la = Å
V y
R

X-ra E hc/l
0.202 red
U

(e) For deuteron ld = Å a tte p=h/l


V Incident x-rays Sc
JO

For neutral particles (neutron, atom or molecule) : q

h h E0=hc/l f
(a) If E is the energy of the particle, then, = m Recoiling
p0=h/l 0
U

p 2mE Carbon target electron


which consists
ED

h of free electron
(b) If T is the temperature, then, l =
3mkT
sinq
DAVISSON-GERMER EXPERIMENT
q
Idea of de-Broglie wave was tested beautifully in 1926 in an cosq
experiment performed by C. Davission (1881-1958) and L.H. Jermer
(1896-1971). They directed a beam of electrons at a crystal and
observed that the electrons scattered in various directions for a
given crystal orientation.
In this experiment the pattern cosf
formed by the electrons f
reflected from the crystal
lattice of aluminium is almost sinf
identical to that produced by Diagram shows compton scattering of an x-rays by free electron in
X-rays. This strongly a carbon target. The scattered x-rays has less energy than the
incident x-rays. The excess energy is taken by recoiling electrons.
suggests that the electrons
have a wavelength l This effect cannot be explained by classical theory (by wave nature
associated with them and that the Bragg condition for X-ray of light). According to classical model, when X-rays of frequency
diffraction holds true for electron also : n0 is incident on the material containing electrons, then electrons
Bragg’s equation do oscillate & reradiate electromagnetic waves of same frequency
nl = D sin q or nl = 2d sin f. n0 . Hence scattered X-rays has same frequency n0 & same
wavelength as that of incident X-rays.
Diffraction maximum of electrons accelerated with 54 volt is
Compton treated this processes as a collision between a photon
obtained at q = 50º for the Nickel crystal.
Dual Nature of Radiation and Matter 689

& an electron. In this treatment, the photon is assumed as a particle suddenly stopped on a metal of high atomic number.
of energy Properties of X-rays
E = hn0 = hc/l0 ...(1) (i) They are not deflected by electric or magnetic field.
Further, the rest mass of photon is zero (because photon travels (ii) They travel with the speed of light.
with the speed of light) hence the momentum of photon can be (iii) There is no charge on X-rays.
written as (iv) X-rays show both particle and wave nature.
E hc h (v) They are invisible.
p= = = ...(2)
c cl l Continuous and Characteristic X-rays
To dervie the compton shift. Dl, we apply both conservation of Experimental observation and studies of spectra of X-rays reveal
energy & momentum. that X-rays are of two types and so are their respective spectras.
Conservation of energy : Characteristic X-rays and Continuous X-rays.
Characteristic X-rays: The spectra of this group consists of
hc hc
= + Ke ...(3) several radiations with specific sharp wavelengths and frequency
l0 l similar to the spectrum (line) of atoms like hydrogen. The
Where hc/l is energy of scattered X-rays, Ke is kinetic energy of wavelengths of this group show characteristic discrete radiations
recoling electron & hc/l0 is the energy of incoming X-rays. Since emitted by the atoms of the target material. The characteristic
the electron may travel with the speed of light, so we must use X-rays spectra helps us to identify the element of target material.
relativistic expression of Ke in equation (3), and we obtain When the atoms of the target material are bombard with high
energy electrons (or hard X-rays), which posses enough energy

.IN
hc hc
= + g mc 2 - mc 2 ...(4) to penetrate into the atom, knock out the electron of inner shell
l0 l (say K shell, n = 1). When an electron is missing in the ‘K’ shell, an
where m is rest mass of electron and mc2 is the rest mass energy electron from next upper shell makes a quantum jump to fill the
AL
of the electron vacancy in ‘K’ shell. In the transition process the electron radiates
1 energy whose frequency lies in the X-rays region. The frequency
where g = of emitted radiation (i.e. of ph oton ) is given by
N
1 - v2 / c 2
Conservation of momentum : æ 1 1ö
v = RZe2 ç 2 - 2 ÷ ; where R is constant and Z is effective
R

h h è n1 n2 ø e
= cos q + gmv cos f x - component ...(5)
U

l0 l atomic number. Generally Ze is taken to be equal to Z – s, where Z


is proton number or atomic number of the element and s is called
JO

h the screening constant. Due to the presence of the other electrons.


0= sin q - gmvsin f y - component ...(6)
l The charge of the nucleus as seen by the electron will be different
where pe = gmv is the relativistic expression for momentum of in different shells.
U

recoiling electron.
By eliminating v & f from equation (4) to (6), we obtain ek
ED

hv
h (x-ray)
Dl = l - l 0 = (1 - cos q) ...(7)
mc
K L
K L
or Dl = 0.0243(1 - cos q)Å ...(8)
ei
It is clear from expression (7) that compton shift Dl depends on
scattering angle q & not on the wavelength. ei

Keep in Memory Knocking out e - of K shell by incident electron e i


1. The wave nature of light shows up in the phenomena of emission of X-ray photon (Ka- series)
interference, diffraction and polarisation whereas Another vacancy is now created in the ‘L’ shell which is again
photoelectric effect and compton effect shows particle nature filled up by another electron jump from one of the upper shell (M)
of light. which results in the emission of another photon, but of different
2. The maximum kinetic energy of the photoelectrons varies X-rays frequency. This transition continues till outer shells are
linearly with the frequency of incident radiation but is reached. Thus resulting in the emission of series of spectral line.
independent of its intensity. The transitions of electrons from various outer shells to the inner
most ‘K’ shell produces a group of X-rays lines called as
X-RAYS K-series. These radiations are most energetic and most
· The X-rays were discvoered by Prof. Roentgen, a German penetrating. K-series is further divided into Ka , Kb , K g ….
scientist in 1885. He was awarded Nobel Prize for this
depending upon the outer shell from which the transition is made.
discovery in 1901. X-rays are electromagnetic waves.
· The modern apparatus for the production of X-rays was The jump of electrons from outer shells to ‘L’ shell results in
developed by Dr. Coolidge in 1913. L-Series X-rays
· X-rays are produced when fast moving electrons are
EBD_7179
690 PHYSICS

50 kV

Relative intensity
n=a 40 kV
Na
n=4 N 30 kV
M a Mb
n=3 M 20 kV

La M -series
L b Lg
0 0.02 0.04 0.06 0.08 0.10
n=2 L Wavelength (nm)
L-series
Wavelength of X-Rays (Daume Hunt Law)
Ka
Kb
Kg K (i) When an electron is accelerated through a potential
d
difference V then
n=1 K 1
the energy accquired by electron = eV = mv 2 .
K-series 2
(ii) When these high energy electrons fall on target T of high
atomic number, then X-rays are produced, whose wavelength
and so on. is given by
Continuous X-rays : In addition to characteristic X-rays tubes hc hc
emit a continuous spectrum also. The characteristic line spectra is \ eV = hn = ; l= .
l eV

.IN
superimposed on a continuous X-rays spectra of varying
intensities. The wavelength of the continuous X-rays spectra are hc
(iii) The energy of X-rays of wavelength l is E = hn =
independent of material. One important feature of continuous X-
AL l
rays is that they end abruptly at a certain lower wavelength for a (iv) The shortest wavelength of X-rays emitted is
given voltage. If an electron beam of energy eV (electron volts) is hc 1240 1
l= = nm i.e. l µ .
incident on the target material; the electrons are suddenly stopped. eV V V
N
If the whole of the energy is converted to continuous radiation, It is called Daume Hunt law.
R

then lmin (corresponding to energy maximum) = hc/Ve where V is Types of X-Rays


the voltage applied. 1 . Hard X- rays : The X-rays of high frequency or low
U

X-ray photon wavelength are said to be hard X-rays. They have higher
JO

hv penetrating power.
2 . Soft X-rays : The X-rays of longer wavelength are called
K K¢ soft X-rays.
U

Target atom Moseley’s Law


Moseley used different elements as target in the X-ray tube. He
ED

The classical theory of electromagnetism states that the suddenly


found that Ka radiation of different elements were different
accelerated or decelerated electrons emit radiations of
electromagnetic nature called as bremsstrahlung (braking Mathematically, n = a( Z - b)
radiation) and wavelength of such radiation is continuous because where a and b depend on the particular line of the radiation
the loss in energy is statistical. At the peak, the probability of 3
maximum number of electrons producing radiation. For Ka, a = Rc and b = 1
4
The wavelength of X-rays emitted is minimum corresponding to
where R = Rydberg constant and c = speed of light
the electron which hits the target with maximum speed. This In general the wavelength of K - lines are given as
electron is completely stopped and will emit the photon of highest
1 é 1 ù
energy. = R ( Z - 1) 2 ê1 - ú where n = 2, 3, .....
As the electrons lose energy by collision, longer wavelengths are l ë n 2
û
produced the shape of the curve is statistical. Absorption of X-rays
· X-rays are absorbed by the materials according to the
Ka
relation I = I0 e–mx, where m is absorption coefficient and x
is the thickness of the mateiral. Here I is the intensity after
Kb penetrating the material through distance x and I0 is the
La
Intensity

Kg Lb initial intensity of the X-rays.


Ma · The coefficient of absorption (m) of the material is given by
0.6931
m= where x1/2 is the distance after traversing which
0.01 l min 0.1 1.0 x1/2
Wavelength (nm) the intensity of X-rays is reduced to half.
Dual Nature of Radiation and Matter 691

· Absorption coefficient depends on the nature of material 9. de-Broglie wavelength of a particle of K.E., Ek is given by
and wavelength of X-rays i.e. m = cZ4 l3.
h
It means that (a) m µ z4 (b) m µ l3 (c) m µ n–3. l= .
Fluorescence : Certain substances (like quinine sulphate, 2mE k
fluoricine, barium platinocyanide, uranium oxide etc.), when
10. de-Broglie wavelength for a charged particle with charge q
illuminated with light of high frequency (ultraviolet, violet, etc.)
and accelerated through a potental difference V is given by
emit light of lower frequency. The phenomenon is called
fluorescence. h
l= .
· When quinine sulphate is illuminated with ultraviolet or 2mqV
violet light it gives out blue light. The fluroescence of barium
sulphate as well as uranium oxide gives out green light when 11. de-Broglie wavelength of a material particle at temperature
illuminated with ultraviolet or violet light. T is given by
· The house hold tubes are painted from inside with
h
magnesium tungstate or zinc-beryllium silicate. They are l= . , where k is Boltzmann’s constant.
fluorescent materials. The ultraviolet light generated inside 2mkT
the tube falls on the walls, where magnesium tungstate gives Application of X-rays
blue light and zinc beryllium silicate gives yellow orange
light. The mixture of the two produces white light. If the Following are some important and useful applications of X-rays.
inner side of the tube is painted with cadmium borate it 1. Scientific applications: The diffraction of X-rays at crystals

.IN
gives fluorescence of pink light and when painted with zinc opened new dimension to X-rays crystallography. Various
silicate, it gives fluorescence of green light. diffraction patterns are used to determine internal structure
· The fluorescence occurs as long as the material is of crystals. The spacing and dispositions of atoms of a crystal
AL
illuminated. can be precisely determined by using Bragg’s law :
Phosphorescence : Fluorescent materials emit light only so long nl = 2d sin q.
as light is incident on them. There are certain susbstances which 2. Industrial applications: Since X-rays can penetrate through
N
continue emitting light for some time after the light incident on various materials, they are used in industry to detect defects
them is stopped. This phenomenon is called phosphorescence.
R

in metallic structures in big machines, railway tracks and


For example, if we make blue light incident on a zinc-sulphide
bridges. X-rays are used to analyse the composition of alloys
(ZnS) screen, then it produces phosphorescence of green colour.
U

Calcium sulpide and barium sulphide, after absorbing sunlight, and pearls.
3. In radio therapy: X-rays can cause damage to the tissues of
JO

produce blue phosphorescence for some time. Time of


phosphorescence is different for different materials. body (cells are ionized and molecules are broken). So X-rays
damages the malignant growths like cancer and tumors which
Keep in Memory
U

are dangerous to life, when is used in proper and controlled


1. The stopping potential (and hence the maximum kinetic intensities.
ED

energy of emitted electrons) is independent of the intensity 4. In medicine and surgery: X-rays are absorbed more in heavy
of light but that the saturation current (and hence the number elements than the lighter ones. Since bones (containing
of emitted photoelectrons) is proportional to the intensity. calcium and phosphorus) absorb more X-rays than the
2. Photoelctric effect doesn't take place below the threshold surrounding tissues (containing light elements like H , C , O ),
frequency for the photometal used.
their shadow is casted on the photographic plate. So the
3. In compton effect, the change in wavelength is independent
cracks or fracture in bones can be easily located. Similarly
of incident photon as well as of the nature of the scatterer,
but depends only on the angle of scattering (q). intestine and digestive system abnormalities are also detected
by X-rays.
h Example 1.
4. The quantity (= 0.02426Å) is called Compton
mc An electron is moving with velocity 107 m/s on a circular
wavelength. path of radius 0.57 cm in a magnetic field of 10–2 Wb/m2.
5. The maximum wavelength change possible in compton Find the value of e/m for electron.
effect is 0.05Å. Solution :
6. Compton effect can't be observed for visible light rays. e v
7. In compton effect, the direction of recoil electron is given e v B = ( mv 2 / r ) or =
m Br
lsinq e 107
by tan f = . \ = = 1.76 × 1011 coulomb/kg.
l '- l cos q m 10 - 2 ´ 0.0057
8. The kinetic energy of recoil electron is given by
Example 2.
é x(1 - cos q ) ù hn Find the ratio of specific charge e/m of a proton to that of
T = hn ê ú , where x =
ë 1 + x(1 - cos q ) û mc 2 an a particle.
EBD_7179
692 PHYSICS

Solution : (d) The K.E. of emitted electrons does not depend upon
æ e ö the intensity of light. Hence if the intensity of incident
æeö æeö
=ç ÷ 2e
ç ÷
÷ and çè m ÷ø = 4 m
light be doubled, the energy will remain unchanged.
è m ø proton çè m p ø a p Example 5.
(e / m) proton (e / m p ) 2 What will be the ratio of the de-Broglie wavelength of
\ = = or 2 : 1 proton and a particle of same energy?
(e / m ) a (2 e / 4 m p ) 1
Solution :
Example 3.
h h
Determine the ratio of momentum of an electron and an l= =
alpha particle which are accelerated from rest by a mv 2mE
potential difference of 100 V. 1
Solution : \ E= mv2 or 2 m E = m2v2 or mv = 2mE
2
1 lp æm ö
Q m v 2 = e V or v = (2 e V / m)1 / 2 4
2 so, = ç a ÷= = 2 or l p : l a = 2 : 1
la ç mp ÷ 1
\ p = m v = (2 m e V)1 / 2 è ø

Now p e = (2 m e ´ e ´100)1 / 2 Example 6.


and p a = (4 m a ´ e ´100) 1/ 2 Calculate the energy and momentum of a photon of
wavelength 6600Å.

.IN
\ p e / p a = (m e / 2 m a ) Solution :
Energy of photon
Example 4.
hc 6.62 ´ 10 -34 ´ 3 ´ 108
E= = = 3 ´ 10 -19 J
The work function of cesium is 1.8 eV. Light of 5000 Å is
AL l -10
incident on it. Calculate (a) threshold frequency and 6600 ´ 10
Momentum of photon
threshold wavelength. (b) maximum K.E. of emitted
h 6.6 ´ 10-34
N
electrons. (c) maximum velocity of emitted electrons p= = = 10-27 kg m / sec
(d) if the intensity of the incident light be doubled, then l 6600 ´ 10 -10
R

what will be the maximum K.E. of the emitted Example 7.


electrons? (h = 6.6 × 10–34 joule second, mass of electron
U

Find the number of photons emitted per second by a


m = 9.0×10–31 kg and speed of light c = 3 x 108 m/s). 25 watt source of monochromatic light of wavelength 6000Å.
JO

Solution : Solution :
(a) W0 = h v 0 or v 0 = h / W0 Energy of one photon
W0 = 1.8 ´ (1.6 ´10 -19 ) = 2.9 × 10–19 joule hc 6.62 ´ 10 -34 ´ 3 ´ 108
U

E = hn = = - 10
= 3.315 ´ 10 -19 J
l 6000 ´ 10
2.9 ´ 10 -19 joule
ED

W0 No. of photons emitted per second


\ n0 = =
h 6.6 ´ 10 -34 joule second Total energy emitted per second P
= =
\ Threshold frequency = 4.4 × 1014 sec–1 Energy of the photon E
Threshold wavelength
25
= ´ 10 -19 = 7.54 ´ 1019
c 3.0 ´108 m / s 3.315
l0 = =
n 0 4.4 ´1014 s -1 Example 8.
= 6.8 × 10–7 metre = 6800 Å In a photoelectric experiment, with light of wavelength l,
the fastest electron has speed v. If the exciting wavelength
hc is changed to 3l/4, then find the speed of the fastest emitted
(b) E k = hn - W0 = - W0
l electron.
(6.63 ´ 10 -34 ) (3 ´ 108 ) Solution :
= - (2.9 ´ 10 -19 joule)
-10
5000 ´ 10 1 hc
= (4.0 – 2.9) ×10–19 = 1.1 × 10–19 joule. mv 2 = - f
2 l
1
(c) E k = mv2max 1 hc 4hc
2 m¢ = -f = -f
2 (3l / 4) 3l
æ 2 Ek ö é 2 ´ (1.1´10 -19 ) ù
\ v max = ç m ÷ = ê - 31 ú
Clearly, v ¢ >
4
è ø êë 9.0 ´ 10 ûú v
3
5
= 5.0 × 10 m/sec.
Dual Nature of Radiation and Matter 693

Example 9. The de-Broglie wavelength associated with an electron in


What is the de-Broglie wavelength associated with (a) an this case is of the order of X-ray wavelengths.
electron moving with a speed of 5.4 × 106 m/s, and (b) a Example 11.
ball of mass 150 g travelling at 30.0 m/s? If the Ka radiation of Mo (Z = 42) has a wavelength of
Solution : 0.71 Å, calculate wavelength of the corresponding radiation
(a) For the electron of Cu, i.e., Ka for Cu (Z = 29) assuming s = 1.
Mass m = 9.11 ×10–31 kg, speed v = 5.4 × 106 m/s. Then, Solution :
momentum p = m v
= 9.11 ×10–31 (kg) × 5.4 × 106 (m/s)
According to Moseley’s law : v = a(Z - 1)
p = 4.92 × 10–24 kg m/s Þ (Z - 1)2 µ v or (Z - 1)2 µ 1/ l
-34
6.63 ´ 10 Js
de-Broglie wavelength, l = h/p = (ZM0 - 1) 2 l Cu
-24
4.92 ´ 10 kg m / s Þ =
(ZCu - 1) 2 l Mo
= 0.135 nm
(b) For the ball: Mass m' = 0.150 kg, (Z M0 - 1) 2
Speed v' = 30.0 m/s. Þ l Cu = l Mo
(ZCu - 1) 2
Then momentum p' = m'v'
2
= 0.150 (kg) × 30.0 (m/s) æ 41ö
= 0.71 ´ ç ÷ Å = 1.52Å
p' = 4.50 kg m/s è 28 ø

.IN
de Broglie wavelength l' = h/p' Example 12.
-34
6.63 ´ 10 Js The wavelength of Ka x-rays of two metals ‘A’ and ‘B’ are
= = 1.47 × 10–34 m
4.50 kg m / s
AL 4 1
The de-Broglie wavelength of electron is comparable and respectively, where ‘R’ is Rydbergg
1875R 675R
with X-ray wavelengths. However, for the ball it is about constant. Find the number of elements lying between A and
N
10–19 times the size of the proton, quite beyond B according to their atomic numbers.
experimental measurement.
R

Solution :
Example 10.
é1 1 ù
U

What is the de Broglie wavelength associated with an 1


Using = R(Z - 1)2 ê 2 - 2 ú
electron, accelerated through a potential difference of 100 l ëê n 2 n1 úû
JO

volt?
For a particle, n 1 = 2, n2 = 1
Solution :
Accelerating potential V = 100 V. The de Broglie wavelength, 1875 R æ 3ö
For metal A : = R (Z1 - 1) 2 ç ÷ Þ Z1 = 26
U

h 1.227 4 è 4ø
l= = nm
ED

p V æ 3ö
For metal B : 675 R = R (Z2 - 1)2 ç ÷ Þ Z2 = 31
1.227 è 4ø
l= nm = 0.123nm Therefore, 4 elements lie between A and B.
100
CONCEPT MAP
694
Photoelectric emission
Thermionic emission Field emission Emission of free electrons
Emission of electrons Emission of electrons by from the surface of metals
by suitably heating of applying a very strong when light radiation of
metal surface. electric field to a metal suitable frequency fall on it

Methods of electron
ED
emission
U 1
Photon Tiny packets mv 2max = h(v - v0 ) = hv - hv0
of light energy 2
Work function æ1 1 ö
Minimum energy required Electron emission Energy of a photon = hc ç - ÷
JO
to just escape electron from Emission of electrons E = hn è l l0 ø
metal surface f0 = hv0 from the surface of metal
U
De-Broglie wavelength
R
h h h N Effect of potential on
l= = = Effect of intensity of light
p mv 2ev m on photo current for a fixed photoelectric current. For a
DUAL NATURE OF frequency of incident fixed frequency and intensity
RADIATION AND radiation. Photoelectric of incident light photo electric
AL
Wave nature of current µintensity of current increases with increase
MATTE R
particles De-Broglie incident light in the potential
hypothesis Light has dual nature
.IN
wave and particle like
Davisson and Germer nature
experiment Confirms
the wave nature
of electrons Stopping potential
Photocell Stopping potential Photoelectric current
Uses of photocell varies linearly with becomes zero at a particular
Converts a change in
Count the persons the frequency of value of negative potential
intensity of illumination
entering an auditorium incident radiation for
into a change in photo current v0called stopping potential
burglar alarm, in a given photosensitive
It is a technological application or cut-off potential
motion picture and material
of photoelectric effect.
television
PHYSICS

EBD_7179
Dual Nature of Radiation and Matter 695

1. In which of the following, emission of electrons does not 12. Einstein’s photoelectric equation is E k = hn - f . In this
take place?
(a) Thermionic emission (b) X-rays emission equation Ek refers to
(c) Photoelectric emission (d) Secondary emission (a) kinetic energy of all the emitted electrons
2. In Davison-Germer experiment, an electron beam is incident (b) mean kinetic energy of emitted electrons
on a crystal. The reflected beam consists of (c) maximum kinetic energy of emitted electrons
(a) photons (b) protons (d) minimum kinetic energy of emitted electrons
(c) x-rays (d) electrons 13. If E1, E2, E3 are the respective kinetic energies of an electron,
3. The de-Broglie wavelength of an electron moving in the nth
an alpha-particle and a proton, each having the same
Bohr orbit of radius r is given by
de-Broglie wavelength, then
2 pr nr nr (a) E1 > E3 > E2 (b) E2 > E3 > E1
(a) (b) npr (c) (d)
n 2p p (c) E1 > E2 > E3 (d) E1 = E2 = E3
4. The kinetic energy of electron (in electron volt) moving with 14. When the speed of electrons increase, then the value of its
a velocity of 4 × 106 m/s will be
specific charge
(a) 60 eV (b) 50 eV (c) 30 eV (d) 45.5 eV

.IN
5. Photoelectric effect is the phenomenon in which (a) increases
(a) photons come out of a metal when it is hit by a beam of (b) decreases
electrons.
AL(c) ramains unchanged
(b) photons come out of the nucleus of an atom under the (d) increases upto some velocity and then begins to
action of an electric field. decrease
(c) electrons come out of a metal with a constant velocity 15. In an electron gun the control grid is given a negative
N
which depends on the frequency and intensity of potential relative to cathode in order to
incident light wave.
R

(a) decelerate electrons


(d) electons come out of a metal with different velocities
not greater than a certain value which depends only (b) repel electrons and thus to control the number of
U

on the frequency of the incident light wave and not on electrons passing through it
its intensity. (c) to select electrons of same velocity and to converge
JO

6. The momentum of a photon of wavelength l is them along the axis.


(a) hl (b) h/l (c) l/h (d) h/cl (d) to decrease the kinetic energy of electrons
7. A photo sensitive metal is not emitting photo-electrons when 16. X-rays are produced in X-ray tube operating at a given
U

irradiated. It will do so when threshold is crossed. To cross accelerating voltage. The wavelength of the continuous
the threshold we need to increase
ED

X-rays has values from


(a) intensity (b) frequency
(c) wavelength (d) None of these (a) 0 to ¥
8. A photoelectric cell is a device which (b) lmin to ¥, where lmin > 0
(a) converts light into electricity (c) 0 to lmax, where lmax < ¥
(b) converts electricity into light (d) lmin to lmax, where 0 < lmin < lmax < ¥
(c) stores light 17. As intensity of incident light increases
(d) stores electricity (a) photoelectric current increase
9. A particle with rest mass m0 is moving with speed of light c. (b) K.E. of emitted photoelectron increases
The de-Broglie wavelength associated with it will be (c) photoelectric current decreases
(a) µ (b) zero (c) m0 c/h (d) hn/m0c (d) K.E. of emitted photoelectrons decreases
10. Einstein’s work on photoelectric effect provided support
18. Which of the following shows par.ticle nature of light?
for the equation
(a) E = hn (b) E = mc2 (a) Refraction (b) Interference
(c) Polarization (d) Photoelectric effect
- Rhc 1
(c) E = (d) K.E. = mv 2 19. Which of the following when falls on a metal will emit
2 2
n photoelectrons ?
11. The maximum velocity of an electron emitted by light of (a) UV radiations (b) Infrared radiation
wavelength l incident on the surface of a metal of work-
(c ) Radio waves (d) Microwaves
function f is
20. Light of frequency v falls on a material of threshold
2( hc + lf ) 2( hc + lf ) frequency v0. Maximum kinetic energy of emitted electron
(a) (b)
ml ml is proportional to
2( hc - lf ) 2( hl - f) (a) v–v0 (b) v
(c) (d)
ml m (c) v – v0 (d) v 0
EBD_7179
696 PHYSICS

21. The ratio of de-Broglie wavelengths of proton and 24. The energy of a photon of wavelength l is
a -particle having same kinetic energy is hc
(a) (a) hc l (b)
2 : 1 (b) 2 2 : 1 (c) 2 : 1 (d) 4 : 1 l
22. White X-rays are called white due to the fact that l lh
(a) they are electromagnetic radiations having nature (c) (d)
hc c
same as that of white light.
25. In the Davisson and Germer experiment, the velocity of
(b) they are produced most abundantly in X ray tubes.
electrons emitted from the electron gun can be increased by
(c) they have a continuous wavelength range.
(d) they can be converted to visible light using coated (a) increasing the potential difference between the anode
screens and photographic plates are affected by and filament
them just like light. (b) increasing the filament current
23. In case of electrons and photons having the same
wavelength. What is same for them? (c) decreasing the filament current
(a) Energy (b) Velocity (d) decreasing the potential difference between the anode
(c) Momentum (d) Angular momentum and filament

.IN
1. In a photoelectric experiment the stopping potential for the 8.
AL
If the energy of a photon is 10 eV, then its momentum is
incident light of wavelength 4000Å is 2 volt. If the wavelength (a) 5.33 × 10–23 kg m/s (b) 5.33 × 10–25 kg m/s
be changed to 3000 Å, the stopping potential will be –29
(c) 5.33 × 10 kg m/s (d) 5.33 × 10–27 kg m/s
N
(a) 2 V (b) zero 9. The photoelectric threshold of Tungsten is 2300Å. The energy
R

(c) less than 2 V (d) more than 2 V of the electrons ejected from the surface by ultraviolet light
2. A proton and a-particle are accelerated through the same of wavelength 1800Å is
U

potential difference. The ratio of their de-Broglie wavelength (a) 0.15 eV (b) 1.5 eV (c) 15 eV (d) 150 eV
will be
JO

10. The maximum kinetic energy (Emax) of photoelectrons


(a) 1 : 1 (b) 1 : 2 (c) 2 : 1 (d) 2 2 : 1 emitted in a photoelectric cell varies with frequency (n) as
shown in the graph. The slope of the graph is equal to
3. An electron is accelerated by a p.d. of 1000 V. Its velocity
U

will be (a) charge of the electron E max


(a) 3.78 × 107 m/s (b) 1.89 × 107 m/s
ED

(c) 5.67 × 107 m/s (d) 0.95 × 107 m/s e


(b) of the electron
4. The photoelectric work function for a metal surface is m
4.125 eV. The cut-off wavelength for this surface is (c) work function of the emitter
(a) 4125 Å (b) 3000 Å (c) 6000 Å (d) 2062 Å n n
(d) Plank’s constant 0
5. The energy of a photon of green light of wavelength 5000Å
11. Ultraviolet radiation of 6.2 eV falls on an aluminium surface
is
(workfunction 4.2 eV). The kinetic energy in joule of the
(a) 3.459 × 10–19 joule (b) 3.973 × 10–19 joule faster electron emitted is approximately
(c) 4.132 × 10–19 joule (d) 8453 × 10–19 joule (a) 3 × 10–21 (b) 3 × 10–19
6. 4eV is the energy of incident photon and the work function (c) 3 × 10 –17 (d) 3 × 10–15
is 2eV. The stopping potential will be 12. A light having wavelength 300 nm fall on a metal surface.
(a) 2V (b) 4V (c ) 6 V (d) 2 2 V The work function of metal is 2.54 eV, what is stopping
potential ?
3
7. The velocity of a body of rest mass m o is c (where c is (a) 2.3 V (b) 2.59 V (c) 1.59 V (d) 1.29 V
2
the velocity of light in vacuum). The mass of this body is: 13. A and B are two metals with threshold frequencies
1.8 × 1014 Hz and 2.2 × 1014 Hz. Two identical photons of
æ 3ö æ1ö energy 0.825 eV each are incident on them. Then
(a) ç ÷m (b) ç ÷m o
ç 2 ÷ o è 2ø photoelectrons are emitted in (Take h = 6.6 × 10–34 Js)
è ø
(a) B alone (b) A alone
(c) 3m o (d) 2m0 (c) neither A nor B (d) both A and B.
Dual Nature of Radiation and Matter 697

14. The de-Broglie wavelength of a proton (mass = 1.6 × 23. In a photo-emissive cell with exciting wavelength l, the
10–27 kg) accelerated through a potential difference of 1 kV fastest electron has speed V. If the exciting wavelength is
is changed to 3l/4, the speed of the fastest emitted electron
(a) 600 A (b) 0.9 × 10–12m will be :
(c) 7 Å (d) 0.9 nm. (a) n (¾)1 / 2 (b) v(4 / 3)1 / 2
15. An ionisation chamber, with parallel conducting plates as
anode and cathodes has singly charged positive ions per (c) less the v(4 / 3)1 / 2 (d) greater than v(4 / 3)1 / 2
cm3. The electrons are moving toward the anode with 24. The momentum of photon whose frequency f is
velocity 0.4 m/s. The current density from anode to cathode
is X104 m A/m2. The velocity of positive ions moving hf hc h c
(a) (b) (c ) (d)
towards cathode is c f f hf
(a) 0.1 m/s (b) 0.4 m/s (c) zero (d) 1.6 m/s 25. A small photocell is placed at a distance of 4 m from a
16. A parallel beam of light is incident normally on a plane surface photosensitive surface. When light falls on the surface the
absorbing 40% of the light and reflecting the rest. If the current is 5 mA. If the distance of cell is decreased to 1 m,
incident beam carries 60 watt of power, the force exerted by it on the current will become
the surface is æ 5ö
(a) 3.2 × 10–8 N (b) 3.2 × 10–7 N (a) 1.25 mA (b) ç ÷ mA
–7 è 16 ø
(c) 5.12 × 10 N (d) 5.12 × 10–8 N
(c) 20 mA (d) 80 mA

.IN
17. Radiations of two photon’s energy, twice and ten times the
work function of metal are incident on the metal surface 26. If 5% of the energy supplied to a bulb is radiated as visible
successsively. The ratio of maximum velocities of light, the number of visible quanta emitted per second by a
100 W bulb, assuming the wavelength of visible light to be
photoelectrons emitted in two cases is
AL
5.6 × 10–5 cm, is
(a) 1 : 2 (b) 1 : 3 (c) 1 : 4 (d) 1 : 1
(a) 1.4 × 1019 (b) 1.4 × 1020
18. A photocell is illuminated by a small bright source placed 1 m 19 (d) 2 × 1020
N
(c) 2 × 10
away. When the same source of light is placed 2 m away, the
27. A material particle with a rest mass m0 is moving with speed
number of electrons emitted by photocathode are reduced by
R

of light c. The de-Broglie wavelength associated is given


a factor of
by
U

(a) 1/8 (b) 1/16 (c) 1/2 (d) 1/4


19. In a photoelectric effect measurement, the stopping potential h m0c
(a) (b) (c) zero (d) ¥
JO

for a given metal is found to be V0 volt when radiation of m0c h


wavelength l0 is used. If radiation of wavelength 2 l0 is 28. Which one of the following graphs represents the variation
used with the same metal then the stopping potential (in of maximum kinetic energy (EK) of the emitted electrons
U

volt) will be with frequency u in photoelectric effect correctly ?


ED

V0
(a) (b) 2 V0 EK EK
2

hc hc
(c) V0 + (d) V0 -
2el 0 2el 0 (a) (b)
20. An electron of mass m and charge e initially at rest gets u u
accelerated by a constant electric field E. The rate of change
of de-Broglie wavelength of this electron at time t ignoring
relativistic effects is EK EK

-h -eht - mh -h
(a) (b) (c) (d)
e Et 2
E e Et 2 eE
(c) (d)
21. The wavelength of Ka-line characteristic X-rays emitted by u u
an element is 0.32 Å. The wavelength of Kb-line emitted by u0
the same element will be
29. In a photoelectric effect experiment, for radiation with
(a) 0.32 Å (b) 0.39 Å (c) 0.49 Å (d) 0.27 Å frequency u0 with hu0 = 8eV, electrons are emitted with
22. The X-rays of wavelength 0.5 Å are scattered by a target. energy 2 eV. What is the energy of the electrons emitted for
What will be the energy of incident X-rays, if these are
incoming radiation of frequency 1.25 u0 ?
scattered at an angle of 72º ?
(a) 1 eV (b) 3.25 eV
(a) 12.41 keV (b) 6.2 keV
(c) 18.6 keV (d) 24.82 keV (c) 4 eV (d) 9.25 eV.
EBD_7179
698 PHYSICS

30. Two insulating plates are both uniformly charged in 38. The de-Broglie wavelength of a neutron at 927°C is l.
such a way that the potential difference between them is What will be its wavelength at 27 °C ?
V2 – V1 = 20 V. (i.e., plate 2 is at a higher potential). The l
plates are separated by d = 0.1 m and can be treated as (a) (b) l (c) 2 l (d) 4 l
2
infinitely large. An electron is released from rest on the inner 39. The maximum distance between interatomic lattice planes is
surface of plate 1. What is its speed when it hits plate 2? 15 Å. The maximum wavelength of X-rays which are
(e = 1.6 × 10–19 C, me = 9.11 × 10–31 kg) diffracted by this crystal will be
(a) 15 Å (b) 20 Å (c) 30 Å (d) 45 Å
Y 40. A monochromatic source of light operating at 200 W emits
4 × 1020 photons per second. Find the wavelength of light.
0.1 m (a) 400 mm (b) 200 nm
X (c) 4 × 10–10 Å (d) None of these
41. An X-ray tube is operated at 15 kV. Calculate the upper limit
of the speed of the electrons striking the target.
1 2 (a) 7.26 × 107 m/s (b) 7.62 × 107 m/s
(a) 2.65 × 106 m/s (b) 7.02 × 1012 m/s (c) 7.62 × 107 cm/s (d) 7.26 × 109 m/s
(c) 1.87 × 106 m/s (d) 32 × 10–19 m/s 42. All electrons ejected from a surface by incident light of
wavelength 200nm can be stopped before travelling 1m in
31. A steel ball of mass m is moving with a kinetic energy K. The

.IN
the direction of uniform electric field of 4N/C. The work
de-Broglie wavelength associated with the ball is
function of the surface is
h h
AL (a) 4 eV (b) 6.2 eV (c) 2 eV (d) 2.2 eV
(a) (b) 43. The stopping potential (V0) versus frequency (v) plot of a
2mK 2mK
substance is shown in figure, the threshold wavelength is
h
N
(c) (d) meaningless V0
2mK
2
R

32. If the X-ray tube is working at 20 kV then the minimum


wavelength of X-rays will be
U

1
(a) 0.31 Å (b) 0.62 Å (c) 0.93 Å (d) 0.47 Å
JO

33. In an electron gun, the potential difference between the


filament and plate is 3000 V. What will be the velocity of 4 5 6 7 8
v × 1014 Hz
electron emitting from the gun?
U

(a) 3 × 108 m/s (b) 3.18 × 107 m/s (a) 5 × 1014m


ED

7
(c) 3.52 × 10 m/s (d) 3.26 × 107 m/s (b) 6000 Å
34. Which metal will be suitable for a photoelectric cell using (c) 5000 Å
light of wavelength 4000Å. The work functions of sodium (d) Cannot be estimated from given data
44. A photon of 1.7 × 10–13 joule is absorbed by a material
and copper are respectively 2.0 eV and 4.0 eV.
under special circumstances. The correct statement is
(a) Sodium (b) Copper (a) Electrons of the atom of absorbed material will go the
(c) Both (d) None of these higher energy states
35. What is the energy of ka X-ray photon of copper (Z = 29) ? (b) Electron and positron pair will be created
(a) 7.99 keV(b) 8.29 keV (c) 8.25 keV (d) 7.19 keV (c) Only positron will be produced
36. When X-rays of wavelength 0.5 Å would be transmitted by (d) Photoelectric effect will occur and electron will be
produced
an aluminium tube of thickness 7 mm, its intensity remains
45. Light from a hydrogen discharge tube is incident on
one-fourth. The attenuation coefficient of aluminium for
the cathode of a photoelectric cell, the work function of
these X-rays is the cathode surface is 4.2 eV. In order to reduce the
(a) 0.188 mm–1 (b) 0.189 mm–1 photocurrent to zero the voltage of the anode relative to the
(c) 0.198 mm –1 (d) None of these cathode must be made
37. An X-ray tube with Cu target is operated at 25 kV. The (a) – 4.2 V (b) – 9.4 V
glancing angle for a NaCl. Crystal for the Cu ka line is 15.8°. (c) – 17.8 V (d) + 9.4 V
Find the wavelength of this line. 46. The glancing angle in a X-ray diffraction is 30º and the
(d for NaCl = 2.82 Å, h = 6.62 × 10–27 erg-sec) wavelength of X-rays used is 20 nm. The interplanar spacing
(a) 3.06 Å (b) 1.53 Å of the crystal dffracting these X-rays will be
(c) 0.75 Å (d) None of these (a) 40 nm (b) 20 nm (c) 15 nm (d) 10 nm
Dual Nature of Radiation and Matter 699

47. The frequency and work function of an incident photon are 57. A 200 W sodium street lamp emits yellow light of wavelength
v and f0. If v0 is the threshold frequency then necessary 0.6 µm. Assuming it to be 25% efficient in converting
condition for the emission of photoelectron is electrical energy to light, the number of photons of yellow
light it emits per second is
v0 (a) 1.5 × 1020 (b) 6 × 1018
(a) v < v0 (b) v=
2 (c) 62 × 1020 (d) 3 × 1019
(c) v ³ v0 (d) None of these 58. Monochromatic radiation emitted when electron on
48. The work function of aluminium is 4.2 eV. If two photons, hydrogen atom jumps from first excited to the ground state
each of energy 3.5 eV strike an electron of aluminium, then irradiates a photosensitive material. The stopping potential
emission of electrons is measured to be 3.57 V. The threshold frequency of the
(a) will be possible materials is
(b) will not be possible (a) 4 × 1015 Hz (b) 5 × 1015 Hz
(c) Data is incomplete (c) 1.6 × 1015 Hz (d) 2.5 × 1015 Hz
(d) Depends upon the density of the surface 59. The magnitude of the de-Broglie wavelength (l) of electron
(e), proton (p), neutron (n) and a-particle (a) all having the
49. For intensity I of a light of wavelenght 5000Å the
same energy of 1 MeV, in the increasing order will follow the
photoelectron saturation current is 0.40 µA and stopping
sequence
potential is 1.36 V, the work function of metal is
(a) le, lp, ln, la (b) le, ln, lp, la
(a) 2.47 eV (b) 1.36 eV

.IN
(c) la, ln, lp, le (d) lp, le, la, ln
(c) 1.10 eV (d) 0.43 eV
60. For a given photosensitive material and frequency (>
50. A source S1 is producing, 1015 photons per second of AL threshold frequency) of incident radiation, the photoelectric
wavelength 5000 Å. Another source S2 is producing current varies with the intensity of incident light as
1.02×1015 photons per second of wavelength 5100Å Then,
(power of S2) to the (power of S1) is equal to : Current Current
N
(a) 1.00 (b) 1.02 (c) 1.04 (d) 0.98
R

51. The potential difference that must be applied to stop the


fastest photoelectrons emitted by a nickel surface, having
U

work function 5.01 eV, when ultraviolet light of 200 nm falls (a) (b)
on it, must be
JO

Intensity Intensity
(a) 2.4 V (b) – 1.2 V (c) – 2.4 V (d) 1.2 V
52. Photoelectric emmision occurs only when the incident light
Current
U

has more than a certain minimum Current


(a) power (b) wavelength
ED

(c) intensity (d) frequency


53. In photoelectric emission process from a metal of work (c) (d)
function 1.8 eV, the kinetic energy of most energetic
electrons is 0.5 eV. The corresponding stopping potential is Intensity
(a) 1.8 V (b) 1.2 V (c) 0.5 V (d) 2.3 V Intensity
54. The threshold frequency for a photosensitive metal is 61. The anode voltage of a photocell is kept fixed. The
3.3 × 1014 Hz. If light of frequency 8.2 × 1014 Hz is incident wavelength l of the light falling on the cathode is gradually
changed. The plate current I of the photocell varies as
on this metal, the cut-off voltage for the photoelectric
follows
emission is nearly
(a) 2 V (b) 3 V (c) 5 V (d) 1 V I I
55. If the momentum of electron is changed by P, then the de
Broglie wavelength associated with it changes by 0.5%.
The initial momentum of electron will be (a) (b)

P O l O l
(a) 200 P (b) 400 P (c) (d) 100 P
200
56. Two radiations of photons energies 1 eV and 2.5 eV, I I
successively illuminate a photosensitive metallic surface of
work function 0.5 eV. The ratio of the maximum speeds of
(c) (d)
the emitted electrons is
(a) 1 : 4 (b) 1 : 2 (c) 1 : 1 (d) 1 : 5 O l O l
EBD_7179
700 PHYSICS

62. Radiations of intensity 0.5 W/m2 are striking a metal plate. 1 2


The pressure on the plate is (a) ms -1 (b) ms -1
137 137
(a) 0.166 × 10–8 N/m2 (b) 0.332 × 10–8 N/m2
1 2
(c) 0.111 × 10–8 N/m2 (d) 0.083 × 10–8 N/m2 (c) (d)
137 137
63. Hard X-rays for the study of fractures in bones should have
a minimum wavelength of 10–11 m. The accelerating voltage 70. The threshold frequency for photoelectric effect on sodium
for electrons in X-ray machine should be corresponds to a wavelength of 5000 Å. Its work function is
(a) < 124 kV (a) 4 × 10–19 J (b) 1 J
(b) > 124 kV (c) 2 × 10–19 J (d) 3 × 10–19 J
(c) between 60 kV and 70 kV DIRECTIONS (Qs. 71 to 75) : Each question contains
STATEMENT-1 and STATEMENT-2. Choose the correct answer
(d) = 100 kV
(ONLY ONE option is correct ) from the following-
64. The wavelength of a 1 keV photon is 1.24 × 10–9 m. What is
(a) Statement-1 is false, Statement-2 is true
the frequency of 1 MeV photon ?
(b) Statement-1 is true, Statement-2 is true; Statement-2 is a
(a) 1.24 × 1015 (b) 2.4 × 1020
correct explanation for Statement-1
(c) 1.24 × 1018 (d) 2 × 4 × 1023

.IN
(c) Statement-1 is true, Statement-2 is true; Statement-2 is not
65. Photoelectric work function of a metal is 1eV. Light of
a correct explanation for Statement-1
wavelength l = 3000 Å falls on it. The photo electrons come
(d) Statement-1 is true, Statement-2 is false
out with velocity
AL
71. Statement-1 : In process of photoelectric emission, all
(a) 10 metres/sec (b) 102 metres/sec
emitted electrons do not have same kinetic energy.
N
(c) 104 metres/sec (d) 106 metres/sec
Statement-2 : If radiation falling on photosensitive surface
66. Energy levels A, B, C of a certain atom correspond to
R

of a metal consists of different wavelength then energy


increasing values of energy i.e., EA < EB < EC. If l1, l2, l3 are
acquired by electrons absorbing photons of different
U

the wavelengths of radiation corresponding to the


wavelengths shall be different.
transitions C to B, B to A and C to A respectively, which of
JO

72. Statement-1 : Though light of a single frequency


the following relation is correct?
(monochromatic) is incident on a metal, the energies of
l1l 2 emitted photoelectrons are different.
U

(a) l 3 = l1 + l 2 (b) l3 =
l1 + l 2 Statement-2 : The energy of electrons emitted from inside
ED

the metal surface, is lost in collision with the other atoms in


(c) l1 + l 2 + l 3 = 0 (d) l 3 2 = l1 2 + l 2 2 the metal.

67. A radio transmitter operates at a freqency 880 kHz and a 73. Statement-1 : The de-Broglie wavelength of a molecule (in a
power of 10 kW. The number of photons emitted per second sample of ideal gas) varies inversely as the square root of
is absolute temperature.
(a) 1.72 × 1031 (b) 1.327 × 1025 Statement-2 : The rms velocity of a molecule (in a sample of
(c) 1.327 × 1037 (d) 1.327 × 1045 ideal gas) depends on temperature.
68. The momentum of a photon of an electromagnetic radiation 74. Statement-1 : Photoelectric saturation current increases
is 3.3 × 10–29 kgms–1. What is the frequency of the associated with the increase in frequency of incident light.
waves ?
Statement-2 : Energy of incident photons increases with
[h = 6.6 × 10–34 Js; c = 3 × 108 ms–1) increase in frequency and as a result photoelectric current
(a) 1.5 × 1013 Hz (b) 7.5 × 1012 Hz increases.
(c) 6.0 × 103 Hz (d) 3.0 × 103 Hz 75. Statement-1 : Photosensitivity of a metal is high if its work
function is small.
e2
69. The quantity has a value Statement-2 : Work function = hf0 where f0 is the threshold
2h e 0 c
frequency.
Dual Nature of Radiation and Matter 701

r
Exemplar Questions 6. An electron is moving with an initial velocity v = v0 i and
r
1. A particle is dropped from a height H. The de-Broglie is in a magnetic field B = B0 j . Then, it's de-Broglie
wavelength of the particle as a function of height is wavelength
proportional to (a) remains constant
(a) H (b) H1/2 (b) increases with time
(c) H0 (d) H–1/2 (c) decreases with me
2. The wavelength of a photon needed to remove a proton (d) increases and decreases periodically
from a nucleus which is bound to the nuclear with 1 MeV
7. An electron (mass m) with an initial velocity v = v0 i (v0 > 0) is
energy is nearly
(a) 1.2 nm (b) 1.2 × 10–3 nm in an electric field E = – E0 î (E0 = constant > 0). It's
de-Broglie wavelength at time t is given by
(c) 1.2 × 10–6 nm (d) 1.2 × 10 nm
3. Consider a beam of electrons (each electron with energy E0) l0 æ eE 0 t ö

.IN
(a)
æ eE0 t ö
(b) ç 1 + mv ÷
incident on a metal surface kept in an evacuated chamber. è 0 ø
ç1 + m v ÷
Then, AL è 0ø
(a) no electrons will be emitted as only photons can emit (c) l0 (d) l0t
electrons 8. An electron (mass m) with an initial velocity v = v0ˆi is in
(b) electrons can be emitted but all with an energy, E0 an electric field E = E ˆj . If l = h /mv, it's de-Broglie
N
0 0
(c) electrons can be emitted with any energy, with a wavelnegth at time t is given by
R

maximum of E0 – f (f is the work function)


e 2 E 20 t 2
(d) electrons can be emitted with any energy, with a l0 1 +
U

(a) l0 (b)
maximum of E0 m 2 v02
JO

4. Consider figure given below. Suppose the voltage applied l0 l0


to A is increased. The diffracted beam will have the maximum e2 E 20 t 2 æ e 2 E 02 t 2 ö
at value of q that (c) 1+ (d) ç1 + 2 2 ÷
U

m 2 v02 ç m v0 ÷
(a) will be larger than the earlier value è ø
ED

(b) will be the same as the earlier value NEET/AIPMT (2013-2017) Questions
(c) will be less than the earlier value 9. For photoelectric emission from certain metal the cut-off
(d) will depend on the target frequency is n. If radiation of frequency 2n impinges on the
– + metal plate, the maximum possible velocity of the emitted
HT
electron will be (m is the electron mass) [2013]
(a) hn / m (b) 2 hn / m

A Electron beam
Nickel
target
(c) 2 hn / m (d) hn / ( 2m )
10. The wavelength le of an electron and lp of a photon are of
Electron
LT gun same energy E are related by [2013]
Diffracted
Movable electron
Vaccum
chamber (a) l p µ l e (b) l p µ l e
collector beam

1
To galvanometer
(c) lp µ (d) l p µ l 2e
5. A proton, a neutron, an electron and an a-particle have le
same energy. Then, their de-Broglie wavelengths compare 11. A source of light is placed at a distance of 50 cm from a
as photocell and the stopping potential is found to be V0. If
(a) lp = ln > le > la the distance between the light source and photocell is made
(b) la < lp = ln > le 25 cm, the new stopping potential will be
(a) 2V0 (b) V0/2 [NEET Kar. 2013]
(c) le < lp = ln > la
(d) le = lp = ln = la (c) V0 (d) 4V0
EBD_7179
702 PHYSICS

12. The de-Broglie wavelength of neutron in thermal equilibrium 18. A photoelectric surface is illuminated successively by
at temperature T is [NEET Kar. 2013]
l
30.8 3.08 monochromatic light of wavelength l and . If the maximum
Å Å 2
(a) (b)
T T kinetic energy of the emitted photoelectrons in the second
case is 3 times that in the first case, the work function of the
0.308 0.0308
(c) Å (d) Å surface of the material is :
T T (h = Planck's constant, c = speed of light) [2015 RS]
13. When the energy of the incident radiation is incredased by hc 2hc
20%, the kinetic energy of the photoelectrons emitted from (a) (b)
l l
a metal surface increased from 0.5 eV to 0.8 eV. The work
function of the metal is : [2014] hc hc
(c) (d)
(a) 0.65 eV (b) 1.0 eV 3l 2l
(c) 1.3 eV (d) 1.5 eV 19. An electron of mass m and a photon have same energy E.
14. If the kinetic energy of the particle is increased to 16 times The ratio of de-Broglie wavelengths associated with them
its previous value, the percentage change in the de-Broglie is : [2016]
wavelength of the particle is : [2014] 1 1
(a) 25 (b) 75 1 æ E ö2
(b) æç
E ö2
(a)
(c) 60 (d) 50 ç ÷
c è 2m ø è 2m ÷ø

.IN
15. Which of the following figures represents the variation of
particle momentum and the associated de-Broglie 1
1 1 æ 2m ö 2
wavelength? [2015] (c) (d)
c(2mE) 2 ç ÷
xc è E ø
p
AL
p
20. When a metallic surface is illuminated with radiation of
wavelength l, the stopping potential is V. If the same surface
N
(a) (b) is illuminated with radiation of wavelength 2l, the stopping
R

V
l potential is . The threshold wavelength for the metallic
l 4
U

p p surface is : [2016]
(a) 4l (b) 5l
JO

(c) (d) 5
(c) l (d) 3l
2
U

21. The de-Broglie wavelength of a neutron in thermal


l l
equilibrium with heavy water at a temperature T (Kelvin)
ED

16. A certain metallic surface is illuminated with monochromatic and mass m, is :- [2017]
light of wavelength l. The stopping potential for photo-
electric current for this light is 3V0. If the same surface is h 2h
(a) (b)
illuminated with light of wavelength 2l, the stopping 3mkT 3mkT
potential is V0. The threshold wavelength for this surface
for photo-electric effect is [2015] 2h h
(c) (d)
l mkT mkT
(a) 4l (b) 22. The photoelectric threshold wavelength of silver is 3250 ×
4
10–10m. The velocity of the electron ejected from a silver
l surface by ultraviolet light of wavelength 2536 × 10–10 m is
(c) (d) 6l
6 (Given h = 4.14 × 10–15 eVs and c = 3 × 108 ms–1) [2017]
17. Light of wavelength 500 nm is incident on a metal with work (a) » 0.6 × 106 ms–1
function 2.28 eV. The de Broglie wavelength of the emitted (b) » 61 × 103 ms–1
electron is: [2015 RS] (c) » 0.3 × 106 ms–1
(a) < 2.8 × 10-9 m (b) ³. 2.8 × 10-9 m (d) » 6 × 105 ms–1
(c) £ 2.8 × 10-12 m (d) < 2.8 × 10-10 m
Dual Nature of Radiation and Matter 703

Hints & Solutions


EXERCISE - 1 22. (c)
1. (b) 2. (d) 3. (a) 4. (d) 5. (d) h h
6. (b) 7. (b) 8. (a) 23. (c) Momentum p = Þl= (photon)
l p
h h 1- v2 / c2 If l photon = l electron
9. (b) l= = = 0 (Q v = c)
mv m0 v p(momentum)will become same.
10. (a) Einstein’s photo electric effect & compton effect hc
established particle nature of light. These effects can 24. (b) Energy of a photon E = hn =
l
be explained only, when we assume that the light has
particle nature (To explain, Interference & Diffraction 25. (a) In the Davisson and Germer experiment, the velocity
the light must have wave nature. It means that light of electrons emitted from the electron gun can be
has both particle and wave nature, so it is called dual increased by increasing the potential difference
nature of light) between the anode and filament.
1 hc 2( hc - lf ) EXERCISE - 2
11. (c) mv 2 = -f Þ v =
2 l lm

.IN
12. (c) hc
1. (d) eVs = - W0 . If l decreases, Vs increases
1 l
13. (a) According to relation, E = mv 2
2
2.
AL 1
(d) q V = m v 2 or m v = 2q Vm ;
2
2E
=v h h 1
So l = =
N
m i.e. l µ ;
mv 2q V m qm
h
R

l=
2mE lp qama

You might also like